Sie sind auf Seite 1von 705

TJ-SP

Tribunal de Justiça do Estado de São Paulo

Psicólogo Judiciário
Volume I
Edital de Abertura - Concurso Público
MR026-2017
DADOS DA OBRA

Título da obra: Tribunal de Justiça do Estado de São Paulo

Cargo: Psicólogo Judiciário

(Baseada no Edital de Abertura - Concurso Público)

Volume I
• Língua Portuguesa • Atualidades e Deveres dos Servidores Públicos• Informática

Volume II
• Conhecimentos Específicos • Legislação

Autores:
Ana Maria Quiteto
Bruna Pinotti Garcia Oliveira
Mariela Cardoso
Guilherme Cardoso

Gestão de Conteúdos
Emanuela Amaral de Souza

Produção Editorial/Revisão
Elaine Cristina
Igor de Oliveira
Suelen Domenica Pereira

Capa
Rosa Thaina dos Santos

Editoração Eletrônica
Marlene Moreno

Gerente de Projetos
Bruno Fernandes
APRESENTAÇÃO

PARABÉNS! ESTE É O PASSAPORTE PARA SUA APROVAÇÃO.

A Nova Concursos tem um único propósito: mudar a vida das pessoas.


Vamos ajudar você a alcançar o tão desejado cargo público.
Nossos livros são elaborados por professores que atuam na área de Concursos Públicos. Assim a
matéria é organizada de forma que otimize o tempo do candidato. Afinal corremos contra o tempo,
por isso a preparação é muito importante.
Aproveitando, convidamos você para conhecer nossa linha de produtos “Cursos online”, conteúdos
preparatórios e por edital, ministrados pelos melhores professores do mercado.
Estar à frente é nosso objetivo, sempre.
Contamos com índice de aprovação de 87%*.
O que nos motiva é a busca da excelência. Aumentar este índice é nossa meta.
Acesse www.novaconcursos.com.br e conheça todos os nossos produtos.
Oferecemos uma solução completa com foco na sua aprovação, como: apostilas, livros, cursos on-
line, questões comentadas e treinamentos com simulados online.
Desejamos-lhe muito sucesso nesta nova etapa da sua vida!
Obrigado e bons estudos!

*Índice de aprovação baseado em ferramentas internas de medição.

CURSO ONLINE

PASSO 1
Acesse:
www.novaconcursos.com.br/passaporte

PASSO 2
Digite o código do produto no campo indicado no
site.
O código encontra-se no verso da capa da apostila.
*Utilize sempre os 8 primeiros dígitos.
Ex: FV054-17

PASSO 3
Pronto!
Você já pode acessar os conteúdos online.
SUMÁRIO

Língua Portuguesa

1. ortografia oficial;.................................................................................................................................................................................................. 01
2. conjugação de verbos;....................................................................................................................................................................................... 04
3. flexão de gênero, número e grau;................................................................................................................................................................. 18
4. regência e concordância;.................................................................................................................................................................................. 23
5. emprego de pronomes e crases;................................................................................................................................................................... 35
6. formas de tratamento;....................................................................................................................................................................................... 49
7. pontuação;.............................................................................................................................................................................................................. 57
8. figuras de sintaxe;................................................................................................................................................................................................ 61
9. análise sintática;.................................................................................................................................................................................................... 64
10. orações e seus termos;.................................................................................................................................................................................... 64
11. coordenação e subordinação;...................................................................................................................................................................... 64
12. acentuação;.......................................................................................................................................................................................................... 75
13. colocação pronominal e verbal;................................................................................................................................................................... 78
14. emprego de preposição;................................................................................................................................................................................ 78
15. conjunção e sinonímia;.................................................................................................................................................................................... 79
16. morfo-sintaxe...................................................................................................................................................................................................... 83

Atualidades e Deveres dos Servidores Públicos

1. Questões relacionadas a fatos políticos, econômicos e sociais e culturais, nacionais e internacionais, ocorridos a partir
do 2º semestre de 2016, divulgados na mídia local e/ou nacional...................................................................................................... 01
2. Estatuto dos Funcionários Públicos Civis do Estado de São Paulo (Lei n.º 10.261/68) - artigos 239 a 250; com as alte-
rações vigentes até a publicação deste Edital;.............................................................................................................................................. 34
3. Lei Federal nº 8.429/92 (Lei de Improbidade Administrativa) artigos 1º ao 11º – com as alterações vigentes até a
publicação do Edital................................................................................................................................................................................................ 56

Informática

MS-Windows 10: conceito de pastas, diretórios, arquivos e atalhos, área de trabalho, área de transferência, manipula-
ção de arquivos e pastas, uso dos menus, programas e aplicativos, interação com o conjunto de aplicativos MS-Office
2016, ............................................................................................................................................................................................................................. 01
MS-Word 2016: estrutura básica dos documentos, edição e formatação de textos, cabeçalhos, parágrafos, fontes, colu-
nas, marcadores simbólicos e numéricos, tabelas, impressão, controle de quebras e numeração de páginas, legendas,
índices, inserção de objetos, campos predefinidos, caixas de texto.................................................................................................... 03
MSExcel 2016: estrutura básica das planilhas, conceitos de células, linhas, colunas, pastas e gráficos, elaboração de ta-
belas e gráficos, uso de fórmulas, funções e macros, impressão, inserção de objetos, campos predefinidos, controle de
quebras e numeração de páginas, obtenção de dados externos, classificação de dados.......................................................... 49
Correio Eletrônico: uso de correio eletrônico, preparo e envio de mensagens, anexação de arquivos................................ 81
Internet: navegação internet, conceitos de URL, links, sites, busca e impressão de páginas..................................................... 90
LÍNGUA PORTUGUESA

1. ortografia oficial;.................................................................................................................................................................................................. 01
2. conjugação de verbos;....................................................................................................................................................................................... 04
3. flexão de gênero, número e grau;................................................................................................................................................................. 18
4. regência e concordância;.................................................................................................................................................................................. 23
5. emprego de pronomes e crases;................................................................................................................................................................... 35
6. formas de tratamento;....................................................................................................................................................................................... 49
7. pontuação;.............................................................................................................................................................................................................. 57
8. figuras de sintaxe;................................................................................................................................................................................................ 61
9. análise sintática;.................................................................................................................................................................................................... 64
10. orações e seus termos;.................................................................................................................................................................................... 64
11. coordenação e subordinação;...................................................................................................................................................................... 64
12. acentuação;.......................................................................................................................................................................................................... 75
13. colocação pronominal e verbal;................................................................................................................................................................... 78
14. emprego de preposição;................................................................................................................................................................................ 78
15. conjunção e sinonímia;.................................................................................................................................................................................... 79
16. morfo-sintaxe...................................................................................................................................................................................................... 83
LÍNGUA PORTUGUESA

O fonema z:
1. ORTOGRAFIA OFICIAL;
Escreve-se com S e não com Z:
*os sufixos: ês, esa, esia, e isa, quando o radical é substan-
tivo, ou em gentílicos e títulos nobiliárquicos: freguês, freguesa,
A ortografia é a parte da língua responsável pela grafia freguesia, poetisa, baronesa, princesa, etc.
correta das palavras. Essa grafia baseia-se no padrão culto *os sufixos gregos: ase, ese, ise e ose: catequese, metamor-
da língua. fose.
As palavras podem apresentar igualdade total ou par- *as formas verbais pôr e querer: pôs, pus, quisera, quis, qui-
cial no que se refere a sua grafia e pronúncia, mesmo ten- seste.
do significados diferentes. Essas palavras são chamadas *nomes derivados de verbos com radicais terminados em
de homônimas (canto, do grego, significa ângulo / canto, “d”: aludir - alusão / decidir - decisão / empreender - empresa /
do latim, significa música vocal). As palavras homônimas difundir - difusão
dividem-se em homógrafas, quando têm a mesma grafia *os diminutivos cujos radicais terminam com “s”: Luís - Lui-
(gosto, substantivo e gosto, 1ª pessoa do singular do verbo sinho / Rosa - Rosinha / lápis - lapisinho
gostar) e homófonas, quando têm o mesmo som (paço, pa- *após ditongos: coisa, pausa, pouso
lácio ou passo, movimento durante o andar). *em verbos derivados de nomes cujo radical termina com
Quanto à grafia correta em língua portuguesa, devem- “s”: anális(e) + ar - analisar / pesquis(a) + ar - pesquisar
se observar as seguintes regras:
Escreve-se com Z e não com S:
O fonema s: *os sufixos “ez” e “eza” das palavras derivadas de adjetivo:
macio - maciez / rico - riqueza
Escreve-se com S e não com C/Ç as palavras substan- *os sufixos “izar” (desde que o radical da palavra de origem
tivadas derivadas de verbos com radicais em nd, rg, rt, pel, não termine com s): final - finalizar / concreto - concretizar
*como consoante de ligação se o radical não terminar com s:
corr e sent: pretender - pretensão / expandir - expansão /
pé + inho - pezinho / café + al - cafezal ≠ lápis + inho - lapisinho
ascender - ascensão / inverter - inversão / aspergir aspersão
/ submergir - submersão / divertir - diversão / impelir - im-
O fonema j:
pulsivo / compelir - compulsório / repelir - repulsa / recorrer
- recurso / discorrer - discurso / sentir - sensível / consentir
Escreve-se com G e não com J:
- consensual
*as palavras de origem grega ou árabe: tigela, girafa, gesso.
*estrangeirismo, cuja letra G é originária: sargento, gim.
Escreve-se com SS e não com C e Ç os nomes deri- *as terminações: agem, igem, ugem, ege, oge (com poucas
vados dos verbos cujos radicais terminem em gred, ced, exceções): imagem, vertigem, penugem, bege, foge.
prim ou com verbos terminados por tir ou meter: agredir
- agressivo / imprimir - impressão / admitir - admissão / Observação: Exceção: pajem
ceder - cessão / exceder - excesso / percutir - percussão / *as terminações: ágio, égio, ígio, ógio, ugio: sortilégio, lití-
regredir - regressão / oprimir - opressão / comprometer - gio, relógio, refúgio.
compromisso / submeter - submissão *os verbos terminados em ger e gir: eleger, mugir.
*quando o prefixo termina com vogal que se junta com *depois da letra “r” com poucas exceções: emergir, surgir.
a palavra iniciada por “s”. Exemplos: a + simétrico - assimé- *depois da letra “a”, desde que não seja radical terminado
trico / re + surgir - ressurgir com j: ágil, agente.
*no pretérito imperfeito simples do subjuntivo. Exem- Escreve-se com J e não com G:
plos: ficasse, falasse *as palavras de origem latinas: jeito, majestade, hoje.
*as palavras de origem árabe, africana ou exótica: jiboia,
Escreve-se com C ou Ç e não com S e SS os vocábulos manjerona.
de origem árabe: cetim, açucena, açúcar *as palavras terminada com aje: aje, ultraje.
*os vocábulos de origem tupi, africana ou exótica: cipó,
Juçara, caçula, cachaça, cacique O fonema ch:
*os sufixos aça, aço, ação, çar, ecer, iça, nça, uça, uçu,
uço: barcaça, ricaço, aguçar, empalidecer, carniça, caniço, Escreve-se com X e não com CH:
esperança, carapuça, dentuço *as palavras de origem tupi, africana ou exótica: abacaxi,
*nomes derivados do verbo ter: abster - abstenção / de- muxoxo, xucro.
ter - detenção / ater - atenção / reter - retenção *as palavras de origem inglesa (sh) e espanhola (J): xampu,
*após ditongos: foice, coice, traição lagartixa.
*palavras derivadas de outras terminadas em te, to(r): *depois de ditongo: frouxo, feixe.
marte - marciano / infrator - infração / absorto - absorção *depois de “en”: enxurrada, enxoval.

Observação: Exceção: quando a palavra de origem não


derive de outra iniciada com ch - Cheio - (enchente)

1
LÍNGUA PORTUGUESA

Escreve-se com CH e não com X: 04. (TRF - 1ª REGIÃO - TÉCNICO JUDICIÁRIO - FCC/2011)
*as palavras de origem estrangeira: chave, chumbo, As palavras estão corretamente grafadas na seguinte frase:
chassi, mochila, espadachim, chope, sanduíche, salsicha. (A) Que eles viajem sempre é muito bom, mas não é boa
a ansiedade com que enfrentam o excesso de passageiros
As letras e e i: nos aeroportos.
*os ditongos nasais são escritos com “e”: mãe, põem. (B) Comete muitos deslises, talvez por sua espontaneida-
Com “i”, só o ditongo interno cãibra. de, mas nada que ponha em cheque sua reputação de pes-
*os verbos que apresentam infinitivo em -oar, -uar são soa cortês.
escritos com “e”: caçoe, tumultue. Escrevemos com “i”, os (C) Ele era rabugento e tinha ojeriza ao hábito do sócio
verbos com infinitivo em -air, -oer e -uir: trai, dói, possui. de descançar após o almoço sob a frondoza árvore do pátio.
- atenção para as palavras que mudam de sentido (D) Não sei se isso influe, mas a persistência dessa mágoa
quando substituímos a grafia “e” pela grafia “i”: área (su- pode estar sendo o grande impecilho na superação dessa
perfície), ária (melodia) / delatar (denunciar), dilatar (expan- sua crise.
dir) / emergir (vir à tona), imergir (mergulhar) / peão (de (E) O diretor exitou ao aprovar a retenção dessa alta
estância, que anda a pé), pião (brinquedo). quantia, mas não quiz ser taxado de conivente na concessão
de privilégios ilegítimos.
Fonte: http://www.pciconcursos.com.br/aulas/portu-
gues/ortografia 05.Em qual das alternativas a frase está corretamente es-
crita?
Questões sobre Ortografia A) O mindingo não depositou na cardeneta de poupansa.
B) O mendigo não depositou na caderneta de poupança.
01. (TRE/AP - TÉCNICO JUDICIÁRIO – FCC/2011) Entre C) O mindigo não depozitou na cardeneta de poupanssa.
as frases que seguem, a única correta é: D) O mendingo não depozitou na carderneta de poupansa.
a) Ele se esqueceu de que?
b) Era tão ruím aquele texto, que não deu para distri-
06.(IAMSPE/SP – ATENDENTE – [PAJEM] - CCI) – VU-
bui-lo entre os presentes.
NESP/2011) Assinale a alternativa em que o trecho – Mas ela
c) Embora devessemos, não fomos excessivos nas crí-
cresceu ... – está corretamente reescrito no plural, com o ver-
ticas.
bo no tempo futuro.
d) O juíz nunca negou-se a atender às reivindicações
(A) Mas elas cresceram...
dos funcionários.
(B) Mas elas cresciam...
e) Não sei por que ele mereceria minha consideração.
(C) Mas elas cresçam...
(D) Mas elas crescem...
02. (Escrevente TJ SP – Vunesp/2013). Assinale a alter-
nativa cujas palavras se apresentam flexionadas de acordo (E) Mas elas crescerão...
com a norma- -padrão.
(A) Os tabeliãos devem preparar o documento. 07. (IAMSPE/SP – ATENDENTE – [PAJEM – CCI] – VU-
(B) Esses cidadões tinham autorização para portar fuzis. NESP/2011 - ADAPTADA) Assinale a alternativa em que o tre-
(C) Para autenticar as certidãos, procure o cartório local. cho – O teste decisivo e derradeiro para ele, cidadão ansioso e
(D) Ao descer e subir escadas, segure-se nos corrimãos. sofredor...– está escrito corretamente no plural.
(E) Cuidado com os degrais, que são perigosos! (A) Os testes decisivo e derradeiros para eles, cidadãos
ansioso e sofredores...
03. (Agente de Vigilância e Recepção – VUNESP – 2013). (B) Os testes decisivos e derradeiros para eles, cidadães
Suponha-se que o cartaz a seguir seja utilizado para infor- ansioso e sofredores...
mar os usuários sobre o festival Sounderground. (C) Os testes decisivos e derradeiros para eles, cidadãos
Prezado Usuário ansiosos e sofredores...
________ de oferecer lazer e cultura aos passageiros do (D) Os testes decisivo e derradeiros para eles, cidadões
metrô, ________ desta segunda-feira (25/02), ________ 17h30, ansioso e sofredores...
começa o Sounderground, festival internacional que presti- (E) Os testes decisivos e derradeiros para eles, cidadães
gia os músicos que tocam em estações do metrô. ansiosos e sofredores...
Confira o dia e a estação em que os artistas se apresen-
tarão e divirta-se! 08. (MPE/RJ – TÉCNICO ADMINISTRATIVO – FUJB/2011)
Para que o texto atenda à norma-padrão, devem-se Assinale a alternativa em que a frase NÃO contraria a norma
preencher as lacunas, correta e respectivamente, com as culta:
expressões A) Entre eu e a vida sempre houve muitos infortúnios, por
A) A fim ...a partir ... as isso posso me queixar com razão.
B) A fim ...à partir ... às B) Sempre houveram várias formas eficazes para ultra-
C) A fim ...a partir ... às passarmos os infortúnios da vida.
D) Afim ...a partir ... às C) Devemos controlar nossas emoções todas as vezes que
E) Afim ...à partir ... as vermos a pobreza e a miséria fazerem parte de nossa vida.

2
LÍNGUA PORTUGUESA

D) É difícil entender o por quê de tanto sofrimento, prin- 4-) Fiz a correção entre parênteses:
cipalmente daqueles que procuram viver com dignidade e (A) Que eles viajem sempre é muito bom, mas não é
simplicidade. boa a ansiedade com que enfrentam o excesso de passa-
E) As dificuldades por que passamos certamente nos geiros nos aeroportos.
fazem mais fortes e preparados para os infortúnios da (B) Comete muitos deslises (deslizes), talvez por sua
vida. espontaneidade, mas nada que ponha em cheque (xeque)
sua reputação de pessoa cortês.
09.Assinale a alternativa cuja frase esteja incorreta: (C) Ele era rabugento e tinha ojeriza ao hábito do sócio
A) Porque essa cara? de descançar (descansar) após o almoço sob a frondoza
B) Não vou porque não quero. (frondosa) árvore do pátio.
C) Mas por quê? (D) Não sei se isso influe (influi), mas a persistência des-
D) Você saiu por quê? sa mágoa pode estar sendo o grande impecilho (empeci-
lho) na superação dessa sua crise.
10-) (GOVERNO DO ESTADO DE ALAGOAS – TÉCNICO (E) O diretor exitou (hesitou) ao aprovar a retenção des-
FORENSE - CESPE/2013 - adaptada) Uma variante igual- sa alta quantia, mas não quiz (quis) ser taxado de conivente
mente correta do termo “autópsia” é autopsia. na concessão de privilégios ilegítimos.
( ) Certo
( ) Errado 5-)
A) O mindingo não depositou na cardeneta de poupan-
GABARITO
sa. = mendigo/caderneta/poupança
C) O mindigo não depozitou na cardeneta de poupans-
01.E 02. D 03. C 04. A 05. B
sa. = mendigo/caderneta/poupança
06. E 07. C 08. E 09. A 10. C
D) O mendingo não depozitou na carderneta de pou-
pansa. =mendigo/depositou/caderneta/poupança
RESOLUÇÃO
6-) Futuro do verbo “crescer”: crescerão. Teremos: mas
1-)
elas crescerão...
(A) Ele se esqueceu de que? = quê?
(B) Era tão ruím (ruim) aquele texto, que não deu para
distribui-lo (distribuí-lo) entre os presentes. 7-) Como os itens apresentam o mesmo texto, a alter-
(C) Embora devêssemos (devêssemos) , não fomos ex- nativa correta já indica onde estão as inadequações nos
cessivos nas críticas. demais itens.
(D) O juíz (juiz) nunca (se) negou a atender às reivindi-
cações dos funcionários. 8-) Fiz as correções entre parênteses:
(E) Não sei por que ele mereceria minha consideração. A) Entre eu (mim) e a vida sempre houve muitos infor-
túnios, por isso posso me queixar com razão.
2-) B) Sempre houveram (houve) várias formas eficazes
(A) Os tabeliãos devem preparar o documento. = ta- para ultrapassarmos os infortúnios da vida.
beliães C) Devemos controlar nossas emoções todas as vezes
(B) Esses cidadões tinham autorização para portar fuzis. que vermos (virmos) a pobreza e a miséria fazerem parte
= cidadãos de nossa vida.
(C) Para autenticar as certidãos, procure o cartório lo- D) É difícil entender o por quê (o porquê) de tanto so-
cal. = certidões frimento, principalmente daqueles que procuram viver com
(E) Cuidado com os degrais, que são perigosos = de- dignidade e simplicidade.
graus E) As dificuldades por que (= pelas quais; correto) pas-
samos certamente nos fazem mais fortes e preparados
3-) Prezado Usuário para os infortúnios da vida.
A fim de oferecer lazer e cultura aos passageiros do me-
trô, a partir desta segunda-feira (25/02), às 17h30, come- 9-) Por que essa cara? = é uma pergunta e o pronome
ça o Sounderground, festival internacional que prestigia os está longe do ponto de interrogação.
músicos que tocam em estações do metrô.
Confira o dia e a estação em que os artistas se apresen- 10-) autopsia s.f., autópsia s.f.; cf. autopsia
tarão e divirta-se! (fonte: http://www.academia.org.br/abl/cgi/cgilua.exe/
A fim = indica finalidade; a partir: sempre separado; sys/start.htm?sid=23)
antes de horas: há crase RESPOSTA: “CERTO”.

3
LÍNGUA PORTUGUESA

- Irregulares: são aqueles cuja flexão provoca altera-


2. CONJUGAÇÃO DE VERBOS; ções no radical ou nas desinências: faço fiz farei fi-
zesse.
- Defectivos: são aqueles que não apresentam conju-
VERBO gação completa. Classificam-se em impessoais, unipessoais
e pessoais:
Verbo é a classe de palavras que se flexiona em pes- * Impessoais: são os verbos que não têm sujeito. Nor-
soa, número, tempo, modo e voz. Pode indicar, entre outros malmente, são usados na terceira pessoa do singular. Os
processos: ação (correr); estado (ficar); fenômeno (chover); principais verbos impessoais são:
ocorrência (nascer); desejo (querer). ** haver, quando sinônimo de existir, acontecer, reali-
O que caracteriza o verbo são as suas flexões, e não os zar-se ou fazer (em orações temporais).
seus possíveis significados. Observe que palavras como Havia poucos ingressos à venda. (Havia = Existiam)
corrida, chuva e nascimento têm conteúdo muito próximo Houve duas guerras mundiais. (Houve = Aconteceram)
ao de alguns verbos mencionados acima; não apresentam, Haverá reuniões aqui. (Haverá = Realizar-se-ão)
porém, todas as possibilidades de flexão que esses verbos Deixei de fumar há muitos anos. (há = faz)
possuem.
** fazer, ser e estar (quando indicam tempo)
Estrutura das Formas Verbais Faz invernos rigorosos no Sul do Brasil.
Do ponto de vista estrutural, uma forma verbal pode Era primavera quando a conheci.
apresentar os seguintes elementos: Estava frio naquele dia.
- Radical: é a parte invariável, que expressa o significa-
do essencial do verbo. Por exemplo: fal-ei; fal-ava; fal-am. ** Todos os verbos que indicam fenômenos da natureza
(radical fal-) são impessoais: chover, ventar, nevar, gear, trovejar, amanhe-
- Tema: é o radical seguido da vogal temática que indica cer, escurecer, etc. Quando, porém, se constrói, “Amanheci
a conjugação a que pertence o verbo. Por exemplo: fala-r mal-humorado”, usa-se o verbo “amanhecer” em sentido
São três as conjugações: 1ª - Vogal Temática - A - (falar), figurado. Qualquer verbo impessoal, empregado em senti-
2ª - Vogal Temática - E - (vender), 3ª - Vogal Temática - I - do figurado, deixa de ser impessoal para ser pessoal.
(partir). Amanheci mal-humorado. (Sujeito desinencial: eu)
- Desinência modo-temporal: é o elemento que desig- Choveram candidatos ao cargo. (Sujeito: candidatos)
na o tempo e o modo do verbo. Por exemplo: Fiz quinze anos ontem. (Sujeito desinencial: eu)
falávamos ( indica o pretérito imperfeito do indicativo.)
falasse ( indica o pretérito imperfeito do subjuntivo.) ** São impessoais, ainda:
- Desinência número-pessoal: é o elemento que desig- 1. o verbo passar (seguido de preposição), indicando
na a pessoa do discurso ( 1ª, 2ª ou 3ª) e o número (singular tempo: Já passa das seis.
ou plural): 2. os verbos bastar e chegar, seguidos da preposição
falamos (indica a 1ª pessoa do plural.) de, indicando suficiência: Basta de tolices. Chega de blas-
falavam (indica a 3ª pessoa do plural.) fêmias.
3. os verbos estar e ficar em orações tais como Está
Observação: o verbo pôr, assim como seus derivados bem, Está muito bem assim, Não fica bem, Fica mal, sem re-
(compor, repor, depor, etc.), pertencem à 2ª conjugação, pois ferência a sujeito expresso anteriormente. Podemos, ainda,
a forma arcaica do verbo pôr era poer. A vogal “e”, apesar nesse caso, classificar o sujeito como hipotético, tornando-
de haver desaparecido do infinitivo, revela-se em algumas se, tais verbos, então, pessoais.
formas do verbo: põe, pões, põem, etc. 4. o verbo deu + para da língua popular, equivalente de
“ser possível”. Por exemplo:
Formas Rizotônicas e Arrizotônicas Não deu para chegar mais cedo.
Ao combinarmos os conhecimentos sobre a estrutura Dá para me arrumar uns trocados?
dos verbos com o conceito de acentuação tônica, perce- * Unipessoais: são aqueles que, tendo sujeito, conju-
bemos com facilidade que nas formas rizotônicas o acento gam-se apenas nas terceiras pessoas, do singular e do
tônico cai no radical do verbo: opino, aprendam, nutro, por plural.
exemplo. Nas formas arrizotônicas, o acento tônico não cai A fruta amadureceu.
no radical, mas sim na terminação verbal: opinei, aprende- As frutas amadureceram.
rão, nutriríamos. Obs.: os verbos unipessoais podem ser usados como
verbos pessoais na linguagem figurada: Teu irmão amadu-
Classificação dos Verbos receu bastante.
Classificam-se em:
- Regulares: são aqueles que possuem as desinências Entre os unipessoais estão os verbos que significam vo-
normais de sua conjugação e cuja flexão não provoca alte- zes de animais; eis alguns: bramar: tigre, bramir: crocodilo,
rações no radical: canto cantei cantarei cantava cacarejar: galinha, coaxar: sapo, cricrilar: grilo
cantasse.

4
LÍNGUA PORTUGUESA

Os principais verbos unipessoais são:


1. cumprir, importar, convir, doer, aprazer, parecer, ser (preciso, necessário, etc.):
Cumpre trabalharmos bastante. (Sujeito: trabalharmos bastante.)
Parece que vai chover. (Sujeito: que vai chover.)
É preciso que chova. (Sujeito: que chova.)

2. fazer e ir, em orações que dão ideia de tempo, seguidos da conjunção que.
Faz dez anos que deixei de fumar. (Sujeito: que deixei de fumar.)
Vai para (ou Vai em ou Vai por) dez anos que não vejo Cláudia. (Sujeito: que não vejo Cláudia)
Obs.: todos os sujeitos apontados são oracionais.

* Pessoais: não apresentam algumas flexões por motivos morfológicos ou eufônicos. Por exemplo:
- verbo falir. Este verbo teria como formas do presente do indicativo falo, fales, fale, idênticas às do verbo falar - o que
provavelmente causaria problemas de interpretação em certos contextos.

- verbo computar. Este verbo teria como formas do presente do indicativo computo, computas, computa - formas de
sonoridade considerada ofensiva por alguns ouvidos gramaticais. Essas razões muitas vezes não impedem o uso efetivo de
formas verbais repudiadas por alguns gramáticos: exemplo disso é o próprio verbo computar, que, com o desenvolvimento
e a popularização da informática, tem sido conjugado em todos os tempos, modos e pessoas.

- Abundantes: são aqueles que possuem mais de uma forma com o mesmo valor. Geralmente, esse fenômeno costuma
ocorrer no particípio, em que, além das formas regulares terminadas em -ado ou -ido, surgem as chamadas formas curtas
(particípio irregular). Observe:

INFINITIVO PARTICÍPIO REGULAR PARTICÍPIO IRREGULAR


Anexar Anexado Anexo
Dispersar Dispersado Disperso
Eleger Elegido Eleito
Envolver Envolvido Envolto

INFINITIVO PARTICÍPIO REGULAR PARTICÍPIO IRREGULAR


Imprimir Imprimido Impresso
Matar Matado Morto
Morrer Morrido Morto
Pegar Pegado Pego
Soltar Soltado Solto

- Anômalos: são aqueles que incluem mais de um radical em sua conjugação. Por exemplo: Ir, Pôr, Ser, Saber (vou, vais,
ides, fui, foste, pus, pôs, punha, sou, és, fui, foste, seja).

- Auxiliares: São aqueles que entram na formação dos tempos compostos e das locuções verbais. O verbo principal,
quando acompanhado de verbo auxiliar, é expresso numa das formas nominais: infinitivo, gerúndio ou particípio.

Vou espantar as moscas.


(verbo auxiliar) (verbo principal no infinitivo)

Está chegando a hora do debate.


(verbo auxiliar) (verbo principal no gerúndio)

Os noivos foram cumprimentados por todos os presentes.


(verbo auxiliar) (verbo principal no particípio)

Obs.: os verbos auxiliares mais usados são: ser, estar, ter e haver.

5
LÍNGUA PORTUGUESA

Conjugação dos Verbos Auxiliares

SER - Modo Indicativo


Presente Pret.Perfeito Pretérito Imp. Pret.Mais-Que-Perf. Fut.do Pres. Fut. Do Pretérito
sou fui era fora serei seria
és foste eras foras serás serias
é foi era fora será seria
somos fomos éramos fôramos seremos seríamos
sois fostes éreis fôreis sereis seríeis
são foram eram foram serão seriam

SER - Modo Subjuntivo

Presente Pretérito Imperfeito Futuro


que eu seja se eu fosse quando eu for
que tu sejas se tu fosses quando tu fores
que ele seja se ele fosse quando ele for
que nós sejamos se nós fôssemos quando nós formos
que vós sejais se vós fôsseis quando vós fordes
que eles sejam se eles fossem quando eles forem

SER - Modo Imperativo

Afirmativo Negativo
sê tu não sejas tu
seja você não seja você
sejamos nós não sejamos nós
sede vós não sejais vós
sejam vocês não sejam vocês

SER - Formas Nominais

Infinitivo Impessoal Infinitivo Pessoal Gerúndio Particípio


ser ser eu sendo sido
seres tu

Infinitivo Impessoal Infinitivo Pessoal Gerúndio Particípio


ser ele
sermos nós
serdes vós
serem eles

ESTAR - Modo Indicativo


Presente Pret. perf. Pret. Imperf. Pret.Mais-Que-Perf. Fut.doPres. Fut.do Preté.
estou estive estava estivera estarei estaria
estás estiveste estavas estiveras estarás estarias
está esteve estava estivera estará estaria
estamos estivemos estávamos estivéramos estaremos estaríamos
estais estivestes estáveis estivéreis estareis estaríeis
estão estiveram estavam estiveram estarão estariam

ESTAR - Modo Subjuntivo e Imperativo


Presente Pretérito Imperfeito Futuro Afirmativo Negativo
esteja estivesse estiver
estejas estivesses estiveres está estejas
esteja estivesse estiver esteja esteja
estejamos estivéssemos estivermos estejamos estejamos
estejais estivésseis estiverdes estai estejais
estejam estivessem estiverem estejam estejam

6
LÍNGUA PORTUGUESA

ESTAR - Formas Nominais

Infinitivo Impessoal Infinitivo Pessoal Gerúndio Particípio


estar estar estando estado
estares
estar
estarmos
estardes
estarem

HAVER - Modo Indicativo


Presente Pret. Perf. Pret. Imper. Pret.Mais-Que-Perf. Fut. Do Pres. Fut. Do Preté.
hei houve havia houvera haverei haveria
hás houveste havias houveras haverás haverias
há houve havia houvera haverá haveria
havemos houvemos havíamos houvéramos haveremos haveríamos
haveis houvestes havíeis houvéreis havereis haveríeis
hão houveram haviam houveram haverão haveriam

HAVER - Modo Subjuntivo e Imperativo

Presente Pretérito Imperfeito Futuro Afirmativo Negativo


haja houvesse houver
hajas houvesses houveres há hajas
haja houvesse houver haja haja
hajamos houvéssemos houvermos hajamos hajamos
hajais houvésseis houverdes havei hajais
hajam houvessem houverem hajam hajam

HAVER - Formas Nominais

Infinitivo Impessoal Infinitivo Pessoal Gerúndio Particípio


haver haver havendo havido
haveres
haver
havermos
haverdes
haverem

TER - Modo Indicativo

Presente Pret. Perf. Pret. Imper. Preté.Mais-Que-Perf. Fut. Do Pres. Fut. Do Preté.
Tenho tive tinha tivera terei teria
tens tiveste tinhas tiveras terás terias
tem teve tinha tivera terá teria
temos tivemos tínhamos tivéramos teremos teríamos
tendes tivestes tínheis tivéreis tereis teríeis
têm tiveram tinham tiveram terão teriam

TER - Modo Subjuntivo e Imperativo

Presente Pretérito Imperfeito Futuro Afirmativo Negativo


Tenha tivesse tiver
tenhas tivesses tiveres tem tenhas
tenha tivesse tiver tenha tenha
tenhamos tivéssemos tivermosWW tenhamos tenhamos
tenhais tivésseis tiverdes tende tenhais
tenham tivessem tiverem tenham tenham

7
LÍNGUA PORTUGUESA

- Pronominais: São aqueles verbos que se conjugam com Formas Nominais


os pronomes oblíquos átonos me, te, se, nos, vos, se, na mes-
ma pessoa do sujeito, expressando reflexibilidade (pronomi- Além desses três modos, o verbo apresenta ainda for-
nais acidentais) ou apenas reforçando a ideia já implícita no mas que podem exercer funções de nomes (substantivo,
próprio sentido do verbo (reflexivos essenciais). Veja: adjetivo, advérbio), sendo por isso denominadas formas
- 1. Essenciais: são aqueles que sempre se conjugam com nominais. Observe:
os pronomes oblíquos me, te, se, nos, vos, se. São poucos: abs- - Infinitivo Impessoal: exprime a significação do verbo
ter-se, ater- -se, apiedar-se, atrever-se, dignar-se, arrepender- de modo vago e indefinido, podendo ter valor e função de
se, etc. Nos verbos pronominais essenciais a reflexibilidade já substantivo. Por exemplo:
está implícita no radical do verbo. Por exemplo: Arrependi-me Viver é lutar. (= vida é luta)
de ter estado lá. A ideia é de que a pessoa representada pelo É indispensável combater a corrupção. (= combate à)
sujeito (eu) tem um sentimento (arrependimento) que recai
sobre ela mesma, pois não recebe ação transitiva nenhuma
O infinitivo impessoal pode apresentar-se no presen-
vinda do verbo; o pronome oblíquo átono é apenas uma par-
te (forma simples) ou no passado (forma composta). Por
tícula integrante do verbo, já que, pelo uso, sempre é conju-
exemplo:
gada com o verbo. Diz-se que o pronome apenas serve de re-
É preciso ler este livro.
forço da ideia reflexiva expressa pelo radical do próprio verbo.
Veja uma conjugação pronominal essencial (verbo e res- Era preciso ter lido este livro.
pectivos pronomes):
Eu me arrependo - Infinitivo Pessoal: é o infinitivo relacionado às três
Tu te arrependes pessoas do discurso. Na 1ª e 3ª pessoas do singular, não
Ele se arrepende apresenta desinências, assumindo a mesma forma do im-
Nós nos arrependemos pessoal; nas demais, flexiona-se da seguinte maneira:
Vós vos arrependeis 2ª pessoa do singular: Radical + ES Ex.: teres(tu)
Eles se arrependem 1ª pessoa do plural: Radical + MOS Ex.: termos (nós)
2ª pessoa do plural: Radical + DES Ex.: terdes (vós)
- 2. Acidentais: são aqueles verbos transitivos diretos em 3ª pessoa do plural: Radical + EM Ex.: terem (eles)
que a ação exercida pelo sujeito recai sobre o objeto repre- Por exemplo: Foste elogiado por teres alcançado uma
sentado por pronome oblíquo da mesma pessoa do sujeito; boa colocação.
assim, o sujeito faz uma ação que recai sobre ele mesmo. Em
geral, os verbos transitivos diretos ou transitivos diretos e in- - Gerúndio: o gerúndio pode funcionar como adjetivo
diretos podem ser conjugados com os pronomes menciona- ou advérbio. Por exemplo:
dos, formando o que se chama voz reflexiva. Por exemplo: Saindo de casa, encontrei alguns amigos. (função de ad-
Maria se penteava. vérbio)
A reflexibilidade é acidental, pois a ação reflexiva pode ser Nas ruas, havia crianças vendendo doces. (função de ad-
exercida também sobre outra pessoa. Por exemplo: jetivo)
Maria penteou-me. Na forma simples, o gerúndio expressa uma ação em
Observações: curso; na forma composta, uma ação concluída. Por exem-
- Por fazerem parte integrante do verbo, os pronomes plo:
oblíquos átonos dos verbos pronominais não possuem fun- Trabalhando, aprenderás o valor do dinheiro.
ção sintática.
Tendo trabalhado, aprendeu o valor do dinheiro.
- Há verbos que também são acompanhados de prono-
mes oblíquos átonos, mas que não são essencialmente pro-
nominais, são os verbos reflexivos. Nos verbos reflexivos, os
- Particípio: quando não é empregado na formação
pronomes, apesar de se encontrarem na pessoa idêntica à do
sujeito, exercem funções sintáticas. Por exemplo: dos tempos compostos, o particípio indica geralmente o
Eu me feri. = Eu(sujeito) - 1ª pessoa do singular me (ob- resultado de uma ação terminada, flexionando-se em gê-
jeto direto) - 1ª pessoa do singular nero, número e grau. Por exemplo:
Terminados os exames, os candidatos saíram.
Modos Verbais
Quando o particípio exprime somente estado, sem ne-
Dá-se o nome de modo às várias formas assumidas pelo nhuma relação temporal, assume verdadeiramente a fun-
verbo na expressão de um fato. Em Português, existem três ção de adjetivo (adjetivo verbal). Por exemplo: Ela foi a alu-
modos: na escolhida para representar a escola.
Indicativo - indica uma certeza, uma realidade: Eu sempre
estudo. Tempos Verbais
Subjuntivo - indica uma dúvida, uma possibilidade: Talvez
eu estude amanhã. Tomando-se como referência o momento em que se
Imperativo - indica uma ordem, um pedido: Estuda agora, fala, a ação expressa pelo verbo pode ocorrer em diversos
menino. tempos. Veja:

8
LÍNGUA PORTUGUESA

1. Tempos do Indicativo
- Presente - Expressa um fato atual: Eu estudo neste colégio.
- Pretérito Imperfeito - Expressa um fato ocorrido num momento anterior ao atual, mas que não foi completamente
terminado: Ele estudava as lições quando foi interrompido.
- Pretérito Perfeito - Expressa um fato ocorrido num momento anterior ao atual e que foi totalmente terminado: Ele
estudou as lições ontem à noite.
- Pretérito-Mais-Que-Perfeito - Expressa um fato ocorrido antes de outro fato já terminado: Ele já tinha estudado as
lições quando os amigos chegaram. (forma composta) Ele já estudara as lições quando os amigos chegaram. (forma simples).
- Futuro do Presente - Enuncia um fato que deve ocorrer num tempo vindouro com relação ao momento atual: Ele
estudará as lições amanhã.
- Futuro do Pretérito - Enuncia um fato que pode ocorrer posteriormente a um determinado fato passado: Se eu tivesse
dinheiro, viajaria nas férias.

2. Tempos do Subjuntivo
- Presente - Enuncia um fato que pode ocorrer no momento atual: É conveniente que estudes para o exame.
- Pretérito Imperfeito - Expressa um fato passado, mas posterior a outro já ocorrido: Eu esperava que ele vencesse o
jogo.

Obs.: o pretérito imperfeito é também usado nas construções em que se expressa a ideia de condição ou desejo. Por
exemplo: Se ele viesse ao clube, participaria do campeonato.

- Futuro do Presente - Enuncia um fato que pode ocorrer num momento futuro em relação ao atual: Quando ele vier
à loja, levará as encomendas.

Obs.: o futuro do presente é também usado em frases que indicam possibilidade ou desejo. Por exemplo: Se ele vier à
loja, levará as encomendas.

Presente do Indicativo
1ª conjugação 2ª conjugação 3ª conjugação Desinência pessoal
CANTAR VENDER PARTIR
cantO vendO partO O
cantaS vendeS parteS S
canta vende parte -
cantaMOS vendeMOS partiMOS MOS
cantaIS vendeIS partIS IS
cantaM vendeM parteM M

Pretérito Perfeito do Indicativo


1ª conjugação 2ª conjugação 3ª conjugação Desinência pessoal
CANTAR VENDER PARTIR
canteI vendI partI I
cantaSTE vendeSTE partISTE STE
cantoU vendeU partiU U
cantaMOS vendeMOS partiMOS MOS
cantaSTES vendeSTES partISTES STES
cantaRAM vendeRAM partiRAM RAM

Pretérito mais-que-perfeito
1ª conjugação 2ª conjugação 3ª conjugação Des. temporal (1ª/2ª e 3ª conj.) Desinência pessoal
CANTAR VENDER PARTIR
cantaRA vendeRA partiRA RA Ø
cantaRAS vendeRAS partiRAS RA S
cantaRA vendeRA partiRA RA Ø
cantáRAMOS vendêRAMOS partíRAMOS RA MOS
cantáREIS vendêREIS partíREIS RE IS
cantaRAM vendeRAM partiRAM RA M

9
LÍNGUA PORTUGUESA

Pretérito Imperfeito do Indicativo


1ª conjugação 2ª conjugação 3ª conjugação
CANTAR VENDER PARTIR
cantAVA vendIA partIA
cantAVAS vendIAS partAS
CantAVA vendIA partIA
cantÁVAMOS vendÍAMOS partÍAMOS
cantÁVEIS vendÍEIS partÍEIS
cantAVAM vendIAM partIAM

Futuro do Presente do Indicativo


1ª conjugação 2ª conjugação 3ª conjugação
CANTAR VENDER PARTIR
cantar ei vender ei partir ei
cantar ás vender ás partir ás
cantar á vender á partir á
cantar emos vender emos partir emos
cantar eis vender eis partir eis
cantar ão vender ão partir ão

Futuro do Pretérito do Indicativo


1ª conjugação 2ª conjugação 3ª conjugação
CANTAR VENDER PARTIR
cantarIA venderIA partirIA
cantarIAS venderIAS partirIAS
cantarIA venderIA partirIA
cantarÍAMOS venderÍAMOS partirÍAMOS
cantarÍEIS venderÍEIS partirÍEIS
cantarIAM venderIAM partirIAM

Presente do Subjuntivo
Para se formar o presente do subjuntivo, substitui-se a desinência -o da primeira pessoa do singular do presente do
indicativo pela desinência -E (nos verbos de 1ª conjugação) ou pela desinência -A (nos verbos de 2ª e 3ª conjugação).

1ª conjug. 2ª conjug. 3ª conju. Des. temporal Des.temporal Desinên. pessoal


1ª conj. 2ª/3ª conj.
CANTAR VENDER PARTIR
cantE vendA partA E A Ø
cantES vendAS partAS E A S
cantE vendA partA E A Ø
cantEMOS vendAMOS partAMOS E A MOS
cantEIS vendAIS partAIS E A IS
cantEM vendAM partAM E A M

Pretérito Imperfeito do Subjuntivo

Para formar o imperfeito do subjuntivo, elimina-se a desinência -STE da 2ª pessoa do singular do pretérito perfeito, ob-
tendo-se, assim, o tema desse tempo. Acrescenta-se a esse tema a desinência temporal -SSE mais a desinência de número
e pessoa correspondente.

1ª conjugação 2ª conjugação 3ª conjugação Des. temporal Desinência pessoal


1ª /2ª e 3ª conj.
CANTAR VENDER PARTIR
cantaSSE vendeSSE partiSSE SSE Ø
cantaSSES vendeSSES partiSSES SSE S
cantaSSE vendeSSE partiSSE SSE Ø
cantáSSEMOS vendêSSEMOS partíSSEMOS SSE MOS
cantáSSEIS vendêSSEIS partíSSEIS SSE IS
cantaSSEM vendeSSEM partiSSEM SSE M

10
LÍNGUA PORTUGUESA

Futuro do Subjuntivo

Para formar o futuro do subjuntivo elimina-se a desinência -STE da 2ª pessoa do singular do pretérito perfeito, obtendo-
se, assim, o tema desse tempo. Acrescenta-se a esse tema a desinência temporal -R mais a desinência de número e pessoa
correspondente.

1ª conjugação 2ª conjugação 3ª conjugação Des. temporal Desinência pessoal


1ª /2ª e 3ª conj.
CANTAR VENDER PARTIR
cantaR vendeR partiR Ø
cantaRES vendeRES partiRES R ES
cantaR vendeR partiR R Ø
cantaRMOS vendeRMOS partiRMOS R MOS
cantaRDES vendeRDES partiRDES R DES
cantaREM vendeREM PartiREM R EM

Modo Imperativo

Imperativo Afirmativo

Para se formar o imperativo afirmativo, toma-se do presente do indicativo a 2ª pessoa do singular (tu) e a segunda
pessoa do plural (vós) eliminando-se o “S” final. As demais pessoas vêm, sem alteração, do presente do subjuntivo. Veja:

Presente do Indicativo Imperativo Afirmativo Presente do Subjuntivo


Eu canto --- Que eu cante
Tu cantas CantA tu Que tu cantes
Ele canta Cante você Que ele cante
Nós cantamos Cantemos nós Que nós cantemos
Vós cantais CantAI vós Que vós canteis
Eles cantam Cantem vocês Que eles cantem

Imperativo Negativo
Para se formar o imperativo negativo, basta antecipar a negação às formas do presente do subjuntivo.

Presente do Subjuntivo Imperativo Negativo


Que eu cante ---
Que tu cantes Não cantes tu
Que ele cante Não cante você
Que nós cantemos Não cantemos nós
Que vós canteis Não canteis vós
Que eles cantem Não cantem eles

Observações:

- No modo imperativo não faz sentido usar na 3ª pessoa (singular e plural) as formas ele/eles, pois uma ordem, pedido
ou conselho só se aplicam diretamente à pessoa com quem se fala. Por essa razão, utiliza-se você/vocês.
- O verbo SER, no imperativo, faz excepcionalmente: sê (tu), sede (vós).

Infinitivo Pessoal
1ª conjugação 2ª conjugação 3ª conjugação
CANTAR VENDER PARTIR
cantar vender partir
cantarES venderES partirES
cantar vender partir
cantarMOS venderMOS partirMOS
cantarDES venderDES partirDES
cantarEM venderEM partirEM

11
LÍNGUA PORTUGUESA

Questões sobre Verbo 05.(POLÍCIA MILITAR DO ESTADO DO ACRE – ALUNO


SOLDADO COMBATENTE – FUNCAB/2012) No trecho: “O
01. (Agente Polícia Vunesp 2013) Considere o trecho a crescimento econômico, se associado à ampliação do empre-
seguir. go, PODE melhorar o quadro aqui sumariamente descrito.”, se
É comum que objetos ___________ esquecidos em locais passarmos o verbo destacado para o futuro do pretérito do
públicos. Mas muitos transtornos poderiam ser evitados se as indicativo, teremos a forma:
pessoas _____________ a atenção voltada para seus pertences, A) puder.
conservando-os junto ao corpo. B) poderia.
Assinale a alternativa que preenche, correta e respecti- C) pôde.
vamente, as lacunas do texto. D) poderá.
(A) sejam … mantesse E) pudesse.
(B) sejam … mantivessem
(C) sejam … mantém 06. (Escrevente TJ SP Vunesp 2013) Assinale a alternativa
(D) seja … mantivessem em que todos os verbos estão empregados de acordo com a
(E) seja … mantêm norma-padrão.
(A) Enviaram o texto, para que o revíssemos antes da im-
02. (Escrevente TJ SP Vunesp 2012-adap.) Na frase –… os pressão definitiva.
níveis de pessoas sem emprego estão apresentando quedas (B) Não haverá prova do crime se o réu se manter em si-
sucessivas de 2005 para cá. –, a locução verbal em destaque lêncio.
expressa ação (C) Vão pagar horas-extras aos que se disporem a traba-
(A) concluída. lhar no feriado.
(B) atemporal. (D) Ficarão surpresos quando o verem com a toga...
(C) contínua. (E) Se você quer a promoção, é necessário que a requera
(D) hipotética. a seu superior.
(E) futura.
07. (Papiloscopista Policial Vunesp 2013-adap.) Assinale a
03. (Escrevente TJ SP Vunesp 2013-adap.) Sem querer alternativa que substitui, corretamente e sem alterar o sentido
estereotipar, mas já estereotipando: trata-se de um ser cujas da frase, a expressão destacada em – Se a criança se perder,
interações sociais terminam, 99% das vezes, diante da per- quem encontrá-la verá na pulseira instruções para que envie
gunta “débito ou crédito?”. uma mensagem eletrônica ao grupo ou acione o código na
Nesse contexto, o verbo estereotipar tem sentido de internet.
(A) considerar ao acaso, sem premeditação. (A) Caso a criança se havia perdido…
(B) aceitar uma ideia mesmo sem estar convencido dela. (B) Caso a criança perdeu…
(C) adotar como referência de qualidade. (C) Caso a criança se perca…
(D) julgar de acordo com normas legais. (D) Caso a criança estivera perdida…
(E) classificar segundo ideias preconcebidas. (E) Caso a criança se perda…

04. (Escrevente TJ SP Vunesp 2013) Assinale a alterna- 08. (Agente de Apoio Operacional – VUNESP – 2013-
tiva contendo a frase do texto na qual a expressão verbal adap.). Assinale a alternativa em que o verbo destacado está
destacada exprime possibilidade. no tempo futuro.
(A) ... o cientista Theodor Nelson sonhava com um sis- A) Os consumidores são assediados pelo marketing …
tema capaz de disponibilizar um grande número de obras B) … somente eles podem decidir se irão ou não comprar.
literárias... C) É como se abrissem em nós uma “caixa de necessida-
(B) Funcionando como um imenso sistema de informa- des”…
ção e arquivamento, o hipertexto deveria ser um enorme D) … de onde vem o produto…?
arquivo virtual. E) Uma pesquisa mostrou que 55,4% das pessoas…
(C) Isso acarreta uma textualidade que funciona por 09. (Papiloscopista Policial – VUNESP – 2013). Assinale a
associação, e não mais por sequências fixas previamente alternativa em que a concordância das formas verbais desta-
estabelecidas. cadas se dá em conformidade com a norma-padrão da língua.
(D) Desde o surgimento da ideia de hipertexto, esse (A) Chegou, para ajudar a família, vários amigos e vizinhos.
conceito está ligado a uma nova concepção de textuali- (B) Haviam várias hipóteses acerca do que poderia ter
dade... acontecido com a criança.
(E) Criou, então, o “Xanadu”, um projeto para disponibi- (C) Fazia horas que a criança tinha saído e os pais já esta-
lizar toda a literatura do mundo... vam preocupados.
(D) Era duas horas da tarde, quando a criança foi encon-
trada.
(E) Existia várias maneiras de voltar para casa, mas a crian-
ça se perdeu mesmo assim.

12
LÍNGUA PORTUGUESA

10. (Agente de Escolta e Vigilância Penitenciária – VU- 8-)


NESP – 2013-adap.). Leia as frases a seguir. A) Os consumidores são assediados pelo marketing =
I. Havia onze pessoas jogando pedras e pedaços de ma- presente
deira no animal. C) É como se abrissem em nós uma “caixa de necessi-
II. Existiam muitos ferimentos no boi. dades”… = pretérito do Subjuntivo
III. Havia muita gente assustando o boi numa avenida D) … de onde vem o produto…? = presente
movimentada. E) Uma pesquisa mostrou que 55,4% das pessoas… =
Substituindo-se o verbo Haver pelo verbo Existir e este pretérito perfeito
pelo verbo Haver, nas frases, têm-se, respectivamente:
A) Existia – Haviam – Existiam 9-)
B) Existiam – Havia – Existiam (A) Chegaram, para ajudar a família, vários amigos e
C) Existiam – Haviam – Existiam vizinhos.
D) Existiam – Havia – Existia (B) Havia várias hipóteses acerca do que poderia ter
E) Existia – Havia – Existia acontecido com a criança.
(D) Eram duas horas da tarde, quando a criança foi en-
GABARITO contrada.
(E) Existiam várias maneiras de voltar para casa, mas a
01. B 02. C 03. E 04. B 05. B criança se perdeu mesmo assim.
06. A 07. C 08. B 09. C 10. D
10-) I. Havia onze pessoas jogando pedras e pedaços
RESOLUÇÃO de madeira no animal.
II. Existiam muitos ferimentos no boi.
1-) É comum que objetos sejam esquecidos em III. Havia muita gente assustando o boi numa avenida
locais públicos. Mas muitos transtornos poderiam ser evi- movimentada.
tados se as pessoas mantivessem a atenção voltada para Haver – sentido de existir= invariável, impessoal;
seus pertences, conservando-os junto ao corpo. existir = variável. Portanto, temos:
I – Existiam onze pessoas...
2-) os níveis de pessoas sem emprego estão apresen- II – Havia muitos ferimentos...
tando quedas sucessivas de 2005 para cá. –, a locução ver- III – Existia muita gente...
bal em destaque expressa ação contínua (= não concluída)
Verbos irregulares são verbos que sofrem alterações
3-) Sem querer estereotipar, mas já estereotipando: tra- em seu radical ou em suas desinências, afastando-se do
ta-se de um ser cujas interações sociais terminam, 99% das modelo a que pertencem.
vezes, diante da pergunta “débito ou crédito?”. No português, para verificar se um verbo sofre altera-
Nesse contexto, o verbo estereotipar tem sentido de ções, basta conjugá-lo no presente e no pretérito perfeito
classificar segundo ideias preconcebidas. do indicativo. Ex: faço – fiz, trago – trouxe, posso - pude.
Não é considerada irregularidade a alteração gráfica
4-) (B) Funcionando como um imenso sistema de infor- do radical de certos verbos para conservação da regulari-
mação e arquivamento, o hipertexto deveria ser um enor- dade fônica. Ex: embarcar – embarco, fingir – finjo.
me arquivo virtual. = verbo no futuro do pretérito
Exemplo de conjugação do verbo “dar” no presente
5-) Conjugando o verbo “poder” no futuro do pretérito do indicativo:
do Indicativo: eu poderia, tu poderias, ele poderia, nós po- Eu dou
deríamos, vós poderíeis, eles poderiam. O sujeito da oração Tu dás
é crescimento econômico (singular), portanto, terceira pes- Ele dá
soa do singular (ele) = poderia. Nós damos
6-) Vós dais
(B) Não haverá prova do crime se o réu se mantiver em Eles dão
silêncio.
(C) Vão pagar horas-extras aos que se dispuserem a tra- Percebe-se que há alteração do radical, afastando-se
balhar no feriado. do original “dar” durante a conjugação, sendo considera-
(D) Ficarão surpresos quando o virem com a toga... do verbo irregular.
(E) Se você quiser a promoção, é necessário que a re- Exemplo: Conjugação do verbo valer:
queira a seu superior.

7-) Caso a criança se perca…(perda = substantivo: Hou-


ve uma grande perda salarial...)

13
LÍNGUA PORTUGUESA

Modo Indicativo Gerúndio do verbo valer = valendo


Presente
eu valho Modo Subjuntivo
tu vales
ele vale Presente
nós valemos que eu valha
vós valeis que tu valhas
eles valem que ele valha
que nós valhamos
Pretérito Perfeito do Indicativo que vós valhais
eu vali que eles valham
tu valeste
ele valeu Pretérito Imperfeito do Subjuntivo
nós valemos se eu valesse
vós valestes se tu valesses
eles valeram se ele valesse
se nós valêssemos
Pretérito Imperfeito do Indicativo se vós valêsseis
eu valia se eles valessem
tu valias
ele valia Futuro do Subjuntivo
nós valíamos quando eu valer
vós valíeis quando tu valeres
eles valiam quando ele valer
quando nós valermos
Pretérito Mais-que-perfeito do Indicativo quando vós valerdes
eu valera quando eles valerem
tu valeras
Imperativo
ele valera
Imperativo Afirmativo
nós valêramos
--
vós valêreis
vale tu
eles valeram
valha ele
valhamos nós
Futuro do Presente do Indicativo
valei vós
eu valerei
valham eles
tu valerás
ele valerá Imperativo Negativo
nós valeremos --
vós valereis não valhas tu
eles valerão não valha ele
não valhamos nós
Futuro do Pretérito do Indicativo não valhais vós
eu valeria não valham eles
tu valerias
ele valeria Infinitivo
nós valeríamos Infinitivo Pessoal
vós valeríeis por valer eu
eles valeriam por valeres tu
por valer ele
Mais-que-perfeito Composto do Indicativo por valermos nós
eu tinha valido por valerdes vós
tu tinhas valido por valerem eles
ele tinha valido
nós tínhamos valido
vós tínheis valido
eles tinham valido

14
LÍNGUA PORTUGUESA

Infinitivo Impessoal = valer Vozes do Verbo

Particípio = Valido Dá-se o nome de voz à forma assumida pelo verbo para
indicar se o sujeito gramatical é agente ou paciente da
Acompanhe abaixo uma lista com os principais verbos ação. São três as vozes verbais:
irregulares:
- Ativa: quando o sujeito é agente, isto é, pratica a ação
Dizer expressa pelo verbo. Por exemplo:
Presente do indicativo: Digo, dizes, diz, dizemos, di- Ele fez o trabalho.
zeis, dizem. sujeito agente ação objeto (paciente)
Pretérito perfeito do indicativo: Disse, disseste, disse,
dissemos, dissestes, disseram. - Passiva: quando o sujeito é paciente, recebendo a
Futuro do presente do indicativo: Direi, dirás, dirá, ação expressa pelo verbo. Por exemplo:
diremos, direis, dirão. O trabalho foi feito por ele.
sujeito paciente ação agente da passiva
Fazer
Presente do indicativo: Faço, fazes, faz, fazemos, fa- - Reflexiva: quando o sujeito é ao mesmo tempo agen-
zeis, fazem. te e paciente, isto é, pratica e recebe a ação. Por exemplo:
Pretérito perfeito do indicativo: Fiz, fizeste, fez, fize- O menino feriu-se.
mos, fizestes, fizeram.
Futuro do presente do indicativo: Farei, farás, fará, Obs.: não confundir o emprego reflexivo do verbo com
faremos, fareis, farão. a noção de reciprocidade: Os lutadores feriram-se. (um ao
outro)
Ir
Presente do indicativo: Vou, vais, vai, vamos, ides, vão. Formação da Voz Passiva
Pretérito perfeito do indicativo: Fui, foste, foi, fomos,
fostes, foram. A voz passiva pode ser formada por dois processos:
Futuro do presente do indicativo: Irei, irás, irá, ire- analítico e sintético.
mos, ireis, irão.
Futuro do subjuntivo: For, fores, for, formos, fordes, 1- Voz Passiva Analítica
forem. Constrói-se da seguinte maneira: Verbo SER + particípio
do verbo principal. Por exemplo:
Querer A escola será pintada.
Presente do indicativo: Quero, queres, quer, queremos, O trabalho é feito por ele.
quereis, querem.
Pretérito perfeito do indicativo: Quis, quiseste, quis, Obs.: o agente da passiva geralmente é acompanhado
quisemos, quisestes, quiseram. da preposição por, mas pode ocorrer a construção com a
Presente do subjuntivo: Queira, queiras, queira, quei- preposição de. Por exemplo: A casa ficou cercada de solda-
ramos, queirais, queiram. dos.
- Pode acontecer ainda que o agente da passiva não
Ver esteja explícito na frase: A exposição será aberta amanhã.
Presente do indicativo: Vejo, vês, vê, vemos, vedes, - A variação temporal é indicada pelo verbo auxiliar
veem. (SER), pois o particípio é invariável. Observe a transforma-
Pretérito perfeito do indicativo: Vi, viste, viu, vimos, ção das frases seguintes:
vistes, viram. a) Ele fez o trabalho. (pretérito perfeito do indicativo)
Futuro do presente do indicativo:Verei, verás, verá, O trabalho foi feito por ele. (pretérito perfeito do indi-
veremos, vereis, verão. cativo)
Futuro do subjuntivo: Vir, vires, vir, virmos, virdes, vi-
rem. b) Ele faz o trabalho. (presente do indicativo)
O trabalho é feito por ele. (presente do indicativo)
Vir
Presente do indicativo: Venho, vens, vem, vimos, vin- c) Ele fará o trabalho. (futuro do presente)
des, vêm. O trabalho será feito por ele. (futuro do presente)
Pretérito perfeito do indicativo: Vim, vieste, veio, vie-
mos, viestes, vieram. - Nas frases com locuções verbais, o verbo SER assume
Futuro do presente do indicativo: Virei, virás, virá, vi- o mesmo tempo e modo do verbo principal da voz ativa.
remos, vireis, virão. Observe a transformação da frase seguinte:
Futuro do subjuntivo: Vier, vieres, vier, viermos, vier- O vento ia levando as folhas. (gerúndio)
des, vierem. As folhas iam sendo levadas pelo vento. (gerúndio)

15
LÍNGUA PORTUGUESA

Obs.: é menos frequente a construção da voz passiva - Inversamente, usamos formas ativas com sentido pas-
analítica com outros verbos que podem eventualmente sivo:
funcionar como auxiliares. Por exemplo: A moça ficou mar- Há coisas difíceis de entender. (= serem entendidas)
cada pela doença. Mandou-o lançar na prisão. (= ser lançado)

2- Voz Passiva Sintética - Os verbos chamar-se, batizar-se, operar-se (no sentido


A voz passiva sintética ou pronominal constrói-se com cirúrgico) e vacinar-se são considerados passivos, logo o
o verbo na 3ª pessoa, seguido do pronome apassivador SE. sujeito é paciente.
Por exemplo: Chamo-me Luís.
Abriram-se as inscrições para o concurso. Batizei-me na Igreja do Carmo.
Destruiu-se o velho prédio da escola. Operou-se de hérnia.
Obs.: o agente não costuma vir expresso na voz passiva Vacinaram-se contra a gripe.
sintética.
Fonte: http://www.soportugues.com.br/secoes/morf/
morf54.php
Curiosidade: A palavra passivo possui a mesma raiz la-
tina de paixão (latim passio, passionis) e ambas se relacio-
Questões sobre Vozes dos Verbos
nam com o significado sofrimento, padecimento. Daí vem
o significado de voz passiva como sendo a voz que expres- 01. (COLÉGIO PEDRO II/RJ – ASSISTENTE EM ADMINIS-
sa a ação sofrida pelo sujeito. Na voz passiva temos dois TRAÇÃO – AOCP/2010) Em “Os dados foram divulgados on-
elementos que nem sempre aparecem: SUJEITO PACIENTE tem pelo Instituto Sou da Paz.”, a expressão destacada é
e AGENTE DA PASSIVA. (A) adjunto adnominal.
(B) sujeito paciente.
Conversão da Voz Ativa na Voz Passiva (C) objeto indireto.
(D) complemento nominal.
Pode-se mudar a voz ativa na passiva sem alterar subs- (E) agente da passiva.
tancialmente o sentido da frase.
Gutenberg inventou a imprensa (Voz Ativa) 02. (FCC-COPERGÁS – Auxiliar Técnico Administrativo -
Sujeito da Ativa objeto Direto 2011) Um dia um tufão furibundo abateu-o pela raiz. Trans-
pondo- -se a frase acima para a voz passiva, a forma
A imprensa foi inventada por Gutenberg (Voz Passiva) verbal resultante será:
Sujeito da Passiva Agente da Passiva (A) era abatido.
(B) fora abatido.
Observe que o objeto direto será o sujeito da passiva, o (C) abatera-se.
sujeito da ativa passará a agente da passiva e o verbo ativo (D) foi abatido.
assumirá a forma passiva, conservando o mesmo tempo. (E) tinha abatido
Observe mais exemplos:
- Os mestres têm constantemente aconselhado os alunos. 03. (TRE/AL – TÉCNICO JUDICIÁRIO – FCC/2010)
Os alunos têm sido constantemente aconselhados pelos ... valores e princípios que sejam percebidos pela socie-
mestres. dade como tais.
- Eu o acompanharei. Transpondo para a voz ativa a frase acima, o verbo pas-
Ele será acompanhado por mim. sará a ser, corretamente,
(A) perceba.
(B) foi percebido.
Obs.: quando o sujeito da voz ativa for indeterminado,
(C) tenham percebido.
não haverá complemento agente na passiva. Por exemplo:
(D) devam perceber.
Prejudicaram-me. / Fui prejudicado.
(E) estava percebendo.
Saiba que: 04. (TJ/RJ – TÉCNICO DE ATIVIDADE JUDICIÁRIA SEM
- Aos verbos que não são ativos nem passivos ou refle- ESPECIALIDADE – FCC/2012) As ruas estavam ocupadas
xivos, são chamados neutros. pela multidão...
O vinho é bom. A forma verbal resultante da transposição da frase aci-
Aqui chove muito. ma para a voz ativa é:
(A) ocupava-se.
- Há formas passivas com sentido ativo: (B) ocupavam.
É chegada a hora. (= Chegou a hora.) (C) ocupou.
Eu ainda não era nascido. (= Eu ainda não tinha nascido.) (D) ocupa.
És um homem lido e viajado. (= que leu e viajou) (E) ocupava.

16
LÍNGUA PORTUGUESA

05. (TRF - 5ª REGIÃO - TÉCNICO JUDICIÁRIO - FCC/2012) A GABARITO


frase que NÃO admite transposição para a voz passiva está em:
(A) Quando Rodolfo surgiu... 01. E 02. D 03. A 04. E 05. A
(B) ... adquiriu as impressoras... 06. B 07. C 08. D 09. A 10. D
(C) ... e sustentar, às vezes, família numerosa.
(D) ... acolheu-o como patrono. RESOLUÇÃO
(E) ... que montou [...] a primeira grande folhetaria do Recife ...
1-) No enunciado temos uma oração com a voz passiva
06. (TRF - 4ª REGIÃO – TÉCNICO JUDICIÁRIO – FCC/2010) O do verbo. Transformando-a em ativa, teremos: “O Instituto
engajamento moral e político não chegou a constituir um deslo- Sou da Paz divulgou dados”. Nessa, “Instituto Sou da Paz”
camento da atenção intelectual de Said ... funciona como sujeito da oração, ou seja, na passiva sua
Transpondo-se a frase acima para a voz passiva, a forma função é a de agente da passiva. O sujeito paciente é “os
verbal resultante é: dados”.
a) se constituiu.
b) chegou a ser constituído. 2-) Um dia um tufão furibundo abateu-o pela raiz. = Ele
c) teria chegado a constituir. foi abatido...
d) chega a se constituir.
e) chegaria a ser constituído. 3-) ... valores e princípios que sejam percebidos pela
sociedade como tais = dois verbos na voz passiva, então
07. (METRÔ/SP – TÉCNICO SISTEMAS METROVIÁRIOS CI- teremos um na ativa: que a sociedade perceba os valores
VIL – FCC/2014 - ADAPTADA) ...’sertanejo’ indicava indistinta- e princípios...
mente as músicas produzidas no interior do país...
Transpondo-se a frase acima para a voz passiva, a forma 4-) As ruas estavam ocupadas pela multidão = dois ver-
verbal resultante será: bos na passiva, um verbo na ativa:
(A) vinham indicadas. A multidão ocupava as ruas.
(B) era indicado.
(C) eram indicadas. 5-)
(D) tinha indicado. B = as impressoras foram adquiridas...
(E) foi indicada. C = família numerosa é sustentada...
08. (GOVERNO DO ESTADO DO RIO DE JANEIRO – PRO- D – foi acolhido como patrono...
CON – AGENTE ADMINISTRATIVO – CEPERJ/2012 - adaptada) E – a primeira grande folhetaria do Recife foi montada...
Um exemplo de construção na voz passiva está em: 6-) O engajamento moral e político não chegou a consti-
(A) “A Gulliver recolherá 6 mil brinquedos” tuir um deslocamento da atenção intelectual de Said = dois
(B) “o consumidor pode solicitar a devolução do dinheiro” verbos na voz ativa, mas com presença de preposição e, um
(C) “enviar o brinquedo por sedex” deles, no infinitivo, então o verbo auxiliar “ser” ficará no in-
(D) “A empresa também é obrigada pelo Código de Defesa finitivo (na voz passiva) e o verbo principal (constituir) ficará
do Consumidor” no particípio: Um deslocamento da atenção intelectual de
(E) “A empresa fez campanha para recolher” Said não chegou a ser constituído pelo engajamento...

09. (METRÔ/SP –SECRETÁRIA PLENO – FCC/2010) Trans- 7-)’sertanejo’ indicava indistintamente as músicas pro-
pondo-se para a voz passiva a construção Mais tarde vim a duzidas no interior do país.
entender a tradução completa, a forma verbal resultante será: As músicas produzidas no país eram indicadas pelo ser-
(A) veio a ser entendida. tanejo, indistintamente.
(B) teria entendido.
(C) fora entendida. 8-)
(D) terá sido entendida. (A) “A Gulliver recolherá 6 mil brinquedos” = voz ativa
(E) tê-la-ia entendido. (B) “o consumidor pode solicitar a devolução do dinhei-
ro” = voz ativa
10. (INFRAERO – CADASTRO RESERVA OPERACIONAL (C) “enviar o brinquedo por sedex” = voz ativa
PROFISSIONAL DE TRÁFEGO AÉREO – FCC/2011 - ADAPTADA) (D) “A empresa também é obrigada pelo Código de De-
... ele empreende, de maneira quase clandestina, a série Mu- fesa do Consumidor” = voz passiva
lheres. (E) “A empresa fez campanha para recolher” = voz ativa
Transpondo-se a frase acima para a voz passiva, a forma
verbal resultante será: 9-)Mais tarde vim a entender a tradução completa...
(A) foi empreendida. A tradução completa veio a ser entendida por mim.
(B) são empreendidos. 10-) ele empreende, de maneira quase clandestina, a sé-
(C) foi empreendido. rie Mulheres.
(D) é empreendida. A série de mulheres é empreendida por ele, de maneira
(E) são empreendidas. quase clandestina.

17
LÍNGUA PORTUGUESA

Substantivo Concreto: é aquele que designa o ser que


3. FLEXÃO DE GÊNERO, NÚMERO E GRAU; existe, independentemente de outros seres.
Obs.: os substantivos concretos designam seres do
mundo real e do mundo imaginário.
SUBSTANTIVO Seres do mundo real: homem, mulher, cadeira, cobra,
Brasília, etc.
Tudo o que existe é ser e cada ser tem um nome. Subs- Seres do mundo imaginário: saci, mãe-d’água, fantas-
tantivo é a classe gramatical de palavras variáveis, as quais ma, etc.
denominam os seres. Além de objetos, pessoas e fenôme-
nos, os substantivos também nomeiam: Observe agora:
Beleza exposta
-lugares: Alemanha, Porto Alegre... Jovens atrizes veteranas destacam-se pelo visual.
-sentimentos: raiva, amor...
-estados: alegria, tristeza... O substantivo beleza designa uma qualidade.
-qualidades: honestidade, sinceridade...
-ações: corrida, pescaria... Substantivo Abstrato: é aquele que designa seres que
dependem de outros para se manifestar ou existir.
Pense bem: a beleza não existe por si só, não pode ser
Morfossintaxe do substantivo observada. Só podemos observar a beleza numa pessoa
ou coisa que seja bela. A beleza depende de outro ser para
Nas orações de língua portuguesa, o substantivo em ge- se manifestar. Portanto, a palavra beleza é um substantivo
ral exerce funções diretamente relacionadas com o verbo: abstrato.
atua como núcleo do sujeito, dos complementos verbais Os substantivos abstratos designam estados, qualida-
(objeto direto ou indireto) e do agente da passiva. Pode des, ações e sentimentos dos seres, dos quais podem ser
ainda funcionar como núcleo do complemento nominal ou abstraídos, e sem os quais não podem existir: vida (estado),
do aposto, como núcleo do predicativo do sujeito, do ob- rapidez (qualidade), viagem (ação), saudade (sentimento).
jeto ou como núcleo do vocativo. Também encontramos
substantivos como núcleos de adjuntos adnominais e de
3 - Substantivos Coletivos
adjuntos adverbiais - quando essas funções são desempe-
nhadas por grupos de palavras.
Ele vinha pela estrada e foi picado por uma abelha, outra
abelha, mais outra abelha.
Classificação dos Substantivos
Ele vinha pela estrada e foi picado por várias abelhas.
Ele vinha pela estrada e foi picado por um enxame.
1- Substantivos Comuns e Próprios
Observe a definição: s.f. 1: Povoação maior que vila, com
muitas casas e edifícios, dispostos em ruas e avenidas (no Note que, no primeiro caso, para indicar plural, foi ne-
Brasil, toda a sede de município é cidade). 2. O centro de cessário repetir o substantivo: uma abelha, outra abelha,
uma cidade (em oposição aos bairros). mais outra abelha...
No segundo caso, utilizaram-se duas palavras no plural.
Qualquer “povoação maior que vila, com muitas casas No terceiro caso, empregou-se um substantivo no sin-
e edifícios, dispostos em ruas e avenidas” será chamada gular (enxame) para designar um conjunto de seres da
cidade. Isso significa que a palavra cidade é um substantivo mesma espécie (abelhas).
comum.
Substantivo Comum é aquele que designa os seres de O substantivo enxame é um substantivo coletivo.
uma mesma espécie de forma genérica: cidade, menino,
homem, mulher, país, cachorro. Substantivo Coletivo: é o substantivo comum que, mes-
Estamos voando para Barcelona. mo estando no singular, designa um conjunto de seres da
mesma espécie.
O substantivo Barcelona designa apenas um ser da es-
pécie cidade. Esse substantivo é próprio. Substantivo Pró- Substantivo coletivo Conjunto de:
prio: é aquele que designa os seres de uma mesma espécie assembleia pessoas reunidas
de forma particular: Londres, Paulinho, Pedro, Tietê, Brasil. alcateia lobos
acervo livros
2 - Substantivos Concretos e Abstratos antologia trechos literários selecionados
arquipélago ilhas
LÂMPADA MALA banda músicos
bando desordeiros ou malfeitores
Os substantivos lâmpada e mala designam seres com banca examinadores
existência própria, que são independentes de outros seres. batalhão soldados
São substantivos concretos. cardume peixes

18
LÍNGUA PORTUGUESA

caravana viajantes peregrinos Substantivo Composto: é aquele formado por dois ou


cacho frutas mais elementos. Outros exemplos: beija-flor, passatempo.
cáfila camelos
cancioneiro canções, poesias líricas Substantivos Primitivos e Derivados
colmeia abelhas Meu limão meu limoeiro,
chusma gente, pessoas meu pé de jacarandá...
concílio bispos
congresso parlamentares, cientistas. O substantivo limão é primitivo, pois não se originou de
elenco atores de uma peça ou filme nenhum outro dentro de língua portuguesa.
esquadra navios de guerra
enxoval roupas Substantivo Primitivo: é aquele que não deriva de ne-
falange soldados, anjos nhuma outra palavra da própria língua portuguesa. O subs-
fauna animais de uma região tantivo limoeiro é derivado, pois se originou a partir da pa-
feixe lenha, capim lavra limão.
flora vegetais de uma região
frota navios mercantes, ônibus Substantivo Derivado: é aquele que se origina de outra
girândola fogos de artifício palavra.
horda bandidos, invasores
junta médicos, bois, credores, examinadores Flexão dos substantivos
júri jurados
legião soldados, anjos, demônios O substantivo é uma classe variável. A palavra é variável
leva presos, recrutas quando sofre flexão (variação). A palavra menino, por exem-
malta malfeitores ou desordeiros plo, pode sofrer variações para indicar:
manada búfalos, bois, elefantes, Plural: meninos Feminino: menina
matilha cães de raça Aumentativo: meninão Diminutivo: menininho
Flexão de Gênero
molho chaves, verduras
multidão pessoas em geral
Gênero é a propriedade que as palavras têm de indicar
ninhada pintos
sexo real ou fictício dos seres. Na língua portuguesa, há dois
nuvem insetos (gafanhotos, mosquitos, etc.)
gêneros: masculino e feminino. Pertencem ao gênero mas-
penca bananas, chaves
culino os substantivos que podem vir precedidos dos artigos
pinacoteca pinturas, quadros
o, os, um, uns. Veja estes títulos de filmes:
quadrilha ladrões, bandidos O velho e o mar
ramalhete flores Um Natal inesquecível
rebanho ovelhas Os reis da praia
récua bestas de carga, cavalgadura
repertório peças teatrais, obras musicais Pertencem ao gênero feminino os substantivos que po-
réstia alhos ou cebolas dem vir precedidos dos artigos a, as, uma, umas:
romanceiro poesias narrativas A história sem fim
revoada pássaros Uma cidade sem passado
sínodo párocos As tartarugas ninjas
talha lenha Substantivos Biformes e Substantivos Uniformes
tropa muares, soldados
turma estudantes, trabalhadores Substantivos Biformes (= duas formas): ao indicar nomes
vara porcos de seres vivos, geralmente o gênero da palavra está relacio-
nado ao sexo do ser, havendo, portanto, duas formas, uma
Formação dos Substantivos para o masculino e outra para o feminino. Observe: gato –
gata, homem – mulher, poeta – poetisa, prefeito - prefeita
Substantivos Simples e Compostos
Chuva - subst. Fem. 1 - água caindo em gotas sobre a Substantivos Uniformes: são aqueles que apresentam
terra. uma única forma, que serve tanto para o masculino quanto
O substantivo chuva é formado por um único elemento para o feminino. Classificam-se em:
ou radical. É um substantivo simples. - Epicenos: têm um só gênero e nomeiam bichos: a co-
bra macho e a cobra fêmea, o jacaré macho e o jacaré fêmea.
Substantivo Simples: é aquele formado por um único - Sobrecomuns: têm um só gênero e nomeiam pessoas:
elemento. a criança, a testemunha, a vítima, o cônjuge, o gênio, o ídolo,
Outros substantivos simples: tempo, sol, sofá, etc. Veja o indivíduo.
agora: O substantivo guarda-chuva é formado por dois - Comuns de Dois Gêneros: indicam o sexo das pes-
elementos (guarda + chuva). Esse substantivo é composto. soas por meio do artigo: o colega e a colega, o doente e a
doente, o artista e a artista.

19
LÍNGUA PORTUGUESA

Saiba que: Substantivos de origem grega terminados Sobrecomuns:


em ema ou oma, são masculinos: o fonema, o poema, o Entregue as crianças à natureza.
sistema, o sintoma, o teorema.
- Existem certos substantivos que, variando de gênero, A palavra crianças refere-se tanto a seres do sexo mas-
variam em seu significado: o rádio (aparelho receptor) e a culino, quanto a seres do sexo feminino. Nesse caso, nem
rádio (estação emissora) o capital (dinheiro) e a capital (ci- o artigo nem um possível adjetivo permitem identificar o
dade) sexo dos seres a que se refere a palavra. Veja:
A criança chorona chamava-se João.
Formação do Feminino dos Substantivos Biformes A criança chorona chamava-se Maria.

- Regra geral: troca-se a terminação -o por –a: aluno - Outros substantivos sobrecomuns:
aluna. a criatura = João é uma boa criatura. Maria é uma boa
- Substantivos terminados em -ês: acrescenta-se -a ao criatura.
masculino: freguês - freguesa o cônjuge = O cônjuge de João faleceu. O cônjuge de
- Substantivos terminados em -ão: fazem o feminino de Marcela faleceu
três formas:
- troca-se -ão por -oa. = patrão – patroa Comuns de Dois Gêneros:
- troca-se -ão por -ã. = campeão - campeã Motorista tem acidente idêntico 23 anos depois.
-troca-se -ão por ona. = solteirão - solteirona
Quem sofreu o acidente: um homem ou uma mulher?
Exceções: barão – baronesa ladrão- ladra sultão - É impossível saber apenas pelo título da notícia, uma
sultana vez que a palavra motorista é um substantivo uniforme.
A distinção de gênero pode ser feita através da análise
- Substantivos terminados em -or: do artigo ou adjetivo, quando acompanharem o substanti-
- acrescenta-se -a ao masculino = doutor – doutora vo: o colega - a colega; o imigrante - a imigrante; um jovem
- troca-se -or por -triz: = imperador - imperatriz - uma jovem; artista famoso - artista famosa; repórter fran-
- Substantivos com feminino em -esa, -essa, -isa: cônsul cês - repórter francesa
- consulesa / abade - abadessa / poeta - poetisa / duque -
duquesa / conde - condessa / profeta - profetisa - A palavra personagem é usada indistintamente nos
dois gêneros.
- Substantivos que formam o feminino trocando o -e a) Entre os escritores modernos nota-se acentuada pre-
final por -a: elefante - elefanta ferência pelo masculino: O menino descobriu nas nuvens os
personagens dos contos de carochinha.
- Substantivos que têm radicais diferentes no masculino b) Com referência a mulher, deve-se preferir o feminino:
O problema está nas mulheres de mais idade, que não acei-
e no feminino: bode – cabra / boi - vaca
tam a personagem.
- Diz-se: o (ou a) manequim Marcela, o (ou a) modelo
- Substantivos que formam o feminino de maneira es-
fotográfico Ana Belmonte.
pecial, isto é, não seguem nenhuma das regras anteriores:
Observe o gênero dos substantivos seguintes:
czar – czarina réu - ré
Masculinos: o tapa, o eclipse, o lança-perfume, o dó
Formação do Feminino dos Substantivos Uniformes
(pena), o sanduíche, o clarinete, o champanha, o sósia, o
Epicenos:
maracajá, o clã, o hosana, o herpes, o pijama, o suéter, o
Novo jacaré escapa de policiais no rio Pinheiros. soprano, o proclama, o pernoite, o púbis.
Não é possível saber o sexo do jacaré em questão. Isso Femininos: a dinamite, a derme, a hélice, a omoplata, a
ocorre porque o substantivo jacaré tem apenas uma forma cataplasma, a pane, a mascote, a gênese, a entorse, a libido,
para indicar o masculino e o feminino. a cal, a faringe, a cólera (doença), a ubá (canoa).
Alguns nomes de animais apresentam uma só forma
para designar os dois sexos. Esses substantivos são cha- - São geralmente masculinos os substantivos de ori-
mados de epicenos. No caso dos epicenos, quando houver gem grega terminados em -ma: o grama (peso), o quilo-
a necessidade de especificar o sexo, utilizam-se palavras grama, o plasma, o apostema, o diagrama, o epigrama, o
macho e fêmea. telefonema, o estratagema, o dilema, o teorema, o trema, o
A cobra macho picou o marinheiro. eczema, o edema, o magma, o estigma, o axioma, o traco-
A cobra fêmea escondeu-se na bananeira. ma, o hematoma.

Exceções: a cataplasma, a celeuma, a fleuma, etc.

20
LÍNGUA PORTUGUESA

Gênero dos Nomes de Cidades: - Os substantivos terminados em al, el, ol, ul flexionam-
se no plural, trocando o “l” por “is”: quintal - quintais; cara-
Com raras exceções, nomes de cidades são femininos. col – caracóis; hotel - hotéis. Exceções: mal e males, cônsul
e cônsules.
A histórica Ouro Preto.
A dinâmica São Paulo. - Os substantivos terminados em “il” fazem o plural de
A acolhedora Porto Alegre. duas maneiras:
Uma Londres imensa e triste. - Quando oxítonos, em “is”: canil - canis
Exceções: o Rio de Janeiro, o Cairo, o Porto, o Havre. - Quando paroxítonos, em “eis”: míssil - mísseis.
Obs.: a palavra réptil pode formar seu plural de duas
Gênero e Significação: maneiras: répteis ou reptis (pouco usada).
- Os substantivos terminados em “s” fazem o plural de
Muitos substantivos têm uma significação no masculino duas maneiras:
e outra no feminino. Observe: o baliza (soldado que, que à - Quando monossilábicos ou oxítonos, mediante o
frente da tropa, indica os movimentos que se deve realizar acréscimo de “es”: ás – ases / retrós - retroses
em conjunto; o que vai à frente de um bloco carnavalesco, - Quando paroxítonos ou proparoxítonos, ficam inva-
manejando um bastão), a baliza (marco, estaca; sinal que riáveis: o lápis - os lápis / o ônibus - os ônibus.
marca um limite ou proibição de trânsito), o cabeça (chefe),
a cabeça (parte do corpo), o cisma (separação religiosa, dissi- - Os substantivos terminados em “ao” fazem o plural de
dência), a cisma (ato de cismar, desconfiança), o cinza (a cor três maneiras.
cinzenta), a cinza (resíduos de combustão), o capital (dinhei- - substituindo o -ão por -ões: ação - ações
ro), a capital (cidade), o coma (perda dos sentidos), a coma - substituindo o -ão por -ães: cão - cães
(cabeleira), o coral (pólipo, a cor vermelha, canto em coro), - substituindo o -ão por -ãos: grão - grãos
a coral (cobra venenosa), o crisma (óleo sagrado, usado na
administração da crisma e de outros sacramentos), a crisma - Os substantivos terminados em “x” ficam invariáveis:
(sacramento da confirmação), o cura (pároco), a cura (ato de o látex - os látex.
curar), o estepe (pneu sobressalente), a estepe (vasta planície
de vegetação), o guia (pessoa que guia outras), a guia (docu-
Plural dos Substantivos Compostos
mento, pena grande das asas das aves), o grama (unidade de
peso), a grama (relva), o caixa (funcionário da caixa), a caixa
-A formação do plural dos substantivos compostos de-
(recipiente, setor de pagamentos), o lente (professor), a lente
pende da forma como são grafados, do tipo de palavras
(vidro de aumento), o moral (ânimo), a moral (honestidade,
que formam o composto e da relação que estabelecem en-
bons costumes, ética), o nascente (lado onde nasce o Sol), a
tre si. Aqueles que são grafados sem hífen comportam-se
nascente (a fonte), o maria-fumaça (trem como locomotiva
como os substantivos simples: aguardente/aguardentes,
a vapor), maria-fumaça (locomotiva movida a vapor), o pala
girassol/girassóis, pontapé/pontapés, malmequer/
(poncho), a pala (parte anterior do boné ou quepe, antepa-
ro), o rádio (aparelho receptor), a rádio (estação emissora), o malmequeres.
voga (remador), a voga (moda, popularidade). O plural dos substantivos compostos cujos elementos
Flexão de Número do Substantivo são ligados por hífen costuma provocar muitas dúvidas e
discussões. Algumas orientações são dadas a seguir:
Em português, há dois números gramaticais: o singular,
que indica um ser ou um grupo de seres, e o plural, que
indica mais de um ser ou grupo de seres. A característica - Flexionam-se os dois elementos, quando formados
do plural é o “s” final. de:
substantivo + substantivo = couve-flor e couves-flores
Plural dos Substantivos Simples substantivo + adjetivo = amor-perfeito e amores-per-
feitos
- Os substantivos terminados em vogal, ditongo oral e adjetivo + substantivo = gentil-homem e gentis-homens
“n” fazem o plural pelo acréscimo de “s”: pai – pais; ímã – numeral + substantivo = quinta-feira e quintas-feiras
ímãs; hífen - hifens (sem acento, no plural). Exceção: cânon
- cânones. - Flexiona-se somente o segundo elemento, quando
formados de:
- Os substantivos terminados em “m” fazem o plural em verbo + substantivo = guarda-roupa e guarda-roupas
“ns”: homem - homens. palavra invariável + palavra variável = alto-falante e
alto- -falantes
- Os substantivos terminados em “r” e “z” fazem o plural palavras repetidas ou imitativas = reco-reco e reco-recos
pelo acréscimo de “es”: revólver – revólveres; raiz - raízes.

Atenção: O plural de caráter é caracteres.

21
LÍNGUA PORTUGUESA

- Flexiona-se somente o primeiro elemento, quando Plural dos Nomes Próprios Personativos
formados de:
substantivo + preposição clara + substantivo = água- Devem-se pluralizar os nomes próprios de pessoas sem-
de-colônia e águas-de-colônia pre que a terminação preste-se à flexão.
substantivo + preposição oculta + substantivo = cava- Os Napoleões também são derrotados.
lo-vapor e cavalos-vapor As Raquéis e Esteres.
substantivo + substantivo que funciona como determi-
nante do primeiro, ou seja, especifica a função ou o tipo do Plural dos Substantivos Estrangeiros
termo anterior:
palavra-chave - palavras-chave, bomba-relógio - bom- Substantivos ainda não aportuguesados devem ser es-
bas-relógio, notícia-bomba - notícias-bomba, homem-rã - critos como na língua original, acrescentando-se “s” (exceto
homens-rã, peixe- -espada - peixes-espada. quando terminam em “s” ou “z”): os shows, os shorts, os jazz.

- Permanecem invariáveis, quando formados de: Substantivos já aportuguesados flexionam-se de acordo


verbo + advérbio = o bota-fora e os bota-fora com as regras de nossa língua: os clubes, os chopes, os jipes,
verbo + substantivo no plural = o saca-rolhas e os sa- os esportes, as toaletes, os bibelôs, os garçons, os réquiens.
ca-rolhas Observe o exemplo:
- Casos Especiais Este jogador faz gols toda vez que joga.
o louva-a-deus e os louva-a-deus O plural correto seria gois (ô), mas não se usa.
o bem-te-vi e os bem-te-vis
o bem-me-quer e os bem-me-queres Plural com Mudança de Timbre
o joão-ninguém e os joões-ninguém.
Certos substantivos formam o plural com mudança de
timbre da vogal tônica (o fechado / o aberto). É um fato fo-
Plural das Palavras Substantivadas
nético chamado metafonia (plural metafônico).
As palavras substantivadas, isto é, palavras de outras
Singular Plural
classes gramaticais usadas como substantivo, apresentam,
corpo (ô) corpos (ó)
no plural, as flexões próprias dos substantivos.
esforço esforços
Pese bem os prós e os contras.
fogo fogos
O aluno errou na prova dos noves. forno fornos
Ouça com a mesma serenidade os sins e os nãos. fosso fossos
imposto impostos
Obs.: numerais substantivados terminados em “s” ou olho olhos
“z” não variam no plural: Nas provas mensais consegui mui- osso (ô) ossos (ó)
tos seis e alguns dez. ovo ovos
poço poços
Plural dos Diminutivos porto portos
posto postos
Flexiona-se o substantivo no plural, retira-se o “s” final tijolo tijolos
e acrescenta-se o sufixo diminutivo.
pãe(s) + zinhos = pãezinhos Têm a vogal tônica fechada (ô): adornos, almoços, bolsos,
animai(s) + zinhos = animaizinhos esposos, estojos, globos, gostos, polvos, rolos, soros, etc.
botõe(s) + zinhos = botõezinhos Obs.: distinga-se molho (ô) = caldo (molho de carne), de
chapéu(s) + zinhos = chapeuzinhos molho (ó) = feixe (molho de lenha).
farói(s) + zinhos = faroizinhos
tren(s) + zinhos = trenzinhos Particularidades sobre o Número dos Substantivos
colhere(s) + zinhas = colherezinhas
flore(s) + zinhas = florezinhas - Há substantivos que só se usam no singular: o sul, o
mão(s) + zinhas = mãozinhas norte, o leste, o oeste, a fé, etc.
papéi(s) + zinhos = papeizinhos - Outros só no plural: as núpcias, os víveres, os pêsames,
nuven(s) + zinhas = nuvenzinhas as espadas/os paus (naipes de baralho), as fezes.
funi(s) + zinhos = funizinhos - Outros, enfim, têm, no plural, sentido diferente do sin-
túnei(s) + zinhos = tuneizinhos gular: bem (virtude) e bens (riquezas), honra (probidade, bom
pai(s) + zinhos = paizinhos nome) e honras (homenagem, títulos).
pé(s) + zinhos = pezinhos - Usamos às vezes, os substantivos no singular, mas com
pé(s) + zitos = pezitos sentido de plural:
Aqui morreu muito negro.
Celebraram o sacrifício divino muitas vezes em capelas
improvisadas.

22
LÍNGUA PORTUGUESA

Flexão de Grau do Substantivo No primeiro caso, o metrô é o lugar a que vou; no se-
gundo caso, é o meio de transporte por mim utilizado. A
Grau é a propriedade que as palavras têm de exprimir as oração “Cheguei no metrô”, popularmente usada a fim de
variações de tamanho dos seres. Classifica-se em: indicar o lugar a que se vai, possui, no padrão culto da lín-
- Grau Normal - Indica um ser de tamanho considerado gua, sentido diferente. Aliás, é muito comum existirem di-
normal. Por exemplo: casa vergências entre a regência coloquial, cotidiana de alguns
verbos, e a regência culta.
- Grau Aumentativo - Indica o aumento do tamanho do Para estudar a regência verbal, agruparemos os verbos
ser. Classifica-se em: de acordo com sua transitividade. A transitividade, porém,
Analítico = o substantivo é acompanhado de um adjetivo não é um fato absoluto: um mesmo verbo pode atuar de
que indica grandeza. Por exemplo: casa grande. diferentes formas em frases distintas.
Sintético = é acrescido ao substantivo um sufixo indica-
dor de aumento. Por exemplo: casarão. Verbos Intransitivos

- Grau Diminutivo - Indica a diminuição do tamanho do Os verbos intransitivos não possuem complemento. É
ser. Pode ser: importante, no entanto, destacar alguns detalhes relativos
Analítico = substantivo acompanhado de um adjetivo aos adjuntos adverbiais que costumam acompanhá-los.
que indica pequenez. Por exemplo: casa pequena.
Sintético = é acrescido ao substantivo um sufixo indica- - Chegar, Ir
dor de diminuição. Por exemplo: casinha. Normalmente vêm acompanhados de adjuntos adver-
biais de lugar. Na língua culta, as preposições usadas para
indicar destino ou direção são: a, para.
4. REGÊNCIA E CONCORDÂNCIA; Fui ao teatro.
Adjunto Adverbial de Lugar
REGÊNCIA
Ricardo foi para a Espanha.
Adjunto Adverbial de Lugar
Dá-se o nome de regência à relação de subordinação que
ocorre entre um verbo (ou um nome) e seus complementos.
- Comparecer
Ocupa-se em estabelecer relações entre as palavras, criando
O adjunto adverbial de lugar pode ser introduzido por
frases não ambíguas, que expressem efetivamente o sentido
desejado, que sejam corretas e claras. em ou a.
Comparecemos ao estádio (ou no estádio) para ver o úl-
Regência Verbal timo jogo.

Termo Regente: VERBO Verbos Transitivos Diretos

A regência verbal estuda a relação que se estabelece entre Os verbos transitivos diretos são complementados por
os verbos e os termos que os complementam (objetos diretos objetos diretos. Isso significa que não exigem preposição
e objetos indiretos) ou caracterizam (adjuntos adverbiais). para o estabelecimento da relação de regência. Ao em-
O estudo da regência verbal permite-nos ampliar nossa ca- pregar esses verbos, devemos lembrar que os pronomes
pacidade expressiva, pois oferece oportunidade de conhecer- oblíquos o, a, os, as atuam como objetos diretos. Esses pro-
mos as diversas significações que um verbo pode assumir com nomes podem assumir as formas lo, los, la, las (após formas
a simples mudança ou retirada de uma preposição. Observe: verbais terminadas em -r, -s ou -z) ou no, na, nos, nas (após
A mãe agrada o filho. -> agradar significa acariciar, con- formas verbais terminadas em sons nasais), enquanto lhe e
tentar. lhes são, quando complementos verbais, objetos indiretos.
A mãe agrada ao filho. -> agradar significa “causar agrado São verbos transitivos diretos, dentre outros: abando-
ou prazer”, satisfazer. nar, abençoar, aborrecer, abraçar, acompanhar, acusar, ad-
Logo, conclui-se que “agradar alguém” é diferente de mirar, adorar, alegrar, ameaçar, amolar, amparar, auxiliar,
“agradar a alguém”. castigar, condenar, conhecer, conservar,convidar, defender,
eleger, estimar, humilhar, namorar, ouvir, prejudicar, prezar,
Saiba que: proteger, respeitar, socorrer, suportar, ver, visitar.
O conhecimento do uso adequado das preposições é um Na língua culta, esses verbos funcionam exatamente
dos aspectos fundamentais do estudo da regência verbal (e como o verbo amar:
também nominal). As preposições são capazes de modificar Amo aquele rapaz. / Amo-o.
completamente o sentido do que se está sendo dito. Veja Amo aquela moça. / Amo-a.
os exemplos: Amam aquele rapaz. / Amam-no.
Cheguei ao metrô. Ele deve amar aquela mulher. / Ele deve amá-la.
Cheguei no metrô.

23
LÍNGUA PORTUGUESA

Obs.: os pronomes lhe, lhes só acompanham esses ver- - O uso dos pronomes oblíquos átonos deve ser feito
bos para indicar posse (caso em que atuam como adjuntos com particular cuidado. Observe:
adnominais). Agradeci o presente. / Agradeci-o.
Quero beijar-lhe o rosto. (= beijar seu rosto) Agradeço a você. / Agradeço-lhe.
Prejudicaram-lhe a carreira. (= prejudicaram sua carreira) Perdoei a ofensa. / Perdoei-a.
Conheço-lhe o mau humor! (= conheço seu mau humor) Perdoei ao agressor. / Perdoei-lhe.
Paguei minhas contas. / Paguei-as.
Verbos Transitivos Indiretos Paguei aos meus credores. / Paguei-lhes.

Os verbos transitivos indiretos são complementados por Informar


objetos indiretos. Isso significa que esses verbos exigem - Apresenta objeto direto ao se referir a coisas e objeto
uma preposição para o estabelecimento da relação de re- indireto ao se referir a pessoas, ou vice-versa.
gência. Os pronomes pessoais do caso oblíquo de terceira Informe os novos preços aos clientes.
pessoa que podem atuar como objetos indiretos são o “lhe”, Informe os clientes dos novos preços. (ou sobre os novos
o “lhes”, para substituir pessoas. Não se utilizam os prono- preços)
mes o, os, a, as como complementos de verbos transitivos
indiretos. Com os objetos indiretos que não representam - Na utilização de pronomes como complementos, veja
pessoas, usam-se pronomes oblíquos tônicos de terceira as construções:
pessoa (ele, ela) em lugar dos pronomes átonos lhe, lhes. Informei-os aos clientes. / Informei-lhes os novos preços.
Os verbos transitivos indiretos são os seguintes: Informe-os dos novos preços. / Informe-os deles. (ou so-
- Consistir - Tem complemento introduzido pela prepo- bre eles)
sição “em”: A modernidade verdadeira consiste em direitos
iguais para todos. Obs.: a mesma regência do verbo informar é usada
para os seguintes: avisar, certificar, notificar, cientificar, pre-
- Obedecer e Desobedecer - Possuem seus complemen- venir.
Comparar
tos introduzidos pela preposição “a”:
Quando seguido de dois objetos, esse verbo admite as
Devemos obedecer aos nossos princípios e ideais.
preposições “a” ou “com” para introduzir o complemento
Eles desobedeceram às leis do trânsito.
indireto.
- Responder - Tem complemento introduzido pela pre-
Comparei seu comportamento ao (ou com o) de uma
posição “a”. Esse verbo pede objeto indireto para indicar “a
criança.
quem” ou “ao que” se responde.
Pedir
Respondi ao meu patrão.
Esse verbo pede objeto direto de coisa (geralmente na
Respondemos às perguntas. forma de oração subordinada substantiva) e indireto de
Respondeu-lhe à altura. pessoa.
Pedi-lhe favores.
Obs.: o verbo responder, apesar de transitivo indireto Objeto Indireto Objeto Direto
quando exprime aquilo a que se responde, admite voz pas-
siva analítica. Veja: Pedi-lhe que se mantivesse em silêncio.
O questionário foi respondido corretamente. Objeto Indireto Oração Subordinada Substantiva
Todas as perguntas foram respondidas satisfatoriamente. Objetiva Direta
- Simpatizar e Antipatizar - Possuem seus complementos Saiba que:
introduzidos pela preposição “com”. - A construção “pedir para”, muito comum na lingua-
Antipatizo com aquela apresentadora. gem cotidiana, deve ter emprego muito limitado na língua
Simpatizo com os que condenam os políticos que gover- culta. No entanto, é considerada correta quando a palavra
nam para uma minoria privilegiada. licença estiver subentendida.
Peço (licença) para ir entregar-lhe os catálogos em casa.
Verbos Transitivos Diretos e Indiretos Observe que, nesse caso, a preposição “para” introduz
uma oração subordinada adverbial final reduzida de infini-
Os verbos transitivos diretos e indiretos são acompanha- tivo (para ir entregar-lhe os catálogos em casa).
dos de um objeto direto e um indireto. Merecem destaque,
nesse grupo: Agradecer, Perdoar e Pagar. São verbos que - A construção “dizer para”, também muito usada po-
apresentam objeto direto relacionado a coisas e objeto indi- pularmente, é igualmente considerada incorreta.
reto relacionado a pessoas. Veja os exemplos:
Agradeço aos ouvintes a audiência. Preferir
Objeto Indireto Objeto Direto Na língua culta, esse verbo deve apresentar objeto indi-
reto introduzido pela preposição “a”. Por Exemplo:
Paguei o débito ao cobrador. Prefiro qualquer coisa a abrir mão de meus ideais.
Objeto Direto Objeto Indireto Prefiro trem a ônibus.

24
LÍNGUA PORTUGUESA

Obs.: na língua culta, o verbo “preferir” deve ser usado CHAMAR


sem termos intensificadores, tais como: muito, antes, mil - Chamar é transitivo direto no sentido de convocar, so-
vezes, um milhão de vezes, mais. A ênfase já é dada pelo licitar a atenção ou a presença de.
prefixo existente no próprio verbo (pre). Por gentileza, vá chamar sua prima. / Por favor, vá cha-
má-la.
Mudança de Transitividade X Mudança de Significado Chamei você várias vezes. / Chamei-o várias vezes.

Há verbos que, de acordo com a mudança de transitivi- - Chamar no sentido de denominar, apelidar pode apre-
dade, apresentam mudança de significado. O conhecimen- sentar objeto direto e indireto, ao qual se refere predicativo
to das diferentes regências desses verbos é um recurso lin- preposicionado ou não.
guístico muito importante, pois além de permitir a correta A torcida chamou o jogador mercenário.
interpretação de passagens escritas, oferece possibilidades A torcida chamou ao jogador mercenário.
expressivas a quem fala ou escreve. Dentre os principais, A torcida chamou o jogador de mercenário.
estão: A torcida chamou ao jogador de mercenário.

AGRADAR CUSTAR
- Agradar é transitivo direto no sentido de fazer cari- - Custar é intransitivo no sentido de ter determinado
nhos, acariciar. valor ou preço, sendo acompanhado de adjunto adverbial:
Sempre agrada o filho quando o revê. / Sempre o agrada Frutas e verduras não deveriam custar muito.
quando o revê. - No sentido de ser difícil, penoso, pode ser intransitivo
Cláudia não perde oportunidade de agradar o gato. / ou transitivo indireto.
Cláudia não perde oportunidade de agradá-lo.
Muito custa viver tão longe da família.
- Agradar é transitivo indireto no sentido de causar Verbo Oração Subordinada Substantiva Subjetiva
Intransitivo Reduzida de Infinitivo
agrado a, satisfazer, ser agradável a. Rege complemento
introduzido pela preposição “a”.
Custa-me (a mim) crer que tomou realmente aquela atitude.
O cantor não agradou aos presentes.
Objeto Oração Subordinada Substantiva Subjetiva
O cantor não lhes agradou.
Indireto Reduzida de Infinitivo
ASPIRAR
Obs.: a Gramática Normativa condena as construções
- Aspirar é transitivo direto no sentido de sorver, inspi-
que atribuem ao verbo “custar” um sujeito representado
rar (o ar), inalar: Aspirava o suave aroma. (Aspirava-o) por pessoa. Observe:
- Aspirar é transitivo indireto no sentido de desejar, ter Custei para entender o problema.
como ambição: Aspirávamos a melhores condições de vida. Forma correta: Custou-me entender o problema.
(Aspirávamos a elas)
IMPLICAR
Obs.: como o objeto direto do verbo “aspirar” não é - Como transitivo direto, esse verbo tem dois sentidos:
pessoa, mas coisa, não se usam as formas pronominais áto- a) dar a entender, fazer supor, pressupor: Suas atitudes
nas “lhe” e “lhes” e sim as formas tônicas “a ele (s)”, “ a ela implicavam um firme propósito.
(s)”. Veja o exemplo: Aspiravam a uma existência melhor. (= b) Ter como consequência, trazer como consequência,
Aspiravam a ela) acarretar, provocar: Liberdade de escolha implica amadure-
cimento político de um povo.
ASSISTIR
- Assistir é transitivo direto no sentido de ajudar, pres- - Como transitivo direto e indireto, significa compro-
tar assistência a, auxiliar. Por exemplo: meter, envolver: Implicaram aquele jornalista em questões
As empresas de saúde negam-se a assistir os idosos. econômicas.
As empresas de saúde negam-se a assisti-los.
Obs.: no sentido de antipatizar, ter implicância, é transi-
- Assistir é transitivo indireto no sentido de ver, presen- tivo indireto e rege com preposição “com”: Implicava com
ciar, estar presente, caber, pertencer. Exemplos: quem não trabalhasse arduamente.
Assistimos ao documentário.
Não assisti às últimas sessões. PROCEDER
Essa lei assiste ao inquilino. - Proceder é intransitivo no sentido de ser decisivo, ter
cabimento, ter fundamento ou portar-se, comportar-se,
Obs.: no sentido de morar, residir, o verbo “assistir” é agir. Nessa segunda acepção, vem sempre acompanhado
intransitivo, sendo acompanhado de adjunto adverbial de de adjunto adverbial de modo.
lugar introduzido pela preposição “em”: Assistimos numa As afirmações da testemunha procediam, não havia
conturbada cidade. como refutá-las.
Você procede muito mal.

25
LÍNGUA PORTUGUESA

- Nos sentidos de ter origem, derivar-se (rege a prepo- SIMPATIZAR


sição” de”) e fazer, executar (rege complemento introduzi- Transitivo indireto e exige a preposição “com”: Não sim-
do pela preposição “a”) é transitivo indireto. patizei com os jurados.
O avião procede de Maceió.
Procedeu-se aos exames. NAMORAR
O delegado procederá ao inquérito. É transitivo direto, ou seja, não admite preposição: Ma-
ria namora João.
QUERER
- Querer é transitivo direto no sentido de desejar, ter Obs: Não é correto dizer: “Maria namora com João”.
vontade de, cobiçar.
Querem melhor atendimento. OBEDECER
Queremos um país melhor. É transitivo indireto, ou seja, exige complemento com
a preposição “a” (obedecer a): Devemos obedecer aos pais.
- Querer é transitivo indireto no sentido de ter afeição,
estimar, amar. Obs: embora seja transitivo indireto, esse verbo pode
Quero muito aos meus amigos. ser usado na voz passiva: A fila não foi obedecida.
Ele quer bem à linda menina.
Despede-se o filho que muito lhe quer. VER
É transitivo direto, ou seja, não exige preposição: Ele viu
VISAR o filme.
- Como transitivo direto, apresenta os sentidos de mi-
rar, fazer pontaria e de pôr visto, rubricar. Regência Nominal
O homem visou o alvo.
O gerente não quis visar o cheque. É o nome da relação existente entre um nome (subs-
tantivo, adjetivo ou advérbio) e os termos regidos por esse
- No sentido de ter em vista, ter como meta, ter como nome. Essa relação é sempre intermediada por uma prepo-
objetivo, é transitivo indireto e rege a preposição “a”. sição. No estudo da regência nominal, é preciso levar em
O ensino deve sempre visar ao progresso social. conta que vários nomes apresentam exatamente o mesmo
Prometeram tomar medidas que visassem ao bem-estar regime dos verbos de que derivam. Conhecer o regime de
público. um verbo significa, nesses casos, conhecer o regime dos
nomes cognatos. Observe o exemplo: Verbo obedecer e os
ESQUECER – LEMBRAR nomes correspondentes: todos regem complementos in-
- Lembrar algo – esquecer algo troduzidos pela preposição a. Veja:
- Lembrar-se de algo – esquecer-se de algo (pronomi- Obedecer a algo/ a alguém.
nal) Obediente a algo/ a alguém.

No 1º caso, os verbos são transitivos diretos, ou seja, Apresentamos a seguir vários nomes acompanhados
exigem complemento sem preposição: Ele esqueceu o livro. da preposição ou preposições que os regem. Observe-os
No 2º caso, os verbos são pronominais (-se, -me, etc) e atentamente e procure, sempre que possível, associar es-
exigem complemento com a preposição “de”. São, portan- ses nomes entre si ou a algum verbo cuja regência você
to, transitivos indiretos: conhece.
- Ele se esqueceu do caderno.
- Eu me esqueci da chave.
- Eles se esqueceram da prova.
- Nós nos lembramos de tudo o que aconteceu.

Há uma construção em que a coisa esquecida ou lem-


brada passa a funcionar como sujeito e o verbo sofre leve
alteração de sentido. É uma construção muito rara na lín-
gua contemporânea, porém, é fácil encontrá-la em textos
clássicos tanto brasileiros como portugueses. Machado de
Assis, por exemplo, fez uso dessa construção várias vezes.
- Esqueceu-me a tragédia. (cair no esquecimento)
- Lembrou-me a festa. (vir à lembrança)

O verbo lembrar também pode ser transitivo direto e


indireto (lembrar alguma coisa a alguém ou alguém de al-
guma coisa).

26
LÍNGUA PORTUGUESA

Substantivos

Admiração a, por Devoção a, para, com, por Medo a, de


Aversão a, para, por Doutor em Obediência a
Atentado a, contra Dúvida acerca de, em, sobre Ojeriza a, por
Bacharel em Horror a Proeminência sobre
Capacidade de, para Impaciência com Respeito a, com, para com, por

Adjetivos
Acessível a Diferente de Necessário a
Acostumado a, com Entendido em Nocivo a
Afável com, para com Equivalente a Paralelo a
Agradável a Escasso de Parco em, de
Alheio a, de Essencial a, para Passível de
Análogo a Fácil de Preferível a
Ansioso de, para, por Fanático por Prejudicial a
Apto a, para Favorável a Prestes a
Ávido de Generoso com Propício a
Benéfico a Grato a, por Próximo a
Capaz de, para Hábil em Relacionado com
Compatível com Habituado a Relativo a
Contemporâneo a, de Idêntico a Satisfeito com, de, em, por
Contíguo a Impróprio para Semelhante a
Contrário a Indeciso em Sensível a
Curioso de, por Insensível a Sito em
Descontente com Liberal com Suspeito de
Desejoso de Natural de Vazio de

Advérbios
Longe de Perto de

Obs.: os advérbios terminados em -mente tendem a seguir o regime dos adjetivos de que são formados: paralela a;
paralelamente a; relativa a; relativamente a.

Fonte: http://www.soportugues.com.br/secoes/sint/sint61.php

Questões sobre Regência Nominal e Verbal

01. (Administrador – FCC – 2013-adap.).


... a que ponto a astronomia facilitou a obra das outras ciências ...
O verbo que exige o mesmo tipo de complemento que o grifado acima está empregado em:
A) ...astros que ficam tão distantes ...
B) ...que a astronomia é uma das ciências ...
C) ...que nos proporcionou um espírito ...
D) ...cuja importância ninguém ignora ...
E) ...onde seu corpo não passa de um ponto obscuro ...

02.(Agente de Apoio Administrativo – FCC – 2013-adap.).


... pediu ao delegado do bairro que desse um jeito nos filhos do sueco.
O verbo que exige, no contexto, o mesmo tipo de complementos que o grifado acima está empregado em:
A) ...que existe uma coisa chamada exército...
B) ...como se isso aqui fosse casa da sogra?
C) ...compareceu em companhia da mulher à delegacia...
D) Eu ensino o senhor a cumprir a lei, ali no duro...
E) O delegado apenas olhou-a espantado com o atrevimento.

27
LÍNGUA PORTUGUESA

03.(Agente de Defensoria Pública – FCC – 2013-adap.). 06. (Papiloscopista Policial – VUNESP – 2013). Assina-
... constava simplesmente de uma vareta quebrada em le a alternativa correta quanto à regência dos termos em
partes desiguais... destaque.
O verbo que exige o mesmo tipo de complemento que (A) Ele tentava convencer duas senhoras a assumir a
o grifado acima está empregado em: responsabilidade pelo problema.
A) Em campos extensos, chegavam em alguns casos a (B) A menina tinha o receio a levar uma bronca por ter
extremos de sutileza. se perdido.
B) ...eram comumente assinalados a golpes de machado (C) A garota tinha apenas a lembrança pelo desenho
nos troncos mais robustos. de um índio na porta do prédio.
C) Os toscos desenhos e os nomes estropiados deso- (D) A menina não tinha orgulho sob o fato de ter se
rientam, não raro, quem... perdido de sua família.
D) Koch-Grünberg viu uma dessas marcas de caminho (E) A família toda se organizou para realizar a procura
na serra de Tunuí... à garotinha.
E) ...em que tão bem se revelam suas afinidades com o
gentio, mestre e colaborador... 07. (Analista de Sistemas – VUNESP – 2013). Assinale
a alternativa que completa, correta e respectivamente, as
04. (Agente Técnico – FCC – 2013-adap.). lacunas do texto, de acordo com as regras de regência.
... para lidar com as múltiplas vertentes da justiça... Os estudos _______ quais a pesquisadora se reportou já
O verbo que exige o mesmo tipo de complemento que assinalavam uma relação entre os distúrbios da imagem
o da frase acima se encontra em: corporal e a exposição a imagens idealizadas pela mídia.
A) A palavra direito, em português, vem de directum, do A pesquisa faz um alerta ______ influência negativa que a
verbo latino dirigere... mídia pode exercer sobre os jovens.
B) ...o Direito tem uma complexa função de gestão das A) dos … na
sociedades... B) nos … entre a
C) ...o de que o Direito [...] esteja permeado e regulado C) aos … para a
pela justiça. D) sobre os … pela
D) Essa problematicidade não afasta a força das aspira- E) pelos … sob a
ções da justiça...
E) Na dinâmica dessa tensão tem papel relevante o sen- 08. (Analista em Planejamento, Orçamento e Finanças
timento de justiça. Públicas – VUNESP – 2013). Considerando a norma-padrão
da língua, assinale a alternativa em que os trechos desta-
05. (Escrevente TJ SP – Vunesp 2012) Assinale a alter- cados estão corretos quanto à regência, verbal ou nominal.
nativa em que o período, adaptado da revista Pesquisa A) O prédio que o taxista mostrou dispunha de mais de
Fapesp de junho de 2012, está correto quanto à regência dez mil tomadas.
nominal e à pontuação. B) O autor fez conjecturas sob a possibilidade de haver
(A) Não há dúvida que as mulheres ampliam, rapida- um homem que estaria ouvindo as notas de um oboé.
mente, seu espaço na carreira científica ainda que o avanço C) Centenas de trabalhadores estão empenhados de
seja mais notável em alguns países, o Brasil é um exemplo, criar logotipos e negociar.
do que em outros. D) O taxista levou o autor a indagar no número de to-
(B) Não há dúvida de que, as mulheres, ampliam rapida- madas do edifício.
mente seu espaço na carreira científica; ainda que o avanço E) A corrida com o taxista possibilitou que o autor repa-
seja mais notável, em alguns países, o Brasil é um exemplo!, rasse a um prédio na marginal.
do que em outros.
(C) Não há dúvida de que as mulheres, ampliam ra- 09. (Assistente de Informática II – VUNESP – 2013). As-
pidamente seu espaço, na carreira científica, ainda que o sinale a alternativa que substitui a expressão destacada na
avanço seja mais notável, em alguns países: o Brasil é um frase, conforme as regras de regência da norma-padrão da
exemplo, do que em outros. língua e sem alteração de sentido.
(D) Não há dúvida de que as mulheres ampliam rapida- Muitas organizações lutaram a favor da igualdade de
mente seu espaço na carreira científica, ainda que o avanço direitos dos trabalhadores domésticos.
seja mais notável em alguns países – o Brasil é um exemplo A) da
– do que em outros. B) na
(E) Não há dúvida que as mulheres ampliam rapidamen- C) pela
te, seu espaço na carreira científica, ainda que, o avanço D) sob a
seja mais notável em alguns países (o Brasil é um exemplo) E) sobre a
do que em outros.
GABARITO

01. D 02. D 03. A 04. A 05. D


06. A 07. C 08. A 09. C

28
LÍNGUA PORTUGUESA

RESOLUÇÃO 6-)
(B) A menina tinha o receio de levar uma bronca por ter
1-) ... a que ponto a astronomia facilitou a obra das ou- se perdido.
tras ciências ... (C) A garota tinha apenas a lembrança do desenho de
Facilitar – verbo transitivo direto um índio na porta do prédio.
A) ...astros que ficam tão distantes ... = verbo de ligação (D) A menina não tinha orgulho do fato de ter se perdi-
B) ...que a astronomia é uma das ciências ... = verbo de do de sua família.
ligação (E) A família toda se organizou para realizar a procura
C) ...que nos proporcionou um espírito ... = verbo transi- pela garotinha.
tivo direto e indireto
E) ...onde seu corpo não passa de um ponto obscuro = 7-) Os estudos aos quais a pesquisadora se reportou
verbo transitivo indireto já assinalavam uma relação entre os distúrbios da imagem
corporal e a exposição a imagens idealizadas pela mídia.
2-) ... pediu ao delegado do bairro que desse um jeito A pesquisa faz um alerta para a influência negativa
nos filhos do sueco. que a mídia pode exercer sobre os jovens.
Pedir = verbo transitivo direto e indireto
A) ...que existe uma coisa chamada EXÉRCITO... = tran- 8-)
sitivo direto B) O autor fez conjecturas sobre a possibilidade de ha-
B) ...como se isso aqui fosse casa da sogra? =verbo de ver um homem que estaria ouvindo as notas de um oboé.
ligação C) Centenas de trabalhadores estão empenhados em
C) ...compareceu em companhia da mulher à delegacia... criar logotipos e negociar.
=verbo intransitivo D) O taxista levou o autor a indagar sobre o número de
E) O delegado apenas olhou-a espantado com o atrevi- tomadas do edifício.
mento. =transitivo direto E) A corrida com o taxista possibilitou que o autor repa-
rasse em um prédio na marginal.
3-) ... constava simplesmente de uma vareta quebrada
em partes desiguais...
9-) Muitas organizações lutaram pela igualdade de
Constar = verbo intransitivo
direitos dos trabalhadores domésticos.
B) ...eram comumente assinalados a golpes de machado
nos troncos mais robustos. =ligação
CONCORDÂNCIA
C) Os toscos desenhos e os nomes estropiados deso-
rientam, não raro, quem... =transitivo direto
D) Koch-Grünberg viu uma dessas marcas de caminho Ao falarmos sobre a concordância verbal, estamos nos
na serra de Tunuí... = transitivo direto referindo à relação de dependência estabelecida entre um
E) ...em que tão bem se revelam suas afinidades com o termo e outro mediante um contexto oracional. Desta fei-
gentio, mestre e colaborador...=transitivo direto ta, os agentes principais desse processo são representados
pelo sujeito, que no caso funciona como subordinante; e o
4-) ... para lidar com as múltiplas vertentes da justiça... verbo, o qual desempenha a função de subordinado.
Lidar = transitivo indireto Dessa forma, temos que a concordância verbal caracte-
B) ...o Direito tem uma complexa função de gestão das riza-se pela adaptação do verbo, tendo em vista os quesi-
sociedades... =transitivo direto tos “número e pessoa” em relação ao sujeito. Exemplifican-
C) ...o de que o Direito [...] esteja permeado e regulado do, temos: O aluno chegou atrasado. Temos que o verbo
pela justiça. =ligação apresenta-se na terceira pessoa do singular, pois faz refe-
D) Essa problematicidade não afasta a força das aspira- rência a um sujeito, assim também expresso (ele). Como
ções da justiça... =transitivo direto e indireto poderíamos também dizer: os alunos chegaram atrasados.
E) Na dinâmica dessa tensão tem papel relevante o sen-
timento de justiça. =transitivo direto Casos referentes a sujeito simples

5-) A correção do item deve respeitar as regras de pontua- 1) Em caso de sujeito simples, o verbo concorda com
ção também. Assinalei apenas os desvios quanto à regência o núcleo em número e pessoa: O aluno chegou atrasado.
(pontuação encontra-se em tópico específico)
(A) Não há dúvida de que as mulheres ampliam, 2) Nos casos referentes a sujeito representado por subs-
(B) Não há dúvida de que (erros quanto à pon- tantivo coletivo, o verbo permanece na terceira pessoa do
tuação) singular: A multidão, apavorada, saiu aos gritos.
(C) Não há dúvida de que as mulheres, (erros quanto Observação:
à pontuação) - No caso de o coletivo aparecer seguido de adjunto
(E) Não há dúvida de que as mulheres ampliam rapida- adnominal no plural, o verbo permanecerá no singular ou
mente, seu espaço na carreira científica, ainda que, o avan- poderá ir para o plural:
ço seja mais notável em alguns países (o Brasil é um exem- Uma multidão de pessoas saiu aos gritos.
plo) do que em outros. Uma multidão de pessoas saíram aos gritos.

29
LÍNGUA PORTUGUESA

3) Quando o sujeito é representado por expressões par- Observações:


titivas, representadas por “a maioria de, a maior parte de, - Caso o verbo apareça anteposto à expressão de porcen-
a metade de, uma porção de” entre outras, o verbo tanto tagem, esse deverá concordar com o numeral: Aprovaram a
pode concordar com o núcleo dessas expressões quanto decisão da diretoria 50% dos funcionários.
com o substantivo que a segue: A maioria dos alunos resol- - Em casos relativos a 1%, o verbo permanecerá no sin-
veu ficar. A maioria dos alunos resolveram ficar. gular: 1% dos funcionários não aprovou a decisão da diretoria.
- Em casos em que o numeral estiver acompanhado de
4) No caso de o sujeito ser representado por expres- determinantes no plural, o verbo permanecerá no plural: Os
sões aproximativas, representadas por “cerca de, perto de”, 50% dos funcionários apoiaram a decisão da diretoria.
o verbo concorda com o substantivo determinado por elas:
Cerca de mil candidatos se inscreveram no concurso. 11) Nos casos em que o sujeito estiver representado por pro-
nomes de tratamento, o verbo deverá ser empregado na terceira
5) Em casos em que o sujeito é representado pela ex- pessoa do singular ou do plural: Vossas Majestades gostaram
pressão “mais de um”, o verbo permanece no singular: Mais das homenagens. Vossa Majestade agradeceu o convite.
de um candidato se inscreveu no concurso de piadas.
Observação: 12) Casos relativos a sujeito representado por substantivo
- No caso da referida expressão aparecer repetida ou próprio no plural se encontram relacionados a alguns aspec-
associada a um verbo que exprime reciprocidade, o verbo, tos que os determinam:
necessariamente, deverá permanecer no plural: - Diante de nomes de obras no plural, seguidos do verbo
Mais de um aluno, mais de um professor contribuíram na ser, este permanece no singular, contanto que o predicati-
campanha de doação de alimentos. vo também esteja no singular: Memórias póstumas de Brás
Mais de um formando se abraçaram durante as soleni- Cubas é uma criação de Machado de Assis.
dades de formatura. - Nos casos de artigo expresso no plural, o verbo também
permanece no plural: Os Estados Unidos são uma potência
6) Quando o sujeito for composto da expressão “um mundial.
dos que”, o verbo permanecerá no plural: Esse jogador foi - Casos em que o artigo figura no singular ou em que ele
um dos que atuaram na Copa América. nem aparece, o verbo permanece no singular: Estados Unidos
é uma potência mundial.
7) Em casos relativos à concordância com locuções pro-
nominais, representadas por “algum de nós, qual de vós, Casos referentes a sujeito composto
quais de vós, alguns de nós”, entre outras, faz-se necessário
nos atermos a duas questões básicas: 1) Nos casos relativos a sujeito composto de pessoas gra-
- No caso de o primeiro pronome estar expresso no maticais diferentes, o verbo deverá ir para o plural, estando
plural, o verbo poderá com ele concordar, como poderá relacionado a dois pressupostos básicos:
também concordar com o pronome pessoal: Alguns de nós - Quando houver a 1ª pessoa, esta prevalecerá sobre as
o receberemos. / Alguns de nós o receberão. demais: Eu, tu e ele faremos um lindo passeio.
- Quando o primeiro pronome da locução estiver ex- - Quando houver a 2ª pessoa, o verbo poderá flexionar na
presso no singular, o verbo permanecerá, também, no sin- 2ª ou na 3ª pessoa: Tu e ele sois primos. Tu e ele são primos.
gular: Algum de nós o receberá.
2) Nos casos em que o sujeito composto aparecer ante-
8) No caso de o sujeito aparecer representado pelo pro- posto ao verbo, este permanecerá no plural: O pai e seus dois
nome “quem”, o verbo permanecerá na terceira pessoa do filhos compareceram ao evento.
singular ou poderá concordar com o antecedente desse
pronome: Fomos nós quem contou toda a verdade para 3) No caso em que o sujeito aparecer posposto ao verbo,
ela. / Fomos nós quem contamos toda a verdade para ela. este poderá concordar com o núcleo mais próximo ou per-
manecer no plural: Compareceram ao evento o pai e seus dois
9) Em casos nos quais o sujeito aparece realçado pela filhos. Compareceu ao evento o pai e seus dois filhos.
palavra “que”, o verbo deverá concordar com o termo que
antecede essa palavra: Nesta empresa somos nós que toma- 4) Nos casos relacionados a sujeito simples, porém com
mos as decisões. / Em casa sou eu que decido tudo. mais de um núcleo, o verbo deverá permanecer no singular:
Meu esposo e grande companheiro merece toda a felicidade
10) No caso de o sujeito aparecer representado por ex- do mundo.
pressões que indicam porcentagens, o verbo concordará
com o numeral ou com o substantivo a que se refere essa 5) Casos relativos a sujeito composto de palavras sinôni-
porcentagem: 50% dos funcionários aprovaram a decisão mas ou ordenado por elementos em gradação, o verbo po-
da diretoria. / 50% do eleitorado apoiou a decisão. derá permanecer no singular ou ir para o plural: Minha vitória,
minha conquista, minha premiação são frutos de meu esforço.
/ Minha vitória, minha conquista, minha premiação é fruto de
meu esforço.

30
LÍNGUA PORTUGUESA

Concordância nominal é o ajuste que fazemos aos de- f) Um(a) e outro(a), num(a) e noutro(a)
mais termos da oração para que concordem em gênero e - Após essas expressões o substantivo fica sempre no
número com o substantivo. Teremos que alterar, portanto, singular e o adjetivo no plural.
o artigo, o adjetivo, o numeral e o pronome. Além disso, Renato advogou um e outro caso fáceis.
temos também o verbo, que se flexionará à sua maneira. Pusemos numa e noutra bandeja rasas o peixe.
Regra geral: O artigo, o adjetivo, o numeral e o prono-
me concordam em gênero e número com o substantivo. g) É bom, é necessário, é proibido
- A pequena criança é uma gracinha. - Essas expressões não variam se o sujeito não vier pre-
- O garoto que encontrei era muito gentil e simpático. cedido de artigo ou outro determinante.
Canja é bom. / A canja é boa.
Casos especiais: Veremos alguns casos que fogem à É necessário sua presença. / É necessária a sua presença.
regra geral mostrada acima. É proibido entrada de pessoas não autorizadas. / A en-
a) Um adjetivo após vários substantivos trada é proibida.
- Substantivos de mesmo gênero: adjetivo vai para o
plural ou concorda com o substantivo mais próximo. h) Muito, pouco, caro
- Como adjetivos: seguem a regra geral.
- Irmão e primo recém-chegado estiveram aqui.
Comi muitas frutas durante a viagem.
- Irmão e primo recém-chegados estiveram aqui.
Pouco arroz é suficiente para mim.
Os sapatos estavam caros.
- Substantivos de gêneros diferentes: vai para o plural
masculino ou concorda com o substantivo mais próximo. - Como advérbios: são invariáveis.
- Ela tem pai e mãe louros. Comi muito durante a viagem.
- Ela tem pai e mãe loura. Pouco lutei, por isso perdi a batalha.
- Adjetivo funciona como predicativo: vai obrigatoria- Comprei caro os sapatos.
mente para o plural. i) Mesmo, bastante
- O homem e o menino estavam perdidos. - Como advérbios: invariáveis
- O homem e sua esposa estiveram hospedados aqui. Preciso mesmo da sua ajuda.
Fiquei bastante contente com a proposta de emprego.
b) Um adjetivo anteposto a vários substantivos
- Adjetivo anteposto normalmente concorda com o - Como pronomes: seguem a regra geral.
mais próximo. Seus argumentos foram bastantes para me convencer.
Comi delicioso almoço e sobremesa. Os mesmos argumentos que eu usei, você copiou.
Provei deliciosa fruta e suco.
j) Menos, alerta
- Adjetivo anteposto funcionando como predicativo: - Em todas as ocasiões são invariáveis.
concorda com o mais próximo ou vai para o plural. Preciso de menos comida para perder peso.
Estavam feridos o pai e os filhos. Estamos alerta para com suas chamadas.
Estava ferido o pai e os filhos.
k) Tal Qual
c) Um substantivo e mais de um adjetivo - “Tal” concorda com o antecedente, “qual” concorda
- antecede todos os adjetivos com um artigo. com o consequente.
Falava fluentemente a língua inglesa e a espanhola. As garotas são vaidosas tais qual a tia.
Os pais vieram fantasiados tais quais os filhos.
- coloca o substantivo no plural.
l) Possível
Falava fluentemente as línguas inglesa e espanhola.
- Quando vem acompanhado de “mais”, “menos”, “me-
lhor” ou “pior”, acompanha o artigo que precede as ex-
d) Pronomes de tratamento
pressões.
- sempre concordam com a 3ª pessoa. A mais possível das alternativas é a que você expôs.
Vossa Santidade esteve no Brasil. Os melhores cargos possíveis estão neste setor da em-
presa.
e) Anexo, incluso, próprio, obrigado As piores situações possíveis são encontradas nas favelas
- Concordam com o substantivo a que se referem. da cidade.
As cartas estão anexas.
A bebida está inclusa. m) Meio
Precisamos de nomes próprios. - Como advérbio: invariável.
Obrigado, disse o rapaz. Estou meio (um pouco) insegura.

- Como numeral: segue a regra geral.


Comi meia (metade) laranja pela manhã.

31
LÍNGUA PORTUGUESA

n) Só 03. (Escrevente TJ-SP – Vunesp/2012) Leia o texto para


- apenas, somente (advérbio): invariável. responder à questão.
Só consegui comprar uma passagem. _________dúvidas sobre o crescimento verde. Primeiro,
não está claro até onde pode realmente chegar uma po-
- sozinho (adjetivo): variável. lítica baseada em melhorar a eficiência sem preços ade-
Estiveram sós durante horas. quados para o carbono, a água e (na maioria dos países
pobres) a terra. É verdade que mesmo que a ameaça dos
Fonte: preços do carbono e da água em si ___________diferença, as
http://www.brasilescola.com/gramatica/concordancia- companhias não podem suportar ter de pagar, de repente,
verbal.htm digamos, 40 dólares por tonelada de carbono, sem qual-
quer preparação. Portanto, elas começam a usar preços-
Questões sobre Concordância Nominal e Verbal -sombra. Ainda assim, ninguém encontrou até agora uma
maneira de quantificar adequadamente os insumos bási-
01.(TRE/AL – TÉCNICO JUDICIÁRIO – FCC/2010) A con- cos. E sem eles a maioria das políticas de crescimento verde
cordância verbal e nominal está inteiramente correta na sempre ___________ a segunda opção.
frase: (Carta Capital, 27.06.2012. Adaptado)
(A) A sociedade deve reconhecer os princípios e va-
lores que determinam as escolhas dos governantes, para De acordo com a norma-padrão da língua portuguesa,
conferir legitimidade a suas decisões. as lacunas do texto devem ser preenchidas, correta e res-
(B) A confiança dos cidadãos em seus dirigentes de- pectivamente, com:
vem ser embasados na percepção dos valores e princípios (A) Restam… faça… será
que regem a prática política. (B) Resta… faz… será
(C) Eleições livres e diretas é garantia de um verdadei- (C) Restam… faz... serão
ro regime democrático, em que se respeita tanto as liber- (D) Restam… façam… serão
dades individuais quanto as coletivas. (E) Resta… fazem… será
(D) As instituições fundamentais de um regime demo-
crático não pode estar subordinado às ordens indiscrimi- 04 (Escrevente TJ SP – Vunesp/2012) Assinale a alterna-
nadas de um único poder central. tiva em que o trecho
(E) O interesse de todos os cidadãos estão voltados – Ainda assim, ninguém encontrou até agora uma ma-
para o momento eleitoral, que expõem as diferentes opi- neira de quantificar adequadamente os insumos básicos.–
niões existentes na sociedade. está corretamente reescrito, de acordo com a norma-pa-
drão da língua portuguesa.
02. (Agente Técnico – FCC – 2013). As normas de con- (A) Ainda assim, temos certeza que ninguém encontrou
cordância verbal e nominal estão inteiramente respeita- até agora uma maneira adequada de se quantificar os in-
das em: sumos básicos.
A) Alguns dos aspectos mais desejáveis de uma boa (B) Ainda assim, temos certeza de que ninguém encon-
leitura, que satisfaça aos leitores e seja veículo de apri- trou até agora uma maneira adequada de os insumos bá-
moramento intelectual, estão na capacidade de criação do sicos ser quantificados.
autor, mediante palavras, sua matéria-prima. (C) Ainda assim, temos certeza que ninguém encontrou
B) Obras que se considera clássicas na literatura sempre até agora uma maneira adequada para que os insumos
delineia novos caminhos, pois é capaz de encantar o leitor básicos sejam quantificado.
ao ultrapassar os limites da época em que vivem seus au- (D) Ainda assim, temos certeza de que ninguém encon-
tores, gênios no domínio das palavras, sua matéria-prima. trou até agora uma maneira adequada para que os insu-
C) A palavra, matéria-prima de poetas e romancistas, mos básicos seja quantificado.
lhe permitem criar todo um mundo de ficção, em que per- (E) Ainda assim, temos certeza de que ninguém encon-
sonagens se transformam em seres vivos a acompanhar os trou até agora uma maneira adequada de se quantificarem
leitores, numa verdadeira interação com a realidade. os insumos básicos.
D) As possibilidades de comunicação entre autor e lei-
tor somente se realiza plenamente caso haja afinidade de 05. (FUNDAÇÃO CASA/SP - AGENTE ADMINISTRATIVO
ideias entre ambos, o que permite, ao mesmo tempo, o - VUNESP/2011 - ADAPTADA) Observe as frases do texto:
crescimento intelectual deste último e o prazer da leitura. I. Cerca de 75 por cento dos países obtêm nota nega-
E) Consta, na literatura mundial, obras-primas que tiva...
constitui leitura obrigatória e se tornam referências por II. ... à Venezuela, de Chávez, que obtém a pior classi-
seu conteúdo que ultrapassa os limites de tempo e de ficação do continente americano (2,0)...
época. Assim como ocorre com o verbo “obter” nas frases I e
II, a concordância segue as mesmas regras, na ordem dos
exemplos, em:

32
LÍNGUA PORTUGUESA

(A) Todas as pessoas têm boas perspectivas para o d) Intelectuais que têm compromisso apenas com a ver-
próximo ano. Será que alguém tem opinião diferente da dade, ainda que conscientes de que esta é até certo ponto
maioria? relativa, costumam encontrar muito mais detratores que
(B) Vem muita gente prestigiar as nossas festas juni- admiradores.
nas. Vêm pessoas de muito longe para brincar de qua- e) No final do século XX já não se via muitos intelec-
drilha. tuais e escritores como Edward Said, que não apenas era
(C) Pouca gente quis voltar mais cedo para casa. Qua- notícia pelos livros que publicavam como pelas posições
se todos quiseram ficar até o nascer do sol na praia. que corajosamente assumiam.
(D) Existem pessoas bem intencionadas por aqui, mas
também existem umas que não merecem nossa atenção. 09. (TRF - 2ª REGIÃO - TÉCNICO JUDICIÁRIO -
(E) Aqueles que não atrapalham muito ajudam. FCC/2012) O verbo que, dadas as alterações entre pa-
rênteses propostas para o segmento grifado, deverá ser
06. (TRF - 5ª REGIÃO - TÉCNICO JUDICIÁRIO - colocado no plural, está em:
FCC/2012) Os folheteiros vivem em feiras, mercados, pra- (A) Não há dúvida de que o estilo de vida... (dúvidas)
ças e locais de peregrinação. (B) O que não se sabe... (ninguém nas regiões do pla-
O verbo da frase acima NÃO pode ser mantido no neta)
plural caso o segmento grifado seja substituído por: (C) O consumo mundial não dá sinal de trégua... (O
(A) Há folheteiros que consumo mundial de barris de petróleo)
(B) A maior parte dos folheteiros (D) Um aumento elevado no preço do óleo reflete-se
(C) O folheteiro e sua família no custo da matéria-prima... (Constantes aumentos)
(D) O grosso dos folheteiros (E) o tema das mudanças climáticas pressiona os es-
(E) Cada um dos folheteiros forços mundiais... (a preocupação em torno das mudanças
climáticas)
07. (TRF - 5ª REGIÃO - TÉCNICO JUDICIÁRIO -
FCC/2012) Todas as formas verbais estão corretamente 10. (CETESB/SP – ESCRITURÁRIO - VUNESP/2013) Assi-
flexionadas em: nale a alternativa em que a concordância das formas ver-
(A) Enquanto não se disporem a considerar o cordel bais destacadas está de acordo com a norma-padrão da
sem preconceitos, as pessoas não serão capazes de fruir língua.
dessas criações poéticas tão originais. (A) Fazem dez anos que deixei de trabalhar em higie-
(B) Ainda que nem sempre detenha o mesmo status nização subterrânea.
atribuído à arte erudita, o cordel vem sendo estudado (B) Ainda existe muitas pessoas que discriminam os
hoje nas melhores universidades do país. trabalhadores da área de limpeza.
(C) Rodolfo Coelho Cavalcante deve ter percebido que (C) No trabalho em meio a tanta sujeira, havia altos
a situação dos cordelistas não mudaria a não ser que eles riscos de se contrair alguma doença.
mesmos requizessem o respeito que faziam por merecer. (D) Eu passava a manhã no subterrâneo: quando era
(D) Se não proveem do preconceito, a desvalorização sete da manhã, eu já estava fazendo meu serviço.
e a pouca visibilidade dessa arte popular tão rica só pode (E) As companhias de limpeza, apenas recentemente,
ser resultado do puro e simples desconhecimento. começou a adotar medidas mais rigorosas para a prote-
(E) Rodolfo Coelho Cavalcante entreveu que os pro- ção de seus funcionários.
blemas dos cordelistas estavam diretamente ligados à
falta de representatividade. GABARITO

08. (TRF - 4ª REGIÃO – TÉCNICO JUDICIÁRIO – 01. A 02. A 03. A 04. E 05. A
FCC/2010) Observam-se corretamente as regras de con- 06. E 07. |B 08. D 09. D 10. C
cordância verbal e nominal em:
a) O desenraizamento, não só entre intelectuais como RESOLUÇÃO
entre os mais diversos tipos de pessoas, das mais sofis-
ticadas às mais humildes, são cada vez mais comuns nos 1-) Fiz os acertos entre parênteses:
dias de hoje. (A) A sociedade deve reconhecer os princípios e va-
b) A importância de intelectuais como Edward Said e lores que determinam as escolhas dos governantes, para
Tony Judt, que não se furtaram ao debate sobre questões conferir legitimidade a suas decisões.
polêmicas de seu tempo, não estão apenas nos livros que (B) A confiança dos cidadãos em seus dirigentes de-
escreveram. vem (deve) ser embasados (embasada) na percepção dos
c) Nada indica que o conflito no Oriente Médio entre valores e princípios que regem a prática política.
árabes e judeus, responsável por tantas mortes e tanto so- (C) Eleições livres e diretas é (são) garantia de um ver-
frimento, estejam próximos de serem resolvidos ou pelo dadeiro regime democrático, em que se respeita (respei-
menos de terem alguma trégua. tam) tanto as liberdades individuais quanto as coletivas.

33
LÍNGUA PORTUGUESA

(D) As instituições fundamentais de um regime demo- 5-) Em I, obtêm está no plural; em II, no singular. Vamos
crático não pode (podem) estar subordinado (subordina- aos itens:
das) às ordens indiscriminadas de um único poder central. (A) Todas as pessoas têm (plural) ... Será que alguém tem
(E) O interesse de todos os cidadãos estão (está) volta- (singular)
dos (voltado) para o momento eleitoral, que expõem (ex- (B) Vem (singular) muita gente... Vêm pessoas (plural)
põe) as diferentes opiniões existentes na sociedade. (C) Pouca gente quis (singular)... Quase todos quiseram
2-) (plural)
A) Alguns dos aspectos mais desejáveis de uma boa lei- (D) Existem (plural) pessoas ... mas também existem umas
tura, que satisfaça aos leitores e seja veículo de aprimora- (plural)
mento intelectual, estão na capacidade de criação do autor, (E) Aqueles que não atrapalham muito ajudam (ambas as
mediante palavras, sua matéria-prima. = correta formas estão no plural)
B) Obras que se consideram clássicas na literatura sem-
pre delineiam novos caminhos, pois são capazes de encan- 6-)
tar o leitor ao ultrapassarem os limites da época em que A - Há folheteiros que vivem (concorda com o objeto “fo-
vivem seus autores, gênios no domínio das palavras, sua lheterios”)
matéria-prima. B – A maior parte dos folheteiros vivem/vive (opcional)
C) A palavra, matéria-prima de poetas e romancistas, C – O folheteiro e sua família vivem (sujeito composto)
lhes permite criar todo um mundo de ficção, em que per- D – O grosso dos folheteiros vive/vivem (opcional)
sonagens se transformam em seres vivos a acompanhar os E – Cada um dos folheteiros vive = somente no singular
leitores, numa verdadeira interação com a realidade.
D) As possibilidades de comunicação entre autor e lei- 7-) Coloquei entre parênteses a forma verbal correta:
tor somente se realizam plenamente caso haja afinidade (A) Enquanto não se disporem (dispuserem) a considerar
de ideias entre ambos, o que permite, ao mesmo tempo, o o cordel sem preconceitos, as pessoas não serão capazes de
crescimento intelectual deste último e o prazer da leitura. fruir dessas criações poéticas tão originais.
(B) Ainda que nem sempre detenha o mesmo status atri-
E) Constam, na literatura mundial, obras-primas que
buído à arte erudita, o cordel vem sendo estudado hoje nas
constituem leitura obrigatória e se tornam referências por
melhores universidades do país.
seu conteúdo que ultrapassa os limites de tempo e de épo-
(C) Rodolfo Coelho Cavalcante deve ter percebido que a
ca.
situação dos cordelistas não mudaria a não ser que eles mes-
mos requizessem (requeressem) o respeito que faziam por
3-) _Restam___dúvidas
merecer.
mesmo que a ameaça dos preços do carbono e da água
(D) Se não proveem (provêm) do preconceito, a desvalo-
em si __faça __diferença rização e a pouca visibilidade dessa arte popular tão rica só
a maioria das políticas de crescimento verde sempre pode (podem) ser resultado do puro e simples desconheci-
____será_____ a segunda opção. mento.
Em “a maioria de”, a concordância pode ser dupla: tanto (E) Rodolfo Coelho Cavalcante entreveu (entreviu) que os
no plural quanto no singular. Nas alternativas não há “res- problemas dos cordelistas estavam diretamente ligados à fal-
tam/faça/serão”, portanto a A é que apresenta as opções ta de representatividade.
adequadas.
8-) Fiz as correções entre parênteses:
4-) a) O desenraizamento, não só entre intelectuais como en-
(A) Ainda assim, temos certeza de que ninguém encon- tre os mais diversos tipos de pessoas, das mais sofisticadas às
trou até agora uma maneira adequada de se quantificar os mais humildes, são (é) cada vez mais comuns (comum) nos
insumos básicos. dias de hoje.
(B) Ainda assim, temos certeza de que ninguém encon- b) A importância de intelectuais como Edward Said e Tony
trou até agora uma maneira adequada de os insumos bási- Judt, que não se furtaram ao debate sobre questões polê-
cos serem quantificados. micas de seu tempo, não estão (está) apenas nos livros que
(C) Ainda assim, temos certeza de que ninguém encon- escreveram.
trou até agora uma maneira adequada para que os insu- c) Nada indica que o conflito no Oriente Médio entre ára-
mos básicos sejam quantificados. bes e judeus, responsável por tantas mortes e tanto sofrimen-
(D) Ainda assim, temos certeza de que ninguém encon- to, estejam (esteja) próximos (próximo) de serem (ser) resolvi-
trou até agora uma maneira adequada para que os insu- dos (resolvido) ou pelo menos de terem (ter) alguma trégua.
mos básicos sejam quantificados. d) Intelectuais que têm compromisso apenas com a ver-
(E) Ainda assim, temos certeza de que ninguém encon- dade, ainda que conscientes de que esta é até certo ponto
trou até agora uma maneira adequada de se quantificarem relativa, costumam encontrar muito mais detratores que ad-
os insumos básicos. = correta miradores.
e) No final do século XX já não se via (viam) muitos inte-
lectuais e escritores como Edward Said, que não apenas era
(eram) notícia pelos livros que publicavam como pelas posi-
ções que corajosamente assumiam.

34
LÍNGUA PORTUGUESA

9-) Em termos morfológicos, os pronomes são palavras


(A) Não há dúvida de que o estilo de vida... (dúvidas) = variáveis em gênero (masculino ou feminino) e em número
“há” permaneceria no singular (singular ou plural). Assim, espera-se que a referência atra-
(B) O que não se sabe ... (ninguém nas regiões do pla- vés do pronome seja coerente em termos de gênero e nú-
neta) = “sabe” permaneceria no singular mero (fenômeno da concordância) com o seu objeto, mes-
(C) O consumo mundial não dá sinal de trégua ... (O mo quando este se apresenta ausente no enunciado.
consumo mundial de barris de petróleo) = “dá” permane-
ceria no singular Fala-se de Roberta. Ele quer participar do desfile da nossa
(D) Um aumento elevado no preço do óleo reflete-se no escola neste ano.
custo da matéria-prima... Constantes aumentos) = “reflete” [nossa: pronome que qualifica “escola” = concordância
passaria para “refletem-se” adequada]
(E) o tema das mudanças climáticas pressiona os esfor- [neste: pronome que determina “ano” = concordância
ços mundiais... (a preocupação em torno das mudanças cli- adequada]
máticas) = “pressiona” permaneceria no singular [ele: pronome que faz referência à “Roberta” = concor-
dância inadequada]
10-) Fiz as correções: Existem seis tipos de pronomes: pessoais, possessivos,
(A) Fazem dez anos = faz (sentido de tempo = singular) demonstrativos, indefinidos, relativos e interrogativos.
(B) Ainda existe muitas pessoas = existem
(C) No trabalho em meio a tanta sujeira, havia altos ris- Pronomes Pessoais
cos
(D) Eu passava a manhã no subterrâneo: quando era São aqueles que substituem os substantivos, indicando
sete da manhã = eram diretamente as pessoas do discurso. Quem fala ou escreve
(E) As companhias de limpeza, apenas recentemente, assume os pronomes “eu” ou “nós”, usa os pronomes “tu”,
começou = começaram “vós”, “você” ou “vocês” para designar a quem se dirige e
“ele”, “ela”, “eles” ou “elas” para fazer referência à pessoa ou
às pessoas de quem fala.
5. EMPREGO DE PRONOMES E CRASES; Os pronomes pessoais variam de acordo com as funções
que exercem nas orações, podendo ser do caso reto ou do
caso oblíquo.
PRONOME
Pronome Reto
Pronome é a palavra que se usa em lugar do nome, ou
a ele se refere, ou que acompanha o nome, qualificando-o Pronome pessoal do caso reto é aquele que, na sentença,
de alguma forma. exerce a função de sujeito ou predicativo do sujeito.
Nós lhe ofertamos flores.
A moça era mesmo bonita. Ela morava nos meus sonhos!
[substituição do nome] Os pronomes retos apresentam flexão de número, gêne-
A moça que morava nos meus sonhos era mesmo bonita! ro (apenas na 3ª pessoa) e pessoa, sendo essa última a prin-
[referência ao nome] cipal flexão, uma vez que marca a pessoa do discurso. Dessa
Essa moça morava nos meus sonhos! forma, o quadro dos pronomes retos é assim configurado:
[qualificação do nome]
- 1ª pessoa do singular: eu
Grande parte dos pronomes não possuem significados - 2ª pessoa do singular: tu
fixos, isto é, essas palavras só adquirem significação dentro - 3ª pessoa do singular: ele, ela
de um contexto, o qual nos permite recuperar a referên- - 1ª pessoa do plural: nós
cia exata daquilo que está sendo colocado por meio dos - 2ª pessoa do plural: vós
pronomes no ato da comunicação. Com exceção dos pro- - 3ª pessoa do plural: eles, elas
nomes interrogativos e indefinidos, os demais pronomes
têm por função principal apontar para as pessoas do dis- Atenção: esses pronomes não costumam ser usados
curso ou a elas se relacionar, indicando-lhes sua situação como complementos verbais na língua-padrão. Frases como
no tempo ou no espaço. Em virtude dessa característica, “Vi ele na rua”, “Encontrei ela na praça”, “Trouxeram eu até
os pronomes apresentam uma forma específica para cada aqui”, comuns na língua oral cotidiana, devem ser evitadas
pessoa do discurso. na língua formal escrita ou falada. Na língua formal, devem
ser usados os pronomes oblíquos correspondentes: “Vi-o na
Minha carteira estava vazia quando eu fui assaltada. rua”, “Encontrei-a na praça”, “Trouxeram-me até aqui”.
[minha/eu: pronomes de 1ª pessoa = aquele que fala] Obs.: frequentemente observamos a omissão do prono-
Tua carteira estava vazia quando tu foste assaltada? me reto em Língua Portuguesa. Isso se dá porque as pró-
[tua/tu: pronomes de 2ª pessoa = aquele a quem se fala] prias formas verbais marcam, através de suas desinências,
A carteira dela estava vazia quando ela foi assaltada. as pessoas do verbo indicadas pelo pronome reto: Fizemos
[dela/ela: pronomes de 3ª pessoa = aquele de quem se fala] boa viagem. (Nós)

35
LÍNGUA PORTUGUESA

Pronome Oblíquo fiz + o = fi-lo


fazeis + o = fazei-lo
Pronome pessoal do caso oblíquo é aquele que, na sen- dizer + a = dizê-la
tença, exerce a função de complemento verbal (objeto di-
reto ou indireto) ou complemento nominal. Quando o verbo termina em som nasal, o pronome assu-
Ofertaram-nos flores. (objeto indireto) me as formas no, nos, na, nas. Por exemplo:
viram + o: viram-no
Obs.: em verdade, o pronome oblíquo é uma forma repõe + os = repõe-nos
variante do pronome pessoal do caso reto. Essa variação retém + a: retém-na
indica a função diversa que eles desempenham na oração: tem + as = tem-nas
pronome reto marca o sujeito da oração; pronome oblíquo
marca o complemento da oração. Pronome Oblíquo Tônico
Os pronomes oblíquos sofrem variação de acordo com
a acentuação tônica que possuem, podendo ser átonos ou Os pronomes oblíquos tônicos são sempre precedidos
tônicos. por preposições, em geral as preposições a, para, de e com.
Por esse motivo, os pronomes tônicos exercem a função de
Pronome Oblíquo Átono objeto indireto da oração. Possuem acentuação tônica forte.
O quadro dos pronomes oblíquos tônicos é assim con-
São chamados átonos os pronomes oblíquos que não figurado:
são precedidos de preposição. Possuem acentuação tônica - 1ª pessoa do singular (eu): mim, comigo
fraca: Ele me deu um presente. - 2ª pessoa do singular (tu): ti, contigo
O quadro dos pronomes oblíquos átonos é assim con- - 3ª pessoa do singular (ele, ela): ele, ela
figurado: - 1ª pessoa do plural (nós): nós, conosco
- 1ª pessoa do singular (eu): me - 2ª pessoa do plural (vós): vós, convosco
- 3ª pessoa do plural (eles, elas): eles, elas
- 2ª pessoa do singular (tu): te
- 3ª pessoa do singular (ele, ela): o, a, lhe
Observe que as únicas formas próprias do pronome tô-
- 1ª pessoa do plural (nós): nos
nico são a primeira pessoa (mim) e segunda pessoa (ti). As
- 2ª pessoa do plural (vós): vos
demais repetem a forma do pronome pessoal do caso reto.
- 3ª pessoa do plural (eles, elas): os, as, lhes
- As preposições essenciais introduzem sempre prono-
mes pessoais do caso oblíquo e nunca pronome do caso
Observações:
reto. Nos contextos interlocutivos que exigem o uso da lín-
O “lhe” é o único pronome oblíquo átono que já se gua formal, os pronomes costumam ser usados desta forma:
apresenta na forma contraída, ou seja, houve a união en- Não há mais nada entre mim e ti.
tre o pronome “o” ou “a” e preposição “a” ou “para”. Por Não se comprovou qualquer ligação entre ti e ela.
acompanhar diretamente uma preposição, o pronome Não há nenhuma acusação contra mim.
“lhe” exerce sempre a função de objeto indireto na oração. Não vá sem mim.
Os pronomes me, te, nos e vos podem tanto ser objetos
diretos como objetos indiretos. Atenção: Há construções em que a preposição, apesar
Os pronomes o, a, os e as atuam exclusivamente como de surgir anteposta a um pronome, serve para introduzir
objetos diretos. uma oração cujo verbo está no infinitivo. Nesses casos, o
Os pronomes me, te, lhe, nos, vos e lhes podem combi- verbo pode ter sujeito expresso; se esse sujeito for um pro-
nar-se com os pronomes o, os, a, as, dando origem a for- nome, deverá ser do caso reto.
mas como mo, mos , ma, mas; to, tos, ta, tas; lho, lhos, lha, Trouxeram vários vestidos para eu experimentar.
lhas; no-lo, no-los, no-la, no-las, vo-lo, vo-los, vo-la, vo-las. Não vá sem eu mandar.
Observe o uso dessas formas nos exemplos que seguem:
- Trouxeste o pacote? - A combinação da preposição “com” e alguns pronomes
- Sim, entreguei-to ainda há pouco. originou as formas especiais comigo, contigo, consigo, conos-
- Não contaram a novidade a vocês? co e convosco. Tais pronomes oblíquos tônicos frequente-
- Não, no-la contaram. mente exercem a função de adjunto adverbial de compa-
No português do Brasil, essas combinações não são nhia.
usadas; até mesmo na língua literária atual, seu emprego Ele carregava o documento consigo.
é muito raro. - As formas “conosco” e “convosco” são substituídas por
“com nós” e “com vós” quando os pronomes pessoais são
Atenção: Os pronomes o, os, a, as assumem formas reforçados por palavras como outros, mesmos, próprios, to-
especiais depois de certas terminações verbais. Quando o dos, ambos ou algum numeral.
verbo termina em -z, -s ou -r, o pronome assume a forma Você terá de viajar com nós todos.
lo, los, la ou las, ao mesmo tempo que a terminação verbal Estávamos com vós outros quando chegaram as más no-
é suprimida. Por exemplo: tícias.
Ele disse que iria com nós três.

36
LÍNGUA PORTUGUESA

Pronome Reflexivo

São pronomes pessoais oblíquos que, embora funcionem como objetos direto ou indireto, referem-se ao sujeito da
oração. Indicam que o sujeito pratica e recebe a ação expressa pelo verbo.
O quadro dos pronomes reflexivos é assim configurado:
- 1ª pessoa do singular (eu): me, mim.
Eu não me vanglorio disso.
Olhei para mim no espelho e não gostei do que vi.

- 2ª pessoa do singular (tu): te, ti.


Assim tu te prejudicas.
Conhece a ti mesmo.

- 3ª pessoa do singular (ele, ela): se, si, consigo.


Guilherme já se preparou.
Ela deu a si um presente.
Antônio conversou consigo mesmo.

- 1ª pessoa do plural (nós): nos.


Lavamo-nos no rio.

- 2ª pessoa do plural (vós): vos.


Vós vos beneficiastes com a esta conquista.

- 3ª pessoa do plural (eles, elas): se, si, consigo.


Eles se conheceram.
Elas deram a si um dia de folga.

A Segunda Pessoa Indireta

A chamada segunda pessoa indireta manifesta-se quando utilizamos pronomes que, apesar de indicarem nosso interlo-
cutor (portanto, a segunda pessoa), utilizam o verbo na terceira pessoa. É o caso dos chamados pronomes de tratamento,
que podem ser observados no quadro seguinte:

Pronomes de Tratamento

Vossa Alteza V. A. príncipes, duques


Vossa Eminência V. Ema.(s) cardeais
Vossa Reverendíssima V. Revma.(s) acerdotes e bispos
Vossa Excelência V. Ex.ª (s) altas autoridades e oficiais-generais
Vossa Magnificência V. Mag.ª (s) reitores de universidades
Vossa Majestade V. M. reis e rainhas
Vossa Majestade Imperial V. M. I. Imperadores
Vossa Santidade V. S. Papa
Vossa Senhoria V. S.ª (s) tratamento cerimonioso
Vossa Onipotência V. O. Deus

Também são pronomes de tratamento o senhor, a senhora e você, vocês. “O senhor” e “a senhora” são empregados no
tratamento cerimonioso; “você” e “vocês”, no tratamento familiar. Você e vocês são largamente empregados no português
do Brasil; em algumas regiões, a forma tu é de uso frequente; em outras, pouco empregada. Já a forma vós tem uso restrito
à linguagem litúrgica, ultraformal ou literária.

Observações:
a) Vossa Excelência X Sua Excelência : os pronomes de tratamento que possuem “Vossa (s)” são empregados em relação
à pessoa com quem falamos: Espero que V. Ex.ª, Senhor Ministro, compareça a este encontro.

*Emprega-se “Sua (s)” quando se fala a respeito da pessoa.
Todos os membros da C.P.I. afirmaram que Sua Excelência, o Senhor Presidente da República, agiu com propriedade.

37
LÍNGUA PORTUGUESA

- Os pronomes de tratamento representam uma forma indireta de nos dirigirmos aos nossos interlocutores. Ao tratar-
mos um deputado por Vossa Excelência, por exemplo, estamos nos endereçando à excelência que esse deputado suposta-
mente tem para poder ocupar o cargo que ocupa.

- 3ª pessoa: embora os pronomes de tratamento dirijam-se à 2ª pessoa, toda a concordância deve ser feita com a 3ª pes-
soa. Assim, os verbos, os pronomes possessivos e os pronomes oblíquos empregados em relação a eles devem ficar na 3ª pessoa.
Basta que V. Ex.ª cumpra a terça parte das suas promessas, para que seus eleitores lhe fiquem reconhecidos.

- Uniformidade de Tratamento: quando escrevemos ou nos dirigimos a alguém, não é permitido mudar, ao longo do
texto, a pessoa do tratamento escolhida inicialmente. Assim, por exemplo, se começamos a chamar alguém de “você”, não
poderemos usar “te” ou “teu”. O uso correto exigirá, ainda, verbo na terceira pessoa.
Quando você vier, eu te abraçarei e enrolar-me-ei nos teus cabelos. (errado)
Quando você vier, eu a abraçarei e enrolar-me-ei nos seus cabelos. (correto)
Quando tu vieres, eu te abraçarei e enrolar-me-ei nos teus cabelos. (correto)

Pronomes Possessivos

São palavras que, ao indicarem a pessoa gramatical (possuidor), acrescentam a ela a ideia de posse de algo (coisa pos-
suída).
Este caderno é meu. (meu = possuidor: 1ª pessoa do singular)

NÚMERO PESSOA PRONOME


singular primeira meu(s), minha(s)
singular segunda teu(s), tua(s)
singular terceira seu(s), sua(s)
plural primeira nosso(s), nossa(s)
plural segunda vosso(s), vossa(s)
plural terceira seu(s), sua(s)

Note que: A forma do possessivo depende da pessoa gramatical a que se refere; o gênero e o número concordam com
o objeto possuído: Ele trouxe seu apoio e sua contribuição naquele momento difícil.

Observações:

1 - A forma “seu” não é um possessivo quando resultar da alteração fonética da palavra senhor: Muito obrigado, seu José.

2 - Os pronomes possessivos nem sempre indicam posse. Podem ter outros empregos, como:
a) indicar afetividade: Não faça isso, minha filha.

b) indicar cálculo aproximado: Ele já deve ter seus 40 anos.

c) atribuir valor indefinido ao substantivo: Marisa tem lá seus defeitos, mas eu gosto muito dela.

3- Em frases onde se usam pronomes de tratamento, o pronome possessivo fica na 3ª pessoa: Vossa Excelência trouxe
sua mensagem?

4- Referindo-se a mais de um substantivo, o possessivo concorda com o mais próximo: Trouxe-me seus livros e anotações.

5- Em algumas construções, os pronomes pessoais oblíquos átonos assumem valor de possessivo: Vou seguir-lhe os
passos. (= Vou seguir seus passos.)

Pronomes Demonstrativos

Os pronomes demonstrativos são utilizados para explicitar a posição de uma certa palavra em relação a outras ou ao
contexto. Essa relação pode ocorrer em termos de espaço, no tempo ou discurso.
No espaço:
Compro este carro (aqui). O pronome este indica que o carro está perto da pessoa que fala.
Compro esse carro (aí). O pronome esse indica que o carro está perto da pessoa com quem falo, ou afastado da pessoa que fala.
Compro aquele carro (lá). O pronome aquele diz que o carro está afastado da pessoa que fala e daquela com quem falo.

38
LÍNGUA PORTUGUESA

Atenção: em situações de fala direta (tanto ao vivo - Em frases como a seguinte, este se refere à pessoa
quanto por meio de correspondência, que é uma moda- mencionada em último lugar; aquele, à mencionada em
lidade escrita de fala), são particularmente importantes o primeiro lugar: O referido deputado e o Dr. Alcides eram
este e o esse - o primeiro localiza os seres em relação ao amigos íntimos; aquele casado, solteiro este. [ou então: este
emissor; o segundo, em relação ao destinatário. Trocá-los solteiro, aquele casado]
pode causar ambiguidade.
Dirijo-me a essa universidade com o objetivo de solicitar - O pronome demonstrativo tal pode ter conotação irô-
informações sobre o concurso vestibular. (trata-se da univer- nica: A menina foi a tal que ameaçou o professor?
sidade destinatária).
Reafirmamos a disposição desta universidade em partici- - Pode ocorrer a contração das preposições a, de, em
par no próximo Encontro de Jovens. (trata-se da universida- com pronome demonstrativo: àquele, àquela, deste, desta,
de que envia a mensagem). disso, nisso, no, etc: Não acreditei no que estava vendo. (no
No tempo: = naquilo)
Este ano está sendo bom para nós. O pronome este se
refere ao ano presente. Pronomes Indefinidos
Esse ano que passou foi razoável. O pronome esse se
refere a um passado próximo. São palavras que se referem à terceira pessoa do dis-
Aquele ano foi terrível para todos. O pronome aquele curso, dando-lhe sentido vago (impreciso) ou expressando
está se referindo a um passado distante. quantidade indeterminada.
Alguém entrou no jardim e destruiu as mudas recém-
- Os pronomes demonstrativos podem ser variáveis ou -plantadas.
invariáveis, observe:
Variáveis: este(s), esta(s), esse(s), essa(s), aquele(s), aque- Não é difícil perceber que “alguém” indica uma pessoa
la(s). de quem se fala (uma terceira pessoa, portanto) de forma
Invariáveis: isto, isso, aquilo.
imprecisa, vaga. É uma palavra capaz de indicar um ser hu-
- Também aparecem como pronomes demonstrativos:
mano que seguramente existe, mas cuja identidade é des-
- o(s), a(s): quando estiverem antecedendo o “que” e
conhecida ou não se quer revelar. Classificam-se em:
puderem ser substituídos por aquele(s), aquela(s), aquilo.
- Pronomes Indefinidos Substantivos: assumem o lu-
Não ouvi o que disseste. (Não ouvi aquilo que disseste.)
gar do ser ou da quantidade aproximada de seres na frase.
Essa rua não é a que te indiquei. (Esta rua não é aquela
São eles: algo, alguém, fulano, sicrano, beltrano, nada, nin-
que te indiquei.)
guém, outrem, quem, tudo.
- mesmo(s), mesma(s): Estas são as mesmas pessoas que Algo o incomoda?
o procuraram ontem. Quem avisa amigo é.

- próprio(s), própria(s): Os próprios alunos resolveram - Pronomes Indefinidos Adjetivos: qualificam um ser
o problema. expresso na frase, conferindo-lhe a noção de quantidade
aproximada. São eles: cada, certo(s), certa(s).
- semelhante(s): Não compre semelhante livro. Cada povo tem seus costumes.
Certas pessoas exercem várias profissões.
- tal, tais: Tal era a solução para o problema.
Note que: Ora são pronomes indefinidos substantivos,
Note que: ora pronomes indefinidos adjetivos:
- Não raro os demonstrativos aparecem na frase, em algum, alguns, alguma(s), bastante(s) (= muito, muitos),
construções redundantes, com finalidade expressiva, para demais, mais, menos, muito(s), muita(s), nenhum, nenhuns,
salientar algum termo anterior. Por exemplo: Manuela, essa nenhuma(s), outro(s), outra(s), pouco(s), pouca(s), qualquer,
é que dera em cheio casando com o José Afonso. Desfrutar quaisquer, qual, que, quanto(s), quanta(s), tal, tais, tanto(s),
das belezas brasileiras, isso é que é sorte! tanta(s), todo(s), toda(s), um, uns, uma(s), vários, várias.
Menos palavras e mais ações.
- O pronome demonstrativo neutro ou pode represen- Alguns se contentam pouco.
tar um termo ou o conteúdo de uma oração inteira, caso Os pronomes indefinidos podem ser divididos em va-
em que aparece, geralmente, como objeto direto, predi- riáveis e invariáveis. Observe:
cativo ou aposto: O casamento seria um desastre. Todos o Variáveis = algum, nenhum, todo, muito, pouco, vário,
pressentiam. tanto, outro, quanto, alguma, nenhuma, toda, muita, pouca,
vária, tanta, outra, quanta, qualquer, quaisquer, alguns, ne-
- Para evitar a repetição de um verbo anteriormente ex- nhuns, todos, muitos, poucos, vários, tantos, outros, quantos,
presso, é comum empregar-se, em tais casos, o verbo fazer, algumas, nenhumas, todas, muitas, poucas, várias, tantas,
chamado, então, verbo vicário (= que substitui, que faz as outras, quantas.
vezes de): Ninguém teve coragem de falar antes que ela o Invariáveis = alguém, ninguém, outrem, tudo, nada,
fizesse. algo, cada.

39
LÍNGUA PORTUGUESA

São locuções pronominais indefinidas: O trabalho que eu fiz refere-se à corrupção. (= o qual)
A cantora que acabou de se apresentar é péssima. (= a
cada qual, cada um, qualquer um, quantos quer (que), qual)
quem quer (que), seja quem for, seja qual for, todo aquele Os trabalhos que eu fiz referem-se à corrupção. (= os quais)
(que), tal qual (= certo), tal e qual, tal ou qual, um ou outro, As cantoras que se apresentaram eram péssimas. (= as
uma ou outra, etc. quais)
Cada um escolheu o vinho desejado.
- O qual, os quais, a qual e as quais são exclusivamente
Indefinidos Sistemáticos pronomes relativos: por isso, são utilizados didaticamente
para verificar se palavras como “que”, “quem”, “onde” (que
Ao observar atentamente os pronomes indefinidos, per- podem ter várias classificações) são pronomes relativos. To-
cebemos que existem alguns grupos que criam oposição dos eles são usados com referência à pessoa ou coisa por
de sentido. É o caso de: algum/alguém/algo, que têm sen- motivo de clareza ou depois de determinadas preposições:
tido afirmativo, e nenhum/ninguém/nada, que têm sentido Regressando de São Paulo, visitei o sítio de minha tia, o qual
negativo; todo/tudo, que indicam uma totalidade afirmati- me deixou encantado. (O uso de “que”, neste caso, geraria
va, e nenhum/nada, que indicam uma totalidade negativa; ambiguidade.)
alguém/ninguém, que se referem à pessoa, e algo/nada, Essas são as conclusões sobre as quais pairam muitas dúvi-
que se referem à coisa; certo, que particulariza, e qualquer, das? (Não se poderia usar “que” depois de sobre.)
que generaliza.
Essas oposições de sentido são muito importantes na - O relativo “que” às vezes equivale a o que, coisa que, e se
construção de frases e textos coerentes, pois delas muitas refere a uma oração: Não chegou a ser padre, mas deixou de
vezes dependem a solidez e a consistência dos argumen- ser poeta, que era a sua vocação natural.
tos expostos. Observe nas frases seguintes a força que os
pronomes indefinidos destacados imprimem às afirmações - O pronome “cujo” não concorda com o seu anteceden-
te, mas com o consequente. Equivale a do qual, da qual, dos
de que fazem parte:
quais, das quais.
Nada do que tem sido feito produziu qualquer resultado
Este é o caderno cujas folhas estão rasgadas.
prático.
(antecedente) (consequente)
Certas pessoas conseguem perceber sutilezas: não são
pessoas quaisquer.
- “Quanto” é pronome relativo quando tem por antece-
dente um pronome indefinido: tanto (ou variações) e tudo:
Pronomes Relativos
Emprestei tantos quantos foram necessários.
(antecedente)
São aqueles que representam nomes já mencionados Ele fez tudo quanto havia falado.
anteriormente e com os quais se relacionam. Introduzem (antecedente)
as orações subordinadas adjetivas. - O pronome “quem” se refere a pessoas e vem sempre
O racismo é um sistema que afirma a superioridade de precedido de preposição.
um grupo racial sobre outros. É um professor a quem muito devemos.
(afirma a superioridade de um grupo racial sobre outros (preposição)
= oração subordinada adjetiva).
O pronome relativo “que” refere-se à palavra “sistema” - “Onde”, como pronome relativo, sempre possui antece-
e introduz uma oração subordinada. Diz-se que a palavra dente e só pode ser utilizado na indicação de lugar: A casa
“sistema” é antecedente do pronome relativo que. onde morava foi assaltada.
O antecedente do pronome relativo pode ser o prono- - Na indicação de tempo, deve-se empregar quando ou
me demonstrativo o, a, os, as. em que.
Não sei o que você está querendo dizer. Sinto saudades da época em que (quando) morávamos no
Às vezes, o antecedente do pronome relativo não vem exterior.
expresso.
Quem casa, quer casa. - Podem ser utilizadas como pronomes relativos as pa-
lavras:
Observe: - como (= pelo qual): Não me parece correto o modo como
Pronomes relativos variáveis = o qual, cujo, quanto, os você agiu semana passada.
quais, cujos, quantos, a qual, cuja, quanta, as quais, cujas, - quando (= em que): Bons eram os tempos quando podía-
quantas. mos jogar videogame.
Pronomes relativos invariáveis = quem, que, onde.
Note que: - Os pronomes relativos permitem reunir duas orações
- O pronome “que” é o relativo de mais largo emprego, numa só frase.
sendo por isso chamado relativo universal. Pode ser subs- O futebol é um esporte.
tituído por o qual, a qual, os quais, as quais, quando seu O povo gosta muito deste esporte.
antecedente for um substantivo. O futebol é um esporte de que o povo gosta muito.

40
LÍNGUA PORTUGUESA

- Numa série de orações adjetivas coordenadas, pode O pronome oblíquo átono pode assumir três posições
ocorrer a elipse do relativo “que”: A sala estava cheia de na oração em relação ao verbo:
gente que conversava, (que) ria, (que) fumava. 1. próclise: pronome antes do verbo
2. ênclise: pronome depois do verbo
Pronomes Interrogativos 3. mesóclise: pronome no meio do verbo

São usados na formulação de perguntas, sejam elas di- Próclise


retas ou indiretas. Assim como os pronomes indefinidos,
referem-se à 3ª pessoa do discurso de modo impreciso. A próclise é aplicada antes do verbo quando temos:
São pronomes interrogativos: que, quem, qual (e varia- - Palavras com sentido negativo:
ções), quanto (e variações). Nada me faz querer sair dessa cama.
Quem fez o almoço?/ Diga-me quem fez o almoço. Não se trata de nenhuma novidade.
Qual das bonecas preferes? / Não sei qual das bonecas
preferes. - Advérbios:
Quantos passageiros desembarcaram? / Pergunte quan- Nesta casa se fala alemão.
tos passageiros desembarcaram. Naquele dia me falaram que a professora não veio.

Sobre os pronomes: - Pronomes relativos:


A aluna que me mostrou a tarefa não veio hoje.
O pronome pessoal é do caso reto quando tem função Não vou deixar de estudar os conteúdos que me falaram.
de sujeito na frase. O pronome pessoal é do caso oblíquo
quando desempenha função de complemento. Vamos en- - Pronomes indefinidos:
tender, primeiramente, como o pronome pessoal surge na Quem me disse isso?
frase e que função exerce. Observe as orações: Todos se comoveram durante o discurso de despedida.
1. Eu não sei essa matéria, mas ele irá me ajudar.
2. Maria foi embora para casa, pois não sabia se devia - Pronomes demonstrativos:
lhe ajudar. Isso me deixa muito feliz!
Aquilo me incentivou a mudar de atitude!
Na primeira oração os pronomes pessoais “eu” e “ele”
exercem função de sujeito, logo, são pertencentes ao caso - Preposição seguida de gerúndio:
reto. Já na segunda oração, observamos o pronome “lhe” Em se tratando de qualidade, o Brasil Escola é o site mais
exercendo função de complemento, e, consequentemente, indicado à pesquisa escolar.
é do caso oblíquo.
Os pronomes pessoais indicam as pessoas do discurso, - Conjunção subordinativa:
o pronome oblíquo “lhe”, da segunda oração, aponta para Vamos estabelecer critérios, conforme lhe avisaram.
a segunda pessoa do singular (tu/você): Maria não sabia se
devia ajudar.... Ajudar quem? Você (lhe). Ênclise
Importante: Em observação à segunda oração, o em-
prego do pronome oblíquo “lhe” é justificado antes do ver- A ênclise é empregada depois do verbo. A norma culta
bo intransitivo “ajudar” porque o pronome oblíquo pode não aceita orações iniciadas com pronomes oblíquos áto-
estar antes, depois ou entre locução verbal, caso o verbo nos. A ênclise vai acontecer quando:
principal (no caso “ajudar”) esteja no infinitivo ou gerúndio. - O verbo estiver no imperativo afirmativo:
Eu desejo lhe perguntar algo. Amem-se uns aos outros.
Eu estou perguntando-lhe algo. Sigam-me e não terão derrotas.

Os pronomes pessoais oblíquos podem ser átonos ou - O verbo iniciar a oração:


tônicos: os primeiros não são precedidos de preposição, Diga-lhe que está tudo bem.
diferentemente dos segundos que são sempre precedidos Chamaram-me para ser sócio.
de preposição.
- Pronome oblíquo átono: Joana me perguntou o que eu - O verbo estiver no infinitivo impessoal regido da pre-
estava fazendo. posição “a”:
- Pronome oblíquo tônico: Joana perguntou para mim o Naquele instante os dois passaram a odiar-se.
que eu estava fazendo. Passaram a cumprimentar-se mutuamente.
- O verbo estiver no gerúndio:
A colocação pronominal é a posição que os pronomes Não quis saber o que aconteceu, fazendo-se de despreo-
pessoais oblíquos átonos ocupam na frase em relação ao cupada.
verbo a que se referem. São pronomes oblíquos átonos: Despediu-se, beijando-me a face.
me, te, se, o, os, a, as, lhe, lhes, nos e vos.

41
LÍNGUA PORTUGUESA

- Houver vírgula ou pausa antes do verbo: 04. (Papiloscopista Policial – Vunesp – 2013). Assinale a
Se passar no concurso em outra cidade, mudo-me no alternativa em que o pronome destacado está posicionado
mesmo instante. de acordo com a norma-padrão da língua.
Se não tiver outro jeito, alisto-me nas forças armadas. (A) Ela não lembrava-se do caminho de volta.
(B) A menina tinha distanciado-se muito da família.
Mesóclise (C) A garota disse que perdeu-se dos pais.
(D) O pai alegrou-se ao encontrar a filha.
A mesóclise acontece quando o verbo está flexionado (E) Ninguém comprometeu-se a ajudar a criança.
no futuro do presente ou no futuro do pretérito:
A prova realizar-se-á neste domingo pela manhã. (= ela 05. (Escrevente TJ SP – Vunesp 2011). Assinale a alterna-
se realizará) tiva cujo emprego do pronome está em conformidade com
Far-lhe-ei uma proposta irrecusável. (= eu farei uma a norma padrão da língua.
proposta a você) (A) Não autorizam-nos a ler os comentários sigilosos.
(B) Nos falaram que a diplomacia americana está aba-
Questões sobre Pronome lada.
(C) Ninguém o informou sobre o caso WikiLeaks.
01. (Escrevente TJ SP – Vunesp/2012). (D) Conformado, se rendeu às punições.
Restam dúvidas sobre o crescimento verde. Primeiro, não (E) Todos querem que combata-se a corrupção.
está claro até onde pode realmente chegar uma política ba-
seada em melhorar a eficiência sem preços adequados para 06. (Papiloscopista Policial = Vunesp - 2013). Assinale
o carbono, a água e (na maioria dos países pobres) a terra. a alternativa correta quanto à colocação pronominal, de
É verdade que mesmo que a ameaça dos preços do carbono acordo com a norma-padrão da língua portuguesa.
e da água faça em si diferença, as companhias não podem (A) Para que se evite perder objetos, recomenda-se que
suportar ter de pagar, de repente, digamos, 40 dólares por eles sejam sempre trazidos junto ao corpo.
tonelada de carbono, sem qualquer preparação. Portanto, (B) O passageiro ao lado jamais imaginou-se na situa-
elas começam a usar preços-sombra. Ainda assim, ninguém ção de ter de procurar a dona de uma bolsa perdida.
encontrou até agora uma maneira de quantificar adequada-
(C) Nos sentimos impotentes quando não conseguimos
mente os insumos básicos. E sem eles a maioria das políticas
restituir um objeto à pessoa que o perdeu.
de crescimento verde sempre será a segunda opção.
(D) O homem se indignou quando propuseram-lhe que
(Carta Capital, 27.06.2012. Adaptado)
abrisse a bolsa que encontrara.
Os pronomes “elas” e “eles”, em destaque no texto, re-
(E) Em tratando-se de objetos encontrados, há uma ten-
ferem- -se, respectivamente, a
dência natural das pessoas em devolvê-los a seus donos.
(A) dúvidas e preços.
(B) dúvidas e insumos básicos.
(C) companhias e insumos básicos. 07. (Agente de Apoio Operacional – VUNESP – 2013).
(D) companhias e preços do carbono e da água. Há pessoas que, mesmo sem condições, compram produ-
(E) políticas de crescimento e preços adequados. tos______ não necessitam e______ tendo de pagar tudo______
prazo.
02. (Agente de Apoio Administrativo – FCC – 2013- Assinale a alternativa que preenche as lacunas, correta
adap.). Fazendo-se as alterações necessárias, o trecho gri- e respectivamente, considerando a norma culta da língua.
fado está corretamente substituído por um pronome em: A) a que … acaba … à
A) ...sei tratar tipos como o senhor. − sei tratá-lo B) com que … acabam … à
B) ...erguendo os braços desalentado... − erguendo- C) de que … acabam … a
lhes desalentado D) em que … acaba … a
C) ...que tem de conhecer as leis do país? − que tem de E) dos quais … acaba … à
conhecê-lo?
D) ...não parecia ser um importante industrial... − não
parecia ser-lhe 08. (Agente de Apoio Socioeducativo – VUNESP – 2013-
E) incomodaram o general... − incomodaram-no adap.). Assinale a alternativa que substitui, correta e res-
pectivamente, as lacunas do trecho.
03.(Agente de Defensoria Pública – FCC – 2013-adap.). ______alguns anos, num programa de televisão, uma jo-
A substituição do elemento grifado pelo pronome cor- vem fazia referência______ violência______ o brasileiro estava
respondente, com os necessários ajustes, foi realizada de sujeito de forma cômica.
modo INCORRETO em: A) Fazem... a ... de que
A) mostrando o rio= mostrando-o. B) Faz ...a ... que
B) como escolher sítio= como escolhê-lo. C) Fazem ...à ... com que
C) transpor [...] as matas espessas= transpor-lhes. D) Faz ...à ... que
D) Às estreitas veredas[...] nada acrescentariam = nada E) Faz ...à ... a que
lhes acrescentariam.
E) viu uma dessas marcas= viu uma delas.

42
LÍNGUA PORTUGUESA

09. (TRF 3ª região- Técnico Judiciário - /2014) 4-)


As sereias então devoravam impiedosamente os tripu- (A) Ela não se lembrava do caminho de volta.
lantes. (B) A menina tinha se distanciado muito da família.
... ele conseguiu impedir a tripulação de perder a cabe- (C) A garota disse que se perdeu dos pais.
ça... (E) Ninguém se comprometeu a ajudar a criança
... e fez de tudo para convencer os tripulantes...
Fazendo-se as alterações necessárias, os segmentos 5-)
grifados acima foram corretamente substituídos por um (A) Não nos autorizam a ler os comentários sigilosos.
pronome, na ordem dada, em: (B) Falaram-nos que a diplomacia americana está aba-
(A) devoravam-nos − impedi-la − convencê-los lada.
(B) devoravam-lhe − impedi-las − convencer-lhes (D) Conformado, rendeu-se às punições.
(C) devoravam-no − impedi-las − convencer-lhes (E) Todos querem que se combata a corrupção.
(D) devoravam-nos − impedir-lhe − convencê-los
(E) devoravam-lhes − impedi-la − convencê-los 6-)
(B) O passageiro ao lado jamais se imaginou na situa-
10. (Agente de Vigilância e Recepção – VUNESP – 2013- ção de ter de procurar a dona de uma bolsa perdida.
adap.). No trecho, – Em ambos os casos, as câmeras dos (C) Sentimo-nos impotentes quando não consegui-
estabelecimentos felizmente comprovam os acontecimen- mos restituir um objeto à pessoa que o perdeu.
tos, e testemunhas vão ajudar a polícia na investigação. (D) O homem indignou-se quando lhe propuseram
– de acordo com a norma-padrão, os pronomes que subs- que abrisse a bolsa que encontrara.
tituem, corretamente, os termos em destaque são: (E) Em se tratando de objetos encontrados, há uma
A) os comprovam … ajudá-la. tendência natural das pessoas em devolvê-los a seus do-
B) os comprovam …ajudar-la. nos.
C) os comprovam … ajudar-lhe. 7-) Há pessoas que, mesmo sem condições, compram
D) lhes comprovam … ajudar-lhe. produtos de que não necessitam e acabam tendo
E) lhes comprovam … ajudá-la. de pagar tudo a prazo.

GABARITO 8-) Faz alguns anos, num programa de televisão, uma


jovem fazia referência à violência a que o brasileiro
01. C 02. E 03. C 04. D 05. C estava sujeito de forma cômica.
06. A 07. C 08. E 09. A 10. A Faz, no sentido de tempo passado = sempre no sin-
gular
RESOLUÇÃO
9-)
1-) Restam dúvidas sobre o crescimento verde. Primei- devoravam - verbo terminado em “m” = pronome
ro, não está claro até onde pode realmente chegar uma oblíquo no/na (fizeram-na, colocaram-no)
política baseada em melhorar a eficiência sem preços ade- impedir - verbo transitivo direto = pede objeto direto;
quados para o carbono, a água e (na maioria dos países “lhe” é para objeto indireto
pobres) a terra. É verdade que mesmo que a ameaça dos convencer - verbo transitivo direto = pede objeto di-
preços do carbono e da água faça em si diferença, as com- reto; “lhe” é para objeto indireto
panhias não podem suportar ter de pagar, de repente, di- (A) devoravam-nos − impedi-la − convencê-los
gamos, 40 dólares por tonelada de carbono, sem qualquer
preparação. Portanto, elas começam a usar preços-som- 10-) – Em ambos os casos, as câmeras dos estabe-
bra. Ainda assim, ninguém encontrou até agora uma ma- lecimentos felizmente comprovam os acontecimentos, e
neira de quantificar adequadamente os insumos básicos. testemunhas vão ajudar a polícia na investigação.
E sem eles a maioria das políticas de crescimento verde felizmente os comprovam ... ajudá-la
sempre será a segunda opção. (advérbio)

2-)
A) ...sei tratar tipos como o senhor. − sei tratá-los
B) ...erguendo os braços desalentado... − erguendo-os
desalentado
C) ...que tem de conhecer as leis do país? − que tem de
conhecê-las ?
D) ...não parecia ser um importante industrial... − não
parecia sê-lo

3-) transpor [...] as matas espessas= transpô-las

43
LÍNGUA PORTUGUESA

CRASE

A palavra crase é de origem grega e significa “fusão”, “mistura”. Na língua portuguesa, é o nome que se dá à “jun-
ção” de duas vogais idênticas. É de grande importância a crase da preposição “a” com o artigo feminino “a” (s), com o
“a” inicial dos pronomes aquele(s), aquela (s), aquilo e com o “a” do relativo a qual (as quais). Na escrita, utilizamos o
acento grave ( ` ) para indicar a crase. O uso apropriado do acento grave depende da compreensão da fusão das duas
vogais. É fundamental também, para o entendimento da crase, dominar a regência dos verbos e nomes que exigem a
preposição “a”. Aprender a usar a crase, portanto, consiste em aprender a verificar a ocorrência simultânea de uma pre-
posição e um artigo ou pronome. Observe:
Vou a + a igreja.
Vou à igreja.

No exemplo acima, temos a ocorrência da preposição “a”, exigida pelo verbo ir (ir a algum lugar) e a ocorrência do artigo
“a” que está determinando o substantivo feminino igreja. Quando ocorre esse encontro das duas vogais e elas se unem, a
união delas é indicada pelo acento grave. Observe os outros exemplos:
Conheço a aluna.
Refiro-me à aluna.
No primeiro exemplo, o verbo é transitivo direto (conhecer algo ou alguém), logo não exige preposição e a crase não
pode ocorrer. No segundo exemplo, o verbo é transitivo indireto (referir--se a algo ou a alguém) e exige a preposição “a”.
Portanto, a crase é possível, desde que o termo seguinte seja feminino e admita o artigo feminino “a” ou um dos pronomes
já especificados.

Casos em que a crase NÃO ocorre:

- diante de substantivos masculinos:


Andamos a cavalo.
Fomos a pé.
Passou a camisa a ferro.
Fazer o exercício a lápis.
Compramos os móveis a prazo.

- diante de verbos no infinitivo:


A criança começou a falar.
Ela não tem nada a dizer.

Obs.: como os verbos não admitem artigos, o “a” dos exemplos acima é apenas preposição, logo não ocorrerá crase.

- diante da maioria dos pronomes e das expressões de tratamento, com exceção das formas senhora, senhorita
e dona:
Diga a ela que não estarei em casa amanhã.
Entreguei a todos os documentos necessários.
Ele fez referência a Vossa Excelência no discurso de ontem.
Peço a Vossa Senhoria que aguarde alguns minutos.

Os poucos casos em que ocorre crase diante dos pronomes podem ser identificados pelo método: troque a palavra
feminina por uma masculina, caso na nova construção surgir a forma ao, ocorrerá crase. Por exemplo:
Refiro-me à mesma pessoa. (Refiro-me ao mesmo indivíduo.)
Informei o ocorrido à senhora. (Informei o ocorrido ao senhor.)
Peça à própria Cláudia para sair mais cedo. (Peça ao próprio Cláudio para sair mais cedo.)

- diante de numerais cardinais:


Chegou a duzentos o número de feridos.
Daqui a uma semana começa o campeonato.

44
LÍNGUA PORTUGUESA

Casos em que a crase SEMPRE ocorre:

- diante de palavras femininas:


Amanhã iremos à festa de aniversário de minha colega.
Sempre vamos à praia no verão.
Ela disse à irmã o que havia escutado pelos corredores.
Sou grata à população.
Fumar é prejudicial à saúde.
Este aparelho é posterior à invenção do telefone.

- diante da palavra “moda”, com o sentido de “à moda de” (mesmo que a expressão moda de fique subentendida):
O jogador fez um gol à (moda de) Pelé.
Usava sapatos à (moda de) Luís XV.
Estava com vontade de comer frango à (moda de) passarinho.
O menino resolveu vestir-se à (moda de) Fidel Castro.

- na indicação de horas:
Acordei às sete horas da manhã.
Elas chegaram às dez horas.
Foram dormir à meia-noite.

- em locuções adverbiais, prepositivas e conjuntivas de que participam palavras femininas. Por exemplo:
à tarde às ocultas às pressas à medida que
à noite às claras às escondidas à força
à vontade à beça à larga à escuta
às avessas à revelia à exceção de à imitação de
à esquerda às turras às vezes à chave
à direita à procura à deriva à toa
à luz à sombra de à frente de à proporção que
à semelhança de às ordens à beira de

Crase diante de Nomes de Lugar

Alguns nomes de lugar não admitem a anteposição do artigo “a”. Outros, entretanto, admitem o artigo, de modo que
diante deles haverá crase, desde que o termo regente exija a preposição “a”. Para saber se um nome de lugar admite ou não
a anteposição do artigo feminino “a”, deve-se substituir o termo regente por um verbo que peça a preposição “de” ou “em”.
A ocorrência da contração “da” ou “na” prova que esse nome de lugar aceita o artigo e, por isso, haverá crase. Por exemplo:

Vou à França. (Vim da [de+a] França. Estou na [em+a] França.)


Cheguei à Grécia. (Vim da Grécia. Estou na Grécia.)
Retornarei à Itália. (Vim da Itália. Estou na Itália)
Vou a Porto Alegre. (Vim de Porto Alegre. Estou em Porto Alegre.)

*- Dica da Zê!: use a regrinha “Vou A volto DA, crase HÁ; vou A volto DE, crase PRA QUÊ?”
Ex: Vou a Campinas. = Volto de Campinas.
Vou à praia. = Volto da praia.

- ATENÇÃO: quando o nome de lugar estiver especificado, ocorrerá crase. Veja:


Retornarei à São Paulo dos bandeirantes. = mesmo que, pela regrinha acima, seja a do “VOLTO DE”
Irei à Salvador de Jorge Amado.

Crase diante dos Pronomes Demonstrativos Aquele (s), Aquela (s), Aquilo

Haverá crase diante desses pronomes sempre que o termo regente exigir a preposição “a”. Por exemplo:

Refiro-me a + aquele atentado.


Preposição Pronome
Refiro-me àquele atentado.

45
LÍNGUA PORTUGUESA

O termo regente do exemplo acima é o verbo transi- Se a palavra distância não estiver especificada, a crase
tivo indireto referir (referir-se a algo ou alguém) e exige não pode ocorrer. Por exemplo:
preposição, portanto, ocorre a crase. Observe este outro Os militares ficaram a distância.
exemplo: Gostava de fotografar a distância.
Aluguei aquela casa. Ensinou a distância.
O verbo “alugar” é transitivo direto (alugar algo) e não Dizem que aquele médico cura a distância.
exige preposição. Logo, a crase não ocorre nesse caso. Veja Reconheci o menino a distância.
outros exemplos: Observação: por motivo de clareza, para evitar ambigui-
Dediquei àquela senhora todo o meu trabalho. dade, pode-se usar a crase. Veja:
Quero agradecer àqueles que me socorreram. Gostava de fotografar à distância.
Refiro-me àquilo que aconteceu com seu pai. Ensinou à distância.
Não obedecerei àquele sujeito. Dizem que aquele médico cura à distância.
Assisti àquele filme três vezes.
Espero aquele rapaz. Casos em que a ocorrência da crase é FACULTATIVA
Fiz aquilo que você disse.
Comprei aquela caneta. - diante de nomes próprios femininos:
Observação: é facultativo o uso da crase diante de no-
Crase com os Pronomes Relativos A Qual, As Quais mes próprios femininos porque é facultativo o uso do ar-
tigo. Observe:
A ocorrência da crase com os pronomes relativos a qual Paula é muito bonita. Laura é minha amiga.
e as quais depende do verbo. Se o verbo que rege esses A Paula é muito bonita. A Laura é minha amiga.
pronomes exigir a preposição “a”, haverá crase. É possí-
vel detectar a ocorrência da crase nesses casos utilizando a Como podemos constatar, é facultativo o uso do artigo
substituição do termo regido feminino por um termo regi- feminino diante de nomes próprios femininos, então pode-
do masculino. Por exemplo: mos escrever as frases abaixo das seguintes formas:
Entreguei o cartão a Paula. Entreguei o cartão a Roberto.
A igreja à qual me refiro fica no centro da cidade. Entreguei o cartão à Paula. Entreguei o cartão ao Ro-
O monumento ao qual me refiro fica no centro da cidade. berto.

Caso surja a forma ao com a troca do termo, ocorrerá a - diante de pronome possessivo feminino:
crase. Veja outros exemplos: Observação: é facultativo o uso da crase diante de pro-
São normas às quais todos os alunos devem obedecer. nomes possessivos femininos porque é facultativo o uso do
Esta foi a conclusão à qual ele chegou. artigo. Observe:
Várias alunas às quais ele fez perguntas não souberam Minha avó tem setenta anos. Minha irmã está esperando
responder nenhuma das questões. por você.
A sessão à qual assisti estava vazia. A minha avó tem setenta anos. A minha irmã está espe-
rando por você.
Crase com o Pronome Demonstrativo “a” Sendo facultativo o uso do artigo feminino diante de
pronomes possessivos femininos, então podemos escrever
A ocorrência da crase com o pronome demonstrativo as frases abaixo das seguintes formas:
“a” também pode ser detectada através da substituição do Cedi o lugar a minha avó. Cedi o lugar a meu avô.
termo regente feminino por um termo regido masculino. Cedi o lugar à minha avó. Cedi o lugar ao meu avô.
Veja:
Minha revolta é ligada à do meu país. - depois da preposição até:
Meu luto é ligado ao do meu país. Fui até a praia. ou Fui até à praia.
As orações são semelhantes às de antes. Acompanhe-o até a porta. ou Acompanhe-o até à porta.
Os exemplos são semelhantes aos de antes. A palestra vai até as cinco horas da tarde. ou A palestra
Suas perguntas são superiores às dele. vai até às cinco horas da tarde.
Seus argumentos são superiores aos dele.
Sua blusa é idêntica à de minha colega.
Seu casaco é idêntico ao de minha colega.

A Palavra Distância

Se a palavra distância estiver especificada, determinada,


a crase deve ocorrer. Por exemplo: Sua casa fica à distância
de 100km daqui. (A palavra está determinada)
Todos devem ficar à distância de 50 metros do palco. (A
palavra está especificada.)

46
LÍNGUA PORTUGUESA

Questões sobre Crase A) leitura apressada e sem profundidade.


B) cada um de nós neste formigueiro.
01.( Escrevente TJ SP – Vunesp/2012) No Brasil, as dis- C) exemplo de obras publicadas recentemente.
cussões sobre drogas parecem limitar-se ______aspectos ju- D) uma comunicação festiva e virtual.
rídicos ou policiais. É como se suas únicas consequências E) respeito de autores reconhecidos pelo público.
estivessem em legalismos, tecnicalidades e estatísticas cri-
minais. Raro ler ____respeito envolvendo questões de saúde 05. (Agente de Escolta e Vigilância Penitenciária – VU-
pública como programas de esclarecimento e prevenção, de NESP – 2013).
tratamento para dependentes e de reintegração desses____ O Instituto Nacional de Administração Prisional (INAP)
vida. Quantos de nós sabemos o nome de um médico ou também desenvolve atividades lúdicas de apoio______ res-
clínica ____quem tentar encaminhar um drogado da nossa socialização do indivíduo preso, com o objetivo de prepará-
própria família? -lo para o retorno______ sociedade. Dessa forma, quando em
(Ruy Castro, Da nossa própria família. Folha de S.Paulo, liberdade, ele estará capacitado______ ter uma profissão e
17.09.2012. Adaptado) uma vida digna.
(Disponível em: www.metropolitana.com.br/blog/
As lacunas do texto devem ser preenchidas, correta e qual_e_a_importancia_da_ressocializacao_de_presos. Aces-
respectivamente, com: so em: 18.08.2012. Adaptado)
(A) aos … à … a … a
(B) aos … a … à … a Assinale a alternativa que preenche, correta e respecti-
(C) a … a … à … à vamente, as lacunas do texto, de acordo com a norma-pa-
(D) à … à … à … à drão da língua portuguesa.
(E) a … a … a … a A) à … à … à
B) a … a … à
02. (Agente de Apoio Administrativo – FCC – 2013).Leia C) a … à … à
o texto a seguir.
D) à … à ... a
Foi por esse tempo que Rita, desconfiada e medrosa, cor-
E) a … à … a
reu ______ cartomante para consultá-la sobre a verdadeira
causa do procedimento de Camilo. Vimos que ______ carto-
06. (TRIBUNAL DE JUSTIÇA DO ESTADO DE SÃO PAU-
mante restituiu--lhe ______ confiança, e que o rapaz repreen-
LO - ESCREVENTE TÉCNICO JUDICIÁRIO – VUNESP/2013)
deu-a por ter feito o que fez.
Assinale a alternativa que completa as lacunas do trecho a
(Machado de Assis. A cartomante. In: Várias histórias.
seguir, empregando o sinal indicativo de crase de acordo
Rio de Janeiro: Globo, 1997, p. 6)
Preenchem corretamente as lacunas da frase acima, na com a norma-padrão.
ordem dada:
A) à – a – a Não nos sujeitamos ____ corrupção; tampouco cederemos
B) a – a – à espaço ____ nenhuma ação que se proponha ____ prejudicar
C) à – a – à nossas instituições.
D) à – à – a (A) à … à … à
E) a – à – à (B) a … à … à
(C) à … a … a
03 (POLÍCIA CIVIL/SP – AGENTE POLICIAL - VU- (D) à … à … a
NESP/2013) De acordo com a norma-padrão da língua (E) a … a … à
portuguesa, o acento indicativo de crase está corretamente
empregado em: 07. (Agente de Escolta e Vigilância Penitenciária – VU-
(A) A população, de um modo geral, está à espera de NESP – 2013-adap) O acento indicativo de crase está cor-
que, com o novo texto, a lei seca possa coibir os acidentes. retamente empregado em:
(B) A nova lei chega para obrigar os motoristas à repen- A) Tendências agressivas começam à ser relacionadas
sarem a sua postura. com as dificuldades para lidar com as frustrações de seus
(C) A partir de agora os motoristas estarão sujeitos à desejos.
punições muito mais severas. B) A agressividade impulsiva deve-se à perturbações
(D) À ninguém é dado o direito de colocar em risco a nos mecanismos biológicos de controle emocional.
vida dos demais motoristas e de pedestres. C) A violência urbana é comparada à uma enfermidade.
(E) Cabe à todos na sociedade zelar pelo cumprimento D) Condições de risco aliadas à exemplo de impunidade
da nova lei para que ela possa funcionar. alimentam a violência crescente nas cidades.
E) Um ambiente desfavorável à formação da personali-
04. (Agente Técnico – FCC – 2013-adap.) Claro que não dade atinge os mais vulneráveis.
me estou referindo a essa vulgar comunicação festiva e
efervescente.
O vocábulo a deverá receber o sinal indicativo de crase
se o segmento grifado for substituído por:

47
LÍNGUA PORTUGUESA

08. (Agente de Vigilância e Recepção – VUNESP – 2013). 3-)


O sinal indicativo de crase está correto em: (A) A população, de um modo geral, está à espera (dá
A) Este cientista tem se dedicado à uma pesquisa na para substituir por “esperando”) de que
área de biotecnologia. (B) A nova lei chega para obrigar os motoristas à repen-
B) Os pais não podem ser omissos e devem se dedicar sarem (antes de verbo)
à educação dos filhos. (C) A partir de agora os motoristas estarão sujeitos à
C) Nossa síndica dedica-se integralmente à conservar as punições (generalizando, palavra no plural)
instalações do prédio. (D) À ninguém (pronome indefinido)
D) O bombeiro deve dedicar sua atenção à qualquer (E) Cabe à todos (pronome indefinido)
detalhe que envolva a segurança das pessoas. 4-) Claro que não me estou referindo à leitura apressa-
E) É função da política é dedicar-se à todo problema da e sem profundidade.
que comprometa o bem-estar do cidadão. a cada um de nós neste formigueiro. (antes de prono-
me indefinido)
09. (TRF - 5ª REGIÃO - TÉCNICO JUDICIÁRIO - FCC/2012) a exemplo de obras publicadas recentemente. (palavra
O detetive Gervase Fen, que apareceu em 1944, é um ho- masculina)
mem de face corada, muito afeito ...... frases inteligentes e a uma comunicação festiva e virtual. (artigo indefinido)
citações dos clássicos; sua esposa, Dolly, uma dama meiga e a respeito de autores reconhecidos pelo público. (pa-
sossegada, fica sentada tricotando tranquilamente, impassí- lavra masculina)
vel ...... propensão de seu marido ...... investigar assassinatos.
(Adaptado de P.D.James, op.cit.) 5-) O Instituto Nacional de Administração Prisional
Preenchem corretamente as lacunas da frase acima, na (INAP) também desenvolve atividades lúdicas de apoio___à__
ordem dada: ressocialização do indivíduo preso, com o objetivo de prepa-
(A) à - à - a rá--lo para o retorno___à__ sociedade. Dessa forma, quando
(B) a - à - a em liberdade, ele estará capacitado__a___ ter uma profissão
(C) à - a - à
e uma vida digna.
(D) a - à - à
- Apoio a ? Regência nominal pede preposição;
(E) à - a – a
- retorno a? regência nominal pede preposição;
- antes de verbo no infinitivo não há crase.
10. (POLÍCIA MILITAR DO ESTADO DO ACRE – ALUNO
SOLDADO COMBATENTE – FUNCAB/2012) Em qual das op-
6-) Vamos por partes!
ções abaixo o acento indicativo de crase foi corretamente
- Quem se sujeita, sujeita-se A algo ou A alguém, por-
indicado?
tanto: pede preposição;
A) O dia fora quente, mas à noite estava fria e escura.
B) Ninguém se referira à essa ideia antes. - quem cede, cede algo A alguém, então teremos obje-
C) Esta era à medida certa do quarto. to direto e indireto;
D) Ela fechou a porta e saiu às pressas. - quem se propõe, propõe-se A alguma coisa.
E) Os rapazes sempre gostaram de andar à cavalo. Vejamos:
Não nos sujeitamos À corrupção; tampouco cederemos
GABARITO espaço A nenhuma ação que se proponha A prejudicar
nossas instituições.
01. B 02. A 03. A 04. A 05. D * Sujeitar A + A corrupção;
06.C 07. E 08. B 09.B 10. D * ceder espaço (objeto direto) A nenhuma ação (objeto
indireto. Não há acento indicativo de crase, pois “nenhu-
RESOLUÇÃO ma” é pronome indefinido);
* que se proponha A prejudicar (objeto indireto, no
1-) limitar-se _aos _aspectos jurídicos ou policiais. caso, oração subordinada com função de objeto indireto.
Raro ler __a__respeito (antes de palavra masculina Não há acento indicativo de crase porque temos um verbo
não há crase) no infinitivo – “prejudicar”).
de reintegração desses_à_ vida. (reintegrar a + a
vida = à) 7-)
o nome de um médico ou clínica __a_quem tentar en- A) Tendências agressivas começam à ser relacionadas
caminhar um drogado da nossa própria família? (antes de com as dificuldades para lidar com as frustrações de seus
pronome indefinido/relativo) desejos. (antes de verbo no infinitivo não há crase)
B) A agressividade impulsiva deve-se à perturbações
2-) correu _à (= para a ) cartomante para consultá-la so- nos mecanismos biológicos de controle emocional. (se
bre a verdadeira causa do procedimento de Camilo. Vimos o “a” está no singular e antecede palavra no plural, não há
que _a__cartomante (objeto direto)restituiu-lhe ___a___ crase)
confiança (objeto direto), e que o rapaz repreendeu-a por C) A violência urbana é comparada à uma enfermidade.
ter feito o que fez. (artigo indefinido)

48
LÍNGUA PORTUGUESA

D) Condições de risco aliadas à exemplo de impunida- condição e precisam estar adequados à categoria hierár-
de alimentam a violência crescente nas cidades. (palavra quica da pessoa a quem nos dirigimos. E mais, exige-se, em
masculina) discurso falado ou escrito, uma homogeneidade na forma
E) Um ambiente desfavorável à formação da personali- de tratamento, não só nos pronomes como também nos
dade atinge os mais vulneráveis. = correta (regência nomi- verbos.
nal: desfavorável a?) No entanto, as formas de tratamento não são do co-
nhecimento de todos. Para tanto, a partir do Manual da
8-) Presidência da República, apresentaremos as discrimina-
A) Este cientista tem se dedicado à uma pesquisa na ções de usos dos pronomes de tratamento:
área de biotecnologia. (artigo indefinido) São de uso consagrado: Vossa Excelência, para as se-
B) Os pais não podem ser omissos e devem se dedicar à guintes autoridades:
educação dos filhos. = correta (regência verbal: dedicar a ) a) do Poder Executivo
C) Nossa síndica dedica-se integralmente à conservar as Presidente da República;
instalações do prédio. (verbo no infinitivo) Vice-Presidente da República;
D) O bombeiro deve dedicar sua atenção à qualquer Ministro de Estado;
detalhe que envolva a segurança das pessoas. (pronome Secretário-Geral da Presidência da República;
indefinido) Consultor-Geral da República;
E) É função da política é dedicar-se à todo problema Chefe do Estado-Maior das Forças Armadas;
que comprometa o bem-estar do cidadão. (pronome in- Chefe do Gabinete Militar da Presidência da República;
definido) Chefe do Gabinete Pessoal do Presidente da República;
Secretários da Presidência da República;
9-) Afeito a frases (generalizando, já que o “a” está no Procurador – Geral da República;
singular e “frases”, no plural) Governadores e Vice-Governadores de Estado e do Dis-
Impassível à propensão (regência nominal: pede pre- trito Federal;
posição) Chefes de Estado – Maior das Três Armas;
A investigar (antes de verbo no infinitivo não há acen- Oficiais Generais das Forças Armadas;
to indicativo de crase) Embaixadores;
Secretário Executivo e Secretário Nacional de Ministérios;
Sequência: a / à / a.
Secretários de Estado dos Governos Estaduais;
Prefeitos Municipais.
10-)
A) O dia fora quente, mas à noite = mas a noite (artigo e
b) do Poder Legislativo:
substantivo. Diferente de: Estudo à noite = período do dia)
Presidente, Vice–Presidente e Membros da Câmara dos
B) Ninguém se referira à essa ideia antes.= a essa (antes
Deputados e do Senado Federal;
de pronome demonstrativo)
Presidente e Membros do Tribunal de Contas da União;
C) Esta era à medida certa do quarto. = a medida (artigo Presidente e Membros dos Tribunais de Contas Estaduais;
e substantivo, no caso. Diferente da conjunção proporcio- Presidente e Membros das Assembleias Legislativas Es-
nal: À medida que lia, mais aprendia) taduais;
D) Ela fechou a porta e saiu às pressas. = correta (advér- Presidente das Câmaras Municipais.
bio de modo = apressadamente)
E) Os rapazes sempre gostaram de andar à cavalo. = c) do Poder Judiciário:
palavra masculina Presidente e Membros do Supremo Tribunal Federal;
Presidente e Membros do Superior Tribunal de Justiça;
Presidente e Membros do Superior Tribunal Militar;
6. FORMAS DE TRATAMENTO; Presidente e Membros do Tribunal Superior Eleitoral;
Presidente e Membros do Tribunal Superior do Trabalho;
Presidente e Membros dos Tribunais de Justiça;
Pronomes de tratamento na redação oficial Presidente e Membros dos Tribunais Regionais Federais;
Presidente e Membros dos Tribunais Regionais Eleitorais;
A redação Oficial é a maneira para o poder público re- Presidente e Membros dos Tribunais Regionais do Tra-
digir atos normativos. Para redigi-los, muitas regras fazem- balho;
se necessárias. Entre elas, escrever de forma clara, concisa, Juízes e Desembargadores;
sem muito comprometimento, bem como um uso adequa- Auditores da Justiça Militar.”
do das formas de tratamento. Tais regras, acompanhadas
de uma boa redação, com um bom uso da linguagem, as- O vocativo a ser empregado em comunicações dirigi-
seguram que os atos normativos sejam bem executados. das aos Chefes do Poder é Excelentíssimo Senhor, seguido
No Poder Público, a todo momento nós nos depara- do cargo respectivo: Excelentíssimo Senhor Presidente da
mos com situações em que precisamos escrever – ou fa- República; Excelentíssimo Senhor Presidente do Congresso
lar – com pessoas com as quais não temos familiaridade. Nacional; Excelentíssimo Senhor Presidente do Supremo Tri-
Nesses casos, os pronomes de tratamento assumem uma bunal Federal.

49
LÍNGUA PORTUGUESA

E mais: As demais autoridades serão tratadas com o vo- A Impessoalidade


cativo Senhor, seguido do cargo respectivo: Senhor Sena-
dor, Senhor Juiz, Senhor Ministro, Senhor Governador. A finalidade da língua é comunicar, quer pela fala, quer pela
O Manual ainda preceitua que a forma de tratamento escrita. Para que haja comunicação, são necessários: a) alguém
“Digníssimo” fica abolida para as autoridades descritas aci- que comunique, b) algo a ser comunicado, e c) alguém que
ma, afinal, a dignidade é condição primordial para que tais receba essa comunicação. No caso da redação oficial, quem
cargos públicos sejam ocupados. comunica é sempre o Serviço Público (este ou aquele Minis-
Fica ainda dito que doutor não é forma de tratamento, tério, Secretaria, Departamento, Divisão, Serviço, Seção); o que
mas titulação acadêmica de quem defende tese de douto- se comunica é sempre algum assunto relativo às atribuições
rado. Portanto, é aconselhável que não se use discrimina- do órgão que comunica; o destinatário dessa comunicação ou
damente tal termo. é o público, o conjunto dos cidadãos, ou outro órgão público,
AS COMUNICAÇÕES OFICIAIS do Executivo ou dos outros Poderes da União.
Percebe-se, assim, que o tratamento impessoal que deve
ser dado aos assuntos que constam das comunicações oficiais
1. ASPECTOS GERAIS DA REDAÇÃO OFICIAL
decorre:
a) da ausência de impressões individuais de quem comu-
O que é Redação Oficial
nica: embora se trate, por exemplo, de um expediente assina-
do por Chefe de determinada Seção, é sempre em nome do
Em uma frase, pode-se dizer que redação oficial é a Serviço Público que é feita a comunicação. Obtém-se, assim,
maneira pela qual o Poder Público redige atos norma- uma desejável padronização, que permite que comunicações
tivos e comunicações. Interessa-nos tratá-la do ponto de elaboradas em diferentes setores da Administração guardem
vista do Poder Executivo. entre si certa uniformidade;
A redação oficial deve caracterizar-se pela impessoa- b) da impessoalidade de quem recebe a comunicação,
lidade, uso do padrão culto de linguagem, clareza, con- com duas possibilidades: ela pode ser dirigida a um cidadão,
cisão, formalidade e uniformidade. Fundamentalmente sempre concebido como público, ou a outro órgão público.
esses atributos decorrem da Constituição, que dispõe, no Nos dois casos, temos um destinatário concebido de forma
artigo 37: “A administração pública direta, indireta ou fun- homogênea e impessoal;
dacional, de qualquer dos Poderes da União, dos Estados, c) do caráter impessoal do próprio assunto tratado: se o
do Distrito Federal e dos Municípios obedecerá aos princí- universo temático das comunicações oficiais restringe-se a
pios de legalidade, impessoalidade, moralidade, publicidade questões que dizem respeito ao interesse público, é natural
e eficiência (...)”. Sendo a publicidade e a impessoalidade que não caiba qualquer tom particular ou pessoal.
princípios fundamentais de toda administração pública, Desta forma, não há lugar na redação oficial para impres-
claro que devem igualmente nortear a elaboração dos sões pessoais, como as que, por exemplo, constam de uma
atos e comunicações oficiais. carta a um amigo, ou de um artigo assinado de jornal, ou mes-
Não se concebe que um ato normativo de qualquer na- mo de um texto literário. A redação oficial deve ser isenta da
tureza seja redigido de forma obscura, que dificulte ou im- interferência da individualidade que a elabora.
possibilite sua compreensão. A transparência do sentido A concisão, a clareza, a objetividade e a formalidade de que
dos atos normativos, bem como sua inteligibilidade, são nos valemos para elaborar os expedientes oficiais contribuem,
requisitos do próprio Estado de Direito: é inaceitável que ainda, para que seja alcançada a necessária impessoalidade.
um texto legal não seja entendido pelos cidadãos. A publi-
cidade implica, pois, necessariamente, clareza e concisão. A Linguagem dos Atos e Comunicações Oficiais
Fica claro também que as comunicações oficiais são
A necessidade de empregar determinado nível de lingua-
necessariamente uniformes, pois há sempre um único co-
gem nos atos e expedientes oficiais decorre, de um lado, do
municador (o Serviço Público) e o receptor dessas comu-
próprio caráter público desses atos e comunicações; de outro,
nicações ou é o próprio Serviço Público (no caso de expe-
de sua finalidade. Os atos oficiais, aqui entendidos como atos
dientes dirigidos por um órgão a outro) – ou o conjunto de caráter normativo, ou estabelecem regras para a conduta
dos cidadãos ou instituições tratados de forma homogê- dos cidadãos, ou regulam o funcionamento dos órgãos pú-
nea (o público). blicos, o que só é alcançado se em sua elaboração for empre-
A redação oficial não é necessariamente árida e infensa gada a linguagem adequada. O mesmo se dá com os expe-
à evolução da língua. É que sua finalidade básica – co- dientes oficiais, cuja finalidade precípua é a de informar com
municar com impessoalidade e máxima clareza – impõe clareza e objetividade.
certos parâmetros ao uso que se faz da língua, de maneira As comunicações que partem dos órgãos públicos federais
diversa daquele da literatura, do texto jornalístico, da cor- devem ser compreendidas por todo e qualquer cidadão bra-
respondência particular, etc. sileiro. Para atingir esse objetivo, há que evitar o uso de uma
Apresentadas essas características fundamentais da re- linguagem restrita a determinados grupos. Não há dúvida de
dação oficial, passemos à análise pormenorizada de cada que um texto marcado por expressões de circulação restrita,
uma delas. como a gíria, os regionalismos vocabulares ou o jargão téc-
nico, tem sua compreensão dificultada.

50
LÍNGUA PORTUGUESA

Ressalte-se que há necessariamente uma distância en- A formalidade de tratamento vincula-se, também, à ne-
tre a língua falada e a escrita. Aquela é extremamente di- cessária uniformidade das comunicações. Ora, se a admi-
nâmica, reflete de forma imediata qualquer alteração de nistração federal é una, é natural que as comunicações que
costumes, e pode eventualmente contar com outros ele- expede sigam um mesmo padrão. O estabelecimento des-
mentos que auxiliem a sua compreensão, como os gestos, se padrão exige que se atente para todas as características
a entoação, etc., para mencionar apenas alguns dos fatores da redação oficial e que se cuide, ainda, da apresentação
responsáveis por essa distância. Já a língua escrita incorpo- dos textos.
ra mais lentamente as transformações, tem maior vocação A clareza datilográfica, o uso de papéis uniformes para
para a permanência e vale-se apenas de si mesma para co- o texto definitivo e a correta diagramação do texto são in-
municar. dispensáveis para a padronização.
Os textos oficiais, devido ao seu caráter impessoal e sua
finalidade de informar com o máximo de clareza e concisão, Concisão e Clareza
requerem o uso do padrão culto da língua. Há consenso de
A concisão é antes uma qualidade do que uma carac-
que o padrão culto é aquele em que a) se observam as re-
terística do texto oficial. Conciso é o texto que consegue
gras da gramática formal e b) se emprega um vocabulário
transmitir um máximo de informações com um mínimo de
comum ao conjunto dos usuários do idioma. É importante
palavras. Para que se redija com essa qualidade, é funda-
ressaltar que a obrigatoriedade do uso do padrão culto na mental que se tenha, além de conhecimento do assunto
redação oficial decorre do fato de que ele está acima das sobre o qual se escreve, o necessário tempo para revisar o
diferenças lexicais, morfológicas ou sintáticas regionais, texto depois de pronto. É nessa releitura que muitas vezes
dos modismos vocabulares, das idiossincrasias linguísticas, se percebem eventuais redundâncias ou repetições desne-
permitindo, por essa razão, que se atinja a pretendida com- cessárias de ideias.
preensão por todos os cidadãos. O esforço de sermos concisos atende, basicamente, ao
Lembre-se de que o padrão culto nada tem contra a princípio de economia linguística, à mencionada fórmula
simplicidade de expressão, desde que não seja confundida de empregar o mínimo de palavras para informar o má-
com pobreza de expressão. De nenhuma forma o uso do ximo. Não se deve, de forma alguma, entendê-la como
padrão culto implica emprego de linguagem rebuscada, economia de pensamento, isto é, não se devem eliminar
nem dos contorcionismos sintáticos e figuras de linguagem passagens substanciais do texto no afã de reduzi-lo em ta-
próprios da língua literária. manho. Trata-se exclusivamente de cortar palavras inúteis,
Pode-se concluir, então, que não existe propriamente redundâncias, passagens que nada acrescentem ao que já
um “padrão oficial de linguagem”; o que há é o uso do foi dito.
padrão culto nos atos e comunicações oficiais. É claro que A clareza deve ser a qualidade básica de todo texto ofi-
haverá preferência pelo uso de determinadas expressões, cial. Pode-se definir como claro aquele texto que possibili-
ou será obedecida certa tradição no emprego das formas ta imediata compreensão pelo leitor. No entanto a clareza
sintáticas, mas isso não implica, necessariamente, que se não é algo que se atinja por si só: ela depende estritamente
consagre a utilização de uma forma de linguagem buro- das demais características da redação oficial. Para ela con-
crática. O jargão burocrático, como todo jargão, deve ser correm:
evitado, pois terá sempre sua compreensão limitada. - a impessoalidade, que evita a duplicidade de interpre-
A linguagem técnica deve ser empregada apenas em tações que poderia decorrer de um tratamento personalis-
situações que a exijam, sendo de evitar o seu uso indis- ta dado ao texto;
- o uso do padrão culto de linguagem, em princípio, de
criminado. Certos rebuscamentos acadêmicos, e mesmo o
entendimento geral e por definição avesso a vocábulos de
vocabulário próprio a determinada área, são de difícil en-
circulação restrita, como a gíria e o jargão;
tendimento por quem não esteja com eles familiarizado.
- a formalidade e a padronização, que possibilitam a
Deve-se ter o cuidado, portanto, de explicitá-los em comu-
imprescindível uniformidade dos textos;
nicações encaminhadas a outros órgãos da administração - a concisão, que faz desaparecer do texto os excessos
e em expedientes dirigidos aos cidadãos. linguísticos que nada lhe acrescentam.
É pela correta observação dessas características que
Formalidade e Padronização se redige com clareza. Contribuirá, ainda, a indispensável
releitura de todo texto redigido. A ocorrência, em textos
As comunicações oficiais devem ser sempre formais, oficiais, de trechos obscuros e de erros gramaticais provém
isto é, obedecem a certas regras de forma: além das já principalmente da falta da releitura que torna possível sua
mencionadas exigências de impessoalidade e uso do pa- correção.
drão culto de linguagem, é imperativo, ainda, certa forma- A revisão atenta exige, necessariamente, tempo. A pres-
lidade de tratamento. Não se trata somente da eterna dúvi- sa com que são elaboradas certas comunicações quase
da quanto ao correto emprego deste ou daquele pronome sempre compromete sua clareza. Não se deve proceder à
de tratamento para uma autoridade de certo nível; mais do redação de um texto que não seja seguida por sua revi-
que isso, a formalidade diz respeito à polidez, à civilidade são. “Não há assuntos urgentes, há assuntos atrasados”, diz
no próprio enfoque dado ao assunto do qual cuida a co- a máxima. Evite-se, pois, o atraso, com sua indesejável re-
municação. percussão no redigir.

51
LÍNGUA PORTUGUESA

Pronomes de Tratamento Identificação do Signatário

Concordância com os Pronomes de Tratamento Excluídas as comunicações assinadas pelo Presidente


da República, todas as demais comunicações oficiais de-
Os pronomes de tratamento (ou de segunda pessoa vem trazer o nome e o cargo da autoridade que as expede,
indireta) apresentam certas peculiaridades quanto à con- abaixo do local de sua assinatura. A forma da identificação
cordância verbal, nominal e pronominal. Embora se refiram deve ser a seguinte:
à segunda pessoa gramatical (à pessoa com quem se fala, (espaço para assinatura)
ou a quem se dirige a comunicação), levam a concordância Nome
para a terceira pessoa. É que o verbo concorda com o subs- Chefe da Secretaria-Geral da Presidência da República
tantivo que integra a locução como seu núcleo sintático:
“Vossa Senhoria nomeará o substituto”; “Vossa Excelência (espaço para assinatura)
conhece o assunto”. Nome
Da mesma forma, os pronomes possessivos referidos a Ministro de Estado da Justiça
pronomes de tratamento são sempre os da terceira pessoa: Para evitar equívocos, recomenda-se não deixar a as-
“Vossa Senhoria nomeará seu substituto” (e não “Vossa ... sinatura em página isolada do expediente. Transfira para
vosso...”). essa página ao menos a última frase anterior ao fecho.
Já quanto aos adjetivos referidos a esses pronomes, o
gênero gramatical deve coincidir com o sexo da pessoa a Forma de diagramação
que se refere, e não com o substantivo que compõe a lo-
cução. Assim, se nosso interlocutor for homem, o correto Os documentos do Padrão Ofício devem obedecer à se-
é “Vossa Excelência está atarefado”, “Vossa Senhoria deve guinte forma de apresentação:
estar satisfeito”; se for mulher, “Vossa Excelência está atare- - deve ser utilizada fonte do tipo Times New Roman de
fada”, “Vossa Senhoria deve estar satisfeita”. corpo 12 no texto em geral, 11 nas citações, e 10 nas notas
No envelope, o endereçamento das comunicações di-
de rodapé;
rigidas às autoridades tratadas por Vossa Excelência, terá a
- para símbolos não existentes na fonte Times New Ro-
seguinte forma:
man poder-se-á utilizar as fontes Symbol e Wingdings;
- é obrigatório constar a partir da segunda página o
A Sua Excelência o Senhor
número da página;
Fulano de Tal
- os ofícios, memorandos e anexos destes poderão ser
Ministro de Estado da Justiça
impressos em ambas as faces do papel. Neste caso, as mar-
70.064-900 – Brasília. DF
A Sua Excelência o Senhor gens esquerda e direta terão as distâncias invertidas nas
Senador Fulano de Tal páginas pares (“margem espelho”);
Senado Federal - o campo destinado à margem lateral esquerda terá,
70.165-900 – Brasília. DF no mínimo, 3,0 cm de largura;
- o início de cada parágrafo do texto deve ter 2,5 cm de
Senhor Ministro, distância da margem esquerda;
- o campo destinado à margem lateral direita terá 1,5
Submeto a Vossa Excelência projeto (...) cm;
- deve ser utilizado espaçamento simples entre as li-
Fechos para Comunicações nhas e de 6 pontos após cada parágrafo, ou, se o editor
de texto utilizado não comportar tal recurso, de uma linha
O fecho das comunicações oficiais possui, além da fi- em branco;
nalidade de arrematar o texto, a de saudar o destinatário. - não deve haver abuso no uso de negrito, itálico, subli-
Os modelos para fecho que vinham sendo utilizados fo- nhado, letras maiúsculas, sombreado, sombra, relevo, bor-
ram regulados pela Portaria no 1 do Ministério da Justiça, das ou qualquer outra forma de formatação que afete a
de 1937, que estabelecia quinze padrões. Com o fito de elegância e a sobriedade do documento;
simplificá-los e uniformizá-los, este Manual estabelece o - a impressão dos textos deve ser feita na cor preta em
emprego de somente dois fechos diferentes para todas as papel branco. A impressão colorida deve ser usada apenas
modalidades de comunicação oficial: para gráficos e ilustrações;
a) para autoridades superiores, inclusive o Presiden- - todos os tipos de documentos do Padrão Ofício de-
te da República: Respeitosamente, vem ser impressos em papel de tamanho A-4, ou seja, 29,7
b) para autoridades de mesma hierarquia ou de hie- x 21,0 cm;
rarquia inferior: Atenciosamente, - deve ser utilizado, preferencialmente, o formato de
arquivo Rich Text nos documentos de texto;
Ficam excluídas dessa fórmula as comunicações dirigi- - dentro do possível, todos os documentos elaborados
das a autoridades estrangeiras, que atendem a rito e tra- devem ter o arquivo de texto preservado para consulta
dição próprios, devidamente disciplinados no Manual de posterior ou aproveitamento de trechos para casos aná-
Redação do Ministério das Relações Exteriores. logos;

52
LÍNGUA PORTUGUESA

- para facilitar a localização, os nomes dos arquivos de- no próprio documento e, no caso de falta de espaço, em
vem ser formados da seguinte maneira: folha de continuação. Esse procedimento permite formar
tipo do documento + número do documento + pala- uma espécie de processo simplificado, assegurando maior
vras-chaves do conteúdo transparência à tomada de decisões, e permitindo que se
Ex.: “Of. 123 - relatório produtividade ano 2002” historie o andamento da matéria tratada no memorando.

Aviso e Ofício Forma e Estrutura


Quanto a sua forma, o memorando segue o modelo do
Definição e Finalidade padrão ofício, com a diferença de que o seu destinatário
Aviso e ofício são modalidades de comunicação oficial deve ser mencionado pelo cargo que ocupa. Ex:
praticamente idênticas. A única diferença entre eles é que Ao Sr. Chefe do Departamento de Administração
o aviso é expedido exclusivamente por Ministros de Estado, Ao Sr. Subchefe para Assuntos Jurídicos
para autoridades de mesma hierarquia, ao passo que o ofí-
cio é expedido para e pelas demais autoridades. Ambos têm Exposição de Motivos
como finalidade o tratamento de assuntos oficiais pelos ór-
gãos da Administração Pública entre si e, no caso do ofício, Definição e Finalidade
também com particulares. Exposição de motivos é o expediente dirigido ao Presi-
dente da República ou ao Vice-Presidente para: a) informá-
Forma e Estrutura -lo de determinado assunto; b) propor alguma medida; ou
Quanto a sua forma, aviso e ofício seguem o modelo c) submeter a sua consideração projeto de ato normativo.
do padrão ofício, com acréscimo do vocativo, que invoca o Em regra, a exposição de motivos é dirigida ao Presi-
destinatário, seguido de vírgula. dente da República por um Ministro de Estado.
Exemplos: Nos casos em que o assunto tratado envolva mais de
Excelentíssimo Senhor Presidente da República um Ministério, a exposição de motivos deverá ser assinada
Senhora Ministra por todos os Ministros envolvidos, sendo, por essa razão,
Senhor Chefe de Gabinete chamada de interministerial.
Forma e Estrutura
Devem constar do cabeçalho ou do rodapé do ofício as Formalmente, a exposição de motivos tem a apresenta-
seguintes informações do remetente: ção do padrão ofício. A exposição de motivos, de acordo
– nome do órgão ou setor; com sua finalidade, apresenta duas formas básicas de es-
– endereço postal; trutura: uma para aquela que tenha caráter exclusivamente
– telefone e e-mail. informativo e outra para a que proponha alguma medida
ou submeta projeto de ato normativo.
OBS: Estas informações estão ausentes no memorando, No primeiro caso, o da exposição de motivos que sim-
pois trata-se de comunicação interna, destinatário e reme- plesmente leva algum assunto ao conhecimento do Presi-
tente possuem o mesmo endereço. No caso se o Aviso é de dente da República, sua estrutura segue o modelo antes
um Ministério para outro Ministério, também não precisa referido para o padrão ofício.
especificar o endereço. O Ofício é enviado para outras ins-
tituições, logo, são necessárias as informações do remeten- Mensagem
te e o endereço do destinatário para que o ofício possa ser
entregue e o remetente possa receber resposta. Definição e Finalidade
É o instrumento de comunicação oficial entre os Chefes
Memorando dos Poderes Públicos, notadamente as mensagens enviadas
pelo Chefe do Poder Executivo ao Poder Legislativo para in-
Definição e Finalidade formar sobre fato da Administração Pública; expor o plano
O memorando é a modalidade de comunicação entre de governo por ocasião da abertura de sessão legislativa;
unidades administrativas de um mesmo órgão, que podem submeter ao Congresso Nacional matérias que dependem
estar hierarquicamente em mesmo nível ou em nível dife- de deliberação de suas Casas; apresentar veto; enfim, fazer
rente. Trata-se, portanto, de uma forma de comunicação e agradecer comunicações de tudo quanto seja de interes-
eminentemente interna. se dos poderes públicos e da Nação.
Pode ter caráter meramente administrativo, ou ser em- Minuta de mensagem pode ser encaminhada pelos Mi-
pregado para a exposição de projetos, ideias, diretrizes, nistérios à Presidência da República, a cujas assessorias ca-
etc. a serem adotados por determinado setor do serviço berá a redação final.
público. As mensagens mais usuais do Poder Executivo ao Con-
Sua característica principal é a agilidade. A tramitação gresso Nacional têm as seguintes finalidades:
do memorando em qualquer órgão deve pautar-se pela ra- - encaminhamento de projeto de lei ordinária, comple-
pidez e pela simplicidade de procedimentos burocráticos. mentar ou financeira;
Para evitar desnecessário aumento do número de comu- - encaminhamento de medida provisória;
nicações, os despachos ao memorando devem ser dados - indicação de autoridades;

53
LÍNGUA PORTUGUESA

- pedido de autorização para o Presidente ou o Vice- Se necessário o arquivamento, deve-se fazê-lo com có-
-Presidente da República ausentarem-se do País por mais pia do fax e não com o próprio fax, cujo papel, em certos
de 15 dias; modelos, deteriora-se rapidamente.
- encaminhamento de atos de concessão e renovação
de concessão de emissoras de rádio e TV; Forma e Estrutura
- encaminhamento das contas referentes ao exercício Os documentos enviados por fax mantêm a forma e a
anterior; estrutura que lhes são inerentes.
- mensagem de abertura da sessão legislativa; É conveniente o envio, juntamente com o documento
- comunicação de sanção (com restituição de autógra- principal, de folha de rosto, e de pequeno formulário com
fos); os dados de identificação da mensagem a ser enviada, con-
- comunicação de veto; forme exemplo a seguir:
- outras mensagens.
[Órgão Expedidor]
Forma e Estrutura
[setor do órgão expedidor]
As mensagens contêm: a) a indicação do tipo de ex-
[endereço do órgão expedidor]
pediente e de seu número, horizontalmente, no início da
Destinatário:____________________________________
margem esquerda; b) vocativo, de acordo com o pronome
No do fax de destino:_______________ Data:___/___/___
de tratamento e o cargo do destinatário, horizontalmente,
no início da margem esquerda (Excelentíssimo Senhor Pre- Remetente: ____________________________________
sidente do Senado Federal); c) o texto, iniciando a 2 cm do Tel. p/ contato:____________ Fax/correio eletrônico:____
vocativo; d) o local e a data, verticalmente a 2 cm do final No de páginas: ________No do documento:____________
do texto, e horizontalmente fazendo coincidir seu final com
a margem direita. Observações:___________________________________

A mensagem, como os demais atos assinados pelo Pre- Correio Eletrônico


sidente da República, não traz identificação de seu signa-
tário. Definição e finalidade

Telegrama O correio eletrônico (“e-mail”), por seu baixo custo e


celeridade, transformou-se na principal forma de comuni-
Definição e Finalidade cação para transmissão de documentos.
Com o fito de uniformizar a terminologia e simplificar
os procedimentos burocráticos, passa a receber o título de Forma e Estrutura
telegrama toda comunicação oficial expedida por meio de
telegrafia, telex, etc. Um dos atrativos de comunicação por correio eletrô-
Por tratar-se de forma de comunicação dispendiosa aos nico é sua flexibilidade. Assim, não interessa definir forma
cofres públicos e tecnologicamente superada, deve restrin- rígida para sua estrutura. Entretanto, deve-se evitar o uso
gir-se o uso do telegrama apenas àquelas situações que de linguagem incompatível com uma comunicação oficial.
não seja possível o uso de correio eletrônico ou fax e que O campo “assunto” do formulário de correio eletrôni-
a urgência justifique sua utilização e, também em razão de co mensagem deve ser preenchido de modo a facilitar a
seu custo elevado, esta forma de comunicação deve pau- organização documental tanto do destinatário quanto do
tar-se pela concisão. remetente.
Para os arquivos anexados à mensagem deve ser utili-
Forma e Estrutura
zado, preferencialmente, o formato Rich Text. A mensagem
Não há padrão rígido, devendo-se seguir a forma e
que encaminha algum arquivo deve trazer informações mí-
a estrutura dos formulários disponíveis nas agências dos
nimas sobre seu conteúdo.
Correios e em seu sítio na Internet.
Sempre que disponível, deve-se utilizar recurso de con-
Fax firmação de leitura. Caso não seja disponível, deve constar
da mensagem pedido de confirmação de recebimento.
Definição e Finalidade
O fax (forma abreviada já consagrada de fac-símile) é Valor documental
uma forma de comunicação que está sendo menos usada
devido ao desenvolvimento da Internet. É utilizado para a Nos termos da legislação em vigor, para que a mensa-
transmissão de mensagens urgentes e para o envio ante- gem de correio eletrônico tenha valor documental, e para
cipado de documentos, de cujo conhecimento há premên- que possa ser aceito como documento original, é neces-
cia, quando não há condições de envio do documento por sário existir certificação digital que ateste a identidade do
meio eletrônico. Quando necessário o original, ele segue remetente, na forma estabelecida em lei.
posteriormente pela via e na forma de praxe.

54
LÍNGUA PORTUGUESA

ELEMENTOS DE ORTOGRAFIA E GRAMÁTICA Esses seriam os padrões básicos para as orações, ou seja, as
frases que possuem apenas um verbo conjugado. Na constru-
Problemas de Construção de Frases ção de períodos, as várias funções podem ocorrer em ordem
inversa à mencionada, misturando-se e confundindo-se. Não
A clareza e a concisão na forma escrita são alcançadas interessa aqui análise exaustiva de todos os padrões existentes
principalmente pela construção adequada da frase, “a menor na língua portuguesa. O que importa é fixar a ordem normal
unidade autônoma da comunicação”, na definição de Celso dos elementos nesses seis padrões básicos. Acrescente-se que
Pedro Luft. períodos mais complexos, compostos por duas ou mais ora-
A função essencial da frase é desempenhada pelo predi- ções, em geral podem ser reduzidos aos padrões básicos (de
cado, que, para Adriano da Gama Kury, pode ser entendido que derivam). Os problemas mais frequentemente encontra-
como “a enunciação pura de um fato qualquer”. Sempre que dos na construção de frases dizem respeito à má pontuação,
a frase possuir pelo menos um verbo, recebe o nome de pe- à ambiguidade da ideia expressa, à elaboração de falsos pa-
ríodo, que terá tantas orações quantos forem os verbos não ralelismos, erros de comparação, etc. Decorrem, em geral, do
auxiliares que o constituem. desconhecimento da ordem das palavras na frase. Indicam-se,
Outra função relevante é a do sujeito – mas não indis- a seguir, alguns desses defeitos mais comuns e recorrentes na
pensável, pois há orações sem sujeito, ditas impessoais –, construção de frases, registrados em documentos oficiais.
de quem se diz algo, cujo núcleo é sempre um substantivo.
Sempre que o verbo o exigir, teremos nas orações substan- Sujeito
tivos (nomes ou pronomes) que desempenham a função de Como dito, o sujeito é o ser de quem se fala ou que exe-
complementos (objetos direto e indireto, predicativo e com- cuta a ação enunciada na oração. Ele pode ter complemen-
plemento adverbial). Função acessória desempenham os ad- to, mas não ser complemento. Devem ser evitadas, portanto,
juntos adverbiais, que vêm geralmente ao final da oração, construções como:
mas que podem ser ou intercalados aos elementos que de- Errado: É tempo do Congresso votar a emenda.
sempenham as outras funções, ou deslocados para o início Certo: É tempo de o Congresso votar a emenda.
da oração. Errado: Apesar das relações entre os países estarem cor-
Temos, assim, a seguinte ordem de colocação dos ele- tadas, (...).
mentos que compõem uma oração (Observação: os parênte- Certo: Apesar de as relações entre os países estarem cor-
ses indicam os elementos que podem não ocorrer): tadas, (...).
(sujeito) - verbo - (complementos) - (adjunto adverbial). Errado: Não vejo mal no Governo proceder assim.
Certo: Não vejo mal em o Governo proceder assim.
Podem ser identificados seis padrões básicos para as ora- Errado: Antes destes requisitos serem cumpridos, (...).
ções pessoais (i. é, com sujeito) na língua portuguesa (a fun- Certo: Antes de estes requisitos serem cumpridos, (...).
ção que vem entre parênteses é facultativa e pode ocorrer Errado: Apesar da Assessoria ter informado em tempo, (...).
em ordem diversa): Certo: Apesar de a Assessoria ter informado em tempo, (...).
1. Sujeito - verbo intransitivo - (Adjunto Adverbial)
O Presidente - regressou - (ontem). Frases Fragmentadas
A fragmentação de frases “consiste em pontuar uma
2. Sujeito - verbo transitivo direto - objeto direto - (adjunto oração subordinada ou uma simples locução como se fosse
adverbial) uma frase completa”. Decorre da pontuação errada de uma
O Chefe da Divisão - assinou - o termo de posse - (na ma- frase simples. Embora seja usada como recurso estilístico na
nhã de terça-feira). literatura, a fragmentação de frases deve ser evitada nos tex-
tos oficiais, pois muitas vezes dificulta a compreensão. Ex.:
3. Sujeito - verbo transitivo indireto - objeto indireto - (ad- Errado: O programa recebeu a aprovação do Congresso
junto adverbial). Nacional. Depois de ser longamente debatido.
O Brasil - precisa - de gente honesta - (em todos os setores). Certo: O programa recebeu a aprovação do Congresso
Nacional, depois de ser longamente debatido.
4. Sujeito - verbo transitivo direto e indireto - obj. direto - Certo: Depois de ser longamente debatido, o programa
obj. indireto - (adj. Adv.) recebeu a aprovação do Congresso Nacional.
Os desempregados - entregaram - suas reivindicações - ao Errado: O projeto de Convenção foi oportunamente sub-
Deputado - (no Congresso). metido ao Presidente da República, que o aprovou. Consulta-
das as áreas envolvidas na elaboração do texto legal.
5. Sujeito - verbo transitivo indireto - complemento adver- Certo: O projeto de Convenção foi oportunamente subme-
bial - (adjunto adverbial) tido ao Presidente da República, que o aprovou, consultadas
A reunião do Grupo de Trabalho - ocorrerá - em Buenos as áreas envolvidas na elaboração do texto legal.
Aires - (na próxima semana).
O Presidente - voltou - da Europa - (na sexta-feira) Fontes:
http://www.redacaooficial.com.br/redacao_oficial_publi-
6. Sujeito - verbo de ligação - predicativo - (adjunto ad- cacoes_ver.php?id=2
verbial) http://portuguesxconcursos.blogspot.com.br/p/redacao-
O problema - será - resolvido - prontamente. -oficial-para-concursos.html

55
LÍNGUA PORTUGUESA

ATIVIDADES c) assunto: resumo do teor do documento


d) destinatário: o nome e o cargo da pessoa a quem é
1-) (TRIBUNAL DE JUSTIÇA DO ESTADO DO ACRE – TÉC- dirigida a comunicação. No caso do ofício deve ser incluído
NICO EM MICROINFORMÁTICA - CESPE/2012) O correio também o endereço.
eletrônico é uma forma de comunicação célere, na qual e) texto;
deve ser utilizada linguagem compatível com a comunica- f) fecho;
ção oficial, embora não seja definida uma forma rígida para g) assinatura do autor da comunicação; e
sua estrutura. h) identificação do signatário
( ) Certo ( ) Errado
(Fonte: http://webcache.googleusercontent.com/
O correio eletrônico (“e-mail”), por seu baixo custo e search?q=cache:omaLJnt2UtQJ:www.planalto.gov.br/cci-
celeridade, transformou-se na principal forma de comuni- vil_03/manual/Manual_Rich_RedPR2aEd.rtf+&cd=1&hl=p-
cação para transmissão de documentos. t-BR&ct=clnk&gl=br)
Um dos atrativos de comunicação por correio eletrô- RESPOSTA: “ERRADO”.
nico é sua flexibilidade. Assim, não interessa definir forma
rígida para sua estrutura. Entretanto, deve-se evitar o uso 5-) (MINISTÉRIO DO DESENVOLVIMENTO, INDÚSTRIA E
de linguagem incompatível com uma comunicação oficial COMÉRCIO EXTERIOR – ANALISTA TÉCNICO ADMINISTRATI-
(v. 1.2 A Linguagem dos Atos e Comunicações Oficiais). VO – CESPE/2014) Em “Vossa Excelência deve estar satisfeita
(Fonte: http://www.planalto.gov.br/ccivil_03/manual/ com os resultados das negociações”, o adjetivo estará corre-
manual.htm) tamente empregado se dirigido a ministro de Estado do sexo
RESPOSTA: “CERTO”. masculino, pois o termo “satisfeita” deve concordar com a
locução pronominal de tratamento “Vossa Excelência”.
2-) (POLÍCIA CIVIL DO ESTADO DE ALAGOAS – AGENTE ( ) Certo ( ) Errado
DE POLÍCIA – CESPE/2012) O vocativo a ser empregado em Se a pessoa, no caso o ministro, for do sexo feminino (mi-
comunicações dirigidas ao chefe do Poder Executivo da Re- nistra), o adjetivo está correto; mas, se for do sexo masculino,
pública Federativa do Brasil é Excelentíssimo Senhor. o adjetivo sofrerá flexão de gênero: satisfeito. O pronome
( ) Certo ( ) Errado de tratamento é apenas a maneira como tratar a autoridade,
não regendo as demais concordâncias.
(...) O vocativo a ser empregado em comunicações diri- RESPOSTA: “ERRADO”.
gidas aos Chefes de Poder é Excelentíssimo Senhor, segui-
do do cargo respectivo: 6-) (ACADEMIA DE POLÍCIA DO ESTADO DE MINAS GERAIS
Excelentíssimo Senhor Presidente da República (...) – TÉCNICO ASSISTENTE DA POLÍCIA CIVIL - FUMARC/2013)
(Fonte: http://www.planalto.gov.br/ccivil_03/manual/ Sobre a Redação Oficial, NÃO é correto afirmar que
manual.htm) (A) exige emprego do padrão formal de linguagem.
RESPOSTA: “CERTO”. (B) deve permitir uma única interpretação e ser estrita-
mente impessoal.
3-) (GOVERNO DO ESTADO DE ALAGOAS – TÉCNICO (C) sua finalidade básica é comunicar com impessoalida-
FORENSE - CESPE/2013) A concisão, uma das qualidades de e máxima clareza.
essenciais ao texto oficial, para a qual concorrem o domí- (D) dispensa a formalidade de tratamento, uma vez que o
nio do assunto tratado e a revisão textual, consiste em se comunicador e o receptor são o Serviço Público.
transmitir, no texto escrito, o máximo de informações em-
pregando-se um mínimo de palavras. As comunicações oficiais devem ser sempre formais, isto
( ) Certo ( ) Errado é, obedecem a certas regras de forma: além das (...) exigên-
cias de impessoalidade e uso do padrão culto de linguagem,
É a qualidade esperada de um bom texto, assim ele não é imperativo, ainda, certa formalidade de tratamento. Não se
se torna prolixo: “fala, fala, mas não diz nada!”. trata somente da eterna dúvida quanto ao correto emprego
RESPOSTA: “CERTO”. deste ou daquele pronome de tratamento para uma autori-
dade de certo nível (...); mais do que isso, a formalidade diz
4-) (GOVERNO DO ESTADO DE ALAGOAS – TÉCNICO respeito à polidez, à civilidade no próprio enfoque dado ao
FORENSE - CESPE/2013) Na parte superior do ofício, do assunto do qual cuida a comunicação.
aviso e do memorando, antes do assunto, devem constar (Fonte: http://www.planalto.gov.br/ccivil_03/manual/ma-
o nome e o endereço da autoridade a quem é direcionada nual.htm_)
a comunicação. RESPOSTA: “D”.
( ) Certo ( ) Errado
O aviso, o ofício e o memorando devem conter as se- 7-) (ACADEMIA DE POLÍCIA DO ESTADO DE MINAS
guintes partes: GERAIS – TÉCNICO ASSISTENTE DA POLÍCIA CIVIL - FU-
a) tipo e número do expediente, seguido da sigla do MARC/2013 - adaptada) “Na revisão de um expediente, de-
órgão que o expede: ve-se avaliar, ainda, se ele será de fácil compreensão por
b) local e data em que foi assinado, por extenso, com seu destinatário. O que nos parece óbvio pode ser des-
alinhamento à direita: conhecido por terceiros. O domínio que adquirimos sobre

56
LÍNGUA PORTUGUESA

certos assuntos em decorrência de nossa experiência pro- Ata é um documento administrativo que tem a finali-
fissional muitas vezes faz com que os tomemos como de dade de registrar de modo sucinto a sequência de even-
conhecimento geral, o que nem sempre é verdade. Explici- tos de uma reunião ou assembleia de pessoas com um fim
te, desenvolva, esclareça, precise os termos técnicos, o sig- específico. É característica da Ata apresentar um resumo,
nificado das siglas e abreviações e os conceitos específicos cronologicamente disposto, de modo infalível, de todo o
que não possam ser dispensados.” desenrolar da reunião.
(Manual de Redação Oficial da Presidência da Repúbli- (Fonte: https://www.10emtudo.com.br/aula/ensino/a_
ca. p. 14). redacao_oficial_ata/)
RESPOSTA: “ERRADO”.
Sobre a Redação Oficial, pode-se concluir que
(A) a concisão de um texto está relacionada ao grau de 10-) (TRE/PA- ANALISTA JUDICIÁRIO – FGV/2011) Se-
especificação dos termos. gundo o Manual de Redação da Presidência da República,
(B) a padronização de termos e conceitos viabiliza a uni- NÃO se deve usar Vossa Excelência para
formidade dos documentos. (A) embaixadores.
(C) a revisão possibilita a substituição de termos, muitas (B) conselheiros dos Tribunais de Contas estaduais.
vezes, desconhecidos pelo leitor. (C) prefeitos municipais.
(D) claro é o texto que exige releituras mais aprofun- (D) presidentes das Câmaras de Vereadores.
dadas. (E) vereadores.

Através da leitura do excerto e das próprias alternativas, (...) O uso do pronome de tratamento Vossa Senhoria
chegamos à conclusão de que um texto, principalmente (abreviado V. Sa.) para vereadores está correto, sim. Numa
oficial, deve priorizar a revisão. Câmara de Vereadores só se usa Vossa Excelência para o
RESPOSTA: “C”. seu presidente, de acordo com o Manual de Redação da
Presidência da República (1991).
8-) (CNJ – TÉCNICO JUDICIÁRIO – CESPE/2013) O expe- (Fonte: http://www.linguabrasil.com.br/nao-tropece-
diente adequado para a comunicação entre ministros de detail.php?id=393)
RESPOSTA: “E”.
Estado é a mensagem.
( ) Certo ( ) Errado
Mensagem – é o instrumento de comunicação oficial
7. PONTUAÇÃO;
entre os Chefes dos Poderes Públicos, notadamente as
mensagens enviadas pelo Chefe do Poder Executivo ao Po-
der Legislativo para informar sobre fato da Administração
Os sinais de pontuação são marcações gráficas que
Pública; expor o plano de governo por ocasião da aber-
servem para compor a coesão e a coerência textual, além
tura de sessão legislativa; submeter ao Congresso Nacio-
de ressaltar especificidades semânticas e pragmáticas. Ve-
nal matérias que dependem de deliberação de suas Casas; jamos as principais funções dos sinais de pontuação co-
apresentar veto; enfim, fazer e agradecer comunicações de nhecidos pelo uso da língua portuguesa.
tudo quanto seja de interesse dos poderes públicos e da
Nação. Ponto
Aviso e Ofício - são modalidades de comunicação ofi- 1- Indica o término do discurso ou de parte dele.
cial praticamente idênticas. A única diferença entre eles é - Façamos o que for preciso para tirá-la da situação em
que o aviso é expedido exclusivamente por Ministros de que se encontra.
Estado, para autoridades de mesma hierarquia, ao passo - Gostaria de comprar pão, queijo, manteiga e leite.
que o ofício é expedido para e pelas demais autoridades. - Acordei. Olhei em volta. Não reconheci onde estava.
Ambos têm como finalidade o tratamento de assuntos ofi-
ciais pelos órgãos da Administração Pública entre si e, no 2- Usa-se nas abreviações - V. Exª. - Sr.
caso do ofício, também com particulares.
(Fonte: http://www.fontedosaber.com/portugues/re- Ponto e Vírgula ( ; )
dacao-oficial-dicas-e-macetes.html) 1- Separa várias partes do discurso, que têm a mesma
RESPOSTA: “ERRADO”. importância.
- “Os pobres dão pelo pão o trabalho; os ricos dão pelo
9-) (ANP – CONHECIMENTO BÁSICO PARA TODOS OS pão a fazenda; os de espíritos generosos dão pelo pão a vida;
CARGOS – CESPE/2013) Na redação de uma ata, devem- os de nenhum espírito dão pelo pão a alma...” (VIEIRA)
se relatar exaustivamente, com o máximo de detalhamento
possível, incluindo-se os aspectos subjetivos, as discussões, 2- Separa partes de frases que já estão separadas por
as propostas, as resoluções e as deliberações ocorridas em vírgulas.
reuniões e eventos que exigem registro. - Alguns quiseram verão, praia e calor; outros, monta-
( ) Certo ( ) Errado nhas, frio e cobertor.

57
LÍNGUA PORTUGUESA

3- Separa itens de uma enumeração, exposição de mo- Usa-se a vírgula:


tivos, decreto de lei, etc. - Para marcar intercalação:
- Ir ao supermercado; a) do adjunto adverbial: O café, em razão da sua abun-
- Pegar as crianças na escola; dância, vem caindo de preço.
- Caminhada na praia; b) da conjunção: Os cerrados são secos e áridos. Estão
- Reunião com amigos. produzindo, todavia, altas quantidades de alimentos.
c) das expressões explicativas ou corretivas: As indústrias
Dois pontos não querem abrir mão de suas vantagens, isto é, não querem
1- Antes de uma citação abrir mão dos lucros altos.
- Vejamos como Afrânio Coutinho trata este assunto:
- Para marcar inversão:
2- Antes de um aposto a) do adjunto adverbial (colocado no início da oração):
- Três coisas não me agradam: chuva pela manhã, frio à Depois das sete horas, todo o comércio está de portas fecha-
tarde e calor à noite. das.
b) dos objetos pleonásticos antepostos ao verbo: Aos
3- Antes de uma explicação ou esclarecimento pesquisadores, não lhes destinaram verba alguma.
- Lá estava a deplorável família: triste, cabisbaixa, viven- c) do nome de lugar anteposto às datas: Recife, 15 de
do a rotina de sempre. maio de 1982.
- Para separar entre si elementos coordenados (dispos-
4- Em frases de estilo direto tos em enumeração):
Maria perguntou: Era um garoto de 15 anos, alto, magro.
- Por que você não toma uma decisão? A ventania levou árvores, e telhados, e pontes, e animais.

Ponto de Exclamação - Para marcar elipse (omissão) do verbo:


1- Usa-se para indicar entonação de surpresa, cólera, Nós queremos comer pizza; e vocês, churrasco.
susto, súplica, etc. - Para isolar:
- Sim! Claro que eu quero me casar com você! - o aposto: São Paulo, considerada a metrópole brasileira,
2- Depois de interjeições ou vocativos possui um trânsito caótico.
- Ai! Que susto! - o vocativo: Ora, Thiago, não diga bobagem.
- João! Há quanto tempo!
Ponto de Interrogação Fontes: http://www.infoescola.com/portugues/pontua-
Usa-se nas interrogações diretas e indiretas livres. cao/
“- Então? Que é isso? Desertaram ambos?” (Artur Aze- http://www.brasilescola.com/gramatica/uso-da-virgula.
vedo) htm

Reticências Questões sobre Pontuação


1- Indica que palavras foram suprimidas.
- Comprei lápis, canetas, cadernos... 01. (Agente Policial – Vunesp – 2013). Assinale a alterna-
tiva em que a pontuação está corretamente empregada, de
2- Indica interrupção violenta da frase. acordo com a norma-padrão da língua portuguesa.
“- Não... quero dizer... é verdad... Ah!” (A) Diante da testemunha, o homem abriu a bolsa e, em-
bora, experimentasse, a sensação de violar uma intimidade,
3- Indica interrupções de hesitação ou dúvida procurou a esmo entre as coisinhas, tentando encontrar
- Este mal... pega doutor? algo que pudesse ajudar a revelar quem era a sua dona.
(B) Diante, da testemunha o homem abriu a bolsa e, em-
4- Indica que o sentido vai além do que foi dito bora experimentasse a sensação, de violar uma intimidade,
- Deixa, depois, o coração falar... procurou a esmo entre as coisinhas, tentando encontrar
algo que pudesse ajudar a revelar quem era a sua dona.
Vírgula (C) Diante da testemunha, o homem abriu a bolsa e, em-
bora experimentasse a sensação de violar uma intimidade,
Não se usa vírgula procurou a esmo entre as coisinhas, tentando encontrar
*separando termos que, do ponto de vista sintático, li- algo que pudesse ajudar a revelar quem era a sua dona.
gam-se diretamente entre si: (D) Diante da testemunha, o homem, abriu a bolsa e,
- entre sujeito e predicado. embora experimentasse a sensação de violar uma intimida-
Todos os alunos da sala foram advertidos. de, procurou a esmo entre as coisinhas, tentando, encontrar
Sujeito predicado algo que pudesse ajudar a revelar quem era a sua dona.
(E) Diante da testemunha, o homem abriu a bolsa e, em-
- entre o verbo e seus objetos. bora, experimentasse a sensação de violar uma intimidade,
O trabalho custou sacrifício aos realizadores. procurou a esmo entre as coisinhas, tentando, encontrar
V.T.D.I. O.D. O.I. algo que pudesse ajudar a revelar quem era a sua dona.

58
LÍNGUA PORTUGUESA

02. (CNJ – TÉCNICO JUDICIÁRIO – CESPE/2013 - ADAP- O projeto Escola de Bicicleta está distribuindo bicicletas
TADA) Jogadores de futebol de diversos times entraram em de bambu para 4600 alunos da rede pública de São Pau-
campo em prol do programa “Pai Presente”, nos jogos do lo(A) o programa desenvolve ainda oficinas e cursos para as
Campeonato Nacional em apoio à campanha que visa 4 re- crianças utilizarem a bicicleta de forma segura e correta(B) os
duzir o número de pessoas que não possuem o nome do pai alunos ajudam a traçar ciclorrotas e participam de atividades
em sua certidão de nascimento. (...) sobre cidadania e reciclagem(C) as escolas participantes se
A oração subordinada “que não possuem o nome do pai tornam também centros de descarte de garrafas PET(D) des-
em sua certidão de nascimento” não é antecedida por vírgu- tinadas depois para reciclagem(E) o programa possibilitará
la porque tem natureza restritiva. o retorno das bicicletas pela saúde das crianças e transfor-
( ) Certo ( ) Errado mação das comunidades em lugares melhores para se viver.
(Adaptado de Vida Simples, abril de 2012, edição 117)
03.(BNDES – TÉCNICO ADMINISTRATIVO – BNDES/2012) a) A
Em que período a vírgula pode ser retirada, mantendo-se o b) B
sentido e a obediência à norma-padrão? c) C
(A) Quando o técnico chegou, a equipe começou o trei- d) D
no. e) E
(B) Antônio, quer saber as últimas novidades dos es-
portes? 07. (DETRAN - OFICIAL ESTADUAL DE TRÂNSITO – VU-
(C) As Olimpíadas de 2016 ocorrerão no Rio, que se pre- NESP/2013) Assinale a alternativa correta quanto ao uso da
para para o evento. pontuação.
(D) Atualmente, várias áreas contribuem para o aprimo- (A) Segundo alguns psicólogos, é possível, em certas cir-
ramento do desportista. cunstâncias, ceder à frustração para que a raiva seja aliviada.
(E) Eis alguns esportes que a Ciência do Esporte ajuda: (B) Dirigir pode aumentar, nosso nível de estresse, por-
judô, natação e canoagem. que você está junto; com os outros motoristas cujos com-
portamentos, são desconhecidos.
04. (BANPARÁ/PA – TÉCNICO BANCÁRIO – ESPP/2012)
(C) Os motoristas, devem saber, que os carros podem
Assinale a alternativa em que a pontuação está correta.
ser uma extensão de nossa personalidade.
a) Meu grande amigo Pedro, esteve aqui ontem!
(D) A ira de trânsito pode ocasionar, acidentes e; aumen-
b) Foi solicitado, pelo diretor o comprovante da tran-
tar os níveis de estresse em alguns motoristas.
sação.
(E) Os congestionamentos e o número de motoristas na
c) Maria, você trouxe os documentos?
rua, são as principais causas da ira de trânsito.
d) O garoto de óculos leu, em voz alta o poema.
08. (ACADEMIA DE POLÍCIA DO ESTADO DE MINAS
e) Na noite de ontem o vigia percebeu, uma movimen-
GERAIS – TÉCNICO ASSISTENTE DA POLÍCIA CIVIL - FU-
tação estranha. MARC/2013) “Paciência, minha filha, este é apenas um ciclo
econômico e a nossa geração foi escolhida para este vexame,
05. (Papiloscopista Policial – Vunesp – 2013 – adap.). As- você aí desse tamanho pedindo esmola e eu aqui sem nada
sinale a alternativa em que a frase mantém-se correta após para te dizer, agora afasta que abriu o sinal.”
o acréscimo das vírgulas. No período acima, as vírgulas foram empregadas em
(A) Se a criança se perder, quem encontrá-la, verá na “Paciência, minha filha, este é [...]”, para separar
pulseira instruções para que envie, uma mensagem eletrô- (A) aposto.
nica ao grupo ou acione o código na internet. (B) vocativo.
(B) Um geolocalizador também, avisará, os pais de onde (C) adjunto adverbial.
o código foi acionado. (D) expressão explicativa.
(C) Assim que o código é digitado, familiares cadastra- 09. (INFRAERO – CADASTRO RESERVA OPERACIONAL
dos, recebem automaticamente, uma mensagem dizendo PROFISSIONAL DE TRÁFEGO AÉREO – FCC/2011) O período
que a criança foi encontrada. corretamente pontuado é:
(D) De fabricação chinesa, a nova pulseirinha, chega pri- (A) Os filmes que, mostram a luta pela sobrevivência em
meiro às, areias do Guarujá. condições hostis nem sempre conseguem agradar, aos es-
(E) O sistema permite, ainda, cadastrar o nome e o te- pectadores.
lefone de quem a encontrou e informar um ponto de re- (B) Várias experiências de prisioneiros, semelhantes en-
ferência tre si, podem ser reunidas e fazer parte de uma mesma his-
tória ficcional.
06. (DNIT – TÉCNICO ADMINISTRATIVO – ESAF/2013) (C) A história de heroísmo e de determinação que nem
Para que o fragmento abaixo seja coerente e gramatical- sempre, é convincente, se passa em um cenário marcado,
mente correto, é necessário inserir sinais de pontuação. pelo frio.
Assinale a posição em que não deve ser usado o sinal de (D) Caminhar por um extenso território gelado, é correr
ponto, e sim a vírgula, para que sejam respeitadas as re- riscos iminentes que comprometem, a sobrevivência.
gras gramaticais. Desconsidere os ajustes nas letras iniciais (E) Para os fugitivos que se propunham, a alcançar a
minúsculas. liberdade, nada poderia parecer, realmente intransponível.

59
LÍNGUA PORTUGUESA

GABARITO 5-) Assinalei com (X) onde estão as pontuações inade-


quadas
01. C 02. C 03. D 04. C 05. E (A) Se a criança se perder, quem encontrá-la , (X) verá
06. D 07. A 08. B 09.B na pulseira instruções para que envie , (X) uma mensagem
eletrônica ao grupo ou acione o código na internet.
RESOLUÇÃO (B) Um geolocalizador também , (X) avisará , (X) os pais
de onde o código foi acionado.
1- Assinalei com um (X) as pontuações inadequadas (C) Assim que o código é digitado, familiares cadastra-
(A) Diante da testemunha, o homem abriu a bolsa e, dos , (X) recebem ( , ) automaticamente, uma mensagem
embora, (X) experimentasse , (X) a sensação de violar uma dizendo que a criança foi encontrada.
intimidade, procurou a esmo entre as coisinhas, tentando (D) De fabricação chinesa, a nova pulseirinha , (X) chega
encontrar algo que pudesse ajudar a revelar quem era a primeiro às , (X) areias do Guarujá.
sua dona.
(B) Diante , (X) da testemunha o homem abriu a bolsa 6-)
e, embora experimentasse a sensação , (X) de violar uma O projeto Escola de Bicicleta está distribuindo bicicletas de
intimidade, procurou a esmo entre as coisinhas, tentando bambu para 4600 alunos da rede pública de São Paulo(A). O
encontrar algo que pudesse ajudar a revelar quem era a programa desenvolve ainda oficinas e cursos para as crianças
sua dona. utilizarem a bicicleta de forma segura e correta(B). Os alunos
(D) Diante da testemunha, o homem , (X) abriu a bolsa ajudam a traçar ciclorrotas e participam de atividades sobre
e, embora experimentasse a sensação de violar uma inti- cidadania e reciclagem(C). As escolas participantes se tornam
midade, procurou a esmo entre as coisinhas, tentando , (X) também centros de descarte de garrafas PET(D), destinadas
encontrar algo que pudesse ajudar a revelar quem era a depois para reciclagem(E). O programa possibilitará o retor-
sua dona. no das bicicletas pela saúde das crianças e transformação das
(E) Diante da testemunha, o homem abriu a bolsa e, comunidades em lugares melhores para se viver.
embora , (X) experimentasse a sensação de violar uma in-
A vírgula deve ser colocada após a palavra “PET”, posi-
timidade, procurou a esmo entre as coisinhas, tentando ,
ção (D), pois antecipa um termo explicativo.
(X) encontrar algo que pudesse ajudar a revelar quem era
a sua dona.
7-) Fiz as indicações (X) das pontuações inadequadas:
(A) Segundo alguns psicólogos, é possível, em certas cir-
2-) A oração restringe o grupo que participará da cam-
cunstâncias, ceder à frustração para que a raiva seja aliviada.
panha (apenas os que não têm o nome do pai na certidão
(B) Dirigir pode aumentar, (X) nosso nível de estresse,
de nascimento). Se colocarmos uma vírgula, a oração tor-
nar-se-á “explicativa”, generalizando a informação, o que porque você está junto; (X) com os outros motoristas cujos
dará a entender que TODAS as pessoa não têm o nome do comportamentos, (X) são desconhecidos.
pai na certidão. (C) Os motoristas, (X) devem saber, (X) que os carros po-
RESPOSTA: “CERTO”. dem ser uma extensão de nossa personalidade.
(D) A ira de trânsito pode ocasionar, (X) acidentes e; (X)
3-) aumentar os níveis de estresse em alguns motoristas.
(A) Quando o técnico chegou, a equipe começou o trei- (E) Os congestionamentos e o número de motoristas na
no. = mantê-la (termo deslocado) rua, (X) são as principais causas da ira de trânsito.
(B) Antônio, quer saber as últimas novidades dos espor- 8-) Paciência, minha filha, este é... = é o termo usado
tes? = mantê-la (vocativo) para se dirigir ao interlocutor, ou seja, é um vocativo.
(C) As Olimpíadas de 2016 ocorrerão no Rio, que se pre-
para para o evento. 9-) Fiz as marcações (X) onde as pontuações estão ina-
= mantê-la (explicação) dequadas ou faltantes:
(D) Atualmente, várias áreas contribuem para o aprimo- (A) Os filmes que,(X) mostram a luta pela sobrevivência
ramento do desportista. em condições hostis nem sempre conseguem agradar, (X)
= pode retirá-la (advérbio de tempo) aos espectadores.
(E) Eis alguns esportes que a Ciência do Esporte ajuda: (B) Várias experiências de prisioneiros, semelhantes en-
judô, natação e canoagem. tre si, podem ser reunidas e fazer parte de uma mesma his-
= mantê-la (enumeração) tória ficcional.
(C) A história de heroísmo e de determinação (X) que
4-) Assinalei com (X) a pontuação inadequada ou faltante: nem sempre, (X) é convincente, se passa em um cenário
a) Meu grande amigo Pedro, (X) esteve aqui ontem! marcado, (X) pelo frio.
b) Foi solicitado, (X) pelo diretor o comprovante da (D) Caminhar por um extenso território gelado, (X) é
transação. correr riscos iminentes (X) que comprometem, (X) a sobre-
c) Maria, você trouxe os documentos? vivência.
d) O garoto de óculos leu, em voz alta (X) o poema. (E) Para os fugitivos que se propunham, (X) a alcançar
e) Na noite de ontem (X) o vigia percebeu, (X) uma mo- a liberdade, nada poderia parecer, (X) realmente intrans-
vimentação estranha. ponível.

60
LÍNGUA PORTUGUESA

- a parte pelo todo.


8. FIGURAS DE SINTAXE; Vários brasileiros vivem sem teto, ao relento. (teto
substitui casa)

FIGURAS DE LINGUAGEM - o efeito pela causa.


Suou muito para conseguir a casa própria. (suor substitui
Segundo Mauro Ferreira, a importância em reconhecer o trabalho)
figuras de linguagem está no fato de que tal conhecimento,
além de auxiliar a compreender melhor os textos literários, Perífrase
deixa-nos mais sensíveis à beleza da linguagem e ao É a designação de um ser através de alguma de suas
significado simbólico das palavras e dos textos. características ou atributos, ou de um fato que o celebrizou.
Definição: Figuras de linguagem são certos recursos A Veneza Brasileira também é palco de grandes
não--convencionais que o falante ou escritor cria para dar espetáculos. (Veneza Brasileira = Recife)
maior expressividade à sua mensagem. A Cidade Maravilhosa está tomada pela violência.
(Cidade Maravilhosa = Rio de Janeiro)
Metáfora
É o emprego de uma palavra com o significado de outra Antítese
em vista de uma relação de semelhanças entre ambas. É Consiste no uso de palavras de sentidos opostos.
uma comparação subentendida. Nada com Deus é tudo.
Minha boca é um túmulo. Tudo sem Deus é nada.
Essa rua é um verdadeiro deserto.
Eufemismo
Comparação Consiste em suavizar palavras ou expressões que são
Consiste em atribuir características de um ser a outro, desagradáveis.
em virtude de uma determinada semelhança.
Ele foi repousar no céu, junto ao Pai. (repousar no céu
O meu coração está igual a um céu cinzento.
= morrer)
O carro dele é rápido como um avião.
Os homens públicos envergonham o povo. (homens
públicos = políticos)
Prosopopeia
É uma figura de linguagem que atribui características
Hipérbole
humanas a seres inanimados. Também podemos chamá-la
É um exagero intencional com a finalidade de tornar
de PERSONIFICAÇÃO.
mais expressiva a ideia.
O céu está mostrando sua face mais bela.
O cão mostrou grande sisudez. Ela chorou rios de lágrimas.
Muitas pessoas morriam de medo da perna cabeluda.
Sinestesia
Consiste na fusão de impressões sensoriais diferentes Ironia
(mistura dos cinco sentidos). Consiste na inversão dos sentidos, ou seja, afirmamos o
Raquel tem um olhar frio, desesperador. contrário do que pensamos.
Aquela criança tem um olhar tão doce. Que alunos inteligentes, não sabem nem somar.
Se você gritar mais alto, eu agradeço.
Catacrese
É o emprego de uma palavra no sentido figurado por Onomatopeia
falta de um termo próprio. Consiste na reprodução ou imitação do som ou voz
O menino quebrou o braço da cadeira. natural dos seres.
A manga da camisa rasgou. Com o au-au dos cachorros, os gatos desapareceram.
Miau-miau. – Eram os gatos miando no telhado a noite
Metonímia toda.
É a substituição de uma palavra por outra, quando
existe uma relação lógica, uma proximidade de sentidos Aliteração
que permite essa troca. Ocorre metonímia quando Consiste na repetição de um determinado som
empregamos: consonantal no início ou interior das palavras.
O rato roeu a roupa do rei de Roma.
- O autor pela obra.
Li Jô Soares dezenas de vezes. (a obra de Jô Soares) Elipse
Consiste na omissão de um termo que fica subentendido
- o continente pelo conteúdo. no contexto, identificado facilmente.
O ginásio aplaudiu a seleção. (ginásio está substituindo Após a queda, nenhuma fratura.
os torcedores)

61
LÍNGUA PORTUGUESA

Zeugma São conhecidas pelo nome de figuras de pensamento


Consiste na omissão de um termo já empregado os recursos estilísticos utilizados para incrementar o
anteriormente. significado das palavras no seu aspecto semântico.
Ele come carne, eu verduras.
São oito as figuras de pensamento:
Pleonasmo
Consiste na intensificação de um termo através da sua 1) Antítese
repetição, reforçando seu significado. É a aproximação de palavras ou expressões de sentidos
Nós cantamos um canto glorioso. opostos. O contraste que se estabelece serve para dar uma
ênfase aos conceitos envolvidos, o que não ocorreria com
Polissíndeto a exposição isolada dos mesmos. Exemplos:
É a repetição da conjunção entre as orações de um Viverei para sempre ou morrerei tentando.
período ou entre os termos da oração. Do riso se fez o pranto.
Chegamos de viagem e tomamos banho e saímos para Hoje fez sol, ontem, porém, choveu muito.
dançar.
2) Apóstrofe
Assíndeto É assim denominado o chamamento do receptor
Ocorre quando há a ausência da conjunção entre duas da mensagem, seja ele de natureza imaginária ou não. É
orações. utilizada para dar ênfase à expressão e realiza-se por meio
Chegamos de viagem, tomamos banho, depois saímos do vocativo. Exemplos:
para dançar. Deus! Ó Deus! Onde estás que não respondes?
Pai Nosso, que estais no céu;
Anacoluto Ó meu querido Santo António;
Consiste numa mudança repentina da construção
sintática da frase. 3) Paradoxo
Ele, nada podia assustá-lo. É uma proposição aparentemente absurda, resultante
- Nota: o anacoluto ocorre com frequência na linguagem da união de ideias que se contradizem referindo-se ao
falada, quando o falante interrompe a frase, abandonando mesmo termo. Os paradoxos viciosos são denominados
o que havia dito para reconstruí-la novamente. Oxímoros (ou oximoron). Exemplos:
“Menino do Rio / Calor que provoca arrepio...”
Anáfora “Amor é fogo que arde sem se ver; / É ferida que dói e
Consiste na repetição de uma palavra ou expressão não se sente; / É um contentamento descontente; / É dor que
para reforçar o sentido, contribuindo para uma maior desatina sem doer;” (Camões)
expressividade.
Cada alma é uma escada para Deus, 4) Eufemismo
Cada alma é um corredor-Universo para Deus, Consiste em empregar uma expressão mais suave,
Cada alma é um rio correndo por margens de Externo mais nobre ou menos agressiva, para atenuar uma verdade
Para Deus e em Deus com um sussurro noturno. tida como penosa, desagradável ou chocante. Exemplos:
(Fernando Pessoa) “E pela paz derradeira que enfim vai nos redimir Deus
lhe pague”. (Chico Buarque).
Silepse paz derradeira = morte
Ocorre quando a concordância é realizada com a ideia
e não sua forma gramatical. Existem três tipos de silepse: 5) Gradação
gênero, número e pessoa. Na gradação temos uma sequência de palavras que
- De gênero: Vossa excelência está preocupado com as intensificam a mesma ideia. Exemplo:
notícias. (a palavra vossa excelência é feminina quanto à “Aqui... além... mais longe por onde eu movo o passo.”
forma, mas nesse exemplo a concordância se deu com a (Castro Alves).
pessoa a que se refere o pronome de tratamento e não
com o sujeito). 6) Hipérbole
- De número: A boiada ficou furiosa com o peão e É a expressão intencionalmente exagerada com o
derrubaram a cerca. (nesse caso a concordância se deu com intuito de realçar uma ideia, proporcionando uma imagem
a ideia de plural da palavra boiada). emocionante e de impacto. Exemplos:
- De pessoa: As mulheres decidimos não votar em “Faz umas dez horas que essa menina penteia esse
determinado partido até prestarem conta ao povo. (nesse cabelo”.
tipo de silepse, o falante se inclui mentalmente entre os Ele morreu de tanto rir.
participantes de um sujeito em 3ª pessoa).

Fonte:http://juliobattisti.com.br/tutoriais/josebferraz/
figuraslinguagem001.asp

62
LÍNGUA PORTUGUESA

7) Ironia Os cavalinhos correndo,


Ocorre ironia quando, pelo contexto, pela entonação, E nós, cavalões, comendo...
pela contradição de termos, pretende-se questionar O Brasil politicando,
certo tipo de pensamento. A intenção é depreciativa ou Nossa! A poesia morrendo...
sarcástica. Exemplos: O sol tão claro lá fora,
Parece um anjinho aquele menino, briga com todos que O sol tão claro, Esmeralda,
estão por perto. E em minhalma — anoitecendo!
“Moça linda, bem tratada, / três séculos de família, / Manuel Bandeira
burra como uma porta: / um amor.” (Mário de Andrade).
Notamos que em todos os versos há a omissão do verbo
8) Prosopopeia ou Personificação estar, sendo este facilmente identificado pelo contexto.
Consiste na atribuição de ações, qualidades ou
características humanas a seres não humanos. Exemplos: Zeugma
Chora, viola. Ao contrário da elipse, na zeugma ocorre a omissão
A morte mostrou sua face mais sinistra. de um termo já expresso no discurso. Constatemos: Maria
O morro dos ventos uivantes. gosta de Matemática, eu de Português.
Observamos que houve a omissão do verbo gostar.

Figuras de construção ou sintaxe integram as Anáfora


chamadas figuras de linguagem, representando um Essa figura de linguagem se caracteriza pela repetição
subgrupo destas. Dessa forma, tendo em vista o padrão não intencional de um termo no início de um período, frase ou
convencional que prevalece nas figuras de linguagem (ou verso. Observemos um caso representativo:
seja, a subjetividade, a sensibilidade por parte do emissor, A Estrela
deixando às claras seus aspectos estilísticos), devemos
compreender sua denominação. Em outras palavras, por Vi uma estrela tão alta,
que “figuras de construção ou sintaxe”? Vi uma estrela tão fria!
Podemos afirmar que assim se denominam em virtude Vi uma estrela luzindo
de apresentarem algum tipo de modificação na estrutura Na minha vida vazia.
da oração, tendo em vista os reais e já ressaltados objetivos
Era uma estrela tão alta!
da enunciação (do discurso) – sendo o principal conferir
Era uma estrela tão fria!
ênfase a ela.
Era uma estrela sozinha
Assim sendo, comecemos entendendo que, em termos
Luzindo no fim do dia.
convencionais, a estrutura sintática da nossa língua se
[...]
perfaz de uma sequência, demarcada pelos seguintes
Manuel Bandeira
elementos:
Notamos a utilização de termos que se repetem
SUJEITO + PREDICADO + COMPLEMENTO sucessivamente em cada verso da criação de Manuel
Bandeira.
(Nós) CHEGAMOS ATRASADOS À REUNIÃO.
Polissíndeto
Temos, assim, um sujeito oculto – nós; um predicado Figura cuja principal característica se define pela
verbal – chegamos atrasados; e um complemento, repetição enfática do conectivo, geralmente representado
representado por um adjunto adverbial de lugar – à reunião. pela conjunção coordenada “e”. Observemos um verso
Quando há uma ruptura dessa sequência lógica, extraído de uma criação de Olavo Bilac, intitulada “A um
materializada pela inversão de termos, repetição ou até poeta”: “Trabalha e teima, e lima, e sofre, e sua!”
mesmo omissão destes, é justamente aí que as figuras em
questão se manifestam. Desse modo, elas se encontram Assíndeto
muito presentes na linguagem literária, na publicitária e Diferentemente do que ocorre no polissíndeto,
na linguagem cotidiana de forma geral. Vejamos cada uma manifestado pela repetição da conjunção, no assíndeto
delas de modo particular: ocorre a omissão deste. Vejamos: Vim, vi, venci (Júlio César)
Depreendemos que se trata de orações assindéticas,
Elipse justamente pela omissão do conectivo “e”.
Tal figura se caracteriza pela omissão de um termo na
oração não expresso anteriormente, contudo, facilmente Anacoluto
identificado pelo contexto. Vejamos um exemplo: Trata-se de uma figura que se caracteriza pela
interrupção da sequência lógica do pensamento, ou seja,
Rondó dos cavalinhos em termos sintáticos, afirma-se que há uma mudança na
[...] construção do período, deixando algum termo desligado
do restante dos elementos. Vejamos:

63
LÍNGUA PORTUGUESA

Essas crianças de hoje, elas estão muito evoluídas. O verbo auxiliar é o que se relaciona com o sujeito,
Notamos que o termo em destaque, que era para por isso concorda com este, ou seja, se o sujeito estiver
representar o sujeito da oração, encontra-se desligado no singular, o verbo auxiliar também ficará no singular; se
dos demais termos, não cumprindo, portanto, nenhuma o sujeito estiver no plural, o verbo auxiliar também ficará
função sintática. no plural. Na Língua Portuguesa os verbos auxiliares são os
seguintes: ser, estar, ter, haver, dever, poder, ir, dentre outros.
Inversão (ou Hipérbato) O verbo principal é o que indica se o sujeito possui uma
Trata-se da inversão da ordem direta dos termos da qualidade, se ele pratica uma ação ou se a sofre. É o mais
oração. Constatemos: Eufórico chegou o menino. importante da locução. Na Língua Portuguesa, o verbo
Deduzimos que o predicativo do sujeito (pois se trata principal surge sempre no infinitivo (terminado em –ar, -er,
de um predicado verbo-nominal) encontra-se no início da ou –ir), no gerúndio (terminado em –ndo) ou no particípio
oração, quando este deveria estar expresso no final, ou (terminado em –ado ou –ido, dentre outras terminações).
seja: O menino chegou eufórico. Veja alguns exemplos de locuções verbais:
Pleonasmo Os funcionários FORAM CONVOCADOS pelo diretor.
Figura que consiste na repetição enfática de uma ideia (aux.: SER; princ.: CONVOCAR)
antes expressa, tanto do ponto de vista sintático quanto Os estudantes ESTÃO RESPONDENDO às questões.
semântico, no intuito de reforçar a mensagem. Exemplo: (aux.: ESTAR; princ.: RESPONDER)
Vivemos uma vida tranquila. Os trabalhadores TÊM ENFRENTADO muitos problemas.
O termo em destaque reforça uma ideia antes (aux.: TER; princ.: ENFRENTAR)
ressaltada, uma vez que viver já diz respeito à vida. Temos O vereador HAVIA DENUNCIADO seus companheiros.
uma repetição de ordem semântica. (aux.: HAVER; princ.: DENUNCIAR)
A ele nada lhe devo. Os alunos DEVEM ESTUDAR todos os dias. (aux.: DEVER;
princ.: ESTUDAR)
Percebemos que o pronome oblíquo (lhe) faz referência
à terceira pessoa do singular, já expressa. Trata-se,
Sujeito:
portanto, de uma repetição de ordem sintática demarcada
pelo que chamamos de objeto direto pleonástico.
Para se descobrir qual o sujeito do verbo (ou da locução
verbal), deve-se perguntar a ele (ou a ela) o seguinte:
Observação importante: O pleonasmo utilizado sem
Que(m) é que ..........? A resposta será o sujeito. Por exemplo,
a intenção de conferir ênfase ao discurso, torna-se o que
analisemos a primeira frase dentre as apresentadas acima:
denominamos de vício de linguagem – ocorrência que deve
Os funcionários foram convocados pelo diretor.
ser evitada. Como, por exemplo: subir para cima, descer
para baixo, entrar para dentro, entre outras circunstâncias
linguísticas. O princípio é o verbo. Procura-se, portanto, o verbo: é
a locução verbal foram convocados. - - Pergunta-se a ela:
Que(m) é que foi convocado?
- Resposta: Os funcionários.
9. ANÁLISE SINTÁTICA; - O sujeito da oração, então, é o seguinte: os funcionários.
10. ORAÇÕES E SEUS TERMOS; Encontrado o sujeito, parte-se para a análise do verbo:
11. COORDENAÇÃO E SUBORDINAÇÃO; Se ele indicar que o sujeito possui uma qualidade, um
estado ou um modo de ser, sem praticar ação alguma, será
denominado de VERBO DE LIGAÇÃO. Os verbos de ligação
O princípio é o verbo. mais comuns são os seguintes: ser, estar, parecer, ficar,
permanecer e continuar. Não se esqueça, porém, de que só
Essa é a premissa fundamental da Sintaxe, que é a parte será verbo de ligação o que indicar qualidade, estado ou
da gramática que estuda as palavras enquanto elementos de modo de ser do sujeito, sem praticar ação alguma. Observe
uma frase, as suas relações de concordância, de subordinação as seguintes frases:
e de ordem. Significa que, ao se realizar a análise sintática O político continuou seu discurso mesmo com todas as
de uma oração, sempre se inicia pelo verbo. É a partir dele vaias recebidas.
que se descobre qual o sujeito da oração, se há a indicação Continuar, nesta frase, não é de ligação já que não
de qualidade, estado ou modo de ser do sujeito, se ele indica qualidade do sujeito, e sim ação.
pratica uma ação ou se a sofre, se há complemento verbal,
se há circunstância (adjunto adverbial), etc. A professora estava na sala de aula.
Nem sempre o verbo se apresenta sozinho em uma Estar, nesta frase, não é de ligação já que não indica
oração. Em muitos casos, surgem dois ou mais verbos qualidade do sujeito, e sim fato.
juntos, para indicar que se pratica ou se sofre uma ação, ou
que o sujeito possui uma qualidade. A essa junção, dá-se A garota estava muito alegre.
o nome de locução verbal. Toda locução verbal é formada Estar é verbo de ligação porque indica qualidade do
por um verbo auxiliar (ou mais de um) e um verbo principal sujeito.
(somente um).

64
LÍNGUA PORTUGUESA

Se o verbo indicar que o sujeito pratica uma ação, ou Questões sobre Análise Sintática
que participa ativamente de um fato, será denominado de
VERBO INTRANSITIVO ou VERBO TRANSITIVO, de acordo 01. (Agente de Apoio Administrativo – FCC – 2013). Os
com o seguinte: trabalhadores passaram mais tempo na escola...
O segmento grifado acima possui a mesma função
- Quem ............ , ................. : Todo verbo que se encaixar sintática que o destacado em:
nessa frase será INTRANSITIVO. Por exemplo, o verbo A) ...o que reduz a média de ganho da categoria.
correr: Quem corre, corre. B) ...houve mais ofertas de trabalhadores dessa classe.
C) O crescimento da escolaridade também foi
- Quem ............ , ................. algo/alguém: Todo verbo impulsionado...
que se encaixar nessa frase será TRANSITIVO DIRETO. Por D) ...elevando a fatia dos brasileiros com ensino
exemplo, o verbo comer: Quem come, come algo; ou o médio...
verbo amar: Quem ama, ama alguém. E) ...impulsionado pelo aumento do número de
universidades...
- Quem ............ , ................. + prep. + algo/alguém:
Todo verbo que se encaixar nessa frase será TRANSITIVO 02.(Agente de Defensoria Pública – FCC – 2013). Donos
INDIRETO. Por exemplo, o verbo gostar: Quem gosta, gosta de uma capacidade de orientação nas brenhas selvagens [...],
de algo ou de alguém. As preposições mais comuns são as sabiam os paulistas como...
seguintes: a, de, em, por, para, sem e com. O segmento em destaque na frase acima exerce a
mesma função sintática que o elemento grifado em:
- Quem ............ , ................. algo/alguém + prep. + algo/ A) Nas expedições breves serviam de balizas ou
alguém: Todo verbo que se encaixar nessa frase será mostradores para a volta.
TRANSITIVO DIRETO E INDIRETO - também denominado B) Às estreitas veredas e atalhos [...], nada acrescentariam
de BITRANSITIVO. Por exemplo, o verbo mostrar: Quem aqueles de considerável...
mostra, mostra algo a alguém; ou o verbo informar: Quem C) Só a um olhar muito exercitado seria perceptível o
informa, informa alguém de algo ou Quem informa, sinal.
informa algo a alguém. D) Uma sequência de tais galhos, em qualquer
floresta, podia significar uma pista.
É importante salientar que um verbo só será E) Alguns mapas e textos do século XVII apresentam-
TRANSITIVO se houver complemento (objeto direto ou nos a vila de São Paulo como centro...
objeto indireto). A análise de um verbo depende, portanto,
do ambiente sintático em que ele se encontra. Um verbo 03. Há complemento nominal em:
que aparentemente seja transitivo direto pode ser, na A)Você devia vir cá fora receber o beijo da madrugada.
realidade, intransitivo, caso não haja complemento. Por B)... embora fosse quase certa a sua possibilidade de
exemplo, observe a seguinte frase: ganhar a vida.
O pior cego é aquele que não quer ver. C)Ela estava na janela do edifício.
O verbo “ver” é, aparentemente, transitivo direto, uma D)... sem saber ao certo se gostávamos dele.
vez que se encaixa na frase Quem vê, vê algo. Ocorre, E)Pouco depois começaram a brincar de bandido e
porém, que não há o “algo”. O pior cego é aquele que não mocinho de cinema.
quer ver o quê? Não aparece na oração; não há, portanto,
o objeto direto. Como não o há, o verbo não pode ser 04. (ESPM-SP) Em “esta lhe deu cem mil contos”, o termo
transitivo direto, e sim intransitivo. destacado é:
Observe, agora, esta frase: Quem dá aos pobres, A) pronome possessivo
empresta a Deus. B) complemento nominal
Os verbos “dar” e “emprestar” são, aparentemente, C) objeto indireto
transitivos diretos e indiretos, uma vez que se encaixam D) adjunto adnominal
nas frases Quem dá, dá algo a alguém e Quem empresta, E) objeto direto
empresta algo a alguém. Ocorre, porém, que não há
o “algo”. Quem dá o que aos pobres empresta o que a 05. Assinale a alternativa correta e identifique o sujeito
Deus? Não aparece na oração; não há, portanto, o objeto das seguintes orações em relação aos verbos destacados:
direto. Como não o há, os verbos não podem ser transitivos - Amanhã teremos uma palestra sobre qualidade de
diretos e indiretos, e sim somente transitivos indiretos. vida.
- Neste ano, quero prestar serviço voluntário.
FONTE: http://www.gramaticaonline.com.br/texto/1231
A)Tu – vós
B)Nós – eu
C)Vós – nós
D) Ele - tu

65
LÍNGUA PORTUGUESA

06. Classifique o sujeito das orações destacadas no 4-) esta lhe deu cem mil contos = o verbo DAR é
texto seguinte e, a seguir, assinale a sequência correta. bitransitivo, ou seja, transitivo direto e indireto, portanto
É notável, nos textos épicos, a participação do precisa de dois complementos – dois objetos: direto e
sobrenatural. É frequente a mistura de assuntos relativos ao indireto.
nacionalismo com o caráter maravilhoso. Nas epopeias, Deu o quê? = cem mil contos (direto)
os deuses tomam partido e interferem nas aventuras dos Deu a quem? lhe (=a ele, a ela) = indireto
heróis, ajudando-os ou atrapalhando- -os.
A)simples, composto 5-) - Amanhã ( nós ) teremos uma palestra sobre
B)indeterminado, composto qualidade de vida.
C)simples, simples - Neste ano, ( eu ) quero prestar serviço voluntário.
D) oculto, indeterminado
6-) É notável, nos textos épicos, a participação do
07. (ESPM-SP) “Surgiram fotógrafos e repórteres”. sobrenatural. É frequente a mistura de assuntos relativos
Identifique a alternativa que classifica corretamente ao nacionalismo com o caráter maravilhoso. Nas epopeias,
a função sintática e a classe morfológica dos termos os deuses tomam partido e interferem nas aventuras dos
destacados: heróis, ajudando-os ou atrapalhando-os.
A) objeto indireto – substantivo Ambos os termos apresentam sujeito simples
B) objeto direto - substantivo
C) sujeito – adjetivo 7-) Surgiram fotógrafos e repórteres.
D) objeto direto – adjetivo O sujeito está deslocado, colocado na ordem indireta
E) sujeito - substantivo (final da oração). Portanto: função sintática: sujeito
(composto); classe morfológica (classe de palavras):
GABARITO substantivos.
01. C 02. D 03. B 04. C 05. B 06. C 07. E Períodos Compostos
RESOLUÇÃO
O período composto caracteriza-se por possuir mais de
uma oração em sua composição. Sendo Assim:
1-) Os trabalhadores passaram mais tempo na escola
- Eu irei à praia. (Período Simples = um verbo, uma
= SUJEITO
oração)
A) ...o que reduz a média de ganho da categoria. =
- Estou comprando um protetor solar, depois irei à praia.
objeto direto
(Período Composto =locução verbal, verbo, duas orações)
B) ...houve mais ofertas de trabalhadores dessa classe.
= objeto direto - Já me decidi: só irei à praia, se antes eu comprar
C) O crescimento da escolaridade também foi um protetor solar. (Período Composto = três verbos, três
impulsionado... = sujeito paciente orações).
D) ...elevando a fatia dos brasileiros com ensino médio... Cada verbo ou locução verbal sublinhada acima
= objeto direto corresponde a uma oração. Isso implica que o primeiro
E) ...impulsionado pelo aumento do número de exemplo é um período simples, pois tem apenas uma
universidades... = agente da passiva oração, os dois outros exemplos são períodos compostos,
pois têm mais de uma oração.
2-) Donos de uma capacidade de orientação nas brenhas Há dois tipos de relações que podem se estabelecer
selvagens [...], sabiam os paulistas como... = SUJEITO entre as orações de um período composto: uma relação de
A) Nas expedições breves = ADJUNTO ADVERBIAL coordenação ou uma relação de subordinação.
B) nada acrescentariam aqueles de considerável...= Duas orações são coordenadas quando estão juntas
adjunto adverbial em um mesmo período (ou seja, em um mesmo bloco
C) seria perceptível o sinal. = predicativo de informações, marcado pela pontuação final), mas têm,
D) Uma sequência de tais galhos = sujeito ambas, estruturas individuais, como é o exemplo de:
E) apresentam-nos a vila de São Paulo como = objeto - Estou comprando um protetor solar, depois irei à praia.
direto (Período Composto)
Podemos dizer:
3-) 1. Estou comprando um protetor solar.
A) o beijo da madrugada. = adjunto adnominal 2. Irei à praia.
B)a sua possibilidade de ganhar a vida. = complemento Separando as duas, vemos que elas são independentes.
nominal (possibilidade de quê?) É esse tipo de período que veremos: o Período
C)na janela do edifício. = adjunto adnominal Composto por Coordenação.
D)... sem saber ao certo se gostávamos dele. = objeto Quanto à classificação das orações coordenadas, temos
indireto dois tipos: Coordenadas Assindéticas e Coordenadas
E) a brincar de bandido e mocinho de cinema = objeto Sindéticas.
indireto

66
LÍNGUA PORTUGUESA

Coordenadas Assindéticas Questões sobre Orações Coordenadas

São orações coordenadas entre si e que não são ligadas 01. A oração “Não se verificou, todavia, uma
através de nenhum conectivo. Estão apenas justapostas. transplantação integral de gosto e de estilo” tem valor:
A) conclusivo
B) adversativo
Coordenadas Sindéticas C) concessivo
D) explicativo
Ao contrário da anterior, são orações coordenadas E) alternativo
entre si, mas que são ligadas através de uma conjunção
coordenativa. Esse caráter vai trazer para esse tipo de oração 02. “Estudamos, logo deveremos passar nos exames”. A
uma classificação. As orações coordenadas sindéticas oração em destaque é:
são classificadas em cinco tipos: aditivas, adversativas, a) coordenada explicativa
alternativas, conclusivas e explicativas. b) coordenada adversativa
c) coordenada aditiva
Orações Coordenadas Sindéticas Aditivas: suas d) coordenada conclusiva
principais conjunções são: e, nem, não só... mas também, e) coordenada assindética
não só... como, assim... como.
- Não só cantei como também dancei. 03. (Agente Educacional – VUNESP – 2013-adap.) Releia
- Nem comprei o protetor solar, nem fui à praia. o seguinte trecho:
- Comprei o protetor solar e fui à praia. Joyce e Mozart são ótimos, mas eles, como quase toda a
cultura humanística, têm pouca relevância para nossa vida
Orações Coordenadas Sindéticas Adversativas: prática.
suas principais conjunções são: mas, contudo, todavia, Sem que haja alteração de sentido, e de acordo com a
norma- -padrão da língua portuguesa, ao se substituir o
entretanto, porém, no entanto, ainda, assim, senão.
termo em destaque, o trecho estará corretamente reescrito
- Fiquei muito cansada, contudo me diverti bastante.
em:
- Ainda que a noite acabasse, nós continuaríamos
A) Joyce e Mozart são ótimos, portanto eles, como
dançando.
quase toda a cultura humanística, têm pouca relevância
- Não comprei o protetor solar, mas mesmo assim fui à
para nossa vida prática.
praia.
B) Joyce e Mozart são ótimos, conforme eles, como
quase toda a cultura humanística, têm pouca relevância
Orações Coordenadas Sindéticas Alternativas: suas para nossa vida prática.
principais conjunções são: ou... ou; ora...ora; quer...quer; C) Joyce e Mozart são ótimos, assim eles, como quase
seja...seja. toda a cultura humanística, têm pouca relevância para
- Ou uso o protetor solar, ou uso o óleo bronzeador. nossa vida prática.
- Ora sei que carreira seguir, ora penso em várias carreiras D) Joyce e Mozart são ótimos, todavia eles, como quase
diferentes. toda a cultura humanística, têm pouca relevância para nossa
- Quer eu durma quer eu fique acordado, ficarei no vida prática.
quarto. E) Joyce e Mozart são ótimos, pois eles, como quase
toda a cultura humanística, têm pouca relevância para
Orações Coordenadas Sindéticas Conclusivas: suas nossa vida prática.
principais conjunções são: logo, portanto, por fim, por
conseguinte, consequentemente, pois (posposto ao verbo) 04. (Analista Administrativo – VUNESP – 2013-adap.) Em
- Passei no vestibular, portanto irei comemorar. – ...fruto não só do novo acesso da população ao automóvel
- Conclui o meu projeto, logo posso descansar. mas também da necessidade de maior número de viagens...
- Tomou muito sol, consequentemente ficou adoentada. –, os termos em destaque estabelecem relação de
- A situação é delicada; devemos, pois, agir A) explicação.
B) oposição.
Orações Coordenadas Sindéticas Explicativas: suas C) alternância.
principais conjunções são: isto é, ou seja, a saber, na verdade, D) conclusão.
pois (anteposto ao verbo). E) adição.
- Só passei na prova porque me esforcei por muito tempo.
- Só fiquei triste por você não ter viajado comigo. 05. Analise a oração destacada: Não se desespere, que
- Não fui à praia, pois queria descansar durante o estaremos a seu lado sempre.
Domingo. Marque a opção correta quanto à sua classificação:
A) Coordenada sindética aditiva.
Fonte: http://www.infoescola.com/portugues/oracoes- B) Coordenada sindética alternativa.
coordenadas-assindeticas-e-sindeticas/ C) Coordenada sindética conclusiva.
D) Coordenada sindética explicativa.

67
LÍNGUA PORTUGUESA

06. A frase abaixo em que o conectivo E tem valor RESOLUÇÃO


adversativo é:
A) “O gesto é fácil E não ajuda em nada”. 1-) “Não se verificou, todavia, uma transplantação
B )“O que vemos na esquina E nos sinais de trânsito...”. integral de gosto e de estilo” = conjunção adversativa,
C) “..adultos submetem crianças E adolescentes à tarefa portanto: oração coordenada sindética adversativa
de pedir esmola”.
D) “Quem dá esmola nas ruas contribui para a 2-) Estudamos, logo deveremos passar nos exames =
manutenção da miséria E prejudica o desenvolvimento da a oração em destaque não é introduzida por conjunção,
sociedade”. então: coordenada assindética
E) “A vida dessas pessoas é marcada pela falta de
dinheiro, de moradia digna, emprego, segurança, lazer, 3-) Joyce e Mozart são ótimos, mas eles... = conjunção
cultura, acesso à saúde E à educação”. (e ideia) adversativa
A) Joyce e Mozart são ótimos, portanto eles, como
07. Assinale a alternativa em que o sentido da conjunção quase toda a cultura humanística, têm pouca relevância
sublinhada está corretamente indicado entre parênteses. para nossa vida prática. = conclusiva
A) Meu primo formou-se em Direito, porém não B) Joyce e Mozart são ótimos, conforme eles, como
pretende trabalhar como advogado. (explicação) quase toda a cultura humanística, têm pouca relevância
B) Não fui ao cinema nem assisti ao jogo. (adição) para nossa vida prática. = conformativa
C) Você está preparado para a prova; por isso, não se C) Joyce e Mozart são ótimos, assim eles, como quase
preocupe. (oposição) toda a cultura humanística, têm pouca relevância para
D) Vá dormir mais cedo, pois o vestibular será amanhã. nossa vida prática. = conclusiva
(alternância) E) Joyce e Mozart são ótimos, pois eles, como quase
E) Os meninos deviam correr para casa ou apanhariam toda a cultura humanística, têm pouca relevância para
toda a chuva. (conclusão) nossa vida prática. = explicativa
Dica: conjunção pois como explicativa = dá para eu
substituir por porque; como conclusiva: substituo por
portanto.
08. Analise sintaticamente as duas orações destacadas
no texto “O assaltante pulou o muro, mas não penetrou na
4-) fruto não só do novo acesso da população ao
casa, nem assustou seus habitantes.” A seguir, classifique-
automóvel mas também da necessidade de maior número
as, respectivamente, como coordenadas:
de viagens... estabelecem relação de adição de ideias, de
A) adversativa e aditiva.
fatos
B) explicativa e aditiva.
C) adversativa e alternativa. 5-) Não se desespere, que estaremos a seu lado sempre.
D) aditiva e alternativa. = conjunção explicativa (= porque) - coordenada
sindética explicativa
09. Um livro de receita é um bom presente porque ajuda
as pessoas que não sabem cozinhar. A palavra “porque” 6-)
pode ser substituída, sem alteração de sentido, por A) “O gesto é fácil E não ajuda em nada”. = mas não
A) entretanto. ajuda (ideia contrária)
B) então. B )“O que vemos na esquina E nos sinais de trânsito...”.
C) assim. = adição
D) pois. C) “..adultos submetem crianças E adolescentes à tarefa
E) porém. de pedir esmola”. = adição
D) “Quem dá esmola nas ruas contribui para a
10- Na oração “Pedro não joga E NEM ASSISTE”, manutenção da miséria E prejudica o desenvolvimento da
temos a presença de uma oração coordenada que pode sociedade”. = adição
ser classificada em: E) “A vida dessas pessoas é marcada pela falta de
A) Coordenada assindética; dinheiro, de moradia digna, emprego, segurança, lazer,
B) Coordenada assindética aditiva; cultura, acesso à saúde E à educação”. = adição
C) Coordenada sindética alternativa;
D) Coordenada sindética aditiva. 7-)
A) Meu primo formou-se em Direito, porém não
GABARITO pretende trabalhar como advogado. = adversativa
C) Você está preparado para a prova; por isso, não se
01. B 02. E 03. D 04. E 05. D preocupe. = conclusão
06. A 07. B 08. A 09. D 10. D D) Vá dormir mais cedo, pois o vestibular será amanhã.
= explicativa
E) Os meninos deviam correr para casa ou apanhariam
toda a chuva. = alternativa

68
LÍNGUA PORTUGUESA

8-) - mas não penetrou na casa = conjunção adversativa O garoto perguntou qual era o telefone da
- nem assustou seus habitantes = conjunção aditiva moça.
Oração Subordinada
9-) Um livro de receita é um bom presente porque ajuda Substantiva
as pessoas que não sabem cozinhar.
= conjunção explicativa: pois Não sabemos por que a vizinha se mudou.
Oração Subordinada Substantiva
10-) E NEM ASSISTE= conjunção aditiva (ideia de adição,
soma de fatos) = Coordenada sindética aditiva. Classificação das Orações Subordinadas
Substantivas

Observe o exemplo abaixo de Vinícius de Moraes: De acordo com a função que exerce no período, a
oração subordinada substantiva pode ser:
“Eu sinto que em meu gesto existe o
teu gesto.” a) Subjetiva
Oração Principal Oração Subordinada É subjetiva quando exerce a função sintática de sujeito
do verbo da oração principal. Observe:
Observe que na oração subordinada temos o verbo
“existe”, que está conjugado na terceira pessoa do singular É fundamental o seu comparecimento à
do presente do indicativo. As orações subordinadas reunião.
que apresentam verbo em qualquer dos tempos finitos Sujeito
(tempos do modo do indicativo, subjuntivo e imperativo),
são chamadas de orações desenvolvidas ou explícitas. É fundamental que você compareça à reunião.
Podemos modificar o período acima. Veja: Oração Principal Oração Subordinada Substantiva
Subjetiva
Eu sinto existir em meu gesto o teu gesto.
Oração Principal Oração Subordinada Atenção: Observe que a oração subordinada substantiva
pode ser substituída pelo pronome “ isso”. Assim, temos
A análise das orações continua sendo a mesma: “Eu um período simples:
sinto” é a oração principal, cujo objeto direto é a oração É fundamental isso. ou Isso é fundamental.
subordinada “existir em meu gesto o teu gesto”. Note que
a oração subordinada apresenta agora verbo no infinitivo. Dessa forma, a oração correspondente a “isso” exercerá
Além disso, a conjunção “que”, conectivo que unia as duas a função de sujeito
orações, desapareceu. As orações subordinadas cujo verbo Veja algumas estruturas típicas que ocorrem na oração
surge numa das formas nominais (infinitivo - flexionado principal:
ou não -, gerúndio ou particípio) chamamos orações 1- Verbos de ligação + predicativo, em construções
reduzidas ou implícitas. do tipo: É bom - É útil - É conveniente - É certo - Parece certo
- É claro - Está evidente - Está comprovado
Obs.: as orações reduzidas não são introduzidas É bom que você compareça à minha festa.
por conjunções nem pronomes relativos. Podem ser,
eventualmente, introduzidas por preposição. 2- Expressões na voz passiva, como: Sabe-se - Soube-se
- Conta-se - Diz-se - Comenta-se - É sabido - Foi anunciado
1) ORAÇÕES SUBORDINADAS SUBSTANTIVAS - Ficou provado
Sabe-se que Aline não gosta de Pedro.
A oração subordinada substantiva tem valor de
substantivo e vem introduzida, geralmente, por conjunção 3- Verbos como: convir - cumprir - constar - admirar -
integrante (que, se). importar - ocorrer - acontecer
Convém que não se atrase na entrevista.
Suponho que você foi à biblioteca hoje. Obs.: quando a oração subordinada substantiva é
Oração Subordinada Substantiva subjetiva, o verbo da oração principal está sempre na 3ª.
pessoa do singular.
Você sabe se o presidente já chegou?
Oração Subordinada Substantiva b) Objetiva Direta
A oração subordinada substantiva objetiva direta exerce
Os pronomes interrogativos (que, quem, qual) também função de objeto direto do verbo da oração principal.
introduzem as orações subordinadas substantivas, bem
como os advérbios interrogativos (por que, quando, onde, Todos querem sua aprovação no concurso.
como). Veja os exemplos: Objeto Direto

69
LÍNGUA PORTUGUESA

Todos querem que você seja aprovado. (= Todos e) Predicativa


querem isso) A oração subordinada substantiva predicativa exerce
Oração Principal oração Subordinada Substantiva papel de predicativo do sujeito do verbo da oração principal e
Objetiva Direta vem sempre depois do verbo ser.
Nosso desejo era sua desistência.
As orações subordinadas substantivas objetivas diretas Predicativo do Sujeito
desenvolvidas são iniciadas por:
Nosso desejo era que ele desistisse. (= Nosso desejo era
- Conjunções integrantes “que” (às vezes elíptica) e “se”: isso)
A professora verificou se todos alunos estavam presentes. Oração Subordinada Substantiva
Predicativa
- Pronomes indefinidos que, quem, qual, quanto (às
vezes regidos de preposição), nas interrogações indiretas: Obs.: em certos casos, usa-se a preposição expletiva “de”
O pessoal queria saber quem era o dono do carro para realce. Veja o exemplo: A impressão é de que não fui bem
importado. na prova.

- Advérbios como, quando, onde, por que, quão (às f) Apositiva


vezes regidos de preposição), nas interrogações indiretas: A oração subordinada substantiva apositiva exerce função
Eu não sei por que ela fez isso. de aposto de algum termo da oração principal.
Fernanda tinha um grande sonho: a chegada do dia de seu
c) Objetiva Indireta casamento. Aposto
A oração subordinada substantiva objetiva indireta atua (Fernanda tinha um grande sonho: isso.)
como objeto indireto do verbo da oração principal. Vem
precedida de preposição. Fernanda tinha um grande sonho: que o dia do seu
casamento chegasse.
Meu pai insiste em meu estudo. Oração Subordinada Substantiva Apositiva
Objeto Indireto
2) ORAÇÕES SUBORDINADAS ADJETIVAS
Meu pai insiste em que eu estude. (= Meu pai insiste
nisso) Uma oração subordinada adjetiva é aquela que possui valor
Oração Subordinada Substantiva Objetiva e função de adjetivo, ou seja, que a ele equivale. As orações
Indireta vêm introduzidas por pronome relativo e exercem a função de
adjunto adnominal do antecedente. Observe o exemplo:
Obs.: em alguns casos, a preposição pode estar elíptica
na oração. Esta foi uma redação bem-sucedida.
Marta não gosta (de) que a chamem de senhora. Substantivo Adjetivo (Adjunto Adnominal)
Oração Subordinada Substantiva Objetiva
Indireta Note que o substantivo redação foi caracterizado pelo
adjetivo bem-sucedida. Nesse caso, é possível formarmos outra
d) Completiva Nominal construção, a qual exerce exatamente o mesmo papel. Veja:
A oração subordinada substantiva completiva nominal
completa um nome que pertence à oração principal e Esta foi uma redação que fez sucesso.
também vem marcada por preposição. Oração Principal Oração Subordinada Adjetiva
Sentimos orgulho de seu comportamento.
Complemento Nominal Perceba que a conexão entre a oração subordinada adjetiva
e o termo da oração principal que ela modifica é feita pelo
de que você se comportou. (= pronome relativo “que”. Além de conectar (ou relacionar) duas
Sentimos orgulho disso.) orações, o pronome relativo desempenha uma função sintática
Oração Subordinada Substantiva Completiva na oração subordinada: ocupa o papel que seria exercido pelo
Nominal termo que o antecede.
Obs.: para que dois períodos se unam num período
Lembre-se: as orações subordinadas substantivas composto, altera-se o modo verbal da segunda oração.
objetivas indiretas integram o sentido de um verbo,
enquanto que orações subordinadas substantivas Atenção: Vale lembrar um recurso didático para reconhecer
completivas nominais integram o sentido de um nome. o pronome relativo “que”: ele sempre pode ser substituído por:
Para distinguir uma da outra, é necessário levar em conta o qual - a qual - os quais - as quais
o termo complementado. Essa é, aliás, a diferença entre Refiro-me ao aluno que é estudioso.
o objeto indireto e o complemento nominal: o primeiro Essa oração é equivalente a:
complementa um verbo, o segundo, um nome. Refiro-me ao aluno o qual estuda.

70
LÍNGUA PORTUGUESA

Forma das Orações Subordinadas Adjetivas 3) ORAÇÕES SUBORDINADAS ADVERBIAIS

Quando são introduzidas por um pronome relativo Uma oração subordinada adverbial é aquela que exerce
e apresentam verbo no modo indicativo ou subjuntivo, a função de adjunto adverbial do verbo da oração principal.
as orações subordinadas adjetivas são chamadas Dessa forma, pode exprimir circunstância de tempo, modo,
desenvolvidas. Além delas, existem as orações subordinadas fim, causa, condição, hipótese, etc. Quando desenvolvida,
adjetivas reduzidas, que não são introduzidas por pronome vem introduzida por uma das conjunções subordinativas
relativo (podem ser introduzidas por preposição) e (com exclusão das integrantes). Classifica-se de acordo com
apresentam o verbo numa das formas nominais (infinitivo, a conjunção ou locução conjuntiva que a introduz.
gerúndio ou particípio).
Ele foi o primeiro aluno que se apresentou. Durante a madrugada, eu olhei você dormindo.
Ele foi o primeiro aluno a se apresentar. Oração Subordinada Adverbial

No primeiro período, há uma oração subordinada Observe que a oração em destaque agrega uma
adjetiva desenvolvida, já que é introduzida pelo pronome circunstância de tempo. É, portanto, chamada de oração
relativo “que” e apresenta verbo conjugado no pretérito subordinada adverbial temporal. Os adjuntos adverbiais são
perfeito do indicativo. No segundo, há uma oração termos acessórios que indicam uma circunstância referente,
subordinada adjetiva reduzida de infinitivo: não há via de regra, a um verbo. A classificação do adjunto
pronome relativo e seu verbo está no infinitivo. adverbial depende da exata compreensão da circunstância
que exprime. Observe os exemplos abaixo:
Classificação das Orações Subordinadas Adjetivas Naquele momento, senti uma das maiores emoções de
minha vida.
Na relação que estabelecem com o termo que Quando vi a estátua, senti uma das maiores emoções de
caracterizam, as orações subordinadas adjetivas podem minha vida.
atuar de duas maneiras diferentes. Há aquelas que
restringem ou especificam o sentido do termo a que No primeiro período, “naquele momento” é um adjunto
se referem, individualizando-o. Nessas orações não há adverbial de tempo, que modifica a forma verbal “senti”. No
marcação de pausa, sendo chamadas subordinadas segundo período, esse papel é exercido pela oração “Quando
adjetivas restritivas. Existem também orações que realçam vi a estátua”, que é, portanto, uma oração subordinada
um detalhe ou amplificam dados sobre o antecedente, que já adverbial temporal. Essa oração é desenvolvida, pois é
se encontra suficientemente definido, as quais denominam- introduzida por uma conjunção subordinativa (quando) e
se subordinadas adjetivas explicativas. apresenta uma forma verbal do modo indicativo (“vi”, do
Exemplo 1: pretérito perfeito do indicativo). Seria possível reduzi-la,
Jamais teria chegado aqui, não fosse a gentileza de um obtendo-se:
homem que passava naquele momento. Ao ver a estátua, senti uma das maiores emoções de
Oração Subordinada Adjetiva minha vida.
Restritiva
A oração em destaque é reduzida, pois apresenta uma
Nesse período, observe que a oração em destaque das formas nominais do verbo (“ver” no infinitivo) e não é
restringe e particulariza o sentido da palavra “homem”: introduzida por conjunção subordinativa, mas sim por uma
trata-se de um homem específico, único. A oração limita preposição (“a”, combinada com o artigo “o”).
o universo de homens, isto é, não se refere a todos os Obs.: a classificação das orações subordinadas adverbiais
homens, mas sim àquele que estava passando naquele é feita do mesmo modo que a classificação dos adjuntos
momento. adverbiais. Baseia-se na circunstância expressa pela oração.
Exemplo 2:
O homem, que se considera racional, muitas vezes age Circunstâncias Expressas
animalescamente. pelas Orações Subordinadas Adverbiais
Oração Subordinada Adjetiva Explicativa a) Causa
A ideia de causa está diretamente ligada àquilo que
Nesse período, a oração em destaque não tem sentido provoca um determinado fato, ao motivo do que se declara
restritivo em relação à palavra “homem”; na verdade, essa na oração principal. “É aquilo ou aquele que determina um
oração apenas explicita uma ideia que já sabemos estar acontecimento”.
contida no conceito de “homem”. Principal conjunção subordinativa causal: PORQUE
Saiba que: Outras conjunções e locuções causais: como (sempre
A oração subordinada adjetiva explicativa é separada introduzido na oração anteposta à oração principal), pois,
da oração principal por uma pausa, que, na escrita, é pois que, já que, uma vez que, visto que.
representada pela vírgula. É comum, por isso, que a As ruas ficaram alagadas porque a chuva foi muito forte.
pontuação seja indicada como forma de diferenciar as Como ninguém se interessou pelo projeto, não houve
orações explicativas das restritivas; de fato, as explicativas alternativa a não ser cancelá-lo.
vêm sempre isoladas por vírgulas; as restritivas, não. Já que você não vai, eu também não vou.

71
LÍNGUA PORTUGUESA

b) Consequência e) Comparação
As orações subordinadas adverbiais consecutivas As orações subordinadas adverbiais comparativas
exprimem um fato que é consequência, que é efeito do estabelecem uma comparação com a ação indicada pelo
que se declara na oração principal. São introduzidas pelas verbo da oração principal.
conjunções e locuções: que, de forma que, de sorte que, Principal conjunção subordinativa comparativa: COMO
tanto que, etc., e pelas estruturas tão...que, tanto...que, Ele dorme como um urso.
tamanho...que.
Principal conjunção subordinativa consecutiva: QUE Saiba que: É comum a omissão do verbo nas orações
(precedido de tal, tanto, tão, tamanho) subordinadas adverbiais comparativas. Por exemplo:
É feio que dói. (É tão feio que, em consequência, causa Agem como crianças. (agem)
dor.) Oração Subordinada Adverbial Comparativa
Nunca abandonou seus ideais, de sorte que acabou No entanto, quando se comparam ações diferentes,
concretizando-os. isso não ocorre. Por exemplo: Ela fala mais do que faz.
Não consigo ver televisão sem bocejar. (Oração Reduzida (comparação do verbo falar e do verbo fazer).
de Infinitivo)
f) Conformidade
c) Condição As orações subordinadas adverbiais conformativas
Condição é aquilo que se impõe como necessário para indicam ideia de conformidade, ou seja, exprimem uma
a realização ou não de um fato. As orações subordinadas regra, um modelo adotado para a execução do que se
adverbiais condicionais exprimem o que deve ou não declara na oração principal.
ocorrer para que se realize ou deixe de se realizar o fato Principal conjunção subordinativa conformativa:
expresso na oração principal. CONFORME
Principal conjunção subordinativa condicional: SE Outras conjunções conformativas: como, consoante e
Outras conjunções condicionais: caso, contanto que, segundo (todas com o mesmo valor de conforme).
Fiz o bolo conforme ensina a receita.
desde que, salvo se, exceto se, a não ser que, a menos que,
Consoante reza a Constituição, todos os cidadãos têm
sem que, uma vez que (seguida de verbo no subjuntivo).
direitos iguais.
Se o regulamento do campeonato for bem elaborado,
certamente o melhor time será campeão.
g) Finalidade
Uma vez que todos aceitem a proposta, assinaremos o
As orações subordinadas adverbiais finais indicam a
contrato.
intenção, a finalidade daquilo que se declara na oração
Caso você se case, convide-me para a festa.
principal.
Principal conjunção subordinativa final: A FIM DE QUE
d) Concessão Outras conjunções finais: que, porque (= para que) e a
As orações subordinadas adverbiais concessivas locução conjuntiva para que.
indicam concessão às ações do verbo da oração principal, Aproximei-me dela a fim de que ficássemos amigos.
isto é, admitem uma contradição ou um fato inesperado. A Felipe abriu a porta do carro para que sua namorada
ideia de concessão está diretamente ligada ao contraste, à entrasse.
quebra de expectativa.
Principal conjunção subordinativa concessiva: EMBORA h) Proporção
Utiliza-se também a conjunção: conquanto e as locuções As orações subordinadas adverbiais proporcionais
ainda que, ainda quando, mesmo que, se bem que, posto exprimem ideia de proporção, ou seja, um fato simultâneo
que, apesar de que. ao expresso na oração principal.
Só irei se ele for. Principal locução conjuntiva subordinativa proporcional:
À PROPORÇÃO QUE
A oração acima expressa uma condição: o fato de “eu” ir Outras locuções conjuntivas proporcionais: à medida
só se realizará caso essa condição seja satisfeita. Compare que, ao passo que. Há ainda as estruturas: quanto maior...
agora com: (maior), quanto maior...(menor), quanto menor...(maior),
Irei mesmo que ele não vá. quanto menor...(menor), quanto mais...(mais), quanto mais...
(menos), quanto menos...(mais), quanto menos...(menos).
A distinção fica nítida; temos agora uma concessão: À proporção que estudávamos, acertávamos mais
irei de qualquer maneira, independentemente de sua ida. questões.
A oração destacada é, portanto, subordinada adverbial Visito meus amigos à medida que eles me convidam.
concessiva. Observe outros exemplos: Quanto maior for a altura, maior será o tombo.
Embora fizesse calor, levei agasalho.
Conquanto a economia tenha crescido, pelo menos i) Tempo
metade da população continua à margem do mercado de As orações subordinadas adverbiais temporais
consumo. acrescentam uma ideia de tempo ao fato expresso na oração
Foi aprovado sem estudar (= sem que estudasse / principal, podendo exprimir noções de simultaneidade,
embora não estudasse). (reduzida de infinitivo) anterioridade ou posterioridade.

72
LÍNGUA PORTUGUESA

Principal conjunção subordinativa temporal: QUANDO As expressões mais denso e menos trânsito, no título,
Outras conjunções subordinativas temporais: enquanto, estabelecem entre si uma relação de
mal e locuções conjuntivas: assim que, logo que, todas as (A) comparação e adição.
vezes que, antes que, depois que, sempre que, desde que, etc. (B) causa e consequência.
Quando você foi embora, chegaram outros convidados. (C) conformidade e negação.
Sempre que ele vem, ocorrem problemas. (D) hipótese e concessão.
Mal você saiu, ela chegou. (E) alternância e explicação
Terminada a festa, todos se retiraram. (= Quando
terminou a festa) (Oração Reduzida de Particípio) 02. (Agente de Escolta e Vigilância Penitenciária –
VUNESP – 2013). No trecho – Tem surtido um efeito positivo
Fonte: http://www.soportugues.com.br/secoes/sint/ por eles se tornarem uma referência positiva dentro da
sint29.php unidade, já que cumprem melhor as regras, respeitam o
próximo e pensam melhor nas suas ações, refletem antes
Questões sobre Orações Subordinadas de tomar uma atitude. – o termo em destaque estabelece
entre as orações uma relação de
01. (Papiloscopista Policial – Vunesp/2013). A) condição.
B) causa.
Mais denso, menos trânsito C) comparação.
D) tempo.
As grandes cidades brasileiras estão congestionadas e E) concessão.
em processo de deterioração agudizado pelo crescimento
econômico da última década. Existem deficiências evidentes 03. (UFV-MG) As orações subordinadas substantivas
em infraestrutura, mas é importante também considerar o que aparecem nos períodos abaixo são todas subjetivas,
planejamento urbano. exceto:
Muitas grandes cidades adotaram uma abordagem de A) Decidiu-se que o petróleo subiria de preço.
desconcentração, incentivando a criação de diversos centros B) É muito bom que o homem, vez por outra, reflita
urbanos, na visão de que isso levaria a uma maior facilidade sobre sua vida.
de deslocamento. C) Ignoras quanto custou meu relógio?
Mas o efeito tem sido o inverso. A criação de diversos D) Perguntou-se ao diretor quando seríamos recebidos.
centros e o aumento das distâncias multiplicam o número de E) Convinha-nos que você estivesse presente à reunião
viagens, dificultando o investimento em transporte coletivo e
aumentando a necessidade do transporte individual. 04. (Agente de Vigilância e Recepção – VUNESP – 2013).
Se olharmos Los Angeles como a região que levou a Considere a tirinha em que se vê Honi conversando com
desconcentração ao extremo, ficam claras as consequências. seu Namorado Lute.
Numa região rica como a Califórnia, com enorme
investimento viário, temos engarrafamentos gigantescos que
viraram característica da cidade.
Os modelos urbanos bem-sucedidos são aqueles com
elevado adensamento e predominância do transporte
coletivo, como mostram Manhattan e Tóquio.
O centro histórico de São Paulo é a região da cidade
mais bem servida de transporte coletivo, com infraestrutura
de telecomunicação, água, eletricidade etc. Como em outras
grandes cidades, essa deveria ser a região mais adensada da
metrópole. Mas não é o caso. Temos, hoje, um esvaziamento
gradual do centro, com deslocamento das atividades para
diversas regiões da cidade.
A visão de adensamento com uso abundante de
transporte coletivo precisa ser recuperada. Desse modo, será
possível reverter esse processo de uso cada vez mais intenso
do transporte individual, fruto não só do novo acesso da
população ao automóvel, mas também da necessidade de
maior número de viagens em função da distância cada vez
maior entre os destinos da população.
(Henrique Meirelles, Folha de S.Paulo, 13.01.2013.
Adaptado)
(Dik Browne, Folha de
S. Paulo, 26.01.2013)

73
LÍNGUA PORTUGUESA

É correto afirmar que a expressão contanto que 08. (Analista em Planejamento, Orçamento e Finanças
estabelece entre as orações relação de Públicas – VUNESP – 2013-adap.) No trecho – “Fio,
A) causa, pois Honi quer ter filhos e não deseja trabalhar disjuntor, tomada, tudo!”, insiste o motorista, com tanto
depois de casada. orgulho que chega a contaminar-me. –, a construção tanto
B) comparação, pois o namorado espera ter sucesso como ... que estabelece entre as construções [com tanto orgulho]
cantor romântico. e [que chega a contaminar-me] uma relação de
C) tempo, pois ambos ainda são adolescentes, mas já A) condição e finalidade.
pensam em casamento. B) conformidade e proporção.
D) condição, pois Lute sabe que exercendo a profissão de C) finalidade e concessão.
músico provavelmente ganhará pouco. D) proporção e comparação.
E) finalidade, pois Honi espera que seu futuro marido E) causa e consequência.
torne-se um artista famoso.
09. “Os Estados Unidos são considerados hoje um país
05. (Analista Administrativo – VUNESP – 2013). Em – bem mais fechado – embora em doze dias recebam o
Apesar da desconcentração e do aumento da extensão mesmo número de imigrantes que o Brasil em um ano.”
urbana verificados no Brasil, é importante desenvolver e A alternativa que substitui a expressão em negrito, sem
adensar ainda mais os diversos centros já existentes... –, sem prejuízo ao conteúdo, é:
que tenha seu sentido alterado, o trecho em destaque está A) já que.
corretamente reescrito em: B) todavia.
A) Mesmo com a desconcentração e o aumento C) ainda que.
da Extensão urbana verificados no Brasil, é importante D) entretanto.
desenvolver e adensar ainda mais os diversos centros já E) talvez.
existentes...
B) Uma vez que se verifica a desconcentração e o aumento 10. (Escrevente TJ SP – Vunesp – 2013) Assinale a
da extensão urbana no Brasil, é importante desenvolver e alternativa que substitui o trecho em destaque na frase
adensar ainda mais os diversos centros já existentes...
– Assinarei o documento, contanto que garantam sua
C) Assim como são verificados a desconcentração e
autenticidade. – sem que haja prejuízo de sentido.
o aumento da extensão urbana no Brasil, é importante
(A) desde que garantam sua autenticidade.
desenvolver e adensar ainda mais os diversos centros já
(B) no entanto garantam sua autenticidade.
existentes...
(C) embora garantam sua autenticidade.
D) Visto que com a desconcentração e o aumento
(D) portanto garantam sua autenticidade.
da extensão urbana verificados no Brasil, é importante
(E) a menos que garantam sua autenticidade.
desenvolver e adensar ainda mais os diversos centros já
existentes...
E) De maneira que, com a desconcentração e o aumento GABARITO
da extensão urbana verificados no Brasil, é importante
desenvolver e adensar ainda mais os diversos centros já 01. B 02. B 03. C 04. D 05. A
existentes... 06. C 07. D 08. E 09. C 10. A

06. (Analista Administrativo – VUNESP – 2013). Em – É RESOLUÇÃO


fundamental que essa visão de adensamento com uso
abundante de transporte coletivo seja recuperada para que 1-) mais denso e menos trânsito = mais denso,
possamos reverter esse processo de uso… –, a expressão em consequentemente, menos trânsito, então: causa e
destaque estabelece entre as orações relação de consequência
A) consequência.
B) condição. 2-) já que cumprem melhor as regras = estabelece
C) finalidade. entre as orações uma relação de causa com a consequência
D) causa. de “tem um efeito positivo”.
E) concessão. 3-) Ignoras quanto custou meu relógio? = oração
subordinada substantiva objetiva direta
07. (Analista de Sistemas – VUNESP – 2013 – adap.). A oração não atende aos requisitos de tais orações, ou
Considere o trecho: “Como as músicas eram de protesto, seja, não se inicia com verbo de ligação, tampouco pelos
naquele mesmo ano foi enquadrado na lei de segurança verbos “convir”, “parecer”, “importar”, “constar” etc., e
nacional pela ditadura militar e exilado.” O termo Como, em também não inicia com as conjunções integrantes “que”
destaque na primeira parte do enunciado, expressa ideia de e “se”.
A) contraste e tem sentido equivalente a porém.
B) concessão e tem sentido equivalente a mesmo que. 4-) a expressão contanto que estabelece uma relação
C) conformidade e tem sentido equivalente a conforme. de condição (condicional)
D) causa e tem sentido equivalente a visto que.
E) finalidade e tem sentido equivalente a para que.

74
LÍNGUA PORTUGUESA

5-) Apesar da desconcentração e do aumento da extensão Como podemos observar, os vocábulos possuem mais
urbana verificados no Brasil = conjunção concessiva de uma sílaba, mas em nossa língua existem aqueles com
B) Uma vez que se verifica a desconcentração e o uma sílaba somente: são os chamados monossílabos que,
aumento da extensão urbana no Brasil, = causal quando pronunciados, apresentam certa diferenciação quan-
C) Assim como são verificados a desconcentração e o to à intensidade.
aumento da extensão urbana no Brasil = comparativa Tal diferenciação só é percebida quando os pronunciamos
D) Visto que com a desconcentração e o aumento da em uma dada sequência de palavras. Assim como podemos
extensão urbana verificados no Brasil = causal observar no exemplo a seguir:
E) De maneira que, com a desconcentração e o aumento
da extensão urbana verificados no Brasil = consecutivas “Sei que não vai dar em nada,
Seus segredos sei de cor”.
6-) para que possamos = conjunção final (finalidade)
Os monossílabos classificam-se como tônicos; os demais,
7-) “Como as músicas eram de protesto = expressa ideia como átonos (que, em, de).
de causa da consequência “foi enquadrado” = causa e tem
sentido equivalente a visto que. Os acentos
acento agudo (´) – Colocado sobre as letras «a», «i», «u» e
8-) com tanto orgulho que chega a contaminar-me. – a sobre o «e» do grupo “em” - indica que estas letras represen-
construção estabelece uma relação de causa e consequência. tam as vogais tônicas de palavras como Amapá, caí, público,
(a causa da “contaminação” – consequência) parabéns. Sobre as letras “e” e “o” indica, além da tonicidade,
timbre aberto.Ex.: herói – médico – céu (ditongos abertos)
9-) Os Estados Unidos são considerados hoje um país acento circunflexo (^) – colocado sobre as letras “a”, “e”
bem mais fechado – embora em doze dias recebam o e “o” indica, além da tonicidade, timbre fechado: Ex.: tâmara
mesmo número de imigrantes que o Brasil em um ano.” = – Atlântico – pêssego – supôs
conjunção concessiva: ainda que
acento grave (`) – indica a fusão da preposição “a” com
artigos e pronomes. Ex.: à – às – àquelas – àqueles
10-) contanto que garantam sua autenticidade. =
trema ( ¨ ) – De acordo com a nova regra, foi totalmente
conjunção condicional = desde que
abolido das palavras. Há uma exceção: é utilizado em palavras
derivadas de nomes próprios estrangeiros. Ex.: mülleriano (de
Müller)
12. ACENTUAÇÃO;
til (~) – indica que as letras “a” e “o” representam vogais
A acentuação é um dos requisitos que perfazem as regras nasais. Ex.: coração – melão – órgão – ímã
estabelecidas pela Gramática Normativa. Esta se compõe de
algumas particularidades, às quais devemos estar atentos, Regras fundamentais:
procurando estabelecer uma relação de familiaridade e,
consequentemente, colocando-as em prática na linguagem Palavras oxítonas:
escrita. Acentuam-se todas as oxítonas terminadas em: “a”, “e”,
À medida que desenvolvemos o hábito da leitura e a prá- “o”, “em”, seguidas ou não do plural(s): Pará – café(s) – cipó(s)
tica de redigir, automaticamente aprimoramos essas com- – armazém(s)
petências, e logo nos adequamos à forma padrão. Essa regra também é aplicada aos seguintes casos:
Monossílabos tônicos terminados em “a”, “e”, “o”, segui-
Regras básicas – Acentuação tônica dos ou não de “s”. Ex.: pá – pé – dó – há
A acentuação tônica implica na intensidade com que são Formas verbais terminadas em “a”, “e”, “o” tônicos, se-
pronunciadas as sílabas das palavras. Aquela que se dá de guidas de lo, la, los, las. Ex. respeitá-lo – percebê-lo – compô-lo
forma mais acentuada, conceitua-se como sílaba tônica. As
demais, como são pronunciadas com menos intensidade, Paroxítonas:
são denominadas de átonas. Acentuam-se as palavras paroxítonas terminadas em:
De acordo com a tonicidade, as palavras são classificadas - i, is : táxi – lápis – júri
como: - us, um, uns : vírus – álbuns – fórum
Oxítonas – São aquelas cuja sílaba tônica recai sobre a - l, n, r, x, ps : automóvel – elétron - cadáver – tórax – fórceps
última sílaba. Ex.: café – coração – cajá – atum – caju – papel - ã, ãs, ão, ãos : ímã – ímãs – órfão – órgãos
Paroxítonas – São aquelas em que a sílaba tônica recai -- Dica da Zê!: Memorize a palavra LINURXÃO. Para quê?
na penúltima sílaba. Ex.: útil – tórax – táxi – leque – retrato – Repare que essa palavra apresenta as terminações das paroxí-
passível tonas que são acentuadas: L, I N, U (aqui inclua UM = fórum),
R, X, Ã, ÃO. Assim ficará mais fácil a memorização!
Proparoxítonas - São aquelas em que a sílaba tônica
está na antepenúltima sílaba. Ex.: lâmpada – câmara – tím- -ditongo oral, crescente ou decrescente, seguido ou não
pano – médico – ônibus de “s”: água – pônei – mágoa – jóquei

75
LÍNGUA PORTUGUESA

Regras especiais: Não se acentuam o “i” e o “u” que formam hiato quan-
do seguidos, na mesma sílaba, de l, m, n, r ou z. Ra-ul, ru-
Os ditongos de pronúncia aberta “ei”, “oi” (ditongos -im, con-tri-bu-in-te, sa-ir, ju-iz
abertos), que antes eram acentuados, perderam o acento
de acordo com a nova regra, mas desde que estejam em Não se acentuam as letras “i” e “u” dos hiatos se esti-
palavras paroxítonas. verem seguidas do dígrafo nh. Ex: ra-i-nha, ven-to-i-nha.

* Cuidado: Se os ditongos abertos estiverem em uma Não se acentuam as letras “i” e “u” dos hiatos se vierem
palavra oxítona (herói) ou monossílaba (céu) ainda são precedidas de vogal idêntica: xi-i-ta, pa-ra-cu-u-ba
acentuados. Ex.: herói, céu, dói, escarcéu.
As formas verbais que possuíam o acento tônico na raiz,
Antes Agora com “u” tônico precedido de “g” ou “q” e seguido de “e” ou
assembléia assembleia “i” não serão mais acentuadas. Ex.:
idéia ideia
geléia geleia Antes Depois
jibóia jiboia apazigúe (apaziguar) apazigue
apóia (verbo apoiar) apoia averigúe (averiguar) averigue
paranóico paranoico argúi (arguir) argui

Quando a vogal do hiato for “i” ou “u” tônicos, acom- Acentuam-se os verbos pertencentes à terceira pessoa do
panhados ou não de “s”, haverá acento. Ex.: saída – faísca plural de: ele tem – eles têm / ele vem – eles vêm (verbo vir)
– baú – país – Luís
A regra prevalece também para os verbos conter, obter,
Observação importante: reter, deter, abster.
ele contém – eles contêm
Não serão mais acentuados “i” e “u” tônicos, formando
ele obtém – eles obtêm
hiato quando vierem depois de ditongo: Ex.:
ele retém – eles retêm
ele convém – eles convêm
Antes Agora
bocaiúva bocaiuva
Não se acentuam mais as palavras homógrafas que an-
feiúra feiura
tes eram acentuadas para diferenciá-las de outras seme-
Sauípe Sauipe
lhantes (regra do acento diferencial). Apenas em algumas
exceções, como:
O acento pertencente aos encontros “oo” e “ee” foi A forma verbal pôde (terceira pessoa do singular do
abolido. Ex.: pretérito perfeito do modo indicativo) ainda continua sen-
do acentuada para diferenciar-se de pode (terceira pessoa
Antes Agora do singular do presente do indicativo). Ex:
crêem creem Ela pode fazer isso agora.
lêem leem Elvis não pôde participar porque sua mão não deixou...
vôo voo
enjôo enjoo O mesmo ocorreu com o verbo pôr para diferenciar da
preposição por.
- Agora memorize a palavra CREDELEVÊ. São os verbos - Quando, na frase, der para substituir o “por” por “co-
que, no plural, dobram o “e”, mas que não recebem mais locar”, estaremos trabalhando com um verbo, portanto:
acento como antes: CRER, DAR, LER e VER. “pôr”; nos outros casos, “por” preposição. Ex:
Faço isso por você.
Repare: Posso pôr (colocar) meus livros aqui?
1-) O menino crê em você
Os meninos creem em você. Questões sobre Acentuação Gráfica
2-) Elza lê bem!
Todas leem bem! 01. (TJ/SP – AGENTE DE FISCALIZAÇÃO JUDICIÁRIA –
3-) Espero que ele dê o recado à sala. VUNESP/2010) Assinale a alternativa em que as palavras
Esperamos que os garotos deem o recado! são acentuadas graficamente pelos mesmos motivos que
4-) Rubens vê tudo! justificam, respectivamente, as acentuações de: década,
Eles veem tudo! relógios, suíços.
(A) flexíveis, cartório, tênis.
* Cuidado! Há o verbo vir: (B) inferência, provável, saída.
Ele vem à tarde! (C) óbvio, após, países.
Eles vêm à tarde! (D) islâmico, cenário, propôs.
(E) república, empresária, graúda.

76
LÍNGUA PORTUGUESA

02. (TRIBUNAL DE JUSTIÇA DO ESTADO DE SÃO PAU- GABARITO


LO - ESCREVENTE TÉCNICO JUDICIÁRIO – VUNESP/2013)
Assinale a alternativa com as palavras acentuadas segundo 01. E 02. D 03. E 04. C 05. E
as regras de acentuação, respectivamente, de intercâmbio 06. C 07. D 08. B 09. E
e antropológico.
(A) Distúrbio e acórdão. RESOLUÇÃO
(B) Máquina e jiló.
(C) Alvará e Vândalo. 1-) Década = proparoxítona / relógios = paroxítona ter-
(D) Consciência e características. minada em ditongo / suíços = regra do hiato
(E) Órgão e órfãs. (A) flexíveis e cartório = paroxítonas terminadas em di-
tongo / tênis = paroxítona terminada em “i” (seguida de “s”)
03. (TRIBUNAL DE JUSTIÇA DO ESTADO DO ACRE – (B) inferência = paroxítona terminada em ditongo / pro-
TÉCNICO EM MICROINFORMÁTICA - CESPE/2012) As pa- vável = paroxítona terminada em “l” / saída = regra do hiato
lavras “conteúdo”, “calúnia” e “injúria” são acentuadas de (C) óbvio = paroxítona terminada em ditongo / após =
acordo com a mesma regra de acentuação gráfica. oxítona terminada em “o” + “s” / países = regra do hiato
( ) CERTO ( ) ERRADO (D) islâmico = proparoxítona / cenário = paroxítona ter-
04. (TRIBUNAL DE JUSTIÇA DO ESTADO DE MINAS GE- minada em ditongo / propôs = oxítona terminada em “o”
RAIS – OFICIAL JUDICIÁRIO – FUNDEP/2010) Assinale a + “s”
afirmativa em que se aplica a mesma regra de acentuação. (E) república = proparoxítona / empresária = paroxítona
A) tevê – pôde – vê terminada em ditongo / graúda = regra do hiato
B) únicas – histórias – saudáveis
C) indivíduo – séria – noticiários 2-) Para que saibamos qual alternativa assinalar, primeiro
D) diário – máximo – satélite temos que classificar as palavras do enunciado quanto à po-
sição de sua sílaba tônica:
05. (ANATEL – TÉCNICO ADMINISTRATIVO – CES- Intercâmbio = paroxítona terminada em ditongo; Antro-
PE/2012) Nas palavras “análise” e “mínimos”, o emprego pológico = proparoxítona (todas são acentuadas). Agora,
do acento gráfico tem justificativas gramaticais diferentes. vamos à análise dos itens apresentados:
(...) CERTO ( ) ERRADO (A) Distúrbio = paroxítona terminada em ditongo; acór-
dão = paroxítona terminada em “ão”
06. (ANCINE – TÉCNICO ADMINISTRATIVO – CES- (B) Máquina = proparoxítona; jiló = oxítona terminada
PE/2012) Os vocábulos “indivíduo”, “diária” e “paciência” em “o”
recebem acento gráfico com base na mesma regra de (C) Alvará = oxítona terminada em “a”; Vândalo = pro-
acentuação gráfica. paroxítona
(...) CERTO ( ) ERRADO (D) Consciência = paroxítona terminada em ditongo; ca-
racterísticas = proparoxítona
07. (BACEN – TÉCNICO DO BANCO CENTRAL – CES- (E) Órgão e órfãs = ambas: paroxítona terminada em “ão”
GRANRIO/2010) As palavras que se acentuam pelas mes- e “ã”, respectivamente.
mas regras de “conferência”, “razoável”, “países” e “será”,
respectivamente, são 3-) “Conteúdo” é acentuada seguindo a regra do hiato;
a) trajetória, inútil, café e baú. calúnia = paroxítona terminada em ditongo; injúria = paro-
b) exercício, balaústre, níveis e sofá. xítona terminada em ditongo.
c) necessário, túnel, infindáveis e só. RESPOSTA: “ERRADO”.
d) médio, nível, raízes e você.
e) éter, hífen, propôs e saída. 4-)
A) tevê – pôde – vê
08. (CORREIOS – CARTEIRO – CESPE/2011) São acentua- Tevê = oxítona terminada em “e”; pôde (pretérito perfei-
dos graficamente de acordo com a mesma regra de acen- to do Indicativo) = acento diferencial (que ainda prevalece
tuação gráfica os vocábulos após o Novo Acordo Ortográfico) para diferenciar de “pode”
A) também e coincidência. – presente do Indicativo; vê = monossílaba terminada em “e”
B) quilômetros e tivéssemos. B) únicas – histórias – saudáveis
C) jogá-la e incrível. Únicas = proparoxítona; história = paroxítona terminada
D) Escócia e nós. em ditongo; saudáveis = paroxítona terminada em ditongo.
E) correspondência e três. C) indivíduo – séria – noticiários
Indivíduo = paroxítona terminada em ditongo; séria =
09. (IBAMA – TÉCNICO ADMINISTRATIVO – CESPE/2012) paroxítona terminada em ditongo; noticiários = paroxítona
As palavras “pó”, “só” e “céu” são acentuadas de acordo terminada em ditongo.
com a mesma regra de acentuação gráfica. D) diário – máximo – satélite
(...) CERTO ( ) ERRADO Diário = paroxítona terminada em ditongo; máximo =
proparoxítona; satélite = proparoxítona.

77
LÍNGUA PORTUGUESA

5-) Análise = proparoxítona / mínimos = proparoxíto-


na. Ambas são acentuadas pela mesma regra (antepenúlti- 13. COLOCAÇÃO PRONOMINAL E VERBAL;
ma sílaba é tônica, “mais forte”).
RESPOSTA: “ERRADO”.

6-) Indivíduo = paroxítona terminada em ditongo; diá- “CARO CANDIDATO, O TÓPICO ACIMA FOI
ria = paroxítona terminada em ditongo; paciência = paro- ABORDADO NO DECORRER DA MATÉRIA”
xítona terminada em ditongo. Os três vocábulos são acen-
tuados devido à mesma regra.
RESPOSTA: “CERTO”.
14. EMPREGO DE PREPOSIÇÃO;
7-) Vamos classificar as palavras do enunciado:
1-) Conferência = paroxítona terminada em ditongo
2-) razoável = paroxítona terminada em “l’ PREPOSIÇÃO
3-) países = regra do hiato
4-) será = oxítona terminada em “a” Preposição é uma palavra invariável que serve para
ligar termos ou orações. Quando esta ligação acontece,
a) trajetória, inútil, café e baú. normalmente há uma subordinação do segundo termo em
Trajetória = paroxítona terminada em ditongo; inútil = pa- relação ao primeiro. As preposições são muito importantes
roxítona terminada em “l’; café = oxítona terminada em “e” na estrutura da língua, pois estabelecem a coesão textual
b) exercício, balaústre, níveis e sofá. e possuem valores semânticos indispensáveis para a com-
Exercício = paroxítona terminada em ditongo; balaústre preensão do texto.
= regra do hiato; níveis = paroxítona terminada em “i + s”;
sofá = oxítona terminada em “a”. Tipos de Preposição
c) necessário, túnel, infindáveis e só. 1. Preposições essenciais: palavras que atuam exclusiva-
Necessário = paroxítona terminada em ditongo; túnel mente como preposições: a, ante, perante, após, até, com,
= paroxítona terminada em “l’; infindáveis = paroxítona contra, de, desde, em, entre, para, por, sem, sob, sobre, trás,
terminada em “i + s”; só = monossílaba terminada em “o”. atrás de, dentro de, para com.
d) médio, nível, raízes e você. 2. Preposições acidentais: palavras de outras classes
Médio = paroxítona terminada em ditongo; nível = pa- gramaticais que podem atuar como preposições: como,
roxítona terminada em “l’; raízes = regra do hiato; será = durante, exceto, fora, mediante, salvo, segundo, senão, visto.
oxítona terminada em “a”. 3. Locuções prepositivas: duas ou mais palavras valendo
e) éter, hífen, propôs e saída. como uma preposição, sendo que a última palavra é uma
Éter = paroxítona terminada em “r”; hífen = paroxítona delas: abaixo de, acerca de, acima de, ao lado de, a respeito
terminada em “n”; propôs = oxítona terminada em “o + s”; de, de acordo com, em cima de, embaixo de, em frente a, ao
saída = regra do hiato. redor de, graças a, junto a, com, perto de, por causa de, por
cima de, por trás de.
8-) A preposição, como já foi dito, é invariável. No entanto
A) também e coincidência. pode unir-se a outras palavras e assim estabelecer concor-
Também = oxítona terminada em “e + m”; coincidência dância em gênero ou em número. Ex: por + o = pelo por
= paroxítona terminada em ditongo + a = pela.
B) quilômetros e tivéssemos. Vale ressaltar que essa concordância não é caracterís-
Quilômetros = proparoxítona; tivéssemos = proparoxí- tica da preposição, mas das palavras às quais ela se une.
tona Esse processo de junção de uma preposição com outra
C) jogá-la e incrível. palavra pode se dar a partir de dois processos:
Oxítona terminada em “a”; incrível = paroxítona termi- 1. Combinação: A preposição não sofre alteração.
nada em “l’ preposição a + artigos definidos o, os
D) Escócia e nós. a + o = ao
Escócia = paroxítona terminada em ditongo; nós = mo- preposição a + advérbio onde
nossílaba terminada em “o + s” a + onde = aonde
E) correspondência e três. 2. Contração: Quando a preposição sofre alteração.
Correspondência = paroxítona terminada em ditongo;
três = monossílaba terminada em “e + s” Preposição + Artigos
De + o(s) = do(s)
9-) Pó = monossílaba terminada em “o”; só = monos- De + a(s) = da(s)
sílaba terminada em “o”; céu = monossílaba terminada em De + um = dum
ditongo aberto “éu”. De + uns = duns
RESPOSTA: “ERRADO”. De + uma = duma
De + umas = dumas

78
LÍNGUA PORTUGUESA

Em + o(s) = no(s) 2. Algumas relações semânticas estabelecidas por meio


Em + a(s) = na(s) das preposições:
Em + um = num Destino = Irei para casa.
Em + uma = numa Modo = Chegou em casa aos gritos.
Em + uns = nuns Lugar = Vou ficar em casa;
Em + umas = numas Assunto = Escrevi um artigo sobre adolescência.
A + à(s) = à(s) Tempo = A prova vai começar em dois minutos.
Por + o = pelo(s) Causa = Ela faleceu de derrame cerebral.
Por + a = pela(s) Fim ou finalidade = Vou ao médico para começar o tra-
tamento.
Preposição + Pronomes Instrumento = Escreveu a lápis.
De + ele(s) = dele(s) Posse = Não posso doar as roupas da mamãe.
De + ela(s) = dela(s) Autoria = Esse livro de Machado de Assis é muito bom.
De + este(s) = deste(s) Companhia = Estarei com ele amanhã.
De + esta(s) = desta(s) Matéria = Farei um cartão de papel reciclado.
De + esse(s) = desse(s) Meio = Nós vamos fazer um passeio de barco.
De + essa(s) = dessa(s) Origem = Nós somos do Nordeste, e você?
De + aquele(s) = daquele(s) Conteúdo = Quebrei dois frascos de perfume.
De + aquela(s) = daquela(s) Oposição = Esse movimento é contra o que eu penso.
De + isto = disto Preço = Essa roupa sai por R$ 50 à vista.
De + isso = disso
De + aquilo = daquilo Fonte:
De + aqui = daqui http://www.infoescola.com/portugues/preposicao/
De + aí = daí
De + ali = dali
De + outro = doutro(s) 15. CONJUNÇÃO E SINONÍMIA;
De + outra = doutra(s)
Em + este(s) = neste(s)
Em + esta(s) = nesta(s) CONJUNÇÃO
Em + esse(s) = nesse(s)
Em + aquele(s) = naquele(s) Morfossintaxe
Em + aquela(s) = naquela(s)
Em + isto = nisto Para definir o que é artigo é preciso mencionar suas
Em + isso = nisso
relações com o substantivo. Assim, nas orações da língua
Em + aquilo = naquilo
portuguesa, o artigo exerce a função de adjunto adnominal
A + aquele(s) = àquele(s)
do substantivo a que se refere. Tal função independe da
A + aquela(s) = àquela(s)
função exercida pelo substantivo:
A + aquilo = àquilo
A existência é uma poesia.
Uma existência é a poesia.
Dicas sobre preposição
Conjunção é a palavra invariável que liga duas orações
1. O “a” pode funcionar como preposição, pronome pes-
soal oblíquo e artigo. Como distingui-los? Caso o “a” seja ou dois termos semelhantes de uma mesma oração. Por
um artigo, virá precedendo um substantivo. Ele servirá para exemplo:
determiná-lo como um substantivo singular e feminino. A menina segurou a boneca e mostrou quando viu as
A dona da casa não quis nos atender. amiguinhas.
Como posso fazer a Joana concordar comigo? Deste exemplo podem ser retiradas três informações:
- Quando é preposição, além de ser invariável, liga dois 1-) segurou a boneca 2-) a menina mostrou 3-) viu as
termos e estabelece relação de subordinação entre eles. amiguinhas
Cheguei a sua casa ontem pela manhã.
Não queria, mas vou ter que ir à outra cidade para procu- Cada informação está estruturada em torno de um ver-
rar um tratamento adequado. bo: segurou, mostrou, viu. Assim, há nessa frase três ora-
ções:
- Se for pronome pessoal oblíquo estará ocupando o 1ª oração: A menina segurou a boneca 2ª oração: e
lugar e/ou a função de um substantivo. mostrou 3ª oração: quando viu as amiguinhas.
Temos Maria como parte da família. / Nós a temos como A segunda oração liga-se à primeira por meio do “e”, e
parte da família a terceira oração liga-se à segunda por meio do “quando”.
Creio que conhecemos nossa mãe melhor que ninguém. / As palavras “e” e “quando” ligam, portanto, orações.
Creio que a conhecemos melhor que ninguém.

79
LÍNGUA PORTUGUESA

Observe: Gosto de natação e de futebol. - CONFORMATIVAS


Nessa frase as expressões de natação, de futebol são par- Principais conjunções conformativas: como, segundo,
tes ou termos de uma mesma oração. Logo, a palavra “e” está conforme, consoante
ligando termos de uma mesma oração. Cada um colhe conforme semeia.
Expressam uma ideia de acordo, concordância, confor-
Morfossintaxe da Conjunção midade.

As conjunções, a exemplo das preposições, não exercem - CONSECUTIVAS


propriamente uma função sintática: são conectivos. Expressam uma ideia de consequência.
Classificação Principais conjunções consecutivas: que (após “tal”,
- Conjunções Coordenativas “tanto”, “tão”, “tamanho”).
- Conjunções Subordinativas Falou tanto que ficou rouco.

Conjunções coordenativas - FINAIS


Expressam ideia de finalidade, objetivo.
Dividem-se em: Todos trabalham para que possam sobreviver.
- ADITIVAS: expressam a ideia de adição, soma. Ex. Gosto Principais conjunções finais: para que, a fim de que, por-
de cantar e de dançar. que (=para que),
Principais conjunções aditivas: e, nem, não só...mas tam-
bém, não só...como também. - PROPORCIONAIS
Principais conjunções proporcionais: à medida que,
- ADVERSATIVAS: Expressam ideias contrárias, de oposi- quanto mais, ao passo que, à proporção que.
ção, de compensação. Ex. Estudei, mas não entendi nada. À medida que as horas passavam, mais sono ele tinha.
Principais conjunções adversativas: mas, porém, contudo,
todavia, no entanto, entretanto. - TEMPORAIS

- ALTERNATIVAS: Expressam ideia de alternância. Principais conjunções temporais: quando, enquanto,


Ou você sai do telefone ou eu vendo o aparelho. logo que.
Principais conjunções alternativas: Ou...ou, ora...ora, quer... Quando eu sair, vou passar na locadora.
quer, já...já.
- CONCLUSIVAS: Servem para dar conclusões às orações. Diferença entre orações causais e explicativas
Ex. Estudei muito, por isso mereço passar.
Principais conjunções conclusivas: logo, por isso, pois (de- Quando estudamos Orações Subordinadas Adverbiais
pois do verbo), portanto, por conseguinte, assim. (OSA) e Coordenadas Sindéticas (CS), geralmente nos de-
paramos com a dúvida de como distinguir uma oração
- EXPLICATIVAS: Explicam, dão um motivo ou razão. Ex. É causal de uma explicativa. Veja os exemplos:
melhor colocar o casaco porque está fazendo muito frio lá fora. 1º) Na frase “Não atravesse a rua, porque você pode ser
Principais conjunções explicativas: que, porque, pois (antes atropelado”:
do verbo), porquanto. a) Temos uma CS Explicativa, que indica uma justificati-
va ou uma explicação do fato expresso na oração anterior.
Conjunções subordinativas b) As orações são coordenadas e, por isso, independen-
tes uma da outra. Neste caso, há uma pausa entre as ora-
- CAUSAIS ções que vêm marcadas por vírgula.
Principais conjunções causais: porque, visto que, já que, Não atravesse a rua. Você pode ser atropelado.
uma vez que, como (= porque). Outra dica é, quando a oração que antecede a OC (Ora-
Ele não fez o trabalho porque não tem livro. ção Coordenada) vier com verbo no modo imperativo, ela
será explicativa.
- COMPARATIVAS Façam silêncio, que estou falando. (façam= verbo im-
Principais conjunções comparativas: que, do que, tão... perativo)
como, mais...do que, menos...do que. 2º) Na frase “Precisavam enterrar os mortos em outra
Ela fala mais que um papagaio. cidade porque não havia cemitério no local.”
a) Temos uma OSA Causal, já que a oração subordinada
- CONCESSIVAS (parte destacada) mostra a causa da ação expressa pelo
Principais conjunções concessivas: embora, ainda que, verbo da oração principal. Outra forma de reconhecê-la é
mesmo que, apesar de, se bem que. colocá-la no início do período, introduzida pela conjunção
Indicam uma concessão, admitem uma contradição, um como - o que não ocorre com a CS Explicativa.
fato inesperado. Traz em si uma ideia de “apesar de”. Como não havia cemitério no local, precisavam enterrar
Embora estivesse cansada, fui ao shopping. (= apesar de os mortos em outra cidade.
estar cansada) b) As orações são subordinadas e, por isso, totalmente
Apesar de ter chovido fui ao cinema. dependentes uma da outra.

80
LÍNGUA PORTUGUESA

SIGNIFICAÇÃO DAS PALAVRAS Só o contexto é que determina a significação dos ho-


mônimos. A homonímia pode ser causa de ambiguidade,
Quanto à significação, as palavras são divididas nas se- por isso é considerada uma deficiência dos idiomas.
guintes categorias: O que chama a atenção nos homônimos é o seu aspec-
to fônico (som) e o gráfico (grafia). Daí serem divididos em:
Sinônimos: são palavras de sentido igual ou aproxima-
do. Exemplo: Homógrafos Heterofônicos: iguais na escrita e dife-
- Alfabeto, abecedário. rentes no timbre ou na intensidade das vogais.
- Brado, grito, clamor. - Rego (substantivo) e rego (verbo).
- Extinguir, apagar, abolir, suprimir. - Colher (verbo) e colher (substantivo).
- Justo, certo, exato, reto, íntegro, imparcial. - Jogo (substantivo) e jogo (verbo).
- Apoio (verbo) e apoio (substantivo).
Na maioria das vezes não é indiferente usar um sinô- - Para (verbo parar) e para (preposição).
nimo pelo outro. Embora irmanados pelo sentido comum, - Providência (substantivo) e providencia (verbo).
os sinônimos diferenciam-se, entretanto, uns dos outros, - Às (substantivo), às (contração) e as (artigo).
por matizes de significação e certas propriedades que o - Pelo (substantivo), pelo (verbo) e pelo (contração de
escritor não pode desconhecer. Com efeito, estes têm sen- per+o).
tido mais amplo, aqueles, mais restrito (animal e quadrúpe-
de); uns são próprios da fala corrente, desataviada, vulgar, Homófonos Heterográficos: iguais na pronúncia e di-
outros, ao invés, pertencem à esfera da linguagem culta, ferentes na escrita.
literária, científica ou poética (orador e tribuno, oculista e - Acender (atear, pôr fogo) e ascender (subir).
oftalmologista, cinzento e cinéreo). - Concertar (harmonizar) e consertar (reparar, emen-
A contribuição Greco-latina é responsável pela existên- dar).
cia, em nossa língua, de numerosos pares de sinônimos. - Concerto (harmonia, sessão musical) e conserto (ato
de consertar).
Exemplos:
- Cegar (tornar cego) e segar (cortar, ceifar).
- Adversário e antagonista.
- Apreçar (determinar o preço, avaliar) e apressar (ace-
- Translúcido e diáfano.
lerar).
- Semicírculo e hemiciclo.
- Cela (pequeno quarto), sela (arreio) e sela (verbo se-
- Contraveneno e antídoto.
lar).
- Moral e ética.
- Censo (recenseamento) e senso (juízo).
- Colóquio e diálogo.
- Cerrar (fechar) e serrar (cortar).
- Transformação e metamorfose.
- Paço (palácio) e passo (andar).
- Oposição e antítese. - Hera (trepadeira) e era (época), era (verbo).
- Caça (ato de caçar), cassa (tecido) e cassa (verbo cas-
O fato linguístico de existirem sinônimos chama-se si- sar = anular).
nonímia, palavra que também designa o emprego de si- - Cessão (ato de ceder), seção (divisão, repartição) e
nônimos. sessão (tempo de uma reunião ou espetáculo).
Antônimos: são palavras de significação oposta. Exem- Homófonos Homográficos: iguais na escrita e na pro-
plos: núncia.
- Ordem e anarquia. - Caminhada (substantivo), caminhada (verbo).
- Soberba e humildade. - Cedo (verbo), cedo (advérbio).
- Louvar e censurar. - Somem (verbo somar), somem (verbo sumir).
- Mal e bem. - Livre (adjetivo), livre (verbo livrar).
- Pomos (substantivo), pomos (verbo pôr).
A antonímia pode originar-se de um prefixo de senti- - Alude (avalancha), alude (verbo aludir).
do oposto ou negativo. Exemplos: Bendizer/maldizer, sim-
pático/antipático, progredir/regredir, concórdia/discórdia, Parônimos: são palavras parecidas na escrita e na pro-
explícito/implícito, ativo/inativo, esperar/desesperar, co- núncia: Coro e couro, cesta e sesta, eminente e iminente,
munista/anticomunista, simétrico/assimétrico, pré-nupcial/ tetânico e titânico, atoar e atuar, degradar e degredar, cé-
pós-nupcial. tico e séptico, prescrever e proscrever, descrição e discri-
ção, infligir (aplicar) e infringir (transgredir), osso e ouço,
Homônimos: são palavras que têm a mesma pronún- sede (vontade de beber) e cede (verbo ceder), comprimento
cia, e às vezes a mesma grafia, mas significação diferente. e cumprimento, deferir (conceder, dar deferimento) e diferir
Exemplos: (ser diferente, divergir, adiar), ratificar (confirmar) e retifi-
- São (sadio), são (forma do verbo ser) e são (santo). car (tornar reto, corrigir), vultoso (volumoso, muito grande:
- Aço (substantivo) e asso (verbo). soma vultosa) e vultuoso (congestionado: rosto vultuoso).

81
LÍNGUA PORTUGUESA

Polissemia: Uma palavra pode ter mais de uma signi- 04. Há uma alternativa errada. Assinale-a:
ficação. A esse fato linguístico dá-se o nome de polissemia. a) A eminente autoridade acaba de concluir uma via-
Exemplos: gem política.
- Mangueira: tubo de borracha ou plástico para regar as b) A catástrofe torna-se iminente.
plantas ou apagar incêndios; árvore frutífera; grande curral c) Sua ascensão foi rápida.
de gado. d) Ascenderam o fogo rapidamente.
- Pena: pluma, peça de metal para escrever; punição; dó. e) Reacendeu o fogo do entusiasmo.
- Velar: cobrir com véu, ocultar, vigiar, cuidar, relativo ao
véu do palato. 05. Há uma alternativa errada. Assinale-a:
Podemos citar ainda, como exemplos de palavras po- a) cozer = cozinhar; coser = costurar
lissêmicas, o verbo dar e os substantivos linha e ponto, que b) imigrar = sair do país; emigrar = entrar no país
têm dezenas de acepções. c) comprimento = medida; cumprimento = saudação
d) consertar = arrumar; concertar = harmonizar
Sentido Próprio e Sentido Figurado: as palavras po- e) chácara = sítio; xácara = verso
dem ser empregadas no sentido próprio ou no sentido figu-
rado. Exemplos: 06. Assinale o item em que a palavra destacada está
- Construí um muro de pedra. (sentido próprio). incorretamente aplicada:
- Ênio tem um coração de pedra. (sentido figurado). a) Trouxeram-me um ramalhete de flores fragrantes.
- As águas pingavam da torneira, (sentido próprio). b) A justiça infligiu a pena merecida aos desordeiros.
- As horas iam pingando lentamente, (sentido figurado). c) Promoveram uma festa beneficiente para a creche.
d) Devemos ser fiéis ao cumprimento do dever.
Denotação e Conotação: Observe as palavras em des- e) A cessão de terras compete ao Estado.
taque nos seguintes exemplos:
- Comprei uma correntinha de ouro. 07. O ...... do prefeito foi ..... ontem.
- Fulano nadava em ouro. a) mandado - caçado
No primeiro exemplo, a palavra ouro denota ou designa b) mandato - cassado
simplesmente o conhecido metal precioso, tem sentido pró- c) mandato - caçado
prio, real, denotativo. d) mandado - casçado
No segundo exemplo, ouro sugere ou evoca riquezas, e) mandado - cassado
poder, glória, luxo, ostentação; tem o sentido conotativo,
possui várias conotações (ideias associadas, sentimentos, 08. Marque a alternativa cujas palavras preenchem cor-
evocações que irradiam da palavra). retamente as respectivas lacunas, na frase seguinte: “Ne-
cessitando ...... o número do cartão do PIS, ...... a data de
EXERCÍCIOS meu nascimento.”
a) ratificar, proscrevi
01. Estava ....... a ....... da guerra, pois os homens ....... nos b) prescrever, discriminei
erros do passado. c) descriminar, retifiquei
a) eminente, deflagração, incidiram d) proscrever, prescrevi
b) iminente, deflagração, reincidiram e) retificar, ratifiquei
c) eminente, conflagração, reincidiram
d) preste, conflaglação, incidiram 09. “A ......... científica do povo levou-o a .... de feiticeiros
e) prestes, flagração, recindiram os ..... em astronomia.”
a) insipiência tachar expertos
02. “Durante a ........ solene era ........ o desinteresse do b) insipiência taxar expertos
mestre diante da ....... demonstrada pelo político”. c) incipiência taxar espertos
a) seção - fragrante - incipiência d) incipiência tachar espertos
b) sessão - flagrante - insipiência e) insipiência taxar espertos
c) sessão - fragrante - incipiência
d) cessão - flagrante - incipiência 10. Na oração: Em sua vida, nunca teve muito ......, apre-
e) seção - flagrante - insipiência sentava-se sempre ...... no ..... de tarefas ...... . As palavras
adequadas para preenchimento das lacunas são:
03. Na ..... plenária estudou-se a ..... de direitos territoriais a) censo - lasso - cumprimento - eminentes
a ..... . b) senso - lasso - cumprimento - iminentes
a) sessão - cessão - estrangeiros c) senso - laço - comprimento - iminentes
b) seção - cessão - estrangeiros d) senso - laço - cumprimento - eminentes
c) secção - sessão - extrangeiros e) censo - lasso - comprimento - iminentes
d) sessão - seção - estrangeiros
e) seção - sessão - estrangeiros Respostas: (01.B)(02.B)(03.A)(04.D)(05.B)(06.C)(07.B)
(08.E)(09.A)(10.B)

82
LÍNGUA PORTUGUESA

Assinalei com um “X” onde há pontuação inadequada


16. MORFO-SINTAXE. ou faltante:
(A) Um levantamento mostrou, (X) que os adolescentes
americanos consomem (X) em média (X) 357 calorias, (X)
“CARO CANDIDATO, O TÓPICO ACIMA FOI diárias dessa fonte.
ABORDADO NO DECORRER DA MATÉRIA” (B) Um levantamento mostrou que, (X) os adolescentes
americanos consomem, em média (X) 357 calorias diárias
dessa fonte.
EXERCÍCIOS COMPLEMENTARES (C) Um levantamento mostrou que os adolescentes
americanos consomem, em média, 357 calorias diárias des-
1-) (FUNDAÇÃO UNIVERSIDADE FEDERAL DO ABC/ sa fonte.
SP – ADMINISTRADOR - VUNESP/2013) Assinale a al- (D) Um levantamento, (X) mostrou que os adolescentes
ternativa correta quanto à concordância, de acordo americanos, (X) consomem (X) em média (X) 357 calorias
com a norma-padrão da língua portuguesa. diárias dessa fonte.
(A) A má distribuição de riquezas e a desigualdade (E) Um levantamento mostrou que os adolescentes
social está no centro dos debates atuais. americanos, (X) consomem (X) em média (X) 357 calorias
(B) Políticos, economistas e teóricos diverge em re- diárias, (X) dessa fonte.
lação aos efeitos da desigualdade social.
(C) A diferença entre a renda dos mais ricos e a dos RESPOSTA: “C”.
mais pobres é um fenômeno crescente.
(D) A má distribuição de riquezas tem sido muito 3-) (TRT/RO E AC – ANALISTA JUDICIÁRIO –
criticado por alguns teóricos. FCC/2011) Estão plenamente observadas as normas de
(E) Os debates relacionado à distribuição de rique- concordância verbal na frase:
zas não são de exclusividade dos economistas. a) Destinam-se aos homens-placa um lugar visível
nas ruas e nas praças, ao passo que lhes é suprimida a
Realizei a correção nos itens:
visibilidade social.
(A) A má distribuição de riquezas e a desigualdade so-
b) As duas tábuas em que se comprimem o famige-
cial está = estão
rado homem-placa carregam ditos que soam irônicos,
(B) Políticos, economistas e teóricos diverge = diver-
como “compro ouro”.
gem
c) Não se compara aos vexames dos homens-placa
(C) A diferença entre a renda dos mais ricos e a dos
a exposição pública a que se submetem os guardadores
mais pobres é um fenômeno crescente.
de carros.
(D) A má distribuição de riquezas tem sido muito criti-
cado = criticada d) Ao se revogarem o emprego de carros-placa na
(E) Os debates relacionado = relacionados propaganda imobiliária, poupou-se a todos uma de-
monstração de mau gosto.
RESPOSTA: “C”. e) Não sensibilizavam aos possíveis interessados
em apartamentos de luxo a visão grotesca daqueles ve-
2-) (COREN/SP – ADVOGADO – VUNESP/2013) Se- lhos carros-placa.
guindo a norma-padrão da língua portuguesa, a frase
– Um levantamento mostrou que os adolescentes ame- Fiz as correções entre parênteses:
ricanos consomem em média 357 calorias diárias dessa a) Destinam-se (destina-se) aos homens-placa um lu-
fonte. – recebe o acréscimo correto das vírgulas em: gar visível nas ruas e nas praças, ao passo que lhes é supri-
(A) Um levantamento mostrou, que os adolescentes mida a visibilidade social.
americanos consomem em média 357 calorias, diárias b) As duas tábuas em que se comprimem (comprime)
dessa fonte. o famigerado homem-placa carregam ditos que soam irô-
(B) Um levantamento mostrou que, os adolescentes nicos, como “compro ouro”.
americanos consomem, em média 357 calorias diárias c) Não se compara aos vexames dos homens-placa a
dessa fonte. exposição pública a que se submetem os guardadores de
(C) Um levantamento mostrou que os adolescentes carros.
americanos consomem, em média, 357 calorias diárias d) Ao se revogarem (revogar) o emprego de carros-
dessa fonte. -placa na propaganda imobiliária, poupou-se a todos uma
(D) Um levantamento, mostrou que os adolescentes demonstração de mau gosto.
americanos, consomem em média 357 calorias diárias e) Não sensibilizavam (sensibilizava) aos possíveis in-
dessa fonte. teressados em apartamentos de luxo a visão grotesca da-
(E) Um levantamento mostrou que os adolescentes queles velhos carros-placa.
americanos, consomem em média 357 calorias diárias,
dessa fonte. RESPOSTA: “C”.

83
LÍNGUA PORTUGUESA

4-) (TRE/PA- ANALISTA JUDICIÁRIO – FGV/2011) 6-) (TRE/PA- ANALISTA JUDICIÁRIO – FGV/2011)
Assinale a palavra que tenha sido acentuada seguindo a Segundo o Manual de Redação da Presidência da Repú-
mesma regra que distribuídos. blica, NÃO se deve usar Vossa Excelência para
(A) sócio (A) embaixadores.
(B) sofrê-lo (B) conselheiros dos Tribunais de Contas estaduais.
(C) lúcidos (C) prefeitos municipais.
(D) constituí (D) presidentes das Câmaras de Vereadores.
(E) órfãos (E) vereadores.

Distribuímos = regra do hiato (...) O uso do pronome de tratamento Vossa Senhoria


(A) sócio = paroxítona terminada em ditongo (abreviado V. Sa.) para vereadores está correto, sim. Numa
(B) sofrê-lo = oxítona (não se considera o pronome Câmara de Vereadores só se usa Vossa Excelência para o seu
oblíquo. Nunca!) presidente, de acordo com o Manual de Redação da Presi-
(C) lúcidos = proparoxítona dência da República (1991).
(D) constituí = regra do hiato (diferente de “constitui” (Fonte: http://www.linguabrasil.com.br/nao-tropece-de-
– oxítona: cons-ti-tui) tail.php?id=393)
(E) órfãos = paroxítona terminada em “ão”
RESPOSTA: “E”.
RESPOSTA: “D”.
7-) (TRE/AL – TÉCNICO JUDICIÁRIO – FCC/2010)
5-) (TRT/PE – ANALISTA JUDICIÁRIO – FCC/2012) ... valores e princípios que sejam percebidos pela so-
A concordância verbal está plenamente observada na ciedade como tais.
frase: Transpondo para a voz ativa a frase acima, o verbo
(A) Provocam muitas polêmicas, entre crentes e passará a ser, corretamente,
materialistas, o posicionamento de alguns religiosos e (A) perceba.
(B) foi percebido.
parlamentares acerca da educação religiosa nas escolas
(C) tenham percebido.
públicas.
(D) devam perceber.
(B) Sempre deverão haver bons motivos, junto
(E) estava percebendo.
àqueles que são contra a obrigatoriedade do ensino
religioso, para se reservar essa prática a setores da ini-
... valores e princípios que sejam percebidos pela so-
ciativa privada.
ciedade como tais = dois verbos na voz passiva, então te-
(C) Um dos argumentos trazidos pelo autor do tex-
remos um na ativa: que a sociedade perceba os valores e
to, contra os que votam a favor do ensino religioso na
princípios...
escola pública, consistem nos altos custos econômicos
que acarretarão tal medida. RESPOSTA: “A”
(D) O número de templos em atividade na cidade
de São Paulo vêm gradativamente aumentando, em 8-) (TRE/AL – TÉCNICO JUDICIÁRIO – FCC/2010)
proporção maior do que ocorrem com o número de es- A concordância verbal e nominal está inteiramente cor-
colas públicas. reta na frase:
(E) Tanto a Lei de Diretrizes e Bases da Educação (A) A sociedade deve reconhecer os princípios e
como a regulação natural do mercado sinalizam para valores que determinam as escolhas dos governantes,
as inconveniências que adviriam da adoção do ensino para conferir legitimidade a suas decisões.
religioso nas escolas públicas. (B) A confiança dos cidadãos em seus dirigentes
devem ser embasados na percepção dos valores e prin-
(A) Provocam = provoca (o posicionamento) cípios que regem a prática política.
(B) Sempre deverão haver bons motivos = deverá haver (C) Eleições livres e diretas é garantia de um verda-
(C) Um dos argumentos trazidos pelo autor do texto, deiro regime democrático, em que se respeita tanto as
contra os que votam a favor do ensino religioso na escola liberdades individuais quanto as coletivas.
pública, consistem = consiste. (D) As instituições fundamentais de um regime de-
(D) O número de templos em atividade na cidade de mocrático não pode estar subordinado às ordens indis-
São Paulo vêm gradativamente aumentando, em propor- criminadas de um único poder central.
ção maior do que ocorrem = ocorre (E) O interesse de todos os cidadãos estão voltados
(E) Tanto a Lei de Diretrizes e Bases da Educação como para o momento eleitoral, que expõem as diferentes
a regulação natural do mercado sinalizam para as inconve- opiniões existentes na sociedade.
niências que adviriam da adoção do ensino religioso nas
escolas públicas.

RESPOSTA: “E”.

84
LÍNGUA PORTUGUESA

Fiz os acertos entre parênteses: Se a gente entrasse (verbo no singular) na serraria, ve-
(A) A sociedade deve reconhecer os princípios e va- ria = entrasse / veria.
lores que determinam as escolhas dos governantes, para
conferir legitimidade a suas decisões. RESPOSTA: “C”.
(B) A confiança dos cidadãos em seus dirigentes de-
vem (deve) ser embasados (embasada) na percepção dos 11-) (TRE/AL – ANALISTA JUDICIÁRIO – FCC/2010)
valores e princípios que regem a prática política. A pontuação está inteiramente adequada na frase:
(C) Eleições livres e diretas é (são) garantia de um ver- a) Será preciso, talvez, redefinir a infância já que as
dadeiro regime democrático, em que se respeita (respei- crianças de hoje, ao que tudo indica nada mais têm a ver
tam) tanto as liberdades individuais quanto as coletivas. com as de ontem.
(D) As instituições fundamentais de um regime demo- b) Será preciso, talvez redefinir a infância: já que as
crático não pode (podem) estar subordinado (subordina- crianças, de hoje, ao que tudo indica nada têm a ver, com
das) às ordens indiscriminadas de um único poder central. as de ontem.
(E) O interesse de todos os cidadãos estão (está) vol- c) Será preciso, talvez: redefinir a infância, já que as
tados (voltado) para o momento eleitoral, que expõem crianças de hoje ao que tudo indica, nada têm a ver com
(expõe) as diferentes opiniões existentes na sociedade. as de ontem.
d) Será preciso, talvez redefinir a infância? - já que as
RESPOSTA: “A”. crianças de hoje ao que tudo indica, nada têm a ver com
as de ontem.
9-) (TRE/AL – ANALISTA JUDICIÁRIO – FCC/2010) e) Será preciso, talvez, redefinir a infância, já que as
A frase que admite transposição para a voz passiva é: crianças de hoje, ao que tudo indica, nada têm a ver com
(A) O cúmulo da ilusão é também o cúmulo do sa- as de ontem.
grado.
(B) O conceito de espetáculo unifica e explica uma Devido à igualdade textual entre os itens, a apresentação
grande diversidade de fenômenos. da alternativa correta indica quais são as inadequações nas
demais.
(C) O espetáculo é ao mesmo tempo parte da so-
ciedade, a própria sociedade e seu instrumento de uni-
RESPOSTA: “E”.
ficação.
(D) As imagens fluem desligadas de cada aspecto
12-) (POLÍCIA MILITAR DO ESTADO DO ACRE –
da vida (...).
ALUNO SOLDADO COMBATENTE – FUNCAB/2012) No
(E) Por ser algo separado, ele é o foco do olhar ilu-
trecho: “O crescimento econômico, se associado à am-
dido e da falsa consciência.
pliação do emprego, PODE melhorar o quadro aqui su-
mariamente descrito.”, se passarmos o verbo destacado
(A) O cúmulo da ilusão é também o cúmulo do sagra- para o futuro do pretérito do indicativo, teremos a forma:
do. A) puder.
(B) O conceito de espetáculo unifica e explica uma B) poderia.
grande diversidade de fenômenos. C) pôde.
- Uma grande diversidade de fenômenos é unificada e D) poderá.
explicada pelo conceito... E) pudesse.
(C) O espetáculo é ao mesmo tempo parte da socieda-
de, a própria sociedade e seu instrumento de unificação. Conjugando o verbo “poder” no futuro do pretérito do
(D) As imagens fluem desligadas de cada aspecto da Indicativo: eu poderia, tu poderias, ele poderia, nós pode-
vida (...). ríamos, vós poderíeis, eles poderiam. O sujeito da oração é
(E) Por ser algo separado, ele é o foco do olhar iludido crescimento econômico (singular), portanto, terceira pessoa
e da falsa consciência. do singular (ele) = poderia.

RESPOSTA: “B”. RESPOSTA: “B”.

10-) (MPE/AM - AGENTE DE APOIO ADMINISTRA- 13-) (TRE/AP - TÉCNICO JUDICIÁRIO – FCC/2011)
TIVO - FCC/2013) “Quando a gente entra nas serrarias, Entre as frases que seguem, a única correta é:
vê dezenas de caminhões parados”, revelou o analista a) Ele se esqueceu de que?
ambiental Geraldo Motta. b) Era tão ruím aquele texto, que não deu para distri-
Substituindo-se Quando por Se, os verbos subli- bui-lo entre os presentes.
nhados devem sofrer as seguintes alterações: c) Embora devessemos, não fomos excessivos nas
(A) entrar − vira críticas.
(B) entrava − tinha visto d) O juíz nunca negou-se a atender às reivindicações
(C) entrasse − veria dos funcionários.
(D) entraria − veria e) Não sei por que ele mereceria minha conside-
(E) entrava − teria visto ração.

85
LÍNGUA PORTUGUESA

(A) Ele se esqueceu de que? = quê? (C) … soubéssemos respeitar os mais velhos! / E
(B) Era tão ruím (ruim) aquele texto, que não deu para quando eles falassem nós calaríamos a boca!
distribui-lo (distribuí-lo) entre os presentes. (D) … saberemos respeitar os mais velhos! / E quan-
(C) Embora devêssemos (devêssemos) , não fomos ex- do eles falarem nós calaremos a boca!
cessivos nas críticas. (E) … sabemos respeitar os mais velhos! / E quando
(D) O juíz (juiz) nunca (se) negou a atender às reivindi- eles falam nós calamos a boca!
cações dos funcionários.
(E) Não sei por que ele mereceria minha consideração. No presente: nós sabemos / eles falam.

RESPOSTA: “E”. RESPOSTA: “E”.

14-) (FUNDAÇÃO CASA/SP - AGENTE ADMINIS- 16-) (UNESP/SP - ASSISTENTE TÉCNICO ADMINIS-
TRATIVO - VUNESP/2011 - ADAPTADA) Observe as TRATIVO - VUNESP/2012) A correlação entre as formas
frases do texto: verbais está correta em:
I, Cerca de 75 por cento dos países obtêm nota ne- (A) Se o consumo desnecessário vier a crescer, o
gativa... planeta não resistiu.
II,... à Venezuela, de Chávez, que obtém a pior clas- (B) Se todas as partes do mundo estiverem com alto
sificação do continente americano (2,0)... poder de consumo, o planeta em breve sofrerá um co-
Assim como ocorre com o verbo “obter” nas frases lapso.
I e II, a concordância segue as mesmas regras, na ordem (C) Caso todo prazer, como o da comida, o da bebi-
dos exemplos, em: da, o do jogo, o do sexo e o do consumo não conheces-
(A) Todas as pessoas têm boas perspectivas para o se distorções patológicas, não haverá vícios.
próximo ano. Será que alguém tem opinião diferente (D) Se os meios tecnológicos não tivessem se tor-
da maioria? nado tão eficientes, talvez as coisas não ficaram tão
(B) Vem muita gente prestigiar as nossas festas ju- baratas.
ninas. Vêm pessoas de muito longe para brincar de qua- (E) Se as pessoas não se propuserem a consumir
drilha. conscientemente, a oferta de produtos supérfluos cres-
(C) Pouca gente quis voltar mais cedo para casa. cia.
Quase todos quiseram ficar até o nascer do sol na praia.
(D) Existem pessoas bem intencionadas por aqui, Fiz as correções necessárias:
mas também existem umas que não merecem nossa (A) Se o consumo desnecessário vier a crescer, o plane-
atenção. ta não resistiu = resistirá
(E) Aqueles que não atrapalham muito ajudam. (B) Se todas as partes do mundo estiverem com alto
poder de consumo, o planeta em breve sofrerá um colapso.
Em I, obtêm está no plural; em II, no singular. Vamos (C) Caso todo prazer, como o da comida, o da bebida,
aos itens: o do jogo, o do sexo e o do consumo não conhecesse dis-
(A) Todas as pessoas têm (plural) ... Será que alguém torções patológicas, não haverá = haveria
tem (singular) (D) Se os meios tecnológicos não tivessem se tornado
(B) Vem (singular) muita gente... Vêm pessoas (plural) tão eficientes, talvez as coisas não ficaram = ficariam (ou
(C) Pouca gente quis (singular)... Quase todos quise- teriam ficado)
ram (plural) (E) Se as pessoas não se propuserem a consumir cons-
(D) Existem (plural) pessoas ... mas também existem cientemente, a oferta de produtos supérfluos crescia =
umas (plural) crescerá
(E) Aqueles que não atrapalham muito ajudam (ambas
as formas estão no plural) RESPOSTA: “B”.

RESPOSTA: “A”. 17-) (TJ/SP – AGENTE DE FISCALIZAÇÃO JUDICIÁ-


15-) (CETESB/SP - ANALISTA ADMINISTRATIVO - RIA – VUNESP/2010) Assinale a alternativa que preen-
RECURSOS HUMANOS - VUNESP/2013 - ADAPTADA) che adequadamente e de acordo com a norma culta a
Considere as orações: … sabíamos respeitar os mais lacuna da frase: Quando um candidato trêmulo ______ eu
velhos! / E quando eles falavam nós calávamos a boca! lhe faria a pergunta mais deliciosa de todas.
Alterando apenas o tempo dos verbos destacados (A) entrasse
para o tempo presente, sem qualquer outro ajuste, (B) entraria
tem-se, de acordo com a norma-padrão da língua por- (C) entrava
tuguesa: (D) entrar
(A) … soubemos respeitar os mais velhos! / E quan- (E) entrou
do eles falaram nós calamos a boca!
(B) … saberíamos respeitar os mais velhos! / E quan-
do eles falassem nós calaríamos a boca!

86
LÍNGUA PORTUGUESA

O verbo “faria” está no futuro do pretérito, ou seja, in- Década = proparoxítona / relógios = paroxítona termi-
dica que é uma ação que, para acontecer, depende de ou- nada em ditongo / suíços = regra do hiato
tra. Exemplo: Quando um candidato entrasse, eu faria / Se (A) flexíveis e cartório = paroxítonas terminadas em
ele entrar, eu farei / Caso ele entre, eu faço... ditongo / tênis = paroxítona terminada em “i” (seguida
de “s”)
RESPOSTA: “A”. (B) inferência = paroxítona terminada em ditongo /
provável = paroxítona terminada em “l” / saída = regra do
18-) (TJ/SP – AGENTE DE FISCALIZAÇÃO JUDICIÁ- hiato
RIA – VUNESP/2010 - ADAPTADA) (C) óbvio = paroxítona terminada em ditongo / após
Assinale a alternativa de concordância que pode ser = oxítona terminada em “o” + “s” / países = regra do hiato
considerada correta como variante da frase do texto – (D) islâmico = proparoxítona / cenário = paroxítona
A maioria considera aceitável que um convidado che- terminada em ditongo / propôs = oxítona terminada em
gue mais de duas horas ... “o” + “s”
(A) A maioria dos cariocas consideram aceitável (E) república = proparoxítona / empresária = paroxíto-
que um convidado chegue mais de duas horas... na terminada em ditongo / graúda = regra do hiato
(B) A maioria dos cariocas considera aceitáveis que
um convidado chegue mais de duas horas... RESPOSTA: “E”.
(C) As maiorias dos cariocas considera aceitáveis
que um convidado chegue mais de duas horas... 20-) (POLÍCIA CIVIL/SP – AGENTE POLICIAL - VU-
(D) As maiorias dos cariocas consideram aceitáveis NESP/2013) De acordo com a norma- padrão da
que um convidado chegue mais de duas horas... língua portuguesa, o acento indicativo de crase está
(E) As maiorias dos cariocas consideram aceitável corretamente empregado em:
que um convidado cheguem mais de duas horas... (A) A população, de um modo geral, está à espera
de que, com o novo texto, a lei seca possa coibir os aci-
Fiz as indicações: dentes.
(A) A maioria dos cariocas consideram (ou considera, (B) A nova lei chega para obrigar os motoristas à
tanto faz) aceitável que um convidado chegue mais de repensarem a sua postura.
duas horas... (C) A partir de agora os motoristas estarão sujeitos
(B) A maioria dos cariocas considera (ok) aceitáveis à punições muito mais severas.
(aceitável) que um convidado chegue mais de duas horas... (D) À ninguém é dado o direito de colocar em risco
(C) As (A) maiorias (maioria) dos cariocas considera (ok) a vida dos demais motoristas e de pedestres.
aceitáveis (aceitável) que um convidado chegue mais de (E) Cabe à todos na sociedade zelar pelo cumpri-
duas horas... mento da nova lei para que ela possa funcionar.
(D) As (A) maiorias (maioria) dos cariocas consideram
(ok) aceitáveis (aceitável) que um convidado chegue mais (A) A população, de um modo geral, está à espera (dá
de duas horas... para substituir por “esperando”) de que
(E) As (A) maiorias (maioria) dos cariocas consideram (B) A nova lei chega para obrigar os motoristas à re-
(ok) aceitável que um convidado cheguem (chegue) mais pensarem (antes de verbo)
de duas horas... (C) A partir de agora os motoristas estarão sujeitos à
punições (generalizando, palavra no plural)
RESPOSTA: “A”. (D) À ninguém (pronome indefinido)
(E) Cabe à todos (pronome indefinido)
19-) (TJ/SP – AGENTE DE FISCALIZAÇÃO JUDICIÁ-
RIA – VUNESP/2010) Assinale a alternativa em que as RESPOSTA: “A”.
palavras são acentuadas graficamente pelos mesmos
motivos que justificam, respectivamente, as acentua-
ções de: década, relógios, suíços.
(A) flexíveis, cartório, tênis.
(B) inferência, provável, saída.
(C) óbvio, após, países.
(D) islâmico, cenário, propôs.
(E) república, empresária, graúda.

87
LÍNGUA PORTUGUESA

(TRIBUNAL DE JUSTIÇA DO ESTADO DE SÃO PAULO 21-) (TRIBUNAL DE JUSTIÇA DO ESTADO DE SÃO
- ESCREVENTE TÉCNICO JUDICIÁRIO – VUNESP/2013 - PAULO - ESCREVENTE TÉCNICO JUDICIÁRIO – VU-
ADAPTADO) Leia o texto, para responder às questões NESP/2013) Assinale a alternativa contendo passagem
de números 21 e 22. em que o autor simula dialogar com o leitor.
Veja, aí estão eles, a bailar seu diabólico “pas de (A) Acalme-se, conterrâneo. Acostume-se com sua
deux” (*): sentado, ao fundo do restaurante, o cliente existência plebeia.
paulista acena, assovia, agita os braços num agônico (B) Ô, companheiro, faz meia hora que eu cheguei...
polichinelo; encostado à parede, marmóreo e impassí- (C) Veja, aí estão eles, a bailar seu diabólico “pas de
vel, o garçom carioca o ignora com redobrada atenção. deux”.
O paulista estrebucha: “Amigô?!”, “Chefê?!”, “Parcei- (D) Sim, meu caro paulista...
rô?!”; o garçom boceja, tira um fiapo do ombro, olha (E) Ah, paulishhhhta otááário...
pro lustre.
Eu disse “cliente paulista”, percebo a redundância: Em “meu caro paulista”, o autor está dirigindo-se a nós,
o paulista é sempre cliente. Sem querer estereotipar, leitores.
mas já estereotipando: trata-se de um ser cujas inte-
rações sociais terminam, 99% das vezes, diante da per- RESPOSTA: “D”.
gunta “débito ou crédito?”.[...] Como pode ele entender
que o fato de estar pagando não garantirá a atenção do 22-) (TRIBUNAL DE JUSTIÇA DO ESTADO DE SÃO
garçom carioca? Como pode o ignóbil paulista, nascido PAULO - ESCREVENTE TÉCNICO JUDICIÁRIO – VU-
e criado na crua batalha entre burgueses e proletários, NESP/2013) O contexto em que se encontra a passa-
compreender o discreto charme da aristocracia? gem – Se deixou de bajular os príncipes e princesas do
Sim, meu caro paulista: o garçom carioca é antes século 19, passou a servir reis e rainhas do 20 (2.º pará-
de tudo um nobre. Um antigo membro da corte que grafo) – leva a concluir, corretamente, que a menção a
esconde, por trás da carapinha entediada, do descaso (A) príncipes e princesas constitui uma referência
e da gravata borboleta, saudades do imperador. [...] em sentido não literal.
Se deixou de bajular os príncipes e princesas do século (B) reis e rainhas constitui uma referência em sen-
19, passou a servir reis e rainhas do 20: levou gim tô- tido não literal.
nicas para Vinicius e caipirinhas para Sinatra, uísques (C) príncipes, princesas, reis e rainhas constitui uma
para Tom e leites para Nelson, recebeu gordas gorjetas referência em sentido não literal.
de Orson Welles e autógrafos de Rockfeller; ainda hoje (D) príncipes, princesas, reis e rainhas constitui uma
fala de futebol com Roberto Carlos e ouve conselhos de referência em sentido literal.
João Gilberto. Continua tão nobre quanto sempre foi, (E) reis e rainhas constitui uma referência em sen-
seu orgulho permanece intacto. tido literal.
Até que chega esse paulista, esse homem bidimen-
sional e sem poesia, de camisa polo, meia soquete e Pela leitura do texto infere-se que os “reis e rainhas”
sapatênis, achando que o jacarezinho de sua Lacoste é do século 20 são as personalidades da mídia, os “famosos”
um crachá universal, capaz de abrir todas as portas. Ah, e “famosas”. Quanto a príncipes e princesas do século 19,
paulishhhhta otááário, nenhum emblema preencherá o esses eram da corte, literalmente.
vazio que carregas no peito - pensa o garçom, antes de
conduzi-lo à última mesa do restaurante, a caminho do RESPOSTA: “B”.
banheiro, e ali esquecê-lo para todo o sempre.
Veja, veja como ele se debate, como se debaterá 23-) (TRIBUNAL DE JUSTIÇA DO ESTADO DE SÃO
amanhã, depois de amanhã e até a Quarta-Feira de Cin- PAULO - ESCREVENTE TÉCNICO JUDICIÁRIO – VU-
zas, maldizendo a Guanabara, saudoso das várzeas do NESP/2013) O sentido de marmóreo (adjetivo) equiva-
Tietê, onde a desigualdade é tão mais organizada: “Ô, le ao da expressão de mármore. Assinale a alternativa
companheirô, faz meia hora que eu cheguei, dava pra contendo as expressões com sentidos equivalentes, res-
ver um cardápio?!”. Acalme-se, conterrâneo. pectivamente, aos das palavras ígneo e pétreo.
Acostume-se com sua existência plebeia. O garçom (A) De corda; de plástico.
carioca não está aí para servi-lo, você é que foi ao res- (B) De fogo; de madeira.
taurante para homenageá-lo. (C) De madeira; de pedra.
(Antonio Prata, Cliente paulista, garçom carioca. Folha (D) De fogo; de pedra.
de S.Paulo, 06.02.2013) (E) De plástico; de cinza.

(*) Um tipo de coreografia, de dança. Questão que pode ser resolvida usando a lógica ou as-
sociação de palavras! Veja: a ignição do carro lembra-nos
fogo, combustão... Pedra, petrificado. Encontrou a respos-
ta?

RESPOSTA: “D”.

88
LÍNGUA PORTUGUESA

(TRIBUNAL DE JUSTIÇA DO ESTADO DE SÃO PAULO Vamos às análises:


- ESCREVENTE TÉCNICO JUDICIÁRIO – VUNESP/2013 - A - Se deixou de bajular os príncipes e princesas do
ADAPTADO) Para responder às questões de números século 19 = a conjunção inicial é condicional.
24 e 25, considere a seguinte passagem: Sem querer B - antes de conduzi-lo à última mesa do restaurante =
estereotipar, mas já estereotipando: trata-se de um ser conjunção temporal (dá-nos noção de tempo)
cujas interações sociais terminam, 99% das vezes, dian- C - para homenageá-lo = nessa oração temos a noção
te da pergunta “débito ou crédito?”. do motivo (qual a finalidade) da ação de “ter ido ao restau-
rante”, segundo o texto
24-) (TRIBUNAL DE JUSTIÇA DO ESTADO DE D - que carregas no peito – o “que” funciona como
SÃO PAULO - ESCREVENTE TÉCNICO JUDICIÁRIO – pronome relativo (podemos substituí-lo por “o qual” car-
VUNESP/2013) Nesse contexto, o verbo estereotipar regas no peito)
tem sentido de E - tira um fiapo do ombro – temos aqui uma oração
(A) considerar ao acaso, sem premeditação. assindética (sem conjunção “final”)
(B) aceitar uma ideia mesmo sem estar convencido
dela. RESPOSTA: “C”.
(C) adotar como referência de qualidade.
(D) julgar de acordo com normas legais. 27-) (TRIBUNAL DE JUSTIÇA DO ESTADO DE SÃO
(E) classificar segundo ideias preconcebidas. PAULO - ESCREVENTE TÉCNICO JUDICIÁRIO – VU-
NESP/2011) Em – A falta de modos dos homens da Casa
Classificar conforme regras conhecidas, mas não con- de Windsor é proverbial, mas o príncipe Edward dizendo
firmadas se verdadeiras. bobagens para estranhos no Quirguistão incomodou a
embaixadora americana.
RESPOSTA: “E”. A conjunção destacada pode ser substituída por
A) portanto. (B) como. (C) no entanto. (D)
25-) (TRIBUNAL DE JUSTIÇA DO ESTADO DE porque. (E) ou.
SÃO PAULO - ESCREVENTE TÉCNICO JUDICIÁRIO –
VUNESP/2013) Nessa passagem, a palavra cujas tem O “mas” é uma conjunção adversativa, dando a ideia de
sentido de oposição entre as informações apresentadas pelas orações,
(A) lugar, referindo-se ao ambiente em que ocorre a o que acontece no enunciado da questão. Em “A”, temos
pergunta mencionada. uma conclusiva; “B”, comparativa; “C”, adversativa; “D”, ex-
(B) posse, referindo-se às interações sociais do pau- plicativa; “E”, alternativa.
lista.
(C) dúvida, pois a decisão entre débito ou crédito RESPOSTA: “C”.
ainda não foi tomada.
(D) tempo, referindo-se ao momento em que ter- 28-) (TRIBUNAL DE JUSTIÇA DO ESTADO DE SÃO
minam as interações sociais. PAULO - ESCREVENTE TÉCNICO JUDICIÁRIO – VU-
(E) condição em que se deve dar a transação finan- NESP/2013) Assinale a alternativa contendo palavra
ceira mencionada. formada por prefixo.
(A) Máquina.
O pronome “cujo” geralmente nos dá o sentido de (B) Brilhantismo.
posse: O livros cujas folhas (lê-se: as folhas dos livros). (C) Hipertexto.
(D) Textualidade.
RESPOSTA: “B”. (E) Arquivamento.

26-) (TRIBUNAL DE JUSTIÇA DO ESTADO DE SÃO A – Máquina = sem acréscimo de afixos (prefixo ou
PAULO - ESCREVENTE TÉCNICO JUDICIÁRIO – VU- sufixo)
NESP/2013) Assinale a alternativa em que a oração B - Brilhantismo. = acréscimo de sufixo (ismo)
destacada expressa finalidade, em relação à outra que C – Hipertexto = acréscimo de prefixo (hiper)
compõe o período. D – Textualidade = acréscimo de sufixo (idade)
(A) Se deixou de bajular os príncipes e princesas do E – Arquivamento = acréscimo de sufixo (mento)
século 19, passou a servir reis e rainhas do 20...
(B) Pensa o garçom, antes de conduzi-lo à última RESPOSTA: “C”.
mesa do restaurante...
(C) Você é que foi ao restaurante para homenageá-
-lo.
(D) ... nenhum emblema preencherá o vazio que
carregas no peito ...
(E) O garçom boceja, tira um fiapo do ombro...

89
LÍNGUA PORTUGUESA

(TRIBUNAL DE JUSTIÇA DO ESTADO DE SÃO PAULO 31-) (TRIBUNAL DE JUSTIÇA DO ESTADO DE SÃO
- ESCREVENTE TÉCNICO JUDICIÁRIO – VUNESP/2013 - PAULO - ESCREVENTE TÉCNICO JUDICIÁRIO – VU-
ADAPTADA) Para responder a esta questão, conside- NESP/2013) Assinale a alternativa com as palavras
re as palavras destacadas nas seguintes passagens do acentuadas segundo as regras de acentuação, respec-
texto: tivamente, de intercâmbio e antropológico.
Desde o surgimento da ideia de hipertexto... (A) Distúrbio e acórdão.
... informações ligadas especialmente à pesquisa (B) Máquina e jiló.
acadêmica, (C) Alvará e Vândalo.
... uma “máquina poética”, algo que funcionasse (D) Consciência e características.
por analogia e associação... (E) Órgão e órfãs.
Quando o cientista Vannevar Bush [...] concebeu a
ideia de hipertexto... Para que saibamos qual alternativa assinalar, primeiro
... 20 anos depois de seu artigo fundador... temos que classificar as palavras do enunciado quanto à
posição de sua sílaba tônica:
29-) As palavras destacadas que expressam ideia Intercâmbio = paroxítona terminada em ditongo; An-
de tempo são: tropológico = proparoxítona (todas são acentuadas). Ago-
(A) algo, especialmente e Quando. ra, vamos à análise dos itens apresentados:
(B) Desde, especialmente e algo. (A) Distúrbio = paroxítona terminada em ditongo;
(C) especialmente, Quando e depois. acórdão = paroxítona terminada em “ão”
(D) Desde, Quando e depois. (B) Máquina = proparoxítona; jiló = oxítona terminada
(E) Desde, algo e depois. em “o”
(C) Alvará = oxítona terminada em “a”; Vândalo = pro-
As palavras que nos dão a noção, ideia de tempo são: paroxítona
desde, quando e depois. (D) Consciência = paroxítona terminada em ditongo;
características = proparoxítona
RESPOSTA: “D”.
(E) Órgão e órfãs = ambas: paroxítona terminada em
“ão” e “ã”, respectivamente.
30- (TRIBUNAL DE JUSTIÇA DO ESTADO DE SÃO
PAULO - ESCREVENTE TÉCNICO JUDICIÁRIO – VU-
RESPOSTA: “D”.
NESP/2013) Assinale a alternativa contendo frase com
redação de acordo com a norma-padrão de concor-
32-) (TRIBUNAL DE JUSTIÇA DO ESTADO DE SÃO
dância.
(A) Pensava na necessidade de ser substituído de PAULO - ESCREVENTE TÉCNICO JUDICIÁRIO – VU-
imediato os métodos existentes. NESP/2013) Na passagem – Nesse contexto, governos e
(B) Substitui-se os métodos de recuperação de empresas estão fechando o cerco contra a corrupção e a
informações que se ligava especialmente à pesquisa fraude, valendo-se dos mais variados mecanismos... – a
acadêmica. oração destacada expressa, em relação à anterior, sen-
(C) No hipertexto, a textualidade funciona por se- tido que responde à pergunta:
quências fixas que se estabeleceram previamente. (A) “Quando?”
(D) O inventor pensava em textos que já deveria (B) “Por quê?”
estar disponíveis em rede. (C) “Como?”
(E) Era procurado por ele máquinas com as quais (D) “Para quê?”
pudesse capturar o brilhantismo anárquico da imagi- (E) “Onde?”
nação humana.
Questão que envolve conhecimento de coesão e coe-
Coloquei entre parênteses a correção: rência. Se perguntássemos à primeira oração “COMO o
(A) Pensava na necessidade de ser substituído (serem governo está fechando o cerco contra a corrupção?”, ob-
substituídos) de imediato os métodos existentes. teríamos a resposta apresentada pela oração em destaque.
(B) Substitui-se (substituem-se) os métodos de recu-
peração de informações que se ligava (ligavam) especial- RESPOSTA: “C”.
mente à pesquisa acadêmica.
(C) No hipertexto, a textualidade funciona por se-
quências fixas que se estabeleceram previamente.
(D) O inventor pensava em textos que já deveria (de-
veriam) estar disponíveis em rede.
(E) Era procurado (eram procuradas) por ele máquinas
com as quais pudesse capturar o brilhantismo anárquico
da imaginação humana.

RESPOSTA: “C”.

90
LÍNGUA PORTUGUESA

33-) (TRIBUNAL DE JUSTIÇA DO ESTADO DE SÃO 35-) (TRIBUNAL DE JUSTIÇA DO ESTADO DE SÃO
PAULO - ESCREVENTE TÉCNICO JUDICIÁRIO – VU- PAULO – ADVOGADO - VUNESP/2013) Analise a propa-
NESP/2013) Assinale a alternativa em que todos os ver- ganda do programa 5inco Minutos.
bos estão empregados de acordo com a norma-padrão.
(A) Enviaram o texto, para que o revíssemos antes da
impressão definitiva.
(B) Não haverá prova do crime se o réu se manter
em silêncio.
(C) Vão pagar horas-extras aos que se disporem a
trabalhar no feriado.
(D) Ficarão surpresos quando o verem com a toga...
(E) Se você quer a promoção, é necessário que a re-
quera a seu superior.

Realizei a correção entre parênteses:


(A) Enviaram o texto, para que o revíssemos antes da
impressão definitiva.
(B) Não haverá prova do crime se o réu se manter (man-
tiver) em silêncio. Em norma-padrão da língua portuguesa, a frase da
(C) Vão pagar horas-extras aos que se disporem (dispu- propaganda, adaptada, assume a seguinte redação:
serem) a trabalhar no feriado. (A) 5INCO MINUTOS: às vezes, dura mais, mas não
(D) Ficarão surpresos quando o verem (virem) com a matem-na porisso.
toga... (B) 5INCO MINUTOS: as vezes, dura mais, mas não
(E) Se você quer a promoção, é necessário que a requera matem-na por isso.
(requeira) a seu superior. (C) 5INCO MINUTOS: às vezes, dura mais, mas não
RESPOSTA: “A”.
a matem por isso.
(D) 5INCO MINUTOS: as vezes, dura mais, mas não
34-) (TRIBUNAL DE JUSTIÇA DO ESTADO DE SÃO
lhe matem por isso.
PAULO - ESCREVENTE TÉCNICO JUDICIÁRIO – VU-
(E) 5INCO MINUTOS: às vezes, dura mais, mas não
NESP/2013) Assinale a alternativa que completa as lacu-
a matem porisso.
nas do trecho a seguir, empregando o sinal indicativo de
crase de acordo com a norma-padrão.
A questão envolve colocação pronominal e ortografia.
Não nos sujeitamos ____ corrupção; tampouco cede-
remos espaço ____ nenhuma ação que se proponha ____ Comecemos pela mais fácil: ortografia! A palavra “por isso”
prejudicar nossas instituições. é escrita separadamente. Assim, já descartamos duas alter-
(A) à … à … à nativas (“A” e “E”). Quanto à colocação pronominal, temos
(B) a … à … à a presença do advérbio “não”, que sabemos ser um “ímã”
(C) à … a … a para o pronome oblíquo, fazendo-nos aplicar a regra da
(D) à … à … a próclise (pronome antes do verbo). Então, a forma correta
(E) a … a … à é “mas não A matem” (por que A e não LHE? Porque quem
mata, mata algo ou alguém, objeto direto. O “lhe” é usado
Vamos por partes! para objeto indireto. Se não tivéssemos a conjunção “mas”
- Quem se sujeita, sujeita-se A algo ou A alguém, por- nem o advérbio “não”, a forma “matem-na” estaria correta,
tanto: pede preposição; já que, após vírgula, o ideal é que utilizemos ênclise – pro-
- quem cede, cede algo A alguém, então teremos objeto nome oblíquo após o verbo).
direto e indireto;
- quem se propõe, propõe-se A alguma coisa. RESPOSTA: “C”.
Vejamos:
Não nos sujeitamos À corrupção; tampouco cederemos
espaço A nenhuma ação que se proponha A prejudicar nos-
sas instituições.
* Sujeitar A + A corrupção;
* ceder espaço (objeto direto) A nenhuma ação (objeto
indireto. Não há acento indicativo de crase, pois “nenhuma”
é pronome indefinido);
* que se proponha A prejudicar (objeto indireto, no
caso, oração subordinada com função de objeto indireto.
Não há acento indicativo de crase porque temos um verbo
no infinitivo – “prejudicar”).
RESPOSTA: “C”.

91
LÍNGUA PORTUGUESA

36-) (TRIBUNAL DE JUSTIÇA DO ESTADO DE SÃO 37-) (TRIBUNAL DE JUSTIÇA DO ESTADO DE SÃO
PAULO – ADVOGADO - VUNESP/2013) Falha no Face- PAULO – ADVOGADO - VUNESP/2013) A ideia central do
book ______________ dados de 6 milhões de usuários. Nú- texto pode ser sintetizada da seguinte forma, em confor-
meros de telefone e e-mails de parte dos usuários do midade com a norma-padrão da língua portuguesa:
site ______________ para download a partir da ferramenta (A) Daqui à pouco teremos à passagem gratuita.
“Baixe uma cópia dos seus dados”, presente na seção (B) Não existe condições de se implantar a passagem
“Geral” da categoria “Privacidade”, sem o consenti- gratuita.
mento dos cadastrados da rede social. (C) É necessário a implementação da passagem gra-
(http://veja.abril.com.br, 21.06.2013. Adaptado) tuita.
(D) O povo prefere mais passagem paga que gratuita.
Em norma-padrão da língua portuguesa, as lacunas (E) A passagem barata é preferível à gratuita.
do texto devem ser preenchidas, respectivamente, com
(A) expõe … estava disponível Fiz as correções entre parênteses:
(B) expõe … estavam disponíveis (A) Daqui à (a) pouco teremos à (a) passagem gratuita.
(C) expõem … estavam disponível (B) Não existe (existem) condições de se implantar a passa-
(D) expõem … estava disponível gem gratuita.
(E) expõem … estava disponíveis (C) É necessário (necessária) a implementação da passagem
gratuita.
Sublinhei os sujeitos das orações para facilitar a per- (D) O povo prefere mais passagem paga que (paga à) gra-
cepção da concordância verbal: tuita.
Falha no Facebook expõe dados de 6 milhões de usuá- (E) A passagem barata é preferível à gratuita.
rios. O verbo “preferir” pede preposição: Prefiro água a vinho (e
Números de telefone e e-mails de parte dos usuários não: “do que vinho”)
do site estavam disponíveis
“expõe” e “estavam disponíveis”. RESPOSTA: “E”.

RESPOSTA: “B”. 38-) (TRIBUNAL DE JUSTIÇA DO ESTADO DE SÃO PAU-


LO – ADVOGADO - VUNESP/2013) Na passagem – ... e au-
(TRIBUNAL DE JUSTIÇA DO ESTADO DE SÃO PAU- sência de candidatos para preenchê-las. –, substituindo-se
LO – ADVOGADO - VUNESP/2013 - ADAPTADA) Leia o o verbo preencher por concorrer e atendendo-se à norma-
texto para responder às questões de números 37 e 38. -padrão, obtém-se:
Metrópoles desenvolvidas arcam com parte do cus- (A) … e ausência de candidatos para concorrer a elas.
to do transporte público. Fazem-no não só por populis- (B) … e ausência de candidatos para concorrer à elas.
mo dos políticos locais mas também para imprimir mais (C) … e ausência de candidatos para concorrer-lhes.
eficiência ao sistema. E, se a discussão se dá em termos (D) … e ausência de candidatos para concorrê-las.
de definir o nível ideal de subsídio, a gratuidade deixa (E) … e ausência de candidatos para lhes concorrer.
de ser um delírio para tornar-se a posição mais extrema
Vamos por exclusão: “à elas” está errada, já que não temos
num leque de possibilidades.
acento indicativo de crase antes de pronome pessoal; quando
Sou contra a tarifa zero, porque ela traz uma ou-
temos um verbo no infinitivo, podemos usar a construção: ver-
tra classe de problemas que já foi bem analisada pelo
bo + preposição + pronome pessoal. Por exemplo: Dar a eles
pessoal da teoria dos jogos: se não houver pagamento
(ao invés de “dar-lhes”).
individual, aumenta a tendência de as pessoas usarem
ônibus até para andar de uma esquina a outra, o que é
RESPOSTA: “A”.
ruim para o sistema e para a saúde.
Para complicar mais, vale lembrar que a discus- 39-) (TRIBUNAL DE JUSTIÇA DO ESTADO DE SÃO
são surge no contexto de prefeituras com orçamentos PAULO – ADVOGADO - VUNESP/2013) A Polícia Militar
apertados e áreas ainda mais prioritárias como educa- prendeu, nesta semana, um homem de 37 anos, acusado
ção e saúde para atender. de ____________ de drogas e ____________ à avó de 74 anos de
(Hélio Schwartsman, Tarifa zero, um delírio? Folha de idade. Ele foi preso em __________ com uma pequena quanti-
S.Paulo, 21.06.2013. Adaptado) dade de drogas no bairro Irapuá II, em Floriano, após várias
denúncias de vizinhos. De acordo com o Comandante do
3.º BPM, o acusado era conhecido na região pela atuação
no crime.
(www.cidadeverde.com/floriano. Acesso em 23.06.2013.
Adaptado)

92
LÍNGUA PORTUGUESA

De acordo com a norma-padrão da língua portugue- 41-) (TRIBUNAL DE JUSTIÇA DO ESTADO DE SÃO
sa, as lacunas do texto devem ser preenchidas, respecti- PAULO – ADVOGADO - VUNESP/2013) Considerando o
vamente, com: contexto, assinale a alternativa em que há termos empre-
(A) tráfico … mal-tratos … flagrante gados em sentido figurado.
(B) tráfego … maltratos … fragrante (A) Outro dia, meu pai veio me visitar… (1.º parágrafo)
(C) tráfego … maus-trato … flagrante (B) … e trouxe uma caixa de caquis, lá de Sorocaba.
(D) tráfico … maus-tratos … flagrante (1.º parágrafo)
(E) tráfico … mau-trato … fragrante (C) … devem ficar escondidos de mim, guardados
numa caixa… (último parágrafo)
Questão de ortografia. Vamos às exclusões: Polícia tra- (D) Enquanto comia, eu pensava… (1.º parágrafo)
balha com criminosos pegos em “flagrante”, no “flagra”; (E) … botei numa tigela na varanda e comemos um
“fragrante” relaciona-se a aroma, fragrância. Assim, já des- por um… (1.º parágrafo)
cartamos os itens “B” e “E”. “Tráfego” tem relação com trân-
sito, transitar, trafegar. “Tráfico” é o que consideramos ilegal, Sublinhei os termos que estão relacionados (os pronomes
praticado por traficante. Descartamos o item “C” também. e verbos retomam os seguintes substantivos abaixo):
Sobrou-nos “Maus-tratos”/mal-tratos. O tratamento dado Meus amigos e amigas e parentes queridos são como os
à avó foi ruim, mau (adjetivo). Sendo assim, o correto é caquis...
“maus-tratos”. Quando os encontro, relembro como é prazeroso vê-los...
devem ficar escondidos de mim, guardados numa caixa,
RESPOSTA:”D”. lá em Sorocaba...
(TRIBUNAL DE JUSTIÇA DO ESTADO DE SÃO PAULO Através da leitura acima, percebemos que o autor refere-
– ADVOGADO - VUNESP/2013 - ADAPTADA) Leia o texto -se aos amigos, amigas e parentes. Ao dizer que ficam guar-
para responder às questões de números 40 e 41. dados em caixas, obviamente, está utilizando uma linguagem
Outro dia, meu pai veio me visitar e trouxe uma caixa conotativa, figurada.
de caquis, lá de Sorocaba. Eu os lavei, botei numa tigela
na varanda e comemos um por um, num silêncio reveren- RESPOSTA: “C”.
cial, nos olhando de vez em quando. Enquanto comia, eu 42-) (TRIBUNAL DE JUSTIÇA DO ESTADO DO RIO DE
pensava: Deus do céu, como caqui é bom! Caqui é maravi- JANEIRO - ANALISTA DE SISTEMAS - FCC/2012) Com as
lhoso! O que tenho feito eu desta curta vida, tão afastado alterações propostas entre parênteses para o segmento
dos caquis?! grifado nas frases abaixo, o verbo que se mantém cor-
Meus amigos e amigas e parentes queridos são como retamente no singular é:
os caquis: nunca os encontro. Quando os encontro, relem- (A) a modernização do Rio se teria feito (as obras
bro como é prazeroso vê-los, mas depois que vão embo- de modernização)
ra me esqueço da revelação. Por que não os vejo sempre, (B) Mas nunca se esquece ele de que (esses autores)
toda semana, todos os dias desta curta vida? (C) por que vem passando a mais bela das cidades
Já sei: devem ficar escondidos de mim, guardados do Brasil (as mais belas cidades do Brasil)
numa caixa, lá em Sorocaba. (D) continua a haver um Rio de Janeiro do tempo
(Antônio Prata, Apolpando. Folha de S.Paulo, 29.05.2013) dos Franceses (tradições no Rio de Janeiro)
(E) do que a cidade parece ter de eterno (as belezas
40-) (TRIBUNAL DE JUSTIÇA DO ESTADO DE SÃO da cidade)
PAULO – ADVOGADO - VUNESP/2013) A oração – … nun- Fiz as anotações ao lado:
ca os encontro. (2.º parágrafo) – assume, em voz passiva, a (A) a modernização do Rio se teria feito (as obras de
seguinte redação: modernização) = se teriam feito
(A) … eu nunca encontro eles. (B) Mas nunca se esquece ele de que (esses autores) =
(B) … eles nunca têm sido encontrados por mim. se esquecem
(C) … nunca se encontram eles. (C) por que vem passando a mais bela das cidades do
(D) … eu nunca os tenho encontrado. Brasil (as mais belas cidades do Brasil) = por que vêm pas-
(E) … eles nunca são encontrados por mim. sando
(D) continua a haver um Rio de Janeiro do tempo dos
“Traduzindo” a oração destacada: “eu nunca encontro Franceses (tradições no Rio de Janeiro) = continua a haver
eles” (Observação: colocação pronominal feita dessa forma (E) do que a cidade parece ter de eterno (as belezas da
apenas para esclarecer a voz verbal!). Ao passarmos da voz cidade) = parecem ter
ativa para a voz passiva, teremos a seguinte construção: “eles
nunca são encontrados por mim”. RESPOSTA: “D”.
RESPOSTA: “E”.

93
LÍNGUA PORTUGUESA

43-) (TRIBUNAL DE JUSTIÇA DO ESTADO DO RIO DE 45-) (TRIBUNAL DE JUSTIÇA DO ESTADO DO RIO
JANEIRO - ANALISTA DE SISTEMAS - FCC/2012) Os ver- DE JANEIRO - ANALISTA DE SISTEMAS - FCC/2012) ... e
bos que exigem o mesmo tipo de complemento estão chegou à conclusão de que o funcionário passou o dia
empregados nos segmentos transcritos em: inteiro tomando café.
(A) A vida é triste e complicada. // ... mergulhemos Do mesmo modo que se justifica o sinal indicativo
de corpo e alma no cafezinho. de crase em destaque na frase acima, está correto o seu
(B) ... alguém dará o nosso recado sem endereço. // emprego em:
A vida é triste e complicada. (A) e chegou à uma conclusão totalmente inespe-
(C) Tinha razão o rapaz... // Depois de esperar duas rada.
ou três horas... (B) e chegou então à tirar conclusões precipitadas.
(D) Para quem espera nervosamente... // Depois de (C) e chegou à tempo de ouvir as conclusões finais.
esperar duas ou três horas... (D) e chegou finalmente à inevitável conclusão.
(E) Tinha razão o rapaz... // ... mergulhemos de cor- (E) e chegou à conclusões as mais disparatadas.
po e alma no cafezinho.
Vamos por exclusão:
Análise abaixo: (A) e chegou à uma = não há acento grave antes de
(A) A vida é = verbo de ligação // ... mergulhemos = artigo indefinido
intransitivo (B) e chegou então à tirar = não há acento grave antes
(B) ... alguém dará = transitivo direto e indireto (no de verbo no infinitivo
contexto, apenas direto) // A vida é = verbo de ligação (C) e chegou à tempo = não há acento grave antes de
(C) Tinha = transitivo direto // Depois de esperar = palavra masculina
transitivo direto (D) e chegou finalmente à inevitável conclusão.
(D) Para quem espera = pode ser considerado intransi- (E) e chegou à conclusões = não há acento grave quan-
tivo – (NESTE CONTEXTO) // Depois de esperar = transitivo do a preposição está no singular e a palavra que a acom-
direto panha não tem a presença do artigo definido (há genera-
(E) Tinha = transitivo direto // ... mergulhemos = in- lização). Haveria acento se a construção fosse: “chegou às
transitivo conclusões as mais disparatadas”.

RESPOSTA: “C”. RESPOSTA: “D”.

44-) (TRIBUNAL DE JUSTIÇA DO ESTADO DO RIO DE


JANEIRO - ANALISTA DE SISTEMAS - FCC/2012) A frase
que admite transposição para a voz PASSIVA é:
(A) Quando a Bem-amada vier com seus olhos tris-
tes...
(B) O chapéu dele está aí...
(C) ... chegou à conclusão de que o funcionário...
(D) Leio a reclamação de um repórter irritado...
(E) ... precisava falar com um delegado...

A única alternativa que possibilita a transposição para


a voz passiva é a: A reclamação de um repórter irritado foi
lida por mim”.

RESPOSTA: “D”.

94
ATUALIDADES E DEVERES DOS SERVIDORES PÚBLICOS

1. Questões relacionadas a fatos políticos, econômicos e sociais e culturais, nacionais e internacionais, ocorridos a partir
do 2º semestre de 2016, divulgados na mídia local e/ou nacional...................................................................................................... 01
2. Estatuto dos Funcionários Públicos Civis do Estado de São Paulo (Lei n.º 10.261/68) - artigos 239 a 250; com as alte-
rações vigentes até a publicação deste Edital;.............................................................................................................................................. 34
3. Lei Federal nº 8.429/92 (Lei de Improbidade Administrativa) artigos 1º ao 11º – com as alterações vigentes até a pu-
blicação do Edital..................................................................................................................................................................................................... 56
ATUALIDADES E DEVERES DOS SERVIDORES PÚBLICOS

A sessão esteve lotada durante as seis horas, acompa-


1. QUESTÕES RELACIONADAS A FATOS nhada por manifestantes contra e a favor da PEC. No meio
POLÍTICOS, ECONÔMICOS E SOCIAIS E da reunião, por cerca de 10 minutos, o presidente precisou
CULTURAIS, NACIONAIS E interrompeu os trabalhos devido a um curto-circuito no
sistema de som que os obrigou a mudar de sala.
INTERNACIONAIS, OCORRIDOS A PARTIR
Ao final da sessão, cerca de 100 estudantes protesta-
DO 2º SEMESTRE DE 2016, DIVULGADOS NA ram nos corredores do Senado. Com gritos de ordem, di-
MÍDIA LOCAL E/OU NACIONAL. ziam não à PEC.

A PROPOSTA
POLÍTICA A Proposta de emenda à Constituição restringe as des-
pesas do governo ao IPCA (Índice Nacional de Preços ao
PEC que limita gastos é aprovada em comissão do Consumidor Amplo) dos 12 meses anteriores, e tem du-
Senado sob protestos ração de duas décadas, com possibilidade de mudança na
A CCJ (Comissão de Constituição e Justiça) aprovou forma de limitar os gastos a partir do décimo ano.
nesta quarta-feira (9) a PEC que estabelece um limite para O texto final prevê maior folga em saúde e educação.
os gastos do governo por vinte anos. A proposta é uma das Nessas áreas, a correção do piso doas gastos só valerá a
prioridades da gestão Michel Temer no Congresso em 2016. partir de 2018, ou seja, o ano base a ser levado em consi-
Essa é a primeira etapa da tramitação da proposta cuja pró- deração será 2017.
xima fase é a apreciação do plenário. Além disso, o relatório estabelece ainda que a base de
Depois de mais de seis horas de discussão, com parecer cálculo do piso da saúde em 2017 será de 15% da receita
favorável do relator Eunício Oliveira (PMDB-CE), a proposta líquida, e não de 13,7%, como previa o texto original.
foi avalizada por 19 votos a favor e 7 contrários. O presiden- Fonte: Folha.com – (09/11/2016)
te da CCJ, senador José Maranhão (PMDB-MA), não votou.
Não houve mudanças ao texto aprovado na Câmara PEC 55 é aprovada no Senado em primeiro turno
no fim de outubro, apesar das tentativas da oposição. O O Senado aprovou na noite desta terça-feira a Propos-
peemedebista rejeitou todas as emendas apresentadas, a ta de Emenda à Constituição que estabelece um teto para
maioria por petistas. os gastos públicos. Por 61 votos a 14, o texto base da PEC
A oposição tentou, por meio das emendas ao texto 55 – antiga PEC 241 – foi aprovado na primeira votação,
principal, alterar o cálculo do teto, excluir da proposta des- mas ainda passará por mais três sessões de discussão e
pesas com saúde, educação, segurança e reajustes de ser- mais uma votação.
vidores, bem como impor a realização de referendo para A proposta, que prevê o congelamento dos gastos pú-
confirmar os termos da PEC. blicos por até 20 anos, foi aprovada em dois turnos pela
O senador Eunício Oliveira, líder do PMDB, partido do Câmara antes de chegar ao Senado. Tornou-se prioridade
presidente da República, Michel Temer, não acatou nenhum do Governo Michel Temer que vê na medida a possibili-
pedido de modificar o texto, reforçando o discurso do go- dade de reequilibrar as contas públicas. Por outro lado, a
verno de que não há prejuízo para saúde e educação. proposta sofre oposição de parte dos especialistas e ativis-
Ele destacou que programas essenciais, por exemplo, tas, que veem na regra ameaça a investimentos em saúde
vinculados à educação, como fies (Fundo de Financiamento e educação.
Estudantil) e Fundeb (Fundo de Manutenção e Desenvolvi- Poucas horas antes de a sessão no Senado começar,
mento da Educação Básica), já estão excluídos do teto. milhares de manifestantes tomaram as ruas no entorno do
“O Senado não deve adiar o início da produção dos Congresso para protestar contra a PEC e pedir pelo fora
efeitos do novo regime fiscal. A PEC não proíbe aumento de Temer. A Polícia usou bombas de gás para reprimir a mani-
gastos em nenhuma área, apenas requer que esses gastos festação, que seguiu pelo início doa noite. Não há registros
estejam submetidos a um limite. Se for necessário expandir oficiais sobre feridos.
despesas com segurança e ciência e tecnologia, por exem- Fonte: El País Brasil – (30/11/2016)
plo, ou em qualquer outra área, o Congresso será soberano
ao fazê-lo.”, afirmou o senador do PMDB. Câmara conclui votação do pacote anticorrupção
A aprovação da PEC nesta quarta cumpre o cronogra- com mudanças no texto principal
ma acertado pelo presidente do Senado, Renan Calheiros Em mais uma derrota ao relator Onyx Lorenzoni (DEM-
(PMDB-AL), com a base governista e a oposição. Este calen- -RS), o plenário da Câmara dos Deputados suprimiu do
dário prevê a apreciação do texto no plenário em primeiro pacote das medidas anticorrupção o item que tratava da
e segundo turnos nos dias 29 de novembro e 13 de dezem- responsabilização civil e criminal de dirigentes partidários
bro, respectivamente. O peemedebista pretende promulgar decorrente da desaprovação das contas das legendas e de
a PEC em 15 de dezembro, antes do início do recesso Legis- atos ilícitos atribuídos ao partido.
lativo do fim do ano. Na votação do último destaque da madrugada desta
Além das emendas rejeitadas por Eunício, também fo- quarta-feira (30), 328 deputados votaram por retirar a pro-
ram apresentados três votos em separado: dos senadores posta do texto, contra 32 votos e uma abstenção. Os depu-
Roberto Requião (PMDB-PR), Randolfe Rodrigues (Rede-AP) tados concluíram assim a votação das medidas, com várias
e Vanessa Grazziotin (PCdoB-AM). Todos foram rejeitados. mudanças no texto que veio da comissão especial.

1
ATUALIDADES E DEVERES DOS SERVIDORES PÚBLICOS

O texto propunha que a responsabilização pessoal O resultado não agradou o procurador da República,
ocorresse “quando verificada irregularidade grave e insa- Deltan Dallagnol, coordenador da força-tarefa da Lava Jato
nável resultante de conduta dolosa que importe enriqueci- no Paraná. “Está sendo aprovada a lei da intimidação contra
mento ilícito e lesão ao patrimônio do partido. promotores, juízes e grandes investigações”, escreveu Dal-
Fonte: Istoé – (30/11/2016) lagnol no Twitter.
Fonte: Exame.com – (30/11/2016)
Para Câmara, juiz e MP podem responder por abuso
de autoridade Novo relatório anticorrupção reduz número de me-
O plenário da Câmara dos Deputados aprovou na didas de 17 para 12
madrugada desta quarta-feira (30), por 313 votos a 132 e Após mais de 6 horas, o deputado Onyx Lorenzoni
cinco abstenções, a possibilidade de juízes e membros do (DEM-RS) apresentou a sua terceira versão do relatório do
Ministério Público serem processados por crimes de abuso Projeto de Lei 4.850/16, que trata das chamadas medidas de
de autoridade. combate à corrupção. O novo relatório reduz as propostas
A emenda faz parte do texto-base do substitutivo do de 17 para 12. O texto foi fechado depois de Lorenzoni ter
pacote de medidas anticorrupção, que foi aprovado mais ouvidos as sugestões de diversas bancadas partidárias, que
cedo, por 450 deputados. Apenas um parlamentar votou pressionaram por mudanças, e de ter se reunido com o pre-
contra: Zé Geraldo (PT-PA). sidente da Câmara dos Deputados, Rodrigo Maia (DEM-RJ).
Apresentada pela bancada do PDT, a emenda lista as “É um momento complexo que estamos enfrentando
situações em que juízes e promotores poderão ser proces- com um tema que todos reconhecem que é difícil. Desde
sados por abuso de autoridade, com pena de seis meses a a sua compreensão até a construção de um entendimento
dois anos de reclusão. A responsabilização dos agentes pú- que nos permita responder à sociedade brasileira”, disse.
blicos foi incluída no parecer de Onyx Lorenzoni (DEM-RS). “Fizemos a separação dos assuntos que são comuns e que
Segundo a emenda aprovada, os membros do Minis- serão trabalhados nesse projetos de lei e daqueles que não
tério Público podem responder pelo crime de abuso de vamos trabalhar”. De acordo com o presidente do colegia-
autoridade se, entre outros motivos, promoverem a “ins- do, Joaquim Passarinho, os deputados começaram ontem
tauração de procedimento sem que existem indícios míni- (22) a discutir o texto, encerraram a sessão por volta das
mos de prática de algum delito”. Além da “sanção penal”, o 23h40 e a intenção é que ele seja votado hoje (23). Para tan-
procurador ou promotor poderia estar “sujeito a indenizar to, Passarinho convocou nova reunião, marcada para as 9h.
o denunciado pelos danos materiais, morais ou à imagem A nova versão do texto manteve no texto a responsa-
que houver provocado”. bilização dos partidos políticos e criminalização do caixa
Os magistrados, por sua vez, seriam punidos em oito dois. De acordo com a proposta, passa a ser considerado
situações diferentes, entre elas, se “expressar, por qualquer crime de caixa dois arrecadar, receber, manter, movimentar,
meio de comunicação, opinião sobre processo pendente gastar ou utilizar valores, bens ou serviços estimáveis em
de julgamento”. dinheiro, paralelamente à contabilidade exigida pela legis-
Diante da ausência da proposta de anistia do caixa dois, lação eleitoral, com pena de reclusão de dois a cinco anos,
esse é o destaque mais polêmico da matéria. A criação do e multa. As penas serão aplicadas em dobro se os recur-
crime de autoridade para juízes e promotores contou com sos forem provenientes de fontes vedadas pela legislação
o apoio de 20 dos 28 partidos com representação na Casa. eleitoral. Incorre na mesma pena o doador de campanha.
PPS, PV, PSOL e Rede Sustentabilidade votaram contra O relator manteve também o escalonamento de penas de
a permissão para processar juízes e promotores. Já o DEM, acordo com os valores desviados e o fim da prescrição re-
PHS, PROS e PSDB liberaram suas bancadas. troativa das ações penais.
Nos corredores da Câmara, os parlamentares reforça-
vam que a decisão não representava uma afronta ao Minis- Exclusões
tério Público ou uma tentativa de travar o andamento das Foi mantida a exclusão do texto da previsão de crime de
investigações da Operação Lava Jato. Deputados destaca- responsabilidade para juízes e promotores, um dos princi-
ram que a iniciativa é apenas uma ofensiva da Câmara para pais pontos de pressão dos deputados que queriam que a
conter abusos. medida, excluída por Lorenzoni após reunião com integran-
Querem dizer que quem vota por essa emenda, vota tes da Força Tarefa da Operação Lava Jato, fosse reincorpo-
contra Lava Jato. Que falácia, que absurdo”, afirmou o de- rada ao texto. Lorenzoni disse que vai encaminhar o tema
putado André Figueiredo (PDT-CE). No mesmo sentido, Al- para o presidente da Casa para que ele seja tratado como
berto Fraga (DEM-DF) disse que o objetivo dos parlamen- um projeto a parte. “Com a gente debatendo com os agen-
tares não é parar a Lava Jato. “Queremos parar abusos”. tes públicos que serão alvos da nossa proposta legislativa,
Após a votação do texto na Câmara, o projeto será vo- vamos equilibrar a discussão e ouvir com mais calma todos
tado no Senado antes de ser enviado à sanção. Vale lem- os envolvidos”, disse.
brar que o presidente do Senado, Renan Calheiros (PMDB- O deputado disse vai pedir celeridade na tramitação da
-AL), é um dos maiores defensores da aprovação de uma Proposta de Emenda à Constituição (PEC) 291/13, do Sena-
lei que puna abusos de autoridade. do, que regulamenta o regime disciplinar da magistratura
O projeto foi aprovado dois dias antes de Renan rece- e do Ministério Público e que vai pedir que os órgãos do
ber o juiz Sérgio Moro para um debate sobre um projeto Judiciário e do Ministério Público encaminhem sugestões
de lei de abuso de autoridade no plenário do Senado. de projetos sobre o tema.

2
ATUALIDADES E DEVERES DOS SERVIDORES PÚBLICOS

Também foram retiradas pontos do trecho que tratam “Vamos votar um substitutivo que muda bastante o
do uso da prisão preventiva para assegurar a devolução do conteúdo do texto, muda quase tudo, cerca de 70%”, disse
dinheiro desviado; propostas encaminhadas pela Ordem o deputado Vicente Cândido, vice-líder do PT e próximo ao
dos Advogados do Brasil (OAB) que tratam da ação popular. presidente da Câmara, Rodrigo Maia (DEM-RJ).
O ponto que diz respeito aos acordo de cooperação in- Segundo o deputado, há um acordo para que o texto
ternacional também foi retirado, após pressão de integran- siga, “de imediato”, para ser apreciado no Senado, para, em
tes da Polícia Federal, assim como em relação às equipes de seguida, ser sancionado pelo presidente Michel Temer.
cooperação internacional. “Mesmo com o acordo firmado, Ele, no entanto, negou que a pressa exista por conta da
os líderes pediram ao longo do dia de hoje e dos últimos expectativa da homologação das delações dos executivos
dias que tudo aquilo que excedesse as dez medidas fosse da Odebrecht, no âmbito da Operação Lava Jato.
retirado”, justificou Lorenzoni. Outra medida que foi retirada Para justificar a aprovação de um texto mais favorável à
diz respeito a incorporação da decisão recente do Supremo classe política, deputados alegam que o relator das medidas
Tribunal Federal (STF) de que a pena já possa ser cumprida anticorrupção descumpriu o acordo com os líderes e votou
após condenação em segunda instância. um texto diferente do que foi acertado com as bancadas.
Madrugada. O presidente da Câmara deixou a Casa por
Alterações volta das duas da manhã dizendo que não conhecia o texto
O relator também fez alterações no trecho do projeto aprovado e que, por isso, não sabia dizer o que iria aconte-
que trata dos testes de integridade para funcionários públi- cer durante a votação do pacote no plenário. “Eu não vi o
cos. Segundo Lorenzoni, com os ajustes, o teste permaneceu texto, então não posso dizer o que vai ser aprovado.”
com efeitos administrativos. “A demissão não pode ocorrer Maia também negou que haverá uma anistia aos políti-
com base apenas no teste. Tem que ser aplicado em 100% cos que praticaram caixa 2 e defendeu a tese de que, como
dos servidores ou agentes públicos, tem que ser primeiro o crime passará a ser tipificado somente após a aprovação
treinados para depois ser integrados. Vai ser parte integran- da proposta, não há como punir quem praticou atos dessa
te da formação de funcionários públicos”, disse. natureza antes.
Lorenzoni manteve a criminalização do enriquecimento O pacote das medidas anticorrupção foi aprovado por
ilícito de funcionários públicos e a eliminação de barreiras
unanimidade na comissão. A votação foi concluída somente
para o confisco de bens de criminosos (por meio da chama-
depois da meia noite. Por conta do horário, Maia desistiu
da extinção de domínio e do confisco alargado).
de votar o texto no plenário na madrugada desta quinta. A
Permanece no texto a previsão da criação e acesso a
sessão foi suspensa, mas os principais líderes da Casa con-
uma base de dados com informações de agentes públicos
tinuaram no gabinete da presidência da Casa discutindo
relativas à situação econômica ou financeira de pessoas físi-
que estratégia adotar diante do texto que foi aprovado no
cas ou jurídicas pelo Tribunal de Contas da União com o Mi-
colegiado.
nistério da Transparência, Fiscalização e Controladoria-Geral
da União e o Ministério Público. Fonte: Estadão.com – (24/11/2016)
Lorenzoni disse que também manteve no texto a previ-
são do chamado acordo de culpa pelo qual, por vontade do Nova fase da Lava Jato mira operadores que lava-
réu, em acordo com o advogado é realizado um acordo re- ram mais de R$ 50 milhões
conhecendo a validade das acusações do inquérito policial. A Polícia Federal realiza a 36ª fase da Operação Lava
“Daí o réu pode fazer o ajuste com o Ministério Público, com Jato nesta quinta-feira (10). Batizada de “Dragão”, ela mira
a participação do advogado, e vai ao juiz para homologa- dois operadores financeiros especializados na lavagem de
ção”, disse Lorenzoni. “Isso tem que gerar, no mínimo, uma recursos de grandes empreiteiras envolvidas no esquema
redução de um terço da pena. Além disso, algo em torno de de corrupção na Petrobras, segundo a força-tarefa.
30% de todos os processos da área criminal terminarão na As investigações apontam que os dois suspeitos são
fase inicial e vai haver uma desobstrução da Justiça brasilei- responsáveis por lavar mais de R$ 50 milhões para as em-
ra, reduzindo drasticamente”, disse. presas. Eles foram apontados, em delação, como opera-
Fonte: Istoé – (23/11/2016) dores utilizados para pagamentos indevidos pelo Setor de
Operações Estruturadas, o departamento da propina na
Deputados articulam derrubar pacote anticorrupção estatal.
no plenário Foram expedidos 18 mandados para três Estados: Cea-
Após a aprovação do pacote de medidas contra a cor- rá, São Paulo e Paraná. Deles, são 16 de busca e apreensão e
rupção na comissão especial da Câmara, líderes de pratica- dois de prisão preventiva (por tempo indeterminado).
mente todos os partidos, com exceção da Rede e do PSOL, O lobista Adir Assad, já preso pela Lava Jato em Curiti-
vão tentar derrubar o texto do relator Onyx Lorenzoni (DEM- ba, é um dos alvos de um dos mandados de prisão. Inves-
-RS) e aprovar um projeto substitutivo no plenário da Casa. tigado em diversas operações contra corrupção, Assad foi
A votação está prevista para ocorrer nesta quinta-feira, alvo de três mandados de prisão em pouco mais de um ano:
24. O novo texto deverá incluir as duas medidas que ficaram Lava Jato, no Paraná, Operação Saqueador, no Rio, e Opera-
de fora do pacote aprovado nesta quarta: a anistia à prática ção Pripyat, desdobramento da Lava Jato no Rio.
do caixa 2 nas campanhas eleitorais e a previsão de punir O outro alvo de mandado de prisão é o advogado Ro-
magistrados e integrantes do Ministério Público Federal drigo Tacla Duran, que está no exterior. Segundo o MPF-PR
por crime de responsabilidade. (Ministério Público Federal do Paraná), ele foi “responsável

3
ATUALIDADES E DEVERES DOS SERVIDORES PÚBLICOS

por lavar dezenas de milhões de reais por intermédio de Repasses


pessoas jurídicas por ele controladas”. Segundo um delator, As investigações apontam que Assad, por meio de trans-
as offshores utilizadas por Duran eram chamadas de “kibe” e ferências de contas mantidas por suas empresas em território
“dragão” --esta última que dá nome à operação deflagrada nacional, repassou R$ 24,3 milhões a Duran.
desta quinta. A operação apura as práticas, dentre outros crimes, de
Em entrevista na manhã desta quinta-feira (10) na sede corrupção, manutenção não declarada de valores no exterior
da Polícia Federal em Curitiba, os procuradores Julio Motta e lavagem de dinheiro. As ações são realizadas nas cidades de
Noronha e Roberson Pozzobon disseram ter encontrado, ao Jaguaruana (CE), São Paulo, Barueri (SP), Santana do Parnaíba
longo das investigações iniciais, indícios que mostram um (SP), Curitiba e Londrina (PR). Ao menos 90 agentes da Polícia
quadro de corrupção sistêmica e lavagem de dinheiro contí- Federal estão envolvidos na operação.
nua envolvendo operadores financeiros, agentes públicos e Fonte: Portal UOL – (10/11/2016)
empreiteiros.
Assad e Duran seriam os profissionais do esquema en- Senado aprova proposta de reforma política
carregados de trazer dinheiro de origem ilícita do exterior Em primeiro turno, senadores aprovam proposta que
e transformá-lo em dinheiro legal disponível em espécie no acaba com coligações, para deputados e vereadores, cria
Brasil, supostamente originário da prestação de serviços. cláusula de barreira e pune políticos eleitos que mudarem de
Segundo os procuradores, as investigações já mostra- partido. PEC segue para votação em segundo turno. Na pri-
ram que havia também uma relação estruturada entre Assad meira votação, o Senado aprovou nesta quarta-feira (09/11)
e Duran: “Duran e Assad tinham relacionamento constante. a proposta de emenda constitucional (PEC) 36 que acaba
Profissionais do crime que se comunicam entre si, coope- com as coligações partidárias em eleições proporcionais, para
ram”, apontou Pozzobon. vereadores e deputados, e estipula uma cláusula de barreira
Os procuradores justificaram o pedido de prisão preven- para os partidos políticos. A proposta, aprovada por 58 vo-
tiva de Duran dizendo que ele não volta ao Brasil desde abril tos a favor e 13 contra, deverá ser votada em segundo turno
deste ano, morando nos Estados Unidos, e que ele tem dupla pelos senadores e, se aprovada por no mínimo três quintos
cidadania, incluindo a espanhola. “[deixá-lo solto é] Perigo dos parlamentares, ou seja, 49 de 81, segue para a avaliação
para a ordem pública”, disse o procurador Noronha. O Mi- da Câmara dos Deputados, onde também precisa passar por
nistério Público ainda não tinha recebido a confirmação da duas votações para entrar em vigor.
prisão de Duran na manhã desta quinta-feira. A PEC, de autoria dos senadores Aécio Neves e Ricardo
Ferraço, ambos do PSDB, acaba com as coligações partidárias
Uso de empresas nas eleições proporcionais a partir de 2020. Pela atual legis-
O advogado recebeu R$ 36 milhões de empreiteiras in- lação, os partidos podem fazer coligações para eleger verea-
vestigadas na Lava Jato, entre elas, a UTC, Mendes Júnior e dores e deputados. Dessa maneira, os votos alcançados pelas
EIT. Os dados constam em pedido de quebra de sigilo de legendas coligadas são somados e são eleitos os candidatos
empresas que, segundo o MPF, são suspeitas de escoar a mais votados da sigla.
propina da Mendes Júnior. Somente da Mendes Júnior, o es-
critório Tacla Duran Sociedade de Advogados, entre 2011 e Cláusula de desempenho
2013, recebeu R$ 25,5 milhões. A proposta também estabelece uma cláusula de barreira
Da UTC foram R$ 9,1 milhões e da EIT outros R$ 2 mi- que visa diminuir o número de legendas partidárias no país.
lhões. “Diversos envolvidos no caso valeram-se dessas em- Pelo texto, os partidos precisam alcançar 2% dos votos válidos
presas a fim de gerar recursos para realizar pagamentos de no pleito para a Câmara dos Deputados, distribuídos em, ao
propina”, diz o MPF. menos, 14 unidades federativas, em 2018, e 3% a partir de
No caso da Mendes Júnior, os repasses ao operador são 2022, com 2% dos votos válidos em 14 unidades federativas.
explicados na proposta de delação premiada em negociação A cláusula limita, desta maneira, o acesso das legendas
com a Procuradoria-geral da República. Os valores seriam a verbas do fundo partidário e ao tempo de propaganda em
destinados a agentes públicos envolvidos em irregularidades rádio e televisão, além da estrutura funcional do Congresso.
em obras da Petrobras e no governo do Rio de Janeiro. Candidatos que forem eleitos por partidos que não alcança-
Até então, os investigadores tinham conhecimento ape- rem esse limite terão o mandato garantido, mas com estru-
nas da atuação de Duran em transações envolvendo as con- tura menor na Câmara e sem direito a ocupar cargos de lide-
tras secretas da Odebrecht. Com a quebra de sigilo das cons- rança e participar de comissões. Eles poderão ainda optar por
trutoras, os investigadores descobriram que duas empresas mudar para outra legenda sem penalização.
de Duran foram beneficiárias de pagamentos milionários. A medida estabelece que legendas menores podem se
Além de Odebrecht, Mendes Júnior, UTC e EIT, o MPF reunir para atuar como uma, mas para isso devem concorrer
também mapeou a relação de Tacla Duran com ao menos juntas e atuar com um único partido durante todo o mandato.
outras duas empreiteiras e dois operadores presos pela Lava A proposta da cláusula de desempenho é polêmica. Opo-
Jato. A Treviso, de Julio Camargo, operador da Toyo Setal e sitores da medida alegam que ela afetaria partidos tradi-
atualmente delator, repassou R$ 350 mil para o escritório do cionais como o PC do B, PPS e PV, além de legendas mais
advogado. Por sua vez, outra empresa de Tacla Duran, a Eco- recentes como Psol e Rede. Senadores de oposição, da
nocell do Brasil, repassou R$ 3,5 milhões para empresas de Rede, PCdoB e PT, tentaram reduzir as cláusulas de barrei-
Adir Assad, apontado como operador da Delta Engenharia ra para 1% em 2018, 1,5% em 2022 e 2% em 2026, mas a
e de outras construtoras. emenda foi rejeitada.

4
ATUALIDADES E DEVERES DOS SERVIDORES PÚBLICOS

Se a cláusula já estivesse em vigor, ela reduziria o fun- Na carta de demissão, na qual se referiu ao presidente
cionamento parlamentar de 14 legendas das 27 que inte- da República como “fraterno amigo”, Geddel escreveu que
gram atualmente o Congresso. “avolumaram-se as críticas” sobre ele e disse que o “limite da
A PEC 36 também reforça a fidelidade partidária e de- dor que suporta” é o sofrimento da família, em Salvador”. “É
termina a perda de mandato para políticos eleitos que tro- hora de sair”, afirmou na carta.
carem de partidos, menos que seja para concorrer a outra Na mensagem, ele também pediu desculpas a Temer
vaga. pela dimensão das “interpretações dadas”, referindo-se à
A votação do segundo turno no Senado sobre a PEC acusação de Marcelo Calero de que o pressionou para de-
deve ocorrer no dia 23 de novembro. sembargar a construção de um condomínio de luxo em Sal-
Fonte: Portal Terra – (09/11/2016) vador barrado pelo Iphan.
No texto, Geddel afirma que retornará à Bahia, mas se-
Fora do governo, Calero acusa Geddel de pressioná- guirá como “ardoroso torcedor” do governo. Ele também
-lo para liberar obra aproveitou para agradecer o apoio e a colaboração na apro-
De saída do governo, o ministro da Cultura, Marcelo Ca-
vação de “importantes medidas” para o país.
lero, acusa o ministro Geddel Vieira Lima (Governo) de tê-lo
pressionado a produzir um parecer técnico para favorecer
A queda de Geddel
seus interesses pessoais.
Calero diz que o articulador político do governo Temer o Até então um dos homens forte de Temer no Planalto,
procurou pelo menos cinco vezes —por telefone e pessoal- Geddel começou a balançar no cargo de ministro da Secre-
mente— para que o Iphan (Instituto do Patrimônio Histórico taria de Governo na semana passada, quando Calero con-
e Artístico Nacional), órgão subordinado à Cultura, aprovas- cedeu uma entrevista ao jornal “Folha de S. Paulo” denun-
se o projeto imobiliário La Vue Ladeira da Barra, nos arre- ciando a pressão do ex-colega da Esplanada dos Ministérios.
dores de uma área tombada em Salvador, base de Geddel. A turbulência política provocada pela denúncia chegou
Nas palavras do agora ex-ministro, Geddel disse em ao gabinete presidencial nesta quinta (24), quando foi re-
pelo menos duas dessas conversas possuir um apartamento velado o teor do depoimento prestado nesta semana por
no empreendimento que dependia de autorização federal Calero à Polícia Federal (PF). O ex-ministro disse aos policiais
para sair do papel. que, durante uma audiência no Palácio do Planalto, Temer
“Entendi que tinha contrariado de maneira muito con- interveio em favor dos interesses do então ministro da Se-
tundente um interesse máximo de um dos homens fortes do cretaria de Governo.
governo”, afirmou. Calero, que pediu demissão na última sexta (18), gravou
No lugar de Calero, assume o deputado Roberto Freire a conversa que teve na semana passada com Temer no Pla-
(PPS-SP). nalto, informou o Bom Dia Brasil. Procurado pela TV Globo,
Fonte: Folha.com – (19/11/2016) Calero disse que não pode falar desse assunto. Segundo o
G1 apurou, ele entregou cópia da gravação à PF, que enca-
Geddel pede demissão após crise gerada por denún- minhou o material para o Supremo Tribunal Federal (STF).
cia de ex-ministro
O ministro da Secretaria de Governo, Geddel Vieira Lima, Depoimento à PF
pediu demissão na manhã desta sexta-feira (25) por meio de No depoimento à PF prestado na última quarta (23), o
uma carta enviada por e-mail ao presidente Michel Temer. ex-ministro disse ter sido “enquadrado” por Temer a fim de
De Salvador, onde está desde quarta-feira (23), ele conver- encontrar uma “saída” para desembargar a construção do
sou por telefone com o presidente depois de enviar o e-mail. condomínio La Vue, na capital baiana, no qual Geddel com-
Geddel é acusado pelo ex-ministro da Cultura Marcelo
prou um apartamento.
Calero de tê-lo pressionado para liberar uma obra no centro
Após o depoimento de Marcelo Calero à PF vazar na
histórico de Salvador. Geddel é proprietário de um aparta-
imprensa, o porta-voz da Presidência da República, Alexan-
mento em um edifício cuja construção foi embargada pelo
Instituto do Patrimônio Histórico e Artístico Nacional (Iphan), dre Parola, afirmou que Temer procurou o ex-ministro da
subordinado ao Ministério da Cultura. Devido ao episódio, Cultura para resolver o “impasse” entre ele, Calero, e o chefe
Calero pediu demissão na semana passada. da Secretaria de Governo (leia a íntegra do pronunciamento
Segundo a assessoria do Palácio do Planalto, Temer de Parola ao final desta reportagem).
aceitou o pedido de Geddel, que era responsável pela arti- Segundo o colunista do G1 Matheus Leitão, a Procura-
culação política do governo. doria Geral da República (PGR) havia decidido pedir ao STF a
O presidente chegou ao Planalto nesta sexta, por volta abertura de uma investigação para apurar se Geddel fez trá-
às 10h, e, imediatamente, se reuniu com assessores próxi- fico de influência ao pressionar o ex-colega da Esplanadas.
mos, como o secretário de Comunicação Social, Márcio Frei- A PGR recebeu nesta quinta-feira (24) o depoimento
tas. Em seguida, recebeu a carta de demissão do ministro. que Calero prestou à Polícia Federal. O documento inicial-
Geddel é o sexto ministro a deixar o governo desde mente foi enviado ao Supremo, que o encaminhou para a
que Michel Temer assumiu a Presidência, em maio. Antes análise dos procuradores da República. A presidente do STF,
dele, caíram Romero Jucá (Planejamento), Fabiano Silvei- ministra Cármen Lúcia, remeteu o depoimento à PGR an-
ra (Transparência), Fábio Medina Osório (AGU), Henrique tes de mandar sortear o caso para relatoria de algum dos
Eduardo Alves (Turismo) e Marcelo Calero (Cultura). minstros do tribunal.

5
ATUALIDADES E DEVERES DOS SERVIDORES PÚBLICOS

Na condição de ministro, Geddel tinha direito ao cha- Parentes


mado “foro privilegiado”, ou seja, ser investigado e proces- Familiares do ministro da Secretaria de Governo inte-
sado pelo STF, a mais alta Corte do país. Agora, diante da gram a defesa do empreendimento imobiliário de Salvador
demissão do ministro, o caso pode ser remetido à primeira barrado pelo Iphan, no qual ele afirma ter comprado um
instância da Justiça. imóvel, publicou na quarta-feira o jornal “Folha de S.Paulo”.
Segundo o jornal, um primo e um sobrinho de Geddel
Temer ‘surpreso’ atuam como representantes do empreendimento La Vue
Nesta quinta, após Geddel pedir demissão do cargo, o Ladeira da Barra junto ao Iphan.
líder do governo na Câmara, André Moura (PSC-SE), disse A publicação afirmou que, em um documento anexado
que o presidente Temer está “surpreso” com o conteúdo do ao processo administrativo que tramitou junto ao Iphan,
depoimento de Calero. a empresa Porto Ladeira da Barra Empreendimento – res-
“O presidente estava muito surpreso ontem [quinta, 24] ponsável pelo La Vue, interditado pelo órgão ligado ao
à noite com todo esse envolvimento do nome dele pelo [ex- Ministério da Cultura – nomeou como procuradores os ad-
] ministro Calero”, disse. vogados Igor Andrade Costa, Jayme Vieira Lima Filho e o
Assim como já havia feito o porta-voz da Presidência, estagiário Afrísio Vieira Lima Neto.
Alexandre Parola, nesta quinta, André Moura também rea- Ainda de acordo com a “Folha”, Jayme é primo de Ge-
firmou que o presidente, ao procurar Calero, “só tentou so- ddel e também seria sócio dele no restaurante Al Mare, em
lucionar um conflito interno” entre ministros. “Ele [Temer] Salvador. Já o estagiário Afrísio Vieira Lima Neto é filho do
deu o caminho técnico para que a coisa fosse solucionada. deputado federal Lúcio Vieira Lima (PMDB-BA), irmão do
O caminho técnico é exatamente uma avaliação jurídica pela ministro da Secretaria de Governo.
Advocacia-Geral da União”, afirmou. A procuração, informou o jornal, foi assinada em 17
de maio de 2016, cinco dias depois de Geddel assumir o
Comissão de Ética comando da Secretaria de Governo.
Na segunda-feira (21), a Comissão de Ética Pública da Fonte: G1 – (25/11/2016)
Presidência da República decidiu abrir um processo para in-
vestigar a conduta de Geddel no episódio relatado pelo ex- “Eu não estava patrocinando interesse privado”,
ministro da Cultura. afirma Temer sobre caso Geddel
O colegiado fiscaliza eventuais conflitos de interesse en- O presidente Michel Temer negou que tenha interfe-
volvendo integrantes do governo, mas não tem poder para rido para arbitrar um conflito de natureza privada do ex-
punir nenhum servidor público, apenas pode recomendar ao ministro da Secretaria de Governo, Geddel Vieira Lima. No
chefe do Executivo sanções a integrantes do governo, entre entanto, Temer admitiu que o pedido de Geddel para que
as quais demissões. Nos últimos dias, Geddel admitiu que é fosse liberada a construção de um prédio de 30 andares
proprietário de um apartamento no empreendimento, con- em área histórica de salvador foi “muito inadequado”.
firmou que procurou o então ministro da Cultura para tratar Segundo o ex-ministro da Cultura Marcelo Calero, o
do embargo à obra, mas negou que tivesse pressionado Ca- político baiano o pressionou para que conseguisse autori-
lero para liberar a construção do edifício. zação para a obra junto ao Instituto do Patrimônio Históri-
co e Artístico Nacional (Iphan). Calero levou o caso a Temer
A obra embargada e disse que o presidente o “enquadrou” a encontrar uma
O empreendimento imobiliário pivô da saída de Marce- solução para a questão, remetendo o caso à Advocacia Ge-
lo Calero do Ministério da Cultura foi embargado pela dire- ral da União (AGU).
ção nacional do Iphan em razão de estar localizado em uma Segundo Temer, uma das conversas com Calero teria
área tombada como patrimônio cultural da União, sujeita a sido gravada pelo então titular da pasta. “Eu não estava
regramento especial. Os construtores pretendem erguer um patrocinando interesse privado, data vênia. Enfim, disse até
prédio com 31 andares, mas o Iphan autorizou a construção ao ministro, foi uma inadequação, foi muito inadequado,
de, no máximo, 13 pavimentos. não pode ser feito”, afirmou o presidente.
Com vista privilegiada para a Baía de Todos-os-Santos, Temer deu a declaração ao ser questionado sobre qual
o condomínio La Vue começou a ser construído em outu- “conflito institucional” ele estava arbitrando ao sugerir que
bro de 2015. O metro quadrado dos apartamentos – um por o caso fosse remetido à AGU, ou se estava interferindo em
andar – custa em torno de R$ 10 mil. O edifício tem aparta- uma questão particular.
mentos com quatro suítes de 259m² e uma cobertura cha- O presidente disse que o conflito institucional era entre
mada “Top House” de 450 m². Os imóveis no La Vue variam o Iphan da Bahia, que havia autorizado a obra, e o Iphan
de R$ 2,6 milhões a R$ 4,5 milhões. federal, que havia barrado o empreendimento.
No sábado, o instituto informou que a obra foi embar- Juristas ouvidos pela BBC Brasil, porém, dizem que
gada após estudos técnicos apontarem impacto do em- não há conflito entre diferentes órgãos nesse caso, já que
preendimento em cinco imóveis tombados da vizinhança o Iphan da Bahia está hierarquicamente subordinado ao
do condomínio: o forte e farol de Santo Antônio, o forte de Iphan federal.
Santa Maria, o conjunto arquitetônico do Outeiro de Santo A própria AGU manifestou-se na quinta-feira, por meio
Antônio (que inclui o forte de São Diogo), além da própria de nota, informando que “a presidência do Iphan é com-
Igreja de Santo Antônio (leia mais sobre os argumentos do petente para a anulação de ato da Superintendência es-
Iphan ao final desta reportagem). tadual e que poderia decidir o caso concreto”. Segundo

6
ATUALIDADES E DEVERES DOS SERVIDORES PÚBLICOS

a assessoria da AGU, essa decisão já havia sido dada pela Cheque de R$ 1 mi do PMDB para campanha em
procuradoria do Iphan, órgão ligado à AGU, antes do Iphan 2014 foi nominal a Temer
federal barrar a obra. Um cheque de doação no valor de R$ 1 milhão do di-
Foi a primeira vez que Temer comentou o caso publi- retório nacional do PMDB nominal à campanha do então
camente. O presidente disse também que seu perfil “não é candidato a vice-presidente Michel Temer em 10 de julho
autoritário” e que sempre atua para resolver conflitos. “Sem- de 2014 diverge, segundo a defesa da ex-presidente Dilma
pre que houver conflito entre quem quer que seja eu vou Rousseff, da versão do empreiteiro Otávio Azevedo, da An-
arbitrar”, afirmou. drade Gutierrez.
Geddel tem um apartamento no prédio que aguardava O PMDB reafirmou, em nota, que “sempre arrecadou
autorização para ser construído. Ao apresentar sua carta de recursos seguindo os parâmetros legais em vigência no
demissão na sexta-feira, ele deixou clara sua relação de ami- país”. “Doações de empresas eram permitidas e perfeita-
zade com Temer, ao se referir ao presidente como “meu fra- mente de acordo com as normas da Justiça Eleitoral nas
terno amigo” e “meu querido amigo”. eleições citadas.” O partido destacou que todas as suas
Ao deixar a coletiva, Temer foi questionado se sabia que
contas eleitorais “em todos esses anos” foram aprovadas.
Geddel era dono do imóvel no empreendimento que tentava
Em seu depoimento ao Tribunal Superior Eleitoral, na
liberar. O presidente desconversou e disse que “soube nesse
ação que pede a cassação da chapa Dilma-Temer, em se-
episódio”.
Diante da insistência sobre quando teve conhecimento, tembro deste ano, Otávio Azevedo declarou que o valor foi
ele afirmou que foi “na quinta-feira”, aparentemente se refe- doado ao diretório nacional do PT.
rindo a conversa com Calero antes da demissão do ministro, As doações de empresas para os diretórios dos parti-
no último dia 18. dos era uma prática comum até o Supremo Tribunal Fede-
ral vetar repasses de pessoas jurídicas nas eleições, nova
“Gravíssimo” regra que entrou em vigor neste ano. Os diretórios é que
Neste domingo (27), Temer criticou Calero, dizendo ser decidiam, então, os candidatos destinatários dos valores
“indigno” e “gravíssimo” que um ministro gravasse uma con- doados e registrados na Justiça Eleitoral.
versa com o presidente. “Espero que essas gravações venham Em seu relato ao TSE, Otávio Azevedo - um dos de-
a público”, disse Temer, que disse ainda cogitar fazer grava- latores da Operação Lava Jato - afirmou que a Andrade
ções oficiais das audiências na Presidência da República. Gutierrez doou em março de 2014, R$ 1 milhão ao diretório
Na entrevista, que contou com a presença do presiden- nacional do PT que, posteriormente, teria repassado à cam-
te da Câmara dos Deputados, Rodrigo Maia (PMDB-RJ), e o panha em 14 de julho.
presidente do Senado, Renan Calheiros (PMDB-AL), Temer Esse valor, segundo o empresário, teria sido pago
anunciou o que chamou de “ajustamento institucional” para como parte de um acerto de propina de 1% dos contratos
impedir a tramitação de qualquer proposta de anistia a po- da Andrade com o governo Dilma.
líticos que tenham praticado o caixa dois - movimentação Como previa a legislação no período, os diretórios
irregular de recursos de campanha eleitoral. eram obrigados a identificar o responsável pelas doações
Na quinta-feira, a votação de um projeto com medidas que chegavam à sigla e depois eram encaminhados aos
anticorrupção acabou adiada depois de vir à tona uma arti- candidatos.
culação em prol de uma emenda para anistiar quem tivesse O cheque e os registros da prestação de contas, se-
feito uso de caixa dois em eleições passadas - nos bastidores gundo a defesa de Dilma perante o TSE, mostrariam que o
da Câmara, chegou a circular um texto de uma emenda que repasse de R$ 1 milhão feito naquele ano foi para o diretó-
previa livrar, em todas as esferas (cível, criminal e eleitoral). rio nacional do PMDB. Posteriormente, o diretório encami-
“Estamos aqui para revelar que, há uma unanimidade da-
nhou os valores para a campanha da chapa Dilma-Temer.
queles dos poderes Legislativo e Executivo”, afirmou o pre-
A defesa de Dilma Rousseff no processo acusou o de-
sidente. “Não há a menor condição de se patrocinar, de se
lator de prestar falso depoimento à Justiça Eleitoral e pediu
levar adiante essa proposta”, declarou Temer, que disse ser
preciso “ouvir a voz das ruas” em relação à anistia. ao Ministério Público que apure o caso.
Maia voltou a dizer que nunca tinha sido a intenção do Ao ser indagado em setembro pelo ministro Herman
Legislativo de anistiar crimes e culpou uma “confusão de Benjamin, relator da Ação Judicial Eleitoral no TSE, sobre
comunicação” pela polêmica. “Estamos discutindo algo que as doações feitas pela Andrade Gutierrez aos vários can-
não existe”, afirmou. Calheiros disse que uma eventual pro- didatos e partidos, Otávio disse não haver uma distinção
posta de anistia não terá chances no Congresso. no caixa da empresa sobre os repasses feitos aos políticos.
A entrevista foi convocada no sábado por Temer e repre- Ele afirmou, contudo, que “certamente” o R$ 1 milhão
sentou uma rara aparição de mídia conjunta dos principais doado ao PT em março daquele ano seria decorrente do
líderes dos Poderes Executivo e Legislativo, o que pareceu in- acerto de propinas da Andrade com os ex-ministros petis-
dicar a preocupação do Planalto com a repercussão da crise tas Antonio Palocci e Ricardo Berzoini.
política detonada tanto pela renúncia de Geddel - a sexta de Ele também reafirmou que parte dos recursos que
um ministro nos seis meses de governo do pemedebista - e eram doados ao PMDB vinham de um acerto de propinas
a polêmica causada pela possibilidade de anistia do caixa da empreiteira com o partido referente às obras da usina
dois. de Belo Monte, citada pelos delatores da Andrade e que
Fonte: Último Segundo – (27/11/2016) está sob investigação da Lava Jato.

7
ATUALIDADES E DEVERES DOS SERVIDORES PÚBLICOS

Diante disso, o ministro do Tribunal Superior Eleitoral O criminalista Fernando Fernandes, defensor de Garoti-
(TSE) Herman Benjamin, relator do processo que pode levar nho, afirmou que ‘a prisão é ilegal’. Ele vai recorrer ao Tribunal
à cassação da chapa vitoriosa de Dilma Rousseff e Michel Te- Regional Eleitoral para tentar revogar o decreto de prisão ex-
mer nas eleições de 2014, determinou uma acareação entre pedido pelo juiz da zona eleitoral de Campos. Um argumento
Edinho Silva, que atuou como tesoureiro da campanha da da defesa é que o ex-governador não foi candidato nas elei-
petista, e o executivo Otávio Azevedo. ções municipais.
A acareação foi marcada para quinta-feira, 17, às 18h, no Fonte: Istoé – (16/11/2016)
TSE. A decisão do ministro acolhe pedido dos advogados que
representam o PSDB, responsáveis pela ação contra a chapa Sérgio Cabral é preso pela Operação Lava Jato
Dilma/Temer. O ex-governador do Rio de Janeiro Sérgio Cabral (PMDB) foi
A decisão de Herman foi tomada depois de a defesa preso pela Polícia Federal na manhã desta quinta-feira (17) em
de Dilma apresentar ao TSE uma série de documentos que seu apartamento, no Leblon, zona sul. Ele é alvo de dois manda-
apontam que Temer foi o beneficiário de uma doação de R$ dos de prisão, sendo um deles expedido pelo juiz Sérgio Moro,
1 milhão da Andrade Gutierrez. responsável pela Operação Lava Jato na primeira instância. Esta
A defesa de Otávio não quis comentar o assunto. foi a 37ª fase da operação que investiga um esquema de corrup-
ção na Petrobras.
Defesa O outro mandado é do juiz Marcelo Bretas, da 7ª Vara Crimi-
“O PMDB reafirma que sempre arrecadou recursos se- nal do Rio, e faz parte da Operação Calicute, um desdobramen-
guindo os parâmetros legais em vigência no país. Doações de to da Operação Lava Jato, deflagrada pela PF junto com o MPF
empresas eram permitidas e perfeitamente de acordo com (Ministério Público Federal) e a Receita Federal. Segundo as in-
as normas da Justiça Eleitoral nas eleições citadas. Em todos vestigações, Cabral liderava um grupo de pessoas que recebiam
esses anos, após fiscalização e análise acurada do Tribunal propina de empreiteiras que tinham contratos com o governo
Superior Eleitoral, todas as contas do PMDB foram aprovadas estadual. Uma das obras investigadas é a reforma no estádio do
não sendo encontrados nenhum indício de irregularidade”. Maracanã. O prejuízo estimado é de R$ 224 milhões, de acordo
Fonte: Portal UOL – (10/11/2016) com o MPF.
A mulher de Cabral e ex-primeira dama, Adriana Ancelmo,
Garotinho, ex-governador do Rio, é preso pela PF também foi levada para a Polícia Federal, a fim de cumprir man-
A Polícia Federal prendeu nesta quarta-feira, 16, o ex-go- dado de condução coercitiva (quando a pessoa é encaminhada
vernador do Rio Anthony Garotinho (PR). A prisão foi pedida para prestar esclarecimentos em sede policial). O casal deixou o
pelo Ministério Público Eleitoral. prédio onde mora sob gritos de “bandido” e “ladrão”, por volta
Agentes da delegacia da PF em Campos de Goytacazes, das 7h. Os policiais federais usaram spray de pimenta para dis-
a 270 km do Rio, reduto eleitoral de Garotinho, cumpriram persar um grupo de manifestantes, que se colocou em frente ao
o mandado na residência do ex-governador no Flamengo, carro da PF.
zona sul do Rio. Cabral é o segundo ex-governador do Rio preso em menos
Rosinha Garotinho, mulher do ex-governador, é prefeita 24 horas. Ontem, a PF prendeu Anthony Garotinho (PR) em uma
de Campos dos Goytacazes. Anthony Garotinho é secretário investigação sobre esquema de compra de votos em Campos
de governo do município. Anthony Garotinho governou o dos Goytacazes (RJ) comandada pelo Ministério Público Eleitoral.
Rio entre 1999 e 2002. Segundo a PF, 230 policiais federais cumprem 38 manda-
A ordem de prisão contra Garotinho foi decretada pelo dos de busca e apreensão, dez mandados de prisão --sendo oito
juiz Glaucenir Silva de Oliveira, da 100.ª Zona Eleitoral, em preventivos (sem prazo) e dois temporários (com prazo deter-
Campos. minado) -- e 14 mandados de condução coercitiva expedidos
Garotinho é alvo da Operação Chequinho, que investiga pela 7ª Vara Federal Criminal do Rio de Janeiro. As dez ordens
esquema de compra de votos em Campos. A PF mira o Pro- de prisão decretadas pela Justiça fluminense foram cumpridas.
grama Cheque Cidadão que teria sido usado para cooptar Além disso, mais 14 mandados de busca e apreensão, dois
eleitores no último pleito no município situado ao Norte do mandados de prisão preventiva e um mandado de prisão tem-
Estado do Rio. porária foram expedidos pela 13ª Vara Federal de Curitiba, de
Em outubro, a PF prendeu três vereadores de Campos Moro.
por suposto envolvimento no esquema – Kellenson Ayres O MPF informou que a Justiça ainda determinou o seques-
Figueiredo de Souza (PR), Miguel Ribeiro Machado (PSL) e tro e arresto de bens de Cabral e outras 11 pessoas físicas e 41
Ozeias Martins (PSDB).Outro alvo da Operação Chequinho é pessoas jurídicas.
a secretária de Desenvolvimento Humano e Social da prefei- São investigados os crimes de pertencimento a organiza-
tura de Campos Ana Alice Ribeiro Lopes de Alvarenga. ção criminosa, corrupção passiva, corrupção ativa, lavagem de
O ex-governador do Rio e atual secretário de Governo dinheiro, entre outros. Também participam das diligências deze-
de Campos foi preso por volta de 10h30 desta quarta-feira, nove procuradores do MPF e cinco auditores fiscais da Receita
16, no prédio onde reside à Rua Senador Vergueiro, Flamen- Federal.
go. Agentes da PF informaram que ele não foi algemado. Segundo o MPF, a partir de delações de executivos das em-
Alertado da presença de policiais na portaria do edifí- preiteiras Andrade Gutierrez e Carioca Engenharia descobriu-se
cio para cumprimento do mandado de prisão, Garotinho um esquema de pagamento de propina a Cabral e a pessoas do
desceu e se entregou. Na garagem, uma viatura da PF já o seu círculo para que fossem garantidos contratos de obras com
aguardava. o governo do Rio.

8
ATUALIDADES E DEVERES DOS SERVIDORES PÚBLICOS

“As investigações apontam para a prática de corrupção na gas. O programa cresceu e se expandiu, mas começou a
contratação de diversas obras conduzidas no governo de enfrentar problemas na metade do segundo do mandato
Sérgio Cabral, entre elas, a reforma do Maracanã para re- do ex-governador.
ceber a Copa de 2014, o denominado PAC Favelas e o Arco Além do constante cenário de crise na segurança pú-
Metropolitano, financiadas ou custeadas com recursos fe- blica, Cabral teve sua imagem seriamente afetada por su-
derais”, diz o MPF. cessivas denúncias e escândalos de corrupção. A relação
No primeiro semestre, executivos da empreiteira An- dele com empresários do ramo da construção civil, sobre-
drade Gutierrez relataram em delação premiada o acerto tudo o dono da Delta Construções, Fernando Cavendish,
de propinas sobre obras de urbanização do conjunto de fa- foi investigado - em julho desse ano, o empreiteiro acabou
velas de Manguinhos, além da cobrança de um percentual sendo preso pela Polícia Federal na Operação Saqueador.
na obra de reforma do estádio do Maracanã. Outro duro golpe que arranhou a reputação de Ca-
De acordo com os ex-executivos, Cabral teria recebido bral foi o sumiço do pedreiro Amarildo de Souza, em 2013,
R$ 60 milhões de propina na reforma do estádio que rece- após ter sido levado por PMs da UPP Rocinha, favela da
beu a final da Copa. O consórcio da obra teria sido definido zona sul carioca, à base da unidade. O desparecimento e
em 2009, antes mesmo da licitação. O custo foi de R$ 1,2 morte presumida do morador da comunidade virou uma
bilhão. bandeira das manifestações contra a gestão do político.
Os investigadores constataram que outras empreitei- Em 2014, seu último ano de mandato, Cabral renun-
ras também podem ter participado do esquema durante a ciou ao cargo a fim de abrir espaço para o então vice, Luiz
gestão Cabral. “Foi identificado que integrantes da orga- Fernando Pezão (PMDB). Na época, seu nome chegou a ser
nização criminosa de Sérgio Cabral amealharam e lavaram cogitado pra disputar a Presidência.
fortuna imensa, inclusive mediante a aquisição de bens de Apesar da impopularidade do antecessor, Pezão foi
luxo, assim como a prestação de serviços de consultoria reeleito governador do Rio em 2014, ano no qual o Estado
fictícios”, apontou o MPF. já enfrentava reflexos da crise financeira que chegaria ao
O nome da operação (Calicute) é uma referência às ápice nos anos seguintes.
tormentas enfrentadas pelo navegador português e desco- Atualmente, dada a agonizante situação das finanças
bridor do Brasil, Pedro Álvares Cabral, a caminho das Índias. fluminenses, o Executivo tenta aprovar um conjunto de
medidas de austeridade por meio de um projeto de lei, que
foi apelidado de “pacote das maldades”. Entre as propos-
Na mira da Lava Jato
tas, estão cortes de gastos, extinção de programas sociais,
Em Curitiba, os desdobramentos das investigações da
aumento de impostos e elevação na contribuição previ-
Operação Lava Jato revelaram, de acordo com o MPF, que
denciária dos servidores públicos.
houve pagamento de propina diretamente a Cabral em
Fonte: Portal UOL – (17/11/2016)
razão de um contrato firmado entre a Petrobras e a em-
preiteira Andrade Gutierrez. O acordo diz respeito a obras
STF marca 1º julgamento contra Renan após 9 anos
de terraplanagem no Comperj (Complexo Petroquímico do da denúncia
Rio). A Presidente do Supremo Tribunal Federal (STF), Cár-
Cabral teria recebido, entre os anos de 2007 e 2011, ao men Lúcia, marcou para o dia 1º dezembro o julgamento
menos R$ 2,7 milhões da empreiteira Andrade Gutierrez sobre a denúncia apresentada pela Procuradoria-Geral da
“por meio de entregas de dinheiro em espécie”, informou República (PGR) contra o presidente do Senado, Renan Ca-
o MPF, com repasses realizados por executivos da empre- lheiros (PMDB-AL). Se a denúncia for aceita, o parlamentar
sa para emissários do ex-governador, inclusive na sede da se tornará réu no Supremo. O relator do processo é o mi-
empreiteira em São Paulo. nistro Edson Fachin.
Além disso, os investigadores descobriram evidências Segundo a denúncia, que tramita no STF desde 2013,
do crime de lavagem de dinheiro. Apenas dois investiga- Renan teria usado o lobista de uma empreiteira para pagar
dos, entre os anos de 2009 e 2015, teriam efetuado paga- pensão a uma filha que teve fora do casamento. O peeme-
mentos em espécie, de diversos produtos e serviços, em debista também é acusado de ter adulterado documentos
valores que se aproximam de R$ 1 milhão. O crime de lava- para justificar os pagamentos. Renan nega as acusações. O
gem prevê pena entre três e dez anos de reclusão; o crime caso foi revelado em 2007.
de corrupção, entre dois e 12 anos, e o crime de integrar A defesa de Renan sustenta que o senador “já escla-
organização criminosa, pena entre três e oito anos. receu todos os fatos relativos a esta questão e é o maior
interessado no esclarecimento definido do episódio”. Se-
Quem é Sérgio Cabral gundo a assessoria de Renan, o parlamentar “foi o autor do
Liderança do PMDB no Rio, Cabral foi eleito gover- pedido de investigação das falsas denúncias em 2007, há
nador do Estado em 2006 e reeleito quatro anos depois. quase dez anos.”
Antes, havia sido deputado estadual --chegou a presidir a Em fevereiro deste ano, Fachin já tinha pautado a ação
Assembleia Legislativa do Rio-- e senador fluminense. para julgamento, mas, no mesmo mês, foi retirada da pauta
Ganhou força à frente do Executivo principalmente depois que a defesa de Renan Calheiros apresentou um
pela implementação das UPPs (Unidades de Polícia Pacifi- recurso alegando a existência de uma falha na tramitação
cadora), a partir de 2008, projeto que consiste na ocupação do processo.
policial das comunidades dominadas pelo tráfico de dro- Fonte: Portal Terra – (24/11/2016)

9
ATUALIDADES E DEVERES DOS SERVIDORES PÚBLICOS

ECONOMIA Letargia
Agostini avaliou que a economia brasileira está em esta-
OCDE prevê queda maior do PIB no Brasil em 2016 e do de “letargia”, e, por conta disso, ele revisou suas projeções
estagnação em 2017 para PIB deste ano e do próximo. Aprofundou de 3,1% para
A Organização para a Cooperação e Desenvolvimento Eco- 3,49% a previsão de retração em 2016 e reduziu para menos
nômico (OCDE) prevê uma queda maior da economia brasileira de 0,98% a expectativa de crescimento no ano que vem, que
em 2016 e uma retomada do crescimento apenas em 2018. A era de 1,1%. “A revisão para cima do PIB de 2017 recai sobre
instituição revisou nesta segunda-feira (28) suas projeções de a perspectiva de melhora vigorosa dos fatores de produção
crescimento para a economia global. a partir do segundo semestre de 2017, com destaque aos in-
Antes, a OCDE projetava uma queda de 3,3% no PIB brasi- vestimentos privados, bem como pela retomada do mercado
leiro em 2016, número que foi ampliado para 3,4%. Para 2017, de crédito com estímulo da queda da taxa de juros e início de
a estimativa melhorou: a instituição passou a projetar a estag- recuperação do mercado de trabalho, além do efeito estatís-
nação da economia no ano que vem, contra uma previsão an- tico da base de comparação menor”, explicou.
terior de queda de 0,3% no PIB. Para 2018, a expectativa é de Se as novas previsões de Agostini para o PIB se confir-
crescimento de 1,2% no PIB. marem, em 2016, o Brasil cai para o nono lugar entre as 10
A OCDE espera uma aceleração do crescimento global em maiores economias do planeta, atrás da Itália e da Índia. Em
um ritmo superior ao que projetava anteriormente. Em seu Pa- 2017, o país voltaria para a oitava colocação no ranking lide-
norama Econômico, a organização estimou que o crescimento rado por Estados Unidos, seguido por China, Japão, Alema-
global será de 2,9% este ano, 3,3% em 2017 e chegará a 3,6% nha, Reino Unido, França e Índia.
em 2018. Fonte: Correio Braziliense – (30/11/2016)
Um dos motivos para a revisão foi a vitória de Donald
Trump nas eleições americanas. Os cortes de impostos planeja- Governo vai autorizar despesas de R$ 16,2 bi no Or-
dos pela administração de Donald Trump e os gastos públicos çamento de 2016
devem aquecer a economia dos Estados Unidos, disse a OCDE. A receita obtida com multa e imposto pagos dentro do
programa de regularização de recursos ilegais no exterior
Brasil é destaque negativo do G20 deixa ao governo uma reserva técnica, até o final do ano,
O relatório pervê que o Brasil tenha o pior desempenho de R$ 16,2 bilhões, divulgou nesta terça (22) o Ministério do
entre o G20 (grupo das maiores economias do mundo) em Planejamento.
2016 e em 2017. Já o país com a previsão de pior desempenho Até 30 de novembro, o governo publicará um decreto
do G20 depois do Brasil será a Argentina, cujo PIB deve cair sobre como o valor será gasto.
1,7%. No relatório do quinto bimestre publicado pela pasta,
Na outra ponta estão a Índia (7,4%), que lidera o ranking, o governo aumentou sua previsão para a receita líquida de
seguida por China (6,7%), Indonésia (5%) e Espanha (3,2%). No 2016 em R$ 17,8 bilhões na comparação com a publicação
que vem, de acordo com a organização, a Índia deve continuar do bimestre anterior. Ao mesmo tempo, conta com despesas
na liderança (7,6%). Completam o topo da lista China (6,4%), obrigatórias R$ 2,1 bilhões maiores do que o relatório ante-
Indonésia (5%) e Turquia (3,3%). rior, deixando uma reserva de pouco mais de R$ 16 bilhões.
Fonte: G1 – (28/11/2016) O destino desse dinheiro, de acordo com o ministro do
Planejamento, Dyogo de Oliveira, será debatido nas próxi-
Resultado do PIB do Brasil no terceiro trimestre é o mas semanas. “Nossa prioridade é o pagamento de restos a
pior do mundo pagar, o que abre espaço pra termos um desempenho me-
A queda de 0,8% do Produto Interno Bruto (PIB) no tercei- lhor nos próximos anos”, disse.
ro trimestre, descontados os efeitos sazonais, fez o Brasil en- De acordo com ele, o governo avaliou os restos a pagar
colher 2,9% na comparação com o mesmo período de 2015. que possui atualmente para determinar o que é “executável”,
Esse resultado deixou o país na lanterna do desempenho de ou seja, o que pode ser liquidado pelo governo de forma
39 nações que já divulgaram o PIB e representam 83% do PIB imediata. “Fizemos essa análise e o que pode ser pago fica
mundial, conforme ranking levantado pela Austin Rating. bem acima dos R$ 16,2 bilhões de reserva”, disse. Ou seja, se
“Novamente, o Brasil foi superado pelas economias da decidir usar toda a reserva técnica para quitar restos a pagar,
Grécia, Ucrânia e Rússia, que nas edições anteriores estavam o governo atualmente consegue fazê-lo.
com desempenhos piores”, destacou o economista-chefe da Questionado sobre um possível uso dos recursos para
Austin, Alex Agostini. Segundo ele, outras economias que apre- ajuda aos Estados, o ministro não negou a possibilidade.
sentaram resultados muito ruins nas edições anteriores, como “Isso será debatido nos próximos dias e será decidido nos
a Venezuela, ainda não divulgaram seus resultados e isso colo- próximos dias”, afirmou. Oliveira declarou ainda que a meta
ca o país com o pior desempenho do mundo até o momento. fiscal, que determina um deficit de R$ 170,5 bilhões em 2016,
Esse ranking da Austin é liderado por Filipinas, que cresceu será cumprida “fielmente como previsto”.
7,1% no terceiro trimestre deste ano comparado com o mesmo A pasta informou que com a decisão da ministra Rosa
período de 2015, seguido por China, em segundo lugar, com Weber, do STF (Supremo Tribunal Federal), de que a parcela
alta de 6,7% no PIB, na mesma base de comparação. Além do da multa da repatriação seja dividida com estados, o go-
Brasil, apenas Rússia e Noruega tiveram queda no mesmo verno reservou R$ 5 bilhões para caso tenha que pagar o
intervalo, de 0,6% e 0,9%, respectivamente. montante aos entes da federação.

10
ATUALIDADES E DEVERES DOS SERVIDORES PÚBLICOS

Se não sair uma decisão final sobre esses recursos da O inquérito administrativo é um desdobramento da
multa ainda neste ano, o montante passa para o ano que Operação Lava Jato e foi subsidiado pela celebração do
vem. “Se a decisão sair em 2017 e for favorável aos estados, acordo de leniência com a construtora Andrade Gutierrez
receberão no ano que vem, se for favorável ao governo, entra e com executivos e ex-executivos da empresa, em setem-
como receita do governo”, exemplificou Oliveira. bro deste ano. Segundo o Cade, a assinatura do acordo foi
mantida em sigilo para preservar as investigações.
PIB MENOR Por meio do acordo, firmado com o Ministério Públi-
O ministro afirmou ainda que ontem o governo comu- co Federal do Paraná, por meio da força-tarefa da Lava
nicou ao Congresso a recente revisão para baixo da projeção Jato, os signatários admitem sua participação, fornecem
para o PIB de 2017, mas ressaltou que “não cabe ao governo” informações e apresentam documentos probatórios para
reestimar receitas e despesas para 2017. colaborar com as investigações sobre o suposto cartel. As
Nesta segunda (21), a secretaria de Política Econômica, empresas inicialmente apontadas como participantes da
do Ministério da Fazenda, admitiu que o governo espera um provável conduta anticompetitiva são a Andrade Gutierrez
crescimento do PIB (Produto Interno Bruto) de somente 1% no Engenharia, Construções e Comércio, a Camargo Corrêa e
ano que vem, uma redução considerável na comparação com a Construtora Norberto Odebrecht, além de, pelo menos,
a última projeção, que era de 1,6%. seis executivos e ex-executivos de alto escalão dessas em-
Com a expectativa de um PIB menor, as receitas esperadas presas.
também se reduzem, e o atual Orçamento do ano que vem Segundo o Cade, os contatos entre os concorrentes
ainda leva em conta uma alta de 1,6%. “Não cabe ao gover- teriam se iniciado em julho de 2009, com a divisão do gru-
no federal, neste momento, realizar revisão das estimativas de po formado pelas empresas Andrade Gutierrez, Camargo
2017. Vamos fazer isso no início do próximo ano, quando rea- Corrêa e Odebrecht em dois consórcios. Segundo relatos,
lizarmos a programação orçamentária para 2017”, declarou. ao longo do processo de preparação das propostas, as
Ele reafirmou o argumento do Ministério da Fazenda de empresas teriam alinhado parâmetros como premissas da
que a revisão do PIB não significa necessariamente uma redu- construção, divisão de riscos entre construtoras e investi-
ção na receita, já que o governo espera que entrem recursos dores e contingenciamento dos riscos. Tal alinhamento vi-
do novo projeto de repatriação e de concessões. sava criar uma paridade de condições e de preços entre
Fonte: Folha.com – (22/11/2016) as empresas, o que não é esperado entre concorrentes, e
buscava garantir a viabilidade de um pacto para a posterior
CAE aprova projeto que pode limitar taxa de juros do divisão da construção da usina entre elas.
cartão de crédito em 28% Apesar de o leilão ter sido vencido por outro consór-
A Comissão de Assuntos Econômicos (CAE) do Senado cio, as três concorrentes teriam adaptado o prévio ajuste
aprovou nesta terça-feira, 29, um projeto que pode limitar os quando foram posteriormente contratadas para a efetiva
juros do cartão de créditos a duas vezes a taxa do Certificado construção de Belo Monte na modalidade Concorrência
de Depósito Interbancário (CDI). A proposta seguirá para vo- Privada. Para tanto, as três empresas teriam novamente ali-
tação em Plenário. nhado variáveis que impactariam nas propostas de preço a
A taxa do CDI mantém-se próxima à taxa básica de juros serem apresentadas separadamente pelas empresas.
(Selic), que corresponde atualmente a 14% ao ano. Assim, se O leilão foi vencido pelo Consórcio Norte Energia,
o projeto fosse transformado em lei nesta terça, a taxa anual formado pelas empresas Eletrobras, Chesf, Eletronorte,
dos cartões de crédito ficaria limitada ao dobro dessa quantia: Queiroz Galvão, Galvão Engenharia e outras empresas. As
28%. empresas Andrade Gutierrez, Camargo Corrêa e Odebrecht
“Altera a Lei nº 4.595, de 31 de dezembro de 1964, que foram contratadas pela Norte Energia, tendo dividido entre
dispõe sobre a política e as instituições monetárias, bancárias si o montante de 50% da construção da usina hidrelétrica.
e creditícias, cria o Conselho Monetário Nacional e dá outras Segundo o Cade, os contatos anticompetitivos duraram
providências, para limitar os juros de cartão de crédito”, diz o até, pelo menos, julho de 2011, quando foram assinados
texto do projeto. os contratos referentes às obras de construção da usina
Autor do projeto, o senador Ivo Cassol (PP-RO) disse que hidrelétrica de Belo Monte.
os juros abusivos exigem limites regulatórios. Para Cassol, as A Norte Energia ainda não se manifestou sobre a inves-
taxas de juros “ainda são exorbitantes”, especialmente as co- tigação do Cade.
bradas em empréstimos na modalidade do rotativo do cartão Em nota, a Andrade Gutierrez informa que o acordo
de crédito. com o Cade está em linha com sua postura, desde o fecha-
Fonte: Istoé – (29/11/2016) mento do acordo de leniência com o Ministério Público, de
continuar colaborando com as investigações em curso. A
Cade investiga se houve cartel em leilão de Belo empresa também afirma que continuará realizando audito-
Monte rias internas para esclarecer fatos do passado que possam
O Conselho Administrativo de Defesa Econômica (Cade) ser do interesse da Justiça e dos órgãos competentes. “A
instaurou hoje (16) um inquérito para investigar a existência Andrade Gutierrez afirma ainda que acredita ser esse o me-
de um suposto cartel na licitação para a concessão da Usina lhor caminho para a construção de uma relação cada vez
Hidrelétrica de Belo Monte, realizado em 2010. Também será mais transparente entre os setores público e privado”, diz
investigado processo de contratação para a construção da a empresa.
usina, localizada no Rio Xingu (PA). Fonte: Agência Brasil – (16/11/2016)

11
ATUALIDADES E DEVERES DOS SERVIDORES PÚBLICOS

Governo do Rio Grande do Sul decreta estado de Pacote inclui privatização e prevê ao menos 1.200
calamidade financeira demissões
Um dia após anunciar um pacote de medidas para ten- Ontem, o governador José Ivo Sartori (PMDB) anun-
tar melhorar as contas do Estado, o governo do Rio Grande ciou um pacote de medidas na área financeira que prevê
do Sul decretou nesta terça-feira (22) estado de calamida- a extinção de nove fundações, o fim de três secretarias de
de financeira na administração pública estadual, conforme Estado, a privatização de uma companhia pública (a Corag,
decreto publicado no Diário Oficial. O decreto entrou em Companhia Riograndense de Artes Gráficas) e alteração
vigor hoje. nas regras do estatuto do funcionalismo público.
De acordo com a publicação, a decisão foi tomada Além disso, aumenta a alíquota da Previdência Social
considerando que “a crise da economia brasileira está atin- dos servidores ativos e inativos dos atuais 13,25% para 14%
gindo fortemente a capacidade de financiamento do setor - o impacto na receita deve chegar a R$ 130 milhões. Tam-
público”. O Rio Grande do Sul vive uma grave situação fi- bém limita as pensões previdenciárias ao teto constitucio-
nanceira que tem resultado no parcelamento dos salários nal dos desembargadores do Estado. A projeção é que haja
dos servidores e sucateamento das polícias, entre outros a demissão de 1,2 mil servidores.
problemas. Também transfere o pagamento do funcionalismo para
O decreto diz também que “a queda estimada do Pro- um calendário que varia do quinto dia útil (até R$ 1.300)
duto Interno Bruto (PIB), considerados os anos de 2015 e para o vigésimo dia útil (acima de R$ 6.000). A maior par-
2016, chegará a mais de 7%, com trágicas consequências te das matrículas (ou salários de R$ 2.900 mensais), pela
para a arrecadação de tributos”. proposta, deverá receber até o décimo dia útil. Hoje, pela
Ainda segundo o texto, a decisão levou em conta a ne- Constituição estadual, os servidores têm de ser pagos até o
cessidade de ações, no curto prazo, para fazer frente à crise último dia útil de cada mês.
e garantir a continuidade da prestação de serviços públicos Outra proposta é de pagar apenas 50% do 13º dos ser-
essenciais, notadamente nas áreas da segurança pública, vidores de 2016, remetendo a outra parcela para novem-
da saúde e da educação. bro de 2017. A economia em caso de aprovação seria de
A publicação define que secretários de estado e diri- R$ 600 milhões.
gentes máximos de órgãos e entidades da administração Os repasses ao Judiciário e Legislativo, além disso, se-
pública estadual, sob a coordenação da Secretaria da Casa rão calculados de acordo com a Receita Corrente Líquida
Civil, ficam autorizados a adotar medidas excepcionais ne- do estado, limitados ao orçamento. A previsão de econo-
cessárias à racionalização de todos os serviços públicos, mia para o Tesouro, nesse caso, é de R$ 575,7 milhões con-
salvo aqueles considerados essenciais. siderando dados referentes a 2015. O ganho anual no fluxo
As medidas em questão ainda serão estudadas, e ainda financeiro foi calculado em R$ 2,6 bilhões pelo governo.
não está claro como elas afetarão a população. Também reduz em 30% os créditos fiscais presumidos
referentes a 2016, 2017 e 2018 com impacto financeiro de
“Formalização da gravidade da crise” R$ 300 milhões por ano e antecipa o recolhimento do ICMS
Ouvido pelo UOL, o economista Raul Velloso, especia- pelas empresas do dia 21 para o dia 12. O pacote, com 38
lista em finanças públicas, afirmou que o estado de cala- projetos, será enviado nesta terça-feira (22) à Assembleia
midade financeira não está amparado em nenhuma lei. Em Legislativa – 30 deles tramitarão em regime de urgência.
sua opinião, a iniciativa é apenas uma forma de o governo Fonte: Portal UOL – (22/11/2016)
gaúcho comunicar a gravidade da crise do Estado.
Em nota, a Secretaria da Casa Civil do Rio Grande do Trump espalha incertezas na economia e no comér-
Sul afirmou que o “decreto orientará na seleção de novas cio mundial
medidas necessárias” e “formaliza a situação de extrema A chegada inesperada de Donald Trump à Casa Branca,
gravidade da gestão pública e a prioridade para as mu- sem um claro programa econômico, mas repleto de amea-
danças”. ças de veto a vários acordos comerciais, espalha incerte-
Ainda na nota, o secretário adjunto da Casa Civil, José za sobre a maior economia do mundo, que responde por
Guilherme Kliemann, disse que “o decreto é mais um ins- 24,5% do PIB mundial. As dúvidas surgem quando a eco-
trumento que engajará todas as áreas na busca de uma nomia mundial sofre com um fraco crescimento. O TTIP, o
gestão mais eficiente, enxuta e buscando identificar me- acordo comercial entre os EUA e a UE, já aparece como a
didas adicionais àquelas previstas nos decretos de contin- primeira vítima da era Trump.
genciamento que vêm sendo adotados desde janeiro de O comércio mundial está desacelerando praticamente
2015”. desde o início da Grande Recessão. Mas tanto o FMI quan-
“A Casa Civil solicitará às principais áreas do governo to o G-20 temem guerras comerciais, com medidas prote-
que busquem em suas estruturas alternativas de redução cionistas que reduzam ainda mais os volumes. A chegada
de despesas eletivas, que possam ser eliminadas sem pre- de Trump e seu “América, primeiro” é um risco adicional: o
juízo aos serviços essenciais”, afirmou Kliemann. novo presidente dos Estados Unidos não é exatamente um
Ele também declarou na nota que na próxima semana fã dos acordos comerciais e sugeriu que vai desmantelar
serão realizadas reuniões com áreas específicas para “elen- o pacto assinado por Obama com os países do Sudeste
car medidas de contenção”. Asiático (conhecido como TTP) e que vai congelar as nego-
ciações do TTIP, o acordo entre EUA e a UE.

12
ATUALIDADES E DEVERES DOS SERVIDORES PÚBLICOS

Em Bruxelas, o presidente do Conselho Europeu, Do- Apesar desses presságios, Julio Cañero, diretor do Ins-
nald Tusk, e o chefe da Comissão Europeia, Jean-Claude tituto Franklin da Universidade de Alcalá, está “moderada-
Juncker, pediram na quarta-feira uma reunião de cúpula mente otimista”. Admite que implantar o programa como
com os Estados Unidos para discutir essa e outras ques- foi explicado na campanha significaria uma curva perigosa
tões. Mas a Europa dá praticamente por perdido o TTIP, para a economia mundial, mas lembra que Trump “deverá
antes da resposta popular que gerou um pacto similar com negociar tudo com seu partido, que está contra o protecio-
o Canadá e as duras críticas que recebeu o acordo com os nismo e as tarifas”. Quando chegar ao Gabinete Oval, acres-
Estados Unidos na França e na Alemanha. centa, “ele terá que ser mais pragmático. Entre outras coisas
A negociação “entrou em uma pausa natural”, disse o porque se você colocar barreiras à entrada de produtos chi-
vice-presidente da UE, Jyrki Katainen. A agenda de Trump neses, Pequim pode vender a dívida dos EUA e o proble-
“vai contra o livre comércio”, disse Katainen, que ainda as- ma será de Trump. Wall Street vai recolocá-lo em seu lugar,
sim fez um chamado para “não subestimar” o interesse nos porque não pode prejudicar as grandes empresas com suas
acordos comerciais entre as autoridades e o setor empre- medidas”.
sarial norte-americano. De acordo com o escritório de advocacia Baker & Mc-
A Europa não é a única região do mundo que sente Kinzey, “a volatilidade e a incerteza são a nova situação de
a ameaça comercial: Trump sublinhou repetidas vezes que normalidade para as organizações globais” e afirmam que
vai enfrentar a “concorrência desleal” da China. A Europa as empresas procuram “estabilidade e continuidade e Trump
começa a emitir sinais na mesma direção, e isso poderia foi eleito por ser um forasteiro que quer a agitar o establish-
impor tarifas maiores a produtos subsidiados pelos chine- ment político, mas a maneira como vai jogar no mundo dos
ses como o aço. Se, além disso, os bancos centrais também negócios não está clara”. Esse é o problema.
se envolverem em uma guerra de guerrilha para desvalo- Fonte: El País Brasil – (10/11/2016)
rizar as taxas de câmbio, todos esses movimentos irão na
mesma direção: uma guerra comercial — ainda devemos CULTURA E SOCIEDADE
ver se alta ou baixa intensidade — e um retorno ao nacio-
nalismo econômico que coloque obstáculos ao comércio. Morre aos 82 anos o cantor e compositor Leonard
Cohen
As dúvidas fazem o PIB cair Morreu na noite desta quinta-feira, 10, Leonard Cohen.
O HSBC, o maior banco da Europa, publicou um re- A notícia foi dada em sua página no Facebook, que pede
latório forte sobre a chegada de Trump: “O aumento da privacidade para a família neste momento. “É com profunda
incerteza costuma muitas vezes levar a um crescimento tristeza que reportamos que o lendário poeta, compositor e
mais fraco pelo atraso nos investimentos e o gasto dos artista, Leonard Cohen faleceu. Nós perdemos um dos mais
consumidores, assim como um crescimento mais fraco do reverenciados e prolíficos visionários da música”, diz o co-
emprego”. municado.
Os especialistas coincidem em destacar que um dos O compositor tinha acabado de lançar seu 14º disco,
maiores riscos é que haja uma queda dos impostos, com o You Want It Darker, depois de ter excursionado por cinco
crescimento dos gastos, o que poderia gerar inflação. Essa anos. Não se sabe ainda a causa da morte, mas, em suas
situação iria levar a um aumento das taxas de juros e um últimas aparições, o canadense discursava com a voz fraca
possível aumento do dólar. e ofegante.
Um dos controladores do mercado, a agência de classi- Fonte: Estadão.com – (11/11/2016)
ficação Fitch, não demorou para se posicionar sobre Trump.
Não vai rebaixar a classificação da dívida dos EUA, mas ad- Aborto até o terceiro mês não é crime, decide turma
vertiu que se ele colocasse em prática as medidas anuncia- do Supremo
das, “teriam um efeito negativo para as finanças públicas”. A maioria da primeira turma do STF (Supremo Tribunal
Admite as “incertezas” sobre o programa de Trump, Federal) firmou o entendimento, nesta terça-feira (29), de
o grau que tentará realizá-las e sua capacidade de imple- que praticar aborto nos três primeiros meses de gestação
mentá-las. “Esta última vai depender da cooperação entre não é crime. Votaram dessa forma os ministros Luís Roberto
o presidente e as maiorias republicanas no Congresso e no Barroso, Rosa Weber e Edson Fachin.
Senado, e até que ponto os democratas do Senado pode- A decisão é sobre um caso específico, em um habeas
rão bloquear as medidas propostas”, afirma. corpus que revogou a prisão preventiva de cinco pessoas
que trabalhavam numa clínica clandestina de aborto em Du-
Ambiente político menos estável que de Caxias (RJ), mas pode ser considerada um passo à
Na mesma linha se manifesta Bill Papadakis, estrategis- frente na descriminalização do ato, desde que no início da
ta de investimentos do banco suíço Lombard Odier. “A me- gravidez.
nor visibilidade das políticas de Trump poderia gerar algu- Embora a decisão tenha se dado em um caso específi-
ma volatilidade nos mercados financeiros no curto prazo. co, outros magistrados, de outras instâncias, poderão, a seu
Embora a longo prazo, os riscos são menos claros, já que critério, adotar o entendimento da primeira turma do STF.
o impacto sobre o crescimento teria que ser compensado O relator, ministro Marco Aurélio, já havia concedido li-
pelo aumento da incerteza causada por um ambiente polí- minar em 2014 para soltar os cinco médicos e funcionários
tico menos estável”. da clínica fluminense. Seu fundamento era que não exis-

13
ATUALIDADES E DEVERES DOS SERVIDORES PÚBLICOS

tiam os requisitos legais para a prisão preventiva (como Papa concede a padres decisão de perdoar quem
ameaça à ordem pública e risco à investigação e à aplica- comete abortos
ção da lei). Nesse processo, nenhuma mulher que praticou O papa Francisco concedeu hoje (21) aos sacerdotes
aborto na clínica foi denunciada. a decisão de absolver ou não as pessoas que cometeram
Em agosto deste ano, quando foi a julgamento o mé- aborto e procuram a Igreja Católica para se redimir. A orien-
rito do habeas corpus, Barroso pediu vista. Em seu voto, tação foi publicada na carta apostólica Misericordia et Mise-
nesta terça, ele concordou com a revogação das prisões ria, divulgada pelo Vaticano. O texto marca o encerramento
pelos motivos apontados por Marco Aurélio, mas trouxe do Ano Santo do Jubileu, que foi dedicado ao tema da mi-
um segundo fundamento. Para ele, os artigos do Código sericórdia.
Penal que criminalizam o aborto no primeiro trimestre de A carta apostólica estabelece uma série de novas ins-
gestação violam direitos fundamentais da mulher. truções para que a misericórdia seja adotada como prática
As violações são, segundo o voto de Barroso, à auto- diária entre os católicos. Dessa forma, os sacerdotes ficam
nomia da mulher, à sua integridade física e psíquica, a seus livres para decidir perdoar ou não uma pessoa que cometeu
direitos sexuais e reprodutivos e à igualdade de gênero. aborto. Isso abre caminho para médicos e mulheres que já
“Na medida em que é a mulher que suporta o ônus inte- cometeram ou participaram de abortos. Até hoje, os dois
gral da gravidez, e que o homem não engravida, somente eram impedidos automaticamente de comungar na Igreja
haverá igualdade plena se a ela for reconhecido o direito e o status só podia ser revertido em casos específicos por
de decidir acerca da sua manutenção ou não”, escreveu o bispos ou delegados.
ministro sobre o direito à igualdade de gênero. “Com todas as minhas forças, digo que o aborto é um
“O direito à integridade psicofísica protege os indiví- pecado grave, porque coloca fim a uma vida inocente”, afir-
duos contra interferências indevidas e lesões aos seus cor- mou o papa. Mas peço aos sacerdotes que sejam guias e
pos e mentes, relacionando-se, ainda, ao direito à saúde e à deem apoio e conforto no acompanhamento dos peniten-
segurança. Ter um filho por determinação do direito penal tes”, ressaltou o líder católico “Para que nenhum obstáculo
constitui grave violação à integridade física e psíquica de se coloque entre o pedido de reconciliação e o perdão de
uma mulher”, afirmou também o ministro. Deus, concedo, a partir de hoje, a todos os sacerdotes, na
força de seus ministérios, a faculdade de absolver os que
Além disso, segundo Barroso, a criminalização do
os procuram pelo pecado do aborto”, determinou Francisco.
aborto causa uma discriminação contra as mulheres po-
Além da questão do aborto, o papa validou as confis-
bres, que não podem recorrer a um procedimento médico
sões celebradas por sacerdotes lefebrvianos e oficializou o
público e seguro, enquanto as que têm condições pagam
trabalho dos “missionários da misericórdia”, postos criados
clínicas particulares.
durante o Jubileu para “escutar e perdoar os fiéis”. No texto,
Ainda de acordo com o voto de Barroso, que foi acom-
Francisco disse que a misericórdia é um “valor social” que
panhado por Weber e Fachin, os principais países demo-
deve “restituir a dignidade de milhões de pessoas”. Por isso,
cráticos e desenvolvidos, como Estados Unidos, Alemanha, ele também criou o Dia Mundial dos Pobres, que será cele-
Reino Unido, Canadá, França, Itália, Espanha, Portugal e brado em toda a Itália católica.
Holanda, não criminalizam o aborto na fase inicial da ges- Em um claro recado à ala conservadora do Vaticano, o
tação. O prazo de três meses foi tirado da comparação com papa escreveu em sua carta apostólica que “nada que um
esses países. pecador arrependido coloque diante da misericórdia de
Os dois outros ministros da primeira turma, Marco Au- Deus pode permanecer sem o seu abraço e o seu perdão.
rélio e Luiz Fux, não se manifestaram sobre a descriminali- Comunicar a certeza do Deus que ama não é um exercício
zação do aborto no início da gravidez. No caso específico, retórico, mas uma condição de credibilidade do próprio sa-
eles também votaram pela revogação das prisões preven- cerdócio”, disse Francisco. Jorge Mario Bergoglio também
tivas, com base apenas na ausência dos requisitos legais usou a carta para responder a quatro cardeais conservado-
para mantê-las. res que lhe haviam questionado sobre a exortação apostó-
Barroso destaca, em sua decisão, que o aborto não lica Amoris Laetitia (A Alegria do Amor), lançada em 8 de
é algo bom, e que o papel do Estado deve ser evitá-lo, abril e que fala sobre a família na sociedade atual. Assinada
mas com educação sexual, distribuição de contraceptivos e pelo cardeal Raymund Leo Burke, a carta acusa Francisco
apoio às mulheres que desejarem manter a gravidez, mas de apoiar o reconhecimento do divórcio. Como o papa não
que não tenham condições. respondeu ao documento, os cardeais resolveram torná-lo
O que foi julgado na primeira turma foi um habeas cor- público.
pus para reverter a prisão preventiva dos cinco acusados. O “Quando o caminho da vida nupcial é interrompido pelo
mérito desse caso continua a ser julgado na Justiça do Rio. sofrimento, pela traição e solidão, a experiência da miseri-
No próximo dia 7, o plenário do Supremo julgará a córdia nos permite olhar para todas as dificuldades com a
possibilidade de aborto em casos em que mulher for infec- atitude do amor de Deus, que não se cansa de acolher e de
tada pelo vírus da zika. acompanhar”, ratificou Francisco. Dessa forma, o papa pede,
Em 2012, a corte decidiu, por 8 votos a 2, que a inter- mais uma vez, que cada caso de separação matrimonial seja
rupção de gravidez no caso de fetos com anencefalia com- analisado de maneira independente pelos sacerdotes.
provada não é crime. Na ocasião, Barroso, que ainda não O Ano Santo Extraordinário da Misericórdia foi encerra-
era ministro, advogou a favor da descriminalização. do ontem (20), com uma missa celebrada por Francisco no
Fonte: Folha.com – (29/11/2016) Vaticano diante de 70 mil pessoas. Tradicionalmente, o Ju-

14
ATUALIDADES E DEVERES DOS SERVIDORES PÚBLICOS

bileu acontece somente a cada 25 anos. O último tinha sido A Chapecoense tinha previsto voar direto para Medel-
em 2000, portanto, só ocorreria em 2025. Mas Francisco lín. A falta de um acordo de comércio aeronáutico impediu,
resolveu convocar um Jubileu extraordinário com o tema no entanto, que a empresa boliviana Lamia levasse a equipe
da Misericórdia. O Ano Santo foi iniciado em novembro de de São Paulo até Medellín, informou a repórter Talita Bedinelli.
2015 e encerrado agora. Esta operação só é permitida no Brasil para empresas brasilei-
Fonte: Agência Brasil – (21/11/2016) ras ou colombianas. Por isso, a equipe teve que viajar de São
Paulo para Santa Cruz (Bolívia), em um voo comercial da Boli-
Tragédia com avião da Chapecoense mata 71 na Co- viana de Aviación, o que atrasou o charter, que partiu do aero-
lômbia porto de Viru Viru para Medellín. As autoridades aeronáuticas
Uma nova tragédia aérea voltou a abalar o mundo do da Bolívia ressaltaram que o avião partiu de Santa Cruz com
futebol na segunda-feira. O avião em que viajava a equi- tudo “em ordem”, sem defeitos e em boas condições de voo.
pe brasileira da Chapecoense caiu quando estava prestes Recentemente esta mesma empresa tinha sido usada pela se-
a chegar ao seu destino, o aeroporto de Medellín, onde na leção argentina para realizar várias viagens.
quarta-feira estava previsto jogar a partida final da Copa Em um primeiro momento pensou-se que no avião viaja-
Sul-Americana com o Atlético Nacional. No avião viajavam vam 81 pessoas. Depois de verificar as pessoas que tinham em-
77 pessoas, 71 morreram e seis sobreviveram ao acidente: barcado no Brasil e na Bolívia, ficou comprovado que tinham
dois membros da tripulação, três jogadores e um jornalista. embarcado 68 passageiros de origem brasileira e 9 membros
Pelo menos duas dessas vítimas estão em estado grave. As da tripulação de origem boliviana, totalizando 77 pessoas. As
autoridades ainda não confirmaram as razões que fizeram quatro pessoas que não iam no avião correspondem a Luciano
com que o avião, um charter da empresa boliviana Lamia, Buligon, prefeito de Chapecó, a cidade sede da Chapecoense;
caísse quando estava tão perto de seu destino. Plínio Nes Filho, dirigente da Chapecoense; Gelson Merisio,
O avião que transportava a Chapecoense, um Avro Re- deputado de Santa Catarina e Iván Carlos Agnoletto, jornalista
gional Jet 85 (RJ85), deveria ter chegado a Medellín por A equipe brasileira, da cidade de Chapecó, no Estado de
volta das dez da noite da Colômbia. Pouco antes de iniciar Santa Catarina, estava indo a Medellín para disputar a final da
sua descida, perdeu contato com a torre de controle. Os Copa Sul-Americana contra o Atlético Nacional. A alegria dos
pilotos tinham alertado, de acordo com várias fontes, so- jogadores frente a tamanha conquista ficou refletido em um
vídeo distribuído pelas redes, no qual os jogadores se mos-
bre “falhas elétricas”, embora as autoridades não tenham
travam animados antes de deixar Santa Cruz para Medellín. A
confirmado a causa exata do acidente. Uma das hipóteses
Chapecoense virou a surpresa da última temporada no futebol
considerada é que o avião ficou sem combustível. O in-
latino-americano. Ninguém esperava que um time tão modes-
cidente ocorreu nas proximidades do Cerro El Gordo, no
to, que esteve prestes a desaparecer por questões econômicas
município de La Unión, em um lugar relativamente perto
alguns anos atrás, pudesse deixar no caminho clubes como
do aeroporto José María Córdova, a uma hora de Medellín,
San Lorenzo de Almagro, independente ou Junior de Barran-
a segunda maior cidade da Colômbia.
quilla, também colombiano. Sua jornada na Copa Sul-Ame-
No momento do acidente, as condições meteoroló- ricana levou a que muitos o classificassem como o Leicester
gicas na área eram boas, de acordo com as autoridades brasileiro, referindo-se ao atual campeão da liga inglesa. O
locais. No entanto, a falta de luz e a forte chuva que caiu Atlético Nacional pediu que a Copa fosse entregue, simbolica-
durante toda a noite dificultaram as já em si complicadas mente, ao clube brasileiro.
tarefas de resgate. O avião caiu em um lugar íngreme, a O presidente da Colômbia, Juan Manuel Santos, falou
meia hora a pé da estrada mais próxima. Até 150 pessoas logo após a notícia do incidente com seu homólogo brasileiro,
da Aviação Civil, da Força Aérea Colombiana e de agências Michel Temer. Este, por sua vez, expressou sua tristeza com as
humanitárias participaram do resgate. Nas buscas, 70% dos famílias das vítimas: “Nesta hora triste em que a tragédia se
corpos foram encontrados na fuselagem do avião, enquan- abate sobre dezenas de famílias brasileiras manifesto a minha
to espalhados pelo terreno estavam os outros 30%, entre solidariedade”. O Ministério das Relações Exteriores da Colôm-
eles os dos sobreviventes. As equipes de resgate encontra- bia trabalha agora para facilitar a repatriação dos corpos com a
ram destroços da aeronave 500 metros ao redor do local maior brevidade possível.
do acidente. Depois de conseguir retirar os corpos, após o As mensagens de solidariedade e apoio às vítimas não de-
meio-dia na hora local, foram encontradas as duas caixas moraram, especialmente no mundo do futebol. Do Real Ma-
pretas, que fornecerão mais informações sobre as causas drid, que observou um minuto de silêncio, até a solidariedade
do acidente. de figuras como Messi ou Maradona, o futebol voltou a se
As pessoas que foram resgatadas vivas são: Ximena mostrar unido. O acidente da Chapecoense nos remete a ou-
Suárez, auxiliar de voo; Erwin Tumiri, técnico do avião; os tras tragédias semelhantes sofridas por times de futebol. Em
jogadores Alan Luciano Ruschel, Jackson Folmman e Hé- 1949, caiu o avião em que voltava a equipe italiana do Torino
lio Hermito Zampier. O corpo deste último foi recuperado depois de jogar em Lisboa. Morreram 42 pessoas, incluindo
cinco horas após o acidente. Além deles, sobreviveu à tra- quase a totalidade dos jogadores. Nove anos depois, o avião
gédia o jornalista Rafael Malmorbida, que em um primeiro que transportava o Manchester United de Munique sofreu
momento foi identificado como Rafael Henzel. O goleiro um acidente. Oito jogadores, dois diretores e o técnico
da equipe brasileira Danilo Padilha foi resgatado vivo, mas morreram. Sobreviveram sete jogadores, incluindo a lenda
morreu antes de chegar a algum dos hospitais para onde do United, Bobby Charlton.
os feridos foram levados. Fonte: El País Brasil – (30/11/2016)

15
ATUALIDADES E DEVERES DOS SERVIDORES PÚBLICOS

Casos de chikungunya devem aumentar em 2017, O estudo registrou ainda 601 mortes pela doença este
prevê Ministério da Saúde ano, contra 933 no mesmo período de 2015 – uma redu-
O ministro da Saúde, Ricardo Barros, disse hoje (24) que ção de 35,6%. Também reduziram pela metade os casos de
o governo prevê um aumento significativo de casos de in- dengue grave, que passaram de 1.616 para 803, e a quase um
fecção pelo vírus Chikungunya no Brasil em 2017. Os casos terço os casos de dengue com sinais de alarme, que caíram
confirmados da doença aumentaram 15 vezes de 2015 para de 20.352 para 7.730.
este ano (de 8.528 para 134.910) e os suspeitas, quase dez
vezes (de 26.763 para 251.051). Aumento da chikungunya
Barros destacou ainda que, para 2017, a expectativa da O levantamento aponta 251.051 casos suspeitos de febre
pasta é de que os casos de infecção por dengue e pelo vírus chikungunya identificados no país este ano, sendo 134.910
Zika se mantenham estáveis em relação ao que foi registrado confirmados. No mesmo período do ano passado, o total foi
em 2016. “Estamos nos preparando para um aumento de ca- de 26.763 casos suspeitos e 8.528 confirmados.
sos de chikungunya”, enfatizou o ministro. Ao todo, 138 mortes pela doença foram registradas nos
Este ano, pelo menos 138 óbitos por febre chikungunya seguintes estados: Pernambuco (54), Paraíba (31), Rio Grande
foram registrados nos seguintes estados: Pernambuco (54), do Norte (19), Ceará (14), Bahia (5), Rio de Janeiro (5), Mara-
Paraíba (31), Rio Grande do Norte (19), Ceará (14), Bahia (5), nhão (5), Alagoas (2), Piauí (1), Amapá (1) e Distrito Federal
Rio de Janeiro (5), Maranhão (5), Alagoas (2), Piauí (1), Amapá (1).
(1) e Distrito Federal (1). Atualmente, 2.281 municípios brasi- Atualmente, 2.281 municípios brasileiros já registraram
leiros já registraram casos da doença. casos de infecção pelo vírus Chikungunya.
Dados divulgados pelo ministério apontam que 855 ci-
dades brasileiras estão em situação de alerta ou de risco de Incidência de Zika
surto de dengue, chikungunya e Zika. O número representa Em relação ao vírus Zika, foram identificados 208.867
37,4% dos municípios pesquisados. casos prováveis no país até o dia 22 de outubro. O número
Fonte: Jornal do Brasil – (24/11/2016) representa uma taxa de incidência de 102,2 casos para cada
100 mil habitantes. Foram confirmadas ainda três mortes
Aedes: 885 cidades estão em situação de alerta ou
pela doença este ano, além de 16.696 casos prováveis de in-
risco de surto
fecção entre gestantes.
Dados divulgados hoje (24) pelo Ministério da Saúde re-
O Sudeste tem a maior parte de casos prováveis (83.884),
velam que 855 cidades brasileiras estão em situação de alerta
seguido pelo Nordeste (75.762), Centro-Oeste (30.969), Nor-
ou de risco de surto de dengue, chikungunya e zika. O núme-
te (12.200) e Sul (1.052). Considerando a proporção por habi-
ro representa 37,4% dos municípios pesquisados pela pasta
tantes, o Centro-Oeste encabeça a lista, com 200,5 casos para
no Levantamento Rápido de Índices para Aedes aegypti (LI-
RAa), que é o mosquito transmissor das três doenças. cada 100 mil habitantes. Em seguida estão Nordeste (133,9),
Das 22 capitais que participaram do estudo, Cuiabá está Sudeste (97), Norte (69,8) e Sul (3,6).
em situação de risco e outras nove em situação alerta: Ara- A transmissão autóctone (originária no Brasil) foi confir-
caju, Salvador, Rio Branco, Belém, Boa Vista, Vitória, Goiânia, mada em abril de 2015 e as notificações de casos ao Minis-
Recife e Manaus. Outras 12 aparecem como em situação sa- tério da Saúde se tornaram obrigatórias em fevereiro deste
tisfatória: São Luís, Palmas, Fortaleza, João Pessoa, Teresina, ano, por isso não há comparações com anos anteriores.
Belo Horizonte, São Paulo, Rio de Janeiro, Macapá, Florianó-
polis, Campo Grande e Brasília. Adesão
O ministério não recebeu informações sobre as capitais Das 3.704 cidades que estavam aptas a participar do Le-
Maceió, Porto Velho e Curitiba. Já Natal e Porto Alegre utili- vantamento Rápido de Índices para Aedes aegypti (LIRAa),
zam outra metodologia para medição de focos do mosquito. 2.284 integram a edição deste ano – o equivalente a 62,6%
Depósitos de água como toneis, tambores e caixas do total.
d’água foram os principais tipos de criadouro do mosquito Realizado entre outubro e novembro, o estudo é consi-
registrados nas regiões Nordeste e Sul. No Sudeste, predo- derado ferramenta fundamental para o controle do mosqui-
minou o depósito domiciliar, categoria em que se enqua- to. Com base nas informações coletadas, o gestor pode iden-
dram vasos de plantas, garrafas, piscinas e calhas. No Norte e tificar o tipo de depósito predominante e priorizar medidas
no Centro-Oeste, a maioria dos focos foi encontrada no lixo. para conter a proliferação do vetor no município.
Atualmente, o levantamento é feito por meio de adesão
Redução da dengue voluntária, mas a expectativa do governo é que a participa-
Os dados mostram uma queda de 5,5% no número de ção passe a ser obrigatória para cidades com mais de 2 mil
casos de dengue este ano, comparado ao mesmo período imóveis. A proposta será apresentada na próxima reunião da
do ano passado: foram 1.458.355 casos ocorrências até que Comissão Intergestores Tripartite, constituída por represen-
até 22 de outubro deste amp e 1.543.000 casos até a mesma tantes do Ministério da Saúde e de representantes de secre-
data em 2015. tarias estaduais e municipais, marcada para 8 de dezembro.
Entre as regiões do país, o Sudeste e o Nordeste apre- “O número [de municípios participantes] é crescente,
sentam o maior número de casos, com 848.587 e 322.067, mas queremos deixar obrigatório”, reforçou o ministro da
respectivamente. Em seguida, aparecem o Centro-Oeste Saúde, Ricardo Barros.
(177.644), o Sul (72.114) e o Norte (37.943). Fonte: Jornal do Brasil – (24/11/2016)

16
ATUALIDADES E DEVERES DOS SERVIDORES PÚBLICOS

VIOLÊNCIA Dois dos cinco rapazes desaparecidos estavam sendo


investigados pela Polícia Civil por suspeita de envolvimen-
Corpos de 4 dos 5 jovens mortos em chacina em SP to na morte de Rodrigo: Cesar e Caíque. Apesar disso, o
são enterrados guarda que foi preso e mais outros dois agentes de Santo
Os corpos de quatro jovens mortos em uma chacina André decidiram fazer uma investigação paralela e ilegal
ocorrida na Grande São Paulo, em 21 de outubro, foram se- por conta própria.
pultados em uma cerimônia coletiva na tarde deste sábado O guarda preso era amigo de Rodrigo e instrutor de ti-
(12), no Cemitério da Vila Alpina, na Zona Leste de São Pau- ros na Guarda Civil Municipal de Santo André. A Justiça em
lo. Familiares e amigos fizeram orações e gritaram “Justiça”. Mogi das Cruzes decretou a prisão temporária do guarda
O secretário de Segurança Pública de SP, Mágino Alves, por 30 dias. Não há confirmação se ele constituiu advoga-
confirmou a identidade da quinta vítima, Jones Ferreira Ja- do para defendê-lo.
nuário, 30 anos, nesta sexta-feira (11). O corpo dele será se-
pultado no Cemitério da Vila Formosa neste domingo (13). PMs
Jonathan Moreira Ferreira, de 18 anos; César Augusto Além de guardas civis, a investigação apura a suspeita
Gomes Silva, de 19; Caique Henrique Machado Silva, 18; de que policiais militares também poderiam estar envolvi-
Robson Fernando Donato de Paula, 16, que é cadeirante, e dos na chacina dos cinco jovens. Isso porque cartuchos de
Januário desapareceram quando se dirigiam a uma festa em calibre .40 – munição adotada por PMs – também foram
Ribeirão Pires, no ABC Paulista. Os corpos deles foram en- encontrados perto do local onde os cinco corpos estavam
contrados em 6 de novembro, numa área rural em Mogi das em Mogi das Cruzes.
Cruzes, interior paulista. A Corregedoria da PM apura o caso. O Tribunal de Jus-
tiça Militar chegou a decretar segredo na investigação.
Investigação Além disso, uma mensagem enviada por Jonathan para
Nesta sexta-feira (11), a diretora do Departamento Es- uma amiga às 23h do dia 21 de outubro dizia que estava
tadual de Homicídios e Proteção à Pessoa (DHPP), Elisabete passando por uma blitz policial. Ele relatou num áudio ter
Sato, afirmou que um guarda civil metropolitano de Santo
sofrido “enquadro” e “esculacho”. O G1 teve acesso à gra-
André usou perfil falso nas redes sociais de uma “mulher bo-
vação (ouça abaixo).
nita” e com “seios voluptuosos” para atrair os cinco jovens.
Outro indicativo é o fato de os corpos terem sido lo-
O guarda é Rodrigo Gonçalves Oliveira, que teve a pri-
calizados a 3 km de distância de um sítio usado por PMs,
são temporária decretada pela Justiça de Mogi das Cruzes.
onde também foram encontrados e apreendidas munições.
Segundo a polícia, ele confessou ter “armado uma cilada”
O secretário ressaltou, no entanto, que era prematuro
para os rapazes.
falar em envolvimento da PM no caso. Agora a investiga-
Mais dois guardas civis de Santo André, no ABC, são in-
vestigados por suspeita de participação no desaparecimen- ção aponta que ele “estava certo”. “Indicação inicial apon-
to e na chacina dos jovens. tava para um outro lado e hoje estamos assistindo uma
“Ainda é cedo para que a gente afirme que está total- outra realidade”, falou Mágino.
mente desvendado o crime, mas outras pessoas já estão Familiares dos mortos que relataram ter sido ameçados
prestando depoimento visando a apuração de participação por PMs podem ser incluídos no programa de proteção à
nesses múltiplos homicídios”, disse nesta manhã o secretário testemunha, segundo o Conselho Estadual de Defesa dos
da Segurança Pública (SSP) do estado de São Paulo, Mágino Direitos da Pessoa Humana (Condepe).
Alves Barbosa Filho, em entrevista coletiva sobre o caso. Os parentes devem se reunir na tarde desta sexta-feira
De acordo com a investigação, o grupo saiu de carro da com peritos do IML para tratar da liberação dos corpos e
Zona Leste da capital em direção a uma suposta festa em saber como eles foram executados. As vítimas devem ser
Ribeirão Pires para encontrar as garotas que conheceram na enterradas numa cerimônia conjunta neste final de sema-
rede social. O veículo foi localizado abandonado no dia 23 na. A data e local ainda não foram definidos.
de outubro. Fonte: G1 – (12/11/2016)
Robson foi morto a facadas – a cabeça dele teria sido
cortada. Os outros quatro rapazes foram assassinados por Caso Yoki: Elize Matsunaga começa a ser julgada em
disparos de armas calibres 38 e 12 – munições usadas por SP nesta segunda
guardas civis. Um dos crimes mais violentos da história policial bra-
sileira pode ter um veredicto esta semana, em São Paulo,
Vingança quatro anos e meio depois de chocar o país. A partir das
Segundo o DHPP, a chacina dos cinco rapazes ocorreu 9h30 desta segunda-feira (28), Elize Matsunaga, 34, vai a
para vingar a morte do guarda civil Rodrigo Lopes Sabino, júri popular no Fórum Criminal da Barra Funda (zona oeste
de 30 anos. Ele foi assassinado a tiros em Santo André no de São Paulo) acusada de matar, esquartejar e ocultar o
dia 24 de setembro. Seu carro acabou levado e queimado corpo do marido, o empresário Marcos Matsunaga, 42, em
em seguida perto da região onde as vítimas moravam em maio de 2012. Ela está presa desde aquele ano na Peniten-
São Paulo. ciária do Tremembé, e, se condenada à pena máxima pedi-
O caso de Rodrigo foi tratado inicialmente como la- da pela acusação, pode pegar até 33 anos de reclusão –30
trocínio (roubo seguido de morte). Dois criminosos teriam por homicídio triplamente qualificado e três por ocultação
participado do crime e fugido. e destruição de cadáver.

17
ATUALIDADES E DEVERES DOS SERVIDORES PÚBLICOS

A previsão do Tribunal de Justiça de São Paulo é que o Para defesa, “qualquer pessoa poderia praticar o mes-
julgamento dure até cinco dias, principalmente em função mo ato”
do número de testemunhas arroladas, 20 --dez da defesa e A advogada Roselle Soglio, que defende Elize, decla-
dez da acusação. Outras duas testemunhas podem ser ouvi- rou que a defesa argumentará contra as três qualificadoras
das, no caso, dois peritos. apresentadas pela acusação.
O crime aconteceu no apartamento do casal, em 19 de “O promotor quer colocar a ré como uma mulher cruel,
maio de 2012, na Vila Leopoldina, zona oeste da capital. Sa- ou que não prestava, e nada disso é verdade. Basta ler o
cos com partes do corpo do empresário começaram a ser processo para entender que ela era uma mãe dedicada e
encontrados dois dias depois em Cotia, na Grande São Pau- uma mulher incrível, mas que, com o desenrolar de uma
lo, pela GCM (Guarda Civil Metropolitana). O empresário era briga, acabou resultado nisso”, disse.
um dos sócios do grupo alimentício Yoki. Segundo a advogada, Elize, que tem bom comporta-
As investigações começaram com a suspeita de seques- mento e trabalha no setor de costura do presídio, “está
tro, mas, com a descoberta das partes do corpo, chegaram muito ansiosa” pelo júri.
a Elize, que disse ter conhecido o empresário por meio de “Ela quer demais que esse julgamento aconteça a partir
um site de relacionamentos. Dias depois de ser presa, ela desta segunda, e quer principalmente que as pessoas en-
confessou ter matado o marido com um tiro e o esquar- tendam o ato praticado por ela e que a verdade seja dita”,
tejado, sob a alegação de ter agido em desespero, já que afirmou.
Matsunaga a maltrataria, a trairia com uma prostituta e a Indagada se o fato de o corpo do empresário ter sido
ameaçaria com a separação e a tomada da guarda da filha esquartejado e ocultado pode dificultar o entendimento
única do casal. dos jurados por um eventual abrandamento da pena, a ad-
Para o Ministério Público, Elize matou o marido porque vogada resumiu: “Nada é fácil, e apenas aqueles que se dis-
o casamento deles já estaria “arruinado”, e, com a morte do puserem a entender e estiverem dispostos a ouvir os dois
empresário, ela ficaria com a guarda da filha – única herdeira lados e compreender toda a circunstância do que ocorreu é
do marido. que conhecerão a verdade sobre os fatos”, falou.
“Ela agiu por vingança, pois tinha prévio conhecimento “Teve um ato de um mãe desesperada, e, nesse senti-
de que o Marcos estava saindo com outra pessoa. Isso não do, acredito que qualquer pessoa poderia praticar o mes-
é só percepção: eles dormiam em quartos separados, o ca-
mo ato –ele não está fora de nenhum padrão se houver a
samento estava arruinado, e a mim ficou claro que o desa-
ameaça de seu filho ser tomado e você ser internada como
parecimento dele resolvia dois problemas: ela se livraria do
louca. Ela não queria matar o Marcos, mas isso aconteceu
ódio com que estava e ficaria com a guarda natural da filha,
no transcorrer de uma briga; é algo bastante complexo”,
que era a herdeira, então, ficaria com a herança – que era 30
definiu.
vezes mais que a pensão que ela receberia caso se separas-
Entre as testemunhas arroladas pela defesa de Elize,
sem”, afirmou o promotor de justiça José Carlos Cosenzo.
estão peritos do local do crime, médicos legistas, uma de-
O MP vai defender que o homicídio teve três qualifica-
doras: motivo torpe, uso de recurso que impossibilitasse a legada de polícia e amigos que conheceriam a rotina do
defesa da vítima (no caso, um tiro) e meio cruel, já que, para casal.
a acusação, o empresário foi esquartejado ainda vivo. Con- A filha do casal vive com os pais do empresário até
forme o promotor, isso estaria provado tecnicamente pela que a Justiça, em outro processo, decida em definitivo pela
autópsia, que descobriu sangue nas vias respiratórias e no guarda da criança. Órfã de pai e mãe, a família de Elize vive
pulmão, situação comum, por exemplo, com a degola. no interior do Paraná.
“Está demonstrado claramente no processo que esse é Fonte: Portal UOL – (28/11/2016)
um crime de ódio incontido”, classificou Cosenzo.
“Ele até puxava a cadeira para ela”, afirma advogado da Elize Matsunaga volta a chorar e júri é suspenso
família após detetive depor
Contratado pela família do empresário, o advogado Luiz Julgamento de Elize Matsunaga, acusada de matar e
Flávio D’Urso vai atuar como assistente da acusação durante esquartejar o marido Marcos Kitano Matsunaga, foi sus-
o júri. “O Marcos era apaixonado pela Elize, até puxava a penso por volta das 19h15 desta segunda-feira (28) após
cadeira para ela, atendia todos os desejos dela, era extrema- o depoimento de três testemunhas: duas babás do casal e
mente carinhoso e dava todo o apoio material e financeiro. o detetive contratado por Elize para flagrar o marido com
Isso está largamente comprovado no processo –embora ela uma amante. O júri será retomado na manhã desta terça-
diga que, no dia dos fatos, ele teria dado um tapa na mu- feira (29). Elize vai dormir numa Centro de Detenção Provi-
lher”, afirmou. sória (CDP). Mais 16 testemunhas ainda devem ser ouvidas.
Conforme D’Urso, entre as testemunhas da acusação es- Elize chorou várias vezes durante o primeiro dia do júri.
tão um irmão e uma prima de Marcos, além do reverendo da Teve testemunha passando mal, discussão entre acusação e
igreja anglicana que os casou e policiais. defesa e a discussão sobre a compra de uma serra elétrica
A primeira mulher do empresário –que, segundo Elize, por Elize na véspera do crime, ocorrido em 19 de maio de
nos autos, teria sofrido uma tentativa de assassinato por 2012. “Elize está muito emocionada, ela viu hoje o quanto
parte dele, por envenenamento –se recusou a testemunha. ela foi humilhada pelo marido e rever tudo isso, em de-
“Mas ela nos deu uma declaração em que desmente essa poimentos, é difícil”, justificou Roselle Soglio, advogada de
acusação contra o Marcos”, completou D’Urso. defesa.

18
ATUALIDADES E DEVERES DOS SERVIDORES PÚBLICOS

Elize é ré no processo no qual responde presa pela A compra da serra indica uma premeditação, segundo a
acusação de homicídio doloso triplamente qualificado acusação. Já a defesa minimizou o fato alegando que ela não
(motivo torpe, meio cruel e recurso que dificultou a defesa usou o objeto no esquartejamento. A defesa sustenta que Elize
da vítima), destruição e ocultação de cadáver. Ela confes- matou o marido como reação a um tapa que levou durante
sou que atirou na cabeça da vítima com uma arma e depois uma briga em que a ré contou ao marido que sabia era traída.
a esquartejou em sete partes. A acusação diz que Marcos “É tão claro que essa história da serra elétrica não tem o
estava vivo quando foi decapitado pela esposa. A defesa menor cabimento. Que o crime não é premeditado, que ela
alega que ele já estava morto. O júri é formado por quatro não usou serra elétrica. Se o crime era premeditado, para ela
mulheres e três homens. usar uma serra elétrica, por que que ela não usaria? Então,
Avaliações nao tem logica nesse argumento da acusação”, disse Luciano
Após o encerramento, o promotor José Carlos Consen- Santoro, advogado de Elize.
so fez críticas à defesa de Elize. Na avaliação dele, embora “Essa questão da serra eletrica me parece uma questão
os depoimentos tenham ajudado a endossar as provas do que vai ser objeto de apreciação bastante intensa, da nossa
parte, até pq outras testemunha falam dessa serra elétrica”,
Ministério Público, era esperado que mais testemunhas ti-
disse oLuiz Flávio Borges d’Urso, da acusação.
vessem sido ouvidas.
A babá Amonir respondeu por mais de uma hora as per-
“Eu achei que os trabalhos foram bons, mas eu acho
guntas de acusação e defesa. Na maioria das vezes respondeu
que poderia ter avançado mais. Eu achei muito morosa a
a frase: “Não me recordo”.
colheita, até por uma insistência absurda da defesa. Ficar Elize Matsunaga chorou pelo menos quatro vezes du-
perguntando, perguntando algo que não tinha nenhuma rante o depoimento da primeira testemunha. A última vez foi
substância naquele momento”. quando a advogada de defesa perguntou para a babá fol-
Luiz Flávio D’Urso, assistente de acusação, também guista do casal Mastunaga se Eliza era carinhosa com a filha.
avaliou positivamente as informações obtidas nos três de- A babá folguista disse que no domingo, após o crime,
poimentos. “Em termos de prova para a acusação foi extre- Elize estava chorosa.
mamente importante”, defendeu. Segunda babá
Na visão dele, a revelação de que Elize comprou uma Após intervalo de uma hora, o julgamento foi retomado
serra elétrica nas vésperas de cometer o crime reforçam com depoimento de Mauriceia Golçalves dos Santos, princi-
que a ré já havia planejado executar o marido. “Há elemen- pal babá da filha de Elize. Nervosa, a babá pediu para que
tos que nos trazem convicções de que o crime foi preme- Elize fosse retirada do plenário durante o seu depoimento. O
ditado.” pedido foi atendido. “Depois de tudo o que aconteceu, fiquei
E explica que a premeditação reforça as qualificado- cismada, porque ela pode ficar com raiva de mim por ter fala-
ras – Elize responde por homicídio triplamente qualificado. do da serra”, alegou a babá.
“Do motivo torpe, que é vingança e dinheirinho, o método A babá também negou ter conhecimento de armas de
foi de surpresa, impedindo qualquer defesa da vítima e o fogo na casa, e disse não saber sobre supostas ameaças de
terceiro foi o meio cruel que ainda vai ser objetivo de de- Marcos a Elize. “Nunca fiquei sabendo disso”.
monstração da prova já colhida.” Por volta das 16h, a babá começou a responder às ques-
D’Urso diz ter convicção de que Marcos foi esquarteja- tões do promotor. Disse que Elize estava “um pouco triste”
do ainda vivo. “A prova técnica demonstra que ela, depois quando relatou a ela que Marcos tinha sido encontrado mor-
que disparou, Marcos não morre, e vem a falecer em razão to. Nos dias após a morte, porém, afirmou que a ré tinha um
de asfixia respiratória por aspirar sangue em razão da de- comportamento “normal, normal, normal”. Também destacou
gola. Vale dizer: ela começa a esquarteja-lo vivo.” que Marcos a travava “como uma princesa”, sempre dando
presentes. Durante o depoimento, Mauricéia teve um mal-es-
Para Roselle Soglio, advogada de defesa de Elize, o ma-
tar precisou de atendimento médico.
terial colhido nesta segunda comprova o que já nos autos.
Ela foi questionada incialmente pelo juiz sobre o com-
“O crime nunca foi premeditado. Já no primeiro dia de de-
portamento de Marcos e a relação entre o casal. Afirmou que
poimentos ficou provado pelas três testemunhas que esti-
ele era um bom pai e aparentava ser um homem gentil. A ex-
veram aqui que ela nunca premeditou esse crime.” funcionária disse que não presenciava as brigas do casal. “Eu
“A ênfase na serra elétrica é excelente para defesa. Que pegava a bebezinha e ia descia o elevador”.
a acusação continue falando isso, porque se alguém pre- Mauriceia foi a segunda a depor e passou mal durante o
meditou o crime de comprar uma serra elétrica por que julgamento. Ela pediu para Elize não ficar no plenário durante
não a usou? Essa é única pergunta que tem que ser feita o seu depoimento. A babá confirmou que a ré comprou uma
para acusação”, completa Luciano Santoro, também repre- serra elétrica.
sentante da defesa da ré. Detetive fez flagrante
O terceiro a depor foi o detetive particular William Coelho
Primeira babá contratado por Elize por desconfiar que o marido teria uma
Primeira testemunha a depor no julgamento de Elize amante. Detetive afirmou no depoimento que Elize pediu
Matsunaga, a babá folguista Amonir dos Santos, disse que que ele filmasse “a cara” da amante de Marcos. Ele confir-
sua mãe, a também babá Mauriceia José dos Santos, con- mou ter flagrado o marido com a amante. Elize chorou ao
tou-lhe que Elize comprou uma serra elétrica na véspera da final do depoimento dele. A mulher que seria pivô da briga
morte do marido, Marcos Matsunaga, em maio de 2012. do casal não foi convocada como testemunha.

19
ATUALIDADES E DEVERES DOS SERVIDORES PÚBLICOS

O julgamento começou às 11h16 desta terça no Fórum A Delegacia de Homicídios da Capital (DH) foi ao local
da Barra Funda, em São Paulo. Amonir foi a primeira teste- para apurar as circunstâncias da queda da aeronave. Além
munha a ser ouvida, de um total de 19. O julgamento deve disso, informou que a Coordenadoria de Recursos Especiais
durar até sexta-feira (2). (CORE) foi acionada e está prestando apoio. De acordo
De blazer preto e cabelos presos por uma trança, Eli- com a Polícia Civil, diligências estão sendo feitas.
ze Matsunaga chorou e limpou as lágrimas com um lenço Imagens que circulam por redes sociais mostram o
no início do julgamento em que é acusada de matar e es- momento exato em que o helicóptero cai e, depois, a aero-
quartejar o marido. O júri é formado por quatro mulheres e nave já no chão e muita fumaça saindo das ferragens.
três homens. Elize é ré no processo no qual responde presa Tiroteio mais cedo
pela acusação de homicídio doloso triplamente qualificado Motoristas que passavam pela Linha Amarela, na ma-
(motivo torpe, meio cruel e recurso que dificultou a defesa nhã deste sábado (19) ficaram no meio de um intenso tiro-
da vítima), destruição e ocultação de cadáver. Ela confessou teio. A via expressa chegou a ficar fechada por quase meia
que atirou na cabeça da vítima com uma arma e depois a hora, perto das 10h, no sentido Barra.
esquartejou em sete partes em 19 de maio de 2012. A concessionária Lamsa informou que o tiroteio foi
Logo no início do julgamento, Elize chorou e limpou perto do Viaduto da Estrada do Gabinal, na Zona Oeste,
lágrimas com um lenço enquanto jurados liam resumo do que dá acesso a via expressa. De acordo com a Polícia Mi-
caso. Quando Elize chorou, fotógrafos e cinegrafistas já ti- litar, policiais UPP foram atacados quando passavam pelo
nham saído do plenário. viaduto. Outros PMs foram chamados e houve o tiroteio.
Duas testemunhas foram dispensadas: o delegado Jor- Até as 11h30 não havia informações sobre feridos.
ge Carrasco, arrolado como testemunha de defesa, e o re- Devido à troca de tiros, muitos motoristas, assustados,
verendo Renê Henrique Gotz Licht, que fez o casamento de tentaram voltar na contramão. O tráfego ficou complica-
Elize e Marcos Matsunaga. Ele seria testemunha tanto da do na região, inclusive com reflexos na Avenida Geremário
defesa quanto da acusação. Dantas, em Jacarepaguá, na Zona Oeste, que também dá
Fonte: G1 – (28/11/2016) acesso à Linha Amarela.
Fonte: G1 – (19/11/2016)
Helicóptero da PM cai na Zona Oeste do Rio e qua-
tro policiais morrem
Após queda de helicóptero, polícia faz operação na
Um helicóptero do Grupamento Aeromóvel da Polícia
Cidade de Deus
Militar (GAM) caiu próximo à comunidade Cidade de Deus,
A Polícia Militar do Rio de Janeiro iniciou neste domin-
na Zona Oeste do Rio, no começo da noite deste sábado
go (20) uma operação por tempo indeterminado na Cidade
(19). Quatro policiais militares que estavam na aeronave
de Deus, favela da zona oeste da cidade em que um heli-
morreram na queda. O helicóptero caiu no começo da Ave-
cóptero da PM caiu no sábado, matando quatro policiais.
nida Ayrton Senna, perto do acesso à Linha Amarela.
Pelo menos três homens foram presos durante a ação nes-
A informação da queda foi confirmada ao G1 pelo
te domingo. Um deles estava com três fuzis e duas pistolas,
coordenador de Comunicação Social da Polícia Militar, Ma-
jor Ivan Blaz. Durante o dia, a região foi palco de intensos segundo a PM. O caso foi encaminhado à 32ª DP (Taquara).
tiroteios quando, mais cedo, policiais da Unidade de Polícia A decisão de ocupar a favela por tempo indeterminado
Pacificadora (UPP) Cidade de Deus trocaram tiros com cri- foi tomada pela cúpula de Segurança do Rio, que se reuniu
minosos. após o acidente ainda na noite de sábado. Nesta manhã,
Segundo a GloboNews, os quatro ocupantes do heli- pelo menos sete corpos foram encontrados no interior da
cóptero eram Major Rogério Melo Costa, o capitão William comunidade.
de Freitas Short, o subtenente Camilo Barbosa Carvalho e o O ministro da Justiça, Alexandre de Moraes, ofereceu
sargento Rogério Felix Rainha. apoio à Secretaria de Segurança Pública do Rio de Janeiro
O momento em que o helicóptero começa a perder for- na operação. Em nota, Moraes informou que ofereceu o
ça foi filmada por pessoas que estavam nas proximidades. aparato da Força Nacional de Segurança que está no Rio
De acordo com Blaz, “tudo indica que o helicóptero fez para auxiliar nas ações que estão sendo realizadas na co-
um pouso forçado”. Na região, desde cedo, o Comando de munidade.
Operações Especiais (COE) da PM - composto pelos Bata- Ainda não se sabe se a aeronave sofreu uma pane ou
lhão de Operações Especiais (Bope), Batalhão de Choque e foi atingida por criminosos. Embora a queda tenha ocor-
GAM - estava no local dando apoio à UPP local. rido em meio a confrontos entre policiais, traficantes e
A Lamsa, concessionária que administra a Linha Ama- milicianos, a principal hipótese é que o helicóptero tenha
rela, pediu que motoristas evitassem a região. O Centro sofrido uma pane. Segundo o Instituto Médico-Legal, os
de Operações da Prefeitura do Rio informou que os dois policiais morreram devido à queda e não foram atingidos
sentidos da Linha Amarela foram interditados, na altura da por nenhum disparo.
Cidade de Deus, além de trechos da Ayrton Senna, da altura O acidente resultou na morte do major Rogério Melo
do Via Parque ao acesso à Cidade de Deus, devido a uma Costa, 36, do capitão William de Freitas Schorcht, 37, do
operação policial. subtenente Camilo Barbosa Carvalho, 39, e do sargento
Ao todo, a via expressa ficou interditada por quase três Rogério Felix Rainha, 39. Os corpos chegaram no início da
hroas, de 19h às 21h40 . madrugada ao Instituto Médico-Legal.

20
ATUALIDADES E DEVERES DOS SERVIDORES PÚBLICOS

Uma equipe da PM sobrevoou de helicóptero e lan- Integrantes do Centro de Investigação e Prevenção de


çou pétalas de rosas durante o velório coletivo de três dos Acidentes Aeronáuticos (Cenipa) estiveram na noite de sá-
quatro PMs mortos. O velório ocorreu no Salão Nobre do bado na Cidade de Deus para realizar as ações iniciais de
Batalhão de Choque e foi restrito aos parentes e amigos apuração das causas do acidente. Representantes do centro
dos policiais. de Criminalística da PM e a Delegacia de Homicídios tam-
O corpo do capitão e piloto do helicóptero Willian de bém estiveram no local.
Freitas Schorcht, de 37 anos, seguiu direto para Resende, Em 2009, dois policiais morreram e três ficaram feridos
na Região Sul Fluminense, onde vive a família, e será enter- após um helicóptero da Polícia Militar realizar um pouso for-
rado neste domingo. çado no Morro dos Macacos. A aeronave, parcialmente blin-
O presidente Michel Temer usou sua conta oficial do dada, havia sido atingida por tiros durante uma operação
Twitter para lamentar a morte dos policiais. “Lamentável a policial. Além dos tripulantes mortos, um capitão da PM foi
morte dos 4 PMs que cumpriam o seu dever durante ope- baleado na perna e outros dois policiais tiveram queimadu-
ração no Rio de Janeiro. A minha solidariedade aos familia- ras leves.
res e amigos”, disse. Fonte: Portal UOL – (20/11/2016)
O governador Luiz Fernando Pezão decretou luto ofi-
Moradores da Cidade de Deus encontram corpos de-
cial por três dias pelas mortes dos PMs. “Reconhecemos
saparecidos
e agradecemos a dedicação da Polícia Militar no combate
Moradores da Cidade de Deus retiraram da mata sete
ao crime e, em especial, dos policiais que perderam a vida
corpos de jovens da comunidade, na manhã deste domin-
no exercício de proteger e defender a sociedade. Expres- go. Eles foram colocados pelos pais em uma praça próxima
so meus sentimentos aos parentes e amigos dos militares. ao condomínio Itamar Franco, na localidade do Karatê. Fa-
Vamos seguir em frente em defesa dos cidadãos fluminen- miliares começaram a denunciar o sumiço das pessoas no
ses”, afirmou o governador. sábado após ação da polícia militar no local. Na ocasião,
um helicóptero caiu, resultando na morte de quatro PMs: o
Operação por tempo indeterminado major Rogério Melo Costa, o terceiro-sargento Rogério Félix
Na manhã deste domingo, enquanto policiais milita- Rainha; o capitão William de Freitas Schorcht e o subtenente
res circulavam com apoio de blindados, pessoas faziam Camilo Barbosa de Carvalho.
barricadas incendiando lixo. Foram registrados novos con- Não se sabe ainda se os crimes ocorreram antes ou de-
frontos entre criminosos e policiais. Por causa da atuação pois de a aeronave despencar. O secretário de Segurança,
da polícia no local, algumas ruas estavam interditadas. A Roberto Sá, garantiu que os homicídios já estão sendo in-
Estrada dos Bandeirantes, a Estrada do Gabinal e a Linha vestigados:
Amarela eram opções aos que transitavam pela região. — Essas mortes não vão ficar sem resposta.
A Linha Amarela, via expressa que liga as zonas norte e Após o acidente, o Bope iniciou uma operação na comu-
oeste do Rio, chegou a ser fechada duas vezes devido aos nidade. Até o momento, foram detidas três pessoas e houve
tiroteios. apreensão de armas e drogas. Em sua conta no Twitter, o
Desde a sexta-feira (18) foram registrados intensos presidente Michel Temer lamentou, a morte dos policiais.
confrontos entre criminosos da favela Cidade de Deus. Na “Lamentável a morte dos 4 PMs que cumpriam o seu
manhã de sábado, eles voltaram a se enfrentar e traficantes dever durante operação no Rio de Janeiro. A minha solida-
bloquearam a avenida Edgard Werneck, que é a principal riedade aos familiares e amigos. Reitero minha confiança e
da Cidade de Deus, onde fica a base da UPP, com pneus e apoio ao trabalho das forças policiais, sempre comprome-
lixeiras incendiados. Policiais da Unidade de Polícia Pacifi- tidas no combate ao crime”, escreveu Temer, em seu perfil
cadora (UPP) trocaram tiros com os criminosos. no Twitter.
Neste domingo, comboios militares cercaram a área,
Nas redes sociais, moradores relataram o dia de tiro-
bloqueando acessos e revistando carros e os poucos mora-
teios. “A bala tá comendo na CDD. Só escuto os caras da
dores que se arriscaram a deixar a Cidade de Deus. Do lado
Bope [Batalhão de Operações Policiais Especiais] gritando:
de fora, eram raros os que passavam pela Estrada Marechal
‘Sai da rua morador’”, escreveu um rapaz, uma hora antes
Miguel Salazar Mendes de Moraes, fechada nos dois senti-
de o helicóptero cair. Um traficante postou uma foto, com dos. Já nos fundos da favela — junto ao valão que um dia
um fuzil. “Nada mudou. Nós ‘está’ na pista”, escreveu. foi o Rio Arroio Fundo — moradores brigavam com policiais
Após a queda do helicóptero, a PM lamentou a morte pelo direito de entrar na mata do Karatê para procurar seus
dos policiais. “A Polícia Militar do Estado do Rio de Janeiro filhos. Só conseguiram por volta de 9h30m, quando repórte-
lamenta o falecimento de seus policiais militares e o Co- res chegaram ao local. Aos poucos, grupos foram voltando
mando da Corporação está dedicado a prestar todo apoio com os corpos dos sete homens jovens, com sinais de tiros
às famílias desses policiais”, disse a instituição. e facadas, que foram enfileirados e cobertos por lençóis na
Outra nota foi publicada na página oficial da corpora- principal praça do Karatê.
ção no Facebook. No comunicado é mencionada a mor- — Foram cem pessoas lá pegar os corpos. Eles estavam
te de outro policial ocorrida neste sábado. O 3º Sargento deitados de costas. Meu filho tinha as mãos na cabeça. A
Cristiano Bittencourt Coutinho participava de uma outra maioria tinha tiros nas costas e na cabeça. Eles foram execu-
operação quando foi atingido por um tiro após a viatura tados e a perícia vai mostrar — acusou o pastor Leonardo
em que se encontrava ter sido alvejada no bairro Jacaré. Martins da Silva, pai de Leonardo da Silva Junior, de 22 anos.

21
ATUALIDADES E DEVERES DOS SERVIDORES PÚBLICOS

A mãe de Marlon César, de 23 anos, contou que muitos — Normalmente era só de madrugada, agora passou
moradores foram surpreendidos com confrontos. a ficar normal ouvir tiros durante as manhãs. Ontem, eu
— As pessoas foram pegas de surpresa. Foi um deses- achava que a situação ia se estabilizar depois do meio-dia.
pero. Eu não consegui ter contato com meu filho. Ele pode Mas o tiroteio voltou com tudo à tarde e perdurou até as
estar na mata, mas os policiais não nos deixam entrar para 22h. Outro evento cultural além do meu também teve que
procurar. Isto é desumano. ser cancelado. Houve uma sequência de tiros muito grande
Moradores também acompanharam o início da perícia na tentativa de abater o helicóptero — afirmou Vivi.
a cargo da Divisão de Homicídios, que investigará as mor-
tes. Alguns acusavam policiais que invadiram a favela após OPERAÇÃO POR TEMPO INDETERMINADO
a queda da aeronave. Desde ontem, policiais de vários Batalhões realizam
— Eles eram envolvidos (com o tráfico), mas se rende- uma operação pente-fino na comunidade. Por determina-
ram. Isso é execução, não? — Questionava um parente de ção da cúpula da segurança, a operação que começou na
Leonardo Camilo, de 29 anos, também encontrado morto. manhã de sábado, após bandidos atirarem contra policiais
— Depois que o helicóptero caiu foi um terror. Eles entra- da UPP, continuará por tempo indeterminado.
ram e deram muito tiro. Foi vingança. O clima da comunidade é de apreensão. Pouco movi-
Em entrevista à “Rádio CBN”, Raíssa da Silva Monteiro, mento nas ruas. O serviço de mototáxi foi suspenso por de-
de 20 anos, também afirmou que seu irmão desapareceu terminação policial. Todos os carros que entram ou deixam
na noite de sábado: a Cidade de Deus passam por rigorosa revista. Participam
— Ele ligou para minha mãe às 18h35m e disse que das operações policiais do Choque, do Bope, de Operações
não conseguia falar. Não consigo mais notícia, a gente quer com Cães, os batalhões de Jacarepaguá e da Barra, além de
entrar no mato, mas os policiais estão dando tiro para cima policiais de várias UPPs.
da gente. Na manhã deste domingo, pelo menos três pessoas fo-
O morador Thiago Oliveira acompanhou a busca pelos ram detidas durante a operação da Polícia Militar na Cida-
corpos. Ele é amigo de um dos pais das vítimas. de de Deus. Na ação, foram apreendidos fuzis e drogas. Os
— Os corpos estão com ferimentos de facas, como se policiais ocupam a comunidade à procura de bandidos que
tivessem sido torturados. Parece que encapuzaram as ví- entraram em confronto com agentes no sábado. Em seu
perfil no Facebook, a Polícia Militar prestou uma homena-
timas e começaram a atirar. É ódio gerado por ódio, uma
gem aos PMs mortos, publicando uma mensagem de luto.
crueldade — afirma Thiago, que também diz que circula a
A operação policial interdita ruas no entorno da co-
informação na comunidade sobre a morte de uma criança
munidade. Segundo o Centro de operações da prefeitura,
de quatro anos, por bala perdida.
a Estrada Marechal Miguel Salazar Mendes de Moraes está
Antes dos corpos serem encontrados, um vídeo divul-
fechada em ambos os sentidos, entre as ruas Antonieta
gado nas redes sociais mostrava uma mãe, desesperada,
Campos da Paz e a Edgard Werneck.
tentando achar o filho. A polícia estava impedindo o acesso
Já a Rua Edgard Werneck também está interditada em
de pessoas à mata, onde as vítimas foram encontradas. ambos os sentidos, entre a Estrada Marechal Migual Sa-
— Vou procurar meu filho agora! Meu filho está dentro lazar Mendes de Moraes e a Rua Suzano. As estradas dos
do mato, morto! O sangue é meu! Eu sou mãe! — Gritava. Bandeirantes, do Gabinal e a Linha Amarela são opções
Em nota, a Polícia Civil informou que um procedimento para os motoristas. A UPA do local também está fechada e
foi instaurado na Delegacia de Homicídios para apurar as o atendimento está sendo feito no Lourenço Jorge.
mortes das sete pessoas. Segundo o comunicado, “segue Ainda como consequência do confronto, a estação Di-
em andamento um amplo trabalho de investigação visan- vina Providência, do BRT Transcarioca (Barra da Tijuca-Ae-
do apurar detalhadamente as circunstâncias do ocorrido”. roporto Tom Jobim), foi alvo de atos de vandalismo neste
Neste sábado, a queda de um helicóptero da Polícia sábado. A estação, próxima à Cidade de Deus, teve oito
Militar que participava de uma operação na Cidade de vidros quebrados, além de quatro monitores de TV, perfis
Deus, Zona Oeste do Rio de Janeiro, provocou a morte de metálicos e bancos amassados e geladeira de refrigerante
quatro policiais e iniciou uma onda de pânico entre mo- derrubada. O Consórcio BRT estimou o prejuízo material
radores. Logo após a tragédia, começaram a circular pela em pelo menos R$ 37 mil.
internet informações alarmantes - grande parte delas inve- Fonte: O Globo – (20/11/2016)
rídicas - sobre novos ataques ou mortes.
O clima na Cidade de Deus segue tenso após os tiro- INTERNACIONAL
teios e operações policiais que culminaram com a queda
de um helicóptero da Polícia Militar, com quatro militares ELEIÇÕES NORTE-AMERICANAS
mortos.
A moradora Vivi Salles iria realizar um sarau comemo- Donald Trump vence as eleições dos Estados Unidos
rativo de cinco anos do Poesia de Esquina, movimento cria- Donald Trump, um magnata do setor imobiliário e es-
do por ela para reunir poetas da Cidade de Cidade de Deus, trela de reality shows sem experiência política e com uma
ontem, mas teve que cancelar o evento, por causa dos con- mensagem xenófoba e antissistema, será o próximo presi-
frontos entre criminosos e policiais na região. A poetiza, dente dos Estados Unidos. O republicano Trump derrotou
que tem 26 anos, disse que nunca havia presenciado tanto nas eleições de 8 de novembro a democrata Hillary Clin-
tiroteio durante o dia. ton, uma política experiente e associada ao establishment

22
ATUALIDADES E DEVERES DOS SERVIDORES PÚBLICOS

que não soube se conectar com a coalizão de minorias e oito com um democrata na Casa Branca, não existia candi-
jovens que deu duas vitórias ao presidente Barack Obama. dato mais novo do que Trump, nenhum que representasse
A vitória de Trump, um populista imprevisível no comando melhor do que ele um tapa no sistema, a tentativa de virar a
da maior potência do planeta, lança seu país e o mundo ao página com a classe política de um e outro partido.
desconhecido. A vitória eleitoral deixa uma sociedade fraturada. As mi-
O mundo esperava ver a primeira mulher na presidência norias, as mulheres, os estrangeiros que se sentiram insul-
dos EUA e encontra um demagogo pela frente, um homem tados por Trump deverão se acostumar a vê-lo como pre-
que reavivou algumas das tradições mais tenebrosas do país. sidente. Deixa também uma sociedade com medo. O presi-
A chegada de Trump à Casa Branca é uma ruptura com dente eleito prometeu deportar os 11 milhões de imigrantes
as melhores tradições democráticas dos EUA, com a tran- ilegais, uma operação logística com precedentes históricos
quila alternância entre governantes com visões discrepantes sinistros. O veto à entrada de muçulmanos fere os princípios
do país, mas não nos valores fundamentais que o susten- de igualdade consagrados na Constituição dos EUA.
tam desde sua fundação. Trump, que prometeu construir Sua inexperiência e escassa preparação também são
um muro na fronteira com o México e proibir a entrada de uma incógnita sobre o modo como governará. Uma teoria
muçulmanos nos EUA, demonstrou que um homem prati- é que uma vez no salão oval ficará mais moderado e que,
camente sozinho, contra tudo e contra todos, é capaz de de qualquer forma, o sistema de controle de poderes freie
chegar à sala de comando do poder mundial. Lá terá ao al- qualquer afã autoritário. A outra é que, ainda que esse país
cance da mão a valise com os códigos nucleares e controlará não tenha experimentado um regime ditatorial no passado,
as mais letais forças armadas do planeta, além de possuir as declarações de Trump em campanha preveem um viés
um púlpito único para se dirigir ao seu país e ao resto do autoritário.
mundo. Da Casa Branca poderá se lançar, se cumprir suas Existem momentos em que as grandes nações dão vira-
promessas, a batalhas com países vizinhos como o México, a das bruscas. Quando se trata dos Estados Unidos da Amé-
quem quer obrigar a pagar o muro. O México, vizinho e até rica, a virada afeta a toda a humanidade. O 8 de novembro
agora amigo dos EUA, será o primeiro ponto na agenda do de 2016 pode passar à história como um desses momentos.
presidente Trump. Fonte: El País Brasil – (09/11/2016)
O republicano desmentiu todas as pesquisas que há seis
meses prognosticavam sua derrota. Derrotou os Clinton, a
Como a vitória de Trump pode afetar o Brasil?
família mais poderosa da política norte-americana nas últi-
Em um triunfo inesperado, o republicano Donald Trump
mas três décadas, com exceção dos republicanos Bush, que
foi eleito o novo presidente dos Estados Unidos. Trump con-
também se opunham a ele. Enfrentou a máquina de seu pró-
quistou vários Estados-pêndulo, onde os resultados eram
prio partido, os meios de comunicação, Wall Street, as gran-
imprevisíveis - podiam favorecer tanto um quanto o outro
des capitais europeias e latino-americanas e organizações
partido -, como Flórida, Ohio e Carolina do Norte, garantin-
internacionais como a OTAN.
Seu mérito consistiu em entender o desconforto dos do vantagem sobre Hillary Clinton.
norte-americanos vítimas da tempestade da globalização, as Sua vitória não era indicada pelas pesquisas de opinião,
classes médias que não deixaram de perder poder aquisitivo que apontavam Clinton como novo presidente.
nas últimas décadas, os que viram como a Grande Recessão Mas como o êxito do republicano impacta no Brasil?
paralisava a ascensão social, os que observam desconcerta-
dos as mudanças demográficas e sociais em um país cujas Economia e comércio
elites políticas e econômicas os ignoram. Os brancos da clas- Vários aspectos devem ser levados em conta para res-
se trabalhadora – uma minoria antigamente democrata que ponder a questão.
compete com outras minorias como os latinos e os negros, Um deles é a maneira como os dois candidatos e seus
mas que não tem um status social de vítima – encontrou em partidos encararam a economia e as relações comerciais en-
Trump seu homem providencial. tre os Estados Unidos e o resto do mundo.
Durante a campanha Trump prometeu um Brexit mul- O Brasil se beneficiaria de uma maior abertura dos EUA
tiplicado por 5, em alusão à decisão da Grã-Bretanha, em a produtos brasileiros. Hoje os EUA são o segundo maior
referendo, de sair da União Europeia. E cumpriu. A onda de parceiro comercial do Brasil, atrás da China.
populismo de ambos os lados do Atlântico consegue sua Historicamente, o Partido Republicano, de Trump, de-
maior vitória. É um golpe nas elites norte-americanas e glo- fende o livre comércio e se opõe a medidas protecionistas
bais. E é uma prova de que em tempos de incerteza pode que ajudassem empresas americanas a competir com es-
ganhar um candidato com os sensores para identificar os trangeiras.
medos da sociedade e uma mensagem simplificadora que Assim, um candidato republicano tenderia a ser melhor
identifique o inimigo interno e externo. para os interesses econômicos do Brasil do que um candi-
Os intermináveis escândalos, reais ou inventados, de dato democrata.
Clinton derrubaram sua candidatura. Poucos políticos se Mas Trump inverteu essa lógica ao propor renegociar os
identificavam tanto com o establishment como ela. No final acordos comerciais firmados pelos EUA para preservar em-
das contas, é a esposa de um presidente e os EUA, uma re- pregos no país e reduzir o déficit americano nas transações
pública fundada contra as dinastias, já teve o suficiente com com o resto do mundo.
os presidentes Bush pai e filho. Os norte-americanos que- Se o empresário colocar essas ações em prática, o Bra-
riam provar algo diferente, e em um ano de mudança, após sil poderia ser prejudicado.

23
ATUALIDADES E DEVERES DOS SERVIDORES PÚBLICOS

A professora de Relações Internacionais da ESPM De- “A situação do governo Hillary para o Brasil teria sido
nilde Holzhacker afirma que as consequências seriam ime- mais tranquila porque era mais previsível por qual caminho
diatas e negativas, e causariam o que muitos economistas ela iria. Seria a continuidade do governo Obama, de uma di-
estão chamados de “efeito Trump”. mensão política que tem o reconhecimento do Brasil como
“Como ele fez propostas muito amplas e populistas, relevante, sem muitas mudanças.”
os efeitos econômicos dessas medidas podem ter impac- Pecequilo afirma que o país deve perder relevância
to grande e gerar um caos na economia - principalmente na visão dos Estados Unidos dado o conturbado cenário
porque ele é contrário ao livre comércio, se mostrou pro- interno.
tecionista.” “Eles estão com tanto problema dentro de casa, que o
Mas Holzhacker faz uma ressalva sobre a aplicação Brasil não é uma preocupação.”
dessas medidas. Relação entre Brasil e EUA também vai depender de quí-
“Agora, para saber o quanto ele vai conseguir imple- mica entre Temer e Trump
mentar disso, vamos ter que esperar. Ele é tão imprevisível
e tudo fica tão indefinido que prejudica muito o cenário Questão de química
Especialistas nas relações Brasil-EUA costumam dizer
econômico.”
que os laços entre os dois países dependem em grande me-
dida da química entre seus líderes, independentemente de
Imigração e vistos
seus partidos ou ideologias.
Estima-se que um milhão de brasileiros vivam nos EUA,
Eles afirmam que, embora seguissem tradições políticas
boa parte em situação migratória irregular. bastante distintas, os presidentes Luiz Inácio Lula da Silva
Trump propôs construir um muro na fronteira do país (2003-2011) e George W. Bush (2001-2009) tinham uma re-
com o México e prometeu deportar todos os imigrantes lação tão boa quanto a mantida entre FHC (1995-2002) e Bill
sem documentos. Clinton (1993-2001), que tinham maior afinidade ideológica.
Ele diz que protegerá o “bem-estar econômico de imi- Já a relação entre Barack Obama e Dilma Rousseff nunca
grantes legais” e que a admissão de novos imigrantes le- foi tão próxima e sofreu com a revelação de que o governo
vará em conta suas chances de obter sucesso nos EUA, o americano havia espionado a presidente brasileira.
que em tese favoreceria brasileiros com alta escolaridade Analistas afirmam ainda que Brasil e EUA têm relações
e habilidades específicas que queiram migrar para o país. bastante diversificadas e que os laços devem ser mantidos
Evento de latinos em apoio a Trump, que prometeu qualquer que seja o resultado da eleição em novembro, já
construir um muro para evitar entrada de imigrantes que os dois governos dialogam dentro de estruturas buro-
Outro tema de interesse dos brasileiros é a facilidade cráticas.
para obter vistos americanos. Trump fez poucas menções Do lado brasileiro, há interesse em se aproximar mais
ao sistema de concessão de vistos do país. dos EUA, vença quem vençer. Em entrevista à BBC Brasil em
Hoje, Brasil e EUA negociam a adesão brasileira a um julho, o embaixador brasileiro em Washington, Sérgio Ama-
programa que reduziria a burocracia para viajantes fre- ral, disse que o governo Temer investiria nas relações com as
quentes brasileiros, como executivos. A eliminação dos vis- cinco principais potências globais (EUA, China, Rússia França
tos, porém, ainda parece distante. e Reino Unido).
Para que a isenção possa ser negociada, precisaria ha- Amaral afirmou ainda que, na Embaixada, priorizaria
ver uma redução no índice de vistos rejeitados em con- áreas em que Brasil e EUA têm maior convergência, como
sulados americanos no Brasil, uma exigência da legislação direitos humanos e meio ambiente.
dos EUA. Fonte: G1 – (09/11/2016)

Após escândalo, presidente da Coreia do Sul aceita


Relação com o Brasil
renunciar
O Brasil e a América Latina não foram tratadas como
A presidente da Coreia do Sul, Park Geun-hye, afirmou
temas prioritários nas campanhas dos dois candidatos.
em discurso televisionado nesta terça-feira que deixará seu
Em 2015, Trump citou o Brasil ao listar países que, se-
destino político nas mãos do Legislativo. A declaração foi
gundo ele, tiram vantagem dos Estados Unidos através de surpreendente e vista por alguns analistas como uma tática
práticas comerciais que ele considera injustas. A balança para criar um impasse e ela seguir no poder.
comercial entre os dois países, porém, é favorável aos EUA. Em breve discurso, Park pediu desculpas novamente
Como empresário, Trump é sócio de um hotel no Rio pelo escândalo político que derrubou sua popularidade. Ela
de Janeiro e licenciou sua marca para ser usada por um disse estar aberta para abrir mão do poder, mas deixou nas
complexo de edifícios na zona portuária da cidade. Anun- mãos da Assembleia Nacional determinar se ela deve seguir
ciada em 2012, a obra ainda nem começou. como presidente. A Assembleia Nacional é controlada pela
Para a professora de Relações Internacionais da Uni- oposição e por parlamentares independentes.
fesp Cristina Pecequilo, como Trump não falou nada sobre No poder desde 2013, Park afirmou que cumprirá o pro-
o país e se distanciou de temas ligados à América Latina, cesso legal, prometendo apresentar mais detalhes sobre o
não deve haver muitas mudanças para os brasileiros. No escândalo e responder a questões em data futura não es-
entanto, diferentemente de Hillary, o republicano tem o pecificada. Ela não respondeu a perguntas após o pronun-
elemento de imprevisibilidade. ciamento.

24
ATUALIDADES E DEVERES DOS SERVIDORES PÚBLICOS

A presidente é acusada de permitir que uma amiga de O cronograma apertado poderia levar os principais
longa data extorquisse dinheiro de empresas, usando sua pro- partidos a tentar ganhar tempo para consolidar seus can-
ximidade do poder. Park tem aprovação de apenas 4% entre os didatos presidenciais, e analistas políticos disseram que o
eleitores sul-coreanos. parlamento pode demorar meses para acordar um plano
Para realizar um impeachment, é preciso 200 votos na As- de saída para Park.
sembleia Nacional de 300 integrantes. Atualmente, 172 legisla- “Irei renunciar à minha posição de acordo com a lei as-
dores oposicionistas e independentes querem a saída de Park. sim que se crie uma maneira de transferir o governo de uma
Com isso, para retirar a líder seriam necessários os votos maneira estável que também minimize o vácuo e o tumulto
de 28 dos 128 parlamentares do partido conservador Saenuri, político após a decisão e a discussão dos partidos de oposi-
de Park. Vários deputados governistas já disseram que votarão ção”, disse Park com voz firme.
pela saída da presidente. A imprensa local avalia que entre 30 Fonte: G1 – (29/11/2016)
e 40 legisladores governistas podem votar pelo impeachment.
A votação deve ocorrer antes de 9 de dezembro, quan- Merkel confirma candidatura a 4º mandato na Ale-
do acaba a sessão deste ano da legislatura sul-coreana. Caso manha
o Parlamento vote pelo impeachment, Park seria suspensa e Angela Merkel anunciou neste domingo a seu partido
o número dois do país, o primeiro-ministro Hwang Kyo-ahn, que será candidata a um quarto mandato de chanceler du-
assumiria como presidente interino. rante as eleições legislativas de 2017, em um período em
O Tribunal Constitucional teria então de se pronunciar para que seus partidários a consideram a última defesa contra o
decidir se o impeachment é justificado – a corte teria 180 dias avanço do populismo.
para se pronunciar. A chanceler, de 62 anos, anunciou sua intenção aos
Caso seis dos nove magistrados do Tribunal Constitucional dirigentes da União Democrata Cristã (CDU) durante uma
decidirem que a saída se justifica, Park perderia formalmente o reunião em Berlim, informaram à AFP fontes próximas ao
cargo. A Coreia do Sul teria então 60 dias para eleger um su- partido.
cessor. Se mais de três juízes discordarem, porém, a presidente A chefe de Governo, que deve abordar a questão em
retomaria o posto imediatamente. um encontro com a imprensa às 19H00 (16H00 de Brasília),
expressou a intenção de ser reeleita como presidente da
A amiga da presidente que está no centro do escândalo é
CDU no congresso de dezembro, além de apresentar uma
Choi Soon-sil, de 60 anos, que supostamente usava seus laços
nova candidatura para a chancelaria durante as legislativas.
com a líder para conseguir milhões em doações de companhias
Após 11 anos à frente do país, Merkel já ostenta o re-
sul-coreanas. A presidente e a amiga afirmam ser inocentes.
corde de longevidade entre os atuais governantes ociden-
Fonte: Exame.com – (29/11/2016)
tais.
Julia Klöckner, da CDU e muito próxima a Merkel, de-
Presidente da Coreia do Sul pede que parlamento deci-
fendeu a candidatura e afirmou que a chanceler é “uma
da condições para sua saída garantia de estabilidade confiabilidade em um período tur-
A presidente da Coreia do Sul, Park Geun-hye, pediu nesta bulento”.
terça-feira que o parlamento decida como e quando ela pode
entregar o cargo em reação a um escândalo de tráfico de in- Recorde de Kohl à vista
fluência, levando a crise política do país cada vez mais em dire- A julgar pelas pesquisas, Merkel tem grandes chances
ção a um território desconhecido. de conquistar o quarto mandato como chanceler.
O Partido Democrático, principal sigla da oposição, rejei- Ela entraria assim para a história do país ao superar o
tou a oferta de Park, que classificou como uma manobra para tempo de poder do icônico chanceler do pós-guerra Kon-
escapar do impeachment, e disse que irá continuar com seus rad Adenauer (14 anos) e também o de seu próprio mentor
esforços para apresentar uma moção de impedimento no par- político, Helmut Kohl (16 anos).
lamento, o que pretende fazer até sexta-feira. De acordo com uma pesquisa publicada pelo jornal
Nenhum presidente sul-coreano deixou de completar seu Bild, 55% dos alemães desejam que Merkel permaneça no
mandato único de cinco anos desde que o sistema democráti- cargo, contra 39% de opiniões contrárias. Em agosto, o ín-
co atual foi implementado em 1987. dice favorável à chanceler era de 50%.
“Deixarei ao parlamento tudo a respeito do meu futuro, Merkel se encontra em uma situação paradoxal: elogia-
incluindo a abreviação de meu mandato”, disse Park em um da no exterior, onde as expectativas a seu respeito aumen-
breve discurso televisionado. taram após a vitória de Donald Trump na eleição americana,
Seu gesto dramático impõe o fardo de resolver a crise po- na Alemanha enfrenta um ano eleitoral um tanto fragilizada
lítica ao parlamento, que vem sendo controlado por uma coa- pela polêmica provocada pela decisão de receber um mi-
lizão de partidos opositores desde que o conservador Partido lhão de refugiados no país.
Saenuri de Park perdeu a maioria subitamente nas eleições de Esta semana, o presidente americano Barack Obama
abril. elogiou Merkel em Berlim durante sua última viagem oficial
Se a mandatária renunciar ou um voto de impeachment no como chefe de Estado.
parlamento for confirmado pelo Tribunal Constitucional, será “Se fosse alemão, poderia dar meu apoio”, disse.
preciso realizar uma eleição em 60 dias para escolher um E, diante do avanço das tendências autoritárias no
presidente para um mandato de cinco anos, e o primeiro- mundo, o jornal The New York Times a chamou de “último
ministro irá conduzir o país neste ínterim. baluarte dos valores humanistas no Ocidente”.

25
ATUALIDADES E DEVERES DOS SERVIDORES PÚBLICOS

Mas seu poder está em queda na Alemanha, de acordo Contra aqueles que usam os medos dos cidadãos para
com a revista liberal Die Zeit. Ela conseguiu recuperar parte se promoverem, Merkel lembrou a positiva evolução eco-
da popularidade perdida com a crise migratória, mas o seu nômica do país nos últimos anos e o aumento nas contri-
grupo político registra de 32 a 33% das intenções de voto buições sociais e previdência.
nas pesquisas, quase 10% a menos que nas eleições de 2013. A chanceler lembrou também a “incrível” resposta in-
“O recuo criado pela vitória de Trump afeta Merkel ternacional para a catástrofe da Segunda Guerra Mundial,
quando suas possibilidades de liderança são limitadas: não com a criação das Nações Unidas e da Convenção de Di-
pode contar com a Europa para avançar, não tem um parti- reitos Humanos, e defendeu a continuidade desse caminho
do unido atrás dela e não possui o apoio popular que tinha para “dotar de humanidade a globalização”.
há um ano e meio”, afirma a Die Zeit. Durante a presidência rotativa do G20, que a Alemanha
O atraso no anúncio da candidatura está relacionado assume em dezembro, Merkel pretende continuar com os
com a perda de poder. Após a polêmica sobre a recepção avanços na transparência dos mercados financeiros inter-
aos refugiados, ela teve que lidar com a rebelião da CSU, nacionais e no desenvolvimento do continente africano,
partido aliado bávaro, que ameaçou não apoiar Merkel em que será uma das prioridades de Berlim.
2017, antes de mudar de opinião ante a falta de alternativa. No âmbito da UE, a chanceler reconheceu a necessi-
A chanceler sofreu outro revés este mês ao não conse- dade de lutar contra a falta de credibilidade do bloco e de
guir designar um membro de seu partido como candidato acelerar os processos de tomada de decisões.
para ser presidente em 2017, um posto para o qual foi esco- Para Merkel, é preciso assumir que já não é possível
lhido o social-democrata Frank-Walter Steinmeier. traçar uma linha que separe a política interna da externa e
Por fim, seu terceiro mandato coincidiu com o avanço que a segurança, o bem-estar e a prosperidade dos cida-
de um partido populista na Alemanha, que disputa espaço dãos de cada país dependem das relações internacionais.
com seu partido na direita. O AfD tem grandes chances de Fonte: Exame.com – (23/11/2016)
entrar para o Bundestag (Parlamento), o que nenhum grupo
deste tipo consegue desde 1945. Fidel Castro morre aos 90 anos
Merkel mantém, no entanto, a vantagem sobre os de- O ex-presidente cubano Fidel Castro, um dos mais im-
mais, já que não possui rivais fortes em seu partido e conti- portantes líderes mundiais, morreu na noite desta sexta-
nua sendo muito mais popular que seus adversários social- feira aos 90 anos. Seu irmão mais novo e atual presidente,
democratas. Raúl Castro, anunciou a morte em um anúncio oficial na
Fonte: Exame.com – (20/11/2016) TV estatal.
“O comandante-em-chefe da revolução cubana mor-
Merkel defende alianças com UE e EUA e rejeita iso- reu às 22h29 desta noite (03h29 de sábado em Brasília)”,
lamento disse o presidente, que terminou o anúncio gritando o slo-
A chanceler da Alemanha, Angela Merkel, rejeitou nesta gan: “Até a vitória, sempre”.
quarta-feira os populismos que respondem com soluções O governo cubano decretou nove dias de luto nacional.
fáceis problemas globais e complicados, e garantiu que a Como líder da revolução cubana que derrubou o regi-
Alemanha não se fechará em si mesma e defenderá os valo- me do presidente Fulgencio Batista, em 1959, Fidel Castro
res da economia social de mercado e da justiça social com se manteve na liderança do país por décadas, até se afastar
seus aliados, a União Europeia e os Estados Unidos. da Presidência, por motivos de saúde, em 2006, deixando
Em discurso diante do plenário do parlamento em seu o cargo para o irmão.
primeiro comparecimento público após anunciar que, no Fidel Castro foi o líder mundial não ligado a uma mo-
próximo ano, voltará a concorrer para um novo mandato, narquia mais longevo do século 20, comandando Cuba por
Merkel deixou claro que a Alemanha “não pode resolver so- quase cinco décadas.
zinha os problemas do mundo”, mas garantiu que contribui- Seus apoiadores o veneravam e o consideravam um
rá para isso. libertador, que havia salvo Cuba de um regime autoritário
Merkel apostou no multilateralismo, elogiou o acordo supostamente manipulado pelo imperialismo americano.
de livre-comércio com o Canadá e admitiu que não esta- Mas seus críticos o viam como um sanguinário ditador que
va “contente” com a decisão do presidente eleito dos EUA, levou o país ao caos econômico e à ruína com suas políti-
Donald Trump, de retirar seu país do Acordo de Associação cas comunistas.
Transpacífico (TPP), uma decisão que, na opinião da chance- O anúncio de sua morte foi recebido com consterna-
ler, não beneficiará ninguém. ção em Cuba. Mas em Miami, nos Estados Unidos, onde
A chefe de governo alemã reconheceu que o atual pa- vivem milhares de exilados cubanos que deixaram a ilha
norama internacional e nacional é mais complicado que o de durante o regime comunista, houve buzinaço nas ruas.
alguns anos atrás e assumiu que compreende o medo dos Vizinho incômodo
cidadãos quando princípios que eram óbvios nas sociedades Nas quase cinco décadas em que esteve à frente do
democráticas ocidentais passam a ser questionados, por isso governo de Cuba, Fidel Castro viu dez presidentes ameri-
reiterou a necessidade de que as pessoas se mostrem unidas canos se revezarem na Casa Branca. Inimigo declarado de
frente aos populismos. todos eles, o líder comunista fez de sua ilha uma base de
E, além disso, a chanceler alemã destacou a importância resistência ao poder dos Estados Unidos, que nunca conse-
de combater as mensagens e informações manipuladas e guiram dobrar o regime incômodo a apenas 144 quilôme-
falsas que são disseminadas pela internet. tros de seu território.

26
ATUALIDADES E DEVERES DOS SERVIDORES PÚBLICOS

Nascido em uma família de latifundiários, em 1926, o Colapso soviético


jovem advogado se tornou líder revolucionário, dirigente Cuba “exportou” a sua revolução para outras partes do
comunista e terminou seus dias em uma casa confortável mundo na forma do apoio às guerrilhas marxistas em An-
em Havana, opinando sobre os mais diferentes temas, na gola e Moçambique. Sob embargo econômico dos Estados
coluna que mantinha no jornal Granma. Unidos, recebeu, em todo o tempo, ajuda soviética.
Durante este período, sofreu várias tentativas de as- O colapso da União Soviética, em 1991, foi um duro
sassinato, foi acusado de violar direitos humanos, viu sua golpe na economia cubana, apoiada na cooperação com o
principal aliada, a União Soviética, entrar em colapso. Re- antigo regime comunista.
conheceu, ao fim, erros na condução da economia cubana, A crise na qual o país mergulhou fez milhares de cuba-
que só sobreviveu nos últimos tempos graças ao apoio de nos se lançarem ao mar em embarcações precárias nos
outro amigo, o venezuelano Hugo Chávez, morto em março anos 1990, na esperança de chegar a Miami.
de 2013. O caso do menino Elián González ganhou as manchetes
Para os Estados Unidos, Fidel sempre foi uma lembran- do mundo inteiro. Ele perdeu a mãe em uma viagem peri-
ça constante e incômoda das idéias comunistas que, apesar gosa e, depois de uma longa batalha legal entre parentes
de praticamente abandonadas no resto do mundo, per- em Miami e o pai, que morava em Cuba, foi levado de volta
maneceram vivas na ilha vizinha. Para setores da esquerda para a ilha.
mundial, tornou-se um símbolo de resistência. Entre os bons resultados domésticos de Fidel Castro
estão o serviço de saúde cubano, considerado um dos me-
Revolução lhores da região, e o baixo índice de mortalidade infantil,
Fidel Castro liderou uma invasão ao quartel de Mon- comparável ao dos países mais desenvolvidos.
cada, em Santiago de Cuba, no dia 26 de julho de 1953. O governo de Fidel, no entanto, foi acusado por or-
Apesar de fracassada, a iniciativa marcou o começo da re- ganismos internacionais de perseguição política contra os
volução que acabaria levando-o ao poder. opositores do regime e de violações dos direitos humanos.
Depois de breve período preso, Fidel foi anistiado e se
exilou no México, onde organizou uma expedição que vol-
Aposentadoria
tou a Cuba.
Nos últimos anos, Fidel deu sinais de que teria mode-
Ao lado do argentino Ernesto “Che” Guevara, que co-
rado suas posições. Em 1998, recebeu no país o papa João
nheceu durante o exílio, o jovem cubano montou uma cam-
Paulo 2º.
panha de guerrilha a partir de sua base, na Serra Maestra.
Após anos de grave crise social, o regime voltou a ga-
Em 1959, Fulgêncio Batista deixou o país e Fidel estabe-
nhar fôlego na virada do milênio, com os generosos acor-
leceu um novo governo que prometia devolver a proprieda-
dos de cooperação fechados com a Venezuela do presiden-
de da terra aos agricultores e defender o direito dos pobres.
te Hugo Chávez, grande admirador de Fidel.
Fidel comunista Em 31 de julho de 2006, Fidel surpreendeu o mundo
Desde o começo, Fidel insistiu que sua ideologia era, ao deixar temporariamente o poder por motivos de saúde.
acima de tudo, cubana. “Não há comunismo nem marxis- Por meses, sua saúde foi segredo de Estado, com ru-
mo em nossas ideias, só democracia representativa e justiça mores sobre sua morte. Em fevereiro de 2008, a Assembleia
social”, dizia. Nacional de Cuba aprovou a aposentadoria de Fidel, que
Criticado pelos Estados Unidos pela nacionalização de oficialmente passou o poder ao irmão, Raúl Castro.
empresas de americanos, foi alvo do embargo comercial Fidel trocou o traje militar por roupas casuais. Con-
que vigora até hoje. tinuou a provocar polêmica com suas opiniões sobre as-
Fidel disse que assim foi empurrado para os braços da suntos mais variados, publicados em uma coluna no jornal
União Soviética, liderada por Nikita Kruchev. Com o novo Granma.
aliado, Cuba virou mais um campo de batalha da Guerra Nesse tempo, recebeu várias celebridades políticas em
Fria. sua casa, como o ex-presidente brasileiro Luiz Inácio Lula
Os Estados Unidos tentaram derrubar o governo de Fi- da Silva. Em 2011, foi visitado pelo ex-presidente americano
del em abril de 1961, apoiando um grupo de exilados cuba- Jimmy Carter. Hugo Chávez sempre foi uma visita frequente.
nos em uma desastrosa invasão à praia de Girón, na baía Em setembro de 2010, em uma entrevista à revista The
dos Porcos. Atlantic, Fidel reconheceu que o modelo cubano já não fun-
A CIA, central de inteligência americana, foi acusada cionava. No período, seu irmão, Raúl, já esboçava uma série
pelo líder cubano de tentar assassiná-lo várias vezes, inclu- de reformas econômicas, aprovadas posteriormente.
sive com um charuto explosivo. Diferentemente dos países do leste europeu, cujos
Em 1962, aviões de reconhecimento dos Estados Uni- governos ruíram após o colapso da União Soviética, Fidel
dos detectaram um carregamento de mísseis soviéticos conseguiu manter Cuba sob o regime comunista até sua
rumo a Cuba, criando um impasse entre o presidente ame- morte. Mesmo afastado da Presidência, Fidel permaneceu
ricano, John F. Kennedy, e Kruchev. como secretário-geral do Partido Comunista de Cuba até
Depois de 13 dias de impasse, os soviéticos desistiram abril de 2011.
de instalar mísseis com potencial nuclear em Cuba, em tro- Nos últimos anos, ele se manteve afastado da vida pú-
ca de uma promessa secreta americana de retirar suas ar- blica, com raras aparições e eventuais colunas publicadas na
mas da Turquia. mídia oficial cubana.

27
ATUALIDADES E DEVERES DOS SERVIDORES PÚBLICOS

Em abril deste ano, ele surpreendeu ao discursar no Foi esse ministério, junto com o do Comércio, o prin-
último dia do congresso do Partido Comunista. “Em breve cipal responsável por analisar, nos últimos 23 meses, até
terei 90 anos, algo que nunca imaginei”, afirmou. “Logo se- onde seria possível flexibilizar os limites impostos pelo em-
rei como todos os outros, que para todos nossa hora deve bargo, cuja eliminação está nas mãos do Congresso. Em
chegar”, disse. minoria na Câmara e no Senado, a estratégia do Governo
Fonte: BBC Brasil – (26/11/2016) Obama foi tentar facilitar ao máximo as transações comer-
ciais e os intercâmbios pessoais entre os dois países. Em-
Morte de Fidel Castro amplia dúvidas sobre reapro- bora as principais restrições continuem vigentes, é cada vez
ximação com os EUA mais fácil para os cidadãos norte-americanos fazer transa-
A morte de Fidel Castro acrescenta mais uma incógnita ções comerciais com Cuba ou viajar à ilha.
ao processo de normalização das relações entre os Estados Antes que a morte de Fidel Castro monopolizasse to-
Unidos e Cuba, já colocado em xeque pela vitória eleitoral das as manchetes da imprensa cubana – e mundial –, os
do republicano Donald Trump, que será responsável por meios de comunicação estatais comemoravam justamente
manter – ou interromper – o diálogo aberto há quase dois o restabelecimento, na próxima segunda-feira, dos voos
anos com Havana por seu antecessor democrata, Barack comerciais regulares e diretos entre os EUA e Havana, sus-
Obama. pensos durante mais de 50 anos.
Apesar de o histórico líder revolucionário nunca ter São medidas como esta as que também impulsiona-
ocultado suas reticências quanto ao processo iniciado ram, por sua vez, a continuação das reformas iniciadas com
pelo presidente Raúl Castro, seu irmão, o fato de não fa- a chegada de Raúl Castro ao poder em Cuba, embora não
zer oposição frontal ao degelo foi considerado como uma ao ritmo desejado por Washington, como reconheceu o
aprovação implícita a essa iniciativa diplomática, que não próprio Obama. Uma mudança na atitude de Washington
necessariamente contava com o respaldo de toda a cúpula poderia ter, neste sentido, adverte López-Levy, mais impac-
cubana. to ainda que a morte de Fidel Castro. “Enquanto exista in-
Quis o acaso que a morte de Fidel Castro surpreen- certeza no assunto Trump, a direção cubana vai atuar com
desse Trump em Mar-a-Lago, a mansão da Flórida onde o grande cautela, mas isso não tem a ver com o fato de que
Fidel esteja ou não porque já tinha um papel mais simbóli-
magnata costuma passar férias. Esse Estado é o mais tradi-
co, era uma espécie de força moral, de patriarca revolucio-
cional reduto cubano nos EUA, um lugar outrora claramen-
nário mais que líder dos assuntos do governo”.
te anticastrista, mas que, sobretudo nos últimos anos, pas-
Fonte: El País Brasil – (26/11/2016)
sou a apoiar a política conciliadora de Obama, incluindo a
decisão de restabelecer as relações interrompidas durante
Trump diz que pode acabar com acordo entre EUA
mais de meio século.
e Cuba
Obama fez de tudo para consolidar essa política antes
O presidente eleito dos Estados Unidos, Donald Trump,
de deixar a Casa Branca, o que acontecerá em menos de escreveu em sua conta no Twitter nesta segunda-feira (28)
dois meses. Não só reabriu, já há mais de um ano, a em- que vai acabar com o acordo de seu país com Cuba se a
baixada norte-americana em Havana, gesto replicado por ilha não estiver disposta a oferecer um acordo melhor.
Cuba em Washington, como se tornou, em março, no pri- “Se Cuba não quiser fazer um acordo melhor para o
meiro presidente dos EUA em quase um século a pisar em povo cubano, o povo cubano-americano e os Estados Uni-
solo cubano. A menos de um mês das eleições que definiria dos como um todo, vou acabar com o acordo”, escreveu o
seu sucessor, Obama emitiu uma ordem executiva (espécie magnata.
de medida provisória) com a qual pretendia, nas suas pala- No sábado (26), Trump divulgou um comunicado à im-
vras, tornar “irreversíveis” os avanços obtidos nas relações prensa classificando Fidel Castro como um “ditador brutal
bilaterais. que oprimiu seu próprio povo por quase seis décadas” e
Tudo, porém, se tornou um enorme ponto de interro- que deixa um “legado de pelotões de fuzilamento, roubo,
gação após a vitória do republicano Trump, um bilionário inimaginável sofrimento, pobreza e negação de direitos
pragmático que no passado foi acusado de violar o embar- humanos básicos”.
go econômico a Cuba em busca de negócios lucrativos na No texto, ele afirmou que seu governo “vai fazer todo
ilha. Durante a campanha eleitoral, no entanto, ele prome- o possível para assegurar que o povo cubano possa final-
teu reverter a aproximação com Havana. mente começar sua jornada em direção à prosperidade e
Trump não se contentou apenas em cortejar o voto à liberdade”.
mais abertamente anticastrista em Miami. Já eleito presi- Nas primárias, Trump foi o único pré-candidato repu-
dente, parece confirmar suas promessas ao incluir em sua blicano que apoiou a abertura para Cuba, mas em sua bus-
equipe de governo figuras proeminentes do lobby pró-em- ca de votos na Flórida nas eleições gerais, ele prometeu
bargo, como o advogado Mauricio Clever-Carone, mem- que “revogaria” o acordo do presidente Barack Obama “a
bro da influente organização Democracia Cuba-EUA, que não ser que o regime dos Castro” restaurasse “as liberda-
defende uma “transição incondicional de Cuba à demo- des na ilha”, segundo a agência EFE.
cracia e ao livre mercado”. Ele irá trabalhar com Trump no Seu futuro chefe de gabinete, Reince Priebus, disse no
Departamento do Tesouro, uma peça-chave na aplicação domingo que Trump aguardará para ver “alguns movimen-
– ou flexibilização – do embargo econômico a Cuba e das tos” do governo cubano em relação às liberdades na ilha
sanções contra quem o viola. para decidir como será a relação entre os dois países. “Não

28
ATUALIDADES E DEVERES DOS SERVIDORES PÚBLICOS

vamos ter um acordo unilateral procedente de Cuba sem al- Em uma entrevista à rede britânica “BBC” divulgada
gumas mudanças em seu governo”, disse Priebus na TV Fox, nesta quarta-feira, Brennan também afirmou que Trump
após mencionar a repressão, os prisioneiros políticos e as li- deve ser cauteloso com a Rússia por considerar que Mos-
berdades como a religiosa. cou está por trás de grande parte do sofrimento na Síria.
Grupos do exílio cubano mostraram apoio unânime ao Durante a campanha para as eleições americanas, o po-
aviso de Trump, segundo a EFE. lítico republicano ameaçou abandonar o pacto nuclear en-
Em coletiva de imprensa na Casa Branca, o porta-voz do tre o G5+1 (EUA, Rússia, França, Reino Unido e China, mais
governo americano Josh Earnest disse que os críticos da atual Alemanha) e o Irã e sugeriu que o governo americano teria
política de aproximação “estão dando voltas, tentando justi- uma relação muito mais estreita com o governo russo.
ficar sua lealdade a uma política obviamente fracassada de “Acredito que isto seria desastroso. O fato de uma ad-
isolamento de Cuba, que nunca teve nenhum resultado”. ministração encerrar um acordo feito pela administração
Para o porta-voz da Casa Branca, “dar declarações ruido- anterior não teria precedentes”, disse Brennan à emissora
sas e iniciar um caso de recriminações mútuas amarradas ao
britânica antes de deixar seu cargo em janeiro.
passado não faz nem a democracia nem a liberdade avança-
Para o diretor da CIA, uma medida assim, que qualificou
rem, nem expande oportunidades”.
de “loucura”, ajudaria a fortalecer os políticos linha dura no
“Os críticos da atual política sugerem que, de alguma
Irã.
forma, os Estados Unidos fizeram um pacote de concessões
ao governo cubano. Isso é equivocado. Não há concessões”, Em suas declarações, Brennan ressaltou que há muitas
afirmou. áreas nas quais o novo governo tem que agir com “pru-
Earnest acrescentou que cada presidente que ocupa a dência e disciplina”, como a linguagem utilizada em matéria
Casa Branca deve se perguntar se “estaremos ancorados no terrorista e as relações com a Rússia.
passado, ou se vamos olhar para o futuro. Isso não significa O diretor da CIA opinou que o regime sírio de Bashar al
ignorar o passado, mas fazer que o passado não interfira em Assad e a Rússia foram responsáveis pelo massacre de civis
nossa capacidade de fazer progressos”. no conflito sírio, que qualificou de “degradante”.
Na opinião do chefe de inteligência dos EUA, seu país
O acordo deveria continuar com o respaldo dado pela Administração
No dia 17 de dezembro de 2014, os presidentes Barack de Barack Obama aos rebeldes moderados que lutam con-
Obama e Raúl Castro anunciaram o restabelecimento das re- tra o regime de Assad.
lações dos Estados Unidos e Cuba após mais de 50 anos. O Brennan acrescentou que a Rússia é crucial para o futu-
embargo comercial ao país caribenho, no entanto, permane- ro da Síria, mas se mostrou cético sobre a possibilidade de
ceu. um acordo que ajude a pôr fim à guerra civil.
Na época, Cuba libertou o prisioneiro americano Alan “Não tenho confiança de que os russos vão ceder até
Gross e, em troca, três agentes de inteligência cubanos que que possam ser capazes de conseguir o maior sucesso tá-
estavam presos nos Estados Unidos voltaram à ilha. tico possível no campo de batalha”, disse o diretor da CIA.
O acordo previa medidas como o restabelecimento das Além disso, Brennan advertiu sobre a contínua ameaça
relações diplomáticas entre os dois países, facilitar viagens de terrorista já que há grupos “muito ativos” dentro do Estado
americanos a Cuba, autorização de vendas e exportações de Islâmico (EI) que planejam atentados e querem demonstrar
bens e serviços dos EUA para Cuba, autorização para norte-a- sua capacidade de operar no Ocidente.
mericanos importarem bens de até US$ 400 de Cuba e início O diretor da CIA acrescentou que é preciso cuidado no
de novos esforços para melhorar o acesso de Cuba a teleco- uso da linguagem porque isto pode ser aproveitado por or-
municação e internet.
ganizações terroristas para mostrar que os EUA estão con-
Em agosto de 2015, os EUA reabriram oficialmente sua
tra o islã, algo que, segundo ele, não é assim.
embaixada em Havana. Um mês antes, a embaixada cubana
Trump indicou que quer o congressista Mike Pompeo
em Washington foi reaberta.
para o posto de novo diretor da CIA.
Ao anunciar o acordo, Obama disse que as normalizações
das relações com Cuba encerram uma “abordagem antiqua- Fonte: Portal Terra – (30/11/2016)
da” da política externa americana. Ao justificar a decisão, o
presidente disse que a política “rígida” dos EUA em relação a Senado colombiano referenda acordo de paz com
Cuba nas últimas décadas teve pequeno impacto. O presiden- as Farc
te americano afirmou acreditar que os EUA poderão “fazer O Senado da Colômbia referendou no início da madru-
mais para ajudar o povo cubano” ao negociar com o governo gada desta quarta-feira (30) o acordo de paz assinado no
da ilha. último dia 24 entre o governo e as Farc.
Fonte: G1 – (28/11/2016) “Com 75 votos a favor e nenhum contra, o plenário do
Senado aprovou o novo acordo de paz. Fica pendente sua
CIA adverte Trump que seria “loucura” abandonar aprovação na Câmara dos Deputados”, afirmou o ministro
acordo com Irã do Interior, Juan Fernando Cristo, em comunicado.
O diretor da CIA, John Brennan, advertiu ao presidente De acordo com o ministro, “com a participação de ví-
eleito dos Estados Unidos, Donald Trump, que seria “desastro- timas, representantes de igrejas cristãs, negritudes, indíge-
so” e uma “loucura” deixar o acordo nuclear com o Irã, como nas, foi discutido e aprovado no Senado o novo acordo de
o magnata ameaçou fazer durante a campanha eleitoral. paz que foi assinado na última quinta-feira”.

29
ATUALIDADES E DEVERES DOS SERVIDORES PÚBLICOS

Com este passo, esclareceu Cristo, “a esperada aprova- Atlas, a eficiência do processo também é notável: cerca de
ção na Câmara dos Deputados, nesta quarta-feira, poderá 63% do material utilizado foi convertido em etanol. Trata-
começar a implementar o acordo estipulado entre o gover- se de uma surpresa positiva, já que, geralmente, tentativas
no e as Farc”. de gerar etanol a partir de gás carbônico resultam em uma
O debate no Senado foi aberto pelo principal negocia- série de produtos menos úteis, como etileno e monóxido
dor do governo, Humberto de la Calle, e o Alto Comissaria- de carbono. Os detalhes da reação foram publicados pelos
do para a Paz, Sergio Jaramillo, que fizeram um acordo de pesquisadores em um artigo no periódico Chemistry Select.
confissão em favor do acordo, um texto renovado após a
rejeição no referendo realizado no dia 2 de outubro, após Nanotorres de carbono
ter sido assinado no dia 26 de setembro, em Cartagena. Durante o experimento, os psquisadores usaram um
Nesta quarta, a Câmara dos Deputados estará em ses- catalisador feito de carbono e cobre. Por meio de um mé-
são desde o início da manhã para discutir e referendar o todo de deposição de vapores químicos que tinha amônia
novo acordo de paz. e acetileno como reagente, eles conseguiram organizar o
Fonte: G1 – (30/11/2016) carbono e o cobre em estruturas semelhantes a “torres”.
Essas torres tinham cerca de 50 a 80 nanômetros de altu-
MEIO AMBIENTE ra, e terminavam num pico de cerca de 2 nanômetros de
diâmetro.
Restam menos de 300 onças-pintadas na Mata Quando uma carga de apenas 1.2 volt era aplicada a
Atlântica uma superfície contendo essa estrutura, as partículas de
A onça-pintada está definitivamente ameaçada de ex- cobre nas torres de carbono faziam com que um campo
tinção na Mata Atlântica. Menos de 300 desses magníficos elétrico muito forte fosse criado na região. Esse campo, por
felinos ainda sobrevivem no bioma, espalhados e isolados sua vez, fazia com que correntes elétricas se formassem
em pequenas populações pelo Brasil, Argentina e Paraguai, entre os picos, e essas correntes dimerizavam moléculas de
segundo um trabalho publicado nesta quarta-feira, 16, na CO2 em moléculas de etanol.
revista Scientific Reports. É o mais completo levantamento Para realizar esse processo, o gás carbônico precisa-
já feito sobre a população remanescente de onças-pinta- va ser diluído em água, segundo o Popular Mechanics.
das na Mata Atlântica. Em seguida, a superfície texturizada com essas estruturas
As causas do declínio são óbvias. Cerca de 85% do há- nanométricas era colocada na água e recebia a voltagem.
bitat original das onças-pintadas (ou jaguares, como tam-
Com isso, a reação começava a ocorrer. “Usando materiais
bém são conhecidas) na Mata Atlântica já desapareceu, e
comuns, mas arranjando-os com nanotecnologia, nós con-
apenas 7% das florestas que restam ainda estão em bom
seguimos limitar as reações colaterais e ter o único produto
estado de conservação, com tamanho e alimento suficien-
que queríamos”, disse Adam Rondinone, um dos cientistas
tes para abrigar a espécie, segundo os cientistas. Não bas-
envolvidos no projeto.
tasse isso, as poucas onças sobreviventes são frequente-
mente perseguidas e atacadas por caçadores e fazendeiros.
Salvando o planeta
“Perda e fragmentação de hábitat são as principais
causas de declínio das onças-pintadas, mas a mortalida- Como esse processo transforma um gás estufa em
de induzida pelo homem é a principal ameaça às popula- um combustível, ele pode ser extremamente importante
ções remanescentes”, dizem os autores do trabalho, que no combate ao efeito estufa. O efeito estufa é o aqueci-
incluem pesquisadores do Brasil, Argentina, Paraguai e mento da atmosfera terrestre provocado pelo aumento
Porto Rico. A Mata Atlântica, segundo eles, corre risco de da concentração de gases estufa (como o gás carbônico)
se tornar a primeira floresta no mundo a ter o seu maior nela. Esse aumento, por sua vez, tem como uma de suas
predador extinto. principais causas a queima de combustíveis fósseis, como
Fonte: Istoé.com – (16/11/2016) a gasolina.
Com a possibilidade de transformar um gás estufa em
Sem querer, cientistas descobrem processo que um combustível não-fóssil, abre-se uma porta importan-
pode amenizar efeito estufa te para o combate ao problema. Outras vantagens desse
Uma equipe de pesquisadores do Oak Ridge National processo são que ele é relativamente barato e pode ser ini-
Laboratory (ORNL), nos Estados Unidos, acabou descobrin- ciado em temperatura ambiente - ou seja, não exige muita
do sem querer um processo para transformar CO2 (gás car- energia para ser iniciado, o que o torna ainda mais eficien-
bônico, um gás estufa) em etanol (C2H5OH, um combustí- te. Os cientistas ainda acreditam que, por esses motivos,
vel). A descoberta pode ser extremamente importante no ele pode ser realizado em escala industrial, com possíveis
combate ao efeito estufa. ganhos em eficiência.
Os cientistas estavam testando um catalisador feito de Resolver o problema dos gases estufa é um dos gran-
carbono e cobre arranjados em uma estrutura nanométri- des desafios da humanidade atualmente, e por isso diver-
ca. A ideia deles era utilizar esse catalisador para uma série sas formas de fazê-lo já estão sendo estudadas. Outra pes-
de reações que teriam, ao final, um combustível como re- quisa publicada em 2016 descreve também um processo
sultado (inicialmente, eles imaginaram que seria o meta- para armazenar o gás carbônico da atmosfera na forma de
nol). Para a surpresa deles, no entanto, a primeira etapa calcário a fim de combater o efeito estufa.
dessa reação já teve etanol como produto. Segundo New Fonte: Olhar Digital (Uol) – (18/11/2016)

30
ATUALIDADES E DEVERES DOS SERVIDORES PÚBLICOS

Marrakech inaugura uma nova era nas negociações Os primeiros passos dados em Marrakech para definir
sobre o clima essas regras foram “mínimos”, pois “é preciso muita capa-
A Cúpula do Clima de Marrakech (COP22) iniciou uma cidade diplomática para conseguir o consenso dos cerca
nova etapa nas negociações internacionais para combater de 200 países que participam das cúpulas, algo que a pre-
a mudança climática, na qual despontam novas lideranças sidência marroquina não teve”, disse o especialista em re-
e alianças para conduzir a transição para uma economia lações internacionais Doreen Stabinsky.
baixa em carbono, conforme o Acordo de Paris. No entanto, foi estabelecido um programa de trabalho
A reunião de Marrakech, que foi concluída na madru- até 2018 para concretizar essas normas nos próximos dois
gada deste sábado, representou a celebração da entrada anos e, embora pareça muito tempo, não o é se compara-
em vigor do Acordo de Paris, 11 meses após sua aprovação, do com o Protocolo de Kioto, onde esse processo demorou
em comparação com os sete anos do Protocolo de Kioto. quatro anos, apesar de que o mesmo só incluía obrigações
O novo tratado somou 11 novas ratificações na COP22, para os países ricos, e não para todos, como o Acordo de
incluindo hoje 111 países, que representam mais de 80% Paris.
das emissões mundiais. O que a COP22 deixou evidente é que a transição para
A vitória de Donald Trump nas eleições dos Estados um novo modelo de desenvolvimento baixo em carbono
Unidos, alguém que chamou a mudança climática de “con- está em andamento, e não só por parte dos países, mas
to chinês” e prometeu tirar seu país do pacto climático, caiu de regiões, cidades e grandes empresas, que apresentaram
como uma ducha de água fria na primeira das duas sema- ações, compromissos de financiamento e vontade de com-
nas da cúpula, mas, paradoxalmente, serviu para que todos partilhar conhecimento e soluções neste processo.
os países fizessem um esforço para defender o acordo. Quatro países (Canadá, Alemanha, México e Estados
“Não há um único país que tenha mostrado em Marra- Unidos) registraram na ONU suas estratégias para “des-
kech sua intenção de deixar o acordo”, lembrou à Agência carbonizarem” sua economia em meados deste século e
Efe o comissário de Ação pelo Clima da União Europeia, o outros 18 anunciaram que estão fazendo os últimos pre-
espanhol Miguel Arias Cañete. parativos das suas.
China, Índia e até Arábia Saudita afirmaram que “o As quase 50 nações mais vulneráveis à mudança climá-
Acordo de Paris é um caminho sem volta” e mostraram sua tica prometeram que 100% de sua energia será oriunda de
determinação de trabalhar em nível nacional para realizar fontes renováveis, “assim que for possível”, e um grupo de
países ricos liderados pela Alemanha apresentou a iniciati-
as promessas feitas no mesmo.
va “NDC Partnership” para ajudar os países em desenvolvi-
Resta saber quem assumirá a liderança exercida nesta
mento a reduzir a mudança climática.
matéria pelo governo Obama se Trump cumprir sua pro-
Fonte: Portal Terra – (19/11/2016)
messa de sair do pacto, o que levaria quatro anos, e da
própria convenção de mudança climática da ONU, que foi
CIÊNCIA E TECNOLOGIA
assinada por um presidente republicano, George Bush pai,
há 25 anos.
‘Pescar’, puxar e empurrar: os planos dos cientistas
Fontes da delegação americana - que ainda fazem par- para ‘recolher’ o lixo espacial
te da equipe de Obama - confirmaram para a Agência Efe Desde o início da exploração espacial nos anos de
os “intensos esforços diplomáticos” que estão sendo rea- 1950, toneladas de lixo espacial estão se acumulando na
lizados para que Trump não anunciasse a saída do Acordo órbita da Terra.
de Paris, enquanto um de seus maiores promotores, o se- Para resolver o problema, cientistas planejam enviar ao
cretário de Estado John Kerry, o defendia abertamente em espaço no ano que vem uma nave criada para testar ma-
Marrakech. neiras de se livrar desses detritos.
Ontem à noite, o primeiro-ministro de Fiji, que presidi- Pesquisadores da Universidade de Southampton, no
rá a próxima Cúpula do Clima (COP23), convidou Trump a Reino Unido, monitoram o lixo espacial na órbita terrestre.
visitar a ilha no Pacífico-Sul para ver os efeitos do aumento Eles estimam que haja cerca de 22 mil pedaços com tama-
do nível do mar sobre a mesma. nho superior a 10 centímetros.
“Ninguém vai abandonar o acordo como ocorreu com Detritos menores, porém, chegam à casa dos milhões.
o Protocolo de Kioto. A chave está em quem irá ocupar o O lixo espacial é formado em geral por partes e com-
espaço de liderança que será deixado por Obama”, afir- ponentes de satélites, foguetes e naves descartados duran-
mou a diretora da ONG Greenpeace, Jennifer Morgan, que, te missões espaciais.
junto com outros especialistas, opinou que houve indícios A nave que será colocada em órbita no ano que vem
na CO22 de que a China e países europeus como a Alema- testará maneiras de limpar esse lixo. Ela vai usar uma rede
nha poderiam liderar a transição para um desenvolvimento e um arpão para capturar esses destroços.
com baixas emissões de carbono. Vai usar ainda uma vela para tentar forçar grandes des-
Com o histórico pacto do clima firmado um ano antes, troços a entrar na atmosfera - onde são destruídos pela
a COP22 tinha como objetivo definir um livro de regras para alta temperatura de reentrada.
o mesmo, já que o Acordo de Paris deixou em aberto como Mas haverá outras dificuldades a superar - a principal
seriam implementadas as medidas que propõe, como por delas deve ser o financiamento dessas missões.
exemplo, a maneira na qual os países vão notificar e revisar Cientistas estimam que custará milhões para retirar um
seus compromissos nacionais de redução das emissões. único detrito.

31
ATUALIDADES E DEVERES DOS SERVIDORES PÚBLICOS

Além disso, eles não podem, por questões legais, cap- cala. Esse problema ocorreu logo depois da abertura do
turar lixo espacial aleatoriamente - para recolher um satélite paraquedas e o sinal anormal durou cerca de um segundo
obsoleto, por exemplo, é preciso que o país responsável por - mais do que o esperado pelos cientistas.
ele concorde com isso. Com isso, o sistema de navegação deduziu que a alti-
Mas os pesquisadores alertam que, se nada for feito em tude de Schiaparelli fosse negativa, ou seja, que o módulo
relação ao problema, será impossível manter satélites ou estivesse abaixo da superfície marciana. Foi nesse momen-
operar no espaço no futuro. to que o pouso começou a dar errado e resultou na que-
Fonte: G1 – (28/11/2016) da. A reconstrução do acidente de Schiaparelli, feita por
computador, representa ainda uma “conclusão muito pre-
Cientistas criam bateria de celular que carrega em se- liminar das nossas suposições técnicas”, disse o diretor de
gundos e dura vários dias Robótica da ESA, David Parker.
Recarregar os celulares em poucos segundos e menos Parker ainda explica que, para ter um quadro mais com-
de uma vez por semana poderá ser realidade no futuro. Isso pleto, é preciso esperar até os primeiros meses de 2017.
graças aos novos supercondensadores desenvolvidos por Isso porque será publicado um artigo “de uma comissão
especialistas de nanotecnologia, na Universidade da Flórida de investigação independente em fase de constituição, re-
Central. As informações são da Agência ANSA. querida pelo diretor general da ESA e que será coordenada
Os estudiosos desenvolveram dispositivos que são capa- pelo Inspetor General da ESA”.
zes de armazenar rapidamente mais energia que as tradicio- No entanto, os responsáveis da missão Schiaparelli
nais baterias de lítio e sem perder sua estabilidade energética mantém o otimismo. “Nós aprendemos muitas coisas com
durante mais de 30 mil recargas. Hoje, uma bateria normal- o Schiaparelli que contribuirão diretamente com a segunda
mente começa a perder cada vez mais potência a partir do missão ExoMars, que está sendo desenvolvida com os nos-
18° mês de uso. Em média, isso soma 1,5 mil ciclos com es- sos parceiros internacionais e tem um lançamento previsto
tabilidade intacta. para 2020”, enfatiza Parker.
A pesquisa, publicada na revista especializada “ACS Com a mesma visão, complementa Roberto Battiston,
Nano”, reporta que a nova tecnologia poderá ser expandida presidente da Agência Espacial Italiana (ASI). “ExoMars é
para os carros elétricos. O segredo da inovação está no uso
extremamente importante para a ciência e exploração eu-
de baterias bidimensionais. Muitos pesquisadores já haviam
ropeia. Agora, junto com nossos parceiros norte-america-
tentando usar a técnica no passado, por exemplo, com o gra-
nos do programa, trabalharemos para o sucesso da segun-
feno. Mas ninguém tinha conseguido efetivamente alcançar
da missão”, disse o italiano.
tal potencial.
Fonte: Jornal do Brasil – (23/11/2016)
O grupo norte-americano liderado por Yeonwoon “Eric”
Jung ganhou este desafio tecnológico aproveitando um novo
Anvisa aprova regras para registro de remédio à
enfoque de síntese química, juntamente com superconden-
sadores compostos por milhões de microscópicos fios, reves- base de maconha
tidos por materiais bidimensionais. Dessa forma, o “coração” A Agência Nacional de Vigilância Sanitária (Anvisa)
dos eletrônicos se torna um alto condutor de energia, e com incluiu hoje (22) os derivados da Cannabis sativa, a ma-
mais densidade, energia e potência. conha, na lista de substâncias psicotrópicas, vendidas no
No entanto, o maior empecilho atual seria o tamanho Brasil com receita do tipo A específica para entorpecentes.
dessas baterias, que seriam muito maiores do que as de lí- A norma permite que empresas registrem no país produtos
tio. “[Esses materiais] ainda não estão sendo comercializados, com canabidiol e tetraidrocanabinol como princípio ativo,
mas são uma demonstração da comprovação de um impor- passo necessário para venda de remédios.
tante começo: nossos estudos mostram que terão impactos A medida faz parte da atualização da Portaria nº 344/98,
muito fortes sobre muitas tecnologias”, explicou Jung. que também estabelece que laboratórios registrem os de-
Fonte: Jornal do Brasil – (23/11/2016) rivados em concentração de, no máximo, 30 mg de tetrah-
idrocannabinol (THC) por mililitro e 30 mg de canabidiol
Schiaparelli caiu por ‘erro de cálculo’ em altura, diz por mililitro. Os produtos que tiverem concentração maior
ESA do que a estabelecida continuam proibidos no país.
Novos dados da Agência Espacial Europeia (ESA) revela- Segundo nota da agência reguladora, a medida foi
ram que o módulo Schiaparelli caiu no solo de Marte a 3,7 motivada pela fase final do processo de registro do medi-
km de altura, no dia 19 de outubro. Os novos detalhes da ESA camento Mevatyl®. O produto que, em alguns países da
supõem quais foram os erros da missão. O principal motivo Europa, tem o nome comercial de Sativex, pode vir a ser
do acidente deve-se a uma falha durante a medida de distân- o primeiro obtido da Canabis sativa registrado no país. O
cia do Schiaparelli durante o pouso. A interpretação errada medicamento será indicado para o tratamento de sintomas
de sua altura fez com que o módulo se precipitasse ao início de pacientes adultos com esclerose múltipla.
do procedimento de pouso. Fonte: Jornal do Brasil – (22/11/2016)
De acordo com os cientistas, apesar do radar altímetro
doppler do módulo ter funcionado normalmente, o dispo-
sitivo de medida inercial (IMU - Inertial Measurement Unit),
que registra a velocidade de rotação do veículo, começou
a enviar um sinal saturado, isto é, o valor máximo da es-

32
ATUALIDADES E DEVERES DOS SERVIDORES PÚBLICOS

Questões: 6) A presidente da Coreia do Sul pediu sua renúncia


após um escândalo envolvendo:
1) O Senado Federal aprovou a proposta de reforma a) Corrupção ativa
política, que, entre outras medidas, criou a “cláusula de b) Desvio de recursos públicos
barreira”, para reduzir o número de partidos políticos. c) Corrupção passiva
Segundo tal cláusula, os partidos precisam de: d) Peculato
a) 1% dos votos válidos nas eleições para a Câmara dos e) Tráfico de influência
Deputados em 2018 e, 2% a partir de 2022.
b) 2% dos votos válidos nas eleições para a Câmara dos 7) A Revolução Cubana, ocorrida em 1959, destituiu
Deputados em, pelo menos, 16 Estados, em 2018, e 3% a o ditador Fulgêncio Batista. Tal Revolução teve como
partir de 2022, com 1,5% dos votos válidos em 14 Estados. líder uma importante figura do século XX, que faleceu
c) 2% dos votos válidos nas eleições para a Câmara em novembro de 2016:
dos Deputados, distribuídos em, ao menos, 14 Estados, em a) Raúl Castro
2018, e 3% a partir de 2022, com 2% dos votos válidos em b) Hugo Chávez
14 Estados. c) Nikita Kruchev
d) 2% dos votos válidos nas eleições para a Câmara d) Fidel Castro
dos Deputados, distribuídos em, ao menos, 14 Estados, em e) Evo Morales
2018, e 2% a partir de 2022 em 20 Estados.
d) 1% dos votos válidos nas eleições para a Câmara 8) A Cúpula do Clima de Marrakech, realizada em
dos Deputados, distribuídos em, ao menos, 14 Estados, em novembro de 2016, também é chamada de:
2018, e 2% a partir de 2022 em 20 Estados. a) Cop 15
b) Cop 17
2) Com a saída de Marcelo Calero do Ministério da c) Cop 22
Cultura, após o imbróglio envolvendo o também mi- d) Cop 21
nistro Geddel Vieira Lima, quem assumiu a pasta foi o e) Cop 20
deputado:
a) Roberto Freire Gabarito: 1-C/2-A/3-D/4-C/5-B/6-E/7-D/8-C
b) Eliseu Padilha
c) Torquato Jardim
d) Leonardo Picciani
e) Ricardo Barros

3) No terceiro trimestre de 2016, o PIB brasileiro


apresentou:
a) Queda de 0,5%
b) Alta de 0,1%
c) Alta de 0,3%
d) Queda de 0,8%
e) Queda de 1%

4) Em novembro de 2016, a primeira turma do Su-


premo Tribunal Federal (STF) aprovou a descriminaliza-
ção do aborto, desde que ocorra até o:
a) 1º mês
b) 2º mês
c) 3º mês
d) 4º mês
e) 5º mês

5) O avião que vitimou a equipe da Chapecoense,


deixando 71 mortos, tinha como destino a cidade de:
a) Bogotá, na Colômbia
b) Medellín, na Colômbia
c) Quito, no Equador
d) Buenos Aires, na Argentina
e) Caracas, na Venezuela

33
ATUALIDADES E DEVERES DOS SERVIDORES PÚBLICOS

PROF. Mª BRUNA PINOTTI GARCIA OLIVEIRA Para memorizar: veja que as iniciais das palavras formam
o vocábulo LIMPE, que remete à limpeza esperada da
Advogada e pesquisadora. Doutoranda em Direito, Es- Administração Pública. É de fundamental importância um
tado e Constituição pela Universidade de Brasília – UNB. olhar atento ao significado de cada um destes princípios,
Mestre em Teoria do Direito e do Estado pelo Centro Uni- posto que eles estruturam todas as regras éticas prescritas
versitário Eurípides de Marília – UNIVEM (bolsista CAPES). no Código de Ética e na Lei de Improbidade Administrativa,
Professora de curso preparatório para concursos e univer- tomando como base os ensinamentos de Carvalho Filho1 e
sitária na Universidade Federal de Goiás – UFG. Autora de Spitzcovsky2:
diversos trabalhos científicos publicados em revistas qua- a) Princípio da legalidade: Para o particular, legalidade
lificadas, anais de eventos e livros, notadamente na área significa a permissão de fazer tudo o que a lei não proíbe.
do direito eletrônico, dos direitos humanos e do direito Contudo, como a administração pública representa os
constitucional. interesses da coletividade, ela se sujeita a uma relação
de subordinação, pela qual só poderá fazer o que a lei
expressamente determina (assim, na esfera estatal, é preciso
lei anterior editando a matéria para que seja preservado o
2. ESTATUTO DOS FUNCIONÁRIOS princípio da legalidade). A origem deste princípio está na
PÚBLICOS CIVIS DO ESTADO DE SÃO PAULO criação do Estado de Direito, no sentido de que o próprio
(LEI N.º 10.261/68) - ARTIGOS 239 A 250 Estado deve respeitar as leis que dita.
b) Princípio da impessoalidade: Por força dos
interesses que representa, a administração pública
DISCIPLINA CONSTITUCIONAL está proibida de promover discriminações gratuitas.
Discriminar é tratar alguém de forma diferente dos demais,
Antes de abordarmos o estatuto dos funcionários privilegiando ou prejudicando. Segundo este princípio, a
públicos civis do Estado de São Paulo é importante analisar administração pública deve tratar igualmente todos aqueles
que se encontrem na mesma situação jurídica (princípio
a disciplina constitucional aplicável a estes servidores
da isonomia ou igualdade). Por exemplo, a licitação
públicos.
reflete a impessoalidade no que tange à contratação de
serviços. O princípio da impessoalidade correlaciona-se ao
1) Princípios da Administração Pública
princípio da finalidade, pelo qual o alvo a ser alcançado
Os valores éticos inerentes ao Estado, os quais
pela administração pública é somente o interesse público.
permitem que ele consolide o bem comum e garanta a
Com efeito, o interesse particular não pode influenciar no
preservação dos interesses da coletividade, se encontram
tratamento das pessoas, já que deve-se buscar somente a
exteriorizados em princípios e regras. Estes, por sua vez,
preservação do interesse coletivo.
são estabelecidos na Constituição Federal e em legislações c) Princípio da moralidade: A posição deste princípio
infraconstitucionais, a exemplo das que serão estudadas no artigo 37 da CF representa o reconhecimento de
neste tópico, quais sejam: Decreto n° 1.171/94, Lei n° uma espécie de moralidade administrativa, intimamente
8.112/90 e Lei n° 8.429/92. relacionada ao poder público. A administração pública
Todas as diretivas de leis específicas sobre a ética não atua como um particular, de modo que enquanto
no setor público partem da Constituição Federal, que o descumprimento dos preceitos morais por parte
estabelece alguns princípios fundamentais para a deste particular não é punido pelo Direito (a priori), o
ética no setor público. Em outras palavras, é o texto ordenamento jurídico adota tratamento rigoroso do
constitucional do artigo 37, especialmente o caput, que comportamento imoral por parte dos representantes do
permite a compreensão de boa parte do conteúdo das leis Estado. O princípio da moralidade deve se fazer presente
específicas, porque possui um caráter amplo ao preconizar não só para com os administrados, mas também no âmbito
os princípios fundamentais da administração pública. interno. Está indissociavelmente ligado à noção de bom
Estabelece a Constituição Federal: administrador, que não somente deve ser conhecedor da
lei, mas também dos princípios éticos regentes da função
Artigo 37, CF. A administração pública direta e indireta administrativa. TODO ATO IMORAL SERÁ DIRETAMENTE
de qualquer dos Poderes da União, dos Estados, do Distrito ILEGAL OU AO MENOS IMPESSOAL, daí a intrínseca ligação
Federal e dos Municípios obedecerá aos princípios de com os dois princípios anteriores.
legalidade, impessoalidade, moralidade, publicidade e d) Princípio da publicidade: A administração pública
eficiência e, também, ao seguinte: [...] é obrigada a manter transparência em relação a todos
seus atos e a todas informações armazenadas nos seus
São princípios da administração pública, nesta ordem: bancos de dados. Daí a publicação em órgãos da imprensa
Legalidade e a afixação de portarias. Por exemplo, a própria expressão
Impessoalidade 1 CARVALHO FILHO, José dos Santos. Manual de direito ad-
Moralidade ministrativo. 23. ed. Rio de Janeiro: Lumen juris, 2010.
Publicidade 2 SPITZCOVSKY, Celso. Direito Administrativo. 13. ed. São
Eficiência Paulo: Método, 2011.

34
ATUALIDADES E DEVERES DOS SERVIDORES PÚBLICOS

concurso público (art. 37, II, CF) remonta ao ideário de que que alguns autores tratam veem como distintos os
todos devem tomar conhecimento do processo seletivo de princípios da moralidade e da probidade administrativa,
servidores do Estado. Diante disso, como será visto, se negar mas não há  características que permitam tratar os mesmos
indevidamente a fornecer informações ao administrado como procedimentos distintos, sendo no máximo possível
caracteriza ato de improbidade administrativa. afirmar que a probidade administrativa é um aspecto
No mais, prevê o §1º do artigo 37, CF, evitando que o particular da moralidade administrativa.
princípio da publicidade seja deturpado em propaganda b) Princípio da motivação: É a obrigação conferida ao
político-eleitoral: administrador de motivar todos os atos que edita, gerais
ou de efeitos concretos. É considerado, entre os demais
Artigo 37, §1º, CF. A publicidade dos atos, programas, princípios, um dos mais importantes, uma vez que sem a
obras, serviços e campanhas dos órgãos públicos deverá ter motivação não há o devido processo legal, uma vez que a
caráter educativo, informativo ou de orientação social, fundamentação surge como meio interpretativo da decisão
dela não podendo constar nomes, símbolos ou imagens que que levou à prática do ato impugnado, sendo verdadeiro
caracterizem promoção pessoal de autoridades ou servidores meio de viabilização do controle da legalidade dos atos da
públicos. Administração.
Motivar significa mencionar o dispositivo legal aplicável
Somente pela publicidade os indivíduos controlarão ao caso concreto e relacionar os fatos que concretamente
a legalidade e a eficiência dos atos administrativos. Os levaram à aplicação daquele dispositivo legal. Todos os
instrumentos para proteção são o direito de petição e atos administrativos devem ser motivados para que o
as certidões (art. 5°, XXXIV, CF), além do habeas data e - Judiciário possa controlar o mérito do ato administrativo
residualmente - do mandado de segurança. Neste viés, quanto à sua legalidade. Para efetuar esse controle, devem
ainda, prevê o artigo 37, CF em seu §3º:  ser observados os motivos dos atos administrativos.
Em relação à necessidade de motivação dos atos
Artigo 37, §3º, CF. A lei disciplinará as formas de administrativos vinculados (aqueles em que a lei aponta um
participação do usuário na administração pública único comportamento possível) e dos atos discricionários
direta e indireta, regulando especialmente: (aqueles que a lei, dentro dos limites nela previstos, aponta
I -  as reclamações relativas à prestação dos serviços um ou mais comportamentos possíveis, de acordo com
públicos em geral, asseguradas a manutenção de serviços um juízo de conveniência e oportunidade), a doutrina
de atendimento ao usuário e a avaliação periódica, externa é uníssona na determinação da obrigatoriedade de
e interna, da qualidade dos serviços; motivação com relação aos atos administrativos vinculados;
II -  o acesso dos usuários a registros administrativos e todavia, diverge quanto à referida necessidade quanto aos
a informações sobre atos de governo, observado o disposto atos discricionários.
no art. 5º, X e XXXIII; Meirelles4 entende que o ato discricionário, editado
III -  a disciplina da representação contra o exercício sob os limites da Lei, confere ao administrador uma
negligente ou abusivo de cargo, emprego ou função na margem de liberdade para fazer um juízo de conveniência
e oportunidade, não sendo necessária a motivação. No
administração pública.
entanto, se houver tal fundamentação, o ato deverá
condicionar-se a esta, em razão da necessidade de
e) Princípio da eficiência: A administração pública
observância da Teoria dos Motivos Determinantes. O
deve manter o ampliar a qualidade de seus serviços com
entendimento majoritário da doutrina, porém, é de que,
controle de gastos. Isso envolve eficiência ao contratar
mesmo no ato discricionário, é necessária a motivação para
pessoas (o concurso público seleciona os mais qualificados
que se saiba qual o caminho adotado pelo administrador.
ao exercício do cargo), ao manter tais pessoas em seus
Gasparini5, com respaldo no art. 50 da Lei n. 9.784/98,
cargos (pois é possível exonerar um servidor público por aponta inclusive a superação de tais discussões doutrinárias,
ineficiência) e ao controlar gastos (limitando o teto de pois o referido artigo exige a motivação para todos os atos
remuneração), por exemplo. O núcleo deste princípio é a nele elencados, compreendendo entre estes, tanto os atos
procura por produtividade e economicidade. Alcança os discricionários quanto os vinculados.
serviços públicos e os serviços administrativos internos, se
referindo diretamente à conduta dos agentes. 2) Regras mínimas sobre direitos e deveres dos
Além destes cinco princípios administrativo- servidores
constitucionais diretamente selecionados pelo constituinte, O artigo 37 da Constituição Federal estabelece os
podem ser apontados como princípios de natureza ética princípios da administração pública estudados no tópico
relacionados à função pública a probidade e a motivação: anterior, aos quais estão sujeitos servidores de quaisquer
a) Princípio da probidade:  um princípio constitucional dos Poderes em qualquer das esferas federativas, e, em
incluído dentro dos princípios específicos da licitação, seus incisos, regras mínimas sobre o serviço público:
é o dever de todo o administrador público, o dever de Paulo: Saraiva, 2004.
honestidade e fidelidade com o Estado, com a população, 4 MEIRELLES, Hely Lopes. Direito administrativo brasileiro.
no desempenho de suas funções. Possui contornos mais São Paulo: Malheiros, 1993.
definidos do que a moralidade. Diógenes Gasparini3 alerta 5 GASPARINI, Diógenes. Direito Administrativo. 9ª ed. São
3 GASPARINI, Diógenes. Direito Administrativo. 9ª ed. São Paulo: Saraiva, 2004.

35
ATUALIDADES E DEVERES DOS SERVIDORES PÚBLICOS

Artigo 37, I, CF. Os cargos, empregos e funções públicas são acessíveis aos brasileiros que preencham os requisitos
estabelecidos em lei, assim como aos estrangeiros, na forma da lei.

Aprofundando a questão, tem-se o artigo 5º da Lei nº 8.112/1990, que prevê:

Artigo 5º, Lei nº 8.112/1990. São requisitos básicos para investidura em cargo público:
I - a nacionalidade brasileira;
II - o gozo dos direitos políticos;
III - a quitação com as obrigações militares e eleitorais;
IV - o nível de escolaridade exigido para o exercício do cargo;
V - a idade mínima de dezoito anos;
VI - aptidão física e mental.
§ 1º As atribuições do cargo podem justificar a exigência de outros requisitos estabelecidos em lei. [...]
§ 3º As universidades e instituições de pesquisa científica e tecnológica federais poderão prover seus cargos com
professores, técnicos e cientistas estrangeiros, de acordo com as normas e os procedimentos desta Lei.

Destaca-se a exceção ao inciso I do artigo 5° da Lei nº 8.112/1990 e do inciso I do artigo 37, CF, prevista no artigo 207
da Constituição, permitindo que estrangeiros assumam cargos no ramo da pesquisa, ciência e tecnologia.

Artigo 37, II, CF. A investidura em cargo ou emprego público depende de aprovação prévia em concurso público de
provas ou de provas e títulos, de acordo com a natureza e a complexidade do cargo ou emprego, na forma prevista em lei,
ressalvadas as nomeações para cargo em comissão declarado em lei de livre nomeação e exoneração.

Preconiza o artigo 10 da Lei nº 8.112/1990:

Artigo 10, Lei nº 8.112/90. A nomeação para cargo de carreira ou cargo isolado de provimento efetivo depende de prévia
habilitação em concurso público de provas ou de provas e títulos, obedecidos a ordem de classificação e o prazo de sua
validade.
Parágrafo único. Os demais requisitos para o ingresso e o desenvolvimento do servidor na carreira, mediante promoção,
serão estabelecidos pela lei que fixar as diretrizes do sistema de carreira na Administração Pública Federal e seus regulamentos.

No concurso de provas o candidato é avaliado apenas pelo seu desempenho nas provas, ao passo que nos concursos
de provas e títulos o seu currículo em toda sua atividade profissional também é considerado. Cargo em comissão é o cargo
de confiança, que não exige concurso público, sendo exceção à regra geral.

Artigo 37, III, CF. O prazo de validade do concurso público será de até dois anos, prorrogável uma vez, por igual
período.

Artigo 37, IV, CF. Durante o prazo improrrogável previsto no edital de convocação, aquele aprovado em concurso público
de provas ou de provas e títulos será convocado com prioridade sobre novos concursados para assumir cargo ou emprego,
na carreira.

Prevê o artigo 12 da Lei nº 8.112/1990:

Artigo 12, Lei nº 8.112/1990. O concurso público terá validade de até 2 (dois) anos, podendo ser prorrogado uma única
vez, por igual período.
§1º O prazo de validade do concurso e as condições de sua realização serão fixados em edital, que será publicado no
Diário Oficial da União e em jornal diário de grande circulação.
§ 2º Não se abrirá novo concurso enquanto houver candidato aprovado em concurso anterior com prazo de validade
não expirado.

O edital delimita questões como valor da taxa de inscrição, casos de isenção, número de vagas e prazo de validade.
Havendo candidatos aprovados na vigência do prazo do concurso, ele deve ser chamado para assumir eventual vaga e não
ser realizado novo concurso.
Destaca-se que o §2º do artigo 37, CF, prevê:

Artigo 37, §2º, CF. A não-observância do disposto nos incisos II e III implicará a nulidade do ato e a punição da autoridade
responsável, nos termos da lei.

36
ATUALIDADES E DEVERES DOS SERVIDORES PÚBLICOS

Com efeito, há tratamento rigoroso da responsabilização daquele que viola as diretrizes mínimas sobre o ingresso no
serviço público, que em regra se dá por concurso de provas ou de provas e títulos.

Artigo 37, V, CF. As funções de confiança, exercidas exclusivamente por servidores ocupantes de cargo efetivo, e os cargos
em comissão, a serem preenchidos por servidores de carreira nos casos, condições e percentuais mínimos previstos em
lei, destinam-se apenas às atribuições de direção, chefia e assessoramento.

Observa-se o seguinte quadro comparativo6:

Função de Confiança Cargo em Comissão


Exercidas exclusivamente por servidores ocupantes de Qualquer pessoa, observado o percentual mínimo
cargo efetivo. reservado ao servidor de carreira.
Com concurso público, já que somente pode exercê-la
Sem concurso público, ressalvado o percentual
o servidor de cargo efetivo, mas a função em si não
mínimo reservado ao servidor de carreira.
prescindível de concurso público.
É atribuído posto (lugar) num dos quadros da
Somente são conferidas atribuições e responsabilida-
Administração Pública, conferida atribuições e
de
responsabilidade àquele que irá ocupá-lo
Destinam-se apenas às atribuições de direção, chefia Destinam-se apenas às atribuições de direção,
e assessoramento chefia e assessoramento
De livre nomeação e exoneração no que se refere à
De livre nomeação e exoneração
função e não em relação ao cargo efetivo.

Artigo 37, VI, CF. É garantido ao servidor público civil o direito à livre associação sindical.

A liberdade de associação é garantida aos servidores públicos tal como é garantida a todos na condição de direito
individual e de direito social.

Artigo 37, VII, CF. O direito de greve será exercido nos termos e nos limites definidos em lei específica.

O Supremo Tribunal Federal decidiu que os servidores públicos possuem o direito de greve, devendo se atentar pela
preservação da sociedade quando exercê-lo. Enquanto não for elaborada uma legislação específica para os funcionários
públicos, deverá ser obedecida a lei geral de greve para os funcionários privados, qual seja a Lei n° 7.783/89 (Mandado de
Injunção nº 20).

Artigo 37, VIII, CF. A lei reservará percentual dos cargos e empregos públicos para as pessoas portadoras de deficiência
e definirá os critérios de sua admissão.

Neste sentido, o §2º do artigo 5º da Lei nº 8.112/1990:

Artigo 5º, Lei nº 8.112/90. Às pessoas portadoras de deficiência é assegurado o direito de se inscrever em concurso público
para provimento de cargo cujas atribuições sejam compatíveis com a deficiência de que são portadoras; para tais pessoas
serão reservadas até 20% (vinte por cento) das vagas oferecidas no concurso.

Prossegue o artigo 37, CF:

Artigo 37, IX, CF. A lei estabelecerá os casos de contratação por tempo determinado para atender a necessidade temporária
de excepcional interesse público.

A Lei nº 8.745/1993 regulamenta este inciso da Constituição, definindo a natureza da relação estabelecida entre o
servidor contratado e a Administração Pública, para atender à “necessidade temporária de excepcional interesse público”.
“Em se tratando de relação subordinada, isto é, de relação que comporta dependência jurídica do servidor perante o
Estado, duas opções se ofereciam: ou a relação seria trabalhista, agindo o Estado iure gestionis, sem usar das prerrogativas
de Poder Público, ou institucional, estatutária, preponderando o ius imperii do Estado. Melhor dizendo: o sistema
preconizado pela Carta Política de 1988 é o do contrato, que tanto pode ser trabalhista (inserindo-se na esfera do Direito

6 http://direitoemquadrinhos.blogspot.com.br/2011/03/quadro-comparativo-funcao-de-confianca.html

37
ATUALIDADES E DEVERES DOS SERVIDORES PÚBLICOS

Privado) quanto administrativo (situando-se no campo do Ainda, o artigo 37 da Constituição:


Direito Público). [...] Uma solução intermediária não deixa,
entretanto, de ser legítima. Pode-se, com certeza, abonar Artigo 37, XI, CF. A remuneração e o subsídio dos
um sistema híbrido, eclético, no qual coexistam normas ocupantes de cargos, funções e empregos públicos
trabalhistas e estatutárias, pondo-se em contiguidade os da administração direta, autárquica e fundacional,
vínculos privado e administrativo, no sentido de atender dos membros de qualquer dos Poderes da União,
às exigências do Estado moderno, que procura alcançar os dos Estados, do Distrito Federal e dos Municípios, dos
seus objetivos com a mesma eficácia dos empreendimentos detentores de mandato eletivo e dos demais agentes
não-governamentais”7. políticos e os proventos, pensões ou outra espécie
remuneratória, percebidos cumulativamente ou não,
Artigo 37, X, CF. A remuneração dos servidores incluídas as vantagens pessoais ou de qualquer outra
públicos e o subsídio de que trata o § 4º do art. natureza, não poderão exceder o subsídio mensal, em
39 somente poderão ser fixados ou alterados por lei espécie, dos Ministros do Supremo Tribunal Federal,
específica, observada a iniciativa privativa em cada caso, aplicando-se como limite, nos Municípios, o subsídio do
assegurada revisão geral anual, sempre na mesma data e Prefeito, e nos Estados e no Distrito Federal, o subsídio
sem distinção de índices. mensal do Governador no âmbito do Poder Executivo, o
subsídio dos Deputados Estaduais e Distritais no âmbito
Artigo 37, XV, CF. O subsídio e os vencimentos dos do Poder Legislativo e o subsídio dos Desembargadores
ocupantes de cargos e empregos públicos são irredutíveis, do Tribunal de Justiça, limitado a noventa inteiros e
ressalvado o disposto nos incisos XI e XIV deste artigo e vinte e cinco centésimos por cento do subsídio mensal,
nos arts. 39, § 4º, 150, II, 153, III, e 153, § 2º, I. em espécie, dos Ministros do Supremo Tribunal Federal,
no âmbito do Poder Judiciário, aplicável este limite aos
Artigo 37, §10, CF. É vedada a percepção simultânea membros do Ministério Público, aos Procuradores e aos
de proventos de aposentadoria decorrentes do art. Defensores Públicos.
40 ou dos arts. 42 e 142 com a remuneração de cargo,
emprego ou função pública, ressalvados os cargos Artigo 37, XII, CF. Os vencimentos dos cargos do
acumuláveis na forma desta Constituição, os cargos Poder Legislativo e do Poder Judiciário não poderão ser
eletivos e os cargos em comissão declarados em lei de livre superiores aos pagos pelo Poder Executivo.
nomeação e exoneração.
Prevê a Lei nº 8.112/1990 em seu artigo 42:
Sobre a questão, disciplina a Lei nº 8.112/1990 nos
artigos 40 e 41: Artigo 42, Lei nº 8.112/90. Nenhum servidor poderá
perceber, mensalmente, a título de remuneração,
Art. 40. Vencimento é a retribuição pecuniária pelo importância superior à soma dos valores percebidos como
exercício de cargo público, com valor fixado em lei. remuneração, em espécie, a qualquer título, no âmbito dos
respectivos Poderes, pelos Ministros de Estado, por membros
Art. 41. Remuneração é o vencimento do cargo do Congresso Nacional e Ministros do Supremo Tribunal
efetivo, acrescido das vantagens pecuniárias permanentes Federal. Parágrafo único. Excluem-se do teto de remuneração
estabelecidas em lei. as vantagens previstas nos incisos II a VII do art. 61.
§ 1º A remuneração do servidor investido em função ou
cargo em comissão será paga na forma prevista no art. 62. Com efeito, os §§ 11 e 12 do artigo 37, CF tecem
§ 2º O servidor investido em cargo em comissão de aprofundamentos sobre o mencionado inciso XI:
órgão ou entidade diversa da de sua lotação receberá a
remuneração de acordo com o estabelecido no § 1º do art. Artigo 37, § 11, CF. Não serão computadas, para efeito
93. dos limites remuneratórios de que trata o inciso XI do
§ 3º O vencimento do cargo efetivo, acrescido das caput deste artigo, as parcelas de caráter indenizatório
vantagens de caráter permanente, é irredutível. previstas em lei.
§ 4º É assegurada a isonomia de vencimentos para
cargos de atribuições iguais ou assemelhadas do mesmo Artigo 37, § 12, CF. Para os fins do disposto no inciso
Poder, ou entre servidores dos três Poderes, ressalvadas as XI do caput deste artigo, fica facultado aos Estados e ao
vantagens de caráter individual e as relativas à natureza ou Distrito Federal fixar, em seu âmbito, mediante emenda
ao local de trabalho. às respectivas Constituições e Lei Orgânica, como limite
§ 5º Nenhum servidor receberá remuneração inferior ao único, o subsídio mensal dos Desembargadores do
salário mínimo. respectivo Tribunal de Justiça, limitado a noventa
inteiros e vinte e cinco centésimos por cento do subsídio
mensal dos Ministros do Supremo Tribunal Federal, não
7 VOGEL NETO, Gustavo Adolpho. Contratação de servido-
res para atender a necessidade temporária de excepcional inte-
se aplicando o disposto neste parágrafo aos subsídios dos
resse público. Disponível em: <http://www.planalto.gov.br/ccivil_03/ Deputados Estaduais e Distritais e dos Vereadores.
revista/Rev_39/Artigos/Art_Gustavo.htm>. Acesso em: 23 dez. 2014.

38
ATUALIDADES E DEVERES DOS SERVIDORES PÚBLICOS

Por seu turno, o artigo 37 quanto à vinculação ou Artigo 118, Lei nº 8.112/1990.  Ressalvados os casos
equiparação salarial: previstos na Constituição, é vedada a acumulação
remunerada de cargos públicos.
Artigo 37, XIII, CF. É vedada a vinculação ou § 1o  A proibição de acumular estende-se a cargos,
equiparação de quaisquer espécies remuneratórias para o empregos e funções em autarquias, fundações públicas,
efeito de remuneração de pessoal do serviço público. empresas públicas, sociedades de economia mista da
União, do Distrito Federal, dos Estados, dos Territórios e
Os padrões de vencimentos são fixados por conselho dos Municípios.
de política de administração e remuneração de pessoal, § 2o   A acumulação de cargos, ainda que lícita, fica
integrado por servidores designados pelos respectivos condicionada à comprovação da compatibilidade de
Poderes (artigo 39, caput e § 1º), sem qualquer garantia horários.
constitucional de tratamento igualitário aos cargos que § 3o  Considera-se acumulação proibida a percepção
se mostrem similares. de vencimento de cargo ou emprego público efetivo
com proventos da inatividade, salvo quando os cargos de
Artigo 37, XIV, CF. Os acréscimos pecuniários que decorram essas remunerações forem acumuláveis na
percebidos por servidor público não serão computados atividade.
nem acumulados para fins de concessão de acréscimos
ulteriores. Art. 119, Lei nº 8.112/1990.  O servidor não poderá
exercer mais de um cargo em comissão, exceto no caso
A preocupação do constituinte, ao implantar previsto no parágrafo único do art. 9o, nem ser remunerado
tal preceito, foi de que não eclodisse no sistema pela participação em órgão de deliberação coletiva. 
remuneratório dos servidores, ou seja, evitar que se Parágrafo único.  O disposto neste artigo não se aplica
utilize uma vantagem como base de cálculo de um outro à remuneração devida pela participação em conselhos de
benefício. Dessa forma, qualquer gratificação que venha administração e fiscal das empresas públicas e sociedades
a ser concedida ao servidor só pode ter como base de
de economia mista, suas subsidiárias e controladas, bem
cálculo o próprio vencimento básico. É inaceitável que
como quaisquer empresas ou entidades em que a União,
se leve em consideração outra vantagem até então
direta ou indiretamente, detenha participação no capital
percebida.
social, observado o que, a respeito, dispuser legislação
específica.
Artigo 37, XVI, CF. É vedada a acumulação
remunerada de cargos públicos, exceto, quando
Art. 120, Lei nº 8.112/1990.  O servidor vinculado ao
houver compatibilidade de horários, observado em
regime desta Lei, que acumular licitamente dois cargos
qualquer caso o disposto no inciso XI: a)  a de dois cargos
de professor; b)  a de um cargo de professor com outro, efetivos, quando investido em cargo de provimento em
técnico ou científico; c)  a de dois cargos ou empregos comissão, ficará afastado de ambos os cargos efetivos,
privativos de profissionais de saúde, com profissões salvo na hipótese em que houver compatibilidade de
regulamentadas. horário e local com o exercício de um deles, declarada pelas
autoridades máximas dos órgãos ou entidades envolvidos.
Artigo 37, XVII, CF. A proibição de acumular estende-se
a empregos e funções e abrange autarquias, fundações, “Os artigos 118 a 120 da Lei nº 8.112/90 ao tratarem da
empresas públicas, sociedades de economia mista, acumulação de cargos e funções públicas, regulamentam,
suas subsidiárias, e sociedades controladas, direta ou no âmbito do serviço público federal a vedação genérica
indiretamente, pelo poder público. constante do art. 37, incisos VXI e XVII, da Constituição da
República. De fato, a acumulação ilícita de cargos públicos
Segundo Carvalho Filho8, “o fundamento da proibição constitui uma das infrações mais comuns praticadas por
é impedir que o cúmulo de funções públicas faça com que servidores públicos, o que se constata observando o
o servidor não execute qualquer delas com a necessária elevado número de processos administrativos instaurados
eficiência. Além disso, porém, pode-se observar que o com esse objeto. O sistema adotado pela Lei nº 8.112/90
Constituinte quis também impedir a cumulação de ganhos é relativamente brando, quando cotejado com outros
em detrimento da boa execução de tarefas públicas. estatutos de alguns Estados, visto que propicia ao servidor
[...] Nota-se que a vedação se refere à acumulação incurso nessa ilicitude diversas oportunidades para
remunerada. Em consequência, se a acumulação só regularizar sua situação e escapar da pena de demissão.
encerra a percepção de vencimentos por uma das fontes, Também prevê a lei em comentário, um processo
não incide a regra constitucional proibitiva”. administrativo simplificado (processo disciplinar de rito
A Lei nº 8.112/1990 regulamenta intensamente a sumário) para a apuração dessa infração – art. 133” 9.
questão:
9 MORGATO, Almir. O Regime Disciplinar dos Servidores
8 CARVALHO FILHO, José dos Santos. Manual de direito ad- Públicos da União. Disponível em: <http://www.canaldosconcursos.
ministrativo. 23. ed. Rio de Janeiro: Lumen juris, 2010. com.br/artigos/almirmorgado_artigo1.pdf>. Acesso em: 11 ago. 2013.

39
ATUALIDADES E DEVERES DOS SERVIDORES PÚBLICOS

Artigo 37, XVIII, CF. A administração fazendária públicas. OUSAMOS DISCORDAR. Parece-nos que, se o
e seus servidores fiscais terão, dentro de suas áreas de legislador de um ente federado pretendesse, por exemplo,
competência e jurisdição, precedência sobre os demais autorizar a criação de uma subsidiária de uma fundação
setores administrativos, na forma da lei. pública, NÃO haveria base constitucional para considerar
inválida sua autorização”11.
Artigo 37, XXII, CF. As administrações tributárias Ainda sobre a questão do funcionamento da
da União, dos Estados, do Distrito Federal e dos administração indireta e de suas subsidiárias, destaca-se o
Municípios, atividades essenciais ao funcionamento do previsto nos §§ 8º e 9º do artigo 37, CF:
Estado, exercidas por servidores de carreiras específicas,
terão recursos prioritários para a realização de suas Artigo 37, §8º, CF. A autonomia gerencial, orçamentária
atividades e atuarão de forma integrada, inclusive com e financeira dos órgãos e entidades da administração direta
o compartilhamento de cadastros e de informações fiscais, e indireta poderá ser ampliada mediante contrato, a ser
na forma da lei ou convênio. firmado entre seus administradores e o poder público, que
tenha por objeto a fixação de metas de desempenho para o
“O Estado tem como finalidade essencial a garantia órgão ou entidade, cabendo à lei dispor sobre:
do bem-estar de seus cidadãos, seja através dos serviços I -  o prazo de duração do contrato;
públicos que disponibiliza, seja através de investimentos II -  os controles e critérios de avaliação de desempenho,
na área social (educação, saúde, segurança pública). Para direitos, obrigações e responsabilidade dos dirigentes;
atingir esses objetivos primários, deve desenvolver uma III -  a remuneração do pessoal.
atividade financeira, com o intuito de obter recursos
indispensáveis às necessidades cuja satisfação se Artigo 37, § 9º, CF. O disposto no inciso XI aplica-se às
comprometeu quando estabeleceu o “pacto” constitucional empresas públicas e às sociedades de economia mista
de 1988. [...] A importância da Administração Tributária e suas subsidiárias, que receberem recursos da União,
foi reconhecida expressamente pelo constituinte que dos Estados, do Distrito Federal ou dos Municípios para
acrescentou, no artigo 37 da Carta Magna, o inciso XVIII, pagamento de despesas de pessoal ou de custeio em geral.
estabelecendo a sua precedência e de seus servidores
sobre os demais setores da Administração Pública, dentro Continua o artigo 37, CF:
de suas áreas de competência”10.
Artigo 37, XXI, CF. Ressalvados os casos especificados na
Artigo 37, XIX, CF. Somente por lei específica poderá legislação, as obras, serviços, compras e alienações serão
ser criada autarquia e autorizada a instituição de contratados mediante processo de licitação pública que
empresa pública, de sociedade de economia mista e de assegure igualdade de condições a todos os concorrentes,
fundação, cabendo à lei complementar, neste último caso, com cláusulas que estabeleçam obrigações de pagamento,
definir as áreas de sua atuação. mantidas as condições efetivas da proposta, nos termos da
lei, o qual somente permitirá as exigências de qualificação
Artigo 37, XX, CF. Depende de autorização legislativa, técnica e econômica indispensáveis à garantia do
em cada caso, a criação de subsidiárias das entidades cumprimento das obrigações.
mencionadas no inciso anterior, assim como a participação
de qualquer delas em empresa privada. A Lei nº 8.666, de 21 de junho de 1993, regulamenta o
art. 37, inciso XXI, da Constituição Federal, institui normas
Órgãos da administração indireta somente podem para licitações e contratos da Administração Pública e dá
ser criados por lei específica e a criação de subsidiárias outras providências. Licitação nada mais é que o conjunto
destes dependem de autorização legislativa (o Estado de procedimentos administrativos (administrativos porque
cria e controla diretamente determinada empresa parte da administração pública) para as compras ou
pública ou sociedade de economia mista, e estas, por serviços contratados pelos governos Federal, Estadual ou
sua vez, passam a gerir uma nova empresa, denominada Municipal, ou seja todos os entes federativos. De forma
subsidiária. Ex.: Transpetro, subsidiária da Petrobrás). mais simples, podemos dizer que o governo deve comprar
“Abrimos um parêntese para observar que quase todos os e contratar serviços seguindo regras de lei, assim a licitação
autores que abordam o assunto afirmam categoricamente é um processo formal onde há a competição entre os
que, a despeito da referência no texto constitucional interessados.
a ‘subsidiárias das entidades mencionadas no inciso
anterior’, somente empresas públicas e sociedades de Artigo 37, §5º, CF. A lei estabelecerá os prazos de
economia mista podem ter subsidiárias, pois a relação prescrição para ilícitos praticados por qualquer agente,
de controle que existe entre a pessoa jurídica matriz servidor ou não, que causem prejuízos ao erário, ressalvadas
e a subsidiária seria própria de pessoas com estrutura as respectivas ações de ressarcimento.
empresarial, e inadequada a autarquias e fundações
10 http://www.sindsefaz.org.br/parecer_administracao_tribu- 11 ALEXANDRINO, Marcelo. Direito Administrativo Descom-
taria_sao_paulo.htm plicado. São Paulo: GEN, 2014.

40
ATUALIDADES E DEVERES DOS SERVIDORES PÚBLICOS

A prescrição dos ilícitos praticados por servidor encontra 3) Responsabilidade civil do Estado e de seus
disciplina específica no artigo 142 da Lei nº 8.112/1990: servidores
O instituto da responsabilidade civil é parte integrante
Art. 142, Lei nº 8.112/1990.  A ação disciplinar do direito obrigacional, uma vez que a principal
prescreverá: consequência da prática de um ato ilícito é a obrigação
I - em 5 (cinco) anos, quanto às infrações puníveis com que gera para o seu auto de reparar o dano, mediante
demissão, cassação de aposentadoria ou disponibilidade e o pagamento de indenização que se refere às perdas e
destituição de cargo em comissão; danos. Afinal, quem pratica um ato ou incorre em omissão
II - em 2 (dois) anos, quanto à suspensão; que gere dano deve suportar as consequências jurídicas
III - em 180 (cento e oitenta) dias, quanto á advertência. decorrentes, restaurando-se o equilíbrio social.12
§ 1o  O prazo de prescrição começa a correr da data em A responsabilidade civil, assim, difere-se da penal,
que o fato se tornou conhecido. podendo recair sobre os herdeiros do autor do ilícito até
§ 2o  Os prazos de prescrição previstos na lei penal os limites da herança, embora existam reflexos na ação
aplicam-se às infrações disciplinares capituladas também que apure a responsabilidade civil conforme o resultado na
como crime. esfera penal (por exemplo, uma absolvição por negativa de
§ 3o  A abertura de sindicância ou a instauração de autoria impede a condenação na esfera cível, ao passo que
processo disciplinar interrompe a prescrição, até a decisão uma absolvição por falta de provas não o faz).
final proferida por autoridade competente. A responsabilidade civil do Estado acompanha o
§ 4o  Interrompido o curso da prescrição, o prazo raciocínio de que a principal consequência da prática de
começará a correr a partir do dia em que cessar a um ato ilícito é a obrigação que gera para o seu auto de
interrupção. reparar o dano, mediante o pagamento de indenização que
se refere às perdas e danos. Todos os cidadãos se sujeitam
Prescrição é um instituto que visa regular a perda do às regras da responsabilidade civil, tanto podendo buscar
direito de acionar judicialmente. No caso, o prazo é de 5 o ressarcimento do dano que sofreu quanto respondendo
anos para as infrações mais graves, 2 para as de gravidade por aqueles danos que causar. Da mesma forma, o Estado
intermediária (pena de suspensão) e 180 dias para as
tem o dever de indenizar os membros da sociedade pelos
menos graves (pena de advertência), contados da data
danos que seus agentes causem durante a prestação do
em que o fato se tornou conhecido pela administração
serviço, inclusive se tais danos caracterizarem uma violação
pública. Se a infração disciplinar for crime, valerão os
aos direitos humanos reconhecidos.
prazos prescricionais do direito penal, mais longos, logo,
Trata-se de responsabilidade extracontratual porque
menos favoráveis ao servidor. Interrupção da prescrição
não depende de ajuste prévio, basta a caracterização de
significa parar a contagem do prazo para que, retornando,
elementos genéricos pré-determinados, que perpassam
comece do zero. Da abertura da sindicância ou processo
administrativo disciplinar até a decisão final proferida por pela leitura concomitante do Código Civil (artigos 186, 187
autoridade competente não corre a prescrição. Proferida a e 927) com a Constituição Federal (artigo 37, §6°).
decisão, o prazo começa a contar do zero. Passado o prazo, Genericamente, os elementos da responsabilidade civil
não caberá mais propor ação disciplinar. se encontram no art. 186 do Código Civil:

Artigo 37, §7º, CF. A lei disporá sobre os requisitos e Artigo 186, CC. Aquele que, por ação ou omissão
as restrições ao ocupante de cargo ou emprego da voluntária, negligência ou imprudência, violar direito e
administração direta e indireta que possibilite o acesso a causar dano a outrem, ainda que exclusivamente moral,
informações privilegiadas. comete ato ilícito.

A Lei nº 12.813, de 16 de maio de 2013 dispõe sobre o Este é o artigo central do instituto da responsabilidade
conflito de interesses no exercício de cargo ou emprego civil, que tem como elementos: ação ou omissão voluntária
do Poder Executivo federal e impedimentos posteriores (agir como não se deve ou deixar de agir como se deve),
ao exercício do cargo ou emprego; e revoga dispositivos culpa ou dolo do agente (dolo é a vontade de cometer uma
da Lei nº 9.986, de 18 de julho de 2000, e das Medidas violação de direito e culpa é a falta de diligência), nexo
Provisórias nºs 2.216-37, de 31 de agosto de 2001, e 2.225- causal (relação de causa e efeito entre a ação/omissão e
45, de 4 de setembro de 2001. o dano causado) e dano (dano é o prejuízo sofrido pelo
Neste sentido, conforme seu artigo 1º: agente, que pode ser individual ou coletivo, moral ou
material, econômico e não econômico).
Artigo 1º, Lei nº 12.813/2013. As situações que 1) Dano - somente é indenizável o dano certo, especial
configuram conflito de interesses envolvendo ocupantes e anormal. Certo é o dano real, existente. Especial é o dano
de cargo ou emprego no âmbito do Poder Executivo específico, individualizado, que atinge determinada ou
federal, os requisitos e restrições a ocupantes de cargo ou determinadas pessoas. Anormal é o dano que ultrapassa
emprego que tenham acesso a informações privilegiadas, os os problemas comuns da vida em sociedade (por exemplo,
impedimentos posteriores ao exercício do cargo ou emprego infelizmente os assaltos são comuns e o Estado não
e as competências para fiscalização, avaliação e prevenção 12 GONÇALVES, Carlos Roberto. Responsabilidade Civil. 9.
de conflitos de interesses regulam-se pelo disposto nesta Lei. ed. São Paulo: Saraiva, 2005.

41
ATUALIDADES E DEVERES DOS SERVIDORES PÚBLICOS

responde por todo assalto que ocorra, a não ser que na A responsabilidade civil do servidor exige prévio
circunstância específica possuía o dever de impedir o processo administrativo disciplinar no qual seja
assalto, como no caso de uma viatura presente no local - assegurado contraditório e ampla defesa. Trata-se de
muito embora o direito à segurança pessoal seja um direito responsabilidade civil subjetiva ou com culpa. Havendo
humano reconhecido). ação ou omissão com culpa do servidor que gere dano
2) Agentes públicos - é toda pessoa que trabalhe ao erário (Administração) ou a terceiro (administrado), o
dentro da administração pública, tenha ingressado ou servidor terá o dever de indenizar.
não por concurso, possua cargo, emprego ou função. Não obstante, agentes públicos que pratiquem
Envolve os agentes políticos, os servidores públicos em atos violadores de direitos humanos se sujeitam
geral (funcionários, empregados ou temporários) e os à responsabilidade penal e à responsabilidade
particulares em colaboração (por exemplo, jurado ou administrativa, todas autônomas uma com relação à outra
mesário). e à já mencionada responsabilidade civil.
3) Dano causado quando o agente estava agindo nesta No caso da responsabilidade civil, o Estado é diretamente
qualidade - é preciso que o agente esteja lançando mão das acionado e responde pelos atos de seus servidores que
prerrogativas do cargo, não agindo como um particular. violem direitos humanos, cabendo eventualmente ação de
Sem estes três requisitos, não será possível acionar o regresso contra ele. Contudo, nos casos da responsabilidade
Estado para responsabilizá-lo civilmente pelo dano, por penal e da responsabilidade administrativa aciona-se o
mais relevante que tenha sido a esfera de direitos atingida. agente público que praticou o ato.
Assim, não é qualquer dano que permite a responsabilização São inúmeros os exemplos de crimes que podem ser
civil do Estado, mas somente aquele que é causado por um praticados pelo agente público no exercício de sua função
agente público no exercício de suas funções e que exceda que violam direitos humanos. A título de exemplo, peculato,
as expectativas do lesado quanto à atuação do Estado. consistente em apropriação ou desvio de dinheiro público
É preciso lembrar que não é o Estado em si que viola os (art. 312, CP), que viola o bem comum e o interesse da
direitos humanos, porque o Estado é uma ficção formada coletividade; concussão, que é a exigência de vantagem
por um grupo de pessoas que desempenham as atividades indevida (art. 316, CP), expondo a vítima a uma situação
estatais diversas. Assim, viola direitos humanos não o Estado de constrangimento e medo que viola diretamente sua
em si, mas o agente que o representa, fazendo com que o dignidade; tortura, a mais cruel forma de tratamento
próprio Estado seja responsabilizado por isso civilmente, humano, cuja pena é agravada quando praticada por
pagando pela indenização (reparação dos danos materiais funcionário público (art. 1º, §4º, I, Lei nº 9.455/97); etc.
e morais). Sem prejuízo, com relação a eles, caberá ação de Quanto à responsabilidade administrativa, menciona-
regresso se agiram com dolo ou culpa. se, a título de exemplo, as penalidades cabíveis descritas
Prevê o artigo 37, §6° da Constituição Federal: no art. 127 da Lei nº 8.112/90, que serão aplicadas pelo
funcionário que violar a ética do serviço público, como
Artigo 37, §6º, CF. As pessoas jurídicas de direito público advertência, suspensão e demissão.
e as de direito privado prestadoras de serviços públicos Evidencia-se a independência entre as esferas civil,
responderão pelos danos que seus agentes, nessa penal e administrativa no que tange à responsabilização
qualidade, causarem a terceiros, assegurado o direito do agente público que cometa ato ilícito.
de regresso contra o responsável nos casos de dolo ou Tomadas as exigências de características dos danos
culpa. acima colacionadas, notadamente a anormalidade,
considera-se que para o Estado ser responsabilizado por
Este artigo deixa clara a formação de uma relação um dano, ele deve exceder expectativas cotidianas, isto é,
jurídica autônoma entre o Estado e o agente público que não cabe exigir do Estado uma excepcional vigilância da
causou o dano no desempenho de suas funções. Nesta sociedade e a plena cobertura de todas as fatalidades que
relação, a responsabilidade civil será subjetiva, ou seja, possam acontecer em território nacional.
caberá ao Estado provar a culpa do agente pelo dano Diante de tal premissa, entende-se que a
causado, ao qual foi anteriormente condenado a reparar. responsabilidade civil do Estado será objetiva apenas
Direito de regresso é justamente o direito de acionar o no caso de ações, mas subjetiva no caso de omissões.
causador direto do dano para obter de volta aquilo que Em outras palavras, verifica-se se o Estado se omitiu
pagou à vítima, considerada a existência de uma relação tendo plenas condições de não ter se omitido, isto é, ter
obrigacional que se forma entre a vítima e a instituição que deixado de agir quando tinha plenas condições de fazê-lo,
o agente compõe. acarretando em prejuízo dentro de sua previsibilidade.
Assim, o Estado responde pelos danos que seu agente São casos nos quais se reconheceu a responsabilidade
causar aos membros da sociedade, mas se este agente omissiva do Estado: morte de filho menor em creche
agiu com dolo ou culpa deverá ressarcir o Estado do que municipal, buracos não sinalizados na via pública, tentativa
foi pago à vítima. O agente causará danos ao praticar de assalto a usuário do metrô resultando em morte, danos
condutas incompatíveis com o comportamento ético dele provocados por enchentes e escoamento de águas pluviais
esperado.13 quando o Estado sabia da problemática e não tomou
13 SPITZCOVSKY, Celso. Direito Administrativo. 13. ed. São providência para evitá-las, morte de detento em prisão,
Paulo: Método, 2011. incêndio em casa de shows fiscalizada com negligência, etc.

42
ATUALIDADES E DEVERES DOS SERVIDORES PÚBLICOS

Logo, não é sempre que o Estado será responsabilizado. § 2º A União, os Estados e o Distrito Federal manterão
Há excludentes da responsabilidade estatal, escolas de governo para a formação e o aperfeiçoamento
notadamente: a) caso fortuito (fato de terceiro) ou força dos servidores públicos, constituindo-se a participação
maior (fato da natureza) fora dos alcances da previsibilidade nos cursos um dos requisitos para a promoção na carreira,
do dano; b) culpa exclusiva da vítima. facultada, para isso, a celebração de convênios ou contratos
entre os entes federados.
4) Exercício de mandato eletivo por servidores § 3º Aplica-se aos servidores ocupantes de cargo
públicos público o disposto no art. 7º, IV, VII, VIII, IX, XII, XIII,
A questão do exercício de mandato eletivo pelo XV,XVI, XVII, XVIII, XIX, XX, XXII e XXX, podendo a
servidor público encontra previsão constitucional em seu lei estabelecer requisitos diferenciados de admissão
artigo 38, que notadamente estabelece quais tipos de quando a natureza do cargo o exigir.
mandatos geram incompatibilidade ao serviço público e § 4º O membro de Poder, o detentor de mandato
regulamenta a questão remuneratória: eletivo, os Ministros de Estado e os Secretários Estaduais
e Municipais serão remunerados exclusivamente por
Artigo 38, CF.  Ao servidor público da administração subsídio fixado em parcela única, vedado o acréscimo de
direta, autárquica e fundacional, no exercício de mandato qualquer gratificação, adicional, abono, prêmio, verba de
eletivo, aplicam-se as seguintes disposições: representação ou outra espécie remuneratória, obedecido,
I - tratando-se de mandato eletivo federal, estadual ou em qualquer caso, o disposto no art. 37, X e XI.
distrital, ficará afastado de seu cargo, emprego ou função; § 5º Lei da União, dos Estados, do Distrito Federal e dos
II - investido no mandato de Prefeito, será afastado do Municípios poderá estabelecer a relação entre a maior e
cargo, emprego ou função, sendo-lhe facultado optar pela a menor remuneração dos servidores públicos, obedecido,
sua remuneração; em qualquer caso, o disposto no art. 37, XI.
III - investido no mandato de Vereador, havendo § 6º Os Poderes Executivo, Legislativo e Judiciário
compatibilidade de horários, perceberá as vantagens de seu publicarão anualmente os valores do subsídio e da
cargo, emprego ou função, sem prejuízo da remuneração remuneração dos cargos e empregos públicos.
do cargo eletivo, e, não havendo compatibilidade, será § 7º Lei da União, dos Estados, do Distrito Federal
aplicada a norma do inciso anterior; e dos Municípios disciplinará a aplicação de recursos
IV - em qualquer caso que exija o afastamento para orçamentários provenientes da economia com despesas
o exercício de mandato eletivo, seu tempo de serviço correntes em cada órgão, autarquia e fundação, para
será contado para todos os efeitos legais, exceto para aplicação no desenvolvimento de programas de qualidade
promoção por merecimento; e produtividade, treinamento e desenvolvimento,
V - para efeito de benefício previdenciário, no caso modernização, reaparelhamento e racionalização do
de afastamento, os valores serão determinados como se no serviço público, inclusive sob a forma de adicional ou
exercício estivesse. prêmio de produtividade.
§ 8º A remuneração dos servidores públicos organizados
5) Regime de remuneração e previdência dos em carreira poderá ser fixada nos termos do § 4º.
servidores públicos
Regulamenta-se o regime de remuneração e Artigo 40, CF.  Aos servidores titulares de cargos efetivos
previdência dos servidores públicos nos artigo 39 e 40 da da União, dos Estados, do Distrito Federal e dos Municípios,
Constituição Federal: incluídas suas autarquias e fundações, é assegurado
regime de previdência de caráter contributivo e solidário,
Artigo 39, CF. A União, os Estados, o Distrito Federal mediante contribuição do respectivo ente público, dos
e os Municípios instituirão conselho de política de servidores ativos e inativos e dos pensionistas, observados
administração e remuneração de pessoal, integrado critérios que preservem o equilíbrio financeiro e atuarial e
por servidores designados pelos respectivos Poderes. o disposto neste artigo.
(Redação dada pela Emenda Constitucional nº 19, de 1998 § 1º Os servidores abrangidos pelo regime de
e aplicação suspensa pela ADIN nº 2.135-4, destacando-se previdência de que trata este artigo serão aposentados,
a redação anterior: “A União, os Estados, o Distrito Federal calculados os seus proventos a partir dos valores fixados
e os Municípios instituirão, no âmbito de sua competência, na forma dos §§ 3º e 17:
regime jurídico único e planos de carreira para os servidores I - por invalidez permanente, sendo os proventos
da administração pública direta, das autarquias e das proporcionais ao tempo de contribuição, exceto se
fundações públicas”). decorrente de acidente em serviço, moléstia profissional
§ 1º A fixação dos padrões de vencimento e dos demais ou doença grave, contagiosa ou incurável, na forma da lei;
componentes do sistema remuneratório observará: II - compulsoriamente, com proventos proporcionais ao
I -  a natureza, o grau de responsabilidade e a tempo de contribuição, aos 70 (setenta) anos de idade, ou
complexidade dos cargos componentes de cada carreira; aos 75 (setenta e cinco) anos de idade, na forma de lei
II -  os requisitos para a investidura; complementar;
III -  as peculiaridades dos cargos.

43
ATUALIDADES E DEVERES DOS SERVIDORES PÚBLICOS

III - voluntariamente, desde que cumprido tempo § 10. A lei não poderá estabelecer qualquer forma de
mínimo de dez anos de efetivo exercício no serviço contagem de tempo de contribuição fictício.
público e cinco anos no cargo efetivo em que se dará a § 11. Aplica-se o limite fixado no art. 37, XI, à soma
aposentadoria, observadas as seguintes condições: total dos proventos de inatividade, inclusive quando
a)   sessenta anos de idade e trinta e cinco de decorrentes da acumulação de cargos ou empregos
contribuição, se homem, e cinquenta e cinco anos de idade públicos, bem como de outras atividades sujeitas a
e trinta de contribuição, se mulher; contribuição para o regime geral de previdência social, e ao
b)    sessenta e cinco anos de idade, se homem, e montante resultante da adição de proventos de inatividade
sessenta anos de idade, se mulher, com proventos com remuneração de cargo acumulável na forma desta
proporcionais ao tempo de contribuição. Constituição, cargo em comissão declarado em lei de livre
§ 2º Os proventos de aposentadoria e as pensões, nomeação e exoneração, e de cargo eletivo.
por ocasião de sua concessão, não poderão exceder a § 12. Além do disposto neste artigo, o regime de
remuneração do respectivo servidor, no cargo efetivo em previdência dos servidores públicos titulares de cargo
que se deu a aposentadoria ou que serviu de referência efetivo observará, no que couber, os requisitos e critérios
para a concessão da pensão. fixados para o regime geral de previdência social.
§ 3º Para o cálculo dos proventos de aposentadoria, § 13. Ao servidor ocupante, exclusivamente, de cargo
por ocasião da sua concessão, serão consideradas as em comissão declarado em lei de livre nomeação e
remunerações utilizadas como base para as contribuições exoneração bem como de outro cargo temporário ou de
do servidor aos regimes de previdência de que tratam este emprego público, aplica-se o regime geral de previdência
artigo e o art. 201, na forma da lei. social.
§ 4º É vedada a adoção de requisitos e critérios § 14. A União, os Estados, o Distrito Federal e os
diferenciados para a concessão de aposentadoria aos Municípios, desde que instituam regime de previdência
abrangidos pelo regime de que trata este artigo, ressalvados, complementar para os seus respectivos servidores
nos termos definidos em leis complementares, os casos de titulares de cargo efetivo, poderão fixar, para o valor das
servidores: aposentadorias e pensões a serem concedidas pelo regime
I -  portadores de deficiência;
de que trata este artigo, o limite máximo estabelecido
II -  que exerçam atividades de risco;
para os benefícios do regime geral de previdência
III -  cujas atividades sejam exercidas sob condições
social de que trata o art. 201.
especiais que prejudiquem a saúde ou a integridade
§ 15. O regime de previdência complementar de
física.
que trata o § 14 será instituído por lei de iniciativa do
§ 5º Os requisitos de idade e de tempo de contribuição
respectivo Poder Executivo, observado o disposto no art.
serão reduzidos em cinco anos, em relação ao disposto no §
202 e seus parágrafos, no que couber, por intermédio
1º, III, a, para o professor que comprove exclusivamente
de entidades fechadas de previdência complementar,
tempo de efetivo exercício das funções de magistério na
educação infantil e no ensino fundamental e médio. de natureza pública, que oferecerão aos respectivos
§ 6º Ressalvadas as aposentadorias decorrentes dos participantes planos de benefícios somente na modalidade
cargos acumuláveis na forma desta Constituição, é vedada de contribuição definida.
a percepção de mais de uma aposentadoria à conta do § 16. Somente mediante sua prévia e expressa opção,
regime de previdência previsto neste artigo. o disposto nos §§ 14 e 15 poderá ser aplicado ao servidor
§ 7º Lei disporá sobre a concessão do benefício de que tiver ingressado no serviço público até a data da
pensão por morte, que será igual: publicação do ato de instituição do correspondente
I - ao valor da totalidade dos proventos do servidor regime de previdência complementar.
falecido, até o limite máximo estabelecido para os benefícios § 17. Todos os valores de remuneração considerados
do regime geral de previdência social de que trata o art. 201, para o cálculo do benefício previsto no § 3° serão
acrescido de setenta por cento da parcela excedente a este devidamente atualizados, na forma da lei.
limite, caso aposentado à data do óbito; ou § 18. Incidirá contribuição sobre os proventos de
II - ao valor da totalidade da remuneração do servidor aposentadorias e pensões concedidas pelo regime de
no cargo efetivo em que se deu o falecimento, até o limite que trata este artigo que superem o limite máximo
máximo estabelecido para os benefícios do regime geral estabelecido para os benefícios do regime geral de
de previdência social de que trata o art. 201, acrescido de previdência social de que trata o art. 201, com percentual
setenta por cento da parcela excedente a este limite, caso em igual ao estabelecido para os servidores titulares de cargos
atividade na data do óbito. efetivos.
§ 8º É assegurado o reajustamento dos benefícios § 19. O servidor de que trata este artigo que tenha
para preservar-lhes, em caráter permanente, o valor real, completado as exigências para aposentadoria voluntária
conforme critérios estabelecidos em lei. estabelecidas no § 1º, III, a, e que opte por permanecer em
§ 9º O tempo de contribuição federal, estadual ou atividade fará jus a um abono de permanência equivalente
municipal será contado para efeito de aposentadoria ao valor da sua contribuição previdenciária até completar
e o tempo de serviço correspondente para efeito de as exigências para aposentadoria compulsória contidas no
disponibilidade. § 1º, II.

44
ATUALIDADES E DEVERES DOS SERVIDORES PÚBLICOS

§ 20. Fica vedada a existência de mais de um regime Uma vez adquirida a aprovação no estágio probatório,
próprio de previdência social para os servidores o servidor público somente poderá ser exonerado
titulares de cargos efetivos, e de mais de uma unidade nos casos do §1º do artigo 40 da Constituição Federal,
gestora do respectivo regime em cada ente estatal, notadamente: em virtude de sentença judicial transitada
ressalvado o disposto no art. 142, § 3º, X. em julgado; mediante processo administrativo em
§ 21. A contribuição prevista no § 18 deste artigo que lhe seja assegurada ampla defesa; ou mediante
incidirá apenas sobre as parcelas de proventos de procedimento de avaliação periódica de desempenho,
aposentadoria e de pensão que superem o dobro do na forma de lei complementar, assegurada ampla defesa
limite máximo estabelecido para os benefícios do regime (sendo esta lei complementar ainda inexistente no âmbito
geral de previdência social de que trata o art. 201 desta federal.
Constituição, quando o beneficiário, na forma da lei, for
portador de doença incapacitante. 7) Atos de improbidade administrativa
A Lei n° 8.429/1992 trata da improbidade administrativa,
6) Estágio probatório e perda do cargo que é uma espécie qualificada de imoralidade, sinônimo
Estabelece a Constituição Federal em seu artigo 41: de desonestidade administrativa. A improbidade é uma
lesão ao princípio da moralidade, que deve ser respeitado
Artigo 41, CF.  São estáveis após três anos de efetivo estritamente pelo servidor público. O agente ímprobo
exercício os servidores nomeados para cargo de sempre será um violador do princípio da moralidade,
provimento efetivo em virtude de concurso público. pelo qual “a Administração Pública deve agir com boa-fé,
§ 1º O servidor público estável só perderá o cargo: sinceridade, probidade, lhaneza, lealdade e ética”14.
I -  em virtude de sentença judicial transitada em A atual Lei de Improbidade Administrativa foi criada
julgado;
devido ao amplo apelo popular contra certas vicissitudes
II -  mediante processo administrativo em que lhe
do serviço público que se intensificavam com a ineficácia
seja assegurada ampla defesa;
do diploma então vigente, o Decreto-Lei nº 3240/41.
III -  mediante procedimento de avaliação periódica
Decorreu, assim, da necessidade de acabar com os atos
de desempenho, na forma de lei complementar, assegurada
atentatórios à moralidade administrativa e causadores
ampla defesa.
de prejuízo ao erário público ou ensejadores de
§ 2º Invalidada por sentença judicial a demissão
enriquecimento ilícito, infelizmente tão comuns no Brasil.
do servidor estável, será ele reintegrado, e o eventual
Com o advento da Lei nº 8.429/1992, os agentes
ocupante da vaga, se estável, reconduzido ao cargo de
públicos passaram a ser responsabilizados na esfera civil
origem, sem direito a indenização, aproveitado em outro
cargo ou posto em disponibilidade com remuneração pelos atos de improbidade administrativa descritos nos
proporcional ao tempo de serviço. artigos 9º, 10 e 11, ficando sujeitos às penas do art. 12.
§ 3º Extinto o cargo ou declarada a sua desnecessidade, A existência de esferas distintas de responsabilidade
o servidor estável ficará em disponibilidade, com (civil, penal e administrativa) impede falar-se em bis in
remuneração proporcional ao tempo de serviço, até seu idem, já que, ontologicamente, não se trata de punições
adequado aproveitamento em outro cargo. idênticas, embora baseadas no mesmo fato, mas de
§ 4º Como condição para a aquisição da estabilidade, responsabilização em esferas distintas do Direito.
é obrigatória a avaliação especial de desempenho por Destaca-se um conceito mais amplo de agente público
comissão instituída para essa finalidade. previsto pela lei nº 8.429/1992 em seus artigos 1º e 2º
porque o agente público pode ser ou não um servidor
O estágio probatório pode ser definido como um lapso público. Ele poderá estar vinculado a qualquer instituição
de tempo no qual a aptidão e capacidade do servidor ou órgão que desempenhe diretamente o interesse do
serão avaliadas de acordo com critérios de assiduidade, Estado. Assim, estão incluídos todos os integrantes da
disciplina, capacidade de iniciativa, produtividade e administração direta, indireta e fundacional, conforme
responsabilidade. O servidor não aprovado no estágio o preâmbulo da legislação. Pode até mesmo ser uma
probatório será exonerado ou, se estável, reconduzido ao entidade privada que desempenhe tais fins, desde que a
cargo anteriormente ocupado. Não existe vedação para verba de criação ou custeio tenha sido ou seja pública
um servidor em estágio probatório exercer quaisquer em mais de 50% do patrimônio ou receita anual. Caso a
cargos de provimento em comissão ou funções de verba pública que tenha auxiliado uma entidade privada
direção, chefia ou assessoramento no órgão ou entidade a qual o Estado não tenha concorrido para criação ou
de lotação. custeio, também haverá sujeição às penalidades da lei.
Desde a Emenda Constitucional nº 19 de 1998, a Em caso de custeio/criação pelo Estado que seja inferior
disciplina do estágio probatório mudou, notadamente a 50% do patrimônio ou receita anual, a legislação
aumentando o prazo de 2 anos para 3 anos. Tendo em ainda se aplica. Entretanto, nestes dois casos, a sanção
vista que a norma constitucional prevalece sobre a lei patrimonial se limitará ao que o ilícito repercutiu
federal, mesmo que ela não tenha sido atualizada, deve- 14 LENZA, Pedro. Curso de direito constitucional esquema-
se seguir o disposto no artigo 41 da Constituição Federal. tizado. 15. ed. São Paulo: Saraiva, 2011.

45
ATUALIDADES E DEVERES DOS SERVIDORES PÚBLICOS

sobre a contribuição dos cofres públicos. Significa que do artigo 10, nas quais o dano ao erário precisa ser
se o prejuízo causado for maior que a efetiva contribuição comprovado. De acordo com o ministro Castro Meira, a
por parte do poder público, o ressarcimento terá que ser conduta culposa ocorre quando o agente não pretende
buscado por outra via que não a ação de improbidade atingir o resultado danoso, mas atua com negligência,
administrativa. imprudência ou imperícia (REsp n° 1.127.143)”18.
A legislação em estudo, por sua vez, divide os atos de Para Carvalho Filho19, não há inconstitucionalidade na
improbidade administrativa em três categorias: modalidade culposa, lembrando que é possível dosar a
a) Ato de improbidade administrativa que importe pena conforme o agente aja com dolo ou culpa.
enriquecimento ilícito (artigo 9º, Lei nº 8.429/1992) O ponto central é lembrar que neste artigo não se exige
O grupo mais grave de atos de improbidade administrativa que o sujeito ativo tenha percebido vantagens indevidas,
se caracteriza pelos elementos: enriquecimento + ilícito + basta o dano ao erário. Se tiver recebido vantagem
resultante de uma vantagem patrimonial indevida + em indevida, incide no artigo anterior. Exceto pela não
razão do exercício de cargo, mandato, emprego, função percepção da vantagem indevida, os tipos exemplificados
ou outra atividade nas entidades do artigo 1° da Lei nº se aproximam muito dos previstos nos incisos do art. 9°.
8.429/1992. c) Ato de improbidade administrativa que atente
O enriquecimento deve ser ilícito, afinal, o Estado não contra os princípios da administração pública (artigo
se opõe que o indivíduo enriqueça, desde que obedeça aos 11, Lei nº 8.429/1992)
ditames morais, notadamente no desempenho de função Nos termos do artigo 11 da Lei nº 8.429/1992,
de interesse estatal. “constitui ato de improbidade administrativa que atenta
Exige-se que o sujeito obtenha vantagem patrimonial contra os princípios da administração pública qualquer
ilícita. Contudo, é dispensável que efetivamente tenha ação ou omissão que viole os deveres de honestidade,
ocorrido dano aos cofres públicos (por exemplo, quando imparcialidade, legalidade, e lealdade às instituições [...]”. O
um policial recebe propina pratica ato de improbidade grupo mais ameno de atos de improbidade administrativa
administrativa, mas não atinge diretamente os cofres se caracteriza pela simples violação a princípios da
públicos). administração pública, ou seja, aplica-se a qualquer
Como fica difícil imaginar que alguém possa se atitude do sujeito ativo que viole os ditames éticos do
enriquecer ilicitamente por negligência, imprudência ou serviço público. Isto é, o legislador pretende a preservação
imperícia, todas as condutas configuram atos dolosos (com dos princípios gerais da administração pública20.
intenção). Não cabe prática por omissão.15 O objeto de tutela são os princípios constitucionais.
b) Ato de improbidade administrativa que importe Basta a vulneração em si dos princípios, sendo dispensáveis
lesão ao erário (artigo 10, Lei nº 8.429/1992) o enriquecimento ilícito e o dano ao erário. Somente é
O grupo intermediário de atos de improbidade possível a prática de algum destes atos com dolo (intenção),
administrativa se caracteriza pelos elementos: causar embora caiba a prática por ação ou omissão.
dano ao erário ou aos cofres públicos + gerando perda Será preciso utilizar razoabilidade e proporcionalidade
patrimonial ou dilapidação do patrimônio público. Assim para não permitir a caracterização de abuso de poder,
como o artigo anterior, o caput descreve a fórmula genérica diante do conteúdo aberto do dispositivo. Na verdade,
e os incisos algumas atitudes específicas que exemplificam trata-se de tipo subsidiário, ou seja, que se aplica quando
o seu conteúdo16. o ato de improbidade administrativa não tiver gerado
Perda patrimonial é o gênero, do qual são espécies: obtenção de vantagem
desvio, que é o direcionamento indevido; apropriação, Com efeito, os atos de improbidade administrativa não
que é a transferência indevida para a própria propriedade; são crimes de responsabilidade. Trata-se de punição na
malbaratamento, que significa desperdício; e dilapidação, esfera cível, não criminal. Por isso, caso o ato configure
que se refere a destruição17. simultaneamente um ato de improbidade administrativa
O objeto da tutela é a preservação do patrimônio desta lei e um crime previsto na legislação penal, o que
público, em todos seus bens e valores. O pressuposto é comum no caso do artigo 9°, responderá o agente por
exigível é a ocorrência de dano ao patrimônio dos sujeitos ambos, nas duas esferas.
passivos. Em suma, a lei encontra-se estruturada da seguinte
Este artigo admite expressamente a variante culposa, forma: inicialmente, trata das vítimas possíveis (sujeito
o que muitos entendem ser inconstitucional. O STJ, passivo) e daqueles que podem praticar os atos de
no REsp n° 939.142/RJ, apontou alguns aspectos improbidade administrativa (sujeito ativo); ainda, aborda
da inconstitucionalidade do artigo. Contudo, “a a reparação do dano ao lesionado e o ressarcimento ao
jurisprudência do STJ consolidou a tese de que é
indispensável a existência de dolo nas condutas descritas 18 BRASIL. Superior Tribunal de Justiça. Improbidade admi-
nistrativa: desonestidade na gestão dos recursos públicos. Disponível
nos artigos 9º e 11 e ao menos de culpa nas hipóteses
em: <http://www.stj.gov.br/portal_stj/publicacao/engine.wsp?tmp.
15 SPITZCOVSKY, Celso. Direito Administrativo. 13. ed. São area=398&tmp.texto=103422>. Acesso em: 26 mar. 2013.
Paulo: Método, 2011. 19 CARVALHO FILHO, José dos Santos. Manual de direito ad-
16 Ibid. ministrativo. 23. ed. Rio de Janeiro: Lumen juris, 2010.
17 CARVALHO FILHO, José dos Santos. Manual de direito ad- 20 SPITZCOVSKY, Celso. Direito Administrativo. 13. ed. São
ministrativo. 23. ed. Rio de Janeiro: Lumen juris, 2010. Paulo: Método, 2011.

46
ATUALIDADES E DEVERES DOS SERVIDORES PÚBLICOS

patrimônio público; após, traz a tipologia dos atos de função pública, fora de tais situações, a perda se dará pela
improbidade administrativa, isto é, enumera condutas de revogação da designação”. Lembra-se que determinadas
tal natureza; seguindo-se à definição das sanções aplicáveis; autoridades se sujeitam a procedimento especial para
e, finalmente, descreve os procedimentos administrativo e perda da função pública, ponto em que não se aplica a Lei
judicial. de Improbidade Administrativa.
No caso do art. 9°, categoria mais grave, o agente - Multa: a lei indica inflexibilidade no limite máximo,
obtém um enriquecimento ilícito (vantagem econômica mas flexibilidade dentro deste limite, podendo os julgados
indevida) e pode ainda causar dano ao erário, por isso, nesta margem optar pela mais adequada. Há ainda
deverá não só reparar eventual dano causado mas variabilidade na base de cálculo, conforme o tipo de ato
também colocar nos cofres públicos tudo o que adquiriu de improbidade (a base será o valor do enriquecimento ou
indevidamente. Ou seja, poderá pagar somente o que o valor do dano ou o valor da remuneração do agente). A
enriqueceu indevidamente ou este valor acrescido do valor natureza da multa é de sanção civil, não possuindo caráter
do prejuízo causado aos cofres públicos (quanto o Estado indenizatório, mas punitivo.
perdeu ou deixou de ganhar). No caso do artigo 10, não - Proibição de receber benefícios: não se incluem
haverá enriquecimento ilícito, mas sempre existirá dano as imunidades genéricas e o agente punido deve ser ao
ao erário, o qual será reparado (eventualmente, ocorrerá menos sócio majoritário da instituição vitimada.
o enriquecimento ilícito, devendo o valor adquirido ser - Proibição de contratar: o agente punido não pode
tomado pelo Estado). Já no artigo 11, o máximo que pode participar de processos licitatórios.
ocorrer é o dano ao erário, com o devido ressarcimento.
Além disso, em todos os casos há perda da função pública. ESTATUTO DOS FUNCIONÁRIOS PÚBLICOS CIVIS
Nas três categorias, são estabelecidas sanções de suspensão DO ESTADO DE SÃO PAULO (LEI Nº 10.261/68)
dos direitos políticos, multa e vedação de contratação ou
percepção de vantagem, graduadas conforme a gravidade Com efeito, o Estatuto dos Funcionários Públicos Civis
do ato. É o que se depreende da leitura do artigo 12 da do Estado de São Paulo (Lei nº 10.261/68) é a lei maior
Lei nº 8.929/1992 como §4º do artigo 37, CF, que prevê: do funcionalismo público estadual, com direitos, deveres,
“Os atos de improbidade administrativa importarão regras de assuntos voltados às carreiras, tempo de serviço,
a suspensão dos direitos políticos, a perda da função pagamento, benefícios e outras questões específicas.
pública, a indisponibilidade dos bens e o ressarcimento
ao erário, na forma e gradação previstas em lei, sem CAPÍTULO VII
Do Direito de Petição
prejuízo da ação penal cabível”.
A única sanção que se encontra prevista na Lei
Art. 239. É assegurado a qualquer pessoa, física ou
nº 8.429/1992 mas não na Constituição Federal
jurídica, independentemente de pagamento, o direito de
é a de multa. (art. 37, §4°, CF). Não há nenhuma
petição contra ilegalidade ou abuso de poder e para
inconstitucionalidade disto, pois nada impediria que o
defesa de direitos.
legislador infraconstitucional ampliasse a relação mínima
§ 1º Qualquer pessoa poderá reclamar sobre abuso,
de penalidades da Constituição, pois esta não limitou tal
erro, omissão ou conduta incompatível no serviço público.
possibilidade e porque a lei é o instrumento adequado
§ 2º Em nenhuma hipótese, a Administração poderá
para tanto21. recusar-se a protocolar, encaminhar ou apreciar a
Carvalho Filho22 tece considerações a respeito de petição, sob pena de responsabilidade do agente.
algumas das sanções:
- Perda de bens e valores: “tal punição só incide sobre Art. 240. Ao servidor é assegurado o direito de
os bens acrescidos após a prática do ato de improbidade. requerer ou representar, bem como, nos termos desta lei
Se alcançasse anteriores, ocorreria confisco, o que restaria complementar, pedir reconsideração e recorrer de decisões,
sem escora constitucional. Além disso, o acréscimo deve no prazo de 30 (trinta) dias, salvo previsão legal específica.
derivar de origem ilícita”.
- Ressarcimento integral do dano: há quem entenda
que engloba dano moral. Cabe acréscimo de correção Estabelece a CF, no art. 5°, XXIV, a) o direito de petição,
monetária e juros de mora. assegurado a todos: “são a todos assegurados, indepen-
- Perda de função pública: “se o agente é titular dentemente do pagamento de taxas: a) o direito de petição
de mandato, a perda se processa pelo instrumento aos Poderes Públicos em defesa de direitos ou contra ile-
de cassação. Sendo servidor estatutário, sujeitar-se-á galidade ou abuso de poder;”. Os artigos acima descrevem
à demissão do serviço público. Havendo contrato de o direito de petição específico dos servidores públicos.
trabalho (servidores trabalhistas e temporários), a perda da Quanto ao direito de petição, de maneira prática,
função pública se consubstancia pela rescisão do contrato cumpre observar que o direito de petição deve resultar
com culpa do empregado. No caso de exercer apenas uma em uma manifestação do Estado, normalmente dirimindo
21 CARVALHO FILHO, José dos Santos. Manual de direito ad- (resolvendo) uma questão proposta, em um verdadeiro
ministrativo. 23. ed. Rio de Janeiro: Lumen juris, 2010. exercício contínuo de delimitação dos direitos e obrigações
22 Ibid. que regulam a vida social e, desta maneira, quando

47
ATUALIDADES E DEVERES DOS SERVIDORES PÚBLICOS

“dificulta a apreciação de um pedido que um cidadão quer CAPÍTULO I


apresentar” (muitas vezes, embaraçando-lhe o acesso à Dos Deveres e das Proibições
Justiça); “demora para responder aos pedidos formulados”
(administrativa e, principalmente, judicialmente) ou “impõe SEÇÃO I
restrições e/ou condições para a formulação de petição”, Dos Deveres
traz a chamada insegurança jurídica, que traz desesperança
e faz proliferar as desigualdades e as injustiças.
Dentro do espectro do direito de petição se insere, por Os deveres do servidor aqui previstos são em muito
exemplo, o direito de solicitar esclarecimentos, de solicitar compatíveis com os previstos no Código de Ética profis-
cópias reprográficas e certidões, bem como de ofertar sional do Servidor Público Civil do Poder Executivo Federal
denúncias de irregularidades. Contudo, o constituinte, (Decreto n° 1.171/94) e na própria Lei nº 8.112/1990, que
talvez na intenção de deixar clara a obrigação dos Poderes fixa o regime jurídico dos servidores públicos civis fede-
Públicos em fornecer certidões, trouxe a letra b) do inciso, rais. Descrevem algumas das condutas esperadas do ser-
o que gera confusões conceituais no sentido do direito de vidor público quando do desempenho de suas funções.
obter certidões ser dissociado do direito de petição. Em resumo, o servidor público deve desempenhar suas
funções com cuidado, rapidez e pontualidade, sendo leal
TÍTULO VI à instituição que compõe, respeitando as ordens de seus
DOS DEVERES, DAS PROIBIÇÕES E DAS superiores que sejam adequadas às funções que desem-
RESPONSABILIDADES penhe e buscando conservar o patrimônio do Estado. No
tratamento do público, deve ser prestativo e não negar o
O regime disciplinar do servidor público civil está esta- acesso a informações que não sejam sigilosas. Caso pre-
belecido basicamente de duas maneiras: deveres e proibi- sencie alguma ilegalidade ou abuso de poder, deve de-
ções. Ontologicamente, são a mesma coisa: ambos deveres nunciar. Tomam-se como base os ensinamentos de Lima25
e proibições são normas protetivas da boa Administração. a respeito destes deveres:
Nas duas hipóteses, violado o preceito, cabível é uma puni-
ção. Deve-se notar, porém, que os deveres constam da lei Art. 241. São deveres do funcionário:
como ações, como conduta positiva; as proibições, ao con- I - ser assíduo e pontual;
trário, são descritas como condutas vedadas ao servidor,
de modo que ele deve abster-se de praticá-las. Os deveres “Dois conceitos diferentes, porém parecidos. Ser assí-
estão inscritos não de modo exaustivo, porque o servidor duo significa ser presente dentro do horário do expedien-
deve obediência a todas as normas legais ou infralegais23. te. O oposto do assíduo é o ausente, o faltoso. Pontual é
“Estes dispositivos preveem, basicamente, um conjunto de aquele servidor que não atrasa seus compromissos. É o
normas de conduta e de proibições impostas pela lei aos que comparece no horário para as reuniões de trabalho
servidores por ela abrangidos, tendo em vista a preven- e demais atividades relacionadas com o exercício do car-
ção, a apuração e a possível punição de atos e omissões go que ocupa. Embora sejam conceitos diferentes, aqui o
que possam por em risco o funcionamento adequado da dever violado, seja por impontualidade, seja por inassidui-
administração pública, do posto de vista ético, do ponto dade (que ainda não aquela inassiduidade habitual de 60
de vista da eficiência e do ponto de vista da legalidade. De- dias ensejadora de demissão), merece reprimenda de ad-
correm, estes dispositivos, do denominado Poder Discipli- vertência, com fins educativos e de correção do servidor”.
nar que é aquele conferido à Administração com o objetivo
de manter sua disciplina interna, na medida em que lhe
atribui instrumentos para punir seus servidores (e também II - cumprir as ordens superiores, representando quando
àqueles que estejam a ela vinculados por um instrumento forem manifestamente ilegais;
jurídico determinado - particulares contratados pela Admi- “O servidor integra a estrutura organizacional do ór-
nistração). [...]”24. gão em que presta suas atribuições funcionais. O Estado
se movimenta através dos seus diversos órgãos. Dentro
dos órgãos públicos, há um escalonamento de cargos e
funções que servem ao cumprimento da vontade do ente
estatal. Este escalonamento, posto em movimento, é o
que vimos até agora chamando de hierarquia. A hierarquia
existe para que do alto escalão até a prática dos admi-
nistrados as coisas funcionem. Disso decorre que quando
23 LIMA, Fábio Lucas de Albuquerque. O regime disciplinar é emitida uma ordem para o servidor subordinado, este
dos servidores federais. Disponível em: <http://www.sato.adm.br/
artigos/o_regime_disciplinar_dos_servidores_federais.htm>. Acesso
deve dar cumprimento ao comando. Porém quando a or-
em: 11 ago. 2013. 25 LIMA, Fábio Lucas de Albuquerque. O regime disciplinar
24 MORGATO, Almir. O Regime Disciplinar dos Servidores dos servidores federais. Disponível em: <http://www.sato.adm.br/
Públicos da União. Disponível em: <http://www.canaldosconcursos. artigos/o_regime_disciplinar_dos_servidores_federais.htm>. Acesso
com.br/artigos/almirmorgado_artigo1.pdf>. Acesso em: 11 ago. 2013. em: 11 ago. 2013.

48
ATUALIDADES E DEVERES DOS SERVIDORES PÚBLICOS

dem é visivelmente ilegal, arbitrária, inconstitucional ou humanos, e quando a proteção do interesse público e ge-
absurda, o servidor não é obrigado a dar seguimento ao ral preponderante o exigir, também devem ser fornecidas
que lhe é ordenado. Quando a ordem é manifestamente as informações. Portanto, o servidor há que ter reserva no
ilegal? Há uma margem de interpretação, principalmente seu comportamento e fala, esquivando-se de revelar o con-
se o servidor subordinado não tiver nenhuma formação teúdo do que se passa no seu trabalho. Se o assunto pulu-
de ordem jurídica. Logo, é o bom senso que irá margear o lante é uma irregularidade absurda, deve então reduzir a
que é flagrantemente inconstitucional”. escrito e representar para que se apure o caso. Deveriam
diminuir as conversas de corredor e se efetivar a apuração
dos fatos através do processo administrativo disciplinar.
III - desempenhar com zelo e presteza os trabalhos de
Os assuntos objeto do serviço merecem reserva. Devem fi-
que for incumbido;
car circunscritos aos servidores designados para o respec-
“O zelo diz respeito às atribuições funcionais e também tivo trabalho interno, não devendo sair da seção ou setor
ao cuidado com a economia do material, os bens da repar- de trabalho, sem o trâmite hierárquico do chefe imedia-
tição e o patrimônio público. Sob o prisma da disciplina to. Se o assunto ou o trabalho, enfim, merecer divulgação
e da conservação dos bens e materiais da repartição, o mais ampla, deve ser contatado o órgão de assessoria de
servidor deve sempre agir com dedicação no desempenho comunicação social, que saberá proceder de forma oficial,
das funções do cargo que ocupa, e que lhe foram atribuí- obedecendo ao bom senso e às leis vigentes”.
das desde o termo de posse. O servidor não é o dono do
cargo. Dono do cargo é o Estado que o remunera. Se o
V - representar aos superiores sobre todas as
referido cargo não lhe pertence, o servidor deve exercer
irregularidades de que tiver conhecimento no exercício de
suas funções com o máximo de zelo que estiver ao seu
suas funções;
alcance. Sua eventual menor capacidade de desempenho,
para não configurar desídia ou insuficiência de desempe- “Todo servidor público é obrigado a dar conhecimento
nho, deverá ser compensada com um maior esforço e de- ao chefe da repartição acerca das irregularidades de que
dicação de sua parte. Se um servidor altamente preparado toma conhecimento no exercício de suas atribuições. Deve
e capaz, vem a praticar atos que configurem desídia ou levar ao conhecimento da chefia imediata pelo sistema
mesmo falta mais grave, poderá vir a ser punido. Porque o hierárquico. Supõe-se que os titulares das chefias ou di-
que se julgará não é a pessoa do servidor, mas a conduta a visões detêm um conhecimento maior de como corrigir o
ele imputável. O zelo não deve se limitar apenas às atribui- erro ou comunicar aos órgãos de controle para a devida
ções específicas de sua atividade. O servidor deve ter zelo apuração. De nada adiantaria o servidor, ciente de um ato
não somente com os bens e interesses imateriais (a ima- irregular, ir comunicar ao público ou a terceiros. Além do
gem, os símbolos, a moralidade, a pontualidade, o sigilo, dever de sigilo, há assuntos que exigem certas reservas,
a hierarquia) como também para com os bens e interesses visando ao bem do serviço público, da segurança nacional
patrimoniais do Estado”. e mesmo da sociedade”.

IV - guardar sigilo sobre os assuntos da repartição e, VI - tratar com urbanidade as pessoas;


especialmente, sobre despachos, decisões ou providências; “No mundo moderno, e máxime em nossa civilização
“O agente público deve guardar sigilo sobre o que se ocidental, o trato tem que ser o mais urbano possível. Ur-
passa na repartição, principalmente quanto aos assuntos bano, nessa acepção, não quer dizer citadino ou oriundo
oficiais. Pela Lei nº 12.527, de 18 de novembro de 2011, da urbe (cidade), mas, sim, educado, civilizado, cordato e
hoje está regulamentado o acesso às informações. Porém, que não possa criar embaraços aos usuários dos serviços
o servidor deve ter cuidado, pois até mesmo o forneci- públicos”.
mento ou divulgação das informações exigem um pro-
cedimento. Maior cuidado há que se ter, quando a infor- VII - residir no local onde exerce o cargo ou, onde
mação possa expor a intimidade da pessoa humana. As autorizado;
informações pessoais dos administrados em geral devem O objetivo da disciplina é impor que o servidor sempre
ser tratadas forma transparente e com respeito à intimi- seja acessível, não precise se locomover grandes distâncias
dade, à vida privada, à honra e à imagem das pessoas, quando solicitado, o que permite ainda que conheça a
bem como às liberdades e garantias individuais, segundo realidade local.
o artigo 31, da Lei nº 21.527, 2011. A exceção para o sigilo
existe, pois, não devemos tratar a questão em termos de VIII - providenciar para que esteja sempre em ordem,
cláusula jurídica de caráter absoluto, podendo ter autori- no assentamento individual, a sua declaração de família;
zada a divulgação ou o acesso por terceiros quando haja Trata-se do dever de manter seus registros atualizados,
previsão legal. Outra exceção é quando há o consentimen- inclusive no que se refere aos membros de sua família.
to expresso da pessoa a que elas se referirem. No caso de
cumprimento de ordem judicial, para a defesa de direitos

49
ATUALIDADES E DEVERES DOS SERVIDORES PÚBLICOS

IX - zelar pela economia do material do Estado e pela SEÇÃO II


conservação do que for confiado à sua guarda ou utilização; Das Proibições
“Esse deve é basilar. Se o agente não zelar pela econo-
mia e pela conservação dos bens públicos presta um des- Art. 242. Ao funcionário é proibido:
serviço à nação que lhe remunera. E como se verá adiante Em contraposição aos deveres do servidor público,
poderá ser causa inclusive de demissão, se não cumprir o existem diversas proibições, que também estão em boa
presente dever, quando por descumprimento dele a gra- parte abrangidas pelo Decreto n° 1.171/94 e pela Lei nº
vidade do fato implicar a infração a normas mais graves”. 8.112/1990. A violação dos deveres ou a prática de alguma
das violações abaixo descritas caracterizam infração admi-
X - apresentar-se convenientemente trajado em serviço nistrativa disciplinar.
ou com uniforme determinado, quando for o caso;
As roupas devem refletir o respeito à instituição e serem “Nas Proibições constata-se, desde logo, sua objetivi-
compatíveis com a função desempenhada. dade e taxatividade, o que veda sua ampliação e o uso de
interpretações analógicas ou sistemáticas visto serem con-
XI - atender prontamente, com preferência sobre qual- dutas restritivas de direitos, sujeitas, portanto, ao princípio
quer outro serviço, às requisições de papéis, documentos, da reserva legal. O descumprimento dessas proibições po-
informações ou providências que lhe forem feitas pelas au- dem inclusive, ensejar o enquadramento penal do servidor,
toridades judiciárias ou administrativas, para defesa do pois muitas das condutas ali descritas, configuram prática
Estado, em Juízo; de delito penal”26.
“Este dever foi insculpido na lei para que o servidor
público trabalhe diuturnamente no sentido de desfazer a
I - (Revogado).
imagem desagradável que o mesmo possui perante a so- II - retirar, sem prévia permissão da autoridade
ciedade. Exige-se que atue com presteza no atendimento a competente, qualquer documento ou objeto existente na
informações solicitadas pela Fazenda Pública. Esta engloba repartição;
o fisco federal, estadual, municipal e distrital. O servidor III - entreter-se, durante as horas de trabalho, em
público tem que ser expedito, diligente, laborioso. Não há palestras, leituras ou outras atividades estranhas ao
mais lugar para o burocrata que se afasta do administra- serviço;
do, dificultando a vida de quem necessita de atendimen- IV - deixar de comparecer ao serviço sem causa
to rápido e escorreito. Entretanto, há um longo caminho a justificada;
ser percorrido até que se atinja um mínimo ideal de aten- V - tratar de interesses particulares na repartição;
dimento e de funcionamento dos órgãos públicos, o que VI - promover manifestações de apreço ou desapreço
deve necessariamente passar por critérios de valorização dentro da repartição, ou tornar-se solidário com elas;
dos servidores bons e de treinamento e qualificação per- VII - exercer comércio entre os companheiros de serviço,
manente dos quadros de pessoal”. promover ou subscrever listas de donativos dentro da
repartição; e
XII - cooperar e manter espírito de solidariedade com os VIII - empregar material do serviço público em serviço
companheiros de trabalho, particular.
O ambiente de trabalho não deve ser um ambiente de
litígio, mas sim de cooperação. Art. 243. É proibido ainda, ao funcionário:
I - fazer contratos de natureza comercial e industrial
XIII - estar em dia com as leis, regulamentos, regimen- com o Governo, por si, ou como representante de outrem;
tos, instruções e ordens de serviço que digam respeito às II - participar da gerência ou administração de
suas funções; e empresas bancárias ou industriais, ou de sociedades
“A função desta norma é de não deixar sem resposta comerciais, que mantenham relações comerciais ou
qualquer que seja a irregularidade cometida. Daí a neces- administrativas com o Governo do Estado, sejam por este
sária correlação nesses casos que temos de fazer do art. subvencionadas ou estejam diretamente relacionadas com
116, inciso III, com a norma violada, e já prevista em outra a finalidade da repartição ou serviço em que esteja lotado;
lei, decreto, instrução, ordem de serviço ou portaria”. III - requerer ou promover a concessão de privilégios,
garantias de juros ou outros favores semelhantes,
XIV - proceder na vida pública e privada na forma que federais, estaduais ou municipais, exceto privilégio de
dignifique a função pública. invenção própria;
O bom comportamento não deve se fazer presente IV - exercer, mesmo fora das horas de trabalho,
somente no exercício das funções. Cabe ao funcionário se emprego ou função em empresas, estabelecimentos
portar bem quando estiver em sua vida privada, na convi- ou instituições que tenham relações com o Governo, em
vência com seus amigos e familiares, bem como nos mo- matéria que se relacione com a finalidade da repartição ou
mentos de lazer. Por melhor que seja como funcionário, serviço em que esteja lotado;
não será aceito aquele que, por exemplo, for visto frequen- 26 MORGATO, Almir. O Regime Disciplinar dos Servidores
temente embriagado ou for sempre denunciado por vio- Públicos da União. Disponível em: <http://www.canaldosconcursos.
lência doméstica. com.br/artigos/almirmorgado_artigo1.pdf>. Acesso em: 11 ago. 2013.

50
ATUALIDADES E DEVERES DOS SERVIDORES PÚBLICOS

V - aceitar representação de Estado estrangeiro, sem mediante designações recíprocas, viola a Constituição Fe-
autorização do Presidente da República; deral.” Obs.: se o concurso pedir pelo entendimento juris-
VI - comerciar ou ter parte em sociedades comerciais nas prudencial, vá pela súmula, mas se não mencionar nada
condições mencionadas no item II deste artigo, podendo, em se atenha ao texto da lei, visto que há pequenas variações
qualquer caso, ser acionista, quotista ou comanditário; entre o texto da súmula e o da lei.
VII - incitar greves ou a elas aderir, ou praticar atos de
sabotagem contra o serviço público; (INCONSTITUCIONAL) CAPÍTULO II
O Supremo Tribunal Federal decidiu que os servidores Das Responsabilidades
públicos possuem o direito de greve, devendo se atentar
pela preservação da sociedade quando exercê-lo. Art. 245. O funcionário é responsável por todos os
Enquanto não for elaborada uma legislação específica para prejuízos que, nessa qualidade, causar à Fazenda Estadual,
os funcionários públicos, deverá ser obedecida a lei geral por dolo ou culpa, devidamente apurados.
de greve para os funcionários privados, qual seja a Lei n° Parágrafo único. Caracteriza-se especialmente a
7.783/89 (Mandado de Injunção nº 20). responsabilidade:
I - pela sonegação de valores e objetos confiados à
VIII - praticar a usura; sua guarda ou responsabilidade, ou por não prestar contas,
IX - constituir-se procurador de partes ou servir de ou por não as tomar, na forma e no prazo estabelecidos
intermediário perante qualquer repartição pública, exceto nas leis, regulamentos, regimentos, instruções e ordens de
quando se tratar de interesse de cônjuge ou parente até serviço;
segundo grau; II - pelas faltas, danos, avarias e quaisquer outros
X - receber estipêndios de firmas fornecedoras ou de prejuízos que sofrerem os bens e os materiais sob sua
entidades fiscalizadas, no País, ou no estrangeiro, mesmo guarda, ou sujeitos a seu exame ou fiscalização;
quando estiver em missão referente à compra de material III - pela falta ou inexatidão das necessárias
ou fiscalização de qualquer natureza; averbações nas notas de despacho, guias e outros
XI - valer-se de sua qualidade de funcionário para documentos da receita, ou que tenham com eles relação; e
desempenhar atividade estranha às funções ou para IV - por qualquer erro de cálculo ou redução contra a
lograr, direta ou indiretamente, qualquer proveito; e Fazenda Estadual.
XII - fundar sindicato de funcionários ou deles fazer
parte. (INCONSTITUCIONAL) Art. 246. O funcionário que adquirir materiais em
Os servidores públicos têm direito de fundar sindicatos, desacordo com disposições legais e regulamentares,
o que é inerente ao direito fundamental à liberdade de será responsabilizado pelo respectivo custo, sem prejuízo das
associação. penalidades disciplinares cabíveis, podendo-se proceder ao
desconto no seu vencimento ou remuneração.
Parágrafo único. Não está compreendida na proibição
dos itens II e VI deste artigo, a participação do funcionário Art. 247. Nos casos de indenização à Fazenda Estadual,
em sociedades em que o Estado seja acionista, bem assim o funcionário será obrigado a repor, de uma só vez, a
na direção ou gerência de cooperativas e associações de importância do prejuízo causado em virtude de alcance,
classe, ou como seu sócio. desfalque, remissão ou omissão em efetuar recolhimento ou
entrada nos prazos legais.
Art. 244. É vedado ao funcionário trabalhar sob as
ordens imediatas de parentes, até segundo grau, salvo Art. 248. Fora dos casos incluídos no artigo anterior,
quando se tratar de função de confiança e livre escolha, a importância da indenização poderá ser descontada do
não podendo exceder a 2 (dois) o número de auxiliares vencimento ou remuneração não excedendo o desconto à
nessas condições. 10ª (décima) parte do valor destes.
É a chamada prática de nepotismo. Do latim nepos, Parágrafo único. No caso do item IV do parágrafo único
neto ou descendente, é o termo utilizado para designar o do art. 245, não tendo havido má-fé, será aplicada a pena
favorecimento de parentes (ou amigos próximos) em detri- de repreensão e, na reincidência, a de suspensão.
mento de pessoas mais qualificadas, especialmente no que
diz respeito à nomeação ou elevação de cargos. Destaca-se Art. 249. Será igualmente responsabilizado o
o teor da súmula vinculante nº 13 do STF: funcionário que, fora dos casos expressamente previstos
Súmula Vinculante nº 13: “A nomeação de cônjuge, nas leis, regulamentos ou regimentos, cometer a pessoas
companheiro ou parente em linha reta, colateral ou por estranhas às repartições, o desempenho de encargos que
afinidade, até o terceiro grau, inclusive, da autoridade no- lhe competirem ou aos seus subordinados.
meante ou de servidor da mesma pessoa jurídica, investi-
do em cargo de direção, chefia ou assessoramento, para o Art. 250. A responsabilidade administrativa não
exercício de cargo em comissão ou de confiança, ou, ainda, exime o funcionário da responsabilidade civil ou
de função gratificada na Administração Pública direta e in- criminal que no caso couber, nem o pagamento da
direta, em qualquer dos Poderes da União, dos Estados, do indenização a que ficar obrigado, na forma dos arts. 247 e
Distrito Federal e dos municípios, compreendido o ajuste 248, o exame da pena disciplinar em que incorrer.

51
ATUALIDADES E DEVERES DOS SERVIDORES PÚBLICOS

§ 1º A responsabilidade administrativa é independente As pessoas jurídicas de direito público e as de direito


da civil e da criminal. privado prestadoras de serviços públicos responderão
§ 2º Será reintegrado ao serviço público, no cargo pelos danos que seus agentes, nessa qualidade, causarem
que ocupava e com todos os direitos e vantagens devidas, a terceiros, assegurado o direito de regresso contra o
o servidor absolvido pela Justiça, mediante simples responsável nos casos de dolo ou culpa.
comprovação do trânsito em julgado de decisão que negue
a existência de sua autoria ou do fato que deu origem à sua Este artigo deixa clara a formação de uma relação
demissão. jurídica autônoma entre o Estado e o agente público que
§ 3º O processo administrativo só poderá ser sobrestado causou o dano no desempenho de suas funções. Nesta
para aguardar decisão judicial por despacho motivado da relação, a responsabilidade civil será subjetiva, ou seja,
autoridade competente para aplicar a pena. caberá ao Estado provar a culpa do agente pelo dano
Segundo Carvalho Filho27, “a responsabilidade se origi- causado, ao qual foi anteriormente condenado a reparar.
na de uma conduta ilícita ou da ocorrência de determinada Direito de regresso é justamente o direito de acionar o
situação fática prevista em lei e se caracteriza pela natureza causador direto do dano para obter de volta aquilo que
do campo jurídico em que se consuma. Desse modo, a res- pagou à vítima, considerada a existência de uma relação
ponsabilidade pode ser civil, penal e administrativa. Cada obrigacional que se forma entre a vítima e a instituição
responsabilidade é, em princípio, independente da outra”. que o agente compõe.
É possível que o mesmo fato gere responsabilidade ci- Assim, o Estado responde pelos danos que seu agente
vil, penal e administrativa, mas também é possível que este causar aos membros da sociedade, mas se este agente
gere apenas uma ou outra espécie de responsabilidade. agiu com dolo ou culpa deverá ressarcir o Estado do que
Daí o fato das responsabilidades serem independentes: o foi pago à vítima. O agente causará danos ao praticar
mesmo fato pode gerar a aplicação de qualquer uma delas, condutas incompatíveis com o comportamento ético dele
cumulada ou isoladamente. esperado.29
O instituto da responsabilidade civil é parte integrante A responsabilidade civil do servidor exige prévio
do direito obrigacional, uma vez que a principal processo administrativo disciplinar no qual seja assegurado
consequência da prática de um ato ilícito é a obrigação contraditório e ampla defesa.
que gera para o seu auto de reparar o dano, mediante Trata-se de responsabilidade civil subjetiva ou com
o pagamento de indenização que se refere às perdas e culpa. Havendo ação ou omissão com culpa do servidor
danos. Afinal, quem pratica um ato ou incorre em omissão que gere dano ao erário (Administração) ou a terceiro
que gere dano deve suportar as consequências jurídicas (administrado), o servidor terá o dever de indenizar.
decorrentes, restaurando-se o equilíbrio social.28 Já a responsabilidade penal do servidor decorre de
A responsabilidade civil, assim, difere-se da penal, uma conduta que a lei penal tipifique como infração penal,
podendo recair sobre os herdeiros do autor do ilícito até ou seja, como crime ou contravenção penal.
os limites da herança, embora existam reflexos na ação O servidor poderá ser responsabilizado apenas penal-
que apure a responsabilidade civil conforme o resultado na mente, uma vez que somente caberá responsabilização
esfera penal (por exemplo, uma absolvição por negativa de civil se o ato tiver causado prejuízo ao erário (elemento
autoria impede a condenação na esfera cível, ao passo que dano).
uma absolvição por falta de provas não o faz). Os crimes contra a Administração Pública se encon-
Genericamente, os elementos da responsabilidade civil tram nos artigos 312 a 326 do Código Penal, mas existem
se encontram no art. 186 do Código Civil: “aquele que, por outros crimes espalhados pela legislação específica.
ação ou omissão voluntária, negligência ou imprudência, Por seu turno, quando o servidor pratica um ilícito ad-
violar direito e causar dano a outrem, ainda que ministrativo, a ele é atribuída responsabilidade adminis-
exclusivamente moral, comete ato ilícito”. Este é o artigo trativa. O ilícito pode verificar-se por conduta comissiva
central do instituto da responsabilidade civil, que tem ou omissiva e os fatos que o configuram são os previstos
como elementos: ação ou omissão voluntária (agir como na legislação estatutária. Por exemplo, as sanções apli-
não se deve ou deixar de agir como se deve), culpa ou dolo cadas pela Comissão de Ética por violação ao Decreto n°
do agente (dolo é a vontade de cometer uma violação de 1.171/94 são administrativas.
direito e culpa é a falta de diligência), nexo causal (relação Se as responsabilidades se cumularem, também as
de causa e efeito entre a ação/omissão e o dano causado) sanções serão cumuladas. Daí afirmar-se que tais respon-
e dano (dano é o prejuízo sofrido pelo agente, que pode sabilidades são independentes, ou seja, não dependem
ser individual ou coletivo, moral ou material, econômico e uma da outra.
não econômico). Determinadas decisões na esfera penal geram exclu-
Prevê o artigo 37, §6° da Constituição Federal: são da responsabilidade nas esferas civil e administrativa,
quais sejam: absolvição por inexistência do fato ou negati-
va de autoria. A absolvição criminal por falta de provas não
27 CARVALHO FILHO, José dos Santos. Manual de direito ad-
ministrativo. 23. ed. Rio de Janeiro: Lumen juris, 2010. gera exclusão da responsabilidade civil e administrativa.
28 GONÇALVES, Carlos Roberto. Responsabilidade Civil. 9. 29 SPITZCOVSKY, Celso. Direito Administrativo. 13. ed. São
ed. São Paulo: Saraiva, 2005. Paulo: Método, 2011.

52
ATUALIDADES E DEVERES DOS SERVIDORES PÚBLICOS

A absolvição proferida na ação penal, em regra, nada 2. (VUNESP/2015 - TJ-SP - Estatístico Judiciário)
prejudica a pretensão de reparação civil do dano ex delicto, Nos termos do que expressamente estabelece a Lei n°
conforme artigos 65, 66 e 386, IV do CPP: “art. 65. Faz coisa 10.261/68, é dever do funcionário público
julgada no cível a sentença penal que reconhecer ter sido o a) cumprir as ordens superiores, mesmo quando forem
ato praticado em estado de necessidade, em legítima defe- manifestamente ilegais.
sa, em estrito cumprimento de dever legal ou no exercício b) residir no local onde exerce o cargo ou onde
regular de direito” (excludentes de antijuridicidade); “art. autorizado.
66. não obstante a sentença absolutória no juízo criminal, c) guardar sigilo sobre os assuntos da repartição,
a ação civil poderá ser proposta quando não tiver sido, ca- exceto sobre despachos, decisões ou providências.
tegoricamente, reconhecida a inexistência material do d) manter sigilo sobre as irregularidades de que tiver
fato”; “art. 386, IV –  estar provado que o réu não concor- conhecimento no exercício de suas funções, deixando
reu para a infração penal”. eventual investigação para as autoridades competentes.
Entendem Fuller, Junqueira e Machado30: “a absolvição e) providenciar para que estejam sempre em ordem
dubitativa (motivada por juízo de dúvida), ou seja, por falta todas as mesas de trabalho da repartição onde exerce suas
de provas, (art. 386, II, V e VII, na nova redação conferida ao funções.
CPP), não empresta qualquer certeza ao âmbito da jurisdi- R: “B”. O artigo 241, em seu inciso VII, fixa como dever
do funcionário “residir no local onde exerce o cargo ou,
ção civil, restando intocada a possibilidade de, na ação civil
onde autorizado”.
de conhecimento, ser provada e reconhecida a existência
do direito ao ressarcimento, de acordo com o grau de cog-
3. (VUNESP/2015 - TJ-SP - Estatístico Judiciário)
nição e convicção próprios da seara civil (na esfera penal,
A Lei n° 10.261/68 dispõe que ao funcionário público é
a decisão de condenação somente pode ser lastreada em proibido
juízo de certeza, tendo em vista o princípio constitucional a) fazer parte dos quadros sociais de qualquer tipo de
do estado de inocência)”. sociedade comercial.
b) deixar de comparecer ao serviço, mesmo que por
EXERCÍCIOS causa justificada.
c) participar da gerência de sociedades comerciais,
1. (VUNESP/2015 - TJ-SP - Escrevente Técnico mesmo daquelas que não mantenham relações comerciais
Judiciário) Escrivão­Diretor da 1 a Vara Cível da Comarca ou administrativas com o Governo do Estado.
X determina que Escrevente Técnico Judiciário, a ele d) exercer, mesmo fora das horas de trabalho, emprego
subordinado, destrua um documento, colocando­-o em uma ou função em qualquer tipo de empresa.
fragmentadora de papel. O Escrevente Técnico Judiciário e) empregar material do serviço público em serviço
percebe que o documento é uma petição assinada e particular.
devidamente protocolada, que deveria ser encartada em R: “E”. Conforme o art. 242, VIII, ao funcionário
um processo que tramitava naquela Vara e que ainda não público é proibido “empregar material do serviço público
havia sido sentenciado. O Escrevente Técnico Judiciário em serviço particular”. Em relação às demais alternativas,
deverá, nos termos do Estatuto dos Funcionários Públicos conforme art. 243, II, é proibido ao funcionário “participar
Civis do Estado de São Paulo, da gerência ou administração de empresas bancárias ou
a) cumprir a ordem, pois é dever do servidor público industriais, ou de sociedades comerciais, que mantenham
cooperar e manter espírito de solidariedade com os relações comerciais ou administrativas com o Governo
companheiros de trabalho. do Estado, sejam por este subvencionadas ou estejam
b) utilizar­se do documento como papel de rascu­nho diretamente relacionadas com a finalidade da repartição
para seu trabalho, considerando que é dever do servidor ou serviço em que esteja lotado”; conforme art. 242, IV,
público zelar pela economia do material do Estado proíbe-se “deixar de comparecer ao serviço sem causa
c) representar ao Juiz da Vara, já que é dever do servidor justificada”; conforme art. 243, IV, é proibido “exercer,
mesmo fora das horas de trabalho, emprego ou função em
público representar contra ordens manifestamente ilegais.
empresas, estabelecimentos ou instituições que tenham
d) desempenhar com zelo e presteza os trabalhos de
relações com o Governo, em matéria que se relacione com
que for incumbido, destruindo o documento.
a finalidade da repartição ou serviço em que esteja lotado”.
e) proceder conforme ordenado pelo Escrivão ­Diretor,
nada dizendo sobre o assunto, pois é dever do servidor 4. (VUNESP/2015 - TJ-SP - Estatístico Judiciário)
público guardar sigilo sobre os assun­tos da repartição. Sobre a responsabilidade dos funcionários públicos, é
R: “C”. Estabelece-se como dever do servidor, no correto afirmar, nos moldes da Lei n° 10.261/68, que
artigo 241, II, “cumprir as ordens superiores, representando a) o funcionário é responsável por todos os prejuízos
quando forem manifestamente ilegais”. que, nessa qualidade, causar à Fazenda Estadual,
independentemente de dolo ou culpa, devidamente
30 FULLER, Paulo Henrique Aranda; JUNQUEIRA, Gustavo Oc-
apurados.
taviano Diniz; MACHADO, Angela C. Cangiano. Processo Penal. 9. ed. b) a responsabilidade administrativa exime o
São Paulo: Revista dos Tribunais, 2010. (Coleção Elementos do Direito) funcionário da responsabilidade civil.

53
ATUALIDADES E DEVERES DOS SERVIDORES PÚBLICOS

c) a responsabilidade administrativa do funcionário 6. (VUNESP/2013 - SEFAZ-SP - Analista em


depende da criminal e da civil. Planejamento, Orçamento e Finanças Públicas) A
d) o funcionário que for absolvido pela justiça em empresa ABC Ltda. comparece ao setor de protocolo de uma
processo criminal, por qualquer motivo, não responderá repartição pública estadual com o objetivo de protocolar
civil e administrativamente pelo mesmo fato. petição pedindo providências contra uma ilegalidade de
e) o processo administrativo só poderá ser sobrestado que foi vítima no serviço público. Considerando o disposto
para aguardar decisão judicial por despacho motivado da no Estatuto dos Funcionários Públicos Civis do Estado
autoridade competente para aplicar a pena. de São Paulo, é correto afirmar, nessa situação, que o
R: “E”. De acordo com o art. 250, §1º, “o processo funcionário do setor de protocolo
administrativo só poderá ser sobrestado para aguardar a) poderá receber a petição, desde que a empresa
decisão judicial por despacho motivado da autoridade ABC pague os emolumentos devidos para esse tipo de
competente para aplicar a pena”. A alternativa “a” está requerimento, conforme estabelecem as normas do
errada porque conforme o art. 245, “funcionário é respectivo órgão público.
responsável por todos os prejuízos que, nessa qualidade, b) deverá receber a petição, já que a empresa ABC tem
causar à Fazenda Estadual, por dolo ou culpa, devidamente esse direito legalmente garantido, independentemente
apurados”; as alternativas “b” e “c” estão erradas porque nos do pagamento de taxas, sob pena de responsabilidade do
termos do art. 250 “a responsabilidade administrativa não servidor se este se recusar a recebê-la.
exime o funcionário da responsabilidade civil ou criminal”; c) não deve receber a petição, posto que esse direito
a alternativa “d” está errada porque o artigo 250 prevê que não é conferido pela lei às pessoas jurídicas, mas somente
o funcionário que for absolvido pela justiça em processo às pessoas físicas.
criminal não responderá civil e administrativamente pelo d) não poderá receber a petição, porque, embora
mesmo fato, apenas se a absolvição se der por inexistência esse direito seja garantido às pessoas físicas e jurídicas de
do fato ou ausência de autoria. forma geral, o objetivo buscado pela empresa ABC, com o
requerimento, não tem previsão legal.
5. (VUNESP/2014 - TJ-SP - Escrevente Técnico e) não deve receber a petição, uma vez que esse tipo
Judiciário) O Estatuto dos Funcionários Públicos Civis do de pedido não pode ser feito diretamente à Administração
Estado de São Paulo prevê, a respeito do direito de petição, Pública, mas deve ser dirigido ao Poder Judiciário.
que R: “B”. A empresa ABC tem direito de peticionar
a) somente a pessoa física poderá peticionar contra contra ilegalidade ou abuso de poder e para defesa
ilegalidade ou abuso de poder e ser isenta do pagamento direitos, independentemente do pagamento de taxas. E
de taxas. o servidor não pode, em nenhuma hipótese, recusar-se a
b) o servidor não poderá recusar-se a protocolar, protocolar, encaminhar ou apreciar a petição, sob pena de
encaminhar ou apreciar a petição, sob pena de responsabilidade, em consonância com o que preveem os
responsabilidade. artigos 239 e 240.
c) qualquer pessoa poderá se utilizar do direito de
petição para comunicar ilegalidade ou abuso de poder, ou 7. (VUNESP/2013 - SEFAZ-SP - Analista em
ainda defender o patrimônio público, desde que recolha a Planejamento, Orçamento e Finanças Públicas) Minerva,
taxa devida. funcionária pública estadual, comovida com a situação
d) não é assegurado ao servidor o direito de requerer de uma amiga que está passando por sérios problemas
ou representar, pedir reconsideração e recorrer de decisões, financeiros e de saúde, resolve ajudá-la promovendo
mesmo diante de manifesta ilegalidade. uma lista de donativos dentro da sua repartição, pedindo
e) a pessoa que queira reclamar sobre abuso, erro, um pequena contribuição de cada colega de trabalho
omissão ou conduta incompatível no serviço público em benefício da referida amiga. Segundo o disposto no
deverá comprovar seu interesse legítimo na questão, sob Estatuto dos Funcionários Públicos Civis do Estado de São
pena de indeferimento da petição. Paulo, essa conduta de Minerva
R: “B”. Considera-se o artigo 239: “É assegurado a a) é proibida por lei.
qualquer pessoa, física ou jurídica, independentemente b) pode ser adotada, desde que devidamente
de pagamento, o direito de petição contra ilegalidade ou autorizada pelo chefe da repartição e que não atrapalhe o
abuso de poder e para defesa de direitos. § 1º Qualquer bom andamento do serviço público
pessoa poderá reclamar sobre abuso, erro, omissão c) se constitui em uma das exceções permitidas por lei
ou conduta incompatível no serviço público. § 2º Em que autoriza Minerva a adotá-la, tendo em vista o pequeno
nenhuma hipótese, a Administração poderá recusar-se valor por ela solicitado e o nobre objetivo de seu ato.
a protocolar, encaminhar ou apreciar a petição, sob pena d) é legalmente permitida.
de responsabilidade do agente”. Ainda, tem-se o artigo e) não é disciplinada por lei e, portanto, nada impede
240: “ao servidor é assegurado o direito de requerer ou Minerva de assim agir.
representar, bem como, nos termos desta lei complementar, R: “A”. Nos termos do art. 242, VII, o funcionário público
pedir reconsideração e recorrer de decisões, no prazo de é proibido de “exercer comércio entre os companheiros de
30 (trinta) dias, salvo previsão legal específica”. serviço, promover ou subscrever listas de donativos dentro
da repartição”.

54
ATUALIDADES E DEVERES DOS SERVIDORES PÚBLICOS

8. (FCC/2012 - TCE-SP - Agente de Fiscalização Item III: correto, porque nos termos do art. 246, “o
Financeira - Administração) De acordo com o Estatuto funcionário que adquirir materiais em desacordo com
dos Funcionários Públicos Civis do Estado de São Paulo, disposições legais e regulamentares, será responsabilizado
a atuação de funcionários públicos na administração de pelo respectivo custo, sem prejuízo das penalidades
sociedades comerciais é disciplinares cabíveis, podendo-se proceder ao desconto
a) permitida, exceto, apenas, se a sociedade tiver objeto no seu vencimento ou remuneração”.
relacionado com a atividade da repartição ou serviço em Item IV: correto, porque nos termos do art. 250, §3º,
que o servidor esteja lotado. “o processo administrativo só poderá ser sobrestado
b) permitida, exceto, apenas, se a sociedade mantiver para aguardar decisão judicial por despacho motivado da
relações comerciais com o Governo do Estado. autoridade competente para aplicar a pena
c) permitida, exceto, apenas, se a sociedade for
subvencionada pelo Governo do Estado. 10. (VUNESP/2011 - TJM-SP - Oficial de Justiça)
d) permitida, em se tratando de sociedade em que Nos termos do Estatuto dos Funcionários Públicos Civis do
o Estado seja acionista, bem como em cooperativas e Estado de São Paulo, é proibido ao funcionário público
associações de classe. I. participar na gerência ou administração de empresas
e) vedada, exceto quando se trate de empresa bancárias ou industriais, ou de sociedades comerciais, que
controlada ou subvencionada pelo Governo do Estado. mantenham relações comerciais ou administrativas com
R: “D”. Nos moldes do art. 194, VII e VIII do estatuto, o Governo do Estado, sejam por este subvencionadas ou
o funcionário é proibido de “participar de gerência ou estejam relacionadas com a finalidade da repartição ou
administração de empresa comercial ou industrial, salvo serviço em que esteja lotado;
órgão da administração pública indireta” e de “exercer II. entreter-se, durante as horas de trabalho, em
comércio ou participar de sociedade comercial, exceto palestras, leituras ou outras atividades estranhas ao serviço;
como acionista cotista ou comanditário”. III. referir-se depreciativamente, em informação,
parecer ou despacho, ou pela imprensa, ou qualquer meio
9. (VUNESP/2011 - TJM-SP - Escrevente Técnico de divulgação, às autoridades constituídas;
Judiciário) Das responsabilidades dos funcionários
IV. exercer, mesmo fora das horas de trabalho, emprego
públicos, pode-se afirmar que
ou função em empresas, estabelecimentos ou instituições
I. a responsabilidade administrativa exime o funcionário
que tenham relações com o Governo, em matéria que se
da responsabilidade civil que no caso couber;
relacione com a finalidade da repartição ou serviço em que
II. nos casos de indenização à Fazenda Estadual, o
esteja lotado.
funcionário será obrigado a repor, de forma parcelada, a
Está correto o contido em
importância do prejuízo causado em virtude do desfalque;
a) I, II e IV, apenas.
III. o funcionário que adquirir materiais em
desacordo com disposições legais e regulamentares será b) III e IV, apenas.
responsabilizado pelo respectivo custo, sem prejuízo das c) I, II e III, apenas.
penalidades disciplinares cabíveis, podendo-se proceder d) I e III, apenas.
ao desconto no seu vencimento ou remuneração; e) I, II, III e IV.
IV. o processo administrativo só poderá ser sobrestado R: “A”. Os itens I, II e IV estão corretos, ao passo que o
para aguardar decisão judicial por despacho motivado da item III está incorreto.
autoridade competente para aplicar a pena. Item I: correto porque o art. 243, II proíbe “participar
Está correto o contido em na gerência ou administração de empresas bancárias ou
a) I e II, apenas. industriais, ou de sociedades comerciais, que mantenham
b) III e IV, apenas. relações comerciais ou administrativas com o Governo
c) I, II e III, apenas. do Estado, sejam por este subvencionadas ou estejam
d) II, III e IV, apenas. relacionadas com a finalidade da repartição ou serviço em
e) I, II, III e IV. que esteja lotado”.
R: “B”. Os itens I e II estão incorretos e os itens III e IV Item II: correto porque o art. 242, III proíbe “entreter-
estão corretos. se, durante as horas de trabalho, em palestras, leituras ou
Item I: incorreto porque nos termos do art. 250, “a outras atividades estranhas ao serviço”.
responsabilidade administrativa não exime o funcionário Item III: está incorreto porque o dispositivo que assim
da responsabilidade civil ou criminal que no caso couber, previa, o artigo 242, I, foi revogado.
nem o pagamento da indenização a que ficar obrigado, na Item IV: correto porque o art. 243, IV proíbe “exercer,
forma dos arts. 247 e 248, o exame da pena disciplinar em mesmo fora das horas de trabalho, emprego ou função em
que incorrer”. empresas, estabelecimentos ou instituições que tenham
Item II: incorreto porque nos termos do art. 247, “nos relações com o Governo, em matéria que se relacione com
casos de indenização à Fazenda Estadual, o funcionário a finalidade da repartição ou serviço em que esteja lotado”.
será obrigado a repor, de uma só vez, a importância
do prejuízo causado em virtude de alcance, desfalque,
remissão ou omissão em efetuar recolhimento ou entrada
nos prazos legais”.

55
ATUALIDADES E DEVERES DOS SERVIDORES PÚBLICOS

LEI N° 8.429 DE 2 DE JUNHO DE 1992


3. LEI FEDERAL Nº 8.429/92
Dispõe sobre as sanções aplicáveis aos agentes públi-
(LEI DE IMPROBIDADE ADMINISTRATIVA)
cos nos casos de enriquecimento ilícito no exercício de
ARTIGOS 1º AO 11º – COM AS ALTERAÇÕES mandato, cargo, emprego ou função na administração
VIGENTES ATÉ A PUBLICAÇÃO DO EDITAL. pública direta, indireta ou fundacional e dá outras pro-
vidências.
O preâmbulo da lei em estudo já traz alguns elementos
A Lei n° 8.429/92 trata da improbidade administrativa, importantes para a sua boa compreensão:
que é uma espécie qualificada de imoralidade, sinônimo de a) o agente público pode estar exercendo mandato,
desonestidade administrativa. A improbidade é uma lesão quando for eleito para tanto; cargo, no caso de um conjun-
ao princípio da moralidade, que deve ser respeitado estri- to de atribuições e responsabilidades conferido a um servi-
tamente pelo servidor público. O agente ímprobo sempre dor submetido a regime estatutário (é o caso do ingresso
será um violador do princípio da moralidade, pelo qual “a por concurso); emprego público, se o servidor se submeter
Administração Pública deve agir com boa-fé, sinceridade, a regime celetista (CLT); funçãopública, que correspon-
probidade, lhaneza, lealdade e ética”31. de à categoria residual, valendo para o servidor que tenha
A atual Lei de Improbidade Administrativa foi criada tais atribuições e responsabilidades mas não exerça cargo
devido ao amplo apelo popular contra certas vicissitudes ou emprego público. Percebe-se que o conceito de agente
do serviço público que se intensificavam com a ineficácia público que se sujeita à lei é o mais amplo possível.
do diploma então vigente, o Decreto-Lei nº 3240/41. De- b) o exercício pode se dar na administração direta, in-
correu, assim, da necessidade de acabar com os atos aten- direta ou fundacional. A administração pública apresenta
tatórios à moralidade administrativa e causadores de pre- uma estrutura direta e outra indireta, com seus respectivos
juízo ao erário público ou ensejadores de enriquecimento órgãos. Por exemplo, são órgãos da administração direta
ilícito, infelizmente tão comuns no Brasil. os ministérios e secretarias, isto é, os órgãos que compõem
a estrutura do Executivo, Legislativo ou Judiciário; são inte-
Com o advento da Lei nº 8.429/92, os agentes públi-
grantes da administração indireta as autarquias, fundações
cos passaram a ser responsabilizados na esfera civil pelos
públicas, empresas públicas e sociedades de economia
atos de improbidade administrativa descritos nos artigos
mista.
9º, 10 e 11, ficando sujeitos às penas do art. 12. A exis-
tência de esferas distintas de responsabilidade (civil, penal
CAPÍTULO I
e administrativa) impede falar-se em bis in idem, já que,
Das Disposições Gerais
ontologicamente, não se trata de punições idênticas, em-
bora baseadas no mesmo fato, mas de responsabilização Art. 1° Os atos de improbidade praticados por qualquer
em esferas distintas do Direito. agente público, servidor ou não, contra a administração
A legislação em estudo, por sua vez, divide os atos de direta, indireta ou fundacional de qualquer dos Poderes
improbidade administrativa em três categorias: da União, dos Estados, do Distrito Federal, dos Municípios,
a) Ato de improbidade administrativa que importe en- de Território, de empresa incorporada ao patrimônio pú-
riquecimento ilícito; blico ou de entidade para cuja criação ou custeio o erário
b) Ato de improbidade administrativa que importe le- haja concorrido ou concorra com mais de cinquenta por
são ao erário; cento do patrimônio ou da receita anual, serão punidos na
c) Ato de improbidade administrativa que atente con- forma desta lei.
tra os princípios da administração pública. Parágrafo único. Estão também sujeitos às penalida-
ATENÇÃO: os atos de improbidade administrativa não des desta lei os atos de improbidade praticados contra o
são crimes de responsabilidade. Trata-se de punição na patrimônio de entidade que receba subvenção, benefício
esfera cível, não criminal. Por isso, caso o ato configure ou incentivo, fiscal ou creditício, de órgão público bem
simultaneamente um ato de improbidade administrativa como daquelas para cuja criação ou custeio o erário haja
desta lei e um crime previsto na legislação penal, o que concorrido ou concorra com menos de cinquenta por cen-
é comum no caso do artigo 9°, responderá o agente por to do patrimônio ou da receita anual, limitando-se, nestes
ambos, nas duas esferas. casos, a sanção patrimonial à repercussão do ilícito sobre a
Em suma, a lei encontra-se estruturada da seguinte contribuição dos cofres públicos.
forma: inicialmente, trata das vítimas possíveis (sujeito pas- “Sujeito passivo é a pessoa que a lei indica como víti-
sivo) e daqueles que podem praticar os atos de improbida- ma do ato de improbidade administrativa”. A lei adota uma
de administrativa (sujeito ativo); ainda, aborda a reparação noção ampla, pela qual são abrangidas entidades que, sem
do dano ao lesionado e o ressarcimento ao patrimônio integrarem a Administração, possuem alguma espécie de
público; após, traz a tipologia dos atos de improbidade ad- conexão com ela.32
ministrativa, isto é, enumera condutas de tal natureza; se- O agente público pode ser ou não um servidor públi-
guindo-se à definição das sanções aplicáveis; e, finalmente, co. O conceito de agente público é melhor delimitado no
descreve os procedimentos administrativo e judicial. artigo seguinte.
31 LENZA, Pedro. Curso de direito constitucional esquematiza- 32 CARVALHO FILHO, José dos Santos. Manual de direito admi-
do. 15. ed. São Paulo: Saraiva, 2011. nistrativo. 23. ed. Rio de Janeiro: Lumen juris, 2010.

56
ATUALIDADES E DEVERES DOS SERVIDORES PÚBLICOS

Ele poderá estar vinculado a qualquer instituição ou ór- A ampla denominação de agentes públicos conferida
gão que desempenhe diretamente o interesse do Estado. pela lei de improbidade administrativa apenas tem efeito
Assim, estão incluídos todos os integrantes da administra- para os fins desta lei, ou seja, visando a imputação dos atos
ção direta, indireta e fundacional, conforme o preâmbulo de improbidade administrativa. Percebe-se a amplitude
da legislação. Pode até mesmo ser uma entidade privada pelos elementos do conceito:
que desempenhe tais fins, desde que a verba de criação a) Tempo: exercício transitório ou definitivo;
ou custeio tenha sido ou seja pública em mais de50% do b) Remuneração: existente ou não;
patrimônio ou receita anual. c) Espécie de vínculo: por eleição, nomeação, designa-
Caso a verba pública que tenha auxiliado uma entida- ção, contratação ou qualquer outra forma de investidura
de privada a qual o Estado não tenha concorrido para ou vínculo, mandato, cargo, emprego ou função;
criação ou custeio, também haverá sujeição às penalida- d) Local do exercício: em qualquer entidade que pos-
des da lei. Em caso de custeio/criação pelo Estado que sa ser sujeito passivo. Por exemplo, o funcionário de uma
seja inferior a 50% do patrimônio ou receita anual, a le- ONG criada pelo Estado é considerado agente público para
gislação ainda se aplica. Entretanto, nestes dois casos, a os efeitos desta lei.
sanção patrimonial se limitará ao que o ilícito repercutiu O terceiro, por sua vez, é aquele que pratica as condu-
sobre a contribuição dos cofres públicos. Significa que se tas de induzir ou concorrer em relação ao agente público,
o prejuízo causado for maior que a efetiva contribuição ou seja, incentivando-o ou mesmo participando direta-
por parte do poder público, o ressarcimento terá que ser mente do ilícito. Este terceiro jamais será pessoa jurídica,
buscado por outra via que não a ação de improbidade deve necessariamente ser pessoa física.
administrativa.
Basicamente, o dispositivo enumera os principais su- Art. 4° Os agentes públicos de qualquer nível ou hie-
jeitos passivos do ato de improbidade administrativa, di- rarquia são obrigados a velar pela estrita observância dos
vidindo-os em três grupos: a) pessoas da administração princípios de legalidade, impessoalidade, moralidade e
direta, diretamente vinculados a União, Estados, Distrito publicidade no trato dos assuntos que lhe são afetos.
Federal ou Municípios; b) pessoas da administração indire- Trata-se de referência expressa aos princípios do art.
ta, isto é, autarquias, fundações públicas, empresas públi- 37, caput, CF. Não se menciona apenas o princípio da efi-
cas e sociedades de economia mista; c) pessoa cuja criação ciência, o que não significa que possa ser desrespeitado,
afinal, ele é abrangido indiretamente.
ou custeio o erário tenha contribuído com mais de 50% do
patrimônio ou receita naquele ano.
Art. 5° Ocorrendo lesão ao patrimônio público por ação
No parágrafo único, a lei enumera os sujeitos passivos
ou omissão, dolosa ou culposa, do agente ou de terceiro, dar-
secundários, que são: a) entidades que recebam subven-
se-á o integral ressarcimento do dano.
ção, benefício ou incentivo creditício pelo Estado; b) pes-
Integral ressarcimento do dano é a devolução corrigida
soa cuja criação ou custeio o erário tenha contribuído com
monetariamente de todos os valores que foram retirados
menos de 50% do patrimônio ou receita naquele ano.
do patrimônio público. No entanto, destaca-se que a lei
garante não só o integral ressarcimento, mas também a
Art. 2° Reputa-se agente público, para os efeitos desta devolução do enriquecimento ilícito: mesmo que a pessoa
lei, todo aquele que exerce, ainda que transitoriamente não cause prejuízo direto ao erário, mas lucre com um ato
ou sem remuneração, por eleição, nomeação, designação, de improbidade administrativa, os valores devem ir para os
contratação ou qualquer outra forma de investidura ou cofres públicos.
vínculo, mandato, cargo, emprego ou função nas enti-
dades mencionadas no artigo anterior. Art. 6° No caso de enriquecimento ilícito, perderá o
agente público ou terceiro beneficiário os bens ou valores
Art. 3° As disposições desta lei são aplicáveis, no que acrescidos ao seu patrimônio.
couber, àquele que, mesmo não sendo agente público, in- Estabelece o artigo 186 do Código Civil: “aquele que,
duza ou concorra para a prática do ato de improbidade por ação ou omissão voluntária, negligência ou imprudên-
ou dele se beneficie sob qualquer forma direta ou indireta. cia, violar direito e causar dano a outrem, ainda que exclu-
Os sujeitos ativos do ato de improbidade administra- sivamente moral, comete ato ilícito”. Este é o artigo central
tiva se dividem em duas categorias: os agentes públicos, do instituto denominado responsabilidade civil, que tem
definidos no art. 2°, e os terceiros, enumerados no art. 3°. como elementos: ação ou omissão voluntária (agir como
“Denomina-se sujeito ativo aquele que pratica o ato não se deve ou deixar de agir como se deve), culpa ou dolo
de improbidade, concorre para sua prática ou dele extrai do agente (dolo é a vontade de cometer uma violação de
vantagens indevidas. É o autor ímprobo da conduta. Em direito e culpa é a falta de diligência), nexo causal (relação
alguns casos, não pratica o ato em si, mas oferece sua co- de causa e efeito entre a ação/omissão e o dano causado) e
laboração, ciente da desonestidade do comportamento, dano (dano é o prejuízo sofrido pelo agente, que pode ser
Em outros, obtém benefícios do ato de improbidade, mui- individual ou coletivo, moral ou material, econômico e não
to embora sabedor de sua origem escusa”33. econômico). É a este instituto que se relacionam as sanções
33 CARVALHO FILHO, José dos Santos. Manual de direito admi- da perda de bens e valores e de ressarcimento integral do
nistrativo. 23. ed. Rio de Janeiro: Lumen juris, 2010. dano.

57
ATUALIDADES E DEVERES DOS SERVIDORES PÚBLICOS

O tipo de dano que é causado pelo agente ao Estado é Seção I


o material. No caso, há um correspondente financeiro direto, Dos Atos de Improbidade Administrativa que Im-
de modo que a condenação será no sentido de pagar ao Es- portam Enriquecimento Ilícito
tado o equivalente ao prejuízo causado.
O agente público e o terceiro que com ele concorra res- Art. 9° Constitui ato de improbidade administrativa
ponderão pelos danos causados ao erário público com seu importando enriquecimento ilícito auferir qualquer tipo de
patrimônio. Inclusive, perderão os valores patrimoniais acres- vantagem patrimonial indevida em razão do exercício de
cidos devido à prática do ato ilícito. O dano causado deverá cargo, mandato, função, emprego ou atividade nas entida-
ser ressarcido em sua totalidade. des mencionadas no art. 1° desta lei, e notadamente:
O grupo mais grave de atos de improbidade adminis-
Art. 7° Quando o ato de improbidade causar lesão ao pa- trativa se caracteriza pelos elementos: enriquecimento +
trimônio público ou ensejar enriquecimento ilícito, caberá a au- ilícito + resultante de uma vantagem patrimonial inde-
toridade administrativa responsável pelo inquérito representar vida + em razão do exercício de cargo, mandato, empre-
ao Ministério Público, para a indisponibilidade dos bens go, função ou outra atividade nas entidades do artigo
do indiciado. 1°:
Parágrafo único. A indisponibilidade a que se refere o caput a) O enriquecimento deve ser ilícito, afinal, o Estado
deste artigo recairá sobre bens que assegurem o integral ressar- não se opõe que o indivíduo enriqueça, desde que obede-
cimento do dano, ou sobre o acréscimo patrimonial resultante ça aos ditames morais, notadamente no desempenho de
do enriquecimento ilícito. função de interesse estatal.
Será oferecida representação ao Ministério Público para b) Exige-se que o sujeito obtenha vantagem patrimo-
que ele postule a indisponibilidade dos bens do indiciado, de nial ilícita. Contudo, é dispensável que efetivamente tenha
modo a garantir que ele não aliene seu patrimônio para não ocorrido dano aos cofres públicos (por exemplo, quando
reparar o ilícito. Por indisponibilidade entende-se bloquear os um policial recebe propina pratica ato de improbidade ad-
bens para que não sejam vendidos ou deteriorados, garan- ministrativa, mas não atinge diretamente os cofres públi-
tindo que o dano possa ser reparado quando da condenação
cos).
judicial. A indisponibilidade será suficiente para dar integral
c) É preciso que a conduta se consume, ou seja, que
ressarcimento ao dano ou retirar todo o acréscimo patrimo-
realmente exista o enriquecimento ilícito decorrente de
nial resultante do ilícito.
uma vantagem patrimonial indevida.
d) Como fica difícil imaginar que alguém possa se en-
Art. 8° O sucessor daquele que causar lesão ao patrimônio
riquecer ilicitamente por negligência, imprudência ou im-
público ou se enriquecer ilicitamente está sujeito às cominações
perícia, todas as condutas configuram atos dolosos (com
desta lei até o limite do valor da herança.
Caso o sujeito ativo faleça no curso da ação de impro- intenção).
bidade administrativa, os herdeiros arcarão com o dever de e) Não cabe prática por omissão.35
ressarcir o dano, claro, nos limites dos bens que ele deixar Entende Carvalho Filho36 que no caso do art. 9° o re-
como herança. quisito é o enriquecimento ilícito, ao passo que “o pres-
suposto exigível do tipo é a percepção de vantagem pa-
CAPÍTULO II trimonial ilícita obtida pelo exercício da função pública
Dos Atos de Improbidade Administrativa em geral. Pressuposto dispensável é o dano ao erário”. O
elemento subjetivo é o dolo, pois fica difícil imaginar que
Como não é possível ser desonesto sem saber que se está um servidor obtenha vantagem indevida por negligência,
agindo desta forma, o elemento comum a todas as hipóteses imprudência ou imperícia (culpa). Da mesma forma, é in-
de improbidade administrativa é o dolo, que consiste na in- compatível com a conduta omissiva, aceitando apenas a
tenção do agente em praticar o ato desonesto (alguns enten- comissiva (ação).
dem como inconstitucionais todas as referências a condutas ATENÇÃO: todas as condutas descritas abaixo são me-
culposas - inclusive parte do STJ). ros exemplos de condutas compostas pelos elementos
Os atos de improbidade administrativa foram divididos genéricos da cabeça do artigo. Com efeito, estando eles
em três grupos, nos artigos 9°, 10 e 11, conforme a gravidade presentes, não importa a ausência de dispositivo expresso
do ato, indo do grupo mais grave ao menos grave. A cada no rol abaixo.
grupo é aplicada uma espécie diferente de sanção no caso
de confirmação da prática do ato apurada na esfera admi- I - receber, para si ou para outrem, dinheiro, bem móvel
nistrativa. ou imóvel, ou qualquer outra vantagem econômica, direta
Nos três artigos do capítulo II, enquanto o caput traz as ou indireta, a título de comissão, percentagem, gratificação
condutas genéricas, os incisos delimitam condutas especí- ou presente de quem tenha interesse, direto ou indireto, que
ficas, que nada mais são do que exemplos de situações do possa ser atingido ou amparado por ação ou omissão de-
caput, logo, os incisos são uma relação meramente exemplifi- corrente das atribuições do agente público;
cativa34, sendo suficiente bem compreender como encontrar 35 SPITZCOVSKY, Celso. Direito Administrativo. 13. ed. São
os requisitos genéricos para fins de provas. Paulo: Método, 2011.
34 CARVALHO FILHO, José dos Santos. Manual de direito admi- 36 CARVALHO FILHO, José dos Santos. Manual de direito admi-
nistrativo. 23. ed. Rio de Janeiro: Lumen juris, 2010. nistrativo. 23. ed. Rio de Janeiro: Lumen juris, 2010.

58
ATUALIDADES E DEVERES DOS SERVIDORES PÚBLICOS

Significa receber qualquer vantagem econômica, in- VI - receber vantagem econômica de qualquer natureza,
clusive presentes, de pessoas que tenham interesse direto direta ou indireta, para fazer declaração falsa sobre medição
ou indireto em que o agente público faça ou deixe de fazer ou avaliação em obras públicas ou qualquer outro serviço,
alguma coisa. ou sobre quantidade, peso, medida, qualidade ou caracterís-
tica de mercadorias ou bens fornecidos a qualquer das enti-
II - perceber vantagem econômica, direta ou indireta, dades mencionadas no art. 1º desta lei;
para facilitar a aquisição, permuta ou locação de bem mó- Da mesma forma, é vedado o recebimento de vanta-
vel ou imóvel, ou a contratação de serviços pelas entidades gens para fazer declarações falsas na avaliação de obras e
referidas no art. 1° por preço superior ao valor de mercado; serviços em geral.
III - perceber vantagem econômica, direta ou indireta,
para facilitar a alienação, permuta ou locação de bem pú- VII - adquirir, para si ou para outrem, no exercício de
mandato, cargo, emprego ou função pública, bens de qual-
blico ou o fornecimento de serviço por ente estatal por preço
quer natureza cujo valor seja desproporcional à evolução do
inferior ao valor de mercado;
patrimônio ou à renda do agente público;
Tratam-se de espécies da conduta do inciso anterior,
A desproporção entre o rendimento percebido no
na qual o fim visado é permitir a aquisição, alienação, troca
exercício das funções e o patrimônio acumulado é um for-
ou locação de bem móvel ou imóvel por preço diverso ao te indício da percepção indevida de vantagens. Claro, se
de mercado. Percebe-se um ato de improbidade que cau- comprovada que a desproporção se deu por outros moti-
sa prejuízo direto ao erário. vos lícitos, não há ato de improbidade administrativa (por
No inciso II, o Estado que compra, troca ou aluga bem exemplo, ganhar na loteria ou receber uma boa herança).
móvel ou imóvel para sua utilização acima do preço de
mercado; no inciso III, um bem móvel ou imóvel perten- VIII - aceitar emprego, comissão ou exercer atividade de
cente ao Estado é vendido, trocado ou alugado em preço consultoria ou assessoramento para pessoa física ou jurídica
inferior ao de mercado. que tenha interesse suscetível de ser atingido ou amparado
por ação ou omissão decorrente das atribuições do agente
IV - utilizar, em obra ou serviço particular, veículos, má- público, durante a atividade;
quinas, equipamentos ou material de qualquer natureza, O agente público não pode trabalhar em funções in-
de propriedade ou à disposição de qualquer das entidades compatíveis com as que desempenha para o Estado, no-
mencionadas no art. 1° desta lei, bem como o trabalho de tadamente quando isso influenciar nas atitudes por ele to-
servidores públicos, empregados ou terceiros contratados madas no exercício das funções públicas. Afinal, aceitando
por essas entidades; uma posição que comprometa sua imparcialidade, o agente
Todo aparato dos órgãos públicos serve para atender prejudicará o interesse público.
ao Estado e, consequentemente, à preservação do bem
comum na sociedade. Logo, quando um servidor público IX - perceber vantagem econômica para intermediar a li-
utiliza esta estrutura material ou pessoal para atender aos beração ou aplicação de verba pública de qualquer natureza;
seus próprios interesses, causa prejuízo direto aos cofres Para que as verbas públicas sejam liberadas ou apli-
públicos e obtém uma vantagem indevida (a natural van- cadas há todo um procedimento estabelecido em lei, não
tagem decorrente do uso de algo que não lhe pertence). cabendo ao servidor violá-lo e muito menos receber van-
tagem por tal violação. Há improbidade, por exemplo, na
fraude em licitação.
V - receber vantagem econômica de qualquer natureza,
direta ou indireta, para tolerar a exploração ou a prática de
X - receber vantagem econômica de qualquer natureza,
jogos de azar, de lenocínio, de narcotráfico, de contrabando,
direta ou indiretamente, para omitir ato de ofício, providência
de usura ou de qualquer outra atividade ilícita,
ou declaração a que esteja obrigado;
Nenhum ato administrativo pode ser praticado ou A percepção de vantagem econômica para omitir qual-
omitido para facilitar condutas como lenocínio (explorar, quer ato que seja obrigado a praticar caracteriza ato de im-
estimular ou facilitar a prostituição), narcotráfico (envol- probidade administrativa.
ver-se em atividades no mundo das drogas, como venda
e distribuição), contrabando (importar ou exportar merca- XI - incorporar, por qualquer forma, ao seu patrimônio
doria proibida), usura (agiotagem, fornecer dinheiro a ju- bens, rendas, verbas ou valores integrantes do acervo patri-
ros absurdos) ou qualquer outra atividade ilícita. Se, ainda monial das entidades mencionadas no art. 1° desta lei;
por cima, se obter vantagem indevida pela tolerância da XII - usar, em proveito próprio, bens, rendas, verbas ou
prática do ilícito, resta caracterizado um ato de improbida- valores integrantes do acervo patrimonial das entidades
de administrativa da espécie mais grave, ora descrita neste mencionadas no art. 1° desta lei.
art. 9° em estudo. Como visto, todo o aparato material e financeiro pro-
piciado para o desempenho das funções públicas perten-
cem à máquina estatal e devem servir ao bem comum, não
cabendo a utilização em proveito próprio, o que gera uma
natural vantagem econômica, sob pena de incidir em im-
probidade administrativa.

59
ATUALIDADES E DEVERES DOS SERVIDORES PÚBLICOS

Seção II I - facilitar ou concorrer por qualquer forma para a in-


Dos Atos de Improbidade Administrativa que Cau- corporação ao patrimônio particular, de pessoa física ou
sam Prejuízo ao Erário jurídica, de bens, rendas, verbas ou valores integrantes do
acervo patrimonial das entidades mencionadas no art. 1º
Art. 10. Constitui ato de improbidade administrativa que desta lei;
causa lesão ao erário qualquer ação ou omissão, dolosa ou II - permitir ou concorrer para que pessoa física ou jurí-
culposa, que enseje perda patrimonial, desvio, apropriação, dica privada utilize bens, rendas, verbas ou valores integran-
malbaratamento ou dilapidação dos bens ou haveres das tes do acervo patrimonial das entidades mencionadas no
entidades referidas no art. 1º desta lei, e notadamente: art. 1º desta lei, sem a observância das formalidades legais
O grupo intermediário de atos de improbidade admi- ou regulamentares aplicáveis à espécie;
nistrativa se caracteriza pelos elementos: causar dano ao III - doar à pessoa física ou jurídica bem como ao ente
erário ou aos cofres públicos + gerando perda patrimo- despersonalizado, ainda que de fins educativos ou assistên-
nial ou dilapidação do patrimônio público. Assim como cias, bens, rendas, verbas ou valores do patrimônio de qual-
o artigo anterior, o caput descreve a fórmula genérica e quer das entidades mencionadas no art. 1º desta lei, sem
os incisos algumas atitudes específicas que exemplificam observância das formalidades legais e regulamentares apli-
o seu conteúdo.37 cáveis à espécie;
a) Perda patrimonial é o gênero, do qual são espécies: Todos os bens, rendas, verbas e valores que integram
desvio, que é o direcionamento indevido; apropriação, que a estrutura da administração pública somente devem ser
é a transferência indevida para a própria propriedade; mal- utilizados por ela. Por isso, não cabe a incorporação de seu
baratamento, que significa desperdício; e dilapidação, que patrimônio ao acervo de qualquer pessoa física ou jurídi-
se refere a destruição.38 ca e mesmo a simples utilização deve obedecer aos dita-
b) É preciso que seja causado dano a uma das pessoas mes legais. Quem agir, aproveitando da função pública,
do art. 1° da lei. No entanto, o enriquecimento ilícito é dis- de modo a permitir tais situações, incide em ato de im-
pensável. probidade administrativa, ainda que não receba nenhuma
c) O crime pode ser praticado por ação ou omissão. vantagem por seu ato (havendo enriquecimento ilícito, está
O objeto da tutela é a preservação do patrimônio pú- presente um ato do art. 9°, categoria mais grave).
blico, em todos seus bens e valores. O pressuposto exigível Aliás, nem ao menos importa se o ato é benéfico, por
é a ocorrência de dano ao patrimônio dos sujeitos passivos. exemplo, uma doação. O patrimônio público deve ser pre-
Este artigo admite expressamente a variante culpo- servado e sua transmissão/utilização deve obedecer a le-
sa, o que muitos entendem ser inconstitucional. O STJ, no gislação vigente.
REsp n° 939.142/RJ, apontou alguns aspectos da inconsti-
tucionalidade do artigo. Contudo, «a jurisprudência do STJ IV - permitir ou facilitar a alienação, permuta ou locação
consolidou a tese de que é indispensável a existência de de bem integrante do patrimônio de qualquer das entidades
dolo nas condutas descritas nos artigos 9º e 11 e ao menos referidas no art. 1º desta lei, ou ainda a prestação de serviço
de culpa nas hipóteses do artigo 10, nas quais o dano ao por parte delas, por preço inferior ao de mercado;
erário precisa ser comprovado. De acordo com o ministro V - permitir ou facilitar a aquisição, permuta ou locação
Castro Meira, a conduta culposa ocorre quando o agente de bem ou serviço por preço superior ao de mercado;
não pretende atingir o resultado danoso, mas atua com ne- Incisos diretamente correlatos aos incisos II e III do arti-
gligência, imprudência ou imperícia (REsp n° 1.127.143)»39. go anterior, exceto pelo fato do sujeito ativo não perceber
Para Carvalho Filho40, não há inconstitucionalidade na mo- vantagem indevida pela sua conduta. Aliás, é exatamente
dalidade culposa, lembrando que é possível dosar a pena pela falta deste elemento que o ato se enquadra na cate-
conforme o agente aja com dolo ou culpa. goria intermediária, e não mais grave, dentro da classifica-
O ponto central é lembrar que neste artigo não se exi- ção das improbidades.
ge que o sujeito ativo tenha percebido vantagens indevi-
das, basta o dano ao erário. Se tiver recebido vantagem VI - realizar operação financeira sem observância das
indevida, incide no artigo anterior. Exceto pela não per- normas legais e regulamentares ou aceitar garantia insufi-
cepção da vantagem indevida, os tipos exemplificados se ciente ou inidônea;
aproximam muito dos previstos nos incisos do art. 9°. VII - conceder benefício administrativo ou fiscal sem
a observância das formalidades legais ou regulamentares
aplicáveis à espécie;
37 SPITZCOVSKY, Celso. Direito Administrativo. 13. ed. São A realização de operações financeiras, como a libera-
Paulo: Método, 2011. ção de verbas e o investimento destas, e a concessão de
38 CARVALHO FILHO, José dos Santos. Manual de direito admi- benefícios são papéis muito importantes desempenhados
nistrativo. 23. ed. Rio de Janeiro: Lumen juris, 2010.
pelo agente público, que deverá cumprir estritamente a lei.
39 BRASIL. Superior Tribunal de Justiça. Improbidade ad-
ministrativa: desonestidade na gestão dos recursos públicos. Disponí-
vel em: <http://www.stj.gov.br/portal_stj/publicacao/engine.wsp?tmp. VIII - frustrar a licitude de processo licitatório ou de pro-
area=398&tmp.texto=103422>. Acesso em: 26 mar. 2013. cesso seletivo para celebração de parcerias com entidades
40 CARVALHO FILHO, José dos Santos. Manual de direito admi- sem fins lucrativos, ou dispensá-los indevidamente; (Altera-
nistrativo. 23. ed. Rio de Janeiro: Lumen juris, 2010. do pela Lei nº 13.019 de 31 de julho de 2014)

60
ATUALIDADES E DEVERES DOS SERVIDORES PÚBLICOS

Processo licitatório é aquele em que se realiza a lici- XV – celebrar contrato de rateio de consórcio público
tação, procedimento detalhado prescrito em lei pelo qual sem suficiente e prévia dotação orçamentária, ou sem ob-
o Estado contrata serviços, adquire produtos, aliena bens, servar as formalidades previstas na lei.
etc. A finalidade de cumprir o procedimento legal de forma A celebração de contratos de qualquer natureza com-
estrita é garantir a preservação do interesse da sociedade, promete diretamente o orçamento público, causando pre-
não cabendo ao agente público passar por cima destas re- juízo ao erário. Por isso, deve-se obedecer as prescrições
gras (Lei n° 8.666/93). legais que disciplinam a celebração de contratos adminis-
trativos, deliberando com responsabilidade a respeito das
IX - ordenar ou permitir a realização de despesas não contratações necessárias e úteis ao bem comum.
autorizadas em lei ou regulamento;
Todas as despesas que podem ser assumidas pelo Po- XVI - facilitar ou concorrer, por qualquer forma, para a
der Público encontram respectiva previsão em alguma lei incorporação, ao patrimônio particular de pessoa física ou
ou diretriz orçamentária. jurídica, de bens, rendas, verbas ou valores públicos trans-
feridos pela administração pública a entidades privadas
X - agir negligentemente na arrecadação de tributo ou mediante celebração de parcerias, sem a observância das
renda, bem como no que diz respeito à conservação do pa- formalidades legais ou regulamentares aplicáveis à espé-
trimônio público; cie;(Incluído pela Lei nº 13.019 de 31 de julho de 2014)
A arrecadação de tributos é essencial para a manuten-
ção da máquina estatal, não podendo o agente público ser XVII - permitir ou concorrer para que pessoa física ou
negligente (se omitir, deixar de ser zeloso) no que tange ao jurídica privada utilize bens, rendas, verbas ou valores pú-
levantamento desta renda. blicos transferidos pela administração pública a entidade
privada mediante celebração de parcerias, sem a observân-
XI - liberar verba pública sem a estrita observância das cia das formalidades legais ou regulamentares aplicáveis à
normas pertinentes ou influir de qualquer forma para a sua espécie; (Incluído pela Lei nº 13.019 de 31 de julho de 2014)
aplicação irregular;
Para que as verbas públicas sejam aplicadas é preciso XVIII - celebrar parcerias da administração pública com
obedecer o procedimento previsto em lei, preservando o entidades privadas sem a observância das formalidades le-
interesse estatal. gais ou regulamentares aplicáveis à espécie; (Incluído pela
Dos incisos VI a XI resta clara a marca desta categoria Lei nº 13.019 de 31 de julho de 2014)
intermediária de atos de improbidade administrativa: que
seja causado prejuízo ao erário, sem que o agente respon- XIX - frustrar a licitude de processo seletivo para cele-
sável pelo dano receba vantagem indevida. A questão é bração de parcerias da administração pública com entida-
preservar o interesse estatal, garantindo que os bens e ver- des privadas ou dispensá-lo indevidamente; (Incluído pela
bas públicas sejam corretamente utilizados, arrecadados e Lei nº 13.019 de 31 de julho de 2014)
investidos.
XX - agir negligentemente na celebração, fiscalização e
XII - permitir, facilitar ou concorrer para que terceiro se análise das prestações de contas de parcerias firmadas pela
enriqueça ilicitamente; administração pública com entidades privadas; (Incluído
Como visto, quanto o agente público obtém vantagem pela Lei nº 13.019 de 31 de julho de 2014)
própria, direta ou indireta, incide nas hipóteses mais graves
do artigo anterior. Caso concorde com o enriquecimento XXI - liberar recursos de parcerias firmadas pela ad-
ilícito de terceiro, por exemplo, seu superior hierárquico, ministração pública com entidades privadas sem a estrita
ou colabore para que ele ocorra, também cometerá ato de observância das normas pertinentes ou influir de qualquer
improbidade administrativa, embora de menor gravidade. forma para a sua aplicação irregular. (Incluído pela Lei nº
13.019 de 31 de julho de 2014)
XIII - permitir que se utilize, em obra ou serviço particu-
lar, veículos, máquinas, equipamentos ou material de qual- Seção III
quer natureza, de propriedade ou à disposição de qualquer Dos Atos de Improbidade Administrativa
das entidades mencionadas no art. 1° desta lei, bem como o que Atentam Contra os Princípios da
trabalho de servidor público, empregados ou terceiros con- Administração Pública
tratados por essas entidades.
Não se deve permitir que terceiros utilizem do aparato Art. 11. Constitui ato de improbidade administrativa
da máquina estatal, tanto material quanto pessoal, mesmo que atenta contra os princípios da administração pública
que não se obtenha vantagem alguma com tal concessão. qualquer ação ou omissão que viole os deveres de honesti-
dade, imparcialidade, legalidade, e lealdade às instituições,
XIV – celebrar contrato ou outro instrumento que tenha e notadamente:
por objeto a prestação de serviços públicos por meio da ges-
tão associada sem observar as formalidades previstas na lei;

61
ATUALIDADES E DEVERES DOS SERVIDORES PÚBLICOS

O grupo mais ameno de atos de improbidade adminis-


trativa se caracteriza pela simples violação a princípios ANOTAÇÕES
da administração pública, ou seja, aplica-se a qualquer
atitude do sujeito ativo que viole os ditames éticos do ser-
viço público. Isto é, o legislador pretende a preservação ___________________________________________________
dos princípios gerais da administração pública.41
a) O objeto de tutela são os princípios constitucionais; ___________________________________________________
b) Basta a vulneração em si dos princípios, sendo dis-
___________________________________________________
pensáveis o enriquecimento ilícito e o dano ao erário;
c) Somente é possível a prática de algum destes atos ___________________________________________________
com dolo (intenção);
d) Cabe a prática por ação ou omissão. ___________________________________________________
Será preciso utilizar razoabilidade e proporcionalida-
de para não permitir a caracterização de abuso de poder, ___________________________________________________
diante do conteúdo aberto do dispositivo. ___________________________________________________
Na verdade, trata-se de tipo subsidiário, ou seja, que
se aplica quando o ato de improbidade administrativa não ___________________________________________________
tiver gerado obtenção de vantagem indevida ou dano ao
erário. ___________________________________________________
I - praticar ato visando fim proibido em lei ou regula-
___________________________________________________
mento ou diverso daquele previsto, na regra de competência;
II - retardar ou deixar de praticar, indevidamente, ato ___________________________________________________
de ofício;
III - revelar fato ou circunstância de que tem ciência em ___________________________________________________
razão das atribuições e que deva permanecer em segredo;
IV - negar publicidade aos atos oficiais; ___________________________________________________
V - frustrar a licitude de concurso público;
___________________________________________________
VI - deixar de prestar contas quando esteja obrigado a
fazê-lo; ___________________________________________________
VII - revelar ou permitir que chegue ao conhecimento
de terceiro, antes da respectiva divulgação oficial, teor de ___________________________________________________
medida política ou econômica capaz de afetar o preço de
mercadoria, bem ou serviço. ___________________________________________________
VIII - descumprir as normas relativas à celebração, fis- ___________________________________________________
calização e aprovação de contas de parcerias firmadas pela
administração pública com entidades privadas. ___________________________________________________
IX - deixar de cumprir a exigência de requisitos de aces-
sibilidade previstos na legislação. ___________________________________________________
É possível perceber, no rol exemplificativo de condutas
___________________________________________________
do artigo 11, que o agente público que pratique qualquer
ato contrário aos ditames da ética, notadamente os origi- ___________________________________________________
nários nos princípios administrativos constitucionais, prati-
ca ato de improbidade administrativa. ___________________________________________________
Com efeito, são deveres funcionais: praticar atos visan-
do o bem comum, agir com efetividade e rapidez, manter ___________________________________________________
sigilo a respeito dos fatos que tenha conhecimento devido
___________________________________________________
a sua função, tornar públicos os atos oficiais, zelar pela boa
realização de atos administrativos em geral (como a reali- ___________________________________________________
zação de concurso público), prestar contas, entre outros.
___________________________________________________

___________________________________________________

___________________________________________________

___________________________________________________

___________________________________________________
41 SPITZCOVSKY, Celso. Direito Administrativo. 13. ed. São ___________________________________________________
Paulo: Método, 2011.

62
INFORMÁTICA

MS-Windows 10: conceito de pastas, diretórios, arquivos e atalhos, área de trabalho, área de transferência, manipula-
ção de arquivos e pastas, uso dos menus, programas e aplicativos, interação com o conjunto de aplicativos MS-Office
2016, ............................................................................................................................................................................................................................. 01
MS-Word 2016: estrutura básica dos documentos, edição e formatação de textos, cabeçalhos, parágrafos, fontes, colu-
nas, marcadores simbólicos e numéricos, tabelas, impressão, controle de quebras e numeração de páginas, legendas,
índices, inserção de objetos, campos predefinidos, caixas de texto.................................................................................................... 03
MSExcel 2016: estrutura básica das planilhas, conceitos de células, linhas, colunas, pastas e gráficos, elaboração de ta-
belas e gráficos, uso de fórmulas, funções e macros, impressão, inserção de objetos, campos predefinidos, controle de
quebras e numeração de páginas, obtenção de dados externos, classificação de dados.......................................................... 49
Correio Eletrônico: uso de correio eletrônico, preparo e envio de mensagens, anexação de arquivos................................ 81
Internet: navegação internet, conceitos de URL, links, sites, busca e impressão de páginas.................................................... 90
INFORMÁTICA

A Cortana permitirá que os usuários façam chamadas


MS-WINDOWS 10: CONCEITO DE no Skype, verifiquem o calendário, agendem e verifiquem
PASTAS, DIRETÓRIOS, ARQUIVOS E compromissos agendados, definam lembretes, configurem
ATALHOS, ÁREA DE TRABALHO, ÁREA o alarme, tomem notas e muito mais.
DE TRANSFERÊNCIA, MANIPULAÇÃO DE Infelizmente, sua disponibilidade no lançamento do
ARQUIVOS E PASTAS, USO DOS MENUS, Windows 10 em 29 de julho de 2015 deve variar depen-
dendo da região.
PROGRAMAS E APLICATIVOS, INTERAÇÃO
COM O CONJUNTO DE
APLICATIVOS MS-OFFICE 2016,

Windows 10

Windows 10 é a mais recente versão do sistema ope-


racional da Microsoft. Multiplataforma, o download do
software pode ser instalado em PCs (via ISO ou Windows
Update) e dispositivos móveis (Windows 10 mobile) como
smartphones e tablets. A versão liberada para computado-
res (Windows 10 e Windows 10 Pro) une a interface clássica
do Windows 7 com o design renovado do Windows 8 e 8.1,
criando um ambiente versátil capaz de se adaptar a telas
de todos os tamanhos e perfeito para uso com teclado e
mouse, como o tradicional desktop. Microsoft Edge
A terceira das 10 novidades no Windows 10 listadas
Podemos citar, dentre outras, as seguintes novidades: neste artigo é o navegador Microsoft Edge. O navegador
substituirá o Internet Explorer como o navegador padrão
Menu Iniciar do Windows.
O novo navegador foi desenvolvido como um app Uni-
O Windows 8 introduziu uma tela inicial que ocupava versal e receberá novas atualizações através da Windows
toda a área do monitor. Muitos usuários não conseguiram Store. Ele utiliza um novo mecanismo de renderização de
se adaptar muito bem e isto fez com que a Microsoft trou- páginas conhecido também pelo nome Edge, inclui supor-
xesse o menu Iniciar de volta no Windows 10. te para HTML5, Dolby Audio e sua interface se ajusta me-
Nesta nova versão do menu Iniciar, os usuários podem lhor a diferentes tamanhos de tela.
fixar tanto os aplicativos tradicionais como os aplicativos Com ele os usuários também poderão fazer anotações
disponibilizados através da Windows Store. em sites da Web (imagem abaixo) e até mesmo usar a Cor-
O menu também pode ser expandido automaticamen- tana. Basicamente a ideia é permitir que a Cortana navegue
te no modo Tablet para se comportar como a tela inicial do na Web com você e assim encontre informações úteis que
Windows 8 e 8.1. podem te ajudar.

Por exemplo, se você visita o site de um restaurante, a


Cortana Cortana encontrará informações como horários de funcio-
namento, telefone, endereço e até mesmo reviews.
A assistente pessoal Cortana foi introduzida pela Mi- Você também poderá fazer perguntas para a Cortana
crosoft no Windows Phone 8.1. Com o Windows 10, ela durante a navegação.
também estará presente nos PCs.

1
INFORMÁTICA

Áreas de trabalho virtuais 08 – Central de Ações


O suporte para áreas de trabalho virtuais é uma das A Central de Ações é a nova central de notificações do
10 novidades no Windows 10 listadas neste artigo. Com Windows 10. Ele funciona de forma similar à Central de Ações
este recurso, os usuários podem manter múltiplas áreas de do Windows Phone 8.1 e também oferece acesso rápido a
trabalho com programas específicos abertos em cada uma recursos como modo Tablet, Bloqueio de Rotação e VPN.
delas. Por exemplo, você pode deixar uma janela do In-
ternet Explorer visível em uma área de trabalho enquanto
trabalha no Word em outra.
Vale lembrar que este recurso já foi oferecido no Win-
dows XP através de um Power Toy chamado Virtual Desk-
top Manager. Um detalhe é que este PowerToy suporta no
máximo de quatro áreas de trabalho virtuais, enquanto que
no Windows 10 é possível criar muitas (20+).

Continuum
O modo Continuum foi criado para uso em aparelhos
híbridos que combinam tablet e notebook. Com este modo
o usuário pode alternar facilmente entre o uso do híbrido
como tablet e como notebook, basicamente combinando a
simplicidade do tablet com a experiência de uso tradicional. Novos aplicativos Email e Calendário
Os novos aplicativos Email e Calendário trazem uma
interface melhorada e oferecem mais recursos do que as
atuais versões para Windows 8.1.
No caso do aplicativo Email, ele conta com um editor
de texto mais rico baseado no app Universal do Word para
Windows 10 e também permite que o usuário utilize um
plano de fundo personalizado para o app.

Quando o usuário usa um híbrido como o HP Pavillion


x360 ou o Lenovo YOGA, por exemplo, o Windows 10 pode
ser configurado para que entre no modo Tablet automati-
camente. Com isso não é necessário perder tempo mexen-
do nas configurações quando for necessário usar o híbrido
como tablet ou como notebook.
O modo Continuum também estará presente no Win-
dows 10 Mobile, a versão do novo sistema operacional da
Microsoft para smartphones e tablets pequenos.
Durante uma demonstração em abril, a Microsoft co-
nectou um smartphone Lumia a um monitor e a um teclado Já o app Calendário ganhou uma interface bem mais
Bluetooth para usar o aparelho em um modo que oferece intuitiva que a da versão para Windows 8.1, permitindo que
mais produtividade. Com isso o smartphone basicamente o usuário crie compromissos e alterne entre modos dia/
se transformou em um PC com área de trabalho e tudo. semana/mês mais facilmente.

Nova Windows Store


Além de oferecer aplicativos Universais e jogos, a nova
Windows Store inclui a nova seção Filmes & TV. A Micro-
soft também já confirmou que ela também oferecerá apli-
cativos Win32 tradicionais.
Outra novidade é a nova “Windows Store for Business”,
que oferecerá aplicativos para usuários finais e aplicativos pri-
vados voltados para ambientes corporativos e organizações.
Por exemplo, uma escola poderá definir um conjunto
específico de aplicativos que serão instalados nos compu-
tadores disponíveis para os alunos.

2
INFORMÁTICA

Novo Painel de Controle moderno O PowerPoint é o programa do Office que se utiliza


A última das 10 novidades no Windows 10 listadas nes- para criar e exibir apresentações visuais. A sua base está no
te artigo é o novo Painel de Controle moderno do sistema desenvolvimento de dispositivos multimídia, que podem in-
operacional. Ele oferece bem mais opções que a versão cluir texto, imagens, vídeos e som.
moderna presente no Windows 8.1, o que é uma boa notí- A administração da informação pessoal e as mensagens
cia para os usuários. de correio electrónico podem ser geridos a partir do Outlook.
O seu principal ponto forte é o correio eletrônico, embora
também disponha de um calendário e de um diretório de
contatos.
O Microsoft Office é um programa que não dispensa de
licença. Por isso, deve ser comprado pelos utilizadores que
pretendam usufruir dos serviços propostos.
Os programas do pacote office são amplamente utiliza-
dos no ambiente corporativo, mas na maioria dos compu-
tadores domésticos, você vai encontrar também pelo me-
nos um processador de texto, um editor de planilhas e um
programa de apresentações, representados pelos populares
Word, Excel e Powerpoint, respectivamente.
Dominar o básico destes programas não é difícil, com
um pouco de pratica e dedicação, você já consegue criar do-
cumentos, relatórios e planilhas simples, bem como algumas
apresentações a partir de modelos prontos.
Fonte: http://www.baboo.com.br/windows/10-novida-
des-no-windows-10/ OFFICE 2016
Lançado em setembro de 2015, é a versão mais recente,
que veio a substituir o Office2013. Entre os novos recursos
MS-WORD 2016: ESTRUTURA BÁSICA DOS estão a possibilidade de criar, abrir, editar e salvar arquivos
DOCUMENTOS, EDIÇÃO E FORMATAÇÃO na nuvem diretamente de seu desktop. Há também o recurso
DE TEXTOS, CABEÇALHOS, PARÁGRAFOS, “Diga-me”- uma caixa de busca localizada na porção superior
da tela, a partir da qual você pode simplesmente digitar o
FONTES, COLUNAS, MARCADORES
nome de ferramentas e comandos para encontrá-los facil-
SIMBÓLICOS E NUMÉRICOS, TABELAS, mente. Agora os aplicativos do office permitem colaboração
IMPRESSÃO, CONTROLE DE QUEBRAS E de arquivos, com edição em tempo real, permitindo que mais
NUMERAÇÃO DE PÁGINAS, LEGENDAS, de um usuário editem ao mesmo tempo, desde que eles es-
ÍNDICES, INSERÇÃO DE OBJETOS, CAMPOS tejam conectados on-line em suas contas Microsoft.
PREDEFINIDOS, CAIXAS DE TEXTO. A interface do office 2016 também recebeu uma sutil
repaginada. As mudanças visuais são quase imperceptíveis,
mas o visual está mais plano, seguindo a tendência do win-
MICROSOFT OFFICE 2016 dows 10. A porção superior da Ribbon dos aplicativos rece-
Microsoft Office é uma suite de aplicações (programas beu uma cor solida, que varia conforme o aplicativo: azul,
de computador) desenvolvida pela Microsoft Corp. (uma verde, roxo etc.
empresa norte-americana fundada em 1975). Trata-se de Há um cinza bem claro na faixa de opções para separar
um conjunto de programas informáticos/software que rea- o documento aberto das ferramentas disponíveis na porção
lizam tarefas de escritório, isto é, que permitem automati- superior.
zar e otimizar as atividades do dia-a-dia de um escritório. Em caso de telas sensíveis ao toque, aceita-se a possibi-
A primeira versão do Microsoft Office foi lançada em lidade de ajustar a interface dos programas para ser mani-
1989 com dois packs básicos: um formado pelo Microsoft pulada pelos dedos. Na verdade, o recurso cria mais espaça-
Word, pelo Microsoft Excel e pelo Microsoft PowerPoint, e mento entre os botões, para facilitar o toque, mesmo nessa
outro ao qual foram acrescentados os programas Microsoft condição, as funcionalidades continuam as mesmas.
Access e Schedule Plus.
O Word é um dos programas mais populares que fa- Trabalhar em equipe ficou mais fácil
zem parte do Microsoft Office. Consiste num processador Com o Office 2016, ficou mais fácil compartilhar docu-
de textos que inclui um corretor ortográfico, um dicionário mentos e trabalhar com outras pessoas ao mesmo tempo.
de sinónimos e a possibilidade de trabalhar com diversas
fontes (tipos de letras). Colabore em documentos
O Excel, por sua vez, é composto por folhas de cálculo. Veja as edições das outras pessoas com a coautoria no
Tem por principal característica a possibilidade de realizar Word, no PowerPoint e no OneNote. O histórico de versão foi
cálculos aritméticos de forma automática, facilitando assim aperfeiçoado e agora permite conferir os instantâneos das
o desenvolvimento dos balanços e das demonstrações fi- versões anteriores do documento durante o processo de
nanceiras. edição.

3
INFORMÁTICA

Compartilhamento simplificado Hello


Compartilhe a partir do próprio documento com ape- É só dizer “Hello” uma única vez, e o Windows faz lo-
nas um clique. Se preferir, use os novos anexos modernos gon no PC e no Office.
do Outlook: é só anexar os arquivos do OneDrive e confi-
gurar automaticamente a permissão. Você não precisa nem Cortana
sair do Outlook. A Cortana ajuda você a fazer seu trabalho no Office.
Integrá-la ao Office 365 facilita diversas tarefas, como a
Office em todos os seus dispositivos preparação de reuniões.
Revise, edite, analise e apresente documentos do Of-
fice 2016 em qualquer um dos seus dispositivos, seja um Aplicativos móveis do Office
PC, um Mac ou telefones e tablets Windows, Apple® e An- Os aplicativos móveis do Office no Windows 10 são rá-
droid™. pidos, fáceis de usar com a tela de toque e otimizados para
a produtividade em praticamente qualquer lugar.
Colabora com você
O Office 2016 colabora com a sua produtividade ofe-
recendo jeitos novos e mais rápidos de alcançar os resulta- Office pelo melhor preço
dos que você quer. Com os planos de assinatura flexíveis do Office 365,
você escolhe a opção ideal para as suas necessidades. As-
Encontre os comandos com o Diga-me sine o plano individual ou um plano para toda a família.
É só dizer para o Word, o Excel ou o PowerPoint o que
quer fazer e o Diga-me leva você até o comando. Instale os aplicativos do Office
O Office 365 vem com os novos aplicativos do Offi-
Verifique os fatos usando a Pesquisa Inteligente da pla- ce 2016 para PC e Mac, como Word, Excel, PowerPoint,
taforma Bing Outlook e OneNote.
A Pesquisa Inteligente usa os termos destacados por
você e outras informações do contexto do documento para 1 TB de armazenamento em nuvem do OneDrive
mostrar os resultados da pesquisa na Web dentro do pró- O OneDrive deixa ao seu alcance aquilo que é impor-
prio documento. tante, como amigos, parentes, projetos e arquivos, em pra-
ticamente qualquer lugar, em qualquer dispositivo.
Ideias a poucos cliques de distância
Usando a previsão com um clique, você transforma Acesso ao suporte técnico gratuito
rapidamente o histórico de dados em uma análise de ten- Precisa de ajuda com o Office 2016? Todo assinante do
dências futuras. Os novos gráficos ajudam a visualizar da- Office 365 recebe suporte técnico gratuito de especialistas
dos complexos. treinados pela Microsoft.

O Office acompanha você


Do trabalho à sua lanchonete preferida, acompanhe
aquilo que importa: amigos, parentes e projetos, em todos
os seus dispositivos.

Aplicativos do Office otimizados para o toque


Toque para ler, editar, mudar o zoom e navegar. Tome
notas ou faça anotações usando a tinta digital.

Tudo em um único lugar com o armazenamento em


nuvem do OneDrive
Salve o arquivo no armazenamento em nuvem e tro-
que de dispositivo sem perder o embalo. Os aplicativos do
Office abrem o documento do jeitinho que você deixou,
em qualquer um dos seus dispositivos.

Perfeito para o Windows 10


Juntos, o Office 2016 e o Windows 10 formam a solu-
ção mais completa do mundo para resolver o que for ne-
cessário.

4
INFORMÁTICA

EDITOR DE TEXTO MS WORD 2016

Novidades do Word 2016 para Windows1


O Word 2016 para Windows tem todas as funcionalidades e recursos conhecidos, com alguns aprimoramentos e novos
recursos do Office 2016.
Veja alguns dos novos recursos.
Realize ações rapidamente com o recurso Diga-me
Observe que há uma caixa de texto na Faixa de Opções do Word 2016 com a mensagem O que você deseja fazer. Esse
é um campo de texto no qual você insere palavras ou frases relacionadas ao que deseja fazer e obtém rapidamente os re-
cursos que pretende usar ou as ações que deseja realizar. Se preferir, use o Diga-me para encontrar ajuda sobre o que está
procurando ou para usar a Pesquisa Inteligente para pesquisar ou definir o termo que você inseriu.

Trabalhe em grupo em tempo real


Ao armazenar um documento online no OneDrive ou no SharePoint e compartilhá-lo com colegas que usam o Word
2016 ou Word Online, vocês podem ver as alterações uns dos outros no documento durante a edição. Após salvar o do-
cumento online, clique em Compartilhar para gerar um link ou enviar um convite por email. Quando seus colegas abrem o
documento e concordam em compartilhar automaticamente as alterações, você vê o trabalho em tempo real.

1 Fonte: https://support.office.com/pt-br/article/Novidades-do-Word-2016-para-Windows-4219dfb5-23fc-4853-95aa-b13a674a6670?ui=pt-
-BR&rs=pt-BR&ad=BR

5
INFORMÁTICA

Ideias para o trabalho que está realizando


A Pesquisa Inteligente da plataforma Bing apresenta as pesquisas diretamente no Word 2016. Quando você seleciona
uma palavra ou frase, clica com o botão direito do mouse sobre ela e escolhe Pesquisa Inteligente, o Painel de ideias é
exibido com as definições, os artigos Wiki e as principais pesquisas relacionadas da Web.

6
INFORMÁTICA

Equações à tinta
Incluir equações matemáticas ficou muito mais fácil. Vá até Inserir > Equação > Equação à Tinta sempre que desejar
incluir uma equação matemática complexa em um documento. Se tiver um dispositivo sensível ao toque, use o dedo ou
uma caneta de toque para escrever equações matemáticas à mão, e o Word 2016 vai convertê-las em texto. Caso não tenha
um dispositivo sensível ao toque, use o mouse para escrever. Você pode também apagar, selecionar e fazer correções à
medida que escreve.

Histórico de versões melhorado


Vá até Arquivo > Histórico para conferir uma lista completa de alterações feitas a um documento e para acessar versões
anteriores.

Compartilhamento mais simples


Clique em Compartilhar para compartilhar seu documento com outras pessoas no SharePoint, no OneDrive ou no One-
Drive for Business ou para enviar um PDF ou uma cópia como um anexo de email diretamente do Word.

Formatação de formas mais rápida


Quando você insere formas da Galeria de Formas, é possível escolher entre uma coleção de preenchimentos predefini-
dos e cores de tema para aplicar rapidamente o visual desejado.

7
INFORMÁTICA

Interface

Tela de trabalho do MS Word


No cabeçalho de nosso programa temos a barra de títulos do documento ,
que como é um novo documento apresenta como título “Documento1”. Na esquerda temos a Barra de acesso rápido,
que permite acessar alguns comandos mais rapidamente como salvar, desfazer. Você pode personalizar essa
barra, clicando no menu de contexto (flecha para baixo) à direita dela.

8
INFORMÁTICA

Mais à esquerda tem a ABA Arquivo.

Através dessa ABA, podemos criar novos documentos, abrir arquivos existentes, salvar documentos, imprimir, preparar
o documento (permite adicionar propriedades ao documento, criptografar, adicionar assinaturas digitais, etc.).

ABAS

Os comandos para a edição de nosso texto agora ficam agrupadas dentro destas guias. Dentro destas guias temos os
grupos de ferramentas, por exemplo, na guia Página inicial, temos “Fonte”, “Parágrafo”, etc., nestes grupos fica visíveis para
os usuários os principais comandos, para acessar os demais comandos destes grupos de ferramentas, alguns destes grupos
possuem pequenas marcações na sua direita inferior.

9
INFORMÁTICA

O Word possui também guias contextuais quando determinados elementos dentro de seu texto são selecionados, por
exemplo, ao selecionar uma imagem, ele criar na barra de guias, uma guia com a possibilidade de manipulação do elemen-
to selecionado.

Explore a galeria de documentos

A galeria de documentos é o local onde você pode criar um documento em branco ou usar um modelo predefinido. A
galeria fica disponível ao abrir o Word ou você pode acessá-la escolhendo Arquivo > Novo se estiver trabalhando em um
documento existente.

10
INFORMÁTICA

Explorar a faixa de opções

Saiba mais sobre a caixa Diga-me

Diga-Me é uma nova ferramenta de pesquisa e está disponível no Word, no PowerPoint e no Excel 2016. Ela exibe os
comandos necessários quando você digita o que deseja fazer. Por exemplo, digite “configurações de fonte” na janela Diga-
me o que você quer fazer. Em seguida, escolha uma das sugestões exibidas ou escolha Obter Ajuda sobre “configurações
de fonte” para abrir o visualizador da Ajuda.

11
INFORMÁTICA

Faça um tour do Word 2016

Quando o Word abrir, clique em Faça um tour ou digite “Bem-vindo ao Word” na caixa Pesquisar modelos online. O
modelo Bem-vindo ao Word é aberto.
Este documento permite que você explore cinco áreas:
- Usar guias dinâmicas de layout e alinhamento
- Colaborar no Modo de Exibição Marcação Simples
- Inserir Imagens e Vídeos Online
- Desfrutar da Leitura
- Editar conteúdo em PDF no Word

Criando um documento

Quando você abre o Word, a galeria de documentos é exibida, permitindo que você escolha o modelo de documento
em branco ou um dos vários outros modelos.

12
INFORMÁTICA

Utilizando um modelo

Quando você inicia um aplicativo do Office, como o Word, o Excel, o PowerPoint, o Visio ou o Access, a primeira coisa
que você vê é uma lista de modelos que podem ser usados para criar seus arquivos e documentos.
Para encontrar modelos para os aplicativos do Office a qualquer momento, selecione Arquivo > Novo. Veja um exem-
plo de como isso é exibido no Word:

Insira uma pesquisa para o tipo de modelo que você está procurando na caixa de pesquisa que diz Procurar modelos on-
line. Para navegar pelos tipos de modelos populares, selecione qualquer uma das palavras-chave abaixo da caixa de pesquisa.

Selecione a miniatura de um modelo para ver uma visualização maior de como ele é. Você pode usar as setas em ambos os
lados da visualização para rolar pelos modelos relacionados. Depois de encontrar um modelo de que você gosta, selecione Criar.

DICA : Se você usa um modelo com frequência, pode fixá-lo para que esteja sempre à mão quando você inicia o apli-
cativo do Office. Basta selecionar o ícone de pino que aparece abaixo da miniatura na lista de modelos.

13
INFORMÁTICA

Modos de documento e compatibilidade


Quando você abre um documento no Word 2016, ele se encontra em um destes modos:
- Modo Word 2013-2016
- Modo de Compatibilidade do Word 2010
- Modo de Compatibilidade do Word 2007
- Modo de Compatibilidade do Word 97-2003

Caso você veja o Modo de Compatibilidade na barra de título, saiba como descobrir em que modo você está:
- Clique em Arquivo > Informações.
- Na seção Inspecionar Documento, clique em Verificar Problemas e em Verificar Compatibilidade.

Clique em Selecionar versões a exibir para verificar se há uma marca de seleção exibida ao lado do nome do modo em
que o documento se encontra.

Ajustar recuos e espaçamento no Word


Quando você quiser fazer alterações precisas nos recuos e no espaçamento ou quando quiser fazer várias alterações de
uma só vez, use as configurações na guia Recuos e Espaçamento na caixa de diálogo Parágrafo.
Selecione o texto que deseja ajustar.
Clique em Layout e clique na seta para o iniciador da caixa de diálogo no grupo Parágrafo.

Na guia  Recuos e Espaçamento, escolha as configurações (veja abaixo os detalhes de cada configuração) e clique
em OK.

Opções da caixa de diálogo Parágrafo


Escolha uma destas opções na caixa de diálogo  Parágrafo. Na parte inferior da caixa de diálogo, a caixa  Visualiza-
ção mostra a aparência das opções antes que você as aplique.

Geral
Escolha À Esquerda para alinhar o texto à esquerda com uma margem
Alinhamento
direita irregular (ou use o atalho de teclado CTRL+L).
EscolhaCentralizar para centralizar o texto com uma borda esquerda e
direita irregulares (CTRL+E).
EscolhaÀ Direita para alinhar o texto à direita com uma margem esquerda
irregular (CTRL+R).
EscolhaJustificar para alinhar o texto à esquerda e à direita, adicionando
espaço entre as palavras (CTRL+J).
O nível no qual o parágrafo aparece no modo de exibição de Estrutura
Nível da estrutura de tópicos
de Tópicos.
Escolha Recolhido por padrão se quiser que o documento seja aberto
com os títulos recolhidos por padrão.

14
INFORMÁTICA

Recuo
Move-se no lado esquerdo do parágrafo de acordo com quantidade que
Para a Esquerda
você escolher.
Move-se no lado direito do parágrafo de acordo com quantidade que você
Para a Direita
escolher.
Especial Escolha Primeira linha > Por para recuar a primeira linha de um parágrafo.
Escolha Deslocamento > Por para criar um recuo deslocado.
Quando você escolher isso, Esquerda e Direita tornam-se Dentro e Fora.
Espelhar recuos
Isso é para impressão de estilo de livro.
Espaçamento
Antes Ajusta a quantidade de espaço antes de um parágrafo.
Depois Ajusta a quantidade de espaço após um parágrafo
Espaçamento entre linhas Escolha Simples para texto com espaçamento simples.
Escolha 1,5 linhas para definir o espaçamento do texto uma vez e meia o do
espaçamento único.
Escolha Duplo para texto com espaçamento duplo.
Escolha Pelo menos > Em para definir a quantidade mínima de espaçamento
necessário para acomodar a maior fonte ou gráfico na linha.
Escolha Exatamente > Em para definir o espaçamento de linha fixa, expresso
em pontos. Por exemplo, se o texto estiver em fonte de 10 pontos, você pode
especificar 12 pontos como o espaçamento entre linhas.
EscolhaMúltiplo > Em para definir o espaçamento de linha como um múltiplo
expresso em números maiores que 1. Por exemplo, definir o espaçamento
entre linhas como 1,15 aumentará o espaço em 15% e definir o espaçamento
entre linhas como 3 aumentará o espaço em 300% (espaçamento triplo).
Escolha Não adicionar espaço entre parágrafos do mesmo estilo quando não
Não adicionar …
quiser espaço adicional entre os parágrafos.

Se você quiser salvar as configurações como padrão, clique em Definir como padrão.

Clicar em Guias… abre a caixa de diálogo Guias, onde você pode definir precisamente as guias.

Inserir imagens
As imagens podem ser inseridas (ou copiadas) a partir de vários locais diferentes, inclusive de um computador, de uma
fonte online como o Bing.com ou de uma página da Web.
Inserir uma imagem a partir de um computador
Clique no local em que deseja inserir a imagem no documento.
Clique em Inserir > Imagens.

Navegue até a imagem que você deseja inserir, selecione-a e clique em Inserir.

OBSERVAÇÃO: Por padrão, o Word insere a imagem em um documento. Mas você pode, de forma alternativa, vincular
seu documento à imagem para reduzir seu tamanho. Para fazê-lo, na caixa de diálogo Inserir Imagem, clique na seta ao lado
de Inserir e clique em Vincular ao Arquivo.

15
INFORMÁTICA

Inserir imagem a partir de uma fonte online


Caso não tenha uma imagem ideal no seu computador, experimente inserir uma a partir de uma fonte online, como o
Bing ou o Flickr.
Clique no local onde deseja inserir a imagem no documento.
Clique em Inserir > Imagens Online.

Na caixa de pesquisa, digite uma palavra ou frase que descreva a imagem desejada e pressione Enter.
Na lista de resultados, clique na imagem desejada e em Inserir.
Inserir uma imagem a partir de uma página da Web
Abra seu documento.
Na página da Web, clique com o botão direito do mouse na imagem que deseja e clique em Copiar.
No seu documento, clique com o botão direito do mouse no local que deseja inserir a imagem e clique em Colar.

Inserir uma tabela


Para inserir rapidamente uma tabela, clique em Inserir > Tabela e mova o cursor sobre a grade até realçar o número
correto de colunas e linhas desejado.

Clique na tabela exibida no documento. Caso seja necessário fazer ajustes, você poderá adicionar colunas e linhas em
uma tabela, excluir linhas ou colunas ou mesclar células.
Quando você clica na tabela, as Ferramentas de Tabela são exibidas.

16
INFORMÁTICA

Use as Ferramentas de Tabela para escolher diferentes cores, estilos de tabela, adicionar uma borda a uma página ou
remover bordas de uma tabela. Você pode até mesmo inserir uma fórmula para fornecer a soma de uma coluna ou linha
de números em uma tabela.
Se você tem um texto que ficará melhor em uma tabela, o Word pode convertê-lo em uma tabela.
Inserir tabelas maiores ou tabelas com comportamentos de largura personalizada
Para obter tabelas maiores e mais controle sobre as colunas, use o comando Inserir Tabela.

Assim, você pode criar uma tabela com mais de dez colunas e oito linhas, além de definir o comportamento de largura
das colunas.
Clique em Inserir > Tabela > Inserir Tabela.
Defina o número de colunas e linhas.

Na seção Comportamento de Ajuste Automático, há três opções para configurar a largura das colunas:
Largura fixa da coluna: você pode deixar o Word definir automaticamente a largura das colunas com Automático ou
pode definir uma largura específica para todas as colunas.
Ajustar-se automaticamente ao conteúdo: isso cria colunas muito estreitas que são expandidas conforme você adiciona
conteúdo.
Ajustar-se automaticamente à janela: isso mudará automaticamente a largura de toda a tabela para ajustar-se ao ta-
manho de seu documento.
Se quiser que as tabelas criadas tenham uma aparência semelhante à da tabela que você está criando, marque a caixa
Lembrar dimensões para novas tabelas.

17
INFORMÁTICA

Projetar sua própria tabela


Se quiser ter mais controle sobre a forma das colunas e linhas de sua tabela ou algo diferente de uma grade básica, a
ferramenta Desenhar Tabela ajuda a desenhar exatamente a tabela que você deseja.

Você mesmo pode desenhar linhas diagonais e células dentro das células.
Clique em Inserir > Tabela > Desenhar Tabela. O ponteiro é alterado para um lápis.
Desenhe um retângulo para fazer as bordas da tabela. Depois, desenhe linhas para as colunas e linhas dentro do re-
tângulo.

Para apagar uma linha, clique na guia Layout de Ferramentas de Tabela, clique em Borracha e clique na linha que você
quer apagar.

18
INFORMÁTICA

Adicionar um cabeçalho ou rodapé


Você pode adicionar muito mais além de números de página aos seus cabeçalhos ou rodapés. Mas para começar, veja
como criar e personalizar um cabeçalho ou rodapé simples.
Clique em Inserir e depois clique em Cabeçalho ou Rodapé.

Dezenas de layouts internos são exibidos. Percorra-os e clique naquele que você deseja.
O espaço de cabeçalho e rodapé será aberto em seu documento, junto com as Ferramentas de Cabeçalho e Rodapé.
Você precisa fechar as Ferramentas de Cabeçalho e Rodapé para poder editar o corpo do seu documento novamente.
Digite o texto desejado no cabeçalho ou no rodapé. A maioria dos cabeçalhos e rodapés tem texto do espaço reserva-
do (por exemplo, “Título do documento”) que você pode digitar diretamente sobre.
DICA: Escolha entre as Ferramentas de Cabeçalho e Rodapé para adicionar mais ao seu cabeçalho ou rodapé, como
data e hora, uma imagem e o nome do autor ou outras informações do documento. Você também pode selecionar opções
para cabeçalhos diferentes em páginas pares e ímpares, além de indicar que não deseja que o cabeçalho ou rodapé apareça
na primeira página.
Quando terminar, clique em Fechar Cabeçalho e Rodapé.

DICA: Sempre que você quiser abrir as Ferramentas de Cabeçalho e Rodapé, clique duas vezes dentro da área de ca-
beçalho ou rodapé.

Adicionar números de página a um cabeçalho ou rodapé no Word


OBSERVAÇÃO: Se você não tiver um cabeçalho ou rodapé, ou se você tiver um cabeçalho ou rodapé que você não
queira manter, para adicionar rapidamente números de página, clique em Inserir > Número de Página e selecione o tipo de
número da página desejado. Isso substituirá qualquer cabeçalho ou rodapé existente.
Se o documento já tem cabeçalhos ou rodapés, você pode usar o código de campo de Número da Página para adicio-
nar números de página sem substituir os cabeçalhos ou rodapés.
Com mais uma etapa, você poderá exibir o número de página como Página X de Y.
Usar o código Campo de página para adicionar números de página a um cabeçalho ou rodapé
Clique duas vezes na área do cabeçalho ou rodapé (próxima à parte superior da ou inferior da página). Isso abre a
guiaDesign em Ferramentas de Cabeçalho e Rodapé.

19
INFORMÁTICA

Posicione o cursor onde você deseja adicionar o número da página. Para colocar o número da página no centro ou no
lado direito da página, faça o seguinte:
Para colocar o número de página no centro, na guia Design, clique em Inserir Tabulação de Alinhamento >Centralizar > OK.
Para colocar o número de página no lado direito da página, na guia Design, clique em Inserir Tabulação de Alinhamen-
to > Direita > OK.
Na guia Inserir, clique em Partes Rápidas e Campo.

Na lista Nomes de campos, clique em Página e em OK.

OBSERVAÇÕES: Para mostrar os números de página como Página X de Y, faça o seguinte:


Digite de após o número de página que você acabou de adicionar.
Na guia Inserir, clique em Partes Rápidas e Campo.
Na lista Nomes de campos, clique em NumPages e em OK.
Para alterar o formato de numeração, na guia Design (em Ferramentas de Cabeçalho e Rodapé), clique em Número de
Página > Formatar Números de Página.

Para retornar ao corpo do documento, clique em Ferramentas de Cabeçalho e Rodapé > Fechar Cabeçalho e Rodapé.

Adicionar números de página no Word


Clique em Inserir > Número de Página, clique em um local (como o Início da Página) e escolha um estilo. O Word nu-
mera as páginas de forma automática.

20
INFORMÁTICA

Quando concluir, clique em Fechar Cabeçalho e Rodapé ou clique duas vezes em qualquer lugar fora da área do cabe-
çalho ou do rodapé.

DICA : O Word numera as páginas de forma automática, mas você pode alterar essa opção se preferir. Por exemplo,
caso não pretenda exibir o número da página na primeira página do documento, clique duas vezes ou dê um toque duplo
na parte superior ou inferior da página para abrir as ferramentas de cabeçalho e rodapé na guia Design e marque a cai-
xa Primeira Página Diferente. Escolha Inserir > Número da Página > Formatar Números de Página para saber mais.

Salvar um documento no Word 2016


O local escolhido para salvar seu documento depende da forma como você planeja usá-lo. Para acessar um documento
em praticamente qualquer lugar, compartilhá-lo com outras pessoas ou trabalhar em conjunto com outras pessoas em
tempo real, salve-o online. Mas onde você deve salvá-lo? Veja algumas dicas para ajudá-lo a decidir:
Use o site de equipe do OneDrive for Business ou do SharePoint para documentos que serão usados por seus colegas.
Escolha uma pasta pessoal do OneDrive para documentos particulares que somente você pode ver ou que deseja com-
partilhar com seus amigos e familiares.
Se pretende trabalhar com um documento no computador que você está usando atualmente, salve em uma pasta
neste computador.

21
INFORMÁTICA

Decida onde salvar seu documento

Use a tabela a seguir para ajudá-lo a escolher um local para salvar seu documento:

LOCAL PARA
USE ESTE PROCEDIMENTO QUANDO QUISER...
SALVAR
Salvar um documento comercial que você provavelmente desejará compartilhar mais tarde
OneDrive
com parceiros de fora de sua equipe ou organização. As opções de compartilhamento
-Organização
permitem escolher as pessoas que você deseja permitir que exibam ou editem o documento.
Salvar um documento comercial que você deseja compartilhar com sua equipe. Para mantê-
Sites -Organização
lo privado, coloque-o em uma biblioteca que não seja compartilhada com outras pessoas.
Salvar um documento pessoal que você deseja manter privado ou que deseja compartilhar
OneDrive - Pessoal
com amigos e familiares.
Salvar um documento em uma pasta no seu computador. Escolha Este PC e escolha uma
Este PC
pasta.
Adicionar um novo local online. Escolha Adicionar um Local e toque ou clique em SharePoint
adicionar um local
do Office 365ou OneDrive.

Salvando Arquivos
É importante ao terminar um documento, ou durante a digitação do mesmo, quando o documento a ser criado é longo,
salvar seu trabalho. Salvar consiste em armazenar se documento em forma de arquivo em seu computador, pendrive, ou
outro dispositivo de armazenamento. Para salvar seu documento, clique no botão salvar no topo da tela. Será aberta uma
tela onde você poderá definir o nome, local e formato de seu arquivo.

Observe na janela de salvar que o Word procura salvar seus arquivos na pasta Documents do usuário, você pode mudar
o local do arquivo a ser salvo, pela parte esquerda da janela. No campo nome do arquivo, o Word normalmente preenche
com o título do documento, como o documento não possui um título, ele pega os primeiros 255 caracteres e atribui como
nome, é aconselhável colocar um nome menor e que se aproxime do conteúdo de seu texto. “Em Tipo a maior mudança, até
versão 2003, os documentos eram salvos no formato”. DOC”, a partir da versão 2010, os documentos são salvos na versão”.
DOCX”, que não são compatíveis com as versões anteriores. Para poder salvar seu documento e manter ele compatível com
versões anteriores do Word, clique na direita dessa opção e mude para Documento do Word 97-2003.

22
INFORMÁTICA

Observe que o nome de seu arquivo agora aparece na barra de títulos.

Abrindo um arquivo do Word


Para abrir um arquivo, você precisa clicar na ABA Arquivo.

Na esquerda da janela, o botão abrir é o segundo abaixo de novo, observe também que ele mostra uma relação de
documentos recentes, nessa área serão mostrados os últimos documentos abertos pelo Word facilitando a abertura. Ao
clicar em abrir, será necessário localizar o arquivo no local onde o mesmo foi salvo.

23
INFORMÁTICA

Caso necessite salvar seu arquivo em outro formato, outro local ou outro nome, clique no botão Office e escolha Salvar
Como.

Visualização do Documento

Podemos alterar a forma de visualização de nosso documento. No rodapé a direta da tela temos o controle de Zoom.·.
Anterior a este controle de zoom temos os botões de forma de visualização de seu documento, que podem
também ser acessados pela Aba Exibir.

Os cinco primeiros botões são os mesmos que temos em miniaturas no rodapé.


• Layout de Impressão: Formato atual de seu documento é o formato de como seu documento ficará na folha im-
pressa.
• Modo de leitura: Ele oculta as barras de seu documento, facilitando a leitura em tela, observe que no rodapé do
documento à direita, ele possui uma flecha apontado para a próxima página. Para sair desse modo de visualização, clique
no botão fechar no topo à direita da tela.
• Layout da Web: Aproxima seu texto de uma visualização na Internet, esse formato existe, pois muitos usuários
postam textos produzidos no Word em sites e blogs na Internet.
• Estrutura de Tópicos: Permite visualizar seu documento em tópicos, o formato terá melhor compreensão quando
trabalharmos com marcadores.
• Rascunho: É o formato bruto, permite aplicar diversos recursos de produção de texto, porém não visualiza como
impressão nem outro tipo de meio.

24
INFORMÁTICA

O terceiro grupo de ferramentas da Aba exibição per-


mite trabalhar com o Zoom da página. Ao clicar no botão
Zoom o Word apresenta a seguinte janela:

Onde podemos utilizar um valor de zoom predefinido,


ou colocarmos a porcentagem desejada, podemos visuali-
zar o documento em várias páginas. E finalizando essa aba
temos as formas de exibir os documentos aberto em uma
mesma seção do Word.

Configuração de Documentos

Um dos principais cuidados que se deve ter com seus


documentos é em relação à configuração da página. A
ABNT (Associação Brasileira de Normas Técnicas) possui um
manual de regras para documentações, então é comum es-
cutar “o documento tem que estar dentro das normas”, não O grupo “Configurar Página”, permite definir as mar-
vou me atentar a nenhuma das normas especificas, porém gens de seu documento, ele possui alguns tamanhos pré-
vou ensinar como e onde estão as opções de configuração definidos, como também personalizá-las.
de um documento. Ao personalizar as margens, é possível alterar as mar-
No Word 2016 a ABA que permite configurar sua pági- gens superior, esquerda, inferior e direita, definir a orien-
na é a ABA Layout da Página. tação da página, se retrato ou paisagem, configurar a fora
de várias páginas, como normal, livro, espelho. Ainda nessa
mesma janela temos a guia Papel.

25
INFORMÁTICA

Em cabeçalhos e rodapés podemos definir se vamos


utilizar cabeçalhos e rodapés diferentes nas páginas pares
e ímpares, e se quero ocultar as informações de cabeçalho
e rodapé da primeira página. Em Página, pode-se definir o
alinhamento do conteúdo do texto na página. O padrão é
o alinhamento superior, mesmo que fique um bom espa-
ço em branco abaixo do que está editado. Ao escolher a
opção centralizada, ele centraliza o conteúdo na vertical. A
opção números de linha permite adicionar numeração as
linhas do documento.

Colunas

Nesta guia podemos definir o tipo de papel, e fonte de


alimentação do papel.

Ao clicar em mais Colunas, é possível personalizar as


suas colunas, o Word disponibiliza algumas opções pré-
definidas, mas você pode colocar em um número maior de
colunas, adicionar linha entre as colunas, definir a largura
e o espaçamento entre as colunas. Observe que se você
pretende utilizar larguras de colunas diferentes é preciso
desmarcar a opção “Colunas de mesma largura”. Atente
também que se preciso adicionar colunas a somente uma
parte do texto, eu preciso primeiro selecionar esse texto.

Alterar a cor ou a tela de fundo no Word 2016 para


Windows
Para aumentar o apelo visual do documento, adicione
uma cor da tela de fundo usando o botão Cor da Página.
A terceira guia dessa janela chama-se Layout. A primei- Você também pode adicionar uma imagem como marca
ra opção dessa guia chama-se seção. Aqui se define como d’água de tela de fundo.
será uma nova seção do documento, vamos aprender mais
frente como trabalhar com seções.

26
INFORMÁTICA

Alterar a cor do plano de fundo


Clique em Design > Cor da Página.
Escolha a cor que deseja em Cores do Tema ou Cores Padrão.

Caso não veja a cor desejada, clique em Mais Cores e escolha a cor desejada usando qualquer uma das opções da caixa
Cores.
Para adicionar gradiente, textura, padrão ou imagem, clique em Efeitos de Preenchimento e, em seguida, clique nas
guias Gradiente, Textura, Padrão ou Imagem para selecionar as opções desejadas.
Os padrões e texturas são replicados (ou organizados lado a lado) para preencher a página toda. Se você salvar um
documento como página Web, as texturas serão salvas como arquivos JPEG e os padrões e gradientes como arquivos PNG.
Remover a cor da tela de fundo
Para remover a cor da página, clique em Design > Cor da Página > Sem Cor.

Adicionar uma imagem como marca d’água em tela de fundo no Word 2016 para Windows
Adicionar uma marca d’água de imagem é uma ótima maneira de aplicar uma marca em seu documento com um logo-
tipo ou adicionar uma tela de fundo atraente. Para inserir uma imagem de tela de fundo rapidamente, adicione-o como uma
marca d’água personalizada. Se você quiser mais opções para ajustar a imagem de tela de fundo, insira-a como um cabeçalho.

27
INFORMÁTICA

Adicionar uma imagem de tela de fundo como uma marca d’água


Este método é rápido, mas ele não lhe dá muitas opções para a formatação da imagem.
Clique em Design > Marca D’água.

Clique em Marca D’água Personalizada.


Clique em Marca d’água de imagem > Selecionar Imagem.
Procure (ou pesquise) a imagem desejada e clique em Inserir.
Selecione uma porcentagem em Escala para inserir a imagem com um tamanho específico. Verifique se colocou uma
porcentagem grande o suficiente para preencher a página ou simplesmente selecione Automático.
Marque a caixa de seleção Desbotar para clarear a imagem de modo que não interfira no texto.
Clique em OK.
Adicionar uma imagem de tela de fundo com mais opções de formatação
Inserir uma imagem de tela de fundo como um cabeçalho é um pouco mais complexo, mas oferece mais opções de
ajuste de fotos.
Clique em Inserir > Cabeçalho > Editar Cabeçalho.
Na guia Ferramentas de Cabeçalho e Rodapé, clique em Imagens.
Vá até a imagem e clique em Inserir.
Na guia Ferramentas de Imagem, clique em Posição e clique na opção centralizada em Com Quebra Automática de
Texto.
Em Ferramentas de Imagem, clique em Quebra de Texto Automática > Atrás do Texto.
Em Ferramentas de Imagem, selecione as opções desejadas no grupo Ajustar. Por exemplo, para dar à imagem uma
aparência desbotada para que ela não compita com o texto, clique em Cor e, em Recolorir, clique na opção Desbotar:

Clique em Ferramentas de Cabeçalho e Rodapé > Fechar Cabeçalho e Rodapé

28
INFORMÁTICA

Selecionando Textos
Embora seja um processo simples, a seleção de textos é indispensável para ganho de tempo na edição de seu texto.
Através da seleção de texto podemos mudar a cor, tamanho e tipo de fonte, etc.

Selecionando pelo Mouse


Ao posicionar o mouse mais a esquerda do texto, o cursor aponta para a direita.
• Ao dar um clique ele seleciona toda a linha
• Ao dar um duplo clique ele seleciona todo o parágrafo.
• Ao dar um triplo clique seleciona todo o texto
Com o cursor no meio de uma palavra:
• Ao dar um clique o cursor se posiciona onde foi clicado
• Ao dar um duplo clique, ele seleciona toda a palavra.
• Ao dar um triplo clique ele seleciona todo o parágrafo
Podemos também clicar, manter o mouse pressionado e arrastar até onde se deseja selecionar. O problema é que se o
mouse for solto antes do desejado, é preciso reiniciar o processo, ou pressionar a tecla SHIFT no teclado e clicar ao final da
seleção desejada. Podemos também clicar onde começa a seleção, pressionar a tecla SHIFT e clicar onde termina a seleção.
É possível selecionar palavras alternadas. Selecione a primeira palavra, pressione CTRL e vá selecionando as partes do texto
que deseja modificar.

Copiar e Colar
O copiar e colar no Word funciona da mesma forma que qualquer outro programa, pode-se utilizar as teclas de atalho
CTRL+C (copiar), CTRL+X (Recortar) e CTRL+V(Colar), ou o primeiro grupo na ABA Pagina Inicial.

Este é um processo comum, porém um cuidado importante é quando se copia texto de outro tipo de meio como, por
exemplo, da Internet. Textos na Internet possuem formatações e padrões deferentes dos editores de texto. Ao copiar um
texto da Internet, se você precisa adequá-lo ao seu documento, não basta apenas clicar em colar, é necessário clicar na
setinha apontando para baixo no botão Colar, escolher Colar Especial.

Observe na imagem que ele traz o texto no formato HTML. Precisa-se do texto limpo para que você possa manipulá-lo,
marque a opção Texto não formatado e clique em OK.

29
INFORMÁTICA

Localizar e Substituir
Ao final da ABA Pagina Inicial temos o grupo edição, dentro dela temos a opção Localizar e a opção Substituir. Clique
na opção Substituir.

A janela que se abre possui três guias, localizar, Substituir e Ir para. A guia substituir que estamos vendo, permite subs-
tituir em seu documento uma palavra por outra. A substituição pode ser feita uma a uma, clicando em substituir, ou pode
ser todas de uma única vez clicando-se no botão Substituir Tudo.
Algumas vezes posso precisar substituir uma palavra por ela mesma, porém com outra cor, ou então somente quando
escrita em maiúscula, etc., nestes casos clique no botão Mais. As opções são:
• Pesquisar: Use esta opção para indicar a direção da pesquisa;
• Diferenciar maiúsculas de minúsculas: Será localizada exatamente a palavra como foi digitada na caixa localizar.
• Palavras Inteiras: Localiza uma palavra inteira e não parte de uma palavra. Ex: Atenciosamente.
• Usar caracteres curinga: Procura somente as palavras que você especificou com o caractere coringa. Ex. Se você
digitou *ão o Word vai localizar todas as palavras terminadas em ão.
• Semelhantes: Localiza palavras que tem a mesma sonoridade, mas escrita diferente. Disponível somente para pa-
lavras em inglês.
• Todas as formas de palavra: Localiza todas as formas da palavra, não será permitida se as opções usar caractere
coringa e semelhantes estiverem marcadas.
• Formatar: Localiza e Substitui de acordo com o especificado como formatação.
• Especial: Adiciona caracteres especiais à caixa localizar. A caixa de seleção usar caracteres curinga.

Formatação de texto

Um dos maiores recursos de uma edição de texto é a possibilidade de se formatar o texto. No Office 2016 a ABA res-
ponsável pela formatação é a Página Inicial e os grupo Fonte, Parágrafo e Estilo.

Formatação de Fonte

A formatação de fonte diz respeito ao tipo de letra, tamanho de letra, cor, espaçamento entre caracteres, etc., para
formatar uma palavra, basta apenas clicar sobre ela, para duas ou mais é necessário selecionar o texto, se quiser formatar
somente uma letra também é necessário selecionar a letra. No grupo Fonte, temos visível o tipo de letra, tamanho, botões
de aumentar fonte e diminuir fonte, limpar formatação, negrito, itálico, sublinhado, observe que ao lado de sublinhado
temos uma seta apontando para baixo, ao clicar nessa seta, é possível escolher tipo e cor de linha.

30
INFORMÁTICA

Ao lado do botão de sublinhado temos o botão Ta-


chado – que coloca um risco no meio da palavra, botão
subscrito e sobrescrito e o botão Maiúsculas e Minúsculas.

Podemos definir a escala da fonte, o espaçamento en-


Este botão permite alterar a colocação de letras maiús- tre os caracteres que pode ser condensado ou comprimido,
culas e minúsculas em seu texto. Após esse botão temos a posição é referente ao sobrescrito e subscrito, permitindo
o de realce – que permite colocar uma cor de fundo para que se faça algo como: .
realçar o texto e o botão de cor do texto.

Podemos também clicar na Faixa no grupo Fonte.

A janela fonte contém os principais comandos de for-


matação e permite que você possa observar as alterações
antes de aplica. Ainda nessa janela temos a opção Avançado.

31
INFORMÁTICA

Kerning: é o acerto entre o espaço dentro das palavras, Cor do Preenchimento do Parágrafo.
pois algumas vezes acontece de as letras ficaram com es-
paçamento entre elas de forma diferente. Uma ferramenta
interessante do Word é a ferramenta pincel, pois com ela
você pode copiar toda a formatação de um texto e aplicar
em outro.

Formatação de parágrafos
A principal regra da formatação de parágrafos é que
independentemente de onde estiver o cursor a formatação
será aplicada em todo o parágrafo, tendo ele uma linha
ou mais. Quando se trata de dois ou mais parágrafos será
necessário selecionar os parágrafos a serem formatados. A
formatação de parágrafos pode ser localizada na ABA Pá-
gina Inicial, e os recuos também na ABA Layout da Página.

Bordas no parágrafo.

No grupo da Guia Página Inicial, temos as opções de


marcadores (bullets e numeração e listas de vários níveis),
diminuir e aumentar recuo, classificação e botão Mostrar
Tudo, na segunda linha temos os botões de alinhamentos:
esquerda, centralizado, direita e justificado, espaçamento
entre linhas, observe que o espaçamento entre linhas pos-
sui uma seta para baixo, permitindo que se possa definir
qual o espaçamento a ser utilizado.

Marcadores e Numeração

Os marcadores e numeração fazem parte do grupo


parágrafos, mas devido a sua importância, merecem um
destaque. Existem dois tipos de marcadores: Símbolos e
Numeração.

32
INFORMÁTICA

A opção vários níveis é utilizada quando nosso texto Onde você poderá escolher a Fonte (No caso acon-
tenha níveis de marcação como, por exemplo, contratos e selha-se a utilizar fontes de símbolos como a Winddings,
petições. Os marcadores do tipo Símbolos como o nome já Webdings), e depois o símbolo. Ao clicar em Imagem, você
diz permite adicionar símbolos a frente de seus parágrafos. poderá utilizar uma imagem do Office, e ao clicar no botão
Se precisarmos criar níveis nos marcadores, basta clicar importar, poderá utilizar uma imagem externa.
antes do inicio da primeira palavra do parágrafo e pressio-
nar a tecla TAB no teclado. Bordas e Sombreamento
Podemos colocar bordas e sombreamentos em nosso
texto. Podem ser bordas simples aplicadas a textos e pará-
grafos. Bordas na página como vimos quando estudamos a
ABA Layout da Página e sombreamentos. Selecione o texto
ou o parágrafo a ser aplicado à borda e ao clicar no botão
de bordas do grupo Parágrafo, você pode escolher uma
borda pré-definida ou então clicar na última opção Bordas
e Sombreamento.
Você pode observar que o Word automaticamente Podemos começar escolhendo uma definição de borda
adicionou outros símbolos ao marcador, você pode alte- (caixa, sombra, 3D e outra), ou pode-se especificar cada
rar os símbolos dos marcadores, clicando na seta ao lado uma das bordas na direita onde diz Visualização. Pode-se
do botão Marcadores e escolhendo a opção Definir Novo pelo meio da janela especificar cor e largura da linha da
Marcador. borda. A Guia Sombreamento permite atribuir um preen-
chimento de fundo ao texto selecionado. Você pode es-
colher uma cor base, e depois aplicar uma textura junto
dessa cor.

Data e Hora
O Word Permite que você possa adicionar um campo
de Data e Hora em seu texto, dentro da ABA Inserir, no
grupo Texto, temos o botão Data e Hora.

Ao clicar em Símbolo, será mostrada a seguinte janela:

Basta escolher o formato a ser aplicado e clicar em OK.


Se precisar que esse campo sempre atualize data, marque
a opção Atualizar automaticamente.

33
INFORMÁTICA

Inserindo Elementos Gráficos

O Word permite que se insira em seus documentos arquivos gráficos como Imagem, Clip-art, Formas, etc., as opções
de inserção estão disponíveis na ABA Inserir, grupo ilustrações.

Formas
Podemos também adicionar formas ao nosso conteúdo do texto

Para desenhar uma forma, o processo é simples, basta clicar na forma desejada e arrastar o mouse na tela para definir
as suas dimensões. Ao desenhar a sua forma a barra passa a ter as propriedades para modificar a forma.

SmartArt

O SmartArt permite ao você adicionar Organogramas ao seu documento. Basta selecionar o tipo de organograma a ser
trabalhado e clique em OK.

34
INFORMÁTICA

WordArt

Para finalizarmos o trabalho com elementos gráficos temo os WordArt que já um velho conhecido da suíte Office, ele
ainda mantém a mesma interface desde a versão do Office 97 No grupo Texto da ABA Inserir temos o botão de WorArt
Selecione um formato de WordArt e clique sobre ele.

Será solicitado a digitação do texto do WordArt. Digite seu texto e clique em OK. Será mostrada a barra do WordArt

Um dos grupos é o Texto, nesse grupo podemos editar o texto digitado e definir seu espaçamento e alinhamentos. No
grupo Estilos de WordArt pode-se mudar a forma do WordArt, depois temos os grupos de Sombra, Efeitos 3D, Organizar
e Tamanho.

Controlar alterações no Word


Quando quiser verificar quem está fazendo alterações em seu documento, ative o recurso Controlar Alterações.
Clique em Revisar > Controlar Alterações.

Agora, o Word está no modo de exibição Marcação Simples. Ele marca todas as alterações feitas por qualquer pessoa
no documento e mostra para você onde elas estão, exibindo uma linha ao lado da margem.

35
INFORMÁTICA

O Word mostra um pequeno balão no local em que IMPORTANTE:  A única maneira de remover alterações
alguém fez um comentário. Para ver o comentário, clique controladas de um documento é aceitá-las ou rejeitá-las.
no respectivo balão. Ao escolherSem Marcação na caixa Exibir para Revisão aju-
da a ver qual será a aparência do documento final, mas isso
apenas oculta temporariamente as alterações controladas.
As alterações não são excluídas e aparecerão novamente
da próxima vez em que o documento for aberto. Para ex-
cluir permanentemente as alterações controladas, aceite-
-as ou rejeite-as.
Clique em Revisar > Próxima > Aceitar ou Rejeitar.
Para ver as alterações, clique na linha próxima à mar-
gem. Isso alterna para o modo de exibição Toda a Marca-
ção do Word.

O Word aceita a alteração ou a remove e depois passa


para a próxima alteração.
Manter o recurso Controlar Alterações ativado Para excluir um comentário, selecione-o e clique
É possível bloquear o recurso Controlar Alterações com em  Revisão  >  Excluir. Para excluir todos os comentários,
uma senha para impedir que outra pessoa o desative. (Lem- clique em Excluir >Excluir Todos os Comentários do Docu-
bre-se da senha para poder desativar esse recurso quando mento.
estiver pronto para aceitar ou rejeitar as alterações.)
DICA:  Antes de compartilhar a versão final do seu do-
Clique em Revisar.
cumento, é uma boa ideia executar o Inspetor de Docu-
Clique na seta ao lado de Controlar Alterações e clique
mento. Essa ferramenta verifica comentários e alterações
em Bloqueio de Controle.
controladas, além de texto oculto, nomes pessoais em pro-
priedades e outras informações que talvez você não que-
ria compartilhar amplamente. Para executar o Inspetor de
Documento,
Imprimir um documento no Word
Antes de imprimir, você pode visualizar o documento e
especificar as páginas que você deseja imprimir.
Visualizar o documento
No menu Arquivo, clique em Imprimir.
Para visualizar cada página, clique nas setas para frente
e para trás, na parte inferior da página.

Digite uma senha e depois digite-a mais uma vez na


caixa Redigite para confirmar.
Clique em OK.
Enquanto as alterações controladas estiverem blo-
queadas, você não poderá desativar o controle de altera-
ções, nem poderá aceitar ou rejeitar essas alterações.
Para liberar o bloqueio, clique na seta ao lado de Con-
trolar Alterações e clique novamente em Bloqueio de Con-
trole. O Word solicitará que você digite sua senha. Depois Quando o texto é pequeno demais e difícil de ler, use
que você digitá-la e clicar em OK, o recurso Controlar Alte- o controle deslizante de zoom na parte inferior da página
rações continuará ativado, mas agora você poderá aceitar para ampliá-lo.
e rejeitar alterações.

Desativar o controle de alterações


Para desativar esse recurso, clique no botão Controlar
Alterações. O Word deixará de marcar novas alterações,
mas todas as alterações já realizadas continuarão marcadas
no documento até que você as remova.
Remover alterações controladas

36
INFORMÁTICA

Escolha o número de cópias e qualquer outra opção No menu Arquivo, clique em Imprimir.


desejada e clique no botão Imprimir. Para imprimir apenas determinadas páginas, algumas das
propriedades do documento ou alterações controladas e co-
mentários, clique na seta em Configurações, ao lado de Im-
primir Todas as Páginas (o padrão), para ver todas as opções.

Para imprimir somente determinadas páginas, siga um


destes procedimentos:
Para imprimir a página mostrada na visualização, sele-
cione a opção Imprimir Página Atual.
Para imprimir páginas consecutivas, como 1 a 3, esco-
lha Impressão Personalizada e insira o primeiro e o último
número das páginas na caixa Páginas.
Imprimir páginas específicas Para imprimir páginas individuais e intervalo de pági-
nas (como a página 3 e páginas 4 a 6) ao mesmo tempo,
escolhaImpressão Personalizada  e digite os números das
páginas e intervalos separados por vírgulas (por exemplo,
3, 4-6).

Estilos
Os estilos podem ser considerados formatações pron-
tas a serem aplicadas em textos e parágrafos. O Word dis-
ponibiliza uma grande quantidade de estilos através do
grupo estilos.

37
INFORMÁTICA

Para aplicar um estilo ao um texto é simples. Se você Será mostrado todos os estilos presentes no documen-
clicar em seu texto sem selecioná-lo, e clicar sobre um esti- to em uma caixa à direita. Na parte de baixo da janela exis-
lo existente, ele aplica o estilo ao parágrafo inteiro, porém tem três botões, o primeiro deles chama-se Novo Estilo,
se algum texto estiver selecionado o estilo será aplicado clique sobre ele.
somente ao que foi selecionado.

Observe na imagem acima que foi aplicado o estilo Tí-


tulo2 em ambos os textos, mas no de cima como foi clicado
somente no texto, o estilo está aplicado ao parágrafo, na
linha de baixo o texto foi selecionado, então a aplicação do
estilo foi somente no que estava selecionado. Ao clicar no
botão Alterar Estilos é possível acessar a diversas defini-
ções de estilos através da opção Conjunto de Estilos.

No exemplo dei o nome de Citações ao meu estilo, defi-


ni que ele será aplicado a parágrafos, que a base de criação
dele foi o estilo corpo e que ao finalizar ele e iniciar um novo
parágrafo o próximo será também corpo. Abaixo definir a
formatação a ser aplicada no mesmo. Na parte de baixo
mantive a opção dele aparecer nos estilos rápidos e que o
mesmo está disponível somente a este documento. Ao fina-
lizar clique em OK. Veja um exemplo do estilo aplicado:

Podemos também se necessário criarmos nossos pró-


prios estilos. Clique na Faixa do grupo Estilo.

Criar um sumário no Word 2016


Para criar um sumário que seja fácil de atualizar, apli-
que estilos de título ao texto que deseja incluir no sumário.
Depois disso, o Word vai compilá-lo automaticamente a
partir desses títulos.
Aplicar estilos de título
Escolha o texto que você deseja incluir no sumário e,
em seguida, na guia Página Inicial, clique em um estilo do
título, comoTítulo 1.

Faça isso para todo o texto que você deseja exibir no


sumário.

38
INFORMÁTICA

Criar um sumário O Word também verifica e marca possíveis erros gra-


O Word usa os títulos no documento para construir um maticais com uma linha ondulada azul:
sumário automático que pode ser atualizado quando você
altera o texto do título, sequência ou nível.
Clique no local que deseja inserir o índice analítico,
normalmente perto do início de um documento.
Clique em Referências > Sumário e escolha um estilo Se os erros de ortografia e gramática não estiverem
de Sumário Automático na lista. marcados, talvez seja necessário habilitar a verificação or-
OBSERVAÇÃO: Se você usar um estilo de Sumário Manual, tográfica e gramatical automática.
o Word não utilizará os títulos para criar um sumário e não será Quando você vir erros ortográficos ou gramaticais, cli-
possível atualizá-lo automaticamente. Em vez disso, o Word que com botão direito do mouse ou mantenha pressio-
usará o texto do espaço reservado para criar um sumário fictí- nada a palavra ou a frase e escolha uma das opções para
cio, e você deverá preencher as entradas manualmente. corrigir o erro.
Habilitar (ou desabilitar) a verificação ortográfica e gra-
matical automática
Clique em Arquivo> Opções> Revisão de Texto.
Você pode optar por verificar a ortografia e a gramá-
tica automaticamente, uma ou outra, ambas ou nenhuma
delas, ou até mesmo outras opções, como a verificação or-
tográfica contextual

Caso prefira  formatar ou personalizar o sumário, é


possível fazê-lo. Por exemplo, você pode alterar a fonte, o
número de níveis de título e optar por mostrar linhas pon-
tilhadas entre as entradas e os números de página. Em Exceções para, você pode optar por ocultar os erros
de gramática e de ortografia em seu documento aberto ou,
Verificar ortografia e gramática no Word 2016 para se deixar as opções desmarcadas mas mantiver qualquer
Windows uma das opções acima delas marcada, todos os documen-
O Word verifica automaticamente possíveis erros de tos novos manterão essas configurações.
ortografia e gramaticais à medida que você digita. Se pre- Clique em OK para salvar suas alterações.
ferir esperar para verificar a ortografia e a gramática quan-
do terminar de escrever, você pode desabilitar a verificação
ortográfica e gramatical.
Verificar a ortografia e a gramática ao digitar
O Word verifica e marca automaticamente possíveis er-
ros de ortografia com uma linha ondulada vermelha:

39
INFORMÁTICA

Verificar a ortografia e a gramática ao mesmo tempo


Verificar a ortografia e a gramática no seu documento é útil quando você quer revisar rapidamente seu texto. Você
pode verificar a existência de possíveis erros e então decidir se concorda com o verificador ortográfico e gramatical.
Clique em Revisão > Ortografia e Gramática (ou pressione F7) para iniciar o verificador ortográfico e gramatical e veja
os resultados nos painéis de tarefas de Ortografia e Gramática.

Se o Word encontrar um possível erro, um painel de tarefas será aberto e mostrará as opções de ortografia e gramática:

Para corrigir um erro, siga um destes procedimentos:


Use uma das sugestões Para usar uma das palavras sugeridas, selecione a palavra na lista de sugestões e clique em
Alterar. (Você também pode clicar em Alterar tudo, se souber que usou essa palavra incorretamente em todo o documento,
para não precisar corrigi-la toda vez que ela aparecer.)
Adicionar uma palavra ao dicionário     Se a palavra estiver correta e se for uma que você deseja que o Word e TODOS
os programas do Office reconheçam, clique em Adicionar para adicioná-la ao dicionário. Isso só funciona para palavras com
grafia incorreta. Não é possível adicionar uma gramática personalizada ao dicionário.
Ignorar a palavra    Clique em Ignorar para ignorar apenas aquela ocorrência ou clique em Ignorar Tudo para ignorar
todas as ocorrências da palavra.
Depois de corrigir ou ignorar algo marcado como um possível erro, o Word passa para o próximo. Quando o Word
concluir a revisão do documento, você verá uma mensagem informando que a verificação ortográfica ou gramatical foi
concluída.
Clique em OK para retornar ao documento.
Verificar novamente problemas de ortografia e gramática
Você também pode forçar uma nova verificação das palavras e da gramática que anteriormente foram ignoradas.
Abra o documento que você deseja verificar novamente.
Clique em Arquivo> Opções> Revisão de Texto.
Em Ao corrigir ortografia e a gramática no Word, clique em Verificar Documento Novamente.
Quando você vir uma mensagem, clique em Sim e clique em OK para fechar a caixa de diálogo Opções do Word.
Em seguida, no seu documento, clique em Revisão > Ortografia e Gramática (ou pressione F7).

40
INFORMÁTICA

Proteger um documento com senha Habilitar macros no modo de exibição Backstage


Ajude a proteger um documento confidencial contra Outra maneira de habilitar macros em um arquivo é
edições indesejadas atribuindo uma senha. Também é pos- possível pelo modo de exibição Microsoft Office Backsta-
sível evitar que um documento seja aberto. ge, o modo de exibição que aparece depois que você clica
Clique em Arquivo > Informações > Proteger Docu- na guia Arquivo,quando a Barra de Mensagens amarela é
mento > Criptografar com Senha. exibida.
Clique na guia Arquivo.
Na área Aviso de Segurança, clique em Habilitar Con-
teúdo.
Em Habilitar Todo o Conteúdo, clique em Sempre habi-
litar o conteúdo ativo deste documento.
O arquivo se tornará um documento confiável.
A imagem a seguir é um exemplo das opções Habilitar
Conteúdo.

Habilitar macros uma vez quando o Aviso de Seguran-


Na caixa Criptografar Documento, digite uma senha e ça for exibido
clique em OK. Use as instruções a seguir para habilitar macros en-
Na caixa Confirmar Senha, digite a senha novamente e quanto o arquivo permanecer aberto. Quando o arquivo
for fechado e, em seguida, reaberto, o aviso aparecerá no-
clique em OK.
vamente.
Observações : 
Clique na guia Arquivo.
Você sempre pode alterar ou remover sua senha.
Na área Aviso de Segurança, clique em Habilitar Con-
As senhas diferenciam maiúsculas de minúsculas. Veri-
teúdo.
fique se a tecla CAPS LOCK está desativada quando digitar
Selecione Opções Avançadas.
uma senha pela primeira vez.
Na caixa de diálogo Opções de Segurança do Micro-
Se você perder ou esquecer uma senha, o Word não
soft Office, clique em Habilitar conteúdo para esta sessão
conseguirá recuperar suas informações. Portanto, guarde para cada macro.
uma cópia da senha em um local seguro ou crie uma senha Clique em OK.
forte da qual se lembrará.
Alterar configurações de macro na Central de Confia-
Habilitar ou desabilitar macros em arquivos do Office bilidade
Uma macro é uma série de comandos que podem ser As configurações de macro estão localizadas na Cen-
usados para automatizar uma tarefa repetida e que po- tral de Confiabilidade. Entretanto, se você trabalha em uma
dem ser executados durante a tarefa. Este artigo apresenta organização, é possível que o administrador do sistema te-
informações sobre os riscos envolvidos no trabalho com nha alterado as configurações padrão para impedir a alte-
macros, e você poderá saber mais sobre como habilitar ou ração das configurações.
desabilitar macros na Central de Confiabilidade. Importante:  Quando você altera as configurações de
macro na Central de Confiabilidade, elas são alteradas ape-
Habilitar macros quando a Barra de Mensagens for exibida nas no programa do Office usado no momento. As confi-
Quando você abre um arquivo que possui macros, a gurações de macro não são alteradas em todos os progra-
barra de mensagens amarela aparece com um ícone de mas do Office.
escudo e o botão Habilitar Conteúdo. Se você conhecer Clique na guia Arquivo.
a(s) macro(s) como sendo de uma fonte confiável, use as Clique em Opções.
instruções a seguir: Clique em Central de Confiabilidade e em Configura-
Na Barra de Mensagens, clique em Habilitar Conteúdo. ções da Central de Confiabilidade.
O arquivo é aberto e se trata de um documento con- Na Central de Confiabilidade, clique em Configurações
fiável. de Macro.
A imagem a seguir é um exemplo da Barra de Mensa- Faça as seleções desejadas.
gens quando há macros no arquivo. Clique em OK.
A imagem a seguir é a área Configurações de Macro da
Central de Confiabilidade.

41
INFORMÁTICA

Use as informações da seção a seguir para saber mais sobre as configurações de macro.

Configurações de macro explicadas


Desabilitar todas as macros sem notificação     As macros e os alertas de segurança sobre macros serão desabilitados.
Desabilitar todas as macros com notificação     As macros serão desabilitadas, mas os alertas de segurança serão exibi-
dos, se houverem macros. Habilite-as, uma de cada vez.
Desabilitar todas as macros, exceto as digitalmente assinadas     As macros são desabilitadas, mas alertas de segurança
serão apresentados se houver macros. No entanto, se a macro estiver assinada digitalmente por um fornecedor confiável,
ela será executada se você confiar no fornecedor. Se não confiar no fornecedor, você será notificado para habilitar a macro
assinada e confiar no fornecedor.
Habilitar todas as macros (não recomendável, pois pode executar um código potencialmente perigoso)     Todas as ma-
cros serão executadas. Essa configuração deixa seu computador vulnerável a códigos potencialmente mal intencionados.
Confiar no acesso ao modelo de objeto do projeto do VBA     Permitir ou não permitir o acesso programático ao modelo
de objeto do Visual Basic for Applications (VBA) de um cliente de automação. Esta opção de segurança destina-se a código
escrito para automatizar um programa do Office e manipular o ambiente e o modelo de objeto VBA.É uma configuração
definida por usuário e por aplicativo, e nega o acesso por padrão, impedindo que programas não autorizados compilem
código de replicação automática prejudicial. Para que os clientes de automação acessem o modelo de objeto do VBA, o
usuário que executa o código deve conceder acesso. Para ativar o acesso, marque a caixa de seleção.
Observação : O Microsoft Publisher e o Microsoft Access não têm a opção Confiar no acesso ao modelo de objeto do
projeto do VBA.

O que são macros, quem são suas criadoras e qual é o risco de segurança?
As macros automatizam tarefas que são usadas frequentemente para economizar o tempo de pressionamento de
teclas e ações do mouse. Muitas macros foram criadas com o uso do VBA (Visual Basic for Applications) e gravadas por
desenvolvedores de software. No entanto, algumas macros podem representar um possível risco para a segurança. Um
usuário mal-intencionado, também conhecido como hacker, pode introduzir uma macro destrutiva em um arquivo que
possa espalhar vírus no computador ou na rede da sua organização.

Criar ou executar uma macro


Para poupar tempo em tarefas que você costuma realizar com frequência, compacte as etapas em uma macro. Em pri-
meiro lugar, grave a macro. Em seguida, você poderá executá-la clicando em um botão da Barra de Ferramentas de Acesso
Rápido ou pressionando uma combinação de teclas. Isso dependerá de como você a configurar.
Vamos começar com a configuração do botão.

42
INFORMÁTICA

Clique em Exibir > Macros > Gravar Macro. Clique na nova macro (cujo nome é algo do tipo Normal.
NovasMacros.<nome da sua macro>) e clique em Adicionar.

Digite um nome para a macro.

Clique em Modificar.

Para usar essa macro em qualquer novo documento


que você criar, verifique se a caixa Armazenar macro em
exibe Todos os Documentos (Normal.dotm).

Para executar uma macro quando você clicar em um


botão, clique em Botão.

43
INFORMÁTICA

Escolha uma imagem de botão, digite o nome deseja- Clique em Exibir > Macros > Gravar Macro.
do e clique em OK duas vezes.

Digite um nome para a macro.

Agora, chegou a hora de gravar as etapas. Clique nos


comandos ou pressione as teclas para cada etapa na tarefa.
O Word grava seus cliques e pressionamentos de teclas.
Observação : Use o teclado para selecionar texto en-
quanto você grava a macro. Macros não gravam seleções
feitas com o mouse. Para usar essa macro em qualquer novo documento
Para parar de gravar, clique em Exibir > Macros > Parar que você criar, verifique se a caixa Armazenar macro em
Gravação. exibe Todos os Documentos (Normal.dotm).

O botão da sua macro aparece na Barra de Ferramen-


tas de Acesso Rápido.

Para executar sua macro quando você pressionar um


atalho do teclado, clique em Teclado.

Para executar a macro, clique no botão.


Criar uma macro com um atalho do teclado

44
INFORMÁTICA

Digite uma combinação de teclas na caixa Pressione a Clique em Exibir > Macros > Exibir Macros.
nova tecla de atalho.
Verifique se essa combinação já não está atribuída a
outro item. Se estiver, tente uma combinação diferente.
Para usar esse atalho de teclado em qualquer novo do-
cumento criado, verifique se a caixa Salvar alterações em
indica Normal.dotm.
Clique em Atribuir.
Agora, chegou a hora de gravar as etapas. Clique nos
comandos ou pressione as teclas para cada etapa na tarefa.
O Word grava seus cliques e pressionamentos de teclas.
Observação: Use o teclado para selecionar texto en-
quanto você grava a macro. Macros não gravam seleções
feitas com o mouse.
Para parar de gravar, clique em Exibir > Macros > Parar Clique em Organizador.
Gravação.

Para executar a macro, pressione as teclas de atalho


do teclado.
Clique na macro que você quer adicionar ao modelo
Executar uma macro Normal.dotm e clique em Copiar.
Para executar uma macro, clique no botão na Barra de Adicionar um botão de macro à faixa de opções
Ferramentas de Acesso Rápido, pressione o atalho de te- Clique em Arquivo > Opções > Personalizar Faixa de
clado ou executar a macro a partir da lista de Macros. Opções.
Clique em Exibir > Macros > Exibir Macros. Em Escolher comandos de, clique em Macros.
Clique na macro desejada.
Em Personalizar a faixa de opções, clique na guia e no
grupo personalizado onde você quer adicionar a macro.
Se não tiver um grupo personalizado, clique em Novo
Grupo e depois clique em Renomear e digite um nome
para o seu grupo personalizado.
Clique em Adicionar.
Clique em Renomear para escolher uma imagem para
a macro e digitar o nome desejado.
Clique em OK duas vezes.

Integração com planilhas


Na lista em Nome da macro, clique na macro a ser exe- A integração entre diversos aplicativos, sempre foi uma
cutada. das principais características da suíte Office da Microsoft.
Clique em Executar. Na versão 2016, além de importar documentos prontos, é
Disponibilizar uma macro em todos os documentos possível criar pequenas janelas de outros programas e si-
Para disponibilizar uma macro de um documento em mular sua funcionalidade plena.
todos os novos documentos, adicione-a ao modelo Nor- A partir de um documento do Word, por exemplo, você
mal.dotm. pode criar planilhas usufruindo dos mesmos recursos do
Abra o documento que contém a macro. Excel, contando até com a Barra de tarefas do programa.
Em vez de abrir dois programas, esta dica permite que você
permaneça na interface do Word, facilitando seu trabalho.

45
INFORMÁTICA

Dentro da aba “Inserir”, clique sobre a opção “Tabela” e escolha “Planilha do Excel”:

Ao invés de uma tabela comum, o que você encontra é uma verdadeira miniatura do Excel com todos os recursos do
aplicativo.

Repare que até a Barra de ferramentas tradicional do Word dá espaço para a o editor de planilhas. Basta selecionar uma
porção do texto ou da planilha para alternar entre as ferramentas de ambos os programas do Office.

Principais Atalhos
CTRL+O Abrir um novo documento em branco
CTRL+A Abrir um arquivo já existente
CTRL+V Colar o texto selecionado ou movido
CTRL+X Recortar o texto selecionado
CTRL+T Selecionar o documento inteiro
CTRL+ENTER Iniciar uma nova página em um mesmo documento
CTRL+N Formata o texto selecionado negrito
CTRL+I Formata o texto selecionado para itálico 
CTRL+S Formata o texto selecionado para sublinhado 
CTRL+J Formata o parágrafo para justificado
CTRL+C Copiar o texto selecionado
CTRL+P Imprimir documento
CTRL+E Formata o parágrafo para centralizado
CTRL+Q Formata o parágrafo para alinhamento à esquerda
CTRL+B ou F12 Salvar como..
CTRL+Z Desfazer a última ação 
CTRL+Y Refazer a última ação 
CTRL+F10 Maximizar ou restaurar a janela 
CTRL+D Alterar formatação de caracteres 
ALT+F4 Sair do Word
CTRL+SHIFT+> Aumentar o tamanho da fonte do texto selecionado
CTRL+SHIFT+< Diminuir o tamanho da fonte do texto selecionado
CTRL+SHIFT+W Aplicar sublinhado somente em palavras
CTRL+ALT+L Aplicar o estilo lista

46
INFORMÁTICA

WORD ONLINE (365)

Note a interface do Word online sendo executado diretamente do navegador de internet, bastante semelhante à do
Word 2016.

Diferenças entre o uso de um documento no navegador e no Word


O Microsoft Word Online permite fazer edições e alterações de formatação básicas em seu documento em um nave-
gador da Web. Para recursos mais avançados, use o comando do Word Online Abrir no Word. Quando você salva o docu-
mento no Word, ele é salvo no site em que você o abriu no Word Online.
O documento que você abre no Word Online é o mesmo que é aberto no aplicativo Word da área de trabalho, mas
alguns recursos funcionam de maneira diferente nos dois ambientes.

Editar um documento no Word Online


Clique em Editar documento > Editar no Word Online para fazer alterações em um documento.
Quando você abre um documento do OneDrive, o Word Online a exibe no modo de exibição de leitura. Para fazer
alterações em seu documento, alterne para o modo de exibição de edição, onde você pode adicionar e excluir conteúdo e
fazer outras coisas, tais como:
- Adicionar tabelas e imagens.
- Aplicar estilos.
- Ajustar a formatação.
- Editar cabeçalhos e rodapés.
Você pode fazer essas tarefas simples no modo de exibição de leitura e no modo de edição:
- Exibir e adicionar comentários.
- Compartilhar um documento para que possa trabalhar com outras pessoas ao mesmo tempo.
- Baixar uma cópia.
- Imprimir.
Lembre-se, para editar um documento no Word Online, clique em Editar documento > Editar no Word Online.

Formatos de arquivo com suporte no Word Online


O Word Online abre documentos nestes formatos:
- Documento do Word (.docx)
- Modelo do Word (.dotx)
- Documento do Word habilitado para macro (.docm) ou modelo do Word habilitado para macro (.dotm)

47
INFORMÁTICA

O documento pode ser aberto, mas as macros não são - Proteção por senha - O Word Online não pode abrir
executadas. documentos que estão criptografados com uma senha.
- Documento do Word 97-2003 (.doc) ou Modelo do - Permissão para modificar - Um documento que re-
Word 97-2003 (.dot) quer uma senha para ser modificado é aberto no Word
O Word Online exibe documentos nestes formatos; po- Online no Modo de Exibição de Leitura, mas não pode ser
rém, para editar um documento no navegador, o Word On- editado no navegador. Clique em Abrir no Word para re-
line salva uma nova cópia dele no formato .docx ou .dotx. mover as restrições à edição.
O Word Online não pode salvar documentos nos formatos - Salvar - Você salva um documento manualmente no
.doc ou .dot. Word Online; não existe um recurso de salvamento auto-
- Formato de texto OpenDocument (.odt) mático.
- PDF (Portable Document Format) - Gerenciar versões - As versões são gerenciadas no
Word Online não pode abrir documentos em outros servidor, e não no Word Online. Caso o documento es-
formatos de arquivo. Por exemplo, não há suporte para
teja armazenado no OneDrive, use o recurso Histórico de
estes formatos: RTF (Formato Rich Text), linguagem HTML,
Versão do OneDrive. Caso ele esteja armazenado em uma
MHTML (MIME HTML).
biblioteca do SharePoint, verifique se o recurso Histórico
de Versão foi configurado para a biblioteca.
Recursos com suporte para exibição e impressão
Quando você exibir ou imprimir um documento no
Word Online, o documento tenha a mesma aparência como Editando e formatando
faz em Layout de impressão exiba as o Word App desktop. Copiar e colar - Copia e cola texto, e copia/cola ima-
Word Online usa um leitor de PDF para imprimir documen- gens da Web. O texto é formatado para combinar com o
tos. Se você quiser os recursos do Word para impressão texto adjacente ao local onde ele é colado no Word Online.
(como ajustar as margens da página), clique em Abrir no Formatação da fonte - Aplica negrito, itálico, sublinha-
Word e imprima no aplicativo de área de trabalho do Word. do, tachado, subscrito, sobrescrito, fonte, tamanho, cor e
Na exibição de um documento, alguns recursos fun- realce. Também é possível limpar a formatação das fontes.
cionam de maneira diferente no navegador e no aplicativo Formatação de parágrafo - Alinha parágrafos à es-
Word da área de trabalho. Para usar recursos que não estão querda, à direita ou centralizados; implementa texto da
disponíveis no Word Online, clique em Abrir no Word. direita para a esquerda ou da esquerda para a direita; au-
- Modo de Exibição de Layout de Impressão - No Modo menta ou diminui o recuo; formata parágrafos como uma
de Exibição de Leitura, o Word Online exibe a aparência lista numerada ou com marcadores. Também é possível
que um documento tem no Modo de Exibição de Layout limpar a formatação.
de Impressão. Outros modos de exibição disponíveis no Numeração e marcadores - Aplica um de diversos es-
aplicativo Word da área de trabalho (Estrutura de Tópicos, tilos internos de numeração ou marcadores.
Rascunho, Layout da Web e Leitura em Tela Inteira) não es- Estilos - Aplica uma opção de uma galeria de estilos
tão disponíveis no Word Online. Da mesma forma, o Painel internos. Também é possível limpar a formatação. Não é
de Navegação, a exibição lado a lado e as janelas divididas possível criar novos estilos ou modificar estilos no Word
não estão disponíveis no Word Online. Online.
- Réguas e linhas de grade - O Word Online não Localizar - O recurso Localizar só está disponível no
exibe réguas e linhas de grade. Modo de Exibição de Leitura. Localizar e Substituir não
está disponível no Word Online.
Recursos que diferem entre o navegador e a área
Zoom - O recurso Zoom só está disponível no Modo
de trabalho
de Exibição de Leitura. Use as configurações de exibição
Quando você edita um documento no Word Online,
do navegador para ampliar ou reduzir o zoom no Modo
ele é apresentado no Modo de Exibição de Edição para que
de Exibição de Edição.
você possa editar e formatar texto. O Modo de Exibição de
Edição não mostra a formatação da página, incluindo ele- Quebras de linha e de página - As quebras entre linhas
mentos como margens, quebras de página, folhas de rosto, e páginas são exibidas no Modo de Exibição de Leitura.
cabeçalhos e rodapés. Além disso, diversos tipos de obje- No Modo de Edição, as quebras de linha são semelhantes
tos são exibidos como espaços reservados. às quebras de parágrafo, e as quebras de página não são
As tabelas a seguir descrevem como o Word Online dá indicadas.
suporte a recursos que podem se aplicar aos seus docu- Colunas - O layout da página é preservado, mas não
mentos. Para usar recursos que não estão disponíveis no pode ser editado no Word Online.
Word Online, clique em Abrir no Word. Tema, cor da página, marcas d’água - O tema e a tela
de fundo da página são preservados no documento, mas
Abrindo e salvando não podem ser editados no Word Online.
- Gerenciamento de Direitos de Informação (IRM) - No Coautoria - Vários autores podem trabalhar simulta-
Outlook Online anexos e bibliotecas do SharePoint que são neamente no Word Online. A coautoria abrange o Word
protegidos por IRM, Word Online abre documentos para 2010 ou posterior e o Word for Mac 2011.
leitura, mas não para edição. Proteção de IRM não pode ser
adicionada aos documentos Word Online.

48
INFORMÁTICA

Revisão Cabeçalhos e rodapés - Estes objetos são exibidos no


Revisores de texto - Verifica a ortografia e define o documento conforme esperado no Modo de Exibição de
idioma de revisão, usando o dicionário interno. O Word Leitura. Cabeçalhos e rodapés, incluindo os números de
Online não usa um dicionário personalizado, nem inclui página, são ocultos no Modo de Exibição de Edição.
verificação gramatical, tradução ou um dicionário de sinô- Notas de rodapé, notas de fim, bibliografia, sumário,
nimos. índice - Estes objetos são exibidos no documento confor-
AutoCorreção - O Word Online corrige erros comuns me esperado no Modo de Exibição de Leitura. No Modo
enquanto você digita, por exemplo, trocando letras que são de Exibição de Edição, eles aparecem como espaços reser-
digitadas na ordem errada (“caxia” é corrigido para “caixa”). vados que você pode excluir, mas não editar ou atualizar.
Se o Word Online fizer uma correção indesejada, pressione Macros - Você pode exibir, editar, imprimir e comparti-
Ctrl+Z para desfazê-la. Diferentemente do aplicativo Word lhar documentos que contêm macros; porém, para execu-
da área de trabalho, o Word Online não inclui uma maneira tar as macros, clique em Abrir no Word.
de personalizar as opções de AutoCorreção. Controles ActiveX, objetos OLE inseridos, linha de assi-
Alterações controladas - As alterações controladas natura - Estes objetos são exibidos no documento confor-
aparecem no Modo de Exibição de Leitura. Para ativá-las me esperado no Modo de Exibição de Leitura. No Modo de
ou desativá-las, clique em Abrir no Word. Exibição de Edição, eles aparecem como espaços reserva-
dos que você pode excluir, mas não editar. Eles não podem
Objetos ser movidos nem redimensionados no Word Online.
Hiperlinks - Insere, edita e segue hiperlinks. Links para Imagens vinculadas, arquivos inseridos - Estes objetos
indicadores e referências cruzadas funcionam, e você pode são preservados no documento, mas são exibidos como
editar o respectivo texto de exibição, mas não o destino, no espaços reservados no Word Online.
Word Online.
Tabelas - Insere tabelas. Seleciona por tabela, coluna,
linha ou célula; exclui por tabela, coluna ou linha; insere MS-EXCEL 2016: ESTRUTURA BÁSICA
linhas e colunas; alinha o texto da célula à esquerda, à di- DAS PLANILHAS, CONCEITOS DE CÉLULAS,
reita ou centralizado. Recursos de tabela mais sofisticados, LINHAS, COLUNAS, PASTAS E GRÁFICOS,
como estilos, tamanho da célula, direção do texto e ordem ELABORAÇÃO DE TABELAS E GRÁFICOS,
de classificação, são preservados no documento, mas não USO DE FÓRMULAS, FUNÇÕES E MACROS,
podem ser configurados no Word Online.
IMPRESSÃO, INSERÇÃO DE OBJETOS,
Imagens - Inserir figuras ou clip-art armazenado no seu
computador ou no Bing. Você pode aplicar um número de
CAMPOS PREDEFINIDOS, CONTROLE DE
estilos de imagem, escrever texto alt e alterar o tamanho, QUEBRAS E NUMERAÇÃO DE PÁGINAS,
mas imagens não podem ser arrastadas para um novo local. OBTENÇÃO DE DADOS EXTERNOS,
Em vez disso, pressione Ctrl + X para recortar e pressione CLASSIFICAÇÃO DE DADOS.
Ctrl + V no novo local para colar. Recursos mais sofistica-
dos para trabalhar com imagens, como o corte, não estão
disponíveis no Word Online. Você não pode criar capturas PLANILHA ELETÔNICA MS EXCEL 2016
de tela diretamente em Word Online, mas capturas de tela Tarefas básicas no Excel2
em uma exibição de documento como imagens no Word O Excel é uma ferramenta incrivelmente poderosa para
Online. tornar significativa uma vasta quantidade de dados. Mas
Formas, gráficos, caixas de texto, SmartArt, WordArt ele também funciona muito bem para cálculos simples e
- Estes objetos são exibidos no documento conforme es- para rastrear de quase todos os tipos de informações. A
perado no Modo de Exibição de Leitura. No Modo de Exi- chave para desbloquear todo esse potencial é a grade de
bição de Edição, eles aparecem como espaços reservados células. As células podem conter números, texto ou fórmu-
que você pode excluir, mas não editar. Eles não podem ser las. Você insere dados nas células e as agrupa em linhas e
movidos nem redimensionados no Word Online. colunas. Isso permite que você adicione seus dados, classi-
Equações, símbolos, tinta - Estes objetos são exi- fique-os e filtre-os, insira-os em tabelas e crie gráficos in-
bidos no documento conforme esperado no Modo de críveis. Vejamos as etapas básicas para você começar.
Exibição de Leitura. No Modo de Exibição de Edição, eles
aparecem como espaços reservados que você pode excluir, Criar uma nova pasta de trabalho
mas não editar. Eles não podem ser movidos nem redimen- Os documentos do Excel são chamados de pastas de
sionados no Word Online. trabalho. Cada pasta de trabalho contém folhas que, nor-
Campos, controles de conteúdo, página de rosto - Es- malmente, são chamadas de planilhas. Você pode adicionar
tes objetos são exibidos no documento conforme espera- quantas planilhas desejar a uma pasta de trabalho ou pode
do no Modo de Exibição de Leitura. No modo de Edição criar novas pastas de trabalho para guardar seus dados se-
o conteúdo dos campos e os controles de conteúdo são paradamente.
exibido, mas não podem ser editados ou atualizados. Uma Clique em Arquivo e em Novo.
folha de rosto aparece como uma série de espaços reserva- Em Novo, que em Pasta de trabalho em branco
dos para elementos como caixas de texto. 2 Fonte: https://support.office.com/pt-br/excel

49
INFORMÁTICA

Aplicar um formato de número


Para distinguir entre os diferentes tipos de números,
adicione um formato, como moeda, porcentagens ou datas.
Selecione as células que contêm números que você de-
seja formatar.
Clique na guia Página Inicial e, em seguida, clique na
seta na caixa Geral.

Insira os dados
Clique em uma célula vazia.
Por exemplo, a célula A1 em uma nova planilha. As cé-
lulas são referenciadas por sua localização na linha e na Selecione um formato de número
coluna da planilha, portanto, a célula A1 fica na primeira
linha da coluna A.
Inserir texto ou números na célula.
Pressione Enter ou Tab para se mover para a célula se-
guinte.

Usar a AutoSoma para adicionar seus dados


Ao inserir números em sua planilha, talvez deseje so-
má-los. Um modo rápido de fazer isso é usar o AutoSoma.
Selecione a célula à direita ou abaixo dos números que
você deseja adicionar.
Clique na guia Página Inicial e, em seguida, clique
em AutoSoma no grupo Edição.

A AutoSoma soma os números e mostra o resultado na


célula selecionada.
Criar uma fórmula simples
Somar números é uma das coisas que você poderá fa-
zer, mas o Excel também pode executar outras operações
matemáticas. Experimente algumas fórmulas simples para Caso você não veja o formato de número que está pro-
adicionar, subtrair, multiplicar ou dividir seus valores. curando, clique em Mais Formatos de Número.
Escolha uma célula e digite um sinal de igual (=).
Isso informa ao Excel que essa célula conterá uma fór- Inserir dados em uma tabela
mula. Um modo simples de acessar grande parte dos recur-
Digite uma combinação de números e operadores de sos do Excel é colocar os dados em uma tabela. Isso per-
cálculos, como o sinal de mais (+) para adição, o sinal de mite que você filtre ou classifique rapidamente os dados.
menos (-) para subtração, o asterisco (*) para multiplicação Selecione os dados clicando na primeira célula e arras-
ou a barra (/) para divisão. tar a última célula em seus dados.
Por exemplo, insira =2+4, =4-2, =2*4 ou =4/2. Para usar o teclado, mantenha a tecla Shift pressionada
Pressione Enter. ao mesmo tempo em que pressiona as teclas de direção
Isso executa o cálculo. para selecionar os dados.
Você também pode pressionar Ctrl+Enter (se você de- Clique no botão Análise Rápida   no canto inferior
seja que o cursor permaneça na célula ativa). direito da seleção.

50
INFORMÁTICA

Para classificar os dados, clique em  Classificar de A a


Z ou Classificar de Z a A.

Clique em Tabelas, mova o cursor para o botão Tabela para


visualizar seus dados e, em seguida, clique no botão Tabela.

Clique na seta   no cabeçalho da tabela de uma coluna. Clique em OK.


Para filtrar os dados, desmarque a caixa de seleção Se-
lecionar tudo e, em seguida, selecione os dados que você Mostrar totais para os números
deseja mostrar na tabela. As ferramentas de Análise Rápida permitem que você
totalize os números rapidamente. Se for uma soma, média
ou contagem que você deseja, o Excel mostra os resultados
do cálculo logo abaixo ou ao lado dos números.
Selecione as células que contêm os números que você
somar ou contar.
Clique no botão Análise Rápida   no canto inferior
direito da seleção.
Clique em Totais, mova o cursos entre os botões para
ver os resultados dos cálculos dos dados e clique no botão
para aplicar os totais.

Adicionar significado aos seus dados


A formatação condicional ou minigráficos podem des-
tacar os dados mais importantes ou mostrar tendências de
dados. Use a ferramenta Análise Rápida para um Visualiza-
ção Dinâmica para experimentar.

51
INFORMÁTICA

Selecione os dados que você deseja examinar mais de- Se você salvou seu trabalho antes, está pronto.
talhadamente. Se esta for a primeira vez que você salva este arquivo:
Clique no botão Análise Rápida   no canto inferior Em Salvar Como, escolha onde salvar sua pasta de tra-
direito da seleção. balho e navegue até uma pasta.
Explore as opções nas guias  Formatação  e  Minigráfi- Na caixa Nome do arquivo, digite um nome para a pas-
cos para ver como elas afetam os dados. ta de trabalho.
Clique em Salvar.

Imprimir o seu trabalho


Clique em Arquivo e, em seguida, clique em Impri-
mir ou pressione Ctrl+P.
Visualize as páginas clicando nas setas Próxima Pági-
na e Página Anterior.

Por exemplo, selecione uma escala de cores na gale-


ria Formatação para diferenciar as temperaturas alta, média
e baixa. A janela de visualização exibe as páginas em preto e
branco ou colorida, dependendo das configurações de sua
impressora.
Se você não gostar de como suas páginas serão im-
pressas, você poderá mudar as margens da página ou adi-
cionar quebras de página.
Clique em Imprimir.

Localizar ou substituir texto e números em uma


Quando gostar da opção, clique nela. planilha do Excel 2016 para Windows
Localize e substitua textos e números usando curingas
Mostrar os dados em um gráfico ou outros caracteres. Você pode pesquisar planilhas, linhas,
A ferramenta Análise Rápida recomenda o gráfico cor- colunas ou pastas de trabalho.
reto para seus dados e fornece uma apresentação visual Em uma planilha, clique em qualquer célula.
com apenas alguns cliques. Na guia Página Inicial, no grupo Edição, clique em Lo-
Selecione as células contendo os dados que você quer calizar e Selecionar.
mostrar em um gráfico.
Clique no botão Análise Rápida   no canto inferior
direito da seleção.
Clique na guia Gráficos, mova entre os gráficos reco-
mendados para ver qual tem a melhor aparência para seus
dados e clique no que desejar.

Siga um destes procedimentos:


Para localizar texto ou números, clique em Localizar.
Para localizar e substituir texto ou números, clique
em Substituir.
Na caixa  Localizar, digite o texto ou os números que
você deseja procurar ou clique na seta da caixa Localizar e,
em seguida, clique em uma pesquisa recente na lista.
Você pode usar caracteres curinga, como um asterisco
OBSERVAÇÃO: O Excel mostra diferentes gráficos nesta ga- (*) ou ponto de interrogação (?), nos critérios da pesquisa:
leria, dependendo do que for recomendado para seus dados. Use o asterisco para localizar qualquer cadeia de carac-
teres. Por exemplo, s*r localizará “ser” e “senhor”.
Salvar seu trabalho Use o ponto de interrogação para localizar um único
Clique no botão Salvar na Barra de Ferramentas de caractere. Por exemplo, s?m localizará “sim” e “som”.
Acesso Rápido ou pressione Ctrl+S. DICA: Você pode localizar asteriscos, pontos de inter-
rogação e caracteres de til (~) nos dados da planilha prece-
dendo-os com um til na caixa Localizar. Por exemplo, para
localizar dados que contenham “?”, use  ~?  como critério
de pesquisa.

52
INFORMÁTICA

Clique em Opções para definir ainda mais a pesquisa e Alterar a largura da coluna e a altura da linha
siga um destes procedimentos: Em uma planilha, você pode especificar uma largura
Para procurar dados em uma planilha ou em uma de coluna de 0 (zero) a 255. Esse valor representa o nú-
pasta de trabalho inteira, na caixa Em, clique em Plani- mero de caracteres que podem ser exibidos em uma célula
lha ou Pasta de Trabalho. formatada com a fonte padrãoTE000127106. A largura de
Para pesquisar dados em linhas ou colunas, na cai- coluna padrão é 8,43 caracteres. Se a largura da coluna for
xa Pesquisar, clique em Por Linhas ou Por Colunas. definida como 0 (zero), a coluna ficará oculta.
Para procurar dados com detalhes específicos, na cai- Você pode especificar uma altura de linha de 0 (zero) a
xa Examinar, clique em Fórmulas, Valores ou Comentários. 409. Esse valor representa a medida da altura em pontos (1
OBSERVAÇÃO: As opções Fórmulas, Valores e Comen- ponto é igual a aproximadamente 1/72 pol. ou 0,035 cm).
tários só estão disponíveis na guia Localizar, e somen- A altura de linha padrão é 12,75 pontos (aproximadamente
te Fórmulas está disponível na guia Substituir. 1/6 pol. ou 0,4 cm). Se a altura da linha for definida como 0
Para procurar dados que diferenciam maiúsculas de (zero), a linha ficará oculta.
minúsculas, marque a caixa de seleção Diferenciar maiús- Se estiver trabalhando no modo de exibição de Layout
culas de minúsculas. da Página (guia Exibir, grupo Modos de Exibição da Pasta
Para procurar células que contenham apenas os carac- de Trabalho, botão Layout da Página), você poderá especi-
teres que você digitou na caixa Localizar, marque a caixa ficar uma largura de coluna ou altura de linha em polega-
de seleção Coincidir conteúdo da célula inteira. das. Nesse modo de exibição, a unidade de medida padrão
Se você deseja procurar texto ou números que tam- é polegada, mas você poderá alterá-la para centímetros ou
bém tenham uma formatação específica, clique em  For- milímetros (Na guia Arquivo, clique em Opções, clique na
mato e faça as suas seleções na caixa de diálogo Localizar categoria Avançado e, em Exibir, selecione uma opção na
Formato. lista Unidades da Régua).
DICA: Se você deseja localizar células que correspon-
dam a uma formato específico, exclua qualquer critério da
Definir uma coluna com uma largura específica
caixa Localizar e selecione a célula que contenha a for-
Selecione as colunas a serem alteradas.
matação que você deseja localizar. Clique na seta ao lado
Na guia Página Inicial, no grupo Células, clique em For-
de Formato, clique em Escolher formato da célula e, em
matar.
seguida, clique na célula que possui a formatação a ser
pesquisada.
Siga um destes procedimentos:
Para localizar texto ou números, clique em  Localizar
Tudo ou Localizar Próxima.
DICA: Quando você clicar em Localizar Tudo, todas as
ocorrências do critério que você estiver pesquisando serão
listadas, e você poderá ir para uma célula clicando nela na
lista. Você pode classificar os resultados de uma pesqui-
sa Localizar Tudo clicando em um título de coluna. Em Tamanho da Célula, clique em Largura da Coluna.
Para substituir texto ou números, digite os caracteres Na caixa Largura da coluna, digite o valor desejado.
de substituição na caixa Substituir por (ou deixe essa caixa Clique em OK.
em branco para substituir os caracteres por nada) e clique DICA: Para definir rapidamente a largura de uma única
em Localizar ou Localizar Tudo. coluna, clique com o botão direito do mouse na coluna
OBSERVAÇÃO: Se a caixa Substituir por não estiver dis- selecionada, clique em Largura da Coluna, digite o valor
ponível, clique na guia Substituir. desejado e clique em OK.
Se necessário, você poderá cancelar uma pesquisa em Alterar a largura da coluna para ajustá-la automatica-
andamento pressionando ESC. mente ao conteúdo (AutoAjuste)
Para substituir a ocorrência realçada ou todas as ocor- Selecione as colunas a serem alteradas.
rências dos caracteres encontrados, clique em Substi- Na guia Página Inicial, no grupo Células, clique em For-
tuir ou Substituir tudo. matar.
DICA: O Microsoft Excel salva as opções de formata-
ção que você define. Se você pesquisar dados na planilha
novamente e não conseguir encontrar caracteres que você
sabe que estão lá, poderá ser necessário limpar as opções
de formatação da pesquisa anterior. Na caixa de diálo-
go Localizar e Substituir, clique na guia Localizar e depois
em  Opções  para exibir as opções de formatação. Clique
na seta ao lado de Formato e clique em Limpar ‘Localizar
formato’. Em Tamanho da Célula, clique em Ajustar Largura da
Coluna Automaticamente.

53
INFORMÁTICA

OBSERVAÇÃO: Para ajustar automaticamente de forma Na guia Página Inicial, no grupo Células, clique em For-


rápida todas as colunas da planilha, clique no botão Sele- matar.
cionar Tudo e, em seguida, clique duas vezes em qualquer
limite entre dois títulos de coluna.

Em Tamanho da Célula, clique em Largura Padrão.


Na caixa Largura padrão da coluna, digite uma nova
medida e clique em OK.
DICA: Para definir a largura de coluna padrão de todas
as novas pastas de trabalho ou planilhas, você poderá criar
um modelo de pasta de trabalho ou de planilha e utilizá-lo
como base para as novas pastas de trabalho ou planilhas.
Fazer com que a largura da coluna corresponda à de
outra coluna Alterar a largura das colunas com o mouse
Selecione uma célula da coluna com a largura deseja- Siga um destes procedimentos:
da. Para alterar a largura de uma coluna, arraste o limite
Pressione Ctrl+C ou, na guia Página Inicial, no gru- do lado direito do título da coluna até que ela fique do
po Área de Transferência, clique em Copiar. tamanho desejado.

Para alterar a largura de várias colunas, selecione as


colunas desejadas e arraste um limite à direita do título de
coluna selecionado.
Para alterar a largura das colunas a fim de ajustá-la ao
Clique com o botão direito do mouse na coluna de conteúdo, selecione as colunas desejadas e clique duas ve-
destino, aponte paraColar Especial e clique no botão Man- zes no limite à direita do título de coluna selecionado.
ter Largura da Coluna Original  Para alterar a largura de todas as colunas da planilha,
clique no botãoSelecionar Tudo e arraste o limite de qual-
quer título de coluna.
Alterar a largura padrão de todas as colunas em uma
planilha ou pasta de trabalho
O valor da largura de coluna padrão indica o número
médio de caracteres da fonte padrão que cabe em uma cé-
lula. É possível especificar outro valor de largura de coluna
padrão para uma planilha ou pasta de trabalho.
Siga um destes procedimentos:
Para alterar a largura de coluna padrão de uma plani-
lha, clique na guia da planilha.
Para alterar a largura de coluna padrão da pasta de tra-
balho inteira, clique com o botão direito do mouse em uma
guia de planilha e, em seguida, clique em Selecionar Todas
as Planilhas no menu de atalhoTE000127572.

54
INFORMÁTICA

Definir uma linha com uma altura específica Para alterar a altura de várias linhas, selecione as linhas
Selecione as linhas a serem alteradas. desejadas e arraste o limite abaixo de um dos títulos de
Na guia Página Inicial, no grupo Células, clique em For- linha selecionados.
matar. Para alterar a altura de todas as linhas da planilha, cli-
que no botão Selecionar Tudo e arraste o limite abaixo de
qualquer título de linha.

Em Tamanho da Célula, clique em Altura da Linha.


Na caixa Altura da linha, digite o valor que você deseja
e, em seguida, clique em OK.
Alterar a altura da linha para ajustá-la ao conteúdo
Selecione as linhas a serem alteradas.
Na guia Página Inicial, no grupo Células, clique em For-
matar.
Para alterar a altura da linha a fim de ajustá-la ao con-
teúdo, clique duas vezes no limite abaixo do título da linha.

Formatar números como moeda no Excel 2016


Para exibir números como valores monetários, forma-
te-os como moeda. Para fazer isso, aplique o formato de
número Moeda ou Contábil às células que deseja formatar.
As opções de formatação de número estão disponíveis na
Em Tamanho da Célula, clique em AutoAjuste da Altura guia Página Inicial, no grupo Número.
da Linha.
DICA: Para ajustar automaticamente de forma rápi-
da todas as linhas da planilha, clique no botão Selecionar
Tudo e, em seguida, clique duas vezes no limite abaixo de
um dos títulos de linha.

Formatar números como moeda


Você pode exibir um número com o símbolo de moe-
da padrão selecionando a célula ou o intervalo de célu-
las e clicando em  Formato de Número de Contabilização 
  no grupo  Número  da guia  Página Inicial. (Se desejar
aplicar o formato Moeda, selecione as células e pressione
Ctrl+Shift+$.)
Alterar outros aspectos de formatação
Selecione as células que você deseja formatar.
Alterar a altura das linhas com o mouse Na guia Página Inicial, clique no Iniciador de Caixa de
Siga um destes procedimentos: Diálogo ao lado deNúmero.
Para alterar a altura de uma linha, arraste o limite
abaixo do título da linha até que ela fique com a altura
desejada.

DICA: Você também pode pressionar Ctrl+1 para abrir


a caixa de diálogoFormatar Células.

55
INFORMÁTICA

Na caixa de diálogo Formatar Células, na lista Catego- Remover formatação de moeda


ria, clique em Moedaou Contábil. Selecione as células que têm formatação de moeda.
Na guia Página Inicial, no grupo Número, clique na cai-
xa de listagem Geral.
As células formatadas com o formato Geral não têm
um formato de número específico.

Operadores e Funções

A função é um método utilizado para tornar mais fácil


e rápido a montagem de fórmulas que envolvem cálculos
mais complexos e vários valores. Existem funções para os
cálculos matemáticos, financeiros e estatísticos. Por exem-
plo, na função: =SOMA (A1:A10) seria o mesmo que (A1+
A2+A3+A4+A5+A6+A7+A8+A9+A10), só que com a fun-
ção o processo passa a ser mais fácil. Ainda conforme o
exemplo pode-se observar que é necessário sempre iniciar
um cálculo com sinal de igual (=) e usa-se nos cálculos a
referência de células (A1) e não somente valores.
A quantidade de argumentos empregados em uma
função depende do tipo de função a ser utilizada. Os ar-
Na caixa Símbolo, clique no símbolo de moeda desejado. gumentos podem ser números, textos, valores lógicos, re-
OBSERVAÇÃO: Se desejar exibir um valor monetário ferências, etc...
sem um símbolo de moeda, clique em Nenhum.
Na caixa Casas decimais, insira o número de casas de- Operadores
cimais desejadas para o número.
Por exemplo, para exibir R$138.691 em vez de colo- Operadores são símbolos matemáticos que permitem
car R$ 138.690,63 na célula, insira 0 na caixa Casas deci- fazer cálculos e comparações entre as células. Os operado-
mais. Conforme você faz alterações, preste atenção ao nú- res são:
mero na caixa Amostra. Ela mostra como a alteração das
casas decimais afetará a exibição de um número.
Na caixa Números negativos selecione o estilo de exi-
bição que você deseja usar para números negativos.
Se não quiser usar as opções existentes para exibir nú-
meros negativos, você pode criar seu próprio formato de
número.
OBSERVAÇÃO:  A caixa  Números negativos  não está
disponível para o formato de número Contábil. O motivo
disso é que constitui prática contábil padrão mostrar nú-
meros negativos entre parênteses.
Para fechar a caixa de diálogo Formatar Células, clique
em OK. Criar uma fórmula simples
Se o Excel exibir ##### em uma célula depois que você Você pode criar uma fórmula simples para adicionar,
aplicar formatação de moeda em seus dados, isso signifi- subtrair, multiplicar ou dividir valores na planilha. As fór-
cará que talvez a célula não seja suficientemente larga para mulas simples sempre começam com um sinal de igual (=),
exibir os dados. Para expandir a largura da coluna, clique seguido de constantes que são valores numéricos e ope-
duas vezes no limite direito da coluna que contém as cé- radores de cálculo como os sinais de mais (+), menos (-),
lulas com o erro #####. Esse procedimento redimensiona asterisco (*) ou barra (/).
automaticamente a coluna para se ajustar ao número. Você Por exemplo, quando você inserir a fórmula =5+2*3, o
também pode arrastar o limite direito até que as colunas Excel multiplicará os últimos dois números e adicionará o
fiquem com o tamanho desejado. primeiro número ao resultado. Seguindo a ordem padrão
das operações matemáticas, a multiplicação e executada
antes da adição.
Na planilha, clique na célula em que você deseja inserir
a fórmula.
Digite o = (sinal de igual) seguido das constantes e dos
operadores que você deseja usar no cálculo.
Você pode inserir quantas constantes e operadores fo-
rem necessários em uma fórmula, até 8.192 caracteres.

56
INFORMÁTICA

DICA : Em vez de digitar as constantes em sua fórmula, Ao selecionar um intervalo de células, você pode arras-
você pode selecionar as células que contêm os valores que tar a borda da seleção da célula para mover a seleção ou
deseja usar e inserir os operadores entre as células da seleção. arrastar o canto da borda para expandir a seleção.
Pressione Enter.
Para adicionar valores rapidamente, você pode usar
a AutoSoma em vez de inserir a fórmula manualmente
(guia Página Inicial, grupo Edição ).
Para avançar mais uma etapa, você pode usar as refe-
rências de célula e nomes em vez dos valores reais em uma
fórmula simples.
Exemplos
Copie os dados de exemplo da tabela a seguir e co-
le-os na célula A1 de uma nova planilha do Excel. Para as 1. A primeira referência de célula é B3, a cor é azul e o
fórmulas mostrarem resultados, selecione-as, pressione F2 intervalo de células tem uma borda azul com cantos qua-
e pressione Enter. Se precisar, você poderá ajustar as largu- drados.
ras das colunas para ver todos os dados. 2. A segunda referência de célula é C3, a cor é verde
e o intervalo de célula tem uma borda verde com cantos
Dados quadrados.
2 OBSERVAÇÃO : Se não houver um canto quadrado em
uma borda codificada por cor, significa que a referência
5 está relacionada a um intervalo nomeado.
Fórmula Descrição Resultado Pressione Enter.
Adiciona os valores nas DICA :  Você também pode inserir uma referência a um
‘=A2+A3 =A2+A3
células A1 e A2 intervalo ou célula nomeada.
Subtrai o valor na célula Exemplo
‘=A2-A3 =A2-A3
A2 do valor em A1 Copie os dados de exemplo da tabela a seguir e cole-
Divide o valor na célula -os na célula A1 de uma nova planilha do Excel. Para as fór-
‘=A2/A3 =A2/A3 mulas mostrarem resultados, selecione-as, pressione F2 e
A1 pelo valor em A2
Multiplica o valor na cé- pressione Enter. Se precisar, você poderá ajustar as larguras
‘=A2*A3 =A2*A3 das colunas para ver todos os dados. Use o comando De-
lula A1 pelo valor em A2
finir Nome (guia  Formulas, grupo  Nomes Definidos) para
Eleva o valor na célula A1
definir “Ativos” (B2:B4) e “Passivos” (C2:C4).
‘=A2^A3 ao valor exponencial es- =A2^A3
pecificado em A2
Fórmula Descrição Resultado Departamento Ativos Passivos
‘=5+2 Adiciona 5 e 2 =5+2 TI 274000 71000
‘=5-2 Subtrai 2 de 5 =5-2 Administrador 67000 18000
‘=5/2 Divide 5 por 2 =5/2 RH 44000 3000
‘=5*2 Multiplica 5 vezes 2 =5*2 Fórmula Descrição Resultado
Eleva 5 à segunda potên- Retorna o to-
‘=5^2 =5^2 tal de ativos
cia
dos três de-
Usar referências de célula em uma fórmula partamentos
Ao criar uma fórmula simples ou uma fórmula que usa no nome defi-
uma função, você pode fazer referência aos dados das cé- ‘=SOMA(Ativos) nido “Ativos”, =SOMA(Ativos)
lulas de uma planilha incluindo referências de célula nos que é defi-
argumentos da fórmula. Por exemplo, quando você insere nido como o
ou seleciona a referência de célula A2, a fórmula usa o valor intervalo de
dessa célula para calcular o resultado. Você também pode células B2:B4.
fazer referência a um intervalo de células. (385000)
Clique na célula na qual deseja inserir a fórmula. Subtrai a
soma do
Na barra de fórmulas  , digite = (sinal de igual). nome defini-
Faça o seguinte: selecione a célula que contém o valor ‘=SOMA(Ativos)- do “Passivos” =SOMA(Ativos)
desejado ou digite sua referência de célula. -SOMA(Passivos) da soma do -SOMA(Passivos)
Você pode fazer referência a uma única célula, a um nome defini-
intervalo de células, a um local em outra planilha ou a um do “Ativos”.
local em outra pasta de trabalho. (293000)

57
INFORMÁTICA

Criar uma fórmula usando uma função Dados


Você pode criar uma fórmula para calcular valores
5 4
na planilha usando uma função. Por exemplo, as fórmu-
las =SOMA(A1:A2) e SOMA(A1,A2) usam ambas a função 2 6
SOMA para adicionar os valores nas células A1 e A2. As 3 8
fórmulas sempre começam com um sinal de igual (=) 7 1
Clique na célula na qual deseja inserir a fórmula. Fórmula Descrição Resultado
Para iniciar a fórmula com a função, clique em Inserir Adiciona todos
Função  . ‘=SOMA(A:A) os números na =SOMA(A:A)
O Excel insere o sinal de igual (=) para você. coluna A
Na caixa Ou selecione uma categoria, selecione Tudo.
Calcula a mé-
dia de todos
‘=MÉDIA(A1:B4) =MÉDIA(A1:B4)
os números no
intervalo A1:B4

Função Soma
A função soma , uma das funções matemáticas e trigo-
nométricas, adiciona os valores. Você pode adicionar valo-
res individuais, referências de células ou intervalos ou uma
mistura de todos os três.
Este vídeo faz parte de um curso de treinamento cha-
mado Adicionar números no Excel 2013.
Sintaxe: SOMA(número1,[número2],...)
Por exemplo:
=SUM(A2:A10)
= SUM(A2:A10, C2:C10)

Nome do
Descrição
argumento
O primeiro número que você
deseja somar. O número pode
número1   
Se você estiver familiarizado com as categorias de fun- ser como “4”, uma referência de
(Obrigatório)
ção, também poderá selecionar uma categoria. célula, como B6, ou um intervalo
Se você não tiver certeza de qual função usar, poderá de células, como B2:B8.
digitar uma pergunta que descreva o que deseja fazer, na Este é o segundo número que
caixa Procure por uma função (por exemplo, “adicionar nú- número2-255    você deseja adicionar. Você pode
meros” retorna a função SOMA). (Opcional) especificar até 255 números des-
Na caixa Selecione uma função, selecione a função que sa forma.
deseja utilizar e clique em OK.
Nas caixas de argumento que forem exibidas para a Soma rápida com a barra de Status
função selecionada, insira os valores, as cadeias de caracte- Se você quiser obter rapidamente a soma de um inter-
res de texto ou referências de célula desejadas. valo de células, tudo o que você precisa fazer é selecionar o
Em vez de digitar as referências de célula, você tam- intervalo e procure no lado inferior direito da janela do Excel.
bém pode selecionar as células que deseja referenciar. Cli-
que em   para expandir novamente a caixa de diálogo.
Depois de concluir os argumentos para a fórmula, cli-
que em OK.
DICA :  Se você usar funções frequentemente, poderá
inserir suas fórmulas diretamente na planilha. Depois de di-
gitar o sinal de igual (=) e o nome da função, poderá obter
informações sobre a sintaxe da fórmula e os argumentos
da função pressionando F1.
Exemplos
Copie a tabela para a célula A1 em uma planilha em
branco no Excel para trabalhar com esses exemplos de fór-
mulas que usam funções.

58
INFORMÁTICA

Barra de Status Exemplo 2 – AutoSoma verticalmente


Esta é a barra de Status e exibe as informações sobre
tudo o que você selecionou, se você tiver uma única célula
ou várias células. Se você com o botão direito na barra de
Status de uma caixa de diálogo do recurso será pop-out
exibindo todas as opções que você pode selecionar. Obser-
ve que ele também exibe valores para o intervalo selecio-
nado se você tiver esses atributos marcados. 

Usando o Assistente de AutoSoma


A maneira mais fácil de adicionar uma fórmula de soma
à sua planilha é usar o Assistente de AutoSoma. Selecione
uma célula vazia diretamente acima ou abaixo do intervalo
que você quer somar e nas guias página inicial ou fórmula
na faixa de opções, pressione AutoSoma > soma. O Assis-
tente de AutoSoma automaticamente detecta o intervalo
para ser somados e criar a fórmula para você. Ele também
pode trabalhar horizontalmente se você selecionar uma
célula para a esquerda ou à direita do intervalo a serem AutoSoma verticalmente
somados. O Assistente de AutoSoma detectou automaticamente
células B2: B5 como o intervalo a serem somados. Tudo o
que você precisa fazer é pressione Enter para confirmá-la.
Se você precisar adicionar/excluir mais células, você pode
manter a tecla Shift > tecla de direção da sua escolha até
que corresponde à sua seleção desejado e pressione Enter
quando terminar.
Guia de função Intellisense: a soma (Número1, [núm2])
flutuantes marca abaixo a função é o guia de Intellisense.
Se você clicar no nome de função ou soma, ele se trans-
formará em um hiperlink azul, que o levará para o tópico
de ajuda para essa função. Se você clicar nos elementos
de função individual, as peças representantes na fórmula
serão realçadas. Nesse caso somente B2: B5 seria realçadas
como há apenas uma referência numérica nesta fórmula.
A marca de Intellisense será exibido para qualquer função.

Exemplo 3 – AutoSoma horizontalmente

Use o Assistente de AutoSoma para rapidamente os


intervalos contíguos de soma
A caixa de diálogo de AutoSoma também permite que
você selecione outras funções comuns, como:
Média
Contar números
Máx
Min
Mais funções

59
INFORMÁTICA

AutoSoma horizontalmente Em seguida, tente validar se suas entradas estão cor-


Exemplo 4 – somar células não-contíguas retas. É muito mais fácil colocar esses valores em células
individuais e usar uma fórmula de soma. Além disso, você
pode formatar os valores quando eles estão nas células,
tornando-as muito mais legível e quando ela estiverem em
uma fórmula.

Somar células não-contíguas


O Assistente de AutoSoma geralmente só funcionará
para intervalos contíguos, portanto se você tiver linhas ou
colunas vazias no seu intervalo de soma, Excel vai parada o
primeiro espaço. Nesse caso você precisaria soma por sele-
ção, onde você pode adicionar os intervalos individuais, um #VALUE! erros de referência de texto em vez de nú-
por vez. Neste exemplo se você tivesse dados na célula B4, meros.
o Excel seria gerar =SUM(C2:C6) desde que ele reconheça Se você usar um fórmula como:
um intervalo contíguo. = A1 + B1 + C1 ou = A1 + A2 + A3
Você pode selecionar rapidamente vários intervalos
não-contíguas com Ctrl + LeftClick. Primeiro, insira “= soma
(“, selecione seu diferentes intervalos e Excel adicionará au-
tomaticamente o separador de vírgula entre intervalos para
você. Pressione enter quando terminar.
Dica: você pode usar ALT + = para adicionar rapida-
mente a função soma para uma célula. Tudo o que você
precisa fazer é selecionar o intervalo (s).
Observação: você pode perceber como o Excel tem
realçado os intervalos de função diferente por cor, e eles
correspondem dentro da própria fórmula, portanto C2: C3 Evite usar o = 1 + 2 ou = métodos A + B
é azul e C5: C6 é vermelho. Excel fará isso para todas as Sua fórmula pode quebrar se houver quaisquer valores
funções, a menos que o intervalo referenciado esteja em não numéricos (texto) nas células referenciadas, que retor-
uma planilha diferente ou em outra pasta de trabalho. Para narão um #VALUE! erro. SOMA ignorará valores de texto e
acessibilidade aprimorada com tecnologia assistencial, lhe dar a soma dos valores numéricos.
você pode usar intervalos nomeados, como “Semana1”,
“Semana2”, etc. e, em seguida, fazer referência a eles, sua
fórmula:
=SUM(Week1,Week2)

Práticas Recomendadas
Esta seção aborda algumas práticas recomendadas
para trabalhar com as funções soma. Grande parte desse
pode ser aplicada a trabalhar com outras funções também.
♣ a = 1 + 2 ou = A + B Method – enquanto você pode
inserir = 1 + 2 + 3 ou = A1 + B1 + C2 e obter resultados
totalmente precisos, esses métodos estão sujeitos a erros
por vários motivos:
Erros de digitação – Imagine tentando inserir valores
mais e/ou muito maiores assim:
= 14598.93 + 65437.90 + 78496.23

60
INFORMÁTICA

SOMA ignora valores de texto Usando uma fórmula como:


#REF! Erro de excluir linhas ou colunas =SUM(A1,A2,A3,B1,B2,B3)
É igualmente erro sujeitos ao inserir ou excluir linhas
dentro do intervalo referenciada pelas mesmas razões. É
muito melhor usar intervalos individuais, como:

=SUM(A1:A3,B1:B3)
Qual será atualizada ao adicionar ou excluir linhas.
Usando operadores matemáticos com soma
Digamos que você deseja aplicar uma porcentagem de
desconto para um intervalo de células que você já somados.
Se você excluir uma linha ou coluna, a fórmula não serão
atualizados para excluir a linha excluída e retornará um #REF!
erro, onde uma função soma atualizará automaticamente.

Fórmulas não atualizar referências ao inserir linhas ou


colunas

= Style A + B fórmulas não os atualizará ao adicionar


linhas ou colunas = SUM(A2:A14) *-25%
Se você inserir uma linha ou coluna, a fórmula não será Resultará em 25% do intervalo somado, entretanto,
atualizada para incluir a linha adicionada, onde uma função que rígido códigos a 25% da fórmula, e pode ser difícil
soma atualizará automaticamente (contanto que você não encontrar mais tarde se precisar alterá-lo. É muito melhor
estiver fora do intervalo referenciado na fórmula). Isso é es- colocar os 25% em uma célula e referenciando que em vez
pecialmente importante se você esperar sua fórmula para disso, onde ele está check-out em Abrir e facilmente alte-
atualizar e não, como ele deixará incompletos resultados rados, assim:
que você não pode capturar. = SUM(A2:A14) * E2
Dividir em vez de multiplicar você simplesmente subs-
titua a “*” com “/”: = SUM(A2:A14)/E2
Adicionando ou retirando de uma soma
i. você pode facilmente adicionar ou subtrair de uma
soma usando + ou - assim:
= SUM(A1:A10) + E2
= SUM(A1:A10)-E2

SOMA 3D
SOMA com referências de célula individuais versus in- Às vezes você precisa somar uma determinada célula
tervalos em várias planilhas. Pode ser tentador clique em cada pla-
nilha e a célula desejada e use apenas «+» para adicionar a
célula valores, mas que é entediante e pode ser propensa,
muito mais assim que apenas tentando construir uma fór-
mula que faz referência apenas uma única folha.

61
INFORMÁTICA

i = Planilha1! A1 + Plan2! A1 + Planilha3! A1


Você pode fazer isso muito mais fácil com um 3D ou soma 3 dimensionais:

= SUM(Sheet1:Sheet3! A1)
Qual somará a célula A1 em todas as planilhas da planilha 1 para a planilha 3.
Isso é especialmente útil em situações em que você tem uma única folha para cada mês (janeiro a dezembro) e você
precisa total-las em uma planilha de resumo.

= SUM(January:December! A2)
Qual somará célula A2 em cada planilha de janeiro a dezembro.

Observação: se suas planilhas tem espaços em seus nomes, como “Janeiro vendas”, então você precisa usar um após-
trofo ao fazer referência os nomes de planilha em uma fórmula:
= SUM(‘January Sales:December Sales’! A2)
SOMA com outras funções
Absolutamente, você pode usar soma com outras funções. Aqui está um exemplo que cria um cálculo da média mensal:

=SUM(A2:L2)/COUNTA(A2:L2)
Que usa a soma de A2:L2 dividido pela contagem de células não vazias em A2:L2 (maio a dezembro estão em branco).

62
INFORMÁTICA

Função SE
A função SE é uma das funções mais populares do Excel e permite que você faça comparações lógicas entre um valor
e aquilo que você espera. Em sua forma mais simples, a função SE diz:
SE(Algo for Verdadeiro, faça tal coisa, caso contrário, faça outra coisa)
Portanto, uma instrução SE pode ter dois resultados. O primeiro resultado é se a comparação for Verdadeira, o segundo
se a comparação for Falsa.

Exemplos de SE simples

=SE(C2=”Sim”;1,2)
No exemplo acima, a célula D2 diz: SE(C2 = Sim, a fórmula retorna um 1 ou um 2)

=SE(C2=1;”Sim”;”Não”)
Neste exemplo, a fórmula na célula D2 diz: SE(C2 = 1, a fórmula retorna Sim e, caso contrário, retorna Não)
Como você pode ver, a função SE pode ser usada para avaliar texto e valores. Ela também pode ser usada para avaliar
erros. Você não está limitado a verificar apenas se um valor é igual a outro e retornar um único resultado; você também
pode usar operadores matemáticos e executar cálculos adicionais dependendo de seus critérios. Também é possível ani-
nhar várias funções SE juntas para realizar várias comparações.
OBSERVAÇÃO : Se você for usar texto em fórmulas, será preciso quebrar o texto entre aspas (por exemplo, “Texto”). A
única exceção é usar VERDADEIRO ou FALSO que o Excel reconhece automaticamente.

Introdução
A melhor maneira de começar a escrever uma instrução SE é pensar sobre o que você está tentando realizar. Que com-
paração você está tentando fazer? Muitas vezes, escrever uma instrução SE pode ser tão simples quanto pensar na lógica
em sua cabeça: “o que aconteceria se essa condição fosse atendida vs. o que aconteceria se não fosse?” Você sempre deve
se certificar de que suas etapas sigam uma progressão lógica; caso contrário, sua fórmula não executará aquilo que você
acha que ela deveria executar. Isso é especialmente importante quando você cria instruções SE complexas (aninhadas).

63
INFORMÁTICA

Mais exemplos de SE

=SE(C2>B2;”Acima do orçamento”;”Dentro do orçamento”)


No exemplo acima, a função SE em D2 está dizendo SE(C2 é maior que B2, retorne “Acima do orçamento”, caso con-
trário, retorne “Dentro do orçamento”)

= SE(C2>B2,C2-B2,0)
Na ilustração acima, em vez de retornar um resultado de texto, vamos retornar um cálculo matemático. A fórmula em E2
está dizendo SE(Valor real for maior que o Valor orçado, subtraia o Valor orçado do Valor real, caso contrário, não retorne nada).

=SE(E7=”Sim”;F5*0,0825;0)
Neste exemplo, a fórmula em F7 está dizendo SE(E7 = “Sim”, calcule o Valor Total em F5 * 8,25%, caso contrário, ne-
nhum Imposto sobre Vendas é cobrado, retorne 0)

64
INFORMÁTICA

Práticas Recomendadas - Constantes


No último exemplo, você vê dois “Sim” e a Taxa de Imposto sobre Vendas (0,0825) inseridos diretamente na fórmula.
Geralmente, não é recomendável colocar constantes literais (valores que talvez precisem ser alterados ocasionalmente) di-
retamente nas fórmulas, pois elas podem ser difíceis de localizar e alterar no futuro. É muito melhor colocar constantes em
suas próprias células, onde elas ficam fora das constantes abertas e podem ser facilmente encontradas e alteradas. Nesse
caso, tudo bem, pois há apenas uma função SE e a Taxa de Imposto sobre Vendas raramente será alterada. Mesmo se isso
acontecer, será fácil alterá-la na fórmula.
Usar SE para verificar se uma célula está em branco
Às vezes, é preciso verificar se uma célula está em branco, geralmente porque você pode não querer uma fórmula exiba
um resultado sem entrada.

Nesse caso, usamos SE com a função ÉCÉL.VAZIA:


=SE(ÉCÉL.VAZIA(D2),”Em branco”,”Não está em branco”)
Que diz SE(D2 está em branco, retorne “Em branco”, caso contrário, retorne “Não está em branco”). Você também poderia
facilmente usar sua própria fórmula para a condição “Não está em branco”. No próximo exemplo usamos “” em vez de ÉCÉL.
VAZIA. “” basicamente significa “nada”.

=SE(D3=””,”Em branco”,”Não está em branco”)


Essa fórmula diz SE(D3 é nada, retorne “Em branco”, caso contrário, retorne “Não está em branco”). Veja um exemplo
de um método muito comum de usar “” para impedir que uma fórmula calcule se uma célula dependente está em branco:
=SE(D3=””;””;SuaFórmula())
SE(D3 é nada, retorne nada, caso contrário, calcule sua fórmula).
Exemplo de SE aninhada

65
INFORMÁTICA

Em casos onde uma simples função SE tem apenas dois Valores em branco e texto são ignorados. O intervalo selecio-
resultados (Verdadeiro ou Falso), as funções se aninhadas SE nado pode conter datas no formato padrão do Excel (exem-
podem ter de 3 a 64 resultados. plos abaixo).
=SE(D2=1;”SIM”;SE(D2=2;”Não”;”Talvez”)) critérios   Obrigatório. Os critérios na forma de um
Na ilustração acima, a fórmula em E2 diz: SE(D2 é igual número, expressão, referência de célula, texto ou função
a 1, retorne “Sim”, caso contrário, SE(D2 é igual a 2, retorne que define quais células serão adicionadas. Por exemplo,
“Não”, caso contrário, retorne “Talvez”). os critérios podem ser expressos como 32, “>32”, B5, “32”,
“maçãs” ou HOJE().
CONT.SE IMPORTANTE : Qualquer critério de texto ou qualquer
Use CONT.SE, uma das funções estatísticas, para contar o critério que inclua símbolos lógicos ou matemáticos deve
número de células que atendem a um critério; por exemplo, estar entre aspas duplas (“). Se os critérios forem numéri-
para contar o número de vezes que uma cidade específica cos, as aspas duplas não serão necessárias.
aparece em uma lista de clientes. intervalo_soma   Opcional. As células reais a serem adi-
cionadas, se você quiser adicionar células diferentes das
Sintaxe especificadas no argumento intervalo. Se o argumento in-
CONT.SE(intervalo, critério) tervalo_soma for omitido, o Excel adicionará as células es-
Por exemplo: pecificadas no argumento intervalo (as mesmas células às
=CONT.SE(A2:A5;”maçãs”) quais os critérios são aplicados).
=CONT.SE(A2:A5,A4) Você pode usar os caracteres curinga – o ponto de in-
terrogação (?) e o asterisco (*) – como o argumento crité-
Nome do argumento Descrição rios. O ponto de interrogação corresponde a qualquer ca-
ractere único; o asterisco corresponde a qualquer sequên-
O grupo de células que
cia de caracteres. Para localizar um ponto de interrogação
você deseja contar. Inter-
ou asterisco real, digite um til (~) antes do caractere.
valo  pode conter números,
Comentários
matrizes, um intervalo no- A função SOMASE retorna valores incorretos quando
intervalo   
meado ou referências que você a utiliza para corresponder cadeias de caracteres com
(obrigatório)
contenham números. Valo- mais de 255 caracteres ou para a cadeia de caracteres #VA-
res em branco e texto são LOR!.
ignorados. O argumento intervalo_soma não precisa ter o mesmo
tamanho e forma que o argumento intervalo. As células
Um número, expressão, re- reais adicionadas são determinadas pelo uso da célula na
ferência de célula ou cadeia extremidade superior esquerda do argumentointervalo_
de texto que determina soma como a célula inicial e, em seguida, pela inclusão das
quais células serão conta- células correspondentes em termos de tamanho e forma
das. no argumento intervalo. Por exemplo:
Por exemplo, você pode
usar um número como 32, Se o intervalo e intervalo_ Então, as células
critérios    (obrigatório)
uma comparação, como “> for soma for reais serão
32”, uma célula como B4 ou
A1:A5 B1:B5 B1:B5
uma palavra como “maçãs”.
CONT.SE usa apenas um A1:A5 B1:B3 B1:B5
único critério. Use  CONT. A1:B4 C1:D4 C1:D4
SES se você quiser usar vá- A1:B4 C1:C2 C1:D4
rios critérios.
Porém, quando os argumentos  intervalo  e  intervalo_
Função SOMASE soma  na função SOMASE não contêm o mesmo número
Você pode usar a função SOMASE para somar os valores de células, o recálculo da planilha pode levar mais tempo
em uma intervalo que atendem aos critérios que você especi- do que o esperado.
ficar. Por exemplo, suponha que, em uma coluna que contém
números, você quer somar apenas os valores que são maio- MÁXIMO (Função MÁXIMO)
res do que 5. Você pode usar a seguinte fórmula: = SOMASE Descrição
(B2:B25,”> 5”). Retorna o valor máximo de um conjunto de valores.
Sintaxe
Sintaxe MÁXIMO(número1, [número2], ...)
SOMASE(intervalo, critérios, [intervalo_soma]) A sintaxe da função MÁXIMO tem os seguintes argu-
A sintaxe da função SOMASE tem os seguintes argumentos: mentos:
intervalo    Necessário. O intervalo de células a ser avalia- Núm1, núm2,...     Núm1 é obrigatório, números sub-
da por critérios. Células em cada intervalo devem ser números sequentes são opcionais. De 1 a 255 números cujo valor
ou nomes, matrizes ou referências que contenham números. máximo você deseja saber.

66
INFORMÁTICA

Comentários Se os argumentos não contiverem números, MÍNIMO


Os argumentos podem ser números, nomes, matrizes retornará 0.
ou referências que contenham números. Os argumentos que são valores de erro ou texto que
Os valores lógicos e representações em forma de texto não podem ser traduzidos em números causam erros.
de números digitados diretamente na lista de argumentos Se você deseja incluir valores lógicos e representações
são contados. de texto dos números em uma referência como parte do
Se um argumento for uma matriz ou referência, apenas cálculo, utilize a função MÍNIMOA.
os números nesta matriz ou referência serão usados. Célu- Exemplo
las vazias, valores lógicos ou texto na matriz ou referência Copie os dados de exemplo da tabela a seguir e cole-os
serão ignorados. na célula A1 de uma nova planilha do Excel. Para as fór-
Se os argumentos não contiverem números, MÁXIMO mulas mostrarem resultados, selecione-as, pressione F2 e
retornará 0. pressione Enter. Se precisar, você poderá ajustar as larguras
Os argumentos que são valores de erro ou texto que das colunas para ver todos os dados.
não podem ser traduzidos em números causam erros.
Se você deseja incluir valores lógicos e representações Dados
de texto dos números em uma referência como parte do 10
cálculo, utilize a função MÁXIMOA.
7
Exemplo
Copie os dados de exemplo da tabela a seguir e co- 9
le-os na célula A1 de uma nova planilha do Excel. Para as 27
fórmulas mostrarem resultados, selecione-as, pressione F2 2
e pressione Enter. Se precisar, você poderá ajustar as largu- Fórmula Descrição Resultado
ras das colunas para ver todos os dados. O menor dos
=MÍNIMO(A2:A6) números no 2
Dados intervalo A2:A6.
10 O menor dos
7 =MIN(A2:A6;0) números no 0
9 intervalo A2:A6 e 0.
27
Média
2 Calcula a média aritmética de uma seleção de valores.
Fórmula Descrição Resultado Em nossa planilha clique na célula abaixo da coluna de ida-
Maior valor no de na linha de valores máximos E17 e monte a seguinte função
=MÁXIMO(A2:A6) 27
intervalo A2:A6. =MEDIA(E4:E13). Com essa função estamos buscando no in-
Maior valor no tervalo das células E4 à E13 qual é valor máximo encontrado.
=MÁXIMO(A2:A6;
intervalo A2:A6 e 30
30) Mesclar células
o valor 30.
A mesclagem combina duas ou mais células para criar
MÍNIMO (Função MÍNIMO) uma nova célula maior. Essa é uma excelente maneira de
Descrição criar um rótulo que se estende por várias colunas. Por
Retorna o menor número na lista de argumentos. exemplo, aqui, as células A1, B1 e C1 foram mescladas para
Sintaxe criar o rótulo “Vendas Mensais” e descrever as informações
MÍNIMO(número1, [número2], ...) nas linhas de 2 a 7.
A sintaxe da função MÍNIMO tem os seguintes argu-
mentos:
Núm1, núm2,...     Núm1 é obrigatório, números sub-
sequentes são opcionais. De 1 a 255 números cujo valor
MÍNIMO você deseja saber.
Comentários
Os argumentos podem ser números, nomes, matrizes
ou referências que contenham números.
Os valores lógicos e representações em forma de texto
de números digitados diretamente na lista de argumentos
são contados.
Se um argumento for uma matriz ou referência, ape-
nas os números daquela matriz ou referência poderão ser
usados. Células vazias, valores lógicos ou valores de erro na
Selecione duas ou mais células adjacentes que você
matriz ou referência serão ignorados.
deseja mesclar.

67
INFORMÁTICA

IMPORTANTE : Verifique se os dados que você deseja agrupar na célula mesclada estão contidos na célula superior
esquerda. Os dados nas outras células mescladas serão excluídos. Para preservar os dados das outras células, copie-os em
outra parte da planilha antes de fazer a mesclagem.
Clique em Início > Mesclar e Centralizar.

Se o botão Mesclar e Centralizar estiver esmaecido, verifique se você não está editando uma célula e se as células que
você quer mesclar não estão dentro de uma tabela.
DICA :  Para mesclar células sem centralizar, clique na seta ao lado de Mesclar e Centralizar e clique em Mesclar Atra-
vés ou Mesclar Células.
Se você mudar de ideia, é possível dividir as células que foram mescladas.

Criar um gráfico no Excel 2016 para Windows


Use o comando Gráficos Recomendados na guia Inserir para criar rapidamente um gráfico ideal para seus dados.
Selecione os dados que você deseja incluir no seu gráfico.
Clique em Inserir > Gráficos Recomendados.

Na guia Gráficos Recomendados, percorra a lista de tipos de gráficos recomendados pelo Excel para seus dados.
Clique em qualquer tipo de gráfico para ver como os seus dados aparecem naquele formato.

68
INFORMÁTICA

DICA : Se não vir um tipo de gráfico que agrade você, clique na guia Todos os Gráficos para ver todos os tipos de gráfico
disponíveis.
Quando você encontrar o tipo de gráfico desejado, clique nele e clique emOK.
Use os botões Elementos do Gráfico, Estilos de Gráfico e Filtros de Gráficopróximos ao canto superior direito do gráfico
para adicionar elementos de gráfico como títulos de eixo ou rótulos de dados, para personalizar a aparência do seu gráfi-
co ou alterar os dados exibidos no gráfico.

DICAS : 
Use as opções nas guias Design e Formatar para personalizar a aparência do gráfico.

Se você não vir essas guias, adicione as Ferramentas de gráfico à faixa de opções clicando em qualquer lugar no gráfico.

69
INFORMÁTICA

Criar uma Tabela Dinâmica no Excel 2016 para analisar dados da planilha
A capacidade de analisar todos os dados da planilha pode ajudar você a tomar decisões de negócios melhores. Porém,
às vezes é difícil saber por onde começar, especialmente quando há muitos dados. O Excel pode ajudar, recomendando e,
em seguida, criando automaticamente Tabelas Dinâmicas que são um excelente recurso para resumir, analisar, explorar e
apresentar dados. Por exemplo, veja uma lista simples de despesas:

Aqui estão os mesmos dados resumidos em uma Tabela Dinâmica:

Criar uma Tabela Dinâmica Recomendada


Se você tiver experiência limitada com Tabelas Dinâmicas ou não souber como começar, uma Tabela Dinâmica Reco-
mendada é uma boa opção. Quando você usa este recurso, o Excel determina um layout significativo, combinando os da-
dos com as áreas mais adequadas da Tabela Dinâmica. Isso oferece um ponto inicial para experimentos adicionais. Depois
que uma Tabela Dinâmica básica é criada, você pode explorar orientações diferentes e reorganizar os campos para obter
os resultados desejados.
Abra a pasta de trabalho onde você deseja criar a Tabela Dinâmica.
Clique em uma célula na lista ou tabela que contém os dados que serão usados na Tabela Dinâmica.
Na guia Inserir, clique em Tabela Dinâmica Recomendada.

70
INFORMÁTICA

O Excel cria uma Tabela Dinâmica em uma nova planilha e exibe a Lista de Campos da Tabela Dinâmica.
Siga um destes procedimentos:

Para Faça isto


Na área NOME DO CAMPO, marque a caixa de seleção para o campo. Por pa-
drão, campos não numéricos são adicionados à áreaLinha, as hierarquias de
Adicionar um campo
data e hora são adicionadas à área Coluna e os campos numéricos são adicio-
nados à área Valores.
Remover um campo Na área NOME DO CAMPO, desmarque a caixa de seleção para o campo.
Arraste o campo de uma área da Lista de Campos da Tabela Dinâmica para ou-
Mover um campo
tra, por exemplo, de Colunas para Linhas.
Atualizar a Tabela Dinâmica Na guia Analisar Tabela Dinâmica, clique em Atualizar.

Criar uma Tabela Dinâmica manualmente


Se você sabe como organizar seus dados, pode criar uma Tabela Dinâmica manualmente.
Abra a pasta de trabalho onde você deseja criar a Tabela Dinâmica.
Clique em uma célula na lista ou tabela que contém os dados que serão usados na Tabela Dinâmica.
Na guia Inserir, clique em Tabela Dinâmica.

Na planilha, os seus dados devem estar envolvidos por uma linha tracejada. Se não estiverem, clique e arraste para
selecionar os dados. Quando você fizer isso, a caixa Tabela/Intervalo será preenchida automaticamente com o intervalo de
células selecionado.
Em Escolher onde deseja que o relatório de tabela dinâmica seja colocado, escolha Nova planilha para colocar a Tabela
Dinâmica em uma nova guia de planilha. Se preferir, clique em Planilha existente e clique na planilha para especificar o local.
DICA : Para analisar várias tabelas em uma Tabela Dinâmica, marque a caixaAdicionar estes dados ao Modelo de Dados.
Clique em OK.

71
INFORMÁTICA

Na Lista de Campos da Tabela Dinâmica, siga um destes procedimentos:

Para Faça isto


Na área NOME DO CAMPO, marque a caixa de seleção para o campo. Por padrão, campos
Adicionar um campo não numéricos são adicionados à áreaLinha, as hierarquias de data e hora são adicionadas à
área Colunae os campos numéricos são adicionados à área Valores.
Remover um campo Na área NOME DO CAMPO, desmarque a caixa de seleção para o campo.
Arraste o campo de uma área da Lista de Campos da Tabela Dinâmica para outra, por exemplo,
Mover um campo
de Colunas para Linhas.
Clique na seta ao lado do campo em Valores > Definições do Campo de Valor e, na caixa Defi-
nições do Campo de Valor, altere o cálculo.

Alterar o cálculo usado


em um campo de valor

Atualizar a Tabela Dinâ-


Na guia Analisar Tabela Dinâmica, clique em Atualizar.
mica

Proteger com senha uma pasta de trabalho


O Excel oferece várias maneiras de proteger uma pasta de trabalho. Você pode solicitar uma senha para abri-la, uma
senha para alterar dados e uma senha para alterar a estrutura do arquivo (adicionar, excluir ou ocultar planilhas). Você pode
também definir uma senha no modo de exibição Backstage para criptografar a pasta de trabalho.
Lembre-se, no entanto, de que esse tipo de proteção nem sempre criptografa os seus dados. Isso só é possível com a
senha criptografada criada no modo de exibição Backstage. Os usuários podem ainda usar ferramentas de terceiros para
ler dados não criptografados.
Vamos começar solicitando senhas para abrir um arquivo e alterar dados.
Clique em Arquivo > Salvar como.
Clique em um local, como Computador ou a página da Web Meu Site.
Clique em uma pasta, como Documentos ou uma das pastas no seu OneDrive, ou clique em Procurar.
Na caixa de diálogo Salvar como, vá até a pasta que você quer usar, abra a lista Ferramentas e clique em Opções Gerais.
Insira a sua senha e clique em OK. Insira a mesma senha para confirmar e clique novamente em OK.
OBSERVAÇÃO:  Para remover uma senha, siga as etapas acima e exclua a senha. Basicamente, basta deixar a senha em
branco. Você pode fazer isso para qualquer tipo de senha usado no Excel.

72
INFORMÁTICA

Você pode digitar uma das duas senhas aqui, uma para abrir o arquivo, outra para mudar o arquivo.

Consulte as anotações abaixo para mais informações.


Para proteger a estrutura da sua pasta de trabalho, faça isto:
Clique em Revisar > Proteger Pasta de Trabalho.
Clique em Estrutura.
Consulte as anotações abaixo para saber mais sobre essa opção e a opçãoWindows.
Digite uma senha na caixa Senha.
Clique em OK e redigite a senha para confirmá-la.
OBSERVAÇÕES : 
Se você digitar a mesma senha para abrir e alterar a pasta de trabalho, os usuários somente precisarão digitar a senha
uma vez.
Se você solicitar somente uma senha para alterar a pasta de trabalho, os usuários podem abrir uma cópia somente
leitura do arquivo, salvá-la com outro nome e alterar seus dados.
Selecionar a opção Estrutura previne outros usuários de visualizar planilhas ocultas, adicionar, mover, excluir ou ocultar
planilhas e renomear planilhas.
Você pode ignorar a opção Windows. Ela está desabilitada nessa versão do Excel.
Sempre é possível saber quando a estrutura da pasta de trabalho está protegida. O botão Proteger Pasta de Traba-
lho acende.

Criptografar a pasta de trabalho com uma senha


No modo de exibição Backstage, você pode definir uma senha para a pasta de trabalho que fornece criptografia.
Clique em Arquivo > Informações > Proteger Pasta de Trabalho >Criptografar com Senha.
Na caixa Criptografar Documento, digite uma senha e clique em OK.
Na caixa Confirmar Senha, digite a senha novamente e clique em OK.
OBSERVAÇÃO :  Para remover uma senha, siga as etapas acima e exclua a senha. Basicamente, basta deixar a senha em
branco. Você pode fazer isso para qualquer tipo de senha usado no Excel.
Por que minha senha desaparece quando salvo no formato do Excel 97-2003?
Você deseja enviar a sua pasta de trabalho protegida por senha para outras pessoas, mas eles ainda estão usando o
Excel 2003, que salva no formato de arquivo Excel 97-2003 (*.xls). Você escolhe “Salvar como” usando o formato 97-2003,
mas então você descobre que a senha definida na pasta de trabalho desapareceu.
Isso acontece porque a sua versão do Excel usa um novo esquema para salvar senhas, e o formato de arquivo anterior
não o reconhece. Como resultado, a senha é descartada ao salvar seu arquivo para o formato do Excel 97-2003. Defina a
senha no arquivo *.xls para proteger a pasta de trabalho novamente.

73
INFORMÁTICA

Proteger uma planilha com ou sem uma senha no Adicionar ou alterar a cor do plano de fundo das
Excel 2016 células
Para ajudar a proteger seus dados de alterações não É possível realçar dados em células utilizando Cor de
intencionais ou intencionais, proteja sua planilha, com ou preenchimento para adicionar ou alterar a cor do plano de
sem senha. Ela impede que outras pessoas removam a pro- fundo ou padrão das células. Veja como:
teção da planilha: a senha deve ser inserida para desprote- Selecione as células que deseja realçar.
ger a planilha. DICAS : Para utilizar uma cor de fundo diferente para a
Por padrão, quando você protege uma planilha, o excel planilha inteira, clique no botão Selecionar Tudo. Isso irá ocul-
bloqueia todas as células nessa planilha. Antes de proteger tar as linhas de grade, mas é possível melhorar a legibilidade
a planilha, desbloqueie quaisquer células que desejar alte- da planilha exibindo bordas ao redor de todas as células.
rar antes de seguir essas etapas.
Clique na guia Revisão e clique em Proteger Planilha.
Verifique se a caixa de seleção Proteger a planilha e o
conteúdo de células bloqueadas está marcada.
Para usar uma senha, digite-a na caixa Senha para des-
proteger a planilha. 
Outros usuários podem remover a proteção se você
não usar uma senha.
IMPORTANTE :  Anote sua senha e armazene-a em lo- Clique em  Página Inicial  > seta ao lado de  Cor de
cal seguro. Nós sinceramente não podemos ajudar você a
recuperar senhas perdidas. Preenchimento  .
Se você digitou uma senha na etapa 3, redigite-a para
confirmá-la.

Em Cores do Tema ou Cores Padrão, selecione a cor


desejada.

Marque ou desmarque as caixas de seleção em Permi-


tir que todos os usuários desta planilha possam e clique
em OK.

OBSERVAÇÕES : 
Para remover a proteção da planilha, clique em Revi-
são, clique emDesproteger Planilha e digite a senha, se ne-
cessário.
Se uma macro não pode executar na planilha protegi-
da, você verá uma mensagem e a macro será interrompida

Para utilizar uma cor personalizada, clique em Mais Co-


res, e em seguida, na caixa de diálogo Cores, selecione a
cor desejada.

74
INFORMÁTICA

DICA : Para aplicar a cor selecionada mais recentemen- Remover cores de célula, padrões, ou efeitos de
te, clique em Cor de Preenchimento  . Você também preenchimento
encontrará até 10 cores personalizadas selecionadas mais Para remover quaisquer cores de fundo, padrões ou
recentemente em Cores recentes. efeitos de preenchimento das células, basta selecioná-las.
Aplicar um padrão ou efeitos de preenchimento. Clique em Página Inicial > seta ao lado de Cor de Preenchi-
Quando você deseja algo mais do que apenas um mento, e então selecione Sem Preenchimento.
preenchimento de cor sólida, experimente aplicar um pa-
drão ou efeitos de preenchimento.
Selecione a célula ou intervalo de células que deseja
formatar.
Clique em Página Inicial > iniciador da caixa de diálo-
go Formatar Células ou pressione Ctrl + Shift + F.

Imprimir cores de célula, padrões ou efeitos de preen-


chimento em cores
Se as opções de impressão estiverem definidas
como Preto e branco ouQualidade de rascunho — seja
propositalmente ou porque a pasta de trabalho contém
Na guia Preenchimento, em Cor de Fundo, selecione a planilhas e gráficos grandes ou complexos que resultaram
cor desejada. na ativação automática do modo de rascunho — não será
possível imprimir as células em cores. Veja aqui como re-
solver isso:
Clique em Layout da Página > iniciador da caixa de diá-
logo Configurar Página.

Na guia Folha, em Imprimir, desmarque as caixas de


seleção Preto e branco e Qualidade de rascunho.
OBSERVAÇÃO : Se você não visualizar cores em sua
planilha, talvez esteja trabalho no modo de alto contraste.
Se não visualizar cores ao visualizar antes de imprimir, tal-
vez nenhuma impressora colorida esteja selecionada.

Principais atalhos
CTRL+Menos (-) — Exibe a caixa de diálogo Excluir
Para utilizar um padrão com duas cores, selecione uma para excluir as células selecionadas.
cor na caixa Cor do Padrão e, em seguida, selecione um CTRL+; — Insere a data atual.
padrão na caixa Estilo do Padrão. CTRL+` — Alterna entre a exibição dos valores da célu-
Para utilizar um padrão com efeitos especiais, clique la e a exibição de fórmulas na planilha.
em Efeitos de Preenchimento, e, em seguida, selecione as CTRL+’ — Copia uma fórmula da célula que está acima
opções desejadas. da célula ativa para a célula ou a barra de fórmulas.
CTRL+1 — Exibe a caixa de diálogo Formatar Células.
DICA : Na caixa Amostra, é possível visualizar o plano CTRL+2 — Aplica ou remove formatação em negrito.
de fundo, o padrão e os efeitos de preenchimento selecio- CTRL+3 — Aplica ou remove formatação em itálico.
nados. CTRL+4 — Aplica ou remove sublinhado.
CTRL+5 — Aplica ou remove tachado.
CTRL+6 — Alterna entre ocultar objetos, exibir objetos
e exibir espaços reservados para objetos.
CTRL+8 — Exibe ou oculta os símbolos de estrutura
de tópicos.
CTRL+9 — Oculta as linhas selecionadas.

75
INFORMÁTICA

CTRL+0 — Oculta as colunas selecionadas. CTRL+Y — Repete o último comando ou ação, se pos-


CTRL+A — Seleciona a planilha inteira. Se a planilha sível.
contiver dados, este comando seleciona a região atual. CTRL+Z — Usa o comando Desfazer para reverter o
Pressionar CTRL+A novamente seleciona a região atual e último comando ou excluir a última entrada digitada.
suas linhas de resumo. Pressionar CTRL+A novamente se- CTRL+SHIFT+Z — Usa o comando Desfazer ou Refazer
leciona a planilha inteira. para reverter ou restaurar a correção automática quando
CTRL+SHIFT+A — Insere os nomes e os parênteses do Marcas Inteligentes de AutoCorreção são exibidas.
argumento quando o ponto de inserção está à direita de CTRL+SHIFT+( — Exibe novamente as linhas ocultas
um nome de função em uma fórmula. dentro da seleção.
CTRL+N — Aplica ou remove formatação em negrito. CTRL+SHIFT+) — Exibe novamente as colunas ocultas
CTRL+C — Copia as células selecionadas. dentro da seleção.
CTRL+C (seguido por outro CTRL+C) — exibe a Área CTRL+SHIFT+& — Aplica o contorno às células sele-
de Transferência. cionadas.
CTRL+D — Usa o comando Preencher Abaixo para co- CTRL+SHIFT+_ — Remove o contorno das células se-
piar o conteúdo e o formato da célula mais acima de um lecionadas.
intervalo selecionado nas células abaixo. CTRL+SHIFT+~ — Aplica o formato de número Geral.
CTRL+F — Exibe a caixa de diálogo Localizar e Substi- CTRL+SHIFT+$ — Aplica o formato Moeda com duas
tuir com a guia Localizar selecionada. casas decimais (números negativos entre parênteses)
SHIFT+F5 — Também exibe essa guia, enquanto SHIF- CTRL+SHIFT+% — Aplica o formato Porcentagem sem
T+F4 repete a última ação de Localizar. casas decimais.
CTRL+SHIFT+F — Abre a caixa de diálogo Formatar CTRL+SHIFT+^ — Aplica o formato de número Expo-
Células com a guia Fonte selecionada. nencial com duas casas decimais.
CTRL+G — Exibe a caixa de diálogo Ir para. (F5 tam- CTRL+SHIFT+# — Aplica o formato Data com dia, mês
bém exibe essa caixa de diálogo.) e ano.
CTRL+H — Exibe a caixa de diálogo Localizar e Substi- CTRL+SHIFT+@ — Aplica o formato Hora com a hora
e os minutos, AM ou PM.
tuir com a guia Substituir selecionada.
CTRL+SHIFT+! — Aplica o formato Número com duas
CTRL+I — Aplica ou remove formatação em itálico.
casas decimais, separador de milhar e sinal de menos (-)
CTRL+K — Exibe a caixa de diálogo Inserir Hiperlink
para valores negativos.
para novos hiperlinks ou a caixa de diálogo Editar Hiperlink
CTRL+SHIFT+* — Seleciona a região atual em torno
para os hiperlinks existentes que estão selecionados.
da célula ativa (a área de dados circunscrita por linhas e
CTRL+N — Cria uma nova pasta de trabalho em branco
colunas vazias).
CTRL+O — Exibe a caixa de diálogo Abrir para abrir ou
CTRL+SHIFT+: — Insere a hora atual.
localizar um arquivo.
CTRL+SHIFT+” –Copia o valor da célula que está acima
CTRL+SHIFT+O — Seleciona todas as células que con- da célula ativa para a célula ou a barra de fórmulas.
têm comentários. CTRL+SHIFT+Mais (+) — Exibe a caixa de diálogo Inse-
CTRL+P — Exibe a caixa de diálogo Imprimir. rir para inserir células em branco.
CTRL+SHIFT+P — Abre a caixa de diálogo Formatar
Células com a guia Fonte selecionada.
CTRL+R — Usa o comando Preencher à Direita para
copiar o conteúdo e o formato da célula mais à esquerda
de um intervalo selecionado nas células à direita.
CTRL+B — Salva o arquivo ativo com seu nome de ar-
quivo, local e formato atual.
CTRL+T — Exibe a caixa de diálogo Criar Tabela.
CTRL+S — Aplica ou remove sublinhado.
CTRL+SHIFT+S — Alterna entre a expansão e a redu-
ção da barra de fórmulas.
CTRL+V — Insere o conteúdo da Área de Transferência
no ponto de inserção e substitui qualquer seleção. Dispo-
nível somente depois de ter recortado ou copiado um ob-
jeto, texto ou conteúdo de célula.
CTRL+ALT+V — Exibe a caixa de diálogo Colar Especial,
disponível somente depois que você recortar ou copiar um
objeto, textos ou conteúdo de célula em uma planilha ou
em outro programa.
CTRL+W — Fecha a janela da pasta de trabalho sele-
cionada.
CTRL+X — Recorta as células selecionadas.

76
INFORMÁTICA

Fórmulas básicas
As primeiras fórmulas aprendidas na escola são as de adição, subtração, multiplicação e divisão. No Excel não é dife-
rente.

Cálculo Fórmula Explicação Exemplo


Para aplicar a fórmula
=SOMA(A1;A2).
de soma você preci-
Dica: Sempre separe a
sa, apenas, selecionar
indicação das células
as células que estarão
com ponto e vírgula (;).
Adição =SOMA(célulaX;célula Y) envolvidas na adição,
Dessa forma, mesmo as
incluindo a sequência
que estiverem em loca-
no campo superior do
lizações distantes serão
programa junto com o
consideradas na adição
símbolo de igual (=)
Segue a mesma lógica
da adição, mas des-
sa vez você usa o sinal
correspondente a conta
Subtração =(célulaX-célulaY) =(A1-A2)
que será feita (-) no lu-
gar do ponto e vírgula
(;), e retira a palavra
“soma” da função
Use o asterisco (*) para
Multiplicação = (célulaX*célulaY) indicar o símbolo de = (A1*A2)
multiplicação
A divisão se dá com a
barra de divisão (/) en-
Divisão =(célulaX/célulaY) =(A1/A2)
tre as células e sem pa-
lavra antes da função

Fórmulas bastante requisitadas


Outros algoritmos que são bastante importantes nas planilhas são aqueles que mostram valores de média, máxima e
mínimo. Mas para usar essas funções, você precisa estabelecer um grupo de células.

Cálculo Fórmula Explicação Exemplo


Você deve usar a palavra
“media” antes das células
indicadas, que são sempre
Média =MEDIA(célula X:célulaY) separadas por dois pontos =MEDIA(A1:A10)
(:) e representam o grupo
total que você precisa cal-
cular
Segue a mesma lógica, mas
Máxima =MAX(célula X:célulaY) =MAX(A1:A10)
usa a palavra “max”
Dessa vez, use a expressão
Mínima =MIN(célula X:célulaY) =MIN(A1:A10)
“min”

77
INFORMÁTICA

Função Se
Essa função trata das condições de valores solicitados. Para que entenda, se você trabalhar em uma loja que precisa
saber se os produtos ainda estão no estoque ou precisam de mais unidades, essa é uma excelente ferramenta. Veja por que:

Cálculo Fórmula Exemplo


Função =se(célulaX<=0 ; “O que =se(B1<=0 ; “a ser enviado” ; “no estoque”)
Se precisa saber 1” ; “o que Essa linguagem diz ao Excel que se o conteúdo da célula B1 é menor ou
precisa saber 2”) igual a zero ele deve exibir a mensagem “a ser enviado” na célula que
contem a fórmula. Caso o conteúdo seja maior que zero, a mensagem
que aparecerá é “no estoque”

*Fonte: http://www.portaleducacao.com.br/informatica/artigos/71948/23-formulas-e-atalhos-que-vao-facilitar-sua-
vida-no-excel#ixzz48neY9XBW
EXCEL ONLINE (365)

Note a interface do Excel online sendo executado diretamente do navegador de internet, bastante semelhante à do
Excel 2016.

Diferenças entre o uso de uma pasta de trabalho no navegador e no Excel

Excel em sua área de trabalho e online com o Office 365 permitem compartilhar vários recursos em comum, como a
capacidade de imprimir e interagir com itens. No entanto há algumas diferenças como formas que não podem ser editadas
em uma janela do navegador.

Formatos de arquivo de pasta de trabalho suportados na janela do navegador


As pastas de trabalho em qualquer um dos seguintes formatos podem ser visualizadas na janela do navegador:
- pasta de trabalho de Excel (. xls,. xlsx)
- Arquivo de pasta de trabalho binária do Excel (.xlsb)
- Arquivo de planilha OpenDocument (.ods)
- Pasta de trabalho do Excel habilitada para macro (.xlsm): Uma pasta de trabalho neste formato poderá ser aberta, mas
as macros não serão executadas na janela do navegador.
Pastas de trabalho em outros formatos, como .csv, geralmente não têm suporte para visualização na janela do nave-
gador.

78
INFORMÁTICA

Recursos da pasta de trabalho com suporte para vi- Filtros, segmentações de dados e controles de linha do
sualização na janela do navegador tempo
Ao visualizar ou imprimir uma pasta de trabalho em Filtros e segmentações de dados geralmente funcio-
uma janela do navegador, alguns recursos poderão funcio- nam em uma janela do navegador, de modo semelhante a
nar de forma diferente, como resumido a seguir. como funcionam no Excel. Filtros e segmentações de dados
geralmente funcionam em uma janela do navegador, de
Cálculos modo semelhante a como funcionam no Excel.
Cálculos e configurações de cálculos, incluindo con-
figurações de cálculo automáticas, manuais e interativas Se você estiver usando os Serviços do Excel ou o Ex-
para intervalos ou planilhas que funcionem em uma janela cel Web App localmente (não na nuvem), a capacidade de
de navegador similar à como funcionam no Excel. exibir determinado conteúdo em uma janela do navegador
depende de os Serviço do Excel ou o Excel Web App serem
Células usados para exibir uma pasta de trabalho. Para obter mais
As células, incluindo os valores das células, as células informações, consulte Comparar os Serviços do Excel no
mescladas e a visão geral do conteúdo da célula, são exi- SharePoint com oExcel Online.
bidas em uma janela de navegador similar à como são exi-
bidas no Excel. Formatação
A formatação da célula, do intervalo de células e for-
Gráficos e tabelas, incluindo relatórios da Tabela Dinâ- matos de números são exibidos na janela do navegador
mica e de Gráfico Dinâmico similar a como são exibidos no Excel.
Em geral, gráficos, tabelas, relatórios de Tabela Dinâmi-
ca e relatórios de Gráfico Dinâmico são exibidos em uma Funções
janela do navegador, de modo semelhante a como são exi- A maioria das funções do Excel funciona na janela do
bidos no Excel. São exceções navegador de modo semelhante a como são exibidas no
Gráficos que fazem parte de um grupo de gráficos
Excel. As exceções estão listadas na seção Funções da pasta
Gráficos que dependem de referências externas
de trabalho deste artigo.
Gráficos que usam certos tipos de fontes de dados que
não são compatíveis com pastas de trabalho visualizadas
Bordas de linha
em uma janela do navegador
A maioria das bordas de linha, mas não todas, é exibida
Se você estiver usando os Serviços do Excel ou o Ex-
na janela do navegador da mesma forma que é exibida no
cel Web App localmente (não na nuvem), a capacidade de
Excel. As cores das linhas, estilos contínuo e de linha dupla,
exibir determinado conteúdo em uma janela do navegador
depende de os Serviço do Excel ou o Excel Web App serem linhas sólidas e tracejadas e linhas finas, médias e grossas
usados para exibir uma pasta de trabalho. Para obter mais são compatíveis.
informações, consulte Comparar os Serviços do Excel no
SharePoint com oExcel Online. Itens nomeados
Itens, tais como intervalos, tabelas, gráficos, relatórios
Conexões de dados de Gráfico Dinâmico, relatórios de Tabela Dinâmica, seg-
Dependendo da configuração do ambiente, alguns tipos mentações de dados e controles de linha do tempo, geral-
de conexões de dados são suportados pelas pastas de traba- mente são exibidos em uma janela do navegador, de modo
lho exibidas em uma janela de navegador, o que permite atua- semelhante a como são exibidos no Excel.
lizar os dados. Para mais informações, consulte Atualizar dados
de uma pasta de trabalho em uma janela de navegador. Se você estiver usando os Serviços do Excel ou o Ex-
cel Web App localmente (não na nuvem), a capacidade de
Datas exibir determinado conteúdo em uma janela do navegador
As datas são exibidas na janela do navegador similar a depende de os Serviço do Excel ou o Excel Web App serem
como são exibidas no Excel. usados para exibir uma pasta de trabalho. Para obter mais
informações, consulte Comparar os Serviços do Excel no
Referências externas SharePoint com oExcel Online.
O último valor conhecido da referência externa é exibi-
do na janela do navegador. As referências externas devem Minigráficos
ser atualizadas usando o Excel. Os minigráficos são exibidos na janela do navegador
Lista de campos de um relatório de Gráfico Dinâmico similar a como são exibidos no Excel.
ou de Tabela Dinâmica
No geral, a Lista de campos de um relatório de Gráfico Tabelas
Dinâmico ou Tabela Dinâmica funciona em uma janela de As tabelas do Excel, incluindo dados, cabeçalhos de co-
navegador de forma similar a como funciona no Excel. Isso luna e linhas de total, são exibidas na janela do navegador
significa que é possível usar a Lista de campos para explo- similar a como são exibidas no Excel.
rar informações e alterar os dados exibidos no relatório de
Gráfico Dinâmico ou de Tabela Dinâmica.

79
INFORMÁTICA

Recursos da pasta de trabalho que podem diferir entre o Segurança e privacidade


navegador e o programa de área de trabalho Pastas de trabalho com configurações de Gerenciamen-
Alguns recursos funcionam de forma diferente na jane- to de Direitos de Informação (IRM) aplicadas no nível da
la do navegador e no Excel. Além disso, alguns recursos da pasta de trabalho não podem ser exibidas em uma janela
pasta de trabalho podem impedir a visualização da pasta do navegador.
de trabalho em uma janela de navegador. Se sua organização estiver usando o Excel Web App (lo-
Se você não pode exibir uma pasta de trabalho na ja- cal) ou o Excel Online (na nuvem) para exibir pastas de tra-
nela do navegador porque contém recursos sem suporte, balho, então as pastas de trabalho que são salvas em biblio-
tente abrir a pasta de trabalho no Excel na área de trabalho. tecas de documentos protegidas por IRM são compatíveis e
podem ser exibidas em uma janela do navegador.
Controles Pastas de trabalho que são enviadas como anexos pro-
Controles, como os controles do ActiveX e controles tegidos por IRM no Outlook Online também são compatí-
da barra de ferramentas do formulário, podem impedir veis e podem ser exibidas em uma janela do navegador.
que uma pasta de trabalho seja visualizada na janela do No entanto, se sua organização estiver usando os Ser-
navegador. viços do Excel no SharePoint Server (local) para exibir pastas
de trabalho, as pastas de trabalho em bibliotecas protegidas
Conexões de dados por IRM não são compatíveis e devem ser abertas no Excel.
Dependendo da configuração do ambiente, alguns ti- IRM não pode ser aplicado no navegador.
pos de conexões de dados podem impedir que uma pasta
Formas
de trabalho seja visualizada na janela do navegador. Para
Não é possível editar pastas de trabalho que contenham
mais informações, consulte Atualizando dados de uma
formas em uma janela de navegador.
pasta de trabalho em uma janela de navegador.
Desfazer e Refazer
Validação de dados Se uma pasta de trabalho for editada por mais de uma
As pastas de trabalho que usam a validação de dados pessoa ao mesmo tempo, os comandos Desfazer e Refazer
podem ser editadas em uma janela do navegador, mas a não estarão mais disponíveis na janela do navegador assim
validação de dados não pode ser configurada no navega- que outro usuário fizer uma alteração.
dor.
Proteção para planilha ou proteção para pasta de tra-
Assinaturas digitais balho
Não é possível visualizar as pastas de trabalho que Não é possível visualizar pastas de trabalho protegidas
usam assinaturas digitais ou invisíveis em uma janela de (criptografadas por senha) em uma janela do navegador.
navegador. As planilhas protegidas (não as pastas de trabalho in-
teiras) podem ser exibidas em uma janela do navegador no
Fontes Excel Online.
Geralmente, as fontes são exibidas no navegador usan- Em um ambiente local onde você use os Serviços do Ex-
do os mesmos estilos e tamanhos com os quais seriam exi- cel ou do Excel Web App, as planilhas protegidas não podem
bidas no Excel. Se uma pasta de trabalho usar uma fonte ser exibidas em uma janela do navegador.
específica que não esteja disponível no dispositivo, ela será Para remover a proteção, abra a pasta de trabalho no
substituída quando a pasta de trabalho for exibida na jane- Excel na área de trabalho.
la do navegador.
XML
Funções Mapas XML, marcas inteligentes inseridas XML e paco-
Alguns tipos de funções na janela do navegador po- tes de expansão XLM evitam que uma pasta de trabalho seja
dem se comportar de forma diferente no Excel. Para obter visualizada em uma janela de navegador.
mais informações, consulte a seção Funções posteriormen-
te neste artigo. Funções da pasta de trabalho no Excel e na janela do na-
vegador
Algumas funções da pasta de trabalho se comportam
Linguagens de macro herdadas
de forma diferente em uma janela de navegador e no Excel.
Não é possível exibir pastas de trabalho que conte-
A tabela a seguir resume como algumas funções funcionam
nham recursos macro herdados, como Funções Macro do
no Excel e em uma pasta de trabalho visualizada em uma
Microsoft Excel 4.0 e folhas de diálogo do Microsoft 5.0, na
janela de navegador.
janela do navegador.
CARACT
Salvar Retorna um caractere especificado por um número. Para
Ao editar uma pasta de trabalho na janela do navega- caracteres não imprimíveis, retorna valores em branco.
dor, as alterações são salvas automaticamente. Retorna um caractere especificado por um número.
Para caracteres não imprimíveis, retorna valores em branco.

80
INFORMÁTICA

HIPERLINK so e, apesar de cada pessoa ter seu endereço próprio,


Retorna um link ativo que você pode clicar e seguir você pode acessar seu e-mail de qualquer computador
Dependendo das configurações da Interatividade da conectado à Internet. Bem, o e-mail mesclou a facilidade
Pasta e da Navegação da Pasta de Trabalho especificadas de uso do correio convencional com a velocidade do tele-
para uma pasta de trabalho, a função HYPERLINK poderá fone, se tornando um dos melhores e mais utilizado meio
retornar uma cadeia de texto de link ativo ou inativo. de comunicação.

INFORMAÇÃO Estrutura e Funcionalidade do e-mail


Retorna o caminho do atual diretório ou pasta no
computador. Como no primeiro e-mail criado por Tomlinson, todos
Retornar um erro #VALUE!. os endereços eletrônicos seguem uma estrutura padrão,
nome do usuário + @ + host, onde:
AGORA » Nome do Usuário – é o nome de login escolhido pelo
Retorna a data e a hora no seu computador usuário na hora de fazer seu e-mail. Exemplo: sergiodecas-
Retorna a data e a hora no servidor tro.
» @ - é o símbolo, definido por Tomlinson, que separa
ALEATÓRIO, ALEATÓRIOENTRE o nome do usuário do seu provedor.
Retorna um número aleatório cada vez que é usado » Host – é o nome do provedor onde foi criado o ende-
Retorna um número aleatório cada vez que é usado reço eletrônico. Exemplo: click21.com.br .
» Provedor – é o host, um computador dedicado ao
RTD (dados em tempo real) serviço 24 horas por dia.
Retorna dados em tempo real de um programa que
suporta automação COM. Vejamos um exemplo real: sergiodecastro@click21.
Se os valores que forem retornados por uma função com.br
RTD forem armazenados em uma pasta de trabalho, es-
ses valores serão exibidos. Se nenhum valor for armaze- A caixa postal é composta pelos seguintes itens:
nado na pasta de trabalho, um erro #N/A será retornado. » Caixa de Entrada – Onde ficam armazenadas as men-
sagens recebidas.
HOJE » Caixa de Saída – Armazena as mensagens ainda não
Retorna a data no seu computador enviadas.
Retorna a data no servidor » E-mails Enviados – Como o nome diz, ficam os e-mails
que foram enviados.
» Rascunho – Guarda as mensagens que você ainda
não terminou de redigir.
CORREIO ELETRÔNICO: USO DE CORREIO » Lixeira – Armazena as mensagens excluídas.
ELETRÔNICO, PREPARO E ENVIO DE
MENSAGENS, ANEXAÇÃO DE ARQUIVOS. Ao redigir mensagem, os seguintes campos estão pre-
sentes:
» Para – é o campo onde será inserido o endereço do
destinatário.
CORREIO ELETRÔNICO » Cc – este campo é utilizado para mandar cópias da
mesma mensagem, ao usar este campo os endereços apa-
O correio eletrônico3 se parece muito com o correio recerão para todos os destinatários.
tradicional. Todo usuário tem um endereço próprio e » Cco – sua funcionalidade é igual ao campo anterior,
uma caixa postal, o carteiro é a Internet. Você escreve no entanto os endereços só aparecerão para os respectivos
sua mensagem, diz pra quem quer mandar e a Internet donos.
cuida do resto. Mas por que o e-mail se popularizou » Assunto – campo destinado ao assunto da mensa-
tão depressa? A primeira coisa é pelo custo. Você não gem.
paga nada por uma comunicação via e-mail, apenas os » Anexos – são dados que são anexados à mensagem
custos de conexão com a Internet. Outro fator é a ra- (imagens, programas, música, arquivos de texto, etc.).
pidez, enquanto o correio tradicional levaria dias para » Corpo da Mensagem – espaço onde será redigida a
entregar uma mensagem, o eletrônico faz isso quase mensagem.
que instantaneamente e não utiliza papel. Por último, a
mensagem vai direto ao destinatário, não precisa passa Alguns nomes podem mudar de servidor para servidor,
de mão-em-mão (funcionário do correio, carteiro, etc.), porém representando as mesmas funções. Além dos destes
fica na sua caixa postal onde somente o dono tem aces- campos tem ainda os botões para EVIAR, ENCAMINHAR e
3 Fonte: http://juliobattisti.com.br/tutoriais/sergiocastro/correioeletronico- EXCLUIR as mensagens, este botões bem como suas fun-
ewebmail001.asp cionalidades veremos em detalhes, mais à frente.

81
INFORMÁTICA

Para receber seus e-mails você não precisa estar co- » Criando seu e-mail
nectado à Internet, pois o e-mail funciona com provedores. Fazer sua conta de e-mail é uma tarefa extremamente
Mesmo você não estado com seu computador ligado, seus simples, eu escolhi o Outlook.com, pois a interface deste
e-mail são recebidos e armazenados na sua caixa postal, WebMail não tem propagandas e isso ajudar muito os en-
localizada no seu provedor. Quando você acessa sua caixa tendimentos, no entanto você pode acessar qualquer dos
postal, pode ler seus e-mail on-line (diretamente na Internet, endereços informados acima ou ainda qualquer outro que
pelo WebMail) ou baixar todos para seu computador através você conheça. O processo de cadastro é muito simples, basta
de programas de correio eletrônico. Um programa muito co- preencher um formulário e depois você terá sua conta de
nhecido é o Outlook Express, o qual detalhar mais à frente. e-mail pronta para ser usada. Vamos aos passos:
A sua caixa postal é identificada pelo seu endereço de 1. Acesse a página do provedor (www.ibestmail.com.br)
e-mail e qualquer pessoa que souber esse endereço, pode ou qualquer outro de sua preferência.
enviar mensagens para você. Também é possível enviar men- 2. Clique no link “Não tem uma conta? Crie uma!”, será
sagens para várias pessoas ao mesmo tempo, para isto basta aberto um formulário, preencha-o observando todos os
usar os campos “Cc” e “Cco” descritos acima. campos. Os campos do formulário têm suas particularidades
Atualmente, devido à grande facilidade de uso, a maioria de provedor para provedor, no entanto todos trazem a mes-
das pessoas acessa seu e-mail diretamente na Internet atra- ma ideia, colher informações do usuário. Este será a primeira
vés do navegador. Este tipo de correio é chamado de Web- parte do seu e-mail e é igual a este em qualquer cadastro, no
Mail. O WebMail é responsável pela grande popularização exemplo temos “@outlook.com”. A junção do nome de usuá-
do e-mail, pois mesmo as pessoas que não tem computador, rio com o nome do provedor é que será seu endereço eletrô-
podem acessar sua caixa postal de qualquer lugar (um cyber, nico. No exemplo ficaria o seguinte: seunome@outlook.com.
casa de um amigo, etc.). Para ter um endereço eletrônico 3. Após preencher todo o formulário clique no botão
basta querer e acessar a Internet, é claro. Existe quase que “Criar conta”, pronto seu cadastro estará efetivado.
uma guerra por usuários. Os provedores, também, disputam Pelo fato de ser gratuito e ter muitos usuários é comum
quem oferece maior espaço em suas caixas postais. Há pouco que muitos nomes já tenham sido cadastrados por outros
tempo encontrar um e-mail com mais de 10 Mb, grátis, não usuários, neste caso será exibida uma mensagem lhe in-
era fácil. Lembro que, quando a Embratel ofereceu o Click21 formando do problema. Isso acontece porque dentro de
com 30 Mb, achei que era muito espaço, mas logo o iBest
um mesmo provedor não pode ter dois nomes de usuários
ofereceu 120 Mb e não parou por ai, a “guerra” continuo cul-
iguais. A solução é procurar outro nome que ainda esteja
minando com o anúncio de que o Google iria oferecer 1 Gb
livre, alguns provedores mostram sugestões como: seuno-
(1024 Mb). A última campanha do GMail, e-mail do Google,
me2005; seunome28, etc. Se ocorrer isso com você (o que é
é de aumentar sua caixa postal constantemente, a última vez
bem provável que acontecerá) escolha uma das sugestões ou
que acessei estava em 2663 Mb.
informe outro nome (não desista, você vai conseguir), finalize
seu cadastro que seu e-mail vai está pronto para ser usado.
WebMail

O WebMail, como descrito acima, é uma aplicação » Entendendo a Interface do WebMail


acessada diretamente na Internet, sem a necessidade de A interface é a parte gráfica do aplicativo de e-mail que
usar programa de correio eletrônico. Praticamente todos os nos liga do mundo externo aos comandos do programa. Es-
e-mails possuem aplicações para acesso direto na Internet. É tes conhecimentos vão lhe servir para qualquer WebMail que
grande o número de provedores que oferecem correio ele- você tiver e também para o Outlook, que é um programa
trônico gratuitamente, logo abaixo segue uma lista dos mais de gerenciamento de e-mails, vamos ver este programa mais
populares. adiante.
» Outlook (antigo Hotmail) – http://www.outlook.com 1. Chegou e-mail? – Este botão serve para atualizar sua
» GMail – http://www.gmail.com caixa de entrar, verificando se há novas mensagens no ser-
» Bol (Brasil on line) – http://www.bol.com.br vidor.
» iG Mail – http://www.ig.com.br 2. Escrever – Ao clicar neste botão a janela de edição de
» Yahoo – http://www.yahoo.com.br e-mail será aberta. A janela de edição é o espaço no qual
você vai redigir, responder e encaminhar mensagens. Seme-
Para criar seu e-mail basta visitar o endereço acima e se- lhante à função novo e-mail do Outlook.
guir as instruções do site. Outro importante fator a ser obser- 3. Contatos – Abre a seção de contatos. Aqui os seus en-
vado é o tamanho máximo permitido por anexo, este foi ou- dereços de e-mail são previamente guardados para utiliza-
tro fator que aumentou muito de tamanho, há pouco tempo ção futura, nesta seção também é possível criar grupos para
a maioria dos provedores permitiam em torno de 2 Mb, mas facilitar o gerenciamento dos seus contatos.
atualmente a maioria já oferecem em média 25 Mb. Além de 4. Configurações – Este botão abre (como o próprio
caixa postal os provedores costumam oferecer serviços de nome já diz) a janela de configurações. Nesta janela podem
agenda e contatos. ser feitas diversas configurações, tais como: mudar senha,
Todos os WebMail acima são ótimos, então fica a crité- definir número de e-mail por página, assinatura, resposta au-
rio de cada um escolher o seu, ou até mesmo os seus, eu, tomática, etc.
por exemplo, procuro aqueles que oferecem uma interface 5. Ajuda – Abre, em outra janela do navegador, uma
com o menor propaganda possível. seção com vários tópicos de ajuda.

82
INFORMÁTICA

6. Sair – Este botão é muito importante, pois é através guração dos e-mails bloqueados e mais abaixo o link para
dele que você vai fechar sua caixa postal, muito recomenda- baixar um plug-in que lhe permite fazer uma configuração
do quando o uso de seu e-mail ocorrer em computadores automática do Outlook Express. Estes dois primeiros links
de terceiros. são os mesmos apresentados no item 10.
7. Espaço – Esta seção é apenas informativa, exibe seu
endereço de e-mail; quantidade total de sua caixa posta; Outlook Express
parte utilizada em porcentagem e um pequeno gráfico.
8. Seção atual – Mostra o nome da seção na qual você O Outlook Express é um programa, dentre vários, para
está, no exemplo a Caixa de Entrada. a troca de mensagens entre pessoas que tenham acesso à
9. Número de Mensagens – Exibe o intervalo de men- Internet.
sagens que estão na tela e também o total da seção sele- Por meio dele você poderá mandar e receber mensa-
cionada. gens (incluindo os e-mails com imagens, música e diversos
10. Caixa de Comandos – Neste menu suspenso estão efeitos) e também ingressar em grupos de notícias para
todos os comandos relacionados com as mensagens exi- trocar ideias e informações.
bidas. Para usar estes comandos, selecione uma ou mais Para adicionar uma conta de e-mail, isto é, para criar
mensagens o comando desejado e clique no botão “OK”. O sua caixa de correio eletrônico no Outlook Express, você
botão “Bloquear”, bloqueia o endereço de e-mail da men- precisa do nome da conta, da senha e dos nomes dos servi-
sagem, útil para bloquear e-mails indesejados. Já o botão dores de e-mail de entrada (geralmente POP3 - Post Office
“Contas externas” abre uma seção para configurar outras Protocol versão 3 - que é o servidor onde ficam armazena-
contas de e-mails que enviarão as mensagens a sua caixa das as mensagens enviadas a você, até que você as receba)
postal. Para o correto funcionamento desta opção é preciso e de saída (geralmente SMTP - Simple Mail Transfer Pro-
que a conta a ser acessada tenha serviço POP3 e SMTP. tocol - que é o servidor que armazena as mensagens e as
11. Lista de Páginas – Este menu suspenso exibe a lista envia, após você escrevê-las.).
de página, que aumenta conforme a quantidade de e-mails Estas informações são fornecidas pelo seu provedor de
na seção. Para acessar selecione a página desejada e clique
serviços de Internet ou do administrador da rede local.
no botão “OK”. Veja que todos os comandos estão dispo-
Para adicionar um grupo de notícias, você precisa do
níveis também na parte inferior, isto para facilitar o uso de
nome do servidor de notícias ao qual deseja se conectar e,
sua caixa postal.
se necessário, do nome de sua conta e senha.
12. Pastas do Sistema – Exibe as pastas padrões de um
correio eletrônico. Caixa de Entrada; Mensagens Enviadas;
Abrir o Outlook
Rascunho e Lixeira. Um detalhe importante é o estilo do
Para abrir o Outlook Express, clique no ícone
nome, quando está normal significa que todas as mensa-
gens foram abertas, porém quando estão em negrito, acu- que está em sua Área de Trabalho, ou na Barra de Tarefas.
sam que há uma ou mais mensagens que não foram lidas,
o número entre parêntese indica a quantidade. Este detalhe Você pode também clicar em Iniciar/Programas/
funciona para todas as pastas e mensagens do correio. Outlook Express.
13. Painel de Visualização – Espaço destinado a exibir as O programa será aberto e você pode começar a ler,
mensagens. Por padrão, ao abrir sua caixa postal, é exibido redigir e responder seus e-mails.
o conteúdo da Caixa de Entrada, mas este painel exibe tam-
bém as mensagens das diversas pastas existentes na sua Barra de Ferramentas
caixa postal. A observação feita no item anterior, sobre ne- A Barra de ferramentas do Outlook Express (mostrados
grito, também é válida para esta seção. Observe as caixas de nas imagens da versão 6.0) apresenta basicamente os itens
seleção localizadas do lado esquerdo de cada mensagem, é numerados na figura abaixo:
através delas que as mensagens são selecionadas. A seleção
de todos os itens ao mesmo tempo, também pode ser feito
pela caixa de seleção do lado esquerdo do título da coluna
“Remetente”. O título das colunas, além de nomeá-las, tam-
bém serve para classificar as mensagens que por padrão
estão classificadas através da coluna “Data”, para usar outra
coluna na classificação basta clicar sobre nome dela.
14. Gerenciador de Pastas – Nesta seção é possível adi-
cionar, renomear e apagar as suas pastas. As pastas são um
modo de organizar seu conteúdo, armazenando suas men-
sagens por temas. Quando seu e-mail é criado não existem
pastas nesta seção, isso deve ser feito pelo usuário de acor-
do com suas necessidades.
15. Contas Externas – Este item é um link que abrirá a
seção onde pode ser feita uma configuração que permitirá
você acessar outras caixas postais diretamente da sua. O
próximo link, como o nome já diz, abre a janela de confi-

83
INFORMÁTICA

1. Criar email: aqui você clica quando quer redigir um e-mail e uma nova mensagem se abre.
2. Responder: quando você recebe uma mensagem e quer mandar uma resposta, basta clicar aqui e escrever sua res-
posta.
3. Responder a todos: quando você recebe um e-mail que foi endereçado a você e a outras pessoas (você pode saber
se isto ocorreu olhando para o campo Cc que aparece em seu painel de visualização) e quer mandar uma resposta para
todos que também receberam esta mensagem, basta clicar em “responder a todos”.
4. Encaminhar: quando você recebe um e-mail e quer mandá-lo para outra (s) pessoa(s), basta clicar em “encaminhar”
e essa mensagem será enviada para o(s) destinatário(s) que você endereçar.
5. Imprimir: quando você quiser imprimir um e-mail, basta clicar nesse botão indicado que uma nova janela se abre e
nela você define o que deseja que seja impresso.
6. Excluir: quando você quiser excluir uma mensagem, basta clicar na mensagem (em sua lista de mensagens) e usar o
botão excluir da barra de ferramentas. Sua mensagem irá para a Pasta Itens excluídos.
7. Enviar e receber: clicando nesse botão, as mensagens que estão em sua Caixa de Saída serão enviadas e as mensa-
gens que estão em seu servidor chegarão a seu Outlook.
8. Endereços: este botão faz com que seu Catálogo de Endereços (seus contatos) se abra.
9. Localizar: este botão é útil quando você quer encontrar uma mensagem que esteja em seu Outlook. Ao clicar em
“Localizar”, uma nova janela se abre e você pode indicar os critérios de sua busca, preenchendo os campos que estão em
branco e clicando em “localizar agora”.

Se você quiser localizar uma mensagem em uma pasta ou uma pessoa que faça parte de seu catálogo de endereços,
pode clicar na setinha que está ao lado da pasta e alguns itens específicos aparecem, como você pode ver na imagem
abaixo:

Basta indicar o que você quer localizar e uma nova janela se abre e você preenche com os dados que interessam para
sua busca.

84
INFORMÁTICA

Obs.: Se você passar o mouse sobre cada um dos botões da barra de ferramentas, poderá ver uma caixa de diálogo que
descreve a função do botão, conforme imagem abaixo:

Ícones de listas de mensagens do Outlook Express


Os ícones a seguir aparecem nos e-mails e indicam a prioridade das mensagens, se as mensagens possuem arquivos
anexados ou ainda se as mensagens estão marcadas como lidas ou não lidas.
Veja o que eles significam:

Como criar uma conta de e-mail


Para adicionar uma conta de e-mail em seu Outlook faça o seguinte:
Entre em contato com seu provedor de serviços de Internet ou do administrador da rede local e informe-se sobre o tipo
de servidor de e-mail usado para a entrada e para a saída dos e-mails.
Você precisará saber o tipo de servidor usado : POP3 (Post Office Protocol), IMAP (Internet Message Access Protocol)
ou HTTP (Hypertext Transfer Protocol). Precisa também saber o nome da conta e a senha, o nome do servidor de e-mail de
entrada e, para POP3 e IMAP, o nome de um servidor de e-mail de saída, geralmente SMTP (Simple Mail Transfer Protocol)

85
INFORMÁTICA

Vamos à configuração:

No menu Ferramentas, clique em Contas.

Irá se abrir uma nova janela chamada Contas na Inter-


net, clique em Adicionar.

Após digitar o “Endereço de correio eletrônico”, no


campo correspondente, clique no botão “Adicionar” e
em “OK”.
Da mesma forma, você vai acrescentando novos no-
mes ao seu “Catálogo de endereços”.
Quando for mandar uma mensagem, ao clicar em
“Para”, esse catálogo se abre e fica bem mais fácil e rápido
localizar o destinatário.
Para acrescentar contatos em sua lista de endereços
Para acrescentar endereços eletrônicos de pessoas
(contatos) em sua lista de endereços faça o seguinte:
Clique em Email e o Assistente para conexão com a In- Clique no botão Endereços.
ternet irá se abrir. Basta seguir as instruções para estabele- Em seguida, selecione “Novo” e “Novo Contato”.
cer uma conexão com um servidor de e-mail ou de notícias Escreva o nome e o novo endereço eletrônico (tipo:
e ir preenchendo os campos de acordo com seus dados. nome@provedor.com.br).
Observação: Cada usuário pode criar várias contas de Clique em OK.
e-mail, repetindo o procedimento descrito acima para cada
conta.

Criar um catálogo de endereços


Antes de mais nada, é preciso definir um Catálogo de
Endereços contendo algumas pessoas e seus respectivos
endereços de e-mail.
Para criar um catálogo de endereços no Outlook Ex-
press (versão 5 e posteriores), faça o seguinte:
Clique, no menu principal do Outlook, em Ferramentas
/ Catálogo de Endereços. Obs.: Caso você tenha recebido um e-mail de alguém e
Uma janela de nome: “Catálogo de Endereços - identi- queira adicionar esta pessoa a seu catálogo de endereços,
dade principal” se abrirá. basta clicar (com o botão direito do mouse) sobre o nome
Clique em “Novo” e escolha a opção: “Novo Contato” do remetende e escolher a opção “Adicionar remetente ao
Mais uma janela se abre: “Propriedades de”. Aqui você catálogo de endereços”.
vai digitar todos os dados que deseja para cada um dos Pronto, esse contato já estará em seu catálogo de en-
contatos. dereços.

86
INFORMÁTICA

Abrir anexos
Para ver um anexo de arquivo, faça o seguinte:
1. No painel de visualização, clique no ícone de clipe de
papel no cabeçalho da mensagem e, em seguida, clique no
nome do arquivo.

Você pode também configurar o Outlook Express para ou então:


que seus destinatários sejam adicionados automaticamen-
te ao catálogo de endereços quando você responder a uma Na parte superior da janela da mensagem, clique duas
mensagem. Para adicionar todos os destinatários de res- vezes no ícone de anexo de arquivo no cabeçalho da men-
postas ao catálogo de endereços, faça o seguinte: sagem.
Clique em Ferramentas.
Clique em Opções.
Na guia Enviar, clique em Incluir automaticamente no ca-
tálogo de endereços os destinatários das minhas respostas.

Salvar um rascunho
Para salvar um rascunho da mensagem para usar mais
tarde, faça o seguinte:
Com sua mensagem aberta, clique em Arquivo.
A seguir, clique em Salvar.

(Quando uma mensagem tem um arquivo anexado,


um ícone de clipe de papel é exibido ao lado dela na lista
de mensagens.)

Salvar anexos
Para salvar um anexo de arquivo de seu e-mail, faça o
seguinte:
Clique na mensagem que tem o arquivo que você quer
salvar.
No menu Arquivo, clique em Salvar anexos.

Você também pode clicar em Salvar como para salvar


uma mensagem de e-mail em outros arquivos de seu com-
putador no formato de e-mail (.eml), texto (.txt) ou HTML
(.htm ou html).

87
INFORMÁTICA

Uma nova janela se abre. Clique no(s) anexo(s) que


você quer salvar.
Antes de clicar em Salvar, confira se o local indicado
na caixa abaixo é onde você quer salvar seus anexos.
(Caso não seja, clique em “Procurar” e escolha outra pas-
ta ou arquivo.)
Clique em Salvar.

Ou, se preferir, utilize o seguinte recurso:


No menu Mensagem, aponte para Definir prioridade e
selecione uma opção de prioridade.

Se você preferir, pode salvar o anexo no painel de vi-


sualização:
Clique no ícone de clipe de papel
Clique em Salvar anexos.
Observação: Esta configuração atribui a prioridade so-
mente para a mensagem que você está redigindo no mo-
mento.

Verificar novas mensagens


Para saber se chegaram novas mensagens, faça o se-
guinte:
Com seu Outlook aberto, clique em Enviar/receber na
barra de ferramentas.
Prioridade de uma mensagem
Ao enviar uma nova mensagem ou uma resposta a um
e-mail, você pode atribuir uma prioridade à mensagem,
para que o destinatário saiba se deve lê-la imediatamen-
te (prioridade alta) ou quando houver tempo (prioridade
baixa). Lembre-se de que todas as mensagens da caixa de saí-
As mensagens com prioridade alta têm um ponto de da serão enviadas também.
exclamação ao seu lado. Se preferir apenas receber ou apenas enviar mensa-
A prioridade baixa é indicada por uma seta para baixo. gens, você pode usar o seguinte recurso:
Se você deixar marcada a opção Normal, nenhum íco- Clique em Ferramentas / Enviar e receber, escolha a
ne referente à prioridade aparecerá ao lado da mensagem. opção que deseja e clique nela.
Para indicar a prioridade de uma mensagem de um
e-mail que você vai mandar, faça o seguinte:
Na janela Nova mensagem, clique em Prioridade na
barra de ferramentas, clique na setinha que está bem ao
lado e selecione a prioridade desejada.
Você pode deixar acionado um “alarme” que avisa
quando chegaram novas mensagens. Veja mais em Alar-
mes.

88
INFORMÁTICA

Navegar pelo Outlook Criar novas pastas


Usando a lista de mensagens e o painel de visualiza- Para organizar seu Outlook, você pode criar ou adicio-
ção, você pode exibir a lista de mensagens e ler mensagens nar quantas pastas quiser.
individuais ao mesmo tempo. No menu Arquivo, clique em Pasta.
A lista Pastas contém pastas de e-mail, servidores de Clique em Nova.
notícias e grupos de notícias e você pode alternar facilmen-
te entre eles.
Você também pode criar novas pastas para organizar e
classificar suas mensagens e configurar regras de mensa-
gens para colocar automaticamente em uma pasta especí-
fica o e-mail de acordo com o assunto, remetente, grupo,
enfim, da forma mais prática para seu uso.
Você também pode criar seus próprios modos de exibi-
ção para personalizar a maneira como visualiza suas men-
sagens.

Pastas Padrões
As pastas padrões do Outlook não podem ser altera-
das. Você poderá criar outras pastas, mas não deve mexer
nas seguintes pastas: Uma nova janela se abrirá.
Na caixa de texto Nome da pasta, digite o nome que
deseja dar à pasta e, em seguida, selecione o local para a
nova pasta.

Lembre-se de que o Outlook Express vai criar sua pasta


nova dentro daquela que estiver selecionada no momen-
1. Caixa de Entrada: local padrão para onde vão as to. Se você selecionar, por exemplo, “Caixa de Entrada” e
mensagens que chegam ao seu Outlook. (Você pode criar solicitar uma nova pasta, esta será posicionada dentro da
pastas e regras para mudar o lugar para o qual suas men- Caixa de Entrada.
sagens devam ser encaminhadas.). Se o que você quer é uma nova pasta, independente
2. Caixa de Saída: aqui ficam os e-mails que você já das que você já criou, selecione sempre o item Pastas Lo-
escreveu e que vai mandar para o(s) destinatário(s). cais .
3. Itens Enviados: nesta pasta ficam guardados os Dê um nome e selecione o local onde quer que fique
e-mails que você já mandou. esta nova pasta que você acabou de criar.
4. Itens Excluídos: aqui ficam as mensagens que você
já excluiu de outra(s) pasta(s), mas continuam em seu Você também pode fazer o seguinte:
Outlook. Abra o Outlook.
5. Rascunhos: as mensagens que você está escrevendo Clique com o botão direito do mouse em “Caixa de
podem ficar guardadas aqui enquanto você não as acaba entrada”
de compor definitivamente. Veja como salvar uma mensa- Aparecerá uma janela com alguns itens. Clique em
gem na pasta Rascunhos. “Nova Pasta”.
Irá se abrir uma nova janela, assim

89
INFORMÁTICA

INTERNET: NAVEGAÇÃO INTERNET,


CONCEITOS DE URL, LINKS, SITES,
BUSCA E IMPRESSÃO DE PÁGINAS.

INTERNET
“Imagine que fosse descoberto um continente tão
vasto que suas dimensões não tivessem fim. Imagine
um mundo novo, com tantos recursos que a ganância
do futuro não seria capaz de esgotar; com tantas oportuni-
dades que os empreendedores seriam poucos para apro-
veitá-las; e com um tipo peculiar de imóvel que se
No campo “Nome da Pasta”, em que escrevi: “Colocar expandiria com o desenvolvimento.”
aqui o nome da nova...” , você escreve o nome que quer dar John P. Barlow
à sua pastinha. Os Estados Unidos temiam que em um ataque nuclear
Agora, basta clicar em “OK” e prontinho. ficassem sem comunicação entre a Casa Branca e o Pentá-
Desta mesma forma, você poderá fazer quantas pastas gono.
quiser. Este meio de comunicação “infalível”, até o fim da dé-
Se você quiser, pode também criar uma pasta para ar- cada de 60, ficou em poder exclusivo do governo conec-
quivo de e-mails fora do programa, dentro de “Meus Do- tando bases militares, em quatro localidades.
cumentos”, por exemplo, ou em um disquete e nela salvar Nos anos 70, seu uso foi liberado para instituições
todos os e-mails que que guardar. norte-americanas de pesquisa que desejassem aprimorar
Para salvar, basta clicar em Arquivo/Salvar como e es- a tecnologia, logo vinte e três computadores foram conec-
colher a pasta onde quer guardar seus e-mails (fora do tados, porém o padrão de conversação entre as máquinas
Outlook). se tornou impróprio pela quantidade de equipamentos.
A mensagem será salva com o nome que aparece no Era necessário criar um modelo padrão e univer-
assunto do e-mail. Se você tiver várias mensagens com o sal para que as máquinas continuassem trocando da-
dos, surgiu então o Protocolo Padrão TCP/IP, que permi-
mesmo assunto, e for guardá-las em uma mesma pasta,
tiria portanto que mais outras máquinas fossem inseridas
renomeie (preencha com um novo nome no espaço que
àquela rede.
aparece na caixa de diálogo, conforme imagem abaixo) na
hora de salvar.
Com esses avanços, em 1972 é criado o correio eletrô-
nico, o E-mail, permitindo a troca de mensagens entre as
máquinas que compunham aquela rede de pesquisa, assim
no ano seguinte a rede se torna internacional.
Na década de 80, a Fundação Nacional de Ciência do
Brasil conectou sua grande rede à ARPANET, gerando aqui-
lo que conhecemos hoje como internet, auxiliando portan-
to o processo de pesquisa em tecnologia e outras áreas a
nível mundial, além de alimentar as forças armadas brasi-
leiras de informação de todos os tipos, até que em 1990
caísse no domínio público.
Com esta popularidade e o surgimento de softwares
de navegação de interface amigável, no fim da década de
90, pessoas que não tinham conhecimentos profundos de
informática começaram a utilizar a rede internacional.

Para manter a mensagem como a original, salve no for- Acesso à Internet


mato correio .eml. O ISP, Internet Service Provider, ou Provedor de Serviço
Veja em Regras para mensagens, sobre como criar re- de Internet, oferece principalmente serviço de acesso à In-
gras para que suas mensagens sejam “colocadas” direta- ternet, adicionando serviços como e-mail, hospedagem de
mente nas pastas em que você criou. sites ou blogs, ou seja, são instituições que se conectam
à Internet com o objetivo de fornecer serviços à ela
relacionados, e em função do serviço classificam-se em:
• Provedores de Backbone: São instituições que cons-
troem e administram backbones de longo alcance, ou seja,
estrutura física de conexão, com o objetivo de fornecer
acesso à Internet para redes locais;

90
INFORMÁTICA

• Provedores de Acesso: São instituições que se conec- HTML, Hyper Text Markut language ou Linguagem de
tam à Internet via um ou mais acessos dedicados e disponi- Marcação de Hipertexto
bilizam acesso à terceiros a partir de suas instalações; É a linguagem com a qual se cria as páginas para a
• Provedores de Informação: São instituições que dis- web.
ponibilizam informação através da Internet. Suas principais características são:
• Portabilidade (Os documentos escritos em HTML de-
Endereço Eletrônico ou URL vem ter aparência semelhante nas diversas plataformas de
Para se localizar um recurso na rede mundial, deve-se trabalho);
conhecer o seu endereço. • Flexibilidade (O usuário deve ter a liberdade de “cus-
Este endereço, que é único, também é considerado sua tomizar” diversos elementos do documento, como o tama-
URL (Uniform Resource Locator), ou Localizador de Recur- nho padrão da letra, as cores, etc);
sos Universal. Boa parte dos endereços apresenta-se assim: • Tamanho Reduzido (Os documentos devem ter
www.xxxx.com.br um tamanho reduzido, a fim de economizar tempo na
Onde: transmissão através da Internet, evitando longos perío-
www = protocolo da World Wide Web dos de espera e congestionamento na rede).
xxx = domínio
com = comercial Browser ou Navegador
br = brasil É o programa específico para visualizar as páginas da
web.
WWW = World Wide Web ou Grande Teia Mundial O Browser lê e interpreta os documentos escritos em
HTML, apresentando as páginas formatadas para os
É um serviço disponível na Internet que possui um con- usuários.
junto de documentos espalhados por toda rede e disponi-
bilizados a qualquer um. ARQUITETURAS DE REDES
Estes documentos são escritos em hipertexto, que uti- As modernas redes de computadores são projetadas
liza uma linguagem especial, chamada HTML. de forma altamente estruturada. Nas seções seguintes exa-
minaremos com algum detalhe a técnica de estruturação.
Domínio
HIERARQUIAS DE PROTOCOLOS
Designa o dono do endereço eletrônico em ques-
Para reduzir a complexidade de projeto, a maioria das
tão, e onde os hipertextos deste empreendimento estão
redes é organizada em camadas ou níveis, cada uma cons-
localizados. Quanto ao tipo do domínio, existem:
truída sobre sua predecessora. O número de camadas, o
.com = Instituição comercial ou provedor de serviço
nome, o conteúdo e a função de cada camada diferem de
.edu = Instituição acadêmica
uma rede para outra. No entanto, em todas as redes, o pro-
.gov = Instituição governamental
pósito de cada camada é oferecer certos serviços às cama-
.mil = Instituição militar norte-americana das superiores, protegendo essas camadas dos detalhes de
.net = Provedor de serviços em redes como os serviços oferecidos são de fato implementados.
.org = Organização sem fins lucrativos A camada n em uma máquina estabelece uma con-
HTTP, Hyper Texto Transfer Protocol ou Protocolo de versão com a camada n em outra máquina. As regras e
Trasferência em Hipertexto convenções utilizadas nesta conversação são chamadas
É um protocolo ou língua específica da internet, res- coletivamente de protocolo da camada n, conforme ilus-
ponsável pela comunicação entre computadores. trado na Figura abaixo para uma rede com sete camadas.
Um hipertexto é um texto em formato digital, e As entidades que compõem as camadas correspondentes
pode levar a outros, fazendo o uso de elementos espe- em máquinas diferentes são chamadas de processos par-
ciais (palavras, frases, ícones, gráficos) ou ainda um Mapa ceiros. Em outras palavras, são os processos parceiros que
Sensitivo o qual leva a outros conjuntos de informação na se comunicam utilizando o protocolo.
forma de blocos de textos, imagens ou sons. Na verdade, nenhum dado é transferido diretamente
Assim, um link ou hiperlink, quando acionado com o da camada n em uma máquina para a camada n em outra
mouse, remete o usuário à outra parte do documento ou máquina. Em vez disso, cada camada passa dados e infor-
outro documento. mações de controle para a camada imediatamente abaixo,
até que o nível mais baixo seja alcançado. Abaixo do nível
Home Page 1 está o meio físico de comunicação, através do qual a co-
Sendo assim, home page designa a página inicial, prin- municação ocorre. Na Figura abaixo, a comunicação virtual
cipal do site ou web page. é mostrada através de linhas pontilhadas e a comunicação
É muito comum os usuários confundirem um Blog ou física através de linhas sólidas.
Perfil no Orkut com uma Home Page, porém são coisas dis-
tintas, aonde um Blog é um diário e um Perfil no Orkut é
um Profile, ou seja um hipertexto que possui informações
de um usuário dentro de uma comunidade virtual.

91
INFORMÁTICA

Na internet, o princípio é o mesmo. Para que o seu


computador seja encontrado e possa fazer parte da rede
mundial de computadores, necessita ter um endereço úni-
co. O mesmo vale para websites: este fica em um servidor,
que por sua vez precisa ter um endereço para ser localiza-
do na internet. Isto é feito pelo endereço IP (IP Address),
recurso que também é utilizado para redes locais, como a
existente na empresa que você trabalha, por exemplo.
O endereço IP é uma sequência de números composta
de 32 bits. Esse valor consiste em um conjunto de quatro
sequências de 8 bits. Cada uma destas é separada por um
ponto e recebe o nome de octeto ou simplesmente byte, já
que um byte é formado por 8 bits. O número 172.31.110.10
é um exemplo. Repare que cada octeto é formado por nú-
meros que podem ir de 0 a 255, não mais do que isso.

Entre cada par de camadas adjacentes há uma interfa-


ce. A interface define quais operações primitivas e serviços
a camada inferior oferece à camada superior. Quando os
projetistas decidem quantas camadas incluir em uma rede
e o que cada camada deve fazer, uma das considerações
mais importantes é definir interfaces limpas entre as cama-
das. Isso requer, por sua vez, que cada camada desempe-
nhe um conjunto específico de funções bem compreendi- A divisão de um IP em quatro partes facilita a organi-
das. Além de minimizar a quantidade de informações que zação da rede, da mesma forma que a divisão do seu en-
deve ser passada de camada em camada, interfaces bem dereço em cidade, bairro, CEP, número, etc, torna possível
definidas também tornam fácil a troca da implementação a organização das casas da região onde você mora. Neste
de uma camada por outra implementação completamente sentido, os dois primeiros octetos de um endereço IP po-
diferente (por exemplo, trocar todas as linhas telefônicas dem ser utilizados para identificar a rede, por exemplo. Em
por canais de satélite), pois tudo o que é exigido da nova uma escola que tem, por exemplo, uma rede para alunos
implementação é que ela ofereça à camada superior exa- e outra para professores, pode-se ter 172.31.x.x para uma
tamente os mesmos serviços que a implementação antiga rede e 172.32.x.x para a outra, sendo que os dois últimos
oferecia. octetos são usados na identificação de computadores.
O conjunto de camadas e protocolos é chamado de
arquitetura de rede. A especificação de arquitetura deve Classes de endereços IP
conter informações suficientes para que um implementa- Neste ponto, você já sabe que os endereços IP podem
dor possa escrever o programa ou construir o hardware de ser utilizados tanto para identificar o seu computador den-
cada camada de tal forma que obedeça corretamente ao tro de uma rede, quanto para identificá-lo na internet.
protocolo apropriado. Nem os detalhes de implementação Se na rede da empresa onde você trabalha o seu com-
nem a especificação das interfaces são parte da arquitetura, putador tem, como exemplo, IP 172.31.100.10, uma má-
pois esses detalhes estão escondidos dentro da máquina e quina em outra rede pode ter este mesmo número, afinal,
não são visíveis externamente. Não é nem mesmo neces-
ambas as redes são distintas e não se comunicam, sequer
sário que as interfaces em todas as máquinas em uma rede
sabem da existência da outra. Mas, como a internet é uma
sejam as mesmas, desde que cada máquina possa usar cor-
rede global, cada dispositivo conectado nela precisa ter um
retamente todos os protocolos.
endereço único. O mesmo vale para uma rede local: nesta,
O endereço IP cada dispositivo conectado deve receber um endereço úni-
Quando você quer enviar uma carta a alguém, você... co. Se duas ou mais máquinas tiverem o mesmo IP, tem-se
Ok, você não envia mais cartas; prefere e-mail ou deixar um então um problema chamado “conflito de IP”, que dificulta
recado no Facebook. Vamos então melhorar este exemplo: a comunicação destes dispositivos e pode inclusive atrapa-
quando você quer enviar um presente a alguém, você ob- lhar toda a rede.
tém o endereço da pessoa e contrata os Correios ou uma Para que seja possível termos tanto IPs para uso em re-
transportadora para entregar. É graças ao endereço que é des locais quanto para utilização na internet, contamos com
possível encontrar exatamente a pessoa a ser presenteada. um esquema de distribuição estabelecido pelas entidades
Também é graças ao seu endereço - único para cada resi- IANA (Internet Assigned Numbers Authority) e ICANN (In-
dência ou estabelecimento - que você recebe suas contas ternet Corporation for Assigned Names and Numbers) que,
de água, aquele produto que você comprou em uma loja basicamente, divide os endereços em três classes principais
on-line, enfim. e mais duas complementares. São elas:

92
INFORMÁTICA

Classe A: 0.0.0.0 até 127.255.255.255 - permite até 128 xão à internet e a compartilhe com todos os dispositivos
redes, cada uma com até 16.777.214 dispositivos conectados; conectados a ele. Com isso, somente este equipamento
Classe B: 128.0.0.0 até 191.255.255.255 - permite até precisará de um endereço IP para acesso à rede mundial
16.384 redes, cada uma com até 65.536 dispositivos; de computadores.
Classe C: 192.0.0.0 até 223.255.255.255 - permite até
2.097.152 redes, cada uma com até 254 dispositivos; Máscara de sub-rede
Classe D: 224.0.0.0 até 239.255.255.255 - multicast; As classes IP ajudam na organização deste tipo de en-
Classe E: 240.0.0.0 até 255.255.255.255 - multicast re- dereçamento, mas podem também representar desperdí-
servado. cio. Uma solução bastante interessante para isso atende
As três primeiras classes são assim divididas para aten- pelo nome de máscara de sub-rede, recurso onde parte
der às seguintes necessidades: dos números que um octeto destinado a identificar dis-
- Os endereços IP da classe A são usados em locais positivos conectados (hosts) é “trocado” para aumentar
onde são necessárias poucas redes, mas uma grande quan- a capacidade da rede. Para compreender melhor, vamos
tidade de máquinas nelas. Para isso, o primeiro byte é utili- enxergar as classes A, B e C da seguinte forma:
zado como identificador da rede e os demais servem como - A: N.H.H.H;
identificador dos dispositivos conectados (PCs, impresso- - B: N.N.H.H;
ras, etc); - C: N.N.N.H.
- Os endereços IP da classe B são usados nos casos
onde a quantidade de redes é equivalente ou semelhante N significa Network (rede) e H indica Host. Com o
à quantidade de dispositivos. Para isso, usam-se os dois uso de máscaras, podemos fazer uma rede do N.N.H.H se
primeiros bytes do endereço IP para identificar a rede e os “transformar” em N.N.N.H. Em outras palavras, as másca-
restantes para identificar os dispositivos; ras de sub-rede permitem determinar quantos octetos e
- Os endereços IP da classe C são usados em locais que bits são destinados para a identificação da rede e quantos
requerem grande quantidade de redes, mas com poucos
são utilizados para identificar os dispositivos.
dispositivos em cada uma. Assim, os três primeiros bytes
Para isso, utiliza-se, basicamente, o seguinte esque-
são usados para identificar a rede e o último é utilizado
ma: se um octeto é usado para identificação da rede, este
para identificar as máquinas.
receberá a máscara de sub-rede 255. Mas, se um octeto
Quanto às classes D e E, elas existem por motivos es-
é aplicado para os dispositivos, seu valor na máscara de
peciais: a primeira é usada para a propagação de pacotes
sub-rede será 0 (zero). A tabela a seguir mostra um exem-
especiais para a comunicação entre os computadores, en-
plo desta relação:
quanto que a segunda está reservada para aplicações futu-
ras ou experimentais.
Vale frisar que há vários blocos de endereços reserva- I d e n t i f i - Identifica-
dos para fins especiais. Por exemplo, quando o endereço Máscara de
Classe Endereço IP cador da dor do com-
começa com 127, geralmente indica uma rede “falsa”, isto sub-rede
rede putador
é, inexistente, utilizada para testes. No caso do endereço
127.0.0.1, este sempre se refere à própria máquina, ou seja, A 10.2.68.12 10 2.68.12 255.0.0.0
ao próprio host, razão esta que o leva a ser chamado de B 172.31.101.25 172.31 101.25 255.255.0.0
localhost. Já o endereço 255.255.255.255 é utilizado para C 192.168.0.10 192.168.0 10 255.255.255.0
propagar mensagens para todos os hosts de uma rede de
maneira simultânea.
Você percebe então que podemos ter redes com más-
cara 255.0.0.0, 255.255.0.0 e 255.255.255.0, cada uma in-
Endereços IP privados
dicando uma classe. Mas, como já informado, ainda pode
Há conjuntos de endereços das classes A, B e C que são
haver situações onde há desperdício. Por exemplo, supo-
privados. Isto significa que eles não podem ser utilizados
nha que uma faculdade tenha que criar uma rede para
na internet, sendo reservados para aplicações locais. São,
cada um de seus cinco cursos. Cada curso possui 20 com-
essencialmente, estes:
putadores. A solução seria então criar cinco redes classe
-Classe A: 10.0.0.0 à 10.255.255.255;
-Classe B: 172.16.0.0 à 172.31.255.255; C? Pode ser melhor do que utilizar classes B, mas ainda
-Classe C: 192.168.0.0 à 192.168.255.255. haverá desperdício. Uma forma de contornar este proble-
ma é criar uma rede classe C dividida em cinco sub-redes.
Suponha então que você tenha que gerenciar uma rede Para isso, as máscaras novamente entram em ação.
com cerca de 50 computadores. Você pode alocar para es- Nós utilizamos números de 0 a 255 nos octetos, mas
tas máquinas endereços de 192.168.0.1 até 192.168.0.50, estes, na verdade, representam bytes (linguagem binária).
por exemplo. Todas elas precisam de acesso à internet. O 255 em binário é 11111111. O número zero, por sua vez,
que fazer? Adicionar mais um IP para cada uma delas? Não. é 00000000. Assim, a máscara de um endereço classe C,
Na verdade, basta conectá-las a um servidor ou equipa- 255.255.255.0, é:
mento de rede - como um roteador - que receba a cone- 11111111.11111111.11111111.00000000

93
INFORMÁTICA

Perceba então que, aqui, temos uma máscara formada exemplo, o site www.infowester.com é requisitado, o sistema
por 24 bits 1: 11111111 + 11111111 + 11111111. Para criar- envia a solicitação a um servidor responsável por termina-
mos as nossas sub-redes, temos que ter um esquema com ções “.com”. Esse servidor localizará qual o IP do endereço
25, 26 ou mais bits, conforme a necessidade e as possibilida- e responderá à solicitação. Se o site solicitado termina com
des. Em outras palavras, precisamos trocar alguns zeros do “.br”, um servidor responsável por esta terminação é consul-
último octeto por 1. tado e assim por diante.

Suponha que trocamos os três primeiros bits do último IPv6


octeto (sempre trocamos da esquerda para a direita), resul- O mundo está cada vez mais conectado. Se, em um
tando em: passado não muito distante, você conectava apenas o PC
11111111.11111111.11111111.11100000 da sua casa à internet, hoje o faz com o celular, com o seu
Se fizermos o número 2 elevado pela quantidade de bits notebook em um serviço de acesso Wi-Fi no aeroporto e
“trocados”, teremos a quantidade possível de sub-redes. Em assim por diante. Somando este aspecto ao fato de cada
nosso caso, temos 2^3 = 8. Temos então a possibilidade de vez mais pessoas acessarem a internet no mundo inteiro,
criar até oito sub-redes. Sobrou cinco bits para o endereça- nos deparamos com um grande problema: o número de
mento dos host. Fazemos a mesma conta: 2^5 = 32. Assim, IPs disponíveis deixa de ser suficiente para toda as (futuras)
temos 32 dispositivos em cada sub-rede (estamos fazendo aplicações.
estes cálculos sem considerar limitações que possam impe- A solução para este grande problema (grande mesmo,
dir o uso de todos os hosts e sub-redes). afinal, a internet não pode parar de crescer!) atende pelo
11100000 corresponde a 224, logo, a máscara resultante nome de IPv6, uma nova especificação capaz de suportar
é 255.255.255.224. até - respire fundo - 340.282.366.920.938.463.463.374.607.
Perceba que esse esquema de “trocar” bits pode ser em- 431.768.211.456 de endereços, um número absurdamente
pregado também em endereços classes A e B, conforme a alto!
necessidade. Vale ressaltar também que não é possível utili-
zar 0.0.0.0 ou 255.255.255.255 como máscara.
IP estático e IP dinâmico
IP estático (ou fixo) é um endereço IP dado permanen-
temente a um dispositivo, ou seja, seu número não muda,
exceto se tal ação for executada manualmente. Como exem-
plo, há casos de assinaturas de acesso à internet via ADSL
onde o provedor atribui um IP estático aos seus assinantes.
Assim, sempre que um cliente se conectar, usará o mesmo
IP.
O IP dinâmico, por sua vez, é um endereço que é dado
a um computador quando este se conecta à rede, mas que
muda toda vez que há conexão. Por exemplo, suponha que
você conectou seu computador à internet hoje. Quando O IPv6 não consiste, necessariamente, apenas no au-
você conectá-lo amanhã, lhe será dado outro IP. Para enten- mento da quantidade de octetos. Um endereço do tipo
der melhor, imagine a seguinte situação: uma empresa tem pode ser, por exemplo:
80 computadores ligados em rede. Usando IPs dinâmicos, a FEDC:2D9D:DC28:7654:3210:FC57:D4C8:1FFF
empresa disponibiliza 90 endereços IP para tais máquinas.
Como nenhum IP é fixo, um computador receberá, quando Finalizando
se conectar, um endereço IP destes 90 que não estiver sen- Com o surgimento do IPv6, tem-se a impressão de que
do utilizado. É mais ou menos assim que os provedores de a especificação tratada neste texto, o IPv4, vai sumir do
internet trabalham. mapa. Isso até deve acontecer, mas vai demorar bastante.
O método mais utilizado na distribuição de IPs dinâmi- Durante essa fase, que podemos considerar de transição,
cos é o protocolo DHCP (Dynamic Host Configuration Pro- o que veremos é a “convivência” entre ambos os padrões.
tocol). Não por menos, praticamente todos os sistemas operacio-
nais atuais e a maioria dos dispositivos de rede estão aptos
IP nos sites a lidar tanto com um quanto com o outro. Por isso, se você
Você já sabe que os sites na Web também necessitam é ou pretende ser um profissional que trabalha com redes
de um IP. Mas, se você digitar em seu navegador www.in- ou simplesmente quer conhecer mais o assunto, procure se
fowester.com, por exemplo, como é que o seu computador aprofundar nas duas especificações.
sabe qual o IP deste site ao ponto de conseguir encontrá-lo? A esta altura, você também deve estar querendo des-
Quando você digitar um endereço qualquer de um site, cobrir qual o seu IP. Cada sistema operacional tem uma
um servidor de DNS (Domain Name System) é consultado. forma de mostrar isso. Se você é usuário de Windows, por
Ele é quem informa qual IP está associado a cada site. O sis- exemplo, pode fazê-lo digitando cmd em um campo do
tema DNS possui uma hierarquia interessante, semelhante a Menu Iniciar e, na janela que surgir, informar ipconfig /all
uma árvore (termo conhecido por programadores). Se, por e apertar Enter. Em ambientes Linux, o comando é ifconfig.

94
INFORMÁTICA

A pesquisa pode ser realizada com uma palavra, refe-


rente ao assunto desejado. Por exemplo, você quer pesquisar
sobre amortecedores, caso não encontre nada como amor-
tecedores, procure como autopeças, e assim sucessivamente.

Barra de endereços

A Barra de Endereços possibilita que se possa navegar em pá-


ginas da internet, bastando para isto digitar o endereço da página.
Alguns sites interessantes:
Perceba, no entanto, que se você estiver conectado a par- • www.diariopopular.com.br (Jornal Diário Popular)
tir de uma rede local - tal como uma rede wireless - visuali- • www.ufpel.tche.br (Ufpel)
zará o IP que esta disponibiliza à sua conexão. Para saber o • www.cefetrs.tche.br (Cefet)
endereço IP do acesso à internet em uso pela rede, você pode • www.servidor.gov.br (Informações sobre servidor público)
visitar sites como whatsmyip.org. • www.siapenet.gog.br (contracheque)
• www.pelotas.com.br (Site Oficial de Pelotas)
Provedor • www.mec.gov.br (Ministério da Educação)
O provedor é uma empresa prestadora de serviços que
oferece acesso à Internet. Para acessar a Internet, é necessário Identificação de endereços de um site
conectar-se com um computador que já esteja na Internet (no Exemplo: http://www.pelotas.com.br
caso, o provedor) e esse computador deve permitir que seus http:// -> (Hiper Text Tranfer Protocol) protocolo de co-
usuários também tenham acesso a Internet. municação
No Brasil, a maioria dos provedores está conectada à WWW -> (World Wide Web) Grande rede mundial
Embratel, que por sua vez, está conectada com outros com- pelotas -> empresa ou organização que mantém o site
putadores fora do Brasil. Esta conexão chama-se link, que é .com -> tipo de organização
a conexão física que interliga o provedor de acesso com a ......br -> identifica o país
Embratel. Neste caso, a Embratel é conhecida como backbo-
ne, ou seja, é a “espinha dorsal” da Internet no Brasil. Pode-se Tipos de Organizações:
imaginar o backbone como se fosse uma avenida de três pis- .edu -> instituições educacionais. Exemplo: michigam.edu
tas e os links como se fossem as ruas que estão interligadas .com -> instituções comerciais. Exemplo: microsoft.com
nesta avenida. .gov -> governamental. Exemplo: fazenda.gov
Tanto o link como o backbone possui uma velocidade .mil -> instalação militar. Exemplo: af.mil
de transmissão, ou seja, com qual velocidade ele transmite .net -> computadores com funções de administrar redes.
os dados. Esta velocidade é dada em bps (bits por segundo). Exemplo: embratel.net
Deve ser feito um contrato com o provedor de acesso, que .org -> organizações não governamentais. Exemplo: care.org
fornecerá um nome de usuário, uma senha de acesso e um
endereço eletrônico na Internet. Home Page
Pela definição técnica temos que uma Home Page é um
URL - Uniform Resource Locator arquivo ASCII (no formato HTML) acessado de computado-
Tudo na Internet tem um endereço, ou seja, uma identifi- res rodando um Navegador (Browser), que permite o acesso
cação de onde está localizado o computador e quais recursos às informações em um ambiente gráfico e multimídia. Todo
este computador oferece. Por exemplo, a URL: em hipertexto, facilitando a busca de informações dentro das
http://www.novaconcursos.com.br Home Pages.
Será mais bem explicado adiante. O endereço de Home Pages tem o seguinte formato:
http://www.endereço.com/página.html
Como descobrir um endereço na Internet?
Por exemplo, a página principal da Pronag:
http://www.pronag.com.br/index.html
Para que possamos entender melhor, vamos exemplificar.
Você estuda em uma universidade e precisa fazer algu-
PLUG-INS
mas pesquisas para um trabalho. Onde procurar as informa-
Os plug-ins são programas que expandem a capacidade
ções que preciso?
do Browser em recursos específicos - permitindo, por exem-
Para isso, existem na Internet os “famosos” sites de pro-
plo, que você toque arquivos de som ou veja filmes em vídeo
cura, que são sites que possuem um enorme banco de dados
(que contém o cadastro de milhares de Home Pages), que dentro de uma Home Page. As empresas de software vêm
permitem a procura por um determinado assunto. Caso a desenvolvendo plug-ins a uma velocidade impressionante.
palavra ou o assunto que foi procurado exista em alguma Maiores informações e endereços sobre plug-ins são en-
dessas páginas, será listado toda esta relação de páginas contradas na página:
encontradas. http://www.yahoo.com/Computers_and_Internet/Soft-
ware/Internet/World_Wide_Web/Browsers/Plug_Ins/Indices/

95
INFORMÁTICA

Atualmente existem vários tipos de plug-ins. Abaixo O navegador não precisa de nenhuma configuração
temos uma relação de alguns deles: especial para exibir uma página da Web, mas é necessário
- 3D e Animação (Arquivos VRML, MPEG, QuickTime, etc.). ajustar alguns parâmetros para que ele seja capaz de enviar
- Áudio/Vídeo (Arquivos WAV, MID, AVI, etc.). e receber algumas mensagens de correio eletrônico e aces-
- Visualizadores de Imagens (Arquivos JPG, GIF, BMP, sar grupos de discussão (news).
PCX, etc.). O World Wide Web foi inicialmente desenvolvido no
- Negócios e Utilitários Centro de Pesquisas da CERN (Conseil Europeen pour la
- Apresentações Recherche Nucleaire), Suíça. Originalmente, o WWW era
um meio para físicos da CERN trocar experiências sobre
FTP - Transferência de Arquivos suas pesquisas através da exibição de páginas de texto. Fi-
Permite copiar arquivos de um computador da Internet cou claro, desde o início, o imenso potencial que o WWW
para o seu computador. possuía para diversos tipos de aplicações, inclusive não
Os programas disponíveis na Internet podem ser: científicas.
• Freeware: Programa livre que pode ser distribuí- O WWW não dispunha de gráficos em seus primór-
do e utilizado livremente, não requer nenhuma taxa para dios, apenas de hipertexto. Entretanto, em 1993, o projeto
sua utilização, e não é considerado “pirataria” a cópia deste WWW ganhou força extra com a inserção de um visualiza-
programa. dor (também conhecido como browser) de páginas capaz
• Shareware: Programa demonstração que pode ser não apenas de formatar texto, mas também de exibir grá-
utilizado por um determinado prazo ou que contém alguns ficos, som e vídeo. Este browser chamava-se Mosaic e foi
limites, para ser utilizado apenas como um teste do progra- desenvolvido dentro da NCSA, por um time chefiado por
ma. Se o usuário gostar ele compra, caso contrário, não usa Mark Andreesen. O sucesso do Mosaic foi espetacular.
mais o programa. Na maioria das vezes, esses programas Depois disto, várias outras companhias passaram a
exibem, de tempos em tempos, uma mensagem avisando produzir browsers que deveriam fazer concorrência ao
que ele deve ser registrado. Outros tipos de shareware têm Mosaic. Mark Andreesen partiu para a criação da Netscape
tempo de uso limitado. Depois de expirado este tempo de Communications, criadora do browser Netscape.
teste, é necessário que seja feito a compra deste programa. Surgiram ainda o Cello, o AIR Mosaic, o SPRY Mosaic,
o Microsoft Internet Explorer, o Mozilla Firefox e muitos
Navegar nas páginas
outros browsers.
Consiste percorrer as páginas na internet a partir de
um documento normal e de links das próprias páginas.
Busca e pesquisa na web
Os sites de busca servem para procurar por um deter-
Como salvar documentos, arquivos e sites
minado assunto ou informação na internet.
Clique no menu Arquivo e na opção Salvar como.
Alguns sites interessantes:
• www.google.com.br
Como copiar e colar para um editor de textos
Selecionar o conteúdo ou figura da página. Clicar com • http://br.altavista.com
o botão direito do mouse e escolha a opção Copiar. • http://cade.search.yahoo.com
• http://br.bing.com/

Como fazer a pesquisa


Digite na barra de endereço o endereço do site de pes-
quisa. Por exemplo:

www.google.com.br

Abra o editor de texto clique em colar

Navegadores
O navegador de WWW é a ferramenta mais importante
para o usuário de Internet. É com ele que se podem vi-
sitar museus, ler revistas eletrônicas, fazer compras e até
participar de novelas interativas. As informações na Web
são organizadas na forma de páginas de hipertexto, cada
um com seu endereço próprio, conhecido como URL. Para
começar a navegar, é preciso digitar um desses endereços
no campo chamado Endereço no navegador. O software
estabelece a conexão e traz, para a tela, a página corres-
pondente.

96
INFORMÁTICA

Em pesquisar pode-se escolher onde será feita a pes- Como escolher palavra-chave
quisa. • Busca com uma palavra: retorna páginas que in-
cluam a palavra digitada.
• “Busca entre aspas”: a pesquisa só retorna páginas
que incluam todos os seus termos de busca, ou seja, toda a
sequência de termos que foram digitadas.
• Busca com sinal de mais (+): a pesquisa retorna
páginas que incluam todas as palavras aleatoriamente na
Os sites de pesquisa em geral não fazem distinção na página.
pesquisa com letras maiúsculas e minúsculas e nem pala- • Busca com sinal de menos (-): as palavras que fi-
vras com ou sem acento. cam antes do sinal de menos são excluídas da pesquisa.
• Resultado de um cálculo: pode ser efetuado um
Opções de pesquisa cálculo em um site de pesquisa.

Por exemplo: 3+4


Irá retornar:

Web: pesquisa em todos os sites

Imagens: pesquisa por imagens anexadas nas páginas.


Exemplo do resultado se uma pesquisa.
O resultado da pesquisa
O resultado da pesquisa é visualizado da seguinte for-
ma:

Grupos: pesquisa nos grupos de discussão da Usenet.


Exemplo: INTRANET
A Intranet ou Internet Corporativa é a implantação de
uma Internet restrita apenas a utilização interna de uma
empresa. As intranets ou Webs corporativas, são redes de
comunicação internas baseadas na tecnologia usada na In-
ternet. Como um jornal editado internamente, e que pode
ser acessado apenas pelos funcionários da empresa.
A intranet cumpre o papel de conectar entre si filiais e
departamentos, mesclando (com segurança) as suas infor-
mações particulares dentro da estrutura de comunicações
Diretórios: pesquisa o conteúdo da internet organiza- da empresa.
dos por assunto em categorias. Exemplo: O grande sucesso da Internet, é particularmente da
World Wide Web (WWW) que influenciou muita coisa na
evolução da informática nos últimos anos.
Em primeiro lugar, o uso do hipertexto (documentos
interligados através de vínculos, ou links) e a enorme fa-
cilidade de se criar, interligar e disponibilizar documentos
multimídia (texto, gráficos, animações, etc.), democratiza-
ram o acesso à informação através de redes de computa-
dores. Em segundo lugar, criou-se uma gigantesca base de
usuários, já familiarizados com conhecimentos básicos de
informática e de navegação na Internet. Finalmente, surgi-
ram muitas ferramentas de software de custo zero ou pe-

97
INFORMÁTICA

queno, que permitem a qualquer organização ou empresa, Para acessar as informações disponíveis na Web cor-
sem muito esforço, “entrar na rede” e começar a acessar e porativa, o funcionário praticamente não precisa ser trei-
colocar informação. O resultado inevitável foi a impressio- nado. Afinal, o esforço de operação desses programas se
nante explosão na informação disponível na Internet, que resume quase somente em clicar nos links que remetem
segundo consta, está dobrando de tamanho a cada mês. às novas páginas. No entanto, a simplicidade de uma in-
Assim, não demorou muito a surgir um novo conceito, tranet termina aí. Projetar e implantar uma rede desse tipo
que tem interessado um número cada vez maior de em- é uma tarefa complexa e exige a presença de profissionais
presas, hospitais, faculdades e outras organizações inte- especializados. Essa dificuldade aumenta com o tamanho
ressadas em integrar informações e usuários: a intranet. da intranet, sua diversidade de funções e a quantidade de
Seu advento e disseminação promete operar uma revolu- informações nela armazenadas.
ção tão profunda para a vida organizacional quanto o apa-
recimento das primeiras redes locais de computadores, no A intranet é baseada em quatro conceitos:
final da década de 80. • Conectividade - A base de conexão dos computa-
dores ligados através de uma rede, e que podem transferir
O que é Intranet? qualquer tipo de informação digital entre si;
O termo “intranet” começou a ser usado em meados • Heterogeneidade - Diferentes tipos de computa-
de 1995 por fornecedores de produtos de rede para se re- dores e sistemas operacionais podem ser conectados de
ferirem ao uso dentro das empresas privadas de tecnolo- forma transparente;
gias projetadas para a comunicação por computador entre • Navegação - É possível passar de um documento a
empresas. Em outras palavras, uma intranet consiste em outro através de referências ou vínculos de hipertexto, que
uma rede privativa de computadores que se baseia nos facilitam o acesso não linear aos documentos;
padrões de comunicação de dados da Internet pública, • Execução Distribuída - Determinadas tarefas de
baseadas na tecnologia usada na Internet (páginas HTML, acesso ou manipulação na intranet só podem ocorrer gra-
e-mail, FTP, etc.) que vêm, atualmente fazendo muito su- ças à execução de programas aplicativos, que podem es-
cesso. Entre as razões para este sucesso, estão o custo de tar no servidor, ou nos microcomputadores que acessam a
implantação relativamente baixo e a facilidade de uso pro- rede (também chamados de clientes, daí surgiu à expres-
piciada pelos programas de navegação na Web, os brow- são que caracteriza a arquitetura da intranet: cliente-servi-
sers. dor). A vantagem da intranet é que esses programas são
ativados através da WWW, permitindo grande flexibilidade.
Objetivo de construir uma Intranet Determinadas linguagens, como Java, assumiram grande
Organizações constroem uma intranet porque ela é importância no desenvolvimento de softwares aplicativos
que obedeçam aos três conceitos anteriores.
uma ferramenta ágil e competitiva. Poderosa o suficien-
te para economizar tempo, diminuir as desvantagens da
Como montar uma Intranet
distância e alavancar sobre o seu maior patrimônio de
capital-funcionários com conhecimentos das operações e
Basicamente a montagem de uma intranet consiste em
produtos da empresa.
usar as estruturas de redes locais existentes na maioria das
empresas, e em instalar um servidor Web.
Aplicações da Intranet
Servidor Web - É a máquina que faz o papel de repo-
Já é ponto pacífico que apoiarmos a estrutura de co-
sitório das informações contidas na intranet. É lá que os
municações corporativas em uma intranet dá para simplifi- clientes vão buscar as páginas HTML, mensagens de e-mail
car o trabalho, pois estamos virtualmente todos na mesma ou qualquer outro tipo de arquivo.
sala. De qualquer modo, é cedo para se afirmar onde a
intranet vai ser mais efetiva para unir (no sentido opera- Protocolos - São os diferentes idiomas de comunica-
cional) os diversos profissionais de uma empresa. Mas em ção utilizados. O servidor deve abrigar quatro protocolos.
algumas áreas já se vislumbram benefícios, por exemplo: O primeiro é o HTTP, responsável pela comunicação do
• Marketing e Vendas - Informações sobre produ- browser com o servidor, em seguida vem o SMTP ligado ao
tos, listas de preços, promoções, planejamento de eventos; envio de mensagens pelo e-mail, e o FTP usado na transfe-
• Desenvolvimento de Produtos - OT (Orientação rência de arquivos. Independentemente das aplicações uti-
de Trabalho), planejamentos, listas de responsabilidades lizadas na intranet, todas as máquinas nela ligadas devem
de membros das equipes, situações de projetos; falar um idioma comum: o TCP/IP, protocolo da Internet.
• Apoio ao Funcionário - Perguntas e respostas, sis-
temas de melhoria contínua (Sistema de Sugestões), ma- Identificação do Servidor e das Estações - Depois de
nuais de qualidade; definidos os protocolos, o sistema já sabe onde achar as
• Recursos Humanos - Treinamentos, cursos, apos- informações e como requisitá-las. Falta apenas saber o
tilas, políticas da companhia, organograma, oportunida- nome de quem pede e de quem solicita. Para isso existem
des de trabalho, programas de desenvolvimento pessoal, dois programas: o DNS que identifica o servidor e o DHCP
benefícios. (Dinamic Host Configuration Protocol) que atribui nome às
estações clientes.

98
INFORMÁTICA

Estações da Rede - Nas estações da rede, os funcio- • Redução de redundância na criação e manutenção
nários acessam as informações colocadas à sua disposição de páginas;
no servidor. Para isso usam o Web browser, software que • Redução de custos de arquivamento;
permite folhear os documentos. • Compartilhamento de recursos e habilidade.
Alguns dos empecilhos são:
Comparando Intranet com Internet • Aplicativos de Colaboração - Os aplicativos de cola-
Na verdade as diferenças entre uma intranet e a In- boração, não são tão poderosos quanto os oferecidos pelos
ternet, é uma questão de semântica e de escala. Ambas programas para grupos de trabalho tradicionais. É necessá-
utilizam as mesmas técnicas e ferramentas, os mesmos rio configurar e manter aplicativos separados, como e-mail e
protocolos de rede e os mesmos produtos servidores. O servidores Web, em vez de usar um sistema unificado, como
conteúdo na Internet, por definição, fica disponível em es- faria com um pacote de software para grupo de trabalho;
cala mundial e inclui tudo, desde uma home-page de al- • Número Limitado de Ferramentas - Há um número
guém com seis anos de idade até as previsões do tempo. limitado de ferramentas para conectar um servidor Web a
A maior parte dos dados de uma empresa não se destina bancos de dados ou outros aplicativos back-end. As intranets
ao consumo externo, na verdade, alguns dados, tais como exigem uma rede TCP/IP, ao contrário de outras soluções de
as cifras das vendas, clientes e correspondências legais, de- software para grupo de trabalho que funcionam com os pro-
vem ser protegidos com cuidado. E, do ponto de vista da tocolos de transmissão de redes local existentes;
escala, a Internet é global, uma intranet está contida den- • Ausência de Replicação Embutida – As intranets não
tro de um pequeno grupo, departamento ou organização apresentam nenhuma replicação embutida para usuários re-
corporativa. No extremo, há uma intranet global, mas ela motos. A HMTL não é poderosa o suficiente para desenvolver
ainda conserva a natureza privada de uma Internet menor. aplicativos cliente/servidor.
A Internet e a Web ficaram famosas, com justa razão,
por serem uma mistura caótica de informações úteis e ir- Como a Intranet é ligada à Internet
relevantes, o meteórico aumento da popularidade de sites
da Web dedicados a índices e mecanismos de busca é uma
medida da necessidade de uma abordagem organizada.
Uma intranet aproveita a utilidade da Internet e da Web
num ambiente controlado e seguro.

Vantagens e Desvantagens da Intranet


Alguns dos benefícios são:
• Redução de custos de impressão, papel, distribuição
de software, e-mail e processamento de pedidos;
• Redução de despesas com telefonemas e pessoal no
suporte telefônico;
• Maior facilidade e rapidez no acesso as informações
técnicas e de marketing;
• Maior rapidez e facilidade no acesso a localizações
remotas;
• Incrementando o acesso a informações da concor-
rência;
• Uma base de pesquisa mais compreensiva; Segurança da Intranet
• Facilidade de acesso a consumidores (clientes) e par- Três tecnologias fornecem segurança ao armazenamen-
ceiros (revendas); to e à troca de dados em uma rede: autenticação, controle de
• Aumento da precisão e redução de tempo no acesso acesso e criptografia.
à informação; Autenticação - É o processo que consiste em verificar se
• Uma única interface amigável e consistente para um usuário é realmente quem alega ser. Os documentos e
aprender e usar; dados podem ser protegidos através da solicitação de uma
• Informação e treinamento imediato (Just in Time); combinação de nome do usuário/senha, ou da verificação do
• As informações disponíveis são visualizadas com cla- endereço IP do solicitante, ou de ambas. Os usuários autenti-
reza; cados têm o acesso autorizado ou negado a recursos especí-
• Redução de tempo na pesquisa a informações; ficos de uma intranet, com base em uma ACL (Access Control
• Compartilhamento e reutilização de ferramentas e List) mantida no servidor Web;
informação;
• Redução no tempo de configuração e atualização dos Criptografia - É a conversão dos dados para um formato
sistemas; que pode ser lido por alguém que tenha uma chave secreta
• Simplificação e/ou redução das licenças de software de descriptografia. Um método de criptografia amplamente
e outros; utilizado para a segurança de transações Web é a tecnolo-
• Redução de custos de documentação; gia de chave pública, que constitui a base do HTTPS - um
• Redução de custos de suporte; protocolo Web seguro;

99
INFORMÁTICA

Firewall - Você pode proporcionar uma comunicação Internet Explorer4


segura entre uma intranet e a Internet através de servi- O Internet Explorer facilita o acesso a sites e ajuda a ver
dores proxy, que são programas que residem no firewall com o máximo de qualidade todo o conteúdo incrível que
e permitem (ou não) a transmissão de pacotes com base você pode encontrar. Depois de aprender alguns gestos
no serviço que está sendo solicitado. Um proxy HTTP, por e truques comuns, você poderá usar seu novo navegador
exemplo, pode permitir que navegadores Webs internos com todo o conforto e aproveitar ao máximo seus sites
da empresa acessem servidores Web externos, mas não o favoritos.
contrário.
Noções básicas sobre navegação
Dispositivos para realização de Cópias de Segurança Mãos à obra. Para abrir o Internet Explorer, toque ou
Os dispositivos para a realização de cópias de seguran- clique no bloco Internet Explorer na tela Inicial.
ça do(s) servidor(es) constituem uma das peças de especial Uma barra de endereços, três formas de usar
importância. Por exemplo, unidades de disco amovíveis A barra de endereços é o seu ponto de partida para
com grande capacidade de armazenamento, tapes... navegar pela Internet. Ela combina barra de endereços e
Queremos ainda referir que para o funcionamento de caixa de pesquisa para que você possa navegar, pesquisar
uma rede existem outros conceitos como topologias/confi- ou receber sugestões em um só local. Ela permanece fora
gurações (rede linear, rede em estrela, rede em anel, rede em do caminho quando não está em uso para dar mais espaço
árvore, rede em malha …), métodos de acesso, tipos de cabos, para os sites. Para que a barra de endereços apareça, passe
protocolos de comunicação, velocidade de transmissão … o dedo de baixo para cima na tela ou clique na barra na
parte inferior da tela se estiver usando um mouse. Há três
EXTRANET maneiras de utilizá-la:
A Extranet de uma empresa é a porção de sua rede de Para navegar. Insira uma URL na barra de endereços
computadores que faz uso da Internet para partilhar com para ir diretamente para um site. Ou toque, ou clique, na
segurança parte do seu sistema de informação. barra de endereços para ver os sites que mais visita (os
A Extranet de uma empresa é a porção de sua rede de sites mais frequentes).
computadores que faz uso da Internet para partilhar com Para pesquisar. Insira um termo na barra de endereços
segurança parte do seu sistema de informação. e toque ou clique em Ir para pesquisar a Internet com o
Tomado o termo em seu sentido mais amplo, o concei- mecanismo de pesquisa padrão.
to confunde-se com Intranet. Uma Extranet também pode Para obter sugestões. Não sabe para onde deseja ir?
ser vista como uma parte da empresa que é estendida a Digite uma palavra na barra de endereços para ver suges-
usuários externos (“rede extra-empresa”), tais como repre- tões de sites, aplicativos e pesquisa enquanto digita. Basta
sentantes e clientes. Outro uso comum do termo Extranet tocar ou clicar em uma das sugestões acima da barra de
ocorre na designação da “parte privada” de um site, onde endereços.
somente “usuários registrados” podem navegar, previa-
mente autenticados por sua senha (login).

Empresa estendida
O acesso à intranet de uma empresa através de um
Portal (internet) estabelecido na web de forma que pessoas
e funcionários de uma empresa consigam ter acesso à in-
tranet através de redes externas ao ambiente da empresa.
Uma extranet é uma intranet que pode ser acessada via
Web por clientes ou outros usuários autorizados. Uma in-
tranet é uma rede restrita à empresa que utiliza as mesmas
tecnologias presentes na Internet, como e-mail, webpages,
servidor FTP etc.
A ideia de uma extranet é melhorar a comunicação en-
tre os funcionários e parceiros além de acumular uma base
de conhecimento que possa ajudar os funcionários a criar Multitarefas com guias e janelas
novas soluções. Com as guias, você pode ter muitos sites abertos em
Exemplificando uma rede de conexões privadas, basea- uma só janela do navegador, para que seja mais fácil abrir,
da na Internet, utilizada entre departamentos de uma em- fechar e alternar os sites. A barra de guias mostra todas
presa ou parceiros externos, na cadeia de abastecimento, as guias ou janelas que estão abertas no Internet Explorer.
trocando informações sobre compras, vendas, fabricação, Para ver a barra de guias, passe o dedo de baixo para cima
distribuição, contabilidade entre outros. (ou clique) na tela.

4 Fonte: Ajuda do Internet Explorer

100
INFORMÁTICA

Personalizando sua navegação


Depois de ter aprendido as noções básicas sobre o uso
do navegador, você poderá alterar suas home pages, adi-
cionar sites favoritos e fixar sites à tela Inicial.
Para escolher suas home pages

Abrindo e alternando as guias


Abra uma nova guia tocando ou clicando no botão
Nova guia . Em seguida, insira uma URL ou um termo de
pesquisa ou selecione um de seus sites favoritos ou mais
visitados.
Alterne várias guias abertas tocando ou clicando nelas
na barra de guias. Você pode ter até 100 guias abertas em
uma só janela. Feche as guias tocando ou clicando em Fe-
char no canto de cada guia.

As home pages são os sites que se abrem sempre que


você inicia uma nova sessão de navegação no Internet Ex-
plorer. Você pode escolher vários sites, como seus sites de
notícias ou blogs favoritos, a serem carregados na abertura
do navegador. Dessa maneira, os sites que você visita com
Usando várias janelas de navegação mais frequência estarão prontos e esperando por você.
Também é possível abrir várias janelas no Internet Ex- Passe o dedo da borda direita da tela e toque em Con-
plorer 11 e exibir duas delas lado a lado. Para abrir uma figurações.
nova janela, pressione e segure o bloco Internet Explorer (Se você estiver usando um mouse, aponte para o can-
(ou clique nele com o botão direito do mouse) na tela Ini- to inferior direito da tela, mova o ponteiro do mouse para
cial e, em seguida, toque ou clique em Abrir nova janela. cima e clique em Configurações.)
Duas janelas podem ser exibidas lado a lado na tela. Toque ou clique em Opções e, em Home pages, toque
Abra uma janela e arraste-a de cima para baixo, para o lado ou clique em Gerenciar.
direito ou esquerdo da tela. Em seguida, arraste a outra Insira a URL de um site que gostaria de definir como
janela a partir do lado esquerdo da tela. home page ou toque ou clique em Adicionar site atual se es-
Dica tiver em um site que gostaria de transformar em home page.
Você pode manter a barra de endereços e as guias en-
caixadas na parte inferior da tela para abrir sites e fazer Para salvar seus sites favoritos
pesquisas rapidamente. Abra o botão Configurações, to- Salvar um site como favorito é uma forma simples de
que ou clique em Opções e, em Aparência, altere Sempre memorizar os sites de que você gosta e que deseja visitar
mostrar a barra de endereços e as guias para Ativado. sempre. (Se você tiver feito a atualização para o Windo-
ws 8.1 a partir do Windows 8 e entrado usando sua conta
da Microsoft, todos os favoritos já existentes terão sido im-
portados automaticamente.)

101
INFORMÁTICA

Vá até um site que deseja adicionar. Lendo, salvando e compartilhando conteúdo da In-
Passe o dedo de baixo para cima (ou clique) para exibir ternet
os comandos de aplicativos. Em seguida, toque ou clique Ao examinar seu conteúdo online favorito, procure
no botão Favoritos  para mostrar a barra de favoritos. pelo ícone Modo de exibição de leitura na barra de en-
Toque ou clique em Adicionar a favoritos e, em se- dereços. O Modo de exibição de leitura retira quaisquer
guida, toque ou clique em Adicionar. itens desnecessários, como anúncios, para que as matérias
sejam destacadas. Toque ou clique no ícone para abrir a
página no modo de exibição de leitura. Quando quiser re-
tornar à navegação, basta tocar ou clicar no ícone nova-
mente.

Para fixar um site na tela Inicial


A fixação de um site cria um bloco na tela Inicial, o
que fornece acesso com touch ao site em questão. Alguns
sites fixados mostrarão notificações quando houver novo
conteúdo disponível. Você pode fixar quantos sites quiser e Um artigo da Internet com o modo de exibição de lei-
organizá-los em grupos na tela Inicial. tura desativado

Para exibir os comandos de aplicativos, passe o dedo Um artigo da Internet com o modo de exibição de lei-
de baixo para cima (ou clique). tura ativado
Toque ou clique no botão Favoritos , toque ou clique Para personalizar as configurações do modo de exibi-
no botão Fixar site  e, em seguida, toque ou clique em ção de leitura
Fixar na Tela Inicial. Passe o dedo da borda direita da tela e toque em Con-
figurações.
Dica (Se você estiver usando um mouse, aponte para o can-
Você pode alternar rapidamente os favoritos e as guias to inferior direito da tela, mova o ponteiro do mouse para
tocando ou clicando no botão Favoritos ou no botão cima e clique em Configurações.)
Guias nos comandos de aplicativos. Toque ou clique em Opções e, em Modo de exibição de
leitura, escolha um estilo de fonte e um tamanho de texto.
Estas são algumas opções de estilo que você pode se-
lecionar.

102
INFORMÁTICA

Use a Navegação InPrivate. Os navegadores armaze-


nam informações como o seu histórico de pesquisa para aju-
dar a melhorar sua experiência. Quando você usa uma guia
InPrivate, pode navegar normalmente, mas os dados como
senhas, o histórico de pesquisa e o histórico de páginas da
Internet são excluídos quando o navegador é fechado. Para
abrir uma nova guia InPrivate, passe o dedo de baixo para
cima na tela (ou clique nela) para mostrar os comandos de
aplicativos, ou toque ou clique no botão Ferramentas de guia
e em Nova guia InPrivate.
Use a Proteção contra Rastreamento e o recurso Do Not
Track para ajudar a proteger sua privacidade. O rastreamento
refere-se à maneira como os sites, os provedores de conteú-
do terceiros, os anunciantes, etc. aprendem a forma como
você interage com eles. Isso pode incluir o rastreamento das
páginas que você visita, os links em que você clica e os pro-
dutos que você adquire ou analisa. No Internet Explorer, você
pode usar a Proteção contra Rastreamento e o recurso Do
Not Track para ajudar a limitar as informações que podem
ser coletadas por terceiros sobre a sua navegação e para ex-
pressar suas preferências de privacidade para os sites que
visita.

FIREFOX5
Firefox é um navegador web de código aberto e multi-
plataforma com versões para Windows, OS X (Mac), Linux e
Android, em variantes de 32 e 64 bits, dependendo da pla-
taforma. O Firefox possui suporte para extensões, navegação
por abas, alerta contra sites maliciosos, suporte para sincro-
nização de informações, gerenciador de senhas, bloqueador
de janelas pop-up, pesquisa integrada, corretor ortográfico,
gerenciador de download, leitor de feeds RSS e outros re-
Para salvar páginas na Lista de Leitura
cursos.
Quando você tiver um artigo ou outro conteúdo que
Além de ser multiplataforma, o Firefox também suporta
deseje ler mais tarde, basta compartilhá-lo com sua Lista
diferentes linguagens, incluindo o português do Brasil (Pt Br).
de Leitura em vez de enviá-lo por email para você mesmo Surgido de um projeto criado por Dave Hyatt e Blake
ou de deixar mais guias de navegação abertas. A Lista de Ross em 2002, somente dois anos depois a plataforma de na-
Leitura é a sua biblioteca pessoal de conteúdo. Você pode vegação pela internet se desmembrou de outras ferramentas
adicionar artigos, vídeos ou outros tipos de conteúdo a ela e se tornou um browser independente. No começo, o Firefox
diretamente do Internet Explorer, sem sair da página em se popularizou apenas entre o nicho de adeptos do “softwa-
que você está. re livre”, e mesmo assim já alcançou dezenas de milhões de
Passe o dedo desde a borda direita da tela e toque em downloads.
Compartilhar. Não demorou muito para que o navegador começasse a
(Se usar um mouse, aponte para o canto superior direi- receber melhorias relevantes e o seu potencial fosse observa-
to da tela, mova o ponteiro do mouse para baixo e clique do por outros perfis de internautas. E foi basicamente assim
em Compartilhar.) que o produto da Fundação Mozilla ganhou seu espaço e
Toque ou clique em Lista de Leitura e, em seguida, em quase desbancou a hegemonia do Internet Explorer.
Adicionar. O link para o conteúdo será armazenado na Lista Seu sistema de abas permite que o usuário navegue em
de Leitura. diversos sites sem a necessidade de abrir várias instâncias
do programa. A função de navegação privativa é muito útil,
Ajudando a proteger sua privacidade pois com ela, o Mozilla Firefox não memoriza histórico, da-
Interagir em redes sociais, fazer compras, estudar, dos fornecidos a páginas e ao campo de pesquisa, lista de
compartilhar e trabalhar: você provavelmente faz tudo isso downloads, cookies e arquivos temporários. Serão preserva-
diariamente na Internet, o que pode disponibilizar suas in- dos apenas arquivos salvos por downloads e novos favoritos.
formações pessoais para outras pessoas. O Internet Explo- Além dessas opções, o navegador continua com as funções
rer ajuda você a se proteger melhor com uma segurança básicas de qualquer outro aplicativo semelhante: gerencia-
reforçada e mais controle sobre sua privacidade. Estas são dor de favoritos, suporte a complementos e sincronização
algumas das maneiras pela quais você pode proteger me- de dados na nuvem.
lhor a sua privacidade durante a navegação: 5 Fonte: Ajuda do Firefox

103
INFORMÁTICA

Principais características

· Navegação em abas;
· A mesma janela pode conter diversas páginas. Abrin-
do os links em segundo plano
Eles já estarão carregados quando você for ler;
· Bloqueador de popups:
· O Firefox já vem com um bloqueador embutido de
popups;
· Pesquisa inteligente;

· O campo de pesquisa pelo Google fica na direita na


barra de ferramentas e abre direto a página com os resul-
tados, poupando o tempo de acesso à página de pesquisa
antes de ter que digitar as palavras chaves. O novo localiza-
dor de palavras na página busca pelo texto na medida em Conheça o Firefox Hello
que você as digita, agilizando a busca; - Converse por vídeo com qualquer pessoa, em qual-
· Favoritos RSS; quer lugar
· A integração do RSS nos favoritos permite que você - É grátis! Não é preciso ter conta ou baixar comple-
fique sabendo das atualizações e últimas notícias dos seus mentos.
sites preferidos cadastrados. Essa função é disponibilizada - Escolha como você quer pesquisar
a partir do Firefox 2;
· Downloads sem perturbação;
· Os arquivos recebidos são salvos automaticamente na
área de trabalho, onde são fáceis de achar. Menos interrup-
ções significam downloads mais rápidos. Claro, essa função
pode ser personalizada sem problemas;
· Você decide como deve ser seu navegador;
· O Firefox é o navegador mais personalizável que exis-
te. Coloque novos botões nas barras de ferramentas, ins-
tale extensões que adiciona novas funções, adicione temas
que modificam o visual do Firefox e coloque mais mecanis-
mos nos campos de pesquisa.

O Firefox pode se tornar o navegador mais adequado


para a sua necessidade: Uma nova maneira de pesquisar, ainda mais inteligente
· Fácil utilização; - Sugestões de pesquisa aparecerão conforme você
· Simples e intuitivo, mas repleto de recursos. O Firefox digita
tem todas as funções que você está acostumado - favori- - Escolha o site certo para cada pesquisa
tos, histórico, tela inteira, zoom de texto para tornar as pá- - Use a estrela para adicionar Favoritos
ginas mais fáceis de ler, e diversas outras funcionalidades
intuitivas;
· Compacto;
· A maioria das distribuições está em torno dos 5MB.
Você leva apenas alguns minutos para copiar o Firefox para
o seu computador em uma conexão discada e segunda em
uma conexão banda larga. A configuração é simples e in-
tuitiva. Logo você estará navegando com essa ferramenta.

Principais novidades
Tudo começa pelo novo e intuitivo menu
- As opções que você mais acessa, todas no mesmo
lugar
- Pensado para facilitar o acesso
- Converse por vídeo com qualquer pessoa diretamen-
te do Firefox

104
INFORMÁTICA

Seus sites favoritos estão mais perto do que nunca


- Adicione e visualize seus Favoritos rapidamente
- Salve qualquer site com apenas um clique

Como funcionam as sugestões de sites?


O Firefox exibe links de sites como miniaturas ou lo-
gotipos na página Nova Aba. Quando usar o Firefox pela Desativar os controles da Nova Aba
primeira vez, verá links para sites da Mozilla. Esses sites Para ocultar tudo na sua página Nova Aba, incluindo os
serão eventualmente substituídos por sites visitados com controles da Nova Aba (ou para escolher a página que abre
mais frequência. em uma nova aba) você pode instalar o complemento New
Tab Override (browser.newtab.url replacement).

Personalizar a página Nova aba


O comportamento padrão do Firefox é exibir os sites
em destaque em uma nova aba. Aprenda como personali-
zar, fixar, remover e reorganizar esses sites.

Fixar

Ocultar ou exibir Sugestões na Nova Aba


Clique no ícone no canto superior esquerdo da suges-
Você pode determinar sua página Nova Aba para exibir tão para fixá-la naquela posição na página.
seus sites mais visitados ou até mesmo nada. Para acessar Dica: Configure o Firefox Sync para sincronizar suas Su-
estes controles clique no ícone da engrenagem no canto gestões fixadas entre os seus outros computadores.
superior direito da nova aba.
Remover
Exibir seus sites principais
Clique no ícone de engrenagem na página Nova Aba e
marque Exibir os sites mais visitados.

Mostrar uma Nova Aba em branco


Para remover todos os sites da página Nova Aba, sele-
cione Exibir página em branco.

105
INFORMÁTICA

Clique no “X” no canto superior direito do site para ex- - Crie uma conta do Sync
cluí-lo da página. Clique no botão de menu   e depois em Entrar no
Nota: Se acidentalmente remover um site, pode recu- Sync. A página de acesso será aberta em uma nova aba.
perá-lo clicando em Desfazer no topo da página. Se muitos
sites foram removidos clique em Restaurar tudo.

Reorganizar

Nota: Se não visualizar uma seção do Sync no menu,


você ainda está usando uma versão antiga do Sync. 
Clique no botão Começar.
Preencha o formulário para criar uma conta e clique
em Sign Up. Anote o endereço de e-mail e a senha usada,
você precisará disso mais tarde para entrar.
Verifique nas suas mensagens se recebeu o link de
verificação e clique nele para confirmar seu endereço de
Clique e arraste uma Sugestão para dentro da posição e-mail. Você já está pronto para começar a usar!
que desejar. Ela será “fixada” nesse novo local.
Adicionar um dos seus favoritos Conecte dispositivos adicionais ao Sync
Você também pode abrir a biblioteca de favoritos e ar- Tudo que precisa fazer é entrar e deixar o Sync fazer o
rastá-los para a página Nova Aba.
resto. Para entrar você precisa do endereço de e-mail e a
Antes de iniciar, configure o Firefox para lembrar o his-
senha que usou no começo da configuração do sync.
tórico.
Clique no botão de menu   , e, em seguida, clique
Clique no botão favoritos e depois em Exibir todos
em Entrar no Sync.
os favoritos para abrir a janela da Biblioteca.
Clique no botão Começar para abrir a página Crie uma
Arraste um favorito para dentro da posição que você
conta Firefox.
quiser.
Clique no link Already have an account? Sign in na par-
te inferior da página.

Insira o e-mail e a senha que você usou para criar sua


Como faço para configurar o Sync no meu compu- nova conta do Sync.
tador? Depois que você tiver entrado, o Firefox Sync começará
O Sync permite compartilhar seus dados e preferên- a sincronização de suas informações através dos seus dis-
cias (como favoritos, histórico, senhas, abas abertas, Lista positivos conectados.
de Leitura e complementos instalados) com todos os seus
dispositivos. Aprenda como configurar o Firefox Sync. Remover um dispositivo do Sync
Importante: O Sync requer a versão mais recente do Clique no botão   para expandir o Menu.
Firefox. Certifique-se de que você atualizou o Firefox em Clique no nome da sua conta no Sync (geralmente seu
quaisquer computadores ou dispositivos Android. endereço de e-mail) para abrir as preferências do Sync.
Configurar o Sync requer duas partes: A criação de Clique em Desconectar. Seu dispositivo não será mais
uma conta no seu dispositivo principal e entrar nesta conta sincronizado.
usando outros dispositivos. Aqui estão os passos em de-
talhes: Crie favoritos para salvar suas páginas favoritas

106
INFORMÁTICA

Os favoritos são atalhos para as páginas da web que Onde posso encontrar meus favoritos?
você mais gosta. A forma mais fácil de encontrar um site para o qual
Como eu crio um favorito? você criou um favorito é digitar seu nome na Barra de En-
Fácil — é só clicar na estrela! dereços. Enquanto você digita, uma lista de sites que já
Para criar um favorito, clique no ícone da estrela na você visitou, adicionou aos favoritos ou colocou tags apa-
Barra de ferramentas. A estrela ficará azul e seu favorito recerá. Sites com favoritos terão uma estrela amarela ao
será adicionado na pasta “Não organizados”. Pronto! seu lado. Apenas clique em um deles e você será levado até
lá instantaneamente.

Dica: Quer adicionar todas as abas de uma só vez? Cli-


que com o botão direito do mouse em qualquer aba e
selecione Adicionar todas as abas.... Dê um nome a pasta
e escolha onde quer guardá-la. Clique adicionar favoritos Como eu organizo os meus favoritos?
para finalizar. Na Biblioteca, você pode ver e organizar todos os seus
favoritos.
Como eu mudo o nome ou onde fica guardado um Clique no botão favoritos e depois em Exibir todos
favorito? os favoritos para abrir a janela da Biblioteca.
Para editar os detalhes do seu favorito, clique nova-
mente na estrela e a caixa Propriedades do favorito apa-
recerá.

Por padrão, os favoritos que você cria estarão localiza-


Na janela Propriedades do favorito você pode modifi- dos na pasta “Não organizados”. Selecione-a na barra late-
car qualquer um dos seguintes detalhes: ral da janela “Biblioteca” para exibir os favoritos que você
Nome: O nome que o Firefox exibe para os favoritos adicionou. Dê um clique duplo em um favorito para abri-lo.
em menus. Enquanto a janela da Biblioteca está aberta, você tam-
Pasta: Escolha em que pasta guardar seu favorito sele- bém pode arrastar favoritos para outras pastas como a
cionando uma do menu deslizante (por exemplo, o Menu “Menu Favoritos”, que exibe seus favoritos no menu aberto
Favoritos ou a Barra dos favoritos). Nesse menu, você tam- pelo botão Favoritos. Se você adicionar favoritos à pasta
bém pode clicar em Selecionar... para exibir uma lista de “Barra de favoritos”, eles aparecerão nela (embaixo da Bar-
todas as pastas de favoritos. ra de navegação).
Tags: Você pode usar tags para ajudá-lo a pesquisar e
organizar seus favoritos. Quando você terminar suas mo-
dificações, clique em Concluir para fechar a caixa.

107
INFORMÁTICA

Criando novas pastas

Clique no botão favoritos e depois em Exibir todos


os favoritos para abrir a janela da Biblioteca.
Clique com o botão direito do mouse na pasta que irá
conter a nova pasta, então selecione Nova pasta....

Como eu ativo a Barra de favoritos?


Se você gostaria de usar a Barra de Favoritos, faça o
seginte:
Clique no botão e escolhe Personalizar.
Clique na lista Exibir/ocultar barras e no final selecione
Barra dos favoritos.
Clique no botão verde Sair da personalização.

Removendo apenas uma página dos Favoritos


Acesse a página que deseja remover nos Favoritos.
Clique no ícone da estrela à direita da sua barra de pes-
quisa. Na janela de nova pasta, digite o nome e (opcional-
mente) uma descrição para a pasta que você deseja criar.
Na janela Editar este favorito, clique Remover Favorito.
Adicionando favoritos em pastas
Clique no botão favoritos e depois em Exibir todos
os favoritos para abrir a janela da Biblioteca.
Clique na pasta que contém atualmente o favorito que
você deseja mover.
Arraste o favorito sobre a pasta e solte o botão para
mover o favorito para a pasta.

Ordenando por nome


Clique no botão favoritos e depois em Exibir todos
os favoritos para abrir a janela da Biblioteca.
Clique com o botão direito do mouse na pasta que de-
Removendo mais de uma página ou pasta dos Favoritos seja ordenar e selecione Ordenar pelo nome. Os Favoritos
Clique no botão favoritos e depois em Exibir todos os serão colocados em ordem alfabética.
favoritos para abrir a janela da Biblioteca.
No painel esquerdo da janela do gerenciador, clique na
pasta que deseja visualizar. Seu conteúdo será mostrado no
painel direito.
No painel direito, selecione os itens que deseja remover.
Com os itens a serem removidos selecionado, clique no
botão Organizar e selecione Excluir.

As alterações efetuadas na janela Biblioteca será refle-


tido na barra lateral, no menu e no botão de favoritos.

108
INFORMÁTICA

Reorganizando manualmente
Clique no botão favoritos e depois em Exibir todos
os favoritos para abrir a janela da Biblioteca.
Clique na pasta que contém o favorito que você deseja
mover para expandi-la.
Clique no favorito que você quer mover e arraste-o
para a posição desejada.

Dica: Mais opções de configuração da página inicial es-


tão disponíveis na janela Opções .

Clique no botão menu e depois em Opções, na ja-


nela que foi aberta vá para o painel Geral.
A partir do menu drop-down, selecione Abrir página
em branco na inicialização ou Restaurar janelas e abas an-
teriores.
Clicando em Usar as páginas abertas, as páginas que
estiverem abertas serão configuradas como páginas ini-
ciais, abrindo cada página em uma aba separada.

Para restaurar as configurações da página inicial, siga


os seguintes passos:
Clique no botão , depois em Opções
Selecione o painel Geral.
Para mover um favorito para uma pasta diferente, ar- Clique no botão Restaurar o padrão localizado logo
raste-o para cima da pasta. abaixo do campo Página Inicial.
As alterações efetuadas na janela Biblioteca serão re-
fletidas na barra lateral, no menu e no botão de favoritos.

Ordenar visualizações na janela Biblioteca


Para ver os seus favoritos em várias ordens de classifi-
cação, use a janela Biblioteca:
Clique no botão favoritos e depois em Exibir todos
os favoritos para abrir a janela da Biblioteca.
No painel esquerdo, clique na pasta que deseja visuali-
zar. O conteúdo será exibido no painel da direita.
Clique no botão Exibir, selecione Ordenar e depois
escolha uma ordem de classificação.
A ordem de classificação na janela Biblioteca é apenas
para visualização, e não vai ser refletido na barra lateral, no
menu ou no botão de favoritos. Feche a janela about:preferences. Quaisquer alterações
feitas serão salvas automaticamente.
Definindo a Página Inicial
Veja como abrir automaticamente qualquer página Sugestões de pesquisa no Firefox
web na inicialização do Firefox ou clicando no botão Pági- Muitos mecanismos de pesquisa (incluindo Yahoo,
na inicial . Google, Bing e outros) fornecem sugestões de pesquisa,
Abra a página web que deseja definir como sua página as quais são baseadas em pesquisas populares que outras
inicial. pessoas fazem e que estão relacionadas com uma pala-
Clique e arraste a aba para cima do botão Página Inicial . vra ou palavras que você inserir. Quando as Sugestões de
Pesquisa estão ativadas, o texto que você digita em um
campo de pesquisa é enviado para o mecanismo de busca,
o qual analisa as palavras e exibe uma lista de pesquisas
relacionadas.

Clique em Sim para definir esta página como sua pá-


gina inicial.

109
INFORMÁTICA

Usando a Barra de Pesquisa


Basta digitar na barra de Pesquisa na sua barra de fer-
ramentas ou na página de Nova Aba.

Como as sugestões de pesquisa funcionam


Se você ver uma sugestão de pesquisa que correspon-
de ao que você está procurando, clique nela para ver re- Enquanto você digita na busca da barra de ferramen-
sultados para aquele termo de pesquisa. Isso pode poupar tas, o seu mecanismo de pesquisa padrão mostra suges-
tempo e ajudar você a encontrar o que está procurando tões para ajudá-lo a procurar mais rápido. Essas sugestões
com menos digitação. são baseadas em pesquisas populares ou em suas pesqui-
Ativar as sugestões de pesquisa enviará as palavras- sas anteriores (se estiver ativado).
chave que você digita num campo de busca para o me-
canismo de pesquisa padrão - a menos que pareça que
você está digitando uma URL ou hostname. Os campos de
pesquisa incluem:
- a barra de pesquisa
- páginas iniciais (como mostrado na imagem acima)
- a barra de endereço (onde as Sugestões de Pesquisa
podem ser desativadas separadamente)
O mecanismo de pesquisa padrão pode coletar essas
informações de acordo com os termos da política de pri-
vacidade deles, e os usuários preocupados sobre essas in-
formações sendo coletadas podem desejar não ativar as
sugestões de pesquisa. As sugestões de pesquisa estão de-
sativadas por padrão no modo de Navegação Privada. Você
deve ativá-las explicitamente em uma janela de navegação
privativa para ativá-las nesse modo.

Ativando ou desativando as sugestões de pesquisa


As sugestões de pesquisa podem ser ativadas ou de-
sativadas a qualquer momento marcando ou desmarcando
a caixa Fornecer sugestões de pesquisa na seção Pesquisar
das opções do Firefox:
Pressione Enter para pesquisar usando o seu mecanis-
mo de pesquisa padrão, ou selecione outro mecanismo de
pesquisa clicando no logotipo.

Mecanismos de pesquisa disponíveis


O Firefox vem com os seguintes mecanismos de pes-
quisa por padrão:
- Google para pesquisar na web através do Google
Nota: O padrão de busca do Google é criptografado
para evitar espionagem.
- Yahoo para pesquisar na web através do Yahoo
- Bing para pesquisar na web através do Microsoft Bing
- BuscaPé para procurar comparações de preços, pro-
Para ver sugestões de pesquisa na barra de endereços, dutos e serviços no site BuscaPé.
marque a opção Mostrar sugestões de pesquisa na barra - DuckDuckGo como mecanismo de pesquisa para
de localização. para usuários que não querem ser rastreados.

110
INFORMÁTICA

- Mercado Livre para procurar por itens à venda ou em Para ver todos os seus downloads, acesse a Biblioteca
leilão no Mercado Livre clicando em Exibir todos os downloads na parte inferior do
- Twitter para procurar pessoas no Twitter painel de Downloads.
- Wikipédia (pt) para pesquisar na enciclopédia online
gratuita Wikipédia Portuguesa.

Gerenciador de Downloads
A Biblioteca e o painel de downloads controlam os ar-
quivos baixado pelo Firefox. Aprenda a gerenciar seus ar-
quivos e configurar as definições de download.

Como faço para acessar meus downloads?


Você pode acessar seus downloads facilmente clicando
no icone download (a seta para baixo na barra de ferra-
mentas). A seta vai aparecer azul para que você saiba
que existem arquivos baixados.
Durante um download, o icone de download muda
para um timer que mostra o progresso do seu download.
O timer volta a ser uma seta quando o download for con-
cluído.
A Biblioteca mostra essas informações para todos os
seus arquivos baixados, a menos que você tenha removido
eles do seu histórico.

Clique no icone download para abrir o painel de down-


loads. O painel Downloads exibe os últimos três arquivos
baixados, juntamente com o tempo, tamanho e fonte do
download:

Como posso gerenciar meus arquivos baixados?


No painel Downloads e na sua Biblioteca, existe um
botão icone a direita de cada arquivo que muda de acordo
com o progresso atual do download.

Pausar: Você pode pausar qualquer download em


progresso clicando com o botão direito no arquivo e se-
lecionando Pausar. Isto pode ser útil, por exemplo, se você
precisa abrir um pequeno download que começou depois
de um download grande. A Pausa de downloads lhe dá a
opção de decidir qual dos seus downloads são mais impor-
tantes.Quando você quiser continuar o download desses
arquivos, clique com o botão direito no arquivo e selecione
Continue.

111
INFORMÁTICA

Cancelar : Se depois de iniciar o download você de- Atalhos de teclado


cidir que não precisa mais do arquivo, cancelar o download Para aqueles de boa memória. Use a tela inteira através
é simples: apenas clique no botão X ao lado do arquivo. do teclado.
Este botão se transformará em um símbolo de atualização, Atalho para alternar o modo Tela inteira: Pressione a
clique novamente para reiniciar o download. tecla F11.
Abrir o arquivo: Quando o download acabar, você pode Nota: Em computadores com teclado compacto (como
dar um clique no arquivo para abrir-lo. netbooks e laptops), pode ser necessário usar a combina-
Abrir pasta : Uma vez que o arquivo tenha concluído ção de teclas fn + F11.
o download, o ícone à direita da entrada do arquivo torna-
se uma pasta. Clique no ícone da pasta para abrir a pasta Histórico
que contém esse arquivo. Toda vez que você navega na internet o Firefox guarda
Remover arquivo da lista: Se você não quiser manter o várias informações suas, como por exemplo: sites que você
registro de um determinado download, simplesmente cli- visitou, arquivos que você baixou, logins ativos, dados de
que com o botão direito no arquivo, então selecione Excluir formulários, entre outros. Toda essa informação é chamada
da lista. Isto irá remover a arquivo da lista, mas não vai de histórico. No entanto, se estiver usando um computador
apagar o arquivo em si. público ou compartilha um computador com alguém, você
Repetir um download : Se por qualquer razão um pode não querer que outras pessoas vejam esses dados.
download não completar, clique no botão a direita do ar-
quivo - um simbolo de atualizar - para reiniciar.
Que coisas estão incluídas no meu histórico?
Limpar downloads: Clique no botão Limpar Downloads
no topo da janela da Biblioteca para limpar todo o histórico
de itens baixados.

Como deixar o Firefox em tela inteira


Tela inteira é um recurso do Firefox que permite que
ele ocupe a tela toda, ótimo para aquelas telinhas aperta-
das de netbooks, aproveitando o máximo da sua HDTV ou
só porque quer!
Ative o modo Tela inteira
Maior é melhor! Preencha sua tela com o Firefox.
Clique no botão menu no lado direito da barra de
ferramentas e selecione Tela inteira.

Histórico de navegação e downloads: Histórico de na-


vegação é a lista de sites que você visitou que são exibidos
no menu Histórico, a lista do Histórico na janela Biblioteca
e a lista de endereços da função de completar automati-
camente da Barra de endereços. Histórico dos downloads
é a lista de arquivos que foram baixados por você e são
exibidos na janela Downloads.
Dados memorizados de formulários e Barra de Pesqui-
sa: O histórico de dados memorizados de formulários inclui
os itens que você preencheu em formulários de páginas
web para a funcionalidade de Preenchimento Automático
de Formulários. O histórico da Barra de Pesquisa inclui os
itens que você pesquisou na Barra de Pesquisa do Firefox.
Cookies: Cookies armazenam informações sobre os
Desative o modo Tela inteira websites que você visita, tais como o estado da sua auten-
Traga o meu computador de volta! Encolha o Firefox ticação e preferências do site. Também incluem informa-
para seu tamanho normal. ções e preferências do site armazenadas por plugins como
Mova o mouse para o topo da tela para fazer a barra de o Adobe Flash. Cookies podem também ser usados por
ferramentas reaparecer terceiros para rastreá-lo entre páginas.
Clique no botão menu no lado direito da barra de Nota: Para poder limpar cookies criados pelo Flash
ferramentas e selecione Tela inteira. você precisa estar usando a versão mais recente do plugin.

112
INFORMÁTICA

Cache: O cache armazena arquivos temporariamente, Finalmente, clique no botão Limpar agora. A janela
tais como páginas web e outras mídias online, que o Firefox será fechada e os itens selecionados serão limpos.
baixou da Internet para tornar o carregamento das páginas Como faço para o Firefox limpar meu histórico auto-
e sites que você já visitou mais rápido. maticamente?
Logins ativos: Caso você tenha se logado em um web- Se você precisa limpar seu histórico sempre que usar o
site que usa autenticação HTTP desde a vez mais recente Firefox, você pode configurá-lo para que isso seja feito au-
que você abriu o Firefox, este site é considerado “ativo”. Ao tomaticamente assim que você sair, assim você não esquece.
limpar estes registros você sai destes sites. Clique no botão , depois em Opções
Dados offline de sites: Se você permitir, um website Selecione o painel Privacidade.
pode guardar informações em seu computador para que Defina O Firefox irá: para Usar minhas configurações.
você possa continuar a utilizá-lo mesmo sem estar conec-
tado à Internet.
Preferências de sites: Preferências de sites, incluindo
o nível de zoom salvo para cada página específica, codi-
ficação de caracteres e as permissões de páginas (como
excessões para bloqueadores de anúncios) estão descritas
em janela de Propriedades da Página.

Como limpo meu histórico?


Clique no botão de menu , selecione Histórico e, em
seguida, Limpar dados de navegação….
Selecione o quanto do histórico você deseja limpar:
Clique no menu suspenso ao lado de Intervalo de tem-
po a limpar para escolher quanto de seu histórico o Firefox Marque a opção Limpar histórico quando o Firefox fechar.
limpará.

Para especificar que tipos de histórico devem ser lim-


pos, clique no botão Configurar..., ao lado de Limpar histó-
rico quando o Firefox fechar.
Em seguida, clique na seta ao lado de Detalhes para Na janela Configurações para a limpeza do histórico,
selecionar exatamente quais informações você quer que marque os itens que você quer que sejam limpos automa-
sejam limpas. ticamente sempre que você sair do Firefox.

113
INFORMÁTICA

Após selecionar os itens a serem limpos, clique em OK Imprimindo uma página web
para fechar a janela Configurações para a limpeza do his-
tórico. Clique no menu e depois em Imprimir.
Feche a janela about:preferences. Quaisquer alterações
feitas serão salvas automaticamente.
Como faço para remover um único site do meu histórico?
Clique no botão , depois em Histórico, em seguida,
clique no link no final da lista Exibir todo o histórico, para
abrir a janela da Biblioteca.
Use o campo Localizar no histórico no canto superior
direito e pressione a tecla Enter para procurar pelo site que
você deseja remover do histórico.
Nos resultados da busca, clique com o botão direito no
site que você deseja remover, e selecione Limpar tudo so-
bre este site. Ou simplesmente selecione o site que deseja
excluir e pressione a tecla ‘Delete’.
Todos os dados de histórico (histórico de navegação e
downloads, cookies, cache, logins ativos, senhas, dados de
formulários, exceções para cookies, imagens, pop-ups) do
site serão removidos.

Na janela de impressão que foi aberta, ajuste as con-


figurações do que você está prestes a imprimir, se for ne-
cessário.
Clique em OK para iniciar a impressão.
Janela de configurações de impressão

Finalmente, feche a janela Biblioteca.

O leitor de PDF
O visualizador de PDF integrado de maneira nativa ao
navegador. Isto significa que agora não é mais necessário
ter que instalar um plugin externo no Mozilla Firefox para
fazê-lo visualizar um documento neste formato. Este visua-
lizador, inclusive, funciona da mesma forma como ocorre
no Google Chrome, que também suporta a visualização de
arquivos PDF nativamente.
Agora, sempre que você clicar em um documento PDF
no navegador, ele será aberto diretamente na tela. Os con- Seção Impressoras:
troles são exibidos na parte superior, com os quais você
pode salvar ou imprimir o documento, bem como usar re- Clique no menu drop-down ao lado de Name para mu-
cursos como zoom, ou ir diretamente para uma página es- dar qual a impressora imprimirá a página que você está
pecífica. Também é possível alternar para o modo de apre- vendo.
sentação e exibir o PDF em tela cheia. Nota: A impressora padrão é a do Windows. Quando
Durante os testes realizados, conseguimos abrir vários uma página da web é impressa com a impressora selecio-
PDFs em diversas abas sem nenhum problema, já que não nada, ela se torna a impressora padrão do Firefox.
houve travamentos. O segredo por trás do leitor é que ele Cique em Propiedades... para mudar o tamanho do pa-
converte os PDFs para o HTML 5. pel, qualidade de impressão e outras configurações espe-
cíficas da impressora.

114
INFORMÁTICA

Seção Intervalo de impressão - Especifique quais pági- Formato e Opções


nas da página web atual será impressa:
Selecione Tudo para imprimir tudo.
Selecione Páginas e coloque o intervalo de páginas
que você quer imprimir. Por exemplo, selecionando “de 1 a
1” imprimirá somente a primeira página.
Selecione Seleção para imprimir somente a parte da
página que você selecionou.
Seção Cópias - Especifique quantas cópias você quer
imprimir.
Se colocar mais do que 1 no campo Número de cópias,
você também pode escolher se quer agrupá-las. Por exem-
plo, se você escolheu fazer 2 cópias e selecionou Juntar,
elas serão impressas na ordem 1, 2, 3, 1, 2, 3. Caso contrá-
rio, elas serão impressas na ordem 1, 1, 2, 2, 3, 3.
Nota: As seguintes configurações são salvas como pre-
ferências do Firefox em uma base por impressora.
SeçãoImprimir bordas - Se você está vendo uma pá-
gina web com bordas, poderá selecionar como as bordas
serão impressas:

Na aba Formato e Opções você pode alterar:


Formato:
Selecione Retrato para a maioria dos documentos e
páginas web.
Selecione Paisagem para páginas e imagens largas.
Escala: Para tentar uma página web em menos folhas
impressas, você pode ajustar a escala. Reduzir para caber
ajusta automaticamente a escala.
Opções: Selecione Imprimir cores e imagens de fundo
para que o Firefox imprima as páginas com cor e imagens
de fundo como elas são mostradas na tela, caso contrário,
Firefox imprimirá com o fundo branco.
Margens e Cabeçalho/ Rodapé

Como apresentado na tela irá imprimir da mesma for-


ma que você vê a página web no Firefox.
O campo selecionado irá imprimir somente o conteúdo
dentro da última borda que você clicou.
Cada campo separadamente irá imprimir o conteúdo
de todas as bordas, mas em páginas separadas.
Mudando a configuração da página
Para alterar a orientação da página, alterar se as cores
e imagens de fundo são impressas, as margens da página,
o que incluir no cabeçalho e rodapé das páginas impressas,
na parte superior da janela do Firefox, clique no botão Fi-
refox, veja mais em Imprimir... (menu Arquivo no Windows
XP) e selecione Configurar página.... A janela de configura-
ção de página irá aparecer.
Nota: As seguintes configurações são salvas como pre-
ferências do Firefox em uma base por impressora.

115
INFORMÁTICA

Na aba Margens e Cabeçalhos/Rodapé você pode al-


terar:
Margens: Você pode colocar a largura da margem se-
paradamente para cima, baixo, esquerda e direita.
Cabeçalho e Rodapé: Use os menus dropdown para
selecionar o que irá aparecer na página impressa. O valor
do dropdown superior esquerdo aparece no canto superior
esquerdo da página; o valor do dropdown superior central
aparece na parte superior central da página, e assim por
diante. Você pode escolher entre:
--em branco--: Nada será impresso.
Título: Imprime o título das páginas web.
Endereço: Imprime o endereço das páginas web.
Data/Hora: Imprime a data e hora em que a página foi
impressa.
Página #: Imprime o número da página. Use as seguintes opções para convidar seus amigos:
Página # de #: Imprime o número da página e o total
de páginas.
Personalizar...: Coloque seu próprio texto de cabeçalho
ou rodapé. Isso pode ser usado pra mostrar o nome da
empresa ou organização no alto ou na parte de baixo de
toda página impressa.
Clique em OK para concluir as alterações e fechar a
janela de configuração de páginas.

Visualizar impressão
Para ver como a página web que você quer imprimir
ficará quando impressa, na parte superior da janela do Fire-
fox, clique no botão Firefox, veja mais em Imprimir...(menu
Arquivo no Windows XP), e selecione Visualizar impressão.
A janela de pré-visualização permite mudar algumas
das opções descritas acima. Acesse a janela de impressão
clicando em Imprimir..., ou a janela de configuração de pá- Copie e cole o link para a sua ferramenta de mensa-
gina clicando em Configurar página.... Clique nas setas ao gens preferida clicando em Copiar Link.
lado do campo Página: para trocar as páginas do docu- Envie o link por e-mail para o seu amigo clicando no
mento. As setas duplas mudam para a primeira ou última botão Enviar link por E-mail. Isso abrirá sua aplicação de
página, as setas únicas vão para a próxima página ou a e-mail padrão.
anterior. Você também pode ajustar a escala e o formato
(veja acima). Compartilhar no Facebook.
Quando seu amigo se juntar à conversa, você verá um
alerta.
Para encerrar a chamada, clique em .
Se juntar a uma conversa
Clique em Fechar para sair da visualização da impres-
são. Recebeu um convite? Se juntar a uma conversa é fácil!
Firefox Hello - conversas por vídeo e voz online Apenas clique no link do seu convite e clique no botão na
O Firefox Hello lhe deixa navergar e discutir páginas página para entrar na conversa.
web com seus amigos diretamente no navegador. Tudo
que você precisa é uma webcam (opcional), um microfone, Controlar suas notificações
e a versão mais recente do Firefox para ligar para os ami- Você pode desligar as notificações no Firefox se você
gos que estão em navegadores suportados pelo WebRTC preferir não ser notificado quando um amigo se juntar:
como Firefox, Chrome, ou Opera. Clique no botão do Hello .
Nota: O Firefox Hello não está disponível na Navega- Clique na engrenagem na parte de baixo do painel e
ção Privada. escolha Desligar notificações.
Iniciar uma conversa
Clique no botão Hello .
Clique em Navegar nessa página com um amigo.

116
INFORMÁTICA

Navegação Privativa Dica: Janelas de navegação privativas tem uma másca-


Quando navega na web, o Firefox lembra de varias in- ra roxa no topo.
formação para você - como os sites visitados. No entan-
to, pode haver momentos em que não deseja que outros
usuários tenham acesso a tais informações, como quando
estiver comprando um presente de aniversário. A navega-
ção privativa permite que navegue na internet sem salvar
informações sobre os sites e páginas visitadas.
A navegação privativa também inclui Proteção contra
rastreamento na navegação privada, a qual impede que
seja rastreado enquanto navega.
Mostraremos a você como funciona. O que a navegação privativa não salva?
Importante: A navegação privativa não o torna anôni- Páginas visitadas: Nenhuma página será adicionada à
mo na Internet. Seu provedor de acesso a internet ou os lista de sites no histórico, lista de história da janela da Bi-
próprios sites ainda podem rastrear as páginas visitadas. blioteca, ou na lista de endereços da Awesome Bar.
Além disso, a navegação privativa não o protege de key- Entradas em formulário e na barra de pesquisa: Nada
loggers ou spywares que podem estar alojados em seu digitado em caixas de texto em páginas web ou na barra de
computador busca será salvo para o autocomplete.
Senhas: Nenhuma senha será salva.
Como abrir uma nova janela privativa? Lista de arquivos baixados: os arquivos que você baixar
Existem duas maneiras de se abrir uma nova Janela Pri- não serão listados na Janela de Downloads depois de de-
vativa. sativar a Navegação Privativa.
Abrir uma nova Janela Privativa vazia Cookies: Cookies armazenam informações sobre os si-
Clique no botão de menu e depois em Nova janela tes que você visita como preferências, status de login, e os
privativa. dados utilizados por plugins, como o Adobe Flash. Cookies
também podem ser utilizados por terceiros para rastreá-lo
através dos sites.
Conteúdo web em Cache e Conteúdo Web off-line e
de dados do usuário ‘: Nenhum arquivo temporário da In-
ternet (cache) ou arquivos armazenados para o uso off-line
serão salvo.
Nota:
Favoritos criados ao usar a Navegação Privativa serão
salvos.
Todos os arquivos que você baixar para o seu compu-
tador durante o uso de navegação privada serão salvos.
O Firefox Hello não está disponível na navegação pri-
vativa.
Posso definir o Firefox para sempre usar a navegação
privativa?
O Firefox está definido para lembrar o histórico por
padrão, mas você pode alterar essa configuração de pri-
vacidade no Firefox Opções (clique no menu Firefox ,
escolha Opções e selecione o painel Privacidade). Quando
Abrir um link em nova janela privativa alterar a configuração do histórico para nunca lembrar o
Clique com o botão direito do mouse e escolha Abrir histórico, isto equivale a estar sempre no modo de nave-
link em uma nova janela privativa no menu contextual. gação privativa.
Importante: Quando o firefox está definido para nunca
lembrar o histórico você não verá uma máscara roxa na
parte superior de cada janela, mesmo que esteja efetiva-
mente no modo de navegação privativa. Para restaurar a
navegação normal, vá para o painel privacidade Opções e
defina o Firefox para lembrar o histórico.
Outras formas de controlar as informações que o
Firefox salva
Você sempre pode remover a navegação recente,
as pesquisas e histórico de download depois de visitar
um site.

117
INFORMÁTICA

Como saber se a minha conexão com um site é segura?


O botão de Identidade do Site (um cadeado) aparece
na sua barra de endereço quando você visita um site segu-
ro. Você pode descobrir rapidamente se a conexão para o
site que estar visualizando é criptografado. Isso deve lhe Para sites usando certificados VE, o botão de identida-
ajudar a evitar sites maliciosos que estão tentando obter de do site exibe tanto um cadeado verde e o nome legal
sua informação pessoal. da companhia ou organização do website, então você sabe
quem está operando ele. Por exemplo, isto mostra que o
mozilla.org é de propriedade da Fundação Mozilla.
Cadeado verde com um triângulo cinza de alerta
Um cadeado verde com um triângulo cinza de alerta
indica que o site é seguro; no entanto, o firefox bloqueou
o conteúdo inseguro e, assim, o site pode não necessaria-
O botão de Identidade do Site estar na barra de ende- mente exibir ou funcionar inteiramente correto.
reço à esquerda do endereço web. Mais comumente, quan-
do visualizando um site seguro, o botão de Identidade do Cadeado cinza com um triângulo amarelo de alerta
Site será um cadeado verde. Um cadeado cinza com um triângulo amarelo de alerta
indica que a conexão entre o Firefox e o website é apenas
parcialmente criptografada e não impede espionagem.

No entanto, em algumas circunstâncias raras, ele tam-


bém pode ser um cadeado verde com um triângulo de
alerta cinza, um cadeado cinza com um triângulo de alerta Nota: Não envie qualquer tipo de informação sensível
amarelo, ou um cadeado cinza com uma linha vermelha. (informação bancária, dados de cartão de crédito, Números
de Seguridade Social, etc.) para sites onde o botão de iden-
tidade do site tem o ícone de triângulo de alerta amarelo.
Cadeado cinza com um traço vermelho
Nota: Clicando no botão à esquerda da barra de en- Um cadeado cinza com um traço vermelho indica que
dereço nos traz o Centro de Controle, o qual lhe permite a conexão entre o Firefox e o website é apenas parcial-
visualizar mais informações detalhadas sobre o estado de mente criptografada e não previne contra espionagem ou
segurança da conexão e alterar algumas configurações de ataque man-in-the-middle.
segurança e privacidade. Aviso: Você nunca deve enviar
qualquer tipo de informação sensível (informação bancá-
ria, dados de cartão de crédito, Números de Seguridade
Social, etc.) para um site sem o ícone de cadeado na barra
de endereço - neste caso não é verificado que você está se Esse ícone não aparecerá a menos que você manual-
comunicando com o site pretendido nem que seus dados mente desativou o bloqueio de conteúdo misto.
estão seguros contra espionagem! Nota: Não envie qualquer tipo de informação sensível
(informação bancária, dados de cartão de crédito, números
Cadeado verde de seguridade social, etc.) para sites onde o botão de iden-
Um cadeado verde (com ou sem um nome de organi- tidade do site tem o ícone de um cadeado cinza com uma
zação) indica que: listra vermelha.
Você está realmente conectado ao website cujjo en-
dereço é exibido na barra de endereço; a conexão não foi Configurações de segurança e senhas
interceptada. Este artigo explica as configurações disponíveis no pai-
A conexão entre o Firefox e o website é criptografada nel Segurança da janela Opções do Firefox.
para evitar espionagem. O painel Segurança contém Opções relacionadas à sua
segurança ao navegar na internet.

Um cadeado verde mais o nome da empresa ou or-


ganização, também em verde, significa que o website está
usando um Certificado de Validação Avançada. Um certi-
ficado de Validação Avançada é um tipo especial de cer-
tificado do site que requer um processo de verificação de
identidade significativamente mais rigoroso do que outros
tipos de certificados.

118
INFORMÁTICA

Configurações de Segurança Existem três tipos de complementos:


Alertar se sites tentarem instalar extensões ou temas - Extensões
O Firefox sempre pedirá a sua confirmação para a ins- Extensões adicionam novas funcionalidades ao Firefox
talação de complementos. Para evitar que tentativas de ou modificam as já existentes. Existem extensões que per-
instalação não requisitadas resultem em instalações aci- mitem bloquear anúncios, baixar vídeos de sites, integrar
dentais, o Firefox exibe um aviso quando um site tentar ins- o Firefox com sites, como o Facebook ou o Twitter, e até
talar um complemento e bloqueia a tentativa de instalação. mesmo adicionar recursos de outros navegadores.
Para permitir que sites específicos instalem complementos, - Aparência
você deve clicar em Exceções…, digitar o endereço do site Existem dois tipos de complementos de aparência:
e clicar em Permitir. Desmarque essa opção para desativar temas completos, que mudam a aparência de botões e
esse aviso para todos os sites. menus, e temas de fundo, que decoram a barra de menu e
Bloquear sites avaliados como focos de ataques: Mar- faixa de abas com uma imagem de fundo
que isso se você quer que o Firefox verifique se o site que - Plugins
você está visitando pode ser uma tentativa de interfirir nas Plugins permitem adicionar suporte para todos os tipos
funções normais do computador ou mandar dados pes- de conteúdo da Internet. Estes geralmente incluem forma-
soais sobre você sem autorização através da Internet. tos patenteados como o Flash, QuickTime e Silverlight que
A ausência deste aviso não garante que o site seja con- são usados para vídeo, áudio, jogos on-line, apresentações
fiável. e muito mais. Plugins são criados e distribuídos por outras
Bloquear sites avaliados como falsos: Marque isso se empresas.
você quer que o Firefox verifique ativamente se o site que Para visualizar quais complementos estão instalados:
você está visitando pode ser uma tentativa de enganar Clique no botão escolha complementos. A aba com-
você fazendo com que passe suas informações pessoais plementos irá abrir.
(isto é frequentemente chamado de “phishing”). Selecione o painel Extensões, Aparência ou Plugins.
Como faço para encontrar e instalar complementos?
Logins Aqui está um resumo para você começar:
Memorizar logins de sites: I Firefox pode salvar com Clique no botão de menu e selecione Complemen-
segurança senhas que você digita em formulários web para tos para abrir a aba do gerenciador de complementos.
facilitar seu acesso aos websites. Desmarque essa opção No gerenciador de complementos, selecione o painel
para impedir o Firefox de memorizar suas senhas. No en- Get Add-ons.
tanto, mesmo com isso marcado, você ainda será questio- Para ver mais informações sobre um complemento ou
nado se deseja salvar ou não as senhas para um site quan- tema, clique nele. Você pode em seguida clicar no botão
do você visitá-lo pela primeira vez. Se você selecionar Nun- verde Adicionar ao Firefox para instalá-lo.
ca para este site, aquele site será adicionado à uma lista de Você também pode pesquisar por complementos es-
exceções. Para acessar essa lista ou para remover sites dela, pecíficos usando a caixa de busca na parte superior. Po-
clique no botão Exceções…. dendo então instalar qualquer complemento que encon-
Usar uma senha mestra: O Firefox pode proteger infor- trar, com o botão Instalar.
mações sensíveis, como senhas salvas e certificados, crip-
tografando eles usando uma senha mestra. Se você criar
uma senha mestra, cada vez que você iniciar o Firefox, será
solicitado que você digite a senha na primeira vez que for
necessário acessar um certificado ou uma senha salva. Você
pode definir, alterar, ou remover a senha mestra marcando
ou desmarcando essa opção ou clicando no botão Modifi-
car senha mestra…. Se uma senha mestra já estiver defini-
da, você precisará digitá-la para alterar ou remover a senha
mestra.
Você pode gerenciar senhas salvas e excluir senhas in-
dividuais clicando no botão Logins salvos….

Encontrar e instalar complementos para adicionar


funcionalidades ao Firefox
Complementos são como os aplicativos que você ins-
tala para adicionar sinos e assobios para o Firefox. Você
pode obter complementos para comparar preços, verificar
o tempo, mudar o visual do Firefox, ouvir música, ou mes-
mo atualizar o seu perfil no Facebook. Este artigo aborda
os diferentes tipos de complementos disponíveis e como
encontrar e instalá-los.

119
INFORMÁTICA

Como desinstalar plugins


Geralmente os plugins vem com seus próprios desinsta-
ladores. Se precisar de ajuda para desinstalar alguns dos plu-
gins mais populares, vá para lista de artigos de plugins e sele-
cione o artigo do respectivo plugin que você quer desinstalar.

Configurações de Conteúdo

O Firefox irá fazer o download do complemento e pode


pedir que você confirme a sua instalação.
Clique em Reiniciar agora se ele aparecer. Seus abas
serão salvas e restauradas após a reinicialização.
Algumas extensões colocam um botão na barra de fer-
ramentas após a instalação..

Como desativar extensões e temas


Ao desativar um complemento ele deixará de funcio-
nar sem ser removido:
Clique no botão de menu   e selecione Complemen-
tos para abrir a aba do gerenciador de complementos.
No gerenciador de complementos, selecione o pai-
nel Extensões ou Aparência.
Selecione o complemento que deseja desativar. DRM Content
Clique no botão Desativar. Reproduzir conteúdo DRM: Por padrão, o Firefox per-
Se surgir uma mensagem em pop-up, clique em Rei- mite a reprodução de conteúdo de áudio e vídeo protegido
niciar agora. As suas abas serão salvas e restauradas ao por Gerencimento de Direitos Digitais (DRM). Ao desmarcar
reiniciar. esta opção essa funcionalidade será desligada.
Para reativar um complemento, encontre-o na lista de
complementos e clique em Ativar, será solicitado reiniciar Notificações
o Firefox. O Firefox lhe permite escolher quais websites tem per-
missão para lhe enviar notificações. Clique em Escolher para
Como desativar plugins fazer alterações na lista de sites permitidos.
Ao desativar um plugin ele irá deixar de funcionar sem Não me perturbe: Selecione esta opção para suspender
ser removido: temporariamente todas as notificações até você fechar e rei-
Clique no botão de menu   e selecione Complemen- niciar o Firefox.
tos para abrir a aba do gerenciador de complementos.
No gerenciador de complementos, selecione o pai- Pop-ups
nel Plugins. Bloquear janelas popup: Por padrão, o Firefox bloqueia
janelas popup inconvenientes em sites da web. Desmarque
Selecione o plugin que deseja desativar.
essa opção para desativar o Bloqueador de Popups. Alguns
Selecione Nunca Ativar no menu de seleção.
sites utilizam popups com funções importantes. Para permitir
Para reativar um plugin, encontre-o na sua lista de plu-
que sites específicos utilizem popups, clique em Exceções…,
gins e clique em Sempre ativo no menu de seleção. digite o domínio do site e clique em Permitir. Para excluir um
site da lista de sites permitidos, selecione-o e clique em Ex-
Como remover extensões e temas cluir o site. Para limpar a lista completamente, clique em Ex-
Clique no botão de menu   e selecione Complemen- cluir tudo.
tos para abrir a aba do gerenciador de complementos.
No gerenciador de complementos, selecione o pai- Fontes e cores
nel Extensões ou Aparência. Fonte padrão e Tamanho: Normalmente as páginas da
Selecione o complemento que você deseja remover. web são exibidas na fonte e tamanho especificados aqui.
Clique no botão Excluir. Entretanto, páginas da web podem definir fontes diferentes,
Se surgir uma mensagem em pop-up, clique em Rei- que serão exibidos a não ser que você especifique o contrá-
niciar agora. As suas abas serão salvas e restauradas ao rio na janela Fontes. Clique no botãoAvançado… para acessar
reiniciar. mais opções de fontes.

120
INFORMÁTICA

Diálogo de fontes Use atalhos do mouse para executar tarefas comuns


Na lista Fontes padrão para, escolha um grupo de no Firefox
caracteres/idioma. Por exemplo, para configurar o grupo Esta é uma lista dos atalhos do mouse mais comuns no
de fontes padrão dos idiomas ocidentais (latinos), clique Mozilla Firefox.
em Latin.. Para um grupo de caracteres/idioma que não es-
teja na lista, clique em Outros Sistemas de Escrita. Comando Atalho
Escolha se a fonte proporcional deverá ser com serifa
(como “Times New Roman”) ou sem serifa (como “Arial”), Voltar Shift + Rolar para baixo
e então especifique o tamanho padrão da fonte propor- Avançar Shift + Rolar para cima
cional. Aumentar Zoom Ctrl + Rolar para cima
Especifique as fontes utilizadas para fontes com seri-
Diminuir Zoom Ctrl + Rolar para baixo
fa, sem serifa e monoespaçada (largura fixa). Você também
pode especificar o tamanho para as fontes monoespaça- Clicar com botão do
Fechar Aba
das. meio na Aba
Você também pode especificar o tamanho mínimo de Clicar com botão do
Abrir link em uma nova Aba
fonte que pode ser exibido na tela. Isso pode ser útil em meio no link
sites que utilizam tamanhos de fonte muito pequenos e clicar com o botão do
pouco legíveis. Nova aba
meio na barra de abas
Páginas podem usar outras fontes: Por padrão, o Fi-
refox exibe as fontes especificadas pelo autor da página. Ctrl + Clicar com botão
Desative essa opção para forçar todos os sites a usar as Abrir em nova Aba em segun- esquerdo no link
fontes padrão. do plano* Clicar com botão do
Codificação de texto para conteudo legado: A codifica- meio no link
ção de caracteres selecionada nessa caixa será a codifica- Ctrl + Shift + Botão es-
Abrir em nova Aba em pri-
ção padrão utilizada para exibir páginas que não especifi- querdo
meiro plano*
quem uma codificação. Shift + Botão do meio
Diálogo de cores Shift  + Clicar com bo-
Cores padrão: Aqui você pode modificar as cores pa- Abrir em uma Nova Janela
tão esquerdo no link
drão de texto e fundo que serão utilizadas nas páginas em
Duplicar Aba ou Favoritos Ctrl + Arrastar Aba
que essas cores não foram especificadas por seu autor. Cli-
que nas amostras de cores para modificá-las. Recarregar (ignorar cache) Shift + Botão recarregar
Usar cores do sistema: Marque essa opção para usar as Salvar como... Alt + Botão esquerdo
cores de fonte e fundo definidas pelo seu Sistema Opera-
cional em vez das cores definidas acima. * Os atalhos para abrir Abas em primeiro e segun-
Aparência padrão dos links: Aqui você pode modificar do plano serão trocadas se a opção Ao abrir um link em
as cores padrão dos links das páginas. Clique nas amostras uma nova Aba, carregá-la em primeiro plano estiver ativa
de cores para modificá-las. no Painel de configurações geral..
Sublinhar: Por padrão, o Firefox sublinha os links das
páginas. Desmarque essa opção para modificar esse com- Atalhos de teclado
portamento. Note que vários sites especificam seus pró- Navegação
prios estilos de links e nesses sites essa opção não tem
efeito.
Páginas podem usar outras cores: Por padrão, o Firefox Comando Atalho
exibe as cores especificadas pelo autor da página. Desa- Alt + ←
tive essa opção para forçar todos os sites a usar as cores Voltar
Backspace
padrão.
Idiomas Alt + →
Algumas páginas oferecem mais de um idioma para Avançar Shift + Backs-
exibição. Clique no botão  Selecionar…para especificar o pace
idioma ou idiomas de sua preferência. Página inicial Alt + Home
Idiomas: Para adicionar um idioma à lista de idiomas
clique emSelecione um idioma para adicionar…, clique Abrir arquivo Ctrl + O
sobre o idioma escolhido e clique no botãoAdicionar.
Exclua um idioma da lista selecionando-o e clicando no F5
Atualizar a página
botão Excluir. Você também pode reordenar os idiomas Ctrl + R
usando os botões Para cima e Para baixo para determi- Ctrl + F5
Atualizar a página (ignorar o cache)
nar a ordem de preferência no caso de haver mais de um Ctrl + Shift + R
idioma disponível.
Parar o carregamento Esc

121
INFORMÁTICA

Página atual 1.    As setas são ferramentas bem conhecidas por to-
dos que já utilizaram um navegador. Elas permitem avançar
Comando Atalho ou voltar nas páginas em exibição, sem maiores detalhes. Ao
manter o botão pressionado sobre elas, você fará com que o
Ir uma tela para baixo Page Down histórico inteiro apareça na janela.
Ir uma tela para cima Page Up 2.   Reenviar dados, atualizar ou recarregar a página.
Ir para o final da página End Todos são sinônimos desta função, ideal para conferir no-
vamente o link em que você se encontra, o que serve para
Ir para o início da página Home situações bem específicas – links de download perdidos,
Ir para o próximo frame F6 imagens que não abriram, erros na diagramação da página.
Ir para o frame anterior Shift + F6 3.   O ícone remete à palavra home (casa) e leva o nave-
gador à página inicial do programa. Mais tarde ensinaremos
Imprimir Ctrl + P você a modificar esta página para qualquer endereço de sua
Salvar página como Ctrl + S preferência.
Mais zoom Ctrl + + 4.   A estrela adiciona a página em exibição aos favoritos,
que nada mais são do que sites que você quer ter a disposi-
Menos zoom Ctrl + - ção de um modo mais rápido e fácil de encontrar.
tamanho normal Ctrl + 0 5.   Abre uma nova aba de navegação, o que permite
visitar outros sites sem precisar de duas janelas diferentes.
Editando 6.   A barra de endereços é o local em que se encontra
o link da página visitada. A função adicional dessa parte no
Comando Atalho Chrome é que ao digitar palavras-chave na lacuna, o me-
canismo de busca do Google é automaticamente ativado e
Copiar Ctrl + C exibe os resultados em questão de poucos segundos.
Recortar Ctrl + X 7. Simplesmente ativa o link que você digitar na lacuna
Apagar Del à esquerda.
8. Abre as opções especiais para a página aberta no na-
Colar Ctrl + V vegador. Falaremos um pouco mais sobre elas em seguida.
Colar (como texto simples) Ctrl + Shift + V 9. Abre as funções gerais do navegador, que serão me-
Refazer Ctrl + Y lhor detalhadas nos próximos parágrafos.
Selecionar tudo Ctrl + A Para Iniciantes
Desfazer Ctrl + Z Se você nunca utilizou um navegador ou ainda tem dú-
vidas básicas sobre essa categoria de programas, continue
GOOGLE CHROME lendo este parágrafo. Do contrário, pule para o próximo e
O Chrome é o mais novo dos grandes navegadores e poupe seu tempo. Aqui falaremos um pouco mais sobre os
já conquistou legiões de adeptos no mundo todo. O pro- conceitos e ações mais básicas do programa.
grama apresenta excelente qualidade em seu desenvolvi- Com o Google Chrome, você acessa os sites da mesma
mento, como quase tudo o que leva a marca Google. O forma que seus semelhantes – IE, Firefox, Opera. Ao executar o
browser não deve nada para os gigantes Firefox e Internet programa, tudo o que você precisa fazer é digitar o endereço do
Explorer e mostra que não está de brincadeira no mundo local que quer visitar. Para acessar o portal Baixaki, por exemplo,
dos softwares. basta escrever baixaki.com.br (hoje é possível dispensar o famo-
Confira nas linhas abaixo um pouco mais sobre o ótimo so “www”, inserido automaticamente pelo programa.)
Google Chrome. No entanto nem sempre sabemos exatamente o link
que queremos acessar. Para isso, digite o nome ou as pa-
Funções visíveis lavras-chave do que você procura na mesma lacuna. Desta
Antes de detalhar melhor os aspectos mais complicados forma o Chrome acessa o site de buscas do Google e exibe
do navegador, vamos conferir todas as funções disponíveis os resultados rapidamente. No exemplo utilizamos apenas a
logo em sua janela inicial. Observe a numeração na imagem palavra “Baixaki”.
abaixo e acompanhe sua explicação logo em seguida:

122
INFORMÁTICA

Abas Configuração

A segunda tarefa importante para quem quer usar o Antes de continuar com as outras funções do Google
Chrome é lidar com suas abas. Elas são ferramentas muito Chrome é legal deixar o programa com a sua cara. Para
úteis e facilitam a navegação. Como citado anteriormente, isso, vamos às configurações. Vá até o canto direito da tela
basta clicar no botão com um “+” para abrir uma nova guia. e procure o ícone com uma chave de boca. Clique nele e
Outra forma de abri-las é clicar em qualquer link ao selecione “Opções”.
pressionar a rodinha do mouse, o que torna tudo ainda
mais rápido. Também é possível utilizar o botão direito so-
bre o novo endereço e escolher a opção “Abrir link em uma
nova guia”.

Liberdade

É muito fácil manipular as abas no Google Chrome. É


possível arrastá-las e mudar sua ordem, além de arrancar
a aba da janela e desta forma abrir outra independente.
Basta segurar a aba com o botão esquerdo do mouse para
testar suas funções. Clicar nelas com a rodinha do mouse
faz com que fechem automaticamente.

Básicas
O botão direito abre o menu de contexto da aba, em Inicialização: aqui é possível definir a página inicial do
que é possível abrir uma nova, recarregar a atual, fechar navegador. Basta selecionar a melhor opção para você e
a guia ou cancelar todas as outras. No teclado você pode configurar as páginas que deseja abrir.
abrir uma nova aba com o comando Ctrl + T ou simples- Página inicial: caso esta tenha sido a sua escolha na aba
mente apertando o F1. anterior, defina qual será a página inicial do Chrome. Tam-
bém é possível escolher se o atalho para a home (aquele
Fechei sem querer! em formato de casinha) aparecerá na janela do navegador.
Pesquisa padrão: como o próprio nome já deixa claro,
Quem nunca fechou uma aba importante acidental- aqui você escolhe o site de pesquisas utilizado ao digitar
mente em um momento de distração? Pensando nisso, na lacuna do programa. O botão “Gerenciar” mostra a lista
o Chrome conta com a função “Reabrir guia fechada” no de mecanismos.
menu de contexto (botão direito do mouse). Basta selecio- Navegador padrão: aqui você pode definir o aplicativo
ná-la para que a última página retorne ao navegador. como seu navegador padrão. Se você optar por isso, sem-
pre que algum software ou link for executado, o Chrome
será automaticamente utilizado pelo sistema.

Coisas pessoais

Senhas: define basicamente se o programa salvará ou


não as senhas que você digitar durante a navegação. A op-
ção “Mostrar senhas salvas” exibe uma tabela com tudo o
que já foi inserido por você.
Preenchimento automático de formulário: define se os
formulários da internet (cadastros e aberturas de contas)
serão sugeridos automaticamente após a primeira digita-
ção.

123
INFORMÁTICA

Dados de navegação: durante o uso do computador, o


Chrome salva os dados da sua navegação para encontrar sites,
links e conteúdos com mais facilidade. O botão “Limpar dados
de navegação” apaga esse conteúdo, enquanto a função “Im-
portar dados” coleta informações de outros navegadores.
Temas: é possível modificar as cores e todo o visual do
navegador. Para isso, clique em “Obter temas” e aplique
um de sua preferência. Para retornar ao normal, selecione
“Redefinir para o tema padrão”.

Pesquise dentro dos sites

Outra ferramenta muito prática do navegador é a pos-


sibilidade de realizar pesquisas diretamente dentro de al-
guns sites, como o próprio portal Baixaki. Depois de usar a
busca normalmente no nosso site pela primeira vez, tudo o
que você precisa fazer é digitar baixaki e teclar o TAB para
que a busca desejada seja feita diretamente na lacuna do
Configurações avançadas Chrome.
Rede: configura um Proxy para a sua rede. (Indicado
para usuários avançados)
Privacidade: aqui há diversas funções de privacidade,
que podem ser marcadas ou desmarcadas de acordo com
suas preferências.
Downloads: esta é a opção mais importante da aba. Em Navegação anônima
“Local de download” é possível escolher a pasta em que os
arquivos baixados serão salvos. Você também pode definir Se você quer entrar em alguns sites sem deixar rastros ou
que o navegador pergunte o local para cada novo down- históricos de navegação no computador, utilize a navegação
load. anônima. Basta clicar no menu com o desenho da chave de
boca e escolher a função “Nova janela anônima”, que também
Downloads pode ser aberta com o comando Ctrl + Shift + N.

Todos os navegadores mais famosos da atualidade


contam com pequenos gerenciadores de download, o que
facilita a vida de quem baixa várias coisas ao mesmo tem-
po. Com o Google Chrome não é diferente. Ao clicar em
um link de download, muitas vezes o programa pergunta-
rá se você deseja mesmo baixar o arquivo, como ilustrado
abaixo:

Gerenciador de tarefas

Uma das funções mais úteis do Chrome é o pequeno


gerenciador de tarefas incluso no programa. Clique com o
botão direito no topo da página (como indicado na figura)
e selecione a função “Gerenciador de tarefas”.

Logo em seguida uma pequena aba aparecerá embai-


xo da janela, mostrando o progresso do download. Você
pode clicar no canto dela e conferir algumas funções es-
peciais para a situação. Além disso, ao selecionar a função
“Mostrar todos os downloads” (Ctrl + J), uma nova aba é
exibida com ainda mais detalhes sobre os arquivos que
você está baixando

124
INFORMÁTICA

Desta forma, uma nova janela aparecerá em sua tela. 3- Um item selecionado do Windows pode ser excluído
Ela controla todas as abas e funções executadas pelo nave- permanentemente, sem colocá-Lo na Lixeira, pressionando-
gador. Caso uma das guias apresente problemas você pode se simultaneamente as teclas
fechá-la individualmente, sem comprometer todo o progra- (A) Ctrl + Delete.
ma. A função é muito útil e evita diversas dores de cabeça. (B) Shift + End.
(C) Shift + Delete.
(D) Ctrl + End.
(E) Ctrl + X.

Comentário: Quando desejamos excluir permanente-


mente um arquivo ou pasta no Windows sem enviar antes
para a lixeira, basta pressionarmos a tecla Shift em conjunto
com a tecla Delete. O Windows exibirá uma mensagem do
tipo “Você tem certeza que deseja excluir permanentemen-
te este arquivo?” ao invés de “Você tem certeza que deseja
enviar este arquivo para a lixeira?”.
QUESTÕES COMENTADAS Resposta: C
4- Qual a técnica que permite reduzir o tamanho de
1- Com relação ao sistema operacional Windows, assi- arquivos, sem que haja perda de informação?
nale a opção correta. (A) Compactação
(A) A desinstalação de um aplicativo no Windows deve (B) Deleção
ser feita a partir de opção equivalente do Painel de Con- (C) Criptografia
trole, de modo a garantir a correta remoção dos arquivos (D) Minimização
relacionados ao aplicativo, sem prejuízo ao sistema opera- (E) Encolhimento adaptativo
cional.
(B) O acionamento simultâneo das teclas CTRL, ALT e Comentários: A compactação de arquivos é uma téc-
DELETE constitui ferramenta poderosa de acesso direto aos nica amplamente utilizada. Alguns arquivos compactados
diretórios de programas instalados na máquina em uso. podem conter extensões ZIP, TAR, GZ, RAR e alguns exem-
(C) O Windows oferece acesso facilitado a usuários de plos de programas compactadores são o WinZip, WinRar,
um computador, pois bastam o nome do usuário e a senha SolusZip, etc.
da máquina para se ter acesso às contas dos demais usuá- Resposta: A
rios possivelmente cadastrados nessa máquina.
(D) O Windows oferece um conjunto de acessórios 05. (TJ/BA – Técnico Judiciário - Tecnologia da Infor-
disponíveis por meio da instalação do pacote Office, entre mação – FGV/2015) Na suite LibreOffice, o componente
eles, calculadora, bloco de notas, WordPad e Paint. Impress destina-se:
(E) O comando Fazer Logoff, disponível a partir do bo- a) à edição de fórmulas matemáticas para documen-
tão Iniciar do Windows, oferece a opção de se encerrar o tos;
Windows, dar saída no usuário correntemente em uso na b) ao gerenciamento de uma ou mais impressoras;
máquina e, em seguida, desligar o computador. c) à edição de imagens e arquivos congêneres;
Comentários: Para desinstalar um programa de forma d) à utilização de algoritmos de programação linear
segura deve-se acessar Painel de Controle / Adicionar ou em planilhas;
remover programas e) à edição de apresentações de slides.
Resposta – Letra A
05. Resposta: E
2- Nos sistemas operacionais como o Windows, as in- O LibreOffice Impress é um programa de apresentação
formações estão contidas em arquivos de vários formatos, de slides similar ao Keynote, presente no iWork, e ao Power-
que são armazenados no disco fixo ou em outros tipos de Point, encontrado na suíte da Microsoft, e destina-se a criar
mídias removíveis do computador, organizados em: e a apresentar slides, sendo possível inserir plano de fundo,
(A) telas. títulos, marcadores, imagens, vídeos, efeitos de transição de
(B) pastas. slides, dentre outras opções.
(C) janelas.
(D) imagens. 6- “O correio eletrônico é um método que permite
(E) programas. compor, enviar e receber mensagens através de sistemas
eletrônicos de comunicação”. São softwares gerenciadores
Comentários: O Windows Explorer, mostra de forma de email, EXCETO:
bem clara a organização por meio de PASTAS, que nada A) Mozilla Thunderbird.
mais são do que compartimentos que ajudam a organizar B) Yahoo Messenger.
os arquivos em endereços específicos, como se fosse um C) Outlook Express.
sistema de armário e gavetas. D) IncrediMail.
Resposta: Letra B E) Microsoft Office Outlook 2003.

125
INFORMÁTICA

Comentários: Podemos citar vários gerenciadores de Comentários: O URL é o endereço (único) de um recur-
e-mail (eletronic mail ou correio eletrônico), mas devemos so na Internet. A questão parece diferenciar um recurso de
memorizar que os sistemas que trabalham o correio eletrô- página, mas na verdade uma página é um recurso (o mais
nico podem funcionar por meio de um software instalado conhecido, creio) da Web. Item verdadeiro.
em nosso computador local ou por meio de um progra- É comum confundir os itens II e III, por isso memorize:
ma que funciona dentro de um navegador, via acesso por down = baixo = baixar para sua máquina, descarregar. II e
Internet. Este programa da Internet, que não precisa ser III são verdadeiros.
instalado, e é chamado de WEBMAIL, enquanto o software
local é o gerenciador de e-mail citado pela questão.
Principais Vantagens do Gerenciador de e-mail:
• Pode ler e escrever mensagens mesmo quando
está desconectado da Internet;
• Permite armazenar as mensagens localmente (no
computador local);
• Permite utilizar várias caixas de e-mail ao mesmo
tempo;
Maiores Desvantagens:
• Ocupam espaço em disco;
• Compatibilidade com os servidores de e-mail
(nem sempre são compatíveis).
A seguir, uma lista de gerenciadores de e-mail (em ne- No item IV encontramos o item falso da questão, o que
grito os mais conhecidos e utilizados atualmente): nos leva ao gabarito – letra C. Anexar um arquivo em men-
Microsoft Office Outlook sagem de e-mail significa copiar e não mover!
Microsoft Outlook Express; Resposta: C.
Mozilla Thunderbird;
08. (EMBASA – Assistente de Saneamento - Técnico
IcrediMail
em Segurança do Trabalho – IBF/2015) Ao receber, por
Eudora
e-mail, um arquivo com o nome “resumo.xlsx” pode-se abrir
Pegasus Mail
esse arquivo, por padrão, com os aplicativos:
Apple Mail (Apple)
a) Microsoft Office Excel e também com o LibreOffice Calc.
Kmail (Linux)
b) Microsoft Office Access e também com o LibreOffice
Windows Mail
Calc.
A questão cita o Yahoo Mail, mas este é um WEBMAIL, c) Microsoft Office Excel e também com o LibreOffice
ou seja, não é instalado no computador local. Logo, é o Math.
gabarito da questão. d) Microsoft Office PowerPoint e também com o Li-
Resposta: B. breOffice Math.
7- Sobre os conceitos de utilização da Internet e cor- 08. Resposta: A
reio eletrônico, analise: No Libreoffice Calc:
I. A URL digitada na barra de Endereço é usada pelos Abrir um arquivo do Microsoft Office
navegadores da Web (Internet Explorer, Mozilla e Goo- Escolha Arquivo - Abrir. Selecione um arquivo do Mi-
gle Chrome) para localizar recursos e páginas da Internet crosoft Office na caixa de diálogo do LibreOffice.
(Exemplo: http://www.google.com.br). O arquivo do MS Office... ...será aberto no módulo
II. Download significa descarregar ou baixar; é a trans- do LibreOffice
ferência de dados de um servidor ou computador remoto MS Word, *.doc, *.docx = LibreOffice Writer
para um computador local. MS Excel, *.xls, *.xlsx = LibreOffice Calc
III. Upload é a transferência de dados de um computa- MS PowerPoint, *.ppt, *.pps, *.pptx = LibreOffice Im-
dor local para um servidor ou computador remoto. press
IV. Anexar um arquivo em mensagem de e-mail signifi-
ca movê-lo definitivamente da máquina local, para envio a 9- Com relação a conceitos de Internet e intranet, assi-
um destinatário, com endereço eletrônico. nale a opção correta.
Estão corretas apenas as afirmativas: (A) Domínio é o nome dado a um servidor que controla
A) I, II, III, IV a entrada e a saída de conteúdo em uma rede, como ocorre
B) I, II na Internet.
C) I, II, III (B) A intranet só pode ser acessada por usuários da
D) I, II, IV Internet que possuam uma conexão http, ao digitarem na
E) I, III, IV barra de endereços do navegador: http://intranet.com.
(C) Um modem ADSL não pode ser utilizado em uma
rede local, pois sua função é conectar um computador à
rede de telefonia fixa.

126
INFORMÁTICA

(D) O modelo cliente/servidor, em que uma máquina - IMAP(Internet Message Access Protocol) – Semelhan-
denominada cliente requisita serviços a outra, denominada te ao POP, no entanto, possui mais recursos e dá ao usuário
servidor, ainda é o atual paradigma de acesso à Internet. a possibilidade de armazenamento e acesso a suas mensa-
(E) Um servidor de páginas web é a máquina que ar- gens de email direto no servidor.
mazena os nomes dos usuários que possuem permissão de - FTP(File Transfer Protocol) – Protocolo para transfe-
acesso a uma quantidade restrita de páginas da Internet. rência de arquivos
Comentários: O modelo cliente/servidor é questionado Resposta: D
em termos de internet pois não é tão robusto quanto re-
des P2P pois, enquanto no primeiro modelo uma queda do 11- Quanto ao Windows Explorer, assinale a opção cor-
servidor central impede o acesso aos usuários clientes, no reta.
segundo mesmo que um servidor “caia” outros servidores (A) O Windows Explorer é utilizado para gerenciar pas-
ainda darão acesso ao mesmo conteúdo permitindo que tas e arquivos e por seu intermédio não é possível acessar o
o download continue. Ex: programas torrent, Emule, Lime- Painel de Controle, o qual só pode ser acessado pelo botão
ware, etc. Iniciar do Windows.
Em relação às outras letras: (B) Para se obter a listagem completa dos arquivos sal-
letra A – Incorreto – Domínio é um nome que serve vos em um diretório, exibindo-se tamanho, tipo e data de
para localizar e identificar conjuntos de computadores na modificação, deve-se selecionar Detalhes nas opções de
Internet e corresponde ao endereço que digitamos no na- Modos de Exibição.
vegador. (C) No Windows Explorer, o item Meus Locais de Rede
letra B – Incorreto – A intranet é acessada da mesma oferece um histórico de páginas visitadas na Internet para
forma que a internet, contudo, o ambiente de acesso a rede acesso direto a elas.
é restrito a uma rede local e não a internet como um todo. (D) Quando um arquivo estiver aberto no Windows e
letra C – Incorreto – O modem ADSL conecta o compu- a opção Renomear for acionada no Windows Explorer com
tador a internet, como o acesso a intranet se faz da mesma o botão direito do mouse,será salva uma nova versão do
forma só que de maneira local, o acesso via ADSL pode sim arquivo e a anterior continuará aberta com o nome antigo.
acessar redes locais. (E) Para se encontrar arquivos armazenados na estrutu-
letra E – Incorreto – Um servidor é um sistema de com- ra de diretórios do Windows, deve-se utilizar o sítio de bus-
putação que fornece serviços a uma rede de computado- ca Google, pois é ele que dá acesso a todos os diretórios de
res. E não necessariamente armazena nomes de usuários e/ máquinas ligadas à Internet.
ou restringe acessos.
Resposta: D Comentários: Na opção Modos de Exibição, os arqui-
vos são mostrados de várias formas como Listas, Miniatu-
10- Com relação à Internet, assinale a opção correta. ras e Detalhes.
(A) A URL é o endereço físico de uma máquina na Inter- Resposta: B
net, pois, por esse endereço, determina-se a cidade onde
está localizada tal máquina. Atenção: Para responder às questões de números
(B) O SMTP é um serviço que permite a vários usuários 12 e 13, considere integralmente o texto abaixo:
se conectarem a uma mesma máquina simultaneamente, Todos os textos produzidos no editor de textos padrão
como no caso de salas de bate-papo. deverão ser publicados em rede interna de uso exclusivo do
(C) O servidor Pop é o responsável pelo envio e recebi- órgão, com tecnologia semelhante à usada na rede mundial
mento de arquivos na Internet. de computadores.
(D) Quando se digita o endereço de uma página web, Antes da impressão e/ou da publicação os textos deve-
o termo http significa o protocolo de acesso a páginas em rão ser verificados para que não contenham erros. Alguns
formato HTML, por exemplo. artigos digitados deverão conter a imagem dos resultados
(E) O protocolo FTP é utilizado quando um usuário de obtidos em planilhas eletrônicas, ou seja, linhas, colunas, va-
correio eletrônico envia uma mensagem com anexo para lores e totais.
outro destinatário de correio eletrônico. Todo trabalho produzido deverá ser salvo e cuidados de-
Comentários: Os itens apresentados nessa questão es- vem ser tomados para a recuperação em caso de perda e
tão relacionados a protocolos de acesso. Segue abaixo os também para evitar o acesso por pessoas não autorizadas às
protocolos mais comuns: informações guardadas.
- HTTP(Hypertext Transfer Protocol) – Protocole de car- Os funcionários serão estimulados a realizar pesquisas
regamento de páginas de Hipertexto –  HTML na internet visando o atendimento do nível de qualidade da
- IP (Internet Protocol) – Identificação lógica de uma informação prestada à sociedade, pelo órgão.
máquina na rede O ambiente operacional de computação disponível para
- POP (Post Office Protocol) – Protocolo de recebimen- realizar estas operações envolve o uso do MS-Windows, do
to de emails direto no PC via gerenciador de emails MS-Office, das ferramentas Internet Explorer e de correio
- SMTP (Simple Mail Transfer Protocol) – Protocolo pa- eletrônico, em português e em suas versões padrões mais
drão de envio de emails utilizadas atualmente.

127
INFORMÁTICA

Observação: Entenda-se por mídia removível disquetes, Resposta: “A”


CD’s e DVD’s graváveis, Pen Drives (mídia removível acopla- Comentários
da em portas do tipo USB) e outras funcionalmente seme- Prompt de Comando é um recurso do Windows que ofe-
lhantes. rece um ponto de entrada para a digitação de comandos do
MSDOS (Microsoft Disk Operating System) e outros coman-
12- As células que contêm cálculos feitos na planilha dos do computador. O mais importante é o fato de que, ao
eletrônica, digitar comandos, você pode executar tarefas no computa-
(A) quando “coladas” no editor de textos, apresentarão dor sem usar a interface gráfica do Windows. O Prompt de
resultados diferentes do original. Comando é normalmente usado apenas por usuários avan-
(B) não podem ser “coladas” no editor de textos. çados.
(C) somente podem ser copiadas para o editor de tex-
tos dentro de um limite máximo de dez linhas e cinco co- 15. (MF – Todos os Cargos – ESAF/2013) As suítes de
lunas. escritório oferecem funções de editoração de textos, pla-
(D) só podem ser copiadas para o editor de texto uma nilha eletrônica, apresentação, editoração de desenhos e
a uma. banco de dados. Um exemplo de suíte de escritório basea-
(E) quando integralmente selecionadas, copiadas e da em software livre é o:
“coladas” no editor de textos, serão exibidas na forma de a) LibreOffice
tabela. b) Microsoft Office
  c) LinuxOffice
Comentários: Sempre que se copia células de uma pla- d) UBUNTU Office
nilha eletrônica e cola-se no Word, estas se apresentam e) BROfficex
como uma tabela simples, onde as fórmulas são esqueci-
das e só os números são colados. 15. Resposta: A
Resposta: E LibreOffice: é uma suíte de aplicativos livre multiplata-
forma para escritório disponível para Windows, Unix, Sola-
13- O envio do arquivo que contém o texto, por meio ris, Linux e Mac OS X. A suíte utiliza o formato OpenDocu-
do correio eletrônico, deve considerar as operações de ment (ODF) — formato homologado como ISO/IEC 26300
(A) anexação de arquivos e de inserção dos endereços e NBR ISO/IEC 26300 — e é também compatível com os
eletrônicos dos destinatários no campo “Cco”. formatos do Microsoft Office, além de outros formatos le-
(B) de desanexação de arquivos e de inserção dos en- gados.
dereços eletrônicos dos destinatários no campo “Para”.
(C) de anexação de arquivos e de inserção dos endere- 16. (Caixa Econômica Federal - Técnico Bancário
ços eletrônicos dos destinatários no campo “Cc”. Novo - CESGRANRIO/2012) O envio e o recebimento
(D) de desanexação de arquivos e de inserção dos en- de um arquivo de textos ou de imagens na internet, en-
dereços eletrônicos dos destinatários no campo “Cco”. tre um servidor e um cliente, constituem, em relação ao
(E) de anexação de arquivos e de inserção dos endere- cliente, respectivamente, um
ços eletrônicos dos destinatários no campo “Para”. (A) download e um upload
  (B) downgrade e um upgrade
Comentários: Claro que, para se enviar arquivos pelo (C) downfile e um upfile
correio eletrônico deve-se recorrer ao uso de anexação, ou (D) upgrade e um downgrade
seja, anexar o arquivo à mensagem. Quando colocamos (E) upload e um download
os endereços dos destinatários no campo Cco, ou seja, no Resposta: “E”.
campo “com cópia oculta”, um destinatário não ficará sa- Comentários:
bendo quem mais recebeu aquela mensagem, o que aten-  Up – Cima / Down – baixo  / Load – Carregar;
de a segurança solicitada no enunciado. Upload – Carregar para cima (enviar).
Resposta: A Download – Carregar para baixo (receber ou “baixar”)
14. (Caixa Econômica Federal - Técnico Bancário
Novo - CESGRANRIO/2012) Usado para o manuseio 17. (CLIN – Auxiliar de Enfermagem do Trabalho –
de arquivos em lotes, também denominados scripts, o COSEAC/2015) São componentes do pacote LibreOffice:
shell de comando é um programa que fornece comuni- a) Base, Calc, Draw, Impress, Math e Writer.
cação entre o usuário e o sistema operacional de forma b) Draw, Math, Writer, Eudora, Impress e Calc.
direta e independente. Nos sistemas operacionais Win- c) Publisher, Base, Impress, Writer, Calc e Draw.
dows XP, esse programa pode ser acessado por meio de d) Paint, Writer, Impress, Math, Draw e Calc.
um comando da pasta Acessórios denominado
(A) Prompt de Comando 17. Resposta: A
(B) Comandos de Sistema O LibreOffice é uma potente suite office; sua interface
(C) Agendador de Tarefas limpa e suas poderosas ferramentas libertam sua criativida-
(D) Acesso Independente de e melhoram sua produtividade.O LibreOffice incorpora
(E) Acesso Direto várias aplicações que a tornam a mais avançada suite office

128
INFORMÁTICA

livre e de código aberto do mercado. O processador de 20. (TRE-RO – Técnico Judiciário - Área Administra-
textos Writer, a planilha Calc, o editor de apresentações Im- tiva – FCC/2013)
press, a aplicação de desenho e fluxogramas Draw, o banco
de dados Base e o editor de equações Math são os compo-
nentes do LibreOffice.

18. (UFMT – Auxiliar em Administração –


UFMT/2014) No LibreOffice Writer, ao clicar-se no botão ¶
, quando se está editando um texto,
a) exporta-se diretamente o arquivo editado para o
formato pdf.
b) apresentam-se na tela os caracteres não imprimí-
veis do texto que está sendo editado.
c) alinha-se à direita o texto todo ou o trecho sele-
cionado.
d) apresenta-se na tela o texto no formato em que
será impresso.

18. Resposta: B
São os caracteres não imprimíveis e marcas de forma-
tação, como espaço em branco, mudança de linha manual,
quebras de seção, entre muitos.

19. (EMBASA – Assistente de Saneamento - Técnico


em Segurança do Trabalho – IBFC/2015)
Ao receber, por e-mail, um arquivo com o nome “re-
sumo.xlsx” pode-se abrir esse arquivo, por padrão, com os
aplicativos: Para realizar a tarefa 3, Paulo utilizou, respectivamente,
a) Microsoft Office Excel e também com o LibreOffice os softwares
Calc. a) Impress e Calc.
b) Microsoft Office Access e também com o LibreOffice b) Writer e Math.
Calc. c) Impress e Lotus.
c) Microsoft Office Excel e também com o LibreOffice d) Writer e Calc.
Math. e) Libre Word e Libre Excel.
d) Microsoft Office PowerPoint e também com o Li-
breOffice Math. Resposta: D
writer e calc , são correspondentes ao word e Excel do
Resposta: A Windows.
A compatibilidade é total, ou seja, um documento cria-
do no Excel pode ser aberto no Calc e vice-versa. 21- (SPPREV – Técnico – Vunesp/2011 - II) No âmbi-
to das URLs, considere o exemplo: protocolo://xxx.yyy.
zzz.br. O domínio de topo (ou TLD, conforme sigla em
inglês) utilizado para classificar o tipo de instituição, no
exemplo dado acima, é o
a) protocolo.
b) xxx.
c) zzz.
d) yyy.
e) br.

Resposta: “C”
Comentários:
a) protocolo. protocolo HTTP
b) xxx. o nome do domínio
c) zzz. o tipo de domínio
d) yyy. subdomínios
e) br. indicação do país ao qual pertence o domínio

129
INFORMÁTICA

22. (TCE-SP – Agente da Fiscalização Financeira - Considere a figura que mostra o Windows Explorer
Sistemas, Gestão de Projetos e Governança de TI – Vu- do Microsoft Windows, em sua configuração original, e
nesp/2015) responda às questões de números 24 e 25.
O LibreOffice possui alguns aplicativos que apresen-
tam funcionalidades semelhantes às apresentadas pelos
aplicativos do MS-Office. O Writer do LibreOffice gera do-
cumentos com a extensão
a) .odb
b) .odp
c) .odt
d) .ots
e) .ppt

Resposta: C
Open Document Text (.odt)

23. (DATAPREV – Médico do Trabalho – Qua-


drix/2012)
Nas questões que avaliam os conhecimentos de no-
ções de informática, a menos que seja explicitamente in-
formado o contrário, considere que todos os programas 24. (TJ/SP – Escrevente Téc. Jud. – Vunesp/2012) O
mencionados estão em configuração padrão, em portu- arquivo zaSetup_en se encontra
guês, que o mouse está configurado para pessoas destras, (A) no disquete.
que expressões como clicar, clique simples e clique duplo (B) no DVD.
referem-se a cliques com o botão esquerdo do mouse e (C) em Meus documentos.
que teclar corresponde à operação de pressionar uma te- (D) no Desktop.
cla e, rapidamente, liberá-la, acionando-a apenas uma vez. (E) na raiz do disco rígido.
Considere também que não há restrições de proteção, de
funcionamento e de uso em relação aos programas, arqui- Resposta: E
vos, diretórios, recursos e equipamentos mencionados. Comentário:
Considere os pacotes de escritório LibreOffice e o MS No Windows Explorer, você pode ver a hierarquia das
Office, ambos em português do Brasil e em suas instala- pastas em seu computador e todos os arquivos e pastas loca-
ções padrões. Assinale a alternativa correta. lizados em cada pasta selecionada. Ele é especialmente útil
a) As apresentações criadas originalmente no MS para copiar e mover arquivos.
PowerPoint precisam ser convertidas para o formato ODF Ele é composto de uma janela dividida em dois painéis:
para poderem ser editadas no LibreOffice Impress. O painel da esquerda é uma árvore de pastas hierarquiza-
b) Documentos criados no LibreOffice Writer para Li- da que mostra todas as unidades de disco, a Lixeira, a área
nux só podem ser salvos no formato ODF e, por isso, não de trabalho ou Desktop (também tratada como uma pasta);
podem ser abertos do ambiente Windows. O painel da direita exibe o conteúdo do item selecionado à
c) No MS Word para Linux, o atalho de teclado para esquerda e funciona de maneira idêntica às janelas do Meu
se executar a verificação ortográfica e gramatical do texto Computador (no Meu Computador, como padrão ele traz a
é F7. janela sem divisão, as é possível dividi-la também clicando
d) O recurso de Tabela Dinâmica só existe no MS Ex- no ícone Pastas na Barra de Ferramentas)
cel, não tem similar no LibreOffice Calc.
e) Planilhas criadas tanto no LibreOffice Calc quanto
no MS Excel podem ser gravadas/exportadas como arqui-
vos PDF.

Resposta: E
Tanto o Calc quanto o excel, em suas versões mais re-
centes possuem o recurso de conversão para PDF

130
INFORMÁTICA

25. (TJ/SP – Escrevente Téc. Jud. – Vunesp/2012) Ao se EXERCÍCIOS COMPLEMENTARES


clicar em , localizado abaixo do menu Favoritos, será
01. (POLÍCIA FEDERAL - PAPILOSCOPIS-
fechado
TA DA POLÍCIA FEDERAL – CESPE/2012) - Acer-
(A) o Meu computador. ca de conceitos de hardware, julgue o item seguinte.
(B) o Disco Local (C:). Diferentemente dos computadores pessoais ou PCs
(C) o painel Pastas. tradicionais, que são operados por meio de teclado e
mouse, os tablets, computadores pessoais portáteis,
(D) Meus documentos. dispõem de recurso touch-screen. Outra diferença entre
(E) o painel de arquivos. esses dois tipos de computadores diz respeito ao fato
de o tablet possuir firmwares, em vez de processadores,
como o PC.
Resposta: C
( ) Certo
( ) Errado
Comentário:
Firmwares não são hardwares, e sim códigos de pro-
gramação existentes no próprio hardware, inclusos em
chips de memória (ROM, PROM, EPROM, EEPROM, flash)
durante sua fabricação. Sua natureza, na maioria das vezes,
é não volátil, ou seja, não perde seus dados durante a au-
sência de energia elétrica, mas quando presentes em tipos
de memória como PROM ou EPROM, podem ser atualiza-
dos.
Por esse motivo, os firmwares não substituem proces-
sadores inteiros.
A seguir, veja alguns modelos de tablets e observe a
presença do processador em sua configuração:

Este botão, contido na barra de ferramentas, exibe/


oculta o painel PASTAS.
Tablet Softronic PHASER KINNO 4GB Android 2.3.4
Tela 7 Polegadas

Características do Produto
Tablet 4GB - Softronic
APRESENTAÇÃO DO PRODUTO: Com o novo Phaserkin-
no Plus, você possui muito mais interatividade e rapidez na
palma de suas mãos, graças ao seu poderoso processador
A10 de 1.2 Ghz, ele consegue ser totalmente multi-tarefas
para você que se desdobra em dez durante o seu dia a
dia, podendo ler um livro, escutar suas músicas e continuar
acompanhando sua vida em redes sociais e sincronizando
e-mails. Tudo isso sem se preocupar com a lentidão do sis-
tema. Para você que precisa estar conectado a todo o mo-
mento, o PhaserKinno Plus ainda oferece suporte a modem
externo. Ele conta com uma tela touchscreen capacitiva de

131
INFORMÁTICA

7 polegadas que permite uma maior sensibilidade e leveza 02. (UFFS - TÉCNICO DE LABORATÓRIO ÁREA
ao tocar na tela de seu tablet, dispensando totalmente o INFORMÁTICA – FEPESE/2012)- São componentes de
uso das inconvenientes canelas stykus. Possui saída mini hardware de um micro-computador:
HDMI, para curtir seus vídeos favoritos da internet ou de a. ( ) Disco rígido, patch-panel, BIOS, firmware,
seu computador, na sua televisão ou projetor, com entrada mouse.
HDMI. Além de acompanhar um lindo case com teclado b. ( ) RJ-11, processador, memória RAM, placa de
para utilização de tablet comparada com a de um note- rede, pen-drive.
book com grande performance. c. ( ) Memória ROM, placa de vídeo, BIOS, proces-
- Modelo: PHASER KINNO. sador, placa mãe.
- Capacidade: 4GB. Expansível para 32GB via Micro SD.
d. ( ) Memória RAM, Memória ROM, Disco rígido,
- Memória: 512MB.
processador, placa e rede.
- Tela:7 Polegadas capacitiva, sensível ao toque.
e. ( ) Memória RAM, BIOS, Disco rígido, processa-
- Câmera:frontal 2 megapixels.
- Conectividade: Wi-Fi - LAN 802.11b/g/n. dor, placa de rede.
- Processador:Allwinner A10 de 1.0~1.2 Ghz.
- Sistema Operacional:Android 2.3.4. Já vimos a respeito de Memória RAM, Memória ROM,
Disco Rígido e Processador.
Placa de rede é um hardware especificamente pro-
jetado para possibilitar a comunicação entre computa-
dores.

Placa de rede
Tablet Multilaser Diamond NB005 8GB Android 2.3
Tela 7 Polegadas RESPOSTA: “D”.
Wi-Fi HDMI
03. (TRE - ANALISTA JUDICIÁRIO – FCC/2012) -
Informações técnicas Em relação a hardware e software, é correto afirmar:
Marca: Multilaser a) Para que um software aplicativo esteja pronto
Capacidade :8 Gb. Memória expansível até 32 GB por para execução no computador, ele deve estar carrega-
cartão micro SD. do na memória flash.
Processador: Boxchip 1.5 GHz. b) O fator determinante de diferenciação entre um
Sistema Operacional: Android. 2.3. processador sem memória cache e outro com esse re-
TV e vídeo: Somente vídeo: Vídeos suportados - MKV curso reside na velocidade de acesso à memória RAM.
(H.264HP), AVI, RM/BMVB, FLV eMPEG-1/2. c) Processar e controlar as instruções executadas
Tamanho da tela: 7 “. LCD Multi toque. no computador é tarefa típica da unidade de aritmé-
Resolução: 800 x 480. tica e lógica.
Wi-Fi:Sim. d) O pendrive é um dispositivo de armazenamento
Resolução: 1.3 megapixels e filmadora digital. removível, dotado de memória flash e conector USB,
Localização
que pode ser conectado em vários equipamentos ele-
Sensores: Sensor de gravidade: gira a tela conforme a
trônicos.
posição do tablet.
e) Dispositivos de alta velocidade, tais como discos
Áudio Formatos suportados: rígidos e placas de vídeo, conectam-se diretamente ao
MP3, WMA, WAV, APE, AC3, FLAC e AAC. processador.
Duração aproximada da bateria:
- 06 horas reproduzindo vídeo ou wi-fi ligado; O pendrive, por ser um dispositivo portátil, de grande
- 48 horas em standby. poder de armazenamento e conector USB (Universal Serial
Alimentação do Tablet: Bus) que permite sua rápida aceitação em vários disposi-
Bateria recarregável. tivos de hardware, popularizou-se rapidamente. Hoje, en-
contramos pendrives de vários GBs, como 2, 4, 8, 16 e até
RESPOSTA: “ERRADO”. 512GB.

132
INFORMÁTICA

A tecnologia USB está sendo largamente utilizada para no disco rígido ainda. Como as informações são mantidas
padronizar entradas e conectores, possibilitando um mes- por pulsos elétricos, caso haja falta de energia, seja pelo
mo tipo de conector para diversos tipos de equipamentos desligamento do computador, seja por uma queda brusca
como mouses, teclados, impressoras e outros. Por esse mo- que cause o desligamento inesperado do equipamento, os
tivo, os equipamentos atuais possuem uma grande quan- dados presentes nesse tipo de memória serão perdidos.
tidade de conectores USB. Além disso, a tecnologia usada
por esses conectores é a Plugand Play, onde basta conectar Veja a seguir imagens ilustrativas da memória RAM.
o dispositivo para que o sistema o reconheça precisando
de poucos ou quase nenhum caminho de configuração
para poder utilizá-lo.
O tipo de memória que o pendrive utiliza - me-
mória flash - é do tipo EEPROM (Electrically-
ErasableProgrammableRead-OnlyMemory), uma
memória não volátil, ou seja, não depende da
permanência de energia elétrica para manter os dados,de
leitura e gravação. Os chips de memória flash ocupam pou-
co espaço físico, mas grande poder de armazenamento.
Veja imagens de pendrives:

Tipos de memória RAM

RESPOSTA: “A”.

05. (TCE/SP - AGENTE DE FISCALIZAÇÃO FINANCEIRA


– FCC/2012) - Sobre os computadores é correto afirmar:
a) O BIOS é um software armazenado em um chip de
Tipos de pendrive memória RAM fixado na placa mãe. Tem a função de ar-
mazenar o Sistema Operacional.
RESPOSTA: “D”. b) A fonte de alimentação transforma a tensão elé-
trica que entra no computador, de 240 V para 110 V, pois
04. (ANE - ANALISTA EDUCACIONAL – NÍVEL I – os componentes internos suportam apenas a tensão de
GRAU A – INSPETOR ESCOLAR – FCC/2012) - Marco Au- 110 V.
rélio estava digitando um documento na sala dos pro- b) Barramentos são circuitos integrados que fazem a
fessores da escola ABCD quando uma queda de energia transmissão física de dados de um dispositivo a outro.
fez com que o computador que usava desligasse. d) Quando o sistema de fornecimento de energia fa-
Após o retorno da energia elétrica, Marco Aurélio lha, um estabilizador comum tem como principal objetivo
ligou o computador e percebeu que havia perdido o manter o abastecimento por meio de sua bateria até que
documento digitado, pois não o havia gravado. Como a energia volte ou o computador seja desligado.
tinha conhecimentos gerais sobre informática, concluiu e) Um bit representa um sinal elétrico de exatos 5 V
que perdera o documento porque, enquanto estava di- que é interpretado pelos componentes de hardware do
gitando, ele estava armazenado em um dispositivo de computador.
hardware que perde seu conteúdo quando o computa-
dor desliga. O nome desse dispositivo é (A)BIOS é a sigla do termo Basic Input/Output System,
a) memória RAM. ou Sistema Básico de Entrada/Saída. É um software gravado
b) HD. na memória não volátil ou memória ROM, que é a sigla para
c) memória ROM. ReadOnlyMemory, ou Memória de Somente Leitura, que não
d) pen drive. altera ou perde os dados com o desligamento ou ausência
de energia do computador. Esse software não armazena o
RAM – Randon AcessMemory, ou Memória de Acesso Sistema Operacional. É o primeiro software que é executado
Randômico, é um hardware considerado como memória quando ligamos o computador.
primária, volátil. Ela mantém os dados armazenados en- (B)A fonte de alimentação do computador é um equipa-
quanto estes estão à disposição das solicitações do proces- mento eletrônico, fixada ao gabinete e ligada aos conectores
sador, mantendo-os através de pulsos elétricos. As infor- da placa mãe e alguns drives. Fornece energia aos demais
mações mantidas nesse tipo de memória são informações componentes da máquina. Ela transforma a corrente elétrica
que estão em uso em um programa em execução, como no alternada (que tem o sentido variável com o tempo) em uma
caso de textos que estão sendo digitados e não foram salvos corrente constante ao longo do tempo.

133
INFORMÁTICA

(C)Os barramentos são como vias de tráfego presentes Podemos citar também o autor Rogério Amigo De Oli-
na placa mãe, por onde sinais elétricos (representando da- veira, que em seu livro Informática – Teoria e Questões de
dos) podem percorrer toda sua extensão se comunicando Concursos com Gabarito, editora Campus, fala a respeito do
com todos os dispositivos. clock, da seguinte maneira:
(D)O estabilizador é um equipamento eletrônico exter- Em um computador, a velocidade do clock se refere ao
no ao gabinete do computador, onde os demais cabos de número de pulsos por segundo gerados por um oscilador (dis-
energia da máquina são ligados. Geralmente, o estabilizador positivo eletrônico que gera sinais), que determina o tempo
é ligado diretamente na rede elétrica e tem a função de es- necessário para o processador executar uma instrução. Assim
tabilizar a tensão desta para evitar danos ao equipamento para avaliar a performance de um processador, medimos a
devido às variações e picos de tensão. quantidade de pulsos gerados em 1 segundo e, para tanto,
(E)BIT é a sigla para BinaryDigit, ou Dígito Binário, que utilizamos uma unidade de medida de frequência, o Hertz.
pode ser representado apenas pelo 0 ou pelo 1 (verdadeiro
ou falso) que representam a menor unidade de informação RESPOSTA: “E”.
transmitida na computação ou informática.
07. (PREFEITURA DE ANGICOS/RN - TÉCNICO EM
RESPOSTA: “C”. MANUTENÇÃO DE COMPUTADOR – ACAPLAM/2012) -
São vários os fatores que causam a não detecção do HD
06. (TCE/SP - AGENTE DE FISCALIZAÇÃO FINAN- pelo Setup. Assim sendo, todas as alternativas abaixo
CEIRA – FCC/2012) - O processador do computador (ou são responsáveis por esse defeito, EXCETO:
CPU) é uma das partes principais do hardware do com- a) HD com defeito físico
putador e é responsável pelos cálculos, execução de ta- b) Defeito na placamãe
refas e processamento de dados. Sobre processadores, c) Defeito no cabo de alimentação do HD
considere: d) Defeito no cabo de dados do HD
I. Contém um conjunto restrito de células de memó- e) HD sem formatação
ria chamados registradores que podem ser lidos e escri-
tos muito mais rapidamente que em outros dispositivos HD é a sigla para Hard Disk e representa o hardwa-
de memória. re responsável pelo armazenamento das informações de
dados salvos pelo usuário, de programas instalados e até
II. Em relação a sua arquitetura, se destacam os mo-
informações presentes em memória virtual para posterior
delos RISC (ReducedInstruction Set Computer) e CISC
uso em processamentos de informação.
(ComplexInstruction Set Computer).
O HD é ligado por um cabo flat ao conector IDE da
III. Possuem um clock interno de sincronização que
placa mãe. Além dessa conexão, há também a conexão do
define a velocidade com que o processamento ocorre.
cabo da fonte de alimentação de energia.
Essa velocidade é medida em Hertz.
Se conectarmos um HD não formatado e ligarmos o
Está correto o que se afirma em
computador, a mensagem de detecção ocorrerá normal-
a) III, apenas.
mente, mas aparecerá outra mensagem que indica que não
b) I e II, apenas. há sistema operacional instalado.
c) II e III, apenas.
d) II, apenas. RESPOSTA: “E”.
e) I, II e III.

O processador é um chip que executa instruções inter-


nas do computador (em geral, operações matemáticas e
lógicas, leitura e gravação de informações). Todas as ações
estão presentes na memória do computador e requisitadas
pelo sistema. A velocidade do processador é medida em
ciclos denominados clocks e sua unidade é expressa atra-
vés de Hz.
Os registradores são unidades de memória que repre-
sentam o meio mais caro e rápido de armazenamento de
dados. Por isso são usados em pequenas quantidades nos
processadores.
Quanto às arquiteturas RISC e CISC, podemos nos valer
das palavras de Nicholas Carter, em seu livro Arquitetura de
Computadores, editora Bookman:
... RISC são arquiteturas de carga-armazenamento, en-
quanto que a maior parte das arquiteturas CISC permite que
outras operações também façam referência à memória.

134
Tribunal de Justiça do Estado de São Paulo

TJ-SP
Psicólogo Judiciário
Volume II
Edital de Abertura - Concurso Público
MR026-2017
DADOS DA OBRA

Título da obra: Tribunal de Justiça do Estado de São Paulo

Cargo: Psicólogo Judiciário

(Baseada no Edital de Abertura - Concurso Público)

Volume I
• Língua Portuguesa • Atualidades e Deveres dos Servidores Públicos• Informática

Volume II
• Conhecimentos Específicos • Legislação

Autores:
Ana Maria Quiteto
Bruna Pinotti Garcia Oliveira
Mariela Cardoso
Guilherme Cardoso

Gestão de Conteúdos
Emanuela Amaral de Souza

Produção Editorial/Revisão
Elaine Cristina
Igor de Oliveira
Suelen Domenica Pereira

Capa
Rosa Thaina dos Santos

Editoração Eletrônica
Marlene Moreno

Gerente de Projetos
Bruno Fernandes
APRESENTAÇÃO

PARABÉNS! ESTE É O PASSAPORTE PARA SUA APROVAÇÃO.

A Nova Concursos tem um único propósito: mudar a vida das pessoas.


Vamos ajudar você a alcançar o tão desejado cargo público.
Nossos livros são elaborados por professores que atuam na área de Concursos Públicos. Assim a
matéria é organizada de forma que otimize o tempo do candidato. Afinal corremos contra o tempo,
por isso a preparação é muito importante.
Aproveitando, convidamos você para conhecer nossa linha de produtos “Cursos online”, conteúdos
preparatórios e por edital, ministrados pelos melhores professores do mercado.
Estar à frente é nosso objetivo, sempre.
Contamos com índice de aprovação de 87%*.
O que nos motiva é a busca da excelência. Aumentar este índice é nossa meta.
Acesse www.novaconcursos.com.br e conheça todos os nossos produtos.
Oferecemos uma solução completa com foco na sua aprovação, como: apostilas, livros, cursos on-
line, questões comentadas e treinamentos com simulados online.
Desejamos-lhe muito sucesso nesta nova etapa da sua vida!
Obrigado e bons estudos!

*Índice de aprovação baseado em ferramentas internas de medição.

CURSO ONLINE

PASSO 1
Acesse:
www.novaconcursos.com.br/passaporte

PASSO 2
Digite o código do produto no campo indicado no
site.
O código encontra-se no verso da capa da apostila.
*Utilize sempre os 8 primeiros dígitos.
Ex: FV054-17

PASSO 3
Pronto!
Você já pode acessar os conteúdos online.
SUMÁRIO

Legislação

Código de Ética do Psicólogo............................................................................................................................................................................211


Constituição da República Federativa do Brasil - TÍTULO II - Dos Direitos e Garantias Fundamentais.........................................01
CAPÍTULO I - Dos Direitos e Deveres Individuais e Coletivos..........................................................................................................................01
CAPÍTULO II - Dos Direitos Sociais...............................................................................................................................................................................21
TÍTULO VIII - Do Ordem Social, CAPÍTULO II - Da Seguridade Social..........................................................................................................27
CAPÍTULO VII - Da Família, da Criança, do Adolescente, do Jovem e do Idoso......................................................................................32
Estatuto da Criança e do Adolescente - Atualizado com a Lei 12.010................................................................................................ 40
Código Civil - Lei 10.406/2002 Arts: 1511 a 1638; 1694 a 1727 e 1728 a 1783............................................................................... 95
Lei Maria da Penha - LEI 11.340/2006............................................................................................................................................................120
Código de Processo Civil – LEI 13.105/2015 Arts: 144 a 149, 156 a 158, 464 a 480, 693 a 699, 747 a 765........................129
Guarda Compartilhada - Lei 11.698/2014.....................................................................................................................................................138
Nova Guarda Compartilhada - Lei 13.058/2014.........................................................................................................................................138
Alienação Parental – Lei 12.318/2010.............................................................................................................................................................139
Sistema único de Assistência Social – SUAS – Lei 12.435/2011...........................................................................................................140
Sistema Nacional de Atendimento Socioeducativo – Lei 12.594/2012.............................................................................................144
Plano Nacional de Promoçâo, Proteção e defesa do direito de crianças e adolescentes à convívência familiar e comuni-
tária - 2006................................................................................................................................................................................................................151
Estatuto do Idoso – Lei 10.741/2003..............................................................................................................................................................179
Lei Brasileira de Inclusão da Pessoa com Deficiência - Lei 13.146/2015..........................................................................................189

Conhecimentos Específicos

Desenvolvimento psicológico: infância e adolescência............................................................................................................................. 01


A formação e rompimento dos laços afetivos.............................................................................................................................................. 08
A importância do ambiente.................................................................................................................................................................................. 08
Consequências das falhas ambientais.............................................................................................................................................................. 08
O papel do pai........................................................................................................................................................................................................... 16
O papel da agressividade no desenvolvimento humano......................................................................................................................... 19
Natureza e origens da tendência antissocial................................................................................................................................................. 22
Prevenção e efeitos da privação materna....................................................................................................................................................... 24
As inter-relações familiares: casamento, conflito conjugal, separação, guarda dos filhos, violência doméstica................ 27
Aspectos psicossociais do envelhecimento.................................................................................................................................................... 49
Aspectos psicossociais do fenômeno da violência...................................................................................................................................... 57
A criança e a separação dos pais........................................................................................................................................................................ 66
Os direitos fundamentais da criança e do adolescente............................................................................................................................. 70
As medidas específicas de proteção à criança e ao adolescente.......................................................................................................... 85
A criança e o adolescente no acolhimento institucional........................................................................................................................... 91
A colocação em família substituta...................................................................................................................................................................101
As medidas socioeducativas...............................................................................................................................................................................105
O psicólogo no atendimento aos casos nas Varas da Infância e da Juventude, nas Varas da Família e das Sucessões,
Violência Doméstica e ao Idoso........................................................................................................................................................................107
Avaliação Psicológica: instrumentais e sua prática na instituição judiciária....................................................................................124
A entrevista psicológica.......................................................................................................................................................................................124
Elaboração de documentos escritos: laudos, relatórios e pareceres psicológicos........................................................................138
O lugar do saber psicológico na instituição judiciária.............................................................................................................................150
Ética profissional.....................................................................................................................................................................................................153
LEGISLAÇÃO

Constituição da República Federativa do Brasil - TÍTULO II - Dos Direitos e Garantias Fundamentais.........................................01


CAPÍTULO I - Dos Direitos e Deveres Individuais e Coletivos..........................................................................................................................01
CAPÍTULO II - Dos Direitos Sociais...............................................................................................................................................................................21
TÍTULO VIII - Do Ordem Social, CAPÍTULO II - Da Seguridade Social..........................................................................................................27
CAPÍTULO VII - Da Família, da Criança, do Adolescente, do Jovem e do Idoso......................................................................................32
Estatuto da Criança e do Adolescente - Atualizado com a Lei 12.010................................................................................................ 40
Código Civil - Lei 10.406/2002 Arts: 1511 a 1638; 1694 a 1727 e 1728 a 1783............................................................................... 95
Lei Maria da Penha - LEI 11.340/2006............................................................................................................................................................120
Código de Processo Civil – LEI 13.105/2015 Arts: 144 a 149, 156 a 158, 464 a 480, 693 a 699, 747 a 765........................129
Guarda Compartilhada - Lei 11.698/2014.....................................................................................................................................................138
Nova Guarda Compartilhada - Lei 13.058/2014.........................................................................................................................................138
Alienação Parental – Lei 12.318/2010.............................................................................................................................................................139
Sistema único de Assistência Social – SUAS – Lei 12.435/2011...........................................................................................................140
Sistema Nacional de Atendimento Socioeducativo – Lei 12.594/2012.............................................................................................144
Plano Nacional de Promoçâo, Proteção e defesa do direito de crianças e adolescentes à convívência familiar e comuni-
tária - 2006................................................................................................................................................................................................................151
Estatuto do Idoso – Lei 10.741/2003..............................................................................................................................................................179
Lei Brasileira de Inclusão da Pessoa com Deficiência - Lei 13.146/2015..........................................................................................189
Código de Ética do/a Assistente Social – Lei 8.662/93............................................................................................................................208
Código de Ética do Psicólogo............................................................................................................................................................................211
LEGISLAÇÃO

CONSTITUIÇÃO DA REPÚBLICA FEDERATIVA


CAPÍTULO I
DO BRASIL
DOS DIREITOS E DEVERES INDIVIDUAIS E
TÍTULO II
COLETIVOS
DOS DIREITOS E GARANTIAS
FUNDAMENTAIS

O capítulo I do título II é intitulado “direitos e deve-


O título II da Constituição Federal é intitulado “Direitos e res individuais e coletivos”. Da própria nomenclatura do
Garantias fundamentais”, gênero que abrange as seguintes capítulo já se extrai que a proteção vai além dos direitos
espécies de direitos fundamentais: direitos individuais e co- do indivíduo e também abrange direitos da coletividade. A
letivos (art. 5º, CF), direitos sociais (genericamente previstos maior parte dos direitos enumerados no artigo 5º do texto
no art. 6º, CF), direitos da nacionalidade (artigos 12 e 13, CF) constitucional é de direitos individuais, mas são incluídos
e direitos políticos (artigos 14 a 17, CF). alguns direitos coletivos e mesmo remédios constitucio-
Em termos comparativos à clássica divisão tridimensio- nais próprios para a tutela destes direitos coletivos (ex.:
nal dos direitos humanos, os direitos individuais (maior parte mandado de segurança coletivo).
do artigo 5º, CF), os direitos da nacionalidade e os direitos
políticos se encaixam na primeira dimensão (direitos civis e 1) Direitos e garantias
políticos); os direitos sociais se enquadram na segunda di- Não obstante, o capítulo vai além da proteção dos di-
mensão (direitos econômicos, sociais e culturais) e os direitos reitos e estabelece garantias em prol da preservação des-
coletivos na terceira dimensão. Contudo, a enumeração de tes, bem como remédios constitucionais a serem utilizados
direitos humanos na Constituição vai além dos direitos que caso estes direitos e garantias não sejam preservados. Nes-
expressamente constam no título II do texto constitucional. te sentido, dividem-se em direitos e garantias as previsões
Os direitos fundamentais possuem as seguintes caracte- do artigo 5º: os direitos são as disposições declaratórias e
rísticas principais: as garantias são as disposições assecuratórias.
a) Historicidade: os direitos fundamentais possuem an- O legislador muitas vezes reúne no mesmo dispositivo
tecedentes históricos relevantes e, através dos tempos, ad- o direito e a garantia, como no caso do artigo 5º, IX:
quirem novas perspectivas. Nesta característica se enquadra
a noção de dimensões de direitos. Artigo 5º, IX, CF. É livre a expressão da atividade inte-
b) Universalidade: os direitos fundamentais pertencem lectual, artística, científica e de comunicação, independente-
a todos, tanto que apesar da expressão restritiva do caput do mente de censura ou licença.
artigo 5º aos brasileiros e estrangeiros residentes no país tem
se entendido pela extensão destes direitos, na perspectiva de O direito é o de liberdade de expressão e a garantia é
prevalência dos direitos humanos. a vedação de censura ou exigência de licença. Em outros
c) Inalienabilidade: os direitos fundamentais não pos- casos, o legislador traz o direito num dispositivo e a garan-
suem conteúdo econômico-patrimonial, logo, são intransfe- tia em outro: a liberdade de locomoção, direito, é colocada
ríveis, inegociáveis e indisponíveis, estando fora do comércio, no artigo 5º, XV, ao passo que o dever de relaxamento da
o que evidencia uma limitação do princípio da autonomia prisão ilegal de ofício pelo juiz, garantia, se encontra no
privada. artigo 5º, LXV1.
d) Irrenunciabilidade: direitos fundamentais não podem Em caso de ineficácia da garantia, implicando em vio-
ser renunciados pelo seu titular devido à fundamentalidade lação de direito, cabe a utilização dos remédios constitu-
material destes direitos para a dignidade da pessoa humana. cionais.
e) Inviolabilidade: direitos fundamentais não podem Atenção para o fato de o constituinte chamar os remé-
deixar de ser observados por disposições infraconstitucionais dios constitucionais de garantias, e todas as suas fórmulas
ou por atos das autoridades públicas, sob pena de nulidades. de direitos e garantias propriamente ditas apenas de di-
f) Indivisibilidade: os direitos fundamentais compõem reitos.
um único conjunto de direitos porque não podem ser anali-
sados de maneira isolada, separada. 2) Brasileiros e estrangeiros
g) Imprescritibilidade: os direitos fundamentais não O caput do artigo 5º aparenta restringir a proteção
se perdem com o tempo, não prescrevem, uma vez que são conferida pelo dispositivo a algumas pessoas, notadamen-
sempre exercíveis e exercidos, não deixando de existir pela te, “aos brasileiros e aos estrangeiros residentes no País”.
falta de uso (prescrição). No entanto, tal restrição é apenas aparente e tem sido in-
h) Relatividade: os direitos fundamentais não podem terpretada no sentido de que os direitos estarão protegi-
ser utilizados como um escudo para práticas ilícitas ou como dos com relação a todas as pessoas nos limites da sobera-
argumento para afastamento ou diminuição da responsabi- nia do país.
lidade por atos ilícitos, assim estes direitos não são ilimi-
tados e encontram seus limites nos demais direitos igual- 1 FARIA, Cássio Juvenal. Notas pessoais tomadas
mente consagrados como humanos. em teleconferência.

1
LEGISLAÇÃO

Em razão disso, por exemplo, um estrangeiro pode in- - Direito à igualdade


gressar com habeas corpus ou mandado de segurança, ou Abrangência
então intentar ação reivindicatória com relação a imóvel Observa-se, pelo teor do caput do artigo 5º, CF, que o
seu localizado no Brasil (ainda que não resida no país). constituinte afirmou por duas vezes o princípio da igual-
Somente alguns direitos não são estendidos a todas as dade:
pessoas. A exemplo, o direito de intentar ação popular exi-
ge a condição de cidadão, que só é possuída por nacionais Artigo 5º, caput, CF. Todos são iguais perante a lei, sem
titulares de direitos políticos. distinção de qualquer natureza, garantindo-se aos brasilei-
ros e aos estrangeiros residentes no País a inviolabilidade
3) Relação direitos-deveres do direito à vida, à liberdade, à igualdade, à segurança e à
O capítulo em estudo é denominado “direitos e garan- propriedade, nos termos seguintes [...].
tias deveres e coletivos”, remetendo à necessária relação
direitos-deveres entre os titulares dos direitos fundamen- Não obstante, reforça este princípio em seu primeiro
inciso:
tais. Acima de tudo, o que se deve ter em vista é a pre-
missa reconhecida nos direitos fundamentais de que não
Artigo 5º, I, CF. Homens e mulheres são iguais em direi-
há direito que seja absoluto, correspondendo-se para cada
tos e obrigações, nos termos desta Constituição.
direito um dever. Logo, o exercício de direitos fundamen-
tais é limitado pelo igual direito de mesmo exercício por Este inciso é especificamente voltado à necessidade de
parte de outrem, não sendo nunca absolutos, mas sempre igualdade de gênero, afirmando que não deve haver ne-
relativos. nhuma distinção sexo feminino e o masculino, de modo
Explica Canotilho2 quanto aos direitos fundamentais: “a que o homem e a mulher possuem os mesmos direitos e
ideia de deveres fundamentais é suscetível de ser entendi- obrigações.
da como o ‘outro lado’ dos direitos fundamentais. Como Entretanto, o princípio da isonomia abrange muito
ao titular de um direito fundamental corresponde um de- mais do que a igualdade de gêneros, envolve uma pers-
ver por parte de um outro titular, poder-se-ia dizer que o pectiva mais ampla.
particular está vinculado aos direitos fundamentais como O direito à igualdade é um dos direitos norteadores
destinatário de um dever fundamental. Neste sentido, um de interpretação de qualquer sistema jurídico. O primeiro
direito fundamental, enquanto protegido, pressuporia um enfoque que foi dado a este direito foi o de direito civil,
dever correspondente”. Com efeito, a um direito funda- enquadrando-o na primeira dimensão, no sentido de que a
mental conferido à pessoa corresponde o dever de respei- todas as pessoas deveriam ser garantidos os mesmos direi-
to ao arcabouço de direitos conferidos às outras pessoas. tos e deveres. Trata-se de um aspecto relacionado à igual-
dade enquanto liberdade, tirando o homem do arbítrio dos
4) Direitos e garantias em espécie demais por meio da equiparação. Basicamente, estaria se
Preconiza o artigo 5º da Constituição Federal em seu falando na igualdade perante a lei.
caput: No entanto, com o passar dos tempos, se percebeu
que não bastava igualar todos os homens em direitos e
Artigo 5º, caput, CF. Todos são iguais perante a lei, sem deveres para torná-los iguais, pois nem todos possuem
distinção de qualquer natureza, garantindo-se aos brasilei- as mesmas condições de exercer estes direitos e deveres.
ros e aos estrangeiros residentes no País a inviolabilidade Logo, não é suficiente garantir um direito à igualdade for-
do direito à vida, à liberdade, à igualdade, à segurança e à mal, mas é preciso buscar progressivamente a igualdade
material. No sentido de igualdade material que aparece o
propriedade, nos termos seguintes [...].
direito à igualdade num segundo momento, pretendendo-
se do Estado, tanto no momento de legislar quanto no de
O caput do artigo 5º, que pode ser considerado um
aplicar e executar a lei, uma postura de promoção de polí-
dos principais (senão o principal) artigos da Constituição ticas governamentais voltadas a grupos vulneráveis.
Federal, consagra o princípio da igualdade e delimita as Assim, o direito à igualdade possui dois sentidos notá-
cinco esferas de direitos individuais e coletivos que mere- veis: o de igualdade perante a lei, referindo-se à aplicação
cem proteção, isto é, vida, liberdade, igualdade, segurança uniforme da lei a todas as pessoas que vivem em socieda-
e propriedade. Os incisos deste artigos delimitam vários de; e o de igualdade material, correspondendo à necessi-
direitos e garantias que se enquadram em alguma destas dade de discriminações positivas com relação a grupos vul-
esferas de proteção, podendo se falar em duas esferas es- neráveis da sociedade, em contraponto à igualdade formal.
pecíficas que ganham também destaque no texto consti-
tucional, quais sejam, direitos de acesso à justiça e direitos Ações afirmativas
constitucionais-penais. Neste sentido, desponta a temática das ações afirmati-
vas,que são políticas públicas ou programas privados cria-
dos temporariamente e desenvolvidos com a finalidade de
reduzir as desigualdades decorrentes de discriminações ou
2 CANOTILHO, José Joaquim Gomes. Direito cons- de uma hipossuficiência econômica ou física, por meio da
titucional e teoria da constituição. 2. ed. Coimbra: Almedi- concessão de algum tipo de vantagem compensatória de
na, 1998, p. 479. tais condições.

2
LEGISLAÇÃO

Quem é contra as ações afirmativas argumenta que, Vedação à tortura


em uma sociedade pluralista, a condição de membro de um De forma expressa no texto constitucional destaca-se
grupo específico não pode ser usada como critério de inclu- a vedação da tortura, corolário do direito à vida, conforme
são ou exclusão de benefícios. Ademais, afirma-se que elas previsão no inciso III do artigo 5º:
desprivilegiam o critério republicano do mérito (segundo o
qual o indivíduo deve alcançar determinado cargo público Artigo 5º, III, CF. Ninguém será submetido a tortura nem
pela sua capacidade e esforço, e não por pertencer a de- a tratamento desumano ou degradante.
terminada categoria); fomentariam o racismo e o ódio; bem
como ferem o princípio da isonomia por causar uma discri- A tortura é um dos piores meios de tratamento de-
minação reversa. sumano, expressamente vedada em âmbito internacional,
Por outro lado, quem é favorável às ações afirmativas como visto no tópico anterior. No Brasil, além da disciplina
defende que elas representam o ideal de justiça compen- constitucional, a Lei nº 9.455, de 7 de abril de 1997 define
satória (o objetivo é compensar injustiças passadas, dívidas os crimes de tortura e dá outras providências, destacando-
históricas, como uma compensação aos negros por tê-los
se o artigo 1º:
feito escravos, p. ex.); representam o ideal de justiça distri-
butiva (a preocupação, aqui, é com o presente. Busca-se
Art. 1º Constitui crime de tortura:
uma concretização do princípio da igualdade material); bem
como promovem a diversidade. I - constranger alguém com emprego de violência ou
Neste sentido, as discriminações legais asseguram a ver- grave ameaça, causando-lhe sofrimento físico ou mental:
dadeira igualdade, por exemplo, com as ações afirmativas, a a) com o fim de obter informação, declaração ou confis-
proteção especial ao trabalho da mulher e do menor, as ga- são da vítima ou de terceira pessoa;
rantias aos portadores de deficiência, entre outras medidas b) para provocar ação ou omissão de natureza crimi-
que atribuam a pessoas com diferentes condições, iguais nosa;
possibilidades, protegendo e respeitando suas diferenças3. c) em razão de discriminação racial ou religiosa;
Tem predominado em doutrina e jurisprudência, inclusive II - submeter alguém, sob sua guarda, poder ou autori-
no Supremo Tribunal Federal, que as ações afirmativas são dade, com emprego de violência ou grave ameaça, a intenso
válidas. sofrimento físico ou mental, como forma de aplicar castigo
pessoal ou medida de caráter preventivo.
- Direito à vida Pena - reclusão, de dois a oito anos.
Abrangência § 1º Na mesma pena incorre quem submete pessoa
O caput do artigo 5º da Constituição assegura a prote- presa ou sujeita a medida de segurança a sofrimento físico
ção do direito à vida. A vida humana é o centro gravitacio- ou mental, por intermédio da prática de ato não previsto
nal em torno do qual orbitam todos os direitos da pessoa em lei ou não resultante de medida legal.
humana, possuindo reflexos jurídicos, políticos, econômicos, § 2º Aquele que se omite em face dessas condutas,
morais e religiosos. Daí existir uma dificuldade em concei- quando tinha o dever de evitá-las ou apurá-las, incorre na
tuar o vocábulo vida. Logo, tudo aquilo que uma pessoa pena de detenção de um a quatro anos.
possui deixa de ter valor ou sentido se ela perde a vida. Sen- § 3º Se resulta lesão corporal de natureza grave ou
do assim, a vida é o bem principal de qualquer pessoa, é o gravíssima, a pena é de reclusão de quatro a dez anos; se
primeiro valor moral inerente a todos os seres humanos4. resulta morte, a reclusão é de oito a dezesseis anos.
No tópico do direito à vida tem-se tanto o direito de § 4º Aumenta-se a pena de um sexto até um terço:
nascer/permanecer vivo, o que envolve questões como I - se o crime é cometido por agente público;
pena de morte, eutanásia, pesquisas com células-tronco e
II – se o crime é cometido contra criança, gestante, por-
aborto; quanto o direito de viver com dignidade, o que
tador de deficiência, adolescente ou maior de 60 (sessenta)
engloba o respeito à integridade física, psíquica e moral, in-
anos; 
cluindo neste aspecto a vedação da tortura, bem como a ga-
rantia de recursos que permitam viver a vida com dignidade. III - se o crime é cometido mediante sequestro.
Embora o direito à vida seja em si pouco delimitado nos § 5º A condenação acarretará a perda do cargo, função
incisos que seguem o caput do artigo 5º, trata-se de um dos ou emprego público e a interdição para seu exercício pelo
direitos mais discutidos em termos jurisprudenciais e socio- dobro do prazo da pena aplicada.
lógicos. É no direito à vida que se encaixam polêmicas dis- § 6º O crime de tortura é inafiançável e insuscetível de
cussões como: aborto de anencéfalo, pesquisa com células graça ou anistia.
tronco, pena de morte, eutanásia, etc. § 7º O condenado por crime previsto nesta Lei, salvo a
hipótese do § 2º, iniciará o cumprimento da pena em regi-
3 SANFELICE, Patrícia de Mello. Comentários aos me fechado.
artigos I e II. In: BALERA, Wagner (Coord.). Comentários
à Declaração Universal dos Direitos do Homem. Brasília: - Direito à liberdade
Fortium, 2008, p. 08. O caput do artigo 5º da Constituição assegura a pro-
4 BARRETO, Ana Carolina Rossi; IBRAHIM, Fábio teção do direito à liberdade, delimitada em alguns incisos
Zambitte. Comentários aos Artigos III e IV. In: BALERA, Wag- que o seguem.
ner (Coord.). Comentários à Declaração Universal dos Di-
reitos do Homem. Brasília: Fortium, 2008, p. 15.

3
LEGISLAÇÃO

Liberdade e legalidade Artigo 5º, IX, CF. É livre a expressão da atividade inte-
Prevê o artigo 5º, II, CF: lectual, artística, científica e de comunicação, indepen-
dentemente de censura ou licença.
Artigo 5º, II, CF. Ninguém será obrigado a fazer ou dei-
xar de fazer alguma coisa senão em virtude de lei. Consolida-se outra perspectiva da liberdade de expres-
são, referente de forma específica a atividades intelectuais,
O princípio da legalidade se encontra delimitado nes- artísticas, científicas e de comunicação. Dispensa-se, com
te inciso, prevendo que nenhuma pessoa será obrigada a relação a estas, a exigência de licença para a manifestação
fazer ou deixar de fazer alguma coisa a não ser que a lei do pensamento, bem como veda-se a censura prévia.
assim determine. Assim, salvo situações previstas em lei, A respeito da censura prévia, tem-se não cabe impe-
a pessoa tem liberdade para agir como considerar conve- dir a divulgação e o acesso a informações como modo de
niente. controle do poder. A censura somente é cabível quando
Portanto, o princípio da legalidade possui estrita rela- necessária ao interesse público numa ordem democrática,
ção com o princípio da liberdade, posto que, a priori, tudo por exemplo, censurar a publicação de um conteúdo de
à pessoa é lícito. Somente é vedado o que a lei expres- exploração sexual infanto-juvenil é adequado.
samente estabelecer como proibido. A pessoa pode fazer O direito à resposta (artigo 5º, V, CF) e o direito à in-
tudo o que quiser, como regra, ou seja, agir de qualquer denização (artigo 5º, X, CF) funcionam como a contrapar-
maneira que a lei não proíba. tida para aquele que teve algum direito seu violado (no-
tadamente inerentes à privacidade ou à personalidade)
Liberdade de pensamento e de expressão em decorrência dos excessos no exercício da liberdade de
O artigo 5º, IV, CF prevê: expressão.

Artigo 5º, IV, CF. É livre a manifestação do pensamen- Liberdade de crença/religiosa


to, sendo vedado o anonimato. Dispõe o artigo 5º, VI, CF:

Consolida-se a afirmação simultânea da liberdade de Artigo 5º, VI, CF. É inviolável a liberdade de consciên-
pensamento e da liberdade de expressão. cia e de crença, sendo assegurado o livre exercício dos
Em primeiro plano tem-se a liberdade de pensamento. cultos religiosos e garantida, na forma da lei, a proteção
aos locais de culto e a suas liturgias.
Afinal, “o ser humano, através dos processos internos de
reflexão, formula juízos de valor. Estes exteriorizam nada
Cada pessoa tem liberdade para professar a sua fé
mais do que a opinião de seu emitente. Assim, a regra
como bem entender dentro dos limites da lei. Não há uma
constitucional, ao consagrar a livre manifestação do pensa-
crença ou religião que seja proibida, garantindo-se que a
mento, imprime a existência jurídica ao chamado direito de
profissão desta fé possa se realizar em locais próprios.
opinião”5. Em outras palavras, primeiro existe o direito de
Nota-se que a liberdade de religião engloba 3 tipos
ter uma opinião, depois o de expressá-la.
distintos, porém intrinsecamente relacionados de liberda-
No mais, surge como corolário do direito à liberdade des: a liberdade de crença; a liberdade de culto; e a liber-
de pensamento e de expressão o direito à escusa por con- dade de organização religiosa.
vicção filosófica ou política: Consoante o magistério de José Afonso da Silva6, entra
na liberdade de crença a liberdade de escolha da religião,
Artigo 5º, VIII, CF. Ninguém será privado de direitos por a liberdade de aderir a qualquer seita religiosa, a liberdade
motivo de crença religiosa ou de convicção filosófica ou (ou o direito) de mudar de religião, além da liberdade de
política, salvo se as invocar para eximir-se de obrigação não aderir a religião alguma, assim como a liberdade de
legal a todos imposta e recusar-se a cumprir prestação al- descrença, a liberdade de ser ateu e de exprimir o agnos-
ternativa, fixada em lei. ticismo, apenas excluída a liberdade de embaraçar o livre
exercício de qualquer religião, de qualquer crença. A liber-
Trata-se de instrumento para a consecução do direito dade de culto consiste na liberdade de orar e de praticar
assegurado na Constituição Federal – não basta permitir os atos próprios das manifestações exteriores em casa ou
que se pense diferente, é preciso respeitar tal posiciona- em público, bem como a de recebimento de contribuições
mento. para tanto. Por fim, a liberdade de organização religiosa
Com efeito, este direito de liberdade de expressão é refere-se à possibilidade de estabelecimento e organização
limitado. Um destes limites é o anonimato, que consiste na de igrejas e suas relações com o Estado.
garantia de atribuir a cada manifestação uma autoria cer- Como decorrência do direito à liberdade religiosa, as-
ta e determinada, permitindo eventuais responsabilizações segurando o seu exercício, destaca-se o artigo 5º, VII, CF:
por manifestações que contrariem a lei.
Tem-se, ainda, a seguinte previsão no artigo 5º, IX, CF: Artigo 5º, VII, CF. É assegurada, nos termos da lei, a pres-
tação de assistência religiosa nas entidades civis e mili-
5 ARAÚJO, Luiz Alberto David; NUNES JÚNIOR, Vi- tares de internação coletiva.
dal Serrano. Curso de direito constitucional. 10. ed. São 6 SILVA, José Afonso da. Curso de direito constitu-
Paulo: Saraiva, 2006. cional positivo. 25. ed. São Paulo: Malheiros, 2006.

4
LEGISLAÇÃO

O dispositivo refere-se não só aos estabelecimentos A respeito, a Lei nº 12.527, de 18 de novembro de 2011
prisionais civis e militares, mas também a hospitais. regula o acesso a informações previsto no inciso XXXIII do
Ainda, surge como corolário do direito à liberdade reli- art. 5º, CF, também conhecida como Lei do Acesso à Infor-
giosa o direito à escusa por convicção religiosa: mação.
Não obstante, estabelece o artigo 5º, XXXIV, CF:
Artigo 5º, VIII, CF. Ninguém será privado de direitos por
motivo de crença religiosa ou de convicção filosófica ou po- Artigo 5º, XXXIV, CF. São a todos assegurados, indepen-
lítica, salvo se as invocar para eximir-se de obrigação legal dentemente do pagamento de taxas:
a todos imposta e recusar-se a cumprir prestação alterna- a) o direito de petição aos Poderes Públicos em defesa
tiva, fixada em lei. de direitos ou contra ilegalidade ou abuso de poder;
b) a obtenção de certidões em repartições públicas,
Sempre que a lei impõe uma obrigação a todos, por para defesa de direitos e esclarecimento de situações de in-
exemplo, a todos os homens maiores de 18 anos o alis- teresse pessoal.
tamento militar, não cabe se escusar, a não ser que tenha
fundado motivo em crença religiosa ou convicção filosó- Quanto ao direito de petição, de maneira prática, cum-
fica/política, caso em que será obrigado a cumprir uma pre observar que o direito de petição deve resultar em uma
prestação alternativa, isto é, uma outra atividade que não manifestação do Estado, normalmente dirimindo (resol-
contrarie tais preceitos. vendo) uma questão proposta, em um verdadeiro exercí-
cio contínuo de delimitação dos direitos e obrigações que
Liberdade de informação regulam a vida social e, desta maneira, quando “dificulta
O direito de acesso à informação também se liga a uma a apreciação de um pedido que um cidadão quer apre-
dimensão do direito à liberdade. Neste sentido, prevê o sentar” (muitas vezes, embaraçando-lhe o acesso à Justiça);
artigo 5º, XIV, CF: “demora para responder aos pedidos formulados” (admi-
nistrativa e, principalmente, judicialmente) ou “impõe res-
Artigo 5º, XIV, CF. É assegurado a todos o acesso à in- trições e/ou condições para a formulação de petição”, traz
formação e resguardado o sigilo da fonte, quando neces- a chamada insegurança jurídica, que traz desesperança e
sário ao exercício profissional. faz proliferar as desigualdades e as injustiças.
Dentro do espectro do direito de petição se insere, por
Trata-se da liberdade de informação, consistente na
exemplo, o direito de solicitar esclarecimentos, de solicitar
liberdade de procurar e receber informações e ideias por
cópias reprográficas e certidões, bem como de ofertar de-
quaisquer meios, independente de fronteiras, sem interfe-
núncias de irregularidades. Contudo, o constituinte, talvez
rência.
na intenção de deixar clara a obrigação dos Poderes Públi-
A liberdade de informação tem um caráter passivo, ao
cos em fornecer certidões, trouxe a letra b) do inciso, o que
passo que a liberdade de expressão tem uma caracterís-
gera confusões conceituais no sentido do direito de obter
tica ativa, de forma que juntas formam os aspectos ativo
e passivo da exteriorização da liberdade de pensamento: certidões ser dissociado do direito de petição.
não basta poder manifestar o seu próprio pensamento, é Por fim, relevante destacar a previsão do artigo 5º, LX,
preciso que ele seja ouvido e, para tanto, há necessidade CF:
de se garantir o acesso ao pensamento manifestado para
a sociedade. Artigo 5º, LX, CF. A lei só poderá restringir a publicida-
Por sua vez, o acesso à informação envolve o direito de de dos atos processuais quando a defesa da intimidade ou
todos obterem informações claras, precisas e verdadeiras a o interesse social o exigirem.
respeito de fatos que sejam de seu interesse, notadamente
pelos meios de comunicação imparciais e não monopoli- Logo,o processo, em regra, não será sigiloso. Apenas
zados (artigo 220, CF). No entanto, nem sempre é possível o será quando a intimidade merecer preservação (ex: pro-
que a imprensa divulgue com quem obteve a informação cesso criminal de estupro ou causas de família em geral) ou
divulgada, sem o que a segurança desta poderia ficar pre- quando o interesse social exigir (ex: investigações que pos-
judicada e a informação inevitavelmente não chegaria ao sam ser comprometidas pela publicidade). A publicidade é
público. instrumento para a efetivação da liberdade de informação.
Especificadamente quanto à liberdade de informação
no âmbito do Poder Público, merecem destaque algumas Liberdade de locomoção
previsões. Outra faceta do direito à liberdade encontra-se no ar-
Primeiramente, prevê o artigo 5º, XXXIII, CF: tigo 5º, XV, CF:

Artigo 5º, XXXIII, CF. Todos têm direito a receber dos Artigo 5º, XV, CF. É livre a locomoção no território
órgãos públicos informações de seu interesse particular, nacional em tempo de paz, podendo qualquer pessoa, nos
ou de interesse coletivo ou geral, que serão prestadas no pra- termos da lei, nele entrar, permanecer ou dele sair com seus
zo da lei, sob pena de responsabilidade, ressalvadas aquelas bens.
cujo sigilo seja imprescindível à segurança da sociedade e
do Estado.

5
LEGISLAÇÃO

A liberdade de locomoção é um aspecto básico do di- sistência médica, evitando algazarras e socorrendo pessoas
reito à liberdade, permitindo à pessoa ir e vir em todo o que tenham algum mal-estar no local. Outro limite é o uso
território do país em tempos de paz (em tempos de guerra de armas, totalmente vedado, assim como de substâncias
é possível limitar tal liberdade em prol da segurança). A ilícitas (Ex: embora a Marcha da Maconha tenha sido auto-
liberdade de sair do país não significa que existe um direito rizada pelo Supremo Tribunal Federal, vedou-se que nela
de ingressar em qualquer outro país, pois caberá à ele, no tal substância ilícita fosse utilizada).
exercício de sua soberania, controlar tal entrada.
Classicamente, a prisão é a forma de restrição da liber- Liberdade de associação
dade. Neste sentido, uma pessoa somente poderá ser pre- No que tange à liberdade de reunião, traz o artigo 5º,
sa nos casos autorizados pela própria Constituição Federal. XVII, CF:
A despeito da normativa específica de natureza penal, re-
força-se a impossibilidade de se restringir a liberdade de Artigo 5º, XVII, CF. É plena a liberdade de associação
locomoção pela prisão civil por dívida. para fins lícitos, vedada a de caráter paramilitar.
Prevê o artigo 5º, LXVII, CF:
A liberdade de associação difere-se da de reunião por
Artigo 5º, LXVII, CF. Não haverá prisão civil por dívi- sua perenidade, isto é, enquanto a liberdade de reunião é
da, salvo a do responsável pelo inadimplemento voluntário exercida de forma sazonal, eventual, a liberdade de asso-
e inescusável de obrigação alimentícia e a do depositário ciação implica na formação de um grupo organizado que
infiel. se mantém por um período de tempo considerável, dotado
de estrutura e organização próprias.
Nos termos da Súmula Vinculante nº 25 do Supremo Por exemplo, o PCC e o Comando vermelho são asso-
Tribunal Federal, “é ilícita a prisão civil de depositário infiel, ciações ilícitas e de caráter paramilitar, pois possuem ar-
qualquer que seja a modalidade do depósito”. Por isso, a mas e o ideal de realizar sua própria justiça paralelamente
única exceção à regra da prisão por dívida do ordenamento à estatal.
é a que se refere à obrigação alimentícia. O texto constitucional se estende na regulamentação
da liberdade de associação.
Liberdade de trabalho O artigo 5º, XVIII, CF, preconiza:
O direito à liberdade também é mencionado no artigo
5º, XIII, CF: Artigo 5º, XVIII, CF. A criação de associações e, na for-
ma da lei, a de cooperativas independem de autorização,
Artigo 5º, XIII, CF. É livre o exercício de qualquer tra- sendo vedada a interferência estatal em seu funcionamento.
balho, ofício ou profissão, atendidas as qualificações
profissionais que a lei estabelecer. Neste sentido, associações são organizações resultan-
tes da reunião legal entre duas ou mais pessoas, com ou
O livre exercício profissional é garantido, respeitados sem personalidade jurídica, para a realização de um obje-
os limites legais. Por exemplo, não pode exercer a profis- tivo comum; já cooperativas são uma forma específica de
são de advogado aquele que não se formou em Direito associação, pois visam a obtenção de vantagens comuns
e não foi aprovado no Exame da Ordem dos Advogados em suas atividades econômicas.
do Brasil; não pode exercer a medicina aquele que não fez Ainda, tem-se o artigo 5º, XIX, CF:
faculdade de medicina reconhecida pelo MEC e obteve o
cadastro no Conselho Regional de Medicina. Artigo 5º, XIX, CF. As associações só poderão ser com-
pulsoriamente dissolvidas ou ter suas atividades suspensas
Liberdade de reunião por decisão judicial, exigindo-se, no primeiro caso, o trânsito
Sobre a liberdade de reunião, prevê o artigo 5º, XVI, CF: em julgado.

Artigo 5º, XVI, CF. Todos podem reunir-se pacificamen- O primeiro caso é o de dissolução compulsória, ou seja,
te, sem armas, em locais abertos ao público, independen- a associação deixará de existir para sempre. Obviamente, é
temente de autorização, desde que não frustrem outra re- preciso o trânsito em julgado da decisão judicial que as-
união anteriormente convocada para o mesmo local, sendo sim determine, pois antes disso sempre há possibilidade
apenas exigido prévio aviso à autoridade competente. de reverter a decisão e permitir que a associação continue
em funcionamento. Contudo, a decisão judicial pode sus-
Pessoas podem ir às ruas para reunirem-se com de- pender atividades até que o trânsito em julgado ocorra, ou
mais na defesa de uma causa, apenas possuindo o dever seja, no curso de um processo judicial.
de informar tal reunião. Tal dever remonta-se a questões de Em destaque, a legitimidade representativa da associa-
segurança coletiva. Imagine uma grande reunião de pes- ção quanto aos seus filiados, conforme artigo 5º, XXI, CF:
soas por uma causa, a exemplo da Parada Gay, que chega
a aglomerar milhões de pessoas em algumas capitais: seria Artigo 5º, XXI, CF. As entidades associativas, quando ex-
absurdo tolerar tal tipo de reunião sem o prévio aviso do pressamente autorizadas, têm legitimidade para represen-
poder público para que ele organize o policiamento e a as- tar seus filiados judicial ou extrajudicialmente.

6
LEGISLAÇÃO

Trata-se de caso de legitimidade processual extraordi- Artigo 5º, XI, CF. A casa é asilo inviolável do indivíduo,
nária, pela qual um ente vai a juízo defender interesse de ninguém nela podendo penetrar sem consentimento do mo-
outra(s) pessoa(s) porque a lei assim autoriza. rador, salvo em caso de flagrante delito ou desastre, ou para
A liberdade de associação envolve não somente o di- prestar socorro, ou, durante o dia, por determinação judicial.
reito de criar associações e de fazer parte delas, mas tam-
bém o de não associar-se e o de deixar a associação, con- O domicílio é inviolável, razão pela qual ninguém pode
forme artigo 5º, XX, CF: nele entrar sem o consentimento do morador, a não ser
EM QUALQUER HORÁRIO no caso de flagrante delito (o
Artigo 5º, XX, CF. Ninguém poderá ser compelido a as- morador foi flagrado na prática de crime e fugiu para seu
sociar-se ou a permanecer associado. domicílio) ou desastre (incêndio, enchente, terremoto...) ou
para prestar socorro (morador teve ataque do coração, está
- Direitos à privacidade e à personalidade sufocado, desmaiado...), e SOMENTE DURANTE O DIA por
determinação judicial.
Abrangência Quanto ao sigilo de correspondência e das comunica-
Prevê o artigo 5º, X, CF: ções, prevê o artigo 5º, XII, CF:

Artigo 5º, X, CF. São invioláveis a intimidade, a vida Artigo 5º, XII, CF. É inviolável o sigilo da correspondência
privada, a honra e a imagem das pessoas, assegurado o e das comunicações telegráficas, de dados e das comunica-
direito a indenização pelo dano material ou moral decorrente ções telefônicas, salvo, no último caso, por ordem judicial,
de sua violação. nas hipóteses e na forma que a lei estabelecer para fins de
investigação criminal ou instrução processual penal.
O legislador opta por trazer correlacionados no mesmo
dispositivo legal os direitos à privacidade e à personalidade. O sigilo de correspondência e das comunicações está
Reforçando a conexão entre a privacidade e a intimida- melhor regulamentado na Lei nº 9.296, de 1996.
de, ao abordar a proteção da vida privada – que, em resu-
mo, é a privacidade da vida pessoal no âmbito do domicílio Personalidade jurídica e gratuidade de registro
e de círculos de amigos –, Silva7 entende que “o segredo Quando se fala em reconhecimento como pessoa pe-
da vida privada é condição de expansão da personalidade”, rante a lei desdobra-se uma esfera bastante específica dos
mas não caracteriza os direitos de personalidade em si. direitos de personalidade, consistente na personalidade ju-
A união da intimidade e da vida privada forma a pri- rídica. Basicamente, consiste no direito de ser reconhecido
vacidade, sendo que a primeira se localiza em esfera mais como pessoa perante a lei.
estrita. É possível ilustrar a vida social como se fosse um
Para ser visto como pessoa perante a lei mostra-se
grande círculo no qual há um menor, o da vida privada, e
necessário o registro. Por ser instrumento que serve como
dentro deste um ainda mais restrito e impenetrável, o da
pressuposto ao exercício de direitos fundamentais, asse-
intimidade. Com efeito, pela “Teoria das Esferas” (ou “Teoria
gura-se a sua gratuidade aos que não tiverem condição de
dos Círculos Concêntricos”), importada do direito alemão,
com ele arcar.
quanto mais próxima do indivíduo, maior a proteção a ser
Aborda o artigo 5º, LXXVI, CF:
conferida à esfera (as esferas são representadas pela intimi-
dade, pela vida privada, e pela publicidade).
“O direito à honra distancia-se levemente dos dois an- Artigo 5º, LXXVI, CF. São gratuitos para os reconheci-
teriores, podendo referir-se ao juízo positivo que a pessoa damente pobres, na forma da lei: a) o registro civil de nas-
tem de si (honra subjetiva) e ao juízo positivo que dela fa- cimento; b) a certidão de óbito.
zem os outros (honra objetiva), conferindo-lhe respeitabi-
lidade no meio social. O direito à imagem também possui O reconhecimento do marco inicial e do marco final
duas conotações, podendo ser entendido em sentido obje- da personalidade jurídica pelo registro é direito individual,
tivo, com relação à reprodução gráfica da pessoa, por meio não dependendo de condições financeiras. Evidente, seria
de fotografias, filmagens, desenhos, ou em sentido subje- absurdo cobrar de uma pessoa sem condições a elabora-
tivo, significando o conjunto de qualidades cultivadas pela ção de documentos para que ela seja reconhecida como
pessoa e reconhecidas como suas pelo grupo social”8. viva ou morta, o que apenas incentivaria a indigência dos
menos favorecidos.
Inviolabilidade de domicílio e sigilo de correspon-
dência Direito à indenização e direito de resposta
Correlatos ao direito à privacidade, aparecem a invio- Com vistas à proteção do direito à privacidade, do di-
labilidade do domicílio e o sigilo das correspondências e reito à personalidade e do direito à imagem, asseguram-se
comunicações. dois instrumentos, o direito à indenização e o direito de
Neste sentido, o artigo 5º, XI, CF prevê: resposta, conforme as necessidades do caso concreto.
Com efeito, prevê o artigo 5º, V, CF:
7 SILVA, José Afonso da. Curso de direito constitu-
cional positivo. 25. ed. São Paulo: Malheiros, 2006. Artigo 5º, V, CF. É assegurado o direito de resposta,
8 MOTTA, Sylvio; BARCHET, Gustavo. Curso de di- proporcional ao agravo, além da indenização por dano ma-
reito constitucional. Rio de Janeiro: Elsevier, 2007. terial, moral ou à imagem.

7
LEGISLAÇÃO

“A manifestação do pensamento é livre e garantida Nesta linha, para Silva11, “efetivamente, esse conjunto
em nível constitucional, não aludindo a censura prévia em de direitos aparelha situações, proibições, limitações e pro-
diversões e espetáculos públicos. Os abusos porventura cedimentos destinados a assegurar o exercício e o gozo de
ocorridos no exercício indevido da manifestação do pensa- algum direito individual fundamental (intimidade, liberda-
mento são passíveis de exame e apreciação pelo Poder Ju- de pessoal ou a incolumidade física ou moral)”.
diciário com a consequente responsabilidade civil e penal Especificamente no que tange à segurança jurídica,
de seus autores, decorrentes inclusive de publicações inju- tem-se o disposto no artigo 5º, XXXVI, CF:
riosas na imprensa, que deve exercer vigilância e controle
da matéria que divulga”9. Artigo 5º, XXXVI, CF. A lei não prejudicará o direito ad-
O  direito de resposta é o direito que uma pessoa quirido, o ato jurídico perfeito e a coisa julgada.
tem de se defender de críticas públicas no mesmo meio
em que foram publicadas garantida exatamente a mes- Pelo inciso restam estabelecidos limites à retroativida-
ma repercussão. Mesmo quando for garantido o direito de da lei.
de resposta não é possível reverter plenamente os da- Define o artigo 6º da Lei de Introdução às Normas do
nos causados pela manifestação ilícita de pensamento, Direito Brasileiro:
razão pela qual a pessoa inda fará jus à indenização.
A manifestação ilícita do pensamento geralmente cau- Artigo 6º, LINDB. A Lei em vigor terá efeito imediato e
sa um dano, ou seja, um prejuízo sofrido pelo agente, que geral, respeitados o ato jurídico perfeito, o direito adquirido
pode ser individual ou coletivo, moral ou material, econô- e a coisa julgada.
mico e não econômico. § 1º Reputa-se ato jurídico perfeito o já consumado
Dano material é aquele que atinge o patrimônio (ma- segundo a lei vigente ao tempo em que se efetuou.
terial ou imaterial) da vítima, podendo ser mensurado fi- § 2º Consideram-se adquiridos assim os direitos que o
nanceiramente e indenizado. seu titular, ou alguém por ele, possa exercer, como aqueles
“Dano moral direto consiste na lesão a um interesse cujo começo do exercício tenha termo pré-fixo, ou condi-
que visa a satisfação ou gozo de um bem jurídico extrapa- ção pré-estabelecida inalterável, a arbítrio de outrem.
trimonial contido nos direitos da personalidade (como a § 3º Chama-se coisa julgada ou caso julgado a decisão
vida, a integridade corporal, a liberdade, a honra, o decoro, judicial de que já não caiba recurso.
a intimidade, os sentimentos afetivos, a própria imagem)
- Direito à propriedade
ou nos atributos da pessoa (como o nome, a capacidade, o
O caput do artigo 5º da Constituição assegura a pro-
estado de família)”10.
teção do direito à propriedade, tanto material quanto inte-
Já o dano à imagem é delimitado no artigo 20 do Có-
lectual, delimitada em alguns incisos que o seguem.
digo Civil:
Função social da propriedade material
Artigo 20, CC. Salvo se autorizadas, ou se necessárias à
O artigo 5º, XXII, CF estabelece:
administração da justiça ou à manutenção da ordem públi-
ca, a divulgação de escritos, a transmissão da palavra, ou a Artigo 5º, XXII, CF. É garantido o direito de proprie-
publicação, a exposição ou a utilização da imagem de uma dade.
pessoa poderão ser proibidas, a seu requerimento e sem pre-
juízo da indenização que couber, se lhe atingirem a honra, A seguir, no inciso XXIII do artigo 5º, CF estabelece o
a boa fama ou a respeitabilidade, ou se se destinarem a fins principal fator limitador deste direito:
comerciais.
Artigo 5º, XXIII, CF. A propriedade atenderá a sua fun-
- Direito à segurança ção social.
O caput do artigo 5º da Constituição assegura a pro-
teção do direito à segurança. Na qualidade de direito in- A propriedade, segundo Silva12, “[...] não pode mais ser
dividual liga-se à segurança do indivíduo como um todo, considerada como um direito individual nem como institui-
desde sua integridade física e mental, até a própria segu- ção do direito privado. [...] embora prevista entre os direi-
rança jurídica. tos individuais, ela não mais poderá ser considerada puro
No sentido aqui estudado, o direito à segurança pes- direito individual, relativizando-se seu conceito e significa-
soal é o direito de viver sem medo, protegido pela soli- do, especialmente porque os princípios da ordem econô-
dariedade e liberto de agressões, logo, é uma maneira de mica são preordenados à vista da realização de seu fim:
garantir o direito à vida. assegurar a todos existência digna, conforme os ditames
da justiça social. Se é assim, então a propriedade privada,
que, ademais, tem que atender a sua função social, fica vin-
culada à consecução daquele princípio”.
9 BONAVIDES, Paulo. Curso de direito constitucio- 11 SILVA, José Afonso da. Curso de direito constitu-
nal. 26. ed. São Paulo: Malheiros, 2011. cional positivo... Op. Cit., p. 437.
10 ZANNONI, Eduardo. El daño en la responsabili- 12 SILVA, José Afonso da. Curso de direito constitu-
dad civil. Buenos Aires: Astrea, 1982. cional positivo. 25. ed. São Paulo: Malheiros, 2006.

8
LEGISLAÇÃO

Com efeito, a proteção da propriedade privada está li- III - desapropriação com pagamento mediante títulos
mitada ao atendimento de sua função social, sendo este o da dívida pública de emissão previamente aprovada pelo
requisito que a correlaciona com a proteção da dignidade Senado Federal, com prazo de resgate de até dez anos, em
da pessoa humana. A propriedade de bens e valores em parcelas anuais, iguais e sucessivas, assegurados o valor real
geral é um direito assegurado na Constituição Federal e, da indenização e os juros legais14.
como todos os outros, se encontra limitado pelos demais
princípios conforme melhor se atenda à dignidade do ser Artigo 184, CF. Compete à União desapropriar por in-
humano. teresse social, para fins de reforma agrária, o imóvel rural
A Constituição Federal delimita o que se entende por que não esteja cumprindo sua função social, mediante
função social: prévia e justa indenização em títulos da dívida agrária,
com cláusula de preservação do valor real, resgatáveis no
Art. 182, caput, CF. A política de desenvolvimento urba- prazo de até vinte anos, a partir do segundo ano de sua
no, executada pelo Poder Público municipal, conforme dire- emissão, e cuja utilização será definida em lei15.
trizes gerais fixadas em lei, tem por objetivo ordenar o pleno
desenvolvimento das funções sociais da cidade e garantir o Artigo 184, § 1º, CF. As benfeitorias úteis e necessárias
bem-estar de seus habitantes. serão indenizadas em dinheiro.

Artigo 182, § 1º, CF. O plano diretor, aprovado pela Câ- No que tange à desapropriação por necessidade ou
mara Municipal, obrigatório para cidades com mais de vinte utilidade pública, prevê o artigo 5º, XXIV, CF:
mil habitantes, é o instrumento básico da política de desen-
volvimento e de expansão urbana. Artigo 5º, XXIV, CF. A lei estabelecerá o procedimento
para desapropriação por necessidade ou utilidade pública,
Artigo 182, § 2º, CF. A propriedade urbana cumpre sua ou por interesse social, mediante justa e prévia indenização
função social quando atende às exigências fundamentais de em dinheiro, ressalvados os casos previstos nesta Constitui-
ordenação da cidade expressas no plano diretor13. ção.
Artigo 186, CF. A função social é cumprida quando a
Ainda, prevê o artigo 182, § 3º, CF:
propriedade rural atende, simultaneamente, segundo crité-
rios e graus de exigência estabelecidos em lei, aos seguintes
Artigo 182, §3º, CF. As desapropriações de imóveis urba-
requisitos:
nos serão feitas com prévia e justa indenização em dinheiro.
I - aproveitamento racional e adequado;
II - utilização adequada dos recursos naturais disponí-
Tem-se, ainda o artigo 184, §§ 2º e 3º, CF:
veis e preservação do meio ambiente;
III - observância das disposições que regulam as rela-
ções de trabalho; Artigo 184, §2º, CF. O decreto que declarar o imóvel
IV - exploração que favoreça o bem-estar dos proprie- como de interesse social, para fins de reforma agrária, auto-
tários e dos trabalhadores. riza a União a propor a ação de desapropriação.

Desapropriação Artigo 184, §3º, CF. Cabe à lei complementar estabelecer


No caso de desrespeito à função social da proprieda- procedimento contraditório especial, de rito sumário, para o
de cabe até mesmo desapropriação do bem, de modo que processo judicial de desapropriação.
pode-se depreender do texto constitucional duas possibi-
lidades de desapropriação: por desrespeito à função social
e por necessidade ou utilidade pública.
A Constituição Federal prevê a possibilidade de desa-
propriação por desatendimento à função social: 14 Nota-se que antes de se promover a desapropria-
ção de imóvel urbano por desatendimento à função social é
Artigo 182, § 4º, CF. É facultado ao Poder Público mu- necessário tomar duas providências, sucessivas: primeiro, o
nicipal, mediante lei específica para área incluída no plano parcelamento ou edificação compulsórios; depois, o estabe-
diretor, exigir, nos termos da lei federal, do proprietário do lecimento de imposto sobre a propriedade predial e territorial
solo urbano não edificado, subutilizado ou não utiliza- urbana progressivo no tempo. Se ambas medidas restarem
do, que promova seu adequado aproveitamento, sob pena, ineficazes, parte-se para a desapropriação por desatendimen-
sucessivamente, de: to à função social.
I - parcelamento ou edificação compulsórios; 15 A desapropriação em decorrência do desatendimen-
II - imposto sobre a propriedade predial e territorial ur- to da função social é indenizada, mas não da mesma maneira
bana progressivo no tempo; que a desapropriação por necessidade ou utilidade pública,
13 Instrumento básico de um processo de planejamen- já que na primeira há violação do ordenamento constitucional
to municipal para a implantação da política de desenvolvimen- pelo proprietário, mas na segunda não. Por isso, indeniza-se
to urbano, norteando a ação dos agentes públicos e privados em títulos da dívida agrária, que na prática não são tão valori-
(Lei n. 10.257/2001 - Estatuto da cidade). zados quanto o dinheiro.

9
LEGISLAÇÃO

A desapropriação por utilidade ou necessidade pública Artigo 5º, XXVI, CF. A pequena propriedade rural, as-
deve se dar mediante prévia e justa indenização em dinhei- sim definida em lei, desde que trabalhada pela família, não
ro. O Decreto-lei nº 3.365/1941 a disciplina, delimitando será objeto de penhora para pagamento de débitos decor-
o procedimento e conceituando utilidade pública, em seu rentes de sua atividade produtiva, dispondo a lei sobre os
artigo 5º: meios de financiar o seu desenvolvimento.

Artigo 5º, Decreto-lei n. 3.365/1941. Consideram-se ca- Assim, se uma pessoa é mais humilde e tem uma pe-
sos de utilidade pública: quena propriedade será assegurado que permaneça com
a) a segurança nacional; ela e a torne mais produtiva.
b) a defesa do Estado; A preservação da pequena propriedade em detrimento
c) o socorro público em caso de calamidade; dos grandes latifúndios improdutivos é uma das diretrizes-
d) a salubridade pública; guias da regulamentação da política agrária brasileira, que
e) a criação e melhoramento de centros de população, tem como principal escopo a realização da reforma agrária.
seu abastecimento regular de meios de subsistência; Parte da questão financeira atinente à reforma agrária
f) o aproveitamento industrial das minas e das jazidas se encontra prevista no artigo 184, §§ 4º e 5º, CF:
minerais, das águas e da energia hidráulica;
g) a assistência pública, as obras de higiene e decoração, Artigo 184, §4º, CF. O orçamento fixará anualmente
casas de saúde, clínicas, estações de clima e fontes medici- o volume total de títulos da dívida agrária, assim como o
nais; montante de recursos para atender ao programa de reforma
h) a exploração ou a conservação dos serviços públicos; agrária no exercício.
i) a abertura, conservação e melhoramento de vias ou
logradouros públicos; a execução de planos de urbanização; Artigo 184, §5º, CF. São isentas de impostos federais, es-
o parcelamento do solo, com ou sem edificação, para sua taduais e municipais as operações de transferência de imó-
melhor utilização econômica, higiênica ou estética; a cons- veis desapropriados para fins de reforma agrária.
trução ou ampliação de distritos industriais;
j) o funcionamento dos meios de transporte coletivo; Como a finalidade da reforma agrária é transformar
k) a preservação e conservação dos monumentos históri-
terras improdutivas e grandes propriedades em atinentes à
cos e artísticos, isolados ou integrados em conjuntos urbanos
função social, alguns imóveis rurais não podem ser abran-
ou rurais, bem como as medidas necessárias a manter-lhes
gidos pela reforma agrária:
e realçar-lhes os aspectos mais valiosos ou característicos e,
ainda, a proteção de paisagens e locais particularmente do-
Art. 185, CF. São insuscetíveis de desapropriação para
tados pela natureza;
fins de reforma agrária:
l) a preservação e a conservação adequada de arquivos,
documentos e outros bens moveis de valor histórico ou ar- I - a pequena e média propriedade rural, assim definida
tístico; em lei, desde que seu proprietário não possua outra;
m) a construção de edifícios públicos, monumentos co- II - a propriedade produtiva.
memorativos e cemitérios; Parágrafo único. A lei garantirá tratamento especial à
n) a criação de estádios, aeródromos ou campos de pou- propriedade produtiva e fixará normas para o cumprimento
so para aeronaves; dos requisitos relativos a sua função social.
o) a reedição ou divulgação de obra ou invento de natu-
reza científica, artística ou literária; Sobre as diretrizes da política agrícola, prevê o artigo
p) os demais casos previstos por leis especiais. 187:

Um grande problema que faz com que processos que Art. 187, CF. A política agrícola será planejada e exe-
tenham a desapropriação por objeto se estendam é a in- cutada na forma da lei, com a participação efetiva do setor
devida valorização do imóvel pelo Poder Público, que ge- de produção, envolvendo produtores e trabalhadores rurais,
ralmente pretende pagar valor muito abaixo do devido, bem como dos setores de comercialização, de armazena-
necessitando o Judiciário intervir em prol da correta ava- mento e de transportes, levando em conta, especialmente:
liação. I - os instrumentos creditícios e fiscais;
Outra questão reside na chamada tredestinação, pela II - os preços compatíveis com os custos de produção e a
qual há a destinação de um bem expropriado (desapro- garantia de comercialização;
priação) a finalidade diversa da que se planejou inicial- III - o incentivo à pesquisa e à tecnologia;
mente. A tredestinação pode ser lícita ou ilícita. Será ilícita IV - a assistência técnica e extensão rural;
quando resultante de desvio do propósito original; e será V - o seguro agrícola;
lícita quando a Administração Pública dê ao bem finalidade VI - o cooperativismo;
diversa, porém preservando a razão do interesse público. VII - a eletrificação rural e irrigação;
VIII - a habitação para o trabalhador rural.
Política agrária e reforma agrária § 1º Incluem-se no planejamento agrícola as atividades
Enquanto desdobramento do direito à propriedade agroindustriais, agropecuárias, pesqueiras e florestais.
imóvel e da função social desta propriedade, tem-se ainda § 2º Serão compatibilizadas as ações de política agríco-
o artigo 5º, XXVI, CF: la e de reforma agrária.

10
LEGISLAÇÃO

As terras devolutas e públicas serão destinadas confor- b) Imóveis até 250 m² – Pode dentro de uma posse
me a política agrícola e o plano nacional de reforma agrá- maior isolar área de 250m² e ingressar com a ação? A juris-
ria (artigo 188, caput, CF). Neste sentido, “a alienação ou a prudência é pacífica que a posse desde o início deve ficar
concessão, a qualquer título, de terras públicas com área restrita a 250m². Predomina também que o terreno deve
superior a dois mil e quinhentos hectares a pessoa física ter 250m², não a área construída (a área de um sobrado,
ou jurídica, ainda que por interposta pessoa, dependerá de por exemplo, pode ser maior que a de um terreno).
prévia aprovação do Congresso Nacional”, salvo no caso c) 5 anos – houve controvérsia porque a Constituição
de alienações ou concessões de terras públicas para fins de Federal de 1988 que criou esta modalidade. E se antes
reforma agrária (artigo 188, §§ 1º e 2º, CF). de 05 de outubro de 1988 uma pessoa tivesse há 4 anos
Os que forem favorecidos pela reforma agrária (ho- dentro do limite da usucapião urbana? Predominou que
mens, mulheres, ambos, qualquer estado civil) não pode- só corria o prazo a partir da criação do instituto, não só
rão negociar seus títulos pelo prazo de 10 anos (artigo 189, porque antes não existia e o prazo não podia correr, como
CF). também não se poderia prejudicar o proprietário.
Consta, ainda, que “a lei regulará e limitará a aquisição d) Moradia sua ou de sua família – não basta ter posse,
ou o arrendamento de propriedade rural por pessoa física é preciso que a pessoa more, sozinha ou com sua família,
ou jurídica estrangeira e estabelecerá os casos que depen- ao longo de todo o prazo (não só no início ou no final).
derão de autorização do Congresso Nacional” (artigo 190, Logo, não cabe acessio temporis por cessão da posse.
CF). e) Nenhum outro imóvel, nem urbano, nem rural, no
Brasil. O usucapiente não prova isso, apenas alega. Se al-
Usucapião guém não quiser a usucapião, prova o contrário. Este re-
Usucapião é o modo originário de aquisição da pro- quisito é verificado no momento em que completa 5 anos.
priedade que decorre da posse prolongada por um lon- Em relação à previsão da usucapião especial rural, des-
go tempo, preenchidos outros requisitos legais. Em outras taca-se o artigo 191, CF:
palavras, usucapião é uma situação em que alguém tem a
posse de um bem por um tempo longo, sem ser incomo- Art. 191, CF. Aquele que, não sendo proprietário de imó-
dado, a ponto de se tornar proprietário. vel rural ou urbano, possua como seu, por cinco anos inin-
A Constituição regulamenta o acesso à propriedade terruptos, sem oposição, área de terra, em zona rural, não
mediante posse prolongada no tempo – usucapião – em superior a cinquenta hectares, tornando-a produtiva por seu
casos específicos, denominados usucapião especial urbana trabalho ou de sua família, tendo nela sua moradia, adqui-
e usucapião especial rural. rir-lhe-á a propriedade.
O artigo 183 da Constituição regulamenta a usucapião Parágrafo único. Os imóveis públicos não serão adquiri-
especial urbana: dos por usucapião.

Art. 183, CF. Aquele que possuir como sua área urbana Além dos requisitos gerais (animus e posse que seja
de até duzentos e cinquenta metros quadrados, por cinco pública, pacífica, ininterrupta e contínua), são exigidos os
anos, ininterruptamente e sem oposição, utilizando-a para seguintes requisitos específicos:
sua moradia ou de sua família, adquirir-lhe-á o domínio, a) Imóvel rural
desde que não seja proprietário de outro imóvel urbano ou b) 50 hectares, no máximo – há também legislação que
rural. estabelece um limite mínimo, o módulo rural (Estatuto da
§ 1º O título de domínio e a concessão de uso serão Terra). É possível usucapir áreas menores que o módulo ru-
conferidos ao homem ou à mulher, ou a ambos, indepen- ral? Tem prevalecido o entendimento de que pode, mas é
dentemente do estado civil. assunto muito controverso.
§ 2º Esse direito não será reconhecido ao mesmo pos- c) 5 anos – pode ser considerado o prazo antes 05 de
suidor mais de uma vez. outubro de 1988 (Constituição Federal)? Depende. Se a
§ 3º Os imóveis públicos não serão adquiridos por usu- área é de até 25 hectares sim, pois já havia tal possibilidade
capião. antes da CF/88. Se área for maior (entre 25 ha e 50 ha) não.
d) Moradia sua ou de sua família – a pessoa deve morar
Além dos requisitos gerais (animus e posse que seja na área rural.
pública, pacífica, ininterrupta e contínua), são exigidos os e) Nenhum outro imóvel.
seguintes requisitos específicos: f) O usucapiente, com seu trabalho, deve ter tornado
a) Área urbana – há controvérsia. Pela teoria da locali- a área produtiva. Por isso, é chamado de usucapião “pro
zação, área urbana é a que está dentro do perímetro urba- labore”. Dependerá do caso concreto.
no. Pela teoria da destinação, mais importante que a locali-
zação é a sua utilização. Ex.: se tem fins agrícolas/pecuários Uso temporário
e estiver dentro do perímetro urbana, o imóvel é rural. Para No mais, estabelece-se uma terceira limitação ao di-
fins de usucapião a maioria diz que prevalece a teoria da reito de propriedade que não possui o caráter definitivo
localização. da desapropriação, mas é temporária, conforme artigo 5º,
XXV, CF:

11
LEGISLAÇÃO

Artigo 5º, XXV, CF. No caso de iminente perigo públi- A elaboração do Código de Defesa do Consumidor foi
co, a autoridade competente poderá usar de propriedade um grande passo para a proteção da pessoa nas relações
particular, assegurada ao proprietário indenização ulterior, de consumo que estabeleça, respeitando-se a condição de
se houver dano. hipossuficiente técnico daquele que adquire um bem ou
faz uso de determinado serviço, enquanto consumidor.
Se uma pessoa tem uma propriedade, numa situação
de perigo, o poder público pode se utilizar dela (ex: montar Propriedade intelectual
uma base para capturar um fugitivo), pois o interesse da Além da propriedade material, o constituinte protege
coletividade é maior que o do indivíduo proprietário. também a propriedade intelectual, notadamente no artigo
5º, XXVII, XXVIII e XXIX, CF:
Direito sucessório
O direito sucessório aparece como uma faceta do di- Artigo 5º, XXVII, CF. Aos autores pertence o direito ex-
reito à propriedade, encontrando disciplina constitucional clusivo de utilização, publicação ou reprodução de suas
no artigo 5º, XXX e XXXI, CF: obras, transmissível aos herdeiros pelo tempo que a lei fixar;

Artigo 5º, XXVIII, CF. São assegurados, nos termos da lei:


Artigo 5º, XXX, CF. É garantido o direito de herança;
a) a proteção às participações individuais em obras
coletivas e à reprodução da imagem e voz humanas, in-
Artigo 5º, XXXI, CF. A sucessão de bens de estrangei- clusive nas atividades desportivas;
ros situados no País será regulada pela lei brasileira em be- b) o direito de fiscalização do aproveitamento eco-
nefício do cônjuge ou dos filhos brasileiros, sempre que não nômico das obras que criarem ou de que participarem aos
lhes seja mais favorável a lei pessoal do de cujus. criadores, aos intérpretes e às respectivas representações
sindicais e associativas;
O direito à herança envolve o direito de receber – seja
devido a uma previsão legal, seja por testamento – bens Artigo 5º, XXIX, CF. A lei assegurará aos autores de in-
de uma pessoa que faleceu. Assim, o patrimônio passa ventos industriais privilégio temporário para sua utiliza-
para outra pessoa, conforme a vontade do falecido e/ou ção, bem como proteção às criações industriais, à proprie-
a lei determine. A Constituição estabelece uma disciplina dade das marcas, aos nomes de empresas e a outros signos
específica para bens de estrangeiros situados no Brasil, as- distintivos, tendo em vista o interesse social e o desenvolvi-
segurando que eles sejam repassados ao cônjuge e filhos mento tecnológico e econômico do País.
brasileiros nos termos da lei mais benéfica (do Brasil ou do
país estrangeiro). Assim, a propriedade possui uma vertente intelectual
que deve ser respeitada, tanto sob o aspecto moral quanto
Direito do consumidor sob o patrimonial. No âmbito infraconstitucional brasileiro,
Nos termos do artigo 5º, XXXII, CF: a Lei nº 9.610, de 19 de fevereiro de 1998, regulamenta os
direitos autorais, isto é, “os direitos de autor e os que lhes
Artigo 5º, XXXII, CF. O Estado promoverá, na forma da são conexos”.
lei, a defesa do consumidor. O artigo 7° do referido diploma considera como obras
intelectuais que merecem a proteção do direito do autor
O direito do consumidor liga-se ao direito à proprieda- os textos de obras de natureza literária, artística ou científi-
de a partir do momento em que garante à pessoa que irá ca; as conferências, sermões e obras semelhantes; as obras
adquirir bens e serviços que estes sejam entregues e pres- cinematográficas e televisivas; as composições musicais;
fotografias; ilustrações; programas de computador; coletâ-
tados da forma adequada, impedindo que o fornecedor se
neas e enciclopédias; entre outras.
enriqueça ilicitamente, se aproveite de maneira indevida da
Os direitos morais do autor, que são imprescritíveis,
posição menos favorável e de vulnerabilidade técnica do
inalienáveis e irrenunciáveis, envolvem, basicamente, o di-
consumidor. reito de reivindicar a autoria da obra, ter seu nome divul-
O Direito do Consumidor pode ser considerado um gado na utilização desta, assegurar a integridade desta ou
ramo recente do Direito. No Brasil, a legislação que o re- modificá-la e retirá-la de circulação se esta passar a afron-
gulamentou foi promulgada nos anos 90, qual seja a Lei nº tar sua honra ou imagem.
8.078, de 11 de setembro de 1990, conforme determinado Já os direitos patrimoniais do autor, nos termos dos
pela Constituição Federal de 1988, que também estabele- artigos 41 a 44 da Lei nº 9.610/98, prescrevem em 70 anos
ceu no artigo 48 do Ato das Disposições Constitucionais contados do primeiro ano seguinte à sua morte ou do
Transitórias: falecimento do último coautor, ou contados do primeiro
ano seguinte à divulgação da obra se esta for de natureza
Artigo 48, ADCT. O Congresso Nacional, dentro de cento audiovisual ou fotográfica. Estes, por sua vez, abrangem,
e vinte dias da promulgação da Constituição, elaborará có- basicamente, o direito de dispor sobre a reprodução, edi-
digo de defesa do consumidor. ção, adaptação, tradução, utilização, inclusão em bases de
dados ou qualquer outra modalidade de utilização; sendo
que estas modalidades de utilização podem se dar a título
oneroso ou gratuito.

12
LEGISLAÇÃO

“Os direitos autorais, também conhecidos como co- Assim, dentro da noção de acesso à justiça, diversos
pyright (direito de cópia), são considerados bens móveis, aspectos podem ser destacados: de um lado, deve criar-se
podendo ser alienados, doados, cedidos ou locados. Res- o Poder Judiciário e se disponibilizar meios para que todas
salte-se que a permissão a terceiros de utilização de cria- as pessoas possam buscá-lo; de outro lado, não basta ga-
ções artísticas é direito do autor. [...] A proteção consti- rantir meios de acesso se estes forem insuficientes, já que
tucional abrange o plágio e a contrafação. Enquanto que para que exista o verdadeiro acesso à justiça é necessário
o primeiro caracteriza-se pela difusão de obra criada ou que se aplique o direito material de maneira justa e célere.
produzida por terceiros, como se fosse própria, a segunda Relacionando-se à primeira onda de acesso à justiça,
configura a reprodução de obra alheia sem a necessária prevê a Constituição em seu artigo 5º, XXXV:
permissão do autor”16.
Artigo 5º, XXXV, CF. A lei não excluirá da apreciação do
- Direitos de acesso à justiça Poder Judiciário lesão ou ameaça a direito.
A formação de um conceito sistemático de acesso à
justiça se dá com a teoria de Cappelletti e Garth, que apon- O princípio da inafastabilidade da jurisdição é o prin-
taram três ondas de acesso, isto é, três posicionamentos cípio de Direito Processual Público subjetivo, também
básicos para a realização efetiva de tal acesso. Tais ondas cunhado como Princípio da Ação, em que a Constituição
foram percebidas paulatinamente com a evolução do Di- garante a necessária tutela estatal aos conflitos ocorrentes
reito moderno conforme implementadas as bases da onda na vida em sociedade. Sempre que uma controvérsia for
anterior, quer dizer, ficou evidente aos autores a emergên- levada ao Poder Judiciário, preenchidos os requisitos de
cia de uma nova onda quando superada a afirmação das admissibilidade, ela será resolvida, independentemente de
premissas da onda anterior, restando parcialmente imple- haver ou não previsão específica a respeito na legislação.
mentada (visto que até hoje enfrentam-se obstáculos ao Também se liga à primeira onda de acesso à justiça,
pleno atendimento em todas as ondas). no que tange à abertura do Judiciário mesmo aos menos
Primeiro, Cappelletti e Garth17 entendem que surgiu favorecidos economicamente, o artigo 5º, LXXIV, CF:
uma onda de concessão de assistência judiciária aos po-
bres, partindo-se da prestação sem interesse de remunera- Artigo 5º, LXXIV, CF. O Estado prestará assistência jurí-
ção por parte dos advogados e, ao final, levando à criação dica integral e gratuita aos que comprovarem insuficiên-
cia de recursos.
de um aparato estrutural para a prestação da assistência
pelo Estado.
O constituinte, ciente de que não basta garantir o aces-
Em segundo lugar, no entender de Cappelletti e Garth18,
so ao Poder Judiciário, sendo também necessária a efeti-
veio a onda de superação do problema na representação
vidade processual, incluiu pela Emenda Constitucional nº
dos interesses difusos, saindo da concepção tradicional de
45/2004 o inciso LXXVIII ao artigo 5º da Constituição:
processo como algo restrito a apenas duas partes indivi-
dualizadas e ocasionando o surgimento de novas institui-
Artigo 5º, LXXVIII, CF. A todos, no âmbito judicial e ad-
ções, como o Ministério Público. ministrativo, são assegurados a razoável duração do pro-
Finalmente, Cappelletti e Garth19 apontam uma terceira cesso e os meios que garantam a celeridade de sua trami-
onda consistente no surgimento de uma concepção mais tação.
ampla de acesso à justiça, considerando o conjunto de ins-  
tituições, mecanismos, pessoas e procedimentos utilizados: Com o tempo se percebeu que não bastava garantir
“[...] esse enfoque encoraja a exploração de uma ampla va- o acesso à justiça se este não fosse célere e eficaz. Não
riedade de reformas, incluindo alterações nas formas de significa que se deve acelerar o processo em detrimento
procedimento, mudanças na estrutura dos tribunais ou a de direitos e garantias assegurados em lei, mas sim que é
criação de novos tribunais, o uso de pessoas leigas ou pa- preciso proporcionar um trâmite que dure nem mais e nem
raprofissionais, tanto como juízes quanto como defensores, menos que o necessário para a efetiva realização da justiça
modificações no direito substantivo destinadas a evitar li- no caso concreto.
tígios ou facilitar sua solução e a utilização de mecanismos
privados ou informais de solução dos litígios. Esse enfoque, - Direitos constitucionais-penais
em suma, não receia inovações radicais e compreensivas,
que vão muito além da esfera de representação judicial”. Juiz natural e vedação ao juízo ou tribunal de ex-
ceção
16 MORAES, Alexandre de. Direitos humanos fun- Quando o artigo 5º, LIII, CF menciona:
damentais: teoria geral, comentários aos artigos 1º a 5º da
Constituição da República Federativa do Brasil, doutrina e ju- Artigo 5º, LIII, CF. Ninguém será processado nem sen-
risprudência. São Paulo: Atlas, 1997. tenciado senão pela autoridade competente”, consolida o
17 CAPPELLETTI, Mauro; GARTH, Bryant. Acesso à princípio do juiz natural que assegura a toda pessoa o direito
Justiça. Tradução Ellen Grace Northfleet. Porto Alegre: Sér- de conhecer previamente daquele que a julgará no processo
gio Antônio Fabris Editor, 1998, p. 31-32. em que seja parte, revestindo tal juízo em jurisdição com-
18 Ibid., p. 49-52 petente para a matéria específica do caso antes mesmo do
19 Ibid., p. 67-73 fato ocorrer.

13
LEGISLAÇÃO

Por sua vez, um desdobramento deste princípio encon- Ainda no que tange ao princípio da anterioridade, tem-
tra-se no artigo 5º, XXXVII, CF: se o artigo 5º, XL, CF:

Artigo 5º, XXXVII, CF. Não haverá juízo ou tribunal de Artigo 5º, XL, CF. A lei penal não retroagirá, salvo para
exceção. beneficiar o réu.

Juízo ou Tribunal de Exceção é aquele especialmente O dispositivo consolida outra faceta do princípio da
criado para uma situação pretérita, bem como não reco- anterioridade: se, por um lado, é necessário que a lei tenha
nhecido como legítimo pela Constituição do país. definido um fato como crime e dado certo tratamento pe-
nal a este fato (ex.: pena de detenção ou reclusão, tempo
Tribunal do júri de pena, etc.) antes que ele ocorra; por outro lado, se vier
A respeito da competência do Tribunal do júri, prevê o uma lei posterior ao fato que o exclua do rol de crimes ou
artigo 5º, XXXVIII, CF:
que confira tratamento mais benéfico (diminuindo a pena
ou alterando o regime de cumprimento, notadamente), ela
Artigo 5º, XXXVIII. É reconhecida a instituição do júri,
será aplicada. Restam consagrados tanto o princípio da ir-
com a organização que lhe der a lei, assegurados:
a) a plenitude de defesa; retroatividade da lei penal in pejus quanto o da retroativi-
b) o sigilo das votações; dade da lei penal mais benéfica.
c) a soberania dos veredictos;
d) a competência para o julgamento dos crimes dolosos Menções específicas a crimes
contra a vida. O artigo 5º, XLI, CF estabelece:

O Tribunal do Júri é formado por pessoas do povo, que Artigo 5º, XLI, CF. A lei punirá qualquer discriminação
julgam os seus pares. Entende-se ser direito fundamental o atentatória dos direitos e liberdades fundamentais.
de ser julgado por seus iguais, membros da sociedade e não
magistrados, no caso de determinados crimes que por sua Sendo assim confere fórmula genérica que remete ao
natureza possuem fortes fatores de influência emocional. princípio da igualdade numa concepção ampla, razão pela
Plenitude da defesa envolve tanto a autodefesa quanto qual práticas discriminatórias não podem ser aceitas. No
a defesa técnica e deve ser mais ampla que a denominada entanto, o constituinte entendeu por bem prever trata-
ampla defesa assegurada em todos os procedimentos judi- mento específico a certas práticas criminosas.
ciais e administrativos. Neste sentido, prevê o artigo 5º, XLII, CF:
Sigilo das votações envolve a realização de votações
secretas, preservando a liberdade de voto dos que com- Artigo 5º, XLII, CF. A prática do racismo constitui crime
põem o conselho que irá julgar o ato praticado. inafiançável e imprescritível, sujeito à pena de reclusão,
A decisão tomada pelo conselho é soberana. Contudo, nos termos da lei.
a soberania dos veredictos veda a alteração das decisões
dos jurados, não a recorribilidade dos julgamentos do Tri- A Lei nº 7.716, de 5 de janeiro de 1989 define os crimes
bunal do Júri para que seja procedido novo julgamento resultantes de preconceito de raça ou de cor. Contra eles
uma vez cassada a decisão recorrida, haja vista preservar não cabe fiança (pagamento de valor para deixar a prisão
o ordenamento jurídico pelo princípio do duplo grau de provisória) e não se aplica o instituto da prescrição (perda
jurisdição.
de pretensão de se processar/punir uma pessoa pelo de-
Por fim, a competência para julgamento é dos crimes
curso do tempo).
dolosos (em que há intenção ou ao menos se assume o
Não obstante, preconiza ao artigo 5º, XLIII, CF:
risco de produção do resultado) contra a vida, que são: ho-
micídio, aborto, induzimento, instigação ou auxílio a sui-
cídio e infanticídio. Sua competência não é absoluta e é Artigo 5º, XLIII, CF. A lei considerará crimes inafian-
mitigada, por vezes, pela própria Constituição (artigos 29, çáveis e insuscetíveis de graça ou anistia a prática da
X / 102, I, b) e c) / 105, I, a) / 108, I). tortura, o tráfico ilícito de entorpecentes e drogas afins, o
terrorismo e os definidos como crimes hediondos, por eles
Anterioridade e irretroatividade da lei respondendo os mandantes, os executores e os que, podendo
O artigo 5º, XXXIX, CF preconiza: evitá-los, se omitirem.

Artigo5º, XXXIX, CF. Não há crime sem lei anterior que Anistia, graça e indulto diferenciam-se nos seguintes
o defina, nem pena sem prévia cominação legal. termos: a anistia exclui o crime, rescinde a condenação e
extingue totalmente a punibilidade, a graça e o indulto
É a consagração da regra do nullum crimen nulla poena apenas extinguem a punibilidade, podendo ser parciais; a
sine praevia lege. Simultaneamente, se assegura o princípio anistia, em regra, atinge crimes políticos, a graça e o in-
da legalidade (ou reserva legal), na medida em que não há dulto, crimes comuns; a anistia pode ser concedida pelo
crime sem lei que o defina, nem pena sem prévia comina- Poder Legislativo, a graça e o indulto são de competência
ção legal, e o princípio da anterioridade, posto que não há exclusiva do Presidente da República; a anistia pode ser
crime sem lei anterior que o defina. concedida antes da sentença final ou depois da condena-

14
LEGISLAÇÃO

ção irrecorrível, a graça e o indulto pressupõem o trânsito A pena de multa ou patrimonial opera uma diminuição
em julgado da sentença condenatória; graça e o indulto do patrimônio do indivíduo delituoso.
apenas extinguem a punibilidade, persistindo os efeitos do A prestação social alternativa corresponde às penas
crime, apagados na anistia; graça é em regra individual e restritivas de direitos, autônomas e substitutivas das penas
solicitada, enquanto o indulto é coletivo e espontâneo. privativas de liberdade, estabelecidas no artigo 44 do Có-
Não cabe graça, anistia ou indulto (pode-se considerar digo Penal.
que o artigo o abrange, pela doutrina majoritária) contra Por seu turno, a individualização da pena deve também
crimes de tortura, tráfico, terrorismo (TTT) e hediondos se fazer presente na fase de sua execução, conforme se
(previstos na Lei nº 8.072 de 25 de julho de 1990). Além depreende do artigo 5º, XLVIII, CF:
disso, são crimes que não aceitam fiança.
Por fim, prevê o artigo 5º, XLIV, CF: Artigo 5º, XLVIII, CF. A pena será cumprida em estabe-
lecimentos distintos, de acordo com a natureza do delito,
Artigo 5º, XLIV, CF. Constitui crime inafiançável e im- a idade e o sexo do apenado.
prescritível a ação de grupos armados, civis ou militares,
contra a ordem constitucional e o Estado Democrático. A distinção do estabelecimento conforme a natureza
do delito visa impedir que a prisão se torne uma faculdade
Personalidade da pena do crime. Infelizmente, o Estado não possui aparato sufi-
A personalidade da pena encontra respaldo no artigo ciente para cumprir tal diretiva, diferenciando, no máximo,
5º, XLV, CF: o nível de segurança das prisões. Quanto à idade, desta-
cam-se as Fundações Casas, para cumprimento de medida
Artigo 5º, XLV, CF. Nenhuma pena passará da pessoa por menores infratores. Quanto ao sexo, prisões costumam
do condenado, podendo a obrigação de reparar o dano e a ser exclusivamente para homens ou para mulheres.
decretação do perdimento de bens ser, nos termos da lei, es- Também se denota o respeito à individualização da
tendidas aos sucessores e contra eles executadas, até o limite pena nesta faceta pelo artigo 5º, L, CF:
do valor do patrimônio transferido.
Artigo 5º, L, CF. Às presidiárias serão asseguradas con-
O princípio da personalidade encerra o comando de o
dições para que possam permanecer com seus filhos duran-
crime ser imputado somente ao seu autor, que é, por seu
te o período de amamentação.
turno, a única pessoa passível de sofrer a sanção. Seria fla-
grante a injustiça se fosse possível alguém responder pelos
Preserva-se a individualização da pena porque é toma-
atos ilícitos de outrem: caso contrário, a reação, ao invés de
da a condição peculiar da presa que possui filho no perío-
restringir-se ao malfeitor, alcançaria inocentes. Contudo, se
do de amamentação, mas também se preserva a dignidade
uma pessoa deixou patrimônio e faleceu, este patrimônio
da criança, não a afastando do seio materno de maneira
responderá pelas repercussões financeiras do ilícito.
precária e impedindo a formação de vínculo pela amamen-
Individualização da pena tação.
A individualização da pena tem por finalidade concre-
tizar o princípio de que a responsabilização penal é sempre Vedação de determinadas penas
pessoal, devendo assim ser aplicada conforme as peculia- O constituinte viu por bem proibir algumas espécies de
ridades do agente. penas, consoante ao artigo 5º, XLVII, CF:
A primeira menção à individualização da pena se en-
contra no artigo 5º, XLVI, CF: Artigo 5º, XLVII, CF. não haverá penas:
a) de morte, salvo em caso de guerra declarada, nos
Artigo 5º, XLVI, CF. A lei regulará a individualização da termos do art. 84, XIX;
pena e adotará, entre outras, as seguintes: b) de caráter perpétuo;
a) privação ou restrição da liberdade; c) de trabalhos forçados;
b) perda de bens; d) de banimento;
c) multa; e) cruéis.
d) prestação social alternativa;
e) suspensão ou interdição de direitos. Em resumo, o inciso consolida o princípio da humani-
dade, pelo qual o “poder punitivo estatal não pode aplicar
Pelo princípio da individualização da pena, a pena deve sanções que atinjam a dignidade da pessoa humana ou que
ser individualizada nos planos legislativo, judiciário e exe- lesionem a constituição físico-psíquica dos condenados”20 .
cutório, evitando-se a padronização a sanção penal. A in- Quanto à questão da pena de morte, percebe-se que o
dividualização da pena significa adaptar a pena ao conde- constituinte não estabeleceu uma total vedação, autorizan-
nado, consideradas as características do agente e do delito. do-a nos casos de guerra declarada. Obviamente, deve-se
A pena privativa de liberdade é aquela que restringe, respeitar o princípio da anterioridade da lei, ou seja, a le-
com maior ou menor intensidade, a liberdade do condena- gislação deve prever a pena de morte ao fato antes dele ser
do, consistente em permanecer em algum estabelecimento 20 BITENCOURT, Cezar Roberto. Tratado de direito
prisional, por um determinado tempo. penal. 16. ed. São Paulo: Saraiva, 2011. v. 1.

15
LEGISLAÇÃO

praticado. No ordenamento brasileiro, este papel é cumpri- Surgem como corolário do devido processo legal o
do pelo Código Penal Militar (Decreto-Lei nº 1.001/1969), contraditório e a ampla defesa, pois somente um procedi-
que prevê a pena de morte a ser executada por fuzilamento mento que os garanta estará livre dos vícios. Neste sentido,
nos casos tipificados em seu Livro II, que aborda os crimes o artigo 5º, LV, CF:
militares em tempo de guerra.
Por sua vez, estão absolutamente vedadas em quais- Artigo 5º, LV, CF. Aos litigantes, em processo judicial ou
quer circunstâncias as penas de caráter perpétuo, de traba- administrativo, e aos acusados em geral são assegurados o
lhos forçados, de banimento e cruéis. contraditório e ampla defesa, com os meios e recursos a
No que tange aos trabalhos forçados, vale destacar ela inerentes.
que o trabalho obrigatório não é considerado um trata-
mento contrário à dignidade do recluso, embora o trabalho O devido processo legal possui a faceta formal, pela
forçado o seja. O trabalho é obrigatório, dentro das condi- qual se deve seguir o adequado procedimento na aplica-
ções do apenado, não podendo ser cruel ou menosprezar ção da lei e, sendo assim, respeitar o contraditório e a am-
a capacidade física e intelectual do condenado; como o pla defesa. Não obstante, o devido processo legal tem sua
trabalho não existe independente da educação, cabe in- faceta material que consiste na tomada de decisões justas,
centivar o aperfeiçoamento pessoal; até mesmo porque o que respeitem os parâmetros da razoabilidade e da pro-
trabalho deve se aproximar da realidade do mundo exter- porcionalidade.
no, será remunerado; além disso, condições de dignidade e
segurança do trabalhador, como descanso semanal e equi- Vedação de provas ilícitas
pamentos de proteção, deverão ser respeitados. Conforme o artigo 5º, LVI, CF:

Respeito à integridade do preso Artigo 5º, LVI, CF. São inadmissíveis, no processo, as pro-
Prevê o artigo 5º, XLIX, CF: vas obtidas por meios ilícitos.

Artigo 5º, XLIX, CF. É assegurado aos presos o respeito Provas ilícitas, por força da nova redação dada ao arti-
à integridade física e moral. go 157 do CPP, são as obtidas em violação a normas cons-
titucionais ou legai, ou seja, prova ilícita é a que viola regra
Obviamente, o desrespeito à integridade física e mo- de direito material, constitucional ou legal, no momento
ral do preso é uma violação do princípio da dignidade da da sua obtenção. São vedadas porque não se pode aceitar
pessoa humana. o descumprimento do ordenamento para fazê-lo cumprir:
Dois tipos de tratamentos que violam esta integridade seria paradoxal.
estão mencionados no próprio artigo 5º da Constituição
Federal. Em primeiro lugar, tem-se a vedação da tortura Presunção de inocência
e de tratamentos desumanos e degradantes (artigo 5º, III, Prevê a Constituição no artigo 5º, LVII:
CF), o que vale na execução da pena.
No mais, prevê o artigo 5º, LVIII, CF: Artigo 5º, LVII, CF. Ninguém será considerado culpado
até o trânsito em julgado de sentença penal condenatória.
Artigo 5º, LVIII, CF. O civilmente identificado não será
submetido a identificação criminal, salvo nas hipóteses Consolida-se o princípio da presunção de inocência,
previstas em lei. pelo qual uma pessoa não é culpada até que, em definitivo,
o Judiciário assim decida, respeitados todos os princípios e
Se uma pessoa possui identificação civil, não há por- garantias constitucionais.
que fazer identificação criminal, colhendo digitais, fotos,
etc. Pensa-se que seria uma situação constrangedora des- Ação penal privada subsidiária da pública
necessária ao suspeito, sendo assim, violaria a integridade Nos termos do artigo 5º, LIX, CF:
moral.
Artigo 5º, LIX, CF. Será admitida ação privada nos cri-
Devido processo legal, contraditório e ampla defesa mes de ação pública, se esta não for intentada no prazo
Estabelece o artigo 5º, LIV, CF: legal.

Artigo 5º, LIV, CF. Ninguém será privado da liberdade ou A chamada ação penal privada subsidiária da pública
de seus bens sem o devido processo legal. encontra respaldo constitucional, assegurando que a omis-
são do poder público na atividade de persecução criminal
Pelo princípio do devido processo legal a legislação não será ignorada, fornecendo-se instrumento para que o
deve ser respeitada quando o Estado pretender punir al- interessado a proponha.
guém judicialmente. Logo, o procedimento deve ser livre
de vícios e seguir estritamente a legislação vigente, sob
pena de nulidade processual.

16
LEGISLAÇÃO

Prisão e liberdade Artigo 5º, LXVI, CF. Ninguém será levado à prisão ou
O constituinte confere espaço bastante extenso no ar- nela mantido, quando a lei admitir a liberdade provisória,
tigo 5º em relação ao tratamento da prisão, notadamente com ou sem fiança.
por se tratar de ato que vai contra o direito à liberdade.
Obviamente, a prisão não é vedada em todos os casos, Mesmo que a pessoa seja presa em flagrante, devido
porque práticas atentatórias a direitos fundamentais impli- ao princípio da presunção de inocência, entende-se que
cam na tipificação penal, autorizando a restrição da liber- ela não deve ser mantida presa quando não preencher os
dade daquele que assim agiu. requisitos legais para prisão preventiva ou temporária.
No inciso LXI do artigo 5º, CF, prevê-se:
Indenização por erro judiciário
Artigo 5º, LXI, CF. Ninguém será preso senão em fla- A disciplina sobre direitos decorrentes do erro judiciá-
grante delito ou por ordem escrita e fundamentada de rio encontra-se no artigo 5º, LXXV, CF:
autoridade judiciária competente, salvo nos casos de trans-
gressão militar ou crime propriamente militar, definidos em Artigo 5º, LXXV, CF. O Estado indenizará o condenado
lei. por erro judiciário, assim como o que ficar preso além do
tempo fixado na sentença.
Logo, a prisão somente se dará em caso de flagrante
delito (necessariamente antes do trânsito em julgado), ou Trata-se do erro em que incorre um juiz na apreciação
em caráter temporário, provisório ou definitivo (as duas e julgamento de um processo criminal, resultando em con-
primeiras independente do trânsito em julgado, preen- denação de alguém inocente. Neste caso, o Estado inde-
chidos requisitos legais e a última pela irreversibilidade da nizará. Ele também indenizará uma pessoa que ficar presa
condenação). além do tempo que foi condenada a cumprir.
Aborda-se no artigo 5º, LXII o dever de comunicação
ao juiz e à família ou pessoa indicada pelo preso: 5) Direitos fundamentais implícitos
Nos termos do § 2º do artigo 5º da Constituição Fe-
Artigo 5º, LXII, CF. A prisão de qualquer pessoa e o lo- deral:
cal onde se encontre serão comunicados imediatamente ao
juiz competente e à família do preso ou à pessoa por ele Artigo 5º, §2º, CF. Os direitos e garantias expressos nesta
indicada. Constituição não excluem outros decorrentes do regime e
dos princípios por ela adotados, ou dos tratados internacio-
Não obstante, o preso deverá ser informado de todos nais em que a República Federativa do Brasil seja parte.
os seus direitos, inclusive o direito ao silêncio, podendo
entrar em contato com sua família e com um advogado, Daí se depreende que os direitos ou garantias podem
conforme artigo 5º, LXIII, CF: estar expressos ou implícitos no texto constitucional. Sen-
do assim, o rol enumerado nos incisos do artigo 5º é ape-
Artigo 5º, LXIII, CF. O preso será informado de seus di- nas exemplificativo, não taxativo.
reitos, entre os quais o de permanecer calado, sendo-lhe
assegurada a assistência da família e de advogado. 6) Tribunal Penal Internacional
Preconiza o artigo 5º, CF em seu § 4º:
Estabelece-se no artigo 5º, LXIV, CF:
Artigo 5º, §4º, CF. O Brasil se submete à jurisdição de
Artigo 5º, LXIV, CF. O preso tem direito à identificação Tribunal Penal Internacional a cuja criação tenha manifes-
dos responsáveis por sua prisão ou por seu interrogatório tado adesão.
policial.  
O Estatuto de Roma do Tribunal Penal Internacional foi
Por isso mesmo, o auto de prisão em flagrante e a ata promulgado no Brasil pelo Decreto nº 4.388 de 25 de se-
do depoimento do interrogatório são assinados pelas au- tembro de 2002. Ele contém 128 artigos e foi elaborado em
toridades envolvidas nas práticas destes atos procedimen- Roma, no dia 17 de julho de 1998, regendo a competência
tais. e o funcionamento deste Tribunal voltado às pessoas res-
Ainda, a legislação estabelece inúmeros requisitos para ponsáveis por crimes de maior gravidade com repercussão
que a prisão seja validada, sem os quais cabe relaxamento, internacional (artigo 1º, ETPI).
tanto que assim prevê o artigo 5º, LXV, CF: “Ao contrário da Corte Internacional de Justiça, cuja ju-
risdição é restrita a Estados, ao Tribunal Penal Internacional
Artigo 5º, LXV, CF. A prisão ilegal será imediatamente compete o processo e julgamento de violações contra indi-
relaxada pela autoridade judiciária. víduos; e, distintamente dos Tribunais de crimes de guerra
da Iugoslávia e de Ruanda, criados para analisarem crimes
Desta forma, como decorrência lógica, tem-se a previ- cometidos durante esses conflitos, sua jurisdição não está
são do artigo 5º, LXVI, CF: restrita a uma situação específica”21.
21 NEVES, Gustavo Bregalda. Direito Internacional

17
LEGISLAÇÃO

Resume Mello22: “a Conferência das Nações Unidas so- f) Legitimidade passiva: pessoa física, agente público
bre a criação de uma Corte Criminal Internacional, reunida ou privado.
em Roma, em 1998, aprovou a referida Corte. Ela é perma- g) Competência: é determinada pela autoridade coa-
nente. Tem sede em Haia. A corte tem personalidade inter- tora, sendo a autoridade imediatamente superior a ela. Ex.:
nacional. Ela julga: a) crime de genocídio; b) crime contra Delegado de Polícia é autoridade coatora, propõe na Vara
a humanidade; c) crime de guerra; d) crime de agressão. Criminal Estadual; Juiz de Direito de uma Vara Criminal é a
Para o crime de genocídio usa a definição da convenção autoridade coatora, impetra no Tribunal de Justiça.
de 1948. Como crimes contra a humanidade são citados: h) Conceito de coação ilegal: encontra-se no artigo
assassinato, escravidão, prisão violando as normas inter- 648, CPP:
nacionais, violação tortura, apartheid, escravidão sexual,
prostituição forçada, esterilização, etc. São crimes de guer- Artigo 648, CPP. A coação considerar-se-á ilegal: I -
ra: homicídio internacional, destruição de bens não justifi- quando não houver justa causa; II - quando alguém estiver
cada pela guerra, deportação, forçar um prisioneiro a servir preso por mais tempo do que determina a lei; III - quando
nas forças inimigas, etc.”. quem ordenar a coação não tiver competência para fazê-lo;
IV - quando houver cessado o motivo que autorizou a coa-
7) Remédios constitucionais ção; V - quando não for alguém admitido a prestar fiança,
Remédios constitucionais são as espécies de ações ju- nos casos em que a lei a autoriza; VI - quando o processo for
diciárias que visam proteger os direitos fundamentais re- manifestamente nulo; VII - quando extinta a punibilidade.
conhecidos no texto constitucional quando a declaração e
a garantia destes não se mostrar suficiente. Assim, o Poder i) Procedimento: regulamentado nos artigos 647 a
Judiciário será acionado para sanar o desrespeito a estes 667 do Código de Processo Penal.
direitos fundamentais, servindo cada espécie de ação para
uma forma de violação. 7.2) Habeas data
O artigo 5º, LXXII, CF prevê:
7.1) Habeas corpus
No que tange à disciplina do habeas corpus, prevê a Artigo 5º, LXXII, CF. Conceder-se-á habeas data: a) para
Constituição em seu artigo 5º, LXVIII:
assegurar o conhecimento de informações relativas à pessoa
do impetrante, constantes de registros ou bancos de dados
Artigo 5º, LXVIII, CF. Conceder-se-á habeas corpus sem-
de entidades governamentais ou de caráter público; b) para
pre que alguém sofrer ou se achar ameaçado de sofrer vio-
a retificação de dados, quando não se prefira fazê-lo por
lência ou coação em sua liberdade de locomoção, por ilega-
processo sigiloso, judicial ou administrativo.
lidade ou abuso de poder.
Tal como o habeas corpus, trata-se de ação gratuita (ar-
Trata-se de ação gratuita, nos termos do artigo 5º, LXX-
VII, CF. tigo 5º, LXXVII, CF).
a) Antecedentes históricos: A Magna Carta inglesa, a) Antecedente histórico: Freedom of Information Act,
de 1215, foi o primeiro documento a mencionar este remé- de 1974.
dio e o Habeas Corpus Act, de 1679, o regulamentou. b) Escopo: proteção do acesso a informações pessoais
b) Escopo: ação que serve para proteger a liberdade constantes de registros ou bancos de dados de entidades
de locomoção. Antes de haver proteção no Brasil por ou- governamentais ou de caráter público, para o conhecimen-
tros remédios constitucionais de direitos que não este, o to ou retificação (correção).
habeas-corpus foi utilizado para protegê-los. Hoje, apenas c) Natureza jurídica: ação constitucional que tutela o
serve à lesão ou ameaça de lesão ao direito de ir e vir. acesso a informações pessoais.
c) Natureza jurídica: ação constitucional de cunho d) Legitimidade ativa: pessoa física, brasileira ou es-
predominantemente penal, pois protege o direito de ir e trangeira, ou por pessoa jurídica, de direito público ou pri-
vir e vai contra a restrição arbitrária da liberdade. vado, tratando-se de ação personalíssima – os dados de-
d) Espécies: preventivo, para os casos de ameaça de vem ser a respeito da pessoa que a propõe.
violação ao direito de ir e vir, conferindo-se um “salvo con- e) Legitimidade passiva: entidades governamentais
duto”, ou repressivo, para quando ameaça já tiver se ma- da Administração Pública Direta e Indireta nas três esferas,
terializado. bem como instituições, órgãos, entidades e pessoas jurídi-
e) Legitimidade ativa: qualquer pessoa pode manejá cas privadas prestadores de serviços de interesse público
-lo, em próprio nome ou de terceiro, bem como o Ministé- que possuam dados relativos à pessoa do impetrante.
rio Público (artigo 654, CPP). Impetrante é o que ingressa f) Competência: Conforme o caso, nos termos da
com a ação e paciente é aquele que está sendo vítima da Constituição, do Supremo Tribunal Federal (art. 102, I, “d”),
restrição à liberdade de locomoção. As duas figuras podem do Superior Tribunal de Justiça (art. 105, I, “b”), dos Tribu-
se concentrar numa mesma pessoa. nais Regionais Federais (art. 108, I, “c”), bem como dos juí-
Público & Direito Internacional Privado. 3. ed. São Paulo: zes federais (art. 109, VIII).
Atlas, 2009. g) Regulamentação específica: Lei nº 9.507, de 12 de
22 MELLO, Celso D. de Albuquerque. Curso de Direito novembro de 1997.
Internacional Público. 14. ed. São Paulo: Saraiva, 2000. h) Procedimento: artigos 8º a 19 da Lei nº 9.507/1997.

18
LEGISLAÇÃO

7.3) Mandado de segurança individual a) Origem: Constituição Federal de 1988.


Dispõe a Constituição no artigo 5º, LXIX: b) Escopo: preservação ou reparação de direito líqui-
do e certo relacionado a interesses transindividuais (indi-
Artigo 5º, LXIX, CF. Conceder-se-á mandado de seguran- viduais homogêneos ou coletivos), e devido à questão da
ça para proteger direito líquido e certo, não amparado por legitimidade ativa, pertencente a partidos políticos e deter-
habeas-corpus ou habeas-data, quando o responsável pela ile- minadas associações.
galidade ou abuso de poder for autoridade pública ou agente c) Natureza jurídica: ação constitucional de natureza
de pessoa jurídica no exercício de atribuições do Poder Público. civil, independente da natureza do ato, de caráter coletivo.
d) Objeto: o objeto do mandado de segurança coleti-
a) Origem: Veio com a finalidade de preencher a lacuna vo são os direitos coletivos e os direitos individuais homo-
decorrente da sistemática do habeas corpus e das liminares gêneos. Tal instituto não se presta à proteção dos direitos
possessórias. difusos, conforme posicionamento amplamente majoritá-
b) Escopo: Trata-se de remédio constitucional com na- rio, já que, dada sua difícil individualização, fica improvável
tureza subsidiária pelo qual se busca a invalidação de atos de
a verificação da ilegalidade ou do abuso do poder sobre tal
autoridade ou a suspensão dos efeitos da omissão adminis-
direito (art. 21, parágrafo único, Lei nº 12.016/09).
trativa, geradores de lesão a direito líquido e certo, por ilega-
e) Legitimidade ativa: como se extrai da própria disci-
lidade ou abuso de poder. São protegidos todos os direitos
líquidos e certos à exceção da proteção de direitos humanos plina constitucional, aliada ao artigo 21 da Lei nº 12.016/09,
à liberdade de locomoção e ao acesso ou retificação de in- é de partido político com representação no Congresso Na-
formações relativas à pessoa do impetrante, constantes de cional, bem como de organização sindical, entidade de
registros ou bancos de dados de entidades governamentais classe ou associação legalmente constituída e em funcio-
ou de caráter público, ambos sujeitos a instrumentos espe- namento há, pelo menos, 1 (um) ano, em defesa de direitos
cíficos. líquidos e certos que atinjam diretamente seus interesses
c) Natureza jurídica: ação constitucional de natureza ou de seus membros.
civil, independente da natureza do ato impugnado (adminis- f) Disciplina específica na Lei nº 12.016/09:
trativo, jurisdicional, eleitoral, criminal, trabalhista).
d) Espécies: preventivo, quando se estiver na iminência Art. 22, Lei nº 12.016/09. No mandado de segurança
de violação a direito líquido e certo, ou reparatório, quando coletivo, a sentença fará coisa julgada limitadamente aos
já consumado o abuso/ilegalidade. membros do grupo ou categoria substituídos pelo impetran-
e) Direito líquido e certo: é aquele que pode ser de- te.
monstrado de plano mediante prova pré-constituída, sem § 1º O mandado de segurança coletivo não induz li-
a necessidade de dilação probatória, isto devido à natureza tispendência para as ações individuais, mas os efeitos da
célere e sumária do procedimento. coisa julgada não beneficiarão o impetrante a título indi-
f) Legitimidade ativa: a mais ampla possível, abrangen- vidual se não requerer a desistência de seu mandado de
do não só a pessoa física como a jurídica, nacional ou estran- segurança no prazo de 30 (trinta) dias a contar da ciência
geira, residente ou não no Brasil, bem como órgãos públicos comprovada da impetração da segurança coletiva.
despersonalizados e universalidades/pessoas formais reco- § 2º No mandado de segurança coletivo, a liminar só
nhecidas por lei. poderá ser concedida após a audiência do representante
g) Legitimidade passiva: A autoridade coatora deve ser judicial da pessoa jurídica de direito público, que deverá se
autoridade pública ou agente de pessoa jurídica no exercício pronunciar no prazo de 72 (setenta e duas) horas.
de atribuições do Poder Público. Neste viés, o art. 6º, §3º, Lei
nº 12.016/09, preceitua que “considera-se autoridade coato-
7.5) Mandado de injunção
ra aquela que tenha praticado o ato impugnado ou da qual
Regulamenta o artigo 5º, LXXI, CF:
emane a ordem para a sua prática”.
h) Competência: Fixada de acordo com a autoridade
coatora. Artigo 5º, LXXI, CF. Conceder-se-á mandado de injun-
i) Regulamentação específica: Lei nº 12.016, de 07 de ção sempre que a falta de norma regulamentadora torne
agosto de 2009. inviável o exercício dos direitos e liberdades constitucionais
j) Procedimento: artigos 6º a 19 da Lei nº 12.016/09. e das prerrogativas inerentes à nacionalidade, à soberania e
à cidadania.
7.4) Mandado de segurança coletivo
A Constituição Federal prevê a possibilidade de ingresso a) Escopo: os dois requisitos constitucionais para que
com mandado de segurança coletivo, consoante ao artigo seja proposto o mandado de injunção são a existência de
5º, LXX: norma constitucional de eficácia limitada que prescreva di-
reitos, liberdades constitucionais e prerrogativas inerentes
Artigo 5º, LXX, CF. O mandado de segurança coletivo pode à nacionalidade, à soberania e à cidadania; além da falta de
ser impetrado por: a) partido político com representação no norma regulamentadores, impossibilitando o exercício dos
Congresso Nacional; b) organização sindical, entidade de clas- direitos, liberdades e prerrogativas em questão. Assim, visa
se ou associação legalmente constituída e em funcionamen- curar o hábito que se incutiu no legislador brasileiro de não
to há pelo menos um ano, em defesa dos interesses de seus regulamentar as normas de eficácia limitada para que elas
membros ou associados. não sejam aplicáveis.

19
LEGISLAÇÃO

b) Natureza jurídica: ação constitucional que objetiva 8) Direitos humanos, tratados internacionais de
a regulamentação de normas constitucionais de eficácia li- proteção aos direitos humanos e repercussão no Direi-
mitada. to brasileiro
c) Legitimidade ativa: qualquer pessoa, nacional ou Estabelece o artigo 5º, § 2º, CF que os direitos e garan-
estrangeira, física ou jurídica, capaz ou incapaz, que titula- tias podem decorrer, dentre outras fontes, dos “tratados
rize direito fundamental não materializável por omissão le- internacionais em que a República Federativa do Brasil seja
gislativa do Poder público, bem como o Ministério Público parte”.
na defesa de seus interesses institucionais. Não se aceita a Para o tratado internacional ingressar no ordenamen-
legitimidade ativa de pessoas jurídicas de direito público. to jurídico brasileiro deve ser observado um procedimento
d) Competência: Supremo Tribunal Federal, quando a complexo, que exige o cumprimento de quatro fases: a ne-
elaboração de norma regulamentadora for atribuição do gociação (bilateral ou multilateral, com posterior assinatura
Presidente da República, do Congresso Nacional, da Câma- do Presidente da República), submissão do tratado assina-
ra dos Deputados, do Senado Federal, das Mesas de uma do ao Congresso Nacional (que dará referendo por meio
dessas Casas Legislativas, do Tribunal de Contas da União, do decreto legislativo), ratificação do tratado (confirmação
da obrigação perante a comunidade internacional) e a pro-
de um dos Tribunais Superiores, ou do próprio Supremo
mulgação e publicação do tratado pelo Poder Executivo23.
Tribunal Federal (art. 102, I, “q”, CF); ao Superior Tribunal de
Notadamente, quando o constituinte menciona os tratados
Justiça, quando a elaboração da norma regulamentadora
internacionais no §2º do artigo 5º refere-se àqueles que te-
for atribuição de órgão, entidade ou autoridade federal, da nham por fulcro ampliar o rol de direitos do artigo 5º, ou
administração direta ou indireta, excetuados os casos da seja, tratado internacional de direitos humanos.
competência do Supremo Tribunal Federal e dos órgãos da O §1° e o §2° do artigo 5° existiam de maneira originária
Justiça Militar, da Justiça Eleitoral, da Justiça do Trabalho e na Constituição Federal, conferindo o caráter de primazia
da Justiça Federal (art. 105, I, “h”, CF); ao Tribunal Superior dos direitos humanos, desde logo consagrando o princípio
Eleitoral, quando as decisões dos Tribunais Regionais Elei- da primazia dos direitos humanos, como reconhecido pela
torais denegarem habeas corpus, mandado de segurança, doutrina e jurisprudência majoritários na época. “O princí-
habeas data ou mandado de injunção (art. 121, §4º, V, CF); pio da primazia dos direitos humanos nas relações interna-
e aos Tribunais de Justiça Estaduais, frente aos entes a ele cionais implica em que o Brasil deve incorporar os tratados
vinculados. quanto ao tema ao ordenamento interno brasileiro e res-
e) Procedimento: Lei nº 13.300/2016. peitá-los. Implica, também em que as normas voltadas à
proteção da dignidade em caráter universal devem ser apli-
7.6) Ação popular cadas no Brasil em caráter prioritário em relação a outras
Prevê o artigo 5º, LXXIII, CF: normas”24.
Regra geral, os tratados internacionais comuns ingres-
Artigo 5º, LXXIII, CF. Qualquer cidadão é parte legítima sam com força de lei ordinária no ordenamento jurídico
para propor ação popular que vise a anular ato lesivo ao pa- brasileiro porque somente existe previsão constitucional
trimônio público ou de entidade de que o Estado participe, à quanto à possibilidade da equiparação às emendas consti-
moralidade administrativa, ao meio ambiente e ao patrimô- tucionais se o tratado abranger matéria de direitos huma-
nio histórico e cultural, ficando o autor, salvo comprovada nos. Antes da emenda alterou o quadro quanto aos trata-
má-fé, isento de custas judiciais e do ônus da sucumbência. dos de direitos humanos, era o que acontecia, mas isso não
significa que tais direitos eram menos importantes devido
a) Origem: Constituição Federal de 1934. ao princípio da primazia e ao reconhecimento dos direitos
b) Escopo: é instrumento de exercício direto da demo- implícitos.
Por seu turno, com o advento da Emenda Constitucio-
cracia, permitindo ao cidadão que busque a proteção da
nal nº 45/04 se introduziu o §3º ao artigo 5º da Constituição
coisa pública, ou seja, que vise assegurar a preservação dos
Federal, de modo que os tratados internacionais de direitos
interesses transindividuais.
humanos foram equiparados às emendas constitucionais,
c) Natureza jurídica: trata-se de ação constitucional, desde que houvesse a aprovação do tratado em cada Casa
que visa anular ato lesivo ao patrimônio público ou de en- do Congresso Nacional e obtivesse a votação em dois tur-
tidade de que o Estado participe, à moralidade administra- nos e com três quintos dos votos dos respectivos membros:
tiva, ao meio ambiente e ao patrimônio histórico e cultural
d) Legitimidade ativa: deve ser cidadão, ou seja, aque- Artigo 5º, §3º, CF. Os tratados e convenções interna-
le nacional que esteja no pleno gozo dos direitos políticos. cionais sobre direitos humanos que forem aprovados, em
e) Legitimidade passiva: ente da Administração Pú- cada Casa do Congresso Nacional, em dois turnos, por três
blica, direta ou indireta, ou então pessoa jurídica que de quintos dos votos dos respectivos membros, serão equivalen-
algum modo lide com a coisa pública. tes às emendas constitucionais.
f) Competência: Será fixada de acordo com a origem
do ato ou omissão a serem impugnados (artigo 5º, Lei nº 23 VICENTE SOBRINHO, Benedito. Direitos Funda-
4.717/65). mentais e Prisão Civil. Porto Alegre: Sérgio Antonio Fabris
g) Regulamentação específica: Lei nº 4.717, de 29 de Editor, 2008.
junho de 1965. 24 PORTELA, Paulo Henrique Gonçalves. Direito In-
h) Procedimento: artigos 7º a 19, Lei nº 4.717/65. ternacional Público e Privado. Salvador: JusPodivm, 2009.

20
LEGISLAÇÃO

Logo, a partir da alteração constitucional, os tratados No que tange aos direitos sociais percebe-se que a
de direitos humanos que ingressarem no ordenamento jurí- igualdade material assume grande relevância. Afinal, esta
dico brasileiro, versando sobre matéria de direitos humanos, categoria de direitos pressupõe uma postura ativa do Es-
irão passar por um processo de aprovação semelhante ao da tado em prol da efetivação. Nem todos podem arcar com
emenda constitucional. suas despesas de saúde, educação, cultura, alimentação e
Contudo, há posicionamentos conflituosos quanto à moradia, assim como nem todos se encontram na posição
possibilidade de considerar como hierarquicamente cons- de explorador da mão-de-obra, sendo a grande maioria da
titucional os tratados internacionais de direitos humanos população de explorados. Estas pessoas estão numa clara
que ingressaram no ordenamento jurídico brasileiro ante- posição de desigualdade e caberá ao Estado cuidar para
riormente ao advento da referida emenda. Tal discussão se que progressivamente atinjam uma posição de igualdade
deu com relação à prisão civil do depositário infiel, prevista real, já que não é por conta desta posição desfavorável que
como legal na Constituição e ilegal no Pacto de São José da se pode afirmar que são menos dignos, menos titulares de
Costa Rica (tratado de direitos humanos aprovado antes da direitos fundamentais.
EC nº 45/04), sendo que o Supremo Tribunal Federal firmou
Logo, a efetivação dos direitos sociais é uma meta a ser
o entendimento pela supralegalidade do tratado de direitos
alcançada pelo Estado em prol da consolidação da igual-
humanos anterior à Emenda (estaria numa posição que para-
dade material. Sendo assim, o Estado buscará o crescente
lisaria a eficácia da lei infraconstitucional, mas não revogaria
a Constituição no ponto controverso). aperfeiçoamento da oferta de serviços públicos com quali-
dade para que todos os nacionais tenham garantidos seus
direitos fundamentais de segunda dimensão da maneira
CAPÍTULO II mais plena possível.
DOS DIREITOS SOCIAIS Há se ressaltar também que o Estado não possui ape-
nas um papel direto na promoção dos direitos econômicos,
sociais e culturais, mas também um indireto, quando por
meio de sua gestão permite que os indivíduos adquiram
A Constituição Federal, dentro do Título II, aborda no condições para sustentarem suas necessidades pertencen-
capítulo II a categoria dos direitos sociais, em sua maioria tes a esta categoria de direitos.
normas programáticas e que necessitam de uma postura in-
terventiva estatal em prol da implementação. 2) Reserva do possível e mínimo existencial
Os direitos assegurados nesta categoria encontram Os direitos sociais serão concretizados gradualmente,
menção genérica no artigo 6º, CF: notadamente porque estão previstos em normas progra-
máticas e porque a implementação deles gera um ônus
Artigo 6º, CF. Art. 6º São direitos sociais a educação, a para o Estado. Diferentemente dos direitos individuais, que
saúde, a alimentação, o trabalho, a moradia, o transpor- dependem de uma postura de abstenção estatal, os direi-
te, o lazer, a segurança, a previdência social, a proteção à tos sociais precisam que o Estado assuma um papel ativo
maternidade e à infância, a assistência aos desampara- em prol da efetivação destes.
dos, na forma desta Constituição.  A previsão excessiva de direitos sociais no bojo de uma
Constituição, a despeito de um instante bem-intencionado
Trata-se de desdobramento da perspectiva do Estado de palavras promovido pelo constituinte, pode levar à ne-
Social de Direito. Em suma, são elencados os direitos huma- gativa, paradoxal – e, portanto, inadmissível – consequên-
nos de 2ª dimensão, notadamente conhecidos como direitos cia de uma Carta Magna cujas finalidades não condigam
econômicos, sociais e culturais. Em resumo, os direitos sociais
com seus próprios prescritos, fato que deslegitima o Poder
envolvem prestações positivas do Estado (diferente dos de
Público como determinador de que particulares respeitem
liberdade, que referem-se à postura de abstenção estatal),
os direitos fundamentais, já que sequer eles próprios, os
ou seja, políticas estatais que visem consolidar o princípio da
igualdade não apenas formalmente, mas materialmente (tra- administradores, conseguem cumprir o que consta de seu
tando os desiguais de maneira desigual). Estatuto Máximo25.
Por seu turno, embora no capítulo específico do Título Tecnicamente, nos direitos sociais é possível invocar
II que aborda os direitos sociais não se perceba uma intensa a cláusula da reserva do possível como argumento para a
regulamentação destes, à exceção dos direitos trabalhistas, não implementação de determinado direito social – seja
o Título VIII da Constituição Federal, que aborda a ordem pela absoluta ausência de recursos (reserva do possível fá-
social, se concentra em trazer normativas mais detalhadas a tica), seja pela ausência de previsão orçamentária nos ter-
respeitos de direitos indicados como sociais. mos do artigo 167, CF (reserva do possível jurídica).
O Ministro Celso de Mello afirmou em julgamento que
1) Igualdade material e efetivação dos direitos sociais os direitos sociais “não pode converter-se em promessa
Independentemente da categoria de direitos que este- constitucional inconsequente, sob pena de o Poder Públi-
ja sendo abordada, a igualdade nunca deve aparecer num co, fraudando justas expectativas nele depositadas pela co-
sentido meramente formal, mas necessariamente material. 25 LAZARI, Rafael José Nadim de. Reserva do possí-
Significa que discriminações indevidas são proibidas, mas vel e mínimo existencial: a pretensão de eficácia da norma
existem certas distinções que não só devem ser aceitas, constitucional em face da realidade. Curitiba: Juruá, 2012, p.
como também se mostram essenciais. 56-57.

21
LEGISLAÇÃO

letividade, substituir, de maneira ilegítima, o cumprimento Artigo 7º, I, CF. Relação de emprego protegida contra
de seu impostergável dever, por um gesto irresponsável de despedida arbitrária ou sem justa causa, nos termos de
infidelidade governamental ao que determina a própria Lei lei complementar, que preverá indenização compensatória,
Fundamental do Estado”26. dentre outros direitos.
Sendo assim, a invocação da cláusula da reserva do
possível, embora viável, não pode servir de muleta para Significa que a demissão, se não for motivada por justa
que o Estado não arque com obrigações básicas. Neste causa, assegura ao trabalhador direitos como indenização
viés, geralmente, quando invocada a cláusula é afastada, compensatória, entre outros, a serem arcados pelo empre-
entendendo o Poder Judiciário que não cabe ao Estado se gador.
eximir de garantir direitos sociais com o simples argumen-
to de que não há orçamento específico para isso – ele de- Artigo 7º, II, CF. Seguro-desemprego, em caso de de-
veria ter reservado parcela suficiente de suas finanças para semprego involuntário.
atender esta demanda.
Com efeito, deve ser preservado o mínimo existencial, Sem prejuízo de eventual indenização a ser recebida
que tem por fulcro limitar a discricionariedade político-ad- do empregador, o trabalhador que fique involuntariamente
ministrativa e estabelecer diretrizes orçamentárias básicas desempregado – entendendo-se por desemprego invo-
a serem seguidas, sob pena de caber a intervenção do Po- luntário o que tenha origem num acordo de cessação do
der Judiciário em prol de sua efetivação. contrato de trabalho – tem direito ao seguro-desemprego,
a ser arcado pela previdência social, que tem o caráter de
3) Princípio da proibição do retrocesso assistência financeira temporária.
Proibição do retrocesso é a impossibilidade de que
uma conquista garantida na Constituição Federal sofra um Artigo 7º, III, CF. Fundo de garantia do tempo de ser-
retrocesso, de modo que um direito social garantido não viço.
pode deixar de o ser.
Conforme jurisprudência, a proibição do retrocesso Foi criado em 1967 pelo Governo Federal para pro-
deve ser tomada com reservas, até mesmo porque segun- teger o trabalhador demitido sem justa causa. O FGTS é
do entendimento predominante as normas do artigo 7º, constituído de contas vinculadas, abertas em nome de
CF não são cláusula pétrea, sendo assim passíveis de alte- cada trabalhador, quando o empregador efetua o primei-
ração. Se for alterada normativa sobre direito trabalhista ro depósito. O saldo da conta vinculada é formado pelos
assegurado no referido dispositivo, não sendo o prejuízo depósitos mensais efetivados pelo empregador, equivalen-
evidente, entende-se válida (por exemplo, houve alteração tes a 8,0% do salário pago ao empregado, acrescido de
do prazo prescricional diferenciado para os trabalhadores atualização monetária e juros. Com o FGTS, o trabalhador
agrícolas). O que, em hipótese alguma, pode ser aceito é tem a oportunidade de formar um patrimônio, que pode
um retrocesso evidente, seja excluindo uma categoria de ser sacado em momentos especiais, como o da aquisição
direitos (ex.: abolir o Sistema Único de Saúde), seja dimi- da casa própria ou da aposentadoria e em situações de
nuindo sensivelmente a abrangência da proteção (ex.: ex- dificuldades, que podem ocorrer com a demissão sem justa
cluindo o ensino médio gratuito). causa ou em caso de algumas doenças graves.
Questão polêmica se refere à proibição do retrocesso:
se uma decisão judicial melhorar a efetivação de um direito Artigo 7º, IV, CF. Salário mínimo, fixado em lei, nacio-
social, ela se torna vinculante e é impossível ao legislador nalmente unificado, capaz de atender a suas necessidades
alterar a Constituição para retirar este avanço? Por um lado, vitais básicas e às de sua família com moradia, alimenta-
a proibição do retrocesso merece ser tomada em conceito ção, educação, saúde, lazer, vestuário, higiene, trans-
amplo, abrangendo inclusive decisões judiciais; por outro porte e previdência social, com reajustes periódicos que
lado, a decisão judicial não tem por fulcro alterar a norma, lhe preservem o poder aquisitivo, sendo vedada sua vincula-
o que somente é feito pelo legislador, e ele teria o direito ção para qualquer fim.
de prever que aquela decisão judicial não está incorporada T
na proibição do retrocesso. A questão é polêmica e não há rata-se de uma visível norma programática da Cons-
entendimento dominante. tituição que tem por pretensão um salário mínimo que
atenda a todas as necessidades básicas de uma pessoa e
4) Direito individual do trabalho de sua família. Em pesquisa que tomou por parâmetro o
O artigo 7º da Constituição enumera os direitos indi- preceito constitucional, detectou-se que “o salário mínimo
viduais dos trabalhadores urbanos e rurais. São os direitos do trabalhador brasileiro deveria ter sido de R$ 2.892,47
individuais tipicamente trabalhistas, mas que não excluem em abril para que ele suprisse suas necessidades básicas e
os demais direitos fundamentais (ex.: honra é um direito da família, segundo estudo divulgado nesta terça-feira, 07,
no espaço de trabalho, sob pena de se incidir em prática pelo Departamento Intersindical de Estatística e Estudos
de assédio moral). Socioeconômicos (Dieese)”27.

27 http://exame.abril.com.br/economia/noticias/salario-
26 RTJ 175/1212-1213, Rel. Min. CELSO DE MELLO. -minimo-deveria-ter-sido-de-r-2-892-47-em-abril

22
LEGISLAÇÃO

Artigo 7º, V, CF. Piso salarial proporcional à extensão e o dispositivo é norma de eficácia limitada, pois depende
à complexidade do trabalho. de lei ordinária, ainda mais porque qualquer norma penal
incriminadora é regida pela legalidade estrita (artigo 5º,
Cada trabalhador, dentro de sua categoria de empre- XXXIX, CF).
go, seja ele professor, comerciário, metalúrgico, bancário,
construtor civil, enfermeiro, recebe um salário base, cha- Artigo 7º, XI, CF. Participação nos lucros, ou resul-
mado de Piso Salarial, que é sua garantia de recebimento tados, desvinculada da remuneração, e, excepcionalmente,
dentro de seu grau profissional. O Valor do Piso Salarial é participação na gestão da empresa, conforme definido em
estabelecido em conformidade com a data base da cate- lei.
goria, por isso ele é definido em conformidade com um
acordo, ou ainda com um entendimento entre patrão e A Participação nos Lucros e Resultado (PLR), que é
trabalhador. conhecida também por Programa de Participação nos Re-
sultados (PPR), está prevista na Consolidação das Leis do
Artigo 7º, VI, CF. Irredutibilidade do salário, salvo o Trabalho (CLT) desde a Lei nº 10.101, de 19 de dezembro
disposto em convenção ou acordo coletivo. de 2000. Ela funciona como um bônus, que é ofertado pelo
empregador e negociado com uma comissão de trabalha-
O salário não pode ser reduzido, a não ser que anão dores da empresa. A CLT não obriga o empregador a forne-
redução implique num prejuízo maior, por exemplo, de- cer o benefício, mas propõe que ele seja utilizado.
missão em massa durante uma crise, situações que devem
ser negociadas em convenção ou acordo coletivo. Artigo 7º, XII, CF. Salário-família pago em razão do
dependente do trabalhador de baixa renda nos termos da lei.
Artigo 7º, VII, CF. Garantia de salário, nunca inferior
ao mínimo, para os que percebem remuneração variável. Salário-família é o benefício pago na proporção do
respectivo número de filhos ou equiparados de qualquer
O salário mínimo é direito de todos os trabalhadores, condição até a idade de quatorze anos ou inválido de qual-
mesmo daqueles que recebem remuneração variável (ex.: quer idade, independente de carência e desde que o salá-
baseada em comissões por venda e metas); rio-de-contribuição seja inferior ou igual ao limite máximo
permitido. De acordo com a Portaria Interministerial MPS/
Artigo 7º, VIII, CF. Décimo terceiro salário com base na MF nº 19, de 10/01/2014, valor do salário-família será de
remuneração integral ou no valor da aposentadoria. R$ 35,00, por filho de até 14 anos incompletos ou inválido,
para quem ganhar até R$ 682,50. Já para o trabalhador que
Também conhecido como gratificação natalina, foi ins- receber de R$ 682,51 até R$ 1.025,81, o valor do salário-
tituída no Brasil pela Lei nº 4.090/1962 e garante que o tra- família por filho de até 14 anos de idade ou inválido de
balhador receba o correspondente a 1/12 (um doze avos) qualquer idade será de R$ 24,66.
da remuneração por mês trabalhado, ou seja, consiste no
pagamento de um salário extra ao trabalhador e ao apo- Artigo 7º, XIII, CF. duração do trabalho normal não su-
sentado no final de cada ano. perior a oito horas diárias e quarenta e quatro semanais,
facultada a compensação de horários e a redução da jorna-
Artigo 7º, IX, CF. Remuneração do trabalho noturno da, mediante acordo ou convenção coletiva de trabalho.
superior à do diurno.
Artigo 7º, XVI, CF. Remuneração do serviço extraor-
O adicional noturno é devido para o trabalho exercido dinário superior, no mínimo, em cinquenta por cento à do
durante a noite, de modo que cada hora noturna sofre a re- normal.
dução de 7 minutos e 30 segundos, ou ainda, é feito acrés-
cimo de 12,5% sobre o valor da hora diurna. Considera-se A legislação trabalhista vigente estabelece que a du-
noturno, nas atividades urbanas, o trabalho realizado entre ração normal do trabalho, salvo os casos especiais, é de
as 22:00 horas de um dia às 5:00 horas do dia seguinte; nas 8 (oito) horas diárias e 44 (quarenta e quatro) semanais,
atividades rurais, é considerado noturno o trabalho execu- no máximo. Todavia, poderá a jornada diária de trabalho
tado na lavoura entre 21:00 horas de um dia às 5:00 horas dos empregados maiores ser acrescida de horas suple-
do dia seguinte; e na pecuária, entre 20:00 horas às 4:00 mentares, em número não excedentes a duas, no máximo,
horas do dia seguinte. para efeito de serviço extraordinário, mediante acordo in-
dividual, acordo coletivo, convenção coletiva ou sentença
Artigo 7º, X, CF. Proteção do salário na forma da lei, normativa. Excepcionalmente, ocorrendo necessidade im-
constituindo crime sua retenção dolosa. periosa, poderá ser prorrogada além do limite legalmente
permitido. A remuneração do serviço extraordinário, des-
Quanto ao possível crime de retenção de salário, não de a promulgação da Constituição Federal, deverá cons-
há no Código Penal brasileiro uma norma que determina a tar, obrigatoriamente, do acordo, convenção ou sentença
ação de retenção de salário como crime. Apesar do artigo normativa, e será, no mínimo, 50% (cinquenta por cento)
7º, X, CF dizer que é crime a retenção dolosa de salário, superior à da hora normal.

23
LEGISLAÇÃO

Artigo 7º, XIV, CF. Jornada de seis horas para o traba- Artigo 7º, XX, CF. Proteção do mercado de trabalho da
lho realizado em turnos ininterruptos de revezamento, mulher, mediante incentivos específicos, nos termos da lei.
salvo negociação coletiva.
Embora as mulheres sejam maioria na população de
O constituinte ao estabelecer jornada máxima de 6 ho- 10 anos ou mais de idade, elas são minoria na população
ras para os turnos ininterruptos de revezamento, expres- ocupada, mas estão em maioria entre os desocupados.
samente ressalvando a hipótese de negociação coletiva, Acrescenta-se ainda, que elas são maioria também na po-
objetivou prestigiar a atuação da entidade sindical. Entre- pulação não economicamente ativa. Além disso, ainda há
tanto, a jurisprudência evoluiu para uma interpretação res- relevante diferença salarial entre homens e mulheres, sen-
tritiva de seu teor, tendo como parâmetro o fato de que do que os homens recebem mais porque os empregadores
o trabalho em turnos ininterruptos é por demais desgas- entendem que eles necessitam de um salário maior para
tante, penoso, além de trazer malefícios de ordem fisio- manter a família. Tais disparidades colocam em evidência
lógica para o trabalhador, inclusive distúrbios no âmbito que o mercado de trabalho da mulher deve ser protegido
psicossocial já que dificulta o convívio em sociedade e com de forma especial.
a própria família.
Artigo 7º, XXI, CF. Aviso prévio proporcional ao tem-
Artigo 7º, XV, CF. Repouso semanal remunerado, pre- po de serviço, sendo no mínimo de trinta dias, nos termos
ferencialmente aos domingos. da lei.
O Descanso Semanal Remunerado é de 24 (vinte e Nas relações de emprego, quando uma das partes de-
quatro) horas consecutivas, devendo ser concedido prefe- seja rescindir, sem justa causa, o contrato de trabalho por
rencialmente aos domingos, sendo garantido a todo traba- prazo indeterminado, deverá, antecipadamente, notificar à
lhador urbano, rural ou doméstico. Havendo necessidade outra parte, através do aviso prévio. O aviso prévio tem
de trabalho aos domingos, desde que previamente auto- por finalidade evitar a surpresa na ruptura do contrato de
rizados pelo Ministério do Trabalho, aos trabalhadores é trabalho, possibilitando ao empregador o preenchimento
assegurado pelo menos um dia de repouso semanal re-
do cargo vago e ao empregado uma nova colocação no
munerado coincidente com um domingo a cada período,
mercado de trabalho, sendo que o aviso prévio pode ser
dependendo da atividade (artigo 67, CLT).
trabalhado ou indenizado.
Artigo 7º, XVII, CF. Gozo de férias anuais remuneradas
Artigo 7º, XXII, CF. Redução dos riscos inerentes ao
com, pelo menos, um terço a mais do que o salário normal.
trabalho, por meio de normas de saúde, higiene e segu-
rança.
O salário das férias deve ser superior em pelo menos
um terço ao valor da remuneração normal, com todos os
adicionais e benefícios aos quais o trabalhador tem direi- Trata-se ao direito do trabalhador a um meio ambiente
to. A cada doze meses de trabalho – denominado período do trabalho salubre. Fiorillo28 destaca que o equilíbrio do
aquisitivo – o empregado terá direito a trinta dias corridos meio ambiente do trabalho está sedimentado na salubrida-
de férias, se não tiver faltado injustificadamente mais de de e na ausência de agentes que possam comprometer a
cinco vezes ao serviço (caso isso ocorra, os dias das férias incolumidade físico-psíquica dos trabalhadores.
serão diminuídos de acordo com o número de faltas).
Artigo 7º, XXIII, CF. Adicional de remuneração para as
Artigo 7º, XVIII, CF. Licença à gestante, sem prejuízo atividades penosas, insalubres ou perigosas, na forma da
do emprego e do salário, com a duração de cento e vinte lei.
dias.
Penoso é o trabalho acerbo, árduo, amargo, difícil, mo-
O salário da trabalhadora em licença é chamado de lesto, trabalhoso, incômodo, laborioso, doloroso, rude, que
salário-maternidade, é pago pelo empregador e por ele não é perigoso ou insalubre, mas penosa, exigindo atenção
descontado dos recolhimentos habituais devidos à Previ- e vigilância acima do comum. Ainda não há na legislação
dência Social. A trabalhadora pode sair de licença a partir específica previsão sobre o adicional de penosidade.
do último mês de gestação, sendo que o período de licen- São consideradas atividades ou operações insalubres
ça é de 120 dias. A Constituição também garante que, do as que se desenvolvem excesso de limites de tolerância
momento em que se confirma a gravidez até cinco meses para: ruído contínuo ou intermitente, ruídos de impacto,
após o parto, a mulher não pode ser demitida. exposição ao calor e ao frio, radiações, certos agentes quí-
micos e biológicos, vibrações, umidade, etc. O exercício de
Artigo 7º, XIX, CF. Licença-paternidade, nos termos fi- trabalho em condições de insalubridade assegura ao traba-
xados em lei. lhador a percepção de adicional, incidente sobre o salário
base do empregado (súmula 228 do TST), ou previsão mais
O homem tem direito a 5 dias de licença-paternidade 28 FIORILLO, Celso Antonio Pacheco. Curso de Direi-
para estar mais próximo do bebê recém-nascido e ajudar a to Ambiental brasileiro. 10. ed. São Paulo: Saraiva, 2009, p.
mãe nos processos pós-operatórios. 21.

24
LEGISLAÇÃO

benéfica em Convenção Coletiva de Trabalho, equivalen- Artigo 7º, XXVII, CF. Proteção em face da automação,
te a 40% (quarenta por cento), para insalubridade de grau na forma da lei.
máximo; 20% (vinte por cento), para insalubridade de grau
médio; 10% (dez por cento), para insalubridade de grau Trata-se da proteção da substituição da máquina pelo
mínimo. homem, que pode ser feita, notadamente, qualificando o
O adicional de periculosidade é um valor devido ao profissional para exercer trabalhos que não possam ser de-
empregado exposto a atividades perigosas. São conside- sempenhados por uma máquina (ex.: se criada uma má-
radas atividades ou operações perigosas, aquelas que, por quina que substitui o trabalhador, deve ser ele qualificado
sua natureza ou métodos de trabalho, impliquem risco para que possa operá-la).
acentuado em virtude de exposição permanente do tra-
balhador a inflamáveis, explosivos ou energia elétrica; e a Artigo 7º, XXVIII, CF. Seguro contra acidentes de tra-
roubos ou outras espécies de violência física nas atividades balho, a cargo do empregador, sem excluir a indenização
profissionais de segurança pessoal ou patrimonial. O valor a que este está obrigado, quando incorrer em dolo ou culpa.
do adicional de periculosidade será o salário do empre-
gado acrescido de 30%, sem os acréscimos resultantes de Atualmente, é a Lei nº 8.213/91 a responsável por tra-
gratificações, prêmios ou participações nos lucros da em- tar do assunto e em seus artigos 19, 20 e 21 apresenta a
presa. definição de doenças e acidentes do trabalho. Não se trata
O Tribunal Superior do Trabalho ainda não tem enten- de legislação específica sobre o tema, mas sim de uma nor-
dimento unânime sobre a possibilidade de cumulação des- ma que dispõe sobre as modalidades de benefícios da pre-
tes adicionais. vidência social. Referida Lei, em seu artigo 19 da preceitua
que acidente do trabalho é o que ocorre pelo exercício do
Artigo 7º, XXIV, CF. Aposentadoria. trabalho a serviço da empresa ou pelo exercício do traba-
lho, provocando lesão corporal ou perturbação funcional
A aposentadoria é um benefício garantido a todo tra- que cause a morte ou a perda ou redução, permanente ou
balhador brasileiro que pode ser usufruído por aquele que temporária, da capacidade para o trabalho.
tenha contribuído ao Instituto Nacional de Seguridade So- Seguro de Acidente de Trabalho (SAT) é uma contri-
cial (INSS) pelos prazos estipulados nas regras da Previ- buição com natureza de tributo que as empresas pagam
dência Social e tenha atingido as idades mínimas previstas. para custear benefícios do INSS oriundos de acidente de
Aliás, o direito à previdência social é considerado um direi- trabalho ou doença ocupacional, cobrindo a aposentadoria
to social no próprio artigo 6º, CF. especial. A alíquota normal é de um, dois ou três por cen-
to sobre a remuneração do empregado, mas as empresas
Artigo 7º, XXV, CF. Assistência gratuita aos filhos e que expõem os trabalhadores a agentes nocivos químicos,
dependentes desde o nascimento até 5 (cinco) anos de ida- físicos e biológicos precisam pagar adicionais diferencia-
de em creches e pré-escolas. dos. Assim, quanto maior o risco, maior é a alíquota, mas
atualmente o Ministério da Previdência Social pode alterar
Todo estabelecimento com mais de 30 funcionárias a alíquota se a empresa investir na segurança do trabalho.
com mais de 16 anos tem a obrigação de oferecer um es- Neste sentido, nada impede que a empresa seja res-
paço físico para que as mães deixem o filho de 0 a 6 meses, ponsabilizada pelos acidentes de trabalho, indenizando
enquanto elas trabalham. Caso não ofereçam esse espaço o trabalhador. Na atualidade entende-se que a possibi-
aos bebês, a empresa é obrigada a dar auxílio-creche a mu- lidade de cumulação do benefício previdenciário, assim
lher para que ela pague uma creche para o bebê de até 6 compreendido como prestação garantida pelo Estado ao
meses. O valor desse auxílio será determinado conforme trabalhador acidentado (responsabilidade objetiva) com
negociação coletiva na empresa (acordo da categoria ou a indenização devida pelo empregador em caso de culpa
convenção). A empresa que tiver menos de 30 funcionárias (responsabilidade subjetiva), é pacífica, estando ampla-
registradas não tem obrigação de conceder o benefício. É mente difundida na jurisprudência do Tribunal Superior do
facultativo (ela pode oferecer ou não). Existe a possibilida- Trabalho;
de de o benefício ser estendido até os 6 anos de idade e
incluir o trabalhador homem. A duração do auxílio-creche Artigo 7º, XXIX, CF. Ação, quanto aos créditos resultan-
e o valor envolvido variarão conforme negociação coletiva tes das relações de trabalho, com prazo prescricional de
na empresa. cinco anos para os trabalhadores urbanos e rurais, até o
limite de dois anos após a extinção do contrato de tra-
Artigo 7º, XXVI, CF. Reconhecimento das convenções e balho.
acordos coletivos de trabalho.
Prescrição é a perda da pretensão de buscar a tutela
Neste dispositivo se funda o direito coletivo do tra- jurisdicional para assegurar direitos violados. Sendo assim,
balho, que encontra regulamentação constitucional nos há um período de tempo que o empregado tem para re-
artigo 8º a 11 da Constituição. Pelas convenções e acor- querer seu direito na Justiça do Trabalho. A prescrição tra-
dos coletivos, entidades representativas da categoria dos balhista é sempre de 2 (dois) anos a partir do término do
trabalhadores entram em negociação com as empresas na contrato de trabalho, atingindo as parcelas relativas aos 5
defesa dos interesses da classe, assegurando o respeito aos (cinco) anos anteriores, ou de 05 (cinco) anos durante a
direitos sociais; vigência do contrato de trabalho.

25
LEGISLAÇÃO

Artigo 7º, XXX, CF. Proibição de diferença de salários, 5) Direito coletivo do trabalho
de exercício de funções e de critério de admissão por motivo Os artigos 8º a 11 trazem os direitos sociais coletivos
de sexo, idade, cor ou estado civil. dos trabalhadores, que são os exercidos pelos trabalha-
dores, coletivamente ou no interesse de uma coletivida-
Há uma tendência de se remunerar melhor homens de, quais sejam: associação profissional ou sindical, greve,
brancos na faixa dos 30 anos que sejam casados, sendo substituição processual, participação e representação clas-
patente a diferença remuneratória para com pessoas de sista29.
diferente etnia, faixa etária ou sexo. Esta distinção atenta A liberdade de associação profissional ou sindical tem
contra o princípio da igualdade e não é aceita pelo consti- escopo no artigo 8º, CF:
tuinte, sendo possível inclusive invocar a equiparação sala-
rial judicialmente. Art. 8º, CF. É livre a associação profissional ou sindi-
cal, observado o seguinte:
Artigo 7º, XXXI, CF. Proibição de qualquer discrimina- I - a lei não poderá exigir autorização do Estado para
ção no tocante a salário e critérios de admissão do trabalha- a fundação de sindicato, ressalvado o registro no órgão
dor portador de deficiência. competente, vedadas ao Poder Público a interferência e a
intervenção na organização sindical;
A pessoa portadora de deficiência, dentro de suas li- II - é vedada a criação de mais de uma organização
mitações, possui condições de ingressar no mercado de sindical, em qualquer grau, representativa de categoria
trabalho e não pode ser preterida meramente por conta de profissional ou econômica, na mesma base territorial, que
sua deficiência. será definida pelos trabalhadores ou empregadores interes-
sados, não podendo ser inferior à área de um Município;
Artigo 7º, XXXII, CF. Proibição de distinção entre traba- III - ao sindicato cabe a defesa dos direitos e interes-
lho manual, técnico e intelectual ou entre os profissionais ses coletivos ou individuais da categoria, inclusive em
respectivos. questões judiciais ou administrativas;
IV - a assembleia geral fixará a contribuição que, em
Os trabalhos manuais, técnicos e intelectuais são igual-
se tratando de categoria profissional, será descontada em
mente relevantes e contribuem todos para a sociedade,
folha, para custeio do sistema confederativo da representa-
não cabendo a desvalorização de um trabalho apenas por
ção sindical respectiva, independentemente da contribuição
se enquadrar numa ou outra categoria.
prevista em lei;
V - ninguém será obrigado a filiar-se ou a manter-se
Artigo 7º, XXXIII, CF. proibição de trabalho noturno,
filiado a sindicato;
perigoso ou insalubre a menores de dezoito e de qual-
VI - é obrigatória a participação dos sindicatos nas
quer trabalho a menores de dezesseis anos, salvo na
condição de aprendiz, a partir de quatorze anos. negociações coletivas de trabalho;
VII - o aposentado filiado tem direito a votar e ser vo-
Trata-se de norma protetiva do adolescente, estabele- tado nas organizações sindicais;
cendo-se uma idade mínima para trabalho e proibindo-se VIII - é vedada a dispensa do empregado sindicali-
o trabalho em condições desfavoráveis. zado a partir do registro da candidatura a cargo de direção
ou representação sindical e, se eleito, ainda que suplente,
Artigo 7º, XXXIV, CF. Igualdade de direitos entre o tra- até um ano após o final do mandato, salvo se cometer falta
balhador com vínculo empregatício permanente e o tra- grave nos termos da lei.
balhador avulso. Parágrafo único. As disposições deste artigo aplicam-se
à organização de sindicatos rurais e de colônias de pes-
Avulso é o trabalhador que presta serviço a várias em- cadores, atendidas as condições que a lei estabelecer.
presas, mas é contratado por sindicatos e órgãos gestores
de mão-de-obra, possuindo os mesmos direitos que um O direito de greve, por seu turno, está previsto no ar-
trabalhador com vínculo empregatício permanente. tigo 9º, CF:
A Emenda Constitucional nº 72/2013, conhecida como
PEC das domésticas, deu nova redação ao parágrafo único Art. 9º É assegurado o direito de greve, competindo aos
do artigo 7º: trabalhadores decidir sobre a oportunidade de exercê-lo e
sobre os interesses que devam por meio dele defender.
Artigo 7º, parágrafo único, CF. São assegurados à cate- § 1º A lei definirá os serviços ou atividades essenciais
goria dos trabalhadores domésticos os direitos previstos e disporá sobre o atendimento das necessidades inadiáveis
nos incisos IV, VI, VII, VIII, X, XIII, XV, XVI, XVII, XVIII, XIX, XXI, da comunidade.
XXII, XXIV, XXVI, XXX, XXXI e XXXIII e, atendidas as condições § 2º Os abusos cometidos sujeitam os responsáveis às
estabelecidas em lei e observada a simplificação do cum- penas da lei.
primento das obrigações tributárias, principais e acessórias,
decorrentes da relação de trabalho e suas peculiaridades, os
previstos nos incisos I, II, III, IX, XII, XXV e XXVIII, bem como 29 LENZA, Pedro. Curso de direito constitucional
a sua integração à previdência social.  esquematizado. 15. ed. São Paulo: Saraiva, 2011.

26
LEGISLAÇÃO

A respeito, conferir a Lei nº 7.783/89, que dispõe sobre Art. 195. A seguridade social será financiada por toda a
o exercício do direito de greve, define as atividades essen- sociedade, de forma direta e indireta, nos termos da lei, me-
ciais, regula o atendimento das necessidades inadiáveis diante recursos provenientes dos orçamentos da União, dos
da comunidade, e dá outras providências. Enquanto não Estados, do Distrito Federal e dos Municípios, e das seguintes
for disciplinado o direito de greve dos servidores públicos, contribuições sociais:
esta é a legislação que se aplica, segundo o STF. I - do empregador, da empresa e da entidade a ela equi-
O direito de participação é previsto no artigo 10, CF: parada na forma da lei, incidentes sobre: 
a) a folha de salários e demais rendimentos do trabalho
Artigo 10, CF. É assegurada a participação dos traba- pagos ou creditados, a qualquer título, à pessoa física que
lhadores e empregadores nos colegiados dos órgãos pú- lhe preste serviço, mesmo sem vínculo empregatício; 
blicos em que seus interesses profissionais ou previdenciá- b) a receita ou o faturamento; 
rios sejam objeto de discussão e deliberação. c) o lucro; 
II - do trabalhador e dos demais segurados da previdên-
Por fim, aborda-se o direito de representação classista cia social, não incidindo contribuição sobre aposentadoria e
no artigo 11, CF: pensão concedidas pelo regime geral de previdência social
de que trata o art. 201; 
Artigo 11, CF. Nas empresas de mais de duzentos em- III - sobre a receita de concursos de prognósticos.
pregados, é assegurada a eleição de um representante IV - do importador de bens ou serviços do exterior, ou de
destes com a finalidade exclusiva de promover-lhes o enten- quem a lei a ele equiparar. 
dimento direto com os empregadores. § 1º - As receitas dos Estados, do Distrito Federal e dos
Municípios destinadas à seguridade social constarão dos
respectivos orçamentos, não integrando o orçamento da
TÍTULO VIII União.
§ 2º A proposta de orçamento da seguridade social
DO ORDEM SOCIAL
será elaborada de forma integrada pelos órgãos responsá-
CAPÍTULO II
veis pela saúde, previdência social e assistência social, ten-
DA SEGURIDADE SOCIAL do em vista as metas e prioridades estabelecidas na lei de
diretrizes orçamentárias, assegurada a cada área a gestão
de seus recursos.
§ 3º A pessoa jurídica em débito com o sistema da
CAPÍTULO II seguridade social, como estabelecido em lei, não poderá
DA SEGURIDADE SOCIAL contratar com o Poder Público nem dele receber benefícios
ou incentivos fiscais ou creditícios.
Seção I § 4º A lei poderá instituir outras fontes destinadas a
DISPOSIÇÕES GERAIS garantir a manutenção ou expansão da seguridade social,
obedecido o disposto no art. 154, I.
O título VIII, que aborda a ordem social, traz este § 5º Nenhum benefício ou serviço da seguridade social
tripé no capítulo II, intitulado “Da Seguridade Social”: poderá ser criado, majorado ou estendido sem a corres-
saúde, previdência e assistência social. pondente fonte de custeio total.
§ 6º As contribuições sociais de que trata este artigo só
Art. 194. A seguridade social compreende um conjunto poderão ser exigidas após decorridos noventa dias da data
integrado de ações de iniciativa dos Poderes Públicos e da da publicação da lei que as houver instituído ou modifica-
sociedade, destinadas a assegurar os direitos relativos à saú- do, não se lhes aplicando o disposto no art. 150, III, «b».
de, à previdência e à assistência social. § 7º São isentas de contribuição para a seguridade
Parágrafo único. Compete ao Poder Público, nos termos social as entidades beneficentes de assistência social que
da lei, organizar a seguridade social, com base nos seguintes atendam às exigências estabelecidas em lei.
objetivos: § 8º O produtor, o parceiro, o meeiro e o arrendatá-
I - universalidade da cobertura e do atendimento; rio rurais e o pescador artesanal, bem como os respectivos
II - uniformidade e equivalência dos benefícios e serviços cônjuges, que exerçam suas atividades em regime de eco-
às populações urbanas e rurais; nomia familiar, sem empregados permanentes, contribui-
III - seletividade e distributividade na prestação dos be- rão para a seguridade social mediante a aplicação de uma
nefícios e serviços; alíquota sobre o resultado da comercialização da produção
IV - irredutibilidade do valor dos benefícios; e farão jus aos benefícios nos termos da lei. 
V - equidade na forma de participação no custeio; § 9º As contribuições sociais previstas no inciso I do
VI - diversidade da base de financiamento; caput deste artigo poderão ter alíquotas ou bases de cál-
VII - caráter democrático e descentralizado da adminis- culo diferenciadas, em razão da atividade econômica, da
tração, mediante gestão quadripartite, com participação dos utilização intensiva de mão-de-obra, do porte da empresa
trabalhadores, dos empregadores, dos aposentados e do Go- ou da condição estrutural do mercado de trabalho. 
verno nos órgãos colegiados. 

27
LEGISLAÇÃO

§ 10. A lei definirá os critérios de transferência de re- § 3º Lei complementar, que será reavaliada pelo menos
cursos para o sistema único de saúde e ações de assistên- a cada cinco anos, estabelecerá:
cia social da União para os Estados, o Distrito Federal e os I - os percentuais de que tratam os incisos II e III do § 2º;  
Municípios, e dos Estados para os Municípios, observada a II – os critérios de rateio dos recursos da União vincu-
respectiva contrapartida de recursos.  lados à saúde destinados aos Estados, ao Distrito Federal e
§ 11. É vedada a concessão de remissão ou anistia das aos Municípios, e dos Estados destinados a seus respectivos
contribuições sociais de que tratam os incisos I, a, e II deste Municípios, objetivando a progressiva redução das dispari-
artigo, para débitos em montante superior ao fixado em lei dades regionais; 
complementar.  III – as normas de fiscalização, avaliação e controle das
§ 12. A lei definirá os setores de atividade econômica despesas com saúde nas esferas federal, estadual, distrital e
para os quais as contribuições incidentes na forma dos in- municipal; 
cisos I, b; e IV do caput, serão não-cumulativas.  IV - (revogado).   
§ 13. Aplica-se o disposto no § 12 inclusive na hipótese § 4º Os gestores locais do sistema único de saúde po-
de substituição gradual, total ou parcial, da contribuição derão admitir agentes comunitários de saúde e agentes de
incidente na forma do inciso I, a, pela incidente sobre a combate às endemias por meio de processo seletivo pú-
receita ou o faturamento.  blico, de acordo com a natureza e complexidade de suas
atribuições e requisitos específicos para sua atuação.
Seção II § 5º Lei federal disporá sobre o regime jurídico, o piso
DA SAÚDE salarial profissional nacional, as diretrizes para os Planos
de Carreira e a regulamentação das atividades de agente
Art. 196. A saúde é direito de todos e dever do Estado, comunitário de saúde e agente de combate às endemias,
garantido mediante políticas sociais e econômicas que visem
competindo à União, nos termos da lei, prestar assistência
à redução do risco de doença e de outros agravos e ao acesso
financeira complementar aos Estados, ao Distrito Federal e
universal e igualitário às ações e serviços para sua promo-
aos Municípios, para o cumprimento do referido piso sa-
ção, proteção e recuperação.
larial.
§ 6º Além das hipóteses previstas no § 1º do art. 41 e
Art. 197. São de relevância pública as ações e serviços de
no § 4º do art. 169 da Constituição Federal, o servidor que
saúde, cabendo ao Poder Público dispor, nos termos da lei,
sobre sua regulamentação, fiscalização e controle, devendo exerça funções equivalentes às de agente comunitário de
sua execução ser feita diretamente ou através de terceiros saúde ou de agente de combate às endemias poderá per-
e, também, por pessoa física ou jurídica de direito privado. der o cargo em caso de descumprimento dos requisitos
específicos, fixados em lei, para o seu exercício. 
Art. 198. As ações e serviços públicos de saúde integram
uma rede regionalizada e hierarquizada e constituem um Art. 199. A assistência à saúde é livre à iniciativa priva-
sistema único, organizado de acordo com as seguintes di- da.
retrizes: § 1º As instituições privadas poderão participar de for-
I - descentralização, com direção única em cada esfera ma complementar do sistema único de saúde, segundo
de governo; diretrizes deste, mediante contrato de direito público ou
II - atendimento integral, com prioridade para as ati- convênio, tendo preferência as entidades filantrópicas e as
vidades preventivas, sem prejuízo dos serviços assistenciais; sem fins lucrativos.
III - participação da comunidade. § 2º É vedada a destinação de recursos públicos para
§ 1º O sistema único de saúde será financiado, nos ter- auxílios ou subvenções às instituições privadas com fins lu-
mos do art. 195, com recursos do orçamento da seguridade crativos.
social, da União, dos Estados, do Distrito Federal e dos Mu- § 3º - É vedada a participação direta ou indireta de em-
nicípios, além de outras fontes.  presas ou capitais estrangeiros na assistência à saúde no
§ 2º A União, os Estados, o Distrito Federal e os Muni- País, salvo nos casos previstos em lei.
cípios aplicarão, anualmente, em ações e serviços públicos § 4º A lei disporá sobre as condições e os requisitos
de saúde recursos mínimos derivados da aplicação de per- que facilitem a remoção de órgãos, tecidos e substâncias
centuais calculados sobre:  humanas para fins de transplante, pesquisa e tratamento,
I - no caso da União, a receita corrente líquida do res- bem como a coleta, processamento e transfusão de sangue
pectivo exercício financeiro, não podendo ser inferior a 15% e seus derivados, sendo vedado todo tipo de comerciali-
(quinze por cento);   zação.
II – no caso dos Estados e do Distrito Federal, o produto
da arrecadação dos impostos a que se refere o art. 155 e dos Art. 200. Ao sistema único de saúde compete, além de
recursos de que tratam os arts. 157 e 159, inciso I, alínea a, outras atribuições, nos termos da lei:
e inciso II, deduzidas as parcelas que forem transferidas aos I - controlar e fiscalizar procedimentos, produtos e subs-
respectivos Municípios;  tâncias de interesse para a saúde e participar da produção
III – no caso dos Municípios e do Distrito Federal, o pro- de medicamentos, equipamentos, imunobiológicos, hemo-
duto da arrecadação dos impostos a que se refere o art. 156 derivados e outros insumos;
e dos recursos de que tratam os arts. 158 e 159, inciso I, II - executar as ações de vigilância sanitária e epidemio-
alínea b e § 3º. lógica, bem como as de saúde do trabalhador;

28
LEGISLAÇÃO

III - ordenar a formação de recursos humanos na área Correlato à participação da comunidade no SUS, tem-
de saúde; se o artigo 198, §§ 4º, 5º e 6º, CF. 
IV - participar da formulação da política e da execução Não há prejuízo à atuação da iniciativa privada no
das ações de saneamento básico; campo da assistência à saúde, questão regulamentada no
V - incrementar, em sua área de atuação, o desenvolvi- artigo 199, CF. Do dispositivo depreende-se uma das ques-
mento científico e tecnológico e a inovação; tões mais polêmicas no âmbito do SUS, que é a comple-
VI - fiscalizar e inspecionar alimentos, compreendido o mentaridade do sistema por parte de instituições privadas,
controle de seu teor nutricional, bem como bebidas e águas mediante contrato ou convênio, desde que sem fins lucra-
para consumo humano; tivos por parte destas instituições. Em verdade, é muito co-
VII - participar do controle e fiscalização da produção, mum que hospitais de ensino de instituições particulares
transporte, guarda e utilização de substâncias e produtos com cursos na área de biológicas busquem este convênio,
psicoativos, tóxicos e radioativos; encontrando frequentemente entraves que não possuem
VIII - colaborar na proteção do meio ambiente, nele natureza jurídica, mas política.
compreendido o do trabalho. Finalizando a disciplina do direito à saúde na Constitui-
ção, que vem a ser complementada no âmbito infracons-
Com certeza, um dos direitos sociais mais invocados e titucional pela Lei nº 8.080 de 19 de setembro de 1990,
que mais necessitam de investimento estatal na atualidade prevê o artigo 200 as atribuições do SUS.
é o direito à saúde. Não coincidentemente, a maior parte
dos casos no Poder Judiciário contra o Estado envolvem a Seção III
invocação deste direito, diante da recusa do Poder públi- DA PREVIDÊNCIA SOCIAL
co em custear tratamentos médicos e cirúrgicos. Em que
pese a invocação da reserva do possível, o Judiciário tem Art. 201. A previdência social será organizada sob a
se guiado pelo entendimento de que devem ser reservados forma de regime geral, de caráter contributivo e de filiação
recursos suficientes para fornecer um tratamento adequa- obrigatória, observados critérios que preservem o equilíbrio
do a todos os nacionais. financeiro e atuarial, e atenderá, nos termos da lei, a: 
O direito à saúde, por seu turno, não tem apenas o I - cobertura dos eventos de doença, invalidez, morte e
aspecto repressivo, propiciando a cura de doenças, mas
idade avançada; 
também o preventivo. Sendo assim, o Estado deve desen-
II - proteção à maternidade, especialmente à gestante; 
volver políticas sociais e econômicas para reduzir o risco de
III - proteção ao trabalhador em situação de desempre-
doenças e agravos, bem como para propiciar o acesso uni-
go involuntário; 
versal e igualitário aos serviços voltado ao seu tratamento.
IV - salário-família e auxílio-reclusão para os dependen-
(art. 196, CF).
tes dos segurados de baixa renda; 
A terceirização e a colaboração de agentes privados
nas políticas de saúde pública é autorizada pela Constitui- V - pensão por morte do segurado, homem ou mulher,
ção, sem prejuízo da atuação direta do Estado (art. 197, CF). ao cônjuge ou companheiro e dependentes, observado o dis-
Sendo assim, ou o próprio Estado implementará as políti- posto no § 2º. 
cas ou fiscalizará, regulamentará e controlará a implemen- § 1º É vedada a adoção de requisitos e critérios diferen-
tação destas por terceiros. ciados para a concessão de aposentadoria aos beneficiá-
O artigo 198, CF aborda o sistema único de saúde, uma rios do regime geral de previdência social, ressalvados os
rede hierarquizada e regionalizada de ações e serviços casos de atividades exercidas sob condições especiais que
públicos de saúde, devendo seguiras seguintes diretrizes: prejudiquem a saúde ou a integridade física e quando se
“descentralização, com direção única em cada esfera de tratar de segurados portadores de deficiência, nos termos
governo”, de forma que haverá direção do SUS nos âm- definidos em lei complementar. 
bitos municipal, estadual e federal, não se concentrando § 2º Nenhum benefício que substitua o salário de con-
o sistema numa única esfera; “atendimento integral, com tribuição ou o rendimento do trabalho do segurado terá
prioridade para as atividades preventivas, sem prejuízo dos valor mensal inferior ao salário mínimo. 
serviços assistenciais”, do que se depreende que a preven- § 3º Todos os salários de contribuição considerados
ção é a melhor saída para um sistema eficaz, não havendo para o cálculo de benefício serão devidamente atualizados,
prejuízo para as atividades repressivas; e “participação da na forma da lei. 
comunidade”. Com efeito, busca-se pela descentralização a § 4º É assegurado o reajustamento dos benefícios para
abrangência ampla dos serviços de saúde, que devem em preservar-lhes, em caráter permanente, o valor real, con-
si também ser amplos – preventivos e repressivos, sendo forme critérios definidos em lei.
que todos agentes públicos e a própria comunidade de- § 5º É vedada a filiação ao regime geral de previdên-
vem se envolver no processo. cia social, na qualidade de segurado facultativo, de pessoa
O direito à saúde encontra regulamentação no âmbito participante de regime próprio de previdência. 
da seguridade social, que também abrange a previdência e § 6º A gratificação natalina dos aposentados e pensio-
a assistência social, sendo financiado com este orçamento, nistas terá por base o valor dos proventos do mês de de-
nos moldes do artigo 198, §1º, CF.  zembro de cada ano. 
A questão orçamentária de incumbência mínima de § 7º É assegurada aposentadoria no regime geral de
cada um dos entes federados tem escopo nos §§ 2º e 3º previdência social, nos termos da lei, obedecidas as seguin-
do artigo 198, CF.  tes condições: 

29
LEGISLAÇÃO

I - trinta e cinco anos de contribuição, se homem, e trinta § 4º Lei complementar disciplinará a relação entre a
anos de contribuição, se mulher;  União, Estados, Distrito Federal ou Municípios, inclusive
II - sessenta e cinco anos de idade, se homem, e sessenta suas autarquias, fundações, sociedades de economia mista
anos de idade, se mulher, reduzido em cinco anos o limite e empresas controladas direta ou indiretamente, enquanto
para os trabalhadores rurais de ambos os sexos e para os patrocinadoras de entidades fechadas de previdência pri-
que exerçam suas atividades em regime de economia fami- vada, e suas respectivas entidades fechadas de previdência
liar, nestes incluídos o produtor rural, o garimpeiro e o pes- privada. 
cador artesanal.  § 5º A lei complementar de que trata o parágrafo ante-
§ 8º Os requisitos a que se refere o inciso I do parágra- rior aplicar-se-á, no que couber, às empresas privadas per-
fo anterior serão reduzidos em cinco anos, para o professor
missionárias ou concessionárias de prestação de serviços
que comprove exclusivamente tempo de efetivo exercício
públicos, quando patrocinadoras de entidades fechadas de
das funções de magistério na educação infantil e no ensino
fundamental e médio.  previdência privada. 
§ 9º Para efeito de aposentadoria, é assegurada a con- § 6º A lei complementar a que se refere o § 4° des-
tagem recíproca do tempo de contribuição na administra- te artigo estabelecerá os requisitos para a designação dos
ção pública e na atividade privada, rural e urbana, hipótese membros das diretorias das entidades fechadas de previ-
em que os diversos regimes de previdência social se com- dência privada e disciplinará a inserção dos participantes
pensarão financeiramente, segundo critérios estabelecidos nos colegiados e instâncias de decisão em que seus inte-
em lei. resses sejam objeto de discussão e deliberação. 
§ 10. Lei disciplinará a cobertura do risco de acidente
do trabalho, a ser atendida concorrentemente pelo regime A previdência social e a assistência social se diferen-
geral de previdência social e pelo setor privado.  ciam principalmente porque a previdência social volta-se
§ 11. Os ganhos habituais do empregado, a qualquer ao pagamento de aposentadoria e benefícios aos seus con-
título, serão incorporados ao salário para efeito de contri- tribuintes, ao passo que a assistência social tem por foco a
buição previdenciária e consequente repercussão em be- oferta de amparo mínimo aos que não contribuíram para a
nefícios, nos casos e na forma da lei.  seguridade social.
§ 12. Lei disporá sobre sistema especial de inclusão O artigo 201, CF, trabalha com a organização da previ-
previdenciária para atender a trabalhadores de baixa renda dência social em regime geral, de caráter contributivo e fi-
e àqueles sem renda própria que se dediquem exclusiva-
liação obrigatória, sendo que devem ser adotados critérios
mente ao trabalho doméstico no âmbito de sua residência,
de preservação de equilíbrio financeiro e atuarial. Nota-se
desde que pertencentes a famílias de baixa renda, garan-
tindo-lhes acesso a benefícios de valor igual a um salário- que todos os trabalhadores ficarão vinculados ao regime e
mínimo.  prestarão contribuição a ele, não havendo a opção de dele
§ 13. O sistema especial de inclusão previdenciária de se desvincular. No mais, são previstas como campos de
que trata o § 12 deste artigo terá alíquotas e carências in- atendimento pela previdência: “I - cobertura dos eventos
feriores às vigentes para os demais segurados do regime de doença, invalidez, morte e idade avançada; II - proteção
geral de previdência social.  à maternidade, especialmente à gestante;  III - proteção
ao trabalhador em situação de desemprego involuntário;
Art. 202. O regime de previdência privada, de caráter IV - salário-família e auxílio-reclusão para os dependentes
complementar e organizado de forma autônoma em relação dos segurados de baixa renda; V - pensão por morte do
ao regime geral de previdência social, será facultativo, ba- segurado, homem ou mulher, ao cônjuge ou companheiro
seado na constituição de reservas que garantam o benefício e dependentes, observado o disposto no § 2º” (ou seja, não
contratado, e regulado por lei complementar.  se aceitando valor inferior ao salário mínimo).
§ 1° A lei complementar de que trata este artigo asse- Os critérios para a concessão de aposentadoria são
gurará ao participante de planos de benefícios de entida- unitários, em regra, conforme o §1º do artigo 201, CF.
des de previdência privada o pleno acesso às informações O valor mínimo de benefício com caráter substitutivo
relativas à gestão de seus respectivos planos. 
de salário de contribuição ou rendimento é de 1 salário
§ 2° As contribuições do empregador, os benefícios
mínimo (artigo 201, §2º, CF).
e as condições contratuais previstas nos estatutos, regu-
lamentos e planos de benefícios das entidades de previ- Os salários de contribuição serão atualizados (artigo
dência privada não integram o contrato de trabalho dos 201, §3º, CF) e os benefícios serão devidamente reajusta-
participantes, assim como, à exceção dos benefícios conce- dos (artigo 201, §4º, CF), tudo com vistas à preservação do
didos, não integram a remuneração dos participantes, nos valor real da contribuição e do benefício.
termos da lei.  Integrante de regime próprio de previdência não pode
§ 3º É vedado o aporte de recursos a entidade de pre- se vincular como segurado facultativo, prestando contri-
vidência privada pela União, Estados, Distrito Federal e buições autônomas, ao regime geral (artigo 201, §5º, CF), o
Municípios, suas autarquias, fundações, empresas públicas, que geraria uma indevida cumulação de benefícios.
sociedades de economia mista e outras entidades públicas, Aposentados e pensionistas também fazem jus ao dé-
salvo na qualidade de patrocinador, situação na qual, em cimo terceiro salário, denominado gratificação natalina, a
hipótese alguma, sua contribuição normal poderá exceder ser calculado com base no valor dos proventos do mês de
a do segurado.  dezembro de cada ano (artigo 201, §6º, CF). 

30
LEGISLAÇÃO

O §7º do artigo 201, CF fixa as condições para a apo- Parágrafo único. É facultado aos Estados e ao Distrito
sentadoria pelo regime geral de previdência social. Profes- Federal vincular a programa de apoio à inclusão e promoção
sor de ensino infantil, fundamental e médio, que tenha ex- social até cinco décimos por cento de sua receita tributária
clusivamente desempenhado estas funções, tem o tempo líquida, vedada a aplicação desses recursos no pagamento
de contribuição reduzido em 5 anos (30 anos para homem de: 
e 25 anos para mulher). I - despesas com pessoal e encargos sociais; 
Se uma pessoa contribuir a dois regimes diversos em II - serviço da dívida; 
períodos diferentes de sua vida contributiva, estes regimes III - qualquer outra despesa corrente não vinculada dire-
se compensarão, ou seja, o tempo de um se acrescerá no tamente aos investimentos ou ações apoiados. 
outro (artigo 201, §9º, CF).
A questão de verba destinada à cobertura do risco de A disciplina da assistência social se dá nos artigos 203 e
acidente de trabalho é disciplinada no §10 do artigo 201, 204 da Constituição. Resta evidente o caráter não contribu-
CF. Atualmente, a Lei nº 6.367/1976 dispõe sobre o seguro tivo do sistema, que se guia pelo princípio da fraternidade,
de acidentes do trabalho a cargo do INPS e dá outras pro- fazendo com que os que possuem melhores condições de
vidências. contribuir o façam e que os que não possuem recebam a
Quanto à incorporação de ganhos habituais ao salário, partir da contribuição destes um tratamento digno mínimo
prevê o §11 do artigo 201, CF pela incorporação para efeito de suas necessidades.
de contribuição previdenciária e consequente repercussão Do disposto, destaque para o inciso IV do artigo 204,
em benefícios, nos casos e na forma da lei. CF, que aborda o Benefício de Prestação Continuada – BPC,
Sobre o sistema especial de inclusão previdenciária, é a “instituído pela Constituição Federal de 1988 e regulamen-
disciplina do artigo 201, §§ 12 e 13, CF. tado pela Lei Orgânica da Assistência Social – LOAS, Lei nº
Por seu turno, o artigo 202, CF volta-se ao regime de 8.742/1993; pelas Leis nº 12.435/2011 e nº 12.470/2011,
previdência privada, que pode se organizar de forma autô- que alteram dispositivos da LOAS e pelos Decretos nº
noma e possui caráter complementar e facultativo. 6.214/2007 e nº 6.564/2008.
Com efeito, a Lei Complementar nº 109, de 29 de maio O BPC é um benefício da Política de Assistência Social,
de 2001, dispõe sobre o Regime de Previdência Comple- que integra a Proteção Social Básica no âmbito do Sistema
mentar e dá outras providências. Único de Assistência Social – SUAS e para acessá-lo não é
necessário ter contribuído com a Previdência Social. É um
Seção IV benefício individual, não vitalício e intransferível, que asse-
DA ASSISTÊNCIA SOCIAL gura a transferência mensal de 1 (um) salário mínimo ao
idoso, com 65 (sessenta e cinco) anos ou mais, e à pessoa
Art. 203. A assistência social será prestada a quem dela com deficiência, de qualquer idade, com impedimentos de
necessitar, independentemente de contribuição à seguridade longo prazo, de natureza física, mental, intelectual ou sen-
social, e tem por objetivos: sorial, os quais, em interação com diversas barreiras, po-
I - a proteção à família, à maternidade, à infância, à dem obstruir sua participação plena e efetiva na sociedade
adolescência e à velhice; em igualdade de condições com as demais pessoas. Em
II - o amparo às crianças e adolescentes carentes; ambos os casos, devem comprovar não possuir meios de
III - a promoção da integração ao mercado de trabalho; garantir o próprio sustento, nem tê-lo provido por sua fa-
IV - a habilitação e reabilitação das pessoas portadoras mília. A renda mensal familiar per capita deve ser inferior a
de deficiência e a promoção de sua integração à vida comu- 1/4 (um quarto) do salário mínimo vigente.
nitária; A gestão do BPC é realizada pelo Ministério do De-
V - a garantia de um salário mínimo de benefício men- senvolvimento Social e Combate à Fome (MDS), por inter-
sal à pessoa portadora de deficiência e ao idoso que com- médio da Secretaria Nacional de Assistência Social (SNAS),
provem não possuir meios de prover à própria manutenção que é responsável pela implementação, coordenação, re-
ou de tê-la provida por sua família, conforme dispuser a lei. gulação, financiamento, monitoramento e avaliação do
Benefício. A operacionalização é realizada pelo Instituto
Art. 204. As ações governamentais na área da assistên- Nacional do Seguro Social (INSS).
cia social serão realizadas com recursos do orçamento da se- Os recursos para o custeio do BPC provêm da Segu-
guridade social, previstos no art. 195, além de outras fontes, ridade Social, sendo administrado pelo MDS e repassado
e organizadas com base nas seguintes diretrizes: ao INSS, por meio do Fundo Nacional de Assistência Social
I - descentralização político-administrativa, cabendo a (FNAS). Atualmente são 3,6 milhões (dados de março de
coordenação e as normas gerais à esfera federal e a coor- 2012) beneficiários do BPC em todo o Brasil, sendo 1,9 mi-
denação e a execução dos respectivos programas às esferas lhões pessoas com deficiência e 1,7 idosos”30.
estadual e municipal, bem como a entidades beneficentes e
de assistência social;
II - participação da população, por meio de organiza-
ções representativas, na formulação das políticas e no con-
trole das ações em todos os níveis. 30 http://www.mds.gov.br/assistenciasocial/benefi-
ciosassistenciais/bpc

31
LEGISLAÇÃO

§ 2º A lei disporá sobre normas de construção dos lo-


gradouros e dos edifícios de uso público e de fabricação
CAPÍTULO VII de veículos de transporte coletivo, a fim de garantir acesso
DA FAMÍLIA, DA CRIANÇA, DO adequado às pessoas portadoras de deficiência.
ADOLESCENTE, DO JOVEM E DO IDOSO § 3º O direito a proteção especial abrangerá os seguin-
tes aspectos:
I - idade mínima de quatorze anos para admissão ao
trabalho, observado o disposto no art. 7º, XXXIII;
CAPÍTULO VII II - garantia de direitos previdenciários e trabalhistas;
Da Família, da Criança, do Adolescente, do Jovem III - garantia de acesso do trabalhador adolescente e jo-
e do Idoso vem à escola; 
IV - garantia de pleno e formal conhecimento da atri-
Art. 226. A família, base da sociedade, tem especial pro- buição de ato infracional, igualdade na relação processual e
teção do Estado. defesa técnica por profissional habilitado, segundo dispuser
§ 1º O casamento é civil e gratuita a celebração. a legislação tutelar específica;
§ 2º O casamento religioso tem efeito civil, nos termos V - obediência aos princípios de brevidade, excepciona-
da lei. lidade e respeito à condição peculiar de pessoa em desenvol-
§ 3º Para efeito da proteção do Estado, é reconhecida a vimento, quando da aplicação de qualquer medida privativa
união estável entre o homem e a mulher como entidade fa- da liberdade;
miliar, devendo a lei facilitar sua conversão em casamento. VI - estímulo do Poder Público, através de assistência
§ 4º Entende-se, também, como entidade familiar a jurídica, incentivos fiscais e subsídios, nos termos da lei, ao
comunidade formada por qualquer dos pais e seus des- acolhimento, sob a forma de guarda, de criança ou adoles-
cendentes. cente órfão ou abandonado;
§ 5º Os direitos e deveres referentes à sociedade con- VII - programas de prevenção e atendimento especia-
jugal são exercidos igualmente pelo homem e pela mulher. lizado à criança, ao adolescente e ao jovem dependente de
§ 6º O casamento civil pode ser dissolvido pelo divór- entorpecentes e drogas afins.
cio.  § 4º A lei punirá severamente o abuso, a violência e a
§ 7º Fundado nos princípios da dignidade da pessoa exploração sexual da criança e do adolescente.
humana e da paternidade responsável, o planejamento fa- § 5º A adoção será assistida pelo Poder Público, na for-
miliar é livre decisão do casal, competindo ao Estado pro- ma da lei, que estabelecerá casos e condições de sua efeti-
piciar recursos educacionais e científicos para o exercício vação por parte de estrangeiros.
desse direito, vedada qualquer forma coercitiva por parte § 6º Os filhos, havidos ou não da relação do casamento,
de instituições oficiais ou privadas. ou por adoção, terão os mesmos direitos e qualificações,
§ 8º O Estado assegurará a assistência à família na pes- proibidas quaisquer designações discriminatórias relativas
soa de cada um dos que a integram, criando mecanismos à filiação.
para coibir a violência no âmbito de suas relações. § 7º No atendimento dos direitos da criança e do ado-
lescente levar-se- á em consideração o disposto no art. 204.
Art. 227. É dever da família, da sociedade e do Estado as- § 8º A lei estabelecerá: 
segurar à criança, ao adolescente e ao jovem, com absoluta I - o estatuto da juventude, destinado a regular os direi-
prioridade, o direito à vida, à saúde, à alimentação, à educa- tos dos jovens; 
ção, ao lazer, à profissionalização, à cultura, à dignidade, ao II - o plano nacional de juventude, de duração decenal,
respeito, à liberdade e à convivência familiar e comunitária, visando à articulação das várias esferas do poder público
além de colocá-los a salvo de toda forma de negligência, para a execução de políticas públicas. 
discriminação, exploração, violência, crueldade e opressão. 
§ 1º O Estado promoverá programas de assistência Art. 228. São penalmente inimputáveis os menores de
integral à saúde da criança, do adolescente e do jovem, dezoito anos, sujeitos às normas da legislação especial.
admitida a participação de entidades não governamentais,
mediante políticas específicas e obedecendo aos seguintes Art. 229. Os pais têm o dever de assistir, criar e educar os
preceitos:  filhos menores, e os filhos maiores têm o dever de ajudar e
I - aplicação de percentual dos recursos públicos desti- amparar os pais na velhice, carência ou enfermidade.
nados à saúde na assistência materno-infantil;
II - criação de programas de prevenção e atendimento Art. 230. A família, a sociedade e o Estado têm o dever
especializado para as pessoas portadoras de deficiência físi- de amparar as pessoas idosas, assegurando sua participação
ca, sensorial ou mental, bem como de integração social do na comunidade, defendendo sua dignidade e bem-estar e
adolescente e do jovem portador de deficiência, mediante o garantindo-lhes o direito à vida.
treinamento para o trabalho e a convivência, e a facilitação § 1º Os programas de amparo aos idosos serão execu-
do acesso aos bens e serviços coletivos, com a eliminação de tados preferencialmente em seus lares.
obstáculos arquitetônicos e de todas as formas de discrimi- § 2º Aos maiores de sessenta e cinco anos é garantida
nação.  a gratuidade dos transportes coletivos urbanos.

32
LEGISLAÇÃO

Em que pese o artigo 6º, CF mencionar exclusivamente Explica Liberati31: “Por absoluta prioridade, devemos
a proteção à maternidade e à infância, o constituinte deixa entender que a criança e o adolescente deverão estar em
clara sua intenção de proteger todos os núcleos vulnerá- primeiro lugar na escala de preocupação dos governan-
veis da família: a proteção da mãe envolve a proteção da tes; devemos entender que, primeiro, devem ser atendidas
família, a proteção da infância abrange também a do ado- todas as necessidades das crianças e adolescentes [...]. Por
lescente e do jovem e, porque não dizer, dos idosos, já que absoluta prioridade, entende-se que, na área administrativa,
estes também estão numa idade em que cuidado especial enquanto não existirem creches, escolas, postos de saúde,
deve ser despendido. Em verdade, há associações que tra- atendimento preventivo e emergencial às gestantes dignas
balham com a proteção integrada de todos estes núcleos moradias e trabalho, não se deveria asfaltar ruas, construir
vulneráveis. praças, sambódromos monumentos artísticos etc., porque a
vida, a saúde, o lar, a prevenção de doenças são importantes
1) Proteção à maternidade e à infância que as obras de concreto que ficam par a demonstrar o po-
Envolve medidas de seguridade e assistência social e der do governante”.
outras voltadas à mãe e ao seu filho, para que ele cresça de A Lei nº 8.069, de 13 de julho de 1990 dispõe sobre o Es-
maneira saudável e na presença do seio materno. tatuto da Criança e do Adolescente e dá outras providências,
seguindo em seus dispositivos a ideologia do princípio da
absoluta prioridade.
2) Proteção da família
No §1º do artigo 227 aborda-se a questão da assistência
O constituinte reconhece a família como base da so-
à saúde da criança e do adolescente. Do inciso I se depreen-
ciedade e a coloca como objeto de especial proteção do de a intrínseca relação entre a proteção da criança e do ado-
Estado já no caput do artigo 226, CF. lescente com a proteção da maternidade e da infância, men-
A fundação tradicional da família se dá pelo casamen- cionada no artigo 6º, CF. Já do inciso II se depreende a pro-
to, de modo que os §§ 1º e 2º do artigo 226 estabelecem: teção de outro grupo vulnerável, que é a pessoa portadora
“§ 1º O casamento é civil e gratuita a celebração. § 2º O de deficiência, valendo lembrar que o Decreto nº 6.949, de 25
casamento religioso tem efeito civil, nos termos da lei”. O de agosto de 2009, que promulga a Convenção Internacional
fato do casamento ser gratuito facilita o acesso a ele. Por sobre os Direitos das Pessoas com Deficiência e seu Protoco-
seu turno, a previsão do casamento civil, aceitando que o lo Facultativo, assinados em Nova York, em 30 de março de
religioso tenha o mesmo efeito, consolida a afirmação do 2007, foi promulgado após aprovação no Congresso Nacio-
Estado laico. Aliás, Lei nº 1.110/1950 regula o reconheci- nal nos moldes da Emenda Constitucional nº 45/2004, tendo
mento dos efeitos civis ao casamento religioso. força de norma constitucional e não de lei ordinária. A preo-
Nos §§ 5º e 6º do artigo 226, CF, mais uma vez, o cons- cupação com o direito da pessoa portadora de deficiência se
tituinte aborda o casamento. Estabelece-se, primeiro, a estende ao §2º do artigo 227, CF: “a lei disporá sobre normas
igualdade entre homem e mulher no casamento e depois de construção dos logradouros e dos edifícios de uso pú-
a possibilidade de dissolução do vínculo conjugal. Quanto blico e de fabricação de veículos de transporte coletivo, a
à dissolução do vínculo conjugal, ressalta-se que não mais fim de garantir acesso adequado às pessoas portadoras de
se exige um lapso temporal entre a separação de fato ou deficiência”.
judicial para a concessão do divórcio, sendo que o divórcio A proteção especial que decorre do princípio da prio-
pode ser postulado desde logo (é o que decorre da Emen- ridade absoluta está prevista no §3º do artigo 227. Liga-se,
da Constitucional nº 66/2010). ainda, à proteção especial, a previsão do §4º do artigo 227:
No entanto, o constituinte está ciente de que o casa- “A lei punirá severamente o abuso, a violência e a exploração
mento não é a única forma de se constituir família, pre- sexual da criança e do adolescente”.
vendo no § 3º o reconhecimento da união estável como Tendo em vista o direito de toda criança e adolescente
entidade familiar e no § 4º o reconhecimento da família de ser criado no seio de uma família, o §5º do artigo 227 da
Constituição prevê que “a adoção será assistida pelo Poder
monoparental.
Público, na forma da lei, que estabelecerá casos e condições
O direito ao planejamento familiar é abordado no §7º
de sua efetivação por parte de estrangeiros”. Neste sentido, a
do artigo 226, CF.
Lei nº 12.010, de 3 de agosto de 2009, dispõe sobre a adoção.
Por fim, o constituinte denota preocupação com a vio- A igualdade entre os filhos, quebrando o paradigma da
lência doméstica, prevendo no §8º do artigo 226. Exemplo Constituição anterior e do até então vigente Código Civil de
de consolidação deste ideário é a Lei nº 11.340/2006, Lei 1916 consta no artigo 227, § 6º, CF: “os filhos, havidos ou não
Maria da Penha, que coíbe a violência doméstica e familiar da relação do casamento, ou por adoção, terão os mesmos
praticada contra a mulher. direitos e qualificações, proibidas quaisquer designações dis-
criminatórias relativas à filiação”.
3) Proteção da criança, do adolescente e do jovem Quando o artigo 227 dispõe no § 7º que “no atendimen-
Já no caput do artigo 227, CF se encontra uma das to dos direitos da criança e do adolescente levar-se-á em
principais diretrizes do direito da criança e do adolescente consideração o disposto no art. 204” tem em vista a adoção
que é o princípio da prioridade absoluta. Significa que cada de práticas de assistência social, com recursos da segurida-
criança e adolescente deve receber tratamento especial do de social, em prol da criança e do adolescente.
Estado e ser priorizado em suas políticas públicas, pois são
o futuro do país e as bases de construção da sociedade. 31 LIBERATI, Wilson Donizeti. O Estatuto da Crian-
ça e do Adolescente: Comentários. São Paulo: IBPS.

33
LEGISLAÇÃO

Por seu turno, o artigo 227, § 8º, CF, preconiza: “A lei IV - São invioláveis a intimidade, a vida privada, a honra
estabelecerá: I - o estatuto da juventude, destinado a re- e a imagem das pessoas, assegurado o direito à indeniza-
gular os direitos dos jovens; II - o plano nacional de juven- ção pelo dano material ou moral decorrente de sua viola-
tude, de duração decenal, visando à articulação das várias ção.
esferas do poder público para a execução de políticas pú- V - É inviolável a liberdade de consciência e de crença,
blicas”. A Lei nº 12.852, de 5 de agosto de 2013, institui sendo assegurado o livre exercício dos cultos religiosos e
o Estatuto da Juventude e dispõe sobre os direitos dos garantida, na forma da lei, a proteção aos locais de culto e
jovens, os princípios e diretrizes das políticas públicas de suas liturgias.
juventude e o Sistema Nacional de Juventude - SINAJUVE. (A) Apenas I, II e III estão corretas.
Mais informações sobre a Política mencionada no inciso II (B) Apenas II, III e IV estão corretas.
e sobre a Secretaria e o Conselho Nacional de Juventude (C) Apenas III e V estão corretas.
que direcionam a implementação dela podem ser obtidas (D) Apenas IV e V estão corretas.
na rede32. (E) Todas as questões estão corretas.
O artigo 228, CF dispõe: “são penalmente inimputáveis
os menores de dezoito anos, sujeitos às normas da legis-
2. (PC/SC - Agente de Polícia - ACAFE/2014) Os re-
lação especial”. Percebe-se que a normativa não está no
médios constitucionais são as formas estabelecidas pela
rol de cláusulas pétreas, razão pela qual seria possível uma
Constituição Federal para concretizar e proteger os direitos
emenda constitucional que alterasse a menoridade penal.
Inclusive, há projetos de lei neste sentido. fundamentais a fim de que sejam assegurados os valores
essenciais e indisponíveis do ser humano.
4) Proteção do idoso Assim, é correto afirmar, exceto:
A segunda parte do artigo 229, CF preconiza que “[...] (A) O habeas corpus pode ser formulado sem advo-
os filhos maiores têm o dever de ajudar e amparar os pais gado, não tendo de obedecer a qualquer formalidade pro-
na velhice, carência ou enfermidade”. Consolida o dever de cessual, e o próprio cidadão prejudicado pode ser o autor.
solidariedade familiar, compensando os pais que criaram (B) O habeas corpus é utilizado sempre que alguém
seus filhos quando tinham condições e que hoje se encon- sofrer ou se achar ameaçado de sofrer violência ou coação
tram na posição de necessitados. Contudo, este dever de em sua liberdade de locomoção, por ilegalidade ou abuso
amparo não é exclusivo da família, conforme se extrai do de poder.
artigo 230, CF. (C) O autor da ação constitucional de habeas corpus
O fato de uma pessoa ter se tornado idosa não a trans- recebe o nome de impetrante; o indivíduo em favor do qual
forma numa parte dispensável da sociedade, que merece se impetra, paciente, podendo ser o mesmo impetrante, e
isolamento. Pelo contrário, suas experiências devem ser va- a autoridade que pratica a ilegalidade, autoridade coatora.
lorizadas e incorporadas nas práticas sociais, tornando-as (D) Caberá habeas corpus em relação a punições dis-
mais adequadas. Assim, Estado, família e sociedade pos- ciplinares militares.
suem o dever compartilhado de conferir assistência aos (E) O habeas corpus será preventivo quando alguém
idosos. se achar ameaçado de sofrer violência, ou repressivo,
quando for concreta a lesão.
EXERCÍCIOS
3. (PC/SC - Agente de Polícia - ACAFE/2014) Ainda
1. (PC/SC - Agente de Polícia - ACAFE/2014) O art. em relação aos outros remédios constitucionais analise as
5º da Constituição Federal trata dos direitos e deveres in- questões a seguir e assinale a alternativa correta.
dividuais e coletivos, espécie do gênero direitos e garan- I - O habeas data assegura o conhecimento de infor-
tias fundamentais (Título II). Assim, apesar de referir-se,
mações relativas à pessoa do impetrante, constantes de
de modo expresso, apenas a direitos e deveres, também
registros ou banco de dados de entidades governamentais
consagrou as garantias fundamentais. (LENZA, Pedro. Di-
ou de caráter público.
reito Constitucional Esquematizado, São Paulo: Saraiva,
2009,13ª. ed., p. 671). II - Será concedido habeas data para a retificação de
Com base na afirmação acima, analise as questões a dados, quando não se prefira fazê-lo por processo sigiloso,
seguir e assinale a alternativa correta. judicial ou administrativo.
I - Os direitos são bens e vantagens prescritos na nor- III - Em se tratando de registro ou banco de dados de
ma constitucional, enquanto as garantias são os instrumen- entidade governamental, o sujeito passivo na ação de ha-
tos através dos quais se assegura o exercício dos aludidos beas data será a pessoa jurídica componente da adminis-
direitos. tração direta e indireta do Estado.
II - O rol dos direitos expressos nos 78 incisos e pa- IV - O mandado de injunção serve para requerer à au-
rágrafos do art. 5º da Constituição Federal é meramente toridade competente que faça uma lei para tornar viável o
exemplificativo. exercício dos direitos e liberdades constitucionais.
III - Os direitos e garantias expressos na Constituição V - O pressuposto lógico do mandado de injunção é a
Federal não excluem outros decorrentes do regime e dos demora legislativa que impede um direito de ser efetivado
princípios por ela adotados, ou dos tratados internacionais pela falta de complementação de uma lei.
em que o Brasil seja parte. (A) Todas as afirmações estão corretas.
32 http://www.juventude.gov.br/politica (B) Apenas I, II e III estão corretas.

34
LEGISLAÇÃO

(C) Apenas II, III e IV estão corretas. 8. (PC/MG - Investigador de Polícia - FUMARC/2014)
(D) Apenas II, III e V estão corretas. A casa é asilo inviolável do indivíduo, podendo-se nela en-
(E) Apenas IV e V estão corretas. trar, sem permissão do morador, EXCETO
(A) em caso de desastre.
4. (PC/SC - Agente de Polícia - ACAFE/2014) O de- (B) em caso de flagrante delito.
vido processo legal estabelecido como direito do cidadão (C) para prestar socorro.
na Constituição Federal configura dupla proteção ao indi- (D) por determinação judicial, a qualquer hora.
víduo, pois atua no âmbito material de proteção ao direito
de liberdade e no âmbito formal, ao assegurar-lhe parida- 9. (Prefeitura de Florianópolis/SC - Administrador -
de de condições com o Estado para defender-se. FGV/2014) Em tema de direitos e garantias fundamentais,
Com base na afirmação acima, analise as questões a o artigo 5º da Constituição da República estabelece que é:
seguir e assinale a alternativa correta. (A) livre a manifestação do pensamento, sendo fo-
I - Ninguém será processado nem sentenciado senão mentado o anonimato;
pela autoridade competente. (B) assegurado o direito de resposta, proporcional ao
II - A lei só poderá restringir a publicidade dos atos agravo, que substitui o direito à indenização por dano ma-
processuais quando a defesa da intimidade ou o interesse terial, moral ou à imagem;
social o exigirem. (C) assegurado a todos o acesso à informação e res-
III - São admissíveis, no processo, as provas obtidas por
guardado o sigilo da fonte, quando necessário ao exercício
meios ilícitos.
profissional;
IV - Ninguém será levado à prisão ou nela mantido,
(D) livre a expressão da atividade intelectual, artística,
quando a lei admitir a liberdade provisória, com ou sem
científica e de comunicação, ressalvados os casos de cen-
fiança.
sura ou licença;
V - Não haverá prisão civil por dívida, nem mesmo a do
(E) direito de todos receber dos órgãos públicos in-
depositário infiel.
(A) Apenas I, II e IV estão corretas. formações de seu interesse particular, sendo vedada a
(B) Apenas I, III e V estão corretas. alegação de sigilo por imprescindibilidade à segurança da
(C) Apenas III e IV estão corretas. sociedade e do Estado.
(D) Apenas IV e V estão corretas.
(E) Todas as questões estão corretas. 10. (TJ-RJ - Técnico de Atividade Judiciária -
FGV/2014) A partir da Emenda Constitucional nº 45/2004,
5. (PC/MG - Investigador de Polícia - FUMARC/2014) os tratados e convenções internacionais sobre direitos hu-
Sobre a Lei Penal, é CORRETO afirmar que manos:
(A) não retroage, salvo para beneficiar o réu. (A) sempre terão a natureza jurídica de lei, exigindo a
(B) não retroage, salvo se o fato criminoso ainda não sua aprovação, pelo Congresso Nacional e a promulgação,
for conhecido. na ordem interna, pelo Chefe do Poder Executivo;
(C) retroage, salvo disposição expressa em contrário. (B) sempre terão a natureza jurídica de emenda cons-
(D) retroage, se ainda não houver processo penal ins- titucional, exigindo, apenas, que a sua aprovação, pelo
taurado. Congresso Nacional, se dê em dois turnos de votação, com
o voto favorável de dois terços dos respectivos membros;
6. (PC/MG - Investigador de Polícia - FUMARC/2014) (C) podem ter a natureza jurídica de emenda constitu-
Sobre as garantias fundamentais estabelecidas na Consti- cional, desde que a sua aprovação, pelo Congresso Nacio-
tuição Federal, é CORRETO afirmar que nal, se dê em dois turnos de votação, com o voto favorável
(A) a Lei Penal é sempre irretroativa. de três quintos dos respectivos membros;
(B) a prática do racismo constitui crime inafiançável e (D) podem ter a natureza jurídica de lei complemen-
imprescritível. tar, desde que o Congresso Nacional venha a aprová-los
(C) não haverá pena de morte em nenhuma circuns- com observância do processo legislativo ordinário;
tância. (E) sempre terão a natureza jurídica de atos de direito
(D) os templos religiosos, entendidos como casas de internacional, não se integrando, em qualquer hipótese, à
Deus, possuem garantia de inviolabilidade domiciliar. ordem jurídica interna.

7. (PC/MG - Investigador de Polícia - FUMARC/2014) 11. (OAB - Exame de Ordem Unificado - FGV/2014)
NÃO figura entre as garantias expressas no artigo 5º da Pedro promoveu ação em face da União Federal e seu
Constituição Federal: pedido foi julgado procedente, com efeitos patrimoniais
(A) a obtenção de certidões em repartições públicas. vencidos e vincendos, não havendo mais recurso a ser in-
(B) a defesa do consumidor, prevista em estatuto pró- terposto. Posteriormente, o Congresso Nacional aprovou
prio. lei, que foi sancionada, extinguindo o direito reconhecido
(C) o respeito à integridade física dos presos, garanti- a Pedro. Após a publicação da referida lei, a Administração
do pela lei de execução penal. Pública federal notificou Pedro para devolver os valores re-
(D) a remuneração do trabalho noturno superior ao cebidos, comunicando que não mais ocorreriam os paga-
diurno, posto que contido na legislação ordinária trabalhis- mentos futuros, em decorrência da norma em foco.
ta.

35
LEGISLAÇÃO

Nos termos da Constituição Federal, assinale a opção (D) é sinônima, em significado e extensão, à teoria do
correta mínimo existencial, examinado à luz da violação dos direi-
(A) A lei não pode retroagir, porque a situação versa tos fundamentais sociais, culturais e econômicos, como o
sobre direitos indisponíveis de Pedro direito à saúde e à educação básica.
(B) A lei não pode retroagir para prejudicar a coisa (E) defende a integridade e a intangibilidade dos di-
julgada formada em favor de Pedro. reitos fundamentais, independentemente das possibilida-
(C) A lei pode retroagir, pois não há direito adquirido des financeiras e orçamentárias do estado.
de Pedro diante de nova legislação.
(D) A lei pode retroagir, porque não há ato jurídico 14. (Prefeitura de Recife/PE - Procurador -
perfeito em favor de Pedro diante de pagamentos pen- FCC/2014) A Emenda Constitucional nº 72, promulgada
dentes. em 2 de abril de 2013, tem por finalidade estabelecer a
igualdade de direitos entre os trabalhadores domésticos e
12. (SP-URBANISMO - Analista Administrativo - os demais trabalhadores urbanos e rurais. Nos termos de
Jurídico - VUNESP/2014) João apresenta requerimento suas disposições, a Emenda
junto à Prefeitura do Município de São Paulo, pleiteando (A) determinou a extensão ao trabalhador doméstico,
que lhe seja informado o número de licitações, na modali- dentre outros, dos direitos à remuneração do serviço ex-
dade pregão, realizadas pela São Paulo Urbanismo desde traordinário superior, no mínimo, em cinquenta por cento a
2010. O pleito de João do normal e à proteção do mercado de trabalho da mulher,
(A) não encontra previsão expressa como direito fun- mediante incentivos específicos.
damental na Constituição Federal, mas, todavia, deverá ser (B) instituiu vedação ao legislador para conferir tra-
acolhido em virtude do texto constitucional prever que a tamento diferenciado aos trabalhadores domésticos, em
lei não excluirá da apreciação do Poder Judiciário lesão ou relação aos trabalhadores urbanos e rurais.
ameaça a direito (C) não determinou a extensão ao trabalhador do-
(B) é constitucionalmente previsto, pois é a todos as- méstico, dentre outros, dos direitos à proteção em face da
segurado, mediante o pagamento de taxa, o direito de pe- automação e à proteção do mercado de trabalho da mu-
tição aos Poderes Públicos em defesa de direitos ou contra lher, mediante incentivos específicos.
ilegalidade ou abuso de poder (D) determinou a extensão ao trabalhador doméstico,
(C) não encontra amparo constitucional, uma vez dentre outros, dos direitos à proteção em face da automa-
que a obtenção de certidões em repartições públicas será ção e ao piso salarial proporcional à extensão e à comple-
atendida apenas se o objeto do pedido for para defesa de xidade do trabalho.
direitos ou para esclarecimento de situações de interesse (E) não determinou a extensão ao trabalhador do-
pessoal. méstico, dentre outros, dos direitos à remuneração do ser-
(D) encontra amparo constitucional, pois todos têm viço extraordinário superior, no mínimo, em cinquenta por
direito a receber dos órgãos públicos informações de seu cento a do normal e ao piso salarial proporcional à exten-
interesse particular, ou de interesse coletivo ou geral, que são e à complexidade do trabalho.
serão prestadas no prazo da lei, sob pena de responsabili-
15. (MDIC - Agente Administrativo - CESPE/2014)
dade, ressalvadas aquelas cujo sigilo seja imprescindível à
Com referência à CF, aos direitos e garantias fundamen-
segurança da sociedade e do Estado.
tais, à organização político-administrativa, à administração
(E) é constitucionalmente previsto, devendo ser res-
pública e ao Poder Judiciário, julgue os itens subsecutivos.
pondido em 48 (quarenta e oito) horas, pois a todos, no
A CF prevê o direito de greve na iniciativa privada e
âmbito judicial e administrativo, são assegurados a razoá-
determina que cabe à lei definir os serviços ou atividades
vel duração do processo e os meios que garantam a cele-
essenciais e dispor sobre o atendimento das necessidades
ridade de sua tramitação.
inadiáveis da comunidade.
Certo ( )
13. (TCE/PI - Assessor Jurídico - FCC/2014) A teoria
Errado ( )
da reserva do possível
(A) significa a inoponibilidade do arbítrio estatal à 16. (TJ/MT - Juiz de Direito - FMP/2014) Assinale a
efetivação dos direitos sociais, econômicos e culturais. alternativa correta.
(B) gira em torno da legitimidade constitucional do (A) O rol de direitos sociais nos incisos do art. 7º e
controle e da intervenção do poder judiciário em tema de seguintes é exaustivo.
implementação de políticas públicas, quando caracteriza- (B) É vedada a redução proporcional do salário do tra-
da hipótese de omissão governamental. balhador sob qualquer hipótese.
(C) considera que as políticas públicas são reservadas (C) É assegurado ao trabalhador o gozo de férias
discricionariamente à análise e intervenção do poder judi- anuais remuneradas com, no mínimo, um terço a mais do
ciário, que as limitará ou ampliará, de acordo com o caso que o salário normal.
concreto.

36
LEGISLAÇÃO

(D) A licença à gestante, sem prejuízo do emprego e D) considera-se o casamento religioso inapto para ge-
do salário, não está constitucionalmente prevista, mas é rar efeito civil, visto que a República Federativa do Brasil
determinada pela CLT. constitui um Estado laico.
(E) O direito à licença paternidade, sem prejuízo do E) é de livre decisão do casal o planejamento familiar,
emprego e do salário, não está constitucionalmente previs- admitindo-se, porém, intervenção coercitiva do Estado
to, mas é determinado pela CLT. para controle da natalidade.

17. (TJ/SE - Titular de Serviços de Notas e de Regis- 20. (DPE/DF - Analista - Assistência Judiciária -
tro - CESPE/2014) Com base nas normas constitucionais FGV/2014) No que concerne à previsão constitucional
que tratam da ordem econômica e financeira e da ordem acerca da seguridade social, é INCORRETO afirmar que:
social, assinale a opção correta. A) a seguridade social engloba os direitos relativos à
(A) A CF assegura às empresas públicas prestadoras
saúde, à previdência e à assistência social.
de serviços públicos a isenção de custas processuais para
B) constitui um, entre vários, dos objetivos da segurida-
litigar em juízo.
(B) Os benefícios da previdência social e a assistência de social a universalidade da cobertura e do atendimento.
social, organizadas sob a forma de regime geral, destinam- C) o caráter democrático e descentralizado da admi-
se exclusivamente aos contribuintes da seguridade social. nistração, um dos objetivos constantes na organização da
(C) A CF prevê a possibilidade de exploração direta seguridade social, é realizado através da gestão tripartite
de atividade econômica pelo Estado somente no caso de nos órgãos colegiados, com participação dos trabalhado-
imprescindibilidade à segurança nacional. res, dos empregadores e do governo.
(D) A desapropriação, pela União, de imóvel rural que D) a participação no custeio da seguridade social deve
não atenda a sua função social, para a realização de refor- ser realizada de forma equânime entre os participantes.
ma agrária, depende de prévia indenização em dinheiro. E) constitui um, entre vários, dos objetivos da segurida-
(E) De acordo com a CF, as universidades, entes com de social a uniformidade e a equivalência dos benefícios e
autonomia didático-científica, patrimonial, administrativa e serviços às populações urbanas e rurais.
de gestão financeira, devem tratar como indissociáveis as
atividades de ensino, pesquisa e extensão. 21. (Prefeitura de Recife/PE - Procurador- FCC/2014)
Nos termos do art. 226 da Constituição Federal, “a família,
18. (DPE/GO - Defensor Público - UFG/2014) A base da sociedade, tem especial proteção do Estado”. Entre
Constituição Federal de 1988 prevê a saúde como direi-
os aspectos abrangidos pelo direito à proteção especial,
to fundamental a ser assegurado ao cidadão. A propósito
segundo o texto constitucional, encontram-se os seguin-
desse direito,
A) assegura-se o fornecimento de medicamentos de tes:
alto custo exclusivamente aos necessitados, devido à infi- A) garantia de direitos previdenciários e trabalhistas; e
nitude das demandas e à finitude dos recursos. obediência aos princípios de brevidade, excepcionalidade
B) é exclusiva do ministério público a legitimidade para e respeito à condição peculiar de pessoa em desenvolvi-
ajuizamento de ação de mandado de segurança com vistas mento, quando da aplicação de qualquer medida privativa
a promover o fornecimento de medicamentos. da liberdade.
C) é vedada à iniciativa privada a exploração econô- B) garantia de direitos previdenciários e trabalhistas; e
mica da assistência à saúde dado o direito fundamental à acesso universal à educação infantil, em creche e pré-esco-
saúde ser consectário do direito à vida. la, às crianças até 5 (cinco) anos de idade.
D) regula-se o sistema único de saúde (SUS) exclusiva- C) erradicação do analfabetismo; e estímulo do Poder
mente por meio da legislação infraconstitucional, visto que Público, através de assistência jurídica, incentivos fiscais e
está fora das matérias constitucionais. subsídios, nos termos da lei, ao acolhimento, sob a forma
E) é vedada a destinação de recursos públicos para au- de guarda, de criança ou adolescente órfão ou abandona-
xílios ou subvenções às instituições privadas de saúde com do.
fins lucrativos. D) punição severa ao abuso, à violência e à exploração
sexual da criança e do adolescente; e garantia às presidiá-
19. (DPE/GO - Defensor Público - UFG/2014) A
rias de condições para que possam permanecer com seus
Constituição Federal de 1988 prevê a família como célula
filhos durante o período de amamentação.
mater da sociedade, ao que goza, assim, de especial prote-
ção do Estado. Por isso, E) punição severa ao abuso, à violência e à exploração
A) concebe-se como família aquela união feita por pes- sexual da criança e do adolescente; e estímulo do Poder
soas de diferentes sexos, desde que formalizada perante as Público, através de assistência jurídica, incentivos fiscais e
autoridades notariais de acordo com a jurisprudência dos subsídios, nos termos da lei, ao acolhimento, sob a forma
Tribunais Superiores. de guarda, de criança ou adolescente órfão ou abandona-
B) entende-se como entidade familiar a comunidade do.
formada por qualquer dos pais e seus descendentes.
C) são exercidos diferentemente pelo homem e pela 22. (Prefeitura de Recife/PE - Procurador- FCC/2014)
mulher, tendo em vista a própria diferença de gênero e os Entre as competências constitucionalmente atribuídas ao
direitos e deveres referentes à sociedade conjugal. Sistema Único de Saúde, encontram-se as seguintes;

37
LEGISLAÇÃO

A) participar da formulação da política e da execução 2. Resposta: “D”. O habeas corpus é garantia previs-
das ações de saneamento básico; e estimular a participação ta no artigo 5º, LXVIII, CF: “conceder-se-á habeas corpus
direta ou indireta de empresas ou capitais estrangeiros na sempre que alguém sofrer ou se achar ameaçado de sofrer
assistência à saúde no País violência ou coação em sua liberdade de locomoção, por
B) colaborar na proteção do meio ambiente, nele com- ilegalidade ou abuso de poder”. A respeito dele, a lei bus-
preendido o do trabalho; e apoiar a habilitação e a reabi- ca torná-lo o mais acessível possível, por ser diretamente
litação das pessoas com deficiência e a promoção de sua relacionado a um direito fundamental da pessoa humana.
integração à vida comunitária. O objeto de tutela é a liberdade de locomoção; a propo-
C) participar da formulação da política e da execução situra não depende de advogado; o que propõe a ação é
das ações de saneamento básico; e apoiar a habilitação e a denominado impetrante e quem será por ela beneficiado é
reabilitação das pessoas com deficiência e a promoção de chamado paciente (podendo a mesma pessoa ser os dois),
sua integração à vida comunitária. contra quem é proposta a ação é a denominada autoridade
D) ordenar a formação de recursos humanos na área coatora; e é possível utilizar habeas corpus repressivamen-
de saúde; e estimular a participação direta ou indireta de te e preventivamente. Por sua vez, a Constituição Federal
empresas ou capitais estrangeiros na assistência à saúde prevê no artigo 142, §2º que “não caberá habeas corpus em
no País. relação a punições disciplinares militares”.
E) colaborar na proteção do meio ambiente, nele com-
preendido o do trabalho; e participar do controle e fisca- 3. Resposta: “A”. No que tange ao tema, destaque
lização da produção, transporte, guarda e utilização de para os seguintes incisos do artigo 5º da CF: “LXXI - conce-
substâncias e produtos psicoativos, tóxicos e radioativos. der-se-á mandado de injunção sempre que a falta de nor-
ma regulamentadora torne inviável o exercício dos direitos
23. (PC/SP - Oficial Administrativo - VUNESP/2014) e liberdades constitucionais e das prerrogativas inerentes
Para fins de obtenção de aposentadoria pelo sistema geral à nacionalidade, à soberania e à cidadania; LXXII - conce-
de previdência social, além de outros requisitos, é necessá- der-se-á habeas data: a) para assegurar o conhecimento de
rio o tempo mínimo de contribuição de. informações relativas à pessoa do impetrante, constantes
(A) trinta e cinco anos para homem ou mulher de registros ou bancos de dados de entidades governa-
(B) trinta anos para homem ou mulher. mentais ou de caráter público; b) para a retificação de da-
(C) trinta e cinco anos, se homem, e trinta anos, se dos, quando não se prefira fazê-lo por processo sigiloso,
mulher. judicial ou administrativo”. Os itens “I” e “II” repetem o teor
(D) vinte anos, se homem, e quinze anos, se mulher do artigo 5º, LXXII, CF. Já o item “III” decorre logicamente
(E) vinte e cinco anos, se homem, e vinte anos, se mu- da previsão dos direitos fundamentais como limitadores da
lher. atuação do Estado, logo, as informações requeridas serão
contra uma entidade governamental da administração di-
RESPOSTAS reta ou indireta. Por sua vez, o item “IV” reflete o artigo 5º,
LXXI, CF, do qual decorre logicamente o item “V”, posto
1. Resposta: “E”. “I” está correta porque a principal que a demora do legislador em regulamentar uma norma
diferença entre direitos e garantias é que os primeiros constitucional de aplicabilidade mediata, que necessita do
servem para determinar os bens jurídicos tutelados e as preenchimento de seu conteúdo, evidencia-se em risco aos
segundas são os instrumentos para assegurar estes (ex: direitos fundamentais garantidos pela Constituição Fede-
direito de liberdade de locomoção – garantia do habeas ral.
corpus). “II” está correta, afinal, o próprio artigo 5º prevê
em seu §2º que “os direitos e garantias expressos nesta 4. Resposta: “A”. Nos termos do artigo 5º, LIII, CF, “nin-
Constituição não excluem outros decorrentes do regime e guém será processado nem sentenciado senão pela autori-
dos princípios por ela adotados, ou dos tratados interna- dade competente”, restando o item “I” correto; pelo artigo
cionais em que a República Federativa do Brasil seja parte”, 5º, LX, CF, “a lei só poderá restringir a publicidade dos atos
fundamento que também demonstra que o item “III” está processuais quando a defesa da intimidade ou o interesse
correto. O item IV traz cópia do artigo 5º, X, CF, que prevê social o exigirem”, motivo pelo qual o item “II” está correto;
que “são invioláveis a intimidade, a vida privada, a honra e e prevê o artigo 5º, LXVI, CF que “ninguém será levado à
a imagem das pessoas, assegurado o direito a indenização prisão ou nela mantido, quando a lei admitir a liberdade
pelo dano material ou moral decorrente de sua violação”; provisória, com ou sem fiança”, confirmando o item “IV”.
o que faz também o item V com relação ao artigo 5º, VI, Por sua vez, o item “III” está incorreto porque “são inad-
CF que diz que “é inviolável a liberdade de consciência e missíveis, no processo, as provas obtidas por meios ilícitos”
de crença, sendo assegurado o livre exercício dos cultos (artigo 5º, LVI, CF); e o item “V” está incorreto porque a ju-
religiosos e garantida, na forma da lei, a proteção aos locais risprudência atual ainda aceita a prisão civil do devedor de
de culto e a suas liturgias”. Sendo assim, todas afirmativas alimentos, sendo que o texto constitucional autoriza tanto
estão corretas. esta quanto a do depositário infiel (artigo 5º, LXVII, CF).

38
LEGISLAÇÃO

5. Resposta: “A”. Preconiza o artigo 5º, XL, CF: “XL - 13. Resposta: “B”. A teoria da reserva do possível bus-
a lei penal não retroagirá, salvo para beneficiar o réu”. ca impedir que se argumente por uma obrigação infinita
Assim, se vier uma lei posterior ao fato que o exclua do do Estado de atender direitos econômicos, sociais e cultu-
rol de crimes ou que confira tratamento mais benéfico (di- rais. No entanto, não pode ser invocada como muleta para
minuindo a pena ou alterando o regime de cumprimento, impedir que estes direitos adquiram efetividade. Se a invo-
notadamente), ela será aplicada. cação da reserva do possível não demonstrar cabalmente
que o Estado não tem condições de arcar com as despesas,
6. Resposta: “B”. Neste sentido, prevê o artigo 5º, XLII, o Poder Judiciário irá intervir e sanar a omissão.
CF: “XLII - a prática do racismo constitui crime inafiançá-
vel e imprescritível, sujeito à pena de reclusão, nos termos 14. Resposta: “C”. A Emenda Constitucional nº
da lei”, restando “B” correta. “A” é incorreta porque a lei pe- 72/2013, que ficou conhecida no curso de seu processo de
nal retroage para beneficiar o réu; “C” é incorreta porque é votação como PEC das domésticas, deu redação ao pará-
aceita a pena de morte para os crimes militares praticados grafo único do artigo 7º, o qual estende alguns dos direi-
em tempo de guerra; “D” é incorreta porque igrejas não tos enumerados nos incisos do caput para a categoria dos
possuem inviolabilidade domiciliar. trabalhadores domésticos, quais sejam: “IV, VI, VII, VIII, X,
XIII, XV, XVI, XVII, XVIII, XIX, XXI, XXII, XXIV, XXVI, XXX, XXXI
7. Resposta: “D”. Embora o direito previsto na alter- e XXXIII e, atendidas as condições estabelecidas em lei e
nativa “D” seja um direito fundamental, não é um direito observada a simplificação do cumprimento das obrigações
individual, logo, não está previsto no artigo 5º, e sim no tributárias, principais e acessórias, decorrentes da relação
artigo 7º, CF, em seu inciso IX (“remuneração do trabalho de trabalho e suas peculiaridades, os previstos nos incisos
noturno superior à do diurno”). I, II, III, IX, XII, XXV e XXVIII, bem como a sua integração à
previdência social”. Os direitos descritos na alternativa “C”
8. Resposta: “D”. A propósito, o artigo 5º, XI, CF dis- estão previstos nos incisos XXVII e XX do artigo 7º da Cons-
põe: “a casa é asilo inviolável do indivíduo, ninguém nela tituição, não estendidos aos empregados domésticos pela
podendo penetrar sem consentimento do morador, salvo emenda.
em caso de flagrante delito ou desastre, ou para prestar so-
corro, ou, durante o dia, por determinação judicial”. Sen- 15. Resposta: “Certo”. O artigo 9º, CF disciplina o di-
do assim, não cabe o ingresso por determinação judicial a reito de greve: “É assegurado o direito de greve, compe-
qualquer hora, mas somente durante o dia. tindo aos trabalhadores decidir sobre a oportunidade de
exercê-lo e sobre os interesses que devam por meio dele
9. Resposta: C. Dispõe o artigo 5º, CF em seu inciso defender. § 1º A lei definirá os serviços ou atividades
XIV: “é assegurado a todos o acesso à informação e res- essenciais e disporá sobre o atendimento das necessida-
guardado o sigilo da fonte, quando necessário ao exercício des inadiáveis da comunidade. § 2º Os abusos cometidos
profissional”. sujeitam os responsáveis às penas da lei”.

10. Resposta: “C”. Estabelece o §3º do artigo 5º,CF: 16. Resposta: “C”. “A” está incorreta porque o rol de
“Os tratados e convenções internacionais sobre direitos direitos sociais do artigo 7º é apenas exemplificativo, não
humanos que forem aprovados, em cada Casa do Con- excluindo outros que decorram das normas trabalhistas,
gresso Nacional, em dois turnos, por três quintos dos votos dos direitos humanos internacionais e das convenções e
dos respectivos membros, serão equivalentes às emendas acordos coletivos; “B” está incorreta porque a redução pro-
constitucionais”. Logo, é necessário o preenchimento de porcional pode ser aceita se intermediada por negociação
determinados requisitos para a incorporação. coletiva, evitando cenário de demissão em massa; “D” está
incorreta porque a licença-gestante encontra arcabouço
11. Resposta: “B”. No que tange à segurança jurídica, constitucional, tal como a licença-paternidade, restando
tem-se o disposto no artigo 5º, XXXVI, CF: “XXXVI - a lei “E” também incorreta (artigo 7º, XVIII e XIX, CF. Sendo as-
não prejudicará o direito adquirido, o ato jurídico perfeito sim, “C” está correta, conforme disposto no artigo 7º: “gozo
e a coisa julgada”. A coisa julgada se formou a favor de de férias anuais remuneradas com, pelo menos, um terço
Pedro e não pode ser quebrada por lei posterior que altere a mais do que o salário normal” (artigo 7º, XVII, CF).
a situação fático-jurídica, sob pena de se atentar contra a
segurança jurídica. 17. Resposta: “E”. “A” está incorreta porque as em-
presas públicas não possuem benefícios fiscais; “B” está
12. Resposta: “D”. Trata-se de garantia constitucio- incorreta porque a assistência social é voltada a não contri-
nal prevista no artigo 5º, XXXIII, CF: “todos têm direito a buintes; “C” está incorreta porque cabe a exploração de ati-
receber dos órgãos públicos informações de seu interes- vidade econômica pelo Estado não só em caso de necessi-
se particular, ou de interesse coletivo ou geral, que serão dade aos imperativos da segurança nacional, mas também
prestadas no prazo da lei, sob pena de responsabilidade, de relevante interesse coletivo (artigo 173, caput, CF); “D”
ressalvadas aquelas cujo sigilo seja imprescindível à segu- está incorreta porque a indenização neste caso se dá por
rança da sociedade e do Estado”. títulos da dívida agrária. Somente resta “E”, pois o ensino
universitário encontra respaldo no artigo 207 da Consti-

39
LEGISLAÇÃO

tuição, tendo autonomia didático-científica, administrativa do Poder Público, através de assistência jurídica, incenti-
e de gestão financeira e patrimonial, e sendo baseado na vos fiscais e subsídios, nos termos da lei, ao acolhimento,
tríade ensino-pesquisa-extensão, disciplina que se estende sob a forma de guarda, de criança ou adolescente órfão ou
a instituições de pesquisa científica e tecnológica. abandonado; VII - programas de prevenção e atendimento
especializado à criança, ao adolescente e ao jovem depen-
18. Resposta: “E”. Os medicamentos de alto custo de- dente de entorpecentes e drogas afins”.
vem ser fornecidos a todos aqueles que o necessitarem,
não somente aos hipossuficientes, até mesmo porque é 22. Resposta: “E”. Neste viés, o artigo 200, CF prevê:
possível que uma pessoa com boa renda não tenha condi- “Ao sistema único de saúde compete, além de outras atri-
ções de arcar com estes. O mandado de segurança buscan- buições, nos termos da lei: I - controlar e fiscalizar procedi-
do o fornecimento de medicamentos pode ser interposto mentos, produtos e substâncias de interesse para a saúde
pelo Ministério Público, mas não somente por ele, tam- e participar da produção de medicamentos, equipamen-
bém pela Defensoria Pública e pelo próprio interessado. O tos, imunobiológicos, hemoderivados e outros insumos;
sistema de saúde está previsto na Lei nº 8.080/1990, mas II - executar as ações de vigilância sanitária e epidemio-
encontra substrato constitucional, especialmente em seu lógica, bem como as de saúde do trabalhador; III - orde-
artigo 6º. Referida lei especial assegura a possibilidade de nar a formação de recursos humanos na área de saúde;
participação de entidades privadas no sistema, desde que IV - participar da formulação da política e da execução das
não possuam fim lucrativo, ou seja, atendam pelo preço ações de saneamento básico; V - incrementar em sua área
do SUS. de atuação o desenvolvimento científico e tecnológico; VI
- fiscalizar e inspecionar alimentos, compreendido o con-
19. Resposta: “B”. A alternativa “A” está incorreta por- trole de seu teor nutricional, bem como bebidas e águas
que o que se equiparou à família foi a união estável entre para consumo humano; VII - participar do controle e fis-
pessoas do mesmo sexo, a qual independe de formaliza- calização da produção, transporte, guarda e utilização
ção escrita. A alternativa “C” está incorreta porque o artigo de substâncias e produtos psicoativos, tóxicos e radioa-
226, §5º, CF prevê a igualdade entre homem e mulher no tivos; VIII - colaborar na proteção do meio ambiente,
casamento: “os direitos e deveres referentes à sociedade nele compreendido o do trabalho”. Conforme grifos, as
conjugal são exercidos igualmente pelo homem e pela mu- atribuições descritas na alternativa “E” estão corretas.
lher”. O casamento religioso, por sua vez, pode gerar efei-
tos civis, conforme artigo 226, §2º, restando “D” incorreta. 23. Resposta: “C”. O §7º do artigo 201, CF fixa as con-
O Estado não pode intervir coativamente no planejamento dições para a aposentadoria pelo regime geral de previ-
familiar, decisão totalmente livre do casal (artigo 226, §7º, dência social: “I - trinta e cinco anos de contribuição, se
CF), restando “E” incorreta. Resta a alternativa “B”, que cor- homem, e trinta anos de contribuição, se mulher; [...]”. 
retamente traz o teor do artigo 226, §4º, CF: “entende-se,
também, como entidade familiar a comunidade formada
por qualquer dos pais e seus descendentes”. ESTATUTO DA CRIANÇA E DO ADOLESCENTE
ATUALIZADO COM A LEI 12.010
20. Resposta: “C”. Observando o artigo 194, parágra-
fo único, VII, CF é possível perceber que a alternativa “C”
está incorreta: “Compete ao Poder Público, nos termos da
lei, organizar a seguridade social, com base nos seguintes Noções introdutórias e disciplina constitucional
objetivos: [...] caráter democrático e descentralizado da
administração, mediante gestão quadripartite, com par- Art. 227. É dever da família, da sociedade e do Es-
ticipação dos trabalhadores, dos empregadores, dos apo- tado assegurar à criança, ao adolescente e ao jovem, com
sentados e do Governo nos órgãos colegiados”. Logo, os absoluta prioridade, o direito à vida, à saúde, à alimenta-
aposentados estão incluídos e a gestão é quadripartite. ção, à educação, ao lazer, à profissionalização, à cultura, à
dignidade, ao respeito, à liberdade e à convivência familiar
21. Resposta: “A”. O artigo 227, §3º, CF fixa os aspec- e comunitária, além de colocá-los a salvo de toda forma
tos que abrangem a proteção especial da criança e do ado- de negligência, discriminação, exploração, violência,
lescente: “I - idade mínima de quatorze anos para admissão crueldade e opressão. 
ao trabalho, observado o disposto no art. 7º, XXXIII; II - ga- § 1º O Estado promoverá programas de assistência
rantia de direitos previdenciários e trabalhistas; III - garan- integral à saúde da criança, do adolescente e do jovem, ad-
tia de acesso do trabalhador adolescente e jovem à escola; mitida a participação de entidades não governamentais,
IV - garantia de pleno e formal conhecimento da atribuição mediante políticas específicas e obedecendo aos seguintes
de ato infracional, igualdade na relação processual e defe- preceitos: 
sa técnica por profissional habilitado, segundo dispuser a I - aplicação de percentual dos recursos públicos desti-
legislação tutelar específica; V - obediência aos princípios nados à saúde na assistência materno-infantil;
de brevidade, excepcionalidade e respeito à condição pe- II - criação de programas de prevenção e atendimen-
culiar de pessoa em desenvolvimento, quando da aplicação to especializado para as pessoas portadoras de deficiência
de qualquer medida privativa da liberdade; VI - estímulo física, sensorial ou mental, bem como de integração social

40
LEGISLAÇÃO

do adolescente e do jovem portador de deficiência, me- § 8º A lei estabelecerá: 


diante o treinamento para o trabalho e a convivência, e a I - o estatuto da juventude, destinado a regular os di-
facilitação do acesso aos bens e serviços coletivos, com a eli- reitos dos jovens; 
minação de obstáculos arquitetônicos e de todas as formas II - o plano nacional de juventude, de duração dece-
de discriminação.  nal, visando à articulação das várias esferas do poder públi-
§ 2º A lei disporá sobre normas de construção dos lo- co para a execução de políticas públicas. 
gradouros e dos edifícios de uso público e de fabricação de
veículos de transporte coletivo, a fim de garantir acesso No caput do artigo 227, CF se encontra uma das prin-
adequado às pessoas portadoras de deficiência. cipais diretrizes do direito da criança e do adolescente que
§ 3º O direito a proteção especial abrangerá os seguin- é o princípio da prioridade absoluta. Significa que cada
tes aspectos: criança e adolescente deve receber tratamento especial do
I - idade mínima de quatorze anos para admissão ao Estado e ser priorizado em suas políticas públicas, pois são
trabalho, observado o disposto no art. 7º, XXXIII; o futuro do país e as bases de construção da sociedade.
II - garantia de direitos previdenciários e trabalhis- A Lei nº 8.069, de 13 de julho de 1990 dispõe sobre o
tas; Estatuto da Criança e do Adolescente e dá outras providên-
III - garantia de acesso do trabalhador adolescente e cias, seguindo em seus dispositivos a ideologia do princí-
jovem à escola;  pio da absoluta prioridade.
IV - garantia de pleno e formal conhecimento da No §1º do artigo 227 aborda-se a questão da assistên-
atribuição de ato infracional, igualdade na relação pro- cia à saúde da criança e do adolescente. Do inciso I se de-
cessual e defesa técnica por profissional habilitado, se- preende a intrínseca relação entre a proteção da criança e
gundo dispuser a legislação tutelar específica; do adolescente com a proteção da maternidade e da infân-
V - obediência aos princípios de brevidade, excepcio- cia, mencionada no artigo 6º, CF. Já do inciso II se depreen-
nalidade e respeito à condição peculiar de pessoa em de- de a proteção de outro grupo vulnerável, que é a pessoa
senvolvimento, quando da aplicação de qualquer medida portadora de deficiência, valendo lembrar que o Decreto nº
privativa da liberdade; 6.949, de 25 de agosto de 2009, que promulga a Convenção
Internacional sobre os Direitos das Pessoas com Deficiência
VI - estímulo do Poder Público, através de assistência
e seu Protocolo Facultativo, assinados em Nova York, em
jurídica, incentivos fiscais e subsídios, nos termos da lei,
30 de março de 2007, foi promulgado após aprovação no
ao acolhimento, sob a forma de guarda, de criança ou ado-
Congresso Nacional nos moldes da Emenda Constitucional
lescente órfão ou abandonado;
nº 45/2004, tendo força de norma constitucional e não de
VII - programas de prevenção e atendimento especia-
lei ordinária. A preocupação com o direito da pessoa porta-
lizado à criança, ao adolescente e ao jovem dependente de
dora de deficiência se estende ao §2º do artigo 227, CF: “a
entorpecentes e drogas afins.
lei disporá sobre normas de construção dos logradouros e
§ 4º A lei punirá severamente o abuso, a violência e a dos edifícios de uso público e de fabricação de veículos
exploração sexual da criança e do adolescente. de transporte coletivo, a fim de garantir acesso adequado
§ 5º A adoção será assistida pelo Poder Público, na às pessoas portadoras de deficiência”.
forma da lei, que estabelecerá casos e condições de sua efe- A proteção especial que decorre do princípio da prio-
tivação por parte de estrangeiros. ridade absoluta está prevista no §3º do artigo 227. Liga-se,
§ 6º Os filhos, havidos ou não da relação do casa- ainda, à proteção especial, a previsão do §4º do artigo 227:
mento, ou por adoção, terão os mesmos direitos e qua- “A lei punirá severamente o abuso, a violência e a explora-
lificações, proibidas quaisquer designações discriminatórias ção sexual da criança e do adolescente”.
relativas à filiação. Tendo em vista o direito de toda criança e adolescente
§ 7º No atendimento dos direitos da criança e do ado- de ser criado no seio de uma família, o §5º do artigo 227 da
lescente levar-se-á em consideração o disposto no art. Constituição prevê que “a adoção será assistida pelo Poder
20433. Público, na forma da lei, que estabelecerá casos e condi-
ções de sua efetivação por parte de estrangeiros”. Neste
33 Art. 204. As ações governamentais na área da as-
sentido, a Lei nº 12.010, de 3 de agosto de 2009, dispõe
sistência social serão realizadas com recursos do orça-
sobre a adoção.
mento da seguridade social, previstos no art. 195, além A igualdade entre os filhos, quebrando o paradigma da
de outras fontes, e organizadas com base nas seguintes Constituição anterior e do até então vigente Código Civil
diretrizes: I - descentralização político-administrativa, de 1916 consta no artigo 227, § 6º, CF: “os filhos, havidos
cabendo a coordenação e as normas gerais à esfera fede- ou não da relação do casamento, ou por adoção, terão os
ral e a coordenação e a execução dos respectivos progra- mesmos direitos e qualificações, proibidas quaisquer de-
mas às esferas estadual e municipal, bem como a entida- signações discriminatórias relativas à filiação”.
des beneficentes e de assistência social; II - participação
da população, por meio de organizações representativas, sua receita tributária líquida, vedada a aplicação desses
na formulação das políticas e no controle das ações em recursos no pagamento de: I - despesas com pessoal e
todos os níveis. Parágrafo único. É facultado aos Estados encargos sociais; II - serviço da dívida; III - qualquer outra
e ao Distrito Federal vincular a programa de apoio à in- despesa corrente não vinculada diretamente aos investi-
clusão e promoção social até cinco décimos por cento de mentos ou ações apoiados.

41
LEGISLAÇÃO

Quando o artigo 227 dispõe no § 7º que “no atendi- - A garantia de pleno e formal conhecimento da atri-
mento dos direitos da criança e do adolescente levar-se-á buição do ato infracional, igualdade na relação processual
em consideração o disposto no art. 204” tem em vista a e defesa técnica por profissional habilitado, segundo dis-
adoção de práticas de assistência social, com recursos da puser a legislação tutelar específica (inciso IV);
seguridade social, em prol da criança e do adolescente. - A obediência aos princípios de brevidade, excepcio-
Por seu turno, o artigo 227, § 8º, CF, preconiza: “A lei nalidade e respeito à condição peculiar de pessoa em de-
estabelecerá: I - o estatuto da juventude, destinado a re- senvolvimento, quando da aplicação de qualquer medida
gular os direitos dos jovens; II - o plano nacional de juven- privativa de liberdade (inciso V);
tude, de duração decenal, visando à articulação das várias - O estímulo do Poder Público, através de assistência
esferas do poder público para a execução de políticas pú- jurídica, incentivos fiscais e subsídios, nos termos da lei, ao
blicas”. A Lei nº 12.852, de 5 de agosto de 2013, institui acolhimento, sob a forma de guarda, de criança ou adoles-
o Estatuto da Juventude e dispõe sobre os direitos dos cente órfão ou abandonado (inciso VI);
jovens, os princípios e diretrizes das políticas públicas de - Programas de prevenção e atendimento especializa-
juventude e o Sistema Nacional de Juventude - SINAJUVE. do à criança, ao adolescente e ao jovem dependente de
Mais informações sobre a Política mencionada no inciso II entorpecentes e drogas afins (inciso VII).
e sobre a Secretaria e o Conselho Nacional de Juventude Prosseguindo, o parágrafo sexto, do art. 227, da Cons-
que direcionam a implementação dela podem ser obtidas tituição, garante o “Princípio da Igualdade entre os Filhos”,
na rede34. ao dispor que os filhos, havidos ou não da relação do ca-
Aprofundando o tema, a cabeça do art. 227, da Lei Fun- samento, ou por adoção, terão os mesmos direitos e quali-
damental, preconiza ser dever da família, da sociedade e do ficações, proibidas quaisquer designações discriminatórias
Estado assegurar à criança, ao adolescente e ao jovem, com relativas à filiação.
absoluta prioridade, o direito à vida, à saúde, à alimentação, Assim, com a Constituição Federal, os filhos não têm
à educação, ao lazer, à profissionalização, à cultura, à dig- mais “valor” para efeito de direitos alimentícios e suces-
nidade, ao respeito, à liberdade e à convivência familiar e sórios. Não se pode falar em um filho receber metade da
comunitária, além de colocá-los a salvo de toda forma de parte que originalmente lhe cabia por ser “bastardo”, en-
negligência, discriminação, exploração, violência, cruelda-
quanto aquele fruto da sociedade conjugal receber a quan-
de e opressão.
tia integral. Aliás, nem mesmo a expressão “filho bastardo”
A leitura do art. 227, caput, da Constituição Federal
pode mais ser utilizada, por representar uma forma de dis-
permite concluir que se adotou, neste país, a chamada
criminação designatória.
“Doutrina da Proteção Integral da Criança”, ao lhe assegu-
Também, o art. 229 traz uma “via de mão dupla” entre
rar a absoluta prioridade em políticas públicas, medidas
pais e filhos, isto é, os pais têm o dever de assistir, criar e
sociais, decisões judiciais, respeito aos direitos humanos,
educar os filhos menores, e os filhos maiores têm o dever
e observância da dignidade da pessoa humana. Neste sen-
tido, o parágrafo único, do art. 5º, do “Estatuto da Crian- de ajudar e amparar os pais na velhice, carência ou en-
ça e do Adolescente”, prevê que a garantia de priorida- fermidade. Tal dispositivo, inclusive, permite que os filhos
de compreende a primazia de receber proteção e socorro peçam alimentos aos pais, e que os pais peçam alimentos
em quaisquer circunstâncias (alínea “a”), a precedência de aos filhos.
atendimento nos serviços públicos ou de relevância pública Por fim, há se mencionar o acrescentado parágrafo oi-
(alínea “b”), a preferência na formulação e na execução das tavo (pela Emenda Constitucional nº 65/2010), ao art. 227,
políticas sociais públicas (alínea “c”), e a destinação privi- da Constituição Federal, segundo o qual a lei estabelecerá
legiada de recursos públicos nas áreas relacionadas com a o estatuto da juventude, destinado a regular os direitos dos
proteção à infância e à juventude (alínea “d”). jovens (inciso I), e o plano nacional de juventude, de du-
Ademais, a proteção à criança, ao adolescente e ao jo- ração decenal, visando à articulação das várias esferas do
vem representa incumbência atribuída não só ao Estado, poder público para a execução de políticas públicas (inciso
mas também à família e à sociedade. Sendo assim, há se II). Nada obstante a exigência constitucional desde 2010,
prestar bastante atenção nas provas de concurso, tendo somente bem recentemente o Estatuto da Juventude foi
em vista que só se costuma colocar o Estado como obser- aprovado (Lei nº 12.852/2013), como visto acima, carecen-
vador da “Doutrina da Proteção Integral”, sendo que isso do, ainda, o Plano Nacional de Juventude de maior regula-
também compete à família e à sociedade. mentação infraconstitucional.
Nesta frequência, o direito à proteção especial abran-
gerá os seguintes aspectos (art. 227, §3º, CF): Evolução histórica
- A idade mínima de dezesseis anos para admissão ao
trabalho, salvo a partir dos quatorze anos, na condição de Na Grécia antiga, a criança era colocada numa posição
aprendiz (inciso I de acordo com o art. 7º, XXXIII, CF, pós-al- de inferioridade, tida como um ser irracional, sem capaci-
teração promovida pela Emenda Constitucional nº 20/98); dade de tomar qualquer tipo de decisão. Trata-se de marco
- A garantia de direitos previdenciários e trabalhistas da cultura grega, que enxergava apenas poucos homens de
(inciso II); posses como cidadãos. Estes homens concentravam para
- A garantia de acesso ao trabalhador adolescente e si o pátrio poder, isto é, o poder do pai. Devido ao pátrio
jovem à escola (inciso III); poder, o pai de família concentrava em suas mãos plena
34 http://www.juventude.gov.br/politica possibilidade de gerir a vida das crianças e adolescentes

42
LEGISLAÇÃO

e estes não tinham nenhuma possibilidade de participar comuns). Em 1964 surge a Política Nacional do Bem-estar
destas decisões. Na Idade Média se manteve o sistema do do Menor (Lei nº 4.513/1964), que criou a FUNABEM. Surge
“pátrio poder”. As crianças eram submetidas ao absoluto novo Código de Menores em 1979 (Lei nº 6.697), cujo ob-
poder do pai e seus destinos seguiam a mesma sorte. jeto era a proteção e vigilância de crianças e adolescentes
A partir da Idade Moderna, com o Renascimento e o em situação irregular. Na década de 80 começa um mo-
Iluminismo, as crianças e os adolescentes saíram ligeira- vimento de reelaboração da concepção de infância e ju-
mente da margem social. A moral da época passa a impor ventude. O destaque repercute na Constituição Federal de
aos pais o dever de educar seus filhos. Entretanto, a educa- 1988 e no Estatuto da Criança e do Adolescente de 1990,
ção costumava ser oferecida apenas aos homens. Aqueles que revogou o Código de Menores e substituiu a doutrina
que possuíam melhores condições enviavam seus filhos da situação irregular pela doutrina da proteção integral35.
para estudarem nas universidades que começavam a des-
pontar na Europa, aqueles que possuíam condições piores Relações jurídicas no direito da criança e do ado-
ao menos passavam a ensinar seus ofícios a estes jovens. lescente
Já as meninas permaneciam marginalizadas das atividades
educacionais e profissionalizantes, apenas lhes era ensina- “As relações jurídicas são formas qualificadas de re-
do como desempenhar atividades domésticas. lações interpessoais, indicando, assim, a ligação entre
Desde o final da Revolução Francesa e, com destaque, pessoas, em razão de algum objeto, devidamente regu-
a partir da Revolução Industrial, que alterou substancial- lada pelo direito. Desta forma, o Direito da Criança e do
mente os modos e métodos de produção, a criança e o Adolescente, sob o aspecto objetivo e formal, representa
adolescente passam a ocupar papel central na sociedade, a disciplina das relações jurídicas entre Crianças e Adoles-
desempenhando atividades trabalhistas de caráter equiva- centes, de um lado, e de outro, a família, a comunidade, a
lente a dos adultos. Foram vítimas de inúmeros acidentes sociedade e o próprio Estado. [...] Percebemos que a inten-
de trabalho, morriam em meio à insalubridade das fábricas, ção dos doutrinadores e do próprio legislador foi, sempre,
então movidas predominantemente a carvão. Foi apenas criar uma doutrina da proteção integral não somente para
com a emergência da Organização Internacional do Tra- a Criança, como, ainda, para o Adolescente, ambos ainda
balho – OIT, em 1919, que aos poucos se consolidou uma em desenvolvimento, posto que, somente com o término
consciência a respeito da necessidade de se limitar a parti- da adolescência é que o menor completará o processo de
cipação das crianças e adolescentes no espaço de trabalho. aquisição de mecanismos mentais relacionados ao pensa-
Este foi o estopim para o reconhecimento da condição es- mento, percepção, reconhecimento, classificação etc. [...]
pecial da criança e do adolescente. Com isso, o Estatuto da Criança e do Adolescente, sabia-
Internacionalmente, a proteção efetiva da criança e mente, se preocupou em envolver não somente a família,
do adolescente começa a tomar corpo com o reconheci- mas, ainda, a comunidade, a sociedade e o próprio Estado,
mento internacional dos direitos humanos e a fundação para que todos, em conjunto, exerçam seus direitos e deve-
da UNICEF. A UNICEF, inicialmente conhecida como Fundo res sem oprimir aqueles que, em condição inferior, viviam
Internacional de Emergência das Nações Unidas para as a mercê da sociedade. Mas, qual a razão dessa inclusão tão
Crianças, foi criada em dezembro de 1946 para ajudar as abrangente? Pois bem, a intenção do Estatuto da Criança
crianças da Europa vítimas da II Guerra Mundial. No início e do Adolescente foi conferir ao menor, de forma integral,
da década de 50 o seu mandato foi alargado para respon- todas as condições para que o mesmo possa desenvolver-
der às necessidades das crianças e das mães nos países em se plenamente, evitando-se, com isso, que haja alguma de-
desenvolvimento. Em 1953, torna-se uma agência perma- ficiência em sua formação. Desta forma, a melhor solução
nente das Nações Unidas, e passa a ocupar-se especial- apresentada pelo legislador foi incluir todos os segmentos
mente das crianças dos países mais pobres da África, Ásia, da sociedade, para que ninguém ficasse isento de qualquer
América Latina e Médio Oriente. Passa então a designar-se responsabilidade, uma vez que a doutrina da proteção in-
Fundo das Nações Unidas para a Infância, mas mantém a tegral apresentada pelo Estatuto da Criança e do Adoles-
sigla que a tornara conhecida em todo o mundo – UNICEF. cente exige a participação de todos, sem qualquer exce-
Desde então, sobrevieram no âmbito das Nações Unidas ção”36. Com efeito, o objeto formal do direito da criança e
documentos bastante relevantes sobre a condição jurídica do adolescente é a proteção jurídica especial da criança e
peculiar da criança, já estudados neste material. do adolescente. Já o objeto material é a própria criança ou
No Brasil, no final do século XIX e início do século XX, adolescente.
foi instituído no Rio de Janeiro o Instituto de Proteção e
Assistência à Infância, primeiro estabelecimento público
nacional de atendimento a crianças e adolescentes. Em se-
guida, veio a Lei nº 4.242/1921, que autorizou o governo 35 DEZEM, Guilherme Madeira; AGUIRRE, João Ri-
a organizar o Serviço de Assistência e Proteção à Infância cardo Brandão; FULLER, Paulo Henrique Aranda. Estatu-
Abandonada e Dellinquente. Em 1927 foi aprovado o pri- to da Criança e do Adolescente. São Paulo: Revista dos
meiro Código de Menores. Em 1941, durante o governo Tribunais, 2009. (Coleção Elementos do Direito)
Vargas, foi criado o Serviço de Assistência ao Menor, cujo 36 MENDES, Moacyr Pereira. As relações jurídicas
fim era dar tratamento penal teoricamente diferenciado decorrentes do Estatuto da Criança e do Adolescente.
aos menores (na prática, eram tratados como criminosos Âmbito Jurídico, Rio Grande, XII, n. 70, nov. 2009.

43
LEGISLAÇÃO

Princípios Sem pretender estabelecer uma definição fechada ou


plena, é possível conceituar dignidade da pessoa humana
Não se pode olvidar que os princípios sempre desem- como o principal valor do ordenamento ético e, por con-
penharam um importante papel social, mas foi somente na sequência, jurídico que pretende colocar a pessoa humana
atual dogmática jurídica que eles adquiriram normativida- como um sujeito pleno de direitos e obrigações na or-
de. Hoje em dia, os princípios servem para condensar va- dem internacional e nacional, cujo desrespeito acarreta a
lores, dar unidade ao sistema e condicionar a atividade do própria exclusão de sua personalidade.
intérprete. Os princípios são normas jurídicas, não meros Aponta Barroso39: “o princípio da dignidade da pessoa
conteúdos axiológicos, aceitando aplicação autônoma37. humana identifica um espaço de integridade moral a ser
Em resumo, a teoria dos princípios chega à presente assegurado a todas as pessoas por sua só existência no
fase do Pós-positivismo com os seguintes resultados já mundo. É um respeito à criação, independente da crença
consolidados: a passagem dos princípios da especulação que se professe quanto à sua origem. A dignidade rela-
metafísica e abstrata para o campo concreto e positivo do ciona-se tanto com a liberdade e valores do espírito como
Direito, com baixíssimo teor de densidade normativa; a com as condições materiais de subsistência”.
transição crucial da ordem jusprivatista (sua antiga inser- O Ministro Alberto Luiz Bresciani de Fontan Pereira, do
ção nos Códigos) para a órbita juspublicística (seu ingresso Tribunal Superior do Trabalho, trouxe interessante conceito
nas Constituições); a suspensão da distinção clássica entre numa das decisões que relatou: “a dignidade consiste na
princípios e normas; o deslocamento dos princípios da es- percepção intrínseca de cada ser humano a respeito dos
fera da jusfilosofia para o domínio da Ciência Jurídica; a direitos e obrigações, de modo a assegurar, sob o foco de
proclamação de sua normatividade; a perda de seu cará- condições existenciais mínimas, a participação saudável e
ter de normas programáticas; o reconhecimento definitivo ativa nos destinos escolhidos, sem que isso importe des-
de sua positividade e concretude por obra sobretudo das tilação dos valores soberanos da democracia e das liber-
Constituições; a distinção entre regras e princípios, como dades individuais. O processo de valorização do indivíduo
espécies diversificadas do gênero norma, e, finalmente, por articula a promoção de escolhas, posturas e sonhos, sem
expressão máxima de todo esse desdobramento doutriná- olvidar que o espectro de abrangência das liberdades in-
rio, o mais significativo de seus efeitos: a total hegemonia dividuais encontra limitação em outros direitos fundamen-
e preeminência dos princípios38. tais, tais como a honra, a vida privada, a intimidade, a ima-
No campo do direito da criança e do adolescente, al- gem. Sobreleva registrar que essas garantias, associadas ao
guns princípios assumem destaque, entre eles: princípio da dignidade da pessoa humana, subsistem como
a) Princípio da prioridade absoluta: previsto nos ar- conquista da humanidade, razão pela qual auferiram pro-
tigos 227, CF e 4º, ECA preconiza que é dever de todos – teção especial consistente em indenização por dano moral
Estado, sociedade, comunidade e família – assegurar com decorrente de sua violação”40.
absoluta prioridade direitos fundamentais às crianças e Para Reale41, a evolução histórica demonstra o domínio
adolescentes. Por isso, estabelece-se com primazia a ado- de um valor sobre o outro, ou seja, a existência de uma
ção de políticas públicas, a destinação de recursos e a pres- ordem gradativa entre os valores; mas existem os valores
tação de serviços essenciais àqueles que se encontram na fundamentais e os secundários, sendo que o valor fonte
faixa etária inferior a 18 anos. é o da pessoa humana. Nesse sentido, são os dizeres de
b) Princípio da proteção integral: previsto no artigo Reale42: “partimos dessa ideia, a nosso ver básica, de que a
1º, ECA estabelece que a proteção da criança e do adoles- pessoa humana é o valor-fonte de todos os valores. O ho-
cente não pode se restringir às situações de irregularidade, mem, como ser natural biopsíquico, é apenas um indivíduo
o que teria um caráter estigmatizante, mas deve abranger entre outros indivíduos, um ente animal entre os demais
todas as situações de vida pelas quais passa a criança e o da mesma espécie. O homem, considerado na sua objeti-
adolescente, mesmo as regulares. Neste sentido, ao se as- vidade espiritual, enquanto ser que só realiza no sentido
segurar direitos na regularidade, evita-se que a criança e o de seu dever ser, é o que chamamos de pessoa. Só o ho-
adolescente caiam em irregularidade. mem possui a dignidade originária de ser enquanto deve
c) Princípio da dignidade da pessoa humana: A dig- ser, pondo-se essencialmente como razão determinante
nidade da pessoa humana é o valor-base de interpretação do processo histórico”.
de qualquer sistema jurídico, internacional ou nacional,
39 BARROSO, Luís Roberto. Interpretação e
que possa se considerar compatível com os valores éticos,
notadamente da moral, da justiça e da democracia. Pensar aplicação da Constituição. 7. ed. São Paulo: Saraiva,
em dignidade da pessoa humana significa, acima de tudo, 2009, p. 382.
colocar a pessoa humana como centro e norte para qual- 40 BRASIL. Tribunal Superior do Trabalho. Recurso
quer processo de interpretação jurídico, seja na elaboração de Revista n. 259300-59.2007.5.02.0202. Relator: Alber-
da norma, seja na sua aplicação. to Luiz Bresciani de Fontan Pereira. Brasília, 05 de setem-
bro de 2012j1. Disponível em: www.tst.gov.br. Acesso em:
17 nov. 2012.
37 Ibid., p.327. 41 REALE, Miguel. Filosofia do direito. 19. ed.
38 BONAVIDES, Paulo. Curso de direito consti- São Paulo: Saraiva, 2002, p. 228.
tucional. 26. ed. São Paulo: Malheiros, 2011, p. 294. 42 Ibid., p. 220.

44
LEGISLAÇÃO

Quando a Constituição Federal assegura a dignidade No entender de Kopelman, para que toda esta legisla-
da pessoa humana como um dos fundamentos da Repúbli- ção fosse realmente válida seria necessário definir melhor,
ca, faz emergir uma nova concepção de proteção de cada de maneira bem precisa, o que se entende por um padrão
membro do seu povo. Tal ideologia de forte fulcro huma- mínimo de benefício ou o que é ‘o melhor’ para os interes-
nista guia a afirmação de todos os direitos fundamentais ses da criança ou do adolescente, de modo que a definição
e confere a eles posição hierárquica superior às normas não fique em aberto para a interpretação de quem detém
organizacionais do Estado, de modo que é o Estado que o poder de decidir em nome deles. Além disso, estas defi-
está para o povo, devendo garantir a dignidade de seus nições deveriam estar em constante revisão, para que não
membros, e não o inverso. acabem sendo ultrapassadas, frente à evolução histórico-
d) Princípio da participação popular: previsto no ar- social dos fatos que geraram a necessidade de sua criação.
tigo 227, §§ 3º e 7º e no artigo 204, II, CF, assegura a par- Superados estes dois pontos, que apesar de potencial-
ticipação popular, através de organizações representativas, mente limitantes do processo de discussão da autonomia
na elaboração de políticas públicas direcionadas à infância da criança e do adolescente não podem ser simplesmente
e à juventude. ignorados, como se não existissem, chega-se ao terceiro e
e) Princípio da excepcionalidade: previsto no artigo
mais importante: a interpretação do conceito de autono-
227, §3º, V, CF assegura que quando da imposição de me-
mia à luz do momento de desenvolvimento em que uma
dida privativa de liberdade esta não será imposta a não
determinada criança ou adolescente se encontra.
ser que se trate de um caso excepcional, em que nenhuma
outra medida sócio-educativa possa ser utilizada. Nesse sentido, diversas características do desenvolvi-
f) Princípio da brevidade: previsto no artigo 227, §3º, mento devem ser levadas em consideração:
V, CF assegura que quando da aplicação de medida privati- 1. Trata-se de um processo que evolui continuamente
va de liberdade esta não se estenderá no tempo, devendo à medida que habilidades se aperfeiçoam, novas capaci-
ser a mais breve possível, perdurando apenas pelo prazo dades são adquiridas, novas vivências são acumuladas e
necessário para a ressocialização do adolescente. No caso, integradas e, portanto, passível de rápidas e extremas mu-
o ECA limita a aplicação de medidas desta natureza ao pra- danças no tempo;
zo máximo de 3 anos. 2. A aquisição das competências é progressiva, não se
g) Princípio da condição peculiar da pessoa em de- dá saltos, como se se tratasse de compartimentos estan-
senvolvimento: a criança e o adolescente estão em pro- ques, e segue sempre uma ordem preestabelecida, sendo,
cesso de formação e de transformação física e psíquica, portanto, razoavelmente previsível;
logo, possuem uma condição peculiar que deve ser respei- 3. Os tempos e o ritmo em que o desenvolvimento se
tada quando da aplicação da lei. processa são muito individualizados, fazendo com que dois
indivíduos de uma mesma idade possam estar em momen-
Autonomia da criança e do adolescente tos diferentes de desenvolvimento;
4. No caso específico da inteligência, o desenvolvimen-
Coloca-se o trecho do trabalho de Cláudio Leone43 em to é extremamente influenciável por fatores extrínsecos ao
que reflete sobre a construção da autonomia do infante: indivíduo: as experiências, os estímulos, o ambiente, a edu-
“Conceitualmente, a análise do respeito à autonomia cação, a cultura, etc., o que também acaba por reforçar sua
de uma criança ou de um adolescente só tem sentido se for evolução extremamente individualizada.
conduzida a partir do conhecimento da evolução de suas Segundo Piaget, a capacidade de operar o pensamen-
competências nas diferentes idades. É de conhecimento to concreto estendendo-o à compreensão do outro e às
de todos que a criança nasce totalmente dependente de possíveis consequências de boa parte dos seus atos se
cuidados alheios e que passa por um processo de desen- aperfeiçoa na idade escolar, entre os 6 e os 11 anos de vida.
volvimento progressivo que a leva a alcançar a completa
Este amadurecimento se completa na adolescência, com a
independência na maturidade, o que, nas sociedades mo-
capacidade crescente de abstração que a criança desenvol-
dernas, se situa por volta dos vinte anos de idade.
ve nesta fase da existência. Como consequência, é possível
Entretanto, para que este processo de análise de sua
autonomia transcorra de maneira isenta, fundamentalmen- admitir que é na segunda fase da adolescência, em geral a
te centrado nas peculiaridades do desenvolvimento do ser partir dos 15 anos, que o indivíduo atingiria as competên-
humano, o primeiro ponto a ser considerado é a necessida- cias necessárias para o exercício de sua autonomia, com-
de de abdicar de alguns conceitos preestabelecidos, como petências estas que necessitariam apenas serem lapidadas
é o caso da atitude paternalista. [...] ao longo das vivências e de uma maior experiência de vida.
O segundo ponto a considerar neste percurso, em ge- Entretanto, isto não significa que a autonomia da crian-
ral decorrente do primeiro, é a própria legislação que, mes- ça e do adolescente só possa (ou deva) ser respeitada a
mo tendo o melhor dos intuitos, praticamente nivela todos partir desta fase.
os menores a uma mesma condição: a de incapacidade, Compete ao pediatra e aos demais profissionais de
criando a necessidade de se ter figuras aptas a decidir e saúde, utilizando suas competências profissionais, definir
responder por eles, como se estas figuras fossem sempre e já desde os primeiros anos de vida em que etapa a criança
inevitavelmente imbuídas das melhores intenções em rela- se encontra ao longo do seu processo evolutivo, tentando
ção à criança e ao adolescente. diferenciar se se está diante de uma tomada de decisão
43 LEONE, Cláudio. A criança, o adolescente e a au- ditada apenas pelo receio do desconhecido, por um capri-
tonomia. Revista Bioética, v. 6, n. 1. cho ou vontade decorrente apenas de sua visão egocên-

45
LEGISLAÇÃO

trica, natural em determinadas idades, ou se a mesma já é O artigo 228, CF dispõe: “são penalmente inimputáveis
o resultado de uma reflexão mais amadurecida. São estes os menores de dezoito anos, sujeitos às normas da legis-
extremos que dão a entender a ampla gama de estágios de lação especial”. Percebe-se que a normativa não está no
desenvolvimento, portanto de autonomia, que entre eles rol de cláusulas pétreas, razão pela qual seria possível uma
podem se apresentar. [...] emenda constitucional que alterasse a menoridade penal.
Novamente, cabe enfatizar que o risco que se corre ao Inclusive, há projetos de lei neste sentido.
se utilizar definições bastante precisas como estas é o de
acabar classificando um indivíduo de maneira dicotômica, Comentários à lei
no caso específico da autonomia, como sendo capaz ou
incapaz, desistindo assim de uma possível análise de sua Parte geral
real capacidade.
Consequentemente, a ausência de uma ou de mais das Título I
características anteriormente citadas não deve ser utilizada Das Disposições Preliminares
para qualificar a criança ou o adolescente como incapaz.
Deve, isto sim, servir de embasamento para que se possa Art. 1º Esta Lei dispõe sobre a proteção integral à
tentar entender como suas decisões se originaram. criança e ao adolescente.
Em face de situações específicas, individualizadas, O princípio da proteção integral se associa ao princípio
como ocorre no dia-a-dia da prática pediátrica, esta é a da prioridade absoluta, colacionado no artigo 4º do ECA e
única forma que o profissional tem de realmente respeitar no artigo 227, CF. “Com a positivação desse princípio tem-
a autonomia da criança ou do adolescente. se também a positivação da proteção integral, que se opõe
A interpretação adequada da legislação e o dimen- à antiga e superada doutrina da situação irregular, que era
sionamento correto da decisão dos pais ou responsáveis prevista no antigo Código de Menores e especificava que
dependerão fundamentalmente deste tipo de análise da sua incidência se restringia aos menores em situação irre-
autonomia da criança ou adolescente. Deste modo, mes- gular, apresentando um conjunto de normas destinadas ao
mo que resulte em situações de conflito entre as posições, tratamento e prevenção dessas situações”44.
servirá de embasamento para um trabalho, muitas vezes Basicamente, tinha-se na doutrina da situação irregu-
lar que era necessário disciplinar um estatuto jurídico da
exaustivo, de apresentação, de reflexão e de discussão de
criança e do adolescente que apenas abordasse situações
argumentos e fatos, capaz de conduzir a uma decisão ama-
em que ele estivesse irregular, seja por uma desproteção,
durecida e o mais isenta possível, que, respeitando a po-
como no caso de abandono, ou pela violação da lei, como
sição da criança ou do adolescente, poderá efetivamente
nos casos de atos infracionais.
redundar em seu benefício.
Entretanto, o direito evoluiu e passou a contemplar
No leque das diferentes situações da prática pediá-
uma noção de proteção mais ampla da criança e do ado-
trica, que se estende desde o recém-nascido no limite de
lescente, que não apenas abordasse situações de irregula-
viabilidade ao qual se quer prestar cuidados intensivos de ridade (embora ainda o fizesse), mas que abrangesse todo
validade questionável naquelas circunstâncias, passando o arcabouço jurídico protetivo da criança e do adolescente.
pelas pesquisas científicas que envolvem crianças e ado-
lescentes, até a criança cujo pátrio poder pertence a pais Art. 2º Considera-se criança, para os efeitos desta Lei, a
adolescentes, portanto autônomos nas decisões que lhes pessoa até doze anos de idade incompletos, e adolescente
dizem respeito, todas estas situações, onde nem sempre o aquela entre doze e dezoito anos de idade.
real interesse que está em jogo é o da criança, mas sim o Parágrafo único. Nos casos expressos em lei, aplica-se
dos responsáveis por ela, clarificam que não há uma única excepcionalmente este Estatuto às pessoas entre dezoito e
resposta ou solução mágica, perfeita, para a questão da vinte e um anos de idade.
autonomia da criança e do adolescente. O Estatuto da Criança e do Adolescente opta por cate-
Na realidade, o que deve existir é a construção conjun- gorizar separadamente estas duas categorias de menores.
ta de uma verdade para aquele momento, amadurecida no Criança é aquele que tem até 12 anos de idade (na data
crescimento e evolução de todos: juízes e legisladores, pais de aniversário de 12 anos, passa a ser adolescente), ado-
ou responsáveis, médicos e profissionais de saúde e, prin- lescente é aquele que tem entre 12 e 18 anos (na data de
cipalmente, a criança ou o adolescente, como parte de um aniversário de 18 anos, passa a ser maior). Em situações
processo de interação franco, sincero, isento e realmente excepcionais o ECA se aplica ao maior de 18 anos, até os
participativo que de fato respeite a autonomia, qualquer 21 anos de idade, por exemplo, no caso do menor infrator
que seja o nível de competência que a criança ou o adoles- sujeito a internação em fundação CASA que tenha 17 anos
cente estejam apresentando para tal”. e 11 meses na data do ato infracional poderá ficar detido
até o limite de seus 20 anos e 11 meses (eis que 3 anos é o
Imputabilidade penal tempo máximo de internação).

Art. 228, CF. São penalmente inimputáveis os meno- 44 DEZEM, Guilherme Madeira; AGUIRRE, João Ri-
res de dezoito anos, sujeitos às normas da legislação es- cardo Brandão; FULLER, Paulo Henrique Aranda. Estatu-
pecial. to da Criança e do Adolescente. São Paulo: Revista dos
Tribunais, 2009. (Coleção Elementos do Direito)

46
LEGISLAÇÃO

Art. 3º A criança e o adolescente gozam de todos os di- Deste artigo 3º do ECA é possível, ainda, extrair o des-
reitos fundamentais inerentes à pessoa humana, sem taque ao princípio da igualdade, no sentido de que há ple-
prejuízo da proteção integral de que trata esta Lei, assegu- na igualdade na garantia de direitos entre todas as crian-
rando-se-lhes, por lei ou por outros meios, todas as opor- ças e adolescentes, não sendo permitido qualquer tipo de
tunidades e facilidades, a fim de lhes facultar o desen- discriminação.
volvimento físico, mental, moral, espiritual e social, em
condições de liberdade e de dignidade. A leitura dos artigos 4º e 5º, em conjunto com outros
Parágrafo único. Os direitos enunciados nesta Lei apli- dispositivos do ECA, por sua vez, permite detectar a pre-
cam-se a todas as crianças e adolescentes, sem discrimi- sença de um tríplice sistema de garantias.
nação de nascimento, situação familiar, idade, sexo, raça, Assim, o Estatuto da Criança e do Adolescente adota
etnia ou cor, religião ou crença, deficiência, condição pessoal uma estrutura que contempla três sistemas de garantia –
de desenvolvimento e aprendizagem, condição econômica, primário, secundário e terciário.
ambiente social, região e local de moradia ou outra condição a) Sistema primário – artigos 4º e 87, ECA – aborda
que diferencie as pessoas, as famílias ou a comunidade em políticas públicas de atendimento de crianças e adolescen-
que vivem. tes.
O artigo 3º volta-se à concretização dos direitos da
criança e do adolescente. Concretização significa viabiliza- Art. 4º É dever da família, da comunidade, da socie-
ção prática, consecução real dos fins que a lei descreve. dade em geral e do poder público assegurar, com abso-
Como se percebe pela leitura até o momento, o legislador luta prioridade, a efetivação dos direitos referentes à vida,
brasileiro preocupou-se em elaborar uma legislação cujo à saúde, à alimentação, à educação, ao esporte, ao lazer,
objetivo é concretizar estes direitos da criança e do adoles- à profissionalização, à cultura, à dignidade, ao respeito, à
cente. Entretanto, a lei é apenas uma carta de intenções. É liberdade e à convivência familiar e comunitária.
necessário colocar seu conteúdo em prática, porque sozi- Parágrafo único. A garantia de prioridade compreen-
nha ela nada faz. de:
A implementação na prática dos direitos da criança e a) primazia de receber proteção e socorro em quais-
do adolescente depende da adoção de posturas por parte quer circunstâncias;
de todos aqueles colocados como responsáveis para tanto: b) precedência de atendimento nos serviços públicos
Estado, sociedade, comunidade e família. Especificamente ou de relevância pública;
no que se refere ao Estado, mostra-se essencial que ele c) preferência na formulação e na execução das po-
desenvolve políticas públicas adequadas em respeito à pe- líticas sociais públicas;
culiar condição do infante. d) destinação privilegiada de recursos públicos nas
“O Direito da Criança e do Adolescente deve ter con- áreas relacionadas com a proteção à infância e à juventude.
dições suficientemente próprias de promoção e concreti- O artigo 4º do ECA colaciona em seu caput teor idênti-
zação de direitos. Para isso deve-se desvencilhar do dog- co ao do caput do artigo 227, CF, onde se encontra uma das
matismo e do mero positivismo jurídico acrítico. O Direito principais diretrizes do direito da criança e do adolescente
da Criança e do Adolescente enquanto ramo autônomo que é o princípio da prioridade absoluta. Significa que cada
do direito é responsável por ressignificar a atuação estatal, criança e adolescente deve receber tratamento especial do
principalmente no campo das políticas públicas e impõe Estado e ser priorizado em suas políticas públicas, pois são
corresponsabilidades compartilhadas”45. o futuro do país e as bases de construção da sociedade.
Vale ressaltar que às crianças e aos adolescentes são Explica Liberati46: “Por absoluta prioridade, devemos
garantidos os mesmos direitos fundamentais que aos entender que a criança e o adolescente deverão estar em
adultos, entretanto, o ECA aprofunda alguns direitos fun- primeiro lugar na escala de preocupação dos governantes;
damentais em espécie, abordando-os na vertente da con- devemos entender que, primeiro, devem ser atendidas to-
dição especial dos que pertencem a este grupo. das as necessidades das crianças e adolescentes [...]. Por
As crianças e adolescentes gozam de igualdade de di- absoluta prioridade, entende-se que, na área administra-
reitos em relação às demais pessoas, podendo usufruir de tiva, enquanto não existirem creches, escolas, postos de
todos eles. O próprio estatuto contempla em seu título II saúde, atendimento preventivo e emergencial às gestantes
os direitos fundamentais da criança e do adolescente, entre dignas moradias e trabalho, não se deveria asfaltar ruas,
eles incluindo-se: vida, saúde, liberdade, respeito, dignida- construir praças, sambódromos monumentos artísticos
de, convivência familiar e comunitária, educação, cultura, etc., porque a vida, a saúde, o lar, a prevenção de doenças
esporte, lazer, profissionalização e proteção no trabalho. são importantes que as obras de concreto que ficam par a
Não se trata de rol taxativo de direitos fundamentais ga- demonstrar o poder do governante”.
rantidos à criança e ao adolescente, eis que ele possui to- O parágrafo único do artigo 4º especifica a abrangência
dos os direitos humanos e fundamentais que as demais da absoluta prioridade, esclarecendo que é necessário con-
pessoas. O título II do ECA tem por objetivo aprofundar ferir atendimento prioritário às crianças e aos adolescentes
especificidades acerca de algumas das categorias de direi- diante de situações de perigo e risco (como no salvamento
tos fundamentais assegurados à criança e ao adolescente. em incêndios e enchentes, etc.), bem como nos serviços
45 http://t.boletimjuridico.com.br/doutrina/texto. 46 LIBERATI, Wilson Donizeti. O Estatuto da Crian-
asp?id=2236 ça e do Adolescente: Comentários. São Paulo: IBPS.

47
LEGISLAÇÃO

públicos em geral (chegada aos hospitais, por exemplo). O artigo 5º ressalta o verdadeiro objetivo geral do ECA:
Além disso, devem ser priorizadas políticas públicas que proteger a criança de qualquer forma de negligência, dis-
favoreçam a criança e o adolescente e também devem ser criminação, exploração, violência, crueldade e opressão.
reservados recursos próprios prioritariamente a eles. Neste sentido, coloca-se a possibilidade de responsabi-
lização de todos que atentarem contra esse propósito. A
Art. 87. São linhas de ação da política de atendimen- responsabilização poderá se dar em qualquer uma das três
to: esferas, isolada ou cumulativamente: penal, respondendo
I - políticas sociais básicas; por crimes e contravenções penais todo aquele que praticá
II - serviços, programas, projetos e benefícios de as- -lo contra criança e adolescente, bem como respondendo
sistência social de garantia de proteção social e de preven- por atos infracionais as crianças e adolescentes que atenta-
ção e redução de violações de direitos, seus agravamentos rem um contra o outro; civil, estabelecendo-se o dever de
ou reincidências; indenizar por danos causados a crianças e a adolescentes,
III - serviços especiais de prevenção e atendimento que se estende a toda e qualquer pessoa física ou jurídica
médico e psicossocial às vítimas de negligência, maus-tra-
que o faça, inclusive o próprio Estado; e administrativa, im-
tos, exploração, abuso, crueldade e opressão;
pondo-se penas disciplinares a funcionários sujeitos a re-
IV - serviço de identificação e localização de pais,
gime jurídico administrativo em trabalhos privados ou em
responsável, crianças e adolescentes desaparecidos;
V - proteção jurídico-social por entidades de defesa cargos, empregos e funções públicos.
dos direitos da criança e do adolescente.
VI - políticas e programas destinados a prevenir ou Art. 6º Na interpretação desta Lei levar-se-ão em conta
abreviar o período de afastamento do convívio familiar os fins sociais a que ela se dirige, as exigências do bem
e a garantir o efetivo exercício do direito à convivência comum, os direitos e deveres individuais e coletivos, e a
familiar de crianças e adolescentes; condição peculiar da criança e do adolescente como pes-
VII - campanhas de estímulo ao acolhimento sob soas em desenvolvimento.
forma de guarda de crianças e adolescentes afastados do
convívio familiar e à adoção, especificamente inter-racial, É pacífico que o processo de interpretação hoje faz
de crianças maiores ou de adolescentes, com necessidades parte do Direito, principalmente se considerada a constan-
específicas de saúde ou com deficiências e de grupos de ir- te evolução da sociedade, demandando diariamente por
mãos.  novos modos de aplicação das normas. Como a sociedade
O artigo 87 descreve linhas de ação na política de é dinâmica e o Direito existe para servi-la, cabe a ele ade-
atendimento, que compõem a delimitação do princípio da quar-se às novas exigências sociais, aplicando-se da ma-
prioridade absoluta na vertente da priorização na adoção neira mais justa à vasta gama de casos concretos. Sobre a
de políticas públicas e na delimitação de recursos financei- interpretação, explica Gonçalves47: “Quando o fato é típico
ros para execução de tais políticas. e se enquadra perfeitamente no conceito abstrato da nor-
ma, dá-se o fenômeno da subsunção. Há casos, no entanto,
b) Sistema secundário – artigos 98 e 101, ECA – abor- em que tal enquadramento não ocorre, não encontrando o
da as medidas de proteção destinadas à criança e ao ado- juiz nenhuma norma aplicável à hipótese sub judice. Deve,
lescente em situação de risco pessoal ou social. então, proceder à integração normativa, mediante o em-
Obs.: as medidas de proteção são estudadas adiante prego da analogia, dos costumes e dos princípios gerais do
neste material. direito. [...] Para verificar se a norma é aplicável ao caso em
c) Sistema terciário – artigo 112, ECA – aborda as julgamento (subsunção) ou se deve proceder à integração
medidas socioeducativas, destinadas à responsabilização
normativa, o juiz procura descobrir o sentido da norma, in-
penal do adolescente infrator, isto é, àquele entre 12 e 18
terpretando-a. Interpretar é descobrir o sentido e o alcance
anos que comete atos infracionais.
da norma jurídica”.
Obs.: as medidas socioeducativas são estudadas adian-
te neste material. A hermenêutica possui 3 categorias de métodos.
O sistema tríplice deve operar de forma harmônica, Quanto às fontes ou origem, a interpretação pode ser
com o acionamento gradual de cada um deles. Nas situa- autêntica ou legislativa, jurisprudencial ou judicial e dou-
ções em que a criança ou adolescente escape ao sistema trinária. Quanto aos meios, pode ser gramatical ou literal,
primário de prevenção, ou seja, nos casos de ineficácia das examinando o texto normativo linguísticamente; lógica ou
políticas públicas específicas, deve ser acionado o sistema racional, apurando o sentido e a finalidade da norma; sis-
secundário, operado predominantemente pelo Conselho temática, analisando a lei de maneira comparativa com ou-
Tutelas. Por sua vez, em casos extremos, é necessário partir tras leis pertencentes à mesma província do Direito (livro,
para a adoção de medidas socioeducativas, operadas pre- título, capítulo, seção, parágrafo); histórica, baseando-se na
dominantemente pelo Ministério Público e pelo Judiciário. verificação dos antecedentes do processo legislativo; so-
ciológica, adaptando o sentido ou finalidade da nor-
Art. 5º Nenhuma criança ou adolescente será objeto de ma às novas exigências sociais (artigo 5°, LINDB). Quanto
qualquer forma de negligência, discriminação, explora- aos resultados pode ser declarativa, quando o texto legal
ção, violência, crueldade e opressão, punido na forma da corresponde ao pensamento do legislador; extensiva ou
lei qualquer atentado, por ação ou omissão, aos seus direitos 47 GONÇALVES, Carlos Roberto. Direito Civil Bra-
fundamentais. sileiro. 9. ed. São Paulo: Saraiva, 2011, v. 1.

48
LEGISLAÇÃO

ampliativa, quando o alcance da lei é mais amplo que o in- § 8º A gestante tem direito a acompanhamento sau-
dicado pelo seu texto; e restritiva, na qual se limita o cam- dável durante toda a gestação e a parto natural cuidadoso,
po de aplicação da lei. Nenhum destes métodos se opera estabelecendo-se a aplicação de cesariana e outras inter-
isoladamente48. venções cirúrgicas por motivos médicos.
O artigo 6º do ECA, tal como o artigo 5º da LINDB, ex- § 9º A atenção primária à saúde fará a busca ativa da
pressa o método de interpretação sociológico, chamando gestante que não iniciar ou que abandonar as consultas de
atenção à interpretação da lei levando em conta os seus pré-natal, bem como da puérpera que não comparecer às
fins sociais, as exigências do bem comum, os direitos e de- consultas pós-parto.
veres individuais e coletivos, e vai além: exige que se leve § 10. Incumbe ao poder público garantir, à gestante e
em conta a condição peculiar da criança e do adolescente. à mulher com filho na primeira infância que se encontrem
Logo, ao se interpretar o ECA não se pode nunca perder de sob custódia em unidade de privação de liberdade, am-
vista que o seu objeto material, a criança e o adolescente, é biência que atenda às normas sanitárias e assistenciais do
extremamente peculiar, dotado de especificidades as quais Sistema Único de Saúde para o acolhimento do filho, em
sempre se deve atentar. articulação com o sistema de ensino competente, visando
ao desenvolvimento integral da criança.
Título II
Dos Direitos Fundamentais
Art. 9º O poder público, as instituições e os emprega-
Capítulo I dores propiciarão condições adequadas ao aleitamento
Do Direito à Vida e à Saúde materno, inclusive aos filhos de mães submetidas a me-
dida privativa de liberdade.
Art. 7º A criança e o adolescente têm direito a proteção § 1º Os profissionais das unidades primárias de saúde
à vida e à saúde, mediante a efetivação de políticas sociais desenvolverão ações sistemáticas, individuais ou coletivas,
públicas que permitam o nascimento e o desenvolvimen- visando ao planejamento, à implementação e à avaliação
to sadio e harmonioso, em condições dignas de existência. de ações de promoção, proteção e apoio ao aleitamento
materno e à alimentação complementar saudável, de for-
Art. 8º É assegurado a todas as mulheres o acesso aos ma contínua.
programas e às políticas de saúde da mulher e de pla- § 2º Os serviços de unidades de terapia intensiva neo-
nejamento reprodutivo e, às gestantes, nutrição adequa- natal deverão dispor de banco de leite humano ou unidade
da, atenção humanizada à gravidez, ao parto e ao puerpé- de coleta de leite humano.
rio e atendimento pré-natal, perinatal e pós-natal integral
no âmbito do Sistema Único de Saúde. Art. 10. Os hospitais e demais estabelecimentos de
§ 1º O atendimento pré-natal será realizado por pro- atenção à saúde de gestantes, públicos e particulares, são
fissionais da atenção primária. obrigados a:
§ 2º Os profissionais de saúde de referência da gestan- I - manter registro das atividades desenvolvidas, através
te garantirão sua vinculação, no último trimestre da ges- de prontuários individuais, pelo prazo de dezoito anos;
tação, ao estabelecimento em que será realizado o parto, II - identificar o recém-nascido mediante o registro
garantido o direito de opção da mulher. de sua impressão plantar e digital e da impressão digital
§ 3º Os serviços de saúde onde o parto for realizado da mãe, sem prejuízo de outras formas normatizadas pela
assegurarão às mulheres e aos seus filhos recém-nascidos autoridade administrativa competente;
alta hospitalar responsável e contrarreferência na atenção III - proceder a exames visando ao diagnóstico e tera-
primária, bem como o acesso a outros serviços e a grupos de pêutica de anormalidades no metabolismo do recém-nasci-
apoio à amamentação.
do, bem como prestar orientação aos pais;
§ 4º Incumbe ao poder público proporcionar assis-
IV - fornecer declaração de nascimento onde constem
tência psicológica à gestante e à mãe, no período pré e
necessariamente as intercorrências do parto e do desenvol-
pós-natal, inclusive como forma de prevenir ou minorar as
consequências do estado puerperal. vimento do neonato;
§ 5º A assistência referida no § 4º deste artigo deverá V - manter alojamento conjunto, possibilitando ao neo-
ser prestada também a gestantes e mães que manifestem nato a permanência junto à mãe.
interesse em entregar seus filhos para adoção, bem como
a gestantes e mães que se encontrem em situação de pri- Art. 11. É assegurado acesso integral às linhas de
vação de liberdade. cuidado voltadas à saúde da criança e do adolescen-
§ 6º A gestante e a parturiente têm direito a 1 (um) te, por intermédio do Sistema Único de Saúde, observado
acompanhante de sua preferência durante o período do o princípio da equidade no acesso a ações e serviços para
pré-natal, do trabalho de parto e do pós-parto imediato. promoção, proteção e recuperação da saúde.
§ 7º A gestante deverá receber orientação sobre alei- § 1º A criança e o adolescente com deficiência serão
tamento materno, alimentação complementar saudável e atendidos, sem discriminação ou segregação, em suas
crescimento e desenvolvimento infantil, bem como sobre necessidades gerais de saúde e específicas de habilitação e
formas de favorecer a criação de vínculos afetivos e de es- reabilitação.
timular o desenvolvimento integral da criança.
48 Ibid.

49
LEGISLAÇÃO

§ 2º Incumbe ao poder público fornecer gratuitamen- Capítulo II


te, àqueles que necessitarem, medicamentos, órteses, Do Direito à Liberdade, ao Respeito e à Dignidade
próteses e outras tecnologias assistivas relativas ao tra-
tamento, habilitação ou reabilitação para crianças e adoles- Art. 15. A criança e o adolescente têm direito à liber-
centes, de acordo com as linhas de cuidado voltadas às suas dade, ao respeito e à dignidade como pessoas humanas
necessidades específicas. em processo de desenvolvimento e como sujeitos de direitos
§ 3º Os profissionais que atuam no cuidado diário ou civis, humanos e sociais garantidos na Constituição e nas
frequente de crianças na primeira infância receberão for- leis.
mação específica e permanente para a detecção de sinais Entre os direitos fundamentais garantidos à criança e
de risco para o desenvolvimento psíquico, bem como para ao adolescente que são especificados e aprofundados no
o acompanhamento que se fizer necessário. ECA estão os direitos à liberdade, ao respeito e à dignida-
de.
Art. 12. Os estabelecimentos de atendimento à saúde,
inclusive as unidades neonatais, de terapia intensiva e de Art. 16. O direito à liberdade compreende os seguintes
cuidados intermediários, deverão proporcionar condições aspectos:
para a permanência em tempo integral de um dos pais I - ir, vir e estar nos logradouros públicos e espaços
ou responsável, nos casos de internação de criança ou ado- comunitários, ressalvadas as restrições legais;
lescente. II - opinião e expressão;
III - crença e culto religioso;
Art. 13. Os casos de suspeita ou confirmação de cas- IV - brincar, praticar esportes e divertir-se;
tigo físico, de tratamento cruel ou degradante e de V - participar da vida familiar e comunitária, sem
maus-tratos contra criança ou adolescente serão obrigato- discriminação;
riamente comunicados ao Conselho Tutelar da respectiva VI - participar da vida política, na forma da lei;
localidade, sem prejuízo de outras providências legais. VII - buscar refúgio, auxílio e orientação.
§ 1º As gestantes ou mães que manifestem interesse O artigo 16 aborda diversas facetas do direito de liber-
em entregar seus filhos para adoção serão obrigatoriamen- dade: locomoção, opinião e expressão, religiosa e política.
te encaminhadas, sem constrangimento, à Justiça da Infân- Cria, ainda, duas facetes específicas deste direito: liberdade
cia e da Juventude. para brincar e divertir-se e liberdade para buscar refúgio,
§ 2º Os serviços de saúde em suas diferentes portas de auxílio e orientação, processos estes essenciais para o de-
entrada, os serviços de assistência social em seu compo- senvolvimento do infante.
nente especializado, o Centro de Referência Especializado
de Assistência Social (Creas) e os demais órgãos do Sistema Art. 17. O direito ao respeito consiste na inviolabilida-
de Garantia de Direitos da Criança e do Adolescente deverão de da integridade física, psíquica e moral da criança e
conferir máxima prioridade ao atendimento das crianças na do adolescente, abrangendo a preservação da imagem, da
faixa etária da primeira infância com suspeita ou confirma- identidade, da autonomia, dos valores, ideias e crenças, dos
ção de violência de qualquer natureza, formulando projeto espaços e objetos pessoais.
terapêutico singular que inclua intervenção em rede e, se
necessário, acompanhamento domiciliar. Art. 18. É dever de todos velar pela dignidade da
criança e do adolescente, pondo-os a salvo de qualquer
Art. 14. O Sistema Único de Saúde promoverá progra- tratamento desumano, violento, aterrorizante, vexató-
mas de assistência médica e odontológica para a pre- rio ou constrangedor.
venção das enfermidades que ordinariamente afetam a po- Os direitos ao respeito e à dignidade abrangem a pro-
pulação infantil, e campanhas de educação sanitária para teção da criança e do adolescente em todas facetas de sua
pais, educadores e alunos. integridade: física, psíquica e moral.
§ 1º É obrigatória a vacinação das crianças nos casos
recomendados pelas autoridades sanitárias. Art. 18-A. A criança e o adolescente têm o direito de
§ 2º O Sistema Único de Saúde promoverá a aten- ser educados e cuidados sem o uso de castigo físico ou
ção à saúde bucal das crianças e das gestantes, de forma de tratamento cruel ou degradante, como formas de cor-
transversal, integral e intersetorial com as demais linhas de reção, disciplina, educação ou qualquer outro pretexto,
cuidado direcionadas à mulher e à criança. pelos pais, pelos integrantes da família ampliada, pelos res-
§ 3º A atenção odontológica à criança terá função ponsáveis, pelos agentes públicos executores de medidas so-
educativa protetiva e será prestada, inicialmente, antes de o cioeducativas ou por qualquer pessoa encarregada de cuidar
bebê nascer, por meio de aconselhamento pré-natal, e, pos- deles, tratá-los, educá-los ou protegê-los.
teriormente, no sexto e no décimo segundo anos de vida, Parágrafo único.  Para os fins desta Lei, considera-se:
com orientações sobre saúde bucal. I - castigo físico: ação de natureza disciplinar ou pu-
§ 4º A criança com necessidade de cuidados odontoló- nitiva aplicada com o uso da força física sobre a criança
gicos especiais será atendida pelo Sistema Único de Saúde. ou o adolescente que resulte em: 
a) sofrimento físico; ou
b) lesão;

50
LEGISLAÇÃO

II - tratamento cruel ou degradante: conduta ou for- Capítulo III


ma cruel de tratamento em relação à criança ou ao adoles- Do Direito à Convivência Familiar e Comunitária
cente que:
a) humilhe; ou Seção I
b) ameace gravemente; ou Disposições Gerais
c) ridicularize.
Quando se aborda o direito à convivência familiar e
Art. 18-B. Os pais, os integrantes da família ampliada, os comunitária no ECA confere-se destaque à distinção entre
responsáveis, os agentes públicos executores de medidas so- família natural e substituta e aos procedimentos que carac-
cioeducativas ou qualquer pessoa encarregada de cuidar terizam a inserção e a retirada da criança e do adolescente
de crianças e de adolescentes, tratá-los, educá-los ou pro- destes ambientes.
tegê-los que utilizarem castigo físico ou tratamento cruel ou
degradante como formas de correção, disciplina, educação Art. 19. É direito da criança e do adolescente ser
ou qualquer outro pretexto estarão sujeitos, sem prejuízo de criado e educado no seio de sua família e, excepcional-
outras sanções cabíveis, às seguintes medidas, que serão mente, em família substituta, assegurada a convivência
aplicadas de acordo com a gravidade do caso: familiar e comunitária, em ambiente que garanta seu desen-
I - encaminhamento a programa oficial ou comunitá- volvimento integral.
rio de proteção à família; § 1o  Toda criança ou adolescente que estiver inserido
II - encaminhamento a tratamento psicológico ou em programa de acolhimento familiar ou institucional
psiquiátrico; terá sua situação reavaliada, no máximo, a cada 6 (seis)
III - encaminhamento a cursos ou programas de meses, devendo a autoridade judiciária competente, com
orientação; base em relatório elaborado por equipe interprofissional ou
IV - obrigação de encaminhar a criança a tratamento multidisciplinar, decidir de forma fundamentada pela possi-
especializado; bilidade de reintegração familiar ou colocação em família
V - advertência. substituta, em quaisquer das modalidades previstas no art.
Parágrafo único.  As medidas previstas neste artigo se- 28 desta Lei. 
rão aplicadas pelo Conselho Tutelar, sem prejuízo de outras § 2o  A permanência da criança e do adolescente em
providências legais.  programa de acolhimento institucional não se prolongará
por mais de 2 (dois) anos, salvo comprovada necessidade
Os artigos 18-A e 18-B foram incluídos no ECA pela Lei que atenda ao seu superior interesse, devidamente funda-
nº 13.010, de 26 de junho de 2014, que estabelece o direito mentada pela autoridade judiciária. 
da criança e do adolescente de serem educados e cuidados § 3º A manutenção ou a reintegração de criança ou
sem o uso de castigos físicos ou de tratamento cruel ou de- adolescente à sua família terá preferência em relação a
gradante. Também ficou conhecida como “Lei do Menino qualquer outra providência, caso em que será esta incluí-
Bernardo”49 e “Lei da Palmada”. da em serviços e programas de proteção, apoio e promoção,
Em que pesem as aparentes boas intenções da lei no nos termos do § 1o do art. 23, dos incisos I e IV do caput do
sentido de evitar situações extremas como a do menino art. 101 e dos incisos I a IV do caput do art. 129 desta Lei. 
Bernardo, assassinado após incontáveis ameaças e agres- § 4º Será garantida a convivência da criança e do
sões físicas por parte de seus responsáveis, seu conteúdo adolescente com a mãe ou o pai privado de liberdade,
é bastante criticado. Afinal, é claro que a lei coloca todo e por meio de visitas periódicas promovidas pelo responsável
qualquer tipo de agressão física no mesmo patamar. Con- ou, nas hipóteses de acolhimento institucional, pela entidade
siderado o teor da lei, mesmo uma palmada numa criança responsável, independentemente de autorização judicial.
é proibida. Como se depreende do artigo 19, a família natural é a
Os críticos da “Lei da Palmada” apontam que ela adota regra e a família substituta é a exceção. A criança e o ado-
uma posição extrema e impõe uma indevida intervenção lescente podem ser inseridos em programa de acolhimen-
do Estado nos ambientes familiares, retirando o poder dis- to familiar ou institucional, pelo limite temporal de 2 anos,
ciplinar garantido aos pais na educação de seus filhos. cujo caráter é o de permitir a sua retirada de uma potencial
Os defensores da “Lei da Palmada” utilizam estudos de ou efetiva situação de risco. Durante este programa, reava-
psicólogos e educadores para argumentar que não é ne- liado a cada 6 meses, se verificará se é possível a reinserção
cessário utilizar qualquer tipo de agressão física, mesmo a no ambiente da família natural (o que é preferencial) ou se
mais leve, para educar uma criança. é o caso de colocação definitiva em família substituta.

Art. 20. Os filhos, havidos ou não da relação do casa-


49 O nome da lei é uma homenagem ao menino mento, ou por adoção, terão os mesmos direitos e qua-
Bernardo Boldrini, morto em abril de 2014, aos 11 anos, lificações, proibidas quaisquer designações discrimina-
em Três Passos (RS). Os acusados são o pai e a ma- tórias relativas à filiação.
drasta do menino, com ajuda de uma amiga e do irmão O artigo 20 destaca a igualdade entre todos os filhos,
dela. Segundo as investigações, Bernardo procurou sejam eles havidos dentro ou fora do casamento, sejam
ajuda para denunciar as ameaças que sofria. eles inseridos em família natural ou substituta.

51
LEGISLAÇÃO

A disciplina sobre a perda e suspensão do poder de fa- De acordo com Santos Neto51: “[...] o exercício da auto-
mília no ECA se encontra em dois blocos, o primeiro do ar- ridade parental pela mãe era admitido apenas em caráter
tigo 21 ao 24, que aborda questões materiais, e o segundo excepcional. Ao homem era dada, em condições normais,
do artigo 155 a 163, que foca em questões procedimentais: a titularidade exclusiva do direito em pauta. Sua vontade
prevalecia e contra ela não havia remédio previsto, salvo, é
Art. 21. O poder familiar será exercido, em igualda- claro, no caso de comportamento abusivo e contrário aos
de de condições, pelo pai e pela mãe, na forma do que interesses dos menores”.
dispuser a legislação civil, assegurado a qualquer deles o O Código Civil de 2002, por seu turno, seguindo a toa-
direito de, em caso de discordância, recorrer à autoridade da da Constituição Federal de 1988, trouxe significativas
judiciária competente para a solução da divergência.  modificações ao instituto em análise, surgindo então o
chamado poder familiar, onde ambos os pais exercem de
Art. 22. Aos pais incumbe o dever de sustento, guar- forma igualitária o poder sobre os filhos menores, equili-
da e educação dos filhos menores, cabendo-lhes ainda, no brando dessa forma a relação familiar.
interesse destes, a obrigação de cumprir e fazer cumprir as O artigo 1.630 do atual Código Civil, sem definir o po-
determinações judiciais. der familiar, dispõe que “os filhos estão sujeitos ao poder
Parágrafo único. A mãe e o pai, ou os responsáveis, têm familiar enquanto menores”, ou seja, enquanto não com-
direitos iguais e deveres e responsabilidades comparti- pletarem dezoito anos ou não alcançarem a maioridade
lhados no cuidado e na educação da criança, devendo ser civil por meio de uma das formas previstas no artigo 5º,
resguardado o direito de transmissão familiar de suas cren- parágrafo único e seus incisos, do mesmo diploma legal.
ças e culturas, assegurados os direitos da criança estabele- No mesmo sentido, o citado artigo 21, ECA.
cidos nesta Lei. O poder familiar, conhecido também como autoridade
parental, é um conjunto de direitos e deveres que são atri-
Art. 23. A falta ou a carência de recursos materiais buídos aos pais para que esses administrem de forma legal
não constitui motivo suficiente para a perda ou a sus- a pessoa dos filhos e também de seus bens.
pensão do poder familiar.  “O poder familiar pode ser definido como um conjunto
§ 1º Não existindo outro motivo que por si só autorize de direitos e obrigações, quanto às pessoas e bens do filho
a decretação da medida, a criança ou o adolescente será menor não emancipado, exercido, em igualdade de condi-
mantido em sua família de origem, a qual deverá obrigato- ções, por ambos os pais, para que possam desempenhar os
riamente ser incluída em serviços e programas oficiais de encargos que a norma jurídica lhes impõe, tendo em vista o
proteção, apoio e promoção. interesse e a proteção do filho”52.
§2º A condenação criminal do pai ou da mãe não im-
plicará a destituição do poder familiar, exceto na hipótese Artigo 1.634, CC. Compete aos pais, quanto à pessoa dos
de condenação por crime doloso, sujeito à pena de reclu- filhos menores:
são, contra o próprio filho ou filha. I – dirigir-lhes a criação e educação;
II – tê-los em sua companhia e guarda;
Art. 24. A perda e a suspensão do poder familiar serão III – conceder-lhes ou negar-lhes consentimento para
decretadas judicialmente, em procedimento contraditório, casarem;
nos casos previstos na legislação civil, bem como na hipótese IV – nomear-lhes tutor por testamento ou documento
de descumprimento injustificado dos deveres e obrigações a autêntico, se o outro dos pais não lhe sobreviver, ou o sobre-
que alude o art. 22.  vivo não puder exercer o poder familiar;
V – representá-los, até aos dezesseis anos, nos atos da
O instituto do poder familiar surgiu no direito romano vida civil, e assisti-los, após essa idade, nos atos em que fo-
e era conhecido naquela época como pátrio poder, pois rem partes, suprindo-lhes o consentimento;
o pai exercia mais poderes sobre os filhos do que a mãe, VI – reclamá-los de quem ilegalmente os detenha;
o que vinha a representar um poder absoluto por parte VII – exigir que lhes prestem obediência, respeito e os
do genitor, inclusive sobre a vida e a morte dos próprios serviços próprios de sua idade e condição.
filhos50.
O Código Civil de 1916 atribuiu o poder familiar ao pai, Em relação às suas características, o poder familiar é
que era considerado o chefe da sociedade conjugal, e a indisponível e decorre da paternidade natural ou legal,
mãe possuía um papel secundário, conforme apontava o por isso, não há a possibilidade de seu titular o transfe-
artigo 380 do Código Civil de 1916 que dizia que “durante rir a terceiros por iniciativa própria, tampouco existindo a
o casamento compete o pátrio poder aos pais, exercendo viabilidade de ocorrer a sua prescrição pelo desuso. O po-
-o o marido com a colaboração da mulher [...]”. der familiar é indelegável, sendo, em regra, irrenunciável e

51 SANTOS NETO, José Antônio de Paula. Do


Pátrio Poder. São Paulo: Revista dos Tribunais, 1994.
50 AKEL, Ana Carolina Silveira. Guarda Compar- 52 DINIZ, Maria Helena. Curso de Direito Civil
tilhada: um Avanço para a Família. 2. ed. São Paulo: Brasileiro: Direito de Família. 22. ed. São Paulo: Sarai-
Atlas, 2010. va, 2007. v. 5.

52
LEGISLAÇÃO

sempre intransferível. Logo, não sendo possível ao titular Artigo 1.638, CC. Perderá por ato judicial o poder fami-
do poder familiar abrir mão de seu dever, a renúncia é in- liar o pai ou a mãe que:
viável, existindo apenas uma exceção, qual seja, a decor- I – castigar imoderadamente o filho;
rente da adoção. II – deixar o filho em abandono;
Existe apenas uma exceção, que é o caso do pedido de III – praticar atos contrários à moral e aos bons costu-
colocação do menor em família substituta, disponibilizan- mes;
do o filho para a adoção, caso em que os direitos e deveres IV – incidir, reiteradamente, nas faltas previstas no arti-
decorrentes do poder familiar serão exercidos por novos go antecedente.
titulares, ou seja, pelos pais adotivos.
A esse respeito, os direitos e deveres dos pais dispos- Nessa linha de pensamento, os artigos 24 e 22 do Es-
tos nos artigos 1.630 a 1.638 do Código Civil regulam, den- tatuto da Criança e do Adolescente, citados anteriormen-
tre os deveres e poderes decorrentes do poder familiar, te, além de preverem a suspensão e destituição do poder
também os de ordem pessoal, ou seja, o cuidado existen- familiar, trazem também os motivos que poderão acarre-
cial do menor, a educação, a correição, e os de ordem ma- tá-las.
terial, que envolvem a administração dos bens dos filhos. São bastante frequentes nos casos de pais que perdem
Os principais atributos do poder familiar são a guarda, a o poder familiar os atos de violência e espancamento; as-
criação e a educação, que se refletem nos deveres dos pais sim como o de abandono, no qual os menores, ao se ve-
para com os filhos; tanto que, não cumprindo algum des- rem abandonados, começam a relacionar-se com pessoas
ses atributos, o detentor do poder poderá sofrer sanções delinquentes e usuárias de drogas, que não vão colaborar
cíveis e até criminais. em nada com seu desenvolvimento e crescimento. Nessas
O poder familiar, conforme disposição do próprio or- situações, os detentores do poder familiar podem sofrer a
denamento civil, não é um instituto irrevogável e pode ser perda do poder familiar.
extinto, suspenso ou destituído a qualquer tempo. Basica- Cabe aqui consignar que, no caso da perda do poder
mente, as formas de extinção se aplicam quando o exercí- familiar, se no decorrer do tempo o menor não vier a ser
cio do poder familiar não é mais necessário; ao passo que adotado por outra família e as causas que levaram a perda
as regras de perda e suspensão constituem casos de priva- do poder desaparecerem, os genitores poderão requerer
ção do exercício do poder familiar pelo descumprimento judicialmente a reintegração do poder familiar, desde que
de seus deveres.
comprovem realmente que o motivo que os levou a perder
Sendo assim, o Estado poderá interferir na relação fa-
esse poder já não existe mais.
miliar, com o objetivo de resguardar os interesses do me-
Por seu turno, pelo que se verifica na análise da legis-
nor, sendo que a lei disciplina os casos em que o titular do
lação, quando o legislador estabeleceu as hipóteses de ex-
poder familiar ficará privado de exercê-lo, seja de forma
tinção do poder familiar, em regra, o fez por perceber que a
temporária ou até mesmo definitiva.
pessoa que a ele se encontrava sujeita adquiriu maturidade
o suficiente para guiar a sua vida, não havendo razão para
Artigo 1.637, CC. Se o pai, ou a mãe, abusar de sua auto-
ridade, faltando aos deveres a eles inerentes ou arruinando que permaneça tal vínculo. Há casos, entretanto, que a vio-
os bens dos filhos, cabe ao juiz, requerendo algum parente, lação aos direitos inerentes ao poder familiar tornou irre-
ou o Ministério Público, adotar a medida que lhe pareça re- versível o seu restabelecimento, como ocorre na adoção.
clamada pela segurança do menor e seus haveres, até sus-
pendendo o poder familiar, quando convenha. Artigo 1.635, CC. Extingue-se o poder familiar: I – pela
Parágrafo único. Suspende-se igualmente o exercício do morte dos pais ou do filho; II – pela emancipação, nos termos
poder familiar ao pai ou à mãe condenados por sentença ir- do artigo 5º, parágrafo único; III – pela maioridade; IV – pela
recorrível, em virtude de crime cuja pena exceda a dois anos adoção; V – por decisão judicial, na forma do artigo 1.638.
de prisão.
A extinção do poder familiar é mais complexa, pois
Nestes casos, a suspensão não será definitiva, é apenas nesta situação os pais, extintos do poder, não poderão
uma sanção imposta pelo Poder Judiciário visando preser- requerer a reintegração do poder familiar se houve inter-
var os interesses dos filhos, assim, diante da comprovação ferência deles para sua extinção. Na maioria dos casos, é
de que os problemas que levaram à suspensão desapare- possível identificar facilmente a existência da extinção do
ceram, o poder familiar poderá retornar aos genitores. poder familiar, por se tratarem de hipóteses objetivas.
Assim sendo, os pais podem ser suspensos de exercer
os direitos e deveres decorrentes do poder familiar quan- Seção II
do ficar evidenciado perante a autoridade competente o Da Família Natural
abuso. Como visto, existe, também, a probabilidade de se
decretar a suspensão do poder familiar, caso um dos pais Dos artigos 25 a 27 o ECA aborda a família natural, nos
seja condenado por crime cuja pena exceda a dois anos de seguintes termos:
prisão.
Há, ainda, as hipóteses de perda ou destituição do po- Art. 25. Entende-se por família natural a comunidade
der familiar, que é a sanção mais grave imposta aos pais formada pelos pais ou qualquer deles e seus descenden-
que faltarem com os deveres em relação aos filhos: tes.

53
LEGISLAÇÃO

Parágrafo único.  Entende-se por família extensa ou § 2o  Tratando-se de maior de 12 (doze) anos de idade,
ampliada aquela que se estende para além da unidade será necessário seu consentimento, colhido em audiência. 
pais e filhos ou da unidade do casal, formada por paren- § 3o  Na apreciação do pedido levar-se-á em conta o
tes próximos com os quais a criança ou adolescente convive grau de parentesco e a relação de afinidade ou de afeti-
e mantém vínculos de afinidade e afetividade.  vidade, a fim de evitar ou minorar as consequências decor-
A família natural é composta por pais e filhos que for- rentes da medida. 
mam vínculo entre si desde o nascimento, por questão bio- § 4o  Os grupos de irmãos serão colocados sob ado-
lógica. ção, tutela ou guarda da mesma família substituta, res-
O conceito de família pode ser visto de uma manei- salvada a comprovada existência de risco de abuso ou outra
ra mais ampla, o que se denomina família extensa ou am- situação que justifique plenamente a excepcionalidade de
pliada. Por exemplo, avós e tios que sejam muito próximos solução diversa, procurando-se, em qualquer caso, evitar o
e participem diretamente do convívio familiar, formando rompimento definitivo dos vínculos fraternais. 
para com a criança e o adolescente vínculos de afinidade
§ 5o  A colocação da criança ou adolescente em família
e afetividade.
substituta será precedida de sua preparação gradativa e
acompanhamento posterior, realizados pela equipe inter-
Art. 26. Os filhos havidos fora do casamento poderão
ser reconhecidos pelos pais, conjunta ou separadamente, profissional a serviço da Justiça da Infância e da Juventude,
no próprio termo de nascimento, por testamento, me- preferencialmente com o apoio dos técnicos responsáveis
diante escritura ou outro documento público, qualquer pela execução da política municipal de garantia do direito à
que seja a origem da filiação. convivência familiar. 
Parágrafo único. O reconhecimento pode preceder o § 6o  Em se tratando de criança ou adolescente indí-
nascimento do filho ou suceder-lhe ao falecimento, se gena ou proveniente de comunidade remanescente de qui-
deixar descendentes. lombo, é ainda obrigatório: 
Como o artigo 20 estabelece a igualdade entre os filhos I - que sejam consideradas e respeitadas sua identidade
havidos dentro ou fora do casamento, sentido em que se social e cultural, os seus costumes e tradições, bem como
compreende que tanto os filhos inseridos no matrimônio suas instituições, desde que não sejam incompatíveis com os
quanto os que não o estão fazem parte da família natural, o direitos fundamentais reconhecidos por esta Lei e pela Cons-
artigo 26 tece detalhes sobre a possibilidade de reconheci- tituição Federal; 
mento do vínculo de filiação, que pode se dar antes mesmo II - que a colocação familiar ocorra prioritariamente
do nascimento do filho ou extrapolar a sua vida, devendo no seio de sua comunidade ou junto a membros da mes-
ser feito em documento público. ma etnia; 
III - a intervenção e oitiva de representantes do órgão
Art. 27. O reconhecimento do estado de filiação é di- federal responsável pela política indigenista, no caso de
reito personalíssimo, indisponível e imprescritível, po- crianças e adolescentes indígenas, e de antropólogos, pe-
dendo ser exercitado contra os pais ou seus herdeiros, sem rante a equipe interprofissional ou multidisciplinar que irá
qualquer restrição, observado o segredo de Justiça. acompanhar o caso. 
O direito à filiação é personalíssimo, indisponível e im-
prescritível. Mesmo que um filho passe a vida toda ou boa Art. 29. Não se deferirá colocação em família substituta
parte de sua vida sem buscar o seu reconhecimento, ele a pessoa que revele, por qualquer modo, incompatibilida-
não se perde. Logo, a ação de investigação de paternidade de com a natureza da medida ou não ofereça ambiente
é imprescritível, pois também o é o vínculo que ela reco-
familiar adequado.
nhece em caso de procedência.
Art. 30. A colocação em família substituta não admitirá
Seção III
Da Família Substituta transferência da criança ou adolescente a terceiros ou
a entidades governamentais ou não-governamentais,
Subseção I sem autorização judicial.
Disposições Gerais
Art. 31. A colocação em família substituta estrangei-
Dos artigos 28 a 32 aborda-se a família substituta, nos ra constitui medida excepcional, somente admissível na
seguintes termos: modalidade de adoção.

Art. 28. A colocação em família substituta far-se-á Art. 32. Ao assumir a guarda ou a tutela, o responsável
mediante guarda, tutela ou adoção, independentemente prestará compromisso de bem e fielmente desempenhar o
da situação jurídica da criança ou adolescente, nos termos encargo, mediante termo nos autos.
desta Lei.
§ 1o  Sempre que possível, a criança ou o adolescen- Existem três formas de colocação em família substituta:
te será previamente ouvido por equipe interprofissional, guarda, tutela e adoção. Neste sentido, a criança é retirada
respeitado seu estágio de desenvolvimento e grau de com- da esfera do poder familiar de ambos os pais (o que pode
preensão sobre as implicações da medida, e terá sua opinião acontecer na tutela e na adoção), ou então permanece
devidamente considerada.  vinculado ao poder familiar de ambos genitores enquanto

54
LEGISLAÇÃO

apenas um ou um terceiro exerce a guarda (o que ocorre § 4º Poderão ser utilizados recursos federais, esta-
apenas na guarda). Trata-se de situação excepcional, eis duais, distritais e municipais para a manutenção dos ser-
que em regra a criança deve permanecer na família natural, viços de acolhimento em família acolhedora, facultando-se o
vinculada ao poder familiar atribuído a ambos os pais. repasse de recursos para a própria família acolhedora.
Neste tipo de circunstância, deve-se buscar sempre
ouvir a criança ou o adolescente. Caso já possua 12 anos Art. 35. A guarda poderá ser revogada a qualquer tem-
completos, a oitiva é obrigatória. Trata-se de respeito à au- po, mediante ato judicial fundamentado, ouvido o Ministério
tonomia da criança e do adolescente. Público.
Os irmãos devem permanecer unidos em qualquer cir-
cunstância, sendo a separação de irmãos medida excep- Na definição de Santos Neto53 a guarda trata-se de um
cional. Por exemplo, se um casal estiver disposto a adotar “direito consistente na posse de menor oponível a terceiros
4 filhos e existirem 4 irmãos, será dada prioridade a ele, e que acarreta deveres de vigilância em relação a este”.
passando na frente dos demais candidatos à adoção. No entender de Ishida54, a guarda é vinculada ao poder
familiar, “todavia, pode ocorrer a separação dos dois ins-
Subseção II
titutos, por exemplo, com a separação judicial do marido
Da Guarda
e da mulher, onde o poder familiar continua pertencendo
aos dois, no entanto um só poderá ficar com a guarda da
Art. 33. A guarda obriga a prestação de assistência
material, moral e educacional à criança ou adolescente, prole”. ou seja, tanto o pai como a mãe são detentores do
conferindo a seu detentor o direito de opor-se a terceiros, poder familiar mesmo após a separação, mas nos tipos de
inclusive aos pais.  guarda comum, apenas um terá a guarda do filho.
§ 1º A guarda destina-se a regularizar a posse de Logo, a dissolução do vínculo conjugal não exclui o
fato, podendo ser deferida, liminar ou incidentalmente, nos poder familiar, mas pode excluir a guarda de um dos pais,
procedimentos de tutela e adoção, exceto no de adoção por reservando-o apenas o direito de visitas, dependendo da
estrangeiros. modalidade de guarda adotada. Com a dissolução da união
§ 2º Excepcionalmente, deferir-se-á a guarda, fora dos conjugal, hoje se estabeleceu que a prole poderá ficar com
casos de tutela e adoção, para atender a situações pecu- o genitor que tiver melhor condições de assistir o filho.
liares ou suprir a falta eventual dos pais ou responsável, “Mesmo que a mãe seja considerada culpada pela se-
podendo ser deferido o direito de representação para a práti- paração, pode o juiz deferir-lhe a guarda dos filhos me-
ca de atos determinados. nores, se estiver comprovado que o pai, por exemplo, é
§ 3º A guarda confere à criança ou adolescente a con- alcoólatra e não tem condições de cuidar bem deles. Não
dição de dependente, para todos os fins e efeitos de direito, se indaga, portanto, quem deu causa à separação e quem
inclusive previdenciários. é o cônjuge inocente, mas qual deles revela melhores con-
§ 4o  Salvo expressa e fundamentada determinação em dições para exercer a guarda dos filhos menores, cujos
contrário, da autoridade judiciária competente, ou quando interesses foram colocados em primeiro plano. A solução
a medida for aplicada em preparação para adoção, o de- será, portanto, a mesma se ambos os pais forem culpados
ferimento da guarda de criança ou adolescente a terceiros pela separação e se a hipótese for de ruptura da vida em
não impede o exercício do direito de visitas pelos pais, comum ou de separação por motivo de doença mental. A
assim como o dever de prestar alimentos, que serão objeto regra amolda-se ao princípio do melhor interesse da crian-
de regulamentação específica, a pedido do interessado ou do ça, identificado como direito fundamental na Constituição
Ministério Público.  Federal (art. 5º, §2º), em razão da ratificação pela Conven-
ção Internacional sobre os Direitos da Criança – ONU/89”55.
Art. 34.  O poder público estimulará, por meio de as-
O juiz antes de decidir o mérito de uma ação de guar-
sistência jurídica, incentivos fiscais e subsídios, o acolhi-
da, separação ou divórcio, tem que determinar a guarda
mento, sob a forma de guarda, de criança ou adolescente
provisória do menor a um dos pais, o qual não se trata de
afastado do convívio familiar. 
§ 1o  A inclusão da criança ou adolescente em pro- um modelo de guarda, mas de definir uma situação mo-
gramas de acolhimento familiar terá preferência a seu mentânea em que a prole se encontra. Somente com o jul-
acolhimento institucional, observado, em qualquer caso, o gamento do mérito será estabelecida a guarda definitiva,
caráter temporário e excepcional da medida, nos termos que deverá adotar um dos modelos de guarda permitida
desta Lei.  no ordenamento jurídico brasileiro.
§ 2o  Na hipótese do § 1o deste artigo a pessoa ou casal
cadastrado no programa de acolhimento familiar poderá
receber a criança ou adolescente mediante guarda, obser- 53 SANTOS NETO, José Antônio de Paula. Do Pá-
vado o disposto nos arts. 28 a 33 desta Lei.  trio Poder. São Paulo: Revista dos Tribunais, 1994.
§ 3º A União apoiará a implementação de serviços 54 ISHIDA, Valter Kenji. Estatuto da Criança e do
de acolhimento em família acolhedora como política Adolescente: Doutrina e Jurisprudência. 9. ed. São Paulo:
pública, os quais deverão dispor de equipe que organize o Atlas, 2008.
acolhimento temporário de crianças e de adolescentes em 55 GONÇALVES, Carlos Roberto. Direito Civil Bra-
residências de famílias selecionadas, capacitadas e acompa- sileiro: Direito de Família. 6. ed. São Paulo: Saraiva, 2009.
nhadas que não estejam no cadastro de adoção. v. 6.

55
LEGISLAÇÃO

A guarda definitiva expressa o modelo de guarda ado- c) Guarda Alternada ou Guarda Partilhada
tado pelos pais. Porém, mesmo tratando-se de guarda de- Nesse modelo de guarda, cada um dos genitores terá
finitiva, tal modelo poderá ser alterado a qualquer tempo, a possibilidade de ter sobre sua guarda o menor ou ado-
pois o que regula o instituto da guarda é o melhor interes- lescente de forma alternada e exclusiva, ou seja, o casal
se do menor e, não sendo isso possível, a guarda é passível determinará o período em que o menor ficará com o pai
de modificação. ou com a mãe, existindo dessa forma sempre uma alter-
a) Guarda Comum ou Guarda Originária nância na guarda jurídica do menor. O período em que a
A guarda comum ou guarda originária não é uma guar- guarda ficará com o pai ou com a mãe na guarda alternada,
da judicial, existindo quando os genitores possuem vínculo poderá ser de um dia, uma semana, uma parte da semana,
matrimonial ou vivem em união estável e moram juntos um mês, um ano, ou até mais, dependendo do acordo dos
com seus filhos, situação na qual exercem plenamente e genitores, sendo que, ao término desse período, os papéis
simultaneamente todos os poderes inerentes do poder fa- se invertem. Os direitos e deveres deste modelo de guarda
miliar e, consequentemente, a guarda. Não existe a figura ficarão sempre com o cônjuge que estiver com a guarda
do guardião e nem arbitramento judicial sobre a questão. do menor, cabendo ao outro os direitos inerentes do não
Ambos pais exercem juntos a guarda em plenitude. guardião, ou seja, o de visita e o de fiscalização56.
b) Guarda Unilateral ou Guarda Única d) Guarda Dividida
Observando-se sempre o princípio do melhor interesse Na guarda dividida, são os próprios pais que contes-
do menor, a guarda excepcionalmente poderá ser atribuí- tam e procuram novos meios de garantir uma maior parti-
da de forma unilateral a uma terceira pessoa quando os cipação na vida da prole, pois neste modelo o menor vive
genitores não estiverem em condições de exercê-la, pois, em um lar fixo e determinado e recebe periodicamente a
conforme determina o artigo 1.583, §1º, do Código Civil, visita do pai ou da mãe que não tem a guarda.
“compreende-se por guarda unilateral a atribuída a um só Na guarda dividida o filho tem um lar fixo e recebe
dos genitores ou a alguém que o substitua”. Neste viés, nele a visita de ambos os genitores em tempos diferentes,
dispõe o artigo 1.584, §5º, do mesmo diploma legal: “Se o sendo que a guarda é exercida por aquele que estiver com
juiz verificar que o filho não deve permanecer sob a guarda a criança, o que é diferente da guarda unilateral, pois nela
do pai ou da mãe, deferirá a guarda à pessoa que revele a guarda é de um dos genitores e o infante vai, em dias
compatibilidade com a natureza da medida, considerados, determinados, receber a visita do outro não guardião.
de preferência, o grau de parentesco e as relações de afini- e) Guarda por Aninhamento ou Nidação
dade e afetividade”. Vale ressaltar que a guarda unilateral A guarda por aninhamento, conhecida também como
também é possível quando nenhum dos pais tem condi- guarda por nidação, ocorre quando a prole possui um lar
ções de exercê-la, por exemplo, atribuindo a guarda aos fixo e os pais se revezam, mudando-se para a casa do(s)
avós ou aos tios. filho(s) em períodos alternados de tempo, para conviver e
Usualmente, nesse contexto, podemos constatar a atender as suas necessidades. Basicamente, os pais se reve-
existência da guarda unilateral, que é atribuída somen- zam na residência do filho.
te a um dos genitores, e da guarda compartilhada, que é f) Guarda Compartilhada
atribuída a ambos, sendo que a previsão das duas moda- Considerando a evolução da família, especialmente a
lidades de guarda encontra-se no artigo 1.583, da Lei n. da mulher na sociedade, bem como o grande número de
11.698/2008, que constitui que “a guarda será unilateral ou separações ocorridas nos últimos anos, o ordenamento ju-
compartilhada”. rídico busca com o instituto da guarda compartilhada evitar
A Lei alterou a redação do artigo 1.583 e passou a es- prejuízos ainda maiores aos filhos de casal separado, que,
tabelecer nos incisos do §2º do dispositivo algumas das além de não terem o convívio diário com um dos genitores,
situações que deverão ser consideradas pelo magistrado têm que vivenciar os problemas conjugais de seus pais e
ao atribuir a guarda a um dos genitores: afeto nas relações ainda se tornarem, em muitos casos, vítimas da Síndrome
com o genitor e com o grupo familiar; saúde e segurança; da Alienação Parental.
e educação. Segundo Akel57, “a guarda compartilhada surgiu da ne-
Na guarda unilateral atribuída a apenas um dos pais, cessidade de se encontrar uma maneira que fosse capaz de
apesar de um dos genitores não ser o guardião, continuam fazer com que os pais, que não mais convivem, e seus filhos
ambos a exercerem a guarda jurídica. A diferença é que, mantivessem os vínculos afetivos latentes, mesmo após o
em virtude da guarda, o genitor guardião tem o poder de rompimento”.
decisão, enquanto o genitor não guardião tem o poder de
fiscalização, podendo contestar a decisão do genitor guar-
56 COSTA, Luiz Jorge Valente Pontes. Guarda
dião e até mesmo recorrer à justiça, caso entenda que a
decisão tomada não seja a melhor para a prole, conforme Conjunta: em busca do maior interesse do menor. Dis-
prevê o artigo 1.583, §3º, do Código Civil: “a guarda unila- ponível em: <http://jus.uol.com.br/revista/texto/13965/
teral obriga o pai ou a mãe que não a detenha a supervi- guarda-conjunta-em-busca-do-maior-interesse-do-me-
sionar os interesses dos filhos”. nor/2>. Acesso em: 16 out. 2010.
57 AKEL, Ana Carolina Silveira. Guarda Compar-
tilhada: um Avanço para a Família. 2. ed. São Paulo:
Atlas, 2010.

56
LEGISLAÇÃO

Com esse propósito, a recente Lei nº 11.698/2008 insti- Subseção IV


tuiu expressamente no ordenamento jurídico o instituto da Da Adoção
guarda compartilhada. Embora tenha sido sancionada em
13 de junho de 2008 e publicada no Diário Oficial da União A disciplina do ECA a respeito da adoção também se
em 16 de junho do mesmo ano, por força da vacatio legis divide em dois blocos, um voltado a aspectos materiais,
instituída no artigo 2º, a lei somente entrou em vigor no do artigo 39 ao 52-D, e outro voltado a aspectos procedi-
país 60 (sessenta) dias após a sua publicação, ou seja, em mentais, notadamente no que se refere à habilitação para a
16 de agosto de 2008 (vide anexo). adoção, do artigo 197-A a 197-D.
A nova lei trás em seu bojo o conceito de guarda com-
partilhada nos seguintes termos: “compreende-se por [...] Art. 39. A adoção de criança e de adolescente reger-se-á
guarda compartilhada a responsabilização conjunta e o segundo o disposto nesta Lei.
exercício de direitos e deveres do pai e da mãe que não § 1o  A adoção é medida excepcional e irrevogável, à
vivam sob o mesmo teto, concernentes ao poder familiar qual se deve recorrer apenas quando esgotados os recursos
dos filhos comuns”. de manutenção da criança ou adolescente na família natural
Assim, de acordo com o novo diploma legal, pode- ou extensa, na forma do parágrafo único do art. 25 desta Lei. 
se verificar que na guarda compartilhada os pais terão os § 2o  É vedada a adoção por procuração. 
mesmos direitos e deveres com relação ao filho, ou seja, as
tarefas serão divididas de forma igualitária, não sobrecar- Art. 40. O adotando deve contar com, no máximo, de-
regando somente um dos genitores. zoito anos à data do pedido, salvo se já estiver sob a guar-
da ou tutela dos adotantes.
Subseção III
Da Tutela Art. 41. A adoção atribui a condição de filho ao adotado,
com os mesmos direitos e deveres, inclusive sucessórios,
Art. 36.  A tutela será deferida, nos termos da lei civil, a desligando-o de qualquer vínculo com pais e parentes, salvo
pessoa de até 18 (dezoito) anos incompletos.  os impedimentos matrimoniais.
Parágrafo único. O deferimento da tutela pressupõe a § 1º Se um dos cônjuges ou concubinos adota o filho do
prévia decretação da perda ou suspensão do poder fa- outro, mantêm-se os vínculos de filiação entre o adotado e o
miliar e implica necessariamente o dever de guarda. cônjuge ou concubino do adotante e os respectivos parentes.
§ 2º É recíproco o direito sucessório entre o adotado,
Art. 37.  O tutor nomeado por testamento ou qual- seus descendentes, o adotante, seus ascendentes, descen-
quer documento autêntico, conforme previsto no parágra- dentes e colaterais até o 4º grau, observada a ordem de
fo único do art. 1.729 da Lei no 10.406, de 10 de janeiro de vocação hereditária.
2002 - Código Civil, deverá, no prazo de 30 (trinta) dias
após a abertura da sucessão, ingressar com pedido des- Art. 42.  Podem adotar os maiores de 18 (dezoito)
tinado ao controle judicial do ato, observando o procedi- anos, independentemente do estado civil. 
mento previsto nos arts. 165 a 170 desta Lei.  § 1º Não podem adotar os ascendentes e os irmãos do
Parágrafo único.  Na apreciação do pedido, serão ob- adotando.
servados os requisitos previstos nos arts. 28 e 29 desta Lei, § 2o  Para adoção conjunta, é indispensável que os ado-
somente sendo deferida a tutela à pessoa indicada na dispo- tantes sejam casados civilmente ou mantenham união es-
sição de última vontade, se restar comprovado que a medida tável, comprovada a estabilidade da família. 
é vantajosa ao tutelando e que não existe outra pessoa em § 3º O adotante há de ser, pelo menos, dezesseis anos
melhores condições de assumi-la.  mais velho do que o adotando.
§ 4o  Os divorciados, os judicialmente separados e os
Art. 38. Aplica-se à destituição da tutela o disposto no ex-companheiros podem adotar conjuntamente, contanto
art. 24. que acordem sobre a guarda e o regime de visitas e desde
que o estágio de convivência tenha sido iniciado na cons-
A tutela é forma de colocação de criança e adolescente tância do período de convivência e que seja comprovada a
em família substituta. Pressupõe, ao contrário da guarda, existência de vínculos de afinidade e afetividade com aquele
a prévia destituição ou suspensão do poder familiar dos não detentor da guarda, que justifiquem a excepcionalidade
pais (família natural). Visa essencialmente a suprir carência da concessão.  
de representação legal, assumindo o tutor tal munus na § 5o  Nos casos do § 4o deste artigo, desde que de-
ausência dos genitores. Na hipótese de os pais serem fale- monstrado efetivo benefício ao adotando, será assegurada
cidos, tiverem sido destituídos ou suspensos do poder fa- a guarda compartilhada, conforme previsto no art. 1.584 da
miliar, ou houverem aderido expressamente ao pedido de Lei no10.406, de 10 de janeiro de 2002 - Código Civil. 
colocação em família substituta, este poderá ser formulado § 6o  A adoção poderá ser deferida ao adotante que,
diretamente em cartório, em petição assinada pelos pró- após inequívoca manifestação de vontade, vier a falecer no
prios requerentes, dispensada a assistência por advogado curso do procedimento, antes de prolatada a sentença.
(art. 166, ECA). Em outras circunstâncias, deve passar pelo
crivo do Judiciário.

57
LEGISLAÇÃO

Art. 43. A adoção será deferida quando apresentar reais § 8o  O processo relativo à adoção assim como outros a
vantagens para o adotando e fundar-se em motivos legíti- ele relacionados serão mantidos em arquivo, admitindo-se
mos. seu armazenamento em microfilme ou por outros meios,
garantida a sua conservação para consulta a qualquer tem-
Art. 44. Enquanto não der conta de sua administração po. 
e saldar o seu alcance, não pode o tutor ou o curador ado- § 9º Terão prioridade de tramitação os processos de
tar o pupilo ou o curatelado. adoção em que o adotando for criança ou adolescente
com deficiência ou com doença crônica.
Art. 45. A adoção depende do consentimento dos pais
ou do representante legal do adotando. Art. 48.  O adotado tem direito de conhecer sua origem
§ 1º O consentimento será dispensado em relação à biológica, bem como de obter acesso irrestrito ao processo
criança ou adolescente cujos pais sejam desconhecidos ou no qual a medida foi aplicada e seus eventuais incidentes,
tenham sido destituídos do poder familiar. após completar 18 (dezoito) anos. 
§ 2º Em se tratando de adotando maior de doze anos Parágrafo único.  O acesso ao processo de adoção pode-
de idade, será também necessário o seu consentimento. rá ser também deferido ao adotado menor de 18 (dezoito)
anos, a seu pedido, assegurada orientação e assistência jurí-
Art. 46. A adoção será precedida de estágio de convi- dica e psicológica. 
vência com a criança ou adolescente, pelo prazo que a auto-
ridade judiciária fixar, observadas as peculiaridades do caso. Art. 49. A morte dos adotantes não restabelece o poder
§ 1o  O estágio de convivência poderá ser dispensado familiar dos pais naturais. 
se o adotando já estiver sob a tutela ou guarda legal do
adotante durante tempo suficiente para que seja possível Art. 50. A autoridade judiciária manterá, em cada co-
avaliar a conveniência da constituição do vínculo.  marca ou foro regional, um registro de crianças e ado-
§ 2o  A simples guarda de fato não autoriza, por si só, a lescentes em condições de serem adotados e outro de
dispensa da realização do estágio de convivência.   pessoas interessadas na adoção.  
§ 3o  Em caso de adoção por pessoa ou casal residente § 1º O deferimento da inscrição dar-se-á após prévia
ou domiciliado fora do País, o estágio de convivência, cum- consulta aos órgãos técnicos do juizado, ouvido o Minis-
prido no território nacional, será de, no mínimo, 30 (trinta) tério Público.
dias.  § 2º Não será deferida a inscrição se o interessado não
§ 4o  O estágio de convivência será acompanhado pela satisfazer os requisitos legais, ou verificada qualquer das
equipe interprofissional a serviço da Justiça da Infância e hipóteses previstas no art. 29.
da Juventude, preferencialmente com apoio dos técnicos res- § 3o  A inscrição de postulantes à adoção será prece-
ponsáveis pela execução da política de garantia do direito à dida de um período de preparação psicossocial e jurídica,
convivência familiar, que apresentarão relatório minucioso orientado pela equipe técnica da Justiça da Infância e da
acerca da conveniência do deferimento da medida.  Juventude, preferencialmente com apoio dos técnicos res-
ponsáveis pela execução da política municipal de garantia
Art. 47. O vínculo da adoção constitui-se por sentença do direito à convivência familiar. 
judicial, que será inscrita no registro civil mediante manda- § 4o  Sempre que possível e recomendável, a prepa-
do do qual não se fornecerá certidão. ração referida no § 3o deste artigo incluirá o contato com
§ 1º A inscrição consignará o nome dos adotantes crianças e adolescentes em acolhimento familiar ou insti-
como pais, bem como o nome de seus ascendentes. tucional em condições de serem adotados, a ser realizado
§ 2º O mandado judicial, que será arquivado, cancelará sob a orientação, supervisão e avaliação da equipe técnica
o registro original do adotado. da Justiça da Infância e da Juventude, com apoio dos téc-
§ 3o  A pedido do adotante, o novo registro poderá ser nicos responsáveis pelo programa de acolhimento e pela
lavrado no Cartório do Registro Civil do Município de sua execução da política municipal de garantia do direito à
residência. convivência familiar. 
§ 4o  Nenhuma observação sobre a origem do ato po- § 5o  Serão criados e implementados cadastros esta-
derá constar nas certidões do registro.   duais e nacional de crianças e adolescentes em condições
§ 5o  A sentença conferirá ao adotado o nome do ado- de serem adotados e de pessoas ou casais habilitados à
tante e, a pedido de qualquer deles, poderá determinar a adoção. 
modificação do prenome.   § 6o  Haverá cadastros distintos para pessoas ou casais
§ 6o  Caso a modificação de prenome seja requerida residentes fora do País, que somente serão consultados na
pelo adotante, é obrigatória a oitiva do adotando, observa- inexistência de postulantes nacionais habilitados nos ca-
do o disposto nos §§ 1o e 2o do art. 28 desta Lei.   dastros mencionados no § 5o deste artigo. 
§ 7o  A adoção produz seus efeitos a partir do trânsito § 7o  As autoridades estaduais e federais em matéria de
em julgado da sentença constitutiva, exceto na hipótese adoção terão acesso integral aos cadastros, incumbindo-
prevista no § 6o do art. 42 desta Lei, caso em que terá força lhes a troca de informações e a cooperação mútua, para
retroativa à data do óbito.  melhoria do sistema. 

58
LEGISLAÇÃO

§ 8o  A autoridade judiciária providenciará, no prazo III - que, em se tratando de adoção de adolescente, este
de 48 (quarenta e oito) horas, a inscrição das crianças e foi consultado, por meios adequados ao seu estágio de de-
adolescentes em condições de serem adotados que não senvolvimento, e que se encontra preparado para a medida,
tiveram colocação familiar na comarca de origem, e das mediante parecer elaborado por equipe interprofissional,
pessoas ou casais que tiveram deferida sua habilitação à observado o disposto nos §§ 1o e 2o do art. 28 desta Lei. 
adoção nos cadastros estadual e nacional referidos no § § 2o  Os brasileiros residentes no exterior terão prefe-
5o deste artigo, sob pena de responsabilidade.  rência aos estrangeiros, nos casos de adoção internacional
§ 9o  Compete à Autoridade Central Estadual zelar pela de criança ou adolescente brasileiro. 
manutenção e correta alimentação dos cadastros, com § 3o  A adoção internacional pressupõe a intervenção
posterior comunicação à Autoridade Central Federal Bra- das Autoridades Centrais Estaduais e Federal em matéria
sileira.  de adoção internacional. 
§ 10.  A adoção internacional somente será deferida se,
após consulta ao cadastro de pessoas ou casais habilitados Art. 52.  A adoção internacional observará o procedi-
à adoção, mantido pela Justiça da Infância e da Juventude mento previsto nos arts. 165 a 170 desta Lei, com as seguin-
na comarca, bem como aos cadastros estadual e nacional tes adaptações: 
referidos no § 5o deste artigo, não for encontrado interes- I - a pessoa ou casal estrangeiro, interessado em adotar
sado com residência permanente no Brasil.  criança ou adolescente brasileiro, deverá formular pedido
§ 11.  Enquanto não localizada pessoa ou casal interes- de habilitação à adoção perante a Autoridade Central em
sado em sua adoção, a criança ou o adolescente, sempre matéria de adoção internacional no país de acolhida, assim
que possível e recomendável, será colocado sob guarda de entendido aquele onde está situada sua residência habitual; 
família cadastrada em programa de acolhimento familiar.  II - se a Autoridade Central do país de acolhida consi-
§ 12.  A alimentação do cadastro e a convocação cri- derar que os solicitantes estão habilitados e aptos para ado-
teriosa dos postulantes à adoção serão fiscalizadas pelo tar, emitirá um relatório que contenha informações sobre a
identidade, a capacidade jurídica e adequação dos solicitan-
Ministério Público. 
tes para adotar, sua situação pessoal, familiar e médica, seu
§ 13.  Somente poderá ser deferida adoção em favor
meio social, os motivos que os animam e sua aptidão para
de candidato domiciliado no Brasil não cadastrado previa-
assumir uma adoção internacional; 
mente nos termos desta Lei quando: 
III - a Autoridade Central do país de acolhida enviará o
I - se tratar de pedido de adoção unilateral; 
relatório à Autoridade Central Estadual, com cópia para a
II - for formulada por parente com o qual a criança ou
Autoridade Central Federal Brasileira; 
adolescente mantenha vínculos de afinidade e afetividade; 
IV - o relatório será instruído com toda a documenta-
III - oriundo o pedido de quem detém a tutela ou guarda ção necessária, incluindo estudo psicossocial elaborado por
legal de criança maior de 3 (três) anos ou adolescente, desde equipe interprofissional habilitada e cópia autenticada da
que o lapso de tempo de convivência comprove a fixação legislação pertinente, acompanhada da respectiva prova de
de laços de afinidade e afetividade, e não seja constatada a vigência; 
ocorrência de má-fé ou qualquer das situações previstas nos V - os documentos em língua estrangeira serão devida-
arts. 237 ou 238 desta Lei.  mente autenticados pela autoridade consular, observados os
§ 14.  Nas hipóteses previstas no § 13 deste artigo, o tratados e convenções internacionais, e acompanhados da
candidato deverá comprovar, no curso do procedimento, respectiva tradução, por tradutor público juramentado; 
que preenche os requisitos necessários à adoção, confor- VI - a Autoridade Central Estadual poderá fazer exigên-
me previsto nesta Lei.  cias e solicitar complementação sobre o estudo psicossocial
do postulante estrangeiro à adoção, já realizado no país de
Art. 51.  Considera-se adoção internacional aquela na acolhida; 
qual a pessoa ou casal postulante é residente ou domiciliado VII - verificada, após estudo realizado pela Autoridade
fora do Brasil, conforme previsto no Artigo 2 da Convenção Central Estadual, a compatibilidade da legislação estrangei-
de Haia, de 29 de maio de 1993, Relativa à Proteção das ra com a nacional, além do preenchimento por parte dos
Crianças e à Cooperação em Matéria de Adoção Internacio- postulantes à medida dos requisitos objetivos e subjetivos
nal, aprovada pelo Decreto Legislativo no 1, de 14 de janeiro necessários ao seu deferimento, tanto à luz do que dispõe
de 1999, e promulgada pelo Decreto no 3.087, de 21 de esta Lei como da legislação do país de acolhida, será expe-
junho de 1999.  dido laudo de habilitação à adoção internacional, que terá
§ 1o  A adoção internacional de criança ou adolescen- validade por, no máximo, 1 (um) ano; 
te brasileiro ou domiciliado no Brasil somente terá lugar VIII - de posse do laudo de habilitação, o interessado
quando restar comprovado:  será autorizado a formalizar pedido de adoção perante o
I - que a colocação em família substituta é a solução Juízo da Infância e da Juventude do local em que se encontra
adequada ao caso concreto;  a criança ou adolescente, conforme indicação efetuada pela
II - que foram esgotadas todas as possibilidades de colo- Autoridade Central Estadual. 
cação da criança ou adolescente em família substituta bra- § 1o  Se a legislação do país de acolhida assim o auto-
sileira, após consulta aos cadastros mencionados no art. 50 rizar, admite-se que os pedidos de habilitação à adoção
desta Lei;  internacional sejam intermediados por organismos creden-
ciados. 

59
LEGISLAÇÃO

§ 2o  Incumbe à Autoridade Central Federal Brasileira § 7o  A renovação do credenciamento poderá ser con-
o credenciamento de organismos nacionais e estrangeiros cedida mediante requerimento protocolado na Autoridade
encarregados de intermediar pedidos de habilitação à ado- Central Federal Brasileira nos 60 (sessenta) dias anteriores
ção internacional, com posterior comunicação às Autorida- ao término do respectivo prazo de validade. 
des Centrais Estaduais e publicação nos órgãos oficiais de § 8o  Antes de transitada em julgado a decisão que con-
imprensa e em sítio próprio da internet.  cedeu a adoção internacional, não será permitida a
§ 3o  Somente será admissível o credenciamento de or- § 9o  Transitada em julgado a decisão, a autoridade ju-
ganismos que:  diciária determinará a expedição de alvará com autoriza-
I - sejam oriundos de países que ratificaram a Conven- ção de viagem, bem como para obtenção de passaporte,
ção de Haia e estejam devidamente credenciados pela Auto- constando, obrigatoriamente, as características da criança
ridade Central do país onde estiverem sediados e no país de ou adolescente adotado, como idade, cor, sexo, eventuais
acolhida do adotando para atuar em adoção internacional sinais ou traços peculiares, assim como foto recente e a
no Brasil;  aposição da impressão digital do seu polegar direito, ins-
II - satisfizerem as condições de integridade moral, com- truindo o documento com cópia autenticada da decisão e
petência profissional, experiência e responsabilidade exigi- certidão de trânsito em julgado. 
das pelos países respectivos e pela Autoridade Central Fede- § 10.  A Autoridade Central Federal Brasileira poderá, a
ral Brasileira;  qualquer momento, solicitar informações sobre a situação
III - forem qualificados por seus padrões éticos e sua for- das crianças e adolescentes adotados. 
mação e experiência para atuar na área de adoção interna- § 11.  A cobrança de valores por parte dos organismos
cional;  credenciados, que sejam considerados abusivos pela Auto-
IV - cumprirem os requisitos exigidos pelo ordenamento ridade Central Federal Brasileira e que não estejam devida-
jurídico brasileiro e pelas normas estabelecidas pela Autori- mente comprovados, é causa de seu descredenciamento. 
dade Central Federal Brasileira.  § 12.  Uma mesma pessoa ou seu cônjuge não podem
§ 4o  Os organismos credenciados deverão ainda:  ser representados por mais de uma entidade credenciada
I - perseguir unicamente fins não lucrativos, nas condi- para atuar na cooperação em adoção internacional. 
ções e dentro dos limites fixados pelas autoridades compe- § 13.  A habilitação de postulante estrangeiro ou domi-
tentes do país onde estiverem sediados, do país de acolhida ciliado fora do Brasil terá validade máxima de 1 (um) ano,
e pela Autoridade Central Federal Brasileira;  podendo ser renovada. 
II - ser dirigidos e administrados por pessoas qualifica- § 14.  É vedado o contato direto de representantes de
das e de reconhecida idoneidade moral, com comprovada organismos de adoção, nacionais ou estrangeiros, com di-
formação ou experiência para atuar na área de adoção in- rigentes de programas de acolhimento institucional ou fa-
ternacional, cadastradas pelo Departamento de Polícia Fe- miliar, assim como com crianças e adolescentes em condi-
deral e aprovadas pela Autoridade Central Federal Brasileira, ções de serem adotados, sem a devida autorização judicial. 
mediante publicação de portaria do órgão federal compe- § 15.  A Autoridade Central Federal Brasileira poderá
tente;  limitar ou suspender a concessão de novos credenciamen-
III - estar submetidos à supervisão das autoridades com- tos sempre que julgar necessário, mediante ato administra-
petentes do país onde estiverem sediados e no país de aco- tivo fundamentado. 
lhida, inclusive quanto à sua composição, funcionamento e
situação financeira;  Art. 52-A.  É vedado, sob pena de responsabilidade e
IV - apresentar à Autoridade Central Federal Brasilei- descredenciamento, o repasse de recursos provenientes de
ra, a cada ano, relatório geral das atividades desenvolvidas, organismos estrangeiros encarregados de intermediar pedi-
bem como relatório de acompanhamento das adoções inter- dos de adoção internacional a organismos nacionais ou a
nacionais efetuadas no período, cuja cópia será encaminha- pessoas físicas. 
da ao Departamento de Polícia Federal;  Parágrafo único.  Eventuais repasses somente poderão
V - enviar relatório pós-adotivo semestral para a Autori- ser efetuados via Fundo dos Direitos da Criança e do Adoles-
dade Central Estadual, com cópia para a Autoridade Central cente e estarão sujeitos às deliberações do respectivo Conse-
Federal Brasileira, pelo período mínimo de 2 (dois) anos. O lho de Direitos da Criança e do Adolescente. 
envio do relatório será mantido até a juntada de cópia au-
tenticada do registro civil, estabelecendo a cidadania do país Art. 52-B.  A adoção por brasileiro residente no exterior
de acolhida para o adotado;  em país ratificante da Convenção de Haia, cujo processo de
VI - tomar as medidas necessárias para garantir que os adoção tenha sido processado em conformidade com a le-
adotantes encaminhem à Autoridade Central Federal Brasi- gislação vigente no país de residência e atendido o disposto
leira cópia da certidão de registro de nascimento estrangeira na Alínea “c” do Artigo 17 da referida Convenção, será auto-
e do certificado de nacionalidade tão logo lhes sejam con- maticamente recepcionada com o reingresso no Brasil. 
cedidos.  § 1o  Caso não tenha sido atendido o disposto na Alínea
§ 5o  A não apresentação dos relatórios referidos no § “c” do Artigo 17 da Convenção de Haia, deverá a sentença
4  deste artigo pelo organismo credenciado poderá acarre-
o ser homologada pelo Superior Tribunal de Justiça.  
tar a suspensão de seu credenciamento.  § 2o  O pretendente brasileiro residente no exterior em
§ 6o  O credenciamento de organismo nacional ou es- país não ratificante da Convenção de Haia, uma vez rein-
trangeiro encarregado de intermediar pedidos de adoção gressado no Brasil, deverá requerer a homologação da sen-
internacional terá validade de 2 (dois) anos.  tença estrangeira pelo Superior Tribunal de Justiça. 

60
LEGISLAÇÃO

Art. 52-C.  Nas adoções internacionais, quando o Brasil jam no caso de casais juntamente com os dados familiares,
for o país de acolhida, a decisão da autoridade competente dados que vão completar não só a ficha do casal ou pessoa
do país de origem da criança ou do adolescente será conhe- que deseja adotar, mais da família onde o menor vai residir
cida pela Autoridade Central Estadual que tiver processado e ter sua formação os documentos pessoais como cópias
o pedido de habilitação dos pais adotivos, que comunicará de certidão de nascimento (quando solteiros), de certidão
o fato à Autoridade Central Federal e determinará as provi- de casamento (quando casados), cópias de declaração de
dências necessárias à expedição do Certificado de Naturali- período de união estável, Cópias de Certidão de identidade
zação Provisório.   (RG), Comprovante da renda da pessoa ou casal, compro-
§ 1o  A Autoridade Central Estadual, ouvido o Ministério vante que demostre que as condições vão suprir a neces-
Público, somente deixará de reconhecer os efeitos daquela sidade de se incluir mais um membro naquele lar, compro-
decisão se restar demonstrado que a adoção é manifesta- vante de moradia em que prove a pessoa ter sua residência
mente contrária à ordem pública ou não atende ao interes- que acolherá o novo integrante.
se superior da criança ou do adolescente.  O interessado em adotar deve juntar atestados de saú-
§ 2o  Na hipótese de não reconhecimento da adoção, de física onde a pessoa vai demonstrar ser capaz de cuidar
prevista no § 1o deste artigo, o Ministério Público deverá e garantir uma boa vida para o adotando e atestado de
imediatamente requerer o que for de direito para resguar- saúde mental, comprovando que a pessoa é capaz legal-
dar os interesses da criança ou do adolescente, comunican- mente.
do-se as providências à Autoridade Central Estadual, que Nos aspectos judiciais inerentes ao caso devem ser
fará a comunicação à Autoridade Central Federal Brasileira juntados Certidão de antecedentes criminais, que demos-
e à Autoridade Central do país de origem.  tra conduta legal do indivíduo perante a sociedade, Cer-
tidão negativa de distribuição civil, além da manifestação
Art. 52-D.  Nas adoções internacionais, quando o Brasil do M.P (Ministério Público) apresentando quesitos a serem
for o país de acolhida e a adoção não tenha sido deferida no respondidos pela equipe interdisciplinar, designação de
país de origem porque a sua legislação a delega ao país de audiência para oitiva de testemunhas e requerentes, soli-
acolhida, ou, ainda, na hipótese de, mesmo com decisão, a citar juntada de documentos complementares que sejam
criança ou o adolescente ser oriundo de país que não tenha necessários. Deve ser obrigatório o estudo psicossocial
aderido à Convenção referida, o processo de adoção seguirá onde se testa a capacidade dos indivíduos para se torna-
rem pai ou mãe, o de incluir mais um membro junto aos
as regras da adoção nacional. 
filhos já existente. Os programas de capacitação também
devem ser aderido a este sistema, levando esclarecimentos
A adoção no Código Civil de 1916 era tratada em seu
aos pretendentes a adoção além, de manter contato com a
capítulo V. A adoção seguia um critério de ter um filho
criança em regime de acolhimento familiar.
para que a família tivesse sucessão e para configurar a fa-
Sendo os adotantes inscritos são chamados por ordem
mília da época que só tinha uma configuração costumeira
cronológica de acordo com a habilitação, e que só poderá
se houvesse pai, mãe e filhos. A idade de 30 (trinta) anos
ser dispensada se for pelo melhor interesse do adotando.
para que as pessoas pudessem não se arrepender do feito. Os seguintes princípios e diretrizes devem guiar a de-
Quando se falava do casamento criava-se o critério de que cisão pela adoção:
tenha se passado 05 (cinco) anos de matrimônio para que - Condição da criança e do adolescente como ser de
o casal possa adotar, pois o ideal era que os filhos fossem direitos: As crianças são detentoras de direitos previstos na
consanguíneos. A adoção poderia se dissolver por vontade lei 8.069/90 e na Constituição Federal de 1988;
das partes. Caso houvessem filhos naturais reconhecidos - Proteção integral e prioritária: Toda e qualquer norma
ou legitimados os filhos adotivos não participavam da su- contida dentro da lei 8.069/90 deve ser voltada a proteção
cessão além de não se desfazer o vínculo com os parentes integral dos interesses do menor;
naturais, somente no que dizia respeito ao pátrio poder. - Responsabilidade primária e solidaria do poder pú-
Houve um grande salto no que concerne a adoção que blico: tanto nos casos ressalvados pelo E.C.A (Estatuto da
foi a letra trazida pela Constituição Federal de 1988 em seu Criança e do Adolescente) quanto nos casos que a CF/88
artigo 227 e com o ECA, que versa sobre todas as garantias (Constituição Federal de 1988) e demais objetos legais es-
constitucionais e no que tange a adoção vem dos artigos pecíficos, é, dever dos três poderes atuarem no que for ne-
39 ao 52-D falando sobre a regularização da adoção e traz cessário para a proteção do menor;
como princípio basilar o melhor interesse da criança, além - Interesse superior da criança e do adolescente: me-
de ter como fundamento o afeto de pai para filho sem que diante a necessidade da intervenção estatal é necessário
haja qualquer diferenciação para com os filhos biológicos. que se dê privilégio ao interesse do que for melhor para a
Ainda no que concerne a adoção, foi criada para dar criança e/ou adolescente. O primeiro interesse a ser obser-
efetividade ao ECA a Lei nº 12.010 de 03 de agosto de 2009, vado é o que trouxer melhor benefício a estes;
que tem o intuito de alterar o direito à convivência familiar - Privacidade: A promoção dos direitos deve ser sem-
mostrando com clareza como deve ocorrer a adoção. pre feita respeitando a privação e o direito da imagem, a
Para ser candidato a adoção deve ter cumprido uma sua vida privada e a intimidade;
série de requisitos para se tornar hábil, estes requisitos são - Intervenção precoce: Ao primeiro sinal de perigo e
elencados na lei. O primeiro deles é a qualificação comple- risco ao menor, o Estado deve intervir para reverter a situa-
ta: da pessoa que deseja adotar ou das pessoas que dese- ção presente;

61
LEGISLAÇÃO

- Intervenção mínima; IV - direito de organização e participação em entida-


- Proporcionalidade e atualidade: Ser a atitude tomada des estudantis;
de acordo com a proporção de perigo existente no mo- V - acesso à escola pública e gratuita próxima de sua
mento, evitando e revertendo o risco de morte; residência.
- Responsabilidade parental: a intervenção deve ser a Parágrafo único. É direito dos pais ou responsáveis
princípio para que os pais tomem responsabilidade sobre ter ciência do processo pedagógico, bem como participar da
seus filhos; definição das propostas educacionais.
- Prevalência da família: Primeiro se dará a preferên-
cia a família natural para que a criança continue com seus Art. 54. É dever do Estado assegurar à criança e ao ado-
pais depois a preferência vai ser da família contínua (família lescente:
natural a primeira e extensa a segunda), em último caso a I - ensino fundamental, obrigatório e gratuito, inclu-
criança/adolescente vai ser direcionada a família substituta. sive para os que a ele não tiveram acesso na idade própria;
- Obrigatoriedade da informação: respeitando o es- II - progressiva extensão da obrigatoriedade e gra-
tágio de desenvolvimento e compreensão da criança ou tuidade ao ensino médio;
adolescente, além de seus pais e responsáveis devem ser III - atendimento educacional especializado aos por-
informados o motivo em que se dá a intervenção e como tadores de deficiência, preferencialmente na rede regular
esta se processou; de ensino;
- Oitiva obrigatória e participação: a criança ou adoles- IV - atendimento em creche e pré-escola às crianças de
cente separados ou na companhia de seus pais, responsá- zero a cinco anos de idade;
veis ou pessoas por ela indicados; bem como seus pais e V - acesso aos níveis mais elevados do ensino, da
responsáveis, tem o direito de serem ouvidas e sua opinião pesquisa e da criação artística, segundo a capacidade de
deve ser considerada pelas autoridades do judiciário. cada um;
- Afastamento da criança e do adolescente do conví- VI - oferta de ensino noturno regular, adequado às
vio familiar: É um processo contencioso que importara aos condições do adolescente trabalhador;
pais o direito do contraditório e da ampla defesa, este pro- VII - atendimento no ensino fundamental, através de
programas suplementares de material didático-escolar,
cedimento é de competência do judiciário (procedimento
transporte, alimentação e assistência à saúde.
judicial) artigo 28 §§ 1° e 2° do E.C.A;
§ 1º O acesso ao ensino obrigatório e gratuito é di-
- Acolhimento familiar: medida de proteção criada para
reito público subjetivo.
o amparo da criança até que seja resolvida a situação;
§ 2º O não oferecimento do ensino obrigatório pelo
- Guia de acolhimento: será elaborado pelo poder Ju-
poder público ou sua oferta irregular importa responsabi-
diciário um guia para encaminhar à criança a entidade de
lidade da autoridade competente.
acolhimento (guia deve conter a causa da retirada e per- § 3º Compete ao poder público recensear os educan-
manência do menor fora da sua família natural além de dos no ensino fundamental, fazer-lhes a chamada e zelar,
todos os dados) junto aos pais ou responsável, pela frequência à escola.
- Plano individual de atendimento: cada criança vai ter
um plano a ser desenvolvido visando sua adaptação a nova Art. 55. Os pais ou responsável têm a obrigação de ma-
família; tricular seus filhos ou pupilos na rede regular de ensino.
- Destituição do poder familiar: Promovida pelo M.P
(Ministério público) sendo para isto necessário prévio aviso Art. 56. Os dirigentes de estabelecimentos de ensino
a entidade de acolhimento familiar, ou, responsáveis pela fundamental comunicarão ao Conselho Tutelar os casos de:
execução da política municipal de garantia do direito a I - maus-tratos envolvendo seus alunos;
convivência familiar. II - reiteração de faltas injustificadas e de evasão es-
- Cadastro de crianças a adolescentes à regime institu- colar, esgotados os recursos escolares;
cional e familiar: é um cadastro para crianças e adolescen- III - elevados níveis de repetência.
tes que necessitem de acolhimento familiar ou institucional
devido a algum registro de maus tratos. Art. 57. O poder público estimulará pesquisas, experiên-
cias e novas propostas relativas a calendário, seriação,
Capítulo IV currículo, metodologia, didática e avaliação, com vistas
Do Direito à Educação, à Cultura, ao Esporte e ao à inserção de crianças e adolescentes excluídos do ensino
Lazer fundamental obrigatório.

Art. 53. A criança e o adolescente têm direito à educa- Art. 58. No processo educacional respeitar-se-ão os va-
ção, visando ao pleno desenvolvimento de sua pessoa, pre- lores culturais, artísticos e históricos próprios do contexto
paro para o exercício da cidadania e qualificação para o tra- social da criança e do adolescente, garantindo-se a estes a
balho, assegurando-se-lhes: liberdade da criação e o acesso às fontes de cultura.
I - igualdade de condições para o acesso e perma-
nência na escola; Art. 59. Os municípios, com apoio dos estados e da
II - direito de ser respeitado por seus educadores; União, estimularão e facilitarão a destinação de recursos
III - direito de contestar critérios avaliativos, podendo e espaços para programações culturais, esportivas e de
recorrer às instâncias escolares superiores; lazer voltadas para a infância e a juventude.

62
LEGISLAÇÃO

Capítulo V Uma vez que o adolescente está autorizado a trabalhar,


Do Direito à Profissionalização e à Proteção no mesmo que na condição de menor aprendiz, possui direitos
Trabalho trabalhistas e previdenciários.

Art. 60. É proibido qualquer trabalho a menores de Art. 66. Ao adolescente portador de deficiência é asse-
quatorze anos de idade, salvo na condição de aprendiz.  gurado trabalho protegido.
Preconiza o artigo 7º, XXXIII, CF a “proibição de tra- O adolescente que possui deficiência não pode ser ex-
balho noturno, perigoso ou insalubre a menores de posto a uma situação de risco em decorrência da atividade
dezoito e de qualquer trabalho a menores de dezesseis laboral.
anos, salvo na condição de aprendiz, a partir de quatorze
anos”. Art. 67. Ao adolescente empregado, aprendiz, em regi-
Portanto, em decorrência da própria norma constitu- me familiar de trabalho, aluno de escola técnica, assistido em
cional, nenhuma criança ou adolescente pode trabalhar an- entidade governamental ou não-governamental, é vedado
tes dos 14 anos de idade. Evidentemente que há algumas trabalho:
exceções a esta regra, devidamente fiscalizadas pelo Con- I - noturno, realizado entre as vinte e duas horas de um
selho Tutelar, como é o caso dos artistas mirins. dia e as cinco horas do dia seguinte;
Entre 14 anos e 16 anos de idade somente será possí- II - perigoso, insalubre ou penoso;
vel o trabalho na condição de menor aprendiz, cuja natu- III - realizado em locais prejudiciais à sua formação e ao
reza é de ensino técnico-profissional, viabilizando a futura seu desenvolvimento físico, psíquico, moral e social;
inserção do adolescente no mercado de trabalho. IV - realizado em horários e locais que não permitam a
A partir dos 16 anos, o menor pode trabalhar, mas não frequência à escola.
no período noturno ou em condições de periculosidade e O menor aprendiz está proibido de trabalhar no perío-
insalubridade. do noturno, em trabalho que o coloque exposto a periculo-
sidade (ex.: em andaimes, em áreas com risco de incêndio
ou choques), insalubridade (ex.: em freezers de frigoríficos,
Art. 61. A proteção ao trabalho dos adolescentes é re-
expostos a radiação) ou penosidade (ex.: excesso de força
gulada por legislação especial, sem prejuízo do disposto
física exigida).
nesta Lei.
Art. 68. O programa social que tenha por base o trabalho
Art. 62. Considera-se aprendizagem a formação téc-
educativo, sob responsabilidade de entidade governamental
nico-profissional ministrada segundo as diretrizes e bases
ou não-governamental sem fins lucrativos, deverá assegurar
da legislação de educação em vigor. ao adolescente que dele participe condições de capacitação
para o exercício de atividade regular remunerada.
Art. 63. A formação técnico-profissional obedecerá aos § 1º Entende-se por trabalho educativo a atividade la-
seguintes princípios: boral em que as exigências pedagógicas relativas ao desen-
I - garantia de acesso e frequência obrigatória ao en- volvimento pessoal e social do educando prevalecem sobre o
sino regular; aspecto produtivo.
II - atividade compatível com o desenvolvimento do § 2º A remuneração que o adolescente recebe pelo tra-
adolescente; balho efetuado ou a participação na venda dos produtos de
III - horário especial para o exercício das atividades. seu trabalho não desfigura o caráter educativo.
Aquele que trabalha na condição de menor aprendiz Os programas sociais voltados à capacitação dos ado-
é obrigado a frequentar a escola, devendo ser facilitadas lescentes devem sempre ter por objetivo educá-lo para que
as condições para que o faça, notadamente pelo estabe- ele adquira condições de inserir-se no mercado de trabalho.
lecimento de horário especial de trabalho. Além disso, a Deve ser ensinado, logo, dele não se deve cobrar tanta pro-
atividade laboral deve ser compatível com as atividades de dutividade, mas sim deve ser avaliado pelo seu aprendizado.
ensino, até mesmo por se tratar de ensino técnico-profis- O fato do trabalho ser remunerado não desvirtua este pro-
sionalizante. pósito.
Ex.: um jovem pode trabalhar no período matutino, fre-
quentar o SENAI na parte da tarde e ir ao colégio no ensino Art. 69. O adolescente tem direito à profissionalização
médio noturno. e à proteção no trabalho, observados os seguintes aspectos,
entre outros:
Art. 64. Ao adolescente até quatorze anos de idade é I - respeito à condição peculiar de pessoa em desenvol-
assegurada bolsa de aprendizagem. vimento;
Toda criança e adolescente que necessitar receberá fo- II - capacitação profissional adequada ao mercado de
mento para que não se desvincule das atividades de ensi- trabalho.
no. Trata-se de incentivo àquele que sem auxílio acabaria Com efeito, profissionalização e proteção no trabalho
entrando em situação irregular e trabalhando. são direitos fundamentais garantidos ao adolescente, exi-
gindo-se neste campo que sua condição peculiar inerente
Art. 65. Ao adolescente aprendiz, maior de quatorze ao processo de aprendizado seja respeitada e que o traba-
anos, são assegurados os direitos trabalhistas e previden- lho sirva para permitir a sua inserção no mercado de tra-
ciários. balho.

63
LEGISLAÇÃO

Título III Parágrafo único.  São igualmente responsáveis pela co-


Da Prevenção municação de que trata este artigo, as pessoas encarrega-
das, por razão de cargo, função, ofício, ministério, profissão
Capítulo I ou ocupação, do cuidado, assistência ou guarda de crianças
Disposições Gerais e adolescentes, punível, na forma deste Estatuto, o injustifi-
cado retardamento ou omissão, culposos ou dolosos. 
Art. 70. É dever de todos prevenir a ocorrência de amea-
ça ou violação dos direitos da criança e do adolescente. Art. 71. A criança e o adolescente têm direito a informa-
ção, cultura, lazer, esportes, diversões, espetáculos e produ-
Art. 70-A. A União, os Estados, o Distrito Federal e os tos e serviços que respeitem sua condição peculiar de pessoa
Municípios deverão atuar de forma articulada na elabora- em desenvolvimento.
ção de políticas públicas e na execução de ações destinadas
a coibir o uso de castigo físico ou de tratamento cruel ou Art. 72. As obrigações previstas nesta Lei não excluem
degradante e difundir formas não violentas de educação de da prevenção especial outras decorrentes dos princípios por
crianças e de adolescentes, tendo como principais ações: ela adotados.
I - a promoção de campanhas educativas permanentes
para a divulgação do direito da criança e do adolescente de Art. 73. A inobservância das normas de prevenção im-
serem educados e cuidados sem o uso de castigo físico ou portará em responsabilidade da pessoa física ou jurídica, nos
de tratamento cruel ou degradante e dos instrumentos de termos desta Lei.
proteção aos direitos humanos;
II - a integração com os órgãos do Poder Judiciário, do Capítulo II
Ministério Público e da Defensoria Pública, com o Conselho Da Prevenção Especial
Tutelar, com os Conselhos de Direitos da Criança e do Ado-
lescente e com as entidades não governamentais que atuam Seção I
na promoção, proteção e defesa dos direitos da criança e do Da informação, Cultura, Lazer, Esportes, Diversões
adolescente; e Espetáculos
III - a formação continuada e a capacitação dos profis-
sionais de saúde, educação e assistência social e dos demais Art. 74. O poder público, através do órgão competente,
agentes que atuam na promoção, proteção e defesa dos di- regulará as diversões e espetáculos públicos, informando so-
reitos da criança e do adolescente para o desenvolvimento bre a natureza deles, as faixas etárias a que não se recomen-
das competências necessárias à prevenção, à identificação dem, locais e horários em que sua apresentação se mostre
de evidências, ao diagnóstico e ao enfrentamento de todas inadequada.
as formas de violência contra a criança e o adolescente; Parágrafo único. Os responsáveis pelas diversões e es-
IV - o apoio e o incentivo às práticas de resolução pací- petáculos públicos deverão afixar, em lugar visível e de fácil
fica de conflitos que envolvam violência contra a criança e acesso, à entrada do local de exibição, informação destacada
o adolescente; sobre a natureza do espetáculo e a faixa etária especificada
V - a inclusão, nas políticas públicas, de ações que visem no certificado de classificação.
a garantir os direitos da criança e do adolescente, desde a
atenção pré-natal, e de atividades junto aos pais e respon- Art. 75. Toda criança ou adolescente terá acesso às di-
sáveis com o objetivo de promover a informação, a reflexão, versões e espetáculos públicos classificados como adequados
o debate e a orientação sobre alternativas ao uso de casti- à sua faixa etária.
go físico ou de tratamento cruel ou degradante no processo Parágrafo único. As crianças menores de dez anos so-
educativo; mente poderão ingressar e permanecer nos locais de apre-
VI - a promoção de espaços intersetoriais locais para a sentação ou exibição quando acompanhadas dos pais ou
articulação de ações e a elaboração de planos de atuação responsável.
conjunta focados nas famílias em situação de violência, com
participação de profissionais de saúde, de assistência social Art. 76. As emissoras de rádio e televisão somente exibi-
e de educação e de órgãos de promoção, proteção e defesa rão, no horário recomendado para o público infanto juvenil,
dos direitos da criança e do adolescente. programas com finalidades educativas, artísticas, culturais e
Parágrafo único.   As famílias com crianças e adoles- informativas.
centes com deficiência terão prioridade de atendimento nas Parágrafo único. Nenhum espetáculo será apresentado
ações e políticas públicas de prevenção e proteção. ou anunciado sem aviso de sua classificação, antes de sua
transmissão, apresentação ou exibição.
Art. 70-B.  As entidades, públicas e privadas, que atuem
nas áreas a que se refere o art. 71, dentre outras, devem Art. 77. Os proprietários, diretores, gerentes e funcioná-
contar, em seus quadros, com pessoas capacitadas a reco- rios de empresas que explorem a venda ou aluguel de fitas
nhecer e comunicar ao Conselho Tutelar suspeitas ou casos de programação em vídeo cuidarão para que não haja ven-
de maus-tratos praticados contra crianças e adolescentes. da ou locação em desacordo com a classificação atribuída
pelo órgão competente.

64
LEGISLAÇÃO

Parágrafo único. As fitas a que alude este artigo deverão § 2º A autoridade judiciária poderá, a pedido dos pais
exibir, no invólucro, informação sobre a natureza da obra e a ou responsável, conceder autorização válida por dois anos.
faixa etária a que se destinam.
Art. 84. Quando se tratar de viagem ao exterior, a auto-
Art. 78. As revistas e publicações contendo material im- rização é dispensável, se a criança ou adolescente:
próprio ou inadequado a crianças e adolescentes deverão ser I - estiver acompanhado de ambos os pais ou respon-
comercializadas em embalagem lacrada, com a advertência sável;
de seu conteúdo. II - viajar na companhia de um dos pais, autorizado ex-
Parágrafo único. As editoras cuidarão para que as capas pressamente pelo outro através de documento com firma
que contenham mensagens pornográficas ou obscenas se- reconhecida.
jam protegidas com embalagem opaca.
Art. 85. Sem prévia e expressa autorização judicial, ne-
Art. 79. As revistas e publicações destinadas ao público nhuma criança ou adolescente nascido em território nacio-
infanto-juvenil não poderão conter ilustrações, fotografias, nal poderá sair do País em companhia de estrangeiro resi-
legendas, crônicas ou anúncios de bebidas alcoólicas, taba- dente ou domiciliado no exterior.
co, armas e munições, e deverão respeitar os valores éticos e
sociais da pessoa e da família. Parte Especial
Art. 80. Os responsáveis por estabelecimentos que ex- Título I
plorem comercialmente bilhar, sinuca ou congênere ou por Da Política de Atendimento
casas de jogos, assim entendidas as que realizem apostas,
ainda que eventualmente, cuidarão para que não seja per- Capítulo I
mitida a entrada e a permanência de crianças e adolescentes Disposições Gerais
no local, afixando aviso para orientação do público.
Art. 86. A política de atendimento dos direitos da criança
Seção II
e do adolescente far-se-á através de um conjunto articulado
Dos Produtos e Serviços
de ações governamentais e não-governamentais, da União,
dos estados, do Distrito Federal e dos municípios.
Art. 81. É proibida a venda à criança ou ao adolescente
de:
Art. 87. São linhas de ação da política de atendimento:
I - armas, munições e explosivos;
I - políticas sociais básicas;
II - bebidas alcoólicas;
II - serviços, programas, projetos e benefícios de assis-
III - produtos cujos componentes possam causar depen-
dência física ou psíquica ainda que por utilização indevida; tência social de garantia de proteção social e de prevenção
IV - fogos de estampido e de artifício, exceto aqueles e redução de violações de direitos, seus agravamentos ou
que pelo seu reduzido potencial sejam incapazes de provocar reincidências;
qualquer dano físico em caso de utilização indevida; III - serviços especiais de prevenção e atendimento mé-
V - revistas e publicações a que alude o art. 78; dico e psicossocial às vítimas de negligência, maus-tratos,
VI - bilhetes lotéricos e equivalentes. exploração, abuso, crueldade e opressão;
IV - serviço de identificação e localização de pais, res-
Art. 82. É proibida a hospedagem de criança ou ado- ponsável, crianças e adolescentes desaparecidos;
lescente em hotel, motel, pensão ou estabelecimento con- V - proteção jurídico-social por entidades de defesa dos
gênere, salvo se autorizado ou acompanhado pelos pais ou direitos da criança e do adolescente.
responsável. VI - políticas e programas destinados a prevenir ou
abreviar o período de afastamento do convívio familiar e a
Seção III garantir o efetivo exercício do direito à convivência familiar
Da Autorização para Viajar de crianças e adolescentes;
VII - campanhas de estímulo ao acolhimento sob forma
Art. 83. Nenhuma criança poderá viajar para fora da co- de guarda de crianças e adolescentes afastados do convívio
marca onde reside, desacompanhada dos pais ou responsá- familiar e à adoção, especificamente inter-racial, de crianças
vel, sem expressa autorização judicial. maiores ou de adolescentes, com necessidades específicas de
§ 1º A autorização não será exigida quando: saúde ou com deficiências e de grupos de irmãos.
a) tratar-se de comarca contígua à da residência da
criança, se na mesma unidade da Federação, ou incluída na Art. 88. São diretrizes da política de atendimento:
mesma região metropolitana; I - municipalização do atendimento;
b) a criança estiver acompanhada: II - criação de conselhos municipais, estaduais e nacional
1) de ascendente ou colateral maior, até o terceiro grau, dos direitos da criança e do adolescente, órgãos deliberativos
comprovado documentalmente o parentesco; e controladores das ações em todos os níveis, assegurada
2) de pessoa maior, expressamente autorizada pelo pai, a participação popular paritária por meio de organizações
mãe ou responsável. representativas, segundo leis federal, estaduais e municipais;

65
LEGISLAÇÃO

III - criação e manutenção de programas específicos, ob- neste artigo, no Conselho Municipal dos Direitos da Crian-
servada a descentralização político-administrativa; ça e do Adolescente, o qual manterá registro das inscrições
IV - manutenção de fundos nacional, estaduais e muni- e de suas alterações, do que fará comunicação ao Conselho
cipais vinculados aos respectivos conselhos dos direitos da Tutelar e à autoridade judiciária.
criança e do adolescente; § 2o  Os recursos destinados à implementação e ma-
V - integração operacional de órgãos do Judiciário, Mi- nutenção dos programas relacionados neste artigo serão
nistério Público, Defensoria, Segurança Pública e Assistência previstos nas dotações orçamentárias dos órgãos públicos
Social, preferencialmente em um mesmo local, para efeito encarregados das áreas de Educação, Saúde e Assistência
de agilização do atendimento inicial a adolescente a quem Social, dentre outros, observando-se o princípio da priori-
se atribua autoria de ato infracional; dade absoluta à criança e ao adolescente preconizado pelo
VI - integração operacional de órgãos do Judiciário, Mi- caput do art. 227 da Constituição Federal e pelo caput e
nistério Público, Defensoria, Conselho Tutelar e encarrega- parágrafo único do art. 4o desta Lei.
dos da execução das políticas sociais básicas e de assistência § 3o  Os programas em execução serão reavaliados pelo
social, para efeito de agilização do atendimento de crianças Conselho Municipal dos Direitos da Criança e do Adoles-
e de adolescentes inseridos em programas de acolhimento cente, no máximo, a cada 2 (dois) anos, constituindo-se
familiar ou institucional, com vista na sua rápida reintegra- critérios para renovação da autorização de funcionamento:
ção à família de origem ou, se tal solução se mostrar com- I - o efetivo respeito às regras e princípios desta Lei, bem
provadamente inviável, sua colocação em família substituta, como às resoluções relativas à modalidade de atendimento
em quaisquer das modalidades previstas no art. 28 desta Lei; prestado expedidas pelos Conselhos de Direitos da Criança e
VII - mobilização da opinião pública para a indispensá- do Adolescente, em todos os níveis; 
vel participação dos diversos segmentos da sociedade; II - a qualidade e eficiência do trabalho desenvolvido,
VIII - especialização e formação continuada dos profis- atestadas pelo Conselho Tutelar, pelo Ministério Público e
sionais que trabalham nas diferentes áreas da atenção à pri- pela Justiça da Infância e da Juventude; 
meira infância, incluindo os conhecimentos sobre direitos da III - em se tratando de programas de acolhimento insti-
criança e sobre desenvolvimento infantil; tucional ou familiar, serão considerados os índices de sucesso
IX - formação profissional com abrangência dos diversos na reintegração familiar ou de adaptação à família substitu-
direitos da criança e do adolescente que favoreça a interse- ta, conforme o caso.
torialidade no atendimento da criança e do adolescente e
seu desenvolvimento integral; Art. 91. As entidades não-governamentais somente po-
X - realização e divulgação de pesquisas sobre desenvol- derão funcionar depois de registradas no Conselho Munici-
vimento infantil e sobre prevenção da violência. pal dos Direitos da Criança e do Adolescente, o qual comuni-
cará o registro ao Conselho Tutelar e à autoridade judiciária
Art. 89. A função de membro do conselho nacional e dos da respectiva localidade.
conselhos estaduais e municipais dos direitos da criança e § 1o  Será negado o registro à entidade que: 
do adolescente é considerada de interesse público relevante a) não ofereça instalações físicas em condições adequa-
e não será remunerada. das de habitabilidade, higiene, salubridade e segurança;
b) não apresente plano de trabalho compatível com os
Capítulo II princípios desta Lei;
Das Entidades de Atendimento c) esteja irregularmente constituída;
d) tenha em seus quadros pessoas inidôneas.
Seção I e) não se adequar ou deixar de cumprir as resoluções e
Disposições Gerais deliberações relativas à modalidade de atendimento pres-
tado expedidas pelos Conselhos de Direitos da Criança e do
Art. 90. As entidades de atendimento são responsáveis Adolescente, em todos os níveis.
pela manutenção das próprias unidades, assim como pelo § 2o  O registro terá validade máxima de 4 (quatro) anos,
planejamento e execução de programas de proteção e so- cabendo ao Conselho Municipal dos Direitos da Criança e
cioeducativos destinados a crianças e adolescentes, em re- do Adolescente, periodicamente, reavaliar o cabimento de
gime de:  sua renovação, observado o disposto no § 1o deste artigo.
I - orientação e apoio sociofamiliar;
II - apoio socioeducativo em meio aberto; Art. 92.  As entidades que desenvolvam programas de
III - colocação familiar; acolhimento familiar ou institucional deverão adotar os se-
IV - acolhimento institucional; guintes princípios:
V - prestação de serviços à comunidade; I - preservação dos vínculos familiares e promoção da
VI - liberdade assistida; reintegração familiar;
VII - semiliberdade; e II - integração em família substituta, quando esgotados
VIII - internação. os recursos de manutenção na família natural ou extensa;
§ 1o  As entidades governamentais e não governamen- III - atendimento personalizado e em pequenos grupos;
tais deverão proceder à inscrição de seus programas, es- IV - desenvolvimento de atividades em regime de coe-
pecificando os regimes de atendimento, na forma definida ducação;

66
LEGISLAÇÃO

V - não desmembramento de grupos de irmãos; Art. 94. As entidades que desenvolvem programas de in-
VI - evitar, sempre que possível, a transferência para ou- ternação têm as seguintes obrigações, entre outras:
tras entidades de crianças e adolescentes abrigados; I - observar os direitos e garantias de que são titulares
VII - participação na vida da comunidade local; os adolescentes;
VIII - preparação gradativa para o desligamento; II - não restringir nenhum direito que não tenha sido
IX - participação de pessoas da comunidade no processo objeto de restrição na decisão de internação;
educativo. III - oferecer atendimento personalizado, em pequenas
§ 1o  O dirigente de entidade que desenvolve progra- unidades e grupos reduzidos;
ma de acolhimento institucional é equiparado ao guardião, IV - preservar a identidade e oferecer ambiente de res-
para todos os efeitos de direito.  peito e dignidade ao adolescente;
§ 2o  Os dirigentes de entidades que desenvolvem pro- V - diligenciar no sentido do restabelecimento e da pre-
gramas de acolhimento familiar ou institucional remeterão servação dos vínculos familiares;
à autoridade judiciária, no máximo a cada 6 (seis) meses, VI - comunicar à autoridade judiciária, periodicamente,
relatório circunstanciado acerca da situação de cada crian- os casos em que se mostre inviável ou impossível o reata-
ça ou adolescente acolhido e sua família, para fins da rea- mento dos vínculos familiares;
valiação prevista no § 1o do art. 19 desta Lei.  VII - oferecer instalações físicas em condições adequa-
§ 3o  Os entes federados, por intermédio dos Poderes das de habitabilidade, higiene, salubridade e segurança e os
Executivo e Judiciário, promoverão conjuntamente a per- objetos necessários à higiene pessoal;
manente qualificação dos profissionais que atuam direta VIII - oferecer vestuário e alimentação suficientes e ade-
ou indiretamente em programas de acolhimento institucio- quados à faixa etária dos adolescentes atendidos;
nal e destinados à colocação familiar de crianças e adoles- IX - oferecer cuidados médicos, psicológicos, odontológi-
centes, incluindo membros do Poder Judiciário, Ministério cos e farmacêuticos;
Público e Conselho Tutelar.  X - propiciar escolarização e profissionalização;
§ 4o  Salvo determinação em contrário da autorida- XI - propiciar atividades culturais, esportivas e de lazer;
de judiciária competente, as entidades que desenvolvem XII - propiciar assistência religiosa àqueles que deseja-
programas de acolhimento familiar ou institucional, se ne- rem, de acordo com suas crenças;
cessário com o auxílio do Conselho Tutelar e dos órgãos XIII - proceder a estudo social e pessoal de cada caso;
de assistência social, estimularão o contato da criança ou
XIV - reavaliar periodicamente cada caso, com intervalo
adolescente com seus pais e parentes, em cumprimento ao
máximo de seis meses, dando ciência dos resultados à auto-
disposto nos incisos I e VIII do caput deste artigo. 
ridade competente;
§ 5o  As entidades que desenvolvem programas de aco-
XV - informar, periodicamente, o adolescente internado
lhimento familiar ou institucional somente poderão rece-
sobre sua situação processual;
ber recursos públicos se comprovado o atendimento dos
XVI - comunicar às autoridades competentes todos os
princípios, exigências e finalidades desta Lei. 
casos de adolescentes portadores de moléstias infectocon-
§ 6o  O descumprimento das disposições desta Lei
pelo dirigente de entidade que desenvolva programas de tagiosas;
acolhimento familiar ou institucional é causa de sua desti- XVII - fornecer comprovante de depósito dos pertences
tuição, sem prejuízo da apuração de sua responsabilidade dos adolescentes;
administrativa, civil e criminal.  XVIII - manter programas destinados ao apoio e acom-
§ 7o  Quando se tratar de criança de 0 (zero) a 3 (três) panhamento de egressos;
anos em acolhimento institucional, dar-se-á especial aten- XIX - providenciar os documentos necessários ao exercí-
ção à atuação de educadores de referência estáveis e quali- cio da cidadania àqueles que não os tiverem;
tativamente significativos, às rotinas específicas e ao atendi- XX - manter arquivo de anotações onde constem data e
mento das necessidades básicas, incluindo as de afeto como circunstâncias do atendimento, nome do adolescente, seus
prioritárias.  pais ou responsável, parentes, endereços, sexo, idade, acom-
panhamento da sua formação, relação de seus pertences e
Art. 93.  As entidades que mantenham programa de demais dados que possibilitem sua identificação e a indivi-
acolhimento institucional poderão, em caráter excepcional dualização do atendimento.
e de urgência, acolher crianças e adolescentes sem prévia § 1o  Aplicam-se, no que couber, as obrigações cons-
determinação da autoridade competente, fazendo comuni- tantes deste artigo às entidades que mantêm programas
cação do fato em até 24 (vinte e quatro) horas ao Juiz da de acolhimento institucional e familiar. 
Infância e da Juventude, sob pena de responsabilidade.  § 2º No cumprimento das obrigações a que alude este
Parágrafo único.  Recebida a comunicação, a autoridade artigo as entidades utilizarão preferencialmente os recur-
judiciária, ouvido o Ministério Público e se necessário com o sos da comunidade.
apoio do Conselho Tutelar local, tomará as medidas neces-
sárias para promover a imediata reintegração familiar da Art. 94-A.  As entidades, públicas ou privadas, que abri-
criança ou do adolescente ou, se por qualquer razão não for guem ou recepcionem crianças e adolescentes, ainda que em
isso possível ou recomendável, para seu encaminhamento a caráter temporário, devem ter, em seus quadros, profissio-
programa de acolhimento familiar, institucional ou a família nais capacitados a reconhecer e reportar ao Conselho Tute-
substituta, observado o disposto no § 2o do art. 101 desta Lei.  lar suspeitas ou ocorrências de maus-tratos.

67
LEGISLAÇÃO

Seção II Art. 100. Na aplicação das medidas levar-se-ão em con-


Da Fiscalização das Entidades ta as necessidades pedagógicas, preferindo-se aquelas que
visem ao fortalecimento dos vínculos familiares e comunitá-
Art. 95. As entidades governamentais e não-governa- rios.
mentais referidas no art. 90 serão fiscalizadas pelo Judiciá- Parágrafo único.  São também princípios que regem a
rio, pelo Ministério Público e pelos Conselhos Tutelares. aplicação das medidas: 
I - condição da criança e do adolescente como sujeitos de
Art. 96. Os planos de aplicação e as prestações de contas direitos: crianças e adolescentes são os titulares dos direitos
serão apresentados ao estado ou ao município, conforme a previstos nesta e em outras Leis, bem como na Constituição
origem das dotações orçamentárias. Federal;  
II - proteção integral e prioritária: a interpretação e apli-
Art. 97. São medidas aplicáveis às entidades de atendi- cação de toda e qualquer norma contida nesta Lei deve ser
mento que descumprirem obrigação constante do art. 94, voltada à proteção integral e prioritária dos direitos de que
crianças e adolescentes são titulares;  
sem prejuízo da responsabilidade civil e criminal de seus di-
III - responsabilidade primária e solidária do poder públi-
rigentes ou prepostos:
co: a plena efetivação dos direitos assegurados a crianças e a
I - às entidades governamentais:
adolescentes por esta Lei e pela Constituição Federal, salvo nos
a) advertência; casos por esta expressamente ressalvados, é de responsabilida-
b) afastamento provisório de seus dirigentes; de primária e solidária das 3 (três) esferas de governo, sem pre-
c) afastamento definitivo de seus dirigentes; juízo da municipalização do atendimento e da possibilidade
d) fechamento de unidade ou interdição de programa. da execução de programas por entidades não governamentais;  
II - às entidades não-governamentais: IV - interesse superior da criança e do adolescente: a inter-
a) advertência; venção deve atender prioritariamente aos interesses e direitos
b) suspensão total ou parcial do repasse de verbas pú- da criança e do adolescente, sem prejuízo da consideração que
blicas; for devida a outros interesses legítimos no âmbito da plurali-
c) interdição de unidades ou suspensão de programa; dade dos interesses presentes no caso concreto;  
d) cassação do registro. V - privacidade: a promoção dos direitos e proteção da
§ 1o  Em caso de reiteradas infrações cometidas por criança e do adolescente deve ser efetuada no respeito pela
entidades de atendimento, que coloquem em risco os di- intimidade, direito à imagem e reserva da sua vida privada; 
reitos assegurados nesta Lei, deverá ser o fato comunicado VI - intervenção precoce: a intervenção das autoridades
ao Ministério Público ou representado perante autoridade competentes deve ser efetuada logo que a situação de perigo
judiciária competente para as providências cabíveis, inclu- seja conhecida;  
sive suspensão das atividades ou dissolução da entidade.  VII - intervenção mínima: a intervenção deve ser exercida
§ 2o  As pessoas jurídicas de direito público e as organi- exclusivamente pelas autoridades e instituições cuja ação seja
zações não governamentais responderão pelos danos que indispensável à efetiva promoção dos direitos e à proteção da
seus agentes causarem às crianças e aos adolescentes, ca- criança e do adolescente; 
racterizado o descumprimento dos princípios norteadores VIII - proporcionalidade e atualidade: a intervenção deve
das atividades de proteção específica.  ser a necessária e adequada à situação de perigo em que a
criança ou o adolescente se encontram no momento em que a
Título II decisão é tomada; 
Das Medidas de Proteção IX - responsabilidade parental: a intervenção deve ser efe-
tuada de modo que os pais assumam os seus deveres para com
a criança e o adolescente; 
Capítulo I
X - prevalência da família: na promoção de direitos e na
Disposições Gerais
proteção da criança e do adolescente deve ser dada preva-
lência às medidas que os mantenham ou reintegrem na sua
Art. 98. As medidas de proteção à criança e ao adoles- família natural ou extensa ou, se isto não for possível, que pro-
cente são aplicáveis sempre que os direitos reconhecidos movam a sua integração em família substituta; 
nesta Lei forem ameaçados ou violados: XI - obrigatoriedade da informação: a criança e o ado-
I - por ação ou omissão da sociedade ou do Estado; lescente, respeitado seu estágio de desenvolvimento e capa-
II - por falta, omissão ou abuso dos pais ou respon- cidade de compreensão, seus pais ou responsável devem ser
sável; informados dos seus direitos, dos motivos que determinaram a
III - em razão de sua conduta. intervenção e da forma como esta se processa; 
XII - oitiva obrigatória e participação: a criança e o adoles-
Capítulo II cente, em separado ou na companhia dos pais, de responsável
Das Medidas Específicas de Proteção ou de pessoa por si indicada, bem como os seus pais ou res-
ponsável, têm direito a ser ouvidos e a participar nos atos e na
Art. 99. As medidas previstas neste Capítulo poderão definição da medida de promoção dos direitos e de proteção,
ser aplicadas isolada ou cumulativamente, bem como sendo sua opinião devidamente considerada pela autorida-
substituídas a qualquer tempo. de judiciária competente, observado o disposto nos §§ 1o e
2o do art. 28 desta Lei. 

68
LEGISLAÇÃO

Art. 101. Verificada qualquer das hipóteses previstas no § 6o  Constarão do plano individual, dentre outros: 
art. 98, a autoridade competente poderá determinar, dentre I - os resultados da avaliação interdisciplinar; 
outras, as seguintes medidas: II - os compromissos assumidos pelos pais ou respon-
I - encaminhamento aos pais ou responsável, me- sável; e 
diante termo de responsabilidade; III - a previsão das atividades a serem desenvolvidas com
II - orientação, apoio e acompanhamento temporá- a criança ou com o adolescente acolhido e seus pais ou res-
rios; ponsável, com vista na reintegração familiar ou, caso seja
III - matrícula e frequência obrigatórias em estabele- esta vedada por expressa e fundamentada determinação ju-
cimento oficial de ensino fundamental; dicial, as providências a serem tomadas para sua colocação
IV - inclusão em serviços e programas oficiais ou co- em família substituta, sob direta supervisão da autoridade
munitários de proteção, apoio e promoção da família, da judiciária. 
criança e do adolescente; § 7o  O acolhimento familiar ou institucional ocorrerá
V - requisição de tratamento médico, psicológico ou no local mais próximo à residência dos pais ou do respon-
psiquiátrico, em regime hospitalar ou ambulatorial; sável e, como parte do processo de reintegração familiar,
VI - inclusão em programa oficial ou comunitário sempre que identificada a necessidade, a família de origem
de auxílio, orientação e tratamento a alcoólatras e toxi- será incluída em programas oficiais de orientação, de apoio
cômanos; e de promoção social, sendo facilitado e estimulado o con-
VII - acolhimento institucional;   tato com a criança ou com o adolescente acolhido. 
VIII - inclusão em programa de acolhimento familiar;  § 8o  Verificada a possibilidade de reintegração familiar,
IX - colocação em família substituta.  o responsável pelo programa de acolhimento familiar ou
§ 1o O acolhimento institucional e o acolhimento fa- institucional fará imediata comunicação à autoridade judi-
miliar são medidas provisórias e excepcionais, utilizáveis ciária, que dará vista ao Ministério Público, pelo prazo de 5
como forma de transição para reintegração familiar ou, não (cinco) dias, decidindo em igual prazo.
sendo esta possível, para colocação em família substituta, § 9o  Em sendo constatada a impossibilidade de rein-
não implicando privação de liberdade.  tegração da criança ou do adolescente à família de ori-
§ 2o  Sem prejuízo da tomada de medidas emergenciais gem, após seu encaminhamento a programas oficiais ou
para proteção de vítimas de violência ou abuso sexual e comunitários de orientação, apoio e promoção social, será
das providências a que alude o art. 130 desta Lei, o afasta-
enviado relatório fundamentado ao Ministério Público, no
mento da criança ou adolescente do convívio familiar é de
qual conste a descrição pormenorizada das providências
competência exclusiva da autoridade judiciária e importará
tomadas e a expressa recomendação, subscrita pelos técni-
na deflagração, a pedido do Ministério Público ou de quem
cos da entidade ou responsáveis pela execução da política
tenha legítimo interesse, de procedimento judicial conten-
municipal de garantia do direito à convivência familiar, para
cioso, no qual se garanta aos pais ou ao responsável legal o
a destituição do poder familiar, ou destituição de tutela ou
exercício do contraditório e da ampla defesa.
guarda. 
§ 3o  Crianças e adolescentes somente poderão ser
encaminhados às instituições que executam programas § 10.  Recebido o relatório, o Ministério Público terá
de acolhimento institucional, governamentais ou não, por o prazo de 30 (trinta) dias para o ingresso com a ação de
meio de uma Guia de Acolhimento, expedida pela autori- destituição do poder familiar, salvo se entender necessária
dade judiciária, na qual obrigatoriamente constará, dentre a realização de estudos complementares ou outras provi-
outros:  dências que entender indispensáveis ao ajuizamento da
I - sua identificação e a qualificação completa de seus demanda. 
pais ou de seu responsável, se conhecidos;  § 11.  A autoridade judiciária manterá, em cada comar-
II - o endereço de residência dos pais ou do responsável, ca ou foro regional, um cadastro contendo informações
com pontos de referência;  atualizadas sobre as crianças e adolescentes em regime de
III - os nomes de parentes ou de terceiros interessados acolhimento familiar e institucional sob sua responsabili-
em tê-los sob sua guarda;  dade, com informações pormenorizadas sobre a situação
IV - os motivos da retirada ou da não reintegração ao jurídica de cada um, bem como as providências tomadas
convívio familiar.  para sua reintegração familiar ou colocação em família
§ 4o  Imediatamente após o acolhimento da criança ou substituta, em qualquer das modalidades previstas no art.
do adolescente, a entidade responsável pelo programa de 28 desta Lei. 
acolhimento institucional ou familiar elaborará um plano § 12.  Terão acesso ao cadastro o Ministério Público, o
individual de atendimento, visando à reintegração familiar, Conselho Tutelar, o órgão gestor da Assistência Social e os
ressalvada a existência de ordem escrita e fundamentada Conselhos Municipais dos Direitos da Criança e do Adoles-
em contrário de autoridade judiciária competente, caso em cente e da Assistência Social, aos quais incumbe deliberar
que também deverá contemplar sua colocação em família sobre a implementação de políticas públicas que permitam
substituta, observadas as regras e princípios desta Lei.  reduzir o número de crianças e adolescentes afastados do
§ 5o  O plano individual será elaborado sob a respon- convívio familiar e abreviar o período de permanência em
sabilidade da equipe técnica do respectivo programa de programa de acolhimento.
atendimento e levará em consideração a opinião da criança
ou do adolescente e a oitiva dos pais ou do responsável. 

69
LEGISLAÇÃO

Art. 102. As medidas de proteção de que trata este Capí- - colocação em família substituta (é utilizada somente
tulo serão acompanhadas da regularização do registro ci- em situações muito graves).
vil.  O Juiz pode aplicar essas medidas isolada ou cumulati-
§ 1º Verificada a inexistência de registro anterior, o as- vamente. Pode, também, substituir uma medida pela outra
sento de nascimento da criança ou adolescente será feito a qualquer tempo (art. 99 do ECA). Antes de aplicar qual-
à vista dos elementos disponíveis, mediante requisição da quer uma dessas medidas, o Juiz deverá ouvir os pais ou
autoridade judiciária. responsáveis, realizar estudo social do caso e ouvir o MP.
§ 2º Os registros e certidões necessários à regulariza- Essa oitiva do MP é obrigatória, sob pena de nulidade (art.
ção de que trata este artigo são isentos de multas, custas e 204 do ECA). Esse rol do art. 101 é taxativo.
emolumentos, gozando de absoluta prioridade.
§ 3o  Caso ainda não definida a paternidade, será  defla- Título III
grado procedimento específico destinado à sua averigua- Da Prática de Ato Infracional
ção, conforme previsto pela Lei no 8.560, de 29 de dezem-
bro de 1992.  Capítulo I
§ 4o  Nas hipóteses previstas no § 3o deste artigo, é dis- Disposições Gerais
pensável o ajuizamento de ação de investigação de pater-
nidade pelo Ministério Público se, após o não compareci- Art. 103. Considera-se ato infracional a conduta des-
mento ou a recusa do suposto pai em assumir a paternida- crita como crime ou contravenção penal.
de a ele atribuída, a criança for encaminhada para adoção. 
§ 5º Os registros e certidões necessários à inclusão, a Art. 104. São penalmente inimputáveis os menores
qualquer tempo, do nome do pai no assento de nascimento de dezoito anos, sujeitos às medidas previstas nesta Lei.
são isentos de multas, custas e emolumentos, gozando de Parágrafo único. Para os efeitos desta Lei, deve ser con-
absoluta prioridade. siderada a idade do adolescente à data do fato.
§ 6º São gratuitas, a qualquer tempo, a averbação re-
querida do reconhecimento de paternidade no assento de Art. 105. Ao ato infracional praticado por criança
nascimento e a certidão correspondente. corresponderão as medidas previstas no art. 101.

As normas de prevenção do ECA são destinadas a Capítulo II


crianças e adolescentes em situação de risco. Existirá situa- Dos Direitos Individuais
ção de risco quando a criança ou o adolescente estiverem
privados de assistência. Essa assistência pode ser material Art. 106. Nenhum adolescente será privado de sua
(quando não se tem onde dormir, o que comer, vestir etc.), liberdade senão em flagrante de ato infracional ou por
moral (quando a criança ou o adolescente permanece em ordem escrita e fundamentada da autoridade judiciária
local inadequado, como locais de prática de jogo, prostitui- competente.
ção etc.) ou jurídica (quando não tem quem o represente). Parágrafo único. O adolescente tem direito à identifi-
O menor que pratica ato infracional está em situação cação dos responsáveis pela sua apreensão, devendo ser
de risco por estar privado de assistência moral. A situação informado acerca de seus direitos.
de risco pode decorrer de ação ou omissão do Poder Pú-
blico; ação ou omissão dos pais ou dos responsáveis; por Art. 107. A apreensão de qualquer adolescente e o local
conduta própria. onde se encontra recolhido serão incontinenti comunicados
O art. 101 do ECA traz um rol das medidas protetivas à autoridade judiciária competente e à família do apreendi-
diante da situação de risco. Essas medidas poderão ser do ou à pessoa por ele indicada.
aplicadas tanto para a criança quanto para o adolescente. Parágrafo único. Examinar-se-á, desde logo e sob pena
São elas: de responsabilidade, a possibilidade de liberação imedia-
- encaminhamento da criança e do adolescente aos ta.
pais ou responsáveis, mediante termo ou responsabilidade;
- orientação, apoio e acompanhamentos temporários Art. 108. A internação, antes da sentença, pode ser de-
por pessoa nomeada pelo Juiz; terminada pelo prazo máximo de quarenta e cinco dias.
- matrícula e frequência obrigatória em estabelecimen- Parágrafo único. A decisão deverá ser fundamentada e
to oficial de ensino fundamental (o Juiz determina aos pais basear-se em indícios suficientes de autoria e materia-
a obrigação); lidade, demonstrada a necessidade imperiosa da medida.
- inclusão em programa comunitário ou oficial de auxí-
lio à família, à criança e ao adolescente; Art. 109. O adolescente civilmente identificado não
- requisição de tratamento médico, psicológico ou psi- será submetido a identificação compulsória pelos ór-
quiátrico em regime hospitalar (internação) ou ambulato- gãos policiais, de proteção e judiciais, salvo para efeito
rial (consultas periódicas); de confrontação, havendo dúvida fundada.
- abrigo em entidade (não se fala em orfanato). A dou-
trina chama de “Tutela de Estado” quando a criança está
em abrigo sob a proteção do Estado;

70
LEGISLAÇÃO

O adolescente não é preso, é apreendido. Art. 113. Aplica-se a este Capítulo o disposto nos arts.
A internação é a medida mais gravosa para o adoles- 99 e 100.
cente. O ECA permite a internação provisória durante o
processo. É fixado o prazo máximo de 45 dias. Os funda- Art. 114. A imposição das medidas previstas nos incisos
mentos para que o Juiz decrete essa internação provisória II a VI do art. 112 pressupõe a existência de provas suficien-
são: indícios suficientes de autoria e materialidade e neces- tes da autoria e da materialidade da infração, ressalva-
sidade da medida. da a hipótese de remissão, nos termos do art. 127.
Esse prazo de internação provisória será descontado Parágrafo único. A advertência poderá ser aplicada
na internação definitiva. Em nenhuma hipótese a criança sempre que houver prova da materialidade e indícios sufi-
poderá ser internada. Criança, que é todo aquele menor cientes da autoria.
de 12 anos, não se sujeita a medida sócio-educativa, mas
apenas a medida de proteção. As medidas socioeducativas dependem de um proce-
dimento judicial, só podendo ser aplicadas pelo Juiz. O ECA
Capítulo III apresenta dois critérios genéricos para a aplicação de me-
Das Garantias Processuais dida socioeducativa:
- capacidade do adolescente para cumprir a medida;
Art. 110. Nenhum adolescente será privado de sua liber- - circunstâncias e gravidade da infração.
dade sem o devido processo legal. A internação é uma exceção, existindo hipóteses legais
para sua aplicação.
Art. 111. São asseguradas ao adolescente, entre outras, A medida de segurança não poderá ser aplicada ao
as seguintes garantias: adolescente, tendo em vista ser medida para maior de ida-
I - pleno e formal conhecimento da atribuição de ato de que apresenta periculosidade. No caso de adolescente
infracional, mediante citação ou meio equivalente; doente mental, será aplicada medida de proteção, poden-
II - igualdade na relação processual, podendo con- do ser requisitado tratamento médico.
frontar-se com vítimas e testemunhas e produzir todas as O Juiz poderá cumular medidas socioeducativas, desde
provas necessárias à sua defesa; que sejam compatíveis (ex.: prestação de serviço à comu-
III - defesa técnica por advogado; nidade cumulada com reparação de danos). Com exceção
IV - assistência judiciária gratuita e integral aos ne- da internação, o Juiz poderá substituir as medidas socioe-
cessitados, na forma da lei; ducativas de acordo com o caso concreto, visto não haver
V - direito de ser ouvido pessoalmente pela autoridade taxatividade.
competente; Se o Promotor discordar com a medida socioeducati-
VI - direito de solicitar a presença de seus pais ou va aplicada, deverá entrar com recurso de apelação. Essa
responsável em qualquer fase do procedimento. apelação do ECA possui juízo de retratação, ou seja, o Juiz
pode voltar atrás na decisão. O Tribunal competente para
Capítulo IV julgar essa apelação é o TJ.
Das Medidas Socioeducativas
Seção II
Seção I Da Advertência
Disposições Gerais
Art. 115. A advertência consistirá em admoestação
Art. 112. Verificada a prática de ato infracional, a autori- verbal, que será reduzida a termo e assinada.
dade competente poderá aplicar ao adolescente as seguintes
medidas: Disposta no art. 115 do ECA, é uma medida sócio-e-
I - advertência; ducativa que consiste em uma admoestação verbal que é
II - obrigação de reparar o dano; aplicada pelo Juiz ao adolescente e que é reduzida a termo.
III - prestação de serviços à comunidade; É destinada a atos de menor gravidade.
IV - liberdade assistida; Para a aplicação da advertência, o Juiz deve levar em
V - inserção em regime de semi-liberdade; consideração a prova da materialidade e indícios suficien-
VI - internação em estabelecimento educacional; tes de autoria. É a única medida que o Juiz poderá aplicar
VII - qualquer uma das previstas no art. 101, I a VI. fundamentando-se somente em indícios de autoria.
§ 1º A medida aplicada ao adolescente levará em con-
ta a sua capacidade de cumpri-la, as circunstâncias e a Seção III
gravidade da infração. Da Obrigação de Reparar o Dano
§ 2º Em hipótese alguma e sob pretexto algum, será
admitida a prestação de trabalho forçado. Art. 116. Em se tratando de ato infracional com refle-
§ 3º Os adolescentes portadores de doença ou defi- xos patrimoniais, a autoridade poderá determinar, se for o
ciência mental receberão tratamento individual e espe- caso, que o adolescente restitua a coisa, promova o ressar-
cializado, em local adequado às suas condições. cimento do dano, ou, por outra forma, compense o prejuízo
da vítima.

71
LEGISLAÇÃO

Parágrafo único. Havendo manifesta impossibilidade, a I - promover socialmente o adolescente e sua família,
medida poderá ser substituída por outra adequada. fornecendo-lhes orientação e inserindo-os, se necessário, em
programa oficial ou comunitário de auxílio e assistência so-
Obrigação de reparar o dano (art. 116 do ECA). Há um cial;
pressuposto: o ato infracional deve ter causado um dano à II - supervisionar a frequência e o aproveitamento es-
vítima. Essa reparação é para a vítima que sofreu o dano. É colar do adolescente, promovendo, inclusive, sua matrícula;
uma medida voltada para o adolescente, então deve ser es- III - diligenciar no sentido da profissionalização do ado-
tabelecida de acordo com a possibilidade de cumprimento lescente e de sua inserção no mercado de trabalho;
pelo adolescente (ex.: devolução da coisa furtada, peque- IV - apresentar relatório do caso.
nos serviços a título de reparação etc.).
A jurisprudência admite que essa reparação de dano É a última medida em que o adolescente permanece
pode ser aplicada à criança (ex.: devolução da coisa furta- com sua família. O Juiz irá determinar um acompanhamen-
da). to permanente ao adolescente, designando, para isso, um
orientador, que poderá ser substituído a qualquer tempo. A
Seção IV lei fixa um prazo mínimo de 6 meses para a duração dessa
Da Prestação de Serviços à Comunidade medida. O orientador terá as seguintes obrigações legais:
- promover socialmente o adolescente, bem como a
Art. 117. A prestação de serviços comunitários consis- sua família, inserindo-os em programas sociais. Promover
te na realização de tarefas gratuitas de interesse geral, socialmente é fazer com que o adolescente realize ativida-
por período não excedente a seis meses, junto a entida- des valorizadas socialmente (teatro, música etc.);
des assistenciais, hospitais, escolas e outros estabelecimen- - supervisionar a frequência e o aproveitamento esco-
tos congêneres, bem como em programas comunitários ou lar do adolescente;
governamentais. - profissionalizar o adolescente (nos termos da EC n.
Parágrafo único. As tarefas serão atribuídas conforme as 20);
- apresentar relatório do caso ao Juiz.
aptidões do adolescente, devendo ser cumpridas durante jor-
nada máxima de oito horas semanais, aos sábados, domin-
Seção VI
gos e feriados ou em dias úteis, de modo a não prejudicar a
Do Regime de Semiliberdade
frequência à escola ou à jornada normal de trabalho.
Art. 120. O regime de semiliberdade pode ser determi-
Disposta no art. 117 do ECA, o adolescente será obri-
nado desde o início, ou como forma de transição para o
gado a prestar serviços em benefício da coletividade. São
meio aberto, possibilitada a realização de atividades exter-
tarefas gratuitas de interesse geral junto a entidades as-
nas, independentemente de autorização judicial.
sistenciais, hospitais, escolas ou estabelecimentos congê- § 1º São obrigatórias a escolarização e a profissio-
neres. nalização, devendo, sempre que possível, ser utilizados os
Como a medida é mais gravosa, a lei fixa um prazo recursos existentes na comunidade.
máximo de 6 meses para essa prestação e um máximo de § 2º A medida não comporta prazo determinado
8 horas semanais. Essas 8 horas poderão ser estabeleci- aplicando-se, no que couber, as disposições relativas à in-
das discricionariamente, desde que não prejudiquem a fre- ternação.
quência ao trabalho e à escola. Deverá ser levada em conta
a aptidão do adolescente para a aplicação da medida. Disposta no art. 120 do ECA, é uma medida que impor-
ta em privação de liberdade ao adolescente que pratica um
Seção V ato infracional mais grave. O adolescente é retirado de sua
Da Liberdade Assistida família e colocado em um estabelecimento apropriado de
semiliberdade, podendo realizar atividades externas (estu-
Art. 118. A liberdade assistida será adotada sempre dar, trabalhar etc.) somente com autorização do diretor do
que se afigurar a medida mais adequada para o fim de estabelecimento, não havendo necessidade de autorização
acompanhar, auxiliar e orientar o adolescente. judicial. Pode ser usada tanto como medida principal quan-
§ 1º A autoridade designará pessoa capacitada para to como medida progressiva ou regressiva.
acompanhar o caso, a qual poderá ser recomendada por A semiliberdade não tem prazo fixado em lei, nem
entidade ou programa de atendimento. mínimo nem máximo. A doutrina e a jurisprudência deter-
§ 2º A liberdade assistida será fixada pelo prazo míni- minam a aplicação da medida por analogia dos prazos da
mo de seis meses, podendo a qualquer tempo ser prorro- internação, tendo como prazo máximo 3 anos. Há a obri-
gada, revogada ou substituída por outra medida, ouvido o gatoriedade de escolarização e profissionalização na semi-
orientador, o Ministério Público e o defensor. liberdade.

Art. 119. Incumbe ao orientador, com o apoio e a su-


pervisão da autoridade competente, a realização dos seguin-
tes encargos, entre outros:

72
LEGISLAÇÃO

Seção VII VII - receber visitas, ao menos, semanalmente;


Da Internação VIII - corresponder-se com seus familiares e amigos;
IX - ter acesso aos objetos necessários à higiene e asseio
Art. 121. A internação constitui medida privativa da pessoal;
liberdade, sujeita aos princípios de brevidade, excepcio- X - habitar alojamento em condições adequadas de hi-
nalidade e respeito à condição peculiar de pessoa em giene e salubridade;
desenvolvimento. XI - receber escolarização e profissionalização;
§ 1º Será permitida a realização de atividades externas, XII - realizar atividades culturais, esportivas e de lazer:
a critério da equipe técnica da entidade, salvo expressa de- XIII - ter acesso aos meios de comunicação social;
terminação judicial em contrário. XIV - receber assistência religiosa, segundo a sua crença,
§ 2º A medida não comporta prazo determinado, de- e desde que assim o deseje;
vendo sua manutenção ser reavaliada, mediante decisão XV - manter a posse de seus objetos pessoais e dispor de
fundamentada, no máximo a cada seis meses. local seguro para guardá-los, recebendo comprovante da-
§ 3º Em nenhuma hipótese o período máximo de inter- queles porventura depositados em poder da entidade;
nação excederá a três anos. XVI - receber, quando de sua desinternação, os docu-
§ 4º Atingido o limite estabelecido no parágrafo ante- mentos pessoais indispensáveis à vida em sociedade.
rior, o adolescente deverá ser liberado, colocado em regi- § 1º Em nenhum caso haverá incomunicabilidade.
me de semiliberdade ou de liberdade assistida. § 2º A autoridade judiciária poderá suspender tem-
§ 5º A liberação será compulsória aos vinte e um anos porariamente a visita, inclusive de pais ou responsável, se
de idade. existirem motivos sérios e fundados de sua prejudicialidade
§ 6º Em qualquer hipótese a desinternação será pre- aos interesses do adolescente.
cedida de autorização judicial, ouvido o Ministério Público.
§ 7o  A determinação judicial mencionada no § 1o pode- Art. 125. É dever do Estado zelar pela integridade física
rá ser revista a qualquer tempo pela autoridade judiciária. e mental dos internos, cabendo-lhe adotar as medidas ade-
quadas de contenção e segurança.
Art. 122. A medida de internação só poderá ser aplicada
quando: Disposta no art. 121 e seguintes do ECA, é a medida re-
I - tratar-se de ato infracional cometido mediante grave servada para os atos infracionais de natureza grave. O ECA
ameaça ou violência a pessoa; estabelece princípios específicos para a internação, pois é
II - por reiteração no cometimento de outras infrações medida de privação de liberdade sempre excepcional.
graves; A internação deve durar o menor tempo possível (prin-
III - por descumprimento reiterado e injustificável da cípio da brevidade), é uma medida de exceção que só de-
medida anteriormente imposta. verá ser utilizada em último caso (princípio da excepcio-
§ 1o  O prazo de internação na hipótese do inciso III nalidade) e deve seguir o princípio do respeito à condição
deste artigo não poderá ser superior a 3 (três) meses, de- peculiar do adolescente como pessoa em desenvolvimen-
vendo ser decretada judicialmente após o devido processo to. Em nenhuma hipótese pode ser aplicada à criança.
legal. O ECA estabelece hipóteses de internação para:
§ 2º Em nenhuma hipótese será aplicada a internação, - prática de ato infracional mediante grave ameaça ou
havendo outra medida adequada. violência à pessoa;
- reiteração de infrações graves;
Art. 123. A internação deverá ser cumprida em entida- - descumprimento reiterado e injustificado da medi-
de exclusiva para adolescentes, em local distinto daquele da anteriormente imposta (é uma hipótese de regressão).
destinado ao abrigo, obedecida rigorosa separação por cri- Neste caso, a internação não pode ultrapassar o prazo de
térios de idade, compleição física e gravidade da infração. 3 meses.
Parágrafo único. Durante o período de internação, inclu- Nas duas primeiras hipóteses, o prazo máximo para in-
sive provisória, serão obrigatórias atividades pedagógicas. ternação é de 3 anos. Por força desse prazo, o ECA poderá
atingir o maior de 18 anos. Em rigor, todas as medidas só-
Art. 124. São direitos do adolescente privado de liberda- cio-educativas poderão atingir o maior de 18 anos.
de, entre outros, os seguintes: A medida só poderá ser aplicada com o devido pro-
I - entrevistar-se pessoalmente com o representante do cesso legal e em nenhuma hipótese poderá ser aplicada à
Ministério Público; criança. Quando o adolescente completar 21 anos, a libe-
II - peticionar diretamente a qualquer autoridade; ração será obrigatória. Caso o adolescente tenha passado
III - avistar-se reservadamente com seu defensor; por internação provisória, esses dias serão computados na
IV - ser informado de sua situação processual, sempre internação (detração). A diferença entre semi-liberdade e
que solicitada; internação é que, nesta, o adolescente depende de auto-
V - ser tratado com respeito e dignidade; rização expressa do juiz para praticar atividades externas,
VI - permanecer internado na mesma localidade ou na- ou seja, o adolescente internado somente se ausentará do
quela mais próxima ao domicílio de seus pais ou responsá- estabelecimento em que se achar se autorizado pelo juiz.
vel;

73
LEGISLAÇÃO

O art. 123 dispõe que o local para a internação deve - o Promotor oferece a representação;
ser distinto do abrigo, devendo-se obedecer a separação - na audiência de apresentação, o menor será ouvido
por idade, composição física (tamanho), sexo e gravidade pelo Juiz, que poderá decidir pela remissão;
do ato infracional. Há, também, a obrigatoriedade de reali- - o representante do MP deverá, obrigatoriamente, ser
zação de atividades pedagógicas. ouvido sobre a possibilidade da remissão antes de ela ser
O art. 124 dispõe sobre direitos específicos dos ado- aplicada. A remissão concedida pelo Juiz causa extinção do
lescentes: processo. Havendo discordância por parte do MP, este de-
- entrevista pessoal com o representante do MP; verá ingressar com uma apelação para reformar a decisão
- entrevista reservada com seu defensor, dentre outros. do Juiz.
As visitas podem ser suspensas pelo juiz, sob o funda- Tanto a doutrina quanto a jurisprudência admitem a
mento de segurança e proteção do menor, entretanto, em cumulação da remissão com uma medida sócio-educativa
nenhuma hipótese o menor poderá ficar incomunicável. que seja compatível (ex.: reparação do dano, advertência
etc.). Neste caso, a remissão é causa de suspensão do pro-
Capítulo V cesso.
Da Remissão O ECA traz quatro requisitos genéricos para a aplicação
da remissão, devendo ficar a critério do membro do MP ou
Art. 126. Antes de iniciado o procedimento judicial do Juiz a sua concessão. São eles:
para apuração de ato infracional, o representante do - circunstâncias e consequências do fato;
Ministério Público poderá conceder a remissão, como - contexto social em que o fato foi praticado;
forma de exclusão do processo, atendendo às circunstâncias - personalidade do agente;
e consequências do fato, ao contexto social, bem como à per- - maior ou menor participação no ato infracional.
sonalidade do adolescente e sua maior ou menor participa- A remissão, quer concedida pelo MP quer pelo Juiz,
ção no ato infracional. não implica confissão de culpa. Existe uma divergência na
Parágrafo único. Iniciado o procedimento, a concessão doutrina em considerar a remissão como um acordo ou
da remissão pela autoridade judiciária importará na suspen- não. A posição majoritária entende que a remissão não é
são ou extinção do processo. um acordo, tendo em vista a lei falar em concessão e, ain-
da, pelo fato de não haver nenhum prejuízo para o adoles-
Art. 127. A remissão não implica necessariamente o cente, não possuindo a remissão nenhum efeito, podendo
reconhecimento ou comprovação da responsabilidade, ser concedida quantas vezes forem necessárias.
nem prevalece para efeito de antecedentes, podendo in-
cluir eventualmente a aplicação de qualquer das medidas
Título IV
previstas em lei, exceto a colocação em regime de semi-li-
Das Medidas Pertinentes aos Pais ou Responsável
berdade e a internação.
Art. 129. São medidas aplicáveis aos pais ou responsá-
Art. 128. A medida aplicada por força da remissão pode-
vel:
rá ser revista judicialmente, a qualquer tempo, mediante
I - encaminhamento a serviços e programas oficiais ou
pedido expresso do adolescente ou de seu representante le-
gal, ou do Ministério Público. comunitários de proteção, apoio e promoção da família;
II - inclusão em programa oficial ou comunitário de au-
Tem por conceito o perdão, a indulgência ao menor. xílio, orientação e tratamento a alcoólatras e toxicômanos;
Podem conceder remissão tanto o MP quanto o Juiz. São III - encaminhamento a tratamento psicológico ou psi-
hipóteses de natureza jurídica diferentes. A remissão ju- quiátrico;
dicial é forma de extinção ou de suspensão do processo IV - encaminhamento a cursos ou programas de orien-
(portanto, pressupõe o processo em curso). Já a remissão tação;
ministerial é forma de exclusão do processo (logo, deve V - obrigação de matricular o filho ou pupilo e acompa-
ser concedida antes do processo - administrativamente). nhar sua frequência e aproveitamento escolar;
Quando a remissão é concedida pelo MP, segue-se o se- VI - obrigação de encaminhar a criança ou adolescente
guinte procedimento: a tratamento especializado;
- o menor é ouvido pelo Promotor que concederá a VII - advertência;
remissão; VIII - perda da guarda;
- o Promotor encaminha a remissão para homologação IX - destituição da tutela;
pelo Juiz; X - suspensão ou destituição do poder familiar.
- se o Juiz não aceitar a remissão, deverá remeter para Parágrafo único. Na aplicação das medidas previstas
o Procurador de Justiça, que poderá insistir na remissão nos incisos IX e X deste artigo, observar-se-á o disposto nos
ou designar outro representante do MP para apresentar arts. 23 e 24.
representação contra o menor. Essa remissão concedida
pelo MP é causa de exclusão do processo, visto que, ao Art. 130. Verificada a hipótese de maus-tratos, opressão
conceder a remissão, inexiste o processo. ou abuso sexual impostos pelos pais ou responsável, a auto-
Quando a remissão é concedida pelo Juiz, segue-se o ridade judiciária poderá determinar, como medida cautelar,
seguinte procedimento: o afastamento do agressor da moradia comum.

74
LEGISLAÇÃO

Parágrafo único.  Da medida cautelar constará, ainda, a IV - encaminhar ao Ministério Público notícia de fato
fixação provisória dos alimentos de que necessitem a criança que constitua infração administrativa ou penal contra os di-
ou o adolescente dependentes do agressor. reitos da criança ou adolescente;
V - encaminhar à autoridade judiciária os casos de sua
Capítulo I competência;
Disposições Gerais VI - providenciar a medida estabelecida pela autoridade
judiciária, dentre as previstas no art. 101, de I a VI, para o
Art. 131. O Conselho Tutelar é órgão permanente e au- adolescente autor de ato infracional;
tônomo, não jurisdicional, encarregado pela sociedade de VII - expedir notificações;
zelar pelo cumprimento dos direitos da criança e do adoles- VIII - requisitar certidões de nascimento e de óbito de
cente, definidos nesta Lei. criança ou adolescente quando necessário;
IX - assessorar o Poder Executivo local na elaboração da
Art. 132.  Em cada Município e em cada Região Admi- proposta orçamentária para planos e programas de atendi-
nistrativa do Distrito Federal haverá, no mínimo, 1 (um) mento dos direitos da criança e do adolescente;
Conselho Tutelar como órgão integrante da administração X - representar, em nome da pessoa e da família, contra
pública local, composto de 5 (cinco) membros, escolhidos a violação dos direitos previstos no art. 220, § 3º, inciso II, da
pela população local para mandato de 4 (quatro) anos, Constituição Federal;
permitida 1 (uma) recondução, mediante novo processo XI - representar ao Ministério Público para efeito das
de escolha.  ações de perda ou suspensão do poder familiar, após esgota-
das as possibilidades de manutenção da criança ou do ado-
Art. 133. Para a candidatura a membro do Conselho lescente junto à família natural;
Tutelar, serão exigidos os seguintes requisitos: XII - promover e incentivar, na comunidade e nos gru-
I - reconhecida idoneidade moral; pos profissionais, ações de divulgação e treinamento para o
II - idade superior a vinte e um anos; reconhecimento de sintomas de maus-tratos em crianças e
III - residir no município. adolescentes.
Parágrafo único.  Se, no exercício de suas atribuições, o
Conselho Tutelar entender necessário o afastamento do con-
Art. 134.  Lei municipal ou distrital disporá sobre o
vívio familiar, comunicará incontinenti o fato ao Ministério
local, dia e horário de funcionamento do Conselho Tu-
Público, prestando-lhe informações sobre os motivos de tal
telar, inclusive quanto à remuneração dos respectivos mem-
entendimento e as providências tomadas para a orientação,
bros, aos quais é assegurado o direito a:
o apoio e a promoção social da família. 
I - cobertura previdenciária; 
II - gozo de férias anuais remuneradas, acrescidas de 1/3 Art. 137. As decisões do Conselho Tutelar somente po-
(um terço) do valor da remuneração mensal; derão ser revistas pela autoridade judiciária a pedido de
III - licença-maternidade; quem tenha legítimo interesse.
IV - licença-paternidade; 
V - gratificação natalina.  Capítulo III
Parágrafo único.  Constará da lei orçamentária munici- Da Competência
pal e da do Distrito Federal previsão dos recursos necessários
ao funcionamento do Conselho Tutelar e à remuneração e Art. 138. Aplica-se ao Conselho Tutelar a regra de com-
formação continuada dos conselheiros tutelares. petência constante do art. 147.
Art. 135.  O exercício efetivo da função de conselheiro Capítulo IV
constituirá serviço público relevante e estabelecerá presun- Da Escolha dos Conselheiros
ção de idoneidade moral. 
Art. 139. O processo para a escolha dos membros do
Capítulo II Conselho Tutelar será estabelecido em lei municipal e rea-
Das Atribuições do Conselho lizado sob a responsabilidade do Conselho Municipal dos
Direitos da Criança e do Adolescente, e a fiscalização do
Art. 136. São atribuições do Conselho Tutelar: Ministério Público. 
I - atender as crianças e adolescentes nas hipóteses pre- § 1o  O processo de escolha dos membros do Conse-
vistas nos arts. 98 e 105, aplicando as medidas previstas no lho Tutelar ocorrerá em data unificada em todo o território
art. 101, I a VII; nacional a cada 4 (quatro) anos, no primeiro domingo do
II - atender e aconselhar os pais ou responsável, aplican- mês de outubro do ano subsequente ao da eleição pre-
do as medidas previstas no art. 129, I a VII; sidencial. 
III - promover a execução de suas decisões, podendo § 2o  A posse dos conselheiros tutelares ocorrerá no dia
para tanto: 10 de janeiro do ano subsequente ao processo de escolha. 
a) requisitar serviços públicos nas áreas de saúde, edu- § 3o  No processo de escolha dos membros do Con-
cação, serviço social, previdência, trabalho e segurança; selho Tutelar, é vedado ao candidato doar, oferecer, pro-
b) representar junto à autoridade judiciária nos casos de meter ou entregar ao eleitor bem ou vantagem pessoal de
descumprimento injustificado de suas deliberações. qualquer natureza, inclusive brindes de pequeno valor. 

75
LEGISLAÇÃO

Capítulo V Contudo, a solução possibilidade pela autotutela era


Dos Impedimentos bastante insatisfatória e fazia com que prevalecesse a lei do
mais forte. Então, surgiu o Estado apresentando um melhor
Art. 140. São impedidos de servir no mesmo Conselho sistema para a solução dos conflitos.
marido e mulher, ascendentes e descendentes, sogro e O Estado assumiu para si o poder-dever de dizer o Di-
genro ou nora, irmãos, cunhados, durante o cunhadio, tio reito, de solucionar os conflitos, conhecido como jurisdi-
e sobrinho, padrasto ou madrasta e enteado. ção. Assim, o Estado irá elaborar as leis (direito material)
Parágrafo único. Estende-se o impedimento do conselheiro, na e prever como elas serão aplicadas (direito processual). A
forma deste artigo, em relação à autoridade judiciária e ao repre- autotutela para a ser punida como regra geral e o Estado
sentante do Ministério Público com atuação na Justiça da Infância exerce a heterotutela por meio da atividade jurisdicional.
e da Juventude, em exercício na comarca, foro regional ou distrital. Jurisdição é o poder-dever do Estado de dizer o Direi-
to. Sendo assim, trata-se de atividade estatal exercida por
Título VI intermédio de um agente constituído com competência
Do Acesso à Justiça para exercê-la, o juiz.
Nos primórdios da humanidade não existia o Direito e
Capítulo I nem existiam as leis, de modo que a justiça era feita pelas
Disposições Gerais próprias mãos, na denominada autotutela. Com a evolução
das instituições, o Estado avocou para si o poder-dever de
Art. 141. É garantido o acesso de toda criança ou ado- solucionar os litígios, o que é feito pela jurisdição.
lescente à Defensoria Pública, ao Ministério Público e ao O poder-dever de dizer o direito é uno, apenas existin-
Poder Judiciário, por qualquer de seus órgãos. do uma separação de funções: o Legislativo regulamenta
§ 1º. A assistência judiciária gratuita será prestada aos
normas gerais e abstratas (função legislativa) e o Judiciário
que dela necessitarem, através de defensor público ou advo-
as aplica no caso concreto (função jurisdicional).
gado nomeado.
Entretanto, vale destacar que na sociedade contempo-
§ 2º As ações judiciais da competência da Justiça da In-
rânea, devido às inúmeras mazelas que se apresentaram
fância e da Juventude são isentas de custas e emolumen-
envolvendo o abarrotamento de processos pelo Judiciário,
tos, ressalvada a hipótese de litigância de má-fé.
passou-se a incentivar a adoção de métodos de autocom-
Art. 142. Os menores de dezesseis anos serão represen- posição, como conciliação, mediação e arbitragem.
tados e os maiores de dezesseis e menores de vinte e um Tradicionalmente, são enumerados pela doutrina os
anos assistidos por seus pais, tutores ou curadores, na forma seguintes princípios inerentes à jurisdição: investidura,
da legislação civil ou processual. porque somente exerce jurisdição quem ocupa o cargo de
Parágrafo único. A autoridade judiciária dará curador es- juiz; aderência ao território, posto que juízes somente têm
pecial à criança ou adolescente, sempre que os interesses des- autoridade no território nacional e nos limites de sua com-
tes colidirem com os de seus pais ou responsável, ou quando petência; indelegabilidade, não podendo o Poder Judiciário
carecer de representação ou assistência legal ainda que even- delegar sua competência; inafastabilidade, pois a lei não
tual. pode excluir da apreciação do Poder Judiciário nenhuma
lesão ou ameaça a direito.
Art. 143. E vedada a divulgação de atos judiciais, poli- Embora a jurisdição seja una, em termos doutrinários
ciais e administrativos que digam respeito a crianças e ado- é possível classificá-la: a) quanto ao objeto – penal, traba-
lescentes a que se atribua autoria de ato infracional. lhista e civil (a civil é subsidiária, envolvendo todo direito
Parágrafo único. Qualquer notícia a respeito do fato não material que não seja penal ou trabalhista, não somente
poderá identificar a criança ou adolescente, vedando-se foto- questões inerentes ao direito civil); b) quanto ao organismo
grafia, referência a nome, apelido, filiação, parentesco, residên- que a exerce – comum (estadual ou federal) ou especial
cia e, inclusive, iniciais do nome e sobrenome.  (trabalhista, militar, eleitoral); c) quanto à hierarquia – su-
perior e inferior.
Art. 144. A expedição de cópia ou certidão de atos a que Neste sentido, com vistas a instrumentalizar a jurisdi-
se refere o artigo anterior somente será deferida pela autorida- ção, impedindo que ela seja exercida de maneira caótica,
de judiciária competente, se demonstrado o interesse e justifi- ela é distribuída entre juízos e foros (órgãos competentes
cada a finalidade. em localidades determinadas). A esta distribuição das par-
celas de jurisdição dá-se o nome de competência.
O homem necessita do convívio social, não é um ser ca- As tutelas jurisdicionais diferenciadas, por sua vez,
paz de viver de maneira autônoma e totalmente desvincula- são aquelas que apresentam procedimentos diversos do
da dos demais. Neste sentido, a imposição de regramentos comum. Possuem procedimentos ditos especiais, os quais
e normas permitiu que a sociedade atingisse o atual grau de buscam garantir um processo mais rápido e compatível
evolução. com as necessidades específicas do direito em discussão.
Obviamente, no ambiente social surgem conflitos de in- No âmbito do direito da criança e do adolescente, tem-se
teresses. Afinal, nem sempre os bens e valores existem em o estabelecimento de uma tutela jurisdicional diferenciada,
quantidade suficiente para atender a todas as pessoas. eis que existem inúmeras regras específicas aplicáveis aos
Inicialmente, estes conflitos eram solucionados pelos pró- processos que envolvem de algum modo criança ou ado-
prios envolvidos, na denominada fase da autotutela. lescente.

76
LEGISLAÇÃO

A noção de jurisdição inclusiva também se aplica à tu- VII - conhecer de casos encaminhados pelo Conselho Tu-
tela jurisdicional da criança e do adolescente. Basicamente, telar, aplicando as medidas cabíveis.
refere-se à propiciação de uma jurisdição que esteja atenta Parágrafo único. Quando se tratar de criança ou ado-
às peculiaridades das minorias e dos grupos vulneráveis. lescente nas hipóteses do art. 98, é também competente a
No caso, as crianças e adolescentes são considerados um Justiça da Infância e da Juventude para o fim de:
grupo vulnerável devido à condição especial que ocupam. a) conhecer de pedidos de guarda e tutela;
b) conhecer de ações de destituição do poder familiar,
Capítulo II perda ou modificação da tutela ou guarda; 
Da Justiça da Infância e da Juventude c) suprir a capacidade ou o consentimento para o casa-
mento;
Seção I d) conhecer de pedidos baseados em discordância pater-
Disposições Gerais na ou materna, em relação ao exercício do poder familiar; 
e) conceder a emancipação, nos termos da lei civil,
Art. 145. Os estados e o Distrito Federal poderão criar quando faltarem os pais;
varas especializadas e exclusivas da infância e da juventude, f) designar curador especial em casos de apresentação
cabendo ao Poder Judiciário estabelecer sua proporcionali- de queixa ou representação, ou de outros procedimentos ju-
dade por número de habitantes, dotá-las de infra-estrutura e diciais ou extrajudiciais em que haja interesses de criança ou
dispor sobre o atendimento, inclusive em plantões. adolescente;
g) conhecer de ações de alimentos;
Seção II h) determinar o cancelamento, a retificação e o supri-
Do Juiz mento dos registros de nascimento e óbito.

Art. 146. A autoridade a que se refere esta Lei é o Juiz da Art. 149. Compete à autoridade judiciária disciplinar,
Infância e da Juventude, ou o juiz que exerce essa função, na através de portaria, ou autorizar, mediante alvará:
forma da lei de organização judiciária local. I - a entrada e permanência de criança ou adolescente,
desacompanhado dos pais ou responsável, em:
Art. 147. A competência será determinada: a) estádio, ginásio e campo desportivo;
I - pelo domicílio dos pais ou responsável; b) bailes ou promoções dançantes;
II - pelo lugar onde se encontre a criança ou adolescente, c) boate ou congêneres;
à falta dos pais ou responsável. d) casa que explore comercialmente diversões eletrôni-
§ 1º. Nos casos de ato infracional, será competente a cas;
autoridade do lugar da ação ou omissão, observadas as re- e) estúdios cinematográficos, de teatro, rádio e televisão.
gras de conexão, continência e prevenção. II - a participação de criança e adolescente em:
§ 2º A execução das medidas poderá ser delegada à a) espetáculos públicos e seus ensaios;
autoridade competente da residência dos pais ou respon- b) certames de beleza.
sável, ou do local onde sediar-se a entidade que abrigar a § 1º Para os fins do disposto neste artigo, a autoridade
criança ou adolescente. judiciária levará em conta, dentre outros fatores:
§ 3º Em caso de infração cometida através de trans- a) os princípios desta Lei;
missão simultânea de rádio ou televisão, que atinja mais b) as peculiaridades locais;
de uma comarca, será competente, para aplicação da pe- c) a existência de instalações adequadas;
nalidade, a autoridade judiciária do local da sede estadual d) o tipo de frequência habitual ao local;
da emissora ou rede, tendo a sentença eficácia para todas e) a adequação do ambiente a eventual participação ou
as transmissoras ou retransmissoras do respectivo estado. frequência de crianças e adolescentes;
f) a natureza do espetáculo.
Art. 148. A Justiça da Infância e da Juventude é compe- § 2º As medidas adotadas na conformidade deste ar-
tente para: tigo deverão ser fundamentadas, caso a caso, vedadas as
I - conhecer de representações promovidas pelo Minis- determinações de caráter geral.
tério Público, para apuração de ato infracional atribuído a
adolescente, aplicando as medidas cabíveis; Seção III
II - conceder a remissão, como forma de suspensão ou Dos Serviços Auxiliares
extinção do processo;
III - conhecer de pedidos de adoção e seus incidentes; Art. 150. Cabe ao Poder Judiciário, na elaboração de sua
IV - conhecer de ações civis fundadas em interesses indi- proposta orçamentária, prever recursos para manutenção de
viduais, difusos ou coletivos afetos à criança e ao adolescen- equipe interprofissional, destinada a assessorar a Justiça da
te, observado o disposto no art. 209; Infância e da Juventude.
V - conhecer de ações decorrentes de irregularidades em
entidades de atendimento, aplicando as medidas cabíveis; Art. 151. Compete à equipe interprofissional dentre ou-
VI - aplicar penalidades administrativas nos casos de in- tras atribuições que lhe forem reservadas pela legislação lo-
frações contra norma de proteção à criança ou adolescente; cal, fornecer subsídios por escrito, mediante laudos, ou ver-

77
LEGISLAÇÃO

balmente, na audiência, e bem assim desenvolver trabalhos Os procedimentos especiais do ECA se referem a: per-
de aconselhamento, orientação, encaminhamento, preven- da e suspensão de poder familiar, destituição de tutela, co-
ção e outros, tudo sob a imediata subordinação à autoridade locação em família substituta, apuração de ato infracional
judiciária, assegurada a livre manifestação do ponto de vista atribuído a adolescente, apuração de irregularidades em
técnico. atendimento, apuração de infração administrativa às nor-
mas de proteção da criança e do adolescente e habilitação
Capítulo III em adoção.
Dos Procedimentos
Seção II
Seção I Da Perda e da Suspensão do Poder Familiar
Disposições Gerais
Art. 155. O procedimento para a perda ou a suspensão
Art. 152. Aos procedimentos regulados nesta Lei apli- do poder familiar terá início por provocação do Ministério
cam-se subsidiariamente as normas gerais previstas na Público ou de quem tenha legítimo interesse.
legislação processual pertinente.
Parágrafo único.  É assegurada, sob pena de responsabi- Art. 156. A petição inicial indicará:
lidade, prioridade absoluta na tramitação dos processos I - a autoridade judiciária a que for dirigida;
e procedimentos previstos nesta Lei, assim como na execu- II - o nome, o estado civil, a profissão e a residência do
ção dos atos e diligências judiciais a eles referentes.  requerente e do requerido, dispensada a qualificação em se
tratando de pedido formulado por representante do Minis-
Art. 153. Se a medida judicial a ser adotada não cor- tério Público;
responder a procedimento previsto nesta ou em outra lei, a III - a exposição sumária do fato e o pedido;
autoridade judiciária poderá investigar os fatos e ordenar IV - as provas que serão produzidas, oferecendo, desde
de ofício as providências necessárias, ouvido o Ministé- logo, o rol de testemunhas e documentos.
rio Público.
Parágrafo único.  O disposto neste artigo não se aplica Art. 157. Havendo motivo grave, poderá a autoridade
judiciária, ouvido o Ministério Público, decretar a suspensão
para o fim de afastamento da criança ou do adolescente de
do poder familiar, liminar ou incidentalmente, até o julga-
sua família de origem e em outros procedimentos necessa-
mento definitivo da causa, ficando a criança ou adolescente
riamente contenciosos. 
confiado a pessoa idônea, mediante termo de responsabili-
dade. 
Art. 154. Aplica-se às multas o disposto no art. 214.
Art. 158. O requerido será citado para, no prazo de dez
A tutela sócio-individual abrange aspectos do direito
dias, oferecer resposta escrita, indicando as provas a serem
da criança e do adolescente voltado à criança e ao adoles- produzidas e oferecendo desde logo o rol de testemunhas
cente individualmente concebidos, isto é, pensados como e documentos.
sujeitos de direitos individuais que possam ser por eles § 1º A citação será pessoal, salvo se esgotados todos os
exercidos. meios para sua realização.
A tutela sócio-educativa abrange aspectos do direito § 2º O requerido privado de liberdade deverá ser cita-
da criança e do adolescente voltados às atividades de en- do pessoalmente.
sino e aprendizagem, tanto no que se refere à educação
formal quanto em relação à educação informal. Art. 159. Se o requerido não tiver possibilidade de cons-
A tutela coletiva volta-se à proteção de direitos difu- tituir advogado, sem prejuízo do próprio sustento e de sua
sos e coletivos da criança e do adolescente. Aos direitos família, poderá requerer, em cartório, que lhe seja nomea-
difusos e coletivos são conferidos mecanismos de tutela do dativo, ao qual incumbirá a apresentação de resposta,
específicos para sua proteção, bem como atribuída com- contando-se o prazo a partir da intimação do despacho de
petência para tanto a órgãos determinados que exercerão nomeação.
um papel representativo. No Brasil, destacam-se institui- Parágrafo único. Na hipótese de requerido privado de
ções como o Ministério Público e a Defensoria Pública. Sem liberdade, o oficial de justiça deverá perguntar, no momento
prejuízo, como visto, há remédios constitucionais que se da citação pessoal, se deseja que lhe seja nomeado defensor.
voltam à proteção de interesses desta categoria, como o
mandado de segurança coletivo e a própria ação popu- Art. 160. Sendo necessário, a autoridade judiciária re-
lar, sem falar na ação civil pública, também mencionada no quisitará de qualquer repartição ou órgão público a apre-
texto constitucional. sentação de documento que interesse à causa, de ofício ou
Considerados os diferentes tipos de tutelas inseridas a requerimento das partes ou do Ministério Público.
no direito da criança e do adolescente, justifica-se a tutela
jurisdicional diferenciada, adaptada à condição em desen- Art. 161. Não sendo contestado o pedido, a autoridade
volvimento da criança e do adolescente, que deve ser ágil, judiciária dará vista dos autos ao Ministério Público, por
efetiva, atenta às peculiaridades do caso concreto. cinco dias, salvo quando este for o requerente, decidindo em
igual prazo.

78
LEGISLAÇÃO

§ 1o  A autoridade judiciária, de ofício ou a requeri- Seção IV


mento das partes ou do Ministério Público, determinará a Da Colocação em Família Substituta
realização de estudo social ou perícia por equipe interpro-
fissional ou multidisciplinar, bem como a oitiva de testemu- Dos artigos 165 a 170 estão descritos procedimentos
nhas que comprovem a presença de uma das causas de sus- adotados na colocação em família substituta:
pensão ou destituição do poder familiar previstas nos arts.
1.637  e  1.638 da Lei no 10.406, de 10 de janeiro de 2002 Art. 165. São requisitos para a concessão de pedidos de
- Código Civil, ou no art. 24 desta Lei.  colocação em família substituta:
§ 2o  Em sendo os pais oriundos de comunidades in- I - qualificação completa do requerente e de seu even-
dígenas, é ainda obrigatória a intervenção, junto à equipe tual cônjuge, ou companheiro, com expressa anuência deste;
profissional ou multidisciplinar referida no § 1o deste arti- II - indicação de eventual parentesco do requerente e de
go, de representantes do órgão federal responsável pela seu cônjuge, ou companheiro, com a criança ou adolescente,
política indigenista, observado o disposto no § 6o do art. especificando se tem ou não parente vivo;
28 desta Lei.  III - qualificação completa da criança ou adolescente e
§ 3o  Se o pedido importar em modificação de guarda, de seus pais, se conhecidos;
será obrigatória, desde que possível e razoável, a oitiva da IV - indicação do cartório onde foi inscrito nascimento,
criança ou adolescente, respeitado seu estágio de desen- anexando, se possível, uma cópia da respectiva certidão;
volvimento e grau de compreensão sobre as implicações V - declaração sobre a existência de bens, direitos ou
da medida.  rendimentos relativos à criança ou ao adolescente.
§ 4o  É obrigatória a oitiva dos pais sempre que esses Parágrafo único. Em se tratando de adoção, observar-
forem identificados e estiverem em local conhecido.  se-ão também os requisitos específicos.
§ 5º Se o pai ou a mãe estiverem privados de liberdade,
a autoridade judicial requisitará sua apresentação para a Art. 166.  Se os pais forem falecidos, tiverem sido
oitiva. destituídos ou suspensos do poder familiar, ou houve-
rem aderido expressamente ao pedido de colocação em
Art. 162. Apresentada a resposta, a autoridade judiciária família substituta, este poderá ser formulado diretamente
dará vista dos autos ao Ministério Público, por cinco dias, em cartório, em petição assinada pelos próprios requerentes,
dispensada a assistência de advogado. 
salvo quando este for o requerente, designando, desde logo,
§ 1o  Na hipótese de concordância dos pais, esses serão
audiência de instrução e julgamento.
ouvidos pela autoridade judiciária e pelo representante do
§ 1º A requerimento de qualquer das partes, do Minis-
Ministério Público, tomando-se por termo as declarações. 
tério Público, ou de ofício, a autoridade judiciária poderá
§ 2o  O consentimento dos titulares do poder familiar
determinar a realização de estudo social ou, se possível, de
será precedido de orientações e esclarecimentos prestados
perícia por equipe interprofissional.
pela equipe interprofissional da Justiça da Infância e da Ju-
§ 2º Na audiência, presentes as partes e o Ministério ventude, em especial, no caso de adoção, sobre a irrevoga-
Público, serão ouvidas as testemunhas, colhendo-se oral- bilidade da medida. 
mente o parecer técnico, salvo quando apresentado por § 3o  O consentimento dos titulares do poder familiar
escrito, manifestando-se sucessivamente o requerente, o será colhido pela autoridade judiciária competente em
requerido e o Ministério Público, pelo tempo de vinte mi- audiência, presente o Ministério Público, garantida a livre
nutos cada um, prorrogável por mais dez. A decisão será manifestação de vontade e esgotados os esforços para ma-
proferida na audiência, podendo a autoridade judiciária, nutenção da criança ou do adolescente na família natural
excepcionalmente, designar data para sua leitura no prazo ou extensa. 
máximo de cinco dias. § 4o  O consentimento prestado por escrito não terá
validade se não for ratificado na audiência a que se refere
Art. 163.  O prazo máximo para conclusão do procedi- o § 3o deste artigo. 
mento será de 120 (cento e vinte) dias.  § 5o  O consentimento é retratável até a data da publi-
Parágrafo único.  A sentença que decretar a perda ou cação da sentença constitutiva da adoção. 
a suspensão do poder familiar será averbada à margem do § 6o  O consentimento somente terá valor se for dado
registro de nascimento da criança ou do adolescente.  após o nascimento da criança. 
§ 7o  A família substituta receberá a devida orientação
Seção III por intermédio de equipe técnica interprofissional a serviço
Da Destituição da Tutela do Poder Judiciário, preferencialmente com apoio dos téc-
nicos responsáveis pela execução da política municipal de
Art. 164. Na destituição da tutela, observar-se-á o pro- garantia do direito à convivência familiar. 
cedimento para a remoção de tutor previsto na lei proces-
sual civil e, no que couber, o disposto na seção anterior. Art. 167. A autoridade judiciária, de ofício ou a reque-
rimento das partes ou do Ministério Público, determinará a
A destituição da tutela pode, assim, ser decretada ju- realização de estudo social ou, se possível, perícia por equi-
dicialmente, em procedimento contraditório, nos casos pe interprofissional, decidindo sobre a concessão de guarda
previstos na legislação civil, bem como na hipótese de des- provisória, bem como, no caso de adoção, sobre o estágio de
cumprimento injustificado dos deveres e obrigações. convivência.

79
LEGISLAÇÃO

Parágrafo único.  Deferida a concessão da guarda pro- Art. 174. Comparecendo qualquer dos pais ou responsá-
visória ou do estágio de convivência, a criança ou o ado- vel, o adolescente será prontamente liberado pela autorida-
lescente será entregue ao interessado, mediante termo de de policial, sob termo de compromisso e responsabilidade de
responsabilidade.   sua apresentação ao representante do Ministério Público, no
mesmo dia ou, sendo impossível, no primeiro dia útil ime-
Art. 168. Apresentado o relatório social ou o laudo pe- diato, exceto quando, pela gravidade do ato infracional e
ricial, e ouvida, sempre que possível, a criança ou o adoles- sua repercussão social, deva o adolescente permanecer sob
cente, dar-se-á vista dos autos ao Ministério Público, pelo internação para garantia de sua segurança pessoal ou ma-
prazo de cinco dias, decidindo a autoridade judiciária em nutenção da ordem pública.
igual prazo.
Art. 175. Em caso de não liberação, a autoridade poli-
Art. 169. Nas hipóteses em que a destituição da tutela, a cial encaminhará, desde logo, o adolescente ao representan-
perda ou a suspensão do poder familiar constituir pressupos- te do Ministério Público, juntamente com cópia do auto de
to lógico da medida principal de colocação em família subs- apreensão ou boletim de ocorrência.
tituta, será observado o procedimento contraditório previsto § 1º Sendo impossível a apresentação imediata, a au-
nas Seções II e III deste Capítulo.   toridade policial encaminhará o adolescente à entidade de
Parágrafo único. A perda ou a modificação da guarda atendimento, que fará a apresentação ao representante do
poderá ser decretada nos mesmos autos do procedimento, Ministério Público no prazo de vinte e quatro horas.
observado o disposto no art. 35. § 2º Nas localidades onde não houver entidade de
atendimento, a apresentação far-se-á pela autoridade po-
Art. 170. Concedida a guarda ou a tutela, observar-se-á licial. À falta de repartição policial especializada, o adoles-
o disposto no art. 32, e, quanto à adoção, o contido no art. cente aguardará a apresentação em dependência separada
47. da destinada a maiores, não podendo, em qualquer hipó-
Parágrafo único.  A colocação de criança ou adolescen- tese, exceder o prazo referido no parágrafo anterior.
te sob a guarda de pessoa inscrita em programa de acolhi-
mento familiar será comunicada pela autoridade judiciária à Art. 176. Sendo o adolescente liberado, a autoridade po-
licial encaminhará imediatamente ao representante do Mi-
entidade por este responsável no prazo máximo de 5 (cinco)
nistério Público cópia do auto de apreensão ou boletim de
dias.  
ocorrência.
Seção V
Art. 177. Se, afastada a hipótese de flagrante, houver in-
Da Apuração de Ato Infracional Atribuído a Ado-
dícios de participação de adolescente na prática de ato infra-
lescente
cional, a autoridade policial encaminhará ao representante
do Ministério Público relatório das investigações e demais
Art. 171. O adolescente apreendido por força de ordem documentos.
judicial será, desde logo, encaminhado à autoridade judiciá-
ria. Art. 178. O adolescente a quem se atribua autoria de
ato infracional não poderá ser conduzido ou transportado
Art. 172. O adolescente apreendido em flagrante de ato em compartimento fechado de veículo policial, em condições
infracional será, desde logo, encaminhado à autoridade po- atentatórias à sua dignidade, ou que impliquem risco à sua
licial competente. integridade física ou mental, sob pena de responsabilidade.
Parágrafo único. Havendo repartição policial especiali-
zada para atendimento de adolescente e em se tratando de Art. 179. Apresentado o adolescente, o representante
ato infracional praticado em co-autoria com maior, prevale- do Ministério Público, no mesmo dia e à vista do auto de
cerá a atribuição da repartição especializada, que, após as apreensão, boletim de ocorrência ou relatório policial, devi-
providências necessárias e conforme o caso, encaminhará o damente autuados pelo cartório judicial e com informação
adulto à repartição policial própria. sobre os antecedentes do adolescente, procederá imediata e
informalmente à sua oitiva e, em sendo possível, de seus pais
Art. 173. Em caso de flagrante de ato infracional come- ou responsável, vítima e testemunhas.
tido mediante violência ou grave ameaça a pessoa, a auto- Parágrafo único. Em caso de não apresentação, o repre-
ridade policial, sem prejuízo do disposto nos arts. 106, pará- sentante do Ministério Público notificará os pais ou respon-
grafo único, e 107, deverá: sável para apresentação do adolescente, podendo requisitar
I - lavrar auto de apreensão, ouvidos as testemunhas e o concurso das polícias civil e militar.
o adolescente;
II - apreender o produto e os instrumentos da infração; Art. 180. Adotadas as providências a que alude o artigo
III - requisitar os exames ou perícias necessários à com- anterior, o representante do Ministério Público poderá:
provação da materialidade e autoria da infração. I - promover o arquivamento dos autos;
Parágrafo único. Nas demais hipóteses de flagrante, II - conceder a remissão;
a lavratura do auto poderá ser substituída por boletim de III - representar à autoridade judiciária para aplicação
ocorrência circunstanciada. de medida sócio-educativa.

80
LEGISLAÇÃO

Art. 181. Promovido o arquivamento dos autos ou con- Art. 186. Comparecendo o adolescente, seus pais ou res-
cedida a remissão pelo representante do Ministério Público, ponsável, a autoridade judiciária procederá à oitiva dos mes-
mediante termo fundamentado, que conterá o resumo dos mos, podendo solicitar opinião de profissional qualificado.
fatos, os autos serão conclusos à autoridade judiciária para § 1º Se a autoridade judiciária entender adequada a
homologação. remissão, ouvirá o representante do Ministério Público,
§ 1º Homologado o arquivamento ou a remissão, a au- proferindo decisão.
toridade judiciária determinará, conforme o caso, o cum- § 2º Sendo o fato grave, passível de aplicação de me-
primento da medida. dida de internação ou colocação em regime de semi-liber-
§ 2º Discordando, a autoridade judiciária fará remessa dade, a autoridade judiciária, verificando que o adolescente
dos autos ao Procurador-Geral de Justiça, mediante des- não possui advogado constituído, nomeará defensor, desig-
pacho fundamentado, e este oferecerá representação, de- nando, desde logo, audiência em continuação, podendo de-
signará outro membro do Ministério Público para apresen- terminar a realização de diligências e estudo do caso.
tá-la, ou ratificará o arquivamento ou a remissão, que só § 3º O advogado constituído ou o defensor nomeado,
então estará a autoridade judiciária obrigada a homologar. no prazo de três dias contado da audiência de apresenta-
ção, oferecerá defesa prévia e rol de testemunhas.
Art. 182. Se, por qualquer razão, o representante do Mi- § 4º Na audiência em continuação, ouvidas as teste-
nistério Público não promover o arquivamento ou conceder munhas arroladas na representação e na defesa prévia,
a remissão, oferecerá representação à autoridade judiciária, cumpridas as diligências e juntado o relatório da equipe
propondo a instauração de procedimento para aplicação da interprofissional, será dada a palavra ao representante do
medida sócio-educativa que se afigurar a mais adequada. Ministério Público e ao defensor, sucessivamente, pelo tempo
§ 1º A representação será oferecida por petição, que de vinte minutos para cada um, prorrogável por mais dez, a
conterá o breve resumo dos fatos e a classificação do ato critério da autoridade judiciária, que em seguida proferirá
infracional e, quando necessário, o rol de testemunhas, po- decisão.
dendo ser deduzida oralmente, em sessão diária instalada
pela autoridade judiciária. Art. 187. Se o adolescente, devidamente notificado, não
§ 2º A representação independe de prova pré-consti- comparecer, injustificadamente à audiência de apresenta-
tuída da autoria e materialidade.
ção, a autoridade judiciária designará nova data, determi-
nando sua condução coercitiva.
Art. 183. O prazo máximo e improrrogável para a con-
clusão do procedimento, estando o adolescente internado
Art. 188. A remissão, como forma de extinção ou sus-
provisoriamente, será de quarenta e cinco dias.
pensão do processo, poderá ser aplicada em qualquer fase
do procedimento, antes da sentença.
Art. 184. Oferecida a representação, a autoridade judi-
ciária designará audiência de apresentação do adolescente,
decidindo, desde logo, sobre a decretação ou manutenção Art. 189. A autoridade judiciária não aplicará qualquer
da internação, observado o disposto no art. 108 e parágrafo. medida, desde que reconheça na sentença:
§ 1º O adolescente e seus pais ou responsável serão I - estar provada a inexistência do fato;
cientificados do teor da representação, e notificados a II - não haver prova da existência do fato;
comparecer à audiência, acompanhados de advogado. III - não constituir o fato ato infracional;
§ 2º Se os pais ou responsável não forem localizados, a IV - não existir prova de ter o adolescente concorrido
autoridade judiciária dará curador especial ao adolescente. para o ato infracional.
§ 3º Não sendo localizado o adolescente, a autoridade Parágrafo único. Na hipótese deste artigo, estando o
judiciária expedirá mandado de busca e apreensão, deter- adolescente internado, será imediatamente colocado em li-
minando o sobrestamento do feito, até a efetiva apresen- berdade.
tação.
§ 4º Estando o adolescente internado, será requisitada Art. 190. A intimação da sentença que aplicar medida de
a sua apresentação, sem prejuízo da notificação dos pais internação ou regime de semi-liberdade será feita:
ou responsável. I - ao adolescente e ao seu defensor;
II - quando não for encontrado o adolescente, a seus pais
Art. 185. A internação, decretada ou mantida pela au- ou responsável, sem prejuízo do defensor.
toridade judiciária, não poderá ser cumprida em estabeleci- § 1º Sendo outra a medida aplicada, a intimação far-
mento prisional. se-á unicamente na pessoa do defensor.
§ 1º Inexistindo na comarca entidade com as carac- § 2º Recaindo a intimação na pessoa do adolescente,
terísticas definidas no art. 123, o adolescente deverá ser deverá este manifestar se deseja ou não recorrer da sen-
imediatamente transferido para a localidade mais próxima. tença.
§ 2º Sendo impossível a pronta transferência, o adoles-
cente aguardará sua remoção em repartição policial, desde
que em seção isolada dos adultos e com instalações apro-
priadas, não podendo ultrapassar o prazo máximo de cinco
dias, sob pena de responsabilidade.

81
LEGISLAÇÃO

Seção VI III - por via postal, com aviso de recebimento, se não for
Da Apuração de Irregularidades em Entidade de encontrado o requerido ou seu representante legal;
Atendimento IV - por edital, com prazo de trinta dias, se incerto ou
não sabido o paradeiro do requerido ou de seu representante
Art. 191. O procedimento de apuração de irregularida- legal.
des em entidade governamental e não-governamental terá
início mediante portaria da autoridade judiciária ou repre- Art. 196. Não sendo apresentada a defesa no prazo le-
sentação do Ministério Público ou do Conselho Tutelar, onde gal, a autoridade judiciária dará vista dos autos do Ministé-
conste, necessariamente, resumo dos fatos. rio Público, por cinco dias, decidindo em igual prazo.
Parágrafo único. Havendo motivo grave, poderá a auto-
ridade judiciária, ouvido o Ministério Público, decretar limi- Art. 197. Apresentada a defesa, a autoridade judiciária
narmente o afastamento provisório do dirigente da entidade, procederá na conformidade do artigo anterior, ou, sendo ne-
mediante decisão fundamentada. cessário, designará audiência de instrução e julgamento.
Parágrafo único. Colhida a prova oral, manifestar-se-ão
Art. 192. O dirigente da entidade será citado para, no
sucessivamente o Ministério Público e o procurador do re-
prazo de dez dias, oferecer resposta escrita, podendo juntar
querido, pelo tempo de vinte minutos para cada um, prorro-
documentos e indicar as provas a produzir.
gável por mais dez, a critério da autoridade judiciária, que
Art. 193. Apresentada ou não a resposta, e sendo neces- em seguida proferirá sentença.
sário, a autoridade judiciária designará audiência de instru-
ção e julgamento, intimando as partes. Seção VIII
§ 1º Salvo manifestação em audiência, as partes e o Da Habilitação de Pretendentes à Adoção
Ministério Público terão cinco dias para oferecer alegações
finais, decidindo a autoridade judiciária em igual prazo. Art. 197-A.  Os postulantes à adoção, domiciliados no
§ 2º Em se tratando de afastamento provisório ou de- Brasil, apresentarão petição inicial na qual conste: 
finitivo de dirigente de entidade governamental, a auto- I - qualificação completa; 
ridade judiciária oficiará à autoridade administrativa ime- II - dados familiares; 
diatamente superior ao afastado, marcando prazo para a III - cópias autenticadas de certidão de nascimento ou
substituição. casamento, ou declaração relativa ao período de união es-
§ 3º Antes de aplicar qualquer das medidas, a autorida- tável; 
de judiciária poderá fixar prazo para a remoção das irregu- IV - cópias da cédula de identidade e inscrição no Ca-
laridades verificadas. Satisfeitas as exigências, o processo dastro de Pessoas Físicas; 
será extinto, sem julgamento de mérito. V - comprovante de renda e domicílio; 
§ 4º A multa e a advertência serão impostas ao dirigen- VI - atestados de sanidade física e mental; 
te da entidade ou programa de atendimento. VII - certidão de antecedentes criminais; 
VIII - certidão negativa de distribuição cível. 
Seção VII
Da Apuração de Infração Administrativa às Nor- Art. 197-B.  A autoridade judiciária, no prazo de 48
mas de Proteção à Criança e ao Adolescente (quarenta e oito) horas, dará vista dos autos ao Ministério
Público, que no prazo de 5 (cinco) dias poderá: 
Art. 194. O procedimento para imposição de penalidade I - apresentar quesitos a serem respondidos pela equipe
administrativa por infração às normas de proteção à criança
interprofissional encarregada de elaborar o estudo técnico a
e ao adolescente terá início por representação do Ministério
que se refere o art. 197-C desta Lei; 
Público, ou do Conselho Tutelar, ou auto de infração elabo-
II - requerer a designação de audiência para oitiva dos
rado por servidor efetivo ou voluntário credenciado, e assi-
nado por duas testemunhas, se possível. postulantes em juízo e testemunhas; 
§ 1º No procedimento iniciado com o auto de infração, III - requerer a juntada de documentos complementares
poderão ser usadas fórmulas impressas, especificando-se a e a realização de outras diligências que entender necessá-
natureza e as circunstâncias da infração. rias. 
§ 2º Sempre que possível, à verificação da infração se-
guir-se-á a lavratura do auto, certificando-se, em caso con- Art. 197-C.  Intervirá no feito, obrigatoriamente, equi-
trário, dos motivos do retardamento. pe interprofissional a serviço da Justiça da Infância e da
Juventude, que deverá elaborar estudo psicossocial, que con-
Art. 195. O requerido terá prazo de dez dias para apre- terá subsídios que permitam aferir a capacidade e o prepa-
sentação de defesa, contado da data da intimação, que será ro dos postulantes para o exercício de uma paternidade ou
feita: maternidade responsável, à luz dos requisitos e princípios
I - pelo autuante, no próprio auto, quando este for lavra- desta Lei. 
do na presença do requerido; § 1o  É obrigatória a participação dos postulantes em
II - por oficial de justiça ou funcionário legalmente habi- programa oferecido pela Justiça da Infância e da Juventu-
litado, que entregará cópia do auto ou da representação ao de preferencialmente com apoio dos técnicos responsáveis
requerido, ou a seu representante legal, lavrando certidão; pela execução da política municipal de garantia do direito

82
LEGISLAÇÃO

à convivência familiar, que inclua preparação psicológica, VII - antes de determinar a remessa dos autos à superior
orientação e estímulo à adoção inter-racial, de crianças instância, no caso de apelação, ou do instrumento, no caso
maiores ou de adolescentes, com necessidades específicas de agravo, a autoridade judiciária proferirá despacho funda-
de saúde ou com deficiências e de grupos de irmãos.  mentado, mantendo ou reformando a decisão, no prazo de
§ 2o  Sempre que possível e recomendável, a etapa cinco dias;
obrigatória da preparação referida no § 1o deste artigo in- VIII - mantida a decisão apelada ou agravada, o escrivão
cluirá o contato com crianças e adolescentes em regime remeterá os autos ou o instrumento à superior instância den-
de acolhimento familiar ou institucional em condições de tro de vinte e quatro horas, independentemente de novo pe-
serem adotados, a ser realizado sob a orientação, supervi- dido do recorrente; se a reformar, a remessa dos autos depen-
derá de pedido expresso da parte interessada ou do Ministé-
são e avaliação da equipe técnica da Justiça da Infância e
rio Público, no prazo de cinco dias, contados da intimação.
da Juventude, com o apoio dos técnicos responsáveis pelo
programa de acolhimento familiar ou institucional e pela Art. 199. Contra as decisões proferidas com base no art.
execução da política municipal de garantia do direito à 149 caberá recurso de apelação.
convivência familiar.
Art. 197-D.  Certificada nos autos a conclusão da par- Art. 199-A.  A sentença que deferir a adoção produz efei-
ticipação no programa referido no art. 197-C desta Lei, a to desde logo, embora sujeita a apelação, que será recebida
autoridade judiciária, no prazo de 48 (quarenta e oito) ho- exclusivamente no efeito devolutivo, salvo se se tratar de ado-
ras, decidirá acerca das diligências requeridas pelo Ministé- ção internacional ou se houver perigo de dano irreparável ou
rio Público e determinará a juntada do estudo psicossocial, de difícil reparação ao adotando. 
designando, conforme o caso, audiência de instrução e jul-
gamento.  Art. 199-B.  A sentença que destituir ambos ou qualquer
Parágrafo único.  Caso não sejam requeridas diligências, dos genitores do poder familiar fica sujeita a apelação, que
ou sendo essas indeferidas, a autoridade judiciária determi- deverá ser recebida apenas no efeito devolutivo. 
nará a juntada do estudo psicossocial, abrindo a seguir vista
dos autos ao Ministério Público, por 5 (cinco) dias, decidindo Art. 199-C.  Os recursos nos procedimentos de adoção e
em igual prazo.  de destituição de poder familiar, em face da relevância das
questões, serão processados com prioridade absoluta, de-
vendo ser imediatamente distribuídos, ficando vedado que
Art. 197-E.  Deferida a habilitação, o postulante será ins-
aguardem, em qualquer situação, oportuna distribuição, e
crito nos cadastros referidos no art. 50 desta Lei, sendo a
serão colocados em mesa para julgamento sem revisão e com
sua convocação para a adoção feita de acordo com ordem parecer urgente do Ministério Público.  
cronológica de habilitação e conforme a disponibilidade de
crianças ou adolescentes adotáveis.  Art. 199-D.  O relator deverá colocar o processo em mesa
§ 1o  A ordem cronológica das habilitações somente para julgamento no prazo máximo de 60 (sessenta) dias, con-
poderá deixar de ser observada pela autoridade judiciária tado da sua conclusão. 
nas hipóteses previstas no § 13 do art. 50 desta Lei, quan- Parágrafo único.  O Ministério Público será intimado da
do comprovado ser essa a melhor solução no interesse do data do julgamento e poderá na sessão, se entender necessá-
adotando.  rio, apresentar oralmente seu parecer. 
§ 2o  A recusa sistemática na adoção das crianças ou
adolescentes indicados importará na reavaliação da habili- Art. 199-E.  O Ministério Público poderá requerer a ins-
tação concedida.  tauração de procedimento para apuração de responsabili-
dades se constatar o descumprimento das providências e do
Capítulo IV prazo previstos nos artigos anteriores.  
Dos Recursos
Capítulo V
Do Ministério Público
Art. 198.  Nos procedimentos afetos à Justiça da Infância
e da Juventude, inclusive os relativos à execução das me-
Art. 200. As funções do Ministério Público previstas nesta
didas socioeducativas, adotar-se-á o sistema recursal da Lei Lei serão exercidas nos termos da respectiva lei orgânica.
no 5.869, de 11 de janeiro de 1973 (Código de Processo Ci-
vil), com as seguintes adaptações: (Redação dada pela Lei nº Art. 201. Compete ao Ministério Público:
12.594, de 2012) I - conceder a remissão como forma de exclusão do pro-
I - os recursos serão interpostos independentemente de cesso;
preparo; II - promover e acompanhar os procedimentos relativos
II - em todos os recursos, salvo nos embargos de decla- às infrações atribuídas a adolescentes;
ração, o prazo para o Ministério Público e para a defesa será III - promover e acompanhar as ações de alimentos e os
sempre de 10 (dez) dias; (Redação dada pela Lei nº 12.594, procedimentos de suspensão e destituição do poder familiar,
de 2012) nomeação e remoção de tutores, curadores e guardiães, bem
III - os recursos terão preferência de julgamento e dis- como oficiar em todos os demais procedimentos da compe-
pensarão revisor; tência da Justiça da Infância e da Juventude; 

83
LEGISLAÇÃO

IV - promover, de ofício ou por solicitação dos interes- a) reduzir a termo as declarações do reclamante, instau-
sados, a especialização e a inscrição de hipoteca legal e a rando o competente procedimento, sob sua presidência;
prestação de contas dos tutores, curadores e quaisquer ad- b) entender-se diretamente com a pessoa ou autoridade
ministradores de bens de crianças e adolescentes nas hipó- reclamada, em dia, local e horário previamente notificados ou
teses do art. 98; acertados;
V - promover o inquérito civil e a ação civil pública para c) efetuar recomendações visando à melhoria dos serviços
a proteção dos interesses individuais, difusos ou coletivos re- públicos e de relevância pública afetos à criança e ao adoles-
lativos à infância e à adolescência, inclusive os definidos no cente, fixando prazo razoável para sua perfeita adequação.
art. 220, § 3º inciso II, da Constituição Federal;
VI - instaurar procedimentos administrativos e, para ins- Art. 202. Nos processos e procedimentos em que não for
truí-los: parte, atuará obrigatoriamente o Ministério Público na defesa
a) expedir notificações para colher depoimentos ou es- dos direitos e interesses de que cuida esta Lei, hipótese em
clarecimentos e, em caso de não comparecimento injusti- que terá vista dos autos depois das partes, podendo juntar do-
ficado, requisitar condução coercitiva, inclusive pela polícia cumentos e requerer diligências, usando os recursos cabíveis.
civil ou militar;
b) requisitar informações, exames, perícias e documen- Art. 203. A intimação do Ministério Público, em qualquer
tos de autoridades municipais, estaduais e federais, da admi- caso, será feita pessoalmente.
nistração direta ou indireta, bem como promover inspeções e
diligências investigatórias; Art. 204. A falta de intervenção do Ministério Público
c) requisitar informações e documentos a particulares e acarreta a nulidade do feito, que será declarada de ofício pelo
instituições privadas; juiz ou a requerimento de qualquer interessado.
VII - instaurar sindicâncias, requisitar diligências inves-
tigatórias e determinar a instauração de inquérito policial, Art. 205. As manifestações processuais do representante
para apuração de ilícitos ou infrações às normas de proteção do Ministério Público deverão ser fundamentadas.
à infância e à juventude;
VIII - zelar pelo efetivo respeito aos direitos e garantias Capítulo VI
legais assegurados às crianças e adolescentes, promovendo Do Advogado
as medidas judiciais e extrajudiciais cabíveis;
IX - impetrar mandado de segurança, de injunção e ha- Art. 206. A criança ou o adolescente, seus pais ou respon-
beas corpus, em qualquer juízo, instância ou tribunal, na de- sável, e qualquer pessoa que tenha legítimo interesse na so-
fesa dos interesses sociais e individuais indisponíveis afetos à lução da lide poderão intervir nos procedimentos de que trata
criança e ao adolescente; esta Lei, através de advogado, o qual será intimado para todos
X - representar ao juízo visando à aplicação de penali- os atos, pessoalmente ou por publicação oficial, respeitado o
dade por infrações cometidas contra as normas de proteção segredo de justiça.
à infância e à juventude, sem prejuízo da promoção da res- Parágrafo único. Será prestada assistência judiciária inte-
ponsabilidade civil e penal do infrator, quando cabível; gral e gratuita àqueles que dela necessitarem.
XI - inspecionar as entidades públicas e particulares de
atendimento e os programas de que trata esta Lei, adotando Art. 207. Nenhum adolescente a quem se atribua a prá-
de pronto as medidas administrativas ou judiciais necessá- tica de ato infracional, ainda que ausente ou foragido, será    
rias à remoção de irregularidades porventura verificadas; processado sem defensor.
XII - requisitar força policial, bem como a colaboração § 1º Se o adolescente não tiver defensor, ser-lhe-á no-
dos serviços médicos, hospitalares, educacionais e de assis- meado pelo juiz, ressalvado o direito de, a todo tempo,
tência social, públicos ou privados, para o desempenho de constituir outro de sua preferência.
suas atribuições. § 2º A ausência do defensor não determinará o adia-
§ 1º A legitimação do Ministério Público para as ações mento de nenhum ato do processo, devendo o juiz nomear
cíveis previstas neste artigo não impede a de terceiros, nas substituto, ainda que provisoriamente, ou para o só efeito
mesmas hipóteses, segundo dispuserem a Constituição e do ato.
esta Lei. § 3º Será dispensada a outorga de mandato, quando se
§ 2º As atribuições constantes deste artigo não ex- tratar de defensor nomeado ou, sido constituído, tiver sido
cluem outras, desde que compatíveis com a finalidade do indicado por ocasião de ato formal com a presença da au-
Ministério Público. toridade judiciária.
§ 3º O representante do Ministério Público, no exercí-
cio de suas funções, terá livre acesso a todo local onde se Capítulo VII
encontre criança ou adolescente. Da Proteção Judicial dos Interesses Individuais, Di-
§ 4º O representante do Ministério Público será res- fusos e Coletivos
ponsável pelo uso indevido das informações e documentos
que requisitar, nas hipóteses legais de sigilo. Art. 208. Regem-se pelas disposições desta Lei as ações
§ 5º Para o exercício da atribuição de que trata o inci- de responsabilidade por ofensa aos direitos assegurados à
so VIII deste artigo, poderá o representante do Ministério criança e ao adolescente, referentes ao não oferecimento ou
Público: oferta irregular:

84
LEGISLAÇÃO

I - do ensino obrigatório; Art. 211. Os órgãos públicos legitimados poderão tomar


II - de atendimento educacional especializado aos por- dos interessados compromisso de ajustamento de sua con-
tadores de deficiência; duta às exigências legais, o qual terá eficácia de título exe-
III - de atendimento em creche e pré-escola às crianças cutivo extrajudicial.
de zero a cinco anos de idade;
IV - de ensino noturno regular, adequado às condições Art. 212. Para defesa dos direitos e interesses protegidos
do educando; por esta Lei, são admissíveis todas as espécies de ações per-
V - de programas suplementares de oferta de material tinentes.
didático-escolar, transporte e assistência à saúde do educan- § 1º Aplicam-se às ações previstas neste Capítulo as
do do ensino fundamental; normas do Código de Processo Civil.
VI - de serviço de assistência social visando à proteção § 2º Contra atos ilegais ou abusivos de autoridade pú-
à família, à maternidade, à infância e à adolescência, bem blica ou agente de pessoa jurídica no exercício de atribui-
como ao amparo às crianças e adolescentes que dele neces- ções do poder público, que lesem direito líquido e certo
sitem; previsto nesta Lei, caberá ação mandamental, que se rege-
VII - de acesso às ações e serviços de saúde; rá pelas normas da lei do mandado de segurança.
VIII - de escolarização e profissionalização dos adoles-
centes privados de liberdade. Art. 213. Na ação que tenha por objeto o cumprimento
IX - de ações, serviços e programas de orientação, apoio de obrigação de fazer ou não fazer, o juiz concederá a tute-
e promoção social de famílias e destinados ao pleno exercício la específica da obrigação ou determinará providências que
do direito à convivência familiar por crianças e adolescentes.  assegurem o resultado prático equivalente ao do adimple-
X - de programas de atendimento para a execução das mento.
medidas socioeducativas e aplicação de medidas de prote- § 1º Sendo relevante o fundamento da demanda e ha-
ção. (Incluído pela Lei nº 12.594, de 2012)  vendo justificado receio de ineficácia do provimento final,
§ 1o As hipóteses previstas neste artigo não excluem da é lícito ao juiz conceder a tutela liminarmente ou após jus-
proteção judicial outros interesses individuais, difusos ou tificação prévia, citando o réu.
coletivos, próprios da infância e da adolescência, protegi- § 2º O juiz poderá, na hipótese do parágrafo anterior
dos pela Constituição e pela Lei.  ou na sentença, impor multa diária ao réu, independente-
§ 2o A investigação do desaparecimento de crianças mente de pedido do autor, se for suficiente ou compatível
ou adolescentes será realizada imediatamente após noti- com a obrigação, fixando prazo razoável para o cumpri-
ficação aos órgãos competentes, que deverão comunicar mento do preceito.
o fato aos portos, aeroportos, Polícia Rodoviária e compa- § 3º A multa só será exigível do réu após o trânsito em
nhias de transporte interestaduais e internacionais, forne- julgado da sentença favorável ao autor, mas será devida
cendo-lhes todos os dados necessários à identificação do desde o dia em que se houver configurado o descumpri-
desaparecido. mento.

Art. 209. As ações previstas neste Capítulo serão propos- Art. 214. Os valores das multas reverterão ao fundo ge-
tas no foro do local onde ocorreu ou deva ocorrer a ação ou rido pelo Conselho dos Direitos da Criança e do Adolescente
omissão, cujo juízo terá competência absoluta para proces- do respectivo município.
sar a causa, ressalvadas a competência da Justiça Federal e § 1º As multas não recolhidas até trinta dias após o
a competência originária dos tribunais superiores. trânsito em julgado da decisão serão exigidas através de
execução promovida pelo Ministério Público, nos mesmos
Art. 210. Para as ações cíveis fundadas em interesses autos, facultada igual iniciativa aos demais legitimados.
coletivos ou difusos, consideram-se legitimados concorren- § 2º Enquanto o fundo não for regulamentado, o di-
temente: nheiro ficará depositado em estabelecimento oficial de cré-
I - o Ministério Público; dito, em conta com correção monetária.
II - a União, os estados, os municípios, o Distrito Federal
e os territórios; Art. 215. O juiz poderá conferir efeito suspensivo aos re-
III - as associações legalmente constituídas há pelo me- cursos, para evitar dano irreparável à parte.
nos um ano e que incluam entre seus fins institucionais a
defesa dos interesses e direitos protegidos por esta Lei, dis- Art. 216. Transitada em julgado a sentença que impuser
pensada a autorização da assembleia, se houver prévia au- condenação ao poder público, o juiz determinará a remessa
torização estatutária. de peças à autoridade competente, para apuração da res-
§ 1º Admitir-se-á litisconsórcio facultativo entre os Mi- ponsabilidade civil e administrativa do agente a que se atri-
nistérios Públicos da União e dos estados na defesa dos bua a ação ou omissão.
interesses e direitos de que cuida esta Lei.
§ 2º Em caso de desistência ou abandono da ação por Art. 217. Decorridos sessenta dias do trânsito em julgado
associação legitimada, o Ministério Público ou outro legiti- da sentença condenatória sem que a associação autora lhe
mado poderá assumir a titularidade ativa. promova a execução, deverá fazê-lo o Ministério Público, fa-
cultada igual iniciativa aos demais legitimados.

85
LEGISLAÇÃO

Art. 218. O juiz condenará a associação autora a pagar Título VII


ao réu os honorários advocatícios arbitrados na conformi- Dos Crimes e Das Infrações Administrativas
dade do § 4º do art. 20 da Lei n.º 5.869, de 11 de janeiro de
1973 (Código de Processo Civil), quando reconhecer que a Capítulo I
pretensão é manifestamente infundada. Dos Crimes
Parágrafo único. Em caso de litigância de má-fé, a as-
sociação autora e os diretores responsáveis pela propositura Seção I
da ação serão solidariamente condenados ao décuplo das
custas, sem prejuízo de responsabilidade por perdas e danos. Disposições Gerais

Art. 219. Nas ações de que trata este Capítulo, não ha- Art. 225. Este Capítulo dispõe sobre crimes praticados
verá adiantamento de custas, emolumentos, honorários pe- contra a criança e o adolescente, por ação ou omissão, sem
riciais e quaisquer outras despesas. prejuízo do disposto na legislação penal.

Art. 220. Qualquer pessoa poderá e o servidor público Art. 226. Aplicam-se aos crimes definidos nesta Lei as
deverá provocar a iniciativa do Ministério Público, prestan- normas da Parte Geral do Código Penal e, quanto ao     pro-
do-lhe informações sobre fatos que constituam objeto de cesso, as pertinentes ao Código de Processo Penal.
ação civil, e indicando-lhe os elementos de convicção.
Art. 227. Os crimes definidos nesta Lei são de ação pú-
Art. 221. Se, no exercício de suas funções, os juízos e blica incondicionada.
tribunais tiverem conhecimento de fatos que possam ensejar
a propositura de ação civil, remeterão peças ao Ministério Seção II
Público para as providências cabíveis. Dos Crimes em Espécie

Art. 222. Para instruir a petição inicial, o interessado Art. 228. Deixar o encarregado de serviço ou o dirigente
poderá requerer às autoridades competentes as certidões e de estabelecimento de atenção à saúde de gestante de man-
informações que julgar necessárias, que serão fornecidas no ter registro das atividades desenvolvidas, na forma e prazo
prazo de quinze dias. referidos no art. 10 desta Lei, bem como de fornecer à par-
turiente ou a seu responsável, por ocasião da alta médica,
Art. 223. O Ministério Público poderá instaurar, sob sua declaração de nascimento, onde constem as intercorrências
presidência, inquérito civil, ou requisitar, de qualquer pes- do parto e do desenvolvimento do neonato:
soa, organismo público ou particular, certidões, informações, Pena - detenção de seis meses a dois anos.
exames ou perícias, no prazo que assinalar, o qual não pode- Parágrafo único. Se o crime é culposo:
rá ser inferior a dez dias úteis. Pena - detenção de dois a seis meses, ou multa.
§ 1º Se o órgão do Ministério Público, esgotadas todas
as diligências, se convencer da inexistência de fundamento Art. 229. Deixar o médico, enfermeiro ou dirigente de
para a propositura da ação cível, promoverá o arquivamen- estabelecimento de atenção à saúde de gestante de identifi-
to dos autos do inquérito civil ou das peças informativas, car corretamente o neonato e a parturiente, por ocasião do
fazendo-o fundamentadamente. parto, bem como deixar de proceder aos exames referidos no
§ 2º Os autos do inquérito civil ou as peças de informa- art. 10 desta Lei:
ção arquivados serão remetidos, sob pena de se incorrer Pena - detenção de seis meses a dois anos.
em falta grave, no prazo de três dias, ao Conselho Superior Parágrafo único. Se o crime é culposo:
do Ministério Público. Pena - detenção de dois a seis meses, ou multa.
§ 3º Até que seja homologada ou rejeitada a promoção
de arquivamento, em sessão do Conselho Superior do Mi- Art. 230. Privar a criança ou o adolescente de sua liber-
nistério público, poderão as associações legitimadas apre- dade, procedendo à sua apreensão sem estar em flagrante
sentar razões escritas ou documentos, que serão juntados de ato infracional ou inexistindo ordem escrita da autorida-
aos autos do inquérito ou anexados às peças de informa- de judiciária competente:
ção. Pena - detenção de seis meses a dois anos.
§ 4º A promoção de arquivamento será submetida a Parágrafo único. Incide na mesma pena aquele que pro-
exame e deliberação do Conselho Superior do Ministério cede à apreensão sem observância das formalidades legais.
Público, conforme dispuser o seu regimento.
§ 5º Deixando o Conselho Superior de homologar a Art. 231. Deixar a autoridade policial responsável pela
promoção de arquivamento, designará, desde logo, outro apreensão de criança ou adolescente de fazer imediata co-
órgão do Ministério Público para o ajuizamento da ação. municação à autoridade judiciária competente e à família
do apreendido ou à pessoa por ele indicada:
Art. 224. Aplicam-se subsidiariamente, no que couber, as Pena - detenção de seis meses a dois anos.
disposições da Lei n.º 7.347, de 24 de julho de 1985.

86
LEGISLAÇÃO

Art. 232. Submeter criança ou adolescente sob sua au- Art. 241.  Vender ou expor à venda fotografia, vídeo ou
toridade, guarda ou vigilância a vexame ou a constrangi- outro registro que contenha cena de sexo explícito ou porno-
mento: gráfica envolvendo criança ou adolescente: 
Pena - detenção de seis meses a dois anos. Pena – reclusão, de 4 (quatro) a 8 (oito) anos, e multa. 

Art. 234. Deixar a autoridade competente, sem justa Art. 241-A.  Oferecer, trocar, disponibilizar, transmitir,
causa, de ordenar a imediata liberação de criança ou ado- distribuir, publicar ou divulgar por qualquer meio, inclusive
lescente, tão logo tenha conhecimento da ilegalidade da por meio de sistema de informática ou telemático, fotogra-
apreensão: fia, vídeo ou outro registro que contenha cena de sexo ex-
Pena - detenção de seis meses a dois anos. plícito ou pornográfica envolvendo criança ou adolescente:  
Pena – reclusão, de 3 (três) a 6 (seis) anos, e multa. 
Art. 235. Descumprir, injustificadamente, prazo fixado § 1o  Nas mesmas penas incorre quem: 
nesta Lei em benefício de adolescente privado de liberdade: I – assegura os meios ou serviços para o armazenamento
Pena - detenção de seis meses a dois anos. das fotografias, cenas ou imagens de que trata o caput deste
artigo; 
Art. 236. Impedir ou embaraçar a ação de autoridade II – assegura, por qualquer meio, o acesso por rede de
judiciária, membro do Conselho Tutelar ou representante do computadores às fotografias, cenas ou imagens de que trata
Ministério Público no exercício de função prevista nesta Lei: o caput deste artigo.
Pena - detenção de seis meses a dois anos.  § 2o  As condutas tipificadas nos incisos I e II do §
1  deste artigo são puníveis quando o responsável legal
o
Art. 237. Subtrair criança ou adolescente ao poder de pela prestação do serviço, oficialmente notificado, deixa
quem o tem sob sua guarda em virtude de lei ou ordem judi- de desabilitar o acesso ao conteúdo ilícito de que trata
cial, com o fim de colocação em lar substituto: o caput deste artigo. 
Pena - reclusão de dois a seis anos, e multa.
 Art. 241-B.  Adquirir, possuir ou armazenar, por qual-
Art. 238. Prometer ou efetivar a entrega de filho ou pu- quer meio, fotografia, vídeo ou outra forma de registro que
pilo a terceiro, mediante paga ou recompensa: contenha cena de sexo explícito ou pornográfica envolven-
Pena - reclusão de um a quatro anos, e multa.
do criança ou adolescente: 
Parágrafo único. Incide nas mesmas penas quem oferece
 Pena – reclusão, de 1 (um) a 4 (quatro) anos, e multa. 
ou efetiva a paga ou recompensa.
 § 1o  A pena é diminuída de 1 (um) a 2/3 (dois ter-
ços) se de pequena quantidade o material a que se refere
Art. 239. Promover ou auxiliar a efetivação de ato des-
o caput deste artigo. 
tinado ao envio de criança ou adolescente para o exterior
 § 2o  Não há crime se a posse ou o armazenamento
com inobservância das formalidades legais ou com o fito de
tem a finalidade de comunicar às autoridades competentes
obter lucro:
Pena - reclusão de quatro a seis anos, e multa. a ocorrência das condutas descritas nos arts. 240, 241, 241-
Parágrafo único. Se há emprego de violência, grave A e 241-C desta Lei, quando a comunicação for feita por:
ameaça ou fraude:   I – agente público no exercício de suas funções; 
Pena - reclusão, de 6 (seis) a 8 (oito) anos, além da pena  II – membro de entidade, legalmente constituída, que
correspondente à violência. inclua, entre suas finalidades institucionais, o recebimento,
o processamento e o encaminhamento de notícia dos cri-
Art. 240.  Produzir, reproduzir, dirigir, fotografar, filmar mes referidos neste parágrafo; 
ou registrar, por qualquer meio, cena de sexo explícito ou  III – representante legal e funcionários responsáveis de
pornográfica, envolvendo criança ou adolescente:   provedor de acesso ou serviço prestado por meio de rede
Pena – reclusão, de 4 (quatro) a 8 (oito) anos, e multa.  de computadores, até o recebimento do material relativo à
§ 1o  Incorre nas mesmas penas quem agencia, faci- notícia feita à autoridade policial, ao Ministério Público ou
lita, recruta, coage, ou de qualquer modo intermedeia a ao Poder Judiciário. 
participação de criança ou adolescente nas cenas referidas  § 3o  As pessoas referidas no § 2o deste artigo deverão
no caputdeste artigo, ou ainda quem com esses contracena.   manter sob sigilo o material ilícito referido. 
§ 2o  Aumenta-se a pena de 1/3 (um terço) se o agente
comete o crime:   Art. 241-C.  Simular a participação de criança ou adoles-
I – no exercício de cargo ou função pública ou a pretexto cente em cena de sexo explícito ou pornográfica por meio de
de exercê-la;   adulteração, montagem ou modificação de fotografia, vídeo
II – prevalecendo-se de relações domésticas, de coabita- ou qualquer outra forma de representação visual: 
ção ou de hospitalidade; ou    Pena – reclusão, de 1 (um) a 3 (três) anos, e multa. 
III – prevalecendo-se de relações de parentesco consan- Parágrafo único.  Incorre nas mesmas penas quem ven-
guíneo ou afim até o terceiro grau, ou por adoção, de tutor, de, expõe à venda, disponibiliza, distribui, publica ou divulga
curador, preceptor, empregador da vítima ou de quem, a por qualquer meio, adquire, possui ou armazena o material
qualquer outro título, tenha autoridade sobre ela, ou com produzido na forma do caput deste artigo. 
seu consentimento.  

87
LEGISLAÇÃO

 Art. 241-D.  Aliciar, assediar, instigar ou constranger, § 2o  As penas previstas no caput deste artigo são au-
por qualquer meio de comunicação, criança, com o fim de mentadas de um terço no caso de a infração cometida ou
com ela praticar ato libidinoso:  induzida estar incluída no rol do art. 1o da Lei no 8.072, de
 Pena – reclusão, de 1 (um) a 3 (três) anos, e multa.  25 de julho de 1990. 
 Parágrafo único.  Nas mesmas penas incorre quem: 
 I – facilita ou induz o acesso à criança de material con- Capítulo II
tendo cena de sexo explícito ou pornográfica com o fim de Das Infrações Administrativas
com ela praticar ato libidinoso; 
 II – pratica as condutas descritas no caput deste artigo Art. 245. Deixar o médico, professor ou responsável por
com o fim de induzir criança a se exibir de forma pornográ- estabelecimento de atenção à saúde e de ensino fundamen-
fica ou sexualmente explícita.  tal, pré-escola ou creche, de comunicar à autoridade com-
petente os casos de que tenha conhecimento, envolvendo
 Art. 241-E.  Para efeito dos crimes previstos nesta Lei, suspeita ou confirmação de maus-tratos contra criança ou
a expressão “cena de sexo explícito ou pornográfica” com- adolescente:
preende qualquer situação que envolva criança ou adoles- Pena - multa de três a vinte salários de referência, apli-
cente em atividades sexuais explícitas, reais ou simuladas, cando-se o dobro em caso de reincidência.
ou exibição dos órgãos genitais de uma criança ou adoles-
cente para fins primordialmente sexuais.  Art. 246. Impedir o responsável ou funcionário de enti-
dade de atendimento o exercício dos direitos constantes nos
Art. 242. Vender, fornecer ainda que gratuitamente ou incisos II, III, VII, VIII e XI do art. 124 desta Lei:
entregar, de qualquer forma, a criança ou adolescente arma, Pena - multa de três a vinte salários de referência, apli-
munição ou explosivo: cando-se o dobro em caso de reincidência.
Pena - reclusão, de 3 (três) a 6 (seis) anos. 
Art. 247. Divulgar, total ou parcialmente, sem autoriza-
Art. 243. Vender, fornecer, servir, ministrar ou entregar, ção devida, por qualquer meio de comunicação, nome, ato
ainda que gratuitamente, de qualquer forma, a criança ou ou documento de procedimento policial, administrativo ou
a adolescente, bebida alcoólica ou, sem justa causa, outros judicial relativo a criança ou adolescente a que se atribua
produtos cujos componentes possam causar dependência fí- ato infracional:
sica ou psíquica: Pena - multa de três a vinte salários de referência, apli-
Pena - detenção de 2 (dois) a 4 (quatro) anos, e multa, se cando-se o dobro em caso de reincidência.
o fato não constitui crime mais grave. § 1º Incorre na mesma pena quem exibe, total ou par-
cialmente, fotografia de criança ou adolescente envolvido
Art. 244. Vender, fornecer ainda que gratuitamente ou em ato infracional, ou qualquer ilustração que lhe diga res-
entregar, de qualquer forma, a criança ou adolescente fogos peito ou se refira a atos que lhe sejam atribuídos, de forma
de estampido ou de artifício, exceto aqueles que, pelo seu a permitir sua identificação, direta ou indiretamente.
reduzido potencial, sejam incapazes de provocar qualquer § 2º Se o fato for praticado por órgão de imprensa ou
dano físico em caso de utilização indevida: emissora de rádio ou televisão, além da pena prevista neste
Pena - detenção de seis meses a dois anos, e multa. artigo, a autoridade judiciária poderá determinar a apreen-
são da publicação ou a suspensão da programação da emis-
Art. 244-A. Submeter criança ou adolescente, como tais sora até por dois dias, bem como da publicação do periódico
definidos no caput do art. 2o desta Lei, à prostituição ou à até por dois números. (Expressão declarada inconstitucional
exploração sexual:  pela ADIN 869-2).
Pena - reclusão de quatro a dez anos, e multa.
§ 1o Incorrem nas mesmas penas o proprietário, o ge- Art. 248. Deixar de apresentar à autoridade judiciária de
rente ou o responsável pelo local em que se verifique a seu domicílio, no prazo de cinco dias, com o fim de regula-
submissão de criança ou adolescente às práticas referidas rizar a guarda, adolescente trazido de outra comarca para a
no caputdeste artigo.  prestação de serviço doméstico, mesmo que autorizado pe-
§ 2o Constitui efeito obrigatório da condenação a cas- los pais ou responsável:
sação da licença de localização e de funcionamento do es- Pena - multa de três a vinte salários de referência, apli-
tabelecimento.  cando-se o dobro em caso de reincidência, independente-
mente das despesas de retorno do adolescente, se for o caso.
Art. 244-B.  Corromper ou facilitar a corrupção de menor
de 18 (dezoito) anos, com ele praticando infração penal ou Art. 249. Descumprir, dolosa ou culposamente, os deve-
induzindo-o a praticá-la:  res inerentes ao poder familiar  ou decorrente de tutela ou
Pena - reclusão, de 1 (um) a 4 (quatro) anos.  guarda, bem assim determinação da autoridade judiciária
§ 1o  Incorre nas penas previstas no caput deste arti- ou Conselho Tutelar: 
go quem pratica as condutas ali tipificadas utilizando-se de Pena - multa de três a vinte salários de referência, apli-
quaisquer meios eletrônicos, inclusive salas de bate-papo da cando-se o dobro em caso de reincidência.
internet. 

88
LEGISLAÇÃO

Art. 250.  Hospedar criança ou adolescente desacompa- Pena - multa de três a vinte salários de referência, du-
nhado dos pais ou responsável, ou sem autorização escrita plicando-se a pena em caso de reincidência, sem prejuízo de
desses ou da autoridade judiciária, em hotel, pensão, motel apreensão da revista ou publicação.
ou congênere: 
Pena – multa.  Art. 258. Deixar o responsável pelo estabelecimento ou o
§ 1º  Em caso de reincidência, sem prejuízo da pena de empresário de observar o que dispõe esta Lei sobre o acesso
multa, a autoridade judiciária poderá determinar o fecha- de criança ou adolescente aos locais de diversão, ou sobre
mento do estabelecimento por até 15 (quinze) dias.  sua participação no espetáculo:
§ 2º  Se comprovada a reincidência em período inferior Pena - multa de três a vinte salários de referência; em
a 30 (trinta) dias, o estabelecimento será definitivamente caso de reincidência, a autoridade judiciária poderá deter-
fechado e terá sua licença cassada.  minar o fechamento do estabelecimento por até quinze dias.

Art. 251. Transportar criança ou adolescente, por qual- Art. 258-A.  Deixar a autoridade competente de provi-
quer meio, com inobservância do disposto nos arts. 83, 84 e denciar a instalação e operacionalização dos cadastros pre-
85 desta Lei: vistos no art. 50 e no § 11 do art. 101 desta Lei: 
Pena - multa de três a vinte salários de referência, apli- Pena - multa de R$ 1.000,00 (mil reais) a R$ 3.000,00
cando-se o dobro em caso de reincidência. (três mil reais). 
Parágrafo único.  Incorre nas mesmas penas a autori-
Art. 252. Deixar o responsável por diversão ou espetá- dade que deixa de efetuar o cadastramento de crianças e de
culo público de afixar, em lugar visível e de fácil acesso, à adolescentes em condições de serem adotadas, de pessoas
entrada do local de exibição, informação destacada sobre a ou casais habilitados à adoção e de crianças e adolescentes
natureza da diversão ou espetáculo e a faixa etária especifi- em regime de acolhimento institucional ou familiar.  
cada no certificado de classificação:
Pena - multa de três a vinte salários de referência, apli- Art. 258-B.  Deixar o médico, enfermeiro ou dirigente de
cando-se o dobro em caso de reincidência. estabelecimento de atenção à saúde de gestante de efetuar
imediato encaminhamento à autoridade judiciária de caso
Art. 253. Anunciar peças teatrais, filmes ou quaisquer re- de que tenha conhecimento de mãe ou gestante interessada
presentações ou espetáculos, sem indicar os limites de idade em entregar seu filho para adoção:  
a que não se recomendem: Pena - multa de R$ 1.000,00 (mil reais) a R$ 3.000,00
Pena - multa de três a vinte salários de referência, du- (três mil reais). 
plicada em caso de reincidência, aplicável, separadamente, Parágrafo único.  Incorre na mesma pena o funcionário
à casa de espetáculo e aos órgãos de divulgação ou publi- de programa oficial ou comunitário destinado à garantia do
cidade. direito à convivência familiar que deixa de efetuar a comu-
nicação referida no caput deste artigo. 
Art. 254. Transmitir, através de rádio ou televisão, espe-
táculo em horário diverso do autorizado ou sem aviso de sua Art. 258-C. Descumprir a proibição estabelecida no in-
classificação: ciso II do art. 81:
Pena - multa de vinte a cem salários de referência; du- Pena - multa de R$ 3.000,00 (três mil reais) a R$
plicada em caso de reincidência a autoridade judiciária po- 10.000,00 (dez mil reais);
derá determinar a suspensão da programação da emissora Medida Administrativa - interdição do estabelecimento
por até dois dias. comercial até o recolhimento da multa aplicada.

Art. 255. Exibir filme, trailer, peça, amostra ou congêne- Disposições Finais e Transitórias
re classificado pelo órgão competente como inadequado às
crianças ou adolescentes admitidos ao espetáculo: Art. 259. A União, no prazo de noventa dias contados da
Pena - multa de vinte a cem salários de referência; na publicação deste Estatuto, elaborará projeto de lei dispondo
reincidência, a autoridade poderá determinar a suspensão sobre a criação ou adaptação de seus órgãos às diretrizes da
do espetáculo ou o fechamento do estabelecimento por até política de atendimento fixadas no art. 88 e ao que estabele-
quinze dias. ce o Título V do Livro II.
Parágrafo único. Compete aos estados e municípios pro-
Art. 256. Vender ou locar a criança ou adolescente fita moverem a adaptação de seus órgãos e programas às dire-
de programação em vídeo, em desacordo com a classificação trizes e princípios estabelecidos nesta Lei.
atribuída pelo órgão competente:
Pena - multa de três a vinte salários de referência; em Art. 260.  Os contribuintes poderão efetuar doações aos
caso de reincidência, a autoridade judiciária poderá deter- Fundos dos Direitos da Criança e do Adolescente nacional,
minar o fechamento do estabelecimento por até quinze dias. distrital, estaduais ou municipais, devidamente comprova-
das, sendo essas integralmente deduzidas do imposto de
Art. 257. Descumprir obrigação constante dos arts. 78 e renda, obedecidos os seguintes limites: 
79 desta Lei:

89
LEGISLAÇÃO

I - 1% (um por cento) do imposto sobre a renda devido § 3o  O pagamento da doação deve ser efetuado até a
apurado pelas pessoas jurídicas tributadas com base no lu- data de vencimento da primeira quota ou quota única do
cro real; e imposto, observadas instruções específicas da Secretaria
II - 6% (seis por cento) do imposto sobre a renda apu- da Receita Federal do Brasil.
rado pelas pessoas físicas na Declaração de Ajuste Anual, § 4o  O não pagamento da doação no prazo estabeleci-
observado o disposto no art. 22 da Lei no 9.532, de 10 de do no § 3o implica a glosa definitiva desta parcela de dedu-
dezembro de 1997. ção, ficando a pessoa física obrigada ao recolhimento da di-
§ 1º - (Revogado) ferença de imposto devido apurado na Declaração de Ajuste
§ 1o-A.  Na definição das prioridades a serem atendidas Anual com os acréscimos legais previstos na legislação. 
com os recursos captados pelos fundos nacional, estaduais § 5o  A pessoa física poderá deduzir do imposto apu-
e municipais dos direitos da criança e do adolescente, serão rado na Declaração de Ajuste Anual as doações feitas, no
consideradas as disposições do Plano Nacional de Promo- respectivo ano-calendário, aos fundos controlados pelos
ção, Proteção e Defesa do Direito de Crianças e Adolescentes Conselhos dos Direitos da Criança e do Adolescente mu-
à Convivência Familiar e Comunitária e as do Plano Nacio- nicipais, distrital, estaduais e nacional concomitantemente
nal pela Primeira Infância.   com a opção de que trata o caput, respeitado o limite pre-
§ 2o   Os conselhos nacional, estaduais e municipais visto no inciso II do art. 260.
dos direitos da criança e do adolescente fixarão critérios de
utilização, por meio de planos de aplicação, das dotações Art. 260-B.  A doação de que trata o inciso I do art. 260
subsidiadas e demais receitas, aplicando necessariamen- poderá ser deduzida:
te percentual para incentivo ao acolhimento, sob a forma I - do imposto devido no trimestre, para as pessoas jurí-
de guarda, de crianças e adolescentes e para programas de dicas que apuram o imposto trimestralmente; e
atenção integral à primeira infância em áreas de maior ca- II - do imposto devido mensalmente e no ajuste anual,
rência socioeconômica e em situações de calamidade.   para as pessoas jurídicas que apuram o imposto anualmen-
§ 3º O Departamento da Receita Federal, do Ministé- te. 
rio da Economia, Fazenda e Planejamento, regulamentará Parágrafo único.  A doação deverá ser efetuada dentro
a comprovação das doações feitas aos fundos, nos termos do período a que se refere a apuração do imposto.
deste artigo.
§ 4º O Ministério Público determinará em cada comar- Art. 260-C.  As doações de que trata o art. 260 desta Lei
ca a forma de fiscalização da aplicação, pelo Fundo Muni- podem ser efetuadas em espécie ou em bens. 
cipal dos Direitos da Criança e do Adolescente, dos incen- Parágrafo único.  As doações efetuadas em espécie de-
tivos fiscais referidos neste artigo. vem ser depositadas em conta específica, em instituição fi-
§ 5o  Observado o disposto no § 4o do art. 3o da Lei no nanceira pública, vinculadas aos respectivos fundos de que
9.249, de 26 de dezembro de 1995, a dedução de que trata trata o art. 260.
o inciso I do caput: 
I - será considerada isoladamente, não se submetendo a Art. 260-D.  Os órgãos responsáveis pela administração
limite em conjunto com outras deduções do imposto; e das contas dos Fundos dos Direitos da Criança e do Adoles-
II - não poderá ser computada como despesa operacio- cente nacional, estaduais, distrital e municipais devem emitir
nal na apuração do lucro real. recibo em favor do doador, assinado por pessoa competente
e pelo presidente do Conselho correspondente, especifican-
Art. 260-A.  A partir do exercício de 2010, ano-calen- do:
dário de 2009, a pessoa física poderá optar pela doação de I - número de ordem; 
que trata o inciso II do caput do art. 260 diretamente em sua II - nome, Cadastro Nacional da Pessoa Jurídica (CNPJ)
Declaração de Ajuste Anual.  e endereço do emitente; 
§ 1o  A doação de que trata o caput poderá ser deduzida III - nome, CNPJ ou Cadastro de Pessoas Físicas (CPF)
até os seguintes percentuais aplicados sobre o imposto apu- do doador;
rado na declaração: IV - data da doação e valor efetivamente recebido; e
I - (VETADO); V - ano-calendário a que se refere a doação.
II - (VETADO);  § 1o  O comprovante de que trata o caput deste artigo
III - 3% (três por cento) a partir do exercício de 2012. pode ser emitido anualmente, desde que discrimine os valo-
§ 2o  A dedução de que trata o caput: res doados mês a mês. 
I - está sujeita ao limite de 6% (seis por cento) do impos- § 2o  No caso de doação em bens, o comprovante
to sobre a renda apurado na declaração de que trata o inciso deve conter a identificação dos bens, mediante descrição
II do caput do art. 260; em campo próprio ou em relação anexa ao comprovante,
II - não se aplica à pessoa física que:  informando também se houve avaliação, o nome, CPF ou
a) utilizar o desconto simplificado; CNPJ e endereço dos avaliadores.
b) apresentar declaração em formulário; ou 
c) entregar a declaração fora do prazo; Art. 260-E.  Na hipótese da doação em bens, o doador
III - só se aplica às doações em espécie; e  deverá:
IV - não exclui ou reduz outros benefícios ou deduções I - comprovar a propriedade dos bens, mediante docu-
em vigor. mentação hábil;

90
LEGISLAÇÃO

II - baixar os bens doados na declaração de bens e direi- Parágrafo único.  O descumprimento do disposto nos
tos, quando se tratar de pessoa física, e na escrituração, no arts. 260-G e 260-I sujeitará os infratores a responder por
caso de pessoa jurídica; e ação judicial proposta pelo Ministério Público, que poderá
III - considerar como valor dos bens doados: atuar de ofício, a requerimento ou representação de qual-
a) para as pessoas físicas, o valor constante da última quer cidadão.
declaração do imposto de renda, desde que não exceda o
valor de mercado; Art. 260-K.  A Secretaria de Direitos Humanos da Pre-
b) para as pessoas jurídicas, o valor contábil dos bens. sidência da República (SDH/PR) encaminhará à Secretaria
Parágrafo único.  O preço obtido em caso de leilão não da Receita Federal do Brasil, até 31 de outubro de cada ano,
será considerado na determinação do valor dos bens doados, arquivo eletrônico contendo a relação atualizada dos Fundos
exceto se o leilão for determinado por autoridade judiciária. dos Direitos da Criança e do Adolescente nacional, distrital,
estaduais e municipais, com a indicação dos respectivos nú-
Art. 260-F.  Os documentos a que se referem os arts. meros de inscrição no CNPJ e das contas bancárias especí-
260-D e 260-E devem ser mantidos pelo contribuinte por um ficas mantidas em instituições financeiras públicas, destina-
prazo de 5 (cinco) anos para fins de comprovação da dedu- das exclusivamente a gerir os recursos dos Fundos.
ção perante a Receita Federal do Brasil.
Art. 260-L.  A Secretaria da Receita Federal do Brasil ex-
Art. 260-G.  Os órgãos responsáveis pela administração pedirá as instruções necessárias à aplicação do disposto nos
das contas dos Fundos dos Direitos da Criança e do Adoles- arts. 260 a 260-K.
cente nacional, estaduais, distrital e municipais devem:
I - manter conta bancária específica destinada exclusi- Art. 261. A falta dos conselhos municipais dos direitos da
vamente a gerir os recursos do Fundo;  criança e do adolescente, os registros, inscrições e alterações
II - manter controle das doações recebidas; e  a que se referem os arts. 90, parágrafo único, e 91 desta Lei
III - informar anualmente à Secretaria da Receita Fede- serão efetuados perante a autoridade judiciária da comarca
ral do Brasil as doações recebidas mês a mês, identificando a que pertencer a entidade.
os seguintes dados por doador: Parágrafo único. A União fica autorizada a repassar aos
a) nome, CNPJ ou CPF; estados e municípios, e os estados aos municípios, os recur-
b) valor doado, especificando se a doação foi em espécie sos referentes aos programas e atividades previstos nesta Lei,
tão logo estejam criados os conselhos dos direitos da criança
ou em bens.
e do adolescente nos seus respectivos níveis.
Art. 260-H.  Em caso de descumprimento das obrigações
Art. 262. Enquanto não instalados os Conselhos Tute-
previstas no art. 260-G, a Secretaria da Receita Federal do
lares, as atribuições a eles conferidas serão exercidas pela
Brasil dará conhecimento do fato ao Ministério Público.
autoridade judiciária.
Art. 260-I.  Os Conselhos dos Direitos da Criança e do
Art. 263. O Decreto-Lei nº 2.848, de 7 de dezembro de
Adolescente nacional, estaduais, distrital e municipais divul- 1940 (Código Penal), passa a vigorar com as seguintes alte-
garão amplamente à comunidade: rações:
I - o calendário de suas reuniões; 1) Art. 121 (...)
II - as ações prioritárias para aplicação das políticas de § 4º No homicídio culposo, a pena é aumentada de um
atendimento à criança e ao adolescente; terço, se o crime resulta de inobservância de regra técnica
III - os requisitos para a apresentação de projetos a se- de profissão, arte ou ofício, ou se o agente deixa de prestar
rem beneficiados com recursos dos Fundos dos Direitos da imediato socorro à vítima, não procura diminuir as conse-
Criança e do Adolescente nacional, estaduais, distrital ou quências do seu ato, ou foge para evitar prisão em flagrante.
municipais; Sendo doloso o homicídio, a pena é aumentada de um terço,
IV - a relação dos projetos aprovados em cada ano-ca- se o crime é praticado contra pessoa menor de catorze anos.
lendário e o valor dos recursos previstos para implementa- 2) Art. 129 (...)
ção das ações, por projeto; § 7º Aumenta-se a pena de um terço, se ocorrer qual-
V - o total dos recursos recebidos e a respectiva destina- quer das hipóteses do art. 121, § 4º.
ção, por projeto atendido, inclusive com cadastramento na § 8º Aplica-se à lesão culposa o disposto no § 5º do
base de dados do Sistema de Informações sobre a Infância e art. 121.
a Adolescência; e 3) Art. 136 (...)
VI - a avaliação dos resultados dos projetos beneficiados § 3º Aumenta-se a pena de um terço, se o crime é prati-
com recursos dos Fundos dos Direitos da Criança e do Ado- cado contra pessoa menor de catorze anos.
lescente nacional, estaduais, distrital e municipais. 4) Art. 213 (...)
Parágrafo único. Se a ofendida é menor de catorze anos:
Art. 260-J.  O Ministério Público determinará, em cada Pena - reclusão de quatro a dez anos.
Comarca, a forma de fiscalização da aplicação dos incenti- 5) Art. 214 (...)
vos fiscais referidos no art. 260 desta Lei. Parágrafo único. Se o ofendido é menor de catorze anos:
Pena - reclusão de três a nove anos.”

91
LEGISLAÇÃO

Art. 264. O art. 102 da Lei n.º 6.015, de 31 de dezembro e) punição severa ao abuso, à violência e à exploração
de 1973, fica acrescido do seguinte item: sexual da criança e do adolescente; e estímulo do Poder
“Art. 102 (...) Público, através de assistência jurídica, incentivos fiscais e
6º) a perda e a suspensão do pátrio poder.” subsídios, nos termos da lei, ao acolhimento, sob a forma
de guarda, de criança ou adolescente órfão ou abandona-
Art. 265. A Imprensa Nacional e demais gráficas da do.
União, da administração direta ou indireta, inclusive funda- R: A. O artigo 227, §3º, CF fixa os aspectos que abran-
ções instituídas e mantidas pelo poder público federal pro- gem a proteção especial da criança e do adolescente: “I -
moverão edição popular do texto integral deste Estatuto, que idade mínima de quatorze anos para admissão ao trabalho,
será posto à disposição das escolas e das entidades de aten- observado o disposto no art. 7º, XXXIII; II - garantia de di-
dimento e de defesa dos direitos da criança e do adolescente. reitos previdenciários e trabalhistas; III - garantia de acesso
do trabalhador adolescente e jovem à escola; IV - garantia
Art. 265-A.   O poder público fará periodicamente am- de pleno e formal conhecimento da atribuição de ato in-
pla divulgação dos direitos da criança e do adolescente nos fracional, igualdade na relação processual e defesa técnica
meios de comunicação social. por profissional habilitado, segundo dispuser a legislação
Parágrafo único.  A divulgação a que se refere o caput tutelar específica; V - obediência aos princípios de brevi-
será veiculada em linguagem clara, compreensível e ade- dade, excepcionalidade e respeito à condição peculiar de
quada a crianças e adolescentes, especialmente às crianças pessoa em desenvolvimento, quando da aplicação de qual-
com idade inferior a 6 (seis) anos. quer medida privativa da liberdade; VI - estímulo do Poder
Público, através de assistência jurídica, incentivos fiscais e
Art. 266. Esta Lei entra em vigor noventa dias após sua subsídios, nos termos da lei, ao acolhimento, sob a forma
publicação. de guarda, de criança ou adolescente órfão ou abandona-
Parágrafo único. Durante o período de vacância deverão do; VII - programas de prevenção e atendimento especiali-
ser promovidas atividades e campanhas de divulgação e es- zado à criança, ao adolescente e ao jovem dependente de
clarecimentos acerca do disposto nesta Lei. entorpecentes e drogas afins”.

Art. 267. Revogam-se as Leis nº 4.513, de 1964, e 6.697, 2. (Alternative Concursos/2017 - Prefeitura de Sul
de 10 de outubro de 1979 (Código de Menores), e as demais Brasil/SC - Agente Educativo) De acordo com o Estatu-
disposições em contrário. to da Criança e do Adolescente, Lei n.º 8.069/90, art. 60, é
proibido qualquer trabalho a menores:
Brasília, 13 de julho de 1990; 169º da Independência e a) De quatorze anos de idade, inclusive na condição de
102º da República. aprendiz.
b) De quatorze anos de idade, salvo na condição de
EXERCÍCIOS aprendiz.
c) De dezesseis anos de idade, salvo na condição de
1. (FCC/2014 - Prefeitura de Recife/PE - Procurador) aprendiz.
Nos termos do art. 226 da Constituição Federal, “a família, d) De dezesseis anos de idade, inclusive na condição
base da sociedade, tem especial proteção do Estado”. Entre de aprendiz.
os aspectos abrangidos pelo direito à proteção especial, e) De dezessete anos de idade, inclusive na condição
segundo o texto constitucional, encontram-se os seguin- de aprendiz.
tes: R: B. Em que pese o teor do art. 64 do ECA, que pode-
a) garantia de direitos previdenciários e trabalhistas; e ria dar a entender que um menor de 14 anos pode traba-
obediência aos princípios de brevidade, excepcionalidade lhar, prevalece o que diz o texto da Constituição Federal:
e respeito à condição peculiar de pessoa em desenvolvi- “Art. 7º São direitos dos trabalhadores urbanos e rurais,
mento, quando da aplicação de qualquer medida privativa além de outros que visem à melhoria de sua condição so-
da liberdade. cial: [...] XXXIII - proibição de trabalho noturno, perigoso ou
b) garantia de direitos previdenciários e trabalhistas; e insalubre a menores de dezoito e de qualquer trabalho a
acesso universal à educação infantil, em creche e pré-esco- menores de dezesseis anos, salvo na condição de aprendiz,
la, às crianças até 5 (cinco) anos de idade. a partir de quatorze anos”. Logo, o menor pode trabalhar
c) erradicação do analfabetismo; e estímulo do Poder em qualquer serviço, desde que não seja noturno, perigoso
Público, através de assistência jurídica, incentivos fiscais e e insalubre, dos 16 aos 18 anos; e entre 14 e 16 anos ape-
subsídios, nos termos da lei, ao acolhimento, sob a forma nas pode trabalhar como aprendiz.
de guarda, de criança ou adolescente órfão ou abandona-
do. 3. (FCC/2016 - AL-MS - Agente de Polícia Legislati-
d) punição severa ao abuso, à violência e à exploração vo) Sobre a adoção, nos termos preconizados pelo Estatu-
sexual da criança e do adolescente; e garantia às presidiá- to da Criança e do Adolescente,
rias de condições para que possam permanecer com seus a) o adotante deve ser, no mínimo, 18 anos mais velho
filhos durante o período de amamentação. que o adotando.
b) é permitida a adoção por procuração.

92
LEGISLAÇÃO

c) se um dos cônjuges adota o filho do outro, mantêm- 5. (COMPERVE/2016 - Câmara de Natal/RN - Guarda
se os vínculos de filiação entre o adotado e o cônjuge do Legislativo) As crianças e os adolescentes, qualificados pelo
adotante e os respectivos parentes. direito hoje vigente como pessoas em desenvolvimento, re-
d) é vedada a adoção conjunta pelos divorciados, se- ceberam do direito positivo brasileiro, tutela especial através
parados judicialmente e pelos ex-companheiros. da Lei nº 8.069, de 13 de julho de 1990, mais conhecida como
e) o estágio de convivência que precede a adoção não Estatuto da Criança e do Adolescente. Seguindo as diretrizes
poderá, em nenhuma hipótese, ser dispensado pela auto- traçadas pela Constituição de 1988, o Estatuto da Criança e
ridade judiciária. do Adolescente trouxe a previsão normativa da absoluta prio-
R: C. Neste sentido, disciplina o art. 41, § 1º, ECA: “Se ridade e de variados direitos fundamentais. Em tal seara, foi
um dos cônjuges ou concubinos adota o filho do outro, determinado que as crianças e os adolescentes têm direito,
mantêm-se os vínculos de filiação entre o adotado e o côn- a) à liberdade, de forma a compreender a liberdade de
juge ou concubino do adotante e os respectivos parentes”. ir, vir e estar nos logradouros públicos e espaços comunitá-
rios, ressalvadas as restrições legais; a liberdade de opinião e
A alternativa “a” está errada porque o adotante deve ser,
de expressão; a liberdade de brincar e de praticar esportes, a
pelo menos, 16 anos mais velho que o adotado e possuir
liberdade de participar da vida familiar e comunitária; a liber-
pelo menos 18 anos (art. 42, § 3º, ECA); a alternativa “b”
dade de buscar refúgio, auxílio e orientação, excetuadas dessa
está incorreta porque é vedada a adoção por procuração, tutela a liberdade de crença e culto religioso e de participar
pois a adoção é ato personalíssimo (art. 39, § 2º, ECA); a da vida política.
alternativa “d” está incorreta porque é possível a adoção b) ao respeito, consistente na inviolabilidade da sua inte-
conjunta desde que preencha os requisitos de serem casa- gridade física, psíquica e moral, abrangendo a preservação da
dos civilmente ou mantenham união estável, comprovada imagem, da identidade, da autonomia, de seus valores, ideias
a estabilidade da família (art. 42, § 1º, ECA); e a alternativa e crenças, excluída a tutela dos seus espaços e objetos pes-
“e” está incorreta porque pode ser dispensado o estágio de soais.
convivência quando o adotando já estiver sob a tutela ou c) de serem educados e cuidados sem o uso de castigo
guarda do adotante (art. 46, § 1º, ECA). físico ou de tratamento cruel ou degradante, como formas de
correção, disciplina, educação ou a qualquer outro pretexto,
4. (FCC/2016 - AL-MS - Agente de Polícia Legislati- por parte dos pais, de integrantes da família ampliada, dos
vo) Sobre a prática de ato infracional à luz do Estatuto da responsáveis, dos agentes públicos executores de medidas
Criança e do Adolescente, é INCORRETO afirmar que a socioeducativas ou por qualquer pessoa encarregada de cui-
a) medida socioeducativa de internação pode ser de- dar deles, tratá-los, educá-los ou protegê-los.
terminada por descumprimento reiterado e injustificável d) de serem criados e educados no seio de sua família
da medida anteriormente imposta. biológica, não se admitindo a sua inserção em família substi-
b) internação, antes da sentença, poderá ser determi- tuta, assegurada a convivência familiar e comunitária, em am-
nada pelo prazo máximo de quarenta e cinco dias. biente que garanta seu desenvolvimento integral.
c) medida socioeducativa de internação não pode- R: C. Nestes termos, preconiza o artigo 18-A do ECA: “A
rá exceder em nenhuma hipótese três anos, liberando-se criança e o adolescente têm o direito de ser educados e cui-
compulsoriamente o menor infrator aos vinte e um anos dados sem o uso de castigo físico ou de tratamento cruel ou
de idade. degradante, como formas de correção, disciplina, educação
d) medida socioeducativa de liberdade assistida será ou qualquer outro pretexto, pelos pais, pelos integrantes da
fixada pelo prazo mínimo de trinta dias, podendo a qual- família ampliada, pelos responsáveis, pelos agentes públi-
cos executores de medidas socioeducativas ou por qualquer
quer tempo ser prorrogada, revogada ou substituída por
pessoa encarregada de cuidar deles, tratá-los, educá-los ou
outra medida, ouvido o orientador, o Ministério Público e
protegê-los”. A alternativa “a” está errada porque o artigo 16
o defensor.
do ECA fixa que o direito à liberdade envolve os seguintes
e) remissão não implica necessariamente o reconhe- aspectos: “I - ir, vir e estar nos logradouros públicos e espa-
cimento ou comprovação da responsabilidade, nem pre- ços comunitários, ressalvadas as restrições legais; II - opinião e
valece para efeito de antecedentes, podendo incluir even- expressão; III - crença e culto religioso; IV - brincar, praticar
tualmente a aplicação de qualquer das medidas previstas esportes e divertir-se; V - participar da vida familiar e comu-
em lei, exceto a colocação em regime de semiliberdade e nitária, sem discriminação; VI - participar da vida política,
a internação. na forma da lei; VII - buscar refúgio, auxílio e orientação”. A
R: D. A lei exige como prazo mínimo de medida so- alternativa “b” está errada porque o artigo 17 do ECA prevê
cioeducativa o período de 6 meses, conforme art. 118, § 2º, que “o direito ao respeito consiste na inviolabilidade da inte-
ECA, não 30 dias conforme a alternativa “d”, razão pela qual gridade física, psíquica e moral da criança e do adolescente,
está incorreta. A alternativa “a” está prevista no art. 122, § abrangendo a preservação da imagem, da identidade, da au-
1º, ECA; a alternativa “b” está prevista no art. 108 do ECA; tonomia, dos valores, ideias e crenças, dos espaços e objetos
a alternativa “c” está prevista no art. 121, §§ 3º e 5º, ECA; a pessoais”. A alternativa “d” está errada porque o artigo 18 do
alternativa “e” está prevista no art. 127 ECA. ECA assegura que “é direito da criança e do adolescente ser
criado e educado no seio de sua família e, excepcionalmen-
te, em família substituta, assegurada a convivência familiar
e comunitária, em ambiente que garanta seu desenvolvi-
mento integral”.

93
LEGISLAÇÃO

6. (FUNRIO/2016 - IF-PA - Assistente de Alunos) 9. (FUNDAÇÃO CASA - Agente Administrativo - VU-


Segundo o Estatuto da Criança e do Adolescente (Lei NESP/2010) Relativamente às Disposições Preliminares do
8.069/90), é considerado criança Estatuto da Criança e do Adolescente, assinale a alternativa
a) a pessoa até seis anos incompletos de idade. correta.
b) a pessoa até oito anos incompletos de idade. a) Considera-se criança a pessoa com até doze anos
c) a pessoa até 12 anos incompletos de idade. completos, e adolescente aquela entre treze e dezoito anos
d) a pessoa até 18 anos incompletos de idade. de idade incompletos.
e) a pessoa até 14 anos incompletos, desde que não b) Nos casos em que a lei determinar, deverá ser cons-
tenha cometido nenhum crime. tantemente aplicado o Estatuto da Criança e do Adolescen-
R: C. O Estatuto da Criança e do Adolescente opta por te às pessoas entre dezenove e vinte anos de idade.
categorizar separadamente estas duas categorias de me- c) A garantia de prioridade para o adolescente com-
nores. Criança é aquele que tem até 12 anos de idade (na preende a primazia na formulação das políticas sociais pú-
data de aniversário de 12 anos, passa a ser adolescente), blicas para o lazer.
adolescente é aquele que tem entre 12 e 18 anos (na data
d) Na aplicação dessa Lei, deverão ser levados em con-
de aniversário de 18 anos, passa a ser maior), conforme o
ta os fins políticos a que ela se destina.
artigo 2º do ECA.
e) Destinação privilegiada de recursos públicos nas
7. (FUNRIO/2016 - IF-PA - Assistente de Alunos) A áreas relacionadas com a proteção à infância e à juventude.
intenção principal do Estatuto da Criança e do Adolescente R: E. Conforme o artigo 4º, parágrafo único, ECA, “a
(Lei nº 8.069/90) é garantia de prioridade compreende: [...] d) destinação pri-
a) prover uma boa escola para que crianças e adoles- vilegiada de recursos públicos nas áreas relacionadas com
centes possam trabalhar o mais cedo possível. a proteção à infância e à juventude”.
b) questionar políticas sociais que venham a proteger
quem não merece. 10. (Prefeitura de Cruzeiro - SP - Auxiliar de Desen-
c) distribuir renda entre os mais empobrecidos da po- volvimento Infantil - Instituto Excelência/2016) Assinale
pulação. a alternativa CORRETA conforme o artigo 15 do ECA:
d) proteger integralmente crianças e adolescentes, ga- a) na dignidade da criança e do adolescente, pondo-os
rantindo políticas públicas neste sentido. a salvo de qualquer tratamento desumano, violento, ater-
e) proteger crianças e adolescentes da prisão. rorizante, vexatório ou constrangedor.
R: D. Conforme o artigo 1º do ECA, “esta Lei dispõe b) no direito de ser educados e cuidados sem o uso de
sobre a proteção integral à criança e ao adolescente”. O castigo físico ou de tratamento cruel ou degradante, como
princípio da proteção integral se associa ao princípio da formas de correção, disciplina, educação ou qualquer outro
prioridade absoluta, colacionado no artigo 4º do ECA e no pretexto, pelos pais, pelos integrantes da família ampliada,
artigo 227, CF. De uma doutrina da situação irregular, o pelos responsáveis, pelos agentes públicos executores de
direito evoluiu e passou a contemplar uma noção de prote- medidas socioeducativas ou por qualquer pessoa encarre-
ção mais ampla da criança e do adolescente, que não ape- gada de cuidar deles, tratá-los, educá-los ou protegê-los.
nas abordasse situações de irregularidade (embora ainda c) no direito à liberdade, ao respeito e à dignidade
o fizesse), mas que abrangesse todo o arcabouço jurídico como pessoas humanas em processo de desenvolvimento
protetivo da criança e do adolescente, que é a doutrina da e como sujeitos de direitos civis, humanos e sociais garan-
proteção integral. tidos na Constituição e nas leis.
d) na inviolabilidade da integridade física, psíquica e
8. (Prefeitura de Cruzeiro - SP - Auxiliar de Desen-
moral da criança e do adolescente, abrangendo a preserva-
volvimento Infantil - Instituto Excelência/2016) O Esta-
ção da imagem, da identidade, da autonomia, dos valores,
tuto da Criança e do Adolescente em seu art. 4º, parágrafo
ideias e crenças, dos espaços e objetos pessoais.
único fixa a garantia de prioridade. Assinale a alternativa
CORRETA que compreende uma dessas prioridades: R: D. Dispõe o ECA em seu artigo 15: “A criança e o
a) Nenhuma criança ou adolescente será objeto de adolescente têm direito à liberdade, ao respeito e à digni-
qualquer forma de negligência, descriminação e explora- dade como pessoas humanas em processo de desenvolvi-
ção. mento e como sujeitos de direitos civis, humanos e sociais
b) É assegurado atendimento integral á saúde da crian- garantidos na Constituição e nas leis”.
ça e do adolescente por intermédio do sistema único de
saúde. 11. (TRT - 1ª REGIÃO - Juiz do Trabalho Substituto
c) Precedência de atendimento, nos serviços públicos - FCC/2016) Sobre o trabalho da criança e do adolescente,
ou de relevância pública. é correto afirmar:
d) Manter alojamento conjunto, possibilitando ao neo- a) É proibido o trabalho de adolescentes em atividades
nato a permanência junto á mãe. lúdicas.
R: C. Conforme o artigo 4º, parágrafo único, ECA, “a b) É proibido para os menores de 16, salvo na condição
garantia de prioridade compreende: [...] b) precedência de de aprendizes.
atendimento nos serviços públicos ou de relevância públi- c) É proibido o trabalho noturno de menores de 16
ca”. anos, salvo na condição de aprendizes.

94
LEGISLAÇÃO

d) É proibido o trabalho de adolescentes em hospitais,


salvo na condição de aprendizes de enfermagem.
e) É proibido o trabalho de crianças em peças teatrais e CÓDIGO CIVIL - Lei 10.406/2002
atividades cinematográficas. Arts: 1511 a 1638; 1694 a 1727 e 1728 a 1783.
R: B. Nos termos do artigo 60, ECA, “é proibido qual-
quer trabalho a menores de quatorze anos de idade, sal-
vo na condição de aprendiz”. Aceita-se o trabalho como
aprendiz entre 14 e 16 anos. A partir dos 16 anos, o adoles- ARTS: 1511 A 1638;
cente pode trabalhar, não necessariamente como aprendiz,
embora a lei fixe outras restrições. LIVRO IV
Do Direito de Família
12. (TRT - 1ª REGIÃO - Juiz do Trabalho Substituto - TÍTULO I
FCC/2016) A formação técnico-profissional do adolescen- Do Direito Pessoal
te NÃO deverá obedecer a SUBTÍTULO I
a) horário especial, estabelecido em lei. Do Casamento
b) horário especial, de acordo com a atividade. CAPÍTULO I
c) peculiaridades do seu desenvolvimento pessoal. Disposições Gerais
d) adequação ao mercado de trabalho.
e) prevalência das atividades educativas sobre as pro- Art. 1.511. O casamento estabelece comunhão plena
dutivas. de vida, com base na igualdade de direitos e deveres dos
R: A. Dispõe o ECA: “Art. 63. A formação técnico-pro- cônjuges.
fissional obedecerá aos seguintes princípios: [...] III - horário
especial para o exercício das atividades”. Especificamente, Art. 1.512. O casamento é civil e gratuita a sua cele-
o horário deve ser fixado sem prejudicar a frequência à es- bração.
Parágrafo único. A habilitação para o casamento, o re-
cola (artigo 67, IV, ECA) e é proibido o trabalho noturno.
gistro e a primeira certidão serão isentos de selos, emolu-
Entretanto, a lei não fixa com precisão o horário de traba-
mentos e custas, para as pessoas cuja pobreza for declara-
lho permitido ao adolescente.
da, sob as penas da lei.
13. (IDECAN/2016 - UFPB - Auxiliar em Assuntos
Art. 1.513. É defeso a qualquer pessoa, de direito pú-
Educacionais) Considerando a prática de ato infracional
blico ou privado, interferir na comunhão de vida instituída
por adolescentes e os direitos individuais assegurados, pela família.
nessa situação, pelo Estatuto da Criança e do Adolescente
(ECA), assinale a alternativa correta. Art. 1.514. O casamento se realiza no momento em que
a) Considera-se ato infracional a conduta descrita como o homem e a mulher manifestam, perante o juiz, a sua von-
crime e não a estabelecida como contravenção penal. tade de estabelecer vínculo conjugal, e o juiz os declara
b) O adolescente não pode ser privado de sua liberda- casados.
de senão em flagrante de ato infracional ou por ordem es-
crita e fundamentada da autoridade judiciária competente. Art. 1.515. O casamento religioso, que atender às exi-
c) O adolescente que comete ato infracional perde o gências da lei para a validade do casamento civil, equipara-
direito à identificação dos responsáveis pela sua apreen- se a este, desde que registrado no registro próprio, produ-
são, devendo, contudo, ser informado acerca de seus di- zindo efeitos a partir da data de sua celebração.
reitos.
d) O adolescente civilmente identificado será subme- Art. 1.516. O registro do casamento religioso submete-
tido à identificação compulsória pelos órgãos policiais, de se aos mesmos requisitos exigidos para o casamento civil.
proteção e judiciais, independente se para efeito de con- § 1o O registro civil do casamento religioso deverá ser
frontação em caso de dúvida fundada. promovido dentro de noventa dias de sua realização, me-
R: B. Conforme preconiza o art. 106, “nenhum adoles- diante comunicação do celebrante ao ofício competente,
cente será privado de sua liberdade senão em flagrante de ou por iniciativa de qualquer interessado, desde que haja
ato infracional ou por ordem escrita e fundamentada da sido homologada previamente a habilitação regulada nes-
autoridade judiciária competente”. te Código. Após o referido prazo, o registro dependerá de
nova habilitação.
§ 2o O casamento religioso, celebrado sem as formali-
dades exigidas neste Código, terá efeitos civis se, a requeri-
mento do casal, for registrado, a qualquer tempo, no regis-
tro civil, mediante prévia habilitação perante a autoridade
competente e observado o prazo do art. 1.532.
§ 3o Será nulo o registro civil do casamento religioso
se, antes dele, qualquer dos consorciados houver contraído
com outrem casamento civil.

95
LEGISLAÇÃO

CAPÍTULO II Parágrafo único. É permitido aos nubentes solicitar ao


Da Capacidade para o casamento juiz que não lhes sejam aplicadas as causas suspensivas
previstas nos incisos I, III e IV deste artigo, provando-se a
Art. 1.517. O homem e a mulher com dezesseis anos inexistência de prejuízo, respectivamente, para o herdeiro,
podem casar, exigindo-se autorização de ambos os pais, para o ex-cônjuge e para a pessoa tutelada ou curatelada;
ou de seus representantes legais, enquanto não atingida a no caso do inciso II, a nubente deverá provar nascimento
maioridade civil. de filho, ou inexistência de gravidez, na fluência do prazo.
Parágrafo único. Se houver divergência entre os pais,
aplica-se o disposto no parágrafo único do art. 1.631. Art. 1.524. As causas suspensivas da celebração do ca-
samento podem ser arguidas pelos parentes em linha reta
Art. 1.518. Até a celebração do casamento podem os de um dos nubentes, sejam consanguíneos ou afins, e pe-
pais ou tutores revogar a autorização. (Redação dada pela los colaterais em segundo grau, sejam também consanguí-
Lei nº 13.146, de 2015) (Vigência) neos ou afins.
Art. 1.519. A denegação do consentimento, quando in-
CAPÍTULO V
justa, pode ser suprida pelo juiz.
Do Processo de Habilitação para o casamento
Art. 1.520. Excepcionalmente, será permitido o ca-
samento de quem ainda não alcançou a idade núbil (art. Art. 1.525. O requerimento de habilitação para o ca-
1517), para evitar imposição ou cumprimento de pena cri- samento será firmado por ambos os nubentes, de próprio
minal ou em caso de gravidez. punho, ou, a seu pedido, por procurador, e deve ser instruí-
do com os seguintes documentos:
CAPÍTULO III I - certidão de nascimento ou documento equivalente;
Dos Impedimentos II - autorização por escrito das pessoas sob cuja de-
pendência legal estiverem, ou ato judicial que a supra;
Art. 1.521. Não podem casar: III - declaração de duas testemunhas maiores, parentes
I - os ascendentes com os descendentes, seja o paren- ou não, que atestem conhecê-los e afirmem não existir im-
tesco natural ou civil; pedimento que os iniba de casar;
II - os afins em linha reta; IV - declaração do estado civil, do domicílio e da resi-
III - o adotante com quem foi cônjuge do adotado e o dência atual dos contraentes e de seus pais, se forem co-
adotado com quem o foi do adotante; nhecidos;
IV - os irmãos, unilaterais ou bilaterais, e demais cola- V - certidão de óbito do cônjuge falecido, de senten-
terais, até o terceiro grau inclusive; ça declaratória de nulidade ou de anulação de casamento,
V - o adotado com o filho do adotante; transitada em julgado, ou do registro da sentença de di-
VI - as pessoas casadas; vórcio.
VII - o cônjuge sobrevivente com o condenado por ho-
micídio ou tentativa de homicídio contra o seu consorte. Art. 1.526.  A habilitação será feita pessoalmente pe-
rante o oficial do Registro Civil, com a audiência do Minis-
Art. 1.522. Os impedimentos podem ser opostos, até o tério Público.
momento da celebração do casamento, por qualquer pes- Parágrafo único.  Caso haja impugnação do oficial, do
soa capaz. Ministério Público ou de terceiro, a habilitação será sub-
Parágrafo único. Se o juiz, ou o oficial de registro, tiver
metida ao juiz.
conhecimento da existência de algum impedimento, será
obrigado a declará-lo.
Art. 1.527. Estando em ordem a documentação, o ofi-
CAPÍTULO IV cial extrairá o edital, que se afixará durante quinze dias nas
Das causas suspensivas circunscrições do Registro Civil de ambos os nubentes, e,
obrigatoriamente, se publicará na imprensa local, se hou-
Art. 1.523. Não devem casar: ver.
I - o viúvo ou a viúva que tiver filho do cônjuge faleci- Parágrafo único. A autoridade competente, havendo
do, enquanto não fizer inventário dos bens do casal e der urgência, poderá dispensar a publicação.
partilha aos herdeiros;
II - a viúva, ou a mulher cujo casamento se desfez por Art. 1.528. É dever do oficial do registro esclarecer os
ser nulo ou ter sido anulado, até dez meses depois do co- nubentes a respeito dos fatos que podem ocasionar a inva-
meço da viuvez, ou da dissolução da sociedade conjugal; lidade do casamento, bem como sobre os diversos regimes
III - o divorciado, enquanto não houver sido homolo- de bens.
gada ou decidida a partilha dos bens do casal;
IV - o tutor ou o curador e os seus descendentes, as- Art. 1.529. Tanto os impedimentos quanto as causas
cendentes, irmãos, cunhados ou sobrinhos, com a pessoa suspensivas serão opostos em declaração escrita e assi-
tutelada ou curatelada, enquanto não cessar a tutela ou nada, instruída com as provas do fato alegado, ou com a
curatela, e não estiverem saldadas as respectivas contas. indicação do lugar onde possam ser obtidas.

96
LEGISLAÇÃO

Art. 1.530. O oficial do registro dará aos nubentes ou a VII - o regime do casamento, com a declaração da data
seus representantes nota da oposição, indicando os funda- e do cartório em cujas notas foi lavrada a escritura ante-
mentos, as provas e o nome de quem a ofereceu. nupcial, quando o regime não for o da comunhão parcial,
Parágrafo único. Podem os nubentes requerer prazo ou o obrigatoriamente estabelecido.
razoável para fazer prova contrária aos fatos alegados, e
promover as ações civis e criminais contra o oponente de Art. 1.537. O instrumento da autorização para casar
má-fé. transcrever-se-á integralmente na escritura antenupcial.

Art. 1.531. Cumpridas as formalidades dos arts. 1.526 e Art. 1.538. A celebração do casamento será imediata-
1.527 e verificada a inexistência de fato obstativo, o oficial mente suspensa se algum dos contraentes:
do registro extrairá o certificado de habilitação. I - recusar a solene afirmação da sua vontade;
II - declarar que esta não é livre e espontânea;
Art. 1.532. A eficácia da habilitação será de noventa III - manifestar-se arrependido.
dias, a contar da data em que foi extraído o certificado. Parágrafo único. O nubente que, por algum dos fatos
mencionados neste artigo, der causa à suspensão do ato,
CAPÍTULO VI não será admitido a retratar-se no mesmo dia.
Da Celebração do Casamento
Art. 1.539. No caso de moléstia grave de um dos nu-
Art. 1.533. Celebrar-se-á o casamento, no dia, hora e bentes, o presidente do ato irá celebrá-lo onde se encon-
lugar previamente designados pela autoridade que houver trar o impedido, sendo urgente, ainda que à noite, perante
de presidir o ato, mediante petição dos contraentes, que se duas testemunhas que saibam ler e escrever.
mostrem habilitados com a certidão do art. 1.531. § 1o A falta ou impedimento da autoridade competen-
te para presidir o casamento suprir-se-á por qualquer dos
Art. 1.534. A solenidade realizar-se-á na sede do cartó- seus substitutos legais, e a do oficial do Registro Civil por
rio, com toda publicidade, a portas abertas, presentes pelo outro ad hoc, nomeado pelo presidente do ato.
menos duas testemunhas, parentes ou não dos contraen- § 2o O termo avulso, lavrado pelo oficial ad hoc, será
tes, ou, querendo as partes e consentindo a autoridade ce- registrado no respectivo registro dentro em cinco dias, pe-
lebrante, noutro edifício público ou particular. rante duas testemunhas, ficando arquivado.
§ 1o Quando o casamento for em edifício particular, fi-
cará este de portas abertas durante o ato. Art. 1.540. Quando algum dos contraentes estiver em
§ 2o Serão quatro as testemunhas na hipótese do pará- iminente risco de vida, não obtendo a presença da autori-
grafo anterior e se algum dos contraentes não souber ou dade à qual incumba presidir o ato, nem a de seu substitu-
não puder escrever. to, poderá o casamento ser celebrado na presença de seis
testemunhas, que com os nubentes não tenham parentes-
Art. 1.535. Presentes os contraentes, em pessoa ou por co em linha reta, ou, na colateral, até segundo grau.
procurador especial, juntamente com as testemunhas e o
oficial do registro, o presidente do ato, ouvida aos nuben- Art. 1.541. Realizado o casamento, devem as testemu-
tes a afirmação de que pretendem casar por livre e espon- nhas comparecer perante a autoridade judicial mais próxi-
tânea vontade, declarará efetuado o casamento, nestes ma, dentro em dez dias, pedindo que lhes tome por termo
termos: “De acordo com a vontade que ambos acabais de a declaração de:
afirmar perante mim, de vos receberdes por marido e mu- I - que foram convocadas por parte do enfermo;
lher, eu, em nome da lei, vos declaro casados.” II - que este parecia em perigo de vida, mas em seu
juízo;
Art. 1.536. Do casamento, logo depois de celebrado, la- III - que, em sua presença, declararam os contraentes,
vrar-se-á o assento no livro de registro. No assento, assina- livre e espontaneamente, receber-se por marido e mulher.
do pelo presidente do ato, pelos cônjuges, as testemunhas, § 1o Autuado o pedido e tomadas as declarações, o
e o oficial do registro, serão exarados: juiz procederá às diligências necessárias para verificar se
I - os prenomes, sobrenomes, datas de nascimento, os contraentes podiam ter-se habilitado, na forma ordiná-
profissão, domicílio e residência atual dos cônjuges; ria, ouvidos os interessados que o requererem, dentro em
II - os prenomes, sobrenomes, datas de nascimento ou quinze dias.
de morte, domicílio e residência atual dos pais; § 2o Verificada a idoneidade dos cônjuges para o ca-
III - o prenome e sobrenome do cônjuge precedente e samento, assim o decidirá a autoridade competente, com
a data da dissolução do casamento anterior; recurso voluntário às partes.
IV - a data da publicação dos proclamas e da celebra- § 3o Se da decisão não se tiver recorrido, ou se ela
ção do casamento; passar em julgado, apesar dos recursos interpostos, o juiz
V - a relação dos documentos apresentados ao oficial mandará registrá-la no livro do Registro dos Casamentos.
do registro; § 4o O assento assim lavrado retrotrairá os efeitos do
VI - o prenome, sobrenome, profissão, domicílio e resi- casamento, quanto ao estado dos cônjuges, à data da ce-
dência atual das testemunhas; lebração.

97
LEGISLAÇÃO

§ 5o Serão dispensadas as formalidades deste e do arti- I - (Revogado); (Redação dada pela Lei nº 13.146, de
go antecedente, se o enfermo convalescer e puder ratificar 2015) (Vigência)
o casamento na presença da autoridade competente e do II - por infringência de impedimento.
oficial do registro.
Art. 1.549. A decretação de nulidade de casamento, pe-
Art. 1.542. O casamento pode celebrar-se mediante los motivos previstos no artigo antecedente, pode ser pro-
procuração, por instrumento público, com poderes espe- movida mediante ação direta, por qualquer interessado, ou
ciais. pelo Ministério Público.
§ 1o A revogação do mandato não necessita chegar ao
conhecimento do mandatário; mas, celebrado o casamen- Art. 1.550. É anulável o casamento:
to sem que o mandatário ou o outro contraente tivessem I - de quem não completou a idade mínima para casar;
ciência da revogação, responderá o mandante por perdas II - do menor em idade núbil, quando não autorizado
e danos. por seu representante legal;
III - por vício da vontade, nos termos dos arts. 1.556 a
§ 2o O nubente que não estiver em iminente risco de
1.558;
vida poderá fazer-se representar no casamento nuncupa-
IV - do incapaz de consentir ou manifestar, de modo
tivo.
inequívoco, o consentimento;
§ 3o A eficácia do mandato não ultrapassará noventa V - realizado pelo mandatário, sem que ele ou o outro
dias. contraente soubesse da revogação do mandato, e não so-
§ 4o Só por instrumento público se poderá revogar o brevindo coabitação entre os cônjuges;
mandato. VI - por incompetência da autoridade celebrante.
§ 1º. Equipara-se à revogação a invalidade do mandato
CAPÍTULO VII judicialmente decretada. (Redação dada pela Lei nº 13.146,
Das Provas do Casamento de 2015) (Vigência)
§ 2º A pessoa com deficiência mental ou intelectual em
Art. 1.543. O casamento celebrado no Brasil prova-se idade núbia poderá contrair matrimônio, expressando sua
pela certidão do registro. vontade diretamente ou por meio de seu responsável ou
Parágrafo único. Justificada a falta ou perda do registro curador. (Incluído pela Lei nº 13.146, de 2015) (Vigência)
civil, é admissível qualquer outra espécie de prova.
Art. 1.551. Não se anulará, por motivo de idade, o casa-
Art. 1.544. O casamento de brasileiro, celebrado no es- mento de que resultou gravidez.
trangeiro, perante as respectivas autoridades ou os côn-
sules brasileiros, deverá ser registrado em cento e oitenta Art. 1.552. A anulação do casamento dos menores de
dias, a contar da volta de um ou de ambos os cônjuges dezesseis anos será requerida:
ao Brasil, no cartório do respectivo domicílio, ou, em sua I - pelo próprio cônjuge menor;
falta, no 1o Ofício da Capital do Estado em que passarem II - por seus representantes legais;
a residir. III - por seus ascendentes.

Art. 1.545. O casamento de pessoas que, na posse do Art. 1.553. O menor que não atingiu a idade núbil po-
estado de casadas, não possam manifestar vontade, ou te- derá, depois de completá-la, confirmar seu casamento,
nham falecido, não se pode contestar em prejuízo da pro- com a autorização de seus representantes legais, se neces-
sária, ou com suprimento judicial.
le comum, salvo mediante certidão do Registro Civil que
prove que já era casada alguma delas, quando contraiu o
Art. 1.554. Subsiste o casamento celebrado por aque-
casamento impugnado.
le que, sem possuir a competência exigida na lei, exercer
publicamente as funções de juiz de casamentos e, nessa
Art. 1.546. Quando a prova da celebração legal do ca- qualidade, tiver registrado o ato no Registro Civil.
samento resultar de processo judicial, o registro da senten-
ça no livro do Registro Civil produzirá, tanto no que toca Art. 1.555. O casamento do menor em idade núbil,
aos cônjuges como no que respeita aos filhos, todos os quando não autorizado por seu representante legal, só po-
efeitos civis desde a data do casamento. derá ser anulado se a ação for proposta em cento e oitenta
dias, por iniciativa do incapaz, ao deixar de sê-lo, de seus
Art. 1.547. Na dúvida entre as provas favoráveis e con- representantes legais ou de seus herdeiros necessários.
trárias, julgar-se-á pelo casamento, se os cônjuges, cujo ca- § 1o O prazo estabelecido neste artigo será contado
samento se impugna, viverem ou tiverem vivido na posse do dia em que cessou a incapacidade, no primeiro caso; a
do estado de casados. partir do casamento, no segundo; e, no terceiro, da morte
do incapaz.
CAPÍTULO VIII § 2o Não se anulará o casamento quando à sua cele-
Da Invalidade do Casamento bração houverem assistido os representantes legais do
incapaz, ou tiverem, por qualquer modo, manifestado sua
Art. 1.548. É nulo o casamento contraído: aprovação.

98
LEGISLAÇÃO

Art. 1.556. O casamento pode ser anulado por vício da Art. 1.562. Antes de mover a ação de nulidade do casa-
vontade, se houve por parte de um dos nubentes, ao con- mento, a de anulação, a de separação judicial, a de divórcio
sentir, erro essencial quanto à pessoa do outro. direto ou a de dissolução de união estável, poderá reque-
rer a parte, comprovando sua necessidade, a separação de
Art. 1.557. Considera-se erro essencial sobre a pessoa corpos, que será concedida pelo juiz com a possível brevi-
do outro cônjuge: dade.
I - o que diz respeito à sua identidade, sua honra e boa
fama, sendo esse erro tal que o seu conhecimento ulterior Art. 1.563. A sentença que decretar a nulidade do casa-
torne insuportável a vida em comum ao cônjuge enganado; mento retroagirá à data da sua celebração, sem prejudicar
II - a ignorância de crime, anterior ao casamento, que, a aquisição de direitos, a título oneroso, por terceiros de
por sua natureza, torne insuportável a vida conjugal; boa-fé, nem a resultante de sentença transitada em julga-
III - a ignorância, anterior ao casamento, de defeito fí- do.
sico irremediável que não caracterize deficiência ou de mo-
léstia grave e transmissível, por contágio ou por herança, Art. 1.564. Quando o casamento for anulado por culpa
capaz de pôr em risco a saúde do outro cônjuge ou de sua de um dos cônjuges, este incorrerá:
descendência; (Redação dada pela Lei nº 13.146, de 2015) I - na perda de todas as vantagens havidas do cônjuge
(Vigência) inocente;
IV - (Revogado). (Redação dada pela Lei nº 13.146, de II - na obrigação de cumprir as promessas que lhe fez
2015) (Vigência) no contrato antenupcial.

Art. 1.558. É anulável o casamento em virtude de coa- CAPÍTULO IX


ção, quando o consentimento de um ou de ambos os côn- Da Eficácia do Casamento
juges houver sido captado mediante fundado temor de
mal considerável e iminente para a vida, a saúde e a honra, Art. 1.565. Pelo casamento, homem e mulher assumem
sua ou de seus familiares. mutuamente a condição de consortes, companheiros e res-
ponsáveis pelos encargos da família.
Art. 1.559. Somente o cônjuge que incidiu em erro, ou § 1o Qualquer dos nubentes, querendo, poderá acres-
sofreu coação, pode demandar a anulação do casamento; cer ao seu o sobrenome do outro.
mas a coabitação, havendo ciência do vício, valida o ato, § 2o O planejamento familiar é de livre decisão do casal,
ressalvadas as hipóteses dos incisos III e IV do art. 1.557. competindo ao Estado propiciar recursos educacionais e
financeiros para o exercício desse direito, vedado qualquer
Art. 1.560. O prazo para ser intentada a ação de anu- tipo de coerção por parte de instituições privadas ou pú-
lação do casamento, a contar da data da celebração, é de: blicas.
I - cento e oitenta dias, no caso do inciso IV do art.
1.550; Art. 1.566. São deveres de ambos os cônjuges:
II - dois anos, se incompetente a autoridade celebrante; I - fidelidade recíproca;
III - três anos, nos casos dos incisos I a IV do art. 1.557; II - vida em comum, no domicílio conjugal;
IV - quatro anos, se houver coação. III - mútua assistência;
§ 1o Extingue-se, em cento e oitenta dias, o direito de IV - sustento, guarda e educação dos filhos;
anular o casamento dos menores de dezesseis anos, conta- V - respeito e consideração mútuos.
do o prazo para o menor do dia em que perfez essa idade;
e da data do casamento, para seus representantes legais Art. 1.567. A direção da sociedade conjugal será exerci-
ou ascendentes. da, em colaboração, pelo marido e pela mulher, sempre no
§ 2o Na hipótese do inciso V do art. 1.550, o prazo para interesse do casal e dos filhos.
anulação do casamento é de cento e oitenta dias, a partir Parágrafo único. Havendo divergência, qualquer dos
da data em que o mandante tiver conhecimento da cele- cônjuges poderá recorrer ao juiz, que decidirá tendo em
bração. consideração aqueles interesses.

Art. 1.561. Embora anulável ou mesmo nulo, se con- Art. 1.568. Os cônjuges são obrigados a concorrer, na
traído de boa-fé por ambos os cônjuges, o casamento, em proporção de seus bens e dos rendimentos do trabalho,
relação a estes como aos filhos, produz todos os efeitos até para o sustento da família e a educação dos filhos, qual-
o dia da sentença anulatória. quer que seja o regime patrimonial.
§ 1o Se um dos cônjuges estava de boa-fé ao celebrar o
casamento, os seus efeitos civis só a ele e aos filhos apro- Art. 1.569. O domicílio do casal será escolhido por
veitarão. ambos os cônjuges, mas um e outro podem ausentar-se
§ 2o Se ambos os cônjuges estavam de má-fé ao cele- do domicílio conjugal para atender a encargos públicos,
brar o casamento, os seus efeitos civis só aos filhos apro- ao exercício de sua profissão, ou a interesses particulares
veitarão. relevantes.

99
LEGISLAÇÃO

Art. 1.570. Se qualquer dos cônjuges estiver em lugar Parágrafo único. O juiz pode recusar a homologação e
remoto ou não sabido, encarcerado por mais de cento e não decretar a separação judicial se apurar que a conven-
oitenta dias, interditado judicialmente ou privado, episodi- ção não preserva suficientemente os interesses dos filhos
camente, de consciência, em virtude de enfermidade ou de ou de um dos cônjuges.
acidente, o outro exercerá com exclusividade a direção da
família, cabendo-lhe a administração dos bens. Art. 1.575. A sentença de separação judicial importa a
separação de corpos e a partilha de bens.
CAPÍTULO X Parágrafo único. A partilha de bens poderá ser feita
Da Dissolução da Sociedade e do vínculo Conjugal mediante proposta dos cônjuges e homologada pelo juiz
ou por este decidida.
Art. 1.571. A sociedade conjugal termina:
I - pela morte de um dos cônjuges; Art. 1.576. A separação judicial põe termo aos deveres
II - pela nulidade ou anulação do casamento; de coabitação e fidelidade recíproca e ao regime de bens.
III - pela separação judicial; Parágrafo único. O procedimento judicial da separação
IV - pelo divórcio. caberá somente aos cônjuges, e, no caso de incapacida-
§ 1o O casamento válido só se dissolve pela morte de de, serão representados pelo curador, pelo ascendente ou
um dos cônjuges ou pelo divórcio, aplicando-se a presun- pelo irmão.
ção estabelecida neste Código quanto ao ausente.
§ 2o Dissolvido o casamento pelo divórcio direto ou por Art. 1.577. Seja qual for a causa da separação judicial e
conversão, o cônjuge poderá manter o nome de casado; o modo como esta se faça, é lícito aos cônjuges restabe-
salvo, no segundo caso, dispondo em contrário a sentença lecer, a todo tempo, a sociedade conjugal, por ato regular
de separação judicial. em juízo.
Parágrafo único. A reconciliação em nada prejudicará o
Art. 1.572. Qualquer dos cônjuges poderá propor a direito de terceiros, adquirido antes e durante o estado de
ação de separação judicial, imputando ao outro qualquer separado, seja qual for o regime de bens.
ato que importe grave violação dos deveres do casamento
Art. 1.578. O cônjuge declarado culpado na ação de
e torne insuportável a vida em comum.
separação judicial perde o direito de usar o sobrenome do
§ 1o A separação judicial pode também ser pedida se
outro, desde que expressamente requerido pelo cônjuge
um dos cônjuges provar ruptura da vida em comum há
inocente e se a alteração não acarretar:
mais de um ano e a impossibilidade de sua reconstituição.
I - evidente prejuízo para a sua identificação;
§ 2o O cônjuge pode ainda pedir a separação judicial
II - manifesta distinção entre o seu nome de família e o
quando o outro estiver acometido de doença mental gra-
dos filhos havidos da união dissolvida;
ve, manifestada após o casamento, que torne impossível a III - dano grave reconhecido na decisão judicial.
continuação da vida em comum, desde que, após uma du- § 1o O cônjuge inocente na ação de separação judicial
ração de dois anos, a enfermidade tenha sido reconhecida poderá renunciar, a qualquer momento, ao direito de usar
de cura improvável. o sobrenome do outro.
§ 3o No caso do parágrafo 2o, reverterão ao cônjuge § 2o Nos demais casos caberá a opção pela conserva-
enfermo, que não houver pedido a separação judicial, os ção do nome de casado.
remanescentes dos bens que levou para o casamento, e
se o regime dos bens adotado o permitir, a meação dos Art. 1.579. O divórcio não modificará os direitos e de-
adquiridos na constância da sociedade conjugal. veres dos pais em relação aos filhos.
Parágrafo único. Novo casamento de qualquer dos
Art. 1.573. Podem caracterizar a impossibilidade da pais, ou de ambos, não poderá importar restrições aos di-
comunhão de vida a ocorrência de algum dos seguintes reitos e deveres previstos neste artigo.
motivos:
I - adultério; Art. 1.580. Decorrido um ano do trânsito em julgado
II - tentativa de morte; da sentença que houver decretado a separação judicial, ou
III - sevícia ou injúria grave; da decisão concessiva da medida cautelar de separação de
IV - abandono voluntário do lar conjugal, durante um corpos, qualquer das partes poderá requerer sua conver-
ano contínuo; são em divórcio.
V - condenação por crime infamante; § 1o A conversão em divórcio da separação judicial dos
VI - conduta desonrosa. cônjuges será decretada por sentença, da qual não consta-
Parágrafo único. O juiz poderá considerar outros fatos rá referência à causa que a determinou.
que tornem evidente a impossibilidade da vida em comum. § 2o O divórcio poderá ser requerido, por um ou por
ambos os cônjuges, no caso de comprovada separação de
Art. 1.574. Dar-se-á a separação judicial por mútuo fato por mais de dois anos.
consentimento dos cônjuges se forem casados por mais
de um ano e o manifestarem perante o juiz, sendo por ele Art. 1.581. O divórcio pode ser concedido sem que haja
devidamente homologada a convenção. prévia partilha de bens.

100
LEGISLAÇÃO

Art. 1.582. O pedido de divórcio somente competirá Art. 1.585. Em sede de medida cautelar de separação
aos cônjuges. de corpos, aplica-se quanto à guarda dos filhos as disposi-
Parágrafo único. Se o cônjuge for incapaz para propor ções do artigo antecedente.
a ação ou defender-se, poderá fazê-lo o curador, o ascen-
dente ou o irmão. Art. 1.586. Havendo motivos graves, poderá o juiz, em
qualquer caso, a bem dos filhos, regular de maneira dife-
CAPÍTULO XI rente da estabelecida nos artigos antecedentes a situação
Da Proteção da Pessoa dos Filhos deles para com os pais.

Art. 1.583.  A guarda será unilateral ou compartilhada. Art. 1.587. No caso de invalidade do casamento, ha-
§ 1o  Compreende-se por guarda unilateral a atribuída vendo filhos comuns, observar-se-á o disposto nos arts.
a um só dos genitores ou a alguém que o substitua (art. 1.584 e 1.586.
1.584, § 5o) e, por guarda compartilhada a responsabiliza-
ção conjunta e o exercício de direitos e deveres do pai e da Art. 1.588. O pai ou a mãe que contrair novas núpcias
mãe que não vivam sob o mesmo teto, concernentes ao não perde o direito de ter consigo os filhos, que só lhe po-
poder familiar dos filhos comuns. derão ser retirados por mandado judicial, provado que não
§ 2o  A guarda unilateral será atribuída ao genitor que são tratados convenientemente.
revele melhores condições para exercê-la e, objetivamente,
mais aptidão para propiciar aos filhos os seguintes fatores: Art. 1.589. O pai ou a mãe, em cuja guarda não este-
I – afeto nas relações com o genitor e com o grupo jam os filhos, poderá visitá-los e tê-los em sua companhia,
familiar; segundo o que acordar com o outro cônjuge, ou for fixado
II – saúde e segurança; pelo juiz, bem como fiscalizar sua manutenção e educação.
III – educação. Parágrafo único.  O direito de visita estende-se a qual-
§ 3o  A guarda unilateral obriga o pai ou a mãe que não quer dos avós, a critério do juiz, observados os interesses
a detenha a supervisionar os interesses dos filhos. da criança ou do adolescente.

Art. 1.584.  A guarda, unilateral ou compartilhada, po- Art. 1.590. As disposições relativas à guarda e pres-
derá ser: tação de alimentos aos filhos menores estendem-se aos
I – requerida, por consenso, pelo pai e pela mãe, ou por maiores incapazes.
qualquer deles, em ação autônoma de separação, de divór-
cio, de dissolução de união estável ou em medida cautelar; SUBTÍTULO II
II – decretada pelo juiz, em atenção a necessidades es- Das Relações de Parentesco
pecíficas do filho, ou em razão da distribuição de tempo CAPÍTULO I
necessário ao convívio deste com o pai e com a mãe. Disposições Gerais
§ 1o  Na audiência de conciliação, o juiz informará ao
pai e à mãe o significado da guarda compartilhada, a sua Art. 1.591. São parentes em linha reta as pessoas que
importância, a similitude de deveres e direitos atribuídos estão umas para com as outras na relação de ascendentes
aos genitores e as sanções pelo descumprimento de suas e descendentes.
cláusulas.
§ 2o  Quando não houver acordo entre a mãe e o pai Art. 1.592. São parentes em linha colateral ou transver-
quanto à guarda do filho, será aplicada, sempre que possí- sal, até o quarto grau, as pessoas provenientes de um só
vel, a guarda compartilhada. tronco, sem descenderem uma da outra.
§ 3o  Para estabelecer as atribuições do pai e da mãe
e os períodos de convivência sob guarda compartilhada, Art. 1.593. O parentesco é natural ou civil, conforme
o juiz, de ofício ou a requerimento do Ministério Público, resulte de consanguinidade ou outra origem.
poderá basear-se em orientação técnico-profissional ou de
equipe interdisciplinar. Art. 1.594. Contam-se, na linha reta, os graus de pa-
§ 4o  A alteração não autorizada ou o descumprimento rentesco pelo número de gerações, e, na colateral, tam-
imotivado de cláusula de guarda, unilateral ou comparti- bém pelo número delas, subindo de um dos parentes até
lhada, poderá implicar a redução de prerrogativas atribuí- ao ascendente comum, e descendo até encontrar o outro
das ao seu detentor, inclusive quanto ao número de horas parente.
de convivência com o filho.
§ 5o  Se o juiz verificar que o filho não deve permanecer Art. 1.595. Cada cônjuge ou companheiro é aliado aos
sob a guarda do pai ou da mãe, deferirá a guarda à pes- parentes do outro pelo vínculo da afinidade.
soa que revele compatibilidade com a natureza da medida, § 1o O parentesco por afinidade limita-se aos ascen-
considerados, de preferência, o grau de parentesco e as dentes, aos descendentes e aos irmãos do cônjuge ou
relações de afinidade e afetividade. companheiro.
§ 2o Na linha reta, a afinidade não se extingue com a
dissolução do casamento ou da união estável.

101
LEGISLAÇÃO

CAPÍTULO II II - quando existirem veementes presunções resultan-


Da Filiação tes de fatos já certos.

Art. 1.596. Os filhos, havidos ou não da relação de ca- Art. 1.606. A ação de prova de filiação compete ao fi-
samento, ou por adoção, terão os mesmos direitos e quali- lho, enquanto viver, passando aos herdeiros, se ele morrer
ficações, proibidas quaisquer designações discriminatórias menor ou incapaz.
relativas à filiação. Parágrafo único. Se iniciada a ação pelo filho, os her-
deiros poderão continuá-la, salvo se julgado extinto o pro-
Art. 1.597. Presumem-se concebidos na constância do cesso.
casamento os filhos:
I - nascidos cento e oitenta dias, pelo menos, depois de CAPÍTULO III
estabelecida a convivência conjugal; Do Reconhecimento dos Filhos
II - nascidos nos trezentos dias subsequentes à disso-
lução da sociedade conjugal, por morte, separação judicial, Art. 1.607. O filho havido fora do casamento pode ser
nulidade e anulação do casamento; reconhecido pelos pais, conjunta ou separadamente.
III - havidos por fecundação artificial homóloga, mes-
mo que falecido o marido; Art. 1.608. Quando a maternidade constar do termo do
IV - havidos, a qualquer tempo, quando se tratar de nascimento do filho, a mãe só poderá contestá-la, provan-
embriões excedentários, decorrentes de concepção artifi- do a falsidade do termo, ou das declarações nele contidas.
cial homóloga;
V - havidos por inseminação artificial heteróloga, des- Art. 1.609. O reconhecimento dos filhos havidos fora
de que tenha prévia autorização do marido. do casamento é irrevogável e será feito:
I - no registro do nascimento;
Art. 1.598. Salvo prova em contrário, se, antes de de- II - por escritura pública ou escrito particular, a ser ar-
corrido o prazo previsto no inciso II do art. 1.523, a mulher quivado em cartório;
contrair novas núpcias e lhe nascer algum filho, este se pre- III - por testamento, ainda que incidentalmente mani-
sume do primeiro marido, se nascido dentro dos trezentos festado;
dias a contar da data do falecimento deste e, do segundo, IV - por manifestação direta e expressa perante o juiz,
se o nascimento ocorrer após esse período e já decorrido o ainda que o reconhecimento não haja sido o objeto único
prazo a que se refere o inciso I do art. 1597. e principal do ato que o contém.
Parágrafo único. O reconhecimento pode preceder o
Art. 1.599. A prova da impotência do cônjuge para ge- nascimento do filho ou ser posterior ao seu falecimento, se
rar, à época da concepção, ilide a presunção da paterni- ele deixar descendentes.
dade.
Art. 1.610. O reconhecimento não pode ser revogado,
Art. 1.600. Não basta o adultério da mulher, ainda que nem mesmo quando feito em testamento.
confessado, para ilidir a presunção legal da paternidade.
Art. 1.611. O filho havido fora do casamento, reconhe-
Art. 1.601. Cabe ao marido o direito de contestar a pa- cido por um dos cônjuges, não poderá residir no lar conju-
ternidade dos filhos nascidos de sua mulher, sendo tal ação gal sem o consentimento do outro.
imprescritível.
Parágrafo único. Contestada a filiação, os herdeiros do Art. 1.612. O filho reconhecido, enquanto menor, ficará
impugnante têm direito de prosseguir na ação. sob a guarda do genitor que o reconheceu, e, se ambos o
reconheceram e não houver acordo, sob a de quem melhor
Art. 1.602. Não basta a confissão materna para excluir atender aos interesses do menor.
a paternidade.
Art. 1.613. São ineficazes a condição e o termo apostos
Art. 1.603. A filiação prova-se pela certidão do termo ao ato de reconhecimento do filho.
de nascimento registrada no Registro Civil.
Art. 1.614. O filho maior não pode ser reconhecido sem
Art. 1.604. Ninguém pode vindicar estado contrário ao o seu consentimento, e o menor pode impugnar o reco-
que resulta do registro de nascimento, salvo provando-se nhecimento, nos quatro anos que se seguirem à maiorida-
erro ou falsidade do registro. de, ou à emancipação.

Art. 1.605. Na falta, ou defeito, do termo de nascimen- Art. 1.615. Qualquer pessoa, que justo interesse tenha,
to, poderá provar-se a filiação por qualquer modo admis- pode contestar a ação de investigação de paternidade, ou
sível em direito: maternidade.
I - quando houver começo de prova por escrito, prove-
niente dos pais, conjunta ou separadamente;

102
LEGISLAÇÃO

Art. 1.616. A sentença que julgar procedente a ação de IV - nomear-lhes tutor por testamento ou documento
investigação produzirá os mesmos efeitos do reconheci- autêntico, se o outro dos pais não lhe sobreviver, ou o so-
mento; mas poderá ordenar que o filho se crie e eduque brevivo não puder exercer o poder familiar;
fora da companhia dos pais ou daquele que lhe contestou V - representá-los, até aos dezesseis anos, nos atos da
essa qualidade. vida civil, e assisti-los, após essa idade, nos atos em que
forem partes, suprindo-lhes o consentimento;
Art. 1.617. A filiação materna ou paterna pode resultar VI - reclamá-los de quem ilegalmente os detenha;
de casamento declarado nulo, ainda mesmo sem as condi- VII - exigir que lhes prestem obediência, respeito e os
ções do putativo. serviços próprios de sua idade e condição.

Seção III
CAPÍTULO IV
Da Suspensão e Extinção do Poder Familiar
Da Adoção
Art. 1.635. Extingue-se o poder familiar:
Art. 1.618.  A adoção de crianças e adolescentes será I - pela morte dos pais ou do filho;
deferida na forma prevista pela Lei no 8.069, de 13 de julho II - pela emancipação, nos termos do art. 5o, parágrafo
de 1990 - Estatuto da Criança e do Adolescente. único;
III - pela maioridade;
Art. 1.619.  A adoção de maiores de 18 (dezoito) anos IV - pela adoção;
dependerá da assistência efetiva do poder público e de V - por decisão judicial, na forma do artigo 1.638.
sentença constitutiva, aplicando-se, no que couber, as re-
gras gerais da Lei no 8.069, de 13 de julho de 1990 - Estatu- Art 1.636. O pai ou a mãe que contrai novas núpcias,
to da Criança e do Adolescente. ou estabelece união estável, não perde, quanto aos filhos
do relacionamento anterior, os direitos ao poder familiar,
Art. 1.620. a 1.629. (REVOGADOS) exercendo-os sem qualquer interferência do novo cônjuge
ou companheiro.
CAPÍTULO V Parágrafo único. Igual preceito ao estabelecido neste
Do Poder familiar artigo aplica-se ao pai ou à mãe solteiros que casarem ou
estabelecerem união estável.
Seção I
Disposições Gerais
Art. 1.637. Se o pai, ou a mãe, abusar de sua autorida-
de, faltando aos deveres a eles inerentes ou arruinando os
Art. 1.630. Os filhos estão sujeitos ao poder familiar, bens dos filhos, cabe ao juiz, requerendo algum parente,
enquanto menores. ou o Ministério Público, adotar a medida que lhe pareça
reclamada pela segurança do menor e seus haveres, até
Art. 1.631. Durante o casamento e a união estável, com- suspendendo o poder familiar, quando convenha.
pete o poder familiar aos pais; na falta ou impedimento de Parágrafo único. Suspende-se igualmente o exercício
um deles, o outro o exercerá com exclusividade. do poder familiar ao pai ou à mãe condenados por senten-
Parágrafo único. Divergindo os pais quanto ao exercí- ça irrecorrível, em virtude de crime cuja pena exceda a dois
cio do poder familiar, é assegurado a qualquer deles recor- anos de prisão.
rer ao juiz para solução do desacordo.
Art. 1.638. Perderá por ato judicial o poder familiar o
Art. 1.632. A separação judicial, o divórcio e a dissolu- pai ou a mãe que:
ção da união estável não alteram as relações entre pais e I - castigar imoderadamente o filho;
filhos senão quanto ao direito, que aos primeiros cabe, de II - deixar o filho em abandono;
terem em sua companhia os segundos. III - praticar atos contrários à moral e aos bons costu-
mes;
IV - incidir, reiteradamente, nas faltas previstas no arti-
Art. 1.633. O filho, não reconhecido pelo pai, fica sob
go antecedente.
poder familiar exclusivo da mãe; se a mãe não for conheci-
da ou capaz de exercê-lo, dar-se-á tutor ao menor. A concepção de família não pode ser compreendia
em caráter restrito. Mais do que uma entidade a ser per-
Seção II cebida sob a ótica do Direito, a família requer a percepção
Do Exercício do Poder Familiar de questões multifatoriais que interferem na sua definição
e na sua retratação. Questão relevante que determina a
Art. 1.634. Compete aos pais, quanto à pessoa dos fi- impossibilidade de se fixar um modelo familiar uniforme
lhos menores: é a necessidade de ser, a família, compreendida de acor-
I - dirigir-lhes a criação e educação; do com as relações sociais ocorridas ao longo do tempo,
II - tê-los em sua companhia e guarda; adaptadas às necessidades sociais, facilmente percebidos
III - conceder-lhes ou negar-lhes consentimento para quando fazemos uma digressão histórica e antropológica
casarem; de tal instituto.

103
LEGISLAÇÃO

O conceito jurídico de família evoluiu com o progresso É, portanto, uma união permanente entre um homem e
da sociedade, condicionando-se, durante muito tempo, ao uma mulher, um contrato totalmente vinculado às normas
casamento religioso e, mais tarde, ao casamento civil disci- públicas, que tem como pressuposto a livre manifestação
plinado legalmente. de vontade dos nubentes. Estabelecendo comunhão plena
O novo referencial da família, pautado na afetividade, de vida, com base na igualdade de direitos e deveres dos
como valor jurídico, é reforçado na Constituição, acompa- cônjuges.
nhando o progresso e a mudança de valores da sociedade. O casamento é civil e gratuita a sua celebração. A ha-
A nossa Constituição definiu o Direito de Família com bilitação para o casamento, o registro e a primeira certidão
base em três linhas: vedação da discriminação entre filhos, serão isentos de selos, emolumentos e custas, para as pes-
igualdade entre homens e mulheres e entidade familiar. soas cuja pobreza for declarada, sob as penas da lei.
Pode-se identificar, pela literalidade da Constituição de Este instituto se realiza no momento em que o homem
1988, 3 espécies exemplificativas e bem definidas de arran- e a mulher manifestam, perante o juiz, a sua vontade de
jos familiares: estabelecer vínculo conjugal, e o juiz os declara casados.
- a matrimonial; Já o casamento religioso, que atender às exigências da
- a decorrente da união estável e lei para a validade do casamento civil, equipara-se a este,
- a monoparental, formada por qualquer dos pais e desde que registrado no registro próprio, produzindo des-
respectiva prole. ta forma efeitos a partir da data de sua celebração. Subme-
tendo aos mesmos requisitos exigidos para o casamento
Os modelos de família presentes em nossa realidade civil.
social devem ser reconhecidos pelo Direito sempre, para - o registro civil do casamento religioso deverá ser
que haja respeito aos valores essenciais dos membros do promovido dentro de noventa dias de sua realização, me-
grupo familiar, especialmente o da dignidade da pessoa diante comunicação do celebrante ao ofício competente,
humana. ou por iniciativa de qualquer interessado, desde que haja
Baseando nos preceitos do direito italiano, pode-se sido homologada previamente a habilitação regulada pelo
afirmar dever o princípio da pluralidade no âmbito familiar Código Civil, Após este prazo o registro dependerá de nova
receber uma interpretação ampla, respeitadora das diver- habilitação.
sas formas de união, a encontrar limite apenas na dignida-
- o casamento religioso, celebrado sem as formalida-
de das pessoas. Não se deve negar um digno tratamento
des exigidas, terá efeitos civis se, a requerimento do casal,
aos diferentes modelos arraigados na sociedade por razões
for registrado, a qualquer tempo, no registo mediante pré-
preconceituosas.
via habilitação perante a autoridade competente e obser-
Com a tutela principal das relações familiares voltada
vado o prazo estipulado pelo artigo 1.532.
para a realização personalística de seus membros, a plu-
- será nulo o registro civil do casamento religioso se,
ralidade de entidades familiares se impõe. Respeitando-se
antes dele, qualquer dos consorciados houver contraído
as diferenças, as pessoas poderão conviver familiarmente
conforme o modelo que melhor represente seus anseios com outrem casamento civil.
pessoais.
Neste contexto, a relação entre pessoas do mesmo Da Capacidade
sexo deve ser apreendida pelo ordenamento jurídico e pe- A capacidade para contrair matrimônios teve sua idade
los operadores do direito, como mais um modelo de enti- núbil equiparada entre homens e mulheres: 16 anos. A con-
dade familiar. dição de discernir mostra-se para o legislador em sintonia
Vários princípios jurídicos agasalham a tutela da par- e que não haveria o porquê de trazer uma variação etária
ceria homoafetiva, sendo uma realidade a demandar igual- relativa ao fato de que a aptidão de ambos se equivalem
mente uma solução jurídica positiva. Isto é defendido, pois nesta nuance. Mostrando-se alerta a isso, não confunde-se
se acredita não haver nenhum empecilho de cunho axioló- neste âmbito os termos capacidade matrimonial e capa-
gico para reconhecerem-se efeitos jurídicos a essa dimen- cidade de ação, já que este último é conseguido através
são privada de formação de núcleos afetivos semelhantes do alcance da maioridade civil, excetuada pelo instituto da
aos modelos de famílias contemporaneamente reconheci- emancipação, ocorrendo, portanto, em princípio, com 18
dos. anos, vislumbrado está aqui o art. o caput do art. 5º: “a
A prevalência da socioafetividade, amparada na dig- menoridade cessa aos dezoito anos completos, quando
nidade da pessoa humana e na solidariedade familiar são a pessoa fica habilitada à pratica de todos os atos da
modernos princípios do Direito de Família hábeis a de- vida civil.”.
monstrar a relativização dos laços parentais de natureza
biológica. Dos Impedimentos e das Causas Suspensivas
Além da capacidade matrimonial, os impedimentos so-
Do Casamento frem uma alteração doutrinária; nos artigos 1.521, 1.523 e
A doutrina majoritária conceitua casamento como um 1.548 encontram-se os denominados impedimentos diri-
ato jurídico negocial, solene, público e complexo, mediante mentes absolutos ou públicos, com efeito ex tunc, causado
o qual um homem e uma mulher constituem família, pela consequentemente por uma nulidade absoluta que há de
livre manifestação de vontade e pelo reconhecimento do ser arbitrada judicialmente em ação posterior, não poden-
Estado. do os nubentes, portanto, se casar nas seguintes hipóteses:

104
LEGISLAÇÃO

- quando são ascendentes e descendentes, partindo-se aí tanto do parentesco oriundo da consanguinidade (parentes-
co natural), quanto da adoção (parentesco civil);
- os parentes por afinidade que encontram-se na linha reta, especialmente noras com sogros e genros com sogras,
além de padrastos com enteadas e enteados com madrastas, ressaltando-se aqui que a linha reta por afinidade não tem
limite em suas gerações, pela quantidade de graus;
- o adotante com a pessoa que foi casada com o adotado e o adotado com quem foi consorte de quem o adotou;
- os irmãos, tanto os unilaterais (que não possuem o mesmo pai e a mesma mãe), quanto os bilaterais ou germanos
(que têm os mesmos pais), e demais colaterais, até o terceiro grau (tios e sobrinhos), sendo que há um entendimento de
que, apresentando documento médico, podem os mesmos contrair matrimônio;
- o adotado com o filho de quem o adotou;
- as pessoas que são casadas; o consorte sobrevivente com o indivíduo condenado pela prática do crime de homicídio,
ou tentativa deste, em face do seu cônjuge;
- além da hipótese do casamento contraído com enfermo mental sem a necessária capacidade de discernimento para
a prática dos atos da vida civil.
O legislador elencou as hipóteses dos impedimentos impedientes, denominados como causas suspensivas, que não
acarretam nem a nulidade absoluta, nem tão pouco, a nulidade relativa, oriunda de ato jurídico anulável, sendo, portanto,
um casamento denominado de irregular, trazendo aos consortes uma sanção que é a impossibilidade de se escolher livre-
mente qual o regime de bens que deve reger o matrimônio do casal, devendo este se dar sob o regime da separação total
de bens.
Abaixo segue um quadro exemplificativo diferenciado causas de impedimento e causas suspensivas para a constituição
do matrimonio. Vale ressaltar que as causas suspensivas são obstáculos transponíveis, suscetíveis de transposição, consti-
tuindo meras recomendações do legislador; podendo ser de ordem privada, ou seja, somente poderão ser opostas pelos
parentes em linha reta de qualquer dos nubentes, e pelos colaterais até o segundo grau.
Já em relação a causas de impedimentos, a decretação da nulidade se dará mediante ação direta, promovida por qual-
quer interessado ou então pelo Ministério Público.

Impedimentos Causas suspensivas


Não podem casar Não DEVEM casar
Se casarem, o casamento será valido, sendo imposto a eles
Se casarem, o casamento será nulo. apenas uma sanção administrativa, qual seja: o regime da
separação obrigatória de bens.
Quem pode arguir: qualquer pessoa capaz, até o momento Quem pode arguir: Os parentes em linha reta de um dos
da celebração do casamento. Se o juiz, ou o oficial de nubentes, sejam consanguíneos ou afins, e pelos colaterais
registro, tiver conhecimento da existência de algum em segundo grau, sejam também consanguíneos ou afins.
impedimento, será obrigado a declará-lo.
São 7 hipóteses São 4 hipóteses
São hipóteses de impedimento:
São hipóteses de suspensão:
1.  os ascendentes com os descendentes, seja o parentesco
1.  O viúvo ou a viúva que tiver filho do cônjuge falecido,
natural ou civil;
enquanto não fizer inventário dos bens do casal e der
2.  os afins em linha reta;
partilha aos herdeiros;
3.  o adotante com quem foi cônjuge do adotado e o
2.  A viúva, ou a mulher cujo casamento se desfez por ser
adotado com quem o foi do adotante;
nulo ou ter sido anulado, até 10 meses depois do começo
4.  os irmãos, unilaterais ou bilaterais, e demais colaterais,
da viuvez, ou da dissolução da sociedade conjugal;
até o terceiro grau inclusive;
3.  O divorciado, enquanto não houver sido homologada
5.  o adotado com o filho do adotante;
ou decidida a partilha dos bens do casal;
6.  as pessoas casadas;
4.  O tutor ou o curador e os seus descendentes,
7.  o cônjuge sobrevivente com o condenado por
ascendentes, irmãos, cunhados ou sobrinhos, com a pessoa
homicídio ou tentativa de homicídio contra o seu consorte.
tutelada ou curatelada, enquanto não cessar a tutela ou
curatela, e não estiverem saldadas as respectivas contas.

No processo de habilitação para o casamento, são reflexos de um procedimento necessário à condição das pessoas
quererem contrair núpcias, encontrando aqui uma lista de documentos necessários para tal ato em si, sendo este o primeiro
ato para a efetivação do matrimonio.
Depois disto será encaminhado ao Ministério Público, para que este se manifeste. Não havendo oposições ao casamen-
to, o terceiro ato deste procedimento consiste na publicação dos editais, que serão afixados nas circunscrições dos noivos,
durante 15 dias.

105
LEGISLAÇÃO

As formalidades preliminares da habilitação para o ca- Casamento Consular


samento serão encerradas com a expedição de uma habilita- Ocorre quando a celebração é feita perante a autorida-
ção, em sentido estrito, por meio da qual os nubentes ficam de diplomática ou consular de ambos os nubentes.
autorizados a se casarem no prazo decadencial de 90 dias a Para que ele acontece deve ser observados alguns re-
contar da data da expedição. quisitos:
O art. 1.528 faz menção ao fato de que é dever do oficial - é preciso que ambos tenham a mesma nacionalidade
do registro esclarecer, orientar, os que pretendem o enlace do cônsul ou autoridade diplomática.
matrimonial no que concerne aos fatos que podem ocasionar - quanto às formalidades e aos impedimentos são da
a invalidade do casamento, bem como sobre os quatro regi- legislação do país de origem dos nubentes.
mes de bens que incidem sobre o aludido negócio jurídico. - o casamento não se realizará se a legislação do país
O referido preceito acima indica uma solução lógica ao em que se localiza o consulado ou embaixada não permitir.
interpretar que ninguém poderia alegar a não ciência a res- A autoridade consular deve ser de carreira (tempo de
peito do que estivesse para ocorrer com o destino de seus exercício de função).
bens inclusive pós matrimônio, o que de muita valia esta in- Quando realizado o casamento consular, este terá que
dicação se reveste. ser registrado em 180 dias, contados da data em que um ou
ambos os cônjuges voltarem ao Brasil.
Existindo o casamento, como já vimos ele pode ser váli-
do ou inválido. Para que o casamento seja válido, também já Casamento por Procuração
demonstrado, é preciso que ele seja realizado mediante todos O nosso ordenamento jurídico permite aos cônjuges
os requisitos estabelecidos em lei. que não possam, por qualquer motivo, estarem presentes na
celebração do seu próprio matrimônio, que se façam repre-
Casamento Putativo sentar por procurador bastante, munido de poderes espe-
O casamento putativo ocorre quando um ou ambos os ciais, necessariamente através de instrumento público, feito
cônjuges desconhece algum impedimento, portanto ele pode por tabelião de notas, com prazo de validade de noventa
ser nulo ou anulável. Para o cônjuge de boa fé, ele produz dias e menção inequívoca do outro contraente.
efeitos de casamento válido. Esses efeitos são desde a cele- Poderá um ou ambos estar ausentes, mas a única exi-
bração do casamento até a data da sentença anulatória, após gência é que o procurador represente apenas um do casal.
a sentença, cessam todos os deveres que são resultantes do Na ausência dos dois, serão necessárias duas procurações,
casamento. com mandatários distintos, um para cada qual dos consor-
Os filhos que porventura nascerem de um casamento pu- tes. A restrição se faz porque sendo o casamento um contra-
tativo, terão seus direitos garantidos. to bilateral, é necessária a manifestação de duas vontades.
O casamento celebrado, mesmo ausentes ambos os
Casamento Nuncupativo e em caso de Moléstia Grave contraentes, cada qual representado por mandatário pró-
O casamento nuncupativo é uma forma especial de prio, é legal, não sendo passível de se tornar nulo ou anulá-
celebração do casamento, onde um dos nubentes está em vel o ato feito por procuração.
iminente risco de vida, assim devido à urgência e a falta de
tempo não foram cumpridas todas as formalidades para a Nota-se que o casamento existe a partir da manifes-
celebração. Mas é dispensada a presença de autoridade, pos- tação de vontade de querer se casar de ambas as partes,
suindo apenas seis testemunhas, desde que não sejam paren- homem e mulher, formando assim uma união, uma famí-
tes dos nubentes. Essas testemunhas precisam ser convoca- lia. Mas para a sua validação é preciso que ele estabeleça
das pelo enfermo e, ouvir do casal a manifestação de vontade os requisitos previstos em lei. Sendo assim, todas as formas
de contrair núpcias. de casamento citadas são formas de casamento válido, pois
Após a celebração as testemunhas devem procurar a au- preenchem todos os requisitos necessários especificados
toridade competente para reduzir à termo as suas declara- pelo nosso ordenamento jurídico.
ções, devendo fazer isso em 10 dias.
Já a celebração em caso de moléstia grave, consiste em Separação e divórcio
um casamento civil, onde um dos nubentes encontra-se em Conforme nosso ordenamento jurídico, a dissolução da
com uma doença grave que o impeça de locomover-se e sociedade conjugal termina com a morte de um dos cônju-
também de adiar tal cerimônia. ges, com a nulidade, ou anulação do casamento, pela sepa-
ração judicial ou então pelo divórcio.
Casamento Religioso com Efeitos Civis Mas em 2010 com a Emenda Constitucional n° 66, alte-
O casamento religioso com efeitos civis é realizado pe- rou-se o artigo 226, § 6° da Constituição Federal, dispondo
rante um ministro de qualquer fé religiosa, logo após a habi- que a separação judicial, segundo posicionamento majoritá-
litação dos nubentes. rio, foi revogada remanescendo os demais institutos.
Existem duas formas de casamento religioso com efeitos Desta forma, o vínculo matrimonial poderá ser desfei-
civis: com habilitação prévia (onde será apresentado ao minis- to de maneira mais rápida e menos custosa. Sendo que as
tro religioso o certificado de habilitação e ele irá arquivá-lo, o principais alterações foram a fim da separação judicial, mas
registro civil tem que ser feito dentro do prazo decadencial também põem fim ao divórcio indireto, posto não haver
que é de 90 dias da celebração) e com habilitação posterior mais o que ser convertido. Outra questão que foi abolida é
(onde os nubentes podem requerer o registro a qualquer em relação ao prazo mínimo para a dissolução do vínculo
tempo). matrimonial.

106
LEGISLAÇÃO

Com isso ficam tacitamente revogados os artigos Como dispõe o artigo 1.584, verificando que os filhos
1.572 a 1.578 e 1.580. não devem permanecer sob a guarda do pai ou da mãe, o
juiz deferirá a sua guarda à pessoa que revele compatibi-
A referida emenda tem efeito ex nunc, ela não retroa- lidade com a natureza da medida, de preferência levando
ge e nem altera relações jurídicas pretéritas, ou seja, é a em conta o grau de parentesco e relação de afinidade e
proteção ao direito adquirido. Quem já está separado ju- afetividade, de acordo com a referida lei.
dicialmente continua neste mesmo estado civil, por isso dá Havendo motivos graves, poderá o juiz, em qualquer
permanência da possibilidade do divórcio por conversão. caso, a bem dos filhos, regular de maneira diferente da es-
Uma outra questão relevante a cerca desta alteração tabelecida nos artigos 1.583 a 1.585 – a situação deles para
estabelecida pela EC 66, diz respeito aos processos de se- com os pais.
paração que estão em curso, estes não podem ser conver- O pai ou a mãe que contrair novas núpcias não perde o
tidos de ofício pelo magistrado, mas extintos ante a perda direito de ter consigo os filhos, que só lhe poderão ser reti-
superveniente do objeto a não ser que as partes requeiram rados por mandado judicial, provado que não são tratados
a conversão. convenientemente.
As disposições relativas à guarda e prestação de ali-
Divórcio extrajudicial mentos aos filhos menores estendem-se aos maiores in-
Esta matéria sofreu mudanças significativas através da capazes.
Lei Federal n° 11.441/07 que trouxe importante inovação Cumpre ressaltar que em relação a guarda dos filhos,
ao permitir o divórcio administrativo, realizado perante estes deverão ficar com quem tiver melhores condições
um tabelião e desde que não haja litígio entre os divorcia- psicológicas e morais, devendo levar em conta o interesse
dos, nem incapazes envolvidos. A presença do advogado é do menor.
imprescindível, sendo possível, inclusive, um só causídico Com base nesta premissa, da importância do interesse
para ambos os clientes. do menor, a Lei 12.398/2011 alterou a redação do artigo
O procedimento é simples, estando toda a documenta- 888, inciso VII do Código de Processo Civil e acrescentou o
ção em dia, o tabelião lavrará a escritura pública do divór- parágrafo único do artigo 1.589 CC, estendendo o direito
cio extrajudicial, que será o título hábil tanto à averbação de visita aos avós e também estendendo o direito a guarda.
deste à margem do registro de casamento, quanto para Art. 888 - O juiz poderá ordenar ou autorizar, na pen-
dência da ação principal ou antes de sua propositura:
eventual alteração do registro imobiliário ou do nome de
VII – a guarda e a educação dos filhos, regulado o direi-
uma das partes.
to de visita que, no interessa da criança ou do adolescente,
Com a extinção da separação judicial não cabe falar em
pode, a critério do juiz, ser extensivo a cada um dos avós;
separação extrajudicial.
Relações de Parentesco
Os efeitos que são geridos pelo divórcio, assim como o
Doutrinadores conceituam relações de parentesco
casamento, são de ordem pessoal e patrimonial em relação
como sendo as pessoas que se unem a uma família em
aos divorciados e seus filhos. razão de vínculo conjugal ou união estável, de parentesco
Um desses efeitos é em relação ao nome de casado, por consanguinidade ou outra origem, e da afinidade.
como é uma faculdade do nubente, que poderá acrescer Parentesco tem pela doutrina, em sua definição, dois
ou não ao seu nome o sobrenome do outro cônjuge, na sentidos: em sentido estrito é o que abrange o consan-
dissolução também cabe a mesma regra, é uma faculdade guíneo, ou seja, a relação que vincula entre si pessoas que
manter o nome ou retornar ao nome de solteiro. descendem umas das outras, ou de um mesmo tronco.
Em relação ao direito a alimentos, cumpre ressaltar que Já em sentido amplo, no entanto, inclui o parentesco por
se trata de um direito do cônjuge a alimento prestado pelo afinidade e o decorrente da adoção ou de outra origem,
outro cônjuge. como modalidades de técnicas de reprodução assistida.
Os alimentos serão devidos levando-se em conta ba- Podemos então definir parentesco como o vínculo jurí-
sicamente o binômio necessidade/utilidade. O quantum dico que une duas pessoas por questões consanguíneas ou
devido será fixado na justa medida entre a capacidade do por determinação legal. O parentesco proveniente de afi-
alimentante e a necessidade do alimentado, mas manten- nidade surge com a ocorrência do casamento ou de união
do o padrão de vida do cônjuge enquanto estava casado e estável, onde o cônjuge ou o companheiro se vincula aos
não somente é apenas o que se come. parentes consanguíneos do outro, daí surgindo o paren-
Se o divórcio for consensual, os próprios divorciados tesco por afinidade.
deverão acordar sobre o montante a ser pago. Mas se o di- O parentesco consanguíneo pode ser então dividido
vórcio for judicial e cumulado com alimentos, o juiz é quem em: consanguíneo na linha reta e consanguíneo na linha
decidirá de forma equilibrada. colateral ou transversal.
O grau de parentesco é definido em linha reta ascen-
Proteção dos filhos dente e/ou descendente pelo número de gerações que
No caso de dissolução da sociedade ou do vínculo separam as pessoas entre si, pois os indivíduos que estão
conjugal pela separação judicial por mútuo consentimen- uns para com os outros na relação de ascendentes e/ou
to ou pelo divórcio direto consensual, observar-se-á o que descendentes são sempre parentes, independentemente
cônjuges acordarem sobre a guarda dos filhos. da distância em graus entre os mesmos.

107
LEGISLAÇÃO

O grau de parentesco na linha colateral ou transversal A presunção de paternidade em caso de viuvez é atri-
é contado partindo-se do parente cujo grau de parentesco buível ao segundo marido se o nascimento se deu após
se pretende determinar, subindo-se em linha reta, contan- 300 dias da dissolução da sociedade conjugal anterior e se
do-se cada grau, até o ascendente comum, descendo-se o nascimento se deu após 180 dias do início da segunda
em seguida até encontrar o outro parente, mas o paren- sociedade conjugal.
tesco entre os colaterais vai até o quarto grau, limitação Já na presunção de paternidade e impotência, o legis-
imposta pelo novo diploma civil. lador questiona se a impotência generandi (de gerar) foi
Já em relação à definição do grau de parentesco por constatada no período em que teria ocorrido a concepção.
afinidade, o procedimento é o mesmo, colocando-se ape- A ocorrência dessa concepção presume o impedimento
nas a posição do cônjuge ou companheiro no lugar ocu- para gerar. Provada a impotência do cônjuge desaparece a
pado naturalmente pelo outro, observando-se que o pa- presunção de paternidade.
rentesco por afinidade na linha colateral estende-se até o No caso de adultério, não basta à confissão da mulher
2º grau (cunhado/cunhada) e, na linha reta, estende-se ao de que cometeu adultério para ilidir a presunção legal de
infinito e, mesmo após a dissolução do casamento, não se paternidade. Neste caso é preciso a existência de exame de
extingue. DNA e eventual reconhecimento pelo pai biológico.
Desta forma, se o marido não contestar a paternidade,
Filiação a mulher não poderá contestá-la, pois sua confissão não
Como demonstra o artigo 1.596, filiação é filiação é possui valor probante.
uma relação de parentesco em linha reta de primeiro grau, Esta filiação em regra geral será feita em prova pela
estabelecida entre pais e filhos, seja por relação de vínculo certidão de nascimento, ou na falta do documento, por
sanguíneo ou de outra origem legal, como por exemplo, qualquer modo admissível em direito, isto quando houver
nos casos de adoção ou de reprodução assistida como uti- começo de prova por escrito, vindo dos pais, conjunta ou
separadamente, ou ainda quando existir intensas presun-
lização de material genético de outra pessoa estranha ao
ções resultantes de fatos já certos.
casal.
É irrevogável o reconhecimento do filho mesmo quan-
A Constituição Federal, no artigo 227, § 6° disciplina
do feito em testamento, sendo que havido fora do casa-
a igualdade existente entre os filhos, não admitindo
mento, reconhecido por um dos cônjuges, não poderá re-
qualquer distinção ou denominação que o diferencie
sidir no lar conjugal sem o consentimento do outro, sendo
dos demais.
ineficazes a condição e o termo apostos ao ato de reconhe-
Art. 227 – cimento do filho.
§ 6° - os filhos, havidos ou não da relação do casa- O reconhecimento do filho maior não pode acontecer
mento, ou por adoção, terão os mesmos direitos e qualifi- sem o consentimento do filho. Se for menor, este poderá
cações, proibidas quaisquer designações descriminatórias impugnar nos quatros anos que se seguirem à maioridade
relativas à filiação. ou a emancipação.
Ação de investigação de paternidade é o meio pelo
Importante também ressaltar em relação a insemina- qual se pode reconhecer judicialmente a filiação de al-
ção artificial homóloga, ou seja, é a técnica de reprodução guém. Qualquer filho poderá propor a ação de investiga-
assistida havida com material genético (semem e óvulo) do ção de paternidade.
próprio casal. Pela lei, nestes casos a presunção da filiação Já na ação negatória de paternidade, conforme esta-
na constância do casamento sempre existirá, mesmo se o belece o artigo 1.601, p autor é o marido que pretende a
nascimento, ou a fecundação, ou ainda a inseminação, se declaração de que não é pai do filho nascido de sua mu-
der após a morte, desde que presentes autorização expres- lher. Sendo esta ação imprescritível por expressa disposi-
sa e prévia do marido. ção legal. Se o marido não ajuizar, ninguém poderá o fazer.
A inseminação artificial heteróloga, é a fecundação Em relação a impugnação ao estado de filiação, o filho
realizada com material genético de pelo menos um terceiro que está registrado como filho, que no prazo de 4 anos a
aproveitando ou não semem ou óvulo de um ou outro côn- contar ou da sua emancipação, ou da maioridade, a impug-
juge, razão pela qual exige autorização do outro cônjuge, nação deste registro, com o subsequente cancelamento de
sob pena de não se presumir filho do casal. mesmo.
Esta presunção é relativa, podendo ser afastadas por Como já demonstrado com as inseminações artificiais,
prova em sentido contrário. o Direito não pode negar a necessidade de acomodar no-
Presumem-se concebidos na constância do casamento: vos problemas jurídicos, como o direito subjetivo de al-
1. Os filhos nascidos 180 dias, pelo menos, depois de guém a conhecer, exclusivamente, de sua antecedência
estabelecida a convivência conjugal. biológica, de seu DNA, sem que isto importe numa impu-
2. Os filhos nascidos 300 dias após a dissolução da so- tação de paternidade indesejada.
ciedade conjugal – seja pela morte, separação, nulidade e
anulação do casamento. Poder Familiar
3. Os filhos havidos por fecundação artificial homólo- Significa o conjunto de direitos e deveres atribuídos
ga, mesmo que falecido o marido. aos pais, em relação aos filhos menores e seus bens. É ir-
4. Os filhos havidos a qualquer tempo, quando se tra- renunciável, indelegável e imprescritível, desta forma os
tar de embriões excedentários, decorrente de concepção pais não podem renunciar a ele, bem como transferi-lo a
artificial homóloga. outrem.

108
LEGISLAÇÃO

O art.1.630 traz em seu texto que os filhos estão su- e que os pais tem sobre o menor o poder familiar, inde-
jeitos ao poder familiar enquanto menores. Cabe ressal- pendente de separação judicial, divórcio ou dissolução da
tar que o dispositivo engloba os filhos menores, havidos união estável, lhes garantido o direito de visita e fiscaliza-
ou não no casamento e adotivos, e que compete a am- ção. Quando estes restarem casados e fizerem o exercício
bos os pais o exercício do poder, de forma igualitária de do poder familiar conjuntamente, têm direito ao exercício
condições, sendo que no caso de divergência poderá ser de forma igualitária, estando sujeitos a condições que sus-
solucionado por via judicial. Na ausência de um dos pais, pendam ou extinguem o poder familiar.
o outro possui o exercício com exclusividade e a família é
identificada como monoparental. 1694 A 1727 E
A separação judicial, o divórcio e a dissolução da união
estável, não alteram o poder familiar. Somente se altera SUBTÍTULO III
com relação à guarda do menor, sendo que este fica so- Dos Alimentos
mente com um dos pais, assegurado ao outro o acompa-
nhamento por meios de visita e fiscalização. Art. 1.694. Podem os parentes, os cônjuges ou compa-
A doutrina determina que nenhum dos pais perde o nheiros pedir uns aos outros os alimentos de que neces-
poder familiar com a separação judicial ou divórcio. O pá- sitem para viver de modo compatível com a sua condição
trio poder ou poder familiar decorre da paternidade e da social, inclusive para atender às necessidades de sua edu-
filiação e não do casamento, tanto que o mais recente có- cação.
digo se reporta também à união estável. A guarda normal- § 1o Os alimentos devem ser fixados na proporção das
mente ficará com um deles, assegurado ao outro o direito necessidades do reclamante e dos recursos da pessoa obri-
de visita. gada.
O poder familiar incumbe aos pais na criação e educa- § 2o Os alimentos serão apenas os indispensáveis à
ção dos filhos menores, tê-los em sua guarda e administrar subsistência, quando a situação de necessidade resultar de
os atos da vida civil enquanto forem menores, conforme culpa de quem os pleiteia.
exposto no art. 1.634. Incumbe aos pais também a adminis-
tração dos bens dos filhos, bem como o direito ao usufruto Art. 1.695. São devidos os alimentos quando quem os
destes. pretende não tem bens suficientes, nem pode prover, pelo
O poder família tem sua extinção prevista no artigo seu trabalho, à própria mantença, e aquele, de quem se
seguinte. A qual se dá por fatos de plenos direito, fatos reclamam, pode fornecê-los, sem desfalque do necessário
naturais ou por decisão judicial. O artigo supracitado defi- ao seu sustento.
ne as causas de extinção, sendo elas: morte dos pais ou do
filho, emancipação, maioridade, adoção e decisão judicial Art. 1.696. O direito à prestação de alimentos é recípro-
na forma do art.1.638. co entre pais e filhos, e extensivo a todos os ascendentes,
A extinção do poder familiar pela morte do pai justifi- recaindo a obrigação nos mais próximos em grau, uns em
ca-se pelo motivo que na morte deste, desaparece o titular falta de outros.
de tal direito. Já na morte do filho, na sua emancipação
ou maioridade, faz desaparecer a razão do poder familiar, Art. 1.697. Na falta dos ascendentes cabe a obrigação
que é a proteção do menor. A adoção por sua vez extingue aos descendentes, guardada a ordem de sucessão e, fal-
o poder familiar quanto ao pai natural, o qual transfere o tando estes, aos irmãos, assim germanos como unilaterais.
pátrio poder ao pai adotivo.
Já a extinção por decisão judicial se dá na forma do art. Art. 1.698. Se o parente, que deve alimentos em pri-
1.638, nas seguintes hipóteses: meiro lugar, não estiver em condições de suportar total-
a) castigo imoderado ao filho; mente o encargo, serão chamados a concorrer os de grau
b) abandono do filho; imediato; sendo várias as pessoas obrigadas a prestar ali-
c) prática de atos contrários à moral e aos bons cos- mentos, todas devem concorrer na proporção dos respec-
tumes; tivos recursos, e, intentada ação contra uma delas, poderão
d) reiteração de faltas aos deveres inerentes ao poder as demais ser chamadas a integrar a lide.
familiar.
Nas infrações menos graves, pode o juiz aplicar sanção Art. 1.699. Se, fixados os alimentos, sobrevier mudança
de suspensão do poder familiar com o intuito de proteção na situação financeira de quem os supre, ou na de quem
ao menor. É temporária, pois tem duração somente quanto os recebe, poderá o interessado reclamar ao juiz, confor-
ao tempo necessário, de forma que, desaparecendo a cau- me as circunstâncias, exoneração, redução ou majoração
sa, os pais podem recuperar o poder familiar. do encargo.
Desta forma, o poder familiar em síntese é o dever dos
pais do momento do nascimento do filho até enquanto es- Art. 1.700. A obrigação de prestar alimentos transmite-
tes permanecerem menores, de criá-los e educá-los, nunca se aos herdeiros do devedor, na forma do art. 1.694.
de forma imoderada, administrando seus bens e nunca ul-
trapassando os limites da lei. O poder familiar não distin-
gue o filho concebido durante o casamento, ou fora deste

109
LEGISLAÇÃO

Art. 1.701. A pessoa obrigada a suprir alimentos poderá Os alimentos devem ser fixados na proporção das ne-
pensionar o alimentando, ou dar-lhe hospedagem e sus- cessidades do reclamante e dos recursos da pessoa obri-
tento, sem prejuízo do dever de prestar o necessário à sua gada.
educação, quando menor. Os alimentos serão apenas os indispensáveis à subsis-
Parágrafo único. Compete ao juiz, se as circunstâncias tência, quando a situação de necessidade resultar de culpa
o exigirem, fixar a forma do cumprimento da prestação. de quem os pleiteia.
São devidos quando quem os pretende não tem bens
Art. 1.702. Na separação judicial litigiosa, sendo um dos suficientes, nem pode prover, pelo seu trabalho, à própria
cônjuges inocente e desprovido de recursos, prestar-lhe-á mantença, e aquele, de quem se reclamam, pode fornecê
o outro a pensão alimentícia que o juiz fixar, obedecidos os -los, sem desfalque do necessário ao seu sustento.
critérios estabelecidos no art. 1.694. O direito à prestação de alimentos é recíproco entre
pais e filhos, e extensivo a todos os ascendentes, recain-
Art. 1.703. Para a manutenção dos filhos, os cônjuges do a obrigação nos mais próximos em grau, uns em falta
separados judicialmente contribuirão na proporção de de outros, Na falta dos ascendentes cabe a obrigação aos
seus recursos. descendentes, guardada a ordem de sucessão e, faltando
estes, aos irmãos.
Art. 1.704. Se um dos cônjuges separados judicialmen- Se o parente, que deve alimentos em primeiro lugar,
te vier a necessitar de alimentos, será o outro obrigado a não estiver em condições de suportar totalmente o encar-
prestá-los mediante pensão a ser fixada pelo juiz, caso não go, serão chamados a concorrer os de grau imediato; sen-
tenha sido declarado culpado na ação de separação judi- do várias as pessoas obrigadas a prestar alimentos, todas
cial. devem concorrer na proporção dos respectivos recursos,
Parágrafo único. Se o cônjuge declarado culpado vier a e, intentada ação contra uma delas, poderão as demais ser
necessitar de alimentos, e não tiver parentes em condições chamadas a integrar a lide.
de prestá-los, nem aptidão para o trabalho, o outro côn- Se, fixados os alimentos, sobrevier mudança na situa-
juge será obrigado a assegurá-los, fixando o juiz o valor ção financeira de quem os supre, ou na de quem os recebe,
indispensável à sobrevivência. poderá o interessado reclamar ao juiz, conforme as circuns-
tâncias, exoneração, redução ou majoração do encargo.
Art. 1.705. Para obter alimentos, o filho havido fora do Sendo, portanto que a obrigação de prestar alimentos
casamento pode acionar o genitor, sendo facultado ao juiz transmite-se aos herdeiros do devedor.
determinar, a pedido de qualquer das partes, que a ação se Para a manutenção dos filhos, os cônjuges separados
processe em segredo de justiça. judicialmente contribuirão na proporção de seus recursos.
Se um desses vier a necessitar de alimentos, será o outro
Art. 1.706. Os alimentos provisionais serão fixados pelo obrigado a prestá-los mediante pensão a ser fixada pelo
juiz, nos termos da lei processual. juiz. Com o casamento ou a união estável cessa o dever de
prestar alimentos do antigo companheiro ou companheira.
Art. 1.707. Pode o credor não exercer, porém lhe é ve- Para obter alimentos, o filho havido fora do casamento
dado renunciar o direito a alimentos, sendo o respectivo pode acionar o genitor, sendo facultado ao juiz determinar,
crédito insuscetível de cessão, compensação ou penhora. a pedido de qualquer das partes, que a ação se processe
em segredo de justiça.
Art. 1.708. Com o casamento, a união estável ou o con- As prestações alimentícias, de qualquer natureza, serão
cubinato do credor, cessa o dever de prestar alimentos. atualizadas segundo índice oficial regularmente estabele-
Parágrafo único. Com relação ao credor cessa, tam- cido.
bém, o direito a alimentos, se tiver procedimento indigno São hipóteses que tornam alguém obrigado a prestar
em relação ao devedor. alimentos:
- voluntários: por ato de vontade inter vivos (contrato)
Art. 1.709. O novo casamento do cônjuge devedor não ou causa mortis (testamento com legado de alimentos)
extingue a obrigação constante da sentença de divórcio. - indenizatórios: por ato ilícito, como causar a morte
ou lesão corporal. P.ex: atropelamento
Art. 1.710. As prestações alimentícias, de qualquer na- - legais: imposta por lei, sendo a que se aplica ao Di-
tureza, serão atualizadas segundo índice oficial regular- reito de Família.
mente estabelecido.
O foro competente para cobrança de alimentos é o do
Alimentos domicilio do alimentando, prescrevendo em 02 anos o di-
Podem os parentes, os cônjuges ou companheiros pe- reito desta cobrança, observando-se que tal prescrição não
dir uns aos outros os alimentos de que necessitem para corre contra os absolutamente incapazes.
viver de modo compatível com a sua condição social, inclu- Em não se cumprindo com o pagamento dos alimen-
sive para atender às necessidades de sua educação. tos, poderá a outra parte acionar o judiciário, através de
Trata-se de direito indisponível, intransacionável, impe- uma ação de execução de alimentos, para que o devedor
nhorável, incompensável, incessível, inerente à pessoa, que o faça forçosamente, se mesmo assim, não o fizer caberá
impõe o pagamento, in natura ou em espécie, de presta- prisão civil do alimentante ou a penhora de bens, podendo
ções aptas à manutenção da vida digna. ser cumulativos.

110
LEGISLAÇÃO

Nesta ação, poderá ser cobrados os alimentos devidos Art. 1.714. O bem de família, quer instituído pelos côn-
relativos a 3 meses anteriores e os seguintes a ação (sob juges ou por terceiro, constitui-se pelo registro de seu títu-
pena de prisão) e 21 meses anteriores aos anteriores descri- lo no Registro de Imóveis.
tos (sob pena de penhora).
Os alimentos em Direito de Família podem ser: Art. 1.715. O bem de família é isento de execução por
- naturais: o mínimo necessário para a subsistência da dívidas posteriores à sua instituição, salvo as que provie-
pessoa. rem de tributos relativos ao prédio, ou de despesas de con-
- civis: o necessário para manter o mesmo padrão de domínio.
vida da família. Parágrafo único. No caso de execução pelas dívidas
- provisionais: fixados em ação cautelar de alimentos referidas neste artigo, o saldo existente será aplicado em
provisionais, para garantir a sobrevivência do alimentando outro prédio, como bem de família, ou em títulos da dívida
durante o processo, e custear a demanda. pública, para sustento familiar, salvo se motivos relevantes
- provisórios: devidos a partir da citação em ação pelo aconselharem outra solução, a critério do juiz.
rito especial da Lei de Alimentos (Lei 5.478/68), havendo
prova pré-constituída do direito a alimentos. Art. 1.716. A isenção de que trata o artigo antecedente
- definitivos: fixados por sentença em ação de alimen- durará enquanto viver um dos cônjuges, ou, na falta destes,
tos, separação, anulação de casamento. até que os filhos completem a maioridade.
Vale ressaltar que não há obrigação alimentícia entre Art. 1.717. O prédio e os valores mobiliários, constituí-
parentes por afinidade. Obedecida à ordem preferencial, dos como bem da família, não podem ter destino diverso
não se pode escolher contra quem se quer cobrar alimentos. do previsto no art. 1.712 ou serem alienados sem o con-
Dentro da mesma categoria, cobra-se de todos. Havendo sentimento dos interessados e seus representantes legais,
mais de um alimentante, todos deverão pagar conforme
ouvido o Ministério Público.
suas possibilidades, ou seja, paga mais quem tem mais.
Art. 1.718. Qualquer forma de liquidação da entidade
SUBTÍTULO IV
administradora, a que se refere o § 3o do art. 1.713, não
Do Bem de Família
atingirá os valores a ela confiados, ordenando o juiz a sua
transferência para outra instituição semelhante, obedecen-
Art. 1.711. Podem os cônjuges, ou a entidade familiar,
mediante escritura pública ou testamento, destinar parte do-se, no caso de falência, ao disposto sobre pedido de
de seu patrimônio para instituir bem de família, desde que restituição.
não ultrapasse um terço do patrimônio líquido existente ao
tempo da instituição, mantidas as regras sobre a impenho- Art. 1.719. Comprovada a impossibilidade da manuten-
rabilidade do imóvel residencial estabelecida em lei especial. ção do bem de família nas condições em que foi instituído,
Parágrafo único. O terceiro poderá igualmente instituir poderá o juiz, a requerimento dos interessados, extingui-lo
bem de família por testamento ou doação, dependendo a ou autorizar a sub-rogação dos bens que o constituem em
eficácia do ato da aceitação expressa de ambos os cônjuges outros, ouvidos o instituidor e o Ministério Público.
beneficiados ou da entidade familiar beneficiada.
Art. 1.720. Salvo disposição em contrário do ato de ins-
Art. 1.712. O bem de família consistirá em prédio resi- tituição, a administração do bem de família compete a am-
dencial urbano ou rural, com suas pertenças e acessórios, bos os cônjuges, resolvendo o juiz em caso de divergência.
destinando-se em ambos os casos a domicílio familiar, e po- Parágrafo único. Com o falecimento de ambos os côn-
derá abranger valores mobiliários, cuja renda será aplicada juges, a administração passará ao filho mais velho, se for
na conservação do imóvel e no sustento da família. maior, e, do contrário, a seu tutor.

Art. 1.713. Os valores mobiliários, destinados aos fins Art. 1.721. A dissolução da sociedade conjugal não ex-
previstos no artigo antecedente, não poderão exceder o va- tingue o bem de família.
lor do prédio instituído em bem de família, à época de sua Parágrafo único. Dissolvida a sociedade conjugal pela
instituição. morte de um dos cônjuges, o sobrevivente poderá pedir a
§ 1o Deverão os valores mobiliários ser devidamente extinção do bem de família, se for o único bem do casal.
individualizados no instrumento de instituição do bem de
família. Art. 1.722. Extingue-se, igualmente, o bem de família
§ 2o Se se tratar de títulos nominativos, a sua instituição com a morte de ambos os cônjuges e a maioridade dos
como bem de família deverá constar dos respectivos livros filhos, desde que não sujeitos a curatela.
de registro.
§ 3o O instituidor poderá determinar que a administração O bem de família pode ser classificado em duas espé-
dos valores mobiliários seja confiada a instituição financeira, cies voluntário e legal.
bem como disciplinar a forma de pagamento da respectiva O bem de família voluntário é o instituído por ato de
renda aos beneficiários, caso em que a responsabilidade vontade do casal ou de entidade familiar, mediante for-
dos administradores obedecerá às regras do contrato de malização do registro de imóveis, deflagrando dois efeitos
depósito. fundamentais: impenhorabilidade limitada (significa que o

111
LEGISLAÇÃO

imóvel torna-se isento de dívidas futuras, salvo obrigações tada pela finalidade que a norteia, ou seja, a manutenção
tributárias referentes ao bem e despesas condominiais - da garantia de moradia, de subsistência e de respeito ao
art. 1.715) e inalienabilidade relativa (uma vez inscrito princípio constitucional da dignidade da pessoa humana.
como bem de família voluntário, ele só poderá ser alienado Desta forma, se o imóvel pertencente as pessoas solteiras,
com a autorização dos interessados, cabendo ao Ministério separadas ou viúvas tem por fim o exercício desse direito,
Público intervir quando houver participação de incapaz - conclui-se que à eles se aplica o conceito de bem de fa-
art. 1.717). mília.
Para evitar fraudes, ficou limitado o valor do bem de Neste sentido, o Superior Tribunal de Justiça tem fir-
família voluntário ao teto de 1/3 do patrimônio liquido de mado jurisprudência que pacifica o entendimento sobre
seus instituidores. Com relação ao bem de família legal , re- situações não previstas expressamente na lei, mas que
gulado pela Lei 8.009/90, diz respeito à impenhorabilidade são constantes na vida dos brasileiros. Imóvel habitado
legal do bem de família, independentemente de inscrição por irmão do dono ou por pessoa separada, único imó-
voluntária em cartório, e que convive com o bem de família vel alugado, penhorabilidade dos móveis dentro do imóvel
voluntário. Assim, se há duas casas, a proteção se dá na de impenhorável... Seja qual for a hipótese, o Tribunal da Ci-
menor valor, contudo, será protegida a de maior valor se dadania aplica a lei tendo em vista os fins sociais a que ela
se destina.
os proprietários a inscreverem como bem de família volun-
Sob esse enfoque, a lei do bem de família visa a pre-
tário. Ressalte-se que, esse bem de família não tem teto de
servar o devedor do constrangimento do despejo que o
valor.
relegue ao desabrigo. O entendimento levou o STJ a ga-
rantir o benefício da impenhorabilidade legal a pequenos
Art. 1º O imóvel residencial próprio do casal, ou da empreendimentos nitidamente familiares, cujos sócios são
entidade familiar, é impenhorável e não responderá por integrantes da família e, muitas vezes, o local de funciona-
qualquer tipo de dívida civil, comercial, fiscal, previ- mento confunde-se com a própria moradia.
denciária ou de outra natureza, contraída pelos cônju-
ges ou pelos pais ou filhos que sejam seus proprietários TÍTULO III
e nele residam, salvo nas hipóteses previstas nesta lei. DA UNIÃO ESTÁVEL
Parágrafo único. A impenhorabilidade compreen-
de o imóvel sobre o qual se assentam a construção, as Art. 1.723. É reconhecida como entidade familiar a
plantações, as benfeitorias de qualquer natureza e to- união estável entre o homem e a mulher, configurada na
dos os equipamentos, inclusive os de uso profissional, convivência pública, contínua e duradoura e estabelecida
ou móveis que guarnecem a casa, desde que quitados. com o objetivo de constituição de família.
§ 1o A união estável não se constituirá se ocorrerem os
Art. 3º A impenhorabilidade é oponível em qual- impedimentos do art. 1.521; não se aplicando a incidência
quer processo de execução civil, fiscal, previdenciária, do inciso VI no caso de a pessoa casada se achar separada
trabalhista ou de outra natureza , salvo se movido: de fato ou judicialmente.
I - em razão dos créditos de trabalhadores da pró- § 2o As causas suspensivas do art. 1.523 não impedirão
pria residência e das respectivas contribuições previ- a caracterização da união estável.
denciárias;
II - pelo titular do crédito decorrente do financia- Art. 1.724. As relações pessoais entre os companheiros
mento destinado à construção ou à aquisição do imó- obedecerão aos deveres de lealdade, respeito e assistência,
vel, no limite dos créditos e acréscimos constituídos em e de guarda, sustento e educação dos filhos.
função do respectivo contrato;
III - pelo credor de pensão alimentícia; Art. 1.725. Na união estável, salvo contrato escrito en-
tre os companheiros, aplica-se às relações patrimoniais, no
IV - para cobrança de impostos, predial ou territo-
que couber, o regime da comunhão parcial de bens.
rial, taxas e contribuições devidas em função do imóvel
familiar;
Art. 1.726. A união estável poderá converter-se em ca-
V - para execução de hipoteca sobre o imóvel ofe- samento, mediante pedido dos companheiros ao juiz e as-
recido como garantia real pelo casal ou pela entidade sento no Registro Civil.
familiar;
VI - por ter sido adquirido com produto de crime ou Art. 1.727. As relações não eventuais entre o homem
para execução de sentença penal condenatória a ressar- e a mulher, impedidos de casar, constituem concubinato.
cimento, indenização ou perdimento de bens.
VII - por obrigação decorrente de fiança concedida É reconhecida como entidade familiar a união estável
em contrato de locação. (grifos nossos) entre o homem e a mulher, configurada na convivência pú-
blica, contínua e duradoura e estabelecida com o objetivo
Em outubro de 2008, o Superior Tribunal de Justiça de constituição de família.
editou a Súmula 364 para estender o conceito de impe- A união estável não se constituirá se ocorrerem os im-
nhorabilidade de bem de família ao imóvel pertencente a pedimentos constantes no artigo 1.521; não se aplicando a
pessoas solteiras, separadas e viúvas. Afinal, a regra da im- incidência do inciso VI no caso de a pessoa casada se achar
penhorabilidade do bem de família deve ser sempre pau- separada de fato ou judicialmente.

112
LEGISLAÇÃO

Art. 1.521. Não podem casar: A união estável, porém, não dispõe de qualquer condi-
I - os ascendentes com os descendentes, seja o pa- cionante. Nasce do vínculo afetivo e se tem por constituída
rentesco natural ou civil; a partir do momento em que a relação se torna ostensiva,
II - os afins em linha reta; passando a ser reconhecida e aceita socialmente. Não há
III - o adotante com quem foi cônjuge do adotado e qualquer interferência estatal para sua formação, sendo
o adotado com quem o foi do adotante; inócuo tentar impor restrições ou impedimentos. Tanto
IV - os irmãos, unilaterais ou bilaterais, e demais co- é assim que as provas da existência da união estável são
laterais, até o terceiro grau inclusive; circunstanciais, dependem de testemunhas que saibam do
V - o adotado com o filho do adotante; relacionamento ou de documentos que tragam indícios de
VI - as pessoas casadas; sua vigência.
VII - o cônjuge sobrevivente com o condenado por Em se tratando de convivência pública, contínua e du-
homicídio ou tentativa de homicídio contra o seu con- radoura, impositivo o reconhecimento de sua existência. O
sorte. simples desatendimento a alguma das vedações impedi-
tivas do casamento, não subtrai da relação o objetivo de
constituição de família. Por exemplo, o relacionamento
As causas suspensivas do artigo 1.523 não impedirão a
do homicida com o cônjuge sobrevivente da vítima, que
caracterização da união estável.
atende a todos as exigências de longevidade, publicidade
e continuidade, não se pode ter por inexistente. A tentativa
Art. 1.523. Não devem casar: de impedir seu reconhecimento dispõe de nítido caráter
I - o viúvo ou a viúva que tiver filho do cônjuge fa- punitivo.
lecido, enquanto não fizer inventário dos bens do casal São limitações de caráter temporário, que não afetam
e der partilha aos herdeiros; a existência, a validade ou a eficácia do casamento. Como
II - a viúva, ou a mulher cujo casamento se desfez para estabelecer a união estável inexiste qualquer formali-
por ser nulo ou ter sido anulado, até dez meses depois dade, a remissão é absolutamente ilógica.
do começo da viuvez, ou da dissolução da sociedade A exceção aberta, autorizando o reconhecimento da
conjugal; união estável na hipótese de ser a pessoa casada, mas estar
III - o divorciado, enquanto não houver sido homo- separada de fato ou judicialmente, trata-se de verdadeira
logada ou decidida a partilha dos bens do casal; manobra legal para, a contrario sensu, excluir da figura jurí-
IV - o tutor ou o curador e os seus descendentes, dica da união estável o que a doutrina chama de concubi-
ascendentes, irmãos, cunhados ou sobrinhos, com a nato adulterino, ou impuro, ou concubinagem.
pessoa tutelada ou curatelada, enquanto não cessar a O fato de não haver qualquer menção à obrigação ali-
tutela ou curatela, e não estiverem saldadas as respec- mentar, não pode dar ensejo a que se pense que houve a
tivas contas. exclusão de dito direito. A referência do art. 1.694 – que fala
Parágrafo único. É permitido aos nubentes solicitar em conviventes – assegurando o direito a alimentos, bas-
ao juiz que não lhes sejam aplicadas as causas suspensi- ta para suprir a necessidade de uma melhor explicitação.
vas previstas nos incisos I, III e IV deste artigo, provan- Também não é estabelecida a presunção de colaboração
do-se a inexistência de prejuízo, respectivamente, para mútua na aquisição dos bens. Tal, no entanto, não pode
o herdeiro, para o ex-cônjuge e para a pessoa tutelada afastar o estado condominial e consequente partição igua-
ou curatelada; no caso do inciso II, a nubente deverá litária do patrimônio. Para isso é suficiente a determinação
provar nascimento de filho, ou inexistência de gravi- de aplicação supletiva do regime da comunhão parcial dos
dez, na fluência do prazo. bens feita no art. 1.725.
A promoção do cônjuge à condição de herdeiro neces-
sário é uma novidade. Porém, indevido excluir da parceria
As relações pessoais entre os companheiros obedece-
estável a sucessão necessária, condição a que o cônjuge
rão aos devedores de lealdade, respeito e assistência, e de
foi guindado pelo art. 1.845. De todo descabida, por con-
guarda, sustento e educação dos filhos. sequência, a disparidade de tratamento que resultou em
Na união estável, salvo contrato escrito entre os com- severas sequelas, dando margens a gritantes injustiças.
panheiros, aplica-se às relações patrimoniais, no que cou- O art. 1.829 estabelece que o cônjuge concorre em
ber, o regime da comunhão parcial de bens. iguais proporções com os descendentes, sejam filhos co-
A união estável poderá converter-se em casamento, muns ou filhos só do de cujus. Mas aos companheiros so-
mediante pedido dos companheiros ao juiz e assento no mente concede o mesmo direito se concorrerem com os
Registro Civil. filhos comuns. Limitando a concorrência à metade do qui-
As relações não eventuais entre o homem e a mulher, nhão, se os herdeiros forem filhos só do autor da herança,
impedidos de casar, constituem concubinato. distinção que não é feita quanto ao vínculo matrimonial.
Necessário ressaltar que há de se reconhecer como Como não integra o companheiro a ordem de vocação he-
pertinente e coerente haver a norma codificada, copiado reditária, concorre com os herdeiros sucessíveis, ou seja,
a definição da união estável já consolidada na legislação os colaterais até o quarto grau. Nessa hipótese, percebe
infraconstitucional. Sendo uma maneira de impor parâme- somente um terço da herança, ficando a maior parte (2/3)
tros objetivos para regular relações nascidas do afeto, aca- para sobrinhos netos ou primos do companheiro falecido.
baria deixando à margem do manto legal inúmeras situa- Tal não ocorre quando há casamento, pois o cônjuge ante-
ções que constituem entidades familiares dignas de tutela. cede os colaterais na ordem de vocação hereditária.

113
LEGISLAÇÃO

A realidade social aceitou as relações afetivas constituí- Art. 1.733. Aos irmãos órfãos dar-se-á um só tutor.
das fora do casamento. Outra não foi à saída da moderna § 1o No caso de ser nomeado mais de um tutor por
doutrina e da mais vanguardista jurisprudência senão bus- disposição testamentária sem indicação de precedência,
car um novo conceito de família. entende-se que a tutela foi cometida ao primeiro, e que os
Acabou por se definir família pela só presença de um outros lhe sucederão pela ordem de nomeação, se ocor-
vínculo afetivo. Mudaram os paradigmas da família. O ca- rer morte, incapacidade, escusa ou qualquer outro impe-
samento deixou de ser seu traço identificador. A entidade dimento.
familiar não mais tem por finalidade precípua e exclusiva a § 2o Quem institui um menor herdeiro, ou legatário seu,
função reprodutiva. Assim, imperativo incluir no Direito de poderá nomear-lhe curador especial para os bens deixa-
Família, como espécie do gênero união estável, as relações dos, ainda que o beneficiário se encontre sob o poder fa-
homossexuais, chamadas de uniões homoafetivas, e que, miliar, ou tutela.
tanto quanto as uniões heteroafetivas têm por razão de
existir o afeto entre os conviventes. Hoje a discriminação Art. 1.734.  As crianças e os adolescentes cujos pais
não é mais aceitável. Traduz puro preconceito de ordem forem desconhecidos, falecidos ou que tiverem sido sus-
sexual. pensos ou destituídos do poder familiar terão tutores no-
meados pelo Juiz ou serão incluídos em programa de co-
1728 A 1783. locação familiar, na forma prevista pela Lei no 8.069, de 13
de julho de 1990 - Estatuto da Criança e do Adolescente. 
TÍTULO IV
Da Tutela, da Curatela e da Tomada de Decisão Apoia- Seção II
da Dos Incapazes de Exercer a Tutela
(Redação dada pela Lei nº 13.146, de 2015)
CAPÍTULO I Art. 1.735. Não podem ser tutores e serão exonerados
Da Tutela da tutela, caso a exerçam:
Seção I I - aqueles que não tiverem a livre administração de
Dos Tutores seus bens;
II - aqueles que, no momento de lhes ser deferida a
Art. 1.728. Os filhos menores são postos em tutela: tutela, se acharem constituídos em obrigação para com o
I - com o falecimento dos pais, ou sendo estes julgados menor, ou tiverem que fazer valer direitos contra este, e
ausentes; aqueles cujos pais, filhos ou cônjuges tiverem demanda
II - em caso de os pais decaírem do poder familiar. contra o menor;
III - os inimigos do menor, ou de seus pais, ou que
Art. 1.729. O direito de nomear tutor compete aos pais, tiverem sido por estes expressamente excluídos da tutela;
em conjunto. IV - os condenados por crime de furto, roubo, estelio-
Parágrafo único. A nomeação deve constar de testa- nato, falsidade, contra a família ou os costumes, tenham ou
mento ou de qualquer outro documento autêntico. não cumprido pena;
V - as pessoas de mau procedimento, ou falhas em
Art. 1.730. É nula a nomeação de tutor pelo pai ou pela probidade, e as culpadas de abuso em tutorias anteriores;
mãe que, ao tempo de sua morte, não tinha o poder fami- VI - aqueles que exercerem função pública incompatí-
liar. vel com a boa administração da tutela.

Art. 1.731. Em falta de tutor nomeado pelos pais in- Seção III
cumbe a tutela aos parentes consanguíneos do menor, por Da Escusa dos Tutores
esta ordem:
I - aos ascendentes, preferindo o de grau mais próximo Art. 1.736. Podem escusar-se da tutela:
ao mais remoto; I - mulheres casadas;
II - aos colaterais até o terceiro grau, preferindo os II - maiores de sessenta anos;
mais próximos aos mais remotos, e, no mesmo grau, os III - aqueles que tiverem sob sua autoridade mais de
mais velhos aos mais moços; em qualquer dos casos, o juiz três filhos;
escolherá entre eles o mais apto a exercer a tutela em be- IV - os impossibilitados por enfermidade;
nefício do menor. V - aqueles que habitarem longe do lugar onde se haja
de exercer a tutela;
Art. 1.732. O juiz nomeará tutor idôneo e residente no VI - aqueles que já exercerem tutela ou curatela;
domicílio do menor: VII - militares em serviço.
I - na falta de tutor testamentário ou legítimo;
II - quando estes forem excluídos ou escusados da tu- Art. 1.737. Quem não for parente do menor não poderá
tela; ser obrigado a aceitar a tutela, se houver no lugar parente
III - quando removidos por não idôneos o tutor legíti- idôneo, consanguíneo ou afim, em condições de exercê-la.
mo e o testamentário.

114
LEGISLAÇÃO

Art. 1.738. A escusa apresentar-se-á nos dez dias sub- Art. 1.747. Compete mais ao tutor:
sequentes à designação, sob pena de entender-se renun- I - representar o menor, até os dezesseis anos, nos atos
ciado o direito de alegá-la; se o motivo escusatório ocorrer da vida civil, e assisti-lo, após essa idade, nos atos em que
depois de aceita a tutela, os dez dias contar-se-ão do em for parte;
que ele sobrevier. II - receber as rendas e pensões do menor, e as quan-
tias a ele devidas;
Art. 1.739. Se o juiz não admitir a escusa, exercerá o III - fazer-lhe as despesas de subsistência e educação,
nomeado a tutela, enquanto o recurso interposto não tiver bem como as de administração, conservação e melhora-
provimento, e responderá desde logo pelas perdas e danos mentos de seus bens;
que o menor venha a sofrer. IV - alienar os bens do menor destinados a venda;
V - promover-lhe, mediante preço conveniente, o ar-
Seção IV rendamento de bens de raiz.
Do Exercício da Tutela
Art. 1.748. Compete também ao tutor, com autorização
Art. 1.740. Incumbe ao tutor, quanto à pessoa do me- do juiz:
nor: I - pagar as dívidas do menor;
I - dirigir-lhe a educação, defendê-lo e prestar-lhe ali- II - aceitar por ele heranças, legados ou doações, ainda
mentos, conforme os seus haveres e condição; que com encargos;
II - reclamar do juiz que providencie, como houver por III - transigir;
bem, quando o menor haja mister correção; IV - vender-lhe os bens móveis, cuja conservação não
III - adimplir os demais deveres que normalmente ca- convier, e os imóveis nos casos em que for permitido;
bem aos pais, ouvida a opinião do menor, se este já contar V - propor em juízo as ações, ou nelas assistir o menor,
doze anos de idade. e promover todas as diligências a bem deste, assim como
defendê-lo nos pleitos contra ele movidos.
Art. 1.741. Incumbe ao tutor, sob a inspeção do juiz, Parágrafo único. No caso de falta de autorização, a efi-
administrar os bens do tutelado, em proveito deste, cum- cácia de ato do tutor depende da aprovação ulterior do
prindo seus deveres com zelo e boa-fé. juiz.

Art. 1.742. Para fiscalização dos atos do tutor, pode o Art. 1.749. Ainda com a autorização judicial, não pode
juiz nomear um protutor. o tutor, sob pena de nulidade:
I - adquirir por si, ou por interposta pessoa, mediante
Art. 1.743. Se os bens e interesses administrativos exi- contrato particular, bens móveis ou imóveis pertencentes
girem conhecimentos técnicos, forem complexos, ou reali- ao menor;
zados em lugares distantes do domicílio do tutor, poderá II - dispor dos bens do menor a título gratuito;
este, mediante aprovação judicial, delegar a outras pessoas III - constituir-se cessionário de crédito ou de direito,
físicas ou jurídicas o exercício parcial da tutela. contra o menor.

Art. 1.744. A responsabilidade do juiz será: Art. 1.750. Os imóveis pertencentes aos menores sob
I - direta e pessoal, quando não tiver nomeado o tutor, tutela somente podem ser vendidos quando houver mani-
ou não o houver feito oportunamente; festa vantagem, mediante prévia avaliação judicial e apro-
II - subsidiária, quando não tiver exigido garantia legal vação do juiz.
do tutor, nem o removido, tanto que se tornou suspeito.
Art. 1.751. Antes de assumir a tutela, o tutor declarará
Art. 1.745. Os bens do menor serão entregues ao tutor tudo o que o menor lhe deva, sob pena de não lhe poder
mediante termo especificado deles e seus valores, ainda cobrar, enquanto exerça a tutoria, salvo provando que não
que os pais o tenham dispensado. conhecia o débito quando a assumiu.
Parágrafo único. Se o patrimônio do menor for de valor
considerável, poderá o juiz condicionar o exercício da tute- Art. 1.752. O tutor responde pelos prejuízos que, por
la à prestação de caução bastante, podendo dispensá-la se culpa, ou dolo, causar ao tutelado; mas tem direito a ser
o tutor for de reconhecida idoneidade. pago pelo que realmente despender no exercício da tutela,
salvo no caso do art. 1.734, e a perceber remuneração pro-
Art. 1.746. Se o menor possuir bens, será sustentado e porcional à importância dos bens administrados.
educado a expensas deles, arbitrando o juiz para tal fim as § 1o Ao protutor será arbitrada uma gratificação módi-
quantias que lhe pareçam necessárias, considerado o ren- ca pela fiscalização efetuada.
dimento da fortuna do pupilo quando o pai ou a mãe não § 2o São solidariamente responsáveis pelos prejuízos as
as houver fixado. pessoas às quais competia fiscalizar a atividade do tutor, e
as que concorreram para o dano.

115
LEGISLAÇÃO

Seção V Art. 1.759. Nos casos de morte, ausência, ou interdição


Dos Bens do Tutelado do tutor, as contas serão prestadas por seus herdeiros ou
representantes.
Art. 1.753. Os tutores não podem conservar em seu
poder dinheiro dos tutelados, além do necessário para as Art. 1.760. Serão levadas a crédito do tutor todas as
despesas ordinárias com o seu sustento, a sua educação e a despesas justificadas e reconhecidamente proveitosas ao
administração de seus bens. menor.
§ 1o Se houver necessidade, os objetos de ouro e prata,
pedras preciosas e móveis serão avaliados por pessoa idô- Art. 1.761. As despesas com a prestação das contas se-
nea e, após autorização judicial, alienados, e o seu produto rão pagas pelo tutelado.
convertido em títulos, obrigações e letras de responsabili-
dade direta ou indireta da União ou dos Estados, atenden- Art. 1.762. O alcance do tutor, bem como o saldo con-
do-se preferentemente à rentabilidade, e recolhidos ao es- tra o tutelado, são dívidas de valor e vencem juros desde o
tabelecimento bancário oficial ou aplicado na aquisição de julgamento definitivo das contas.
imóveis, conforme for determinado pelo juiz.
§ 2o O mesmo destino previsto no parágrafo antece- Seção VII
dente terá o dinheiro proveniente de qualquer outra pro- Da Cessação da Tutela
cedência.
§ 3o Os tutores respondem pela demora na aplicação Art. 1.763. Cessa a condição de tutelado:
dos valores acima referidos, pagando os juros legais desde I - com a maioridade ou a emancipação do menor;
o dia em que deveriam dar esse destino, o que não os exime II - ao cair o menor sob o poder familiar, no caso de
da obrigação, que o juiz fará efetiva, da referida aplicação. reconhecimento ou adoção.

Art. 1.754. Os valores que existirem em estabelecimen- Art. 1.764. Cessam as funções do tutor:
to bancário oficial, na forma do artigo antecedente, não se I - ao expirar o termo, em que era obrigado a servir;
poderão retirar, senão mediante ordem do juiz, e somente: II - ao sobrevir escusa legítima;
III - ao ser removido.
I - para as despesas com o sustento e educação do tu-
telado, ou a administração de seus bens;
Art. 1.765. O tutor é obrigado a servir por espaço de
II - para se comprarem bens imóveis e títulos, obriga-
dois anos.
ções ou letras, nas condições previstas no § 1o do artigo
Parágrafo único. Pode o tutor continuar no exercício da
antecedente;
tutela, além do prazo previsto neste artigo, se o quiser e o
III - para se empregarem em conformidade com o dis-
juiz julgar conveniente ao menor.
posto por quem os houver doado, ou deixado;
IV - para se entregarem aos órfãos, quando emancipa- Art. 1.766. Será destituído o tutor, quando negligente,
dos, ou maiores, ou, mortos eles, aos seus herdeiros. prevaricador ou incurso em incapacidade.
Seção VI A tutela constitui instituto do direito assistencial de
Da Prestação de Contas família, que existe para preencher um espaço vazio, qual
seja a falta da autoridade parental. Tem sua justificativa no
Art. 1.755. Os tutores, embora o contrário tivessem dis- interesse da criança e do adolescente menor de 18 anos e
posto os pais dos tutelados, são obrigados a prestar contas não emancipado que, sem os pais, seja porque órfãos, seja
da sua administração. em face de possível perda da autoridade parental, carecem
de administração.
Art. 1.756. No fim de cada ano de administração, os tu- A principal característica da tutela, é a supressão da
tores submeterão ao juiz o balanço respectivo, que, depois falta de capacidade de menores aos quais tenham os pais
de aprovado, se anexará aos autos do inventário. falecido ou encontra-se ausentes ou estejam destituídos
do pátrio poder
Art. 1.757. Os tutores prestarão contas de dois em dois Tal instituto encontra também abrigo na Lei nº.8.069/90
anos, e também quando, por qualquer motivo, deixarem o - Estatuto da Criança e do Adolescente, como sucedâneo
exercício da tutela ou toda vez que o juiz achar conveniente. do poder familiar, porém, dele se difere, pois há limitações
Parágrafo único. As contas serão prestadas em juízo, e e obrigações legais impostas ao tutor.
julgadas depois da audiência dos interessados, recolhendo No mais, a lei e consequentemente a doutrina classifi-
o tutor imediatamente a estabelecimento bancário oficial cam em três as espécies de tutela: testamentária, legítima
os saldos, ou adquirindo bens imóveis, ou títulos, obriga- e dativa.
ções ou letras, na forma do § 1o do art. 1.753. A testamentária é aquela em que os pais no exercício
do poder familiar, nomeiam por testamento ou por outro
Art. 1.758. Finda a tutela pela emancipação ou maiori- documento autêntico, tutor para a sua prole. Este docu-
dade, a quitação do menor não produzirá efeito antes de mento pode ser por escritura pública ou particular, desde
aprovadas as contas pelo juiz, subsistindo inteira, até en- que as assinaturas dos pais estejam reconhecidas por tabe-
tão, a responsabilidade do tutor. lião, que lhes confira autenticidade.

116
LEGISLAÇÃO

Tal nomeação testamentária de tutor, deve ser realiza- CAPÍTULO II


da pelos pais seus genitores, de forma consensual, pois, a Da Curatela
disposição de última vontade de um deles não pode so- Seção I
brepor a vontade do outro. Se um dos pais for falecido Dos Interditos
ou tenha sido destituído do poder familiar, o outro poderá
fazer unilateralmente tal nomeação. Art. 1.767. Estão sujeitos a curatela:
A nomeação de tutor aos filhos, estenderá seus efeitos I - aqueles que, por causa transitória ou permanente,
apenas após a morte dos testamenteiros, e desde que estes não puderem exprimir sua vontade; (Redação dada pela Lei
detiverem o poder familiar. Caso não seja observado este nº 13.146, de 2015) (Vigência)
requisito, é nula a nomeação do tutor. II - (Revogado); (Redação dada pela Lei nº 13.146, de
A tutela legitima é aquela que na falta da testamentá- 2015) (Vigência)
ria, a lei incumbe aos parentes consanguíneos do menor o III - os ébrios habituais e os viciados em tóxico; (Reda-
dever de tutela. A ordem preferencial de nomeação está ção dada pela Lei nº 13.146, de 2015) (Vigência)
elencada no artigo 1.731, impondo o encargo aos ascen- IV - (Revogado); (Redação dada pela Lei nº 13.146, de
dentes e aos colaterais até o terceiro grau, preferindo os 2015) (Vigência)
mais próximos aos mais remotos e os mais velhos aos mais V - os pródigos.
moços, quando do mesmo grau.
Mas é importante salientar que o Magistrado não está Art. 1.768. O processo que define os termos da cura-
vinculado a obedecer esta ordem, vez que a finalidade da tela deve ser promovido: (Redação dada pela Lei nº 13.146,
tutela é atender o melhor interesse do menor, buscando de 2015) (Vigência)
sempre o seu bem estar. I - pelos pais ou tutores; (Vide Lei nº 13.146, de 2015)
A terceira espécies de tutela que a lei contempla é a (Vigência)
chamada dativa. Esta, porém, só poderá ser aplicada subsi- II - pelo cônjuge, ou por qualquer parente;
diariamente as duas anteriores. Desta forma, se não houve III - pelo Ministério Público. (Vide Lei nº 13.146, de
tutela testamentária por parte dos pais e tampouco foi en- 2015) (Vigência)
contrado algum parente do menor em condições de pres- IV - pela própria pessoa. (Incluído pela Lei nº 13.146,
tar-lhe a tutela, ou ainda, quando os incumbidos se escu- de 2015) (Vigência)
saram ou foram excluídos ou removidos da tutela, deverá
então o juiz nomear pessoa idônea. Art. 1.769. O Ministério Público somente promoverá o
Temos também a tutela prevista no artigo 1.734, desti- processo que define os termos da curatela: (Redação dada
nada aos menores abandonados, por decisão judicial, este pela Lei nº 13.146, de 2015) (Vigência)
deverá ser recolhido a estabelecimento público, ou lhe no- I - nos casos de deficiência mental ou intelectual; (Re-
meado tutor, o menor que se encontrar em situação de dação dada pela Lei nº 13.146, de 2015) (Vigência)
abandono. II - se não existir ou não promover a interdição alguma
Nos casos de destituição ou suspensão do poder fa- das pessoas designadas nos incisos I e II do artigo antece-
miliar, falecimento dos pais, necessário se faz a nomeação dente;
de tutor e a inserção do menor em família substituta. A lei III - se, existindo, forem menores ou incapazes as pes-
prevê também que em caso de irmãos órfãos dar-se-á um soas mencionadas no inciso II. (Redação dada pela Lei nº
só tutor. 13.146, de 2015) (Vigência)
A tutela é um encargo legal de ordem pública, mas esta
obrigação poderá ser recusada, desde que fundamentadas Art. 1.770. Nos casos em que a interdição for promovi-
e justificadas ao juiz. da pelo Ministério Público, o juiz nomeará defensor ao su-
É dever do tutor zelar pela boa administração dos bens posto incapaz; nos demais casos o Ministério Público será
do menor, conservando-os e melhorando-os, e o juiz, o de o defensor.
inspecionar sua conduta. Mas se o juiz julgar necessário,
poderá nomear um protutor, que terá a incumbência de Art. 1.771. Antes de se pronunciar acerca dos termos
fiscalizar os atos do tutor. da curatela, o juiz, que deverá ser assistido por equipe mul-
Desde que autorizado, judicialmente, pagar dívidas tidisciplinar, entrevistará pessoalmente o interditando. (Re-
do pupilo; aceitar heranças; legados ou doações em nome dação dada pela Lei nº 13.146, de 2015) (Vigência)
dele; transigir; vender bens móveis cuja conservação não
for conveniente e os imóveis quando permitido; represen- Art. 1.772. O juiz determinará, segundo as potencia-
tar ou assistir o pupilo em ações judiciais. lidades da pessoa, os limites da curatela, circunscritos às
Para exercer a tutela é necessário que o tutor tenha a restrições constantes do art. 1.782, e indicará curador. (Re-
sua idoneidade incólume, bem como não haja nenhuma dação dada pela Lei nº 13.146, de 2015) (Vigência)
espécie de conflito entre o ele e o seu pupilo. Parágrafo único. Para a escolha do curador, o juiz le-
A tutela cessa quando o tutelado atingir a maioridade, vará em conta a vontade e as preferências do interditando,
adquirindo a plena capacidade civil; quando o menor tute- a ausência de conflito de interesses e de influência indevi-
lado for emancipado; ou quando for reconhecido, pelo pai, da, a proporcionalidade e a adequação às circunstâncias
como filho ou, ainda, quando o menor for adotado. da pessoa. (Incluído pela Lei nº 13.146, de 2015) (Vigência)

117
LEGISLAÇÃO

Art. 1.773. A sentença que declara a interdição produz Art. 1.783. Quando o curador for o cônjuge e o regi-
efeitos desde logo, embora sujeita a recurso. me de bens do casamento for de comunhão universal, não
será obrigado à prestação de contas, salvo determinação
Art. 1.774. Aplicam-se à curatela as disposições concer- judicial.
nentes à tutela, com as modificações dos artigos seguintes.
A curatela constitui instituto mais amplo a se dirigir
Art. 1.775. O cônjuge ou companheiro, não separado não apenas aos demais incapazes, absolutos ou relativos,
judicialmente ou de fato, é, de direito, curador do outro, maiores de dezoito anos, como ainda ao nascituro, ao au-
quando interdito. sente, ao revel citado por edital, entre outras opções.
§1º Na falta do cônjuge ou companheiro, é curador le- É o encargo deferido por lei a alguém capaz, para reger
gítimo o pai ou a mãe; na falta destes, o descendente que a pessoa e administrar os bens de quem, em regra maior,
se demonstrar mais apto. não pode fazê-lo por si mesmo.
§ 2º Entre os descendentes, os mais próximos prece- A curatela assemelha-se à tutela por seu caráter as-
dem aos mais remotos. sistencial, destinando-se, igualmente, à proteção de inca-
§ 3º Na falta das pessoas mencionadas neste artigo, pazes. Por essa razão, a ela são aplicáveis as disposições
legais relativas à tutela, com apenas algumas modificações.
compete ao juiz a escolha do curador.
Vigoram para o curador as escusas voluntárias e proi-
bitórias; é obrigado a prestar caução, quando exigida pelo
Art. 1.775-A. Na nomeação de curador para a pessoa
Juiz, e a prestar contas; cabem-lhe os direitos e deveres es-
com deficiência, o juiz poderá estabelecer curatela com- pecificados no capítulo que trata da tutela; somente pode
partilhada a mais de uma pessoa. (Incluído pela Lei nº alienar bens imóveis mediante prévia avaliação judicial e
13.146, de 2015) (Vigência) autorização do juiz etc.
Apesar dessa semelhança, os dois institutos não se
Art. 1.776. (Revogado pela Lei nº 13.146, de 2015) (Vi- confundem. Podem ser apontadas as seguintes diferenças:
gência) - a tutela é destinada à menores de 18 anos de idade,
enquanto a curatela é deferida, em regra, à maiores;
Art. 1.777. As pessoas referidas no inciso I do art. 1.767 - a tutela pode ser testamentária, com nomeação do
receberão todo o apoio necessário para ter preservado o tutor pelos pais; a curatela é sempre deferida pelo juiz;
direito à convivência familiar e comunitária, sendo evita- - a tutela abrange a pessoa e os bens do menor, en-
do o seu recolhimento em estabelecimento que os afaste quanto a curatela pode compreender somente a adminis-
desse convívio. (Redação dada pela Lei nº 13.146, de 2015) tração dos bens do incapaz, como no caso dos pródigos;
(Vigência) - os poderes do curador são mais restritos do que os
do tutor.
Art. 1.778. A autoridade do curador estende-se à pes- A curatela apresenta cinco características:
soa e aos bens dos filhos do curatelado, observado o art. - seus fins são assistenciais – destinados aos que não po-
5º. dem por si só regerem sua pessoa e administrar seus bens;
- tem caráter eminentemente publicista – advém do
Seção II fato de ser dever do Estado zelar pelos interesses dos inca-
Da Curatela do Nascituro e do Enfermo ou Portador de pazes, este dever, no entanto, é delegado a pessoas capa-
Deficiência Física zes e idôneas, que passam a exercer um múnus público, ao
serem nomeadas curadoras;
Art. 1.779. Dar-se-á curador ao nascituro, se o pai fale- - tem, também, caráter supletivo da capacidade – o
cer estando grávida a mulher, e não tendo o poder familiar. curador tem o encargo de representar ou assistir o seu
curatelado, cabendo em todos os casos de incapacidade
Parágrafo único. Se a mulher estiver interdita, seu cura-
não suprida pela tutela;
dor será o do nascituro.
- é temporária – perdurando somente enquanto a cau-
sa da incapacidade se mantiver (cessada a causal, levanta-
Art. 1.780. (Revogado pela Lei nº 13.146, de 2015) (Vi- se a interdição);
gência) - a sua decretação requer certeza absoluta da incapa-
cidade.
Seção III O art. 3º do Código Civil menciona os absolutamente
Do Exercício da Curatela incapazes de exercer pessoalmente os seus direitos e que de-
vem ser representados, sob pena de nulidade do ato (art. 166, I).
Art. 1.781. As regras a respeito do exercício da tutela Art. 3o São absolutamente incapazes de exercer pes-
aplicam-se ao da curatela, com a restrição do art. 1.772 e soalmente os atos da vida civil:
as desta Seção. I - os menores de dezesseis anos;
II - os que, por enfermidade ou deficiência mental, não
Art. 1.782. A interdição do pródigo só o privará de, sem tiverem o necessário discernimento para a prática desses
curador, emprestar, transigir, dar quitação, alienar, hipote- atos;
car, demandar ou ser demandado, e praticar, em geral, os III - os que, mesmo por causa transitória, não puderem
atos que não sejam de mera administração. exprimir sua vontade.

118
LEGISLAÇÃO

Art. 166. É nulo o negócio jurídico quando: § 1º Para formular pedido de tomada de decisão apoia-
I - celebrado por pessoa absolutamente incapaz; da, a pessoa com deficiência e os apoiadores devem apre-
E o art. 4º enumera os relativamente incapazes, do- sentar termo em que constem os limites do apoio a ser
tados de algum discernimento e por isso, autorizados a oferecido e os compromissos dos apoiadores, inclusive o
participar dos atos jurídicos de seu interesse, desde que prazo de vigência do acordo e o respeito à vontade, aos
devidamente assistidos por seus representantes legais, sob direitos e aos interesses da pessoa que devem apoiar. (In-
pena de anulabilidade (art. 171, I), salvo algumas hipóteses cluído pela Lei nº 13.146, de 2015) (Vigência)
restritas em que se lhes permitem atuarem sozinhos. § 2º O pedido de tomada de decisão apoiada será re-
Art. 4o São incapazes, relativamente a certos atos, ou à querido pela pessoa a ser apoiada, com indicação expressa
maneira de os exercer: das pessoas aptas a prestarem o apoio previsto no caput
I - os maiores de dezesseis e menores de dezoito anos; deste artigo. (Incluído pela Lei nº 13.146, de 2015) (Vigên-
II - os ébrios habituais, os viciados em tóxicos, e os que, cia)
por deficiência mental, tenham o discernimento reduzido; § 3º Antes de se pronunciar sobre o pedido de tomada
III - os excepcionais, sem desenvolvimento mental de decisão apoiada, o juiz, assistido por equipe multidis-
completo; ciplinar, após oitiva do Ministério Público, ouvirá pessoal-
mente o requerente e as pessoas que lhe prestarão apoio.
IV - os pródigos.
(Incluído pela Lei nº 13.146, de 2015) (Vigência)
Parágrafo único. A capacidade dos índios será regulada
§ 4º A decisão tomada por pessoa apoiada terá valida-
por legislação especial.
de e efeitos sobre terceiros, sem restrições, desde que es-
Art. 171. Além dos casos expressamente declarados na teja inserida nos limites do apoio acordado. (Incluído pela
lei, é anulável o negócio jurídico: Lei nº 13.146, de 2015) (Vigência)
I - por incapacidade relativa do agente; § 5º Terceiro com quem a pessoa apoiada mantenha
Assim como na tutela, a curatela pressupõe o devido relação negocial pode solicitar que os apoiadores contra
processo legal judicial a ser promovida pelos pais ou tuto- -assinem o contrato ou acordo, especificando, por escrito,
res, pelo cônjuge, por qualquer parente, ou pelo Ministério sua função em relação ao apoiado. (Incluído pela Lei nº
Público Estadual. 13.146, de 2015) (Vigência)
A legitimidade ativa do MP é condicionada as questões § 6º Em caso de negócio jurídico que possa trazer ris-
específicas sobre as quais deve a aludida Instituição atuar, co ou prejuízo relevante, havendo divergência de opiniões
como, por exemplo, em casos de doença mental grave, ou entre a pessoa apoiada e um dos apoiadores, deverá o juiz,
se ninguém requerer a interdição, ou então, se a interdição ouvido o Ministério Público, decidir sobre a questão. (In-
versar sobre incapazes. cluído pela Lei nº 13.146, de 2015) (Vigência)
Dentre as curadorias especiais podem ser menciona- § 7º Se o apoiador agir com negligência, exercer pres-
das: são indevida ou não adimplir as obrigações assumidas,
a) a instituída pelo testador para os bens deixados a poderá a pessoa apoiada ou qualquer pessoa apresentar
herdeiro ou legatário menor; denúncia ao Ministério Público ou ao juiz. (Incluído pela Lei
b) a que se dá à herança jacente; nº 13.146, de 2015) (Vigência)
c) a que se dá ao filho, sempre que no exercício do po- § 8º Se procedente a denúncia, o juiz destituirá o apoia-
der familiar colidirem os interesses do pai com os daquele; dor e nomeará, ouvida a pessoa apoiada e se for de seu
d) a dada ao incapaz que não tiver representante legal interesse, outra pessoa para prestação de apoio. (Incluído
ou, se o tiver, seus interesses conflitarem com os daqueles; pela Lei nº 13.146, de 2015) (Vigência)
e) a conferida ao réu preso; § 9º A pessoa apoiada pode, a qualquer tempo, solici-
f) a que se dá ao revel citado por edital ou com hora tar o término de acordo firmado em processo de tomada
certa, que se fizer revel. de decisão apoiada. (Incluído pela Lei nº 13.146, de 2015)
(Vigência)
Se o interditado restabelecer a saúde psíquica, poderá
§ 10. O apoiador pode solicitar ao juiz a exclusão de
formalizar judicialmente o pedido de levantamento da in-
sua participação do processo de tomada de decisão apoia-
terdição, com isso poderá cessar a curatela.
da, sendo seu desligamento condicionado à manifestação
do juiz sobre a matéria. (Incluído pela Lei nº 13.146, de
CAPÍTULO III 2015) (Vigência)
Da Tomada de Decisão Apoiada § 11. Aplicam-se à tomada de decisão apoiada, no que
(Incluído pela Lei nº 13.146, de 2015) (Vigência) couber, as disposições referentes à prestação de contas na
curatela. (Incluído pela Lei nº 13.146, de 2015) (Vigência)
Art. 1.783-A. A tomada de decisão apoiada é o proces-
so pelo qual a pessoa com deficiência elege pelo menos 2 A tomada de decisão apoiada é a maneira encontrada
(duas) pessoas idôneas, com as quais mantenha vínculos para dar maior proteção à pessoa com deficiência, se dá
e que gozem de sua confiança, para prestar-lhe apoio na através de um processo pelo qual a pessoa com deficiên-
tomada de decisão sobre atos da vida civil, fornecendo- cia elege pelo menos duas pessoas idôneas, com as quais
lhes os elementos e informações necessários para que pos- mantenha vínculos e que gozem de sua confiança, para
sa exercer sua capacidade. (Incluído pela Lei nº 13.146, de prestar-lhe apoio na tomada de decisão sobre atos da vida
2015) (Vigência) civil, fornecendo-lhes os elementos e informações necessá-
rios para que possa exercer sua capacidade.

119
LEGISLAÇÃO

Este procedimento de dá pela apresentação de um (D) A guarda compartilhada dos filhos poderá ser re-
termo em que constem os limites do apoio a ser oferecido querida pelos pais em consenso, mas não poderá ser de-
e os compromissos dos apoiadores, inclusive o prazo de cretada pelo juiz;
vigência do acordo e o respeito à vontade, aos direitos e (E) O direito de visita não pode se estender aos avós.
aos interesses da pessoa que devem apoiar. O pedido de
tomada de decisão apoiada será requerido pela pessoa a 04. (MPE/RJ - Analista do Ministério Público – Pro-
ser apoiada, com indicação expressa das pessoas aptas a cessual - FGV/2016). Felícia, dezenove anos de idade,
prestarem o apoio previsto no caput deste artigo. após ter sido criada por sua tia Deise desde que tinha qua-
Esta decisão tomada por pessoa apoiada terá validade tro anos de idade, foi adotada por ela em procedimento ao
e efeitos sobre terceiros, sem restrições, desde que esteja qual os pais biológicos não anuíram. É correto afirmar que
inserida nos limites do apoio acordado. a adoção em questão é ato:
O seu término se dá a qualquer tempo, solicitar o tér- (A) inexistente, já que é imprescindível, na hipótese, a
mino de acordo firmado em processo de tomada de deci- concordância dos pais biológicos;
são apoiada. O apoiador pode solicitar ao juiz a exclusão (B) nulo, já que é imprescindível, na hipótese, a concor-
de sua participação do processo de tomada de decisão dância dos pais biológicos;
apoiada, sendo seu desligamento condicionado à manifes- (C) nulo, já que é imprescindível, na hipótese, ao menos
tação do juiz sobre a matéria. a concordância da mãe biológica;
(D) válido, já que não há exigência legal quanto à con-
Questões cordância dos pais biológicos para o ato em questão;
(E) anulável, já que é imprescindível, na hipótese, a
01. (UECE – Advogado - FUNECE/2017). Consideran- concordância dos pais biológicos..
do os impedimentos ao matrimônio elencados no Código
Civil Brasileiro, NÃO pode(m) casar: 05. (TJ/RS - Psicólogo Judiciário - FAURGS/2016).
(A) o adotante com quem foi cônjuge do adotado e o A legislação expressa no Código Civil referente aos inter-
adotado com quem o foi do adotante. ditos define condições dos indivíduos sujeitos à curatela
(B) o viúvo ou a viúva que tiver filho do cônjuge faleci- (art. 1.767).
do, enquanto não fizer inventário dos bens do casal e der Assinale a alternativa que NÃO apresenta uma dessas
partilha aos herdeiros. condições previstas e descritas no referido artigo.
(C) o tutor ou o curador e os seus descendentes, as- (A) Ébrios habituais.
cendentes, irmãos, cunhados ou sobrinhos, com a pessoa (B) Pródigos.
tutelada ou curatelada, enquanto não cessar a tutela ou (C) Quem, por causa transitória ou permanente, não
curatela, e não estiverem saldadas as respectivas contas. puder exprimir sua vontade.
(D) o divorciado, enquanto não houver sido homologa- (D) Viciados em tóxico.
da ou decidida a partilha dos bens do casal. (E) Filhos menores de 18 anos.

02. (MPE/RS - Secretário de Diligências - MPE/ Respostas:


RS/2017). Assinale a alternativa correta acerca dos precei- 01. A
tos alusivos ao casamento, nos termos do Código Civil.
(A) É anulável o casamento do incapaz de consentir ou 02. A
manifestar, de modo inequívoco, o consentimento. 03. C
(B) A direção da sociedade conjugal será exercida, pre- 04. D
ferencialmente, pelo marido, sempre no interesse do casal 05. E
e dos filhos.
(C) O divórcio somente poderá ser concedido com a
prévia partilha dos bens. LEI MARIA DA PENHA - LEI 11.340/2006
(D) Não há impedimento legal para o casamento do
adotado com o filho do adotante.
(E) É nulo o casamento celebrado por autoridade in-
competente. LEI Nº 11.340, DE 7 DE AGOSTO DE 2006.

03. (MPE/PR - Promotor Substituto - MPE/PR/2016). Cria mecanismos para coibir a violência doméstica e
Assinale a alternativa correta: familiar contra a mulher, nos termos do § 8o do art. 226 da
(A) A guarda dos filhos será, sempre, compartilhada; Constituição Federal, da Convenção sobre a Eliminação de
(B) Na guarda compartilhada, o tempo de convívio com Todas as Formas de Discriminação contra as Mulheres e da
os filhos deve ser dividido de forma idêntica entre a mãe Convenção Interamericana para Prevenir, Punir e Erradicar
e com o pai; a Violência contra a Mulher; dispõe sobre a criação dos Jui-
(C) Na guarda compartilhada, a cidade considerada zados de Violência Doméstica e Familiar contra a Mulher;
base de moradia dos filhos será aquela que melhor atender altera o Código de Processo Penal, o Código Penal e a Lei
aos interesses dos filhos; de Execução Penal; e dá outras providências.

120
LEGISLAÇÃO

O PRESIDENTE DA REPÚBLICA Faço saber que o Con- III - em qualquer relação íntima de afeto, na qual o
gresso Nacional decreta e eu sanciono a seguinte Lei: agressor conviva ou tenha convivido com a ofendida, inde-
pendentemente de coabitação.
TÍTULO I Parágrafo único.  As relações pessoais enunciadas nes-
DISPOSIÇÕES PRELIMINARES te artigo independem de orientação sexual.
Art. 6o  A violência doméstica e familiar contra a mulher
Art. 1o  Esta Lei cria mecanismos para coibir e prevenir a constitui uma das formas de violação dos direitos huma-
violência doméstica e familiar contra a mulher, nos termos nos.
do § 8o do art. 226 da Constituição Federal, da Convenção sobre a Elimi-
nação de Todas as Formas de Violência contra a Mulher, da
CAPÍTULO II
Convenção Interamericana para Prevenir, Punir e Erradicar
a Violência contra a Mulher e de outros tratados interna- DAS FORMAS DE VIOLÊNCIA DOMÉSTICA E FAMI-
cionais ratificados pela República Federativa do Brasil; dis- LIAR
põe sobre a criação dos Juizados de Violência Doméstica CONTRA A MULHER
e Familiar contra a Mulher; e estabelece medidas de as-
sistência e proteção às mulheres em situação de violência Art. 7o  São formas de violência doméstica e familiar
doméstica e familiar. contra a mulher, entre outras:
Art. 2o  Toda mulher, independentemente de classe, I - a violência física, entendida como qualquer conduta
raça, etnia, orientação sexual, renda, cultura, nível educa- que ofenda sua integridade ou saúde corporal;
cional, idade e religião, goza dos direitos fundamentais II - a violência psicológica, entendida como qualquer
inerentes à pessoa humana, sendo-lhe asseguradas as conduta que lhe cause dano emocional e diminuição da au-
oportunidades e facilidades para viver sem violência, pre- to-estima ou que lhe prejudique e perturbe o pleno desen-
servar sua saúde física e mental e seu aperfeiçoamento volvimento ou que vise degradar ou controlar suas ações,
moral, intelectual e social. comportamentos, crenças e decisões, mediante ameaça,
Art. 3o  Serão asseguradas às mulheres as condições constrangimento, humilhação, manipulação, isolamento,
para o exercício efetivo dos direitos à vida, à segurança, à vigilância constante, perseguição contumaz, insulto, chan-
saúde, à alimentação, à educação, à cultura, à moradia, ao tagem, ridicularização, exploração e limitação do direito de
acesso à justiça, ao esporte, ao lazer, ao trabalho, à cida-
ir e vir ou qualquer outro meio que lhe cause prejuízo à
dania, à liberdade, à dignidade, ao respeito e à convivência
saúde psicológica e à autodeterminação;
familiar e comunitária.
§ 1o  O poder público desenvolverá políticas que visem III - a violência sexual, entendida como qualquer con-
garantir os direitos humanos das mulheres no âmbito das duta que a constranja a presenciar, a manter ou a partici-
relações domésticas e familiares no sentido de resguardá par de relação sexual não desejada, mediante intimidação,
-las de toda forma de negligência, discriminação, explora- ameaça, coação ou uso da força; que a induza a comerciali-
ção, violência, crueldade e opressão. zar ou a utilizar, de qualquer modo, a sua sexualidade, que
§ 2o  Cabe à família, à sociedade e ao poder público a impeça de usar qualquer método contraceptivo ou que
criar as condições necessárias para o efetivo exercício dos a force ao matrimônio, à gravidez, ao aborto ou à prosti-
direitos enunciados no caput. tuição, mediante coação, chantagem, suborno ou manipu-
Art. 4o  Na interpretação desta Lei, serão considerados lação; ou que limite ou anule o exercício de seus direitos
os fins sociais a que ela se destina e, especialmente, as con- sexuais e reprodutivos;
dições peculiares das mulheres em situação de violência IV - a violência patrimonial, entendida como qualquer
doméstica e familiar. conduta que configure retenção, subtração, destruição
parcial ou total de seus objetos, instrumentos de trabalho,
TÍTULO II documentos pessoais, bens, valores e direitos ou recursos
DA VIOLÊNCIA DOMÉSTICA E FAMILIAR CONTRA A econômicos, incluindo os destinados a satisfazer suas ne-
MULHER cessidades;
CAPÍTULO I
V - a violência moral, entendida como qualquer condu-
DISPOSIÇÕES GERAIS
ta que configure calúnia, difamação ou injúria.
Art. 5o  Para os efeitos desta Lei, configura violência
doméstica e familiar contra a mulher qualquer ação ou TÍTULO III
omissão baseada no gênero que lhe cause morte, lesão, DA ASSISTÊNCIA À MULHER EM SITUAÇÃO DE
sofrimento físico, sexual ou psicológico e dano moral ou VIOLÊNCIA DOMÉSTICA E FAMILIAR
patrimonial: (Vide Lei complementar nº 150, de 2015) CAPÍTULO I
I - no âmbito da unidade doméstica, compreendida DAS MEDIDAS INTEGRADAS DE PREVENÇÃO
como o espaço de convívio permanente de pessoas, com ou
sem vínculo familiar, inclusive as esporadicamente agregadas; Art. 8o  A política pública que visa coibir a violência
II - no âmbito da família, compreendida como a comu- doméstica e familiar contra a mulher far-se-á por meio de
nidade formada por indivíduos que são ou se consideram um conjunto articulado de ações da União, dos Estados, do
aparentados, unidos por laços naturais, por afinidade ou Distrito Federal e dos Municípios e de ações não-governa-
por vontade expressa; mentais, tendo por diretrizes:

121
LEGISLAÇÃO

I - a integração operacional do Poder Judiciário, do § 2o  O juiz assegurará à mulher em situação de vio-
Ministério Público e da Defensoria Pública com as áreas lência doméstica e familiar, para preservar sua integridade
de segurança pública, assistência social, saúde, educação, física e psicológica:
trabalho e habitação; I - acesso prioritário à remoção quando servidora pú-
II - a promoção de estudos e pesquisas, estatísticas e blica, integrante da administração direta ou indireta;
outras informações relevantes, com a perspectiva de gê- II - manutenção do vínculo trabalhista, quando neces-
nero e de raça ou etnia, concernentes às causas, às conse- sário o afastamento do local de trabalho, por até seis me-
quências e à frequência da violência doméstica e familiar ses.
contra a mulher, para a sistematização de dados, a serem § 3o  A assistência à mulher em situação de violência
unificados nacionalmente, e a avaliação periódica dos re- doméstica e familiar compreenderá o acesso aos benefícios
sultados das medidas adotadas; decorrentes do desenvolvimento científico e tecnológico,
III - o respeito, nos meios de comunicação social, dos incluindo os serviços de contracepção de emergência, a
valores éticos e sociais da pessoa e da família, de forma
profilaxia das Doenças Sexualmente Transmissíveis (DST) e
a coibir os papéis estereotipados que legitimem ou exa-
da Síndrome da Imunodeficiência Adquirida (AIDS) e ou-
cerbem a violência doméstica e familiar, de acordo com o
tros procedimentos médicos necessários e cabíveis nos ca-
estabelecido no inciso III do art. 1o, no inciso IV do art. 3o e
no inciso IV do art. 221 da Constituição Federal; sos de violência sexual.
IV - a implementação de atendimento policial especia-
lizado para as mulheres, em particular nas Delegacias de CAPÍTULO III
Atendimento à Mulher; DO ATENDIMENTO PELA AUTORIDADE POLICIAL
V - a promoção e a realização de campanhas educati-
vas de prevenção da violência doméstica e familiar contra a Art. 10.  Na hipótese da iminência ou da prática de vio-
mulher, voltadas ao público escolar e à sociedade em geral, lência doméstica e familiar contra a mulher, a autoridade
e a difusão desta Lei e dos instrumentos de proteção aos policial que tomar conhecimento da ocorrência adotará, de
direitos humanos das mulheres; imediato, as providências legais cabíveis.
VI - a celebração de convênios, protocolos, ajustes, Parágrafo único.  Aplica-se o disposto no caput deste
termos ou outros instrumentos de promoção de parceria artigo ao descumprimento de medida protetiva de urgên-
entre órgãos governamentais ou entre estes e entidades cia deferida.
não-governamentais, tendo por objetivo a implementação Art. 11.  No atendimento à mulher em situação de vio-
de programas de erradicação da violência doméstica e fa- lência doméstica e familiar, a autoridade policial deverá,
miliar contra a mulher; entre outras providências:
VII - a capacitação permanente das Polícias Civil e Mi- I - garantir proteção policial, quando necessário, co-
litar, da Guarda Municipal, do Corpo de Bombeiros e dos municando de imediato ao Ministério Público e ao Poder
profissionais pertencentes aos órgãos e às áreas enuncia- Judiciário;
dos no inciso I quanto às questões de gênero e de raça ou II - encaminhar a ofendida ao hospital ou posto de saú-
etnia; de e ao Instituto Médico Legal;
VIII - a promoção de programas educacionais que dis- III - fornecer transporte para a ofendida e seus depen-
seminem valores éticos de irrestrito respeito à dignidade dentes para abrigo ou local seguro, quando houver risco
da pessoa humana com a perspectiva de gênero e de raça de vida;
ou etnia; IV - se necessário, acompanhar a ofendida para asse-
IX - o destaque, nos currículos escolares de todos os
gurar a retirada de seus pertences do local da ocorrência
níveis de ensino, para os conteúdos relativos aos direitos
ou do domicílio familiar;
humanos, à equidade de gênero e de raça ou etnia e ao
V - informar à ofendida os direitos a ela conferidos
problema da violência doméstica e familiar contra a mu-
lher. nesta Lei e os serviços disponíveis.
Art. 12.  Em todos os casos de violência doméstica e
CAPÍTULO II familiar contra a mulher, feito o registro da ocorrência, de-
DA ASSISTÊNCIA À MULHER EM SITUAÇÃO DE verá a autoridade policial adotar, de imediato, os seguintes
VIOLÊNCIA DOMÉSTICA E FAMILIAR procedimentos, sem prejuízo daqueles previstos no Código
de Processo Penal:
Art. 9o  A assistência à mulher em situação de violência I - ouvir a ofendida, lavrar o boletim de ocorrência e
doméstica e familiar será prestada de forma articulada e tomar a representação a termo, se apresentada;
conforme os princípios e as diretrizes previstos na Lei Or- II - colher todas as provas que servirem para o esclare-
gânica da Assistência Social, no Sistema Único de Saúde, cimento do fato e de suas circunstâncias;
no Sistema Único de Segurança Pública, entre outras nor- III - remeter, no prazo de 48 (quarenta e oito) horas, ex-
mas e políticas públicas de proteção, e emergencialmente pediente apartado ao juiz com o pedido da ofendida, para
quando for o caso. a concessão de medidas protetivas de urgência;
§ 1o  O juiz determinará, por prazo certo, a inclusão da IV - determinar que se proceda ao exame de corpo de
mulher em situação de violência doméstica e familiar no delito da ofendida e requisitar outros exames periciais ne-
cadastro de programas assistenciais do governo federal, cessários;
estadual e municipal. V - ouvir o agressor e as testemunhas;

122
LEGISLAÇÃO

VI - ordenar a identificação do agressor e fazer juntar CAPÍTULO II


aos autos sua folha de antecedentes criminais, indicando DAS MEDIDAS PROTETIVAS DE URGÊNCIA
a existência de mandado de prisão ou registro de outras Seção I
ocorrências policiais contra ele; Disposições Gerais
VII - remeter, no prazo legal, os autos do inquérito po-
licial ao juiz e ao Ministério Público. Art. 18.  Recebido o expediente com o pedido da ofen-
§ 1o  O pedido da ofendida será tomado a termo pela dida, caberá ao juiz, no prazo de 48 (quarenta e oito) horas:
autoridade policial e deverá conter: I - conhecer do expediente e do pedido e decidir sobre
I - qualificação da ofendida e do agressor; as medidas protetivas de urgência;
II - nome e idade dos dependentes; II - determinar o encaminhamento da ofendida ao ór-
III - descrição sucinta do fato e das medidas protetivas gão de assistência judiciária, quando for o caso;
solicitadas pela ofendida. III - comunicar ao Ministério Público para que adote as
providências cabíveis.
§ 2o  A autoridade policial deverá anexar ao documento
Art. 19.  As medidas protetivas de urgência poderão ser
referido no § 1o o boletim de ocorrência e cópia de todos os
concedidas pelo juiz, a requerimento do Ministério Público
documentos disponíveis em posse da ofendida.
ou a pedido da ofendida.
§ 3o  Serão admitidos como meios de prova os laudos § 1o  As medidas protetivas de urgência poderão ser
ou prontuários médicos fornecidos por hospitais e postos concedidas de imediato, independentemente de audiência
de saúde. das partes e de manifestação do Ministério Público, deven-
do este ser prontamente comunicado.
TÍTULO IV § 2o  As medidas protetivas de urgência serão aplicadas
DOS PROCEDIMENTOS isolada ou cumulativamente, e poderão ser substituídas a
CAPÍTULO I qualquer tempo por outras de maior eficácia, sempre que
DISPOSIÇÕES GERAIS os direitos reconhecidos nesta Lei forem ameaçados ou
violados.
Art. 13.  Ao processo, ao julgamento e à execução das § 3o  Poderá o juiz, a requerimento do Ministério Pú-
causas cíveis e criminais decorrentes da prática de violên- blico ou a pedido da ofendida, conceder novas medidas
cia doméstica e familiar contra a mulher aplicar-se-ão as protetivas de urgência ou rever aquelas já concedidas, se
normas dos Códigos de Processo Penal e Processo Civil e entender necessário à proteção da ofendida, de seus fa-
da legislação específica relativa à criança, ao adolescente e miliares e de seu patrimônio, ouvido o Ministério Público.
ao idoso que não conflitarem com o estabelecido nesta Lei. Art. 20.  Em qualquer fase do inquérito policial ou da
Art. 14.  Os Juizados de Violência Doméstica e Familiar instrução criminal, caberá a prisão preventiva do agressor,
contra a Mulher, órgãos da Justiça Ordinária com compe- decretada pelo juiz, de ofício, a requerimento do Ministério
tência cível e criminal, poderão ser criados pela União, no Público ou mediante representação da autoridade policial.
Distrito Federal e nos Territórios, e pelos Estados, para o Parágrafo único.  O juiz poderá revogar a prisão pre-
processo, o julgamento e a execução das causas decorren- ventiva se, no curso do processo, verificar a falta de motivo
tes da prática de violência doméstica e familiar contra a para que subsista, bem como de novo decretá-la, se sobre-
mulher. vierem razões que a justifiquem.
Parágrafo único.  Os atos processuais poderão realizar- Art. 21.  A ofendida deverá ser notificada dos atos pro-
se em horário noturno, conforme dispuserem as normas de cessuais relativos ao agressor, especialmente dos pertinen-
tes ao ingresso e à saída da prisão, sem prejuízo da intima-
organização judiciária.
ção do advogado constituído ou do defensor público.
Art. 15.  É competente, por opção da ofendida, para os
Parágrafo único.  A ofendida não poderá entregar inti-
processos cíveis regidos por esta Lei, o Juizado:
mação ou notificação ao agressor.
I - do seu domicílio ou de sua residência;
II - do lugar do fato em que se baseou a demanda; Seção II
III - do domicílio do agressor. Das Medidas Protetivas de Urgência que Obrigam
Art. 16.  Nas ações penais públicas condicionadas à o Agressor
representação da ofendida de que trata esta Lei, só será
admitida a renúncia à representação perante o juiz, em au- Art. 22.  Constatada a prática de violência doméstica e
diência especialmente designada com tal finalidade, antes familiar contra a mulher, nos termos desta Lei, o juiz pode-
do recebimento da denúncia e ouvido o Ministério Público. rá aplicar, de imediato, ao agressor, em conjunto ou sepa-
Art. 17.  É vedada a aplicação, nos casos de violência radamente, as seguintes medidas protetivas de urgência,
doméstica e familiar contra a mulher, de penas de cesta entre outras:
básica ou outras de prestação pecuniária, bem como a I - suspensão da posse ou restrição do porte de armas,
substituição de pena que implique o pagamento isolado com comunicação ao órgão competente, nos termos da Lei
de multa. no10.826, de 22 de dezembro de 2003;
II - afastamento do lar, domicílio ou local de convivên-
cia com a ofendida;
III - proibição de determinadas condutas, entre as
quais:

123
LEGISLAÇÃO

a) aproximação da ofendida, de seus familiares e das IV - prestação de caução provisória, mediante depó-
testemunhas, fixando o limite mínimo de distância entre sito judicial, por perdas e danos materiais decorrentes da
estes e o agressor; prática de violência doméstica e familiar contra a ofendida.
b) contato com a ofendida, seus familiares e testemu- Parágrafo único.  Deverá o juiz oficiar ao cartório com-
nhas por qualquer meio de comunicação; petente para os fins previstos nos incisos II e III deste artigo.
c) frequentação de determinados lugares a fim de pre-
servar a integridade física e psicológica da ofendida; CAPÍTULO III
IV - restrição ou suspensão de visitas aos dependentes DA ATUAÇÃO DO MINISTÉRIO PÚBLICO
menores, ouvida a equipe de atendimento multidisciplinar
ou serviço similar; Art. 25.  O Ministério Público intervirá, quando não for
V - prestação de alimentos provisionais ou provisórios. parte, nas causas cíveis e criminais decorrentes da violência
§ 1o  As medidas referidas neste artigo não impedem a doméstica e familiar contra a mulher.
aplicação de outras previstas na legislação em vigor, sem- Art. 26.  Caberá ao Ministério Público, sem prejuízo de
pre que a segurança da ofendida ou as circunstâncias o outras atribuições, nos casos de violência doméstica e fa-
exigirem, devendo a providência ser comunicada ao Minis- miliar contra a mulher, quando necessário:
tério Público. I - requisitar força policial e serviços públicos de saúde,
§ 2o  Na hipótese de aplicação do inciso I, encontrando- de educação, de assistência social e de segurança, entre
se o agressor nas condições mencionadas no caput e inci- outros;
sos do art. 6o da Lei no 10.826, de 22 de dezembro de 2003, o juiz comunicará II - fiscalizar os estabelecimentos públicos e particu-
ao respectivo órgão, corporação ou instituição as medidas lares de atendimento à mulher em situação de violência
protetivas de urgência concedidas e determinará a restri- doméstica e familiar, e adotar, de imediato, as medidas ad-
ção do porte de armas, ficando o superior imediato do ministrativas ou judiciais cabíveis no tocante a quaisquer
agressor responsável pelo cumprimento da determinação irregularidades constatadas;
judicial, sob pena de incorrer nos crimes de prevaricação III - cadastrar os casos de violência doméstica e familiar
contra a mulher.
ou de desobediência, conforme o caso.
§ 3o  Para garantir a efetividade das medidas protetivas
CAPÍTULO IV
de urgência, poderá o juiz requisitar, a qualquer momento,
DA ASSISTÊNCIA JUDICIÁRIA
auxílio da força policial.
§ 4o  Aplica-se às hipóteses previstas neste artigo, no
Art. 27.  Em todos os atos processuais, cíveis e crimi-
que couber, o disposto no caput e nos §§ 5o e 6º do art. 461 da Lei no
5.869, de 11 de janeiro de 1973 (Código de Processo Civil).
nais, a mulher em situação de violência doméstica e fami-
liar deverá estar acompanhada de advogado, ressalvado o
previsto no art. 19 desta Lei.
Seção III Art. 28.  É garantido a toda mulher em situação de vio-
Das Medidas Protetivas de Urgência à Ofendida lência doméstica e familiar o acesso aos serviços de De-
fensoria Pública ou de Assistência Judiciária Gratuita, nos
Art. 23.  Poderá o juiz, quando necessário, sem prejuízo termos da lei, em sede policial e judicial, mediante atendi-
de outras medidas: mento específico e humanizado.
I - encaminhar a ofendida e seus dependentes a pro-
grama oficial ou comunitário de proteção ou de atendi- TÍTULO V
mento; DA EQUIPE DE ATENDIMENTO MULTIDISCIPLINAR
II - determinar a recondução da ofendida e a de seus
dependentes ao respectivo domicílio, após afastamento do Art. 29.  Os Juizados de Violência Doméstica e Familiar
agressor; contra a Mulher que vierem a ser criados poderão contar
III - determinar o afastamento da ofendida do lar, sem com uma equipe de atendimento multidisciplinar, a ser in-
prejuízo dos direitos relativos a bens, guarda dos filhos e tegrada por profissionais especializados nas áreas psicos-
alimentos; social, jurídica e de saúde.
IV - determinar a separação de corpos. Art. 30.  Compete à equipe de atendimento multidis-
Art. 24.  Para a proteção patrimonial dos bens da so- ciplinar, entre outras atribuições que lhe forem reservadas
ciedade conjugal ou daqueles de propriedade particular da pela legislação local, fornecer subsídios por escrito ao juiz,
mulher, o juiz poderá determinar, liminarmente, as seguin- ao Ministério Público e à Defensoria Pública, mediante lau-
tes medidas, entre outras: dos ou verbalmente em audiência, e desenvolver trabalhos
I - restituição de bens indevidamente subtraídos pelo de orientação, encaminhamento, prevenção e outras me-
agressor à ofendida; didas, voltados para a ofendida, o agressor e os familiares,
II - proibição temporária para a celebração de atos e com especial atenção às crianças e aos adolescentes.
contratos de compra, venda e locação de propriedade em Art. 31.  Quando a complexidade do caso exigir avalia-
comum, salvo expressa autorização judicial; ção mais aprofundada, o juiz poderá determinar a manifes-
III - suspensão das procurações conferidas pela ofen- tação de profissional especializado, mediante a indicação
dida ao agressor; da equipe de atendimento multidisciplinar.

124
LEGISLAÇÃO

Art. 32.  O Poder Judiciário, na elaboração de sua pro- Parágrafo único.  As Secretarias de Segurança Públi-
posta orçamentária, poderá prever recursos para a criação ca dos Estados e do Distrito Federal poderão remeter suas
e manutenção da equipe de atendimento multidisciplinar, informações criminais para a base de dados do Ministério
nos termos da Lei de Diretrizes Orçamentárias. da Justiça.
Art. 39.  A União, os Estados, o Distrito Federal e os
TÍTULO VI Municípios, no limite de suas competências e nos termos
DISPOSIÇÕES TRANSITÓRIAS das respectivas leis de diretrizes orçamentárias, poderão
estabelecer dotações orçamentárias específicas, em cada
Art. 33.  Enquanto não estruturados os Juizados de exercício financeiro, para a implementação das medidas
Violência Doméstica e Familiar contra a Mulher, as varas estabelecidas nesta Lei.
criminais acumularão as competências cível e criminal para Art. 40.  As obrigações previstas nesta Lei não excluem
conhecer e julgar as causas decorrentes da prática de vio- outras decorrentes dos princípios por ela adotados.
lência doméstica e familiar contra a mulher, observadas as Art. 41.  Aos crimes praticados com violência domésti-
previsões do Título IV desta Lei, subsidiada pela legislação ca e familiar contra a mulher, independentemente da pena
processual pertinente. prevista, não se aplica a Lei no 9.099, de 26 de setembro de 1995.
Parágrafo único.  Será garantido o direito de preferên- Art. 42.  O art. 313 do Decreto-Lei no  3.689, de 3 de outubro de
cia, nas varas criminais, para o processo e o julgamento das 1941 
(Código de Processo Penal), passa a vigorar acrescido
causas referidas no caput. do seguinte inciso IV:
“Art. 313.  .................................................
TÍTULO VII ................................................................
DISPOSIÇÕES FINAIS IV - se o crime envolver violência doméstica e familiar
contra a mulher, nos termos da lei específica, para garantir
Art. 34.  A instituição dos Juizados de Violência Domés- a execução das medidas protetivas de urgência.” (NR)
tica e Familiar contra a Mulher poderá ser acompanhada Art. 43.  A alínea f do inciso II do art. 61 do Decreto-Lei
pela implantação das curadorias necessárias e do serviço no 2.848, de 7 de dezembro de 1940 (Código Penal), passa a vigorar com
de assistência judiciária. a seguinte redação:
Art. 35.  A União, o Distrito Federal, os Estados e os Mu- “Art. 61.  ..................................................
nicípios poderão criar e promover, no limite das respectivas .................................................................
competências: II - ............................................................
I - centros de atendimento integral e multidisciplinar .................................................................
para mulheres e respectivos dependentes em situação de f) com abuso de autoridade ou prevalecendo-se de re-
violência doméstica e familiar; lações domésticas, de coabitação ou de hospitalidade, ou
II - casas-abrigos para mulheres e respectivos depen- com violência contra a mulher na forma da lei específica;
dentes menores em situação de violência doméstica e fa- ........................................................... ” (NR)
miliar; Art. 44.  O art. 129 do Decreto-Lei nº 2.848, de 7 de
III - delegacias, núcleos de defensoria pública, serviços dezembro de 1940 (Código Penal), passa a vigorar com as
de saúde e centros de perícia médico-legal especializados seguintes alterações:
no atendimento à mulher em situação de violência domés- “Art. 129.  ..................................................
tica e familiar; ..................................................................
IV - programas e campanhas de enfrentamento da vio- § 9o  Se a lesão for praticada contra ascendente, des-
lência doméstica e familiar; cendente, irmão, cônjuge ou companheiro, ou com quem
V - centros de educação e de reabilitação para os conviva ou tenha convivido, ou, ainda, prevalecendo-se o
agressores. agente das relações domésticas, de coabitação ou de hos-
Art. 36.  A União, os Estados, o Distrito Federal e os Mu- pitalidade:
nicípios promoverão a adaptação de seus órgãos e de seus Pena - detenção, de 3 (três) meses a 3 (três) anos.
programas às diretrizes e aos princípios desta Lei. ..................................................................
Art. 37.  A defesa dos interesses e direitos transindivi- § 11.  Na hipótese do § 9o  deste artigo, a pena será
duais previstos nesta Lei poderá ser exercida, concorrente- aumentada de um terço se o crime for cometido contra
mente, pelo Ministério Público e por associação de atuação pessoa portadora de deficiência.” (NR)
na área, regularmente constituída há pelo menos um ano, Art. 45.  O art. 152 da Lei no 7.210, de 11 de julho de 1984 (Lei de
nos termos da legislação civil. Execução Penal), passa a vigorar com a seguinte redação:
Parágrafo único.  O requisito da pré-constituição po- “Art. 152.  ...................................................
derá ser dispensado pelo juiz quando entender que não há Parágrafo único.  Nos casos de violência doméstica
outra entidade com representatividade adequada para o contra a mulher, o juiz poderá determinar o compareci-
ajuizamento da demanda coletiva. mento obrigatório do agressor a programas de recupera-
Art. 38.  As estatísticas sobre a violência doméstica e ção e reeducação.” (NR)
familiar contra a mulher serão incluídas nas bases de dados Art. 46.  Esta Lei entra em vigor 45 (quarenta e cinco)
dos órgãos oficiais do Sistema de Justiça e Segurança a fim dias após sua publicação.
de subsidiar o sistema nacional de dados e informações Brasília,  7  de  agosto  de 2006; 185o da Independência
relativo às mulheres. e 118o da República.

125
LEGISLAÇÃO

EXERCÍCIOS A violência de gênero produz-se e reproduz-se nas re-


lações de poder onde se entrelaçam as categorias de gêne-
01. (CFO – POLICIA MILITAR/MG – 2011) Quanto ao ro, classe, raça ou etnia. Expressa uma forma particular da
atendimento da mulher/vítima dos crimes estipulados violência global mediatizada pela ordem patriarcal que dá
na Lei Maria da Penha, analise as afirmativas abaixo: aos homens o direito de dominar e controlar suas mulhe-
I. A autoridade policial deve adotar providências res, podendo, para isso, usar a violência. Para os efeitos da
imediatas ao constatar que as medidas protetivas de Lei, configura violência doméstica e familiar contra a mulher
urgência deferida que não foram adotadas em relação qualquer ação ou omissão baseada no gênero que lhe cause
à vítima. morte, lesão, sofrimento físico, sexual ou psicológico e dano
II. Os órgãos policiais devem providenciar transpor- moral ou patrimonial.
te para a ofendida quando for importante colocá-la em
um abrigo. RESPOSTA: “E”.
III. A autoridade policial deverá mandar em 24 ho-
ras um expediente apartado para o Juiz, com a finali- 03. (DPE/AC - Defensor Público - CESPE/2012) Con-
dade de propor a aplicação de medidas protetivas de soante a Lei n.º 11.340/2006 (Lei Maria da Penha), o CP
urgência em relação à ofendida. e o entendimento do STF, a ação penal nos crimes de
ameaça deve ser
A) pública, condicionada à representação da vítima,
Assinale a alternativa correta.
que só pode ser realizada perante o juiz.
A) As afirmativas I, II e III estão corretas.
B) privada; contudo, caso a vítima esteja em situação
B) As afirmativas I, II e III estão incorretas. de vulnerabilidade — em conflito com o representante
C) Apenas a afirmativa I está correta. legal, por exemplo —, o MP poderá intentar ação penal
D) Apenas as afirmativas I e II estão corretas. pública mediante representação.
C) pública incondicionada.
Apenas o item III está incorreto, tendo em vista que a D) pública, condicionada à representação da vítima.
autoridade policial deverá remeter, no prazo de 48 (qua- E) privada, de iniciativa da vítima ou de seus repre-
renta e oito) horas, expediente apartado ao juiz com o pe- sentantes legais.
dido da ofendida, para a concessão de medidas protetivas
de urgência. Nos crimes de ameaça a ação penal será pública condi-
cionada à representação da vítima, nos termos do artigo 147
RESPOSTA: “D”. do Código Penal, bem como entendimento adotado pelo
Supremo Tribunal Federal.
02. (TJ/AL - Analista Judiciário - Serviço Social -
CESPE/2012) A Lei n.º 11.340/2006 (Lei Maria da Pe- RESPOSTA: “D”.
nha) resguarda os direitos da mulher contra a violência
doméstica e familiar, caracterizada como forma de vio- 04. (TJ/RJ - Juiz - VUNESP/2012) As medidas pro-
lação dos direitos humanos. Em relação a essa temáti- tetivas de urgência, da Lei n.º 11.340/06 (Lei Maria da
ca e ao que dispõe a referida norma, assinale a opção Penha)
correta. I. podem ser deferidas pelo Juiz independentemente
A) O termo gênero representa a aceitação do deter- de requerimento ou manifestação do Ministério Público;
minismo biológico como decisivo para a compreensão II. podem ser aplicadas isolada ou cumulativamente,
da relação entre homem e mulher. e poderão ser substituídas a qualquer tempo por outras
B) A referida lei, além de estabelecer que os crimes de maior eficácia;
nela previstos sejam julgados exclusivamente nos juiza- III. como regra, podem ser deferidas tanto na fase de
inquérito policial como na fase de ação penal, sendo que
dos especializados, prevê a aplicação de penas de paga-
a prisão preventiva só pode ser decretada após oferecida
mentos de cestas básicas para os casos de violência de
a denúncia.
menor gravidade.
Completa corretamente a proposição o que se afir-
C) A concessão de medidas protetivas de urgência ma em
está condicionada à realização de audiência das partes A) I, apenas.
e à manifestação do Ministério Público, não podendo B) I e II, apenas.
tais medidas ser novamente concedidas no prazo de C) II e III, apenas.
trinta dias. D) I, II e III.
D) À equipe de atendimento multidisciplinar é ve-
dado emitir, verbalmente, opinião técnica em audiência Considerando o que prevê a Lei nº 11.340/06 sobre as
quando o agressor estiver presente. medidas protetivas de urgência o único item que está in-
E) A violência de gênero, transmitida de geração correto é o “III”, uma vez que a prisão preventiva poderá
para geração, configura modelos patriarcais de família, ser decretada a qualquer momento, desde que presentes os
em que o poder masculino impõe à mulher uma cultura requisitos legais.
de subjugação.
RESPOSTA: “B”.

126
LEGISLAÇÃO

05. (PC/RJ - Inspetor de Polícia - 6º Classe - A Lei Maria da Penha é um dispositivo legal que visa
FEC/2012) Para fins de aplicação das medidas proteti- aumentar o rigor das punições das agressões contra as
vas da Lei Maria da Penha: mulheres quando ocorridas no âmbito doméstico ou fa-
A) agressor e agredida não mais precisam viver jun- miliar. Assim, para efeitos da lei considera-se a violência
tos, mas devem ter coabitado. familiar como a praticada entre pessoas unidas por vínculo
B) só o homem pode ser sujeito passivo das medi- jurídico de natureza familiar (pai, filho, avô) ou por vontade
das protetivas. expressa (casamento, adoção). Vale lembrar que a família
C) agressor e agredida devem viver juntos. fica entendida como indivíduos que são ou se consideram
D) não importa a coabitação e nem a orientação se- parentes, unidos por laços naturais ou por afinidade.
xual de agressor e agredida.
E) a mulher pode ser sujeito passivo das medidas RESPOSTA: “C”.
protetivas, mas somente se mantiver relações de paren-
tesco com a agredida. 07. (Polícia Civil/ES - Escrivão de Polícia - FUN-
CAB/2013) Em todos os casos de violência doméstica e
A Lei Maria da Penha em seu artigo 5º estipula quais familiar contra a mulher, feito o registro da ocorrência,
os critérios básicos para aplicação das medidas proteti- deverá a autoridade policial adotar, de imediato, os se-
vas configurando a violência doméstica e familiar contra guintes procedimentos, sem prejuízo daqueles previs-
a mulher qualquer ação ou omissão baseada no gênero, tos no Código de Processo Penal, conforme determina
praticada no âmbito da unidade doméstica, compreendida a Lei n° 11.340/2006:
como o espaço de convívio permanente de pessoas, com I. Ouvir a ofendida, lavrar o boletim de ocorrência e
ou sem vínculo familiar, inclusive as esporadicamente agre- tomar a retratação a termo da vítima.
gadas; no âmbito da família, compreendida como a comu- II. Remeter, no prazo de 5 (cinco) dias, expediente
nidade formada por indivíduos que são ou se consideram apartado ao Juiz com o pedido da ofendida, para a con-
aparentados, unidos por laços naturais, por afinidade ou cessão de medidas protetivas de urgência.
por vontade expressa; ou em qualquer relação íntima de III. Determinar que se proceda ao exame de corpo
afeto, na qual o agressor conviva ou tenha convivido com a de delito da ofendida e requisitar outros exames peri-
ofendida, independentemente de coabitação. Salientando ciais necessários.
IV. Ordenar a identificação da vítima e fazer juntar
ainda que as relações pessoais enunciadas independem de
aos autos sua folha de antecedentes criminais, indican-
orientação sexual.
do a existência de mandado de prisão ou registro de
outras ocorrências policiais contra ela.
RESPOSTA: “D”.
Assinale a opção que contempla apenas as asserti-
06. (DPE/RR - Defensor Público - CESPE/2013)
vas corretas.
Com base no disposto na Lei Maria da Penha — Lei n.º
A) I e II.
11.340/2006 —, assinale a opção correta. B) II e III.
A) A lei em pauta estabelece a habitualidade das C) III e IV.
condutas como requisito configurador das infrações D) I e III.
nela contempladas, ou seja, como elemento constitu- E) II e IV.
tivo do tipo. Os item “II” e IV estão errados. O prazo para remessa
B) Caso uma empregada doméstica, maior e capaz, de expediente apartado ao juiz com o pedido da ofendida,
ao receber a notícia que será despedida, sob a suspei- para a concessão de medidas protetivas de urgência é de
ta da prática de furtos, agrida seu patrão — este com 48 (quarenta e oito) horas. Deve ser realizada a identifica-
sessenta e sete anos de idade — e fuja, tal conduta da ção do “agressor” fazendo juntar aos autos sua folha de
empregada em face do patrão caracterizará violência antecedentes criminais, dentre outras providências, e não
doméstica expressamente tipificada na lei em questão. da “vítima”.
C) A violência familiar, assim considerada para efei-
tos da lei em pauta, engloba a praticada entre pessoas RESPOSTA: “D”.
unidas por vínculo jurídico de natureza familiar ou por
vontade expressa. 08. (TJ/SP – Juiz – VUNESP/2013) Nos termos da
D) O conflito entre vizinhas de que resulte violência Lei n.º 11.340, de 7 de agosto de 2006 (Lei Maria da
física e agressões verbais constitui evento que integra Penha), constatada a prática de violência doméstica e
a esfera da violência doméstica e familiar de que trata familiar contra a mulher, o Juiz poderá aplicar, de ime-
a lei em apreço. diato, ao agressor, a seguinte medida protetiva de ur-
E) Para a caracterização de violência doméstica e fa- gência, entre outras:
miliar é imprescindível a existência de vínculo familiar A) suspensão definitiva do poder familiar.
entre o agente e o paciente. B) cassação de porte de arma.
C) restrição ou suspensão de visitas aos dependen-
tes menores.
D) suspensão temporária do poder familiar.

127
LEGISLAÇÃO

Constatada a prática de violência doméstica e familiar D) Ante a urgência da situação, a referida lei pre-
contra a mulher, nos termos desta Lei, o juiz poderá aplicar, vê a possibilidade de a autoridade policial proceder, de
de imediato, ao agressor, em conjunto ou separadamente, ofício, ao afastamento provisório do agressor do lar,
as seguintes medidas protetivas de urgência, entre outras: enquanto são adotadas outras medidas pertinentes le-
I - suspensão da posse ou restrição do porte de armas, com galmente estabelecidas.
comunicação ao órgão competente; II - afastamento do lar, E) Determina a lei de regência, de forma expressa,
domicílio ou local de convivência com a ofendida; III - proi- a obrigatoriedade do segredo de justiça, no âmbito cri-
bição de determinadas condutas, entre as quais: a) aproxi- minal, com relação às medidas protetivas adotadas em
mação da ofendida, de seus familiares e das testemunhas, favor de Flávia e aos demais atos do processo.
fixando o limite mínimo de distância entre estes e o agres-
sor; b) contato com a ofendida, seus familiares e testemu- No atendimento à mulher em situação de violência
nhas por qualquer meio de comunicação; c) frequentação doméstica e familiar, a autoridade policial deverá, entre
de determinados lugares a fim de preservar a integridade outras providências: garantir proteção policial, quando ne-
física e psicológica da ofendida; IV - restrição ou suspensão cessário, comunicando de imediato ao Ministério Público e
de visitas aos dependentes menores, ouvida a equipe de ao Poder Judiciário; encaminhar a ofendida ao hospital ou
atendimento multidisciplinar ou serviço similar; V - presta- posto de saúde e ao Instituto Médico Legal; fornecer trans-
porte para a ofendida e seus dependentes para abrigo ou
ção de alimentos provisionais ou provisórios.
local seguro, quando houver risco de vida; se necessário,
acompanhar a ofendida para assegurar a retirada de seus
RESPOSTA: “C”.
pertences do local da ocorrência ou do domicílio familiar;
informar à ofendida os direitos a ela conferidos nesta Lei
e os serviços disponíveis, nos termos do art. 10 da Lei nº
09. (DPE/TO - Defensor Público - CESPE/2013) Flá- 11.340/06. Desta forma, consta expressamente na referida
via, maior, capaz, de trinta e sete anos de idade, mãe de lei o dever da autoridade policial de fornecer transporte
Lúcia, de dezesseis anos de idade, desconfiando que o para Flávia e a filha a local seguro, bem como o acompa-
companheiro, Saulo, de quarenta anos de idade, asse- nhamento ao domicílio para a retirada de seus pertences,
diava sexualmente Lúcia, procurou a delegacia de po- enquanto são adotadas outras medidas protetivas.
lícia, onde foi instaurado o procedimento investigativo
pertinente. Saulo foi conduzido à delegacia e prestou RESPOSTA: “B”.
esclarecimentos, tendo sido indiciado. Ao retornar ao
imóvel do casal, Saulo espancou Flávia, tendo-lhe cau- 10. (PC/SP - Escrivão de Polícia Civil - VUNESP/2013)
sado lesões corporais graves, e expulsou mãe e filha Assinale a alternativa que está de acordo com o dispos-
do imóvel, sob a alegação de ter pago a maior parte to na Lei Maria da Penha (Lei n.º 11.340/2006).
da casa, o que lhe garantia o direito de permanecer no A) Em qualquer fase do inquérito policial ou da ins-
imóvel. Reteve também todos os bens comuns do ca- trução criminal, caberá a prisão preventiva do agressor,
sal. Flávia, imediatamente, em companhia da filha, vol- a ser decretada pela autoridade policial competente,
tou à delegacia de polícia e declarou o seu receio de desde que esta entenda urgente e indispensável a sua
que o agressor voltasse a agredi-la e o fato de não ter aplicação.
para onde ir. Em face dessa situação hipotética, assi- B) Nos casos de violência doméstica e familiar con-
nale a opção correta com base no que dispõe a Lei n.º tra a mulher, poderão ser aplicadas ao réu as penas de
11.340/2006. detenção, reclusão, de pagamento de cesta básica ou
A) De acordo com a referida lei, o juiz, ao receber outras de prestação pecuniária, bem como a imposição
o expediente com a comunicação dos fatos, somente de multa.
poderá decretar a prisão preventiva de Saulo, de ofício, C) Constatada a prática de violência doméstica e fa-
caso exista ação penal ajuizada. Na fase investigativa, miliar contra a mulher, poderá ser aplicada ao agressor,
entre outras, a medida protetiva de urgência de afasta-
a decretação da prisão depende de representação da
mento do lar, domicílio ou local de convivência com a
autoridade policial ou de requerimento do MP.
ofendida, podendo a intimação ser entregue pela ofen-
B) Assegura expressamente a referida lei o dever da
dida diretamente ao agressor.
autoridade policial de fornecer transporte para Flávia e
D) No atendimento à mulher em situação de vio-
a filha a local seguro, bem como o acompanhamento ao lência doméstica e familiar, a autoridade policial deve-
domicílio para a retirada de seus pertences, enquanto rá, entre outras providências, conceder-lhe as medidas
são adotadas outras medidas protetivas. protetivas de urgência cabíveis no caso.
C) Nesse caso, é imprescindível a oitiva prévia de E) O juiz assegurará à mulher em situação de violên-
Saulo antes da imposição das medidas protetivas a Flá- cia doméstica e familiar, para preservar sua integridade
via, inclusive as de natureza patrimonial, sob pena de física e psicológica, a manutenção do vínculo trabalhis-
nulidade da medida, segundo dispositivo expresso da ta, quando necessário o afastamento do local de traba-
lei de regência. lho, por até seis meses.

128
LEGISLAÇÃO

Considerando o disposto na Lei Maria da Penha sobre A) No âmbito da Lei Maria da Penha, nos crimes
a assistência à mulher em situação de violência doméstica de lesão corporal leve, a ação penal é condicionada à
e familiar, a alternativa correta é a “E”, o juiz assegurará à representação. Desta forma, é possível a sua retratação,
mulher em situação de violência doméstica e familiar, para pois não houve o oferecimento da denúncia.
preservar sua integridade física e psicológica, a manuten- B) No âmbito da Lei Maria da Penha, nos crimes de
ção do vínculo trabalhista, quando necessário o afastamen- lesão corporal leve, a ação penal é pública incondicio-
to do local de trabalho, por até seis meses (art. 9º, §2º, II, nada, sendo impossível interromper as investigações e
da Lei). obstar o prosseguimento da ação penal.
C) No âmbito da Lei Maria da Penha, nos crimes de
RESPOSTA: “E”. lesão corporal leve, a ação penal é pública incondicio-
nada, mas é possível a retratação da representação an-
11. (PC/SP - Investigador de Polícia - VUNESP/2013) tes do oferecimento da denúncia.
Fulano, casado com Ciclana, num momento de discus- D) No âmbito da Lei Maria da Penha, nos crimes de
são no lar, destruiu parte dos instrumentos de trabalho lesão corporal leve, a ação penal é pública condiciona-
de sua esposa. Considerando a conduta de Fulano em da à representação, mas como os fatos já foram levados
face do disposto na Lei Maria da Penha, pode-se afir-
ao conhecimento da autoridade policial será impossível
mar que
impedir o prosseguimento das investigações e o ajuiza-
A) Fulano, pela sua conduta, poderá ser submetido
mento da ação penal.
à pena de pagamento de cestas básicas em favor de en-
tidades assistenciais. Com base no recente entendimento do Supremo Tri-
B) Fulano não se sujeitará às penas da Lei Maria da bunal Federal (STF) para que não fique exaurida a proteção
Penha, pois a sua conduta ocorreu apenas dentro do que o Estado deve dar às mulheres, os artigos 12 (inciso I),
ambiente familiar. 16 e 41, da Lei 11.340/2006 (Lei Maria da Penha), devem
C) Fulano estará sujeito à prisão preventiva, a ser ser entendidos no sentido de que não se aplica a Lei nº
decretada pelo juiz, de ofício, a requerimento do Minis- 9.099/95, dos Juizados Especiais – aos crimes da Lei Maria
tério Público ou mediante representação da autoridade da Penha, e que nos crimes de lesão corporal praticados
policial. contra a mulher no ambiente doméstico, mesmo de caráter
D) Fulano não poderá ser processado pela Lei Maria leve, atua-se mediante ação penal pública incondicionada.
da Penha, tendo em vista que esta se destina a proteger
a mulher contra agressões físicas, psicológicas ou mo- RESPOSTA: “B”.
rais, mas não patrimoniais.
E) Ciclana terá direito a obter medida judicial pro-
tetiva de urgência contra Fulano, podendo entregar CÓDIGO DE PROCESSO CIVIL – LEI 13.105/2015
pessoalmente a intimação da respectiva medida ao seu Arts: 144 a 149, 156 a 158, 464 a 480, 693 a
marido. 699, 747 a 765.

A conduta praticada por Fulano estará sujeita à prisão


preventiva, tendo em vista que em qualquer fase do inqué-
rito policial ou da instrução criminal, esta poderá ser decre- Arts: 144 a 149
ta pelo juiz, de ofício, a requerimento do Ministério Público
ou mediante representação da autoridade policial. É o que CAPÍTULO II
dispõe o artigo 20 da Lei Maria da Penha. DOS IMPEDIMENTOS E DA SUSPEIÇÃO
RESPOSTA: “C”.
Art. 144. Há impedimento do juiz, sendo-lhe vedado
exercer suas funções no processo:
12. (OAB - Exame de Ordem Unificado - XIII - Pri-
I - em que interveio como mandatário da parte, oficiou
meira Fase - FGV/2014) Fernanda, durante uma dis-
cussão com seu marido Renato, levou vários socos e como perito, funcionou como membro do Ministério Públi-
chutes. Inconformada com a agressão, dirigiu-se à De- co ou prestou depoimento como testemunha;
legacia de Polícia mais próxima e narrou todo o ocorri- II - de que conheceu em outro grau de jurisdição, ten-
do. Após a realização do exame de corpo de delito, foi do proferido decisão;
constatada a prática de lesão corporal leve por parte de III - quando nele estiver postulando, como defensor
Renato. O Delegado de Polícia registrou a ocorrência e público, advogado ou membro do Ministério Público, seu
requereu as medidas cautelares constantes no Artigo cônjuge ou companheiro, ou qualquer parente, consanguí-
23 da Lei nº 11.340/2006. Após alguns dias e com obje- neo ou afim, em linha reta ou colateral, até o terceiro grau,
tivo de reconciliação com o marido, Fernanda foi nova- inclusive;
mente à Delegacia de Polícia requerendo a cessação das IV - quando for parte no processo ele próprio, seu côn-
investigações para que não fosse ajuizada a ação penal juge ou companheiro, ou parente, consanguíneo ou afim,
respectiva. Diante do caso narrado, de acordo com o em linha reta ou colateral, até o terceiro grau, inclusive;
recente entendimento do Supremo Tribunal Federal, V - quando for sócio ou membro de direção ou de ad-
assinale a afirmativa correta. ministração de pessoa jurídica parte no processo;

129
LEGISLAÇÃO

VI - quando for herdeiro presuntivo, donatário ou em- § 3o Enquanto não for declarado o efeito em que é re-
pregador de qualquer das partes; cebido o incidente ou quando este for recebido com efeito
VII - em que figure como parte instituição de ensino suspensivo, a tutela de urgência será requerida ao substi-
com a qual tenha relação de emprego ou decorrente de tuto legal.
contrato de prestação de serviços; § 4o Verificando que a alegação de impedimento ou de
VIII - em que figure como parte cliente do escritório de suspeição é improcedente, o tribunal rejeitá-la-á.
advocacia de seu cônjuge, companheiro ou parente, con- § 5o Acolhida a alegação, tratando-se de impedimento
sanguíneo ou afim, em linha reta ou colateral, até o terceiro ou de manifesta suspeição, o tribunal condenará o juiz nas
grau, inclusive, mesmo que patrocinado por advogado de custas e remeterá os autos ao seu substituto legal, poden-
outro escritório; do o juiz recorrer da decisão.
IX - quando promover ação contra a parte ou seu ad- § 6o Reconhecido o impedimento ou a suspeição, o tri-
vogado. bunal fixará o momento a partir do qual o juiz não poderia
§ 1o Na hipótese do inciso III, o impedimento só se ve- ter atuado.
rifica quando o defensor público, o advogado ou o mem- § 7o O tribunal decretará a nulidade dos atos do juiz, se
bro do Ministério Público já integrava o processo antes do praticados quando já presente o motivo de impedimento
início da atividade judicante do juiz. ou de suspeição.
§ 2o É vedada a criação de fato superveniente a fim de
caracterizar impedimento do juiz. Art. 147. Quando 2 (dois) ou mais juízes forem paren-
§ 3o O impedimento previsto no inciso III também se tes, consanguíneos ou afins, em linha reta ou colateral, até
verifica no caso de mandato conferido a membro de es- o terceiro grau, inclusive, o primeiro que conhecer do pro-
critório de advocacia que tenha em seus quadros advoga- cesso impede que o outro nele atue, caso em que o segun-
do que individualmente ostente a condição nele prevista, do se escusará, remetendo os autos ao seu substituto legal.
mesmo que não intervenha diretamente no processo. Art. 148. Aplicam-se os motivos de impedimento e de
suspeição:
Art. 145. Há suspeição do juiz: I - ao membro do Ministério Público;
I - amigo íntimo ou inimigo de qualquer das partes ou II - aos auxiliares da justiça;
de seus advogados; III - aos demais sujeitos imparciais do processo.
II - que receber presentes de pessoas que tiverem inte- § 1o A parte interessada deverá arguir o impedimento
resse na causa antes ou depois de iniciado o processo, que ou a suspeição, em petição fundamentada e devidamente
aconselhar alguma das partes acerca do objeto da causa ou instruída, na primeira oportunidade em que lhe couber fa-
que subministrar meios para atender às despesas do litígio; lar nos autos.
III - quando qualquer das partes for sua credora ou de- § 2o O juiz mandará processar o incidente em separado
vedora, de seu cônjuge ou companheiro ou de parentes e sem suspensão do processo, ouvindo o arguido no pra-
destes, em linha reta até o terceiro grau, inclusive; zo de 15 (quinze) dias e facultando a produção de prova,
IV - interessado no julgamento do processo em favor quando necessária.
de qualquer das partes. § 3o Nos tribunais, a arguição a que se refere o § 1o
§ 1o Poderá o juiz declarar-se suspeito por motivo de será disciplinada pelo regimento interno.
foro íntimo, sem necessidade de declarar suas razões. § 4o O disposto nos §§ 1o e 2o não se aplica à arguição
§ 2o Será ilegítima a alegação de suspeição quando: de impedimento ou de suspeição de testemunha.
I - houver sido provocada por quem a alega;
II - a parte que a alega houver praticado ato que signi- CAPÍTULO III
fique manifesta aceitação do arguido. DOS AUXILIARES DA JUSTIÇA
Art. 149.  São auxiliares da Justiça, além de outros cujas
Art. 146. No prazo de 15 (quinze) dias, a contar do atribuições sejam determinadas pelas normas de organiza-
conhecimento do fato, a parte alegará o impedimento ou ção judiciária, o escrivão, o chefe de secretaria, o oficial de
a suspeição, em petição específica dirigida ao juiz do pro- justiça, o perito, o depositário, o administrador, o intérpre-
cesso, na qual indicará o fundamento da recusa, podendo te, o tradutor, o mediador, o conciliador judicial, o partidor,
instruí-la com documentos em que se fundar a alegação e o distribuidor, o contabilista e o regulador de avarias.
com rol de testemunhas.
§ 1o Se reconhecer o impedimento ou a suspeição ao - Diferenças entre conciliador e mediador
receber a petição, o juiz ordenará imediatamente a remes-
sa dos autos a seu substituto legal, caso contrário, determi- O conciliador atuará preferencialmente nos casos em
nará a autuação em apartado da petição e, no prazo de 15 que não houver vínculo anterior entre as partes, poderá
(quinze) dias, apresentará suas razões, acompanhadas de sugerir soluções para o litígio, sendo vedada a utilização de
documentos e de rol de testemunhas, se houver, ordenan- qualquer tipo de constrangimento ou intimidação para que
do a remessa do incidente ao tribunal. as partes conciliem. Já o mediador atuará preferencialmen-
§ 2o Distribuído o incidente, o relator deverá declarar te nos casos em que houver vínculo anterior entre as par-
os seus efeitos, sendo que, se o incidente for recebido: tes, auxiliará aos interessados a compreender as questões
I - sem efeito suspensivo, o processo voltará a correr; e os interesses em conflito, de modo que eles possam, pelo
II - com efeito suspensivo, o processo permanecerá restabelecimento da comunicação, identificar, por si pró-
suspenso até o julgamento do incidente. prios, soluções consensuais que gerem benefícios mútuos.

130
LEGISLAÇÃO

- Princípios que regulam a conciliação e mediação § 2o  Para formação do cadastro, os tribunais devem
realizar consulta pública, por meio de divulgação na rede
A conciliação e a mediação são informadas pelos prin- mundial de computadores ou em jornais de grande circu-
cípios da independência, da imparcialidade, da autonomia lação, além de consulta direta a universidades, a conselhos
da vontade, da confidencialidade, da oralidade, da infor- de classe, ao Ministério Público, à Defensoria Pública e à
malidade e da decisão informada. Tais princípios, em sua Ordem dos Advogados do Brasil, para a indicação de pro-
maioria, foram extraídos da Resolução 125/2010 do CNPJ. fissionais ou de órgãos técnicos interessados.
§ 3o  Os tribunais realizarão avaliações e reavaliações
- Requisitos para ser conciliador ou mediador periódicas para manutenção do cadastro, considerando a
formação profissional, a atualização do conhecimento e a
Os conciliadores, os mediadores e as câmaras privadas experiência dos peritos interessados.
de conciliação e mediação serão inscritos em cadastro na- § 4o Para verificação de eventual impedimento ou mo-
cional e em cadastro de tribunal de justiça ou de tribunal
tivo de suspeição, nos termos dos arts. 148 e 467, o órgão
regional federal, que manterá registro de profissionais ha-
técnico ou científico nomeado para realização da perícia
bilitados, com indicação de sua área profissional.
informará ao juiz os nomes e os dados de qualificação dos
Preenchendo o requisito da capacitação mínima, por
meio de curso realizado por entidade credenciada, confor- profissionais que participarão da atividade.
me parâmetro curricular definido pelo Conselho Nacional § 5o Na localidade onde não houver inscrito no cadas-
de Justiça em conjunto com o Ministério da Justiça, o con- tro disponibilizado pelo tribunal, a nomeação do perito é
ciliador ou o mediador, com o respectivo certificado, pode- de livre escolha pelo juiz e deverá recair sobre profissional
rá requerer sua inscrição no cadastro nacional e no cadas- ou órgão técnico ou científico comprovadamente detentor
tro de tribunal de justiça ou de tribunal regional federal. do conhecimento necessário à realização da perícia.
Efetivado o registro, que poderá ser precedido de con- Art. 157.  O perito tem o dever de cumprir o ofício no
curso público, o tribunal remeterá ao diretor do foro da prazo que lhe designar o juiz, empregando toda sua dili-
comarca, seção ou subseção judiciária onde atuará o con- gência, podendo escusar-se do encargo alegando motivo
ciliador ou o mediador os dados necessários para que seu legítimo.
nome passe a constar da respectiva lista, a ser observada § 1o A escusa será apresentada no prazo de 15 (quinze)
na distribuição alternada e aleatória, respeitado o princípio dias, contado da intimação, da suspeição ou do impedi-
da igualdade dentro da mesma área de atuação profissio- mento supervenientes, sob pena de renúncia ao direito a
nal. alegá-la.
§ 2o  Será organizada lista de peritos na vara ou na se-
- Impedimentos cretaria, com disponibilização dos documentos exigidos
para habilitação à consulta de interessados, para que a no-
No caso de impedimento, o conciliador ou mediador meação seja distribuída de modo equitativo, observadas a
o comunicará imediatamente, de preferência por meio ele- capacidade técnica e a área de conhecimento.
trônico, e devolverá os autos ao juiz do processo ou ao Art. 158. O perito que, por dolo ou culpa, prestar infor-
coordenador do centro judiciário de solução de conflitos, mações inverídicas responderá pelos prejuízos que causar
devendo este realizar nova distribuição. Se a causa de im- à parte e ficará inabilitado para atuar em outras perícias no
pedimento for apurada quando já iniciado o procedimen- prazo de 2 (dois) a 5 (cinco) anos, independentemente das
to, a atividade será interrompida, lavrando-se ata com re- demais sanções previstas em lei, devendo o juiz comunicar
latório do ocorrido e solicitação de distribuição para novo
o fato ao respectivo órgão de classe para adoção das me-
conciliador ou mediador.
didas que entender cabíveis.
- Remuneração
Arts: 464 a 480
O conciliador e o mediador receberão pelo seu traba-
lho remuneração prevista em tabela fixada pelo tribunal, Seção X
conforme parâmetros estabelecidos pelo Conselho Nacio- Da Prova Pericial
nal de Justiça.
Art. 464. A prova pericial consiste em exame, vistoria
Arts: 156 a 158 ou avaliação.
§ 1o O juiz indeferirá a perícia quando:
Seção II I - a prova do fato não depender de conhecimento es-
Do Perito pecial de técnico;
II - for desnecessária em vista de outras provas produ-
Art. 156.  O juiz será assistido por perito quando a pro- zidas;
va do fato depender de conhecimento técnico ou científico. III - a verificação for impraticável.
§ 1o  Os peritos serão nomeados entre os profissionais § 2o De ofício ou a requerimento das partes, o juiz po-
legalmente habilitados e os órgãos técnicos ou científicos derá, em substituição à perícia, determinar a produção de
devidamente inscritos em cadastro mantido pelo tribunal prova técnica simplificada, quando o ponto controvertido
ao qual o juiz está vinculado. for de menor complexidade.

131
LEGISLAÇÃO

§ 3o A prova técnica simplificada consistirá apenas na § 1o No caso previsto no inciso II, o juiz comunicará a
inquirição de especialista, pelo juiz, sobre ponto controver- ocorrência à corporação profissional respectiva, podendo,
tido da causa que demande especial conhecimento cientí- ainda, impor multa ao perito, fixada tendo em vista o va-
fico ou técnico. lor da causa e o possível prejuízo decorrente do atraso no
§ 4o Durante a arguição, o especialista, que deverá ter processo.
formação acadêmica específica na área objeto de seu de- § 2o O perito substituído restituirá, no prazo de 15
poimento, poderá valer-se de qualquer recurso tecnológi- (quinze) dias, os valores recebidos pelo trabalho não rea-
co de transmissão de sons e imagens com o fim de esclare- lizado, sob pena de ficar impedido de atuar como perito
cer os pontos controvertidos da causa. judicial pelo prazo de 5 (cinco) anos.
§ 3o Não ocorrendo a restituição voluntária de que tra-
Art. 465. O juiz nomeará perito especializado no ob- ta o § 2o, a parte que tiver realizado o adiantamento dos
jeto da perícia e fixará de imediato o prazo para a entrega honorários poderá promover execução contra o perito, na
do laudo. forma dos arts. 513 e seguintes deste Código, com funda-
§ 1o Incumbe às partes, dentro de 15 (quinze) dias con- mento na decisão que determinar a devolução do nume-
tados da intimação do despacho de nomeação do perito: rário.
I - arguir o impedimento ou a suspeição do perito, se
for o caso; Art. 469. As partes poderão apresentar quesitos suple-
II - indicar assistente técnico; mentares durante a diligência, que poderão ser respondi-
III - apresentar quesitos. dos pelo perito previamente ou na audiência de instrução
§ 2o Ciente da nomeação, o perito apresentará em 5 e julgamento.
(cinco) dias: Parágrafo único. O escrivão dará à parte contrária
I - proposta de honorários; ciência da juntada dos quesitos aos autos.
II - currículo, com comprovação de especialização; Art. 470. Incumbe ao juiz:
I - indeferir quesitos impertinentes;
III - contatos profissionais, em especial o endereço ele-
II - formular os quesitos que entender necessários ao
trônico, para onde serão dirigidas as intimações pessoais.
esclarecimento da causa.
§ 3o As partes serão intimadas da proposta de hono-
rários para, querendo, manifestar-se no prazo comum de 5
Art. 471. As partes podem, de comum acordo, escolher
(cinco) dias, após o que o juiz arbitrará o valor, intimando-
o perito, indicando-o mediante requerimento, desde que:
se as partes para os fins do art. 95.
I - sejam plenamente capazes;
§ 4o O juiz poderá autorizar o pagamento de até cin- II - a causa possa ser resolvida por autocomposição.
quenta por cento dos honorários arbitrados a favor do pe- § 1o As partes, ao escolher o perito, já devem indicar os
rito no início dos trabalhos, devendo o remanescente ser respectivos assistentes técnicos para acompanhar a realiza-
pago apenas ao final, depois de entregue o laudo e presta- ção da perícia, que se realizará em data e local previamente
dos todos os esclarecimentos necessários. anunciados.
§ 5o Quando a perícia for inconclusiva ou deficiente, o § 2o O perito e os assistentes técnicos devem entregar,
juiz poderá reduzir a remuneração inicialmente arbitrada respectivamente, laudo e pareceres em prazo fixado pelo
para o trabalho. juiz.
§ 6o Quando tiver de realizar-se por carta, poder-se-á § 3o A perícia consensual substitui, para todos os efei-
proceder à nomeação de perito e à indicação de assisten- tos, a que seria realizada por perito nomeado pelo juiz.
tes técnicos no juízo ao qual se requisitar a perícia.
Art. 472. O juiz poderá dispensar prova pericial quan-
Art. 466. O perito cumprirá escrupulosamente o encar- do as partes, na inicial e na contestação, apresentarem, so-
go que lhe foi cometido, independentemente de termo de bre as questões de fato, pareceres técnicos ou documentos
compromisso. elucidativos que considerar suficientes.
§ 1o Os assistentes técnicos são de confiança da parte
e não estão sujeitos a impedimento ou suspeição. Art. 473. O laudo pericial deverá conter:
§ 2o O perito deve assegurar aos assistentes das partes I - a exposição do objeto da perícia;
o acesso e o acompanhamento das diligências e dos exa- II - a análise técnica ou científica realizada pelo perito;
mes que realizar, com prévia comunicação, comprovada III - a indicação do método utilizado, esclarecendo-o
nos autos, com antecedência mínima de 5 (cinco) dias. e demonstrando ser predominantemente aceito pelos es-
pecialistas da área do conhecimento da qual se originou;
Art. 467. O perito pode escusar-se ou ser recusado por IV - resposta conclusiva a todos os quesitos apresenta-
impedimento ou suspeição. dos pelo juiz, pelas partes e pelo órgão do Ministério Pú-
Parágrafo único. O juiz, ao aceitar a escusa ou ao julgar blico.
procedente a impugnação, nomeará novo perito. § 1o No laudo, o perito deve apresentar sua fundamen-
tação em linguagem simples e com coerência lógica, indi-
Art. 468. O perito pode ser substituído quando: cando como alcançou suas conclusões.
I - faltar-lhe conhecimento técnico ou científico; § 2o É vedado ao perito ultrapassar os limites de sua
II - sem motivo legítimo, deixar de cumprir o encargo designação, bem como emitir opiniões pessoais que ex-
no prazo que lhe foi assinado. cedam o exame técnico ou científico do objeto da perícia.

132
LEGISLAÇÃO

§ 3o Para o desempenho de sua função, o perito e os § 3o Quando o exame tiver por objeto a autenticidade
assistentes técnicos podem valer-se de todos os meios da letra e da firma, o perito poderá requisitar, para efeito
necessários, ouvindo testemunhas, obtendo informações, de comparação, documentos existentes em repartições pú-
solicitando documentos que estejam em poder da parte, blicas e, na falta destes, poderá requerer ao juiz que a pes-
de terceiros ou em repartições públicas, bem como instruir soa a quem se atribuir a autoria do documento lance em
o laudo com planilhas, mapas, plantas, desenhos, fotogra- folha de papel, por cópia ou sob ditado, dizeres diferentes,
fias ou outros elementos necessários ao esclarecimento do para fins de comparação.
objeto da perícia.
Art. 479. O juiz apreciará a prova pericial de acordo
Art. 474. As partes terão ciência da data e do local de- com o disposto no art. 371, indicando na sentença os moti-
signados pelo juiz ou indicados pelo perito para ter início a vos que o levaram a considerar ou a deixar de considerar as
produção da prova. conclusões do laudo, levando em conta o método utilizado
pelo perito.
Art. 475. Tratando-se de perícia complexa que abranja
mais de uma área de conhecimento especializado, o juiz Art. 480. O juiz determinará, de ofício ou a requeri-
poderá nomear mais de um perito, e a parte, indicar mais mento da parte, a realização de nova perícia quando a ma-
de um assistente técnico. téria não estiver suficientemente esclarecida.
§ 1o A segunda perícia tem por objeto os mesmos fa-
Art. 476. Se o perito, por motivo justificado, não puder tos sobre os quais recaiu a primeira e destina-se a corrigir
apresentar o laudo dentro do prazo, o juiz poderá conce- eventual omissão ou inexatidão dos resultados a que esta
der-lhe, por uma vez, prorrogação pela metade do prazo conduziu.
originalmente fixado. § 2o A segunda perícia rege-se pelas disposições esta-
belecidas para a primeira.
Art. 477. O perito protocolará o laudo em juízo, no § 3o A segunda perícia não substitui a primeira, caben-
prazo fixado pelo juiz, pelo menos 20 (vinte) dias antes da do ao juiz apreciar o valor de uma e de outra.
audiência de instrução e julgamento.
§ 1o As partes serão intimadas para, querendo, mani- 1- Introdução – conceitos básicos sobre a prova:
festar-se sobre o laudo do perito do juízo no prazo comum
de 15 (quinze) dias, podendo o assistente técnico de cada A palavra “prova” deriva do latim probare (convencer,
uma das partes, em igual prazo, apresentar seu respectivo tornar crível) e, de acordo com José Frederico Marques, é o
parecer. “meio e modo utilizados pelos litigantes com o escopo de
§ 2o O perito do juízo tem o dever de, no prazo de 15 convencer o juiz da veracidade dos fatos por eles alegados,
(quinze) dias, esclarecer ponto: e igualmente, pelo magistrado, para formar sua convicção
I - sobre o qual exista divergência ou dúvida de qual- sobre os fatos que constituem a base empírica da lide.
quer das partes, do juiz ou do órgão do Ministério Público; Torna-se possível reconstruir, historicamente, os aconteci-
II - divergente apresentado no parecer do assistente mentos geradores do litígio, de sorte a possibilitar, com
técnico da parte. a sua qualificação jurídica, um julgamento justo e confor-
§ 3o Se ainda houver necessidade de esclarecimentos, me o Direito” (MARQUES, José Frederico. Curso de Direito
a parte requererá ao juiz que mande intimar o perito ou o Processual Civil – Processo de Conhecimento. São Paulo:
assistente técnico a comparecer à audiência de instrução Saraiva, 1999, p. 336).
e julgamento, formulando, desde logo, as perguntas, sob
forma de quesitos. Os meios de prova expressamente tipificados no novo
§ 4o O perito ou o assistente técnico será intimado por Código de Processo Civil são a ata notarial (art. 384, novi-
meio eletrônico, com pelo menos 10 (dez) dias de antece- dade em relação ao CPC de 1973), o depoimento pessoal
dência da audiência. (arts. 385 a 388), a confissão (arts. 389 a 395), a prova do-
cumental (arts. 405 a 429), a prova testemunhal (fls. 442 a
Art. 478. Quando o exame tiver por objeto a auten- 463), a prova pericial (arts. 464 a 480) e a inspeção judicial
ticidade ou a falsidade de documento ou for de natureza (arts. 481 a 484).
médico-legal, o perito será escolhido, de preferência, entre
os técnicos dos estabelecimentos oficiais especializados, a Entretanto, o novel legislador, mantendo a previsão já
cujos diretores o juiz autorizará a remessa dos autos, bem existente no CPC de 1973, admite outros tipos de prova
como do material sujeito a exame. além dos elencados acima, estabelecendo que “as partes
§ 1o Nas hipóteses de gratuidade de justiça, os órgãos têm o direito de empregar todos os meios legais, bem
e as repartições oficiais deverão cumprir a determinação como os moralmente legítimos, ainda que não especifica-
judicial com preferência, no prazo estabelecido. dos neste Código, para provar a verdade dos fatos em que
§ 2o A prorrogação do prazo referido no § 1o pode ser se funda o pedido ou a defesa e influir na convicção do
requerida motivadamente. juiz” (art. 369).

133
LEGISLAÇÃO

2- A prova pericial no novo CPC: Já de acordo com o novo CPC, o juiz nomeará um peri-
Segundo o dicionário Aurélio, a perícia consiste na “vis- to “especializado” no objeto da perícia e fixará de imediato
toria ou exame de caráter técnico especializado”. o prazo para a entrega do laudo (art. 465, caput).

Já para Fredie Didier Jr, a prova pericial “é aquela pela Desse modo, a novel legislação não faz referência ao
qual a elucidação do fato se dá com ao auxílio de um perito, pré-requisito “nível universitário”, substituindo-o pela ex-
especialista em determinado campo do saber, devidamente pressão “especializado no objeto da perícia”. Entendemos
nomeado pelo juiz, que deve registrar sua opinião técnica e que o profissional “especializado” é aquele que, seja me-
científica no chamado laudo pericial – que poderá ser objeto diante lei, seja mediante regulamentação específica, tenha
de discussão pelas partes e seus assistentes técnicos” (DI- condições de atuar em uma determinada área do conheci-
DIER JR., Fredie; BRAGA, Paula Sarno; OLIVEIRA, Rafael. Curso mento. Por consectário lógico, não havendo lei regulamen-
de Direito Processual Civil: teoria da prova, direito proba- tando determinada área de conhecimento, profissionais de
tório, teoria da precedente, decisão judicial, coisa julgada e qualquer área, em tese, estariam habilitados ou, no míni-
antecipação da tutela. 5. Ed. Rev. Amp. Salvador: JusPodivm,
mo, não teriam nenhum impeditivo para exercer a perícia.
2010, v.2, pág. 225).
Isso confirma o atual entendimento do STJ, no sentido de
que a falta de formação específica do perito não anula o
No mesmo sentido, Humberto Theororo Júnior con-
ceitua a prova pericial como “meio de suprir a carência de laudo pericial (cf. RESP 1383693/DF, relator Ministro Ro-
conhecimentos técnicos de que se ressente o juiz para apu- gério Schieti Cruz, 6ª Turma do STJ, acórdão publicado em
ração dos fatos litigiosos” (THEODORO JÚNIOR, Humberto. 04/02/2015).
Curso de Direito Processual Civil: Teoria geral do direito pro-
cessual civil e processo de conhecimento. 52. Ed. Rev. Amp. Ademais, o mesmo STJ decidiu, recentemente, ser re-
Rio de Janeiro: Forense, 2011, v.1, pág. 486). lativa a nulidade em virtude de eventual irregularidade na
nomeação dos peritos designados para elaborar a prova
O novo Código de Processo Civil incorporou todas as al- técnica, o que exige a manifestação da parte interessada na
terações promovidas ao CPC de 1973 pela lei nº 8.455, de 24 primeira oportunidade em que lhe couber falar nos autos,
de agosto de 1992, detalhando ainda mais o procedimento sob pena de preclusão (RESP nº 1370903/MG, 3ª Turma, re-
de realização da perícia. lator Ministro Ricardo Villas Bôas Cueva, DJ de 31/03/2015).

Nos termos do art. 464, caput, § 1º, I, II e III, do novo O novo CPC exige a apresentação, pelo perito, no pra-
CPC (que reproduziu o art. 420, caput, do CPC de 1973), a zo de 05 (cinco) dias após a sua nomeação, da proposta
prova pericial pode consistir em exame (perícia sobre coisas de honorários, dos contatos profissionais (em especial um
móveis), vistoria (perícia sobre bens imóveis) ou avaliação endereço eletrônico) e de um currículo, com comprovação
(perícia que se presta a aferir o valor de determinado bem de sua especialização na área da perícia (art. 465, § 2º, I, II
ou direito), podendo o juiz indeferir a perícia quando a pro- e III, do novo CPC).
va do fato não depender do conhecimento do técnico, for
desnecessária em vista de outras provas produzidas ou a ve- Segundo a jurisprudência, a teor do disposto nos arts.
rificação for impraticável. 19 e 33 do CPC/73 (correspondentes aos arts. 82 e 95 do
CPC/2015), cabe à parte que requereu a produção de prova
Ademais, o juiz poderá dispensar a prova pericial quan- pericial o ônus de adiantar os honorários periciais, ou ao
do as partes, na inicial e na contestação, apresentarem, so- autor, quando requerida por ambas as partes, ou determi-
bre as questões de fato, pareceres técnicos ou documentos
nada de ofício pelo juiz, sendo que a eventual inversão do
elucidativos que considerar suficientes (art. 472 do novo
ônus probatório pelo juiz não acarreta a obrigação de su-
CPC, que reproduziu o art. 427 do CPC de 1973).
portar as despesas com a perícia, implicando, tão somente,
De acordo com a jurisprudência, em observância ao que a parte requerida arque com as consequências jurídi-
princípio do livre convencimento motivado, não configura cas decorrentes da não produção da prova (cf. AgRg no
cerceamento de defesa o julgamento antecipado da lide, de- AgRg no AREsp 575.905/MS, relator Ministro Raul Araújo,
vidamente fundamentado, sem a produção de prova pericial 4ª Turma, DJ de 29/04/2015).
tida por desnecessária pelo juízo, uma vez que cabe ao ma-
gistrado dirigir a instrução e deferir a produção probatória Outra boa novidade trazida pela novel legislação foi
que considerar necessária à formação do seu convencimen- a possibilidade de utilização da chamada “prova técnica
to (Nesse sentido: AgRg no AREsp nº 169.080/DF, 4ª Turma, simplificada”, no caso de pontos controvertidos de menor
relatora Ministra Maria Isabel Gallotti, DJ de 14/05/2015). complexidade, que consiste na inquirição, pelo juiz, em
audiência, de um especialista com formação acadêmica
A nomeação do perito sofreu sensível alteração no novo específica no objeto da perícia, hipótese na qual será dis-
texto processual. No CPC de 1973, os peritos eram escolhi- pensada a elaboração de um laudo escrito. Tal especialista
dos dentre profissionais de nível universitário, inscritos nos poderá se valer de recursos tecnológicos e transmissão de
órgãos de classe competentes (art. 145, § 1º), sendo que, sons e imagens para esclarecer os pontos controvertidos
nos locais onde não houvesse essa possibilidade, os peritos (art. 464, §§ 2º a 4º).
seriam de livre escolha do juiz (art. 145, § 3º).

134
LEGISLAÇÃO

Essa previsão da lei, sem dúvida, consagra os princípios Interessante é a previsão legal no tocante ao mérito do
da instrumentalidade das formas, da efetividade e da cele- trabalho do perito, uma vez que o CPC de 2015 impõe os
ridade processual, desburocratizando e reduzindo os custos requisitos para a elaboração do laudo pericial, tais como a
das demandas judiciais, já que, sob a égide do CPC de 1973, necessidade de exposição do objeto da perícia, análise téc-
caberia ao perito ou ao assistente técnico manifestarem-se em nica ou científica, indicação do método utilizado e resposta
audiência acerca do laudo pericial escrito produzido, esclare- conclusiva a todos os quesitos, sendo que a fundamenta-
cendo algum ponto cuja relevância fosse considerada impor- ção deverá ser em linguagem simples e coerente (art. 473,
tante para o deslinde da causa, desde que intimados 5 (cinco) parágrafos e incisos do novo CPC).
dias antes da audiência (art. 435, caput e parágrafo único). Isso evitará eventuais abusos por parte dos peritos,
que passarão a ter de pautar o seu trabalho conforme os
O perito pode escusar-se ou ser recusado em razão de ditames legais, evitando-se, assim, a elaboração de lau-
impedimento ou suspeição (art. 467, caput); já os assistentes dos demasiadamente complexos, muitas vezes elaborados
técnicos são de confiança da parte e não estão sujeitos a im- mais com o fim de receber vastos honorários periciais do
pedimento ou suspeição (art. 466, § 1º).
que esclarecer o objeto da demanda em si.
Os prazos da prova pericial também sofreram mudanças,
Os esclarecimentos a serem prestados pelo peri-
uma vez que a eventual arguição de impedimento do perito,
a formulação dos quesitos e a indicação dos assistentes técni- to deverão ser apresentados em um primeiro momento
cos pelas partes poderá ser feita em até 15 (quinze) dias (art. por escrito, e serão bem abrangentes, abarcando tanto o
465, § 1º, I, II e III do novo CPC), havendo uma significativa questionamento das partes como eventuais divergências
melhora em relação ao prazo de 05 (cinco) dias previsto no suscitadas pelos assistentes técnicos, o que não impede,
art. 421, § 1º, I e II, do CPC de 1973. outrossim, a necessidade do comparecimento do expert
em audiência caso o juiz assim determine, caso haja ne-
As partes poderão impugnar a proposta de honorários cessidade de maiores esclarecimentos, devendo o perito,
do perito no prazo de 5 (cinco) dias, cabendo ao juiz decidir nesse último caso, ser intimado por correio eletrônico com
sobre o valor (art. 465, § 3º). pelo menos 10 dias de antecedência (art. 477 §§ 2º a 4º, do
novo CPC).
Ressalte-se, ainda, a previsão, pelo novo CPC, da chamada
“perícia consensual”, que é a possibilidade de as partes, desde O novo Código, por fim, confere ao juiz a possibili-
que sejam capazes, estejam em comum acordo e a matéria dade de destituir e fixar sanções ao perito, caso falte-lhe
tratada admita a auto composição, indicarem o perito, sendo conhecimento técnico ou científico ou deixe de cumprir
essa decisão das partes obrigatória ao juiz, o que prestigia o seu encargo no prazo sem motivo legítimo, o que poderá
princípio da autonomia da vontade das partes. Importante é a ensejar a devolução dos honorários periciais que o profis-
previsão de que a perícia consensual substitui, para todos os sional eventualmente houver recebido, sob pena de, não o
efeitos, a que seria realizada por perito nomeado pelo juiz (art. fazendo, ficar impedido de atuar como perito judicial pelo
471, incisos e parágrafos). prazo de 5 (cinco) anos (art. 468 do novo CPC).
A perícia será protocolada em juízo no prazo definido
pelo juiz, que deverá ser pelo menos 20 (vinte) dias antes da Arts: 693 a 699
audiência de instrução e julgamento (art. 477, caput, do novo
CPC, não inovou em relação ao art. 433, caput, do CPC de CAPÍTULO X
1973).
DAS AÇÕES DE FAMÍLIA
O prazo das partes para a juntada dos pareceres dos as-
Art. 693.  As normas deste Capítulo aplicam-se aos
sistentes técnicos e posterior manifestação dos seus advoga-
dos passa a ser comum de 15 (quinze) dias (art. 477, § 1º), ha- processos contenciosos de divórcio, separação, reconhe-
vendo sensível majoração dos prazos anteriormente previstos cimento e extinção de união estável, guarda, visitação e
na lei (10 e 5 dias, respectivamente). Uma crítica que pode ser filiação.
feita é sobre a natureza desse prazo, que é comum, ao passo Parágrafo único.  A ação de alimentos e a que versar
que poderia ser sucessivo, uma vez que a vista aos advoga- sobre interesse de criança ou de adolescente observarão o
dos somente será útil após o parecer do assistente técnico da procedimento previsto em legislação específica, aplicando-
parte contrária. se, no que couber, as disposições deste Capítulo.

Digna de nota também é a previsão do CPC de 2015 de Art. 694.  Nas ações de família, todos os esforços serão
que as partes poderão ter acesso ao trabalho do perito no empreendidos para a solução consensual da controvérsia,
decorrer das diligências, sendo cientificadas sobre a data e devendo o juiz dispor do auxílio de profissionais de outras
o local da produção da prova (art. 474), podendo inclusive áreas de conhecimento para a mediação e conciliação.
apresentar quesitos suplementares durante a diligência, Parágrafo único.  A requerimento das partes, o juiz
que poderão ser respondidos previamente pelo perito, ou pode determinar a suspensão do processo enquanto os
somente na audiência de instrução e julgamento (art. 469, litigantes se submetem a mediação extrajudicial ou a aten-
caput). dimento multidisciplinar.

135
LEGISLAÇÃO

Art. 695.  Recebida a petição inicial e, se for o caso, to- Art. 750.  O requerente deverá juntar laudo médico
madas as providências referentes à tutela provisória, o juiz para fazer prova de suas alegações ou informar a impossi-
ordenará a citação do réu para comparecer à audiência de bilidade de fazê-lo.
mediação e conciliação, observado o disposto no art. 694. Art. 751.  O interditando será citado para, em dia desig-
§ 1o  O mandado de citação conterá apenas os dados nado, comparecer perante o juiz, que o entrevistará minu-
necessários à audiência e deverá estar desacompanhado ciosamente acerca de sua vida, negócios, bens, vontades,
de cópia da petição inicial, assegurado ao réu o direito de preferências e laços familiares e afetivos e sobre o que mais
examinar seu conteúdo a qualquer tempo. lhe parecer necessário para convencimento quanto à sua
§ 2o A citação ocorrerá com antecedência mínima de 15 capacidade para praticar atos da vida civil, devendo ser re-
(quinze) dias da data designada para a audiência. duzidas a termo as perguntas e respostas.
§ 3o A citação será feita na pessoa do réu. § 1o  Não podendo o interditando deslocar-se, o juiz o
§ 4o Na audiência, as partes deverão estar acompanha- ouvirá no local onde estiver.
§ 2o A entrevista poderá ser acompanhada por especia-
das de seus advogados ou de defensores públicos.
lista.
§ 3o  Durante a entrevista, é assegurado o emprego de
Art. 696.  A audiência de mediação e conciliação pode-
recursos tecnológicos capazes de permitir ou de auxiliar o
rá dividir-se em tantas sessões quantas sejam necessárias interditando a expressar suas vontades e preferências e a
para viabilizar a solução consensual, sem prejuízo de provi- responder às perguntas formuladas.
dências jurisdicionais para evitar o perecimento do direito. § 4o A critério do juiz, poderá ser requisitada a oitiva de
parentes e de pessoas próximas.
Art. 697.  Não realizado o acordo, passarão a incidir, a Art. 752.  Dentro do prazo de 15 (quinze) dias contado
partir de então, as normas do procedimento comum, ob- da entrevista, o interditando poderá impugnar o pedido.
servado o art. 335. § 1o O Ministério Público intervirá como fiscal da ordem
jurídica.
Art. 698.  Nas ações de família, o Ministério Público § 2o O interditando poderá constituir advogado, e, caso
somente intervirá quando houver interesse de incapaz e não o faça, deverá ser nomeado curador especial.
deverá ser ouvido previamente à homologação de acordo. § 3o Caso o interditando não constitua advogado, o seu
cônjuge, companheiro ou qualquer parente sucessível po-
Art. 699.  Quando o processo envolver discussão sobre derá intervir como assistente.
fato relacionado a abuso ou a alienação parental, o juiz, ao Art. 753.  Decorrido o prazo previsto no art. 752, o juiz
tomar o depoimento do incapaz, deverá estar acompanha- determinará a produção de prova pericial para avaliação da
do por especialista. capacidade do interditando para praticar atos da vida civil.
§ 1o  A perícia pode ser realizada por equipe composta
Arts: 747 a 765 por expertos com formação multidisciplinar.
§ 2o O laudo pericial indicará especificadamente, se for o
Seção IX caso, os atos para os quais haverá necessidade de curatela.
Da Interdição Art. 754.  Apresentado o laudo, produzidas as demais
provas e ouvidos os interessados, o juiz proferirá sentença.
Art. 747.  A interdição pode ser promovida: Art. 755.  Na sentença que decretar a interdição, o juiz:
I - pelo cônjuge ou companheiro; I - nomeará curador, que poderá ser o requerente da
interdição, e fixará os limites da curatela, segundo o estado
II - pelos parentes ou tutores;
e o desenvolvimento mental do interdito;
III - pelo representante da entidade em que se encon-
II - considerará as características pessoais do interdito,
tra abrigado o interditando;
observando suas potencialidades, habilidades, vontades e
IV - pelo Ministério Público. preferências.
Parágrafo único.  A legitimidade deverá ser comprova- § 1o A curatela deve ser atribuída a quem melhor possa
da por documentação que acompanhe a petição inicial. atender aos interesses do curatelado.
Art. 748.  O Ministério Público só promoverá interdição § 2o  Havendo, ao tempo da interdição, pessoa incapaz
em caso de doença mental grave: sob a guarda e a responsabilidade do interdito, o juiz atri-
I - se as pessoas designadas nos incisos I, II e III do art. buirá a curatela a quem melhor puder atender aos interes-
747 não existirem ou não promoverem a interdição; ses do interdito e do incapaz.
II - se, existindo, forem incapazes as pessoas mencio- § 3o   A sentença de interdição será inscrita no registro
nadas nos incisos I e II do art. 747. de pessoas naturais e imediatamente publicada na rede
Art. 749.  Incumbe ao autor, na petição inicial, espe- mundial de computadores, no sítio do tribunal a que estiver
cificar os fatos que demonstram a incapacidade do inter- vinculado o juízo e na plataforma de editais do Conselho
ditando para administrar seus bens e, se for o caso, para Nacional de Justiça, onde permanecerá por 6 (seis) meses,
praticar atos da vida civil, bem como o momento em que a na imprensa local, 1 (uma) vez, e no órgão oficial, por 3
incapacidade se revelou. (três) vezes, com intervalo de 10 (dez) dias, constando do
Parágrafo único.  Justificada a urgência, o juiz pode no- edital os nomes do interdito e do curador, a causa da inter-
mear curador provisório ao interditando para a prática de dição, os limites da curatela e, não sendo total a interdição,
determinados atos. os atos que o interdito poderá praticar autonomamente.

136
LEGISLAÇÃO

Art. 756.  Levantar-se-á a curatela quando cessar a cau- Art. 763.  Cessando as funções do tutor ou do curador
sa que a determinou. pelo decurso do prazo em que era obrigado a servir, ser-
§ 1o  O pedido de levantamento da curatela poderá ser lhe-á lícito requerer a exoneração do encargo.
feito pelo interdito, pelo curador ou pelo Ministério Público § 1o Caso o tutor ou o curador não requeira a exonera-
e será apensado aos autos da interdição. ção do encargo dentro dos 10 (dez) dias seguintes à expi-
§ 2o  O juiz nomeará perito ou equipe multidisciplinar ração do termo, entender-se-á reconduzido, salvo se o juiz
para proceder ao exame do interdito e designará audiência o dispensar.
de instrução e julgamento após a apresentação do laudo. § 2o  Cessada a tutela ou a curatela, é indispensável a
§ 3o Acolhido o pedido, o juiz decretará o levantamento prestação de contas pelo tutor ou pelo curador, na forma
da interdição e determinará a publicação da sentença, após da lei civil.
o trânsito em julgado, na forma do art. 755, § 3o, ou, não
sendo possível, na imprensa local e no órgão oficial, por 3 Seção XI
(três) vezes, com intervalo de 10 (dez) dias, seguindo-se a Da Organização e da Fiscalização das Fundações
averbação no registro de pessoas naturais.
§ 4o  A interdição poderá ser levantada parcialmente Art. 764.  O juiz decidirá sobre a aprovação do estatuto
quando demonstrada a capacidade do interdito para prati- das fundações e de suas alterações sempre que o requeira
car alguns atos da vida civil. o interessado, quando:
Art. 757.  A autoridade do curador estende-se à pes- I - ela for negada previamente pelo Ministério Público
soa e aos bens do incapaz que se encontrar sob a guarda e ou por este forem exigidas modificações com as quais o
a responsabilidade do curatelado ao tempo da interdição, interessado não concorde;
salvo se o juiz considerar outra solução como mais conve- II - o interessado discordar do estatuto elaborado pelo
niente aos interesses do incapaz. Ministério Público.
Art. 758.  O curador deverá buscar tratamento e apoio § 1o O estatuto das fundações deve observar o disposto
apropriados à conquista da autonomia pelo interdito. na Lei no 10.406, de 10 de janeiro de 2002 (Código Civil).
§ 2o Antes de suprir a aprovação, o juiz poderá mandar
Seção X fazer no estatuto modificações a fim de adaptá-lo ao obje-
tivo do instituidor.
Disposições Comuns à Tutela e à Curatela
Art. 765.  Qualquer interessado ou o Ministério Público
promoverá em juízo a extinção da fundação quando:
Art. 759.  O tutor ou o curador será intimado a prestar
I - se tornar ilícito o seu objeto;
compromisso no prazo de 5 (cinco) dias contado da:
II - for impossível a sua manutenção;
I - nomeação feita em conformidade com a lei;
III - vencer o prazo de sua existência.
II - intimação do despacho que mandar cumprir o tes-
tamento ou o instrumento público que o houver instituído.
§ 1o  O tutor ou o curador prestará o compromisso por Questões
termo em livro rubricado pelo juiz.
§ 2o Prestado o compromisso, o tutor ou o curador assu- 01) Aplicada em: 2016Banca: FGVÓrgão: Prefeitura de
me a administração dos bens do tutelado ou do interditado. Paulínia - SPProva: Procurador .A respeito das disposições
Art. 760. O tutor ou o curador poderá eximir-se do en- gerais sobre as provas, assinale a afirmativa incorreta.
cargo apresentando escusa ao juiz no prazo de 5 (cinco) a) Não será admitida prova produzida em outro pro-
dias contado: cesso
I - antes de aceitar o encargo, da intimação para prestar b) É possível utilizar a teoria da carga dinâmica do ônus
compromisso; da prova nos casos previstos em lei ou diante de peculiari-
II - depois de entrar em exercício, do dia em que sobre- dades da causa relacionadas à impossibilidade ou à exces-
vier o motivo da escusa. siva dificuldade de produzir a prova ou à maior facilidade
§ 1o  Não sendo requerida a escusa no prazo estabele- de obtenção da prova do fato contrário, de modo a permi-
cido neste artigo, considerar-se-á renunciado o direito de tir que haja a inversão por decisão devidamente motivada.
alegá-la. c) A distribuição do ônus da prova pode ocorrer por
§ 2o O juiz decidirá de plano o pedido de escusa, e, não convenção das partes, salvo quando recair sobre direito in-
o admitindo, exercerá o nomeado a tutela ou a curatela disponível da parte ou tornar excessivamente difícil a uma
enquanto não for dispensado por sentença transitada em parte o exercício do direito.
julgado. d) A parte que alegar direito municipal, estadual, es-
Art. 761.  Incumbe ao Ministério Público ou a quem te- trangeiro ou consuetudinário deverá provar o teor e a vi-
nha legítimo interesse requerer, nos casos previstos em lei, gência, se assim o juiz determinar.
a remoção do tutor ou do curador. e) Preservado o direito de não produzir prova contra
Parágrafo único.  O tutor ou o curador será citado para si própria, incumbe à parte comparecer em juízo, respon-
contestar a arguição no prazo de 5 (cinco) dias, findo o qual dendo ao que lhe for interrogado, colaborar com o juízo na
observar-se-á o procedimento comum. realização de inspeção judicial que for considerada neces-
Art. 762.  Em caso de extrema gravidade, o juiz poderá sária e praticar o ato que lhe for determinado.
suspender o tutor ou o curador do exercício de suas fun-
ções, nomeando substituto interino. RESPOSTA: A

137
LEGISLAÇÃO

§ 5o  Se o juiz verificar que o filho não deve permanecer


sob a guarda do pai ou da mãe, deferirá a guarda à pes-
GUARDA COMPARTILHADA - LEI 11.698/2014 soa que revele compatibilidade com a natureza da medida,
considerados, de preferência, o grau de parentesco e as
relações de afinidade e afetividade.” (NR)
Art. 2o  Esta Lei entra em vigor após decorridos 60 (ses-
LEI Nº 11.698, DE 13 DE JUNHO DE 2008. senta) dias de sua publicação.
Brasília, 13 de junho de 2008; 187o da Independência e
Altera os arts. 1.583 e 1.584 da Lei no 10.406, de 10 de 120o da República.
janeiro de 2002 – Código Civil, para instituir e disciplinar a
guarda compartilhada.

O PRESIDENTE DA REPÚBLICA Faço saber que o Con- NOVA GUARDA COMPARTILHADA -


gresso Nacional decreta e eu sanciono a seguinte Lei: LEI 13.058/2014
Art. 1o  Os arts. 1.583 e 1.584 da Lei no  10.406, de 10
de janeiro de 2002 – Código Civil, passam a vigorar com a
seguinte redação:
“Art. 1.583.  A guarda será unilateral ou compartilhada. LEI Nº 13.058, DE 22 DE DEZEMBRO DE 2014.
§ 1o  Compreende-se por guarda unilateral a atribuída
a um só dos genitores ou a alguém que o substitua (art. Altera os arts. 1.583, 1.584, 1.585 e 1.634 da Lei
1.584, § 5o) e, por guarda compartilhada a responsabiliza- no 10.406, de 10 de janeiro de 2002 (Código Civil), para es-
ção conjunta e o exercício de direitos e deveres do pai e da tabelecer o significado da expressão “guarda compartilha-
mãe que não vivam sob o mesmo teto, concernentes ao da” e dispor sobre sua aplicação.
poder familiar dos filhos comuns.
§ 2o  A guarda unilateral será atribuída ao genitor que A PRESIDENTA DA REPÚBLICA Faço saber que o Con-
revele melhores condições para exercê-la e, objetivamente, gresso Nacional decreta e eu sanciono a seguinte Lei:
mais aptidão para propiciar aos filhos os seguintes fatores: Art. 1o  Esta Lei estabelece o significado da expressão
I – afeto nas relações com o genitor e com o grupo “guarda compartilhada” e dispõe sobre sua aplicação, para
familiar;
o que modifica os arts. 1.583, 1.584, 1.585 e 1.634 da Lei
II – saúde e segurança;
no 10.406, de 10 de janeiro de 2002 (Código Civil).
III – educação.
Art. 2o   A Lei no  10.406, de 10 de janeiro de 2002 (Código Civil), passa a
§ 3o  A guarda unilateral obriga o pai ou a mãe que não
vigorar com as seguintes alterações:
a detenha a supervisionar os interesses dos filhos.
“Art. 1.583.  ......................;;;........................................
§ 4o  (VETADO).” (NR)
.............................................................................................
“Art. 1.584.  A guarda, unilateral ou compartilhada, po-
derá ser: § 2o    Na guarda compartilhada, o tempo de convívio
I – requerida, por consenso, pelo pai e pela mãe, ou por com os filhos deve ser dividido de forma equilibrada com
qualquer deles, em ação autônoma de separação, de divór- a mãe e com o pai, sempre tendo em vista as condições
cio, de dissolução de união estável ou em medida cautelar; fáticas e os interesses dos filhos.
II – decretada pelo juiz, em atenção a necessidades es- I - (revogado);
pecíficas do filho, ou em razão da distribuição de tempo II - (revogado);
necessário ao convívio deste com o pai e com a mãe. III - (revogado).
§ 1o  Na audiência de conciliação, o juiz informará ao § 3º  Na guarda compartilhada, a cidade considerada
pai e à mãe o significado da guarda compartilhada, a sua base de moradia dos filhos será aquela que melhor atender
importância, a similitude de deveres e direitos atribuídos aos interesses dos filhos.
aos genitores e as sanções pelo descumprimento de suas ..............................................................................................
cláusulas. § 5º  A guarda unilateral obriga o pai ou a mãe que não
§ 2o  Quando não houver acordo entre a mãe e o pai a detenha a supervisionar os interesses dos filhos, e, para
quanto à guarda do filho, será aplicada, sempre que possí- possibilitar tal supervisão, qualquer dos genitores sempre
vel, a guarda compartilhada. será parte legítima para solicitar informações e/ou pres-
§ 3o  Para estabelecer as atribuições do pai e da mãe tação de contas, objetivas ou subjetivas, em assuntos ou
e os períodos de convivência sob guarda compartilhada, situações que direta ou indiretamente afetem a saúde física
o juiz, de ofício ou a requerimento do Ministério Público, e psicológica e a educação de seus filhos.” (NR)
poderá basear-se em orientação técnico-profissional ou de “Art. 1.584.  ..................................................................
equipe interdisciplinar. .............................................................................................
§ 4o  A alteração não autorizada ou o descumprimento § 2o    Quando não houver acordo entre a mãe e o pai
imotivado de cláusula de guarda, unilateral ou comparti- quanto à guarda do filho, encontrando-se ambos os geni-
lhada, poderá implicar a redução de prerrogativas atribuí- tores aptos a exercer o poder familiar, será aplicada a guar-
das ao seu detentor, inclusive quanto ao número de horas da compartilhada, salvo se um dos genitores declarar ao
de convivência com o filho. magistrado que não deseja a guarda do menor.

138
LEGISLAÇÃO

§ 3o    Para estabelecer as atribuições do pai e da mãe


e os períodos de convivência sob guarda compartilhada,
o juiz, de ofício ou a requerimento do Ministério Público, ALIENAÇÃO PARENTAL – LEI 12.318/2010
poderá basear-se em orientação técnico-profissional ou de
equipe interdisciplinar, que deverá visar à divisão equilibra-
da do tempo com o pai e com a mãe.
§ 4o  A alteração não autorizada ou o descumprimento LEI Nº 12.318, DE 26 DE AGOSTO DE 2010.
imotivado de cláusula de guarda unilateral ou compartilha-
da poderá implicar a redução de prerrogativas atribuídas Dispõe sobre a alienação parental e altera o art. 236 da
ao seu detentor. Lei no8.069, de 13 de julho de 1990.
§ 5o  Se o juiz verificar que o filho não deve permanecer
sob a guarda do pai ou da mãe, deferirá a guarda a pes- O PRESIDENTE DA REPÚBLICA Faço saber que o Con-
soa que revele compatibilidade com a natureza da medida, gresso Nacional decreta e eu sanciono a seguinte Lei: 
considerados, de preferência, o grau de parentesco e as Art. 1o  Esta Lei dispõe sobre a alienação parental. 
relações de afinidade e afetividade. Art. 2o  Considera-se ato de alienação parental a inter-
§ 6o    Qualquer estabelecimento público ou privado é ferência na formação psicológica da criança ou do adoles-
obrigado a prestar informações a qualquer dos genitores cente promovida ou induzida por um dos genitores, pelos
sobre os filhos destes, sob pena de multa de R$ 200,00 avós ou pelos que tenham a criança ou adolescente sob
(duzentos reais) a R$ 500,00 (quinhentos reais) por dia pelo a sua autoridade, guarda ou vigilância para que repudie
não atendimento da solicitação.” (NR) genitor ou que cause prejuízo ao estabelecimento ou à ma-
“Art. 1.585.  Em sede de medida cautelar de separação nutenção de vínculos com este. 
de corpos, em sede de medida cautelar de guarda ou em Parágrafo único.  São formas exemplificativas de alie-
outra sede de fixação liminar de guarda, a decisão sobre nação parental, além dos atos assim declarados pelo juiz
guarda de filhos, mesmo que provisória, será proferida pre- ou constatados por perícia, praticados diretamente ou com
ferencialmente após a oitiva de ambas as partes perante o auxílio de terceiros:  
juiz, salvo se a proteção aos interesses dos filhos exigir a I - realizar campanha de desqualificação da conduta do
concessão de liminar sem a oitiva da outra parte, aplican- genitor no exercício da paternidade ou maternidade; 
do-se as disposições do art. 1.584.” (NR) II - dificultar o exercício da autoridade parental; 
“Art. 1.634.  Compete a ambos os pais, qualquer que III - dificultar contato de criança ou adolescente com
seja a sua situação conjugal, o pleno exercício do poder genitor; 
familiar, que consiste em, quanto aos filhos: IV - dificultar o exercício do direito regulamentado de
I - dirigir-lhes a criação e a educação; convivência familiar; 
II - exercer a guarda unilateral ou compartilhada nos V - omitir deliberadamente a genitor informações pes-
termos do art. 1.584; soais relevantes sobre a criança ou adolescente, inclusive
III - conceder-lhes ou negar-lhes consentimento para escolares, médicas e alterações de endereço; 
casarem; VI - apresentar falsa denúncia contra genitor, contra
IV - conceder-lhes ou negar-lhes consentimento para familiares deste ou contra avós, para obstar ou dificultar a
viajarem ao exterior; convivência deles com a criança ou adolescente; 
V - conceder-lhes ou negar-lhes consentimento para VII - mudar o domicílio para local distante, sem jus-
mudarem sua residência permanente para outro Município; tificativa, visando a dificultar a convivência da criança ou
VI - nomear-lhes tutor por testamento ou documento adolescente com o outro genitor, com familiares deste ou
autêntico, se o outro dos pais não lhe sobreviver, ou o so- com avós. 
brevivo não puder exercer o poder familiar; Art. 3o  A prática de ato de alienação parental fere di-
VII - representá-los judicial e extrajudicialmente até os reito fundamental da criança ou do adolescente de con-
16 (dezesseis) anos, nos atos da vida civil, e assisti-los, após vivência familiar saudável, prejudica a realização de afeto
essa idade, nos atos em que forem partes, suprindo-lhes o nas relações com genitor e com o grupo familiar, constitui
consentimento; abuso moral contra a criança ou o adolescente e descum-
VIII - reclamá-los de quem ilegalmente os detenha; primento dos deveres inerentes à autoridade parental ou
IX - exigir que lhes prestem obediência, respeito e os decorrentes de tutela ou guarda. 
serviços próprios de sua idade e condição.” (NR) Art. 4o  Declarado indício de ato de alienação parental,
Art. 3o  Esta Lei entra em vigor na data de sua publica- a requerimento ou de ofício, em qualquer momento pro-
ção. cessual, em ação autônoma ou incidentalmente, o proces-
Brasília, 22 de dezembro de 2014; 193o da Independên- so terá tramitação prioritária, e o juiz determinará, com ur-
cia e 126o da República. gência, ouvido o Ministério Público, as medidas provisórias
necessárias para preservação da integridade psicológica
da criança ou do adolescente, inclusive para assegurar sua
convivência com genitor ou viabilizar a efetiva reaproxima-
ção entre ambos, se for o caso. 

139
LEGISLAÇÃO

Parágrafo único.  Assegurar-se-á à criança ou adoles- Art. 9o  (VETADO) 


cente e ao genitor garantia mínima de visitação assistida, Art. 10.  (VETADO) 
ressalvados os casos em que há iminente risco de prejuízo Art. 11.  Esta Lei entra em vigor na data de sua publi-
à integridade física ou psicológica da criança ou do adoles- cação. 
cente, atestado por profissional eventualmente designado
pelo juiz para acompanhamento das visitas.  Brasília,  26  de  agosto  de 2010; 189o da Independên-
Art. 5o  Havendo indício da prática de ato de alienação cia e 122o da República. 
parental, em ação autônoma ou incidental, o juiz, se neces-
sário, determinará perícia psicológica ou biopsicossocial. 
§ 1o  O laudo pericial terá base em ampla avaliação psi- SISTEMA ÚNICO DE ASSISTÊNCIA SOCIAL –
cológica ou biopsicossocial, conforme o caso, compreen- SUAS – Lei 12.435/2011
dendo, inclusive, entrevista pessoal com as partes, exame
de documentos dos autos, histórico do relacionamento do
casal e da separação, cronologia de incidentes, avaliação
da personalidade dos envolvidos e exame da forma como
LEI Nº 12.435, DE 6 DE JULHO DE 2011.
a criança ou adolescente se manifesta acerca de eventual
acusação contra genitor. 
§ 2o  A perícia será realizada por profissional ou equipe Altera a Lei no  8.742, de 7 de dezembro de 1993, que
multidisciplinar habilitados, exigido, em qualquer caso, ap- dispõe sobre a organização da Assistência Social. 
tidão comprovada por histórico profissional ou acadêmico
para diagnosticar atos de alienação parental.   A PRESIDENTA DA REPÚBLICA Faço saber que o Con-
§ 3o  O perito ou equipe multidisciplinar designada gresso Nacional decreta e eu sanciono a seguinte Lei: 
para verificar a ocorrência de alienação parental terá prazo Art. 1o  Os arts. 2o, 3o, 6o, 12, 13, 14, 15, 16, 17, 20, 21, 22,
de 90 (noventa) dias para apresentação do laudo, prorro- 23, 24, 28 e 36 da Lei no 8.742, de 7 de dezembro de 1993,
gável exclusivamente por autorização judicial baseada em passam a vigorar com a seguinte redação:
justificativa circunstanciada.  “Art. 2o  A assistência social tem por objetivos:
Art. 6o  Caracterizados atos típicos de alienação pa- I - a proteção social, que visa à garantia da vida, à re-
rental ou qualquer conduta que dificulte a convivência de dução de danos e à prevenção da incidência de riscos, es-
criança ou adolescente com genitor, em ação autônoma pecialmente:
ou incidental, o juiz poderá, cumulativamente ou não, sem a) a proteção à família, à maternidade, à infância, à
prejuízo da decorrente responsabilidade civil ou criminal adolescência e à velhice;
e da ampla utilização de instrumentos processuais aptos b) o amparo às crianças e aos adolescentes carentes;
a inibir ou atenuar seus efeitos, segundo a gravidade do c) a promoção da integração ao mercado de trabalho;
caso:  d) a habilitação e reabilitação das pessoas com defi-
I - declarar a ocorrência de alienação parental e adver- ciência e a promoção de sua integração à vida comunitária;
tir o alienador;  e
II - ampliar o regime de convivência familiar em favor e) a garantia de 1 (um) salário-mínimo de benefício
do genitor alienado;  mensal à pessoa com deficiência e ao idoso que compro-
III - estipular multa ao alienador;  vem não possuir meios de prover a própria manutenção ou
IV - determinar acompanhamento psicológico e/ou de tê-la provida por sua família;
biopsicossocial;  II - a vigilância socioassistencial, que visa a analisar ter-
V - determinar a alteração da guarda para guarda com-
ritorialmente a capacidade protetiva das famílias e nela a
partilhada ou sua inversão; 
ocorrência de vulnerabilidades, de ameaças, de vitimiza-
VI - determinar a fixação cautelar do domicílio da
ções e danos;
criança ou adolescente; 
VII - declarar a suspensão da autoridade parental.  III - a defesa de direitos, que visa a garantir o pleno
Parágrafo único.  Caracterizado mudança abusiva de acesso aos direitos no conjunto das provisões socioassis-
endereço, inviabilização ou obstrução à convivência fami- tenciais.
liar, o juiz também poderá inverter a obrigação de levar Parágrafo único.  Para o enfrentamento da pobreza, a
para ou retirar a criança ou adolescente da residência do assistência social realiza-se de forma integrada às políticas
genitor, por ocasião das alternâncias dos períodos de con- setoriais, garantindo mínimos sociais e provimento de con-
vivência familiar.  dições para atender contingências sociais e promovendo a
Art. 7o  A atribuição ou alteração da guarda dar-se-á universalização dos direitos sociais.” (NR)
por preferência ao genitor que viabiliza a efetiva convivên- “Art. 3o  Consideram-se entidades e organizações de
cia da criança ou adolescente com o outro genitor nas hi- assistência social aquelas sem fins lucrativos que, isolada
póteses em que seja inviável a guarda compartilhada.  ou cumulativamente, prestam atendimento e assesso-
Art. 8o  A alteração de domicílio da criança ou adoles- ramento aos beneficiários abrangidos por esta Lei, bem
cente é irrelevante para a determinação da competência como as que atuam na defesa e garantia de direitos.
relacionada às ações fundadas em direito de convivência § 1o  São de atendimento aquelas entidades que, de
familiar, salvo se decorrente de consenso entre os genito- forma continuada, permanente e planejada, prestam ser-
res ou de decisão judicial.  viços, executam programas ou projetos e concedem be-

140
LEGISLAÇÃO

nefícios de prestação social básica ou especial, dirigidos IV - realizar o monitoramento e a avaliação da política
às famílias e indivíduos em situações de vulnerabilidade ou de assistência social e assessorar Estados, Distrito Federal e
risco social e pessoal, nos termos desta Lei, e respeitadas Municípios para seu desenvolvimento.” (NR)
as deliberações do Conselho Nacional de Assistência Social “Art. 13.  ..........................................................................
(CNAS), de que tratam os incisos I e II do art. 18. I - destinar recursos financeiros aos Municípios, a título
§ 2o  São de assessoramento aquelas que, de forma con- de participação no custeio do pagamento dos benefícios
tinuada, permanente e planejada, prestam serviços e execu- eventuais de que trata o art. 22, mediante critérios esta-
tam programas ou projetos voltados prioritariamente para o belecidos pelos Conselhos Estaduais de Assistência Social;
fortalecimento dos movimentos sociais e das organizações II - cofinanciar, por meio de transferência automática, o
de usuários, formação e capacitação de lideranças, dirigidos aprimoramento da gestão, os serviços, os programas e os
ao público da política de assistência social, nos termos desta projetos de assistência social em âmbito regional ou local;
Lei, e respeitadas as deliberações do CNAS, de que tratam os .............................................................................................
incisos I e II do art. 18. VI - realizar o monitoramento e a avaliação da política
§ 3o  São de defesa e garantia de direitos aquelas que, de assistência social e assessorar os Municípios para seu
de forma continuada, permanente e planejada, prestam ser- desenvolvimento.” (NR)
viços e executam programas e projetos voltados prioritaria- “Art. 14.  ..........................................................................
mente para a defesa e efetivação dos direitos socioassisten- I -  destinar recursos financeiros para custeio do pa-
ciais, construção de novos direitos, promoção da cidadania, gamento dos benefícios eventuais de que trata o art. 22,
enfrentamento das desigualdades sociais, articulação com mediante critérios estabelecidos pelos Conselhos de Assis-
órgãos públicos de defesa de direitos, dirigidos ao público tência Social do Distrito Federal;
da política de assistência social, nos termos desta Lei, e res- .............................................................................................
peitadas as deliberações do CNAS, de que tratam os incisos VI - cofinanciar o aprimoramento da gestão, os servi-
I e II do art. 18.” (NR) ços, os programas e os projetos de assistência social em
“Art. 6o  A gestão das ações na área de assistência social âmbito local;
fica organizada sob a forma de sistema descentralizado e VII - realizar o monitoramento e a avaliação da política
participativo, denominado Sistema Único de Assistência So-
de assistência social em seu âmbito.” (NR)
cial (Suas), com os seguintes objetivos:
“Art. 15.  .........................................................................
I - consolidar a gestão compartilhada, o cofinanciamen-
I - destinar recursos financeiros para custeio do paga-
to e a cooperação técnica entre os entes federativos que, de
mento dos benefícios eventuais de que trata o art. 22, me-
modo articulado, operam a proteção social não contributiva;
diante critérios estabelecidos pelos Conselhos Municipais
II - integrar a rede pública e privada de serviços, progra-
de Assistência Social;
mas, projetos e benefícios de assistência social, na forma do
.............................................................................................
art. 6o-C;
III - estabelecer as responsabilidades dos entes federati- VI - cofinanciar o aprimoramento da gestão, os servi-
vos na organização, regulação, manutenção e expansão das ços, os programas e os projetos de assistência social em
ações de assistência social; âmbito local;
IV - definir os níveis de gestão, respeitadas as diversida- VII - realizar o monitoramento e a avaliação da política
des regionais e municipais; de assistência social em seu âmbito.” (NR)
V - implementar a gestão do trabalho e a educação per- “Art. 16.  As instâncias deliberativas do Suas, de caráter
manente na assistência social; permanente e composição paritária entre governo e socie-
VI - estabelecer a gestão integrada de serviços e bene- dade civil, são:
fícios; e .............................................................................................
VII - afiançar a vigilância socioassistencial e a garantia Parágrafo único. Os Conselhos de Assistência Social
de direitos. estão vinculados ao órgão gestor de assistência social, que
§ 1o  As ações ofertadas no âmbito do Suas têm por deve prover a infraestrutura necessária ao seu funciona-
objetivo a proteção à família, à maternidade, à infância, à mento, garantindo recursos materiais, humanos e finan-
adolescência e à velhice e, como base de organização, o ter- ceiros, inclusive com despesas referentes a passagens e
ritório. diárias de conselheiros representantes do governo ou da
§ 2o  O Suas é integrado pelos entes federativos, pelos sociedade civil, quando estiverem no exercício de suas atri-
respectivos conselhos de assistência social e pelas entidades buições.” (NR)
e organizações de assistência social abrangidas por esta Lei. “Art. 17.  .......................................................................
§ 3o  A instância coordenadora da Política Nacional de .............................................................................................
Assistência Social é o Ministério do Desenvolvimento Social § 4o  Os Conselhos de que tratam os incisos II, III e IV
e Combate à Fome.” (NR) do art. 16, com competência para acompanhar a execução
“Art. 12.  ....................................................................... da política de assistência social, apreciar e aprovar a pro-
............................................................................................. posta orçamentária, em consonância com as diretrizes das
II - cofinanciar, por meio de transferência automática, o conferências nacionais, estaduais, distrital e municipais, de
aprimoramento da gestão, os serviços, os programas e os acordo com seu âmbito de atuação, deverão ser instituí-
projetos de assistência social em âmbito nacional; dos, respectivamente, pelos Estados, pelo Distrito Federal e
............................................................................................. pelos Municípios, mediante lei específica.” (NR)

141
LEGISLAÇÃO

“Art. 20.  O benefício de prestação continuada é a ga- § 2o  O CNAS, ouvidas as respectivas representações
rantia de um salário-mínimo mensal à pessoa com defi- de Estados e Municípios dele participantes, poderá propor,
ciência e ao idoso com 65 (sessenta e cinco) anos ou mais na medida das disponibilidades orçamentárias das 3 (três)
que comprovem não possuir meios de prover a própria esferas de governo, a instituição de benefícios subsidiários
manutenção nem de tê-la provida por sua família. no valor de até 25% (vinte e cinco por cento) do salário-mí-
§ 1o  Para os efeitos do disposto no caput, a família é nimo para cada criança de até 6 (seis) anos de idade.
composta pelo requerente, o cônjuge ou companheiro, os § 3o  Os benefícios eventuais subsidiários não poderão
pais e, na ausência de um deles, a madrasta ou o padrasto, ser cumulados com aqueles instituídos pelas Leis no 10.954,
os irmãos solteiros, os filhos e enteados solteiros e os me- de 29 de setembro de 2004, e no 10.458, de 14 de maio de
nores tutelados, desde que vivam sob o mesmo teto. 2002.” (NR)
§ 2o  Para efeito de concessão deste benefício, consi- “Art. 23.  Entendem-se por serviços socioassistenciais
dera-se: as atividades continuadas que visem à melhoria de vida
I - pessoa com deficiência: aquela que tem impedimen- da população e cujas ações, voltadas para as necessidades
tos de longo prazo de natureza física, intelectual ou senso-
básicas, observem os objetivos, princípios e diretrizes esta-
rial, os quais, em interação com diversas barreiras, podem
belecidos nesta Lei.
obstruir sua participação plena e efetiva na sociedade com
§ 1o  O regulamento instituirá os serviços socioassis-
as demais pessoas;
II - impedimentos de longo prazo: aqueles que incapa- tenciais.
citam a pessoa com deficiência para a vida independente e § 2o  Na organização dos serviços da assistência social
para o trabalho pelo prazo mínimo de 2 (dois) anos. serão criados programas de amparo, entre outros:
§ 3o  Considera-se incapaz de prover a manutenção da I - às crianças e adolescentes em situação de risco pes-
pessoa com deficiência ou idosa a família cuja renda men- soal e social, em cumprimento ao disposto no art. 227 da
sal  per capita seja inferior a 1/4 (um quarto) do salário- Constituição Federal e na Lei no  8.069, de 13 de julho de
mínimo. 1990 (Estatuto da Criança e do Adolescente);
§ 4o  O benefício de que trata este artigo não pode ser II - às pessoas que vivem em situação de rua.” (NR)
acumulado pelo beneficiário com qualquer outro no âm- “Art. 24.  ........................................................................
bito da seguridade social ou de outro regime, salvo os da .............................................................................................
assistência médica e da pensão especial de natureza inde- § 2o  Os programas voltados para o idoso e a integra-
nizatória. ção da pessoa com deficiência serão devidamente articula-
§ 5o  A condição de acolhimento em instituições de dos com o benefício de prestação continuada estabelecido
longa permanência não prejudica o direito do idoso ou da no art. 20 desta Lei.” (NR)
pessoa com deficiência ao benefício de prestação conti- “Art. 28.  ..........................................................................
nuada. § 1o  Cabe ao órgão da Administração Pública respon-
§ 6o  A concessão do benefício ficará sujeita à avaliação sável pela coordenação da Política de Assistência Social nas
da deficiência e do grau de incapacidade, composta por 3 (três) esferas de governo gerir o Fundo de Assistência So-
avaliação médica e avaliação social realizadas por médicos cial, sob orientação e controle dos respectivos Conselhos
peritos e por assistentes sociais do Instituto Nacional do de Assistência Social.
Seguro Social (INSS). .............................................................................................
...................................................................................” (NR) § 3o   O financiamento da assistência social no Suas
“Art. 21.  ........................................................................ deve ser efetuado mediante cofinanciamento dos 3 (três)
............................................................................................. entes federados, devendo os recursos alocados nos fundos
§ 3o  O desenvolvimento das capacidades cognitivas,
de assistência social ser voltados à operacionalização, pres-
motoras ou educacionais e a realização de atividades não
tação, aprimoramento e viabilização dos serviços, progra-
remuneradas de habilitação e reabilitação, entre outras,
mas, projetos e benefícios desta política.” (NR)
não constituem motivo de suspensão ou cessação do be-
nefício da pessoa com deficiência. “Art. 36.  As entidades e organizações de assistência
§ 4o  A cessação do benefício de prestação continuada social que incorrerem em irregularidades na aplicação dos
concedido à pessoa com deficiência, inclusive em razão do recursos que lhes foram repassados pelos poderes públi-
seu ingresso no mercado de trabalho, não impede nova cos terão a sua vinculação ao Suas cancelada, sem prejuízo
concessão do benefício, desde que atendidos os requisitos de responsabilidade civil e penal.” (NR)
definidos em regulamento.” (NR) Art. 2o  A Lei no 8.742, de 1993, passa a vigorar acrescida
“Art. 22.  Entendem-se por benefícios eventuais as pro- dos seguintes artigos:
visões suplementares e provisórias que integram organica- “Art. 6o-A.  A assistência social organiza-se pelos se-
mente as garantias do Suas e são prestadas aos cidadãos e guintes tipos de proteção:
às famílias em virtude de nascimento, morte, situações de I - proteção social básica: conjunto de serviços, pro-
vulnerabilidade temporária e de calamidade pública. gramas, projetos e benefícios da assistência social que visa
§ 1o  A concessão e o valor dos benefícios de que trata a prevenir situações de vulnerabilidade e risco social por
este artigo serão definidos pelos Estados, Distrito Federal meio do desenvolvimento de potencialidades e aquisições
e Municípios e previstos nas respectivas leis orçamentárias e do fortalecimento de vínculos familiares e comunitários;
anuais, com base em critérios e prazos definidos pelos res-
pectivos Conselhos de Assistência Social.

142
LEGISLAÇÃO

II - proteção social especial: conjunto de serviços, pro- “Art. 6º-D.  As instalações dos Cras e dos Creas de-
gramas e projetos que tem por objetivo contribuir para a vem ser compatíveis com os serviços neles ofertados, com
reconstrução de vínculos familiares e comunitários, a defe- espaços para trabalhos em grupo e ambientes específicos
sa de direito, o fortalecimento das potencialidades e aqui- para recepção e atendimento reservado das famílias e in-
sições e a proteção de famílias e indivíduos para o enfren- divíduos, assegurada a acessibilidade às pessoas idosas e
tamento das situações de violação de direitos. com deficiência.”
Parágrafo único.  A vigilância socioassistencial é um “Art. 6º-E.   Os recursos do cofinanciamento do Suas,
dos instrumentos das proteções da assistência social que destinados à execução das ações continuadas de assistên-
identifica e previne as situações de risco e vulnerabilidade cia social, poderão ser aplicados no pagamento dos profis-
social e seus agravos no território.” sionais que integrarem as equipes de referência, responsá-
“Art. 6º-B.  As proteções sociais básica e especial serão veis pela organização e oferta daquelas ações, conforme
ofertadas pela rede socioassistencial, de forma integrada, percentual apresentado pelo Ministério do Desenvolvi-
diretamente pelos entes públicos e/ou pelas entidades e mento Social e Combate à Fome e aprovado pelo CNAS.
organizações de assistência social vinculadas ao Suas, res-
Parágrafo único.  A formação das equipes de referên-
peitadas as especificidades de cada ação.
cia deverá considerar o número de famílias e indivíduos
§ 1o  A vinculação ao Suas é o reconhecimento pelo
referenciados, os tipos e modalidades de atendimento e as
Ministério do Desenvolvimento Social e Combate à Fome
de que a entidade de assistência social integra a rede so- aquisições que devem ser garantidas aos usuários, confor-
cioassistencial. me deliberações do CNAS.”
§ 2o  Para o reconhecimento referido no § 1o, a entidade “Art. 12-A.  A União apoiará financeiramente o aprimo-
deverá cumprir os seguintes requisitos: ramento à gestão descentralizada dos serviços, programas,
I - constituir-se em conformidade com o disposto no projetos e benefícios de assistência social, por meio do Ín-
art. 3o; dice de Gestão Descentralizada (IGD) do Sistema Único de
II - inscrever-se em Conselho Municipal ou do Distrito Assistência Social (Suas), para a utilização no âmbito dos
Federal, na forma do art. 9o; Estados, dos Municípios e do Distrito Federal, destinado,
III - integrar o sistema de cadastro de entidades de que sem prejuízo de outras ações a serem definidas em regu-
trata o inciso XI do art. 19. lamento, a:
§ 3o  As entidades e organizações de assistência so- I - medir os resultados da gestão descentralizada do
cial vinculadas ao Suas celebrarão convênios, contratos, Suas, com base na atuação do gestor estadual, municipal
acordos ou ajustes com o poder público para a execução, e do Distrito Federal na implementação, execução e moni-
garantido financiamento integral, pelo Estado, de serviços, toramento dos serviços, programas, projetos e benefícios
programas, projetos e ações de assistência social, nos limi- de assistência social, bem como na articulação intersetorial;
tes da capacidade instalada, aos beneficiários abrangidos II - incentivar a obtenção de resultados qualitativos na
por esta Lei, observando-se as disponibilidades orçamen- gestão estadual, municipal e do Distrito Federal do Suas; e
tárias. III - calcular o montante de recursos a serem repas-
§ 4o  O cumprimento do disposto no § 3o será informa- sados aos entes federados a título de apoio financeiro à
do ao Ministério do Desenvolvimento Social e Combate à gestão do Suas.
Fome pelo órgão gestor local da assistência social.” § 1o  Os resultados alcançados pelo ente federado na
“Art. 6º-C.  As proteções sociais, básica e especial, se- gestão do Suas, aferidos na forma de regulamento, serão
rão ofertadas precipuamente no Centro de Referência de considerados como prestação de contas dos recursos a se-
Assistência Social (Cras) e no Centro de Referência Espe- rem transferidos a título de apoio financeiro.
cializado de Assistência Social (Creas), respectivamente, e
§ 2o  As transferências para apoio à gestão descentra-
pelas entidades sem fins lucrativos de assistência social de
lizada do Suas adotarão a sistemática do Índice de Gestão
que trata o art. 3o desta Lei.
Descentralizada do Programa Bolsa Família, previsto no art.
§ 1o  O Cras é a unidade pública municipal, de base
territorial, localizada em áreas com maiores índices de vul- 8o  da Lei no10.836, de 9 de janeiro de 2004, e serão efeti-
nerabilidade e risco social, destinada à articulação dos ser- vadas por meio de procedimento integrado àquele índice.
viços socioassistenciais no seu território de abrangência e à § 3o  (VETADO).
prestação de serviços, programas e projetos socioassisten- § 4o  Para fins de fortalecimento dos Conselhos de As-
ciais de proteção social básica às famílias. sistência Social dos Estados, Municípios e Distrito Federal,
§ 2o  O Creas é a unidade pública de abrangência e ges- percentual dos recursos transferidos deverá ser gasto com
tão municipal, estadual ou regional, destinada à prestação atividades de apoio técnico e operacional àqueles colegia-
de serviços a indivíduos e famílias que se encontram em dos, na forma fixada pelo Ministério do Desenvolvimento
situação de risco pessoal ou social, por violação de direitos Social e Combate à Fome, sendo vedada a utilização dos
ou contingência, que demandam intervenções especializa- recursos para pagamento de pessoal efetivo e de gratifi-
das da proteção social especial. cações de qualquer natureza a servidor público estadual,
§ 3o  Os Cras e os Creas são unidades públicas esta- municipal ou do Distrito Federal.”
tais instituídas no âmbito do Suas, que possuem interface “Art. 24-A.  Fica instituído o Serviço de Proteção e
com as demais políticas públicas e articulam, coordenam Atendimento Integral à Família (Paif), que integra a pro-
e ofertam os serviços, programas, projetos e benefícios da teção social básica e consiste na oferta de ações e servi-
assistência social.” ços socioassistenciais de prestação continuada, nos Cras,

143
LEGISLAÇÃO

por meio do trabalho social com famílias em situação de Parágrafo único.  Os entes transferidores poderão re-
vulnerabilidade social, com o objetivo de prevenir o rom- quisitar informações referentes à aplicação dos recursos
pimento dos vínculos familiares e a violência no âmbito de oriundos do seu fundo de assistência social, para fins de
suas relações, garantindo o direito à convivência familiar e análise e acompanhamento de sua boa e regular utilização.”
comunitária. Art. 3o  Revoga-se o art. 38 da Lei nº 8.742, de 7 de de-
Parágrafo único.  Regulamento definirá as diretrizes e zembro de 1993. 
os procedimentos do Paif.” Art. 4o  Esta Lei entra em vigor na data de sua publicação. 
“Art. 24-B.  Fica instituído o Serviço de Proteção e Brasília,  6  de julho de 2011; 190o  da Independência e
Atendimento Especializado a Famílias e Indivíduos (Paefi), 123 da República. 

que integra a proteção social especial e consiste no apoio,


orientação e acompanhamento a famílias e indivíduos em
situação de ameaça ou violação de direitos, articulando os SISTEMA NACIONAL DE ATENDIMENTO
serviços socioassistenciais com as diversas políticas públi- SOCIOEDUCATIVO – Lei 12.594/2012
cas e com órgãos do sistema de garantia de direitos.
Parágrafo único.  Regulamento definirá as diretrizes e
os procedimentos do Paefi.”
“Art. 24-C.  Fica instituído o Programa de Erradicação LEI Nº 12.594, DE 18 DE JANEIRO DE 2012.
do Trabalho Infantil (Peti), de caráter intersetorial, integran-
te da Política Nacional de Assistência Social, que, no âmbi- Institui o Sistema Nacional de Atendimento Socioedu-
to do Suas, compreende transferências de renda, trabalho cativo (Sinase), regulamenta a execução das medidas so-
social com famílias e oferta de serviços socioeducativos cioeducativas destinadas a adolescente que pratique ato
para crianças e adolescentes que se encontrem em situa- infracional; e altera as Leis nos 8.069, de 13 de julho de 1990
ção de trabalho. (Estatuto da Criança e do Adolescente); 7.560, de 19 de de-
§ 1o  O Peti tem abrangência nacional e será desenvol- zembro de 1986, 7.998, de 11 de janeiro de 1990, 5.537,
vido de forma articulada pelos entes federados, com a par- de 21 de novembro de 1968, 8.315, de 23 de dezembro de
ticipação da sociedade civil, e tem como objetivo contribuir 1991, 8.706, de 14 de setembro de 1993, os Decretos-Leis
para a retirada de crianças e adolescentes com idade infe- nos4.048, de 22 de janeiro de 1942, 8.621, de 10 de janeiro de
rior a 16 (dezesseis) anos em situação de trabalho, ressalva- 1946, e a Consolidação das Leis do Trabalho (CLT), aprovada
da a condição de aprendiz, a partir de 14 (quatorze) anos. pelo Decreto-Lei no 5.452, de 1o de maio de 1943.
§ 2o  As crianças e os adolescentes em situação de tra-
balho deverão ser identificados e ter os seus dados inseri-  proteção, de advertência e de reparação do dano,
dos no Cadastro Único para Programas Sociais do Governo quando aplicadas de forma isolada, serão executadas nos
Federal (CadÚnico), com a devida identificação das situa- próprios autos do processo de conhecimento, respeitado o
ções de trabalho infantil.” disposto nos arts. 143 e 144 da Lei no 8.069, de 13 de julho de 1990 (Estatuto
“Art. 30-A.   O cofinanciamento dos serviços, progra- da Criança e do Adolescente)

mas, projetos e benefícios eventuais, no que couber, e o Art. 39.  Para aplicação das medidas socioeducativas
aprimoramento da gestão da política de assistência social de prestação de serviços à comunidade, liberdade assisti-
no Suas se efetuam por meio de transferências automáticas da, semiliberdade ou internação, será constituído processo
entre os fundos de assistência social e mediante alocação de execução para cada adolescente, respeitado o disposto
de recursos próprios nesses fundos nas 3 (três) esferas de nos arts. 143 e 144 da Lei nº 8.069, de 13 de julho de 1990
governo. (Estatuto da Criança e do Adolescente), e com autuação das
Parágrafo único.  As transferências automáticas de re- seguintes peças: 
cursos entre os fundos de assistência social efetuadas à I - documentos de caráter pessoal do adolescente exis-
conta do orçamento da seguridade social, conforme o art. tentes no processo de conhecimento, especialmente os que
204 da Constituição Federal, caracterizam-se como despe- comprovem sua idade; e
sa pública com a seguridade social, na forma do art. 24 da II - as indicadas pela autoridade judiciária, sempre que
Lei Complementar no 101, de 4 de maio de 2000.” houver necessidade e, obrigatoriamente: 
“Art. 30-B.  Caberá ao ente federado responsável pela a) cópia da representação; 
utilização dos recursos do respectivo Fundo de Assistência b) cópia da certidão de antecedentes; 
Social o controle e o acompanhamento dos serviços, pro- c) cópia da sentença ou acórdão; e 
gramas, projetos e benefícios, por meio dos respectivos ór- d) cópia de estudos técnicos realizados durante a fase
gãos de controle, independentemente de ações do órgão de conhecimento. 
repassador dos recursos.” Parágrafo único.  Procedimento idêntico será observado
“Art. 30-C.  A utilização dos recursos federais descen- na hipótese de medida aplicada em sede de remissão, como
tralizados para os fundos de assistência social dos Esta- forma de suspensão do processo. 
dos, dos Municípios e do Distrito Federal será declarada Art. 40.  Autuadas as peças, a autoridade judiciária en-
pelos entes recebedores ao ente transferidor, anualmente, caminhará, imediatamente, cópia integral do expediente ao
mediante relatório de gestão submetido à apreciação do órgão gestor do atendimento socioeducativo, solicitando
respectivo Conselho de Assistência Social, que comprove a designação do programa ou da unidade de cumprimento
execução das ações na forma de regulamento. da medida. 

144
LEGISLAÇÃO

Art. 41.  A autoridade judiciária dará vistas da proposta § 3o  Admitido o processamento do pedido, a autorida-
de plano individual de que trata o art. 53 desta Lei ao de- de judiciária, se necessário, designará audiência, observan-
fensor e ao Ministério Público pelo prazo sucessivo de 3 do o princípio do § 1o do art. 42 desta Lei. 
(três) dias, contados do recebimento da proposta encami- § 4o  A substituição por medida mais gravosa somente
nhada pela direção do programa de atendimento.  ocorrerá em situações excepcionais, após o devido proces-
§ 1o  O defensor e o Ministério Público poderão reque- so legal, inclusive na hipótese do inciso III do art. 122 da
rer, e o Juiz da Execução poderá determinar, de ofício, a Lei no 8.069, de 13 de julho de 1990 (Estatuto da Criança e do Adolescente), e deve ser: 
realização de qualquer avaliação ou perícia que entende- I - fundamentada em parecer técnico; 
rem necessárias para complementação do plano individual.  II - precedida de prévia audiência, e nos termos do §
§ 2o  A impugnação ou complementação do plano indi- 1o do art. 42 desta Lei. 
vidual, requerida pelo defensor ou pelo Ministério Público, Art. 44.  Na hipótese de substituição da medida ou mo-
deverá ser fundamentada, podendo a autoridade judiciária dificação das atividades do plano individual, a autoridade
indeferi-la, se entender insuficiente a motivação.  judiciária remeterá o inteiro teor da decisão à direção do
§ 3o  Admitida a impugnação, ou se entender que o programa de atendimento, assim como as peças que en-
plano é inadequado, a autoridade judiciária designará, se tender relevantes à nova situação jurídica do adolescente. 
necessário, audiência da qual cientificará o defensor, o Mi- Parágrafo único.  No caso de a substituição da medida
nistério Público, a direção do programa de atendimento, o importar em vinculação do adolescente a outro programa
adolescente e seus pais ou responsável.  de atendimento, o plano individual e o histórico do cum-
§ 4o  A impugnação não suspenderá a execução do pla- primento da medida deverão acompanhar a transferência. 
no individual, salvo determinação judicial em contrário.  Art. 45.  Se, no transcurso da execução, sobrevier sen-
§ 5o  Findo o prazo sem impugnação, considerar-se-á o tença de aplicação de nova medida, a autoridade judiciária
plano individual homologado.  procederá à unificação, ouvidos, previamente, o Ministério
Art. 42.  As medidas socioeducativas de liberdade as- Público e o defensor, no prazo de 3 (três) dias sucessivos,
sistida, de semiliberdade e de internação deverão ser rea- decidindo-se em igual prazo. 
valiadas no máximo a cada 6 (seis) meses, podendo a au- § 1o  É vedado à autoridade judiciária determinar reiní-
toridade judiciária, se necessário, designar audiência, no cio de cumprimento de medida socioeducativa, ou deixar
prazo máximo de 10 (dez) dias, cientificando o defensor, o de considerar os prazos máximos, e de liberação compul-
Ministério Público, a direção do programa de atendimento, sória previstos na Lei no  8.069, de 13 de julho de 1990 (Estatuto da Criança e do
o adolescente e seus pais ou responsável. 
Adolescente)
, excetuada a hipótese de medida aplicada por ato
infracional praticado durante a execução. 
§ 1o  A audiência será instruída com o relatório da equi-
§ 2o  É vedado à autoridade judiciária aplicar nova me-
pe técnica do programa de atendimento sobre a evolução
dida de internação, por atos infracionais praticados ante-
do plano de que trata o art. 52 desta Lei e com qualquer
riormente, a adolescente que já tenha concluído cumpri-
outro parecer técnico requerido pelas partes e deferido
mento de medida socioeducativa dessa natureza, ou que
pela autoridade judiciária. 
tenha sido transferido para cumprimento de medida me-
§ 2o  A gravidade do ato infracional, os antecedentes e
nos rigorosa, sendo tais atos absorvidos por aqueles aos
o tempo de duração da medida não são fatores que, por si, quais se impôs a medida socioeducativa extrema. 
justifiquem a não substituição da medida por outra menos Art. 46.  A medida socioeducativa será declarada ex-
grave.  tinta: 
§ 3o  Considera-se mais grave a internação, em relação I - pela morte do adolescente; 
a todas as demais medidas, e mais grave a semiliberdade, II - pela realização de sua finalidade; 
em relação às medidas de meio aberto.  III - pela aplicação de pena privativa de liberdade, a ser
Art. 43.  A reavaliação da manutenção, da substituição cumprida em regime fechado ou semiaberto, em execução
ou da suspensão das medidas de meio aberto ou de priva- provisória ou definitiva; 
ção da liberdade e do respectivo plano individual pode ser IV - pela condição de doença grave, que torne o ado-
solicitada a qualquer tempo, a pedido da direção do pro- lescente incapaz de submeter-se ao cumprimento da me-
grama de atendimento, do defensor, do Ministério Público, dida; e 
do adolescente, de seus pais ou responsável.  V - nas demais hipóteses previstas em lei. 
§ 1o  Justifica o pedido de reavaliação, entre outros mo- § 1o  No caso de o maior de 18 (dezoito) anos, em cum-
tivos:  primento de medida socioeducativa, responder a proces-
I - o desempenho adequado do adolescente com base so-crime, caberá à autoridade judiciária decidir sobre even-
no seu plano de atendimento individual, antes do prazo da tual extinção da execução, cientificando da decisão o juízo
reavaliação obrigatória;  criminal competente. 
II - a inadaptação do adolescente ao programa e o rei- § 2o  Em qualquer caso, o tempo de prisão cautelar não
terado descumprimento das atividades do plano individual; convertida em pena privativa de liberdade deve ser des-
e  contado do prazo de cumprimento da medida socioedu-
III - a necessidade de modificação das atividades do cativa. 
plano individual que importem em maior restrição da liber- Art. 47.  O mandado de busca e apreensão do adoles-
dade do adolescente.  cente terá vigência máxima de 6 (seis) meses, a contar da
§ 2o  A autoridade judiciária poderá indeferir o pedido, data da expedição, podendo, se necessário, ser renovado,
de pronto, se entender insuficiente a motivação.  fundamentadamente. 

145
LEGISLAÇÃO

Art. 48.  O defensor, o Ministério Público, o adolescen- Art. 50.  Sem prejuízo do disposto no § 1o do art. 121 da Lei no 8.069,
te e seus pais ou responsável poderão postular revisão ju- , a direção do programa
de 13 de julho de 1990 (Estatuto da Criança e do Adolescente)

dicial de qualquer sanção disciplinar aplicada, podendo a de execução de medida de privação da liberdade poderá au-
autoridade judiciária suspender a execução da sanção até torizar a saída, monitorada, do adolescente nos casos de tra-
decisão final do incidente.  tamento médico, doença grave ou falecimento, devidamente
§ 1o  Postulada a revisão após ouvida a autoridade co- comprovados, de pai, mãe, filho, cônjuge, companheiro ou
legiada que aplicou a sanção e havendo provas a produzir irmão, com imediata comunicação ao juízo competente. 
em audiência, procederá o magistrado na forma do § 1o do Art. 51.  A decisão judicial relativa à execução de medida
art. 42 desta Lei.  socioeducativa será proferida após manifestação do defen-
§ 2o  É vedada a aplicação de sanção disciplinar de iso- sor e do Ministério Público. 
lamento a adolescente interno, exceto seja essa imprescin-
dível para garantia da segurança de outros internos ou do CAPÍTULO IV
DO PLANO INDIVIDUAL DE ATENDIMENTO (PIA)
próprio adolescente a quem seja imposta a sanção, sendo
necessária ainda comunicação ao defensor, ao Ministério
Art. 52.  O cumprimento das medidas socioeducativas,
Público e à autoridade judiciária em até 24 (vinte e quatro)
em regime de prestação de serviços à comunidade, liberdade
horas.  assistida, semiliberdade ou internação, dependerá de Plano
Individual de Atendimento (PIA), instrumento de previsão,
CAPÍTULO III registro e gestão das atividades a serem desenvolvidas com
DOS DIREITOS INDIVIDUAIS o adolescente. 
Parágrafo único.  O PIA deverá contemplar a participação
Art. 49.  São direitos do adolescente submetido ao dos pais ou responsáveis, os quais têm o dever de contribuir
cumprimento de medida socioeducativa, sem prejuízo de com o processo ressocializador do adolescente, sendo es-
outros previstos em lei:  ses passíveis de responsabilização administrativa, nos termos
I - ser acompanhado por seus pais ou responsável e do art. 249 da Lei no 8.069, de 13 de julho de 1990 (Estatuto da Criança e do Adolescente),
por seu defensor, em qualquer fase do procedimento ad- civil e criminal. 
ministrativo ou judicial;  Art. 53.  O PIA será elaborado sob a responsabilidade
II - ser incluído em programa de meio aberto quando da equipe técnica do respectivo programa de atendimento,
inexistir vaga para o cumprimento de medida de privação com a participação efetiva do adolescente e de sua família,
da liberdade, exceto nos casos de ato infracional cometido representada por seus pais ou responsável. 
mediante grave ameaça ou violência à pessoa, quando o Art. 54.  Constarão do plano individual, no mínimo: 
adolescente deverá ser internado em Unidade mais próxi- I - os resultados da avaliação interdisciplinar; 
ma de seu local de residência;  II - os objetivos declarados pelo adolescente; 
III - ser respeitado em sua personalidade, intimidade, III - a previsão de suas atividades de integração social e/
liberdade de pensamento e religião e em todos os direitos ou capacitação profissional; 
não expressamente limitados na sentença;  IV - atividades de integração e apoio à família; 
IV - peticionar, por escrito ou verbalmente, diretamen- V - formas de participação da família para efetivo cum-
te a qualquer autoridade ou órgão público, devendo, obri- primento do plano individual; e 
gatoriamente, ser respondido em até 15 (quinze) dias;  VI - as medidas específicas de atenção à sua saúde. 
V - ser informado, inclusive por escrito, das normas de Art. 55.  Para o cumprimento das medidas de semiliber-
dade ou de internação, o plano individual conterá, ainda: 
organização e funcionamento do programa de atendimen-
I - a designação do programa de atendimento mais ade-
to e também das previsões de natureza disciplinar; 
quado para o cumprimento da medida; 
VI - receber, sempre que solicitar, informações sobre
II - a definição das atividades internas e externas, indivi-
a evolução de seu plano individual, participando, obriga- duais ou coletivas, das quais o adolescente poderá participar;
toriamente, de sua elaboração e, se for o caso, reavaliação;  e 
VII - receber assistência integral à sua saúde, conforme III - a fixação das metas para o alcance de desenvolvi-
o disposto no art. 60 desta Lei; e  mento de atividades externas. 
VIII - ter atendimento garantido em creche e pré-esco- Parágrafo único.  O PIA será elaborado no prazo de até
la aos filhos de 0 (zero) a 5 (cinco) anos.  45 (quarenta e cinco) dias da data do ingresso do adolescen-
§ 1o  As garantias processuais destinadas a adolescen- te no programa de atendimento. 
te autor de ato infracional previstas na Lei no 8.069, de 13 de julho Art. 56.  Para o cumprimento das medidas de prestação
de 1990 (Estatuto da Criança e do Adolescente)
, aplicam-se integralmente na de serviços à comunidade e de liberdade assistida, o PIA será
execução das medidas socioeducativas, inclusive no âmbi- elaborado no prazo de até 15 (quinze) dias do ingresso do
to administrativo.  adolescente no programa de atendimento. 
§ 2o  A oferta irregular de programas de atendimento Art. 57.  Para a elaboração do PIA, a direção do respecti-
socioeducativo em meio aberto não poderá ser invocada vo programa de atendimento, pessoalmente ou por meio de
como motivo para aplicação ou manutenção de medida de membro da equipe técnica, terá acesso aos autos do proce-
privação da liberdade.  dimento de apuração do ato infracional e aos dos procedi-
mentos de apuração de outros atos infracionais atribuídos
ao mesmo adolescente. 

146
LEGISLAÇÃO

§ 1o  O acesso aos documentos de que trata o caput de- VIII - estruturação das unidades de internação confor-
verá ser realizado por funcionário da entidade de atendi- me as normas de referência do SUS e do Sinase, visando ao
mento, devidamente credenciado para tal atividade, ou por atendimento das necessidades de Atenção Básica. 
membro da direção, em conformidade com as normas a Art. 61.  As entidades que ofereçam programas de
serem definidas pelo Poder Judiciário, de forma a preservar atendimento socioeducativo em meio aberto e de semili-
o que determinam os arts. 143 e 144 da Lei no 8.069, de 13 de julho berdade deverão prestar orientações aos socioeducandos
de 1990 (Estatuto da Criança e do Adolescente)
.  sobre o acesso aos serviços e às unidades do SUS. 
§ 2o  A direção poderá requisitar, ainda:  Art. 62.  As entidades que ofereçam programas de pri-
I - ao estabelecimento de ensino, o histórico escolar do vação de liberdade deverão contar com uma equipe míni-
adolescente e as anotações sobre o seu aproveitamento;  ma de profissionais de saúde cuja composição esteja em
II - os dados sobre o resultado de medida anterior- conformidade com as normas de referência do SUS. 
mente aplicada e cumprida em outro programa de aten- Art. 63.  (VETADO). 
dimento; e  § 1o  O filho de adolescente nascido nos estabeleci-
III - os resultados de acompanhamento especializado mentos referidos no caput deste artigo não terá tal infor-
anterior.  mação lançada em seu registro de nascimento. 
Art. 58.  Por ocasião da reavaliação da medida, é obri-
§ 2o  Serão asseguradas as condições necessárias para
gatória a apresentação pela direção do programa de aten-
que a adolescente submetida à execução de medida so-
dimento de relatório da equipe técnica sobre a evolução
cioeducativa de privação de liberdade permaneça com o
do adolescente no cumprimento do plano individual. 
Art. 59.  O acesso ao plano individual será restrito aos seu filho durante o período de amamentação. 
servidores do respectivo programa de atendimento, ao Seção II
adolescente e a seus pais ou responsável, ao Ministério Pú- Do Atendimento a Adolescente com Transtorno Mental
blico e ao defensor, exceto expressa autorização judicial.  e com Dependência de Álcool e de Substância Psicoativa 
Art 64.  O adolescente em cumprimento de medida so-
CAPÍTULO V cioeducativa que apresente indícios de transtorno mental,
DA ATENÇÃO INTEGRAL À SAÚDE DE ADOLESCEN- de deficiência mental, ou associadas, deverá ser avaliado
TE EM CUMPRIMENTO DE MEDIDA SOCIOEDUCATIVA por equipe técnica multidisciplinar e multissetorial. 
Seção I § 1o  As competências, a composição e a atuação da
Disposições Gerais equipe técnica de que trata o caput deverão seguir, con-
juntamente, as normas de referência do SUS e do Sinase,
Art. 60.  A atenção integral à saúde do adolescente no na forma do regulamento. 
Sistema de Atendimento Socioeducativo seguirá as seguin- § 2o  A avaliação de que trata o caput subsidiará a ela-
tes diretrizes:  boração e execução da terapêutica a ser adotada, a qual
I - previsão, nos planos de atendimento socioedu- será incluída no PIA do adolescente, prevendo, se necessá-
cativo, em todas as esferas, da implantação de ações de rio, ações voltadas para a família. 
promoção da saúde, com o objetivo de integrar as ações § 3o  As informações produzidas na avaliação de que
socioeducativas, estimulando a autonomia, a melhoria das trata o caput são consideradas sigilosas. 
relações interpessoais e o fortalecimento de redes de apoio § 4o  Excepcionalmente, o juiz poderá suspender a exe-
aos adolescentes e suas famílias;  cução da medida socioeducativa, ouvidos o defensor e o
II - inclusão de ações e serviços para a promoção, pro- Ministério Público, com vistas a incluir o adolescente em
teção, prevenção de agravos e doenças e recuperação da programa de atenção integral à saúde mental que melhor
saúde;  atenda aos objetivos terapêuticos estabelecidos para o seu
III - cuidados especiais em saúde mental, incluindo os caso específico. 
relacionados ao uso de álcool e outras substâncias psicoa-
§ 5o  Suspensa a execução da medida socioeducativa,
tivas, e atenção aos adolescentes com deficiências; 
o juiz designará o responsável por acompanhar e informar
IV - disponibilização de ações de atenção à saúde se-
sobre a evolução do atendimento ao adolescente. 
xual e reprodutiva e à prevenção de doenças sexualmente
transmissíveis;  § 6o  A suspensão da execução da medida socioeduca-
V - garantia de acesso a todos os níveis de atenção à tiva será avaliada, no mínimo, a cada 6 (seis) meses. 
saúde, por meio de referência e contrarreferência, de acor- § 7o  O tratamento a que se submeterá o adolescente
do com as normas do Sistema Único de Saúde (SUS);  deverá observar o previsto na Lei no 10.216, de 6 de abril de 2001, que
VI - capacitação das equipes de saúde e dos profissio- dispõe sobre a proteção e os direitos das pessoas portado-
nais das entidades de atendimento, bem como daqueles ras de transtornos mentais e redireciona o modelo assis-
que atuam nas unidades de saúde de referência voltadas tencial em saúde mental. 
às especificidades de saúde dessa população e de suas fa- § 8o  (VETADO). 
mílias;  Art. 65.  Enquanto não cessada a jurisdição da Infância
VII - inclusão, nos Sistemas de Informação de Saúde do e Juventude, a autoridade judiciária, nas hipóteses tratadas
SUS, bem como no Sistema de Informações sobre Atendi- no art. 64, poderá remeter cópia dos autos ao Ministério
mento Socioeducativo, de dados e indicadores de saúde Público para eventual propositura de interdição e outras
da população de adolescentes em atendimento socioedu- providências pertinentes. 
cativo; e  Art. 66.  (VETADO). 

147
LEGISLAÇÃO

CAPÍTULO VI I - por coação irresistível ou por motivo de força maior; 


DAS VISITAS A ADOLESCENTE EM CUMPRIMENTO II - em legítima defesa, própria ou de outrem. 
DE MEDIDA DE
INTERNAÇÃO CAPÍTULO VIII
DA CAPACITAÇÃO PARA O TRABALHO
Art. 67.  A visita do cônjuge, companheiro, pais ou res-
ponsáveis, parentes e amigos a adolescente a quem foi Art. 76.  O art. 2o do Decreto-Lei no 4.048, de 22 de janeiro de 1942,
aplicada medida socioeducativa de internação observará passa a vigorar acrescido do seguinte § 1o, renumerando-se
dias e horários próprios definidos pela direção do progra- o atual parágrafo único para § 2o: 
ma de atendimento.  “Art. 2o  ......................................................................... 
Art. 68.  É assegurado ao adolescente casado ou que § 1o    As escolas do Senai poderão ofertar vagas aos
viva, comprovadamente, em união estável o direito à visita usuários do Sistema Nacional de Atendimento Socioedu-
íntima.  cativo (Sinase) nas condições a serem dispostas em instru-
Parágrafo único.  O visitante será identificado e regis- mentos de cooperação celebrados entre os operadores do
trado pela direção do programa de atendimento, que emi- Senai e os gestores dos Sistemas de Atendimento Socioe-
tirá documento de identificação, pessoal e intransferível, ducativo locais. 
específico para a realização da visita íntima.  § 2o  ...................................................................... ” (NR) 
Art. 69.  É garantido aos adolescentes em cumprimento Art. 77.  O art. 3o do Decreto-Lei no 8.621, de 10 de janeiro de 1946,
de medida socioeducativa de internação o direito de rece- passa a vigorar acrescido do seguinte § 1o, renumerando-se
ber visita dos filhos, independentemente da idade desses.  o atual parágrafo único para § 2o: 
Art. 70.  O regulamento interno estabelecerá as hipó- “Art. 3o  ......................................................................... 
teses de proibição da entrada de objetos na unidade de § 1o    As escolas do Senac poderão ofertar vagas aos
internação, vedando o acesso aos seus portadores.  usuários do Sistema Nacional de Atendimento Socioedu-
cativo (Sinase) nas condições a serem dispostas em instru-
CAPÍTULO VII mentos de cooperação celebrados entre os operadores do
DOS REGIMES DISCIPLINARES
Senac e os gestores dos Sistemas de Atendimento Socioe-
ducativo locais. 
Art. 71.  Todas as entidades de atendimento socioedu-
§ 2o. ..................................................................... ” (NR) 
cativo deverão, em seus respectivos regimentos, realizar a
Art. 78.  O art. 1o da Lei no 8.315, de 23 de dezembro de 1991, passa a
previsão de regime disciplinar que obedeça aos seguintes
vigorar acrescido do seguinte parágrafo único: 
princípios: 
“Art. 1o  ......................................................................... 
I - tipificação explícita das infrações como leves, médias
Parágrafo único.   Os programas de formação profis-
e graves e determinação das correspondentes sanções; 
II - exigência da instauração formal de processo disci- sional rural do Senar poderão ofertar vagas aos usuários
plinar para a aplicação de qualquer sanção, garantidos a do Sistema Nacional de Atendimento Socioeducativo (Si-
ampla defesa e o contraditório;  nase) nas condições a serem dispostas em instrumentos
III - obrigatoriedade de audiência do socioeducando de cooperação celebrados entre os operadores do Senar e
nos casos em que seja necessária a instauração de proces- os gestores dos Sistemas de Atendimento Socioeducativo
so disciplinar;  locais.” (NR) 
IV - sanção de duração determinada;  Art. 79.  O art. 3o da Lei no 8.706, de 14 de setembro de 1993, passa a
V - enumeração das causas ou circunstâncias que vigorar acrescido do seguinte parágrafo único: 
eximam, atenuem ou agravem a sanção a ser imposta ao “Art. 3o  ......................................................................... 
socioeducando, bem como os requisitos para a extinção Parágrafo único.  Os programas de formação profissio-
dessa;  nal do Senat poderão ofertar vagas aos usuários do Siste-
VI - enumeração explícita das garantias de defesa;  ma Nacional de Atendimento Socioeducativo (Sinase) nas
VII - garantia de solicitação e rito de apreciação dos condições a serem dispostas em instrumentos de coopera-
recursos cabíveis; e  ção celebrados entre os operadores do Senat e os gestores
VIII - apuração da falta disciplinar por comissão com- dos Sistemas de Atendimento Socioeducativo locais.” (NR) 
posta por, no mínimo, 3 (três) integrantes, sendo 1 (um), Art. 80.  O art. 429 do Decreto-Lei no 5.452, de 1o de maio de 1943,
obrigatoriamente, oriundo da equipe técnica.  passa a vigorar acrescido do seguinte § 2o: 
Art. 72.  O regime disciplinar é independente da res- “Art. 429.  .....................................................................
ponsabilidade civil ou penal que advenha do ato cometido.  ............................................................................................. 
Art. 73.  Nenhum socioeducando poderá desempenhar § 2o  Os estabelecimentos de que trata o caput oferta-
função ou tarefa de apuração disciplinar ou aplicação de rão vagas de aprendizes a adolescentes usuários do Siste-
sanção nas entidades de atendimento socioeducativo.  ma Nacional de Atendimento Socioeducativo (Sinase) nas
Art. 74.  Não será aplicada sanção disciplinar sem ex- condições a serem dispostas em instrumentos de coope-
pressa e anterior previsão legal ou regulamentar e o devido ração celebrados entre os estabelecimentos e os gestores
processo administrativo.  dos Sistemas de Atendimento Socioeducativo locais.” (NR) 
Art. 75.  Não será aplicada sanção disciplinar ao socioe-
ducando que tenha praticado a falta: 

148
LEGISLAÇÃO

TÍTULO III “Art. 198.   Nos procedimentos afetos à Justiça da In-


DISPOSIÇÕES FINAIS E TRANSITÓRIAS fância e da Juventude, inclusive os relativos à execução das
medidas socioeducativas, adotar-se-á o sistema recursal
Art. 81.  As entidades que mantenham programas de da Lei no 5.869, de 11 de janeiro de 1973 (Código de Processo Civil), com as seguin-
atendimento têm o prazo de até 6 (seis) meses após a pu- tes adaptações:
blicação desta Lei para encaminhar ao respectivo Conselho ............................................................................................. 
Estadual ou Municipal dos Direitos da Criança e do Adoles- II - em todos os recursos, salvo nos embargos de de-
cente proposta de adequação da sua inscrição, sob pena claração, o prazo para o Ministério Público e para a defesa
de interdição.  será sempre de 10 (dez) dias;
Art. 82.  Os Conselhos dos Direitos da Criança e do ...................................................................................” (NR) 
Adolescente, em todos os níveis federados, com os órgãos “Art. 208.  .....................................................................
responsáveis pelo sistema de educação pública e as enti- ............................................................................................. 
dades de atendimento, deverão, no prazo de 1 (um) ano a X - de programas de atendimento para a execução das
partir da publicação desta Lei, garantir a inserção de ado- medidas socioeducativas e aplicação de medidas de pro-
lescentes em cumprimento de medida socioeducativa na teção.
rede pública de educação, em qualquer fase do período ...................................................................................” (NR) 
letivo, contemplando as diversas faixas etárias e níveis de Art. 87.  A Lei nº 8.069, de 13 de julho de 1990 (Esta-
instrução.  tuto da Criança e do Adolescente), passa a vigorar com as
Art. 83.  Os programas de atendimento socioeducativo seguintes alterações: 
sob a responsabilidade do Poder Judiciário serão, obriga- “Art. 260.   Os contribuintes poderão efetuar doações
toriamente, transferidos ao Poder Executivo no prazo má- aos Fundos dos Direitos da Criança e do Adolescente na-
ximo de 1 (um) ano a partir da publicação desta Lei e de cional, distrital, estaduais ou municipais, devidamente
acordo com a política de oferta dos programas aqui defi- comprovadas, sendo essas integralmente deduzidas do
nidos.  imposto de renda, obedecidos os seguintes limites: 
Art. 84.  Os programas de internação e semiliberdade I - 1% (um por cento) do imposto sobre a renda devi-
sob a responsabilidade dos Municípios serão, obrigatoria- do apurado pelas pessoas jurídicas tributadas com base no
mente, transferidos para o Poder Executivo do respectivo lucro real; e 
Estado no prazo máximo de 1 (um) ano a partir da publi-
II - 6% (seis por cento) do imposto sobre a renda apu-
cação desta Lei e de acordo com a política de oferta dos
rado pelas pessoas físicas na Declaração de Ajuste Anual,
programas aqui definidos. 
observado o disposto no art. 22 da Lei no 9.532, de 10 de dezembro
Art. 85.  A não transferência de programas de atendi- de 1997
.
mento para os devidos entes responsáveis, no prazo deter-
............................................................................................. 
minado nesta Lei, importará na interdição do programa e
§ 5o  Observado o disposto no § 4o do art. 3o da Lei no 9.249, de 26
caracterizará ato de improbidade administrativa do agente de dezembro de 1995
, a dedução de que trata o inciso I do caput: 
responsável, vedada, ademais, ao Poder Judiciário e ao Po-
der Executivo municipal, ao final do referido prazo, a reali- I - será considerada isoladamente, não se submetendo
zação de despesas para a sua manutenção.  a limite em conjunto com outras deduções do imposto; e 
Art. 86.  Os arts. 90, 97, 121, 122, 198 e 208 da Lei no 8.069, II - não poderá ser computada como despesa opera-
de 13 de julho de 1990 (Estatuto da Criança e do Adolescente)
, passam a vigorar com cional na apuração do lucro real.” (NR) 
a seguinte redação:  “Art. 260-A.  A partir do exercício de 2010, ano-calen-
“Art. 90.  ...................................................................... dário de 2009, a pessoa física poderá optar pela doação de
.............................................................................................  que trata o inciso II do caput do art. 260 diretamente em
V - prestação de serviços à comunidade;  sua Declaração de Ajuste Anual. 
VI - liberdade assistida;  § 1o  A doação de que trata o caput poderá ser deduzi-
VII - semiliberdade; e  da até os seguintes percentuais aplicados sobre o imposto
VIII - internação. apurado na declaração: 
....................................................................................” (NR)  I - (VETADO); 
“Art. 97.  (VETADO)”  II - (VETADO); 
“Art. 121.  .................................…………………............ III - 3% (três por cento) a partir do exercício de 2012. 
.............................................................................................  § 2o  A dedução de que trata o caput: 
§ 7o  A determinação judicial mencionada no § 1o pode- I - está sujeita ao limite de 6% (seis por cento) do im-
rá ser revista a qualquer tempo pela autoridade judiciária.” posto sobre a renda apurado na declaração de que trata o
(NR)  inciso II do caput do art. 260; 
“Art. 122.  ..................................................................... II - não se aplica à pessoa física que: 
.............................................................................................  a) utilizar o desconto simplificado; 
§ 1o    O prazo de internação na hipótese do inciso III b) apresentar declaração em formulário; ou 
deste artigo não poderá ser superior a 3 (três) meses, de- c) entregar a declaração fora do prazo; 
vendo ser decretada judicialmente após o devido processo III - só se aplica às doações em espécie; e 
legal. IV - não exclui ou reduz outros benefícios ou deduções
...................................................................................” (NR)  em vigor. 

149
LEGISLAÇÃO

§ 3o  O pagamento da doação deve ser efetuado até a III - considerar como valor dos bens doados: 
data de vencimento da primeira quota ou quota única do a) para as pessoas físicas, o valor constante da última
imposto, observadas instruções específicas da Secretaria declaração do imposto de renda, desde que não exceda o
da Receita Federal do Brasil.  valor de mercado; 
§ 4o  O não pagamento da doação no prazo estabe- b) para as pessoas jurídicas, o valor contábil dos bens. 
lecido no § 3o  implica a glosa definitiva desta parcela de Parágrafo único.  O preço obtido em caso de leilão
dedução, ficando a pessoa física obrigada ao recolhimento não será considerado na determinação do valor dos bens
da diferença de imposto devido apurado na Declaração de doados, exceto se o leilão for determinado por autoridade
Ajuste Anual com os acréscimos legais previstos na legis- judiciária.” 
lação.  “Art. 260-F.  Os documentos a que se referem os arts.
§ 5o  A pessoa física poderá deduzir do imposto apu- 260-D e 260-E devem ser mantidos pelo contribuinte por
rado na Declaração de Ajuste Anual as doações feitas, no um prazo de 5 (cinco) anos para fins de comprovação da
respectivo ano-calendário, aos fundos controlados pelos dedução perante a Receita Federal do Brasil.” 
Conselhos dos Direitos da Criança e do Adolescente mu- “Art. 260-G.  Os órgãos responsáveis pela administra-
nicipais, distrital, estaduais e nacional concomitantemente ção das contas dos Fundos dos Direitos da Criança e do
com a opção de que trata o caput, respeitado o limite pre- Adolescente nacional, estaduais, distrital e municipais de-
visto no inciso II do art. 260.”  vem: 
“Art. 260-B.  A doação de que trata o inciso I do art. 260 I - manter conta bancária específica destinada exclusi-
poderá ser deduzida:  vamente a gerir os recursos do Fundo; 
I - do imposto devido no trimestre, para as pessoas II - manter controle das doações recebidas; e 
jurídicas que apuram o imposto trimestralmente; e  III - informar anualmente à Secretaria da Receita Fede-
II - do imposto devido mensalmente e no ajuste anual, ral do Brasil as doações recebidas mês a mês, identificando
para as pessoas jurídicas que apuram o imposto anualmen- os seguintes dados por doador: 
te.  a) nome, CNPJ ou CPF; 
Parágrafo único.  A doação deverá ser efetuada dentro b) valor doado, especificando se a doação foi em espé-
do período a que se refere a apuração do imposto.”  cie ou em bens.” 
“Art. 260-C.  As doações de que trata o art. 260 desta
“Art. 260-H.  Em caso de descumprimento das obriga-
Lei podem ser efetuadas em espécie ou em bens. 
ções previstas no art. 260-G, a Secretaria da Receita Federal
Parágrafo único.  As doações efetuadas em espécie de-
do Brasil dará conhecimento do fato ao Ministério Público.” 
vem ser depositadas em conta específica, em instituição fi-
“Art. 260-I.  Os Conselhos dos Direitos da Criança e do
nanceira pública, vinculadas aos respectivos fundos de que
Adolescente nacional, estaduais, distrital e municipais di-
trata o art. 260.” 
vulgarão amplamente à comunidade: 
“Art. 260-D.  Os órgãos responsáveis pela administra-
I - o calendário de suas reuniões; 
ção das contas dos Fundos dos Direitos da Criança e do
Adolescente nacional, estaduais, distrital e municipais de- II - as ações prioritárias para aplicação das políticas de
vem emitir recibo em favor do doador, assinado por pessoa atendimento à criança e ao adolescente; 
competente e pelo presidente do Conselho corresponden- III - os requisitos para a apresentação de projetos a
te, especificando:  serem beneficiados com recursos dos Fundos dos Direitos
I - número de ordem;  da Criança e do Adolescente nacional, estaduais, distrital
II - nome, Cadastro Nacional da Pessoa Jurídica (CNPJ) ou municipais; 
e endereço do emitente;  IV - a relação dos projetos aprovados em cada ano-ca-
III - nome, CNPJ ou Cadastro de Pessoas Físicas (CPF) lendário e o valor dos recursos previstos para implementa-
do doador;  ção das ações, por projeto; 
IV - data da doação e valor efetivamente recebido; e  V - o total dos recursos recebidos e a respectiva desti-
V - ano-calendário a que se refere a doação.  nação, por projeto atendido, inclusive com cadastramento
§ 1o  O comprovante de que trata o caput deste artigo na base de dados do Sistema de Informações sobre a In-
pode ser emitido anualmente, desde que discrimine os va- fância e a Adolescência; e 
lores doados mês a mês.  VI - a avaliação dos resultados dos projetos beneficia-
§ 2o  No caso de doação em bens, o comprovante dos com recursos dos Fundos dos Direitos da Criança e do
deve conter a identificação dos bens, mediante descrição Adolescente nacional, estaduais, distrital e municipais.” 
em campo próprio ou em relação anexa ao comprovante, “Art. 260-J.  O Ministério Público determinará, em cada
informando também se houve avaliação, o nome, CPF ou Comarca, a forma de fiscalização da aplicação dos incenti-
CNPJ e endereço dos avaliadores.”  vos fiscais referidos no art. 260 desta Lei. 
“Art. 260-E.  Na hipótese da doação em bens, o doador Parágrafo único.  O descumprimento do disposto nos
deverá:  arts. 260-G e 260-I sujeitará os infratores a responder por
I - comprovar a propriedade dos bens, mediante docu- ação judicial proposta pelo Ministério Público, que poderá
mentação hábil;  atuar de ofício, a requerimento ou representação de qual-
II - baixar os bens doados na declaração de bens e di- quer cidadão.” 
reitos, quando se tratar de pessoa física, e na escrituração,
no caso de pessoa jurídica; e 

150
LEGISLAÇÃO

“Art. 260-K.  A Secretaria de Direitos Humanos da Pre- população mais pobre, que devido a sua situação social
sidência da República (SDH/PR) encaminhará à Secretaria precária, eram vistos como “incapazes” de cuidar de seus
da Receita Federal do Brasil, até 31 de outubro de cada filho, dando sustentação a pratica recorrente da suspensão
ano, arquivo eletrônico contendo a relação atualizada dos provisória do poder familiar ou da destituição dos pais e de
Fundos dos Direitos da Criança e do Adolescente nacio- seus deveres em relação aos filhos.
nal, distrital, estaduais e municipais, com a indicação dos Tais consequências e o crescente número das desigual-
respectivos números de inscrição no CNPJ e das contas dades sociais, em especial no tocante as condições de vida
bancárias específicas mantidas em instituições financeiras das crianças e adolescentes, levaram a sociedade a reava-
públicas, destinadas exclusivamente a gerir os recursos dos liar os padrões da assistência social em geral, e a clamar
Fundos.”  por políticas públicas e legislações que garantissem os
“Art. 260-L.   A Secretaria da Receita Federal do Brasil direitos dessas crianças e adolescentes, especialmente no
expedirá as instruções necessárias à aplicação do disposto que diz respeito à família.
nos arts. 260 a 260-K.”  As mudanças então começaram a acontecer com a
Art. 88.  O parágrafo único do art. 3o da Lei no 12.213, de 20 de promulgação da Constituição Federal, em 1988, do Esta-
janeiro de 2010
, passa a vigorar com a seguinte redação:  tuto da Criança e do Adolescente (ECA), em 1990, da Lei
“Art. 3o  ..........................................................................  Orgânica da Assistência Social (LOAS), em 1993 e com a
Parágrafo único.   A dedução a que se refere ratificação da Convenção sobre os Direitos da Criança em
o caput deste artigo não poderá ultrapassar 1% (um por 1990, provocando rupturas em relação às concepções e
cento) do imposto devido.” (NR)  práticas assistencialistas e institucionalizantes.
Art. 89.  (VETADO).  Essas mudanças passaram a olhar as crianças e adoles-
Art. 90.  Esta Lei entra em vigor após decorridos 90 centes como sujeitos de direitos, instituindo políticas pú-
(noventa) dias de sua publicação oficial. blicas em um sistema conhecido como Sistema de Garantia
Brasília, 18 de janeiro de 2012; 191o  da Independência de Direitos, onde essas crianças e adolescentes são vistas
e 124o da República.  de maneira indissociável do seu contexto sociofamiliar e
comunitário.
Este Plano constitui um marco nas políticas públicas
no Brasil, ao romper com a cultura da institucionalização
PLANO NACIONAL DE PROMOÇÂO, PROTEÇÃO de crianças e adolescentes e ao fortalecer o paradigma da
proteção integral e da preservação dos vínculos familiares
E DEFESA DO DIREITO DE CRIANÇAS E
e comunitários preconizados pelo Estatuto da Criança e do
ADOLESCENTES À CONVÍVÊNCIA FAMILIAR E
Adolescente. O núcleo principal do Plano Nacional é pro-
COMUNITÁRIA - 2006 mover à cultura de valorização, respeito e promoção da
convivência familiar e comunitária.
O PNCFC vem propor ainda o rompimento com a cul-
Plano Nacional de Promoção, Proteção e Defesa tura de institucionalização de crianças e adolescentes e o
do Direito de Crianças e Adolescentes à Convivência fortalecimento do paradigma da proteção integral e da
Familiar e Comunitária preservação dos vínculos familiares e comunitários.
Para tal, faz-se necessário o investimento nas políticas
Introdução públicas de atenção à família;
Os principais temas que orientaram a discussão e a ela-
O Plano Nacional de Promoção, Proteção e Defesa do boração do PNCF foi a valorização da família, e neste con-
Direito de Crianças e Adolescentes à Convivência Familiar e texto as políticas de apoio sociofamiliar e o reordenamento
Comunitária – PNCFC, foi aprovado em 2006 por resolução dos abrigos e a implementação de programas de famílias
conjunta do Conselho Nacional dos Direitos da Criança e acolhedoras. Neste contexto a adoção passa a ser centrada
do Adolescente – CONANDA e do Conselho Nacional de no interesse da criança e do adolescente.
Assistência Social – CNAS, destinado a nortear as ações de
promoção, proteção e defesa do direito de crianças e ado- Marco legal
lescentes à convivência familiar e comunitária.
Tais ações podem ser entendidas como formulação e Em se tratando dos princípios legais, norteadores e
implementação de políticas públicas que assegurem a ga- que dão sustento ao PNCF, temos que a Constituição Fede-
rantia dos direitos das crianças e adolescentes, de forma ra de 1988 foi o marco inicial da transformação da política
integrada e articulada com os demais programas de go- nacional de atenção às crianças e adolescentes.
verno. Em seu artigo 227 a CF/88 estabelece que
O mencionado Plano Nacional foi criado em um con-
texto onde as políticas públicas voltadas às crianças e ado- Constituição Federal
lescentes, não priorizavam a convivência familiar e comu-
nitária, e a legislação era totalmente paternalista, visando Artigo 227. “É dever da família, da sociedade e do Es-
apenas a contenção social, o que acabou gerando um tado assegurar à criança e ao adolescente, com absoluta
aprofundamento nas desigualdades sociais, sobretudo na prioridade, o direito à vida, à saúde, à alimentação, à edu-

151
LEGISLAÇÃO

cação, ao lazer, à profissionalização, à cultura, à dignidade, O reordenamento do serviço de acolhimento institu-


ao respeito, à liberdade e à convivência familiar e comu- cional visa especialmente garantir que as crianças e ado-
nitária, além de colocá-los a salvo de toda forma de ne- lescentes acolhidos, permaneçam como titulares de direi-
gligencia, discriminação, exploração, violência, crueldade e tos, integrados na rede de atendimento de saúde, cultura,
opressão”. educação, nos serviços da comunidade propiciados pelas
A CF/88 ainda acabou ainda com a diferença havida demais organizações, envolvendo projetos de dança, tea-
entre filhos havidos ou não da relação do casamento, ou tro, esporte, lazer, profissionalização e outros que venham
por adoção, estipulando em que todos teriam os mesmos frequentando ou tenham necessidade. Além disso, todos
direitos e qualificações, proibidas quaisquer designações os esforços devem ser realizados para manter juntos os
discriminatórias relativas à filiação. E finalizou preconizan- grupos de irmãos.
do em seu artigo 229 que “Os pais têm o dever de assistir,
criar e educar os filhos menores (...)”. Marco conceitual
Posteriormente com a promulgação do Estatuto da
Criança e do Adolescente – Lei Federal n. 8.069/90, de 13 Existem alguns conceitos básicos que precisam ser
de julho de 1990, em consonância com as diretrizes apro- apresentados para a devida compreensão do Plano Nacio-
vadas na Convenção Internacional dos Direitos da Criança nal de Convivência Familiar, quais sejam: o termo “Família”,
e do Adolescente, em 20 de novembro de 1989, pode se a compreensão da Criança e do Adolescente como sujeitos
considerar que o país adotou uma nova concepção em re- de direitos e a conceituação de “Convivência Familiar e Co-
munitária”.
lação à formulação e implementação das políticas públicas
dirigidas à infância e à adolescência.
“Família”
A diferença da concepção adotada é clara. Enquanto
anteriormente a legislação tratava as crianças e adolescen-
Indispensável, desta forma, citar o artigo 226, §4º da
tes como objeto, sob regras coercitivas, em geral aplicada CF/88 que explicita: “Entende-se, também, como entidade
as crianças em situação de vulnerabilidade social e finan- familiar a comunidade formada por qualquer dos pais ou
ceira, equiparando muitas vezes crianças e adolescentes descendentes”. Já o Estatuto da Criança e do Adolescente,
pobres à delinquentes, o Estatuto da Criança e do Adoles- em seu art. 25, assim preconiza: “Entende-se por família na-
cente, propõe atender a todas as crianças e adolescentes, tural a comunidade formada pelos pais ou qualquer deles e
respeitando-os como “sujeitos de direitos”, dentro da cha- seus descendentes”.
mada Doutrina da Proteção Integral. Dessa forma, independente do tipo de arranjo fami-
Dentre as principais inovações do Estatuto da Criança liar, não importando se a família seja classificada como
e do Adolescente podemos destacar: “nuclear, monoparental ou reconstituída”, a ênfase está na
- Todas as crianças passam a ter direitos iguais, sem existência de vínculos de filiação de origem natural ou ado-
discriminações; tiva, conferindo-lhe igualdade de direitos. Este formato,
- É previsto que as crianças e adolescentes são respon- seguramente, propõe superar o modelo “ideal de família”,
sabilidades da família, do Estado e da sociedade; que tem ênfase na “estrutura” para enfatizar a sua capaci-
- As crianças e adolescentes devem ser considerados dade de exercer a função de proteção, de socialização e de
como prioridade absoluta no conjunto das políticas pú- cuidados das suas crianças e adolescentes, considerando
blicas; também outras possibilidades de arranjos.
- Crianças e adolescentes devem ser tratados como Desta realidade começa a emergir um modelo de fa-
“pessoas em condição peculiar de desenvolvimento”. mília que se estende para além da unidade casal (papai,
Já no tocante a questão do acolhimento institucional, mamãe e criança), da unidade pais e filhos, vivendo ou não
mais conhecido como “abrigamento”, podemos destacar no mesmo domicílio, na concepção tradicional. Este novo
que com a promulgação do ECA, este tipo de serviço pas- modelo considera uma família ampliada, a “família exten-
sou e ainda passa por um processo de reordenamento, ou sa”. Em suma, propõe a compreensão para além do espaço
seja, os serviços estão sendo adequados a realidade das físico, para um modelo que reconhece e valoriza a rede de
novas políticas públicas, bem como das legislações. vínculos. É preciso compreender a diversidade de organi-
zações familiares, a complexidade e riqueza dos vínculos
A nova legislação (ECA) estabelece o caráter de pro-
familiares e comunitários. Mais do que isso, é preciso su-
visoriedade e de excepcionalidade. Ou seja, essa medida
perar o modelo estático e reconhecer um modelo dinâmi-
deverá ser adotada como última alternativa, depois de es-
co, ainda que convivam, concomitantemente, os diferentes
gotadas todas as possibilidades.
modelos e que o tradicional carregue seu valor intrínseco.
Além disso, no capítulo II do Estatuto, que trata do Não se trata aqui de valorizar um novo modelo familiar e
trabalho das Entidades de Atendimento, artigos 90 a 94 e desconstruir o anterior.
no artigo 100, há clareza sobre as responsabilidades das Neste momento o Estatuto da Criança e do Adolescen-
organizações e de seus dirigentes, no sentido de assegu- te passa, no artigo 23, preconiza que a pobreza material,
rar que sejam respeitados todos os direitos da criança e, a falta/carência de recursos materiais não constitui mais
sobretudo, da tarefa de desenvolver ações para a preser- motivo suficiente para a perda ou suspensão do poder fa-
vação dos vínculos familiares. miliar. Não existindo outros motivos que levem o Poder Ju-

152
LEGISLAÇÃO

diciário a afastar a criança ou adolescente da sua família de O afastamento da criança e do adolescente só ocorrerá
origem, compete então exigir que essa família seja obriga- quando não for possível uma intervenção junto à família
toriamente inserida em programas oficiais de auxílio. Essa (origem ou extensa), neste caso, a medida terá por objetivo
atribuição é da responsabilidade dos gestores das políticas garantir o melhor para criança e o adolescente, colocando
sociais no município e pode ser requerida pelo Ministério a salvo de qualquer prejuízo ao seu processo de desenvol-
Público, caso não esteja sendo cumprida. vimento.
Faz-se necessário destacar, que antes do afastamento
A criança e o adolescente como “sujeitos de direitos” da criança e do adolescente e de sua inserção nos servi-
ços ou programas de acolhimento institucional, os órgãos
Nesse sentido, a legislação passa garantir que as competentes devem promover ações de fortalecimento e
crianças e adolescentes sejam vistos como indivíduos au- de inclusão social dessas famílias nas políticas sociais pú-
tônomos e íntegros, dotados de personalidade e vontade blicas, como também em redes sociais e comunitárias, vi-
próprias que, na sua relação com o adulto, não podem ser sando potencializar o núcleo familiar para que este possa
tratados como seres passivos, subalternos ou meros “ob- garantir a convivência saudável de crianças e adolescentes
jetos”, devendo participar das decisões que lhes dizem no seio da família e da comunidade.
respeito, sendo ouvidos e considerados em conformidade O Estatuto da Criança e do Adolescente busca em sua
com suas capacidades e grau de desenvolvimento. essência garantir à criança e ao adolescente os direitos
Significa ainda que a sociedade deve garantir a estas fundamentais inerentes à pessoa humana, sem prejuízo da
proteção integral, logo, estes também devem ser os prin-
crianças e adolescentes o pleno desenvolvimento, no seio
cípios norteadores dos serviços sociais de atendimento à
de uma família e de uma comunidade, ou prestar-lhes cui-
criança e ao adolescente, em especial aquelas que estão
dados alternativos temporários, quando afastados do con-
com os vínculos familiares e comunitários fragilizados e/
vívio com a família de origem, o que passa a ser cumpri-
ou rompidos.
mento de deveres para com a criança e o adolescente e o
exercício da responsabilidade da família, da sociedade e do Diretrizes
Estado.
Todas essas garantias são condições peculiares da crian- São diretrizes básicas do Plano Nacional de Convivên-
ça e do adolescente como pessoa em desenvolvimento cia Familiar e Comunitária:
- Centralidade da família nas políticas públicas: O di-
Convivência Familiar e Comunitária reito das crianças e adolescentes à convivência familiar e
comunitária está relacionado à inclusão social de suas fa-
A convivência familiar e comunitária é um direito re- mílias. Significa dizer que a família é a matriz das relações
servado a toda criança e adolescente de ser criado e edu- sociais, ou seja, é o núcleo de acolhida, convívio, autono-
cado no seio de sua família original, e excepcionalmente mia, sustentabilidade e protagonismo social e deve ser
se necessário, em família substituta, conforme artigo 19 do apoiada e ter acesso a condições para responder ao seu
Estatuto da Criança e do Adolescente. papel no sustento, na guarda e na educação de suas crian-
O direito à convivência familiar e comunitária é tão im- ças e adolescentes, bem como na proteção de seus idosos
portante quanto o direito à vida, à saúde, à alimentação, à e portadores de deficiência.
educação, ao lazer, à profissionalização, à cultura, à digni- - Primazia da responsabilidade do Estado no fomento
dade, ao respeito e à liberdade. de políticas integradas de apoio à família: O Estado é o
Nesse sentido, verifica-se que o fortalecimento de vín- maior responsável por garantir e oferecer as políticas pú-
culos sociais e familiares, bem como a inclusão em ambien- blicas adequadas à prevenção e superação das situações
te onde as relações afetivas são preservadas, assegura a de violação de direitos, possibilitando o fortalecimento dos
criança e o adolescente um melhor desenvolvimento. vínculos familiares e sócio-comunitários. Todo este apoio
Por esta razão, os Governos, tanto Federal, Estadual às famílias, deve ser realizado através da articulação de
atendimento das diversas políticas públicas ofertadas.
quanto o Municipal, devem desenvolver políticas públicas
- Reconhecimento das competências da família na sua
de modo a garantir os direitos de crianças e adolescentes
organização interna e na superação das suas dificuldades:
que estão em situação de vulnerabilidade social e pessoal,
As políticas especiais para promoção, proteção e defesa do
com vínculos fragilizados ou não e ainda, executá-las de
direito de crianças e adolescentes à convivência familiar e
forma articulada com toda a rede de atendimento à criança comunitária não devem ser assistencialistas, ou seja, devem
e adolescente. reconhecer a família como um grupo social capaz de se or-
Dentre essas medidas necessariamente precisa estar o ganizar e reorganizar dentro de seu contexto e a partir de
apoio sociofamiliar as famílias que não conseguem suprir suas demandas e necessidades, bem como rever e recons-
as necessidades básicas de seus filhos, acarretando o afas- truir seus vínculos ameaçados, a partir do apoio recebido
tamento dessas crianças do seu lar, buscando assim, garan- das políticas sociais.
tir a reintegração desta criança ou adolescente na família Reconhece-se aqui uma metodologia de participativa,
de origem ou extensa. Caso isto não seja possível, outras que envolva a família, pactuando responsabilidades e com-
medidas devem ser tomadas, como por exemplo, o enca- promissos, metas e objetivos. As ações são desenvolvidas
minhamento para família substituta. com e pelas famílias.

153
LEGISLAÇÃO

- Respeito à diversidade étnico-cultural, à identidade e - Controle social das políticas públicas: A sociedade, na
orientação sexuais, à equidade de gênero e às particularida- busca de garantir os direitos fundamentais e os princípios
des das condições físicas, sensoriais e mentais: Neste ponto democráticos, deve realizar o controle social das políticas
o objetivo é combater os estigmas sociais através da promo- públicas. Isso significa dizer que precisam existir espaços
ção dos direitos humanos. A meta também é desenvolver democráticos para as discussões das instituições e socieda-
programas sociais que incentivem os laços de solidariedade de civil organizada, com objetivo de fiscalizar e solucionar
social e respeito ao próximo. Os princípios que norteiam está problemas que afligem a população infanto-juvenil.
diretriz são o da discriminação e do respeito à diversidade.
- Fortalecimento da autonomia do adolescente e do jo- Objetivos gerais
vem adulto na elaboração de seu projeto de vida. Nesta di-
retriz, busca-se garantir aos jovens e adolescentes direito de Dentre os objetivos gerais do Plano Nacional de Convi-
participar das formulações das políticas públicas que os afe- vência Familiar e Comunitária, podemos citar:
tam, com direito de voz e de participação. É preciso ouvir es- a) Ampliar, articular e integrar as diversas políticas,
tes jovens e adolescentes, para reconhecer suas habilidades, programas, projetos, serviços e ações de apoio sociofami-
competências, interesses e necessidades. Quando os jovens liar para a promoção, proteção e defesa do direito de crian-
e adolescentes adotam um modelo participativo na socie- ças e adolescentes à convivência familiar e comunitária;
dade, isso fortalece sua autonomia e lhes garante condições b) Difundir uma cultura de promoção, proteção e de-
para elaborar um projeto de vida mais justo e consciente. fesa do direito à convivência familiar e comunitária, em
- Garantia dos princípios de excepcionalidade e proviso- suas mais variadas formas, extensiva a todas as crianças e
riedade dos PFA e de Programa de Acolhimento Institucional adolescentes, com ênfase no fortalecimento ou resgate de
( PAI ): As medidas de afastamento de crianças e adolescen- vínculos com suas famílias de origem;
tes dos seus lares, deve ser vista sempre como uma medida c) Proporcionar, por meio de apoio psicossocial ade-
temporária, com uma ruptura de vínculos temporária, de- quado, a manutenção da criança ou adolescente em seu
vendo imediatamente a Entidade de Acolhimento iniciar o ambiente familiar e comunitário, considerando os recursos
trabalho de restabelecimento do vínculo e das condições de e potencialidades da família natural, da família extensa e da
autonomia desta família. rede social de apoio;
Ou seja, a regra geral é pela permanência e a manuten- d) Fomentar a implementação de Programas de Famí-
ção da família de origem e a exceção, o acolhimento insti- lias Acolhedoras, como alternativa de acolhimento a crian-
tucional. Importante ressaltar, que a decisão pela separação ças e adolescentes que necessitam ser temporariamente
da família é sempre de um técnico competente e somente afastados da família de origem, atendendo aos princípios
será tomada após o esgotamento de todas as possibilidades de excepcionalidade e de provisoriedade, estabelecidos
de manutenção da família de origem ou extensa. A simples pelo Estatuto da Criança e do Adolescente, bem como
precariedade financeira não constitui motivo para o acolhi- assegurando parâmetros técnicos de qualidade no aten-
mento institucional. dimento e acompanhamento às famílias acolhedoras, às
- Reordenamento dos Programas de Acolhimento Insti- famílias de origem, às crianças e aos adolescentes;
tucional: O Reordenamento Institucional surge para reorien- e) Assegurar que o Acolhimento Institucional seja efe-
tar os serviços de acolhimento, que antes eram vistos como tivamente utilizado como medida de caráter excepcional
abrigamento, mantendo crianças e adolescentes isolados e provisório, proporcionando atendimento individualizado,
do convívio social, para se alinharem ao modelo de famí- de qualidade e em pequenos grupos, bem como proceder
lia como a unidade básica da ação social, não sendo mais ao reordenamento institucional das entidades para que se-
concebido este isolamento. Busca-se o desenvolvimento jam adequadas aos princípios, diretrizes e procedimentos
de programas para reorientar as redes públicas e privadas estabelecidos no ECA;
para se alinharem à mudança de paradigma. Os serviços de f) Fomentar a implementação de programas para pro-
reordenamento deverão ter como base, as orientações reco- moção da autonomia do adolescente e/ou jovem egressos
mendadas pelo Conselho Nacional dos Direitos da Criança de programas de acolhimento, desenvolvendo parâmetros
e do Adolescente (Conanda) e pelo Conselho Nacional de para a sua organização, monitoramento e avaliação;
Assistência Social. g) Aprimorar os procedimentos de adoção nacional e
- Adoção centrada no interesse da criança e do adoles- internacional, visando: a) estimular, no País, as adoções de
cente: esse tópico visa esclarecer que a adoção não se trata crianças e adolescentes que, por circunstâncias diversas,
de “dar” um filho a quem ainda não tem, mas sim de uma têm sido preteridos pelos adotantes – crianças maiores e
forma de garantir a estas crianças e adolescentes o direito à adolescentes, com deficiência, com necessidades específi-
convivência familiar e comunitária. Trata-se sempre de uma cas de saúde, afrodescendentes ou pertencentes a mino-
medida excepcional, mas que garante a criança e ao adoles- rias étnicas, dentre outros; b) investir para que todos os
cente o direito de crescer e se desenvolver em uma família processos de adoção no País ocorram em consonância com
substituta. Na intenção de manutenção do conceito de fa- os procedimentos legais previstos no Estatuto da Criança
mília e que se deve preservar a união de grupos de irmão e do Adolescente; e c) garantir que a adoção internacional
num processo de adoção. A adoção é um processo que ne- ocorra somente quando esgotadas todas as tentativas de
cessariamente precisa ocorrer com a intervenção do Poder adoção em território nacional, sendo, nestes casos, priori-
Judiciário, visando dar legalidade e transparência ao ato. zados os países que ratificaram a Convenção de Haia;

154
LEGISLAÇÃO

h) Assegurar estratégias e ações que favoreçam os me- Eixo 1 – Análise da Situação e Sistemas de Infor-
canismos de controle social e a mobilização da opinião pú- mação
blica na perspectiva da implementação do Plano Nacional
de Promoção, Proteção e Defesa do Direito de Crianças e - Levantamento de dados visando o conhecimento em
Adolescentes à Convivência Familiar e Comunitária; relação à situação familiar, identificando os fatores que fa-
i) Aprimorar e integrar mecanismos para o cofinancia- vorecem ou ameaçam a CFC;
mento, pela União, Estados, Distrito Federal e Municípios, - Mapeamento e análise das iniciativas de PASF, PFA,
das ações previstas no Plano Nacional de Promoção, Prote- PAI e Adoção e sua adequação aos marcos legais;
ção e Defesa do Direito de Crianças e Adolescentes à Con- - Aprimoramento e valorização da comunicação entre
vivência Familiar e Comunitária, tendo como referência a os Sistemas de Informação sobre crianças, adolescentes e
absoluta prioridade definida no artigo 227 da Constituição família.
Federal de 1988 e no artigo 4° do Estatuto da Criança e do
Adolescente. Eixo 2 – Atendimento

Resultados programáticos - Articulação e integração entre as políticas públicas;


- Sistematização e difusão de metodologias de traba-
Como resultado da implantação do Plano Nacional, lho com famílias e comunidades;
prevê-se a concretização do fortalecimento das políticas - Ampliação e estruturação da oferta de serviços de
públicas especificadas de proteção à criança e adolescente, ASF, que contribuam para o empoderamento das famílias;
que garantam efetivamente o acesso a políticas, progra- - Reordenamento dos serviços de Acolhimento Institu-
mas, projetos, serviços e ações intersetoriais, materializan- cional e Implementação de PFA;
do dessa forma o direito fundamental dessas crianças e - Implantação, ampliação e implementação de Progra-
adolescentes. mas e serviços de preparação de adolescentes e jovens em
Acolhimento Institucional, para a autonomia.
- Fortalecimento de vínculos familiares de adolescen-
Implementação, monitoramento e avaliação
tes em cumprimento de medida socioeducativa, sobretudo
privativas de liberdade, bem como de filhos com pais pri-
Para a materialização deste direito será necessário:
vados de liberdade;
- Cumprimento integral do Plano nas três esferas do
- Articulação entre os serviços de AI e o SGD, em par-
governo. O Plano Nacional precisa ser integralmente de-
ticular o judiciário, de modo a evitar o “esquecimento” de
senvolvido na União, nos Estados e nos Municípios.
crianças e adolescentes nestas instituições;
- Constituição formal de Comissão Nacional Interseto-
- Aprimoramento dos procedimentos de Adoção na-
rial (Grupo de Trabalho). Necessário à constituição de co-
cional e internacional, priorizando a adoção nacional;
missão intersetorial, para acompanhar o desenvolvimento - Capacitação e assessoramento aos municípios para a
e a implementação dos planos estaduais e municipais. implementação das ações do PNCFC;
- Elaboração de Planos Estaduais e Municipais e cons- - Consolidação de uma rede nacional de identificação e
tituição de Comissões Intersetoriais; localização de crianças e adolescentes desaparecidos e de
- Conselhos dos Direitos da Criança e do Adolescente pais e responsáveis.
(três esferas) assumindo o Plano como prioridade, a partir
de 2007 – viabilizando recursos nos orçamentos; Eixo 3 – Marcos Normativos e Regulatórios
- Participação e integração entre os Conselhos de Di-
reitos da Criança e Setoriais nas três esferas de governo; - Parametrização e regulamentação dos PASF, PFA e
- Corresponsabilidade entre os entes federativos no fi- PAI e de apadrinhamento;
nanciamento para a implementação do Plano. Vale ressaltar - Regulamentação e aplicação dos conceitos de “ex-
que o Plano Nacional já especificou responsabilidades que cepcionalidade e provisoriedade”;
devem ser compartilhadas pelas três esferas de governo. - Regulamentação dos programas e serviços de Aco-
lhimento Familiar;
Plano de Ação - Aprimoramento dos instrumentos legais de proteção
social que ofereçam alternativas e a possibilidade do con-
Foi estipulado, que o conjunto das ações será imple- traditório à suspensão ou destituição do poder familiar.
mentado e implantado no horizonte de 09 anos (2007-
2015), ficando estabelecidos os seguintes intervalos: Eixo 4 – Mobilização, Articulação e Participação
- Curto Prazo: 2007-2008
- Médio Prazo: 2009-2011 - Estratégias de comunicação social para mobilização
- Longo Prazo: 2012-2015 da sociedade e afirmação de novos valores;
- Ações permanentes: 2007-2015 - Mobilização e articulação para a garantia da proviso-
As propostas operacionais PNCFC estão organizadas riedade e excepcionalidade do Acolhimento Institucional;
em quatro eixos estratégicos: - Produção e divulgação de material de orientação e
capacitação;

155
LEGISLAÇÃO

- Articulação e integração de ações entre as três esfe- coroamento destas mudanças aconteceu com a promul-
ras de Poder; gação da Constituição Federal, em 1988, do Estatuto da
- Garantia de recursos para viabilização do Plano. Criança e do Adolescente (ECA), em 1990, da Lei Orgânica
Neste momento, visando garantir uma visão mais da Assistência Social (LOAS), em 1993 e com a ratificação
aprofundada e detalhada sobre o tema, necessários se da Convenção sobre os Direitos da Criança em 1990, pro-
faz a leitura da íntegra do Plano Nacional de Promoção, vocando rupturas em relação às concepções e práticas as-
Proteção e Defesa do Direito de Crianças e Adolescentes à sistencialistas e institucionalizantes.
Convivência Familiar e Comunitária, o qual apresentamos Trata-se da mudança do olhar e do fazer, não apenas
a seguir: das políticas públicas focalizadas na infância e na juventu-
de, mas extensivos aos demais atores sociais do chamado
PLANO NACIONAL DE PROMOÇÃO, DEFESA E GA- Sistema de Garantia de Direitos, implicando a capacidade
RANTIA DO DIREITO DE CRIANÇAS E ADOLESCENTES de ver essas crianças e adolescentes de maneira indissociá-
À CONVIVÊNCIA FAMILIAR E COMUNITÁRIA. vel do seu contexto sociofamiliar e comunitário.
Crianças e adolescentes têm o direito a uma família,
1. Antecedentes cujos vínculos devem ser protegidos pela sociedade e pelo
Estado. Nas situações de risco e enfraquecimento desses
A legislação brasileira vigente reconhece e preconiza vínculos familiares, as estratégias de atendimento deverão
a família,58 enquanto estrutura vital, lugar essencial à hu- esgotar as possibilidades de preservação de tais vínculos,
manização e à socialização da criança e do adolescente, aliando o apoio socioeconômico à elaboração de novas
espaço ideal e privilegiado ao desenvolvimento integral formas de interação, referências morais e afetivas no grupo
dos indivíduos. familiar.
Contudo, a história social da criança, do adolescente e No caso de ruptura desses vínculos, o Estado é o res-
da família foi marcada pela dificuldade da família em pro- ponsável pela proteção das crianças e dos adolescentes, in-
teger e educar seus filhos. O discurso de uma “incapacida- cluindo o desenvolvimento de programas e estratégias que
de” da família foi assumido pelo Poder Público, que passou possam levar a constituição de novos vínculos familiares e
a desenvolver políticas paternalistas voltadas para o con- comunitários, mas sempre tendo em vista a possibilidade
trole e a contenção social, principalmente para a população de resgate dos vínculos originais.
mais pobre. Essa desqualificação das famílias pobres, trata- Diante do desafio de garantir efetivamente o direito à
das como incapazes, deu sustentação ideológica à prática convivência familiar e comunitária de crianças e adolescen-
recorrente da suspensão provisória do poder familiar ou tes, o então Departamento da Criança e do Adolescente
da destituição dos pais e de seus deveres em relação aos (DCA) do Ministério de Justiça (MJ), a Secretaria de Estado
filhos. de Assistência Social (SEAS) do Ministério da Previdência
A engenharia construída com o sistema de proteção e e Assistência Social (MPAS) e o Fundo das Nações Unidas
assistência, sobretudo durante o século passado, permitiu para Infância (UNICEF) reuniram-se, no primeiro semestre
que qualquer criança ou adolescente, por sua condição de de 2002, com a finalidade de discutir os dados apresenta-
pobreza, estivesse sujeita a se enquadrar no raio da ação dos pela Caravana da Comissão de Direitos Humanos da
da justiça e da assistência, que sob o argumento de “pren- Câmara dos Deputados59 sobre os programas de abrigo.
der para proteger” confinavam-nas em grandes institui- A partir dessa situação problema percebeu-se que para
ções totais. ampliar e qualificar o debate fazia-se necessário integrar
Essas representações negativas sobre as famílias cujos novos atores sociais no processo.
filhos formavam o público da assistência social e demais Assim, em agosto de 2002 foi realizado o “Colóquio
políticas sociais tornaram-se parte estratégica das políticas Técnico sobre Rede Nacional de Abrigos”, que contou com
de atendimento, principalmente da infância e da juventu- a participação de Secretarias Estaduais de Assistência So-
de, até muito recentemente. cial, e entidades não-governamentais dos diferentes esta-
O aprofundamento das desigualdades sociais, com dos brasileiros envolvidos com a temática.
todas as suas consequências, principalmente para as con- Nesse evento foram identificadas ações a serem prio-
dições de vida das crianças e dos adolescentes, levou à rizadas, entre elas: a realização de um censo nacional de
revisão dos paradigmas assistenciais cristalizados na so- crianças e adolescentes em abrigos e práticas institucio-
ciedade. O olhar multidisciplinar e intersetorial iluminou a nais e a elaboração de um Plano de Ação para o reorde-
complexidade e multiplicidade dos vínculos familiares. O namento de abrigos. Para o encaminhamento das decisões
58 Com base no texto constitucional e infraconstitucional, deliberadas no Colóquio, constituiu-se o “Comitê Nacional
define-se família como um grupo de pessoas, com laços de consan- para Reordenamento de Abrigos”, com objetivo de esti-
guinidade e/ou de aliança e/ou de afinidade, cujos vínculos circuns- mular mudanças nas políticas e práticas de atendimento,
crevem obrigações recíprocas, organizadas em torno de relações de efetivando uma transição para o novo paradigma legal,
geração e de gênero. A amplitude desta definição derruba qualquer
ideia preconcebida de modelo familiar “normal”. Trata-se, portanto, de 59 A Caravana, realizada de setembro a dezembro de 2001,
saber se a família é capaz de realizar as funções de proteção e de percorreu oito estados brasileiros com o objetivo de verificar a real
socialização das suas crianças e adolescentes em uma diversidade situação dos programas de abrigos para crianças e adolescentes. Os
de arranjos familiares e culturais, mas já não mais de se perguntar resultados da Caravana foram apresentados no Caderno Especial do
pela forma ou estrutura da família. jornal Correio Braziliense datado de 09/01/ 2002.

156
LEGISLAÇÃO

Estatuto da Criança e do Adolescente, a respeito do direi- Comissão Intersetorial foi nomeada por decreto presiden-
to de crianças e adolescentes á convivência familiar e co- cial de 19 de outubro de 2004 e composta por cinco Mi-
munitária. O Comitê foi composto pelos seguintes órgãos nistérios, cada um com obrigação de orçar recursos para a
e organizações: DCA, SEAS, FONSEAS, CNAS, CONANDA, nova política. Foram também convidadas representações
Colegiado do Fórum Nacional de Conselheiros Tutelares, dos três poderes e da sociedade civil. A Comissão Interse-
RENIPAC, UNICEF e Fundação ORSA. Este grupo realizou torial teve noventa dias para a elaboração do documento,
três encontros60 ainda em 2002, e concluiu pela relevân- tendo seu prazo ampliado para abril de 2005.
cia do levantamento nacional de abrigos, porém, dado a A composição dessa Comissão, de acordo com o de-
limitações de recursos e tempo, delimitando o universo da creto, obedeceu à lógica da intersetorialidade. Articulou
pesquisa para os programas de abrigos que faziam parte atores institucionais dos três poderes da República, das
da Rede de Serviço de Ação Continuada ( Rede SAC). 61 62No três esferas de poder, das diferentes políticas sociais bá-
final de 2002 o CONANDA, e o DCA do Ministério de Jus- sicas, da área de planejamento do Governo Federal, das
tiça, aprovaram/alocaram recursos para financiar esta pes- instâncias de participação e controle social que integram
quisa. Em 2003, o Instituto de Pesquisa Econômica Aplicada o Sistema de Garantia de Direitos, das entidades de aten-
(IPEA) iniciou a pesquisa. dimento, bem como do Conselho Nacional dos Direitos da
No inicio de 2004, no seu Planejamento Estratégico Criança e do Adolescente (CONANDA), Fórum Colegiado
para o exercício 2004-2005, o CONANDA elegeu como Nacional dos Conselheiros Tutelares, do Conselho Nacio-
uma de suas prioridades a promoção do direito à convi- nal de Assistência Social (CNAS) e do Conselho Nacional
vência familiar e comunitária de crianças e adolescentes. dos Direitos da Pessoa Portadora de Deficiência (CONADE),
Por parte do Poder Executivo, o Ministro Chefe da Secreta- além de entidades civis de âmbito nacional, que militam
ria Especial dos Direitos Humanos e o Ministro de Estado pelo direito de crianças e adolescentes à convivência fami-
de Desenvolvimento Social e Combate à Fome se articula- liar e comunitária. Participou também desse esforço, o Fun-
ram e propuseram a convocação de outros Ministérios e do das Nações Unidas para a Infância (UNICEF), apoiando
atores numa Comissão Intersetorial. tecnicamente os trabalhos da Comissão mediante a contra-
Nesse novo momento, de maior integração interseto- tação de consultores, e pelo aporte ao debate do “estado
rial, ampliou-se o escopo temático para além da propos- da arte” da discussão internacional que se trava sobre o
ta inicial de reordenamento dos abrigos. A incorporação mesmo tema.
das questões sobre Família e Adoção tornou necessário Além da participação dos seus membros, a Comis-
redimensionar o grupo de trabalho, criando-se a Comis- são Intersetorial não prescindiu da valiosa contribuição de
são Intersetorial que teria, agora, como finalidade superior, colaboradores dos campos jurídico, técnico, acadêmico e
construir subsídios para a elaboração do “Plano Nacional midiático, bem como dos diferentes atores sociais do siste-
de Promoção, Defesa e Garantia do Direito de Crianças e ma de atendimento, entre eles as famílias que participaram
Adolescentes à Convivência Familiar e Comunitária”. Esta deste processo, que proferiram palestras ou deram seus
60 Os três encontros aconteceram em Brasília/DF nas se- depoimentos durante as jornadas de trabalho, enriquecen-
guintes datas: 1ª reunião – 24/09/2002; 2ª reunião – 22/10/2002 e 3ª do sobremaneira a discussão.
reunião – 22/11/2002. Entre novembro de 2004 e março de 2005, a Comissão
61 A decisão pela proposta de Levantamento da Rede de realizou quatro reuniões ordinárias.63 Em cada uma, foram
Abrigos com base na Rede SAC em detrimento de um Censo Nacio- discutidas questões referentes a um dos quatro Eixos Estra-
nal (proposta inicial e mais completa) deu-se em razão de que a rea- tégicos propostos para o Plano de Ação, a saber: a) Análise
lização desse Censo Nacional seria uma tarefa praticamente impos- da situação e sistemas de informação; b) Atendimento; c)
sível diante do quadro de ausência de dados que possibilitassem a
Marcos normativos e regulatórios; d) Mobilização, articula-
identificação de todas as instituições que executavam serviços dessa
natureza, destacando-se: i) Muitos municípios não possuíam Conse-
ção e participação.
lhos dos Direitos da Criança e do Adolescente (órgãos responsáveis Para melhor organização do trabalho a Comissão In-
pelos registros das entidades e serviços de abrigos); ii) O Cadastro tersetorial optou por dividir-se em três Câmaras Técnicas,
existente na SEAS reduzia-se aos serviços de abrigos que recebiam cada uma voltada ao aprofundamento de uma das três
subvenção do Fundo Nacional de Assistência Social, não se tendo áreas temáticas que juntas abarcam as diferentes facetas
dimensionado a representação dessa rede diante do universo das or- do Direito à Convivência Familiar e Comunitária. Em primei-
ganizações que executavam tal medida de proteção e iii) A realização
ro lugar, a família de origem e a comunidade na qual está
de um Censo implicaria na cobertura de toda a rede, necessitando
para tanto de um banco de dados que identificasse a localização de inserida, a importância da preservação dos vínculos familia-
cada um dos abrigos ou instituições que operavam o abrigamento de res e comunitários e o papel das políticas públicas de apoio
crianças e adolescentes. sociofamiliar. Em segundo lugar, a intervenção institucional
62 Ressalta-se que, com a aprovação da NOB/SUAS em ju- nas situações de rompimento ou ameaça de rompimento
lho de 2005 e das portarias nº 440 e nº 442 do MDS, os recursos do dos vínculos familiares, os abrigos e as alternativas ao Aco-
cofinanciamento federal das ações socioassistenciais passam a ser lhimento Institucional, com ênfase na excepcionalidade, na
transferidos por “Pisos de Proteção”, cujos recursos poderão ser uti-
lizados conforme a necessidade local, dentro das ações passíveis de 63 Datas das reuniões: 19 e 20 de novembro de 2004 – Tema:
financiamento por cada piso. Cabe ao gestor local e ao CMAS a de- Análise da situação e sistemas de informação; 16 e 17 de dezembro
finição da rede de atendimento. O Piso de Alta Complexidade I pode de 2004 – Tema: Atendimento; 02 a 04 de março de 2005 – Tema:
ser utilizado para a manutenção dos serviços da rede de acolhimento Marcos normativos e regulatórios; 21 a 23 de março de 2005 – Tema:
para crianças e adolescentes. Mobilização, articulação e participação.

157
LEGISLAÇÃO

brevidade e na provisoriedade destas medidas e, ainda, na direito à convivência familiar e comunitária será de grande
restauração e na preservação dos vínculos familiares. Por relevância.
fim, a necessidade de uma nova família para a criança/ado- O documento ora intitulado como “Plano Nacional de
lescente que perdeu a sua própria. Promoção, Defesa e Garantia do Direito de Crianças e Ado-
As três Câmaras Técnicas que trataram de Políticas de lescentes à Convivência Familiar e Comunitária – Versão
Apoio à Família, Medidas de Abrigamento e Adoção foram Preliminar” – é o produto histórico da elaboração de inú-
coordenados pelo MDS, o UNICEF e a SEDH respectiva- meros atores sociais comprometidos com os direitos das
mente. O UNICEF disponibilizou consultores técnicos para crianças e adolescentes brasileiros. O CONANDA e o CNAS,
sistematizar as contribuições dos participantes da Comis- ao apresentarem o documento esperam contribuir para a
são. construção de um novo patamar conceitual que orientará
Cada uma destas áreas foi objeto de discussões apro- a formulação das políticas para que cada vez mais crianças
fundadas e propositivas, abarcando também suas interfa- e adolescentes tenham seus direitos assegurados e encon-
ces e inter-relações, tentando dar conta da imensa comple- trem na família os elementos necessários ao seu pleno de-
xidade do tema e das múltiplas variáveis que interagem em senvolvimento. Este processo acontece simultaneamente
cada dimensão da realidade focalizada. com um processo de discussão internacional liderado pelo
Em 15 de abril de 2005 o documento contendo os Comitê dos Direitos da Criança da Organização das Nações
“Subsídios para a elaboração do Plano de Promoção, Defe- Unidas (ONU) sobre a necessidade de aprimorar os meca-
sa e Garantia do Direito de Crianças e Adolescentes à Con- nismos de proteção integral dos direitos da criança privada
vivência Familiar e Comunitária” foi apresentado ao MDS dos cuidados parentais, com recomendações em 2004 e
e ao SEDH, em cerimônia oficial, e contou com a presença 2005 da elaboração de nova normativa internacional a esse
dos Conselhos Nacionais dos Direitos da Criança e do Ado- respeito.
lescente (CONANDA) e da Assistência Social (CNAS). Nesta Elaborar um “plano nacional” requer outro e importan-
solenidade ficou definido o prazo de dois meses para que o te desafio: mobilizar ainda mais outros atores sociais para
Poder Executivo Federal fizesse a readequação programáti- que se integrem a esse movimento, que deve ser coletivo
ca e orçamentária e em seguida encaminhasse o documen- e articulado na efetivação de direitos, tornando efetiva a
to ao CONANDA e CNAS para a aprovação conjunta do participação social e, sobretudo possibilitando o avanço na
Plano Nacional. Neste período, o Governo Federal desen- promoção, defesa e garantia do direito à convivência fami-
volveu um importante trabalho de análise das ações, dos liar e comunitária.
programas e dos respectivos orçamentos, acrescentando A promoção, a defesa e a garantia dos direitos das
nas tabelas dos quatros eixos do Plano as estratégias re- crianças e adolescentes à convivência familiar e comunitá-
levantes, possibilitando a inserção da temática em progra- ria envolvem o esforço de toda a sociedade e o compro-
mas bem como sua articulação. Conselheiros do CONAN- misso com uma mudança cultural que atinge as relações
DA e do CNAS participaram como titulares da Comissão familiares, as relações comunitárias e as relações do Estado
Intersetorial, contribuindo e acompanhando todo o pro- com a sociedade. O respeito à diversidade cultural não é
cesso e, sobretudo, informando os respectivos Conselhos contraditório com esta mudança que atravessa os diversos
a respeito. Ao receberem o documento “Subsídios para a grupos socioculturais, na defesa desses direitos. Pelo con-
elaboração do Plano de Promoção, Defesa e Garantia do trário, exige que se amplie a concepção de cidadania para
Direito de Crianças e Adolescentes à Convivência Familiar e incluir as crianças e adolescentes e suas famílias, com suas
Comunitária”, ambos os Conselhos nomearam Comissões necessidades próprias.
(CNAS – a Comissão de Política e CONANDA – uma Sub- Desafio de dimensões estratégicas, sem dúvida, de
comissão ad-hoc da Comissão de Políticas Públicas), para cujo enfrentamento eficaz depende a viabilidade de qual-
discussão e encaminhamento de contribuições. Os Presi- quer projeto de nação e de país que se deseje construir
dentes de ambos os Conselhos lideraram este processo, agora e no futuro. Eis o nosso desafio!
facilitando reunião conjunta entre as Comissões dos Con-
selhos para a consideração do documento. Num momento Marco Legal
de intensa parametrização de ambos – o Sistema Único de
Assistência Social (SUAS) e o Sistema de Garantia de Di- A Constituição Federal estabelece que a “família é a
reitos da Criança e do Adolescente (SGD) – esse esforço base da sociedade” (artigo 226) e que, portanto compete
de deliberação conjunta constitui-se como elemento es- a ela, juntamente com o Estado, a sociedade em geral e
tratégico. Uma política de promoção, defesa e garantia do as comunidades, “assegurar à criança e ao adolescente o
direito da criança e do adolescente à convivência familiar e exercício de seus direitos fundamentais” (artigo 227). Por
comunitária perpassa ambos os sistemas e é fundamental sua vez, o referido artigo, especifica os direitos fundamen-
para o aprimoramento da interface entre eles. Tanto CO- tais especiais da criança e do adolescente, ampliando e
NANDA quanto CNAS são categóricos ao afirmarem que aprofundando aqueles reconhecidos e garantidos para os
este direito só será garantido com a interação de todas as cidadãos adultos no seu artigo 5º. E dentre esses direitos
políticas sociais, com centralidade na família para acesso fundamentais da cidadania está o direito à convivência fa-
a serviços de saúde, a educação de qualidade, geração de miliar e comunitária.
emprego e renda entre outros. Desta forma, as contribui- Em face desse papel de mecanismo de promoção e
ções sobre o papel de cada setor no apoio e garantia do proteção dos direitos humanos, no tocante às relações fa-

158
LEGISLAÇÃO

miliares, a Constituição Federal rompe com o anterior tra- ção, desenvolvimento e sobrevivência”, responsáveis por
tamento diferenciado e discriminatório dado aos filhos em seu cuidado, em especial. Em seu preâmbulo e em muitos
razão da origem do nascimento ou das condições de con- dos seus artigos a Convenção define os direitos da criança
vivência dos pais, determinando a equiparação de filhos realmente num sentido próximo da Declaração dos Direi-
havidos ou não da relação do casamento ou por adoção tos da Criança, da ONU, em 1959, apenas como direito a
(artigo 227 §6º). A mesma Carta Constitucional, em seu ar- uma proteção especial: “a criança tem necessidade de uma
tigo 226 §8º estabelece que ao Estado compete assegurar a proteção especial e de cuidados especiais, notadamente de
assistência à família na pessoa de cada um dos que a inte- uma proteção jurídica, antes e depois de seu nascimento.”
gram, criando mecanismos para coibir violências no âmbito Todavia, em outros pontos, a Convenção avança e acresce
de suas relações. Adiante, no artigo 229 determina que os a esse “direito à proteção especial”, outros tipos de direitos
pais têm o dever de assistir, criar e educar os filhos menores que só podem ser exercidos pelos próprios beneficiários:
e os filhos maiores têm o dever de ajudar e amparar os pais o direito à liberdade de opinião (artigo12), à liberdade de
na velhice, carência ou enfermidade. Consequentemente, expressão (artigo 13), à liberdade de pensamento, de cons-
todo reordenamento normativo e político-institucional que ciência e de religião (artigo 14), à liberdade de associação
se pretenda fazer há de partir das normas constitucionais, (artigo 15). Direitos que pressupõem certo grau de capaci-
marco legal basilar para o presente Plano. dade, de responsabilidade, isto é, que pressupõem sujeitos
Respeitando-se essa hierarquia normativa, quando se de direitos como titulares. As crianças e os adolescentes
tratar desta questão da convivência familiar e comunitá- são seres essencialmente autônomos, mas com capacidade
ria, igualmente deve ser dada prevalência a toda norma- limitada de exercício da sua liberdade e dos seus direitos.
tiva convencional internacional, reguladora da promoção Para efetivação da Convenção sobre os Direitos da
e proteção dos direitos humanos, ratificada em caráter es- Criança, no País é importante que sejam observados os se-
pecial pelo Brasil64 e àquela estabelecida por força de re- guintes princípios:
soluções da Assembleia Geral das Nações Unidas. Assim
sendo, é de se destacar como marcos normativos a serem - Não discriminação;
considerados as Declarações sobre os Direitos da Criança - Interesse superior da criança;
(1924/1959), a Declaração Universal dos Direitos Humanos - Direitos à sobrevivência e ao desenvolvimento;
(1948), a Declaração Americana dos Direitos e Deveres do - Respeito à opinião da criança
Homem (1948), o Pacto de São José da Costa Rica (1969), o
Pacto Internacional dos Direitos Civis e Políticos e o Pacto Regulamentando esses princípios constitucionais e tais
Internacional dos Direitos Econômicos, Sociais e Culturais normas internacionais, a Lei Federal 8.069 de 13 de julho de
(1966, ratificados em 1992). 1990 (Estatuto da Criança e do Adolescente – ECA) reforça o
A Convenção sobre os Direitos da Criança, ratificada papel da família na vida da criança e do adolescente como
pelo Brasil em 24 de setembro de 1990, em especial, tem elemento imprescindível dentro do processo de proteção
um papel superior e preponderante no embasamento da integral, e como um dos objetivos maiores do sistema de
criação ou reforma de toda e qualquer norma reguladora, promoção e defesa dos direitos da infância e adolescência,
nesse campo da família e no embasamento de processos que aquela lei propõe instituir, articulando e integrando to-
de reforma administrativa, de implantação e implementa- das as políticas públicas (sociais, institucionais, econômicas
ção de políticas, programas, serviços e ações públicas. A e infraestruturantes), no sentido da priorização do atendi-
Convenção das Nações Unidas sobre os Direitos da Criança mento direto desse segmento da população, como forma
assegura as duas prerrogativas maiores que a sociedade e de garantia de direitos: fazer com que o atendimento das
o Estado devem conferir à criança e ao adolescente, para necessidades básicas das crianças e dos adolescentes seja
operacionalizar a proteção dos seus Direitos Humanos: cui- realizado como direito do cidadão-criança e do cidadão-a-
dados e responsabilidades. As crianças e os adolescentes dolescente e ao mesmo tempo dever do Estado, da socie-
têm direitos subjetivos à liberdade, à dignidade, à integri- dade e da família, com prioridade absoluta.
dade física, psíquica e moral, à educação, à saúde, à pro- No tocante ao direito à convivência familiar e comuni-
teção no trabalho, à assistência social, à cultura, ao lazer, tária, o Estatuto da Criança e do Adolescente estabeleceu
ao desporto, à habitação, a um meio ambiente de quali- no artigo 19 que toda criança ou adolescente tem direito
dade e outros direitos individuais indisponíveis, sociais, di- de ser criado e educado no seio de sua família e, excepcio-
fusos e coletivos. E consequentemente se postam, como nalmente, em família substituta, assegurada a convivência
credores desses direitos, diante do Estado e da sociedade, familiar e comunitária. Esse dispositivo do Estatuto deve
devedores que devem garantir esses direitos. Não apenas ser considerado, em seguida aos princípios constitucionais
como atendimento de necessidades, desejos e interesses, e convencionais, como o outro marco legal basilar na cons-
mas como Direitos Humanos indivisíveis, como os qualifi- trução do presente Plano. Em função desse princípio o ECA
ca a normativa internacional – como direito a um desen- estabelece a excepcionalidade e a provisoriedade do Aco-
volvimento humano, econômico e social. São pessoas que lhimento Institucional a exemplo do acolhimento em regi-
precisam de alguém, de grupos e instituições, responsá- me de abrigo, obrigando que se assegure a “preservação
veis pela promoção e defesa da sua “participação, prote- dos vínculos familiares e a integração em família substituta
64 Para a Convenção a criança é considerada, no seu artigo quando esgotados os recursos de manutenção na família
1º, menor de 18 anos. de origem” (artigos 92 e 100). Não havendo possibilidade

159
LEGISLAÇÃO

de preservar os vínculos com a sua família natural, o ECA Sendo assim, para este Plano é necessário a verifica-
estabelece que o acolhimento em família substituta se dê ção mínima do discurso dentro dos nossos marcos legal e
provisoriamente via tutela ou guarda ou em definitivo via situacional, sem prejuízo das questões jurídicas conflituais
adoção (artigos 28 a 52 do ECA), sempre por decisão ju- que existam e que venham a existir.67
dicial, processando-se dentro dos princípios e requisitos Entende-se como família natural, nos termos do artigo
previstos na citada Lei 8.069/90, aplicando-se quando for 25 do ECA, a comunidade formada pelos pais ou qualquer
o caso, subsidiariamente, as regras do Código Civil. Nesse deles e seus antecedentes.
ponto, essa regulação das diversas formas de acolhimen- Embora o Estatuto não se utilize expressamente do
to familiar citadas não foi alterado pelo novo Código Civil termo família extensa, entende-se neste documento como
(2002) e por nenhuma outra posterior ao ECA. aquela que inclui, além dos parentes e agregados, todas as
A colocação em família substituta dar-se-á meio de de- demais pessoas que tenham relação de afinidade ou de afe-
cisão judicial e somente tendo lugar quando comprovada- tividade com o núcleo familiar natural, como se depreende
mente representar para a criança e o adolescente a melhor do texto do artigo 28, parágrafo 2º (ECA)68 não se exigindo
medida para sua proteção e desenvolvimento. Essa nova que as pessoas residam no mesmo domicílio.
família deve proporcionar um ambiente familiar adequado
É fato que o acolhimento informal de filhos de outra
(art. 29 do ECA) e devem ser excluídas de sua convivência
pessoa diz respeito à prática secular e disseminada em todo
pessoas dependentes de substâncias entorpecentes, pes-
o país,69 ficando caracterizada esta situação quando os pais,
soas que os submetam a maus-tratos, ou lhes imponham
voluntariamente, delegam seu papel parental a outro mem-
tratamento desumano, violento, aterrorizante, vexatório e
constrangedor ou que pratiquem exploração, abuso, cruel- bro da família extensa ou, simplesmente, por sua omissão,
dade e opressão (artigos 5º, 18 e 19 do ECA). permitem que haja esta transferência. É sabido que não são
Em respeito ao disposto nos artigos 226 e 227 da poucas as crianças e adolescentes que passam a maior par-
Constituição Federal, no tocante ao direito à convivência te de sua vida sendo criadas por parentes, padrinhos ou
familiar e comunitária, as leis orgânicas das políticas sociais amigos próximos dos seus pais (família extensa). Esta prá-
foram sendo editadas e reformadas aprofundando esses tica antiga que se denomina por processo de circulação de
princípios constitucionais, regulamentados pelo Estatuto crianças e de adolescentes70 ainda persiste e persistirá por
da Criança e do Adolescente, tornando-os operacionais, ser natural e culturalmente legitimada. Nestes casos, a re-
com a construção de sistemas de atendimento de direitos, gularização da situação da criança ou do adolescente vai
especializados. Assim, se procedeu com a promulgação da exigir apenas uma solução judicial, consistente na coloca-
Lei Orgânica da Assistência Social, da Lei Orgânica da Saú- ção em família pelos instrumentos jurídicos previstos no art.
de, da Lei de Diretrizes e Bases da Educação. 28 do Estatuto da Criança e do Adolescente.
Sendo assim, não há que se falar em acolhimento “for-
Marco Conceitual65 mal” na hipótese de simples colocação em guarda ou tute-
la, ou até mesmo de adoção de criança ou de adolescente,
O Estatuto da Criança e do Adolescente (ECA) tem uma mantendo-se na sua família extensa, como forma de regu-
redação que facilita a sua interpretação e reduz a possi- larização jurídica de uma situação protetora ou para se efe-
bilidade de divergências extremadas sobre as noções de tivar a reintegração dessas em sua família.
seus institutos. Como toda norma esta possui o seu dis- O termo acolhimento “formal”, a que se refere este Pla-
curso. Este discurso é dotado de racionalidade, na medida no, corresponde à modalidade de atendimento de serviço
em que oferece razões para aquilo que é dito,66 e tudo isto de proteção especial de alta complexidade71 que garanta
decorre da verificação da realidade e dos conceitos que proteção integral, incluindo a moradia, alimentação, higieni-
todos temos desta. Entretanto, o discurso da norma jamais zação, bem como os demais cuidados para crianças e ado-
poderá prescindir de qualquer uma das duas funções: jus- lescentes que se encontram sem referência e/ ou em situa-
tificadora e modificadora.
ção de ameaça, que não possam permanecer em seu núcleo
Muito embora vivamos em um período de exigência
de um processo de positivação do Direito, alguns institutos 67 Em se tratando de interpretação é perfeitamente natural
previstos na legislação e na doutrina sobre os direitos da que aconteça. Assim, justifica-se a elaboração deste marco concei-
criança e do adolescente estão sendo consolidados e, ou- tual e de glossário neste Plano.
tros ainda, construídos. 68 Sobre o conceito de família extensa ligada pela consangui-
65 ‘’Na Oficina de revisão jurídica do presente Plano Nacio- nidade e pela afinidade ver também Saffioti, Heleieth I. B. e Almeida,
nal, organizado pelo CONANDA com especialistas da área em 18 de Suely de Souza. Violência de Gênero: Poder e Impotência. Rio de
abril de 2006, surgiu a necessidade de introduzir maior detalhamento Janeiro, Revinter, 1995, p. 71.
sobre Marco Conceitual, especialmente para dirimir dúvidas quanto 69 SERRA, Márcia Milena Pivatto. O Brasil das muitas mães:
ao conceito de “Acolhimento Familiar” utilizado no texto, antes de aspectos demográficos da circulação de crianças. Tese de Doutorado
submetê-lo à Consulta Pública. Ressalta-se que o conteúdo do Marco no PPG Antropologia – UNICAMP, 2003.
Conceitual não foi objeto de apreciação do Conanda e do CNAS. Por- 70 FONSECA, Cláudia, TERTO, Veriano e ALVES, Caleb F.
tanto, este capítulo, assim como os demais, devem receber as críticas Antropologia, diversidade e direitos humanos: diálogos interdiscipli-
e sugestões pertinentes quando da consulta pública. nares. Porto Alegre: Editora da UFRGS, 2004.
66 FERRAZ JR, Tércio Sampaio. Direito, retórica e comunica- 71 Ministério do Desenvolvimento Social e Combate à Fome
ção. Saraiva: São Paulo, 1973, p. 126. e Secretaria Nacional de Assistência Social – Brasília, 2004, pág. 32.

160
LEGISLAÇÃO

familiar ou comunitário.72 Ou seja, “trata-se de uma prática O Acolhimento Familiar é a modalidade de atendi-
mediada por uma autoridade, com um plano de interven- mento que oferece acolhimento na residência de famílias
ção definido, administrada por um serviço com recursos cadastradas, selecionadas, capacitadas e acompanhadas
disponíveis, conforme política pública estabelecida.”73 para receber crianças e/ou adolescentes com medida de
Este “acolhimento formal”, definido como o ato de criar proteção, que necessitem de acolhimento fora da família
o filho de uma pessoa, não pode ser realizado por pessoa de origem, até que seja possível sua reintegração familiar
da mesma família – natural ou extensa –, mas por uma pes- ou salvo exceções encaminhamento para família substituta.
soa, família ou instituição, que cuide transitoriamente da Embora se constitua um instituto novo no país, esta
criança ou do adolescente com seus direitos violados, até experiência já se encontra consolidada em outros países,
que este possa ser reintegrado à sua família de origem. principalmente nos europeus,75 e se encontra contemplada
Nos casos em que se inviabiliza a reintegração à família de expressamente na Política Nacional de Assistência Social,
origem outras alternativas podem ser consideradas, como ao dispor que dentro dos serviços de proteção social espe-
o encaminhamento para adoção ou eventualmente a indi- cial de alta complexidade está a “Família Acolhedora”.
cação de um acolhimento permanente por parte da família Este atendimento poderá ser efetivado por uma entida-
acolhedora. de governamental ou não-governamental em regime de co-
A criança ou o adolescente que se encontra em situa- locação familiar, nos termos do artigo 90, inciso III, do ECA.
ção de violação permanente de seus direitos deve receber Dentro de nossa sistemática jurídica, este tipo de aco-
as medidas específicas de proteção, traduzidas no âmbito lhimento possui como pressuposto um mandato formal,
da assistência social pelos serviços de proteção especial, uma guarda fixada judicialmente, a ser requerida pelo pro-
conforme estabelecidos no Estatuto da Criança e do Ado- grama de atendimento ao Juízo, em favor da família acolhe-
lescente e na Política Nacional de Assistência Social, res- dora. A manutenção da guarda judicial, que é instrumento
pectivamente. judicial exigível para a regularização deste acolhimento, à
obviedade, estará vinculada à permanência da família aco-
São duas as espécies de Acolhimento: Acolhimento lhedora no Programa. Nesta modalidade de atendimento
Institucional e o Acolhimento Familiar. há supervisão e apoio para aqueles que estão acolhendo as
O Acolhimento Institucional é a modalidade de aten- crianças ou os adolescentes com quem possuem uma obri-
dimento integral institucional, que oferece cuidado e es- gação direta. O programa de atendimento deve ter como
paço para socialização e desenvolvimento de crianças e objetivos não só o cuidado adequado e individualizado da
adolescentes com medida de proteção, que necessitem de criança ou do adolescente acolhido, mas também a viabili-
acolhimento fora da família de origem, até que seja pos- zação do retorno da criança à sua família de origem. 76
sível sua reintegração familiar (natural ou extensa) ou en- Assim como as demais entidades previstas no artigo
caminhamento para família substituta. As modalidades de 90 do ECA, os programas de “Famílias Acolhedoras”, deno-
Acolhimento Institucional são: Casa Lar, República, Casa de minadas também de “Família Guardiã”, “Família de Apoio”,
Passagem, Albergue, entre outros.74 “Família Cuidadora”, “Família Solidária”,77 dentre outras,
Embora estas modalidades de acolhimento não es- deverão se sujeitar ao regime previsto nos artigos 92 e 93
tejam especificadas no artigo 101 do ECA, todas elas se do Estatuto, mesmo porque ausente legislação federal es-
encaixam na hipótese denominada no inciso VII, como en- pecífica.
tidade de abrigo, cujo regime jurídico básico está estabele- As demais denominações se encontram, ad referen-
cido nos artigos 90, 92 e 93 da referida Lei. dum, devidamente descritas no glossário.
As entidades que desenvolvem programas de abrigo
75 conforme estudos divulgados pela International Foster
servem para acolher a criança e o adolescente e prestar-lhe Care Organisation ( IFCO): GEORGE, Shanti & OUDENHOVEN, Nico
plena assistência. O abrigo é o lar coletivo, de pequenas Van. Trad. Maria Soledad Franco. Apostando al Acogimiento Familiar
dimensões, onde o abrigado não está privado da liberdade - Um estúdio comparativo internacional.Bélgica: Garant, 2003.
e deve obedecer aos princípios estabelecidos no artigo 92 76 Na definição de Matilde Luna, Presidente do Instituto Mer-
do ECA. As casas-lares, segundo a Lei no. 7.644, de 18 de cosul Social ( IMS), Buenos Aires, Argentina, “O Acolhimento Familiar
dezembro de 1987, são unidades residenciais sob respon- se define como, entre outras questões, um dos recursos técnicos uti-
sabilidade de mãe social, que abrigue até dez crianças e/ou lizados pelos governos na instrumentação de medidas que resguar-
dem às crianças e aos adolescentes na situação de risco psicosso-
adolescentes. As entidades de abrigo podem manter estas
cial. Isto implica que no menu de programas assistenciais se incluam
casas, nos termos do artigo 16 desta Lei, e possui como os programas de acolhimento como resposta às demandas sociais e
finalidade proporcionar tratamento especial às crianças e no cumprimento da responsabilidade que cabe aos governos, particu-
aos adolescentes, visando dar maior individualidade ao tra- larmente às autoridades das políticas sociais. Sendo um recurso que
tamento. pode adotar diferentes formas na sua implementação, cada país es-
colhe a sua segundo algumas variáveis (...)”. In, “Menores em riesgo
72 Política Nacional de Assistência Social (2.5.2.) – Brasília, y acogimiento familiar. Compartir el compromiso”. Buenos Aires. Ed.
2004. Humanitas, 1994.
73 CABRAL, Cláudia (Org.). “Perspectivas do Acolhimento 77 Vide diversas experiências pelo país, como os projetos do
Familiar no Brasil”. Acolhimento Familiar – experiências e perspecti- Rio de Janeiro; SAPECA, de Campinas/SP; São Bento do Sul/SC,
vas. Rio de Janeiro: Booklink, 2004, p. 11. etc:, in, CABRAL, Cláudia (Org.). “Perspectivas do Acolhimento Fa-
74 Vide rol da Política Nacional de Assistência Social, “2.5.2.” miliar no Brasil”. Acolhimento Familiar – experiências e perspectivas.
– Brasília, 2004. Rio de Janeiro: Booklink, 2004, p. 11.

161
LEGISLAÇÃO

Marco Situacional crescente diminuição da sua capacidade de proteger os


seus membros.80 Criar e educar os filhos, garantindo-lhes
É amplamente reconhecida a importância da família o usufruto de todos os direitos de que são titulares como
no cuidado e no bem-estar de seus integrantes, uma vez pessoas humanas em situação peculiar de desenvolvimen-
que é ela o âmbito privilegiado e primeiro a proporcionar to, tem sido uma tarefa muitas vezes impossível de ser
a garantia de sua sobrevivência, o aporte afetivo funda- cumprida pelas famílias submetidas a condições de vida
mental para o seu pleno desenvolvimento e para a sua precárias, sem garantia de alimento, de moradia, de tra-
saúde mental, a absorção de valores éticos e de conduta, e balho, de assistência à saúde e de todos os serviços que
a sua introdução na cultura e na sociedade em que estão definem uma vida minimamente digna no mundo con-
inseridas. Essa importância adquire contornos ainda mais temporâneo. Além disso, a dinâmica familiar, naturalmen-
decisivos no caso dos indivíduos mais vulneráveis, como te marcada pela ocorrência de entradas e saídas de inte-
as crianças, os adolescentes, os idosos e os doentes. grantes, registra, no caso das famílias pobres, movimentos
O direito fundamental à convivência familiar está con- ainda mais traumáticos, determinados pelas condições so-
sagrado nas normas e instrumentos legislativos. No en- cioeconômicas e pela luta pela sobrevivência: migrações
tanto, a garantia formal desse direito coloca problemas de em busca de novas oportunidades; institucionalização de
ordem prática para a sua implementação. crianças, adolescentes, adultos e idosos; afastamento dos
Ainda hoje, predomina no ideário social o modelo responsáveis por longos períodos em função da ocupação
de família tradicional e abstrato composto por pai, mãe exercida, como o trabalho doméstico, por exemplo, entre
e filhos pequenos. De fato, os laços de parentesco ainda inúmeras outras situações.81
mantêm fortíssima influência na organização das famílias As consequências da desigualdade social e da pobre-
brasileiras. Os dados que vão de 1977 a 1998, mostram za, que tem como resultado a “produção social de crianças
que a grande maioria dos arranjos domiciliares no Brasil vitimadas pela fome, por ausência de abrigo ou por morar
está baseada em relações de parentesco entre pelo menos em habitações precárias, por falta de escolas, por doenças
dois dos moradores, ressaltando que, em 1998, apenas contagiosas, por inexistência de saneamento básico”,82 re-
9,3% dos arranjos domiciliares não eram familiares – maior fletem diretamente na relação entre criança, adolescente e
índice do período.78 violência no cotidiano de famílias brasileiras. Essa situação
A família brasileira está desde há muito em pleno pro- de vulnerabilidade, denominada vitimação, pode desenca-
cesso de mudança, especialmente no que se refere a sua
dear a agressão física e/ou sexual contra crianças e adoles-
composição. Movido por novas práticas e valores sociais,
centes, haja vista que a cronificação da pobreza da família
esse processo muitas vezes passa ao largo da legislação e
contribui para a precarização e deterioração de suas rela-
das políticas públicas que foram desenhadas para a família
ções afetivas e parentais. Nesse sentido, pequenos espa-
modelar.
ços, pouca ou nenhuma privacidade, falta de alimentos e
Neste sentido, a discussão sobre o direito à convivên-
problemas econômicos acabam gerando situações estres-
cia familiar das crianças e dos adolescentes brasileiros em
santes que, direta ou indiretamente, acarretam danos ao
situação de risco envolve questões ainda mais específicas,
desenvolvimento infantil”.83
relacionadas aos diferentes aspectos dos problemas en-
Contudo, a pobreza ou carência de recursos materiais
frentados por eles e suas famílias. Antes de tudo, há que
se considerar que a família, enquanto unidade essencial não constitui motivo suficiente para explicar o fenômeno
de organização da sociedade brasileira sofre as influências da violação de direitos da criança e do adolescente. A vio-
do desenvolvimento socioeconômico e da ação estatal lação de direitos não ocorre em todas as famílias que são
por meio das políticas públicas. E são as famílias pobres as pobres, assim como não é verdade que crianças e adoles-
mais negativamente afetadas pelas consequências destes centes oriundos de famílias de classes de renda mais ele-
processos, quer se considere o impacto das transforma- vadas estão livres da vivência de maus-tratos e da violação
ções sociais e econômicas de longo prazo que vêm rede- de direitos cometidos por seus próprios familiares.
senhando o país desde a década de 1950, com a industria- Existem outros fatores explicativos para a incidência da
lização e a urbanização crescente da população brasileira, violência contra crianças e adolescentes no âmbito familiar,
quer se considere as medidas de política mais recentes, entre eles destaca-se: a história familiar passada ou presen-
tomadas a partir dos anos de 1990 e caracterizadas pelo 80 CAMPOS, M. S. e MIOTO, R.C.T. Política de Assistência
ajuste econômico e pela restrição das políticas sociais. Social e a posição da Família na Política Social Brasileira. In: Ser
Essa família empobrecida, embora tenha peculiari- Social: Revista do Programa de Pós-Graduação em Política Social/
dades na sua forma de organização que lhe possibilitam Universidade de Brasília. Departamento de Serviço Social – v.1, n.1
(1º semestre, 1998). Brasília, SER Social UnB, 1998.
apoiar-se fortemente nas relações de solidariedade pa-
rental ampliada e conterrânea,79 tem experimentado uma 81 FERRARI, Mário; KALOUSTIAN, Silvio M. Introdução. In:
KALOUSTIAN, Silvio M. (Org.). Família Brasileira: a base de tudo São
78 MEDEIROS, M. e OSÓRIO, R. Arranjos domiciliares e ar- Paulo: Cortez; Brasília, DF: UNICEF, 1994..
ranjos nucleares no Brasil: classificação e evolução de 1977 a 1998. 82 Azevedo e Guerra apud AMARO, Sarita. Crianças vítimas
Texto para Discussão no 788. Brasília, IPEA, abril de 2001. de violência: das sombras do sofrimento à genealogia da resistência
79 CARVALHO, M. C. B. A priorização da família na agenda – uma nova teoria científica. Porto Alegre: AGE/EDIPURS, 2003.
da política social. In: KALOUSTIAN, op. cit. 83 Idem.

162
LEGISLAÇÃO

te de violência doméstica; a ocorrência de perturbações O Levantamento Nacional de Abrigos para Crianças e


psicológicas entre os membros das famílias; o desprepa- Adolescentes da Rede SAC87 do Ministério do Desenvol-
ro para a maternidade e/ou paternidade de pais jovens, vimento Social realizado pelo IPEA/CONANDA88 mostrou
inexperientes ou sujeitos a uma gravidez indesejada; a que a institucionalização se mantém, ainda nos dias atuais,
adoção de práticas educativas muito rígidas e autoritárias; como caminho utilizado indiscriminadamente – e, muitas
o isolamento social das famílias que evitam desenvolver vezes, considerado o único possível – para a proteção de
intimidade com pessoas de fora do pequeno círculo fa- infância e adolescência, demonstrando que o princípio da
miliar; a ocorrência de práticas hostis, desprotetoras ou excepcionalidade da medida de abrigo não vem sendo
negligentes em relação às crianças, e fatores situacionais respeitado.
diversos que colocam as famílias frente a circunstâncias Existem cerca de vinte mil crianças e adolescentes
não antecipadas. 84 atendidos nas 589 instituições de abrigos beneficiados
A relação entre pobreza e vitimização de crianças e com recursos do Governo Federal repassado por meio da
adolescentes por parte de seus responsáveis não é, por- Rede de Serviços de Ação Continuada (Rede SAC). Os da-
tanto, direta, pois existem outras mediações que refutam dos levantados mostram características típicas de exclusão
o caráter natural e fatalista com frequência atribuído a social, apontando que os abrigos no Brasil são o locus da
essa associação. Entretanto, não é possível dissociar o pa- pobreza e da menoridade (lugar de menor valor). Ressalta-
drão de convivência familiar das questões mais amplas de se ainda que o perfil de meninos e meninas encontrados
frustração, humilhação, redução dos direitos sociais e pri- nessas instituições em nada corresponde às expectativas
vações causadas pelo desemprego e pela diminuição do da sociedade para adoção, cuja preferência recai nos be-
papel do Estado na garantia da sobrevivência das famílias bês da cor branca e do sexo feminino. Vivendo nos abrigos
por meio da provisão de políticas sociais.85 do país encontram-se, na maioria, meninos (58,5%), afro-
A condição socioeconômica precária das famílias, ao descendentes (63%) e mais velhos, isto é, com idade entre
impor maiores dificuldades para a sobrevivência digna do 7 e 15 anos (61,3%) (Gráfico 1 e 2).
grupo familiar, pode funcionar como um elemento agra-
vante e desencadeador de outros fatores de risco pree-
xistentes. Portanto, tratar do direito à convivência familiar
e comunitária de crianças e adolescentes em situação de
risco é falar das políticas de atenção às suas famílias, ma-
joritariamente pobres.
O Brasil é um país com tradição de atendimento ins-
titucional às crianças e adolescentes em situação de vul-
nerabilidade, tradição essa historicamente forjada na des-
valorização social da parcela da população a que perten-
cem, em sua grande maioria pobre e procedente de etnias
não-brancas e na adaptação dessa população aos padrões
considerados aceitáveis.
A colocação de crianças e adolescentes em instituições
como medida de proteção contra os desvios causados pe-
las condições sociais, econômicas e morais das famílias
pobres ou como medida corretiva de desvios, ao longo da
história social da criança, do adolescente e da família, cris- 87 Ressalta-se que, com a aprovação da NOB/SUAS em julho
talizou as experiências das chamadas instituições totais, de 2005 e das portarias Nº 440 e Nº 442 do MDS, os recursos do
cofinanciamento federal das ações socioassistenciais passam a ser
onde crianças e adolescentes viviam sob rígida disciplina e
transferidos por “Pisos de Proteção”, cujos recursos poderão ser uti-
afastados da convivência familiar e comunitária, visto que lizados conforme a necessidade local, dentro das ações passíveis de
quase todas as atividades pertinentes a suas vidas eram financiamento por cada piso. Cabe ao gestor local e ao CNAS a de-
realizadas intramuros.86 finição da rede de atendimento. O Piso de Alta Complexidade I pode
ser utilizado para a manutenção dos serviços da rede de acolhimento
para crianças e adolescentes.
88 Levantamento realizado pelo IPEA em 2003 e promovido
pela Secretaria Especial dos Direitos Humanos (SEDH) da Presi-
dência da República, por meio da Subsecretaria de Promoção dos
84 Amaro, op. cit. Direitos da Criança do Adolescente (SPDCA) e do Conselho Nacio-
85 FALEIROS, Vicente de Paula. A questão da violência. In: nal de Direitos da Criança e do Adolescente (Conanda). Das cerca
SOUSA JR., José Geraldo de [et al.] organizadores. Educando para de 670 instituições de abrigo que eram beneficiadas, naquele ano,
Direitos Humanos: pautas pedagógicas para a cidadania na universi- por recursos da Rede de Serviços de Ação Continuada (Rede SAC)
dade. Porto Alegre, 2004. do Ministério do Desenvolvimento Social e Combate à Fome, foram
86 RIZZINI, Irma. Assistência à infância no Brasil: uma análise investigados 589 abrigos, ou seja, 88% do total. Essas instituições
de sua construção. Rio de Janeiro, Ed. Universitária Santa Úrsula, acolhiam, no momento da realização da Pesquisa, 19.373 crianças e
1993. adolescentes.

163
LEGISLAÇÃO

Gráfico 1

Brasil – Crianças e adolescentes abrigados por faixa etária, segundo cor


Fonte: IPEA/DISOC (2003). Levantamento Nacional de Abrigos para Crianças e Adolescentes da Rede SAC

2500

2000

1500

1000

500

0
0a3 4a6 7a9 10 a 12 13 a 15 16 a 18 Mais
anos anos anos anos anos anos de 18
anos

Brancos Não-brancos

Gráfico 2

Brasil – Crianças e adolescentes abrigados por faixa etária, segundo sexo


Fonte: IPEA/DISOC (2003). Levantamento Nacional de Abrigos para Crianças e Adolescentes da Rede SAC

3000

2500

2000

1500

1000

500

0
0 a 3 4 a 6 7 a 9 10 a 13 a 16 a Mais
anos anos anos 12 15 18 de 18
anos anos anos anos
Meninos Meninas

Contrariando o senso comum que imaginava serem órfãos as crianças e adolescentes que viviam nos abrigos, o Le-
vantamento Nacional também mostrou que a grande maioria desses meninos e meninas (86,7%) tinha família, sendo que
58,2% mantinham vínculos com os familiares. Apenas 5,8% estavam impedidos judicialmente desse contato com eles e
somente 5% eram órfãos. Esses meninos e meninas viviam, portanto, a paradoxal situação de estarem juridicamente vincu-
lados a uma família que, na prática, havia algum tempo, não exerce a responsabilidade de cuidar deles, principalmente por
causa da pobreza (Gráfico 3).

164
LEGISLAÇÃO

Gráfico 3
Brasil – Crianças e adolescentes abrigados, segundo situação familiar

Com família e com 58,2%


vínculo

Com família e sem 22,7%


vínculo

Impedimento 5,8%
judicial

Família 6,7%
desaparecida

Sem família 4,6%

Fonte: IPEA/DISOC (2003). Levantamento Nacional de Abrigos para Crianças e Adolescentes da Rede SAC

Embora a carência de recursos materiais, de acordo com o ECA, não constitua motivo para a perda ou suspensão do
poder familiar, o Levantamento Nacional identificou que as causas que motivaram o abrigamento da maioria das crianças
e adolescentes encontradas nas instituições de abrigos estavam relacionadas à pobreza, consequência da falha ou inexis-
tência das políticas complementares de apoio aos que delas necessitam. Entre os principais motivos: a pobreza das famílias
(24,1%), o abandono (18,8%), a violência doméstica (11,6%), a dependência química dos pais ou responsáveis incluindo
alcoolismo (11,3%), a vivência de rua (7,0%) e a orfandade (5,2%).

Gráfico 4
Brasil – Motivos do ingresso de crianças e adolescentes em abrigo, segundo a frequência

Carência de recursos materias da família/


24,2%
responsável (pobreza)

Abandono pelos pais ou responsáveis 18,9%

Violência doméstica (maus-tratos físicos


e/ou psicológicos praticados pelos pais ou 11,7%
responsáveis)

Pais ou responsáveis dependentes químicos/


11,4%
alcoolistas

Vivência de rua 7,0%

Órfão (morte dos pais ou responsáveis) 5,2%

Outros motivos 21,6%

165
LEGISLAÇÃO

Fonte: IPEA/DISOC (2003). Levantamento Nacional de Abrigos para Crianças e Adolescentes da Rede SAC.

Se de um lado tem havido por parte das autoridades competentes – Conselho Tutelar e Judiciário – uma aplicação
indiscriminada da medida de abrigo, de outro lado, à saída do abrigo permanece sendo um desafio. O Levantamento Na-
cional apontou que o princípio da brevidade da medida do abrigo estabelecido pelo ECA também não vem sendo cumpri-
do, uma vez que mais da metade das crianças e dos adolescentes abrigados viviam nas instituições há mais de dois anos,
enquanto 32,9% estavam nos abrigos por um período entre dois e cinco anos, 13,3%, entre seis e dez anos, e 6,4%, por
mais de dez anos.
Em relação à possibilidade de adoção a situação também é dramática, uma vez que dentre as crianças e adolescentes
abrigadas nas instituições pesquisadas, apenas 10,7% estavam judicialmente em condições de serem encaminhados para
a adoção.89 Além disso, apenas metade desses meninos e meninas (54%) abrigados tinha processo judicial. A outra me-
tade, por certo, lá estava sem o conhecimento do judiciário, já que muitas crianças e adolescentes foram encaminhadas
aos abrigos pelas próprias famílias (11,1%), pela polícia (5,5%), dentre outras instituições que, judicialmente, não teriam tal
prerrogativa.90
Embora a legislação tenha como regra geral a convivência de crianças e adolescentes com suas famílias naturais – e,
excepcionalmente, com famílias substitutas –, para muitos dos meninos e meninas brasileiros esse direito permanece nega-
do, passando um período significativo da sua infância e adolescência institucionalizadas e afastadas do convívio com suas
famílias e suas comunidades.
É preciso considerar sempre a prioridade a ser dada à manutenção de crianças e adolescentes no arranjo familiar de
origem, seja ele qual for, evitando-se a separação e suas implicações e, sobretudo é necessário pensar em como manter a
vivência familiar e comunitária quando o afastamento é inevitável.
As sequelas para crianças e adolescentes de um período de institucionalização prolongado serão tanto maiores quanto
maior for o tempo de espera, que interfere não só na adaptação em caso de retorno à família de origem, mas também nos
casos de inserção definitiva em outra família.91
Neste sentido, considerando-se que o Acolhimento Institucional ainda cumpre um papel muito importante no cuidado
com crianças e adolescentes em situação de risco no Brasil, é crucial reconhecer a obrigatoriedade de promoção do direito
à convivência familiar e comunitária também recai sobre as entidades que oferecem programas de abrigo. Muito embora,
essa atribuição seja compartilhada por toda a rede de atendimento à criança e ao adolescente, que inclui ainda o Judiciário,
o Ministério Público, os Conselhos Tutelares e de Direitos da Criança e do Adolescente, as organizações civis de defesa de
direitos humanos e o próprio Poder Executivo nos níveis federal, estadual e municipal.
O perfil institucional dos 589 abrigos identificado no Levantamento Nacional aponta que majoritariamente essas insti-
tuições são não-governamentais, orientadas por valores religiosos, dirigidas por voluntários e que dependem fundamen-
talmente de recursos próprios e privadas para o seu funcionamento (Quadro 1).

89 Observados os dispostos nos artigos 166 e 169 do ECA, a adoção requer a destituição do poder familiar e implica no afastamento
definitivo da criança e do adolescente de suas famílias de origem. Assim, para não incorrer em injustiças, é da maior importância que essas
famílias recebam apoio e suporte necessários para sua reestruturação. Em muitos casos, a inadequação dos processos de destituição do poder
familiar pode provocar injustiças com famílias que sequer receberam apoio e/ou tiveram tempo para reintegração de seus filhos. No entanto, é
fundamental chamar atenção para o fato de que o próprio Estatuto estabelece que o abrigo é uma “medida provisória e excepcional, utilizável
como forma de transição para a colocação em família substituta”, não sendo aceitável a permanência indefinida de crianças e adolescentes nas
instituições sem qualquer perspectiva de convivência.
90 Cumpre esclarecer que o ECA estabelece em seu artigo 93: “As entidades que mantenham programas de abrigo poderão, em caráter
excepcional e de urgência, abrigar crianças e adolescentes sem prévia determinação da autoridade competente, fazendo comunicação do fato
até o segundo dia útil imediato.
91 SILVA, Roberto, apud MINISTÉRIO DO DESENVOLVIMENTO SOCIAL, Comitê Nacional para o Reordenamento dos Abrigos. Subsí-
dios para reflexão na aplicação da medida e o funcionamento de programas em regime de abrigo. Brasília: 2003, p. 13 (não publicado).

166
LEGISLAÇÃO

Quadro 1 – Perfil das entidades de abrigo

Não-governamentais..............................................................................................................68,3%
Públicas ....................................................................................................................30,0%
Têm orientação/vínculo religioso................................................................................................................67,2%
Católicos............................62,1%
Evangélicos.......................22,5%
Espíritas.............................12,6%
• Anteriores a 1990 .....................................................................................................................................41,4%
Posteriores a 1990.....................................................................................................................................58,6%
• Dirigidas por voluntários ............................................................................................................................59,3%
Dirigidas por profissionais remunerados....................................................................................................33,4%
• Profissionais do quadro próprio do abrigo..................................................................................................59,2%
Profissionais voluntários ..............................................................................................................................25,3%
• Funcionam sob regime de permanência integral.........................................................................................78,4%
Funcionam sob outros regimes de permanência........................................................................................19,7%
• Recursos próprios e privados no financiamento das entidades não-governamentais.................................61,7%
• Recursos públicos no financiamento das entidades não-governamentais.....................................................32,3%

Fonte: IPEA/Conanda. O Direito à Convivência Familiar e Comunitária: os abrigos para crianças e adolescentes no Brasil.
Brasília, 2004.

Ao analisar com base nos princípios do ECA os aspectos do atendimento realizado pelos abrigos quanto à convivência
familiar, o Levantamento Nacional observou que, em relação às ações de incentivo à convivência das crianças e dos adoles-
centes com suas famílias de origem, a maioria dos programas realiza visitas das crianças e adolescentes aos seus lares, mas
a minoria permite visitas livres dos familiares aos abrigos Entretanto, somente 31,2% realizavam as duas ações conjunta-
mente. Quanto às ações de não-desmembramento de grupos de irmãos, a maioria dos programas priorizava a manutenção
ou a reconstituição de grupos de irmãos, adotava o modelo de “agrupamento vertical”, possibilitando o acolhimento de
irmãos em diferentes idades e recebia tanto meninos quanto meninas. Contudo, somente 27,8% do total das instituições
que desenvolvem programas de abrigo atendiam todas as três ações (Quadro 2).

Quadro 2 – Preservação dos vínculos familiares

ABRIGOS QUE
CRITÉRIOS CONSIDERADOS
ATENDEM (%)
1. Incentivo à convivência com a família de origem
1.1. Promovem visitas de crianças e adolescentes aos lares de suas famílias 65,9%
1.2. Permitem visitas livres dos familiares ao abrigo 41,4%
 Atendem a todos os critérios 31,2%
2. Não-desmembramento de grupos de irmãos abrigados
1.1. Priorizam a manutenção ou reconstituição de grupos de irmãos 66,4%
1.2. Organizam-se sob agrupamento vertical (intervalo entre idades mínima e
62,1%
máxima maior do que 10 anos)
1.3. Atuam em regime de coeducação (meninos e meninas) 62,3%
 Atendem a todos os critérios 27,8%
1+2 6,6%

Fonte: IPEA/DISOC, Levantamento Nacional de Abrigos para Crianças e Adolescentes da Rede SAC (2003)

167
LEGISLAÇÃO

Além do fortalecimento e da manutenção dos vínculos afetivos entre as crianças e adolescentes em abrigos e seus
familiares, o apoio à reestruturação das famílias constitui-se em ação importante e complexa. Muito embora a maioria dos
programas realize atividades de visitas às famílias e acompanhamento social, a minoria realiza reuniões ou grupos de dis-
cussão e encaminha famílias para inserção em programas de proteção social. E ainda, somente 14,1% do total de abrigos
pesquisados realizavam todas as quatro ações de apoio à reestruturação familiar (Quadro 3).

Quadro 3 – Apoio à reestruturação familiar

ABRIGOS QUE
CRITÉRIOS CONSIDERADOS
ATENDEM (%)
1. Realizam visitas domiciliares 78,1%
2. Oferecem acompanhamento social 65,5%
3. Organizam reuniões ou grupos de discussão e apoio 34,5%
4. Encaminham para inserção em programas de auxílio/proteção à família 31,6%
1+2+3+4 14,1%

Fonte: IPEA/DISOC, Levantamento Nacional de Abrigos para Crianças e Adolescentes da Rede SAC (2003)

O relacionamento de crianças e adolescentes abrigados com outras famílias é outra forma de garantir o direito à convi-
vência familiar cujas chances de retorno para suas famílias de origem foram esgotadas. Assim, a colocação em família subs-
tituta configura-se como uma opção frente à tradicional prática brasileira de institucionalização prolongada de crianças e
adolescentes em situação de risco, condenados a viver grande parte de suas vidas privados de qualquer vivência familiar.92
Mesmo que a colocação em família substituta não dependa exclusivamente do trabalho das instituições de abrigo,
elas podem desempenhar um papel fundamental nesse processo, incentivando a convivência de crianças e adolescentes
abrigados com outras famílias por meio de ações como: o incentivo à integração em família substituta sob as formas de
guarda, tutela ou adoção, o envio de relatórios periódicos sobre a situação dos abrigados e de suas famílias para as Varas
da Infância e da Juventude e a manutenção de programas de apadrinhamento afetivo.93 Das 589 instituições pesquisadas,
apenas 22,1% desenvolviam todos esses tipos de ação de incentivo à convivência de crianças e adolescentes abrigados com
outras famílias (Quadro 4).

Quadro 4 – Incentivo à convivência familiar com outras famílias

ABRIGOS QUE
CRITÉRIOS CONSIDERADOS
ATENDEM (%)
1. Incentivam a integração em família substituta (guarda, tutela ou adoção) 67,5%
2. Mantêm programas de apadrinhamento 55,3%
1+2 22,1%

Fonte: IPEA/DISOC, Levantamento Nacional de Abrigos para Crianças e Adolescentes da Rede SAC (2003).

Nos aspectos do atendimento realizado pelos abrigos quanto à convivência comunitária, o Levantamento Nacional
identificou um quadro preocupante em relação às ações de estímulo à participação das crianças e adolescentes na vida da
comunidade local, pois apenas 6,6% dos abrigos pesquisados utilizavam todos os serviços necessários que estavam dispo-
níveis na comunidade, tais como: creche; ensino regular; profissionalização para adolescentes; assistência médica e odon-
tológica; atividades culturais, esportivas e de lazer; e assistência jurídica. A maioria das instituições (80,3%) ainda oferecia
pelo menos um desses serviços diretamente, ou seja, de forma exclusiva dentro do abrigo (Quadro 5).

92 O Estatuto estabelece como princípio a ser seguido pelas entidades de abrigo “a colocação em família substituta, quando esgotados
os recursos de manutenção na família de origem” (Lei 8.069/90, Art. 92, Inc. II).
93 Os programas de apadrinhamento se constituem em alternativa de referência familiar para as crianças e os adolescentes abrigados.

168
LEGISLAÇÃO

Quadro 5 – Participação na vida da comunidade local

ABRIGOS QUE
CRITÉRIO CONSIDERADO
ATENDEM (%)
1. Utilizam serviços especializados existentes na comunidade 6,6 %
2. Oferecem pelo menos um dos serviços de forma exclusiva dentro dos abrigos 80,3%

Fonte: IPEA/DISOC, Levantamento Nacional de Abrigos para Crianças e Adolescentes da Rede SAC (2003)

Muitas das instituições investigadas – a maioria delas – como se viu, surgida durante a vigência do ECA já introduziram
condutas diferentes e programas mais condizentes com as diretrizes legais, ampliando-se no país o elenco de experiências
pautadas pelos princípios da proteção integral e do atendimento individualizado. No entanto, há ainda inúmeras institui-
ções que mantêm práticas que privam quase que totalmente crianças e adolescentes da convivência social. Sobretudo, ain-
da falta estratégia de coordenação das várias atividades desenvolvidas e que poderiam contribuir para a promoção efetiva
da convivência familiar e comunitária daqueles que vivem nesses abrigos.
As questões mais decisivas talvez estejam relacionadas com a falta de integração entre essas instituições e os demais
atores da rede de atendimento, o que dificulta em muito a realização de suas atividades em consonância com os princípios
do ECA.
No tocante às alternativas ao Acolhimento Institucional é importante considerar o acolhimento familiar provisório de
crianças e adolescentes em situação de risco. Com efeito, na busca de garantir o direito à convivência familiar e comuni-
tária às crianças e adolescentes privados do convívio com seus pais, uma primeira opção que poderia ser considerada é o
acolhimento por outros membros da família da criança ou adolescente em risco, a chamada família extensiva. Outra forma
de propiciar vivência em família para esta população seriam as experiências de acolhimento por famílias, que têm surgido
em vários lugares do mundo – sobretudo na Europa, e, mais recentemente no Brasil – sob as denominações de famílias
acolhedoras, guardiãs, madrinhas, entre outras.
É preciso ser destacado, entretanto, que o acolhimento familiar não se apresenta como substituto ao atendimento ins-
titucional ou às políticas de adoção. Ao contrário, deve ser mais uma opção, na busca da melhor medida para cada criança
ou adolescente que teve um ou mais de seus direitos violados.
Na construção de que o atendimento institucional e o acolhimento familiar sejam opções alternativas em vez de exclu-
dentes, busca-se humanizar o cuidado institucional, com a mudança da postura institucional e do quadro de recursos hu-
manos em relação aos abrigados e a suas famílias; a organização de atendimentos complementares, como as experiências
de crianças e adolescentes que vivem em instituições e visitam famílias voluntárias em finais de semana e férias; ou, ainda,
a aproximação do ambiente institucional aos padrões familiares e residenciais, conforme recomendado pelo ECA.
Da mesma forma que temos tradição de atendimento institucional como caminho usual na atenção às crianças e ado-
lescentes em situação de risco, a colocação em família substituta no Brasil é muito mais conhecida na forma de adoção.
No Brasil, ainda não existe a tradição do acolhimento formal por famílias voluntárias e o caminho mais usual de colo-
cação em família substituta acaba sendo a adoção. Entretanto, como se trata de uma medida definitiva, a adoção não deve
ser vista como a única solução para os problemas das crianças pobres brasileiras, inclusive para o problema da institucio-
nalização indiscriminada, como com frequência se apresenta. Antes, a adoção deve ser encarada como uma entre várias
opções, a ser aplicada apenas quando as chances de manutenção ou recuperação dos vínculos com a família de origem,
incluindo-se a ampliada, não existem mais.
É preciso superar a aplicação indiscriminada de medidas que acarretam no afastamento de crianças e de adolescentes
de suas famílias de origem, o que, na prática, tem representado uma forma de vitimizar famílias, crianças e adolescentes
em situação de pobreza.
Desde a consagração da doutrina da proteção integral de crianças e adolescentes com a Constituição de 1988, em se-
guida com o ECA e posteriormente com a LOAS, vem-se exigindo da rede de atendimento, das instituições, dos programas
e dos serviços que prestam atendimento às crianças, aos adolescentes e às famílias a revisão e a mudança de suas práticas,
no sentido de se implantar alternativas que contemplem ações emancipatórias e, sobretudo que garantam os direitos das
crianças e dos adolescentes com prioridade absoluta nas políticas públicas, com decisões fundadas na avaliação do seu
melhor interesse, considerando sua voz e opinião.

169
LEGISLAÇÃO

Diretrizes que possam levar à constituição de novos vínculos familiares


e comunitários em caso de ruptura dos vínculos originais.
A mudança no paradigma do atendimento à criança e Para garantir a qualidade das políticas de apoio às famílias,
adolescente, sobretudo na efetivação do seu direito à con- o Estado tem a responsabilidade de capacitar seus agentes e
vivência familiar e comunitária apresentada na forma opera- de fiscalizar, monitorar e avaliar esses serviços na articulação
cional deste Plano fundamenta-se nas seguintes diretrizes: dos níveis municipal, estadual e federal.
- Reconhecimento das competências da família na
- Centralidade da família nas políticas públicas sua organização interna e na superação de suas dificul-
O direito das crianças e adolescentes à convivência fa- dades
miliar e comunitária está relacionada à inclusão social de As políticas especiais para promoção, defesa e garantia
suas famílias. O reconhecimento da importância da família do direito de crianças e adolescentes à convivência familiar e
no contexto da vida social está explicito no artigo 226 da comunitária devem reconhecer a família como um grupo so-
Constituição Federal do Brasil, na Convenção sobre os Di- cial capaz de se organizar e reorganizar dentro de seu con-
reitos da Criança, no Estatuto da Criança e do Adolescente, texto e a partir de suas demandas e necessidades. Em sua
na Lei Orgânica da Assistência Social e na Declaração dos relação com a sociedade e em sua rede de relações internas,
Direitos Humanos. a família apresenta capacidade de criar soluções para seus
A família é compreendida como um grupo de pessoas problemas, e de rever e reconstruir seus vínculos ameaça-
com laços de consanguinidade, de aliança, de afinidade ou dos, a partir do apoio recebido das políticas sociais.
de solidariedade, cujos vínculos circunscrevem obrigações Reconhecendo a complexidade desse processo, é pre-
recíprocas, organizadas em torno de relações de geração e ciso escutar e respeitar as famílias, seus valores e crenças,
de gênero. Arranjos familiares diversos devem ser respeita- criando com elas soluções que possam ser adequadas ao
dos e reconhecidos como potencialmente capazes de reali- contexto, coerentes com os direitos dos seus membros e
zar as funções de proteção e de socialização de suas crianças consistentes com as políticas sociais.
e adolescentes. - Respeito à diversidade étnico-cultural, à identidade
Sendo assim, “a família, independente de seu formato, é sexual e à equidade de gênero
a mediadora das relações entre os sujeitos e a coletividade e O apoio às famílias deve se pautar pelo respeito à diver-
sidade dos arranjos familiares, às diferenças étnico-raciais e
geradora de modalidades comunitárias de vida”.94 Portanto,
culturais bem como à equidade de gênero, consoante com
diante de situações de risco social e vulnerabilidades vividas
a Constituição Federal. A defesa dos direitos de cidadania
pelas famílias brasileiras, principalmente por pressões gera-
deve ter cunho universalista, considerando todos os atores
das pelos processos de exclusão social e cultural, essas famí-
sociais desenvolvidos no complexo das relações familiares
lias precisam ser apoiadas pelo Estado e pela sociedade, para
e sociais e tendo impacto emancipatório nas desigualda-
cumprir suas responsabilidades. Esse apoio visa a superação
des sociais. Dessa forma, o respeito à diversidade não pode
de vulnerabilidades e riscos vividos por cada família, favo-
ser contraditório com uma ética dos direitos que incentive
recendo e ampliando os recursos socioculturais, materiais, mudanças culturais, por meio do resgate das tradições de
simbólicos e afetivos que contribuem para o fortalecimen- cuidado e afeto nos vínculos familiares e comunitários, em
to desses vínculos. Diante disso, a centralidade da família suas bases de identidade cultural, nem com a construção
no âmbito das políticas públicas se constitui em importante participativa de novas práticas. O respeito à diversidade está
mecanismo para a efetiva garantia do direito à convivência associado à reflexão das famílias sobre suas bases culturais,
familiar e comunitária de crianças e adolescentes. ao combate aos estigmas sociais, à promoção dos direitos
- Primazia da responsabilidade do Estado no fomen- humanos e ao incentivo aos laços de solidariedade social.
to de políticas integradas de apoio à família - Fortalecimento da autonomia do adolescente e do
No cumprimento do princípio da prioridade absoluta à jovem adulto na elaboração do seu projeto de vida
garantia dos direitos da criança e do adolescente, o Estado Sendo a criança e o adolescente sujeitos de direitos, é
deve se responsabilizar por oferecer serviços adequados e necessário reconhecer suas habilidades, competências, inte-
suficientes à prevenção e superação das situações de viola- resses e necessidades específicas, incentivando-os, inclusive
ção de direitos, possibilitando o fortalecimento dos vínculos por meio de espaços de participação nas políticas públicas, à
familiares e sócio-comunitários. O apoio às famílias e seus busca compartilhada de soluções para as questões que lhes
membros deve ser concretizado na articulação eficiente da são próprias.
rede de atendimento das diferentes políticas públicas, ga- Atenção especial deve ser dada aos adolescentes em re-
rantindo o acesso a serviços de educação, de saúde, de ge- gime de Acolhimento Institucional, ou sem possibilidades de
ração de trabalho e renda, de cultura, de esporte, de assis- reatar os vínculos familiares e que requerem soluções par-
tência social, dentre outros. Nas situações de risco e enfra- ticipativas e negociadas para a elaboração de seus projetos
quecimento dos vínculos familiares, as estratégias de atendi- de vida. Os espaços públicos frequentados por crianças e
mento deverão favorecer a reestruturação do grupo familiar adolescentes e as instâncias de formulação de políticas pú-
e a elaboração de novas referências morais e afetivas. Estas blicas constituem importantes instrumentos para exercício
estratégias visam potencializar a família para o exercício de dos direitos de cidadania, sob a perspectiva tanto de in-
suas funções de proteção e socialização e o desenvolvimen- centivar a criatividade no campo das ciências, das artes, da
to de sua autonomia, incluindo o desenvolvimento de ações cultura e dos esportes quanto na formação de lideranças
94 Política Nacional de Assistência Social – PNAS (2004). infanto-juvenis.

170
LEGISLAÇÃO

- Garantia dos princípios de excepcionalidade, bre- atendimento significa reorientar as redes pública e privada,
vidade e provisoriedade nos programas de Acolhimen- que historicamente praticaram o regime de abrigamento,
to Familiar e Acolhimento Institucional95 de crianças e para afinarem-se com a mudança de paradigma propos-
de adolescentes to, de eleger a família como a unidade básica da ação so-
Toda medida de proteção que indique o afastamen- cial, e não mais a criança, o adolescente, o deficiente físico
to da criança e do adolescente de seu contexto familiar, ou o idoso, individualmente e deslocado de seu contexto
podendo ocasionar suspensão temporária ou ruptura dos familiar. Conselhos Municipais dos Direitos da Criança e
vínculos atuais, deve ser uma medida rara, excepcional. do Adolescente, Conselhos Municipais de Assistência So-
Apenas em casos onde a situação de risco e desproteção cial e órgãos públicos repassadores de recursos podem
afetam a integridade do desenvolvimento da criança e do sugerir adequações, tanto nos estatutos quanto nos pro-
adolescente é que se deve pensar no seu afastamento da jetos pedagógicos das entidades, como condição para o
família de origem. A decisão sobre a separação é de gran- registro, para aprovação de projetos e/ou para liberação
de responsabilidade, por parte dos agentes sociais e deve de recursos. Esta diretriz requer ações como: 1) mudança
estar baseada em fundamentação teórica sobre o desen- na sistemática de financiamento das entidades de abrigo,
volvimento infantil, as etapas do ciclo de vida individual e eliminando-se formas que incentivem a manutenção inde-
familiar e a teoria dos vínculos; e deve ter como prioridades finida das crianças e adolescentes na instituição – como o
a comunicação na família e o investimento na reorganiza- financiamento por criança e adolescente atendido; 2) qua-
ção dos laços familiares. lificação dos profissionais que trabalham na entidade; 3)
A análise da situação evita danos ao desenvolvimento estabelecimento de indicadores qualitativos e quantitati-
da criança e do adolescente causados por separações brus- vos de avaliação para a entidade; 4) desenvolvimento ou
cas, longas e desnecessárias. Deve, ainda, considerar a qua- incorporação de metodologias para trabalho com famílias;
lidade das relações, a atitude proativa de seus membros na 5) ênfase na prevenção ao abandono e na recuperação das
requalificação dos vínculos e construção de sua autonomia. competências da família; 6) reconhecimento da autonomia
A decisão por um afastamento definitivo, ou seja, a e da competência da família para bem criar e educar os
destituição do poder familiar, só deve ocorrer após um seus filhos, 7) adequação do espaço físico e do número
investimento eficiente na busca de recursos na família de de crianças e adolescentes atendidos em cada unidade, de
origem, nuclear ou extensa, com acompanhamento profis- forma a garantir o atendimento individualizado e em pe-
sional sistemático e aprofundado de cada caso e conside- quenos grupos, 8) articulação com a rede de serviços e o
rando o tempo de afastamento, a idade da criança e do SGD. As instituições que aceitarem reordenarem-se podem
adolescente e a qualidade das relações. continuar atendendo em regime de abrigamento, em si-
Cabe esclarecer que a expressão “Acolhimento” utiliza- tuações de provisoriedade, brevidade e de excepcionalida-
da neste Plano refere-se aos Programas de Proteção Social de, desde que incluam em seus objetivos o atendimento: 1)
Especial de “Acolhimento Institucional” ou “Acolhimento das famílias das crianças e dos adolescentes abrigados; 2)
Familiar”. Por Acolhimento familiar entende-se a moda- famílias das crianças e adolescentes desabrigadas; 3) famí-
lidade de atendimento que oferece acolhimento na resi- lias da comunidade em situação de vulnerabilidade social
dência de famílias previamente cadastradas, selecionadas, e/ou que sejam encaminhados por agentes do sistema de
capacitadas e acompanhadas para receber crianças e/ou garantia de direitos.
adolescentes com medida de proteção, que necessitem de - Adoção centrada no interesse da criança e do ado-
acolhimento fora da família de origem até que seja possível lescente
sua reintegração familiar ou encaminhamento para família De acordo com o ECA, a colocação em família substitu-
substituta. Por Acolhimento Institucional entende-se a mo- ta, concebida nas formas de guarda, tutela e adoção, é uma
dalidade de atendimento integral institucional que oferece medida de proteção que visa garantir o direito fundamen-
acolhimento, cuidado e espaço para socialização e desen- tal das crianças e adolescentes à convivência familiar e co-
volvimento de crianças e adolescentes com medida de pro- munitária. Entretanto, tradicional e culturalmente a adoção
teção, que necessitem de acolhimento fora da família de foi e ainda é bastante aplicada no Brasil com a finalidade
origem, até que seja possível sua reintegração familiar ou precípua de dar filhos a quem não os tem, estando, portan-
encaminhamento para família substituta. Recebem atual- to, centrado no interesse dos adultos.
mente várias denominações, tais como: “abrigos”, “casas O direito de toda criança e adolescente cujos pais fo-
lares”, “casas de passagem”, entre outros. ram destituídos do poder familiar deve prevalecer sobre
- Reordenamento institucional dos programas de o desejo dos pretendentes a adoção. A orientação deve
Acolhimento Institucional em consonância com as nor- seguir a ideia de “uma família para uma criança” e não de
mativas nacionais, priorizando o desenvolvimento de “uma criança para uma família”. Isso pressupõe a busca de
ações sustentadas nos princípios dos direitos humanos famílias disponíveis a acolherem crianças e adolescentes
O reordenamento institucional se constitui em um novo hoje privados do direito à convivência familiar e comuni-
paradigma na política social que deve ser incorporado por tária. Não se trata mais de procurar crianças para preen-
toda a rede de atendimento social do país. Reordenar o cher o perfil desejado pelos pretendentes, mas sim de in-
95 Medidas de proteção especial, excepcional e temporária formá-los quanto ao perfil dessas crianças e adolescentes
que visam atender crianças e adolescentes que precisam ser afasta- efetivamente disponíveis para adoção. Este é o sentido da
dos provisoriamente de suas famílias de origem. proposta de uma nova cultura para a adoção, que não mais

171
LEGISLAÇÃO

se atém à semelhança biológica, tradicionalmente procura- - Aprimorar os procedimentos de adoção nacional e


da na adoção, mas que entende a adoção como alternativa internacional.
excepcional e extraordinária para assegurar o direito à con- - Fomentar a implementação de programas para pro-
vivência familiar e comunitária. moção da autonomia do adolescente e/ou jovem egressos
A nova cultura da adoção visa estimular, sobretudo, de programas de acolhimento, desenvolvendo parâmetros
as adoções inter-raciais, as adoções tardias96 a adoção de para a sua organização, monitoramento e avaliação;
crianças e adolescentes com deficiências físicas ou men- - Aprimorar os procedimentos de adoção nacional e
tais e a adoção de crianças e adolescentes com doenças internacional, visando: a) estimular, no País, as adoções de
congênitas e afetados pelo vírus HIV/AIDS, para assegurar crianças e adolescentes que, por circunstâncias diversas,
a todos o respeito ao seu direito à convivência familiar e têm sido preteridos pelos adotantes – crianças maiores e
comunitária. adolescentes, com deficiência, com necessidades especí-
- Controle social das políticas públicas ficas de saúde, afrodescendentes ou pertencentes a mi-
Efetivada nas normativas constitucional e infraconsti- norias étnicas, dentre outros; b) investir para que todos
tucionais (Constituição Federal, Convenção sobre os Direi- os processos de adoção no País ocorram em consonân-
tos da Criança, ECA, LOAS, LDB e LOS) a participação po- cia com os procedimentos legais previstos no Estatuto
pular, com caráter democrático e descentralizado se dá em da Criança e do Adolescente; e c) garantir que a adoção
cada esfera do governo, abrangendo o processo de gestão internacional ocorra somente quando esgotadas todas as
político-administrativa-financeira e técnico-operativa. O tentativas de adoção em território nacional, sendo, nestes
controle do Estado deve ser exercido pela sociedade na casos, priorizados os países que ratificaram a Convenção
busca de garantir os direitos fundamentais e os princípios de Haia;
democráticos. - Assegurar estratégias e ações que favoreçam os me-
Os Conselhos e as Conferências são espaços privile- canismos de controle social e a mobilização da opinião
giados para esta participação, mas, também existem outros pública na perspectiva da implementação do Plano Nacio-
como a mídia e os conselhos profissionais. As Conferên- nal de Promoção, Proteção e Defesa do Direito de Crianças
cias avaliam a situação das políticas públicas e da garantia e Adolescentes à Convivência Familiar e Comunitária;
de direitos, definem diretrizes e avaliam os seus avanços. - Aprimorar e integrar mecanismos para o cofinancia-
Os Conselhos têm, dentre outras, a responsabilidade de mento, pela União, Estados, Distrito Federal e Municípios,
formular, deliberar e fiscalizar a política de atendimento, das ações previstas no Plano Nacional de Promoção, Pro-
normatizar, disciplinar, acompanhar e avaliar os serviços. teção e Defesa do Direito de Crianças e Adolescentes à
Avanços na organização e fortalecimento da participação Convivência Familiar e Comunitária, tendo como referên-
da população são necessários, buscando a integração das cia a absoluta prioridade definida no artigo 227 da Consti-
políticas sociais nos níveis federal, estadual e municipal.
tuição Federal de 1988 e no artigo 4° do Estatuto da Crian-
A consolidação de novas representações e práticas das
ça e do Adolescente.
famílias e da sociedade acerca dos direitos das crianças e
adolescentes reside na sustentabilidade de uma mudança
Resultados Programáticos
cultural. A legitimidade desta mudança cultural apoia-se
O direito à convivência familiar e comunitária, assegu-
nos processos participativos e no exercício do controle so-
rado como fundamental na Carta Constitucional e na le-
cial, por meio das instituições da sociedade, sobre a política
gislação infraconstitucional, garantido a todas as crianças
social e na ética da defesa dos direitos.
e adolescentes, demanda iniciativas de diferentes políticas
Objetivos Gerais públicas e sociais.
Essa articulação e intersetorialidade entre as políticas,
- Ampliar, articular e integrar as políticas, os progra- no Sistema de Garantia de Direitos e respaldados pelos
mas, os projetos, os serviços e as ações de apoio socio- seus mecanismos de exigibilidade de direitos, é condição
familiar para a promoção, defesa e garantia do direito a fundamental para que a família, a comunidade, o poder
convivência familiar e comunitária; público e a sociedade em geral assegurem a efetivação
- Difundir uma cultura de promoção, defesa e garantia dos direitos descritos nos artigos 227 da Constituição Fe-
do direito de crianças e adolescentes à convivência familiar deral e 4º do ECA.
e comunitária. O fortalecimento, a efetivação e a consolidação desses
- Parametrizar o Acolhimento Institucional como me- direitos passam necessariamente pela concretização de
dida de caráter excepcional e provisório, assegurando políticas, programas, projetos, serviços e ações que asse-
atendimento individualizado de qualidade e em pequenos gurem aquilo o que antes se constituía em expectativa de
grupos; direito.
- Fomentar e implementar alternativas à institucionali- O Plano Nacional de Promoção, Defesa e Garantia do
zação, na forma de programas de Acolhimento Familiar e Direito de Crianças e Adolescentes à Convivência Familiar
de programas para promoção da autonomia do adolescen- e Comunitária pretende, com sua execução, materializar
te e do jovem adulto; esse direito fundamental, alcançando resultados progra-
96 São consideradas adoções tardias as adoções de crianças máticos a seguir descritos:
acima de dois anos de idade.

172
LEGISLAÇÃO

Família de origem/comunidade - Programas de Acolhimento Institucional e Acolhi-


mento Familiar em constante articulação com o Conselho
- Famílias incluídas, principalmente aquelas em maior Tutelar e a Vara da Infância e Juventude para o acompa-
vulnerabilidade social, nas políticas sociais de educação, saú- nhamento;
de, assistência social, esporte cultura e lazer e tendo acesso - Programas de Acolhimento Institucional e Acolhi-
a: habitação digna; creches; atividades lúdicas, esportivas e mento Familiar assegurando os princípios de excepciona-
culturais que respeitem a diversidade étnico-racial e de gê- lidade, provisoriedade e transitoriedade no atendimento,
nero; escolarização formal e reforço escolar no contra turno priorizando o enfoque nas relações afetivas da criança e do
escolar; tratamento preventivo e curativo da dependência adolescente com suas famílias de origem;
de álcool e outras drogas na rede de saúde, informações/ - Parâmetros nacionais de atendimento para progra-
orientações quanto ao pré-natal tanto no atendimento ao mas de Acolhimento Familiar elaborados conjuntamente
aspecto físico quanto no psicoafetivo, com destaque para pelos Conselhos Nacionais dos Direitos da Criança e do
mães adolescentes; oferta de métodos contraceptivos aos Adolescente e da Assistência Social e critérios de operacio-
que desejarem, apoio às crianças e adolescentes com defi- nalização definidos pelos Conselhos Municipais dos Direi-
ciência e afetados pelo vírus HIV/AIDS e suas famílias; apoio tos da Criança e do Adolescente e da Assistência Social em
sociofamiliar; atividades socioeducativas; atendimento psi- cumprimento pelos programas;
cossocial; transferência de renda; qualificação profissional; - Programas de Acolhimento Familiar funcionando
geração de renda e inclusão no mundo do trabalho; como alternativa ao abrigamento em instituições e reco-
- Famílias estimuladas a buscar e participar em sua co- nhecidos como medida de proteção social e defesa do di-
munidade de diferentes espaços de integração e mobiliza- reito à convivência familiar e comunitária e, portanto incor-
ção social, assegurando por meio do controle social a quali- porados na política de atendimento à infância e juventude
dade dos serviços e, sobretudo favorecendo o dinamismo a e as demais políticas públicas municipais;
diversidade cultural e a sua participação política; - Famílias dos programas de Acolhimento Familiar ca-
- Equipamentos e serviços públicos disponibilizados em pacitadas para o atendimento, para facilitar a reconstru-
quantidade e qualidade suficientes e prontos para atender ção e/ou preservação do vínculo com a família de origem
às demandas da população em situação de vulnerabilidade e para manter grupos de irmãos em um mesmo programa;
social, com programas e ações preventivos à fragilização e/ - Crianças e adolescentes de programas de Acolhimen-
ou rompimento de vínculos;
to Institucional sem perspectivas de adoção, colocadas em
- Família participando ativamente nos projetos políti-
programas de Acolhimento Familiar ou em programas que
copedagógicos dos programas de atendimento governa-
estimulem a passagem para a sua autonomia;
mental e não-governamental de apoio socioeducativo às
- Projeto políticopedagógicos dos programas de Aco-
crianças e adolescentes, incluindo também os programas
lhimento Institucional prevendo: a incompletude institucio-
que atendem adolescentes em conflito com a lei;
nal, o atendimento personalizado e orientado pelo Plano
- Equipamentos, programas e serviços públicos e so-
Individual de Atendimento da criança e do adolescente, o
ciais em permanente articulação entre si e com os Conse-
lhos Tutelares, Vara da Infância e Juventude, Ministério Pú- atendimento em pequenos grupos, a manutenção de gru-
blico, Conselhos de Direitos e Setoriais de políticas públicas, pos de irmãos num mesmo programa, o registro de dados
mantendo uma rede de informações que assessore o aten- de cada criança e adolescente constantemente atualizado,
dimento e acompanhamento das famílias; a preparação para o desligamento e a participação na co-
- Família com vínculos fragilizados incluídas em progra- munidade local;
mas de superação de violação de direitos e fortalecimento - Profissionais do Acolhimento Institucional capaci-
de vínculos familiares; tados permanentemente no trabalho social de famílias e
- Famílias em situação de vulnerabilidade incluídas em atuando sistematicamente no reforço aos vínculos familia-
ações de fortalecimento da autonomia, da independência, res, priorizando o investimento na família de origem e na
da autoestima e da identidade, tendo reconhecidas as dife- defesa do direito à convivência familiar e comunitária das
renças culturais, favorecendo a existência de um contexto crianças e adolescentes que vivem em instituições;
positivo para a criação dos filhos e o desenvolvimento de - Crianças e adolescentes pretendidos à adoção aten-
seus projetos de vida. didos por programas de Acolhimento Institucional sendo
previamente preparados;
Acolhimento Institucional Programas de Famílias Aco- - Aumento dos encaminhamentos de crianças e ado-
lhedoras. lescentes aos programas de Acolhimento Familiar (como
transição para a volta á família de origem), aos programas
- Modalidades de Acolhimento Institucional (Casa de que estimulem a passagem para a autonomia ou encami-
Passagem, Abrigo de pequeno porte, Casa Lar e República) nhadas à Adoção reduzindo a permanência no Acolhimen-
oferecidos na rede de atendimento municipal; to Institucional;
- Todos os programas de Acolhimento Institucional e - Destituição do poder familiar proposto com seguran-
Acolhimento Familiar devidamente registrados no Conse- ça pelos profissionais dos programas quando esgotados
lho Municipal dos Direitos da Criança e do Adolescente todos os investimentos na capacidade de reorganização do
(CMDCA), atendendo as diretrizes do artigo 92 do ECA e contexto que gerou o afastamento da criança e do adoles-
no Conselho Municipal de Assistência Social (CMAS); cente da família de origem, nuclear ou extensa.

173
LEGISLAÇÃO

Adoção nacional e internacional Sistema de Garantia de Direitos da criança e do adoles-


cente
- Aumento do número de famílias pretendentes à ado-
ção disponíveis em acolher criança maiores de cinco anos in- - Políticas públicas e, principalmente, sociais, entre
dependente da raça/etnia,97 deficiência ou estado de saúde; elas: educação, saúde, assistência social, cultura, esporte,
- Famílias adotivas devidamente preparadas e acompa- lazer, trabalho, previdência social, segurança pública, exe-
nhadas pela equipe técnica da Vara da Infância e Juventude cutando suas ações intersetorialmente com qualidade e
(VIJ) da sua comarca e por grupos de apoio à adoção (GAA); chegando efetivamente aos seus destinatários;
- Famílias pretendentes à adoção assessoradas com efi- - Conselhos Tutelares, Judiciário, Poder Executivo,
ciência pela Defensoria Pública e/ou advogado particular Organizações Não-Governamentais, Poder Legislativo,
durante o processo de adoção; Conselhos de Direitos e Setoriais e sociedade em geral
- Metodologia desenvolvida e consensuada entre a VIJ e desempenhando ativamente suas tarefas e responsabili-
o GAA para apresentação da família pretendente à criança e
dades na rede de atendimento às crianças e adolescentes
ao adolescente a serem adotados respeitando o tempo e o
afastados ou em vias de afastarem-se do convívio familiar;
entrosamento gradual entre as partes;
- Conselho Tutelar desempenhando suas prerrogati-
- Crianças e adolescentes com os devidos dados e regis-
vas legais, tendo sua decisão de abrigamento respaldada
tros da sua história de vida de forma que a família preten-
dente tenha condições de adotar suas histórias; num consciencioso diagnóstico da excepcionalidade da
- Estágio de convivência da família adotiva com a crian- medida e conselheiros tutelares com boa estrutura logís-
ça e adolescente autorizado pela VIJ da comarca de origem tica mantendo estreito contato com toda a rede de ser-
devidamente respaldado pelo acompanhamento técnico; viços e sobretudo respaldado pelo Poder Executivo local;
- Busca ativa de pais para crianças e adolescentes priori- - Poder Executivo desempenhando suas prerrogativas
zando a adoção nacional; legais, sendo responsável pelo atendimento à população
- Famílias adotivas frequentando grupos de pais adoti- e contando com equipe profissional em estreita parce-
vos, recebendo atendimento individualizado com frequên- ria com o Conselho Tutelar, realizando o diagnóstico e o
cia sistemática durante o período previamente determinado acompanhamento às famílias de forma preventiva e pro-
(aproximadamente um ano, podendo estender-se caso ne- tetiva por meio medidas de Acolhimento (Institucional ou
cessário), recorrendo sempre que necessário à equipe técni- Familiar), promovendo a proteção social básica e especial
ca da VIJ, da sua região, todas as vezes que necessitar du- da criança e do adolescente em situação de risco;
rante o período de adaptação da criança e do adolescente; - Organizações não-governamentais oferecendo
- Seminários e trocas de experiências auxiliando no apri- complementação ao atendimento oferecido pelo Poder
moramento de metodologias de acompanhamento; Executivo e requisitando a participação ativa da comuni-
- Sociedade brasileira informada sobre adoções inter-ra- dade na solução de seus problemas; atuando de forma
ciais e tardias reduzindo o preconceito às famílias adotivas. integrada com as demais organizações da rede de aten-
- Encaminhamento processual da adoção agilizado, de- dimento, de acordo com a sua missão institucional e as
pois de esgotadas todas as possibilidades de reintegração à necessidades locais de trabalho especializado;
família de origem, evitando a longa permanência de crianças - Sociedade civil organizada participando ativamente
e adolescentes nos programas de Acolhimento Institucional; nos Conselhos de Direitos e Setoriais, deliberando e mo-
- Adoções nacionais bem sucedidas e país sendo reco- nitorando as políticas municipais, oferecendo programas
nhecido pela qualidade com que promove a adoção nacio- de Apoio Sociofamiliar, atividades socioeducativas, gera-
nal; ção de trabalho e renda, Abrigo, Casa lar, Repúblicas e
- Crianças e adolescentes encaminhados para adoção
Acolhimento Familiar;
internacional somente nos casos em que estejam esgotadas
- Poder Judiciário desempenhando suas prerrogativas
todas as tentativas de adoção em território nacional, respei-
legais, aplicando as medidas legais de proteção; contan-
tando a Convenção de Haia de 1993;
do com equipe técnica interdisciplinar própria, eficiente-
- Cadastro nacional de adoção em rede informatizada
e em funcionamento organizado sob responsabilidade da mente articulada com todos os atores sociais da região,
Secretaria Especial dos Direitos Humanos, denominado SI- monitorando a aplicação das medidas legais deliberadas
PIA/INFOADOTE, favorecendo a comunicação fluente entre em juízo; em estreita articulação com o Conselho Tutelar,
diversas Autoridades Centrais Estaduais e o agrupamento o Poder Executivo e a sociedade civil organizada, promo-
de informações relativas às crianças abrigadas e aos pre- vendo a proteção das crianças e adolescentes e prestando
tendentes nacionais e estrangeiros à adoção. atendimento efetivo a suas famílias;
- Poder Legislativo desempenhando suas prerrogati-
97 Considera-se que o termo raça, longe de possuir na atua- vas legais, promovendo a revisão das leis; monitorando o
lidade as conotações biológicas que tinha nos séculos XIX e começo orçamento público, determinando à Prefeitura a aplicação
do XX, é um conceito socialmente construído. Utilizado como indica- da política municipal deliberada no Conselho de Direitos
dor específico das diferenças e desigualdades sociais determinadas
pela cor e, portanto, serve para entender as discriminações raciais
e Setoriais promovendo por meio de audiências públicas
existentes no Brasil. espaço aberto para o controle social com participação de
todos os atores sociais estratégicos;

174
LEGISLAÇÃO

- Conselhos de Direitos e Setoriais desempenhando 3) Elaboração de Planos Estaduais e Municipais em


suas prerrogativas legais, sendo responsáveis pela dis- consonância com o Plano Nacional e constituição de Co-
cussão democrática e elaboração de políticas públicas, en- missões Intersetoriais de acompanhamento do Plano nas
volvendo crianças, adolescentes e suas famílias; controlan- esferas estaduais e municipais;
do as ações do Poder Executivo visando à implementação 4) Conselhos dos Direitos da Criança e do Adolescen-
das políticas e programas de atendimento, acionando a te nas três esferas públicas assumindo o presente Plano
participação da sociedade civil organizada; como prioridade, a partir de 2007, viabilizando recursos
- Fundo dos Direitos da Criança e do Adolescente nos orçamentos, de um modo geral, e, em particular, nos
(FDCA) e Fundo Municipal da Assistência Social (FMAS) Fundos da Infância e Adolescência para a sua implemen-
desempenhando uma gestão ágil e autônoma e contan- tação;
do com a participação dos diversos setores da sociedade; 5) Participação e integração entre os Conselhos de Di-
contando, também, com recursos suficientes para imple- reitos da Criança e Setoriais nas três esferas de governo;
mentar suas propostas; 6) Corresponsabilidade entre os entes federativos no
- Famílias participando ativamente da rede de atendi- financiamento para implementação dos objetivos e ações
mento, sendo lideranças protagonistas na defesa dos di- propostos no presente Plano.
reitos de sua comunidade;
- Sociedade em geral, sendo mobilizada por meio de Atribuições e Competências dos entes federativos
campanhas de divulgação pressionando os Poderes Exe-
cutivo, Legislativo e Judiciário, de forma a garantir a imple- Guardadas as competências e atribuições específicas
mentação e a continuidade das políticas públicas; nas disposições contidas na Constituição Federal, a reali-
- Controle social sobre a execução dos programas e zação do presente Plano somente será possível se for as-
dos orçamentos contando com a participação popular, sumido pelas três esferas públicas (União, Estados e Muni-
além do Estado e da sociedade civil organizada; cípios). Assim, os objetivos e ações propostos no presente
- Agilidade no fluxo de informações e troca entre ato- Plano terão as responsabilidades compartilhadas pelas
res sociais estratégicos garantindo a otimização dos resul- três esferas de governo.
tados no atendimento às crianças e adolescentes e famí-
lias em situação de risco; Competências e atribuições da Comissão de Acompa-
- Sistema de registro e de tratamento de dados para nhamento e Implementação do Plano, comuns às três esfe-
cada caso de criança e adolescente afastado de sua famí-
ras de governo
lia, por intermédio do SIPIA – Módulo de acompanhamen-
to de crianças e adolescentes em programas de Acolhi-
• Articular os atores envolvidos na implementação
mento Familiar e Institucional –, estabelecido e alimentado
para a consecução dos objetivos propostos nos eixos:
por todos os atores do Sistema e programado de forma a
a) análise da situação e sistemas de informação; b)
obter informações que orientem no diagnóstico, acompa-
atendimento; c) marcos normativos e regulatórios; d) mo-
nhamento de cada caso e prognóstico;
bilização, articulação e participação do presente Plano;
- Conselho Municipal de Direitos, de Assistência Social
• Identificar e mensurar os resultados, efeitos e impac-
e a Câmara de Vereadores, por meio de suas assembleias
tos dos objetivos e ações propostas antes, durante e de-
e audiências públicas, se constituindo em espaços privile-
giados para articulação dos atores sociais locais e partici- pois de sua implementação;
pação conjunta na elaboração e monitoramento de polí- • Proporcionar informações necessárias e contribuir
ticas públicas de proteção social e de garantia de direitos. para a tomada de decisões por parte dos responsáveis
pela execução dos objetivos e ações do Plano;
Implementação, Monitoramento e Avaliação • Acompanhar o desenvolvimento das ações e tarefas
referentes à execução do Plano;
O Presente Plano tem como desafio garantir efetiva- • Controlar as ações, as atividades e os resultados pro-
mente o direito de crianças e adolescentes à convivência postos no Plano assegurando o cronograma previsto;
familiar e comunitária, principalmente àquelas que se en- • Socializar informações periodicamente aos diferen-
contram em situação de vulnerabilidade. tes atores do Sistema de Garantia de Direitos e aos Con-
Sendo assim, sua implementação integral é condição selhos de Direitos da Criança e do Adolescente e da Assis-
fundamental para uma real mudança do olhar e do fazer tência Social;
que possibilite a concreta experiência e vivência singular • Avaliar continuamente a implementação do Plano,
da convivência familiar e comunitária para toda criança e nas diferentes esferas ajustando as condições operacio-
adolescente no Brasil. Para a materialização deste direito nais e correção de rumos durante o processo de execução;
será necessário: • Realizar bianualmente a revisão do Plano, de forma
1) Cumprimento integral deste Plano nas três esferas a adequá-lo às deliberações das Conferências Nacionais
de governo; dos Direitos da Criança e do Adolescente e da Assistência
2) Constituição formal de Comissão Nacional Interse- Social.
torial para acompanhamento da implementação do Plano;

175
LEGISLAÇÃO

Específicas à esfera Federal levantamento de informações e dados que auxiliarão no


monitoramento e avaliação do Plano em execução. Cabe
• Articular com as Comissões das esferas estadual e ressaltar que os dados a serem coletados, em sua maioria,
municipal para ampliar o diálogo e acompanhar o desen- devem ser obtidos no município que é o executor das polí-
volvimento das tarefas e ações dos referidos Planos; ticas públicas e a coleta dos dados deve ser anual.
• Produzir informações consolidadas sobre a imple-
mentação do Plano; Diagnóstico da situação de famílias com crianças e
• Socializar as informações consolidadas; adolescentes em Programas de Acolhimento Institucio-
• Cofinanciar as ações necessárias à implementação do nal, em situação de rua e em medida socioeducativa.
presente Plano, bem como dos Planos Estaduais e Muni-
cipais; • Número de famílias com crianças/adolescentes em:
• O Governo Federal deverá apresentar anualmente a) acolhimento institucional b) situação de rua c) em medi-
Relatório de Implementação do Plano Nacional de Promo- da socioeducativa, e outras, comparado com o número de
ção, Proteção e Defesa do Direito de Crianças e Adoles- famílias da população brasileira, observadas as variações
centes à Convivência Familiar e Comunitária, inclusive com de renda, arranjo familiar, meio rural ou urbano e perten-
informações sobre orçamento. cimento étnico;
• Número e perfil das famílias abrangidas pelas dife-
Específicas à esfera Estadual rentes políticas protetivas, por região ou território, ao ano,
inclusive comparando-se as médias nacionais e regionais;
• Dialogar permanentemente com a Comissão Nacio- • Número e perfil de crianças e adolescentes fora do
nal e com os municípios, visando o cumprimento deste convívio familiar devido a: a) por questões de pobreza; b)
Plano; por questões de uso e ou abuso de drogas (lícitas e ilíci-
• Apoiar os municípios no cumprimento deste Plano, tas); c)por violência doméstica; d) por abuso sexual; e) por
inclusive na produção de informações a serem consolida- exploração sexual; inclusive comparado com o número de
das; crianças e adolescentes na população brasileira, observa-
• Produzir informações consolidadas sobre a imple- das as variações de renda, gênero, meio rural ou urbano e
mentação do Plano; pertencimento étnico;
• Socializar as informações consolidadas; • Número de famílias das crianças e adolescentes em
• Encaminhar informações sobre monitoramento e as acolhimento institucional, em programas de famílias aco-
avaliações referentes à implementação do Plano nas esfe- lhedoras, em situação de rua, medida socioeducativa, e ou-
ras Estadual e Municipal em períodos previamente acorda- tras, em programas de transferência de renda, em relação
dos para a Comissão Nacional; ao total de famílias inseridas neste programa por território
• Cofinanciar as ações necessárias à implementação do ano a ano, inclusive observadas as variações de arranjo fa-
presente Plano, bem como dos Planos Estaduais e Muni- miliar, meio rural ou urbano e pertencimento étnico, nas
cipais. médias nacionais e regionais;
• Número de famílias inseridas em programas de assis-
Específicas à esfera Municipal tência, saúde, e outros que perderam a guarda temporária
dos filhos, inclusive comparado com o número de famílias
• Dialogar permanentemente com a Comissão Nacio- da população brasileira observadas as variações de renda,
nal e Estadual; arranjo familiar, meio rural ou urbano e pertencimento ét-
• Produzir informações consolidadas sobre a imple- nico;
mentação do Plano; • Número de famílias atendidas em programas so-
• Socializar as informações consolidadas; cioeducativos da proteção social básica, em relação à to-
• Encaminhar informações sobre monitoramento e as talidade de famílias na mesma faixa de renda no mesmo
avaliações referentes à implementação do Plano na esfera território, ano a ano, observadas as variações de arranjos
Municipal em períodos previamente acordados para a Co- familiares, meio rural ou urbano e pertencimento étnico;
missão Nacional; • Número de famílias inseridas em programas de aten-
• Cofinanciar as ações necessárias à implementação do dimento para prevenção da violência doméstica, em rela-
presente Plano, bem como do Plano Municipal. ção ao total de demanda e às famílias na mesma faixa de
renda, no território, ano a ano, observadas as variações de
Indicadores de eficácia e monitoramento arranjos familiares, meio rural ou urbano e pertencimento
étnico;
Para implementação, monitoramento e avaliação do • Causas geradoras do rompimento dos vínculos fa-
Plano Nacional de Promoção, Proteção e Defesa do Direito miliares, em relação à população com a mesma faixa de
de Crianças e Adolescentes à Convivência Familiar e Comu- renda, por município, por ano, observadas as variações de
nitária faz-se necessária a coleta de informações que pos- arranjos familiares, meio rural ou urbano e pertencimento
sibilitem o acompanhamento da implementação do Plano. étnico, inclusive comparando-se as médias nacionais e re-
Assim, os indicadores abaixo relacionados permitirão o gionais;

176
LEGISLAÇÃO

• Causas motivadores da retirada de crianças e ado- receberam formação em direitos humanos); por questões
lescentes do convívio familiar e comunitário, em relação à de abandono, maus tratos, violência doméstica, abuso e
população com a mesma faixa de renda, por ordem do a) exploração sexual, entre outros, ano a ano;
Juizado b)Conselho tutelar c) própria família, etc. por muni- • Tempo médio de permanência de crianças e adoles-
cípio, por ano, observadas as variações de arranjos familia- centes em acolhimento institucional, por município, por ano,
res, meio rural ou urbano e pertencimento étnico, inclusive inclusive comparando-se por idade, gênero, etnia, condição
comparando-se as médias nacionais e regionais. de saúde e, quando possível, pela renda familiar;
• Número de crianças e adolescentes fora do convívio • Tempo médio de permanência de crianças e adolescen-
familiar por questões de pobreza, em relação à quantidade tes em programas de famílias acolhedoras, família substituta,
de crianças que vivem em família na mesma faixa de ren- por município, por ano, inclusive comparando-se por idade,
da, por ano, observadas as variações de arranjos familia- gênero, etnia, condição de saúde e, quando possível, pela
res, meio rural ou urbano e pertencimento étnico, inclusive renda familiar;
comparando-se as médias nacionais e regionais. • Número de crianças e adolescentes reintegrados à fa-
mília de origem por município, por ano, em relação ao total
Diagnóstico da situação de crianças e adolescentes de crianças e adolescentes em acolhimento institucional e ou
em situação de adoção nacional e internacional. em Programas de Famílias Acolhedoras, inclusive comparan-
do-se por idade, gênero, etnia, condição de saúde e, quando
• Número de adoções de crianças maiores e adoles- possível, pela renda familiar;
centes, afrodescendentes e pertencentes a minorias étni- • Número de crianças e adolescentes reintegradas à fa-
cas, vivendo e convivendo com HIV/AIDS/AIDS em relação mília de origem que retornaram para acolhimento institucio-
ao número de crianças e adolescentes que esperam por nal ou aos Programas de Famílias Acolhedoras, por municí-
adoção nas mesmas condições, por município, por ano; pio, por ano, em relação ao total de crianças e adolescentes
• Número de crianças e adolescentes que esperam por reintegradas à família de origem, inclusive comparando-se
adoção: a) por gênero, b) etnia, c) condição de saúde, d) por idade, gênero, etnia, condição de saúde e, quando possí-
grupo de irmãos e) idade; vel, pela renda familiar;
• Número de adoções que tramitaram pelo cadastro • Considerando a demanda, qual a oferta de políticas pú-
blicas para crianças e adolescentes, em especial àquelas que
das Varas da Infância e Juventude em relação ao universo
estão privadas do direito à convivência familiar e comunitá-
de crianças e adolescentes “prontas para adoção”, por mu-
ria, por município, por ano.
nicípio, por ano;
• Número de entrega de bebês às autoridades judiciá-
Orçamento Público
rias para adoção, por município, por ano; observadas as
variações de arranjos familiares, meio rural ou urbano e
Valor destinado nos orçamentos do Município, do Esta-
pertencimento étnico, inclusive comparando-se as médias
do e da União, por ano, para implantação e implementação
nacionais e regionais; das ações do Plano Nacional de Proteção, Defesa e Garantia
• Considerando o universo de crianças entregues às au- do Direito da Criança e do Adolescente à Convivência Fami-
toridades judiciárias: a) quantas foram encaminhadas para liar e Comunitária.
acolhimento institucional; b) quantas foram encaminhadas
para acolhimento em família extensiva; c) quantas foram Plano de Ação
encaminhadas para programas de famílias acolhedoras; c)
quantas retornaram às suas famílias de origem; por muni- As propostas operacionais deste Plano estão organiza-
cípio, por ano, observadas as variações de arranjos familia- das em quatro eixos estratégicos articulados entre si: 1) Aná-
res, meio rural ou urbano e pertencimento étnico, inclusive lise da situação e sistemas de informação, 2) Atendimento, 3)
comparando-se as médias nacionais e regionais. Marcos normativos e regulatórios e 4) Mobilização, articula-
ção e participação. Os quadros a seguir são resultados de um
Diagnóstico de situação de operação de políticas esforço para propor ações em curto, médio e longo prazo,98
públicas (municipais, estaduais) de proteção às crian- almejando caminhar na direção de uma sociedade que de
ças, aos adolescentes e a suas famílias (assistência so- fato respeite o direito à convivência familiar e comunitária.
cial; saúde – física e mental; tratamentos ao uso e abuso
de drogas – lícitas e ilícitas; etc.) Eixo 1 – Análise da Situação e Sistemas de Informação

• Identificação e perfil das políticas em operação, por São propostos objetivos, ações e estratégias que enfatizam:
município, incluindo as alternativas previstas para as várias - Mapeamento quantitativo e qualitativo sobre família,
condições de vulnerabilidade da criança e do adolescente programas de atendimento e adoção;
e de suas famílias; - Realização e socialização de pesquisas nas esferas
• Metodologia assumida, no nível municipal, para ga- estaduais e municipais sobre convivência familiar e comu-
rantir a integração intersetorial dessas políticas; nitária;
• Informações sobre a quantidade e qualidade dos 98 Para efeito deste documento considerou-se a seguinte
trabalhadores das instituições acolhedoras de crianças e temporalidade: curto prazo – até 2007; médio prazo – de 2007 a 2010
adolescentes e de seu processo de formação (incluindo se e longo prazo – de 2010 a 2016.

177
LEGISLAÇÃO

- Levantamento de pesquisas existentes que auxiliem - Regulamentação e aplicação dos conceitos de pro-
na análise e indicação de critérios de qualidade do aten- visoriedade e excepcionalidade nos programas de Abrigo;
dimento; - Regulamentação dos procedimentos necessários ao
- Identificação de lacunas na oferta de dados dos Siste- reordenamento das instituições e seus programas junto
mas de Informação; aos Conselhos Municipais dos Direitos da Criança e do
- Implementação nacional do SIPIA nos seus módulos: ( Adolescente e de Assistência Social;
I) Registro de violações de direitos, (II) Medidas socioeduca- - Elaboração de parâmetros para definição do papel e
tivas; ( III) Cadastro de Adoções /InfoAdote e (IV) Cadastro função de educador social;
de Conselhos de Direitos e Tutelares; - Estabelecimento de parâmetros para as diferentes
- Implantação e implementação de Conselhos Tutelares modalidades de programas de Acolhimento Institucional e
que faltam no Brasil; para programas de Acolhimento Familiar;
- Elaboração de indicadores de monitoramento e ava- - Regulamentação dos programas de Acolhimento Fa-
liação; miliar;
- Aperfeiçoamento, articulação e integração entre os sis- - Garantia de igualdade, equidade e inclusão em pro-
temas de informação existentes; gramas de Acolhimento Institucional, Acolhimento Familiar
- Capacitação de atores estratégicos para operacionali- e Adoção;
zação dos sistemas propostos. - Aprimoramento de instrumentos legais de proteção
social que oferecem alternativas e a possibilidade do con-
Eixo 2 – Atendimento traditório à suspensão ou destituição do poder familiar;
- Regulamentação da legislação referente à Adoção;
São propostos objetivos, ações e estratégias que enfa- - Estabelecimento de parâmetros que garantam a le-
tizam: galidade dos procedimentos de Adoção nacional e inter-
- Ampliação e estruturação de programas de atendi- nacional.
mento sociofamiliar;
- Sistematização de metodologias participativas de tra- Eixo 4 – Mobilização, Articulação e Participação
balho com famílias e comunidade;
- Reordenamento institucional do sistema de atendi- São propostos objetivos que enfatizam:
mento com ênfase para os programas de abrigos e instru-
- Estabelecimento de estratégias de comunicação so-
mentalização para sua regulamentação;
cial para mobilização da sociedade e afirmação de novos
- Implementação de políticas e programas de acolhi-
valores;
mento familiar;
- Qualificação do trabalho da imprensa;
- Ampliação de programas de emancipação para ado-
- Identificação de pontos de contato e interfaces en-
lescentes e/ou jovens abrigados;
tre o Plano Nacional de Promoção, Defesa e Garantia do
- Construção de parâmetros de atendimento para pro-
Direito de Crianças e Adolescentes à Convivência Familiar
gramas de Acolhimento Institucional, e suas diferentes mo-
dalidades, e Acolhimento Familiar; e Comunitária, e os Parâmetros para a institucionalização e
- Sistematização e socialização de boas práticas no aten- fortalecimento do Sistema de Garantia de Direitos (Reso-
dimento às famílias, no acolhimento familiar, no Acolhimen- lução 105, CONANDA) e do Sistema Nacional de Atendi-
to Institucional e na adoção; mento Socioeducativo (SINASE) e com as demais políticas
- Fortalecimento e integração das ações governamen- públicas e programas governamentais, visando à integra-
tais (intersetorialidade) entre as diferentes políticas e pro- ção de ações;
gramas sociais; - Articulação e integração de ações entre as três esfe-
- Fortalecimento e integração entre os diferentes Con- ras de Poder;
selhos Setoriais; - Ampliação da participação da sociedade e do con-
- Aprimoramento dos procedimentos de adoção, in- trole social;
cluindo a implementação de cadastro nacional de preten- - Mobilização e articulação para a garantia da proviso-
dentes à adoção e de crianças e adolescentes passíveis de riedade, excepcionalidade do Acolhimento Institucional e
serem adotados; para o reordenamento dos Abrigos;
- Formação continuada em todos os níveis da federação, - Inclusão da temática da convivência familiar e comu-
com metodologias participativas, que promovam a mudan- nitária em cursos de formação de educadores, operadores
ça de paradigma necessária à transformação da cultura que do SGD, lideranças comunitárias e religiosas, dentre outros
apoia o direito a convivência familiar e comunitária. atores sociais;
- Inclusão da disciplina “direitos da criança e do ado-
Eixo 3 – Marcos Normativos e Regulatórios lescente” em programas de concursos públicos;
- Garantia de recursos financeiros e orçamentários para
São propostos objetivos, ações e estratégias que enfa- a realização deste Plano.
tizam:
- Aperfeiçoamento dos marcos normativos e regulató-
rios, para a efetivação da promoção, defesa e garantia do
direito à convivência familiar e comunitária.

178
LEGISLAÇÃO

Indicadores de Eficácia e Monitoramento Parágrafo único. A garantia de prioridade compreende:


I – atendimento preferencial imediato e individualizado
- Número de famílias atendidas em programas socioe- junto aos órgãos públicos e privados prestadores de servi-
ducativos da proteção social básica e em relação ao nú- ços à população;
mero de famílias na mesma faixa de renda na população II – preferência na formulação e na execução de políti-
brasileira; cas sociais públicas específicas;
- Proporção entre eventos de ruptura de vínculos fami- III – destinação privilegiada de recursos públicos nas
liares nos territórios atendidos pelos programas de prote- áreas relacionadas com a proteção ao idoso;
ção social básica e no total da população na mesma faixa IV – viabilização de formas alternativas de participação,
de renda; ocupação e convívio do idoso com as demais gerações;
- Número de crianças e adolescentes reintegrados à V – priorização do atendimento do idoso por sua pró-
família de origem por município, por ano; pria família, em detrimento do atendimento asilar, exceto
- Proporção entre o número de crianças e adolescen- dos que não a possuam ou careçam de condições de ma-
tes em Acolhimento Institucional e o número de crianças e nutenção da própria sobrevivência;
adolescentes em Acolhimento Familiar por município, por VI – capacitação e reciclagem dos recursos humanos
ano; nas áreas de geriatria e gerontologia e na prestação de ser-
- Tempo médio de permanência de crianças e adoles- viços aos idosos;
centes no Acolhimento Institucional e Familiar por municí- VII – estabelecimento de mecanismos que favoreçam
pio, por ano; a divulgação de informações de caráter educativo sobre os
- Número de adoções que tramitam regularmente no aspectos biopsicossociais de envelhecimento;
cadastro nacional; VIII – garantia de acesso à rede de serviços de saúde e
- Número de adoções tardias, inter-raciais, com defi- de assistência social locais.
ciência e afetados pelo vírus HIV/AIDS por estado e por IX – prioridade no recebimento da restituição do Im-
ano. posto de Renda. (Incluído pela Lei nº 11.765, de 2008).
Art. 4o Nenhum idoso será objeto de qualquer tipo de
negligência, discriminação, violência, crueldade ou opres-
são, e todo atentado aos seus direitos, por ação ou omis-
ESTATUTO DO IDOSO – Lei 10.741/2003 são, será punido na forma da lei.
§ 1o  É dever de todos prevenir a ameaça ou violação
aos direitos do idoso.
LEI Nº 10.741, DE 1º DE OUTUBRO DE 2003. § 2o  As obrigações previstas nesta Lei não excluem da
prevenção outras decorrentes dos princípios por ela ado-
Dispõe sobre o Estatuto do Idoso e dá outras provi- tados.
dências. Art. 5o  A inobservância das normas de prevenção im-
portará em responsabilidade à pessoa física ou jurídica nos
O PRESIDENTE DA REPÚBLICA Faço saber que o Con- termos da lei.
gresso Nacional decreta e eu sanciono a seguinte Lei: Art. 6o Todo cidadão tem o dever de comunicar à auto-
ridade competente qualquer forma de violação a esta Lei
TÍTULO I que tenha testemunhado ou de que tenha conhecimento.
Disposições Preliminares Art. 7o  Os Conselhos Nacional, Estaduais, do Distrito
Federal e Municipais do Idoso, previstos na Lei no 8.842, de 4 de
Art. 1o É instituído o Estatuto do Idoso, destinado a re- janeiro de 1994
, zelarão pelo cumprimento dos direitos do idoso,
gular os direitos assegurados às pessoas com idade igual definidos nesta Lei.
ou superior a 60 (sessenta) anos.
Art. 2o  O idoso goza de todos os direitos fundamen- TÍTULO II
tais inerentes à pessoa humana, sem prejuízo da proteção Dos Direitos Fundamentais
integral de que trata esta Lei, assegurando-se-lhe, por lei CAPÍTULO I
ou por outros meios, todas as oportunidades e facilidades, Do Direito à Vida
para preservação de sua saúde física e mental e seu aper-
feiçoamento moral, intelectual, espiritual e social, em con- Art. 8o O envelhecimento é um direito personalíssimo e
dições de liberdade e dignidade. a sua proteção um direito social, nos termos desta Lei e da
Art. 3o  É obrigação da família, da comunidade, da so- legislação vigente.
ciedade e do Poder Público assegurar ao idoso, com ab- Art. 9o É obrigação do Estado, garantir à pessoa idosa a
soluta prioridade, a efetivação do direito à vida, à saúde, à proteção à vida e à saúde, mediante efetivação de políticas
alimentação, à educação, à cultura, ao esporte, ao lazer, ao sociais públicas que permitam um envelhecimento saudá-
trabalho, à cidadania, à liberdade, à dignidade, ao respeito vel e em condições de dignidade.
e à convivência familiar e comunitária.

179
LEGISLAÇÃO

CAPÍTULO II IV – atendimento domiciliar, incluindo a internação,


Do Direito à Liberdade, ao Respeito e à Dignidade para a população que dele necessitar e esteja impossibi-
litada de se locomover, inclusive para idosos abrigados e
Art. 10.  É obrigação do Estado e da sociedade, asse- acolhidos por instituições públicas, filantrópicas ou sem
gurar à pessoa idosa a liberdade, o respeito e a dignidade, fins lucrativos e eventualmente conveniadas com o Poder
como pessoa humana e sujeito de direitos civis, políticos, Público, nos meios urbano e rural;
individuais e sociais, garantidos na Constituição e nas leis. V – reabilitação orientada pela geriatria e gerontolo-
§ 1o O direito à liberdade compreende, entre outros, os gia, para redução das sequelas decorrentes do agravo da
seguintes aspectos: saúde.
I – faculdade de ir, vir e estar nos logradouros públicos § 2o  Incumbe ao Poder Público fornecer aos idosos,
e espaços comunitários, ressalvadas as restrições legais; gratuitamente, medicamentos, especialmente os de uso
II – opinião e expressão; continuado, assim como próteses, órteses e outros recur-
III – crença e culto religioso; sos relativos ao tratamento, habilitação ou reabilitação.
IV – prática de esportes e de diversões; § 3o  É vedada a discriminação do idoso nos planos de
V – participação na vida familiar e comunitária; saúde pela cobrança de valores diferenciados em razão da
VI – participação na vida política, na forma da lei; idade.
VII – faculdade de buscar refúgio, auxílio e orientação. § 4o  Os idosos portadores de deficiência ou com limi-
§ 2o O direito ao respeito consiste na inviolabilidade da tação incapacitante terão atendimento especializado, nos
integridade física, psíquica e moral, abrangendo a preser- termos da lei.
vação da imagem, da identidade, da autonomia, de valores, § 5o  É vedado exigir o comparecimento do idoso en-
ideias e crenças, dos espaços e dos objetos pessoais. fermo perante os órgãos públicos, hipótese na qual será
§ 3o  É dever de todos zelar pela dignidade do idoso, admitido o seguinte procedimento:       (Incluído pela Lei nº
colocando-o a salvo de qualquer tratamento desumano, 12.896, de 2013)
violento, aterrorizante, vexatório ou constrangedor. I - quando de interesse do poder público, o agente
promoverá o contato necessário com o idoso em sua resi-
CAPÍTULO III
dência; ou(Incluído pela Lei nº 12.896, de 2013)
Dos Alimentos
II - quando de interesse do próprio idoso, este se fará
representar por procurador legalmente constituído.(Incluí-
Art. 11. Os alimentos serão prestados ao idoso na for-
do pela Lei nº 12.896, de 2013)
ma da lei civil.
§ 6o É assegurado ao idoso enfermo o atendimento do-
Art. 12. A obrigação alimentar é solidária, podendo o
miciliar pela perícia médica do Instituto Nacional do Seguro
idoso optar entre os prestadores.
Social - INSS, pelo serviço público de saúde ou pelo serviço
Art. 13.  As transações relativas a alimentos poderão
ser celebradas perante o Promotor de Justiça ou Defensor privado de saúde, contratado ou conveniado, que integre o
Público, que as referendará, e passarão a ter efeito de título Sistema Único de Saúde - SUS, para expedição do laudo de
executivo extrajudicial nos termos da lei processual civil. saúde necessário ao exercício de seus direitos sociais e de
(Redação dada pela Lei nº 11.737, de 2008) isenção tributária.       (Incluído pela Lei nº 12.896, de 2013)
Art. 14.  Se o idoso ou seus familiares não possuírem Art. 16.  Ao idoso internado ou em observação é as-
condições econômicas de prover o seu sustento, impõe-se segurado o direito a acompanhante, devendo o órgão de
ao Poder Público esse provimento, no âmbito da assistên- saúde proporcionar as condições adequadas para a sua
cia social. permanência em tempo integral, segundo o critério mé-
dico.
CAPÍTULO IV Parágrafo único. Caberá ao profissional de saúde res-
Do Direito à Saúde ponsável pelo tratamento conceder autorização para o
acompanhamento do idoso ou, no caso de impossibilida-
Art. 15.  É assegurada a atenção integral à saúde do de, justificá-la por escrito.
idoso, por intermédio do Sistema Único de Saúde – SUS, Art. 17. Ao idoso que esteja no domínio de suas facul-
garantindo-lhe o acesso universal e igualitário, em con- dades mentais é assegurado o direito de optar pelo trata-
junto articulado e contínuo das ações e serviços, para a mento de saúde que lhe for reputado mais favorável.
prevenção, promoção, proteção e recuperação da saúde, Parágrafo único. Não estando o idoso em condições de
incluindo a atenção especial às doenças que afetam prefe- proceder à opção, esta será feita:
rencialmente os idosos. I – pelo curador, quando o idoso for interditado;
§ 1o  A prevenção e a manutenção da saúde do idoso II – pelos familiares, quando o idoso não tiver curador
serão efetivadas por meio de: ou este não puder ser contactado em tempo hábil;
I – cadastramento da população idosa em base terri- III – pelo médico, quando ocorrer iminente risco de
torial; vida e não houver tempo hábil para consulta a curador ou
II – atendimento geriátrico e gerontológico em ambu- familiar;
latórios; IV – pelo próprio médico, quando não houver curador
III – unidades geriátricas de referência, com pessoal ou familiar conhecido, caso em que deverá comunicar o
especializado nas áreas de geriatria e gerontologia social; fato ao Ministério Público.

180
LEGISLAÇÃO

Art. 18.  As instituições de saúde devem atender aos CAPÍTULO VI


critérios mínimos para o atendimento às necessidades do Da Profissionalização e do Trabalho
idoso, promovendo o treinamento e a capacitação dos pro-
fissionais, assim como orientação a cuidadores familiares e Art. 26. O idoso tem direito ao exercício de atividade
grupos de auto-ajuda. profissional, respeitadas suas condições físicas, intelectuais
Art. 19.  Os casos de suspeita ou confirmação de vio- e psíquicas.
lência praticada contra idosos serão objeto de notificação Art. 27.  Na admissão do idoso em qualquer trabalho
compulsória pelos serviços de saúde públicos e privados à ou emprego, é vedada a discriminação e a fixação de limite
autoridade sanitária, bem como serão obrigatoriamente co- máximo de idade, inclusive para concursos, ressalvados os
municados por eles a quaisquer dos seguintes órgãos: (Re- casos em que a natureza do cargo o exigir.
dação dada pela Lei nº 12.461, de 2011) Parágrafo único. O primeiro critério de desempate em
I – autoridade policial; concurso público será a idade, dando-se preferência ao de
II – Ministério Público; idade mais elevada.
III – Conselho Municipal do Idoso; Art. 28. O Poder Público criará e estimulará programas
IV – Conselho Estadual do Idoso; de:
V – Conselho Nacional do Idoso. I – profissionalização especializada para os idosos,
§ 1o  Para os efeitos desta Lei, considera-se violência aproveitando seus potenciais e habilidades para atividades
contra o idoso qualquer ação ou omissão praticada em local regulares e remuneradas;
público ou privado que lhe cause morte, dano ou sofrimento II – preparação dos trabalhadores para a aposentado-
físico ou psicológico. (Incluído pela Lei nº 12.461, de 2011) ria, com antecedência mínima de 1 (um) ano, por meio de
§ 2o  Aplica-se, no que couber, à notificação compulsória estímulo a novos projetos sociais, conforme seus interes-
prevista no caput deste artigo, o disposto na Lei no 6.259, de 30 de ses, e de esclarecimento sobre os direitos sociais e de ci-
outubro de 1975
. (Incluído pela Lei nº 12.461, de 2011) dadania;
III – estímulo às empresas privadas para admissão de
CAPÍTULO V
idosos ao trabalho.
Da Educação, Cultura, Esporte e Lazer
CAPÍTULO VII
Art. 20. O idoso tem direito a educação, cultura, esporte,
Da Previdência Social
lazer, diversões, espetáculos, produtos e serviços que respei-
tem sua peculiar condição de idade.
Art. 29.  Os benefícios de aposentadoria e pensão do
Art. 21. O Poder Público criará oportunidades de acesso
Regime Geral da Previdência Social observarão, na sua con-
do idoso à educação, adequando currículos, metodologias
e material didático aos programas educacionais a ele des- cessão, critérios de cálculo que preservem o valor real dos
tinados. salários sobre os quais incidiram contribuição, nos termos
§ 1o Os cursos especiais para idosos incluirão conteúdo da legislação vigente.
relativo às técnicas de comunicação, computação e demais Parágrafo único. Os valores dos benefícios em manu-
avanços tecnológicos, para sua integração à vida moderna. tenção serão reajustados na mesma data de reajuste do
§ 2o Os idosos participarão das comemorações de cará- salário-mínimo, pro rata, de acordo com suas respectivas
ter cívico ou cultural, para transmissão de conhecimentos e datas de início ou do seu último reajustamento, com base
vivências às demais gerações, no sentido da preservação da em percentual definido em regulamento, observados os
memória e da identidade culturais. critérios estabelecidos pela Lei no 8.213, de 24 de julho de 1991.
Art. 22.  Nos currículos mínimos dos diversos níveis de Art. 30.  A perda da condição de segurado não será
ensino formal serão inseridos conteúdos voltados ao pro- considerada para a concessão da aposentadoria por ida-
cesso de envelhecimento, ao respeito e à valorização do ido- de, desde que a pessoa conte com, no mínimo, o tempo
so, de forma a eliminar o preconceito e a produzir conheci- de contribuição correspondente ao exigido para efeito de
mentos sobre a matéria. carência na data de requerimento do benefício.
Art. 23. A participação dos idosos em atividades cultu- Parágrafo único. O cálculo do valor do benefício pre-
rais e de lazer será proporcionada mediante descontos de visto no caput observará o disposto no caput e § 2o do art.
pelo menos 50% (cinquenta por cento) nos ingressos para 3o da Lei no 9.876, de 26 de novembro de 1999
, ou, não havendo salários-de-
eventos artísticos, culturais, esportivos e de lazer, bem como contribuição recolhidos a partir da competência de julho
o acesso preferencial aos respectivos locais. de 1994, o disposto no art. 35 da Lei no 8.213, de 1991.
Art. 24. Os meios de comunicação manterão espaços ou Art. 31. O pagamento de parcelas relativas a benefícios,
horários especiais voltados aos idosos, com finalidade infor- efetuado com atraso por responsabilidade da Previdência
mativa, educativa, artística e cultural, e ao público sobre o Social, será atualizado pelo mesmo índice utilizado para os
processo de envelhecimento. reajustamentos dos benefícios do Regime Geral de Previ-
Art. 25. O Poder Público apoiará a criação de universi- dência Social, verificado no período compreendido entre
dade aberta para as pessoas idosas e incentivará a publica- o mês que deveria ter sido pago e o mês do efetivo paga-
ção de livros e periódicos, de conteúdo e padrão editorial mento.
adequados ao idoso, que facilitem a leitura, considerada a Art. 32. O Dia Mundial do Trabalho, 1o de Maio, é a da-
natural redução da capacidade visual. ta-base dos aposentados e pensionistas.

181
LEGISLAÇÃO

CAPÍTULO VIII I - reserva de pelo menos 3% (três por cento) das uni-
Da Assistência Social dades habitacionais residenciais para atendimento aos ido-
sos;      (Redação dada pela Lei nº 12.418, de 2011)
Art. 33.  A assistência social aos idosos será prestada, II – implantação de equipamentos urbanos comunitá-
de forma articulada, conforme os princípios e diretrizes rios voltados ao idoso;
previstos na Lei Orgânica da Assistência Social, na Política III – eliminação de barreiras arquitetônicas e urbanísti-
Nacional do Idoso, no Sistema Único de Saúde e demais cas, para garantia de acessibilidade ao idoso;
normas pertinentes. IV – critérios de financiamento compatíveis com os
Art. 34.  Aos idosos, a partir de 65 (sessenta e cinco) rendimentos de aposentadoria e pensão.
anos, que não possuam meios para prover sua subsistên- Parágrafo único.  As unidades residenciais reservadas
cia, nem de tê-la provida por sua família, é assegurado o para atendimento a idosos devem situar-se, preferen-
benefício mensal de 1 (um) salário-mínimo, nos termos da cialmente, no pavimento térreo.       (Incluído pela Lei nº
Lei Orgânica da Assistência Social – Loas.       (Vide Decreto 12.419, de 2011)
nº 6.214, de 2007)
Parágrafo único. O benefício já concedido a qualquer
CAPÍTULO X
membro da família nos termos do caput não será compu-
tado para os fins do cálculo da renda familiar per capita a Do Transporte
que se refere a Loas.
Art. 35. Todas as entidades de longa permanência, ou Art. 39. Aos maiores de 65 (sessenta e cinco) anos fica
casa-lar, são obrigadas a firmar contrato de prestação de assegurada a gratuidade dos transportes coletivos públi-
serviços com a pessoa idosa abrigada.  cos urbanos e semi-urbanos, exceto nos serviços seletivos
§ 1o  No caso de entidades filantrópicas, ou casa-lar, é e especiais, quando prestados paralelamente aos serviços
facultada a cobrança de participação do idoso no custeio regulares.
da entidade. § 1o  Para ter acesso à gratuidade, basta que o idoso
§ 2o O Conselho Municipal do Idoso ou o Conselho Mu- apresente qualquer documento pessoal que faça prova de
nicipal da Assistência Social estabelecerá a forma de par- sua idade.
ticipação prevista no § 1o, que não poderá exceder a 70% § 2o  Nos veículos de transporte coletivo de que trata
(setenta por cento) de qualquer benefício previdenciário este artigo, serão reservados 10% (dez por cento) dos as-
ou de assistência social percebido pelo idoso. sentos para os idosos, devidamente identificados com a
§ 3o Se a pessoa idosa for incapaz, caberá a seu repre- placa de reservado preferencialmente para idosos.
sentante legal firmar o contrato a que se refere o caput des- § 3o No caso das pessoas compreendidas na faixa etária
te artigo. entre 60 (sessenta) e 65 (sessenta e cinco) anos, ficará a
Art. 36. O acolhimento de idosos em situação de risco
critério da legislação local dispor sobre as condições para
social, por adulto ou núcleo familiar, caracteriza a depen-
exercício da gratuidade nos meios de transporte previstos
dência econômica, para os efeitos legais.      (Vigência)
no caput deste artigo.
CAPÍTULO IX Art. 40. No sistema de transporte coletivo interestadual
Da Habitação observar-se-á, nos termos da legislação específica:      (Re-
gulamento)     (Vide Decreto nº 5.934, de 2006)
Art. 37. O idoso tem direito a moradia digna, no seio da I – a reserva de 2 (duas) vagas gratuitas por veículo
família natural ou substituta, ou desacompanhado de seus para idosos com renda igual ou inferior a 2 (dois) salários-
familiares, quando assim o desejar, ou, ainda, em institui- mínimos;
ção pública ou privada. II – desconto de 50% (cinquenta por cento), no mínimo,
§ 1o  A assistência integral na modalidade de entidade no valor das passagens, para os idosos que excederem as
de longa permanência será prestada quando verificada vagas gratuitas, com renda igual ou inferior a 2 (dois) salá-
inexistência de grupo familiar, casa-lar, abandono ou ca- rios-mínimos.
rência de recursos financeiros próprios ou da família. Parágrafo único. Caberá aos órgãos competentes defi-
§ 2o Toda instituição dedicada ao atendimento ao ido- nir os mecanismos e os critérios para o exercício dos direi-
so fica obrigada a manter identificação externa visível, sob tos previstos nos incisos I e II.
pena de interdição, além de atender toda a legislação per- Art. 41. É assegurada a reserva, para os idosos, nos ter-
tinente. mos da lei local, de 5% (cinco por cento) das vagas nos
§ 3o  As instituições que abrigarem idosos são obriga-
estacionamentos públicos e privados, as quais deverão ser
das a manter padrões de habitação compatíveis com as
posicionadas de forma a garantir a melhor comodidade ao
necessidades deles, bem como provê-los com alimentação
idoso.
regular e higiene indispensáveis às normas sanitárias e com
Art. 42. São asseguradas a prioridade e a segurança do
estas condizentes, sob as penas da lei.
Art. 38. Nos programas habitacionais, públicos ou sub- idoso nos procedimentos de embarque e desembarque
sidiados com recursos públicos, o idoso goza de prioridade nos veículos do sistema de transporte coletivo.       (Reda-
na aquisição de imóvel para moradia própria, observado o ção dada pela Lei nº 12.899, de 2013)
seguinte:

182
LEGISLAÇÃO

TÍTULO III CAPÍTULO II


Das Medidas de Proteção Das Entidades de Atendimento ao Idoso
CAPÍTULO I
Das Disposições Gerais Art. 48. As entidades de atendimento são responsáveis
pela manutenção das próprias unidades, observadas as
Art. 43. As medidas de proteção ao idoso são aplicáveis normas de planejamento e execução emanadas do órgão
sempre que os direitos reconhecidos nesta Lei forem amea- competente da Política Nacional do Idoso, conforme a Lei
çados ou violados: no8.842, de 1994.
I – por ação ou omissão da sociedade ou do Estado; Parágrafo único. As entidades governamentais e não-
II – por falta, omissão ou abuso da família, curador ou governamentais de assistência ao idoso ficam sujeitas à
entidade de atendimento; inscrição de seus programas, junto ao órgão competente
III – em razão de sua condição pessoal. da Vigilância Sanitária e Conselho Municipal da Pessoa Ido-
sa, e em sua falta, junto ao Conselho Estadual ou Nacional
CAPÍTULO II
da Pessoa Idosa, especificando os regimes de atendimento,
Das Medidas Específicas de Proteção
observados os seguintes requisitos:
I – oferecer instalações físicas em condições adequadas
Art. 44. As medidas de proteção ao idoso previstas nes-
de habitabilidade, higiene, salubridade e segurança;
ta Lei poderão ser aplicadas, isolada ou cumulativamente, e
levarão em conta os fins sociais a que se destinam e o forta- II – apresentar objetivos estatutários e plano de traba-
lecimento dos vínculos familiares e comunitários. lho compatíveis com os princípios desta Lei;
Art. 45. Verificada qualquer das hipóteses previstas no art. III – estar regularmente constituída;
43, o Ministério Público ou o Poder Judiciário, a requerimento IV – demonstrar a idoneidade de seus dirigentes.
daquele, poderá determinar, dentre outras, as seguintes medidas: Art. 49. As entidades que desenvolvam programas de
I – encaminhamento à família ou curador, mediante ter- institucionalização de longa permanência adotarão os se-
mo de responsabilidade; guintes princípios:
II – orientação, apoio e acompanhamento temporários; I – preservação dos vínculos familiares;
III – requisição para tratamento de sua saúde, em regime II – atendimento personalizado e em pequenos grupos;
ambulatorial, hospitalar ou domiciliar; III – manutenção do idoso na mesma instituição, salvo
IV – inclusão em programa oficial ou comunitário de em caso de força maior;
auxílio, orientação e tratamento a usuários dependentes de IV – participação do idoso nas atividades comunitárias,
drogas lícitas ou ilícitas, ao próprio idoso ou à pessoa de sua de caráter interno e externo;
convivência que lhe cause perturbação; V – observância dos direitos e garantias dos idosos;
V – abrigo em entidade; VI – preservação da identidade do idoso e oferecimen-
VI – abrigo temporário. to de ambiente de respeito e dignidade.
Parágrafo único. O dirigente de instituição prestadora
TÍTULO IV de atendimento ao idoso responderá civil e criminalmente
Da Política de Atendimento ao Idoso pelos atos que praticar em detrimento do idoso, sem pre-
CAPÍTULO I juízo das sanções administrativas.
Disposições Gerais Art. 50. Constituem obrigações das entidades de aten-
dimento:
Art. 46. A política de atendimento ao idoso far-se-á por I – celebrar contrato escrito de prestação de serviço
meio do conjunto articulado de ações governamentais e
com o idoso, especificando o tipo de atendimento, as obri-
não-governamentais da União, dos Estados, do Distrito Fe-
gações da entidade e prestações decorrentes do contrato,
deral e dos Municípios.
com os respectivos preços, se for o caso;
Art. 47. São linhas de ação da política de atendimento:
II – observar os direitos e as garantias de que são titu-
I – políticas sociais básicas, previstas na Lei no 8.842, de 4 de
janeiro de 1994; lares os idosos;
II – políticas e programas de assistência social, em cará- III – fornecer vestuário adequado, se for pública, e ali-
ter supletivo, para aqueles que necessitarem; mentação suficiente;
III – serviços especiais de prevenção e atendimento IV – oferecer instalações físicas em condições adequa-
às vítimas de negligência, maus-tratos, exploração, abuso, das de habitabilidade;
crueldade e opressão; V – oferecer atendimento personalizado;
IV – serviço de identificação e localização de parentes ou VI – diligenciar no sentido da preservação dos vínculos
responsáveis por idosos abandonados em hospitais e insti- familiares;
tuições de longa permanência; VII – oferecer acomodações apropriadas para recebi-
V – proteção jurídico-social por entidades de defesa dos mento de visitas;
direitos dos idosos; VIII – proporcionar cuidados à saúde, conforme a ne-
VI – mobilização da opinião pública no sentido da par- cessidade do idoso;
ticipação dos diversos segmentos da sociedade no atendi- IX – promover atividades educacionais, esportivas, cul-
mento do idoso. turais e de lazer;

183
LEGISLAÇÃO

X – propiciar assistência religiosa àqueles que deseja- § 1o Havendo danos aos idosos abrigados ou qualquer
rem, de acordo com suas crenças; tipo de fraude em relação ao programa, caberá o afasta-
XI – proceder a estudo social e pessoal de cada caso; mento provisório dos dirigentes ou a interdição da unidade
XII – comunicar à autoridade competente de saúde e a suspensão do programa.
toda ocorrência de idoso portador de doenças infectocon- § 2o A suspensão parcial ou total do repasse de verbas
tagiosas; públicas ocorrerá quando verificada a má aplicação ou des-
XIII – providenciar ou solicitar que o Ministério Público vio de finalidade dos recursos.
requisite os documentos necessários ao exercício da cida- § 3o Na ocorrência de infração por entidade de atendi-
dania àqueles que não os tiverem, na forma da lei; mento, que coloque em risco os direitos assegurados nesta
XIV – fornecer comprovante de depósito dos bens mó- Lei, será o fato comunicado ao Ministério Público, para as
veis que receberem dos idosos; providências cabíveis, inclusive para promover a suspensão
XV – manter arquivo de anotações onde constem data das atividades ou dissolução da entidade, com a proibição
e circunstâncias do atendimento, nome do idoso, respon- de atendimento a idosos a bem do interesse público, sem
sável, parentes, endereços, cidade, relação de seus perten-
prejuízo das providências a serem tomadas pela Vigilância
ces, bem como o valor de contribuições, e suas alterações,
Sanitária.
se houver, e demais dados que possibilitem sua identifica-
ção e a individualização do atendimento; § 4o Na aplicação das penalidades, serão consideradas
XVI – comunicar ao Ministério Público, para as provi- a natureza e a gravidade da infração cometida, os danos
dências cabíveis, a situação de abandono moral ou material que dela provierem para o idoso, as circunstâncias agra-
por parte dos familiares; vantes ou atenuantes e os antecedentes da entidade.
XVII – manter no quadro de pessoal profissionais com
formação específica. CAPÍTULO IV
Art. 51. As instituições filantrópicas ou sem fins lucra- Das Infrações Administrativas
tivos prestadoras de serviço ao idoso terão direito à assis-
tência judiciária gratuita. Art. 56. Deixar a entidade de atendimento de cumprir
as determinações do art. 50 desta Lei:
CAPÍTULO III Pena – multa de R$ 500,00 (quinhentos reais) a R$
Da Fiscalização das Entidades de Atendimento 3.000,00 (três mil reais), se o fato não for caracterizado
como crime, podendo haver a interdição do estabeleci-
Art. 52. As entidades governamentais e não-governa- mento até que sejam cumpridas as exigências legais.
mentais de atendimento ao idoso serão fiscalizadas pelos Parágrafo único. No caso de interdição do estabeleci-
Conselhos do Idoso, Ministério Público, Vigilância Sanitária mento de longa permanência, os idosos abrigados serão
e outros previstos em lei.
transferidos para outra instituição, a expensas do estabele-
Art. 53. O art. 7o da Lei no 8.842, de 1994, passa a vigorar com
cimento interditado, enquanto durar a interdição.
a seguinte redação:
“Art. 7o  Compete aos Conselhos de que trata o art. Art. 57. Deixar o profissional de saúde ou o responsável
6 desta Lei a supervisão, o acompanhamento, a fiscaliza-
o  por estabelecimento de saúde ou instituição de longa per-
ção e a avaliação da política nacional do idoso, no âmbito manência de comunicar à autoridade competente os casos
das respectivas instâncias político-administrativas.” (NR) de crimes contra idoso de que tiver conhecimento:
Art. 54. Será dada publicidade das prestações de con- Pena – multa de R$ 500,00 (quinhentos reais) a R$
tas dos recursos públicos e privados recebidos pelas enti- 3.000,00 (três mil reais), aplicada em dobro no caso de re-
dades de atendimento. incidência.
Art. 55. As entidades de atendimento que descumpri- Art. 58. Deixar de cumprir as determinações desta Lei
rem as determinações desta Lei ficarão sujeitas, sem pre- sobre a prioridade no atendimento ao idoso:
juízo da responsabilidade civil e criminal de seus dirigentes Pena – multa de R$ 500,00 (quinhentos reais) a R$
ou prepostos, às seguintes penalidades, observado o devi- 1.000,00 (um mil reais) e multa civil a ser estipulada pelo
do processo legal: juiz, conforme o dano sofrido pelo idoso.
I – as entidades governamentais:
a) advertência; CAPÍTULO V
b) afastamento provisório de seus dirigentes; Da Apuração Administrativa de Infração às 
c) afastamento definitivo de seus dirigentes; Normas de Proteção ao Idoso
d) fechamento de unidade ou interdição de programa;
II – as entidades não-governamentais:
Art. 59. Os valores monetários expressos no Capítulo IV
a) advertência;
b) multa; serão atualizados anualmente, na forma da lei.
c) suspensão parcial ou total do repasse de verbas pú- Art. 60. O procedimento para a imposição de penali-
blicas; dade administrativa por infração às normas de proteção ao
d) interdição de unidade ou suspensão de programa; idoso terá início com requisição do Ministério Público ou
e) proibição de atendimento a idosos a bem do inte- auto de infração elaborado por servidor efetivo e assinado,
resse público. se possível, por duas testemunhas.

184
LEGISLAÇÃO

§ 1o No procedimento iniciado com o auto de infração § 3o  Antes de aplicar qualquer das medidas, a autori-
poderão ser usadas fórmulas impressas, especificando-se a dade judiciária poderá fixar prazo para a remoção das irre-
natureza e as circunstâncias da infração. gularidades verificadas. Satisfeitas as exigências, o processo
§ 2o Sempre que possível, à verificação da infração se- será extinto, sem julgamento do mérito.
guir-se-á a lavratura do auto, ou este será lavrado dentro § 4o A multa e a advertência serão impostas ao dirigente
de 24 (vinte e quatro) horas, por motivo justificado. da entidade ou ao responsável pelo programa de atendi-
Art. 61. O autuado terá prazo de 10 (dez) dias para a mento.
apresentação da defesa, contado da data da intimação,
que será feita: TÍTULO V
I – pelo autuante, no instrumento de autuação, quando Do Acesso à Justiça
for lavrado na presença do infrator; CAPÍTULO I
II – por via postal, com aviso de recebimento. Disposições Gerais
Art. 62. Havendo risco para a vida ou à saúde do ido-
so, a autoridade competente aplicará à entidade de aten- Art. 69. Aplica-se, subsidiariamente, às disposições des-
dimento as sanções regulamentares, sem prejuízo da ini- te Capítulo, o procedimento sumário previsto no Código de
ciativa e das providências que vierem a ser adotadas pelo Processo Civil, naquilo que não contrarie os prazos previs-
Ministério Público ou pelas demais instituições legitimadas tos nesta Lei.
para a fiscalização. Art. 70. O Poder Público poderá criar varas especializa-
Art. 63. Nos casos em que não houver risco para a vida das e exclusivas do idoso.
ou a saúde da pessoa idosa abrigada, a autoridade com- Art. 71. É assegurada prioridade na tramitação dos pro-
petente aplicará à entidade de atendimento as sanções re- cessos e procedimentos e na execução dos atos e diligên-
gulamentares, sem prejuízo da iniciativa e das providências cias judiciais em que figure como parte ou interveniente
que vierem a ser adotadas pelo Ministério Público ou pelas pessoa com idade igual ou superior a 60 (sessenta) anos,
demais instituições legitimadas para a fiscalização. em qualquer instância.
§ 1o  O interessado na obtenção da prioridade a que
CAPÍTULO VI alude este artigo, fazendo prova de sua idade, requererá o
Da Apuração Judicial de Irregularidades em Entidade benefício à autoridade judiciária competente para decidir o
feito, que determinará as providências a serem cumpridas,
de Atendimento
anotando-se essa circunstância em local visível nos autos
do processo.
Art. 64. Aplicam-se, subsidiariamente, ao procedimen-
§ 2o  A prioridade não cessará com a morte do benefi-
to administrativo de que trata este Capítulo as disposições
ciado, estendendo-se em favor do cônjuge supérstite, com-
das Leis nos 6.437, de 20 de agosto de 1977, e 9.784, de 29 de janeiro de
panheiro ou companheira, com união estável, maior de 60
1999.
(sessenta) anos.
Art. 65. O procedimento de apuração de irregularida-
§ 3o  A prioridade se estende aos processos e procedi-
de em entidade governamental e não-governamental de
mentos na Administração Pública, empresas prestadoras de
atendimento ao idoso terá início mediante petição funda- serviços públicos e instituições financeiras, ao atendimento
mentada de pessoa interessada ou iniciativa do Ministério preferencial junto à Defensoria Publica da União, dos Esta-
Público. dos e do Distrito Federal em relação aos Serviços de Assis-
Art. 66. Havendo motivo grave, poderá a autoridade ju- tência Judiciária.
diciária, ouvido o Ministério Público, decretar liminarmente § 4o  Para o atendimento prioritário será garantido ao
o afastamento provisório do dirigente da entidade ou ou- idoso o fácil acesso aos assentos e caixas, identificados com
tras medidas que julgar adequadas, para evitar lesão aos a destinação a idosos em local visível e caracteres legíveis.
direitos do idoso, mediante decisão fundamentada.
Art. 67.  O dirigente da entidade será citado para, no CAPÍTULO II
prazo de 10 (dez) dias, oferecer resposta escrita, podendo Do Ministério Público
juntar documentos e indicar as provas a produzir.
Art. 68. Apresentada a defesa, o juiz procederá na con- Art. 72. (VETADO)
formidade do art. 69 ou, se necessário, designará audiência Art. 73. As funções do Ministério Público, previstas nesta
de instrução e julgamento, deliberando sobre a necessida- Lei, serão exercidas nos termos da respectiva Lei Orgânica.
de de produção de outras provas. Art. 74. Compete ao Ministério Público:
§ 1o Salvo manifestação em audiência, as partes e o Mi- I – instaurar o inquérito civil e a ação civil pública para
nistério Público terão 5 (cinco) dias para oferecer alegações a proteção dos direitos e interesses difusos ou coletivos, in-
finais, decidindo a autoridade judiciária em igual prazo. dividuais indisponíveis e individuais homogêneos do idoso;
§ 2o Em se tratando de afastamento provisório ou defi- II – promover e acompanhar as ações de alimentos, de
nitivo de dirigente de entidade governamental, a autorida- interdição total ou parcial, de designação de curador espe-
de judiciária oficiará a autoridade administrativa imediata- cial, em circunstâncias que justifiquem a medida e oficiar
mente superior ao afastado, fixando-lhe prazo de 24 (vinte em todos os feitos em que se discutam os direitos de idosos
e quatro) horas para proceder à substituição. em condições de risco;

185
LEGISLAÇÃO

III – atuar como substituto processual do idoso em si- Art. 79. Regem-se pelas disposições desta Lei as ações
tuação de risco, conforme o disposto no art. 43 desta Lei; de responsabilidade por ofensa aos direitos assegurados
IV – promover a revogação de instrumento procurató- ao idoso, referentes à omissão ou ao oferecimento insatis-
rio do idoso, nas hipóteses previstas no art. 43 desta Lei, fatório de:
quando necessário ou o interesse público justificar; I – acesso às ações e serviços de saúde;
V – instaurar procedimento administrativo e, para ins- II – atendimento especializado ao idoso portador de
truí-lo: deficiência ou com limitação incapacitante;
a) expedir notificações, colher depoimentos ou esclare-
III – atendimento especializado ao idoso portador de
cimentos e, em caso de não comparecimento injustificado
doença infectocontagiosa;
da pessoa notificada, requisitar condução coercitiva, inclu-
sive pela Polícia Civil ou Militar; IV – serviço de assistência social visando ao amparo do
b) requisitar informações, exames, perícias e documen- idoso.
tos de autoridades municipais, estaduais e federais, da ad- Parágrafo único. As hipóteses previstas neste artigo não
ministração direta e indireta, bem como promover inspe- excluem da proteção judicial outros interesses difusos, cole-
ções e diligências investigatórias; tivos, individuais indisponíveis ou homogêneos, próprios do
c) requisitar informações e documentos particulares de idoso, protegidos em lei.
instituições privadas; Art. 80. As ações previstas neste Capítulo serão propos-
VI – instaurar sindicâncias, requisitar diligências investi- tas no foro do domicílio do idoso, cujo juízo terá compe-
gatórias e a instauração de inquérito policial, para a apura- tência absoluta para processar a causa, ressalvadas as com-
ção de ilícitos ou infrações às normas de proteção ao idoso; petências da Justiça Federal e a competência originária dos
VII – zelar pelo efetivo respeito aos direitos e garantias Tribunais Superiores.
legais assegurados ao idoso, promovendo as medidas judi- Art. 81. Para as ações cíveis fundadas em interesses di-
ciais e extrajudiciais cabíveis;
fusos, coletivos, individuais indisponíveis ou homogêneos,
VIII – inspecionar as entidades públicas e particulares
consideram-se legitimados, concorrentemente:
de atendimento e os programas de que trata esta Lei, ado-
tando de pronto as medidas administrativas ou judiciais I – o Ministério Público;
necessárias à remoção de irregularidades porventura ve- II – a União, os Estados, o Distrito Federal e os Municí-
rificadas; pios;
IX – requisitar força policial, bem como a colaboração III – a Ordem dos Advogados do Brasil;
dos serviços de saúde, educacionais e de assistência social, IV – as associações legalmente constituídas há pelo me-
públicos, para o desempenho de suas atribuições; nos 1 (um) ano e que incluam entre os fins institucionais a
X – referendar transações envolvendo interesses e di- defesa dos interesses e direitos da pessoa idosa, dispensada
reitos dos idosos previstos nesta Lei. a autorização da assembleia, se houver prévia autorização
§ 1o A legitimação do Ministério Público para as ações estatutária.
cíveis previstas neste artigo não impede a de terceiros, nas § 1o  Admitir-se-á litisconsórcio facultativo entre os Mi-
mesmas hipóteses, segundo dispuser a lei. nistérios Públicos da União e dos Estados na defesa dos in-
§ 2o As atribuições constantes deste artigo não excluem
teresses e direitos de que cuida esta Lei.
outras, desde que compatíveis com a finalidade e atribui-
§ 2o  Em caso de desistência ou abandono da ação por
ções do Ministério Público.
§ 3o O representante do Ministério Público, no exercício associação legitimada, o Ministério Público ou outro legiti-
de suas funções, terá livre acesso a toda entidade de aten- mado deverá assumir a titularidade ativa.
dimento ao idoso. Art. 82. Para defesa dos interesses e direitos protegidos
Art. 75.  Nos processos e procedimentos em que não por esta Lei, são admissíveis todas as espécies de ação per-
for parte, atuará obrigatoriamente o Ministério Público na tinentes.
defesa dos direitos e interesses de que cuida esta Lei, hi- Parágrafo único. Contra atos ilegais ou abusivos de au-
póteses em que terá vista dos autos depois das partes, po- toridade pública ou agente de pessoa jurídica no exercício
dendo juntar documentos, requerer diligências e produção de atribuições de Poder Público, que lesem direito líquido e
de outras provas, usando os recursos cabíveis. certo previsto nesta Lei, caberá ação mandamental, que se
Art. 76. A intimação do Ministério Público, em qualquer regerá pelas normas da lei do mandado de segurança.
caso, será feita pessoalmente. Art. 83. Na ação que tenha por objeto o cumprimento
Art. 77. A falta de intervenção do Ministério Público de obrigação de fazer ou não-fazer, o juiz concederá a tu-
acarreta a nulidade do feito, que será declarada de ofício
tela específica da obrigação ou determinará providências
pelo juiz ou a requerimento de qualquer interessado.
que assegurem o resultado prático equivalente ao adimple-
CAPÍTULO III mento.
Da Proteção Judicial dos Interesses Difusos, Coletivos e § 1o  Sendo relevante o fundamento da demanda e ha-
Individuais Indisponíveis ou Homogêneos vendo justificado receio de ineficácia do provimento final, é
lícito ao juiz conceder a tutela liminarmente ou após justi-
Art. 78. As manifestações processuais do representante ficação prévia, na forma do art. 273 do Código de Processo
do Ministério Público deverão ser fundamentadas. Civil.

186
LEGISLAÇÃO

§ 2o O juiz poderá, na hipótese do § 1o ou na sentença, § 2o Os autos do inquérito civil ou as peças de informa-
impor multa diária ao réu, independentemente do pedido ção arquivados serão remetidos, sob pena de se incorrer
do autor, se for suficiente ou compatível com a obrigação, em falta grave, no prazo de 3 (três) dias, ao Conselho Su-
fixando prazo razoável para o cumprimento do preceito. perior do Ministério Público ou à Câmara de Coordenação
§ 3o A multa só será exigível do réu após o trânsito em e Revisão do Ministério Público.
julgado da sentença favorável ao autor, mas será devida § 3o  Até que seja homologado ou rejeitado o arquiva-
desde o dia em que se houver configurado. mento, pelo Conselho Superior do Ministério Público ou
Art. 84. Os valores das multas previstas nesta Lei rever- por Câmara de Coordenação e Revisão do Ministério Públi-
terão ao Fundo do Idoso, onde houver, ou na falta deste, co, as associações legitimadas poderão apresentar razões
ao Fundo Municipal de Assistência Social, ficando vincula- escritas ou documentos, que serão juntados ou anexados
dos ao atendimento ao idoso. às peças de informação.
Parágrafo único. As multas não recolhidas até 30 (trin- § 4o  Deixando o Conselho Superior ou a Câmara de
ta) dias após o trânsito em julgado da decisão serão exi- Coordenação e Revisão do Ministério Público de homolo-
gidas por meio de execução promovida pelo Ministério gar a promoção de arquivamento, será designado outro
Público, nos mesmos autos, facultada igual iniciativa aos membro do Ministério Público para o ajuizamento da ação.
demais legitimados em caso de inércia daquele.
Art. 85. O juiz poderá conferir efeito suspensivo aos re- TÍTULO VI
cursos, para evitar dano irreparável à parte. Dos Crimes
Art. 86. Transitada em julgado a sentença que impuser
CAPÍTULO I
condenação ao Poder Público, o juiz determinará a remes-
Disposições Gerais
sa de peças à autoridade competente, para apuração da
responsabilidade civil e administrativa do agente a que se
Art. 93.  Aplicam-se subsidiariamente, no que couber,
atribua a ação ou omissão.
Art. 87. Decorridos 60 (sessenta) dias do trânsito em as disposições da Lei no 7.347, de 24 de julho de 1985.
julgado da sentença condenatória favorável ao idoso sem Art. 94. Aos crimes previstos nesta Lei, cuja pena má-
que o autor lhe promova a execução, deverá fazê-lo o Mi- xima privativa de liberdade não ultrapasse 4 (quatro) anos,
nistério Público, facultada, igual iniciativa aos demais legi- aplica-se o procedimento previsto na Lei no 9.099, de 26 de setembro
timados, como assistentes ou assumindo o pólo ativo, em
de 1995
, e, subsidiariamente, no que couber, as disposições
caso de inércia desse órgão. do Código Penal e do Código de Processo Penal. (Vide ADI
Art. 88. Nas ações de que trata este Capítulo, não have- 3.096-5 - STF)
rá adiantamento de custas, emolumentos, honorários peri-
ciais e quaisquer outras despesas. CAPÍTULO II
Parágrafo único. Não se imporá sucumbência ao Mi- Dos Crimes em Espécie
nistério Público.
Art. 89. Qualquer pessoa poderá, e o servidor deverá, Art. 95. Os crimes definidos nesta Lei são de ação pe-
provocar a iniciativa do Ministério Público, prestando-lhe nal pública incondicionada, não se lhes aplicando os arts.
informações sobre os fatos que constituam objeto de ação 181 e 182 do Código Penal.
civil e indicando-lhe os elementos de convicção. Art. 96. Discriminar pessoa idosa, impedindo ou difi-
Art. 90. Os agentes públicos em geral, os juízes e tri- cultando seu acesso a operações bancárias, aos meios de
bunais, no exercício de suas funções, quando tiverem co- transporte, ao direito de contratar ou por qualquer outro
nhecimento de fatos que possam configurar crime de ação meio ou instrumento necessário ao exercício da cidadania,
pública contra idoso ou ensejar a propositura de ação para por motivo de idade:
sua defesa, devem encaminhar as peças pertinentes ao Mi- Pena – reclusão de 6 (seis) meses a 1 (um) ano e multa.
nistério Público, para as providências cabíveis. § 1o  Na mesma pena incorre quem desdenhar, humi-
Art. 91. Para instruir a petição inicial, o interessado po- lhar, menosprezar ou discriminar pessoa idosa, por qual-
derá requerer às autoridades competentes as certidões e quer motivo.
informações que julgar necessárias, que serão fornecidas
§ 2o A pena será aumentada de 1/3 (um terço) se a ví-
no prazo de 10 (dez) dias.
tima se encontrar sob os cuidados ou responsabilidade do
Art. 92. O Ministério Público poderá instaurar sob sua
agente.
presidência, inquérito civil, ou requisitar, de qualquer pes-
Art. 97. Deixar de prestar assistência ao idoso, quando
soa, organismo público ou particular, certidões, informa-
ções, exames ou perícias, no prazo que assinalar, o qual possível fazê-lo sem risco pessoal, em situação de iminente
não poderá ser inferior a 10 (dez) dias. perigo, ou recusar, retardar ou dificultar sua assistência à
§ 1o Se o órgão do Ministério Público, esgotadas todas saúde, sem justa causa, ou não pedir, nesses casos, o so-
as diligências, se convencer da inexistência de fundamento corro de autoridade pública:
para a propositura da ação civil ou de peças informativas, Pena – detenção de 6 (seis) meses a 1 (um) ano e multa.
determinará o seu arquivamento, fazendo-o fundamenta- Parágrafo único. A pena é aumentada de metade, se da
damente. omissão resulta lesão corporal de natureza grave, e tripli-
cada, se resulta a morte.

187
LEGISLAÇÃO

Art. 98. Abandonar o idoso em hospitais, casas de saú- Art. 107.  Coagir, de qualquer modo, o idoso a doar,
de, entidades de longa permanência, ou congêneres, ou contratar, testar ou outorgar procuração:
não prover suas necessidades básicas, quando obrigado Pena – reclusão de 2 (dois) a 5 (cinco) anos.
por lei ou mandado: Art. 108. Lavrar ato notarial que envolva pessoa idosa
Pena – detenção de 6 (seis) meses a 3 (três) anos e mul- sem discernimento de seus atos, sem a devida representa-
ta. ção legal:
Art. 99. Expor a perigo a integridade e a saúde, física ou Pena – reclusão de 2 (dois) a 4 (quatro) anos.
psíquica, do idoso, submetendo-o a condições desumanas
ou degradantes ou privando-o de alimentos e cuidados in- TÍTULO VII
dispensáveis, quando obrigado a fazê-lo, ou sujeitando-o a Disposições Finais e Transitórias
trabalho excessivo ou inadequado:
Pena – detenção de 2 (dois) meses a 1 (um) ano e multa. Art. 109.  Impedir ou embaraçar ato do representante
§ 1o Se do fato resulta lesão corporal de natureza grave: do Ministério Público ou de qualquer outro agente fisca-
Pena – reclusão de 1 (um) a 4 (quatro) anos. lizador:
§ 2o Se resulta a morte: Pena – reclusão de 6 (seis) meses a 1 (um) ano e multa.
Pena – reclusão de 4 (quatro) a 12 (doze) anos. Art. 110. O Decreto-Lei no 2.848, de 7 de dezembro de
Art. 100.  Constitui crime punível com reclusão de 6 1940, Código Penal, passa a vigorar com as seguintes alte-
(seis) meses a 1 (um) ano e multa: rações:
I – obstar o acesso de alguém a qualquer cargo público “Art. 61. ............................................................................
por motivo de idade; ............................................................................
II – negar a alguém, por motivo de idade, emprego ou II - ............................................................................
trabalho; ............................................................................
h) contra criança, maior de 60 (sessenta) anos, enfermo
III – recusar, retardar ou dificultar atendimento ou dei-
ou mulher grávida;
xar de prestar assistência à saúde, sem justa causa, a pessoa
.............................................................................” (NR)
idosa;
“Art. 121. ............................................................................
IV – deixar de cumprir, retardar ou frustrar, sem justo
............................................................................
motivo, a execução de ordem judicial expedida na ação civil
§ 4o No homicídio culposo, a pena é aumentada de 1/3
a que alude esta Lei;
(um terço), se o crime resulta de inobservância de regra
V – recusar, retardar ou omitir dados técnicos indispen-
técnica de profissão, arte ou ofício, ou se o agente deixa
sáveis à propositura da ação civil objeto desta Lei, quando
de prestar imediato socorro à vítima, não procura diminuir
requisitados pelo Ministério Público.
as consequências do seu ato, ou foge para evitar prisão em
Art. 101.  Deixar de cumprir, retardar ou frustrar, sem flagrante. Sendo doloso o homicídio, a pena é aumentada
justo motivo, a execução de ordem judicial expedida nas de 1/3 (um terço) se o crime é praticado contra pessoa me-
ações em que for parte ou interveniente o idoso: nor de 14 (quatorze) ou maior de 60 (sessenta) anos.
Pena – detenção de 6 (seis) meses a 1 (um) ano e multa. .............................................................................” (NR)
Art. 102.  Apropriar-se de ou desviar bens, proventos, “Art. 133. ............................................................................
pensão ou qualquer outro rendimento do idoso, dando- ............................................................................
lhes aplicação diversa da de sua finalidade: § 3o ............................................................................
Pena – reclusão de 1 (um) a 4 (quatro) anos e multa. ............................................................................
Art. 103.  Negar o acolhimento ou a permanência do III – se a vítima é maior de 60 (sessenta) anos.” (NR)
idoso, como abrigado, por recusa deste em outorgar pro- “Art. 140. ............................................................................
curação à entidade de atendimento: ............................................................................
Pena – detenção de 6 (seis) meses a 1 (um) ano e multa. § 3o Se a injúria consiste na utilização de elementos re-
Art. 104. Reter o cartão magnético de conta bancária ferentes a raça, cor, etnia, religião, origem ou a condição de
relativa a benefícios, proventos ou pensão do idoso, bem pessoa idosa ou portadora de deficiência:
como qualquer outro documento com objetivo de assegu- ............................................................................ (NR)
rar recebimento ou ressarcimento de dívida: “Art. 141. ............................................................................
Pena – detenção de 6 (seis) meses a 2 (dois) anos e ............................................................................
multa. IV – contra pessoa maior de 60 (sessenta) anos ou por-
Art. 105. Exibir ou veicular, por qualquer meio de comu- tadora de deficiência, exceto no caso de injúria.
nicação, informações ou imagens depreciativas ou injurio- .............................................................................” (NR)
sas à pessoa do idoso: “Art. 148. ............................................................................
Pena – detenção de 1 (um) a 3 (três) anos e multa. ............................................................................
Art. 106.  Induzir pessoa idosa sem discernimento de § 1o............................................................................
seus atos a outorgar procuração para fins de administração I – se a vítima é ascendente, descendente, cônjuge do
de bens ou deles dispor livremente: agente ou maior de 60 (sessenta) anos.
Pena – reclusão de 2 (dois) a 4 (quatro) anos. ............................................................................” (NR)

188
LEGISLAÇÃO

“Art. 159............................................................................ Art. 116. Serão incluídos nos censos demográficos da-


............................................................................ dos relativos à população idosa do País.
§ 1o  Se o sequestro dura mais de 24 (vinte e quatro) Art. 117. O Poder Executivo encaminhará ao Congres-
horas, se o sequestrado é menor de 18 (dezoito) ou maior so Nacional projeto de lei revendo os critérios de conces-
de 60 (sessenta) anos, ou se o crime é cometido por bando são do Benefício de Prestação Continuada previsto na Lei
ou quadrilha. Orgânica da Assistência Social, de forma a garantir que o
............................................................................” (NR) acesso ao direito seja condizente com o estágio de desen-
“Art. 183............................................................................ volvimento socioeconômico alcançado pelo País.
............................................................................ Art. 118. Esta Lei entra em vigor decorridos 90 (noventa)
III – se o crime é praticado contra pessoa com idade dias da sua publicação, ressalvado o disposto no caput do
igual ou superior a 60 (sessenta) anos.” (NR) art. 36, que vigorará a partir de 1o de janeiro de 2004.
“Art. 244. Deixar, sem justa causa, de prover a subsis- Brasília, 1o de outubro de 2003; 182o da Independência
tência do cônjuge, ou de filho menor de 18 (dezoito) anos e 115o da República.
ou inapto para o trabalho, ou de ascendente inválido ou
maior de 60 (sessenta) anos, não lhes proporcionando os
recursos necessários ou faltando ao pagamento de pensão
alimentícia judicialmente acordada, fixada ou majorada;
deixar, sem justa causa, de socorrer descendente ou ascen- LEI BRASILEIRA DE INCLUSÃO DA PESSOA
dente, gravemente enfermo: COM DEFICIÊNCIA – LEI 13.146/2015
............................................................................” (NR)
Art. 111. O O art. 21 do Decreto-Lei no 3.688, de 3 de outubro de
1941
, Lei das Contravenções Penais, passa a vigorar acrescido
do seguinte parágrafo único: LEI Nº 13.146, DE 6 DE JULHO DE 2015.
“Art. 21............................................................................
............................................................................ Institui a Lei Brasileira de Inclusão da Pessoa com Defi-
Parágrafo único. Aumenta-se a pena de 1/3 (um terço) ciência (Estatuto da Pessoa com Deficiência).
até a metade se a vítima é maior de 60 (sessenta) anos.”
(NR)
A PRESIDENTA DA REPÚBLICA Faço saber que o Con-
Art. 112. O inciso II do § 4o do art. 1o da Lei no 9.455, de 7 de abril de 1997,
gresso Nacional decreta e eu sanciono a seguinte Lei:
passa a vigorar com a seguinte redação:
LIVRO I
“Art. 1o ............................................................................
............................................................................
PARTE GERAL
§ 4o ............................................................................
TÍTULO I
II – se o crime é cometido contra criança, gestante, por-
DISPOSIÇÕES PRELIMINARES
tador de deficiência, adolescente ou maior de 60 (sessenta)
CAPÍTULO I
anos;
............................................................................” (NR) DISPOSIÇÕES GERAIS
Art. 113. O inciso III do art. 18 da Lei no 6.368, de 21 de outubro de
1976
, passa a vigorar com a seguinte redação: Art. 1o  É instituída a Lei Brasileira de Inclusão da Pessoa
“Art. 18............................................................................ com Deficiência (Estatuto da Pessoa com Deficiência), des-
............................................................................ tinada a assegurar e a promover, em condições de igualda-
III – se qualquer deles decorrer de associação ou visar de, o exercício dos direitos e das liberdades fundamentais
a menores de 21 (vinte e um) anos ou a pessoa com idade por pessoa com deficiência, visando à sua inclusão social e
igual ou superior a 60 (sessenta) anos ou a quem tenha, por cidadania.
qualquer causa, diminuída ou suprimida a capacidade de Parágrafo único.  Esta Lei tem como base a Conven-
discernimento ou de autodeterminação: ção sobre os Direitos das Pessoas com Deficiência e seu
............................................................................” (NR) Protocolo Facultativo, ratificados pelo Congresso Nacional
Art. 114. O art 1º da Lei no 10.048, de 8 de novembro de 2000, passa a por meio do Decreto Legislativo no  186, de 9 de julho de 2008, em
vigorar com a seguinte redação: conformidade com o procedimento previsto no § 3o  do art.
“Art. 1o As pessoas portadoras de deficiência, os idosos
5o da Constituição da República Federativa do Brasil
, em vigor para o Brasil, no
com idade igual ou superior a 60 (sessenta) anos, as ges- plano jurídico externo, desde 31 de agosto de 2008, e pro-
tantes, as lactantes e as pessoas acompanhadas por crian- mulgados pelo Decreto no 6.949, de 25 de agosto de 2009, data de início
ças de colo terão atendimento prioritário, nos termos desta de sua vigência no plano interno.
Lei.” (NR) Art. 2o    Considera-se pessoa com deficiência aquela
Art. 115. O Orçamento da Seguridade Social destinará que tem impedimento de longo prazo de natureza física,
ao Fundo Nacional de Assistência Social, até que o Fundo mental, intelectual ou sensorial, o qual, em interação com
Nacional do Idoso seja criado, os recursos necessários, em uma ou mais barreiras, pode obstruir sua participação ple-
cada exercício financeiro, para aplicação em programas e na e efetiva na sociedade em igualdade de condições com
ações relativos ao idoso. as demais pessoas.  

189
LEGISLAÇÃO

§ 1o  A avaliação da deficiência, quando necessária, será racteres ampliados, os dispositivos multimídia, assim como
biopsicossocial, realizada por equipe multiprofissional e in- a linguagem simples, escrita e oral, os sistemas auditivos e
terdisciplinar e considerará:      (Vigência) os meios de voz digitalizados e os modos, meios e forma-
I - os impedimentos nas funções e nas estruturas do tos aumentativos e alternativos de comunicação, incluindo
corpo; as tecnologias da informação e das comunicações;
II - os fatores socioambientais, psicológicos e pessoais; VI - adaptações razoáveis: adaptações, modificações e
III - a limitação no desempenho de atividades; e ajustes necessários e adequados que não acarretem ônus
IV - a restrição de participação. desproporcional e indevido, quando requeridos em cada
§ 2o  O Poder Executivo criará instrumentos para avalia- caso, a fim de assegurar que a pessoa com deficiência pos-
ção da deficiência. sa gozar ou exercer, em igualdade de condições e oportu-
Art. 3o  Para fins de aplicação desta Lei, consideram-se: nidades com as demais pessoas, todos os direitos e liber-
I - acessibilidade: possibilidade e condição de alcance dades fundamentais;
para utilização, com segurança e autonomia, de espaços, VII - elemento de urbanização: quaisquer componen-
mobiliários, equipamentos urbanos, edificações, transpor- tes de obras de urbanização, tais como os referentes a pa-
tes, informação e comunicação, inclusive seus sistemas e vimentação, saneamento, encanamento para esgotos, dis-
tecnologias, bem como de outros serviços e instalações tribuição de energia elétrica e de gás, iluminação pública,
abertos ao público, de uso público ou privados de uso co- serviços de comunicação, abastecimento e distribuição de
letivo, tanto na zona urbana como na rural, por pessoa com água, paisagismo e os que materializam as indicações do
deficiência ou com mobilidade reduzida; planejamento urbanístico;  
II - desenho universal: concepção de produtos, am- VIII - mobiliário urbano: conjunto de objetos existentes
bientes, programas e serviços a serem usados por todas nas vias e nos espaços públicos, superpostos ou adiciona-
as pessoas, sem necessidade de adaptação ou de projeto dos aos elementos de urbanização ou de edificação, de
específico, incluindo os recursos de tecnologia assistiva; forma que sua modificação ou seu traslado não provoque
III - tecnologia assistiva ou ajuda técnica: produtos, alterações substanciais nesses elementos, tais como semá-
equipamentos, dispositivos, recursos, metodologias, es- foros, postes de sinalização e similares, terminais e pontos
tratégias, práticas e serviços que objetivem promover a de acesso coletivo às telecomunicações, fontes de água,
funcionalidade, relacionada à atividade e à participação da
lixeiras, toldos, marquises, bancos, quiosques e quaisquer
pessoa com deficiência ou com mobilidade reduzida, vi-
outros de natureza análoga;
sando à sua autonomia, independência, qualidade de vida
IX - pessoa com mobilidade reduzida: aquela que te-
e inclusão social;
nha, por qualquer motivo, dificuldade de movimentação,
IV - barreiras: qualquer entrave, obstáculo, atitude ou
permanente ou temporária, gerando redução efetiva da
comportamento que limite ou impeça a participação social
mobilidade, da flexibilidade, da coordenação motora ou da
da pessoa, bem como o gozo, a fruição e o exercício de
percepção, incluindo idoso, gestante, lactante, pessoa com
seus direitos à acessibilidade, à liberdade de movimento
criança de colo e obeso;
e de expressão, à comunicação, ao acesso à informação,
à compreensão, à circulação com segurança, entre outros, X - residências inclusivas: unidades de oferta do Servi-
classificadas em: ço de Acolhimento do Sistema Único de Assistência Social
a) barreiras urbanísticas: as existentes nas vias e nos (Suas) localizadas em áreas residenciais da comunidade,
espaços públicos e privados abertos ao público ou de uso com estruturas adequadas, que possam contar com apoio
coletivo; psicossocial para o atendimento das necessidades da pes-
b) barreiras arquitetônicas: as existentes nos edifícios soa acolhida, destinadas a jovens e adultos com deficiên-
públicos e privados; cia, em situação de dependência, que não dispõem de con-
c) barreiras nos transportes: as existentes nos sistemas dições de autossustentabilidade e com vínculos familiares
e meios de transportes; fragilizados ou rompidos;
d) barreiras nas comunicações e na informação: qual- XI - moradia para a vida independente da pessoa com
quer entrave, obstáculo, atitude ou comportamento que deficiência: moradia com estruturas adequadas capazes de
dificulte ou impossibilite a expressão ou o recebimento de proporcionar serviços de apoio coletivos e individualizados
mensagens e de informações por intermédio de sistemas que respeitem e ampliem o grau de autonomia de jovens e
de comunicação e de tecnologia da informação; adultos com deficiência;  
e) barreiras atitudinais: atitudes ou comportamentos XII - atendente pessoal: pessoa, membro ou não da
que impeçam ou prejudiquem a participação social da pes- família, que, com ou sem remuneração, assiste ou presta
soa com deficiência em igualdade de condições e oportu- cuidados básicos e essenciais à pessoa com deficiência no
nidades com as demais pessoas; exercício de suas atividades diárias, excluídas as técnicas ou
f) barreiras tecnológicas: as que dificultam ou impe- os procedimentos identificados com profissões legalmente
dem o acesso da pessoa com deficiência às tecnologias; estabelecidas;
V - comunicação: forma de interação dos cidadãos que XIII - profissional de apoio escolar: pessoa que exerce
abrange, entre outras opções, as línguas, inclusive a Língua atividades de alimentação, higiene e locomoção do estu-
Brasileira de Sinais (Libras), a visualização de textos, o Brail- dante com deficiência e atua em todas as atividades esco-
le, o sistema de sinalização ou de comunicação tátil, os ca- lares nas quais se fizer necessária, em todos os níveis e mo-

190
LEGISLAÇÃO

dalidades de ensino, em instituições públicas e privadas, nológicos, à dignidade, ao respeito, à liberdade, à convi-
excluídas as técnicas ou os procedimentos identificados vência familiar e comunitária, entre outros decorrentes da
com profissões legalmente estabelecidas; Constituição Federal, da Convenção sobre os Direitos das
XIV - acompanhante: aquele que acompanha a pessoa Pessoas com Deficiência e seu Protocolo Facultativo e das
com deficiência, podendo ou não desempenhar as funções leis e de outras normas que garantam seu bem-estar pes-
de atendente pessoal. soal, social e econômico.

CAPÍTULO II Seção Única


DA IGUALDADE E DA NÃO DISCRIMINAÇÃO Do Atendimento Prioritário

Art. 4o  Toda pessoa com deficiência tem direito à igual- Art. 9o  A pessoa com deficiência tem direito a receber
dade de oportunidades com as demais pessoas e não so- atendimento prioritário, sobretudo com a finalidade de:
frerá nenhuma espécie de discriminação. I - proteção e socorro em quaisquer circunstâncias;
§ 1o  Considera-se discriminação em razão da deficiên- II - atendimento em todas as instituições e serviços de
cia toda forma de distinção, restrição ou exclusão, por ação atendimento ao público;
ou omissão, que tenha o propósito ou o efeito de preju- III - disponibilização de recursos, tanto humanos quan-
dicar, impedir ou anular o reconhecimento ou o exercício to tecnológicos, que garantam atendimento em igualdade
dos direitos e das liberdades fundamentais de pessoa com de condições com as demais pessoas;
deficiência, incluindo a recusa de adaptações razoáveis e IV - disponibilização de pontos de parada, estações e
de fornecimento de tecnologias assistivas. terminais acessíveis de transporte coletivo de passageiros
§ 2o  A pessoa com deficiência não está obrigada à frui- e garantia de segurança no embarque e no desembarque;
ção de benefícios decorrentes de ação afirmativa. V - acesso a informações e disponibilização de recursos
Art. 5o    A pessoa com deficiência será protegida de de comunicação acessíveis;
toda forma de negligência, discriminação, exploração, vio- VI - recebimento de restituição de imposto de renda;
lência, tortura, crueldade, opressão e tratamento desuma- VII - tramitação processual e procedimentos judiciais e
no ou degradante. administrativos em que for parte ou interessada, em todos
Parágrafo único.  Para os fins da proteção menciona- os atos e diligências.
da no caput deste artigo, são considerados especialmente § 1o  Os direitos previstos neste artigo são extensivos ao
vulneráveis a criança, o adolescente, a mulher e o idoso, acompanhante da pessoa com deficiência ou ao seu aten-
com deficiência. dente pessoal, exceto quanto ao disposto nos incisos VI e
Art. 6o  A deficiência não afeta a plena capacidade civil VII deste artigo.
da pessoa, inclusive para: § 2o  Nos serviços de emergência públicos e privados, a
I - casar-se e constituir união estável; prioridade conferida por esta Lei é condicionada aos pro-
II - exercer direitos sexuais e reprodutivos; tocolos de atendimento médico.
III - exercer o direito de decidir sobre o número de fi-
lhos e de ter acesso a informações adequadas sobre repro- TÍTULO II
dução e planejamento familiar; DOS DIREITOS FUNDAMENTAIS
IV - conservar sua fertilidade, sendo vedada a esterili- CAPÍTULO I
zação compulsória; DO DIREITO À VIDA
V - exercer o direito à família e à convivência familiar e
comunitária; e Art. 10.  Compete ao poder público garantir a dignida-
VI - exercer o direito à guarda, à tutela, à curatela e de da pessoa com deficiência ao longo de toda a vida. 
à adoção, como adotante ou adotando, em igualdade de Parágrafo único.  Em situações de risco, emergência
oportunidades com as demais pessoas. ou estado de calamidade pública, a pessoa com deficiên-
Art. 7o  É dever de todos comunicar à autoridade com- cia será considerada vulnerável, devendo o poder público
petente qualquer forma de ameaça ou de violação aos di- adotar medidas para sua proteção e segurança.
reitos da pessoa com deficiência. Art. 11.  A pessoa com deficiência não poderá ser obri-
Parágrafo único.  Se, no exercício de suas funções, os gada a se submeter a intervenção clínica ou cirúrgica, a tra-
juízes e os tribunais tiverem conhecimento de fatos que ca- tamento ou a institucionalização forçada.
racterizem as violações previstas nesta Lei, devem remeter Parágrafo único.  O consentimento da pessoa com de-
peças ao Ministério Público para as providências cabíveis. ficiência em situação de curatela poderá ser suprido, na
Art. 8o  É dever do Estado, da sociedade e da família as- forma da lei.
segurar à pessoa com deficiência, com prioridade, a efeti- Art. 12.  O consentimento prévio, livre e esclarecido da
vação dos direitos referentes à vida, à saúde, à sexualidade, pessoa com deficiência é indispensável para a realização
à paternidade e à maternidade, à alimentação, à habitação, de tratamento, procedimento, hospitalização e pesquisa
à educação, à profissionalização, ao trabalho, à previdên- científica.
cia social, à habilitação e à reabilitação, ao transporte, à § 1o   Em caso de pessoa com deficiência em situação
acessibilidade, à cultura, ao desporto, ao turismo, ao lazer, de curatela, deve ser assegurada sua participação, no maior
à informação, à comunicação, aos avanços científicos e tec- grau possível, para a obtenção de consentimento.

191
LEGISLAÇÃO

§ 2o  A pesquisa científica envolvendo pessoa com defi- Parágrafo único.  Os serviços de que trata o caput des-
ciência em situação de tutela ou de curatela deve ser reali- te artigo podem fornecer informações e orientações nas
zada, em caráter excepcional, apenas quando houver indí- áreas de saúde, de educação, de cultura, de esporte, de
cios de benefício direto para sua saúde ou para a saúde de lazer, de transporte, de previdência social, de assistência
outras pessoas com deficiência e desde que não haja outra social, de habitação, de trabalho, de empreendedorismo,
opção de pesquisa de eficácia comparável com participan- de acesso ao crédito, de promoção, proteção e defesa de
tes não tutelados ou curatelados. direitos e nas demais áreas que possibilitem à pessoa com
Art. 13.  A pessoa com deficiência somente será aten- deficiência exercer sua cidadania.
dida sem seu consentimento prévio, livre e esclarecido em
casos de risco de morte e de emergência em saúde, res- CAPÍTULO III
guardado seu superior interesse e adotadas as salvaguardas DO DIREITO À SAÚDE
legais cabíveis.  
Art. 18.  É assegurada atenção integral à saúde da pes-
CAPÍTULO II soa com deficiência em todos os níveis de complexidade,
DO DIREITO À HABILITAÇÃO E À REABILITAÇÃO por intermédio do SUS, garantido acesso universal e igua-
litário.
Art. 14.  O processo de habilitação e de reabilitação é § 1o   É assegurada a participação da pessoa com defi-
um direito da pessoa com deficiência. ciência na elaboração das políticas de saúde a ela destina-
Parágrafo único.  O processo de habilitação e de rea- das.
bilitação tem por objetivo o desenvolvimento de potencia- § 2o  É assegurado atendimento segundo normas éticas
lidades, talentos, habilidades e aptidões físicas, cognitivas, e técnicas, que regulamentarão a atuação dos profissionais
sensoriais, psicossociais, atitudinais, profissionais e artísticas de saúde e contemplarão aspectos relacionados aos direi-
que contribuam para a conquista da autonomia da pessoa tos e às especificidades da pessoa com deficiência, incluin-
com deficiência e de sua participação social em igualdade do temas como sua dignidade e autonomia.
de condições e oportunidades com as demais pessoas. § 3o  Aos profissionais que prestam assistência à pessoa
Art. 15.  O processo mencionado no art. 14 desta Lei com deficiência, especialmente em serviços de habilitação
baseia-se em avaliação multidisciplinar das necessidades, e de reabilitação, deve ser garantida capacitação inicial e
habilidades e potencialidades de cada pessoa, observadas continuada.
as seguintes diretrizes: § 4o  As ações e os serviços de saúde pública destinados
I - diagnóstico e intervenção precoces; à pessoa com deficiência devem assegurar:
II - adoção de medidas para compensar perda ou limi- I - diagnóstico e intervenção precoces, realizados por
tação funcional, buscando o desenvolvimento de aptidões; equipe multidisciplinar;  
III - atuação permanente, integrada e articulada de po- II - serviços de habilitação e de reabilitação sempre
líticas públicas que possibilitem a plena participação social que necessários, para qualquer tipo de deficiência, inclu-
da pessoa com deficiência; sive para a manutenção da melhor condição de saúde e
IV - oferta de rede de serviços articulados, com atuação qualidade de vida;
intersetorial, nos diferentes níveis de complexidade, para III - atendimento domiciliar multidisciplinar, tratamen-
atender às necessidades específicas da pessoa com defi- to ambulatorial e internação;
ciência;   IV - campanhas de vacinação;
V - prestação de serviços próximo ao domicílio da pes- V - atendimento psicológico, inclusive para seus fami-
soa com deficiência, inclusive na zona rural, respeitadas a liares e atendentes pessoais;
organização das Redes de Atenção à Saúde (RAS) nos terri- VI - respeito à especificidade, à identidade de gênero e
tórios locais e as normas do Sistema Único de Saúde (SUS). à orientação sexual da pessoa com deficiência;
Art. 16.  Nos programas e serviços de habilitação e de VII - atenção sexual e reprodutiva, incluindo o direito à
reabilitação para a pessoa com deficiência, são garantidos: fertilização assistida;
I - organização, serviços, métodos, técnicas e recursos VIII - informação adequada e acessível à pessoa com
para atender às características de cada pessoa com defi- deficiência e a seus familiares sobre sua condição de saúde;
ciência; IX - serviços projetados para prevenir a ocorrência e o
II - acessibilidade em todos os ambientes e serviços; desenvolvimento de deficiências e agravos adicionais;
III - tecnologia assistiva, tecnologia de reabilitação, ma- X - promoção de estratégias de capacitação perma-
teriais e equipamentos adequados e apoio técnico profis- nente das equipes que atuam no SUS, em todos os níveis
sional, de acordo com as especificidades de cada pessoa de atenção, no atendimento à pessoa com deficiência, bem
com deficiência; como orientação a seus atendentes pessoais;
IV - capacitação continuada de todos os profissionais XI - oferta de órteses, próteses, meios auxiliares de lo-
que participem dos programas e serviços. comoção, medicamentos, insumos e fórmulas nutricionais,
Art. 17.  Os serviços do SUS e do Suas deverão promo- conforme as normas vigentes do Ministério da Saúde.
ver ações articuladas para garantir à pessoa com deficiên- § 5o   As diretrizes deste artigo aplicam-se também às
cia e sua família a aquisição de informações, orientações e instituições privadas que participem de forma complemen-
formas de acesso às políticas públicas disponíveis, com a tar do SUS ou que recebam recursos públicos para sua ma-
finalidade de propiciar sua plena participação social. nutenção.

192
LEGISLAÇÃO

Art. 19.  Compete ao SUS desenvolver ações destina- CAPÍTULO IV


das à prevenção de deficiências por causas evitáveis, inclu- DO DIREITO À EDUCAÇÃO
sive por meio de:
I - acompanhamento da gravidez, do parto e do puer- Art. 27.  A educação constitui direito da pessoa com
pério, com garantia de parto humanizado e seguro; deficiência, assegurados sistema educacional inclusivo em
II - promoção de práticas alimentares adequadas e sau- todos os níveis e aprendizado ao longo de toda a vida, de
dáveis, vigilância alimentar e nutricional, prevenção e cuida- forma a alcançar o máximo desenvolvimento possível de
do integral dos agravos relacionados à alimentação e nutri- seus talentos e habilidades físicas, sensoriais, intelectuais e
ção da mulher e da criança; sociais, segundo suas características, interesses e necessi-
III - aprimoramento e expansão dos programas de imu- dades de aprendizagem.
nização e de triagem neonatal; Parágrafo único.  É dever do Estado, da família, da co-
IV - identificação e controle da gestante de alto risco. munidade escolar e da sociedade assegurar educação de
Art. 20.  As operadoras de planos e seguros privados de qualidade à pessoa com deficiência, colocando-a a salvo
saúde são obrigadas a garantir à pessoa com deficiência, no de toda forma de violência, negligência e discriminação.
mínimo, todos os serviços e produtos ofertados aos demais Art. 28.  Incumbe ao poder público assegurar, criar, de-
clientes. senvolver, implementar, incentivar, acompanhar e avaliar:
Art. 21. Quando esgotados os meios de atenção à saúde I - sistema educacional inclusivo em todos os níveis e
da pessoa com deficiência no local de residência, será pres- modalidades, bem como o aprendizado ao longo de toda
tado atendimento fora de domicílio, para fins de diagnóstico a vida;
e de tratamento, garantidos o transporte e a acomodação II - aprimoramento dos sistemas educacionais, visando
da pessoa com deficiência e de seu acompanhante.   a garantir condições de acesso, permanência, participação
Art. 22.  À pessoa com deficiência internada ou em ob- e aprendizagem, por meio da oferta de serviços e de recur-
servação é assegurado o direito a acompanhante ou a aten- sos de acessibilidade que eliminem as barreiras e promo-
dente pessoal, devendo o órgão ou a instituição de saúde vam a inclusão plena;
proporcionar condições adequadas para sua permanência III - projeto pedagógico que institucionalize o atendi-
em tempo integral. mento educacional especializado, assim como os demais
§ 1o   Na impossibilidade de permanência do acompa- serviços e adaptações razoáveis, para atender às caracterís-
nhante ou do atendente pessoal junto à pessoa com defi- ticas dos estudantes com deficiência e garantir o seu pleno
ciência, cabe ao profissional de saúde responsável pelo tra- acesso ao currículo em condições de igualdade, promo-
tamento justificá-la por escrito. vendo a conquista e o exercício de sua autonomia;
§ 2o   Na ocorrência da impossibilidade prevista no § IV - oferta de educação bilíngue, em Libras como pri-
1 deste artigo, o órgão ou a instituição de saúde deve ado-

meira língua e na modalidade escrita da língua portuguesa
tar as providências cabíveis para suprir a ausência do acom- como segunda língua, em escolas e classes bilíngues e em
panhante ou do atendente pessoal. escolas inclusivas;
Art. 23.  São vedadas todas as formas de discriminação V - adoção de medidas individualizadas e coletivas em
contra a pessoa com deficiência, inclusive por meio de co- ambientes que maximizem o desenvolvimento acadêmi-
brança de valores diferenciados por planos e seguros priva- co e social dos estudantes com deficiência, favorecendo o
dos de saúde, em razão de sua condição. acesso, a permanência, a participação e a aprendizagem
Art. 24.  É assegurado à pessoa com deficiência o acesso em instituições de ensino;
aos serviços de saúde, tanto públicos como privados, e às VI - pesquisas voltadas para o desenvolvimento de no-
informações prestadas e recebidas, por meio de recursos de vos métodos e técnicas pedagógicas, de materiais didáti-
tecnologia assistiva e de todas as formas de comunicação cos, de equipamentos e de recursos de tecnologia assistiva; 
previstas no inciso V do art. 3o desta Lei. VII - planejamento de estudo de caso, de elaboração
Art. 25.  Os espaços dos serviços de saúde, tanto públi- de plano de atendimento educacional especializado, de
cos quanto privados, devem assegurar o acesso da pessoa organização de recursos e serviços de acessibilidade e de
com deficiência, em conformidade com a legislação em vi- disponibilização e usabilidade pedagógica de recursos de
gor, mediante a remoção de barreiras, por meio de projetos tecnologia assistiva;
arquitetônico, de ambientação de interior e de comunicação VIII - participação dos estudantes com deficiência e de
que atendam às especificidades das pessoas com deficiência suas famílias nas diversas instâncias de atuação da comu-
física, sensorial, intelectual e mental. nidade escolar;
Art. 26.  Os casos de suspeita ou de confirmação de vio- IX - adoção de medidas de apoio que favoreçam o
lência praticada contra a pessoa com deficiência serão obje- desenvolvimento dos aspectos linguísticos, culturais, vo-
to de notificação compulsória pelos serviços de saúde públi- cacionais e profissionais, levando-se em conta o talento,
cos e privados à autoridade policial e ao Ministério Público, a criatividade, as habilidades e os interesses do estudante
além dos Conselhos dos Direitos da Pessoa com Deficiência. com deficiência;
Parágrafo único.  Para os efeitos desta Lei, considera-se X - adoção de práticas pedagógicas inclusivas pelos
violência contra a pessoa com deficiência qualquer ação ou programas de formação inicial e continuada de professo-
omissão, praticada em local público ou privado, que lhe res e oferta de formação continuada para o atendimento
cause morte ou dano ou sofrimento físico ou psicológico. educacional especializado;

193
LEGISLAÇÃO

XI - formação e disponibilização de professores para IV - disponibilização de recursos de acessibilidade e de


o atendimento educacional especializado, de tradutores e tecnologia assistiva adequados, previamente solicitados e
intérpretes da Libras, de guias intérpretes e de profissionais escolhidos pelo candidato com deficiência;
de apoio; V - dilação de tempo, conforme demanda apresentada
XII - oferta de ensino da Libras, do Sistema Braille e de pelo candidato com deficiência, tanto na realização de exa-
uso de recursos de tecnologia assistiva, de forma a ampliar me para seleção quanto nas atividades acadêmicas, me-
habilidades funcionais dos estudantes, promovendo sua diante prévia solicitação e comprovação da necessidade;
autonomia e participação; VI - adoção de critérios de avaliação das provas escri-
XIII - acesso à educação superior e à educação pro- tas, discursivas ou de redação que considerem a singulari-
fissional e tecnológica em igualdade de oportunidades e dade linguística da pessoa com deficiência, no domínio da
condições com as demais pessoas;   modalidade escrita da língua portuguesa;
XIV - inclusão em conteúdos curriculares, em cursos de VII - tradução completa do edital e de suas retificações
nível superior e de educação profissional técnica e tecnoló- em Libras.
gica, de temas relacionados à pessoa com deficiência nos
respectivos campos de conhecimento; CAPÍTULO V
XV - acesso da pessoa com deficiência, em igualdade DO DIREITO À MORADIA
de condições, a jogos e a atividades recreativas, esportivas
e de lazer, no sistema escolar; Art. 31.   A pessoa com deficiência tem direito à mo-
XVI - acessibilidade para todos os estudantes, traba- radia digna, no seio da família natural ou substituta, com
lhadores da educação e demais integrantes da comuni- seu cônjuge ou companheiro ou desacompanhada, ou em
dade escolar às edificações, aos ambientes e às atividades moradia para a vida independente da pessoa com defi-
concernentes a todas as modalidades, etapas e níveis de ciência, ou, ainda, em residência inclusiva.
ensino; § 1o   O poder público adotará programas e ações es-
XVII - oferta de profissionais de apoio escolar; tratégicas para apoiar a criação e a manutenção de mora-
XVIII - articulação intersetorial na implementação de dia para a vida independente da pessoa com deficiência.
políticas públicas. § 2o  A proteção integral na modalidade de residência
§ 1o   Às instituições privadas, de qualquer nível e mo- inclusiva será prestada no âmbito do Suas à pessoa com
dalidade de ensino, aplica-se obrigatoriamente o disposto deficiência em situação de dependência que não disponha
nos incisos I, II, III, V, VII, VIII, IX, X, XI, XII, XIII, XIV, XV, XVI, de condições de autossustentabilidade, com vínculos fa-
XVII e XVIII do caput deste artigo, sendo vedada a cobrança
miliares fragilizados ou rompidos.
de valores adicionais de qualquer natureza em suas men-
Art. 32.  Nos programas habitacionais, públicos ou
salidades, anuidades e matrículas no cumprimento dessas
subsidiados com recursos públicos, a pessoa com deficiên-
determinações.
cia ou o seu responsável goza de prioridade na aquisição
§ 2o  Na disponibilização de tradutores e intérpretes da
de imóvel para moradia própria, observado o seguinte:
Libras a que se refere o inciso XI do caput  deste artigo,
I - reserva de, no mínimo, 3% (três por cento) das uni-
deve-se observar o seguinte:
dades habitacionais para pessoa com deficiência;
I - os tradutores e intérpretes da Libras atuantes na
II - (VETADO);
educação básica devem, no mínimo, possuir ensino médio
completo e certificado de proficiência na Libras; (Vigência) III - em caso de edificação multifamiliar, garantia de
II - os tradutores e intérpretes da Libras, quando dire- acessibilidade nas áreas de uso comum e nas unidades ha-
cionados à tarefa de interpretar nas salas de aula dos cur- bitacionais no piso térreo e de acessibilidade ou de adap-
sos de graduação e pós-graduação, devem possuir nível tação razoável nos demais pisos;
superior, com habilitação, prioritariamente, em Tradução e IV - disponibilização de equipamentos urbanos comu-
Interpretação em Libras.(Vigência) nitários acessíveis;
Art. 29.  (VETADO). V - elaboração de especificações técnicas no projeto
Art. 30.  Nos processos seletivos para ingresso e perma- que permitam a instalação de elevadores.
nência nos cursos oferecidos pelas instituições de ensino § 1o   O direito à prioridade, previsto no caput  deste
superior e de educação profissional e tecnológica, públicas artigo, será reconhecido à pessoa com deficiência benefi-
e privadas, devem ser adotadas as seguintes medidas: ciária apenas uma vez.
I - atendimento preferencial à pessoa com deficiência § 2o  Nos programas habitacionais públicos, os critérios
nas dependências das Instituições de Ensino Superior (IES) de financiamento devem ser compatíveis com os rendi-
e nos serviços; mentos da pessoa com deficiência ou de sua família.
II - disponibilização de formulário de inscrição de exa- § 3o  Caso não haja pessoa com deficiência interessada
mes com campos específicos para que o candidato com nas unidades habitacionais reservadas por força do dis-
deficiência informe os recursos de acessibilidade e de tec- posto no inciso I do caput deste artigo, as unidades não
nologia assistiva necessários para sua participação; utilizadas serão disponibilizadas às demais pessoas.
III - disponibilização de provas em formatos acessíveis Art. 33.  Ao poder público compete:
para atendimento às necessidades específicas do candida- I - adotar as providências necessárias para o cumpri-
to com deficiência; mento do disposto nos arts. 31 e 32 desta Lei; e

194
LEGISLAÇÃO

II - divulgar, para os agentes interessados e beneficiá- § 2o  A habilitação profissional corresponde ao proces-
rios, a política habitacional prevista nas legislações federal, so destinado a propiciar à pessoa com deficiência aquisição
estaduais, distrital e municipais, com ênfase nos dispositi- de conhecimentos, habilidades e aptidões para exercício
vos sobre acessibilidade. de profissão ou de ocupação, permitindo nível suficiente
de desenvolvimento profissional para ingresso no campo
CAPÍTULO VI de trabalho.
DO DIREITO AO TRABALHO § 3o   Os serviços de habilitação profissional, de reabi-
Seção I litação profissional e de educação profissional devem ser
Disposições Gerais dotados de recursos necessários para atender a toda pes-
soa com deficiência, independentemente de sua caracterís-
Art. 34.  A pessoa com deficiência tem direito ao traba- tica específica, a fim de que ela possa ser capacitada para
lho de sua livre escolha e aceitação, em ambiente acessível trabalho que lhe seja adequado e ter perspectivas de obtê
e inclusivo, em igualdade de oportunidades com as demais -lo, de conservá-lo e de nele progredir.
pessoas. § 4o   Os serviços de habilitação profissional, de reabi-
litação profissional e de educação profissional deverão ser
§ 1o  As pessoas jurídicas de direito público, privado ou
oferecidos em ambientes acessíveis e inclusivos.
de qualquer natureza são obrigadas a garantir ambientes
§ 5o  A habilitação profissional e a reabilitação profissio-
de trabalho acessíveis e inclusivos.
nal devem ocorrer articuladas com as redes públicas e pri-
§ 2o  A pessoa com deficiência tem direito, em igualda- vadas, especialmente de saúde, de ensino e de assistência
de de oportunidades com as demais pessoas, a condições social, em todos os níveis e modalidades, em entidades de
justas e favoráveis de trabalho, incluindo igual remuneração formação profissional ou diretamente com o empregador.
por trabalho de igual valor. § 6o    A habilitação profissional pode ocorrer em em-
§ 3o  É vedada restrição ao trabalho da pessoa com defi- presas por meio de prévia formalização do contrato de
ciência e qualquer discriminação em razão de sua condição, emprego da pessoa com deficiência, que será considerada
inclusive nas etapas de recrutamento, seleção, contratação, para o cumprimento da reserva de vagas prevista em lei,
admissão, exames admissional e periódico, permanência no desde que por tempo determinado e concomitante com a
emprego, ascensão profissional e reabilitação profissional, inclusão profissional na empresa, observado o disposto em
bem como exigência de aptidão plena. regulamento.
§ 4o  A pessoa com deficiência tem direito à participação § 7o  A habilitação profissional e a reabilitação profissio-
e ao acesso a cursos, treinamentos, educação continuada, nal atenderão à pessoa com deficiência.
planos de carreira, promoções, bonificações e incentivos
profissionais oferecidos pelo empregador, em igualdade de Seção III
oportunidades com os demais empregados. Da Inclusão da Pessoa com Deficiência no Trabalho
§ 5o    É garantida aos trabalhadores com deficiência
acessibilidade em cursos de formação e de capacitação. Art. 37.  Constitui modo de inclusão da pessoa com de-
Art. 35.   É finalidade primordial das políticas públicas ficiência no trabalho a colocação competitiva, em igualda-
de trabalho e emprego promover e garantir condições de de de oportunidades com as demais pessoas, nos termos
acesso e de permanência da pessoa com deficiência no da legislação trabalhista e previdenciária, na qual devem
campo de trabalho. ser atendidas as regras de acessibilidade, o fornecimento
Parágrafo único.  Os programas de estímulo ao em- de recursos de tecnologia assistiva e a adaptação razoável
preendedorismo e ao trabalho autônomo, incluídos o coo- no ambiente de trabalho.
perativismo e o associativismo, devem prever a participação Parágrafo único.  A colocação competitiva da pessoa
com deficiência pode ocorrer por meio de trabalho com
da pessoa com deficiência e a disponibilização de linhas de
apoio, observadas as seguintes diretrizes:
crédito, quando necessárias.
I - prioridade no atendimento à pessoa com deficiência
com maior dificuldade de inserção no campo de trabalho;
Seção II II - provisão de suportes individualizados que aten-
Da Habilitação Profissional e Reabilitação Profis- dam a necessidades específicas da pessoa com deficiên-
sional cia, inclusive a disponibilização de recursos de tecnologia
assistiva, de agente facilitador e de apoio no ambiente de
Art. 36.  O poder público deve implementar serviços e trabalho;
programas completos de habilitação profissional e de reabi- III - respeito ao perfil vocacional e ao interesse da pes-
litação profissional para que a pessoa com deficiência pos- soa com deficiência apoiada;
sa ingressar, continuar ou retornar ao campo do trabalho, IV - oferta de aconselhamento e de apoio aos empre-
respeitados sua livre escolha, sua vocação e seu interesse. gadores, com vistas à definição de estratégias de inclusão e
§ 1o  Equipe multidisciplinar indicará, com base em crité- de superação de barreiras, inclusive atitudinais;
rios previstos no § 1o do art. 2o desta Lei, programa de habi- V - realização de avaliações periódicas;
litação ou de reabilitação que possibilite à pessoa com de- VI - articulação intersetorial das políticas públicas;
ficiência restaurar sua capacidade e habilidade profissional VII - possibilidade de participação de organizações da
ou adquirir novas capacidades e habilidades de trabalho. sociedade civil.

195
LEGISLAÇÃO

Art. 38.  A entidade contratada para a realização de Art. 43.  O poder público deve promover a participação
processo seletivo público ou privado para cargo, função ou da pessoa com deficiência em atividades artísticas, intelec-
emprego está obrigada à observância do disposto nesta Lei tuais, culturais, esportivas e recreativas, com vistas ao seu
e em outras normas de acessibilidade vigentes. protagonismo, devendo:
I - incentivar a provisão de instrução, de treinamento
CAPÍTULO VII e de recursos adequados, em igualdade de oportunidades
DO DIREITO À ASSISTÊNCIA SOCIAL com as demais pessoas;
II - assegurar acessibilidade nos locais de eventos e nos
Art. 39.  Os serviços, os programas, os projetos e os be- serviços prestados por pessoa ou entidade envolvida na or-
nefícios no âmbito da política pública de assistência social ganização das atividades de que trata este artigo; e
à pessoa com deficiência e sua família têm como objetivo a III - assegurar a participação da pessoa com deficiência
garantia da segurança de renda, da acolhida, da habilitação e em jogos e atividades recreativas, esportivas, de lazer, cul-
da reabilitação, do desenvolvimento da autonomia e da con- turais e artísticas, inclusive no sistema escolar, em igualdade
vivência familiar e comunitária, para a promoção do acesso a de condições com as demais pessoas.
direitos e da plena participação social.
Art. 44.  Nos teatros, cinemas, auditórios, estádios, gi-
§ 1o    A assistência social à pessoa com deficiência, nos
násios de esporte, locais de espetáculos e de conferências e
termos do caput deste artigo, deve envolver conjunto arti-
similares, serão reservados espaços livres e assentos para a
culado de serviços do âmbito da Proteção Social Básica e da
Proteção Social Especial, ofertados pelo Suas, para a garantia pessoa com deficiência, de acordo com a capacidade de lo-
de seguranças fundamentais no enfrentamento de situações tação da edificação, observado o disposto em regulamento.
de vulnerabilidade e de risco, por fragilização de vínculos e § 1o   Os espaços e assentos a que se refere este artigo
ameaça ou violação de direitos. devem ser distribuídos pelo recinto em locais diversos, de
§ 2o   Os serviços socioassistenciais destinados à pessoa boa visibilidade, em todos os setores, próximos aos corre-
com deficiência em situação de dependência deverão contar dores, devidamente sinalizados, evitando-se áreas segrega-
com cuidadores sociais para prestar-lhe cuidados básicos e das de público e obstrução das saídas, em conformidade
instrumentais. com as normas de acessibilidade.
Art. 40.  É assegurado à pessoa com deficiência que não § 2o  No caso de não haver comprovada procura pelos
possua meios para prover sua subsistência nem de tê-la pro- assentos reservados, esses podem, excepcionalmente, ser
vida por sua família o benefício mensal de 1 (um) salário-mí- ocupados por pessoas sem deficiência ou que não tenham
nimo, nos termos da Lei no 8.742, de 7 de dezembro de 1993. mobilidade reduzida, observado o disposto em regulamen-
to.
CAPÍTULO VIII § 3o  Os espaços e assentos a que se refere este artigo
DO DIREITO À PREVIDÊNCIA SOCIAL devem situar-se em locais que garantam a acomodação de,
no mínimo, 1 (um) acompanhante da pessoa com deficiên-
Art. 41.  A pessoa com deficiência segurada do Regime cia ou com mobilidade reduzida, resguardado o direito de
Geral de Previdência Social (RGPS) tem direito à aposentado- se acomodar proximamente a grupo familiar e comunitário.
ria nos termos da Lei Complementar no 142, de 8 de maio de 2013. § 4o  Nos locais referidos no caput deste artigo, deve ha-
ver, obrigatoriamente, rotas de fuga e saídas de emergência
CAPÍTULO IX acessíveis, conforme padrões das normas de acessibilidade,
DO DIREITO À CULTURA, AO ESPORTE, AO TURISMO a fim de permitir a saída segura da pessoa com deficiência
E AO LAZER ou com mobilidade reduzida, em caso de emergência.
§ 5o    Todos os espaços das edificações previstas
Art. 42.  A pessoa com deficiência tem direito à cultura,
no caput deste artigo devem atender às normas de acessi-
ao esporte, ao turismo e ao lazer em igualdade de oportuni-
bilidade em vigor.
dades com as demais pessoas, sendo-lhe garantido o acesso:
§ 6o   As salas de cinema devem oferecer, em todas as
I - a bens culturais em formato acessível;
II - a programas de televisão, cinema, teatro e outras ati- sessões, recursos de acessibilidade para a pessoa com defi-
vidades culturais e desportivas em formato acessível; e ciência.       (Vigência)
III - a monumentos e locais de importância cultural e a § 7o  O valor do ingresso da pessoa com deficiência não
espaços que ofereçam serviços ou eventos culturais e espor- poderá ser superior ao valor cobrado das demais pessoas.
tivos. Art. 45.  Os hotéis, pousadas e similares devem ser cons-
§ 1o  É vedada a recusa de oferta de obra intelectual em truídos observando-se os princípios do desenho universal,
formato acessível à pessoa com deficiência, sob qualquer ar- além de adotar todos os meios de acessibilidade, conforme
gumento, inclusive sob a alegação de proteção dos direitos legislação em vigor.      (Vigência)
de propriedade intelectual. § 1o  Os estabelecimentos já existentes deverão disponi-
§ 2o   O poder público deve adotar soluções destinadas bilizar, pelo menos, 10% (dez por cento) de seus dormitórios
à eliminação, à redução ou à superação de barreiras para a acessíveis, garantida, no mínimo, 1 (uma) unidade acessível.
promoção do acesso a todo patrimônio cultural, observadas § 2o  Os dormitórios mencionados no § 1o  deste artigo
as normas de acessibilidade, ambientais e de proteção do deverão ser localizados em rotas acessíveis.
patrimônio histórico e artístico nacional.

196
LEGISLAÇÃO

CAPÍTULO X § 3o  Para colocação do símbolo internacional de acesso


DO DIREITO AO TRANSPORTE E À MOBILIDADE nos veículos, as empresas de transporte coletivo de passa-
geiros dependem da certificação de acessibilidade emitida
Art. 46.  O direito ao transporte e à mobilidade da pes- pelo gestor público responsável pela prestação do serviço.
soa com deficiência ou com mobilidade reduzida será as- Art. 49.  As empresas de transporte de fretamento
segurado em igualdade de oportunidades com as demais e de turismo, na renovação de suas frotas, são obriga-
pessoas, por meio de identificação e de eliminação de to- das ao cumprimento do disposto nos arts. 46 e 48 desta
dos os obstáculos e barreiras ao seu acesso. Lei.       (Vigência)
§ 1o   Para fins de acessibilidade aos serviços de trans- Art. 50.  O poder público incentivará a fabricação de
porte coletivo terrestre, aquaviário e aéreo, em todas as ju- veículos acessíveis e a sua utilização como táxis e vans, de
risdições, consideram-se como integrantes desses serviços forma a garantir o seu uso por todas as pessoas.
os veículos, os terminais, as estações, os pontos de parada, Art. 51.  As frotas de empresas de táxi devem reservar
o sistema viário e a prestação do serviço. 10% (dez por cento) de seus veículos acessíveis à pessoa
§ 2o  São sujeitas ao cumprimento das disposições des-
com deficiência.
ta Lei, sempre que houver interação com a matéria nela
§ 1o   É proibida a cobrança diferenciada de tarifas ou
regulada, a outorga, a concessão, a permissão, a autoriza-
de valores adicionais pelo serviço de táxi prestado à pes-
ção, a renovação ou a habilitação de linhas e de serviços de
soa com deficiência.
transporte coletivo.
§ 3o  Para colocação do símbolo internacional de acesso § 2o  O poder público é autorizado a instituir incentivos
nos veículos, as empresas de transporte coletivo de passa- fiscais com vistas a possibilitar a acessibilidade dos veícu-
geiros dependem da certificação de acessibilidade emitida los a que se refere o caput deste artigo.
pelo gestor público responsável pela prestação do serviço. Art. 52.  As locadoras de veículos são obrigadas a ofe-
Art. 47.  Em todas as áreas de estacionamento aberto recer 1 (um) veículo adaptado para uso de pessoa com
ao público, de uso público ou privado de uso coletivo e deficiência, a cada conjunto de 20 (vinte) veículos de sua
em vias públicas, devem ser reservadas vagas próximas aos frota.
acessos de circulação de pedestres, devidamente sinaliza- Parágrafo único.  O veículo adaptado deverá ter, no
das, para veículos que transportem pessoa com deficiência mínimo, câmbio automático, direção hidráulica, vidros elé-
com comprometimento de mobilidade, desde que devida- tricos e comandos manuais de freio e de embreagem.
mente identificados.
§ 1o  As vagas a que se refere o caput deste artigo de- TÍTULO III
vem equivaler a 2% (dois por cento) do total, garantida, DA ACESSIBILIDADE
no mínimo, 1 (uma) vaga devidamente sinalizada e com CAPÍTULO I
as especificações de desenho e traçado de acordo com as DISPOSIÇÕES GERAIS
normas técnicas vigentes de acessibilidade.
§ 2o  Os veículos estacionados nas vagas reservadas de- Art. 53.  A acessibilidade é direito que garante à pes-
vem exibir, em local de ampla visibilidade, a credencial de soa com deficiência ou com mobilidade reduzida viver de
beneficiário, a ser confeccionada e fornecida pelos órgãos forma independente e exercer seus direitos de cidadania e
de trânsito, que disciplinarão suas características e condi- de participação social.
ções de uso. Art. 54.  São sujeitas ao cumprimento das disposições
§ 3º  A utilização indevida das vagas de que trata este desta Lei e de outras normas relativas à acessibilidade,
artigo sujeita os infratores às sanções previstas no inciso sempre que houver interação com a matéria nela regulada:
XX do art. 181 da Lei nº 9.503, de 23 de setembro de 1997
I - a aprovação de projeto arquitetônico e urbanístico
(Código de Trânsito Brasileiro).       (Redação dada pela Lei
ou de comunicação e informação, a fabricação de veículos
nº 13.281, de 2016)      (Vigência)
de transporte coletivo, a prestação do respectivo serviço
§ 4o   A credencial a que se refere o §  2o  deste artigo
e a execução de qualquer tipo de obra, quando tenham
é vinculada à pessoa com deficiência que possui compro-
metimento de mobilidade e é válida em todo o território destinação pública ou coletiva;
nacional. II - a outorga ou a renovação de concessão, permissão,
Art. 48.  Os veículos de transporte coletivo terrestre, autorização ou habilitação de qualquer natureza;
aquaviário e aéreo, as instalações, as estações, os portos e III - a aprovação de financiamento de projeto com uti-
os terminais em operação no País devem ser acessíveis, de lização de recursos públicos, por meio de renúncia ou de
forma a garantir o seu uso por todas as pessoas. incentivo fiscal, contrato, convênio ou instrumento con-
§ 1o  Os veículos e as estruturas de que trata o caput des- gênere; e
te artigo devem dispor de sistema de comunicação aces- IV - a concessão de aval da União para obtenção de
sível que disponibilize informações sobre todos os pontos empréstimo e de financiamento internacionais por entes
do itinerário. públicos ou privados.
§ 2o  São asseguradas à pessoa com deficiência priori- Art. 55.  A concepção e a implantação de projetos que
dade e segurança nos procedimentos de embarque e de tratem do meio físico, de transporte, de informação e co-
desembarque nos veículos de transporte coletivo, de acor- municação, inclusive de sistemas e tecnologias da informa-
do com as normas técnicas. ção e comunicação, e de outros serviços, equipamentos e

197
LEGISLAÇÃO

instalações abertos ao público, de uso público ou privado Art. 59.  Em qualquer intervenção nas vias e nos es-
de uso coletivo, tanto na zona urbana como na rural, de- paços públicos, o poder público e as empresas concessio-
vem atender aos princípios do desenho universal, tendo nárias responsáveis pela execução das obras e dos serviços
como referência as normas de acessibilidade. devem garantir, de forma segura, a fluidez do trânsito e a
§ 1o   O desenho universal será sempre tomado como livre circulação e acessibilidade das pessoas, durante e após
regra de caráter geral. sua execução.
§ 2o  Nas hipóteses em que comprovadamente o dese- Art. 60.  Orientam-se, no que couber, pelas regras de
nho universal não possa ser empreendido, deve ser adota- acessibilidade previstas em legislação e em normas técnicas,
da adaptação razoável. observado o disposto na Lei no 10.098, de 19 de dezembro de 2000, no 10.257,
de 10 de julho de 2001
, e no 12.587, de 3 de janeiro de 2012:
§ 3o  Caberá ao poder público promover a inclusão de
I - os planos diretores municipais, os planos diretores
conteúdos temáticos referentes ao desenho universal nas
de transporte e trânsito, os planos de mobilidade urbana e
diretrizes curriculares da educação profissional e tecnoló-
os planos de preservação de sítios históricos elaborados ou
gica e do ensino superior e na formação das carreiras de atualizados a partir da publicação desta Lei;
Estado. II - os códigos de obras, os códigos de postura, as leis de
§ 4o   Os programas, os projetos e as linhas de pes- uso e ocupação do solo e as leis do sistema viário;
quisa a serem desenvolvidos com o apoio de organismos III - os estudos prévios de impacto de vizinhança;
públicos de auxílio à pesquisa e de agências de fomento IV - as atividades de fiscalização e a imposição de san-
deverão incluir temas voltados para o desenho universal. ções; e
§ 5o  Desde a etapa de concepção, as políticas públicas V - a legislação referente à prevenção contra incêndio
deverão considerar a adoção do desenho universal. e pânico.
Art. 56.  A construção, a reforma, a ampliação ou a mu- § 1o  A concessão e a renovação de alvará de funciona-
dança de uso de edificações abertas ao público, de uso mento para qualquer atividade são condicionadas à obser-
público ou privadas de uso coletivo deverão ser executa- vação e à certificação das regras de acessibilidade.
das de modo a serem acessíveis. § 2o  A emissão de carta de habite-se ou de habilitação
§ 1o   As entidades de fiscalização profissional das ati- equivalente e sua renovação, quando esta tiver sido emitida
vidades de Engenharia, de Arquitetura e correlatas, ao anteriormente às exigências de acessibilidade, é condiciona-
anotarem a responsabilidade técnica de projetos, devem da à observação e à certificação das regras de acessibilidade.
Art. 61.  A formulação, a implementação e a manuten-
exigir a responsabilidade profissional declarada de atendi-
ção das ações de acessibilidade atenderão às seguintes pre-
mento às regras de acessibilidade previstas em legislação
missas básicas:
e em normas técnicas pertinentes.
I - eleição de prioridades, elaboração de cronograma e
§ 2o  Para a aprovação, o licenciamento ou a emissão de reserva de recursos para implementação das ações; e
certificado de projeto executivo arquitetônico, urbanístico II - planejamento contínuo e articulado entre os setores
e de instalações e equipamentos temporários ou perma- envolvidos.
nentes e para o licenciamento ou a emissão de certificado Art. 62.  É assegurado à pessoa com deficiência, median-
de conclusão de obra ou de serviço, deve ser atestado o te solicitação, o recebimento de contas, boletos, recibos, ex-
atendimento às regras de acessibilidade. tratos e cobranças de tributos em formato acessível.
§ 3o   O poder público, após certificar a acessibilidade
de edificação ou de serviço, determinará a colocação, em CAPÍTULO II
espaços ou em locais de ampla visibilidade, do símbolo DO ACESSO À INFORMAÇÃO E À COMUNICAÇÃO
internacional de acesso, na forma prevista em legislação e
em normas técnicas correlatas. Art. 63.  É obrigatória a acessibilidade nos sítios da in-
Art. 57.  As edificações públicas e privadas de uso co- ternet mantidos por empresas com sede ou representação
letivo já existentes devem garantir acessibilidade à pessoa comercial no País ou por órgãos de governo, para uso da
com deficiência em todas as suas dependências e serviços, pessoa com deficiência, garantindo-lhe acesso às informa-
tendo como referência as normas de acessibilidade vigen- ções disponíveis, conforme as melhores práticas e diretrizes
de acessibilidade adotadas internacionalmente.
tes.
§ 1o   Os sítios devem conter símbolo de acessibilidade
Art. 58.   O projeto e a construção de edificação de
em destaque.
uso privado multifamiliar devem atender aos preceitos de
§ 2o   Telecentros comunitários que receberem recursos
acessibilidade, na forma regulamentar. públicos federais para seu custeio ou sua instalação e lan
§ 1o   As construtoras e incorporadoras responsáveis houses  devem possuir equipamentos e instalações aces-
pelo projeto e pela construção das edificações a que se síveis.
refere o caput  deste artigo devem assegurar percentual § 3o   Os telecentros e as lan houses de que trata o §
mínimo de suas unidades internamente acessíveis, na for- 2 deste artigo devem garantir, no mínimo, 10% (dez por

ma regulamentar. cento) de seus computadores com recursos de acessibili-


§ 2o   É vedada a cobrança de valores adicionais para dade para pessoa com deficiência visual, sendo assegurado
a aquisição de unidades internamente acessíveis a que se pelo menos 1 (um) equipamento, quando o resultado per-
refere o § 1o deste artigo. centual for inferior a 1 (um).

198
LEGISLAÇÃO

Art. 64.  A acessibilidade nos sítios da internet de que Art. 70.  As instituições promotoras de congressos, se-
trata o art. 63 desta Lei deve ser observada para obtenção minários, oficinas e demais eventos de natureza científico-
do financiamento de que trata o inciso III do art. 54 desta cultural devem oferecer à pessoa com deficiência, no mí-
Lei. nimo, os recursos de tecnologia assistiva previstos no art.
Art. 65.  As empresas prestadoras de serviços de tele- 67 desta Lei.
comunicações deverão garantir pleno acesso à pessoa com Art. 71.  Os congressos, os seminários, as oficinas e os
deficiência, conforme regulamentação específica. demais eventos de natureza científico-cultural promovidos
Art. 66.  Cabe ao poder público incentivar a oferta de ou financiados pelo poder público devem garantir as con-
aparelhos de telefonia fixa e móvel celular com acessibilida- dições de acessibilidade e os recursos de tecnologia assis-
de que, entre outras tecnologias assistivas, possuam possi- tiva.
bilidade de indicação e de ampliação sonoras de todas as Art. 72.  Os programas, as linhas de pesquisa e os pro-
operações e funções disponíveis. jetos a serem desenvolvidos com o apoio de agências de
Art. 67.  Os serviços de radiodifusão de sons e imagens financiamento e de órgãos e entidades integrantes da ad-
devem permitir o uso dos seguintes recursos, entre outros: ministração pública que atuem no auxílio à pesquisa de-
I - subtitulação por meio de legenda oculta; vem contemplar temas voltados à tecnologia assistiva.
II - janela com intérprete da Libras; Art. 73.  Caberá ao poder público, diretamente ou em
III - audiodescrição. parceria com organizações da sociedade civil, promover a
Art. 68.  O poder público deve adotar mecanismos de capacitação de tradutores e intérpretes da Libras, de guias
incentivo à produção, à edição, à difusão, à distribuição e intérpretes e de profissionais habilitados em Braille, audio-
à comercialização de livros em formatos acessíveis, inclusi- descrição, estenotipia e legendagem.
ve em publicações da administração pública ou financiadas
com recursos públicos, com vistas a garantir à pessoa com CAPÍTULO III
deficiência o direito de acesso à leitura, à informação e à DA TECNOLOGIA ASSISTIVA
comunicação.
§ 1o  Nos editais de compras de livros, inclusive para o Art. 74.  É garantido à pessoa com deficiência acesso a
abastecimento ou a atualização de acervos de bibliotecas produtos, recursos, estratégias, práticas, processos, méto-
em todos os níveis e modalidades de educação e de biblio- dos e serviços de tecnologia assistiva que maximizem sua
tecas públicas, o poder público deverá adotar cláusulas de autonomia, mobilidade pessoal e qualidade de vida.
impedimento à participação de editoras que não ofertem Art. 75. O poder público desenvolverá plano específico
sua produção também em formatos acessíveis.   de medidas, a ser renovado em cada período de 4 (quatro)
§ 2o  Consideram-se formatos acessíveis os arquivos di- anos, com a finalidade de:
gitais que possam ser reconhecidos e acessados por soft- I - facilitar o acesso a crédito especializado, inclusive
wares leitores de telas ou outras tecnologias assistivas que com oferta de linhas de crédito subsidiadas, específicas
vierem a substituí-los, permitindo leitura com voz sintetiza- para aquisição de tecnologia assistiva;
da, ampliação de caracteres, diferentes contrastes e impres- II - agilizar, simplificar e priorizar procedimentos de
são em Braille. importação de tecnologia assistiva, especialmente as ques-
§ 3o  O poder público deve estimular e apoiar a adapta- tões atinentes a procedimentos alfandegários e sanitários;
ção e a produção de artigos científicos em formato acessí- III - criar mecanismos de fomento à pesquisa e à pro-
vel, inclusive em Libras. dução nacional de tecnologia assistiva, inclusive por meio
Art. 69.  O poder público deve assegurar a disponibi- de concessão de linhas de crédito subsidiado e de parce-
lidade de informações corretas e claras sobre os diferen- rias com institutos de pesquisa oficiais;
tes produtos e serviços ofertados, por quaisquer meios de IV - eliminar ou reduzir a tributação da cadeia produti-
comunicação empregados, inclusive em ambiente virtual, va e de importação de tecnologia assistiva;
contendo a especificação correta de quantidade, qualida- V - facilitar e agilizar o processo de inclusão de novos
de, características, composição e preço, bem como sobre os recursos de tecnologia assistiva no rol de produtos distri-
eventuais riscos à saúde e à segurança do consumidor com buídos no âmbito do SUS e por outros órgãos governa-
deficiência, em caso de sua utilização, aplicando-se, no que mentais.
couber, os arts. 30 a 41 da Lei no 8.078, de 11 de setembro de 1990. Parágrafo único. Para fazer cumprir o disposto neste
§ 1o  Os canais de comercialização virtual e os anúncios artigo, os procedimentos constantes do plano específico
publicitários veiculados na imprensa escrita, na internet, no de medidas deverão ser avaliados, pelo menos, a cada 2
rádio, na televisão e nos demais veículos de comunicação (dois) anos.
abertos ou por assinatura devem disponibilizar, conforme
a compatibilidade do meio, os recursos de acessibilidade CAPÍTULO IV
de que trata o art. 67 desta Lei, a expensas do fornecedor DO DIREITO À PARTICIPAÇÃO NA VIDA PÚBLICA E
do produto ou do serviço, sem prejuízo da observância do POLÍTICA
disposto nos arts. 36 a 38 da Lei no 8.078, de 11 de setembro de 1990.
§ 2o   Os fornecedores devem disponibilizar, median- Art. 76.  O poder público deve garantir à pessoa com
te solicitação, exemplares de bulas, prospectos, textos ou deficiência todos os direitos políticos e a oportunidade de
qualquer outro tipo de material de divulgação em formato exercê-los em igualdade de condições com as demais pes-
acessível. soas.

199
LEGISLAÇÃO

§ 1o  À pessoa com deficiência será assegurado o direi- Art. 78.  Devem ser estimulados a pesquisa, o desen-
to de votar e de ser votada, inclusive por meio das seguin- volvimento, a inovação e a difusão de tecnologias volta-
tes ações: das para ampliar o acesso da pessoa com deficiência às
I - garantia de que os procedimentos, as instalações, os tecnologias da informação e comunicação e às tecnolo-
materiais e os equipamentos para votação sejam apropria- gias sociais.
dos, acessíveis a todas as pessoas e de fácil compreensão e Parágrafo único.  Serão estimulados, em especial:
uso, sendo vedada a instalação de seções eleitorais exclu- I - o emprego de tecnologias da informação e comu-
sivas para a pessoa com deficiência; nicação como instrumento de superação de limitações
II - incentivo à pessoa com deficiência a candidatar-se funcionais e de barreiras à comunicação, à informação, à
e a desempenhar quaisquer funções públicas em todos educação e ao entretenimento da pessoa com deficiência;
os níveis de governo, inclusive por meio do uso de novas II - a adoção de soluções e a difusão de normas que
tecnologias assistivas, quando apropriado; visem a ampliar a acessibilidade da pessoa com deficiência
III - garantia de que os pronunciamentos oficiais, a à computação e aos sítios da internet, em especial aos ser-
propaganda eleitoral obrigatória e os debates transmiti- viços de governo eletrônico.
dos pelas emissoras de televisão possuam, pelo menos, os
recursos elencados no art. 67 desta Lei; LIVRO II
IV - garantia do livre exercício do direito ao voto e, PARTE ESPECIAL
para tanto, sempre que necessário e a seu pedido, per- TÍTULO I
DO ACESSO À JUSTIÇA
missão para que a pessoa com deficiência seja auxiliada
CAPÍTULO I
na votação por pessoa de sua escolha.
DISPOSIÇÕES GERAIS
§ 2o   O poder público promoverá a participação da
pessoa com deficiência, inclusive quando institucionali-
Art. 79.  O poder público deve assegurar o acesso da
zada, na condução das questões públicas, sem discrimi- pessoa com deficiência à justiça, em igualdade de oportu-
nação e em igualdade de oportunidades, observado o nidades com as demais pessoas, garantindo, sempre que
seguinte: requeridos, adaptações e recursos de tecnologia assistiva.
I - participação em organizações não governamentais § 1o   A fim de garantir a atuação da pessoa com defi-
relacionadas à vida pública e à política do País e em ativi- ciência em todo o processo judicial, o poder público deve
dades e administração de partidos políticos; capacitar os membros e os servidores que atuam no Poder
II - formação de organizações para representar a pes- Judiciário, no Ministério Público, na Defensoria Pública, nos
soa com deficiência em todos os níveis; órgãos de segurança pública e no sistema penitenciário
III - participação da pessoa com deficiência em orga- quanto aos direitos da pessoa com deficiência.
nizações que a representem. § 2o  Devem ser assegurados à pessoa com deficiência
submetida a medida restritiva de liberdade todos os direi-
TÍTULO IV tos e garantias a que fazem jus os apenados sem deficiên-
DA CIÊNCIA E TECNOLOGIA cia, garantida a acessibilidade.
§ 3o  A Defensoria Pública e o Ministério Público toma-
Art. 77.  O poder público deve fomentar o desenvolvi- rão as medidas necessárias à garantia dos direitos previstos
mento científico, a pesquisa e a inovação e a capacitação nesta Lei.
tecnológicas, voltados à melhoria da qualidade de vida Art. 80.  Devem ser oferecidos todos os recursos de
e ao trabalho da pessoa com deficiência e sua inclusão tecnologia assistiva disponíveis para que a pessoa com de-
social. ficiência tenha garantido o acesso à justiça, sempre que
§ 1o   O fomento pelo poder público deve priorizar a figure em um dos polos da ação ou atue como testemunha,
geração de conhecimentos e técnicas que visem à pre- partícipe da lide posta em juízo, advogado, defensor públi-
venção e ao tratamento de deficiências e ao desenvolvi- co, magistrado ou membro do Ministério Público.
mento de tecnologias assistiva e social. Parágrafo único.  A pessoa com deficiência tem garan-
§ 2o  A acessibilidade e as tecnologias assistiva e social tido o acesso ao conteúdo de todos os atos processuais de
seu interesse, inclusive no exercício da advocacia.
devem ser fomentadas mediante a criação de cursos de
Art. 81.   Os direitos da pessoa com deficiência serão
pós-graduação, a formação de recursos humanos e a in-
garantidos por ocasião da aplicação de sanções penais.
clusão do tema nas diretrizes de áreas do conhecimento.
Art. 82.  (VETADO).
§ 3o  Deve ser fomentada a capacitação tecnológica de
Art. 83.  Os serviços notariais e de registro não podem
instituições públicas e privadas para o desenvolvimento negar ou criar óbices ou condições diferenciadas à presta-
de tecnologias assistiva e social que sejam voltadas para ção de seus serviços em razão de deficiência do solicitante,
melhoria da funcionalidade e da participação social da devendo reconhecer sua capacidade legal plena, garantida
pessoa com deficiência. a acessibilidade.
§ 4o  As medidas previstas neste artigo devem ser rea- Parágrafo único.  O descumprimento do disposto
valiadas periodicamente pelo poder público, com vistas no caput deste artigo constitui discriminação em razão de
ao seu aperfeiçoamento. deficiência.

200
LEGISLAÇÃO

CAPÍTULO II Art. 89.  Apropriar-se de ou desviar bens, proventos,


DO RECONHECIMENTO IGUAL PERANTE A LEI pensão, benefícios, remuneração ou qualquer outro rendi-
mento de pessoa com deficiência:
Art. 84.  A pessoa com deficiência tem assegurado o di- Pena - reclusão, de 1 (um) a 4 (quatro) anos, e multa.
reito ao exercício de sua capacidade legal em igualdade de Parágrafo único.  Aumenta-se a pena em 1/3 (um ter-
condições com as demais pessoas. ço) se o crime é cometido:
§ 1o  Quando necessário, a pessoa com deficiência será I - por tutor, curador, síndico, liquidatário, inventarian-
submetida à curatela, conforme a lei. te, testamenteiro ou depositário judicial; ou
§ 2o  É facultado à pessoa com deficiência a adoção de II - por aquele que se apropriou em razão de ofício ou
processo de tomada de decisão apoiada. de profissão.
§ 3o  A definição de curatela de pessoa com deficiência Art. 90.  Abandonar pessoa com deficiência em hospi-
constitui medida protetiva extraordinária, proporcional às tais, casas de saúde, entidades de abrigamento ou congê-
necessidades e às circunstâncias de cada caso, e durará o neres:
menor tempo possível. Pena - reclusão, de 6 (seis) meses a 3 (três) anos, e mul-
§ 4o  Os curadores são obrigados a prestar, anualmente, ta.
contas de sua administração ao juiz, apresentando o balanço Parágrafo único.  Na mesma pena incorre quem não
do respectivo ano. prover as necessidades básicas de pessoa com deficiência
Art. 85.  A curatela afetará tão somente os atos relacio- quando obrigado por lei ou mandado.
nados aos direitos de natureza patrimonial e negocial. Art. 91.  Reter ou utilizar cartão magnético, qualquer
§ 1o    A definição da curatela não alcança o direito ao meio eletrônico ou documento de pessoa com deficiência
próprio corpo, à sexualidade, ao matrimônio, à privacidade, destinados ao recebimento de benefícios, proventos, pen-
à educação, à saúde, ao trabalho e ao voto. sões ou remuneração ou à realização de operações finan-
§ 2o  A curatela constitui medida extraordinária, devendo ceiras, com o fim de obter vantagem indevida para si ou
constar da sentença as razões e motivações de sua defini- para outrem:
ção, preservados os interesses do curatelado. Pena - detenção, de 6 (seis) meses a 2 (dois) anos, e
§ 3o  No caso de pessoa em situação de institucionaliza- multa.
ção, ao nomear curador, o juiz deve dar preferência a pessoa Parágrafo único.  Aumenta-se a pena em 1/3 (um ter-
que tenha vínculo de natureza familiar, afetiva ou comunitá- ço) se o crime é cometido por tutor ou curador.
ria com o curatelado.
Art. 86.  Para emissão de documentos oficiais, não será TÍTULO III
exigida a situação de curatela da pessoa com deficiência. DISPOSIÇÕES FINAIS E TRANSITÓRIAS
Art. 87.  Em casos de relevância e urgência e a fim de
proteger os interesses da pessoa com deficiência em situa- Art. 92.  É criado o Cadastro Nacional de Inclusão da
ção de curatela, será lícito ao juiz, ouvido o Ministério Pú- Pessoa com Deficiência (Cadastro-Inclusão), registro públi-
blico, de oficio ou a requerimento do interessado, nomear, co eletrônico com a finalidade de coletar, processar, sis-
desde logo, curador provisório, o qual estará sujeito, no que tematizar e disseminar informações georreferenciadas que
couber, às disposições do Código de Processo Civil. permitam a identificação e a caracterização socioeconômi-
ca da pessoa com deficiência, bem como das barreiras que
TÍTULO II impedem a realização de seus direitos.
DOS CRIMES E DAS INFRAÇÕES ADMINISTRATIVAS § 1o  O Cadastro-Inclusão será administrado pelo Poder
Executivo federal e constituído por base de dados, instru-
Art. 88.  Praticar, induzir ou incitar discriminação de pes- mentos, procedimentos e sistemas eletrônicos.
soa em razão de sua deficiência: § 2o  Os dados constituintes do Cadastro-Inclusão serão
Pena - reclusão, de 1 (um) a 3 (três) anos, e multa. obtidos pela integração dos sistemas de informação e da
§ 1o  Aumenta-se a pena em 1/3 (um terço) se a vítima base de dados de todas as políticas públicas relacionadas
encontrar-se sob cuidado e responsabilidade do agente. aos direitos da pessoa com deficiência, bem como por in-
§ 2o   Se qualquer dos crimes previstos no caput deste formações coletadas, inclusive em censos nacionais e nas
artigo é cometido por intermédio de meios de comunicação demais pesquisas realizadas no País, de acordo com os pa-
social ou de publicação de qualquer natureza: râmetros estabelecidos pela Convenção sobre os Direitos
Pena - reclusão, de 2 (dois) a 5 (cinco) anos, e multa. das Pessoas com Deficiência e seu Protocolo Facultativo.
§ 3o  Na hipótese do § 2o deste artigo, o juiz poderá de- § 3o   Para coleta, transmissão e sistematização de da-
terminar, ouvido o Ministério Público ou a pedido deste, ain- dos, é facultada a celebração de convênios, acordos, ter-
da antes do inquérito policial, sob pena de desobediência: mos de parceria ou contratos com instituições públicas e
I - recolhimento ou busca e apreensão dos exemplares privadas, observados os requisitos e procedimentos previs-
do material discriminatório; tos em legislação específica.
II - interdição das respectivas mensagens ou páginas de § 4o   Para assegurar a confidencialidade, a privacidade
informação na internet. e as liberdades fundamentais da pessoa com deficiência e
§ 4o   Na hipótese do § 2o  deste artigo, constitui efeito os princípios éticos que regem a utilização de informações,
da condenação, após o trânsito em julgado da decisão, a devem ser observadas as salvaguardas estabelecidas em
destruição do material apreendido. lei.

201
LEGISLAÇÃO

§ 5o  Os dados do Cadastro-Inclusão somente poderão “Art. 428.  ..................................................................


ser utilizados para as seguintes finalidades: ...........................................................................................
I - formulação, gestão, monitoramento e avaliação das § 6o  Para os fins do contrato de aprendizagem, a com-
políticas públicas para a pessoa com deficiência e para provação da escolaridade de aprendiz com deficiência
identificar as barreiras que impedem a realização de seus deve considerar, sobretudo, as habilidades e competências
direitos; relacionadas com a profissionalização.
II - realização de estudos e pesquisas. ...........................................................................................
§ 6o  As informações a que se refere este artigo devem § 8o  Para o aprendiz com deficiência com 18 (dezoito)
ser disseminadas em formatos acessíveis. anos ou mais, a validade do contrato de aprendizagem
Art. 93.  Na realização de inspeções e de auditorias pe- pressupõe anotação na CTPS e matrícula e frequência em
los órgãos de controle interno e externo, deve ser obser- programa de aprendizagem desenvolvido sob orientação
vado o cumprimento da legislação relativa à pessoa com de entidade qualificada em formação técnico-profissional
deficiência e das normas de acessibilidade vigentes. metódica.” (NR)
Art. 94.  Terá direito a auxílio-inclusão, nos termos da “Art. 433.  ..................................................................
lei, a pessoa com deficiência moderada ou grave que: ...........................................................................................
I - receba o benefício de prestação continuada previsto I -  desempenho insuficiente ou inadaptação do
no art. 20 da Lei no 8.742, de 7 de dezembro de 1993, e que passe a exer- aprendiz, salvo para o aprendiz com deficiência quando
cer atividade remunerada que a enquadre como segurado
desprovido de recursos de acessibilidade, de tecnologias
obrigatório do RGPS;
assistivas e de apoio necessário ao desempenho de suas
II - tenha recebido, nos últimos 5 (cinco) anos, o be-
atividades;
nefício de prestação continuada previsto no art. 20 da Lei
..................................................................................” (NR)
no 8.742, de 7 de dezembro de 1993, e que exerça atividade remunerada
que a enquadre como segurado obrigatório do RGPS. Art. 98.  A Lei no 7.853, de 24 de outubro de 1989, passa a vigorar
Art. 95.  É vedado exigir o comparecimento de pessoa com as seguintes alterações:
com deficiência perante os órgãos públicos quando seu “Art. 3o   As medidas judiciais destinadas à proteção
deslocamento, em razão de sua limitação funcional e de de interesses coletivos, difusos, individuais homogêneos e
condições de acessibilidade, imponha-lhe ônus despropor- individuais indisponíveis da pessoa com deficiência pode-
cional e indevido, hipótese na qual serão observados os rão ser propostas pelo Ministério Público, pela Defensoria
seguintes procedimentos: Pública, pela União, pelos Estados, pelos Municípios, pelo
I - quando for de interesse do poder público, o agente Distrito Federal, por associação constituída há mais de 1
promoverá o contato necessário com a pessoa com defi- (um) ano, nos termos da lei civil, por autarquia, por em-
ciência em sua residência; presa pública e por fundação ou sociedade de economia
II - quando for de interesse da pessoa com deficiência, mista que inclua, entre suas finalidades institucionais, a
ela apresentará solicitação de atendimento domiciliar ou proteção dos interesses e a promoção de direitos da pes-
fará representar-se por procurador constituído para essa soa com deficiência.
finalidade. .................................................................................” (NR)
Parágrafo único.  É assegurado à pessoa com deficiên- “Art. 8o   Constitui crime punível com reclusão de 2
cia atendimento domiciliar pela perícia médica e social do (dois) a 5 (cinco) anos e multa:
Instituto Nacional do Seguro Social (INSS), pelo serviço pú- I - recusar, cobrar valores adicionais, suspender, pro-
blico de saúde ou pelo serviço privado de saúde, contra- crastinar, cancelar ou fazer cessar inscrição de aluno em
tado ou conveniado, que integre o SUS e pelas entidades estabelecimento de ensino de qualquer curso ou grau,
da rede socioassistencial integrantes do Suas, quando seu público ou privado, em razão de sua deficiência;
deslocamento, em razão de sua limitação funcional e de II - obstar inscrição em concurso público ou acesso de
condições de acessibilidade, imponha-lhe ônus despropor- alguém a qualquer cargo ou emprego público, em razão
cional e indevido. de sua deficiência;
Art. 96.  O § 6o-A do art. 135 da Lei no 4.737, de 15 de julho de 1965 III - negar ou obstar emprego, trabalho ou promoção
(Código Eleitoral)
, passa a vigorar com a seguinte redação:
à pessoa em razão de sua deficiência;
“Art. 135.  .................................................................
IV - recusar, retardar ou dificultar internação ou deixar
........................................................................................
de prestar assistência médico-hospitalar e ambulatorial à
§ 6o-A.  Os Tribunais Regionais Eleitorais deverão, a
pessoa com deficiência;
cada eleição, expedir instruções aos Juízes Eleitorais para
orientá-los na escolha dos locais de votação, de maneira V - deixar de cumprir, retardar ou frustrar execução de
a garantir acessibilidade para o eleitor com deficiência ou ordem judicial expedida na ação civil a que alude esta Lei;
com mobilidade reduzida, inclusive em seu entorno e nos VI - recusar, retardar ou omitir dados técnicos indis-
sistemas de transporte que lhe dão acesso. pensáveis à propositura da ação civil pública objeto desta
....................................................................................” (NR) Lei, quando requisitados.
Art. 97.  A Consolidação das Leis do Trabalho (CLT), § 1o   Se o crime for praticado contra pessoa com de-
aprovada pelo Decreto-Lei no  5.452, de 1o  de maio de 1943, passa a ficiência menor de 18 (dezoito) anos, a pena é agravada
vigorar com as seguintes alterações: em 1/3 (um terço).

202
LEGISLAÇÃO

§ 2o  A pena pela adoção deliberada de critérios subje- “Art. 93.  (VETADO):
tivos para indeferimento de inscrição, de aprovação e de I - (VETADO);
cumprimento de estágio probatório em concursos públi- II - (VETADO);
cos não exclui a responsabilidade patrimonial pessoal do III - (VETADO);
administrador público pelos danos causados. IV - (VETADO);
§ 3o   Incorre nas mesmas penas quem impede ou di- V - (VETADO).
ficulta o ingresso de pessoa com deficiência em planos § 1o  A dispensa de pessoa com deficiência ou de bene-
privados de assistência à saúde, inclusive com cobrança ficiário reabilitado da Previdência Social ao final de contra-
de valores diferenciados. to por prazo determinado de mais de 90 (noventa) dias e
§ 4o   Se o crime for praticado em atendimento de ur- a dispensa imotivada em contrato por prazo indetermina-
gência e emergência, a pena é agravada em 1/3 (um ter- do somente poderão ocorrer após a contratação de outro
ço).” (NR) trabalhador com deficiência ou beneficiário reabilitado da
Art. 99.  O art. 20 da Lei no 8.036, de 11 de maio de 1990, passa a Previdência Social.
vigorar acrescido do seguinte inciso XVIII: § 2o   Ao Ministério do Trabalho e Emprego incumbe
“Art. 20.  ...................................................................... estabelecer a sistemática de fiscalização, bem como gerar
.............................................................................................. dados e estatísticas sobre o total de empregados e as va-
XVIII -  quando o trabalhador com deficiência, por gas preenchidas por pessoas com deficiência e por bene-
prescrição, necessite adquirir órtese ou prótese para pro- ficiários reabilitados da Previdência Social, fornecendo-os,
moção de acessibilidade e de inclusão social. quando solicitados, aos sindicatos, às entidades represen-
..................................................................................” (NR) tativas dos empregados ou aos cidadãos interessados.
Art. 100.  A Lei no  8.078, de 11 de setembro de 1990 (Código de Defesa do § 3o  Para a reserva de cargos será considerada somente
a contratação direta de pessoa com deficiência, excluído o
Consumidor)
, passa a vigorar com as seguintes alterações:
aprendiz com deficiência de que trata a Consolidação das
“Art. 6o  .......................................................................
Leis do Trabalho (CLT), aprovada pelo Decreto-Lei no 5.452,
............................................................................................
de 1o de maio de 1943.
Parágrafo único.  A informação de que trata o inciso
§ 4o  (VETADO).” (NR)
III do caput deste artigo deve ser acessível à pessoa com
“Art. 110-A.  No ato de requerimento de benefícios
deficiência, observado o disposto em regulamento.” (NR)
operacionalizados pelo INSS, não será exigida apresenta-
“Art. 43.  ......................................................................
ção de termo de curatela de titular ou de beneficiário com
............................................................................................
deficiência, observados os procedimentos a serem estabe-
§ 6o  Todas as informações de que trata o caput deste
lecidos em regulamento.”
artigo devem ser disponibilizadas em formatos acessíveis, Art. 102.  O art. 2o da Lei no 8.313, de 23 de dezembro de 1991, passa
inclusive para a pessoa com deficiência, mediante solici- a vigorar acrescido do seguinte § 3o:
tação do consumidor.” (NR) “Art. 2o  .........................................................................
Art. 101.  A Lei no  8.213, de 24 de julho de 1991, passa a vigorar .............................................................................................
com as seguintes alterações: § 3o   Os incentivos criados por esta Lei somente serão
“Art. 16.  ...................................................................... concedidos a projetos culturais que forem disponibilizados,
I - o cônjuge, a companheira, o companheiro e o fi- sempre que tecnicamente possível, também em formato
lho não emancipado, de qualquer condição, menor de 21 acessível à pessoa com deficiência, observado o disposto
(vinte e um) anos ou inválido ou que tenha deficiência em regulamento.” (NR)
intelectual ou mental ou deficiência grave; Art. 103.  O art. 11 da Lei no 8.429, de 2 de junho de 1992, passa a
............................................................................................ vigorar acrescido do seguinte inciso IX:
III - o irmão não emancipado, de qualquer condição, “Art. 11.  .....................................................................
menor de 21 (vinte e um) anos ou inválido ou que tenha ............................................................................................
deficiência intelectual ou mental ou deficiência grave; IX - deixar de cumprir a exigência de requisitos de
.................................................................................” (NR) acessibilidade previstos na legislação.” (NR)
“Art. 77.  ..................................................................... Art. 104.  A Lei no  8.666, de 21 de junho de 1993, passa a vigorar
............................................................................................ com as seguintes alterações:
§ 2o  .............................................................................. “Art. 3o  .....................................................................
............................................................................................ ..........................................................................................
II - para o filho, a pessoa a ele equiparada ou o irmão, § 2o  ...........................................................................
de ambos os sexos, pela emancipação ou ao completar 21 ..........................................................................................
(vinte e um) anos de idade, salvo se for inválido ou tiver V - produzidos ou prestados por empresas que com-
deficiência intelectual ou mental ou deficiência grave; provem cumprimento de reserva de cargos prevista em lei
................................................................................... para pessoa com deficiência ou para reabilitado da Previ-
§ 4o  (VETADO). dência Social e que atendam às regras de acessibilidade
...................................................................................” (NR) previstas na legislação.
...........................................................................................

203
LEGISLAÇÃO

§ 5o  Nos processos de licitação, poderá ser estabeleci- I - a reintegração com ressarcimento integral de todo o
da margem de preferência para: período de afastamento, mediante pagamento das remu-
I - produtos manufaturados e para serviços nacionais nerações devidas, corrigidas monetariamente e acrescidas
que atendam a normas técnicas brasileiras; e de juros legais;
II - bens e serviços produzidos ou prestados por em- ....................................................................................” (NR)
presas que comprovem cumprimento de reserva de cargos Art. 108. O art. 35 da Lei no 9.250, de 26 de dezembro de 1995, passa
prevista em lei para pessoa com deficiência ou para rea- a vigorar acrescido do seguinte § 5o:
bilitado da Previdência Social e que atendam às regras de “Art. 35.  ......................................................................
acessibilidade previstas na legislação. .............................................................................................
...................................................................................” (NR) § 5o  Sem prejuízo do disposto no inciso IX do parágrafo
“Art. 66-A.  As empresas enquadradas no inciso V do § único do art. 3o da Lei no 10.741, de 1ode outubro de 2003, a pessoa
2o e no inciso II do § 5o do art. 3o desta Lei deverão cumprir, com deficiência, ou o contribuinte que tenha dependente
durante todo o período de execução do contrato, a reserva nessa condição, tem preferência na restituição referida no
de cargos prevista em lei para pessoa com deficiência ou
inciso III do art. 4o e na alínea “c” do inciso II do art. 8o.” (NR)
para reabilitado da Previdência Social, bem como as regras
Art. 109.  A Lei no 9.503, de 23 de setembro de 1997 (Código de Trânsito Brasileiro),
de acessibilidade previstas na legislação.
passa a vigorar com as seguintes alterações:
Parágrafo único.   Cabe à administração fiscalizar o
“Art. 2o  ...........................................................
cumprimento dos requisitos de acessibilidade nos serviços
e nos ambientes de trabalho.” Parágrafo único.  Para os efeitos deste Código, são
Art. 105.  O art. 20 da Lei no 8.742, de 7 de dezembro de 1993, passa consideradas vias terrestres as praias abertas à circulação
a vigorar com as seguintes alterações: pública, as vias internas pertencentes aos condomínios
“Art. 20.  ...................................................................... constituídos por unidades autônomas e as vias e áreas de
............................................................................................. estacionamento de estabelecimentos privados de uso co-
§ 2o   Para efeito de concessão do benefício de pres- letivo.” (NR)
tação continuada, considera-se pessoa com deficiência “Art. 86-A.  As vagas de estacionamento regulamen-
aquela que tem impedimento de longo prazo de natureza tado de que trata o inciso XVII do art. 181 desta Lei de-
física, mental, intelectual ou sensorial, o qual, em interação verão ser sinalizadas com as respectivas placas indicativas
com uma ou mais barreiras, pode obstruir sua participação de destinação e com placas informando os dados sobre a
plena e efetiva na sociedade em igualdade de condições infração por estacionamento indevido.”
com as demais pessoas. “Art. 147-A.   Ao candidato com deficiência auditiva é
............................................................................................ assegurada acessibilidade de comunicação, mediante em-
§ 9o   Os rendimentos decorrentes de estágio supervi- prego de tecnologias assistivas ou de ajudas técnicas em
sionado e de aprendizagem não serão computados para os todas as etapas do processo de habilitação.
fins de cálculo da renda familiar per capita a que se refere § 1o  O material didático audiovisual utilizado em aulas
o § 3o deste artigo. teóricas dos cursos que precedem os exames previstos no
............................................................................................. art. 147 desta Lei deve ser acessível, por meio de subtitu-
§ 11.  Para concessão do benefício de que trata lação com legenda oculta associada à tradução simultânea
o  caput  deste artigo, poderão ser utilizados outros ele- em Libras.
mentos probatórios da condição de miserabilidade do § 2o  É assegurado também ao candidato com deficiên-
grupo familiar e da situação de vulnerabilidade, conforme cia auditiva requerer, no ato de sua inscrição, os serviços
regulamento.” (NR) de intérprete da Libras, para acompanhamento em aulas
Art. 106.  (VETADO). práticas e teóricas.”
Art. 107.  A Lei no 9.029, de 13 de abril de 1995, passa a vigorar com
“Art. 154.  (VETADO).”
as seguintes alterações:
“Art. 181.  ...................................................................
“Art. 1o   É proibida a adoção de qualquer prática dis-
..........................................................................................
criminatória e limitativa para efeito de acesso à relação
XVII - .........................................................................
de trabalho, ou de sua manutenção, por motivo de sexo,
origem, raça, cor, estado civil, situação familiar, deficiência, Infração - grave;
reabilitação profissional, idade, entre outros, ressalvadas, .................................................................................” (NR)
nesse caso, as hipóteses de proteção à criança e ao ado- Art. 110.  O inciso VI e o § 1o do art. 56 da Lei no 9.615, de 24
lescente previstas no inciso XXXIII do art. 7oda Constituição
de março de 1998
, passam a vigorar com a seguinte redação:
Federal.” (NR) “Art. 56.  ....................................................................
“Art. 3o  Sem prejuízo do prescrito no art. 2o desta Lei e ...........................................................................................
nos dispositivos legais que tipificam os crimes resultantes VI - 2,7% (dois inteiros e sete décimos por cento) da
de preconceito de etnia, raça, cor ou deficiência, as infra- arrecadação bruta dos concursos de prognósticos e lo-
ções ao disposto nesta Lei são passíveis das seguintes co- terias federais e similares cuja realização estiver sujeita a
minações: autorização federal, deduzindo-se esse valor do montante
..................................................................................” (NR) destinado aos prêmios;
“Art. 4o  ........................................................................ .............................................................................................

204
LEGISLAÇÃO

§ 1o    Do total de recursos financeiros resultantes do V - acompanhante: aquele que acompanha a pessoa
percentual de que trata o inciso VI do caput, 62,96% (ses- com deficiência, podendo ou não desempenhar as fun-
senta e dois inteiros e noventa e seis centésimos por cen- ções de atendente pessoal;
to) serão destinados ao Comitê Olímpico Brasileiro (COB) VI - elemento de urbanização: quaisquer componen-
e 37,04% (trinta e sete inteiros e quatro centésimos por tes de obras de urbanização, tais como os referentes a pa-
cento) ao Comitê Paralímpico Brasileiro (CPB), devendo vimentação, saneamento, encanamento para esgotos, dis-
ser observado, em ambos os casos, o conjunto de normas tribuição de energia elétrica e de gás, iluminação pública,
aplicáveis à celebração de convênios pela União. serviços de comunicação, abastecimento e distribuição de
..................................................................................” (NR) água, paisagismo e os que materializam as indicações do
Art. 111.  O art. 1o da Lei no 10.048, de 8 de novembro de 2000, passa planejamento urbanístico;
a vigorar com a seguinte redação: VII - mobiliário urbano: conjunto de objetos existentes
“Art. 1o    As pessoas com deficiência, os idosos com nas vias e nos espaços públicos, superpostos ou adicio-
idade igual ou superior a 60 (sessenta) anos, as gestantes, nados aos elementos de urbanização ou de edificação, de
as lactantes, as pessoas com crianças de colo e os obesos forma que sua modificação ou seu traslado não provoque
terão atendimento prioritário, nos termos desta Lei.” (NR) alterações substanciais nesses elementos, tais como semá-
Art. 112.  A Lei no 10.098, de 19 de dezembro de 2000, passa a vigorar foros, postes de sinalização e similares, terminais e pontos
com as seguintes alterações: de acesso coletivo às telecomunicações, fontes de água,
“Art. 2o  ....................................................................... lixeiras, toldos, marquises, bancos, quiosques e quaisquer
I - acessibilidade: possibilidade e condição de alcance outros de natureza análoga;
para utilização, com segurança e autonomia, de espaços, VIII - tecnologia assistiva ou ajuda técnica: produtos,
mobiliários, equipamentos urbanos, edificações, transpor- equipamentos, dispositivos, recursos, metodologias, es-
tes, informação e comunicação, inclusive seus sistemas e tratégias, práticas e serviços que objetivem promover a
tecnologias, bem como de outros serviços e instalações funcionalidade, relacionada à atividade e à participação da
abertos ao público, de uso público ou privados de uso pessoa com deficiência ou com mobilidade reduzida, vi-
coletivo, tanto na zona urbana como na rural, por pessoa sando à sua autonomia, independência, qualidade de vida
com deficiência ou com mobilidade reduzida; e inclusão social;
II - barreiras: qualquer entrave, obstáculo, atitude ou IX - comunicação: forma de interação dos cidadãos
comportamento que limite ou impeça a participação so- que abrange, entre outras opções, as línguas, inclusive a
cial da pessoa, bem como o gozo, a fruição e o exercício Língua Brasileira de Sinais (Libras), a visualização de textos,
de seus direitos à acessibilidade, à liberdade de movimen- o Braille, o sistema de sinalização ou de comunicação tátil,
to e de expressão, à comunicação, ao acesso à informa- os caracteres ampliados, os dispositivos multimídia, assim
ção, à compreensão, à circulação com segurança, entre como a linguagem simples, escrita e oral, os sistemas audi-
outros, classificadas em: tivos e os meios de voz digitalizados e os modos, meios e
a) barreiras urbanísticas: as existentes nas vias e nos formatos aumentativos e alternativos de comunicação, in-
espaços públicos e privados abertos ao público ou de uso cluindo as tecnologias da informação e das comunicações;
coletivo; X - desenho universal: concepção de produtos, am-
b) barreiras arquitetônicas: as existentes nos edifícios bientes, programas e serviços a serem usados por todas
públicos e privados; as pessoas, sem necessidade de adaptação ou de projeto
c) barreiras nos transportes: as existentes nos siste- específico, incluindo os recursos de tecnologia assistiva.”
mas e meios de transportes; (NR)
d) barreiras nas comunicações e na informação: qual- “Art. 3o   O planejamento e a urbanização das vias pú-
quer entrave, obstáculo, atitude ou comportamento que blicas, dos parques e dos demais espaços de uso público
dificulte ou impossibilite a expressão ou o recebimento deverão ser concebidos e executados de forma a torná-los
de mensagens e de informações por intermédio de siste- acessíveis para todas as pessoas, inclusive para aquelas
mas de comunicação e de tecnologia da informação; com deficiência ou com mobilidade reduzida.
III - pessoa com deficiência: aquela que tem impedi- Parágrafo único.  O passeio público, elemento obriga-
mento de longo prazo de natureza física, mental, intelec- tório de urbanização e parte da via pública, normalmente
tual ou sensorial, o qual, em interação com uma ou mais segregado e em nível diferente, destina-se somente à cir-
barreiras, pode obstruir sua participação plena e efetiva culação de pedestres e, quando possível, à implantação de
na sociedade em igualdade de condições com as demais mobiliário urbano e de vegetação.” (NR)
pessoas; “Art. 9o  ........................................................................
IV - pessoa com mobilidade reduzida: aquela que te- Parágrafo único. Os semáforos para pedestres instala-
nha, por qualquer motivo, dificuldade de movimentação, dos em vias públicas de grande circulação, ou que deem
permanente ou temporária, gerando redução efetiva da acesso aos serviços de reabilitação, devem obrigatoria-
mobilidade, da flexibilidade, da coordenação motora ou mente estar equipados com mecanismo que emita sinal
da percepção, incluindo idoso, gestante, lactante, pessoa sonoro suave para orientação do pedestre.” (NR)
com criança de colo e obeso;

205
LEGISLAÇÃO

“Art. 10-A.  A instalação de qualquer mobiliário urbano III - (Revogado);


em área de circulação comum para pedestre que ofereça .............................................................................................
risco de acidente à pessoa com deficiência deverá ser indi- § 1o  ..............................................................................
cada mediante sinalização tátil de alerta no piso, de acordo § 2o  A pessoa com deficiência poderá testemunhar em
com as normas técnicas pertinentes.” igualdade de condições com as demais pessoas, sendo-lhe
“Art. 12-A.  Os centros comerciais e os estabelecimen- assegurados todos os recursos de tecnologia assistiva.”
tos congêneres devem fornecer carros e cadeiras de rodas, (NR)
motorizados ou não, para o atendimento da pessoa com “Art. 1.518.  Até a celebração do casamento podem os
deficiência ou com mobilidade reduzida.” pais ou tutores revogar a autorização.” (NR)
Art. 113.  A Lei no 10.257, de 10 de julho de 2001 (Estatuto da Cidade), passa “Art. 1.548.  ...................................................................
a vigorar com as seguintes alterações: I - (Revogado);
“Art. 3o  ...................................................................... ....................................................................................” (NR)
............................................................................................ “Art. 1.550.  ..................................................................
III - promover, por iniciativa própria e em conjunto com .............................................................................................
os Estados, o Distrito Federal e os Municípios, programas § 1o  ..............................................................................
de construção de moradias e melhoria das condições ha- § 2o  A pessoa com deficiência mental ou intelectual em
bitacionais, de saneamento básico, das calçadas, dos pas- idade núbia poderá contrair matrimônio, expressando sua
seios públicos, do mobiliário urbano e dos demais espaços vontade diretamente ou por meio de seu responsável ou
de uso público; curador.” (NR)
IV - instituir diretrizes para desenvolvimento urbano, “Art. 1.557.  ................................................................
inclusive habitação, saneamento básico, transporte e mo- ............................................................................................
bilidade urbana, que incluam regras de acessibilidade aos III - a ignorância, anterior ao casamento, de defeito fí-
locais de uso público; sico irremediável que não caracterize deficiência ou de mo-
.................................................................................” (NR) léstia grave e transmissível, por contágio ou por herança,
“Art. 41.  .................................................................... capaz de pôr em risco a saúde do outro cônjuge ou de sua
........................................................................................... descendência;
§ 3o  As cidades de que trata o caput deste artigo de- IV - (Revogado).” (NR)
vem elaborar plano de rotas acessíveis, compatível com o “Art. 1.767.  ..................................................................
plano diretor no qual está inserido, que disponha sobre os I - aqueles que, por causa transitória ou permanente,
passeios públicos a serem implantados ou reformados pelo não puderem exprimir sua vontade;
poder público, com vistas a garantir acessibilidade da pes- II - (Revogado);
soa com deficiência ou com mobilidade reduzida a todas III - os ébrios habituais e os viciados em tóxico;
as rotas e vias existentes, inclusive as que concentrem os IV - (Revogado);
focos geradores de maior circulação de pedestres, como ....................................................................................” (NR)
os órgãos públicos e os locais de prestação de serviços “Art. 1.768.  O processo que define os termos da cura-
públicos e privados de saúde, educação, assistência social, tela deve ser promovido:
esporte, cultura, correios e telégrafos, bancos, entre outros, .............................................................................................
sempre que possível de maneira integrada com os sistemas IV - pela própria pessoa.” (NR)
de transporte coletivo de passageiros.” (NR) “Art. 1.769.  O Ministério Público somente promoverá o
Art. 114.  A Lei no 10.406, de 10 de janeiro de 2002 (Código Civil), passa a processo que define os termos da curatela:
vigorar com as seguintes alterações: I - nos casos de deficiência mental ou intelectual;
“Art. 3o   São absolutamente incapazes de exercer pes- ............................................................................................
soalmente os atos da vida civil os menores de 16 (dezes- III - se, existindo, forem menores ou incapazes as pes-
seis) anos. soas mencionadas no inciso II.” (NR)
I - (Revogado); “Art. 1.771.  Antes de se pronunciar acerca dos termos
II - (Revogado); da curatela, o juiz, que deverá ser assistido por equipe mul-
III - (Revogado).” (NR) tidisciplinar, entrevistará pessoalmente o interditando.”
“Art. 4o  São incapazes, relativamente a certos atos ou à (NR)
maneira de os exercer: “Art. 1.772.  O juiz determinará, segundo as potencia-
..................................................................................... lidades da pessoa, os limites da curatela, circunscritos às
II - os ébrios habituais e os viciados em tóxico; restrições constantes do art. 1.782, e indicará curador.
III - aqueles que, por causa transitória ou permanente, Parágrafo único.  Para a escolha do curador, o juiz leva-
não puderem exprimir sua vontade; rá em conta a vontade e as preferências do interditando, a
............................................................................................. ausência de conflito de interesses e de influência indevida,
Parágrafo único.  A capacidade dos indígenas será re- a proporcionalidade e a adequação às circunstâncias da
gulada por legislação especial.” (NR) pessoa.” (NR)
“Art. 228.  ..................................................................... “Art. 1.775-A.  Na nomeação de curador para a pessoa
............................................................................................. com deficiência, o juiz poderá estabelecer curatela com-
II - (Revogado); partilhada a mais de uma pessoa.”

206
LEGISLAÇÃO

“Art. 1.777.   As pessoas referidas no inciso I do art. § 9o  A pessoa apoiada pode, a qualquer tempo, solici-
1.767 receberão todo o apoio necessário para ter preserva- tar o término de acordo firmado em processo de tomada
do o direito à convivência familiar e comunitária, sendo evi- de decisão apoiada.
tado o seu recolhimento em estabelecimento que os afaste § 10.  O apoiador pode solicitar ao juiz a exclusão de
desse convívio.” (NR) sua participação do processo de tomada de decisão apoia-
Art. 115.  O Título IV do Livro IV da Parte Especial da da, sendo seu desligamento condicionado à manifestação
Lei no  10.406, de 10 de janeiro de 2002 (Código Civil), passa a vigorar com a do juiz sobre a matéria.
seguinte redação: § 11.  Aplicam-se à tomada de decisão apoiada, no que
couber, as disposições referentes à prestação de contas na
“TÍTULO IV curatela.”
Da Tutela, da Curatela e da Tomada de Decisão Art. 117.  O art. 1o da Lei no 11.126, de 27 de junho de 2005, passa a
Apoiada” vigorar com a seguinte redação:
“Art. 1o   É assegurado à pessoa com deficiência visual
Art. 116.  O Título IV do Livro IV da Parte Especial da Lei acompanhada de cão-guia o direito de ingressar e de per-
no 10.406, de 10 de janeiro de 2002 (Código Civil), passa a vigorar acrescido do manecer com o animal em todos os meios de transporte e
seguinte Capítulo III: em estabelecimentos abertos ao público, de uso público e
privados de uso coletivo, desde que observadas as condi-
“CAPÍTULO III ções impostas por esta Lei.
Da Tomada de Decisão Apoiada .............................................................................................
§ 2o  O disposto no caput deste artigo aplica-se a todas
Art. 1.783-A.  A tomada de decisão apoiada é o proces- as modalidades e jurisdições do serviço de transporte cole-
so pelo qual a pessoa com deficiência elege pelo menos 2 tivo de passageiros, inclusive em esfera internacional com
(duas) pessoas idôneas, com as quais mantenha vínculos e origem no território brasileiro.” (NR)
que gozem de sua confiança, para prestar-lhe apoio na to- Art. 118.  O inciso IV do art. 46 da Lei no 11.904, de 14 de janeiro de
mada de decisão sobre atos da vida civil, fornecendo-lhes 2009
, passa a vigorar acrescido da seguinte alínea “k”:
os elementos e informações necessários para que possa “Art. 46.  ......................................................................
exercer sua capacidade. ...........................................................................................
§ 1o  Para formular pedido de tomada de decisão apoia- IV - ..............................................................................
da, a pessoa com deficiência e os apoiadores devem apre- ...........................................................................................
sentar termo em que constem os limites do apoio a ser k) de acessibilidade a todas as pessoas.
oferecido e os compromissos dos apoiadores, inclusive o .................................................................................” (NR)
prazo de vigência do acordo e o respeito à vontade, aos Art. 119.  A Lei no 12.587, de 3 de janeiro de 2012, passa a vigorar
direitos e aos interesses da pessoa que devem apoiar. acrescida do seguinte art. 12-B:
§ 2o   O pedido de tomada de decisão apoiada será “Art. 12-B.  Na outorga de exploração de serviço de
requerido pela pessoa a ser apoiada, com indicação ex- táxi, reservar-se-ão 10% (dez por cento) das vagas para
pressa das pessoas aptas a prestarem o apoio previsto condutores com deficiência.
no caput deste artigo. § 1o   Para concorrer às vagas reservadas na forma
§ 3o  Antes de se pronunciar sobre o pedido de tomada do caput deste artigo, o condutor com deficiência deverá
de decisão apoiada, o juiz, assistido por equipe multidis- observar os seguintes requisitos quanto ao veículo utiliza-
ciplinar, após oitiva do Ministério Público, ouvirá pessoal- do:
mente o requerente e as pessoas que lhe prestarão apoio. I - ser de sua propriedade e por ele conduzido; e
§ 4o   A decisão tomada por pessoa apoiada terá vali- II - estar adaptado às suas necessidades, nos termos da
dade e efeitos sobre terceiros, sem restrições, desde que legislação vigente.
esteja inserida nos limites do apoio acordado. § 2o  No caso de não preenchimento das vagas na for-
§ 5o   Terceiro com quem a pessoa apoiada mantenha ma estabelecida no caput deste artigo, as remanescentes
relação negocial pode solicitar que os apoiadores contra devem ser disponibilizadas para os demais concorrentes.”
assinem o contrato ou acordo, especificando, por escrito, Art. 120.  Cabe aos órgãos competentes, em cada esfe-
sua função em relação ao apoiado. ra de governo, a elaboração de relatórios circunstanciados
§ 6o  Em caso de negócio jurídico que possa trazer ris- sobre o cumprimento dos prazos estabelecidos por força
co ou prejuízo relevante, havendo divergência de opiniões das Leis no 10.048, de 8 de novembro de 2000, e no 10.098, de 19 de dezembro de 2000,
entre a pessoa apoiada e um dos apoiadores, deverá o juiz, bem como o seu encaminhamento ao Ministério Público
ouvido o Ministério Público, decidir sobre a questão. e aos órgãos de regulação para adoção das providências
§ 7o  Se o apoiador agir com negligência, exercer pres- cabíveis.
são indevida ou não adimplir as obrigações assumidas, Parágrafo único.  Os relatórios a que se refere
poderá a pessoa apoiada ou qualquer pessoa apresentar o  caput  deste artigo deverão ser apresentados no prazo
denúncia ao Ministério Público ou ao juiz. de 1 (um) ano a contar da entrada em vigor desta Lei.
§ 8o   Se procedente a denúncia, o juiz destituirá o Art. 121.  Os direitos, os prazos e as obrigações pre-
apoiador e nomeará, ouvida a pessoa apoiada e se for de vistos nesta Lei não excluem os já estabelecidos em outras
seu interesse, outra pessoa para prestação de apoio. legislações, inclusive em pactos, tratados, convenções e

207
LEGISLAÇÃO

declarações internacionais aprovados e promulgados pelo


Congresso Nacional, e devem ser aplicados em conformi- CÓDIGO DE ÉTICA DO/A ASSISTENTE
dade com as demais normas internas e acordos internacio- SOCIAL – LEI 8.662/93
nais vinculantes sobre a matéria.
Parágrafo único.  Prevalecerá a norma mais benéfica à
pessoa com deficiência.
Art. 122.  Regulamento disporá sobre a adequação do LEI No 8.662, DE 7 DE JUNHO DE 1993.
disposto nesta Lei ao tratamento diferenciado, simplificado
e favorecido a ser dispensado às microempresas e às em- Dispõe sobre a profissão de Assistente Social e dá ou-
presas de pequeno porte, previsto no § 3o do art. 1o da Lei Complemen- tras providências
tar no 123, de 14 de dezembro de 2006
.
Art. 123.  Revogam-se os seguintes dispositivos:      (Vi- O PRESIDENTE DA REPÚBLICA Faço saber que o Con-
gência) gresso Nacional decreta e eu sanciono a seguinte lei:
I - o inciso II do § 2o do art. 1o da Lei no 9.008, de 21 de março de 1995;
II - os incisos I, II e III do art. 3o da Lei no 10.406, de 10 de janeiro de Art. 1º É livre o exercício da profissão de Assistente So-
2002 
(Código Civil); cial em todo o território nacional, observadas as condições
III - os incisos II e III do art. 228 da Lei no 10.406, de 10 de janeiro estabelecidas nesta lei.
de 2002 
(Código Civil); Art. 2º Somente poderão exercer a profissão de Assis-
IV - o inciso I do art. 1.548 da Lei no 10.406, de 10 de janeiro de tente Social:
2002 
(Código Civil);
I - Os possuidores de diploma em curso de graduação
V - o inciso IV do art. 1.557 da Lei no 10.406, de 10 de janeiro de
em Serviço Social, oficialmente reconhecido, expedido por
2002 
(Código Civil);
estabelecimento de ensino superior existente no País, devi-
VI - os incisos II e IV do art. 1.767 da Lei no 10.406, de 10 de
janeiro de 2002 
(Código Civil); damente registrado no órgão competente;
VII - os arts. 1.776 e 1.780 da Lei no  10.406, de 10 de janeiro de II - os possuidores de diploma de curso superior em
2002 
(Código Civil). Serviço Social, em nível de graduação ou equivalente, ex-
Art. 124.  O § 1o do art. 2o desta Lei deverá entrar em vigor em pedido por estabelecimento de ensino sediado em países
até 2 (dois) anos, contados da entrada em vigor desta Lei. estrangeiros, conveniado ou não com o governo brasileiro,
Art. 125.  Devem ser observados os prazos a seguir dis- desde que devidamente revalidado e registrado em órgão
criminados, a partir da entrada em vigor desta Lei, para o competente no Brasil;
cumprimento dos seguintes dispositivos: III - os agentes sociais, qualquer que seja sua denomi-
I - incisos I e II do § 2o do art. 28, 48 (quarenta e oito) meses; nação com funções nos vários órgãos públicos, segundo o
II - § 6o do art. 44, 48 (quarenta e oito) meses; disposto no art. 14 e seu parágrafo único da Lei nº 1.889,
III - art. 45, 24 (vinte e quatro) meses; de 13 de junho de 1953.
IV - art. 49, 48 (quarenta e oito) meses. Parágrafo único. O exercício da profissão de Assistente
Art. 126.  Prorroga-se até 31 de dezembro de 2021 a Social requer prévio registro nos Conselhos Regionais que
vigência da Lei no 8.989, de 24 de fevereiro de 1995. tenham jurisdição sobre a área de atuação do interessado
Art. 127.  Esta Lei entra em vigor após decorridos 180 nos termos desta lei.
(cento e oitenta) dias de sua publicação oficial. Art. 3º A designação profissional de Assistente Social
Brasília, 6 de julho de 2015; 194o  da Independência e é privativa dos habilitados na forma da legislação vigente.
127o da República. Art. 4º Constituem competências do Assistente Social:
I - elaborar, implementar, executar e avaliar políticas
sociais junto a órgãos da administração pública, direta ou
indireta, empresas, entidades e organizações populares;
II - elaborar, coordenar, executar e avaliar planos, pro-
gramas e projetos que sejam do âmbito de atuação do Ser-
viço Social com participação da sociedade civil;
III - encaminhar providências, e prestar orientação so-
cial a indivíduos, grupos e à população;
IV - (Vetado);
V - orientar indivíduos e grupos de diferentes segmen-
tos sociais no sentido de identificar recursos e de fazer uso
dos mesmos no atendimento e na defesa de seus direitos;
VI - planejar, organizar e administrar benefícios e Ser-
viços Sociais;
VII - planejar, executar e avaliar pesquisas que possam
contribuir para a análise da realidade social e para subsidiar
ações profissionais;

208
LEGISLAÇÃO

VIII - prestar assessoria e consultoria a órgãos da ad- Art. 7º O Conselho Federal de Serviço Social (CFESS) e
ministração pública direta e indireta, empresas privadas e os Conselhos Regionais de Serviço Social (CRESS) consti-
outras entidades, com relação às matérias relacionadas no tuem, em seu conjunto, uma entidade com personalidade
inciso II deste artigo; jurídica e forma federativa, com o objetivo básico de dis-
IX - prestar assessoria e apoio aos movimentos sociais ciplinar e defender o exercício da profissão de Assistente
em matéria relacionada às políticas sociais, no exercício e Social em todo o território nacional.
na defesa dos direitos civis, políticos e sociais da coletivi- 1º Os Conselhos Regionais de Serviço Social (CRESS)
dade; são dotados de autonomia administrativa e financeira, sem
X - planejamento, organização e administração de Ser- prejuízo de sua vinculação ao Conselho Federal, nos ter-
viços Sociais e de Unidade de Serviço Social; mos da legislação em vigor.
XI - realizar estudos sócio-econômicos com os usuários 2º Cabe ao Conselho Federal de Serviço Social (CFESS)
para fins de benefícios e serviços sociais junto a órgãos da e aos Conselhos Regionais de Serviço Social (CRESS), re-
administração pública direta e indireta, empresas privadas presentar, em juízo e fora dele, os interesses gerais e indi-
e outras entidades. viduais dos Assistentes Sociais, no cumprimento desta lei.
Art. 8º Compete ao Conselho Federal de Serviço Social
Art. 5º Constituem atribuições privativas do Assistente
(CFESS), na qualidade de órgão normativo de grau supe-
Social:
rior, o exercício das seguintes atribuições:
I - coordenar, elaborar, executar, supervisionar e avaliar
I - orientar, disciplinar, normatizar, fiscalizar e defender
estudos, pesquisas, planos, programas e projetos na área
o exercício da profissão de Assistente Social, em conjunto
de Serviço Social; com o CRESS;
II - planejar, organizar e administrar programas e pro- II - assessorar os CRESS sempre que se fizer necessário;
jetos em Unidade de Serviço Social; III - aprovar os Regimentos Internos dos CRESS no fó-
III - assessoria e consultoria e órgãos da Administração rum máximo de deliberação do conjunto CFESS/CRESS;
Pública direta e indireta, empresas privadas e outras enti- IV - aprovar o Código de Ética Profissional dos Assis-
dades, em matéria de Serviço Social; tentes Sociais juntamente com os CRESS, no fórum máximo
IV - realizar vistorias, perícias técnicas, laudos periciais, de deliberação do conjunto CFESS/CRESS;
informações e pareceres sobre a matéria de Serviço Social; V - funcionar como Tribunal Superior de Ética Profis-
V - assumir, no magistério de Serviço Social tanto a sional;
nível de graduação como pós-graduação, disciplinas e fun- VI - julgar, em última instância, os recursos contra as
ções que exijam conhecimentos próprios e adquiridos em sanções impostas pelos CRESS;
curso de formação regular; VII - estabelecer os sistemas de registro dos profissio-
VI - treinamento, avaliação e supervisão direta de esta- nais habilitados;
giários de Serviço Social; VIII - prestar assessoria técnico-consultiva aos organis-
VII - dirigir e coordenar Unidades de Ensino e Cursos mos públicos ou privados, em matéria de Serviço Social;
de Serviço Social, de graduação e pós-graduação; IX - (Vetado)
VIII - dirigir e coordenar associações, núcleos, centros Art. 9º O fórum máximo de deliberação da profissão
de estudo e de pesquisa em Serviço Social; para os fins desta lei dar-se-á nas reuniões conjuntas dos
IX - elaborar provas, presidir e compor bancas de exa- Conselhos Federal e Regionais, que inclusive fixarão os li-
mes e comissões julgadoras de concursos ou outras formas mites de sua competência e sua forma de convocação.
de seleção para Assistentes Sociais, ou onde sejam aferidos Art. 10. Compete aos CRESS, em suas respectivas áreas
conhecimentos inerentes ao Serviço Social; de jurisdição, na qualidade de órgão executivo e de primei-
X - coordenar seminários, encontros, congressos e ra instância, o exercício das seguintes atribuições:
eventos assemelhados sobre assuntos de Serviço Social; I - organizar e manter o registro profissional dos Assis-
tentes Sociais e o cadastro das instituições e obras sociais
XI - fiscalizar o exercício profissional através dos Con-
públicas e privadas, ou de fins filantrópicos;
selhos Federal e Regionais;
II - fiscalizar e disciplinar o exercício da profissão de
XII - dirigir serviços técnicos de Serviço Social em enti-
Assistente Social na respectiva região;
dades públicas ou privadas;
III - expedir carteiras profissionais de Assistentes So-
XIII - ocupar cargos e funções de direção e fiscalização ciais, fixando a respectiva taxa;
da gestão financeira em órgãos e entidades representati- IV - zelar pela observância do Código de Ética Profis-
vas da categoria profissional. sional, funcionando como Tribunais Regionais de Ética Pro-
Art. 5o-A.  A duração do trabalho do Assistente Social é fissional;
de 30 (trinta) horas semanais. (Incluído pela Lei nº 12.317, V - aplicar as sanções previstas no Código de Ética Pro-
de 2010). fissional;
Art. 6º São alteradas as denominações do atual Conse- VI - fixar, em assembleia da categoria, as anuidades
lho Federal de Assistentes Sociais (CFAS) e dos Conselhos que devem ser pagas pelos Assistentes Sociais;
Regionais de Assistentes Sociais (CRAS), para, respectiva- VII - elaborar o respectivo Regimento Interno e subme-
mente, Conselho Federal de Serviço Social (CFESS) e Con- tê-lo a exame e aprovação do fórum máximo de delibera-
selhos Regionais de Serviço Social (CRESS). ção do conjunto CFESS/CRESS.

209
LEGISLAÇÃO

Art. 11. O Conselho Federal de Serviço Social (CFESS) 2º No caso de reincidência na mesma infração no pra-
terá sede e foro no Distrito Federal. zo de dois anos, a multa cabível será elevada ao dobro.
Art. 12. Em cada capital de Estado, de Território e no Art. 17. A Carteira de Identificação Profissional expe-
Distrito Federal, haverá um Conselho Regional de Serviço dida pelos Conselhos Regionais de Serviço Social (CRESS),
Social (CRESS) denominado segundo a sua jurisdição, a servirá de prova para fins de exercício profissional e de Car-
qual alcançará, respectivamente, a do Estado, a do Territó- teira de Identidade Pessoal, e terá fé pública em todo o
rio e a do Distrito Federal. território nacional.
1º Nos Estados ou Territórios em que os profissionais Art. 18. As organizações que se registrarem nos CRESS
que neles atuam não tenham possibilidade de instalar um receberão um certificado que as habilitará a atuar na área
Conselho Regional, deverá ser constituída uma delegacia de Serviço Social.
subordinada ao Conselho Regional que oferecer melhores Art. 19. O Conselho Federal de Serviço Social (CFESS)
condições de comunicação, fiscalização e orientação, ou- será mantido:
vido o órgão regional e com homologação do Conselho I - por contribuições, taxas e emolumentos arrecada-
Federal. dos pelos CRESS, em percentual a ser definido pelo fórum
2º Os Conselhos Regionais poderão constituir, dentro máximo instituído pelo art. 9º desta lei;
de sua própria área de jurisdição, delegacias seccionais II - por doações e legados;
para desempenho de suas atribuições executivas e de pri- III - por outras rendas.
meira instância nas regiões em que forem instalados, des- Art. 20. O Conselho Federal de Serviço Social (CFESS)
de que a arrecadação proveniente dos profissionais nelas e os Conselhos Regionais de Serviço Social (CRESS) conta-
atuantes seja suficiente para sua própria manutenção. rão cada um com nove membros efetivos: Presidente, Vice
Art. 13. A inscrição nos Conselhos Regionais sujeita os -Presidente, dois Secretários, dois Tesoureiros e três mem-
Assistentes Sociais ao pagamento das contribuições com- bros do Conselho Fiscal, e nove suplentes, eleitos dentre os
pulsórias (anuidades), taxas e demais emolumentos que fo- Assistentes Sociais, por via direta, para um mandato de três
rem estabelecidos em regulamentação baixada pelo Con- anos, de acordo com as normas estabelecidas em Código
selho Federal, em deliberação conjunta com os Conselhos Eleitoral aprovado pelo fórum instituído pelo art. 9º desta
Regionais. lei.
Art. 14. Cabe às Unidades de Ensino credenciar e co- Parágrafo único. As delegacias seccionais contarão
municar aos Conselhos Regionais de sua jurisdição os cam- com três membros efetivos: um Delegado, um Secretário e
pos de estágio de seus alunos e designar os Assistentes um Tesoureiro, e três suplentes, eleitos dentre os Assisten-
Sociais responsáveis por sua supervisão. tes Sociais da área de sua jurisdição, nas condições previs-
Parágrafo único. Somente os estudantes de Serviço So- tas neste artigo.
cial, sob supervisão direta de Assistente Social em pleno Art. 21. (Vetado).
gozo de seus direitos profissionais, poderão realizar está- Art. 22. O Conselho Federal e os Conselhos Regionais
gio de Serviço Social. terão legitimidade para agir contra qualquer pessoa que
Art. 15. É vedado o uso da expressão Serviço Social por infringir as disposições que digam respeito às prerrogati-
quaisquer pessoas de direito público ou privado que não vas, à dignidade e ao prestígio da profissão de Assistente
desenvolvam atividades previstas nos arts. 4º e 5º desta lei. Social.
Parágrafo único. As pessoas de direito público ou pri- Art. 23. Esta lei entra em vigor na data de sua publica-
vado que se encontrem na situação mencionada neste ar- ção.
tigo terão o prazo de noventa dias, a contar da data da vi- Art. 24. Revogam-se as disposições em contrário e, em
gência desta lei, para processarem as modificações que se especial, a Lei nº 3.252, de 27 de agosto de 1957.
fizerem necessárias a seu integral cumprimento, sob pena Brasília, 7 de junho de 1993; 172º da Independência e
das medidas judiciais cabíveis. 105º da República.
Art. 16. Os CRESS aplicarão as seguintes penalidades
aos infratores dos dispositivos desta Lei:
I - multa no valor de uma a cinco vezes a anuidade
vigente;
II - suspensão de um a dois anos de exercício da pro-
fissão ao Assistente Social que, no âmbito de sua atuação,
deixar de cumprir disposições do Código de Ética, tendo
em vista a gravidade da falta;
III - cancelamento definitivo do registro, nos casos de
extrema gravidade ou de reincidência contumaz.
1º Provada a participação ativa ou conivência de em-
presas, entidades, instituições ou firmas individuais nas in-
frações a dispositivos desta lei pelos profissionais delas de-
pendentes, serão estas também passíveis das multas aqui
estabelecidas, na proporção de sua responsabilidade, sob
pena das medidas judiciais cabíveis.

210
LEGISLAÇÃO

estruturam uma profissão, um código de ética não pode


CÓDIGO DE ÉTICA DO PSICÓLOGO ser visto como um conjunto fixo de normas e imutável no
tempo. As sociedades mudam, as profissões transformam-
se e isso exige, também, uma reflexão contínua sobre o
próprio código de ética que nos orienta.
CÓDIGO DE ÉTICA PROFISSIONAL DO PSICÓLOGO A formulação deste Código de Ética, o terceiro da pro-
Novembro de 2014 fissão de psicólogo no Brasil, responde ao contexto organi-
zativo dos psicólogos, ao momento do país e ao estágio de
RESOLUÇÃO CFP Nº 010/05 desenvolvimento da Psicologia enquanto campo científico
e profissional. Este Código de Ética dos Psicólogos é re-
Aprova o Código de Ética Profissional do Psicólogo. flexo da necessidade, sentida pela categoria e suas enti-
O CONSELHO FEDERAL DE PSICOLOGIA, no uso de dades representativas, de atender à evolução do contexto
suas atribuições legais e regimentais, que lhe são conferi- institucional-legal do país, marcadamente a partir da pro-
das pela Lei no 5.766, de 20 de dezembro de 1971; mulgação da denominada Constituição Cidadã, em 1988,
CONSIDERANDO o disposto no Art. 6º, letra “e”, da Lei e das legislações dela decorrentes.
no 5.766 de 20/12/1971, e o Art. 6º, inciso VII, do Decreto Consoante com a conjuntura democrática vigente, o
nº 79.822 de 17/6/1977; presente Código foi construído a partir de múltiplos espa-
CONSIDERANDO o disposto na Constituição Federal ços de discussão sobre a ética da profissão, suas responsa-
de 1988, conhecida como Constituição Cidadã, que con- bilidades e compromissos com a promoção da cidadania.
solida o Estado Democrático de Direito e legislações dela O processo ocorreu ao longo de três anos, em todo o país,
decorrentes; com a participação direta dos psicólogos e aberto à so-
CONSIDERANDO decisão deste Plenário em reunião ciedade.
realizada no dia 21 de julho de 2005; Este Código de Ética pautou-se pelo princípio geral de
aproximar-se mais de um instrumento de reflexão do que
RESOLVE: de um conjunto de normas a serem seguidas pelo psicólo-
Art. 1º - Aprovar o Código de Ética Profissional do Psi- go. Para tanto, na sua construção buscou-se:
cólogo. a. Valorizar os princípios fundamentais como grandes
Art. 2º - A presente Resolução entrará em vigor no dia eixos que devem orientar a relação do psicólogo com a
27 de agosto de 2005. sociedade, a profissão, as entidades profissionais e a ciên-
Art. 3º - Revogam-se as disposições em contrário, em cia, pois esses eixos atravessam todas as práticas e estas
especial a Resolução CFP n º 002/87. demandam uma contínua reflexão sobre o contexto social
e institucional.
APRESENTAÇÃO b. Abrir espaço para a discussão, pelo psicólogo, dos
Toda profissão define-se a partir de um corpo de prá- limites e interseções relativos aos direitos individuais e
ticas que busca atender demandas sociais, norteado por coletivos, questão crucial para as relações que estabelece
elevados padrões técnicos e pela existência de normas éti- com a sociedade, os colegas de profissão e os usuários ou
cas que garantam a adequada relação de cada profissional beneficiários dos seus serviços.
com seus pares e com a sociedade como um todo. c. Contemplar a diversidade que configura o exercício
Um Código de Ética profissional, ao estabelecer pa- da profissão e a crescente inserção do psicólogo em con-
drões esperados quanto às práticas referendadas pela res- textos institucionais e em equipes multiprofissionais.
pectiva categoria profissional e pela sociedade, procura d. Estimular reflexões que considerem a profissão
fomentar a autorreflexão exigida de cada indivíduo acerca como um todo e não em suas práticas particulares, uma
da sua práxis, de modo a responsabilizá-lo, pessoal e co- vez que os principais dilemas éticos não se restringem a
letivamente, por ações e suas consequências no exercício práticas específicas e surgem em quaisquer contextos de
profissional. A missão primordial de um código de ética atuação.
profissional não é de normatizar a natureza técnica do tra- Ao aprovar e divulgar o Código de Ética Profissional
balho, e, sim, a de assegurar, dentro de valores relevantes do Psicólogo, a expectativa é de que ele seja um instru-
para a sociedade e para as práticas desenvolvidas, um pa- mento capaz de delinear para a sociedade as responsa-
drão de conduta que fortaleça o reconhecimento social bilidades e deveres do psicólogo, oferecer diretrizes para
daquela categoria. a sua formação e balizar os julgamentos das suas ações,
Códigos de Ética expressam sempre uma concepção contribuindo para o fortalecimento e ampliação do signifi-
de homem e de sociedade que determina a direção das cado social da profissão.
relações entre os indivíduos. Traduzem-se em princípios
e normas que devem se pautar pelo respeito ao sujeito PRINCÍPIOS FUNDAMENTAIS
humano e seus direitos fundamentais. Por constituir a ex- I. O psicólogo baseará o seu trabalho no respeito e
pressão de valores universais, tais como os constantes na na promoção da liberdade, da dignidade, da igualdade e
Declaração Universal dos Direitos Humanos; sociocultu- da integridade do ser humano, apoiado nos valores que
rais, que refletem a realidade do país; e de valores que embasam a Declaração Universal dos Direitos Humanos.

211
LEGISLAÇÃO

II. O psicólogo trabalhará visando promover a saúde j) Ter, para com o trabalho dos psicólogos e de ou-
e a qualidade de vida das pessoas e das coletividades e tros profissionais, respeito, consideração e solidariedade, e,
contribuirá para a eliminação de quaisquer formas de ne- quando solicitado, colaborar com estes, salvo impedimen-
gligência, discriminação, exploração, violência, crueldade e to por motivo relevante;
opressão. k) Sugerir serviços de outros psicólogos, sempre que,
III. O psicólogo atuará com responsabilidade social, por motivos justificáveis, não puderem ser continuados
analisando crítica e historicamente a realidade política, pelo profissional que os assumiu inicialmente, fornecendo
econômica, social e cultural. ao seu substituto as informações necessárias à continuida-
IV. O psicólogo atuará com responsabilidade, por meio de do trabalho;
do contínuo aprimoramento profissional, contribuindo l) Levar ao conhecimento das instâncias competentes
para o desenvolvimento da Psicologia como campo cientí- o exercício ilegal ou irregular da profissão, transgressões a
fico de conhecimento e de prática. princípios e diretrizes deste Código ou da legislação pro-
V. O psicólogo contribuirá para promover a universali- fissional.
zação do acesso da população às informações, ao conhe-
cimento da ciência psicológica, aos serviços e aos padrões Art. 2º – Ao psicólogo é vedado:
éticos da profissão. a) Praticar ou ser conivente com quaisquer atos que
VI. O psicólogo zelará para que o exercício profissional caracterizem negligência, discriminação, exploração, vio-
seja efetuado com dignidade, rejeitando situações em que lência, crueldade ou opressão;
a Psicologia esteja sendo aviltada. b) Induzir a convicções políticas, filosóficas, morais,
VII. O psicólogo considerará as relações de poder nos ideológicas, religiosas, de orientação sexual ou a qualquer
contextos em que atua e os impactos dessas relações sobre tipo de preconceito, quando do exercício de suas funções
as suas atividades profissionais, posicionando-se de forma profissionais;
crítica e em consonância com os demais princípios deste c) Utilizar ou favorecer o uso de conhecimento e a uti-
Código. lização de práticas psicológicas como instrumentos de cas-
tigo, tortura ou qualquer forma de violência;
DAS RESPONSABILIDADES DO PSICÓLOGO d) Acumpliciar-se com pessoas ou organizações que
Art. 1º – São deveres fundamentais dos psicólogos: exerçam ou favoreçam o exercício ilegal da profissão de
psicólogo ou de qualquer outra atividade profissional;
a) Conhecer, divulgar, cumprir e fazer cumprir este Có-
e) Ser conivente com erros, faltas éticas, violação de
digo;
direitos, crimes ou contravenções penais praticados por
b) Assumir responsabilidades profissionais somente
psicólogos na prestação de serviços profissionais;
por atividades para as quais esteja capacitado pessoal, teó-
f) Prestar serviços ou vincular o título de psicólogo a
rica e tecnicamente;
serviços de atendimento psicológico cujos procedimentos,
c) Prestar serviços psicológicos de qualidade, em con-
técnicas e meios não estejam regulamentados ou reconhe-
dições de trabalho dignas e apropriadas à natureza desses
cidos pela profissão;
serviços, utilizando princípios, conhecimentos e técnicas
g) Emitir documentos sem fundamentação e qualidade
reconhecidamente fundamentados na ciência psicológica, técnico-científica;
na ética e na legislação profissional; h) Interferir na validade e fidedignidade de instrumen-
d) Prestar serviços profissionais em situações de ca- tos e técnicas psicológicas, adulterar seus resultados ou fa-
lamidade pública ou de emergência, sem visar benefício zer declarações falsas;
pessoal; i) Induzir qualquer pessoa ou organização a recorrer a
e) Estabelecer acordos de prestação de serviços que seus serviços;
respeitem os direitos do usuário ou beneficiário de serviços j) Estabelecer com a pessoa atendida, familiar ou ter-
de Psicologia; ceiro, que tenha vínculo com o atendido, relação que possa
f) Fornecer, a quem de direito, na prestação de servi- interferir negativamente nos objetivos do serviço prestado;
ços psicológicos, informações concernentes ao trabalho a k) Ser perito, avaliador ou parecerista em situações nas
ser realizado e ao seu objetivo profissional; quais seus vínculos pessoais ou profissionais, atuais ou an-
g) Informar, a quem de direito, os resultados decor- teriores, possam afetar a qualidade do trabalho a ser reali-
rentes da prestação de serviços psicológicos, transmitindo zado ou a fidelidade aos resultados da avaliação;
somente o que for necessário para a tomada de decisões l) Desviar para serviço particular ou de outra institui-
que afetem o usuário ou beneficiário; ção, visando benefício próprio, pessoas ou organizações
h) Orientar a quem de direito sobre os encaminhamen- atendidas por instituição com a qual mantenha qualquer
tos apropriados, a partir da prestação de serviços psicoló- tipo de vínculo profissional;
gicos, e fornecer, sempre que solicitado, os documentos m) Prestar serviços profissionais a organizações con-
pertinentes ao bom termo do trabalho; correntes de modo que possam resultar em prejuízo para
i) Zelar para que a comercialização, aquisição, doação, as partes envolvidas, decorrentes de informações privile-
empréstimo, guarda e forma de divulgação do material giadas;
privativo do psicólogo sejam feitas conforme os princípios n) Prolongar, desnecessariamente, a prestação de ser-
deste Código; viços profissionais;

212
LEGISLAÇÃO

o) Pleitear ou receber comissões, empréstimos, doa- Art. 8º – Para realizar atendimento não eventual de
ções ou vantagens outras de qualquer espécie, além dos criança, adolescente ou interdito, o psicólogo deverá obter
honorários contratados, assim como intermediar transa- autorização de ao menos um de seus responsáveis, obser-
ções financeiras; vadas as determinações da legislação vigente:
p) Receber, pagar remuneração ou porcentagem por §1° – No caso de não se apresentar um responsável
encaminhamento de serviços; legal, o atendimento deverá ser efetuado e comunicado às
q) Realizar diagnósticos, divulgar procedimentos ou autoridades competentes;
apresentar resultados de serviços psicológicos em meios §2° – O psicólogo responsabilizar-se-á pelos encami-
de comunicação, de forma a expor pessoas, grupos ou or- nhamentos que se fizerem necessários para garantir a pro-
ganizações. teção integral do atendido.
Art. 3º – O psicólogo, para ingressar, associar-se ou Art. 9º – É dever do psicólogo respeitar o sigilo pro-
permanecer em uma organização, considerará a missão, a
fissional a fim de proteger, por meio da confidencialidade,
filosofia, as políticas, as normas e as práticas nela vigentes
a intimidade das pessoas, grupos ou organizações, a que
e sua compatibilidade com os princípios e regras deste Có-
tenha acesso no exercício profissional.
digo.
Parágrafo único: Existindo incompatibilidade, cabe ao
psicólogo recusar-se a prestar serviços e, se pertinente, Art. 10 – Nas situações em que se configure conflito
apresentar denúncia ao órgão competente. entre as exigências decorrentes do disposto no Art. 9º e
as afirmações dos princípios fundamentais deste Código,
Art. 4º – Ao fixar a remuneração pelo seu trabalho, o excetuando-se os casos previstos em lei, o psicólogo po-
psicólogo: derá decidir pela quebra de sigilo, baseando sua decisão na
a) Levará em conta a justa retribuição aos serviços busca do menor prejuízo.
prestados e as condições do usuário ou beneficiário; Parágrafo único – Em caso de quebra do sigilo previs-
b) Estipulará o valor de acordo com as características to no caput deste artigo, o psicólogo deverá restringir-se a
da atividade e o comunicará ao usuário ou beneficiário an- prestar as informações estritamente necessárias.
tes do início do trabalho a ser realizado;
c) Assegurará a qualidade dos serviços oferecidos in- Art. 11 – Quando requisitado a depor em juízo, o psi-
dependentemente do valor acordado. cólogo poderá prestar informações, considerando o previs-
to neste Código.
Art. 5º – O psicólogo, quando participar de greves ou
paralisações, garantirá que: Art. 12 – Nos documentos que embasam as atividades
a) As atividades de emergência não sejam interrom- em equipe multiprofissional, o psicólogo registrará apenas
pidas; as informações necessárias para o cumprimento dos obje-
b) Haja prévia comunicação da paralisação aos usuá- tivos do trabalho.
rios ou beneficiários dos serviços atingidos pela mesma.
Art. 13 – No atendimento à criança, ao adolescente
Art. 6º – O psicólogo, no relacionamento com profis- ou ao interdito, deve ser comunicado aos responsáveis o
sionais não psicólogos: estritamente essencial para se promoverem medidas em
a) Encaminhará a profissionais ou entidades habilita- seu benefício.
dos e qualificados demandas que extrapolem seu campo
de atuação;
Art. 14 – A utilização de quaisquer meios de registro
b) Compartilhará somente informações relevantes para
e observação da prática psicológica obedecerá às normas
qualificar o serviço prestado, resguardando o caráter confi-
deste Código e a legislação profissional vigente, devendo o
dencial das comunicações, assinalando a responsabilidade,
usuário ou beneficiário, desde o início, ser informado.
de quem as receber, de preservar o sigilo.

Art. 7º – O psicólogo poderá intervir na prestação de Art. 15 – Em caso de interrupção do trabalho do psicó-
serviços psicológicos que estejam sendo efetuados por ou- logo, por quaisquer motivos, ele deverá zelar pelo destino
tro profissional, nas seguintes situações: dos seus arquivos confidenciais.
a) A pedido do profissional responsável pelo serviço; § 1° – Em caso de demissão ou exoneração, o psicólogo
b) Em caso de emergência ou risco ao beneficiário ou deverá repassar todo o material ao psicólogo que vier a
usuário do serviço, quando dará imediata ciência ao pro- substituí-lo, ou lacrá-lo para posterior utilização pelo psi-
fissional; cólogo substituto.
c) Quando informado expressamente, por qualquer § 2° – Em caso de extinção do serviço de Psicologia,
uma das partes, da interrupção voluntária e definitiva do o psicólogo responsável informará ao Conselho Regional
serviço; de Psicologia, que providenciará a destinação dos arquivos
d) Quando se tratar de trabalho multiprofissional e a confidenciais.
intervenção fizer parte da metodologia adotada.

213
LEGISLAÇÃO

Art. 16 – O psicólogo, na realização de estudos, pes- c) Censura pública;


quisas e atividades voltadas para a produção de conheci- d) Suspensão do exercício profissional, por até 30 (trin-
mento e desenvolvimento de tecnologias: ta) dias, ad referendum do Conselho Federal de Psicologia;
a) Avaliará os riscos envolvidos, tanto pelos procedi- e) Cassação do exercício profissional, ad referendum
mentos, como pela divulgação dos resultados, com o obje- do Conselho Federal de Psicologia.
tivo de proteger as pessoas, grupos, organizações e comu-
nidades envolvidas; Art. 22 – As dúvidas na observância deste Código e os
b) Garantirá o caráter voluntário da participação dos casos omissos serão resolvidos pelos Conselhos Regionais
envolvidos, mediante consentimento livre e esclarecido, de Psicologia, ad referendum do Conselho Federal de Psi-
salvo nas situações previstas em legislação específica e res- cologia.
peitando os princípios deste Código;
c) Garantirá o anonimato das pessoas, grupos ou orga- Art. 23 – Competirá ao Conselho Federal de Psicologia
nizações, salvo interesse manifesto destes; firmar jurisprudência quanto aos casos omissos e fazê-la
d) Garantirá o acesso das pessoas, grupos ou organi- incorporar a este Código.
zações aos resultados das pesquisas ou estudos, após seu
encerramento, sempre que assim o desejarem. Art. 24 – O presente Código poderá ser alterado pelo
Conselho Federal de Psicologia, por iniciativa própria ou da
Art. 17 – Caberá aos psicólogos docentes ou supervi- categoria, ouvidos os Conselhos Regionais de Psicologia.
sores esclarecer, informar, orientar e exigir dos estudantes
a observância dos princípios e normas contidas neste Có- Art. 25 – Este Código entra em vigor em 27 de agosto
digo. de 2005.

Art. 18 – O psicólogo não divulgará, ensinará, cede-


rá, emprestará ou venderá a leigos instrumentos e técnicas
psicológicas que permitam ou facilitem o exercício ilegal
da profissão.

Art. 19 – O psicólogo, ao participar de atividade em


veículos de comunicação, zelará para que as informações
prestadas disseminem o conhecimento a respeito das atri-
buições, da base científica e do papel social da profissão.

Art. 20 – O psicólogo, ao promover publicamente seus


serviços, por quaisquer meios, individual ou coletivamente:
a) Informará o seu nome completo, o CRP e seu núme-
ro de registro;
b) Fará referência apenas a títulos ou qualificações
profissionais que possua;
c) Divulgará somente qualificações, atividades e recur-
sos relativos a técnicas e práticas que estejam reconhecidas
ou regulamentadas pela profissão;
d) Não utilizará o preço do serviço como forma de pro-
paganda;
e) Não fará previsão taxativa de resultados;
f) Não fará auto-promoção em detrimento de outros
profissionais;
g) Não proporá atividades que sejam atribuições priva-
tivas de outras categorias profissionais;
h) Não fará divulgação sensacionalista das atividades
profissionais.

DAS DISPOSIÇÕES GERAIS

Art. 21 – As transgressões dos preceitos deste Códi-


go constituem infração disciplinar com a aplicação das se-
guintes penalidades, na forma dos dispositivos legais ou
regimentais:
a) Advertência;
b) Multa;

214
CONHECIMENTOS ESPECÍFICOS - PSICÓLOGO

Desenvolvimento psicológico: infância e adolescência............................................................................................................................. 01


A formação e rompimento dos laços afetivos.............................................................................................................................................. 08
A importância do ambiente.................................................................................................................................................................................. 08
Consequências das falhas ambientais.............................................................................................................................................................. 08
O papel do pai........................................................................................................................................................................................................... 16
O papel da agressividade no desenvolvimento humano......................................................................................................................... 19
Natureza e origens da tendência antissocial................................................................................................................................................. 22
Prevenção e efeitos da privação materna....................................................................................................................................................... 24
As inter-relações familiares: casamento, conflito conjugal, separação, guarda dos filhos, violência doméstica................ 27
Aspectos psicossociais do envelhecimento.................................................................................................................................................... 49
Aspectos psicossociais do fenômeno da violência...................................................................................................................................... 57
A criança e a separação dos pais........................................................................................................................................................................ 66
Os direitos fundamentais da criança e do adolescente............................................................................................................................. 70
As medidas específicas de proteção à criança e ao adolescente.......................................................................................................... 85
A criança e o adolescente no acolhimento institucional........................................................................................................................... 91
A colocação em família substituta...................................................................................................................................................................101
As medidas socioeducativas...............................................................................................................................................................................105
O psicólogo no atendimento aos casos nas Varas da Infância e da Juventude, nas Varas da Família e das Sucessões,
Violência Doméstica e ao Idoso........................................................................................................................................................................107
Avaliação Psicológica: instrumentais e sua prática na instituição judiciária....................................................................................124
A entrevista psicológica.......................................................................................................................................................................................124
Elaboração de documentos escritos: laudos, relatórios e pareceres psicológicos........................................................................138
O lugar do saber psicológico na instituição judiciária.............................................................................................................................150
Ética profissional.....................................................................................................................................................................................................153
CONHECIMENTOS ESPECÍFICOS - PSICÓLOGO

que a criança não consegue fazer. Por exemplo, ele acre-


DESENVOLVIMENTO PSICOLÓGICO: ditava que o pensamento pré-operatório das crianças pe-
INFÂNCIA E ADOLESCÊNCIA. quenas fosse marcado pelo egocentrismo, isto é a incapa-
cidade de ver as coisas pela perspectiva de outra pessoa
(GERRIG, 2005).
O terceiro período (7 aos 11/12anos), é o das opera-
DESENVOLVIMENTO ções concretas, a criança conhece e organiza o mundo de
forma lógica ou operatória. A conversação torna-se possí-
O desenvolvimento é um processo contínuo que tem vel ( já é uma linguagem socializada), pois a fala egocêntri-
início desde a concepção, e tem continuidade após a fe- ca desaparece devido o desejo de trabalhar com os outros
cundação do óvulo, percorrendo a partir da subdivisão ce- (idade escolar), sem que, no entanto, possam discutir dife-
rentes pontos de vista para que cheguem a uma conclusão
lular até que milhões de células sejam formadas. À medida
comum (FERREIRA, 2009).
que as células assumem funções especializadas, dá-se iní-
O estágio operatório formal cobre o período que co-
cio a formação dos sistemas que dão base para a parte fí- meça em torno dos 11 anos. Neste estágio final do de-
sica do desenvolvimento. Porém, o desenvolvimento físico, senvolvimento cognitivo, o pensamento se torna abstrato.
cognitivo, social, afetivo tem continuidade durante todas as Os adolescentes conseguem entender que sua realidade
fases da vida de um sujeito e termina com a morte. específica é apenas uma entre várias imagináveis, e come-
Assim, o fenômeno do desenvolvimento, faz articula- çam a questionar temas profundos relacionados a verdade,
ções e interfaces com várias áreas do conhecimento como: justiça e existência (GERRIG, 2005).
a educação, biologia, sociologia, antropologia, medicina,
entre outros, interagindo com diversos saberes a fim de O que Fazem os Psicólogos do Desenvolvimento?
fomentar suas explicações.
A psicologia do desenvolvimento traz uma compreen- De acordo com GERRIG (2005) os psicólogos do de-
são sobre as transformações psicológicas que ocorrem no senvolvimento propõem teorias para explicar como e por
decorrer do tempo, com auxílio de algumas teorias e teóri- que as pessoas mudam durante a vida. Eles utilizam inves-
cos, bem como Jean Piaget, esses modelos se propõem em tigações normativas para descrever as características de
explicar como as mudanças ocorrem na vida do sujeito e de determinadas idades ou estágios do desenvolvimento. Os
que modo podem ser compreendidas e descritas. estudos longitudinais acompanham os mesmos indivíduos
Tradicionalmente, os primeiros estudos referentes à com o passar do tempo; os modelos transversais estudam
simultaneamente diferentes grupos etários.
psicologia do desenvolvimento faziam menção somente ao
As investigações normativas se caracterizam como um
desenvolvimento da criança e do adolescente. Entretanto,
conjunto de procedimentos que avaliam como seria uma
esse foco vem mudando ao longo dos anos, e hoje, tem-se pessoa, em termo de aparência física, habilidades cogni-
uma ideia que o estudo sobre o desenvolvimento humano tivas, com a finalidade de descrever aquilo que caracteriza
deve abranger todo processo de ciclo vital. uma determinada idade ou estágio do desenvolvimento.
O modelo longitudinal é uma técnica para entender os
As Descobertas de Piaget sobre Desenvolvimento possíveis mecanismos de transformação. Nesse aspecto,
Mental denomina-se uma forma de observar os indivíduos repeti-
damente durante vários anos.
Jean Piaget foi o responsável pela introdução das fases Conforme MOTA (2005) psicólogos do desenvolvimen-
sobre o desenvolvimento mental do indivíduo. Ele acredita- to enfrentam novos desafios no século XXI. As novas con-
va que o desenvolvimento cognitivo do indivíduo poderia cepções de atuação profissional que enfatizam a prevenção
ser subdividido em uma série de quatros estágios orde- e a promoção de saúde fazem com que profissionais de vá-
nados e descontínuos. Esse teórico supõe que as crianças rias áreas busquem na psicologia desenvolvimento subsí-
progridam através dessas fases. dios teóricos e metodológicos para sua prática profissional.
O primeiro período, sensório-motor (0 a 24 meses), Um dos fatos discutidos em questão é o desenvolvi-
baseia-se em uma inteligência que trabalha as percepções mento harmônico do sujeito, que integra vários aspectos
e as ações através dos deslocamentos do próprio corpo. da vida, bem como as características biológicas, sociais,
cognitivas, afetivas que compõem toda a estrutura de um
Neste período a criança não possui representação mental,
indivíduo.
ou seja, para eles os objetos só existem se estiverem em
seu campo visual. A conduta social, neste período, é de iso- Uma Delimitação do Conceito de Psicologia do De-
lamento e indiferenciação, onde o mundo se volta inteira- senvolvimento
mente a própria criança (o mundo é ela) (FERREIRA, 2009).
O estágio pré-operatório vai mais ou menos dos dois Ainda existe uma dificuldade em conceituar o processo
anos aos sete anos de idade. O grande avanço cognitivo de desenvolvimento humano tendo em vista o vasto cam-
nesta fase do desenvolvimento é uma melhor capacidade po de estudo que envolve essa disciplina. Nesse aspecto,
de representar, mentalmente, objetos que não estejam fi- alguns teóricos vêm fazendo algumas especulações a fim
sicamente presentes. Com exceção desta evolução, Piaget de propor novas formas de estudar o desenvolvimento, a
caracteriza o estágio pré-operatório de acordo com aquilo partir de todo o ciclo vital do indivíduo.

1
CONHECIMENTOS ESPECÍFICOS - PSICÓLOGO

Papalia e Olds (2000), por exemplo, definem desen- tura dos dados trazidos por Spitz se apoia nas formulações
volvimento como “o estudo científico de como as pessoas de Freud ao mesmo tempo em que lhe fornece suporte
mudam ou como elas ficam iguais, desde a concepção até teórico, ou seja: por visar a verificação de seus conceitos
a morte”. freudiano, permite maior visibilidade, pois os articula à ex-
Biaggio (1978) argumenta que a especificidade da psi- perimentação, e possibilita um suporte à teoria psicanalíti-
cologia do desenvolvimento humano está em estudar as ca. No exercício de sua pesquisa, Spitz constroi uma teoria
variáveis externas e internas aos indivíduos que levam as própria, nomeando três organizadores essenciais à forma-
mudanças no comportamento em períodos de transição ção psíquica. Ele deixa claro que a teoria freudiana é extensa
rápida (infância, adolescência e envelhecimento). e nova – justificando e entendendo que Freud, enquanto
Algumas teorias contemporâneas do desenvolvimento autor de um novo campo de saber, não pôde se deter na
aceitam que as transformações no processo de desenvolvi- averiguação de seus conceitos em referência aos primeiros
mento acontecem em todas as fases da vida, mas que são anos de vida, pois foi através de sua clínica com adultos que
mais marcantes em períodos rápidos de transição. Desse ele chegou a destacar a importância da sexualidade infan-
modo, é necessário ampliar o escopo em que se configura til na formação psíquica, notoriamente na relação do bebê
o desenvolvimento humano. com sua mãe. É esta relação que se centra o estudo de Spitz,
A visão ampliada acerca da concepção da psicologia que é o objeto de sua pesquisa.
do desenvolvimento faz desse conhecimento bastante im- Coloquemos sucintamente a referência aos três organi-
portante para elaboração de programas de intervenção de zadores em questão. O Primeiro Organizador é a “Gestalt do
cunho preventivo e, sobretudo, na promoção da saúde, rosto humano”, que se forma em torno do segundo mês de
tendo em vista que possibilita uma maior compreensão vida. Inscreve o período pré-objetal que se faz acompanhar
dos processos de desenvolvimento humano. pela aquisição do traço mnemônico da Gestalt do rosto hu-
mano em geral. Este é o primeiro traço de inscrição psíquica
Fonte: (ou a existência de uma representação no psiquismo).
https://psicologado.com/psicologia-geral/desenvolvi- O Segundo Organizador declara a existência no psi-
mento-humano/o-que-e-a-psicologia-do-desenvolvimen- quismo de um espaço outro, enquanto abertura ou fenda.
to © Psicologado.com É impressionante constatar que é deste modo que surge a
presentificação da negatividade – ele deixa claro que uma
OS TRÊS ORGANIZADORES DE SPITZ E OS SIGNIFI- ausência possibilita inscrever a presença como negativida-
CADOS1 de.
Esclareçamos: Spitz constata a inscrição do rosto mater-
Tomemos o primeiro ano de vida (Spitz 2004). Em rela- no, mais precisamente: o rosto da pessoa que faz a função
ção ao bebê, logo após o nascimento, num corpo de fun- de complemento do infans, em sua função de maternagem.
cionamento orgânico, movido pela necessidade de descar- Para nos dar a imagem da formação de tal vínculo, Spitz
ga – que se faz notar pelo choro, por fome, frio, dores... -, faz referência ao “amor egoísta a dois”, que acompanha a
algo no ambiente lhe responde, pois supõe que a existên- comunicação nesta díade.
cia de um pedido neste tempo de ausência de intenção e
de palavras. O Outro materno é o intérprete do bebê. Res- Nesta, a comunicação é visceral, inteiramente afetiva,
ponde às necessidades como se houvesse ali um desejo. É em referência ao “verdadeiro” sentido dos sentimentos vi-
desta forma que Lacan, lendo Freud, trabalha a noção de vido na relação mãe/bebê. Não há espaço para a “mentira”
sujeito como estritamente vinculada ao desejo, mais preci- (ou equívoco), pois a relação flui através da vivência ime-
samente: àquilo que é do âmbito de uma falta. Esta supo- diata apreendida no toque, no tom da voz... Não há espaço
sição no início da vida é essencial, pois coloca aquele que para a mentira – a não ser que as palavras que acompanhem
figura como “objeto no desejo materno” (no caso, o bebê) esta vivência mintam sobre o sentimento em jogo. Neste
numa performance de sujeito, por considerar a existência caso, revela uma possibilidade (ou até mesmo configura-se
de uma subjetividade num tempo precoce – uma vez que, como uma causa, dependendo da dimensão de tal ocorrên-
como a teoria freudiana explicita através de seus estudos, cia) psicopatológica na estruturação psíquica.
a formação da subjetividade ocorre na lógica que concerne Considerando ainda este momento essencial da organi-
ao tempo de Édipo, em torno dos três aos cinco/seis anos zação psíquica, o que se vê construído é o “objeto” do afeto
de vida. infantil. O infans está colocado nesta construção. Narcisica-
O que transparece na obra de Spitz é a construção do mente colocado (representado por uma imagem de si, está
psiquismo em relação à possibilidade de representação e, ao mesmo tempo ausente), chora pelo objeto de amor que,
consequentemente, sua instauração, ou seja, a capacidade neste momento, só pode reconhecer por sua ausência – ou
psíquica (o psiquismo em si). Sua teoria – que se susten- falta. Chora a “ausência” do rosto materno, enquanto este
ta no conhecimento dos textos freudianos – é construída, é o suporte de seu próprio reconhecimento e possibilidade
dissemos, a partir de suas constatações, ou seja, uma série de existência de si mesmo. Conforme Spitz, tal experiência
de experimentos e filmagens das situações infantis. A lei- é verificada num determinado momento em que a crian-
1 LOPES, M. M. de F. Psicanálise e Representação: a teoria de
ça está na presença de outras pessoas (mais precisamente:
René Spitz e a organização psíquica. Brazilian Journal of Health. v. 1, n. na presença de outros rostos) e a mãe não está presente.
3, p. 201-209, Setembro/Dezembro 2010 Neste contexto, o rosto, fonte libidinal do amor “narcísico

2
CONHECIMENTOS ESPECÍFICOS - PSICÓLOGO

a dois”, não mais se encontra. Não mais se encontra em por ainda não ter condições de fazer uso dos símbolos ou
relação à apresentação de outros rostos humanos. O cho- pensamento – aquisição que faz a partir do Terceiro Or-
ro infantil em torno dos oito meses presentifica a angústia ganizador –, reage ao não do outro, que o impede de se
– conforme Spitz denomina: uma angústia enquanto an- aproximar dos objetos de seu interesse, por meio da vivên-
gústia propriamente dita. Então temos: o Segundo Orga- cia de um conflito agudo. Vinculado afetivamente àquele
nizador é a “Angústia propriamente dia”. Este é o nome do de diz não, teme perder seu amor se executar ações co-
organizador. mandadas pelo seu interesse (no caso, descargas imediatas
“Propriamente dita” porque na lógica de seu estudo, no objeto conforme proposta pelo id que desconsidera a
conforme suas averiguações, não é possível a existência da realidade), por outro lado, sucumbe a si mesmo se permitir
angústia sem que haja um psiquismo. A angústia assinala deixar de lado seus desejos imediatos. Deixar de fazer a
a existência de um psiquismo. Neste sentido, cai por terra ação que sua força interior demanda, no sentido de des-
a tese de Otto Rank – e de Melanie Klein – de um trauma carregar tensões íntimas, é “deixar de existir”, mortificar-se,
psíquico no nascimento. Portanto, ao nascer só há possibi- submeter-se inteiramente ao Outro, o que é um retrocesso
lidade de existir traumas fisiológicos, pois não há psiquis- em seu desenvolvimento que o compele à aquisição de sua
mo ao nascer. Neste momento, Spitz junta-se à Freud para independência – um retorno à anterioridade passiva, numa
afirmar que o aparelho psíquico é fruto de uma construção, vivência a dois (o regresso à vivência diádica). Assistimos
fruto da relação com o ambiente. Não há trauma se não há muitas vezes esta luta na forma de birra infantil (em torno
psiquismo. do décimo quinto mês de vida). Quem ganha? Se o outro
Podemos falar em presentificação da negatividade ganhar, a dependência é deflagrada – crianças desenergi-
como a ausência notada deste objeto de amor, que pôde zadas, sem vontade, interesse e extremamente inibidas -,
garantir-se através da constância deste vínculo afetivo. Na alienadas no desejo do Outro, colocadas aí como objetos.
descrição de Spitz, o bebê chora ao perceber a ausência Nesta vertente, a possibilidade de subjetivar-se se perde,
do rosto que ganhou registro de forma marcante no seu pois não houve vivência da falta. Reina a complementari-
psiquismo. Portanto, no psiquismo marca presença – en- dade: do Outro (materno) através de seu objeto (a criança).
quanto na verificação da realidade, ele está ausente. Cho- Esta forma complementar de relacionamento de forma al-
ra a ausência, e se angustia por não ter possibilidade de guma concerne à subjetividade – que conforme afirmamos,
representá-lo – ou entender, por exemplo, que a mãe está através da referência a Lacan, se constroi pela falta.
na sala ao lado, ou no jardim, pois neste mundo de vivência
imediata só há lugar para o que está presente, não há ainda Entretanto – e também no que está em jogo em rela-
a idéia de um outro lugar. Um lugar outro que começa a ção à constituição da sociabilidade que acompanha o nas-
ser presentificado justamente nesta ação de apresentar-se cimento de um sujeito –, as formas extremas de atividade,
como vivência de ausência. Angústia é isto, aquele que a que se traduz pelas satisfações de todas as vontades in-
vive não tem como representá-la (é a vivência da ausência fantis, é irreconciliável ao desejo humano, que precisa de
de uma representação). medida e limite. É neste sentido que o não é essencial à
instauração de um psiquismo que se constituí pela falta.
Por estarmos fazendo referência ao jogo de presença/ O não para atingir seu objetivo – subjetividade que inscre-
ausência – afirmação/negação -, o que está em evidência ve também a socialização –, necessita do apoio do código.
é a dimensão da linguagem, que trabalha binariamente. Com a necessidade deste recurso, não há lugar para a men-
É na ausência do objeto que a linguagem serve à sua re- tira. O verdadeiro se impõe, ou seja: para que o não em seu
presentação. O uso do símbolo (ou representação) teste- sentido de negatividade funcione é essencial que adquira
munha a ausência do objeto. Só se pode fazer uso dele, o sentido semântico de não, e não seja uma palavra vazia
segundo Jean Piaget, após a construção da permanência (ou sem significado).
do objeto – recurso que possibilita à criança, ainda no está- É no Outro do amor infantil – e somente por este ser o
gio sensório-motor, fazer a aquisição de noções de tempo, objeto do afeto infantil - que é atribuído o verdadeiro valor
espaço e causalidade, adquirindo intenção e consciência. desta representação. É necessário que o Outro ame verda-
O uso do símbolo agiliza – e forma - o uso da mente, que deiramente. Isto significa que também para ele algo está
trabalha com representações, abstrações e memória. Nesta vedado, só assim é possível que ele seja representante de
passagem, a economia psíquica muda. A energia em causa uma subjetividade (enquanto desejante e faltante). Em ou-
no sensório-motor – ato de tocar, saborear, mover-se para tros termos, isto é essencial para que cumpra o desígnio de
atingir objetivos – descola-se do manuseio do objeto ao oferecer àquele que depende dele (e dependeu em útero e
“manuseio psíquico” através dos símbolos; é mais econô- no processo de maternagem) a verdadeira possibilidade de
mico e amplifica espaços e recursos, aumentando o univer- nascimento subjetivo, que rompe o elo narcísico.
so mental e a realidade. O que acontece no nível energético? A descrição de
Vemos mais claramente este objetivo de contenção de Spitz não deixa dúvidas: o id buscando descarregar-se –
energia e o uso distinto na forma de abstração quando o num momento em que a criança adquire capacidade de
não ganha significado semântico. Este é o terceiro e último locomoção -, e como potência energética que é, torna-se
Organizador na teoria de Spitz. O infans que ainda se move impossibilitado de atuar sobre os objetos pela força da pa-
pelo mundo afetivamente vinculado ao tempo presente, lavra não. O conflito se instaura: amor do outro ou submis-
reagindo à presença do outro através de seus sentimentos são, dependência. Não encontrando saída – pois se sub-

3
CONHECIMENTOS ESPECÍFICOS - PSICÓLOGO

meter volta a ser apenas complemento do Outro e perde de tratamento. Um dos campos de investigação que neste
a si mesmo, entretanto se executar a ação que demanda contexto se destaca é a adolescência, que ora é tomada
seu desejo perde o amor de seu objeto amado, no caso o como fenômeno social, ora como momento necessário na
Outro materno -, surge, então, a terceira alternativa: uma constituição do sujeito. Como se sabe, não é um conceito
identificação especificamente concernente à efetivação da originalmente psicanalítico, mas surge como fenômeno da
negação enraizada neste não. Não é uma identificação com modernidade (Ariès, 1986) e demanda dos psicanalistas um
aquele que pronuncia esta palavra, mas com o que se faz entendimento a respeito. Este trabalho tem como proposta
presente nesta negativa que impera no Outro materno - a contrapor duas formulações distintas do conceito, a fim de
verdade do não, o não do código em toda sua substância. fazer notar, na diferença, as implicações teóricas.
Ou seja: o efeito presente no “não semântico”. Como o pró- No percurso errante deste conceito – que vagou entre
prio Spitz estudou em O não e o sim... (1978), só na cultura tantos –, a adolescência encontra particular acolhida entre
existe o não, uma vez que o não inexiste na natureza. os representantes da chamada psicologia do ego. Muitas
Em outros termos: há o império da cultura no Ou- referências produzidas nos últimos anos sobre adolescên-
tro materno que se expressa na eficácia do não, ou seja, cia (por exemplo, Osório, 1989; Carvajal, 1996) parecem so-
tornando-o não semântico. Este é o nome de seu Terceiro frer a influência destes autores, que estudam o fenômeno
Organizador. adolescente numa relação direta e inevitável com as trans-
Por fim, pode-se pensar que algo faz passagem, e que formações orgânicas pelas quais cada um passa com o final
se mostra presente no psiquismo do Outro materno, atra- de sua infância e entrada na vida adulta. Aberastury, por
vés da sustentação deste não. Uma f o r ç a exemplo, concebe a adolescência como “um período de
presentificada, capaz de humanizar. contradições, confuso, ambivalente, doloroso, caracteriza-
Energeticamente, a tensão contida das forças do id, do por fricções com o meio familiar e social” (Aberastury,
pela impossibilidade de descarregar-se, encontra sua saí- Knobel, 1991, p. 13), o que parece configurar a chamada
da – faltosa, claro. Provavelmente estamos falando em “su- “crise essencial da adolescência”, como diz Knobel na mes-
blimação”. Esta sofre uma transformação e serve à cultura ma obra. A crise, no entanto, não configura uma patologia,
(especificamente quanto a seus objetivos). mas a síndrome normal da adolescência. Trata-se, portan-
A partir deste contexto, o sujeito – exinfans – passa a to, de uma crise esperada, que cada um deve experimentar.
morar na linguagem – e é por ela representado. A má re- Aberastury, assim como Knobel, não descarta a im-
presentação gera sintomas, ao aprisionar o sujeito. O nome portância do campo social neste processo, na medida em
próprio em processo de reconhecimento segue o roteiro que “tanto as modificações corporais incontroláveis como
de suas representações sociais, ou inova criando seu nome os imperativos do mundo externo... exigem do adolescen-
próprio que deve sempre ser amparado e/ou reconhecido te novas pautas de convivência”. Porém, a interferência do
pela cultura (enquanto lugar do humano). mundo externo parece ocorrer mais no sentido de impor
uma determinada adaptação ao sujeito, como exteriori-
Bibliografia: dade, do que participar das mudanças subjetivas que nele
SPITZ, René A. O primeiro ano de vida, Martins Fontes: estejam ocorrendo, desde seu início. Isto porque ... as mu-
São Paulo, 2013 danças psicológicas que se produzem neste período... são
a correlação de mudanças corporais... Quando o adoles-
ADOLESCÊNCIA: CONCEITO ADOLESCENTE? cente se inclui no mundo adulto com este corpo já madu-
ro, a imagem que tem do seu corpo mudou também sua
Será a adolescência um conceito adolescente? Será identidade, e precisa então adquirir uma ideologia que lhe
preciso mais trabalho clínico e teórico para fazer da ado- permita sua adaptação ao mundo e/ou sua ação sobre ele
lescência algo mais que um fenômeno, um campo de in- para mudá-lo.
vestigação da psicanálise, com consistência e maturidade?
Será preciso mais reflexão teórica acerca da constituição A concepção de corpo implícita aqui diz respeito às
do sujeito, buscando uma metapsicologia capaz de fazer mudanças orgânicas ocorridas, pois trata-se de um ado-
eco às vicissitudes deste emblema da modernidade? Ou lescente num corpo já maduro – se fosse um corpo em seu
adolescência – como é próprio do adolescente, diga-se de significado simbólico, não poderia já estar maduro, uma
passagem – inquieta, na medida em que faz surgir as ner- vez que a adolescência está no início de seu percurso. As-
vuras do referencial teórico que sustenta o conceito? sim, o texto deixa entender que são as mudanças corpo-
Nestes cem anos de psicanálise, muito aconteceu, rais em sua perspectiva orgânica que se destacam no início
não somente no que diz respeito à produção teórica, mas deste processo: daí decorrem as mudanças psicológicas
também em relação às transformações socioculturais que (são correlatas) que, por sua vez, induzem a uma mudança
caracterizaram o período. A clínica sofre os efeitos des- de identidade e à premência de uma ideologia que permita
tas mudanças, pois o paciente de hoje não parece ser o a adaptação ou mudança do mundo – metas últimas do
mesmo que aquele do início do século passado (Pinheiro, processo.
1996). Voltar à reflexão sobre a constituição do sujeito se A ideologia a que se refere parece constituir um “siste-
justifica, portanto, não somente em função de eventuais ma de valores” e “teorias políticas e sociais”, que o adoles-
lacunas teóricas, mas sobretudo pela possibilidade de pen- cente “confronta com (as de) seu meio” e então “se posicio-
sar naqueles que chegam hoje aos consultórios em busca na”. Nesta ideologia do adolescente se configuraria como

4
CONHECIMENTOS ESPECÍFICOS - PSICÓLOGO

um pensamento próprio – no sentido de exclusivamente Esta leitura configura uma expectativa duplamente
seu – e amplo – no sentido de abranger as várias dimen- imobilizadora. No primeiro caso, a adaptação seria a sub-
sões da realidade social – sobre o mundo. A identidade missão direta ao sistema tal como se encontra, sem ques-
adolescente é, por sua vez, para Aberastury, composta por tionamento. No segundo, fica a impressão de que compete
uma “multiplicidade de identificações contemporâneas e ao adolescente a transformação do mundo, expectativa
contraditórias”. Na medida em que esta multiplicidade se sobre a atuação do jovem que em sua idealização tende
mantém e o adolescente ainda não pode “renunciar a as- a fortalecer seu contrário, ou seja, a não transformação;
pectos de si mesmo e não pode sintetizar os que vai adqui- na medida em que demonstra a sua (que ele não sabe ser
rindo”, não pode “adquirir uma identidade coerente”. Sen- também de tantos outros) impotência para tanto. No en-
do assim, espera-se do adolescente que possa harmonizar tanto, se for considerada a articulação entre adaptação e
as contradições pelas quais vem passando, a fim de sair de mudança, como saídas ambas necessárias, torna-se possí-
sua crise e encontrar uma identidade coerente. vel ver a arapuca armada ao sujeito adolescente: caminhos
Por último, há um comentário de Knobel que parece impossíveis de serem efetivados e, concomitantemente,
tornar clara a posição, tanto sua quanto de Aberastury (cf. únicos possíveis, do ponto de vista das imagens oferecidas
foi comentada sua noção de corpo), sobre os determinan- a cada jovem, e frente às quais terá que se posicionar.
tes do processo adolescente. Diz ele:
... não há dúvidas de que o elemento sociocultural in- Pautada numa determinada leitura do texto freudia-
flui com um determinismo específico nas manifestações da no, a formulação de Aberastury e Knobel da adolescência
adolescência, mas também temos que considerar que atrás se mostra, ela própria, adaptativa. Ao apontar o biológico
dessa expressão sociocultural existe um embasamento psi- como fundamento último desta transformação, põe em xe-
cobiológico que lhe dá características universais. que a legitimidade da sexualidade, como construção sin-
Longe de analisar o trabalho destes autores, esta pri- gular do sujeito a partir dos laços que estabelece e está
meira exposição visa assinalar pontos significativos de sua inserido. Efeito da concepção de indivíduo moderno, esta
conceituação. Além do questionamento sobre o alcance de leitura favorece a produção de estigmas em torno da ado-
várias formulações imaginárias acerca da adolescência, vale lescência, desconsiderando ou deixando em segundo pla-
notar a dimensão política atrelada a tal concepção. A partir no a dimensão histórica de que é resultante.
da dualidade estabelecida entre a interioridade e a exte-
Nas últimas décadas, a adolescência se tornou campo
rioridade, fazem uma cisão entre o psíquico e o biológico,
de investigação também de psicanalistas herdeiros da cha-
por um lado, e o social, por outro. Na correlação necessária
mada tradição francesa. Entre eles, Ruffino se propõe a dis-
entre os fatores, uma hierarquia é estabelecida, na qual os
cutir a adolescência como conceito psicanalítico, de modo
primeiros parecem levar a melhor: atrás do social, diz Kno-
a questionar vários problemas presentes na concepção an-
bel, encontra-se o psicobiológico. O saber psicológico po-
terior, mas também fazendo emergir questões complexas
deria encontrar independência de tantos outros cientistas
que põem em relevo determinados fundamentos da psica-
sociais, mas, curiosamente, volta a se atrelar – e quem sabe
depender – do saber médico, responsável pelo corpo em nálise. Ao tomar a adolescência como advento da moder-
seu sentido biológico. nidade, o autor marca uma primeira distinção da anterior: o
Uma vez estabelecida uma dimensão universalizante sujeito da psicanálise possui inscrição histórica, sendo, ele
do psíquico, a chamada crise adolescente se torna neces- próprio, sintoma de um percurso que o antecede. A tese
sária: é preciso que cada sujeito viva um período doloroso, sobre a qual o autor irá trabalhar é que, na ausência de
pautado por contradições, confusão e ambivalência. Esta dispositivos societários tais como os rituais de passagem
experiência subjetiva passa então a ser condição para a in- – que nas sociedades tradicionais possuíam a legitimida-
serção no mundo dos adultos, na qual a adaptação é a re- de de dar sentido a esta transição –, cabe ao sujeito fazer
ferência. Como se a ambivalência e as contradições se res- singularmente a conversão do real ao simbólico. Adolescer,
tringissem a esse momento e a síndrome passasse quan- diz, “é constituir-se de modo a fazer, na ‘interioridade’ da
do este momento patológico fosse superado, sendo ele sua história subjetiva, aquilo que faltou na ‘exterioridade
resultante, em última instância, de fatores orgânicos. Esta social’” (Ruffino, 1996). É neste sentido que a adolescência
forma de entender a adolescência, que encontra tanta res- é “uma atualização da própria estruturalidade do simbólico
sonância nos dias atuais, acaba depurando o sentido social sobre a subjetividade do homem moderno” (1993).
e político que tais experiências possam ter, restringindo-as A partir dos registros RSI formulados por Lacan, oferece
a determinados indivíduos e, consequentemente, normati- uma compreensão da adolescência que evita segmentação
zando-as para que fiquem aí contidas e bem delimitadas. entre o psicológico e o social. Permite, em contraposição,
O próprio conceito de identidade, já bastante questio- a articulação do sujeito psíquico à estrutura simbólica que
nado pela fantasia de unidade que carrega (por exemplo, sustenta o dito corpo social: cabe ao adolescente buscar
Souza, 1994), explicita esse mesmo viés normatizante: es- respostas às questões que ficaram em aberto com a fra-
pera-se deste momento de passagem que o sujeito possa gilização dos rituais de passagem, que dizem respeito ao
sintetizar e harmonizar as contradições que incomodam e “Outro-Sexo”, ao lugar que ocupa na ordem da filiação e à
ameaçam. Difícil, porém, é vislumbrar a conquista de uma ética que sustenta seus atos e escolhas. Mas, lembra o au-
identidade coerente, como é proposto, se as opções para tor, o processo não se cumpre com respostas finais, pois a
o final do processo são a adaptação e/ou a mudança do produção significante não cessa com a suposta idade adul-
mundo. ta. Trata-se de sublinhar o trabalho psíquico exigido, que

5
CONHECIMENTOS ESPECÍFICOS - PSICÓLOGO

envolve o impacto com o real, a antecipação imaginária e a adolescência, mas a saída da infância, sem que isso garanta
operação simbólica propriamente dita (1993), num entrela- a entrada na adolescência, pois, como dissera anteriormen-
çamento não linear, nem sintetizante. te, o impacto sofrido no corpo não se restringe às transfor-
Se Ruffino fala em interioridade e exterioridade, sem- mações fisiológicas (1996). Mas o que representa então a
pre o faz utilizando aspas, o que se justifica pelo fato de saída da infância, senão o confronto com o “buraco” deixa-
que, conforme lembra, “exterior e interior se interpenetram do pela ausência de dispositivos societários compartilha-
na topologia psicanalítica”. Mas por que então continuar dos (rituais de passagem) e, portanto, com a exigência do
se referindo a termos que poderiam fazer retornar à cisão trabalho de simbolização adolescente? O impacto do real
psicológico/ social? Parece menos plausível a intenção de não é constituído por uma duplicidade articulada entre a
manter a proximidade de uma concepção de adolescente puberdade e os aspectos sociais envolvidos que, de algum
vigente, que realça esta cisão. Uma resposta mais consis- modo, incidem no corpo? Se vale lembrar, como fez Ru-
tente seria que, no entrelaçamento dos registros, ainda que ffino, que a revisitada disputa entre o orgânico e o social
o imaginário seja imaginário, tem efeito sobre o simbólico; reproduz uma disputa de saberes em torno de um objeto
ou seja, tanto a imagem de um interior psíquico distante do (1993), no campo psicanalítico, a questão do corpo, em sua
social traduz um aspecto da experiência subjetiva adoles- diferença do biológico, continua exigindo reflexão, assim
cente tal como é vivida quanto a produção simbólica que como outros conceitos aí implicados, como o de pulsão e
singulariza o sujeito é feita a partir das marcas identitárias trauma.
que ficaram inscritas e passaram, necessariamente, pelo Ao tentar assinalar a complexidade de alguns destes
imaginário. conceitos, pretende-se fazer notar que a questão adoles-
Na medida em que o autor entende que “a ‘exteriori- cente configura um campo de investigação que faz sobres-
dade’ é chamada por Lacan de o ‘Campo do Outro’” (1993), sair determinados pilares da metapsicologia psicanalítica,
não poderia Ruffino somente aludir ao (grande) Outro e a qual, por sua vez, não é unívoca. Confirma-se assim o
evitar o primeiro termo? O Outro faz pensar em tantos ou- comentário de Penot, num encontro recente sobre ado-
tros, por ser tão grande e ampliado. Ruffino usa de uma lescência: “... a experiência do adolescente em crise nos
imagem de exterioridade que talvez cumpra a função de incita muito particularmente, com efeito, a reexaminar
sublinhar algo pouco nítido num Outro ampliado: “... o certos fundamentos da teoria psicanalítica...” (1995). Se a
exercício da adolescência porá o sujeito como nunca no concepção proposta por Ruffino oferece saídas a alguns
coração dessas regiões do ‘lá fora’ que permanecem à problemas próprios à concepção de Aberastury, nem por
margem do que caiu sob suspeita: produção literária, gru- isso deixa de explicitar outros, que permeiam a teoria que
pos juvenis e... a psicanálise”. Lá fora em relação ao quê? a sustenta. Assim, o conceito de adolescência se mostra
Supõe-se que àquilo que se costuma olhar quando se fala adolescente não por sua insuficiência, pois isso implicaria
em adolescência, quando o olhar do clínico se prende à no ideal de uma metapsicologia definitiva, mas sim pelo
concretude das paredes que o circunscrevem e a um pa- incômodo que produz, ao pôr em xeque o estabelecido.
drão de adolescência estigmatizante incapaz de notar o su-
jeito que ali insiste. Para o autor, o adolescente se encontra Fonte:
no coração de outras regiões, como a produção literária, MATHEUS, Thiago Corbisier Adolescência: conceito
a experiência entre pares ou a própria psicanálise, como adolescente? 2004
possibilidade de alteridade que oferece. MATHEUS, Thiago Corbisier Adolescência (Coleção Clí-
A pergunta que então surge é: aonde está este “lá nica Psicanalítica). Casa do Psicólogo: São Paulo, 2012
fora”? Aonde esta outra região se inscreve? A hipótese é
que a exterioridade mencionada por Ruffino seja a expres- COMO LIDAR COM O ADOLESCENTE
são da demanda de uma reflexão acerca do Outro que en-
tra em jogo em função da adolescência. De alguma forma, Segundo Melo2, a palavra “adolescência” vem da pala-
o conceito que norteia esta discussão é a problemática vra latina “adolesco”, que significa crescer. É uma fase cheia
noção de realidade, que entre realidade psíquica e efeti- de questionamentos e instabilidade, que se caracteriza por
vidade, realität e wirklichkeit (Freud), realidade psíquica e uma intensa busca de “si mesmo” e da própria identidade,
realidade concreta, continua inquietando os psicanalistas.1 os padrões estabelecidos são questionados, bem como cri-
Outra referência complexa e necessária é a noção de ticadas todas as escolhas de vida feita pelos pais, buscando
corpo, na medida, inclusive, em que nela incide esta du- assim a liberdade e autoafirmação.
plicidade própria à concepção de realidade. Desde o iní- O teórico da adolescência há muito tem concordado
cio, Ruffino busca deixar claro que a adolescência não se que a transição da segunda infância para a idade adulta é
restringe aos efeitos da puberdade, os quais, segundo ele, acompanhada pelo desenvolvimento de uma nova quali-
já desde Freud, dizem respeito aos “processos fisiológicos” dade de mente, caracterizada pela forma de pensar siste-
(1996). Num primeiro momento, considera que a adoles- mática, lógica e hipotética.
cência é um fenômeno “aberto pela puberdade, sob con- Através deste texto tem-se o objetivo de apresentar a
dições específicas da cultura e da história” (1993). Adiante, pesquisa de Piaget acerca do período operacional formal,
nomeia, como deflagrador do processo, “o real do impacto que constitui o ápice do desenvolvimento intelectual expli-
pubertáriosocial”. Num outro momento, ainda, considera citando as principais características deste estágio.
que o efeito causado pela puberdade não é a entrada na 2 MELO, M. A. S.

6
CONHECIMENTOS ESPECÍFICOS - PSICÓLOGO

Estágio Operacional Formal Piagetiano O adolescente exercita ideias no campo do possível e


formula hipóteses, tem o poder de construir à sua vonta-
Piaget afirmava que as mudanças na maneira como os de reflexões e teorias. Com estas capacidades, o adoles-
adolescentes pensam sobre si mesmos, sobre seus relacio- cente começa a definir conceitos e valores. Neste sentido,
namentos pessoais e sobre a natureza da sua sociedade caracteriza-se a adolescência por um egocentrismo cogni-
têm como fonte comum o desenvolvimento de uma nova tivo, pois o adolescente acredita que é capaz de resolver
estrutura lógica que ele chamava de operações formais. todos os problemas que aparecem, considerando as suas
O pensamento operatório formal é o tipo de pensa- próprias concepções como as mais corretas (crença na oni-
mento necessário para qualquer pessoa que tenha de re- potência da reflexão).
solver problemas sistematicamente. A propriedade geral mais importante do pensamento
O adolescente constrói teorias e reflete sobre seu pen- operacional formal, a partir da qual Piaget deriva todas as
samento, o pensamento formal, que constitui uma reflexão demais, refere-se à distinção entre o real e o possível. Ao
da inteligência sobre si mesma, um sistema operatório de contrário da criança que se encontra num período opera-
segunda potência, que opera com proposições. cional concreto, o adolescente, ao começar a examinar um
Segundo Piaget uma das consequências de se adquirir problema com que se defronta, tenta imaginar todas as
pensamento operatório formal é a capacidade de construir relações possíveis que seriam válidas no caso dos dados
provas lógicas em que a conclusão segue a necessidade em questão; a seguir, através de uma combinação de pro-
lógica. Essa habilidade constitui o raciocínio dedutivo. cedimentos de experimentação e de análise lógica, tentan-
O pensamento do adolescente se difere do pensamen-
do verificar quais destas relações possíveis são realmente
to da criança, ou seja, a criança consegue chegar a utili-
verdadeiras.
zar as operações concretas de classes, relações e números,
Uma estratégia cognitiva que tenta determinar a reali-
mas não as utiliza num sistema fundido único e total que
é caracterizado pela lógica do adolescente. O pensamento dade no contexto das possibilidades tem um caráter funda-
liberta-se da experiência direta e as estruturas cognitivas mentalmente hipotético-dedutivo. O adolescente ingressa
da criança adquirem maturidade. Isso significa que a qua- corajosamente no reino hipotético.
lidade potencial do seu pensamento ou raciocínio atinge o O pensamento formal é um pensamento proposicional.
máximo quando as operações formais encontram-se ple- As mais importantes entidades que o adolescente mani-
namente desenvolvidas. pula, ao raciocinar deixaram de ser os dados rudimenta-
A criança não ultrapassa a lógica elementar de agru- res da realidade e passaram a ser afirmações que contem
pamentos ou grupos numéricos aditivos ou multiplicativos, estes dados. O que é realmente alcançado entre os 7 e 11
apresentando deste modo, uma forma elementar de rever- anos de idade, é a cognição organizada de objetos e acon-
sibilidade. O adolescente apresenta a lógica das proposi- tecimentos concretos per se. O adolescente realiza estas
ções relacionando-a a estrutura de classes e das relações. O operações, mas realiza também algo que as transcende,
pensamento formal encontrado nos adolescentes é expli- algo necessário que é precisamente o que faz com que
cado pelo fato de se poderem estabelecer as coordenações seu pensamento seja formal e não mais concreto. Ele toma
entre os objetos que também se originam de determinadas os resultados destas operações concretas, formula-os sob
etapas da maturação deste sujeito. a forma de proposições e continua a operar com eles, ou
No entanto, esta constituição da estrutura, não apenas seja, estabelece vários tipos de conexão lógica entre eles.
tem ligação com o aparato maturacional do sujeito, mas Portanto, as operações formais, na realidade, são opera-
também com o meio social no qual este está inserido. Para ções realizadas com os resultados de operações anteriores.
que o meio social atue sobre os indivíduos é necessário A partir destas considerações pode-se estabelecer um
que estes estejam em condições de assimilar as contribui- paradigma inicial de como os adolescentes pensam. Inicial-
ções desse meio, havendo a necessidade de uma matura- mente organizam os vários elementos dos dados brutos
ção suficiente da capacidade cerebral deste indivíduo. Estes com as técnicas operacionais concretas dos anos interme-
fatores estão relacionados dinamicamente. diários da infância. A seguir estes elementos organizados
Se o adolescente constrói teorias é porque de um lado, são transformados em afirmações ou proposições que po-
tornou-se capaz de reflexão e, de outro, porque sua refle-
dem ser combinadas de várias maneiras. Através do méto-
xão lhe permite fugir do concreto atual na direção do pos-
do de análise combinatória, eles examinam isoladamente
sível e do abstrato. A lógica não é algo “estranho” a vida do
todas as combinações diferentes destas proposições.
sujeito, é justamente a expressão das coordenações opera-
tórias necessárias para atingir determinada ação. Para Piaget o pensamento formal é uma orientação
O pensamento do adolescente tem a necessidade de generalizada, explicita ou implícita, para solução de pro-
construir novas teorias sobre as concepções já dadas, no blemas: uma orientação no sentido de organizar os dados,
meio social, tentando chegar a uma concepção das coisas isolar e controlar variáveis, formular hipóteses e justificar e
que lhe seja própria e que lhe traga mais sucesso que seus provar logicamente os fatos.
antecessores. As operações formais podem ser caracterizadas não
São característicos do processo de pensamento, os só em termos descritivo-verbais gerais, como também em
patamares de desenvolvimento, que leva o nível mais ele- termos das estruturas lógico-matemáticas que são seus
mentar de egocentrismo à descentração, subordinando o modelos abstratos. As operações interposicionais não são
conhecimento sempre a uma constante revisão das pers- ações isoladas sem relações mútuas. Tal como os agrupa-
pectivas. mentos das operações intraposicionais dos anos interme-

7
CONHECIMENTOS ESPECÍFICOS - PSICÓLOGO

diários da infância, elas formam um sistema integrado, e o do Piaget, é o denominador comum importante: a criança
problema consiste em determinar a estrutura formal deste se ocupa, sobretudo com o presente, com o aqui e a agora,
sistema. o adolescente amplia seu âmbito conceitual e inclui o hi-
O conjunto de instrumentos conceituais que Piaget potético, o futuro e o espacialmente remoto. Esta diferença
chama de esquemas operacionais formais encontra-se tem um significado adaptativo. O adolescente começa a
num nível intermediário de generalidade. assumir papéis adultos; para ele o mundo de possibilida-
Grande parte da diferença existente entre o compor- des futuras pessoalmente relevantes - escolha profissional,
tamento diário da criança e do adolescente pode ser ex- escolha do cônjuge, etc. - passa a ser o objeto de reflexão
pressa da seguinte maneira: o adolescente, como a criança mais importante. De modo semelhante, o adulto que ele
vive no presente, mas ao contrário da criança também vive será em breve deverá relacionar-se intelectualmente com
muito na dimensão ausente, isto é, no futuro e o no reino coletividades sociais muito menos concretas e imediatas
do hipotético. Seu mundo conceitual está povoado de teo- do que a família e o círculo de amigos: a cidade, o estado,
rias informais sobre si mesmo e sobre a vida, cheio de pla- os pais, o sindicato, a igreja, etc.
nos para o seu futuro e o da sociedade, em resumo, cheio Em geral, o adolescente pretende inserir-se na socie-
de ideias que transcendem a situação imediata, as relações dade dos adultos por meio de projetos, de programas de
interpessoais atuais, etc. vida, de sistemas muitas vezes teóricos, de planos de refor-
mas políticas ou sociais.
O Pensamento e suas Operações A verdadeira adaptação à sociedade vai-se fazer au-
tomaticamente quando o adolescente, de reformador,
O que surpreende no adolescente é o seu interesse por transformar-se em realizador. A experiência reconcilia o
problemas inabituais, sem relação com as realidades vivi- pensamento formal com a realidade das coisas, o trabalho
das no dia-a-dia, ou por aqueles que antecipam, com uma efetivo e constante, desde que empreendido em situação
ingenuidade desconcertante, as situações futuras do mun- concreta e bem definida, cura todos os devaneios.
do, muitas vezes utópicas, com uma facilidade de elaborar Assim é o desenvolvimento mental, constata-se que
teorias abstratas. Existem alguns que escrevem que criam a unidade profunda dos processos que da construção do
uma filosofia, uma política, uma estética ou outra coisa. universo prático, devido à inteligência sensomotora do lac-
Outros não escrevem, mas falam. tente, chega à reconstrução do mundo pelo pensamento
Por volta de onze a doze anos efetua-se uma transfor- hipotético-dedutivo do adolescente, passando pelo conhe-
mação fundamental no pensamento da criança, que marca cimento do universo concreto devido ao sistema de opera-
o término das operações construídas durante a segunda ções da segunda infância.
infância; é a passagem do pensamento concreto para o Estas construções sucessivas consistem em descen-
“formal” (hipotético-dedutivo). tralização do ponto de vista, imediato e egocêntrico, para
Quais são na realidade, as condições de construção do situá-lo em coordenação mais ampla de relações e noções,
pensamento formal? Para criança, trata-se não somente de de maneira que cada novo agrupamento terminal integre a
aplicar as operações aos objetos, ou melhor, de executar, atividade própria, adaptando-a a uma realidade mais glo-
em pensamento, ações possíveis sobre estes objetos, mas bal. A afetividade liberta-se pouco a pouco do eu para se
de refletir estas operações independentemente dos obje- submeter, graças à reciprocidade e a coordenação dos valo-
tos e de substituí-las por simples proposições. res, às leis da cooperação; a afetividade que atribui valor às
O pensamento concreto é a representação de uma atividades e lhes regula a energia, mas ela atua em conjunto
ação possível, e o formal é a representação de uma repre- com a inteligência, que lhe fornece meios e esclarece fins.
sentação de ações possíveis.
As operações formais fornecem ao pensamento um
novo poder, que consiste em destacá-lo e libertá-lo do real, A FORMAÇÃO E ROMPIMENTO DOS LAÇOS
permitindo-lhe, assim, construir a seu modo as reflexões e AFETIVOS. A IMPORTÂNCIA DO AMBIENTE.
teorias.
CONSEQUÊNCIAS DAS FALHAS AMBIENTAIS.
Processo adaptativo do indivíduo

As reflexões precedentes poderiam levar a crer que o


desenvolvimento mental termina por volta de onze anos COMO SÃO CONSTRUÍDOS OS VÍNCULOS AFETI-
ou doze anos, e que a adolescência é simplesmente uma VOS NO MUNDO DA CRIANÇA3
crise passageira, devida à puberdade, que separa a infância
da idade adulta. A maturação do instinto sexual é marca- Um adulto normal, mentalmente e emocionalmente
da por desequilíbrios momentâneos, que dão um colorido saudável, é o que todos desejamos ser. A construção deste
afetivo muito característico a todo este último período da indivíduo começa, segundo Winnicott, desde o nascimen-
evolução psíquica. to do bebê e seu primeiro contato com sua mãe. (WINNI-
Embora o conteúdo exato das ideias do adolescente COTT, 2008) Refiro-me, aqui, exclusivamente, a mãe, mas
varie, tanto numa mesma cultura como em culturas dife- 3 GOOS, A. F. G. Formação e rompimento dos laços afetivos.
rentes, este fato não deveria obscurecer aquilo que, segun- Araraquara, 2010.

8
CONHECIMENTOS ESPECÍFICOS - PSICÓLOGO

sei perfeitamente que principalmente com as grandes Todavia, aquele bebê que teve uma mãe agitada, an-
mudanças que ocorrem atualmente na sociedade, afetan- siosa, preocupada e, até mesmo, irritada e nervosa duran-
do principalmente as relações familiares, poderíamos cha- te toda a sua gestação, terá todos os requisitos para um
mar essa “mãe” de cuidador ou cuidadora, simplesmente parto difícil ou mesmo com sérios problemas. Esse bebê
aquele que cuida do bebê. Entretanto, como a boa ou a má com uma gestação sempre agitada onde a mãe passava
formação dos vínculos afetivos da criança dependem da a ele todos os hormônios e enzimas de sua ânsia ou es-
dedicação constante e ininterrupta daquela que cuidará do tresse aprendeu a sempre estar ansioso, esperando algo
bebê, Winnicott (2008) atribui à mãe quem melhor realizará eminente acontecer. Seu sono será sempre agitado, marca-
essa recompensante tarefa como veremos mais a seguir. do por despertares repentinos e, então, consequentemen-
te, o bebê não conseguirá distinguir se nesses intervalos
Contudo, acontecimentos fatais durante uma existên-
de sono, já é hora de mamar porque está com fome ou
cia nos acometem e para tanto quando uma mãe vier a
ansioso demais para esperar pelo tempo aproximado en-
faltar, não por negligência, mas por uma fatalidade, o pai tre uma mamada saudável e outra. Dotado, então, de uma
ou mesmo a avó ou ainda algum outro membro da família ansiedade extrema, será até mesmo dificultoso o processo
que se dispuser a tomar conta do bebê que ficou sem a de pegar o seio da mãe de maneira correta de modo a
mãe e o fizer com total dedicação, não poderá ser acusa- não engolir ar demais e leite de menos, causando, desse
do, inconsequentemente, de não ter cumprido sua tarefa modo, cólicas, e sem a quantidade de leite correta, o que
de maneira que a mãe, ela mesma, tivesse realizado esse culminará em fome e em despertares mais frequentes para
feito. Nesse sentido e entendendo da mesma forma que uma nova mamada. Sabemos, é claro, que este não seria
Winnicott (2008) afirmara, usarei, aqui, a palavra mãe para o único fator contribuinte para o surgimento das cólicas,
designar aquela que cuida do bebê, provê seu sustento fí- mas, sim, o entendemos como um grande determinante e/
sico e emocional. ou agravante.
A mulher está grávida. Antes mesmo que ela tome co- Vimos de maneira simples como o bebê é influencia-
nhecimento desse novo mundo que surge em sua vida, o do desde o ventre materno. Passaremos, então, para a fase
após o seu nascimento, logo nas primeiras mamadas. A
bebê já começa a ganhar vida, forma e percepção das coi-
mulher que prezava sua liberdade para trabalhar fora e se
sas que o cerca e, quando mais desenvolvido, do mundo divertir nas horas de lazer, vai cedo descobrir que a pri-
exterior ao ventre materno. vação de tudo em benefício de um pequeno e indefeso
É desde o mais remoto início desta existência que esse ser vai agora reger toda a sua vida. Esse pequeno ser será
pequeno ser reage às emoções pelas quais a mãe passa. como escreveu Melaine Klein “sua majestade, o bebê”. To-
Serão nove meses em que o bebê vai se acostumando a das as suas necessidades serão atendidas pela mãe, mes-
ouvir as vozes daqueles que estarão por perto quando che- mo cansada, de maneira pronta e inquestionável. Não im-
gar à hora dele nascer. As emoções da mãe não são menos porta se a mãe é ou não uma mulher inteligente, instruída
percebidas. Quando a grávida está tranquila e satisfeita, o ou tão pouca experiência de vida que possuir. A verdade é
bebê sente que seu mundo está em perfeita harmonia. São que nada disso conta para o fato de ser, ou não, uma boa
inúmeras conexões neurais, enzimas e hormônios liberados mãe. Não é estranho que algo tão importante como ser
na corrente sanguínea que realizam maravilhas na forma- mãe dependa tão pouco de uma inteligência?
ção do feto. Calmo, tranquilo e sem agitação, ele se desen-
Para que os bebês se convertam, finalmente, em adul-
volverá de modo que nada o atrapalhe. Isso determinará
tos saudáveis, em indivíduos independentes, mas social-
a maneira tranquila que ocorrerá o parto, caso a mãe não mente preocupados, dependem totalmente de que lhes
possua nenhuma doença pré-existente; determinará, tam- seja dado um bom princípio, o qual está segurado, na
bém, a tranquilidade dos intervalos das mamadas e até a natureza, pela existência de um vínculo entre a mãe e o
qualidade do sono do bebê. Durante os nove meses, tudo seu bebê: amor é o nome desse vínculo. Portanto, se você
de aflitivo pode começar a ser trabalhado de modo que ama o seu filhinho, ele estará recebendo um bom princípio.
tudo ocorra com tranquilidade e contribua para uma vida (WINNICOTT, 2008, p. 17)
boa para o bebê. Um bebê que teve sua gestação na sinto-
nia de uma perfeita ordem será um bebê que dorme bem, Quanto à alimentação do bebê, muito se tem discuti-
sem interrupções desnecessárias ao sono. Ele acordará sem do durante a história, visto que nos tempos mais remotos,
estar ansioso ou agitado para mamar. Apenas estará com acreditava-se que se, por exemplo, o bebê tinha raquitismo
fome e, então, sem ansiedade, pegará o seio da melhor ou algum problema intestinal, era devido ao leite materno
maneira ou poderá ser guiado pela mãe, tranquilamente, não ser bom. Hoje, com a quantidade de informações que
dispomos, sabemos bem que não há nada melhor para a
para que isso ocorra sem problemas. Será apenas fome e
alimentação do bebê que o leite materno e do ponto de
não ânsia por mamar. Sendo assim ele não acorda várias
vista nutricional, sabemos que aquela mãe que, por ventu-
vezes antes do tempo adequado para se alimentar, como ra, tiver algum problema em que ela não possa amamentar
se o leite da mãe não fosse suficiente para saciar sua fome. o bebê, também não há motivo para pânico se ela puder
Com a fralda limpa e a fome saciada, a criança tranqui- comprar um bom leite no mercado. Não há o que temer
la não tem motivos para acordar de um sono que tem a quanto à nutrição, os leites disponíveis no mercado pos-
função de prover tudo o que seu organismo precisa para suem toda a sorte de nutrientes que o bebê precisa. Mas
crescer forte e saudável. seria a alimentação somente o ato de nutrir?

9
CONHECIMENTOS ESPECÍFICOS - PSICÓLOGO

Não só Winnicott, mas precisamente ele, discutiu mui- Ao cabo dos nove meses, aproximadamente, o bebê
to a importância da amamentação para a relação entre a inicia suas brincadeiras de atirar as coisas que tem na mão
mãe e o bebê. Essa prática é considerada a primeira e mais no chão e espera que sempre tenha um adulto que pos-
significativa relação na construção dos laços afetivos. “Todo sa apanhá-las. Essa brincadeira, em especial, nos mostra
o processo físico funciona precisamente porque a relação que o bebê está apto a se desvencilhar de algumas coisas;
emocional se está desenvolvendo naturalmente”. (WINNI- se a amamentação teve êxito, o bebê por si só será capaz
COTT, 2008, p.33) de produzir um desmame tranquilo. É a capacidade que o
Imaginem afastar um bebê do contato de sua mãe por bebê tem de abandonar as coisas e nos aproveitando disto,
alegações de que se precisam fazer exames ou tomar va- é a hora perfeita para introduzir um desmame sem proble-
cinas ou, ainda, preencher informações desnecessárias na- mas, sem que pareça obra do acaso, mas uma evolução
quele precioso momento em que o bebê se desliga da mãe nos vínculos já estabelecidos. Nesse caso a amamentação
no nascimento e procura o reencontro de maneira deses- com sucesso produziu, ao longo do período, experiências
perada. O que deveria ser crucial neste momento? mais que suficientes para que o bebê tenha bons sonhos
e ótimas recordações, habilitando-o a aceitar os riscos de
Nada pode ser mais importante para mãe ou para
maneira mais saudável. O desmame é a demolição gradual
o bebê que o contato entre mãe e filho. É a relação mais
das ilusões, que é parte das tarefas que devem ser realiza-
profunda de amor e, portanto, um dos mais importantes
das pelos pais. (WINNICOTT, 2008)
alicerces para a segurança e tranquilidade do bebê. A ama- Quando observamos um bebê que adquire algo que
mentação não é um ato mecanizado e para que produza quer, brinca um pouco e depois o abandona, jogando no
o efeito esperado, não deve ser mecânico o procedimento chão como é de costume um bebê fazer, concluímos que
realizado, em outras palavras, a amamentação não terá o esta criança foi de uma ponta à outra da experiência, po-
mesmo efeito se a enfermeira der a mamadeira desinte- dendo vivê-la desde o seu começo, seu meio e seu fim e
ressadamente e, após terminar, devolver o bebê para mãe iniciando uma consciência sobre o tempo total. Quando es-
ou ainda pior: colocá-lo para dormir. Onde estaria o esta- tamos apressados e ansiosos, não permitimos ao bebê vi-
belecimento de vínculo afetivo neste ato mecanizado? Ou venciar acontecimentos completos e o desenvolvimento e
mesmo nos hospitais, onde insistem em embrulhar o bebê a noção de tempo deste ficam prejudicados. É essa vivência
que não pode usar as mãos para sentir o contato com sua que permite ao bebê construir a consciência de que o que
mãe? Na amamentação todo cenário criado em torno da está em marcha terá um fim. O meio dos acontecimentos
mãe e seu bebê é importante. A vivacidade com que a mãe só poderá ser tolerado se houver a idéia de que existe co-
toma o bebê nos braços, o carinho com que o abraça e meço e fim. Dessa forma a mãe propicia a capacidade para
conversa com ele, o ato de deixar suas mãozinhas livres o bebê ser capaz de desfrutar todas as experiências.
para que toque seu seio, seu próprio rosto, esse contato, Essa riqueza de experiências sobre o que o bebê quer
essa relação entre mãe e filho, é extremamente importante fazer ou o que quer segurar ou soltar, não sabemos pre-
na construção dos laços afetivos iniciais. cisamente quando começa, visto que podemos observar
Para que a amamentação se torne, de fato, o primei- um bebê de apenas três meses tentando levar seu dedo
ro importante passo para a formação dos vínculos afeti- ou sua mão ao seio da mãe, mas por volta dos seis meses,
vos, a amamentação só poderá ter horários estabelecidos quando essas preferências ficam mais claras, é nesta época
quando essa relação que deve ser incentivada ocorrer de que a mãe deve ser precisa sobre o que proibir para que
maneira natural. Se o bebê quiser mamar e sua mãe não o a criança não fique sem saber seus limites. Sabendo que o
fizer porque ainda não é a hora, quero dizer, não chegou o bebê colocará tudo à boca, deve a mãe deixar perto dele
próximo horário da mamada (a cada três horas, aproxima- somente coisas que ele possa levar à boca, para que toda
a ação do bebê para descobrir o mundo não se transforme
damente), o bebê sentirá uma grande angústia, uma ansie-
numa eterna sequência de “sonoros nãos”, confundindo-o.
dade que só é restabelecida se a mãe decidir amamentar o
A mãe, desta forma, evita que o bebê fique desorientado
bebê, quando exigido, por um certo período, voltando aos
sobre o que é bom ou mal para se tocar e aos poucos vai
horários regulares quando possível, sem pressão. A base dizendo pequenas falas como “é quente”, “isto corta”, até
de aceitação da realidade externa é o primeiro período em que ele aprenda o que realmente não deve fazer porque
que a mãe obedece às necessidades do bebê de maneira representa perigo. Sempre que for possível, a mãe deve ex-
natural e a aceitação de um mundo exterior a ele começa plicar o porquê das coisas ao seu filho de maneira a produ-
a se formar tranquilamente. A alimentação infantil bem- zir entendimento e não obediência cega às ordens.
-sucedida é parte essencial para a educação da criança.
Desfrute encontrando o que há para encontrar, à me-
Por outras palavras, a única base autêntica para as re- dida que aparece da pessoa que o vosso bebê é, porque
lações de uma criança com a mãe e o pai, com as outras ele precisa isso de você. De modo que você esperará, sem
crianças e, finalmente, com a sociedade, consiste na primei- pressa, precipitação ou impaciência, que o bebê queira
ra relação bem-sucedida entre a mãe e o bebê, entre duas brincar. É isso, sobretudo, o que indica a existência de uma
pessoas, sem que mesmo uma regra de alimentação regu- vida interior pessoal no bebê. Se ele encontrar em você
lar se interponha entre elas, nem mesmo uma sentença que uma correspondente disposição lúdica, a riqueza íntima do
dite que um bebê deve ser amamentado ao peito materno. bebê desabrochará e as brincadeiras entre a mãe e o bebê
Nos assuntos humanos, os mais complexos só podem evo- tornam-se a melhor parte das relações entre ambos. (WIN-
luir a partir dos mais simples. (WINNICOTT, 2008, p.36) NICOTT, 2008, p.88)

10
CONHECIMENTOS ESPECÍFICOS - PSICÓLOGO

Uma evolução onde a família fornece bases para a se- A criança está constantemente predisposta a odiar
gurança da criança, sem desvios ou entraves, proporciona- alguém e se o pai não estiver presente para servir-lhe de
rá indivíduos saudáveis emocionalmente. alvo, ela detestará a mãe e isso confundi-la-á, visto ser à
Quanto ao desenvolvimento da moralidade na crian- mãe que a criança mais profundamente ama. (WINNICOTT,
ça, o início se dá aos seis meses de vida quando surge a 2008, p.130)
necessidade de separação do objeto por parte da criança
quando o atira ao chão. Inicia-se, portanto, a capacidade Quando pai e mãe unem-se na criação do filho está
de destruição. É essa a hora em que a mãe tem a oportu- montada a base para um bom lar e, por conseguinte, para
nidade de integrar os impulsos de atacar e destruir e de o desenvolvimento normal da criança. Mas o que chama-
dar e compartilhar, humanizando a moralidade do bebê. ríamos de uma criança normal? Seria aquela que nunca se
zanga, não demonstra raiva ou frustração ou mesmo não
Essa dedicação sem pressa e de maneira humana com que
tem seu momento que os pais chamariam de rebeldia?
a mãe integra esses sentimentos no bebê vai formando,
Mesmo a criança sadia de corpo e com intelecto exce-
gradualmente, na criança a noção de responsabilidade, que lente, não necessariamente é considerada normal. O que
a esta época é ainda o sentido de culpa. Essa fase dura dos precisamos saber é se sua personalidade e seu caráter se
seis meses aos dois anos e a mãe por perto para mediar desenvolvem de maneira adequada. Se seu desenvolvimen-
essa relação e essa confusão de sentimentos desordenados to emocional foi comprometido ou houve algum problema,
que surgirão, irá definir a idéia, por parte da criança, que a criança precisará ter um retrocesso e se comportará como
ela pode amar e odiar um objeto ao mesmo tempo. um bebê ou criança menor. Por exemplo, quando a criança
volta a urinar na cama à noite para chamar a atenção dos
A criança torna-se gradativamente apta a tolerar o pais ou afirmar seu direito como indivíduo no caso de pro-
sentimento de angústia (culpa), a respeito dos elementos testo contra a severidade que lhe foi imposta. O mesmo
destrutivos nas experiências instintivas, porque sabe que ocorre até mesmo na fase adulta quando observamos que
haverá uma oportunidade de recompensar e reconstruir [...] alguém se comporta como criança birrenta ou ameaça ter
O equilíbrio aí implícito acarreta um sentido de justo e de um ataque do coração quando se sente frustrada ou com
errado mais profundo do que quaisquer normas meramen- raiva. Para um indivíduo normal, há outras maneiras de en-
te impostas pelos pais. (WINNICOTT, 2008, p.108) frentar uma frustração.
As pessoas precisam recuperar sentimentos que per-
Até agora muito da construção da saúde mental de tenceram à infância a qualquer custo, devido à intensidade
com que foram vividos, daí a necessidade de regressão à
uma criança foi atribuída à mãe, mas somente para explicar
infância sempre que não se consegue resolver os conflitos.
seu papel. Sabemos perfeitamente que o pai constitui um
Se esses sentimentos da infância foram bem resolvidos, o
papel não menos importante na consolidação de todo o ser humano adulto sabe como lidar com os problemas que
amor e dedicação que a mãe vem construindo através de tiver à medida que for crescendo.
seu primeiro contato com o bebê que é a amamentação. A criança deve sempre lançar mão de todos os recursos
Toda mãe sabe que no dia a dia com o bebê muita coisa se que estiverem ao seu alcance para se proteger da raiva,
aprende e aquilo que possa parecer sem sentido ou impor- da angústia e da frustração. Pior será a criança que blo-
tância para quem está fora, é extremamente interessante queada, não usar desses recursos para frear seus sentimen-
para a mãe dividir com seu companheiro e mesmo ouvir- tos considerados ruins. Enquanto a criança sentir repulsa,
-lhe a opinião, mesmo que ainda não concorde. Quando raiva e descontrole em face de algo que a incomoda, ela
o bebê cresce, a riqueza de detalhes aumenta e então os estará protegida mentalmente. As crianças que os pais exi-
vínculos entre mãe e pai se estreitam cada vez mais. gem obediência cega, quando chegarem à adolescência,
Quando o pai entra em cena, é esperado pela mãe que possuirão um sentimento de rebeldia que dificilmente será
o bebê reconheça no pai sentimentos que ele associa a controlado.
mãe como carinho, ternura, prontidão, paciência, ou seja,
se o pai tiver afinidade nas ações com a mãe, rapidamen- A esses recursos, normalmente empregados pelas
te o bebê aceitará esse novo ser em sua vida, e será um crianças, é que chamamos sintomas, e dizemos que uma
grande alívio para mãe poder compartilhar isso com seu criança normal é capaz de ter qualquer espécie de sinto-
mas, em circunstâncias apropriadas. Mas com uma criança
companheiro e seu bebê. No momento em que o pai está
doente, não são os sintomas que constituem a dificuldade;
em casa e participa das brincadeiras das crianças, oferece
é o fato de que os sintomas não estão cumprindo sua ta-
com suas habilidades diferentes recursos para que a crian- refa e constituem tanto um incômodo para a criança como
ça possa intercalar em diferentes momentos de suas brin- para a mãe. (WINNICOTT, 2008)
cadeiras acrescentando sempre novos elementos.
Essa não é a única maneira em que o pai se torna im- Veremos, então, como se concretizam as primeiras ex-
portante. Um lar onde há união entre os cônjuges promove periências de independência da criança. O tão simples bi-
paz e segurança social ao desenvolvimento do bebê e a cho de pelúcia de que a criança se apropria e elege como
criança cresce segura e feliz preferido, que todas as pessoas que cuidam de crianças
O pai é também o ser humano que apoia e sustenta a observam, constitui uma riqueza de elementos para apre-
mãe em sua autoridade com a criança promovendo estabi- ender como a criança elabora suas relações com o mundo
lidades nas relações triangulares. exterior a ela e suas relações de independência.

11
CONHECIMENTOS ESPECÍFICOS - PSICÓLOGO

Esse novo objeto pode ser um bicho de pelúcia, pode tro filho adulto com propensão ao roubo, vê esta mesma
ser o nó que a criança fica fazendo no paninho ou o co- cena com terror nos olhos. Toda sorte de preocupação será
bertor que ela esfrega no rosto até dormir, entre tantos os transferida para o filho menor no sentido de impedir que o
tipos que observamos, e que constitui sua primeira posses- pior aconteça e a história se repita.
são, algo que ela tem afeto e que não pertence a ela como Talvez, será necessário que os pais estabeleçam certas
seu dedo ou sua mão. É o que chamamos de objeto tran- regras para manter o lar em ordem, como é de costume
sitório. É sua primeira relação com o outro, com o mundo. ouvir que a criança não deve pegar açúcar e outras gulo-
Quando isso acontece sabemos que tudo vai bem quanto seimas na despensa, sem o consentimento prévio da mãe.
ao desenvolvimento da criança. É o desenvolvimento do Entretanto, há a criança que rouba algo e não desfruta
sentido de segurança e da sua relação com algo externo daquilo que roubou, como há aquela que rouba por prazer
a ela. e aí sim há motivos para uma grande preocupação. Uma
É claro que existem crianças que não necessitam deste criança assim está doente. Ela não está buscando o objeto
objeto porque talvez precisem da mãe em pessoa o tempo roubado, mas a pessoa de quem esse objeto foi roubado.
todo, ou porque pularam essa parte de transição no desen- Quanto às crianças em que o ato de roubar é apenas
volvimento, fato esse que não quer dizer que a criança não passageiro, deve haver tolerância por parte dos pais que
esteja se desenvolvendo de maneira saudável.
tentam passar por essa fase da maneira mais ajuizada pos-
Não é o tamanho ou o tipo de objeto escolhido pela
sível, e de saber também que pode ser uma boa hora para
criança que importa. O que realmente tem valor são a tex-
estabelecer uma quantia em dinheiro e ensinar a criança
tura e, principalmente, o cheiro que esse objeto adquire.
a usá-lo com sabedoria. Os pais que compreendem essa
Os pais logo aprendem que não se deve lavar este objeto.
Ele ficará sujo e fedido para muitos, mas há, aí, uma razão questão não cairão em sermões para com a criança de
especial. modo a permitir que ela comece a mentir criando assim
Esse é o objeto criado pela imaginação da criança, é um real problema.
sua primeira criação do mundo e visa supri-la de afeto e Considerando, assim, tudo o que foi mencionado até
segurança quando a mãe não está por perto, na hora de agora, constata-se/ baseia-se no princípio de que conhecer
dormir, entre outras situações de angústia pelas quais a a criança desde o início é estreitar não só os laços de afeto,
criança passe. Por esta razão ele não deve ser simplesmen- mas também os laços de compreensão mútua tanto para
te apresentado pela mãe ou por outro membro da família. entender o choro de um bebê bem como um momentâneo
Isso rouba da criança a capacidade de criar e então o pri- afastamento do seu filho com os problemas peculiares da
meiro sentido do objeto transitório se perde. adolescência. Se essa relação foi bem construída, desde o
As técnicas usadas pela criança em momentos de se- início, será mais fácil entender e ajudar seu filho.
paração ou aflição são sem fim. Pode ser o bichinho que
a agrade, pode ser o cobertor ou mesmo uma fralda ma- O que é significativo é a experiência individual de de-
cia, entendendo, assim, que sempre esse objeto usado será senvolvimento desde bebê a uma criança e um adolescen-
de textura macia para proporcionar conforto, que é uma te, numa família que continua existindo e que se considera
das suas funções. Às vezes pode ser uma extensão da mãe capaz de enfrentar os seus próprios problemas localiza-
como seu cabelo que a criança enrola e termina por es- dos – os problemas do mundo em miniatura. Em minia-
fregar no rosto até adormecer ou mesmo o murmurar de tura, sim... mas não menor no que respeita à intensidade
alguns sons que podem acalmá-la até que durma. de sentimentos e riqueza de experiência, menor apenas na
É a partir do interesse da criança nos objetos transitó- acepção relativamente secundária da quantidade e com-
rios que ela desenvolve a capacidade de cuidar dos brin- plexidade. (WINNICOTT, 2008)
quedos e dos animais no futuro.
Em estado saudável, esses fenômenos transitórios evo- Sabemos que a alimentação não é a única maneira de
luem para a capacidade de brincar que é um sintoma do se estabelecer uma relação boa com a criança, entretan-
desenvolvimento saudável da criança. Aos poucos eles sim-
to é a primeira e uma das mais importantes. Sendo assim,
plesmente desaparecem para dar lugar às brincadeiras, que
se essa relação for bem embasada a criança será capaz de
transitam entre o mundo exterior e o sonho.
construir sua relação com a mãe de maneira que essa re-
Dentre muitos dos problemas que a criança pode ma-
lação possa superar todas as frustrações e revezes da vida,
nifestar no decorrer da infância como, por exemplo, aces-
sos de cólera, gritos noturnos, hábitos de asseio, entre até mesmo a perda por separação. (WINNICOTT, 2008)
outros, está o terrível hábito de roubar que deixa a mãe Dentre as muitas situações ideais já expostas, existem
extremamente preocupada e sem ação. Até onde é apenas algumas outras situações a serem consideradas a respeito
uma fase ou algum problema na formação dos laços afeti- da construção positiva dos laços afetivos, ou seja, quando
vos ocorreu? estes se alicerçam no lar ideal.
É de se esperar que a mãe considere apenas uma fase, Uma situação que merece ser observada com muito
toda a boa criança pega suas moedas e se diverte com elas, carinho é o brincar da criança, onde ela exercita muitas de
ou até mesmo mexe em sua bolsa, vez ou outra, e espar- suas fantasias e também traz o mundo real para ser experi-
rama tudo. A mãe sabe que essa é apenas uma fase e até mentado e retratado por ela da maneira que esta concebe
se diverte com isto. Mas para aquela mãe que já tem ou- o mundo exterior.

12
CONHECIMENTOS ESPECÍFICOS - PSICÓLOGO

Um dos grandes exemplos disso é o “brincar de casi- espaço, seu mesmo, onde ninguém tenha que dizer a ela o
nha” ou de “papai e mamãe”. Quando a criança experimen- que pode fazer ou não ou mesmo como algo deve ser feito.
ta o brincar de casinha, ela está recriando o mundo real em Um espaço só dela, um canto onde possa brincar sem se
que vive. Ela distribuirá as tarefas de acordo com que são preocupar se vai sujar ou bagunçar, um lugar, uma parede
cumpridas em sua própria casa. Haverá um dos cônjuges só sua que possa pintar e rabiscar, se expressar sem preo-
retratado por um e outro; outra pessoa será o filho e filha, cupações. Nesse local reservado, ela se sentirá com direito
e assim por diante. Viverão por algumas horas como se fos- e segurança para realizar suas ideias, fantasias, brincar e
sem adultos com grandes responsabilidades e deveres, ar- crescer saudável. Já vimos como a brincadeira, o brincar é
rumarão a casa e até mesmo manterão uma estrutura onde fundamental para a criança. Garante Winnicott (2008) que é
“seus filhos” poderão exercer sua própria espontaneidade. grande a compensação quando a mãe permite que a crian-
Algumas crianças irão recriar a situação em que vivem se ça tenha seus direitos assegurados, desta forma, no início
esta não for ideal. Poderão também retratar “uma vida de e, mais tarde, aumentando os direitos e responsabilidades
novela”, aquela que gostariam de ter no lar se não a tive- das crianças, deixando, por exemplo, que a criança planeje
rem de fato. O importante é que depois de viverem tudo ou ajude a organizar seu aniversário, um passeio ou qual-
isto como “adulto”, elas guardarão os brinquedos e irão quer outra responsabilidade dada a ela que a permita viver
jantar como crianças normais e gulosas, brigando por um como indivíduo onde a família confia e, portanto, cresce de
copo de suco a mais do que o permitido. Se a criança tem maneira maravilhosa.
um lar adequado, pode continuar a descobrir sua espon- Podemos também observar que a criança, como já dito
taneidade. Se na vida real elas possuem ótimos pais e não anteriormente, precisa de um ambiente estável e seguro,
precisam se preocupar com isso, podem continuar a serem com rotinas e horários estabelecidos para sua segurança
pais e mães eles próprios só nas brincadeiras. “Sabemos interior. As crianças não gostam de uma confusão que não
que isso é saudável; se as crianças podem brincar assim cessa, nem da total bagunça que a falta de rotina se ca-
juntas, não precisarão mais tarde que lhes ensinem formar racteriza. Isso faz com que se sintam inseguras para se de-
um lar. Já conhecem o essencial. ” (WINNICOTT, 2008). senvolverem, sentindo-se o tempo todo dependente dos
pais. Não há segurança para voarem sozinhas e, portanto,
A deficiência na formação dos vínculos afetivos e/ para que se arriscarem? A criança pode então ser lesada se
ou seu rompimento a mãe não se mostrar preocupada com os seus direitos. A
mãe que não é dependente e é suficientemente confiante
Podemos observar que à criança faz-se necessário de seu papel como mulher e mãe, saberá deixar seu filho
amor e dedicação para um desenvolvimento adequado. ter direitos, crescer de maneira progressiva, saudável. É cla-
Além disso, ela vai precisar de um lar estável, onde encon- ro que o outro extremo, aquela mãe que permite que os
tre além de amor e dedicação, um lar com atitudes consis- filhos façam tudo como querem, é de igual ou pior modo
tentes, com rotinas para que a criança saiba construir seu prejudicial para todos, principalmente para a criança, que
mundo sem surpresas grandes quando ainda não é hora. perde sua maior orientação para a vida.
A criança cujos pais souberem que para criarem um filho Outro assunto já mencionado anteriormente é a im-
de maneira saudável não é necessário nenhum profundo portância do afeto, também tema central deste trabalho.
conhecimento ou inteligência além do normal, mesmo por- Quanto a esse assunto temos a obrigação de levar em con-
que alguém que possuir o conhecimento em anatomia não sideração a ausência da mãe, seja por qual motivo for.
vai garantir que seu filho seja saudável, essa pessoa deve A criança pode ficar sem a mãe por algum tempo sem
saber que para tanto só é necessário amor profundo e de- se perceber uma mudança imediata uma vez que a crian-
dicação, atenção total àquele ser que precisa de amor para ça conserva, durante algum tempo, dentro de si a imagem
se desenvolver e que se isso não lhe faltar, com certeza viva da mãe. Mas se a mãe de quem essa criança depende
teremos uma base sólida de construção da personalidade se tornar ausente por um limite de tempo que excede a
onde ninguém nem nada no mundo poderão destruir. É capacidade da criança de conservar dentro de si a imagem
exatamente dessa forma que se formará um indivíduo com viva da mãe, isso fará com que o objeto transitório que a
perfeita saúde mental para enfrentar as adversidades da criança usa para acalmar-se quando a mãe não está por
vida. perto, perca seu significado e a criança torna-se, então, in-
capaz de usá-lo. Aqui se perdeu toda área intermediária de
[...] se a mãe não souber ver no filho recém-nascido um contato afetivo. Acaso a mãe retorne ao lar, a criança tem
ser humano, haverá poucas probabilidades de que a saúde que reconstruir todo o afeto novamente e isso leva um de-
mental seja alicerçada com uma solidez tal que a criança, terminado tempo, para que depois, após confiar novamen-
em sua vida posterior, possa ostentar uma personalidade te na mãe e ter seus vínculos afetivos restabelecidos, essa
rica e estável, suscetível não só de adaptar-se ao mundo, criança possa recomeçar a usar os objetos intermediários.
mas também de participar de um mundo que exige adap-
tação. (WINNICOTT, 2008) Os roubos por crianças privadas desse contato e que
estão recuperando-o podem ser considerados como fa-
Importante também se faz a mãe permitir que o filho zendo parte da busca de um objeto transitório, que se per-
viva suas experiências salvaguardando seus direitos dentro de através da morte ou desaparecimento da versão interio-
de casa. É necessário para início que a criança tenha seu rizada da mãe. (WINNICOTT, 2008)

13
CONHECIMENTOS ESPECÍFICOS - PSICÓLOGO

Entretanto, quando a fase do roubo se torna um pouco vo e não ativo. Ele simplesmente reage aos fatores ambien-
mais compulsiva, pede tato e atenção por parte dos pais. tais e não age por condições internas ou fatores genéticos.
No início devem ser tolerantes e depois devem dispensar Os bebês que não recebem um desenvolvimento egoico,
certa dose de atenção todos os dias. É necessário que os são bebês propensos à inquietude, estranhamento, apatia,
pais não perguntem o porquê das coisas o tempo todo, inibição e complacência. É com o desenvolvimento de um
pois ao exigirem uma explicação a qualquer custo, estarão ego forte que as crianças podem começar a formar suas
forçando a criança que já rouba a também mentir e então o memórias e expectativas.
círculo vicioso só aumenta e intensifica a ponto de tornar- Quando o bebê percebe que seu objeto impiedosa-
-se de fato um grande problema. A criança não pode dar mente atacado é o mesmo objeto amado, surge o estágio
a explicação simplesmente porque a ignora e o resultado da preocupação.
poderá ser que em vez de sentir uma culpa insuportável, De acordo com a teoria de Melaine Klein, a criança é
ela opte sem perceber por mentir compulsivamente. agora obrigada a lidar com dois conjuntos de fenômenos.
De qualquer maneira toda a criança que sofreu um de- Uma coisa boa foi atacada e provocou um sentimento sa-
silusionamento está apta a fazer coisas sem um por que; tisfatório na criança. Algo de bom foi formado dentro dela.
criam confusões, roubam pequenas coisas, recusam-se a Mas também tem que lidar com a culpa que emerge deste
defecar no momento apropriado, cortam as flores do jar- fato. Surge, então, uma saída para o problema: a criança
dim, pintam as paredes etc. torna-se capaz de dar em troca, reparar, consertar, devol-
Falando em retroceder, e isso às vezes faz-se necessá- ver aquilo que na fantasia dela foi roubado. Quando tudo
rio, assistimos por diversas vezes o retorno ao estado de vai bem, não se desenvolve um sentido exclusivamente de
bebê em crianças mais velhas. Alguma dificuldade impediu culpa, mas desenvolve-se outro bem mais importante, um
a marcha do progresso e, então, a criança sente necessida- sentido de responsabilidade. A culpa só deve vir à tona
de de regressar ao terreno protegido da infância, a fim de quando o sentido de reparação se torna insuficiente. De
restabelecer os direitos infantis e as leis de desenvolvimen- acordo com as ideias de Françoise Dolto (2003), quanto
to natural segundo Winnicott (2008). Uma das regressões menos existe o sentimento de culpa, mais existe o da res-
típicas é voltar a chupar o dedo, fato este que a criança usa- ponsabilidade entre os seres humanos.
va como objeto transitório quando bebê. Como sabemos, É importante salientar que dificuldades nesse campo,
o objeto transitório é, segundo Winnicott (2008), a base de associadas à repressão de conflitos dolorosos, dão origem
toda a vida cultural do ser humano adulto e faz parte do a várias manifestações neuróticas e distúrbios de humor de
desenvolvimento emocional normal. Se quando bebê hou- qualidade psicótico que virão a se tornar material para a
ver uma privação desta técnica como objeto transitório, tal loucura propriamente dita.
ocorrência poderá acarretar em inquietação e insônia. Isso
faz parte também da formação do afeto, que se perdida, Problemas e transtornos de comportamento de-
revela uma criança carente e demonstrará, quando mais correntes da deficiência da formação e/ou rompimen-
velha, uma tendência a ser antissocial ou até mesmo uma to dos laços afetivos
propensão à delinquência.
Outro problema que pode se manifestar devido à falha Como já foi mencionado anteriormente, o primeiro e
na formação dos vínculos afetivos é a deficiência no pro- mais persistente vínculo afetivo é o da mãe e seu filho. É
cesso de holding. talvez o único vínculo que persiste até a vida adulta, possi-
bilitando afirmação de que a relação entre mãe e filho, mes-
O desenvolvimento, em poucas palavras, é uma função mo depois de separados, quando o filho se torna adulto, é
da herança de um processo de maturação, e da acumu- o vínculo que nem mesmo a morte dissocia. Entretanto de
lação de experiências da vida; mas esse desenvolvimento uma forma um tanto paradoxal, é importante salientar que
só pode ocorrer num ambiente propiciador. A importância o comportamento do tipo agressivo desempenha um pa-
deste ambiente propiciador é absoluta no início, e a seguir pel crucial e decisivo na manutenção dos vínculos afetivos.
relativa; o processo de desenvolvimento pode ser descrito Esse comportamento assume duas formas distintas: pri-
em termos de dependência absoluta, dependência relativa meiros ataques de afugentamento de intrusos e, segundo,
e um caminhar rumo à independência. (WINNICOTT, 2005) a punição de um parceiro errante, seja ele esposa, marido
ou filho. Há provas de que boa parte do comportamento
Outro aspecto que ainda vale mencionar é a formação agressivo de um tipo desconcertante e patológico tem ori-
do ego. Quando a relação mãe-filho é sadia, o ego forma- gem em uma ou outra dessas formas (Bowlby, 2015).
do é um ego reforçado segundo as palavras de Winnicott Os vínculos afetivos e os estados emocionais cami-
(2008). É um ego capaz de desenvolver defesas e organizar nham juntos. Sendo assim, muitas das emoções humanas
padrões pessoais fortemente desenvolvidos e embasados surgem durante a formação, manutenção e rompimento
em padrões hereditários. É com esse ego muito forte que a dos vínculos afetivos.
criança logo se torna ela mesma. Se o ego da mãe é fraco, Em termos subjetivos podemos descrever que a amea-
o ego da criança torna-se reativo ao ambiente. A criança ça da perda gera ansiedade e a perda real causa tristeza, ao
não se desenvolve no aspecto pessoal e suas reações são passo que ambas as situações podem despertar raiva. Fi-
mais contra os revezes ambientais do que devido a fatores nalmente, a manutenção incontestada de um vínculo é ex-
e urgências internas. Como já mencionado, é um ego reati- perimentada como uma fonte de segurança e a renovação

14
CONHECIMENTOS ESPECÍFICOS - PSICÓLOGO

de um vínculo como uma fonte de júbilo (BOWLBY, 2015). quinquênio da infância. Segundo as pesquisas, “as indica-
Portanto, qualquer pessoa interessada em estudar os pro- ções são de que a perda por um dos pais por morte ocorre
blemas na formação dos vínculos afetivos de um indivíduo, com frequência duas vezes maior num grupo de depressi-
vai efetivamente se deparar com distúrbios de personalida- vos do que na população em geral”. (BOWLBY, 2015)
de que muito frequentemente essas pessoas estão sujei-
tas a desenvolverem. Para iniciarmos, é comprovadamente Assim, parece agora razoavelmente certo que, em nu-
produtivo considerar muitos distúrbios psiconeuróticos merosos grupos de pacientes psiquiátricos, a incidência
e de personalidade nos seres humanos como um reflexo de rompimento de vínculos afetivos durante a infância é
de um distúrbio da capacidade para estabelecer vínculos significativamente elevada. [...] As maiores incidências de
afetivos, em virtude de uma falha no desenvolvimento na vínculos afetivos desfeitos incluem tanto os vínculos com
infância ou de um transtorno subsequente (BOWLBY, 2015). os pais como com as mães, e são observados entre os cinco
Aqueles que padecem de distúrbios psiquiátricos – e os catorze anos, tanto quanto nos primeiros cinco anos.
psiconeuróticos sociopáticos ou psicóticos – manifestam Além disso nas condições mais extremas – sociopatia e ten-
sempre uma deterioração da capacidade para estabelecer dências suicidas – não só é provável que uma perda inicial
ou manter vínculos afetivos, uma deterioração que, com tenha ocorrido nos primeiros anos de vida, mas também é
frequência, é grave e duradoura e, em muitos casos, é pri- provável que tenha sido uma perda permanente, seguida
mária derivando de falhas no desenvolvimento, que terão da experiência de repetidas mudanças de figuras parentais.
ocorrido numa infância vivida num ambiente familiar que (BOWLBY, 2001, p. 104)
não foi propício ao desenvolvimento do ser humano (BO-
WLBY, 2015), permitindo-nos classificá-lo como um lar que Há também aquilo que chamamos de efeitos em curto
não é ideal. prazo de vínculos desfeitos. Quando uma criança pequena
Ao examinarem as possíveis causas dos distúrbios psi- se vê entre estranhos e longe da figura dos pais, tal fato
quiátricos na infância, ficou constatado que o problema se torna-se motivo de grande aflição e comprometimento
encontra na ausência de oportunidades para estabelecer posterior nas relações parentais.
vínculos afetivos ou, ainda, as repetidas rupturas dos vín- Nas crianças separadas dos pais foram observados
culos que foram estabelecidos. Foi sistematicamente apu- dois comportamentos antagônicos.
rado que duas síndromes psiquiátricas e duas espécies de De um lado, crianças desligadas emocionalmente e,
sintomas associados são precedidas por uma elevada inci- noutro, crianças extremamente dependentes, requisitan-
dência de vínculos afetivos desfeitos durante a infância. As do atenção dos pais o tempo inteiro. Em sua maioria as
síndromes são a personalidade psicopática (ou sociopática) crianças de dois anos que permaneceram desligadas dos
e a depressão; os sintomas persistentes, a delinquência e o pais por uma ou duas semanas, experimentaram no seu
suicídio (BOWLBY, 2015). regresso uma atitude distante e desligada da mãe. No en-
No psicopata, a capacidade de estabelecer e manter tanto, quando a criança está longe dos pais nos primeiros
os vínculos afetivos são dificultosas ou até mesmo inexis- dias experimenta um grande desespero em querer a mãe
tentes. É constatado que tais indivíduos foram seriamente e, às vezes, chora muito sua falta. Quando finalmente re-
perturbados na infância pela morte, separação ou divór- gressa parece não reconhecer e até mesmo evitá-la. Todo o
cio dos pais ou, ainda, por outros eventos que resultam na comportamento de busca afetiva está ausente e continuará
deficiência ou ruptura dos vínculos afetivos. As incidências assim por um período de tempo, pois essa reaproximação é
desses tipos de problemas são maiores nesses grupos do sempre lenta e gradual e dependerá do tempo em que du-
que em qualquer outro. Adotando como critério a ausência rou o desligamento, como já foi mencionado em capítulos
da mãe durante seis meses ou mais, antes dos seis anos de anteriores. Quando o vínculo é reatado e o comportamento
idade, foi apurada uma incidência de 41% para os socio- de ligação se estabelece, a criança torna-se extremamente
patas e somente 5% para os restantes. Quando o critério é ligada a mãe demandando extensa dedicação de sua par-
ampliado, a incidência aumenta. E, ainda, quando foi ado- te. Se a mãe, por sua vez, não se demonstra disponível, a
tada a ausência da mãe e do pai antes dos dez anos como criança torna-se muito hostil e com comportamento nega-
critérios para pesquisa, foi constatado que o índice sobe tivista, o que não foi notado em crianças que não sofreram
para 65%. separação.
Outro grupo psiquiátrico que mostra incidência mui- Outro assunto tratado por Bowlby (2015) é que o tipo
to alta de perda na infância é a dos pacientes suicidas. As de perda ocorrido durante a infância determina o tipo de
perdas ocorrem na infância, mais precisamente até os cinco depressão que a pessoa poderá ter na vida adulta.
anos de idade, tendo sido causadas não só pela morte de
um dos pais como também por ilegitimidade e o divórcio [...] as mulheres que perderam a mãe por morte ou se-
como nos mostra Bowlby (2015). paração antes dos onze anos de idade, são mais propensas
Outra condição, que está associada às perdas na in- a reagir à perda, ameaça de perda e outras dificuldades e
fância, é a depressão. Entretanto, é importante salientar crises na vida adulta mediante o desenvolvimento de um
que essas perdas não se devem frequentemente por ile- distúrbio depressivo do que mulheres que não experimen-
gitimidade ou divórcio dos pais, mas com mais incidência taram essa perda na infância. Em segundo lugar, se uma
por morte de um deles. A orfandade tende a ser maior dos mulher sofreu uma ou mais perdas de membros da família
cinco aos dez anos de idade e em alguns casos no terceiro por morte ou separação antes dos 17 anos de idade, qual-

15
CONHECIMENTOS ESPECÍFICOS - PSICÓLOGO

quer depressão que se desenvolva subsequentemente é vê inserida no triângulo edípico, denominado organização
susceptível de ser mais grave do que uma mulher que não genital precoce, e, na adolescência, quando a maturação
tenha sofrido perdas desse tipo. Em terceiro lugar, a forma genital obriga a criança a definir seu papel na procriação,
assumida pela perda na infância afeta a forma de qualquer havendo um movimento mais intenso na adolescência para
doença depressiva que possa desenvolver-se mais tarde. que o filho alcance maior autonomia.
Quando a perda na infância foi devida a separação, é pro- Para Aberatury (1991), o lugar do pai, entre seis e doze
vável que qualquer doença que seja subsequentemente meses, não é tão destacado na literatura, como acontece
contraída mostre características de depressão neurótica, com a figura materna, no entanto, o contato corporal entre
com sintomas de ansiedade. Quando a perda se deve a o bebê e o pai, no cotidiano, é referência na organização
morte, qualquer doença que se desenvolva subsequente- psíquica da criança, devido à sua função estruturante para
mente poderá apresentar características de depressão psi- o desenvolvimento do ego. No segundo ano de vida, já
cótica. (Bowlby, 2015) existe a imagem de pai e de mãe, e a figura paterna fica
mais acentuada e tem a função de apoiar o desenvolvi-
Ansiedade, depressão ou até mesmo suicídio são os ti- mento social da criança, auxiliando-a nas dificuldades pe-
pos mais comuns de problemas atribuídos aos rompimen- culiares a este período e no desprendimento necessário da
tos dos laços afetivos. Sabemos que crianças separadas das criança aos costumes da situação familiar, mantidos pela
mães até os primeiros cinco anos de idade são frequentes mãe.
em pacientes mais tarde diagnosticados como psicopatas Muza afirma que o pai aparece como o terceiro im-
ou sociopatas. Sabemos, também, que grandes perdas afe- prescindível para que a criança elabore a perda da relação
tivas (poderão acarretar?) acarretarão, mais tarde, proble- inicial com a mãe, sendo que a criança necessita do pai
mas potencialmente perigosos. Entre essas perdas faz-se para desprender-se da mãe e, ao mesmo tempo, também
necessário citar o luto e o pesar na infância. necessita de um pai e de uma mãe para satisfazer, por iden-
tificação, sua bissexualidade. Este autor afirma, ainda, que
Fonte: o pai passa a representar um princípio de realidade e de
BOWLBY, J. Formação e Rompimento dos laços afeti-
ordem na família, e a criança sente que ela não é mais a
vos. São Paulo: Martins Editora, 2015
única a compartilhar a atenção da mãe.
Cavalcante, apoiada pela teoria junguiana, sustenta
que o arquétipo do pai, vivenciado através da encarnação
O PAPEL DO PAI. no pai real, é o símbolo que promove a estruturação psí-
quica da criança e lhe permite abrir-se para o horizonte
de novas possibilidades. Neste sentido, a identificação da
criança com o universo de seu pai se dá por meio da expe-
O PAI E A FUNÇÃO PATERNA riência da interação, quando ele aparece como interdito na
relação urobórica, entre mãe e filho e a sua presença marca,
É reconhecido como importante o papel do pai no de- simbolicamente, a dinâmica de rompimento desta fase.
senvolvimento da criança e a interação entre pai e filho é
um dos fatores decisivos para o desenvolvimento cognitivo Corneau, fundamentado pelas ideias de Lacan, reafir-
e social, facilitando a capacidade de aprendizagem e a in- ma que o pai é o primeiro outro que a criança encontra fora
tegração da criança na comunidade. A experiência clínica do ventre de sua mãe, sendo ele indistinto para o recém-
tem mostrado que, na vida adulta, as representações dessa -nascido, mas ao bloquear o desejo incestuoso, sua figura
vivência insurgem nas várias possibilidades de construção vai se diferenciando, permitindo o nascimento da interio-
psicoafetiva, com repercussão nas relações sociais. ridade do filho e desfaz, assim, a fusão entre o eu e o não
As teorias psicológicas e as pesquisas científicas afir- eu, o pai encarna inicialmente a não mãe e dá forma a tudo
mam e fundamentam o papel da figura paterna no desen- que não seja ela. A presença do pai é que poderá facilitar à
volvimento e no psiquismo infantil. É pressuposto da teoria criança a passagem do mundo da família para o da socie-
psicanalítica o papel estruturante do pai, a partir da instau- dade. Será permitido o acesso à agressividade, à afirmação
ração do complexo de Édipo. Na trama familiar, o sujeito de si, à capacidade de se defender e de explorar o ambien-
se constrói e sai do estado de natureza para ingressar na te. Este mesmo autor acredita que as crianças que sentem
cultura. Freud, em seu trabalho Leonardo da Vinci e uma o pai próximo e presente sentem-se mais seguras em seus
lembrança da sua infância, afirma: “na maioria dos seres estudos, na escolha de uma profissão ou na tomada de ini-
humanos, tanto hoje como nos tempos primitivos, a ne- ciativas pessoais.
cessidade de se apoiar numa autoridade de qualquer es- A partir de um estudo de caso clínico e de uma rigoro-
pécie é tão imperativa que seu mundo desmorona se essa sa revisão da literatura, relacionada à importância da figu-
autoridade é ameaçada”. Para Aberastury, o pai representa ra paterna na vida dos filhos, Eizirik e Bergamann afirmam
a possibilidade do equilíbrio pensado como regulador da que a ausência paterna tem potencial para gerar conflitos
capacidade da criança investir no mundo real. A necessi- no desenvolvimento psicológico e cognitivo da criança,
dade da figura paterna no processo de desenvolvimento bem como influenciar o desenvolvimento de distúrbios de
infantil ocorre entre seis e doze meses, quando a criança se comportamento.

16
CONHECIMENTOS ESPECÍFICOS - PSICÓLOGO

Shinn revisou os efeitos da ausência paterna no de- enquanto uma relação positiva mãe-adolescente atenuou
senvolvimento cognitivo das crianças e concluiu que, em este risco. Uma forte relação mãe-adolescente também
famílias sem a presença do pai ou nas quais os pais apre- mostrou proteger adolescentes de famílias sem pai do risco
sentavam pouca interação com seus filhos, havia maior as- de distúrbios comportamentais associados ao envolvimen-
sociação com desempenhos pobres em testes cognitivos to com pares com tais problemas.
das crianças. Paschall et al. estudaram os efeitos de ausência pater-
Montgomery observou que crianças com ausência do na, cuidado parental e associação com pares delinquentes
pai biológico têm duas vezes mais probabilidade de repetir entre adolescentes afroamericanos com comportamento
o ano escolar, e que as crianças que apresentam compor- delinquente. Os autores relataram que os achados de es-
tamento violento nas escolas têm 11 vezes mais chance de tudos relacionados ao tema são mistos e inconclusivos e
não conviver na companhia do pai biológico do que crian- que há grande preocupação a respeito da ausência paterna
ças que não têm comportamento violento. Essas crianças, em famílias afroamericanas em relação ao efeito negativo
principalmente meninos, evidenciam maiores dificuldades que isso pode causar no desenvolvimento desses meninos.
nas provas finais e uma média mais baixa de leitura. Nesse estudo, a ausência paterna não foi associada com
Além do papel crucial que o pai exerce na triangulação comportamento delinquente dos filhos e também não foi
pai-mãe-filho, Muza cita que o papel paterno é crucial tam- moderadora da relação entre associação com pares delin-
bém para o desenvolvimento dos filhos na entrada na ado- quentes e comportamento delinquente dos filhos.
lescência, quando a maturação genital obriga a criança a Mas o efeito negativo do fator socioeconômico no
definir o seu papel na procriação. O impacto da ausência do comportamento delinquente foi mais frequente em famí-
pai na adolescência foi estudado por Jones et al., que com- lias com pai ausente. Pfiffner et al. estudaram a associação
pararam a separação psicológica e separação-individuação entre ausência paterna e características antissociais fami-
dos pais de 50 meninos, subdivididos em dois grupos, um liares. Os resultados apontaram que famílias com o pai
de 25 meninos adolescentes que viviam com seus dois pais morando em casa tiveram menos sintomas antissociais na
biológicos e 25 meninos adolescentes que viviam apenas mãe, no pai e na criança do que famílias sem o pai. Ca-
com suas mães biológicas. Os resultados mostraram que racterísticas antissociais foram maiores quando os pais não
os meninos dos dois grupos não diferiram nas medidas de
foram encontrados para participação no estudo. Os auto-
separação-individuação, e que a qualidade da relação mãe-
res concluem que comportamento antissocial em qualquer
-filho mediou muitas manifestações de separação-indivi-
membro da família é mais provável se o pai é ausente ou
duação avaliadas. Estes resultados enfatizam a importância
não-participativo.
da qualidade da relação do filho com sua mãe e com seu
pai como um mediador de muitas dimensões do proces-
O vazio promovido pela ausência do pai, segundo
so de separação-individuação. Segundo Muza, crianças
Ferrari, é formado pela noção das crianças de não serem
que não convivem com o pai acabam tendo problemas de
amadas pelo genitor que está ausente, com uma grande
identificação sexual, dificuldades de reconhecer limites e
de aprender regras de convivência social. Isso mostraria a desvalorização de si mesmas, em consequência disso. Além
dificuldade de internalização de um pai simbólico, capaz de dessa autodesvalorização, ocorrem os sentimentos de cul-
representar a instância moral do indivíduo. Tal falta pode pa por a criança se achar má, por acreditar haver provoca-
se manifestar de diversas maneiras, entre elas uma maior do a separação e até por ter nascido. A criança pensa ser
propensão para o envolvimento com a delinquência. má também por ter sido deixada. O autor coloca que isso
pode gerar reações variadas, desde tristeza e melancolia
Mason et al. abordaram os problemas de comporta- até agressividade e violência. E prossegue dizendo que os
mento associados ao efeito dos pares e ao papel modera- tímidos e temerosos do exterior se fecham em si mesmos,
dor da ausência paterna e da relação mãe-filho. O compor- e os extrovertidos e temerosos do interior de sua história
tamento dos pares e a ausência paterna vêm sendo associa- se vingam no mundo com condutas antissociais. Para Eizi-
dos com maiores índices de distúrbios do comportamento rich e Bergmann e Gomes e Resende, a literatura evidencia
em adolescentes. Pesquisas demonstram que a ausência as modificações na estrutura das famílias contemporâneas,
paterna geralmente tem impacto negativo em crianças e os efeitos negativos da ausência do pai e as repercussões
adolescentes, sendo que estes estariam em maior risco decorrentes dessa ausência, tanto nos aspectos comporta-
para desenvolver problemas de comportamento. O estudo mentais, quanto nas vivências emocionais relacionadas ao
examinou o impacto dos pares, a ausência paterna e a re- complexo de Édipo. Estes autores relacionam a ausência da
lação mãe-filho em 112 adolescentes afroamericanos com figura paterna à produção de variadas expressões de con-
problemas de comportamento. Um modelo moderador foi flitos, defesas e sentimentos de culpa nos filhos.
usado para testar a hipótese de que a ausência do pai (ou Para Gai, atualmente, tenta-se que a intervenção do pai
um equivalente) exacerbaria o impacto negativo de pares seja cada vez mais precoce, inclusive desde o momento do
com distúrbios de comportamento, enquanto uma relação nascimento, onde a sua presença parece aumentar o inte-
mãe-filho positiva seria um fator protetor contra esse risco resse e o envolvimento posterior com a criança.
e quanto à ausência paterna. O modelo moderador sugeriu Como vimos, a privação do pai pode ter consequências
que a ausência paterna ou de equivalente aumentou o im- graves, a longo prazo, com problemas na modulação e na
pacto negativo de pares com problema comportamental, intensidade do afeto.

17
CONHECIMENTOS ESPECÍFICOS - PSICÓLOGO

Santoro afirma que a ausência do pai pode compro- As contribuições de Mahler ao desenvolvimento infan-
meter a saúde da criança, e relata que pesquisas recentes til reforçam as ideias desenvolvidas por Bowlby quanto ao
revelam que a presença da figura paterna ajuda a afastar estabelecimento, através dos cuidados parentais, de uma
problemas como a obesidade e uma série de outros trans- base segura aos filhos. Suas contribuições referem-se à im-
tornos psicológicos. A pediatra Melissa Wake, do Royal portância fornecida às relações de objeto precoces. Mah-
Children’s Hospital, em Melbourne, na Austrália, realizou ler destaca que os três primeiros anos de vida da criança
uma pesquisa com quase 5 mil crianças entre quatro e 5 possuem importantes tarefas estruturantes, cujo alcance e
anos. Ela descobriu que a incidência de sobrepeso e obesi- passagem são determinados por dois fatores: primeiro, a
dade nas crianças em idade pré-escolar tem relação direta dotação genética do bebê, que o impulsiona para o víncu-
com a negligência dos pais. Outras pesquisas demonstram lo com o meio ambiente, permitindo perceber e aceitar os
que as crianças que têm pai presente apresentam nível de cuidados proporcionados pelos pais; e, segundo, a mater-
autoestima superior àquelas que têm pai ausente, com o nagem, ou seja, a presença de uma mãe que verdadeira-
qual não convivem. O pai é um pilar muito importante no mente proporcione esses cuidados.
desenvolvimento de qualquer criança. Quanto maior é a O desenvolvimento do núcleo de confiança básico, por
participação e o envolvimento do pai no crescimento e na meio do qual a criança é encorajada a explorar o mundo
educação da criança, melhor é a qualidade da relação que externo, adquire confiança em si mesma e nos demais in-
se estabelece entre ambos. divíduos, e é de suma importância para a estruturação psí-
Bowlby também reforça a importância dos pais forne- quica da criança. Rohde et al. relatam que a função paterna
cerem uma base segura a partir da qual uma criança ou é fundamental para o desenvolvimento do bebê. Segundo
um adolescente pode explorar o mundo exterior e a ele os autores, tal função é dinâmica, já que o pai represen-
retornar certos, de que serão bem-vindos, nutridos física e ta um sustentáculo afetivo para a mãe interagir com seu
emocionalmente, confortados se houver um sofrimento e bebê e também, ainda nos primeiros anos da criança, deve
encorajados se estiverem ameaçados. A consequência des- funcionar como um fator de divisão da relação simbiótica
sa relação de apego é a construção, por volta da metade mãe-bebê.
do terceiro ano de idade, de um sentimento de confiança Para Pupo, o ideal é que o pai participe dos cuidados
com a criança desde o momento do nascimento: ele deve
e segurança da criança em relação a si mesma e, principal-
assistir ao banho, conversar com o pediatra e enfermeiras,
mente, em relação àqueles que a rodeiam, sejam estes suas
cantar uma cantiga de ninar, ajudar na troca de fraldas e
figuras parentais ou outros integrantes de seu círculo de
no banho. Mesmo se o casal estiver separado, o pai deve
relações sociais.
participar ao máximo possível da rotina de seu filho, per-
Mondardo e Valentina afirmam que um importan-
guntando para aquela pessoa que fica mais tempo com o
te traço do comportamento de apego é a intensidade da
bebê sobre seus gostos e suas preferências. Fazer parte da
emoção que o acompanha, o tipo de emoção que surge
vida de um filho é fazer parte de seu mundo, é conhecê-lo.
de acordo com a relação entre a pessoa apegada e a figura
Desde o útero, a criança já escuta e discrimina a voz
de apego. dos pais devido à diferença de tonalidade. Portanto, o vín-
Lebovici, desenvolvendo estas ideias, reforça que, se culo do bebê com a figura paterna se inicia ainda no útero.
tudo está bem, há satisfação e um senso de segurança, Esta nova configuração social de mudança de papéis na fa-
mas, se esta relação está ameaçada, existem ciúme, ansie- mília, com o pai se tornando mais participante da vida dos
dade e raiva. Se, ocorre uma ruptura, há dor e depressão. filhos, possibilita que, além de provedores, estejam tam-
“Os efeitos perniciosos da privação variam de acordo com bém desejando permanecer guardiões das crianças, quan-
o grau da mesma. A privação traz consigo a angústia, uma do o casal opta pela separação. Esta é uma nova situação
exagerada necessidade de amor, fortes sentimentos de vin- social histórica, com a qual casais têm se deparado com
gança e, em consequência, culpa e depressão”. frequência. Em decorrência da separação, muitos pais estão
Se uma pessoa teve a sorte de crescer em um bom lar solicitando a guarda compartilhada, ou seja, eles querem
comum, ao lado de pais afetivos dos quais pôde contar continuar participando da vida de seus filhos, e, exercendo
com apoio incondicional, conforto e proteção, consegue o papel de pai, pois um novo perfil de pai foi se configuran-
desenvolver estruturas psíquicas suficientemente fortes e do: É um homem oriundo das classes médias ou altas, que
seguras para enfrentar as dificuldades da vida cotidiana. se beneficia de uma formação e de uma renda mais elevada
Nestas condições, crianças seguramente apegadas aos seis que a média. Tem uma profissão liberal que lhe permite,
anos são aquelas que tratam seus pais de uma forma rela- bem como à sua mulher, dispor livremente de seu tempo
xada e amigável, estabelecendo com eles uma intimidade e rejeita a cultura masculina tradicional. A maioria se diz
de forma fácil e sutil, além de manter com eles um fluxo em ruptura com o modelo de sua infância e não quer, por
livre de comunicação. nada, reproduzir o comportamento do pai, considerado
O mesmo autor aponta para as consequências da situ- “frio e distante”. Eles almejam “reparar” sua própria infân-
ação inversa, ou seja, se esta mesma pessoa vem a crescer cia. Finalmente, vivem com mulheres que não têm vontade
em circunstâncias diferentes, seu núcleo de confiança será de ser mães em tempo integral. Aquela figura que comu-
esvaziado, ficando prejudicadas as relações com outros se- mente se tinha somente nos finais de semana, dá lugar a
melhantes, havendo prejuízos nas demais funções de seu um pai mais partícipe, envolvido com o dia-a-dia, com a
desenvolvimento. educação e com o crescimento de seus filhos, priorizando

18
CONHECIMENTOS ESPECÍFICOS - PSICÓLOGO

e garantindo às crianças um ambiente seguro, mediante As experiências de Winnicott durante a Segunda Guer-
um desenvolvimento preservado, em prol da estabilidade ra Mundial, quando trabalhou na coordenação de lares
emocional dos seus filhos. Mas, para Silveira, juíza de direi- para crianças evacuadas consideradas difíceis, foram cen-
to, a Guarda Compartilhada só torna-se viável quando am- trais para um novo olhar sobre o fenômeno da delinquên-
bos os pais têm um firme propósito, especialmente aquele cia e da criminalidade. Nesse ponto voltamos nossa aten-
que não reside com a criança, que é o de cumprir as tarefas, ção para o tema da agressividade, que está contido nesse
que antes da separação eram cumpridas em parceria. Para tema maior da delinquência, devido a sua associação com
esta autora, não basta compartilhar a Guarda, para ela os a destrutividade.
pais devem exercer a sua paternidade com desprendimen- A defesa do equilíbrio entre a participação da realidade
to, amor, determinação e convicção. externa e da realidade interna no funcionamento psíqui-
co, corroborada pelas experiências que acumulou durante
Fonte: suas atividades em Oxfordshire, como psiquiatra consultor
BENCZIK, E. B. P. do plano de evacuação de crianças londrinas para o inte-
rior, onde supervisionou o tratamento de fugitivos e de-
linquentes, levou-o a desenvolver suas reflexões sobre “a
O PAPEL DA AGRESSIVIDADE NO tendência antissocial”.
DESENVOLVIMENTO HUMANO.
Winnicott (1946) afirma que quando uma criança é au-
xiliada nos estágios iniciais por seu próprio lar, ela desen-
volve o que denominou de “ambiente interno”, a capacida-
A AGRESSIVIDADE NA CONCEPÇÃO de para controlar-se. Entretanto, quando isso não acontece,
DE WINNICOTT ou seja, quando a criança não teve a oportunidade de criar
um bom “ambiente interno”, ela “necessita absolutamente
Gostaríamos de, inicialmente, resgatar um pouco da de um controle externo se quiser ser feliz e capaz de brin-
história pessoal de Winnicott para somente então expor car ou trabalhar” (p. 123). Entendida dessa maneira, perce-
sua concepção de agressividade, por acreditar que suas
be-se que a tendência antissocial leva a criança a olhar um
ideias estão estritamente relacionadas com sua vivência
pouco mais longe, “recorrendo à sociedade em vez de re-
profissional.
correr à família ou à escola para lhe fornecer a estabilidade
Winnicott estudou biologia e medicina. Tornou-se pe-
de que necessita a fim de transpor os primeiros e essenciais
diatra e, posteriormente, psiquiatra infantil, antes de se
estágios de seu desenvolvimento emocional” (p. 122).
tornar um psicanalista. Essa trajetória, construída com o
Desse modo, o meio ambiente passa a ter um papel
atendimento a um número bastante elevado de crianças
preponderante, visto que a criança, através dos instintos,
“doentes” e pais aflitos, aliada a outras posturas pessoais,
fez com que ele trilhasse um caminho alternativo, no que atribuirá a outra pessoa a tarefa de cuidar dela. Para que
diz respeito à teoria psicanalítica, aos presentes à época isso aconteça, é necessário que a “deprivação” tenha acon-
(décadas de 1930 e 1940) em que começou a escrever suas tecido em um momento do desenvolvimento do ego do
reflexões sobre o desenvolvimento emocional. Recusan- bebê ou da criança pequena que lhe permita perceber a
do a se aliar tanto ao grupo conduzido por Melanie Klein, causa desse “desastre” como uma falha ou omissão do
quanto ao conduzido por Anna Freud, juntou-se ao chama- ambiente e não como algo interno. Sendo assim, pode-
do Middle Group, do qual faziam parte Michael Balint, Ro- mos dizer que só se desenvolve uma tendência antissocial
nald Fairbairn, Sylvia Payne, Ella Sharpe e Marjorie Brierley. quando a criança já experienciou algo positivo e houve a
Robert Rodman, organizador das cartas de Winnicott que perda desse algo bom por um período maior do que ela
compõem o livro intitulado O Gesto Espontâneo (1990), conseguiu manter viva a memória dessa experiência. “Tudo
comenta o seguinte a respeito dessa escolha: “Tal deci- indica que o momento da privação original ocorre durante
são convinha à pessoa que posteriormente acrescentaria o período em que o ego do bebê ou criança pequena está
o termo “objeto transicional” ao nosso vocabulário, e que em processo de realização da fusão das raízes libidinais e
concentraria seus textos na área interpessoal, o locus do re- agressivas (ou motilidade) do id” (Winnicott, 1956).
lacionamento e da experiência cultural, onde o movimento Winnicott (1946) vê na tendência antissocial um sinal
e a atividade são traços característicos.” (p. XIV). de esperança, pois a criança inicia uma busca pela estabili-
Suas contribuições à psicanálise são marcadas basica- dade que lhe faltou no lar como estratégia para não enlou-
mente pelo destaque que oferece às experiências relacio- quecer. E se conseguir encontrar o sentimento de seguran-
nais concretas. Enquanto Freud coloca na posição primária ça em tempo oportuno, conseguirá, gradualmente, avançar
uma vida pulsional originalmente sem objetos específicos, da dependência, e da necessidade de ser cuidada, para a
Winnicott coloca nessa mesma posição a estrutura rela- independência.
cional do bebê com sua mãe. Ao fazer isso, transformou Clare Winnicott, Ray Shepherd e Madeleine Davis, or-
a teoria da posição depressiva de Melanie Klein no que ganizadores do livro Privação e Delinquência (1994), assi-
ele chama “a fase de preocupação”, caracterizada princi- nalam que Winnicott, ao atribuir à privação a origem da
palmente pela preocupação do bebê com o bem-estar da tendência antissocial, reformulou as ideias presentes na
mãe, entendida aqui mais como uma pessoa real, do que teoria psicanalítica sobre as manifestações da delinquên-
apenas como um produto da fantasia da criança. cia e criminalidade, consideradas como resultantes da an-

19
CONHECIMENTOS ESPECÍFICOS - PSICÓLOGO

siedade ou culpa da ambivalência inconsciente que surge Esse pressuposto tem uma enorme importância na re-
quando o desejo de destruir tem como alvo uma pessoa flexão sobre a criança. Gostaríamos de nos deter um pouco
amada e necessária. mais nele para explicar as várias ideias de Winnicott sobre a
Todos esses posicionamentos sobre a tendência an- importância de oferecer às crianças ambientes emocionais
tissocial aparecem em suas considerações sobre a agres- estáveis e confiáveis.
sividade. Porém, acreditamos ser importante explicitar Para Winnicott (1946), uma criança normal não é aque-
por que após tantos anos consideramos suas ideias como la que se comporta de modo sempre desejável, mas sim
uma ferramenta importante e útil para o entendimento da aquela que usa de todos os meios possíveis para se impor,
agressividade em nossas crianças. Nessa tarefa, recorre- colocando à prova seu poder de desintegrar, destruir, as-
mos à posição defendida por Clare Winnicott, sua segunda sustar, cansar, manobrar, consumir e apropriar-se.
esposa e colega de trabalho, como assistente social psi- Quando encontra no lar um ambiente suficientemente
quiatra e coordenadora dos cinco “lares” supervisionados forte e resistente às suas tentativas de desorganizá-lo, ela
por ele durante a guerra, exposta na introdução do livro se tranquiliza e consegue se sentir livre e capaz de brincar,
Privação e Delinquência (1994). Ao apresentar os argumen- ser uma criança irresponsável. Mas antes que isso aconteça,
tos para a utilização das ideias de Winnicott sobre o tema, ela precisa se conscientizar do quadro de referência, e fará
diz que para conseguirmos enfrentar as ameaças das ma- tentativas para testá-lo, especialmente quando tem alguma
nifestações de privação e delinquência temos que utilizar dúvida quanto à estabilidade dos pais e do lar. Para ele,
o conhecimento obtido por quem enfrentou esse tipo de todos os crimes e delitos que levam as pessoas a serem
problema, assumindo a responsabilidade por casos indivi- julgadas ou retiradas da sociedade têm “seu equivalente
duais. Afirma ainda que: normal na infância, na relação da criança com seu próprio
lar” (p. 121).
Embora as circunstâncias em que Winnicott se encon-
trava fossem anormais por ser tempo de guerra, os conhe- Essa importância atribuída ao lar se associa ao posicio-
cimentos obtidos a partir dessa experiência têm aplicação namento de que a criança deve conhecer o mundo a partir
de seu ambiente familiar. Deve primeiro poder vivenciar,
geral, porque as crianças que sofrem privação e se tornam
contando com isso com um sentimento de segurança, to-
delinquentes têm problemas básicos que se manifestam
dos os sentimentos entre as quatro paredes de seu quarto.
de modos previsíveis, sejam quais forem as circunstâncias.
Somente desse modo, desafiando e até mesmo detestando
Além disso, as crianças que passavam à responsabilidade
os pais, é que a criança pode descobrir a parte mais profun-
de Winnicott eram aquelas que necessitavam de providên-
da de sua natureza. Assim, os pais devem oferecer estabili-
cias especiais porque não podiam ser instaladas em lares
dade suficiente para que a criança possa agir sem medo de
comuns. Em outras palavras, já estavam em dificuldade em
que por suas atitudes deixe de ser compreendida e amada.
seus próprios lares, antes da guerra. A guerra foi quase se-
Para tanto, “os pais terão que ser capazes de mostrar força
cundária para elas, quando não positivamente benéfica (e e firmeza em suas atitudes para com os filhos, e também
isso não raro), na medida em que as removeu de uma situ- compreensão e amor” (Winnicott, 1945, p. 57).
ação intolerável, colocando-as numa situação em que po- Esse empreendimento de conhecer e tolerar a própria
deriam encontrar – e frequentemente encontraram – ajuda agressividade é apontado por Winnicott (1939) como fun-
e alívio (Winnicott, 1994, p. 1). damental para a saúde da criança e das pessoas em geral.
Esclarecidos os motivos pelos quais estamos adotando Ele nos lembra que, infelizmente, a agressividade é uma
as ideias de Winnicott, voltemos ao tema da agressividade. das tendências humanas que mais é dissimulada, desviada
Em carta à Anna Freud no ano de 1948, Winnicott e atribuída a agentes externos, e que isso pode prejudicar
(1990), apesar de fazer restrições à utilidade de um resu- o desenvolvimento das atividades criativas e de trabalho,
mo, expõe alguns dos pilares de seu pensamento sobre a estimuladas pela agressão que não é negada e pela qual se
agressividade. O primeiro diz respeito à sua crença de que assume a responsabilidade pessoal: “por trás de todo jogo,
os problemas do mundo são resultantes da agressividade trabalho e arte está o remorso inconsciente pelo dano cau-
reprimida em cada um e não à agressividade do homem sado na fantasia inconsciente, e um desejo inconsciente de
em geral. Esse seu posicionamento aparece em vários ou- começar a corrigir as coisas” (p. 96).
tros trabalhos, e se explicita com a afirmação “de que todo Com essa visão, repudia as atitudes sentimentalistas
o bem e o mal encontrados no mundo das relações huma- caracterizadas pela negação inconsciente da capacidade
nas serão encontrados no âmago do ser humano” (Winni- de destruição que subjaz a qualquer atividade construti-
cott, 1939, p. 89). O segundo, apresentado como consequ- va. Esse sentimentalismo é algo prejudicial para a criança
ência, defende não a educação das crianças com o objetivo em desenvolvimento, que pode ser impedida de comuni-
de manejar e controlar a agressividade, mas sim o ofereci- car indiretamente sua destrutividade, e por essa razão ter
mento de ambientes emocionais estáveis e confiáveis para que mostrá-la de forma mais direta. A idéia contida então
o maior número possível de bebês e crianças, que possam no segundo pilar de sua concepção sobre agressividade é
proporcionar, a cada um, conhecer e a tolerar como parte assim explicitada:
de si mesmo o conjunto total de sua agressividade (o ávi- É parcialmente falso afirmar que “devemos dar oportu-
do amor primitivo, a destrutividade, a capacidade de odiar, nidade para a expressão criativa, se quisermos neutralizar
etc.). os impulsos destrutivos da criança”. O que se faz necessário

20
CONHECIMENTOS ESPECÍFICOS - PSICÓLOGO

é uma atitude não-sentimentalista em relação a todas as tros comportamentos físicos e verbais, a serviço dos senti-
produções, o que significa a apreciação não tanto do ta- mentos de raiva, ódio e vingança. Essa passagem assinala
lento como da luta que há por trás de qualquer realização, a aceitação da contradição, pois a criança passa a amar e
por menor que seja. Pois com exceção do amor sensual, a odiar simultaneamente. O impulso de morder oferece
nenhuma manifestação de amor é sentida como valiosa um importante exemplo dessa associação entre o amor e
se não implicar agressão reconhecida e controlada... Só se a agressão, que passa a ter um sentido aproximadamente
soubermos que a criança quer derrubar a torre de cubos, a partir dos cinco meses de idade. Esse impulso que no fim
será importante para ela vermos que sabe construí-la (Win- integra-se no prazer obtido durante o ato de comer, en-
nicott, 1939, p. 96). tretanto, originalmente, é o objeto bom, o corpo materno,
O outro pressuposto apresentado a Anna Freud em que excita o morder e produz ideias de morder. Assim, o
1948 (cit. em Winnicott, 1990) complementa o segundo ao alimento acaba por ser aceito como um símbolo do corpo
colocar ao lado da necessidade de todo ser humano tolerar da mãe, do corpo do pai ou de qualquer outra pessoa ama-
e aceitar a própria agressividade a de “respeitar a culpa e a da (Winnicott, 1964, p. 101).
depressão e reconhecer plenamente as tendências de repa- A capacidade de deslocar o desejo de destruir um ob-
ração quando elas existem” (p. 10). Essa articulação entre a jeto bom e necessário para outro objeto é crucial para a
reparação, a agressividade e a depressão foi melhor expli- saúde da criança, e isso coloca em destaque a capacidade
citada por Winnicott (1960) da seguinte maneira: quando de simbolização, pois é através dela que se pode obter o
alguém acumula forças que possibilitam a tolerância à sua alívio relacionado aos conflitos “crus e incômodos”, decor-
destrutividade, essas são objetivadas na reparação. Entre- rentes da verdade pura. A aceitação dos símbolos e o brin-
tanto, se ocorre, por algum motivo, o bloqueio da repa- car, baseado nessa aceitação, apresentam possibilidades
ração, a pessoa torna-se parcialmente incapaz de assumir infinitas para as experiências de vida da criança, pois a ins-
a responsabilidade por seus impulsos destrutivos, o que, trumentalizam para experimentar “tudo o que se encontra
clinicamente, configura a depressão, ou então busca ex- em sua íntima realidade psíquica pessoal, que é a base do
ternamente um responsável por sua destrutividade, através sentimento de identidade em desenvolvimento.
do mecanismo de projeção. Para Winnicott, uma pessoa
será tanto mais saudável quanto menos utilizar esse me- Tanto haverá agressividade como amor” (Winnicott,
canismo, ou seja, quanto mais se reconhecer de forma in- 1964, p. 100).
tegrada, assumindo a responsabilidade por seus impulsos A construção é a outra alternativa importante à des-
truição no processo de amadurecimento da criança. O sur-
agressivos. Sendo assim, a tendência para a agressividade
gimento e manutenção do brincar construtivo é um dos
é inerente à natureza humana, embora ela não se manifeste
mais relevantes sinais de saúde. Ele aparece com o tempo
da mesma forma em todas as pessoas (Winnicott, 1964).
e assim como a confiança não pode ser imposto, mas deve
Para compreender melhor o lugar ocupado pela agressi-
resultar da totalidade das experiências de vida proporcio-
vidade no pensamento desse autor, resgataremos alguns
nadas por aqueles que cuidam da criança (Winnicott, 1964).
outros aspectos que nos parecem relevantes para nosso
Ao uso direto da agressividade, caberá então o com-
objeto de estudo.
bate da realidade externa percebida como má. Nesse mo-
mento, voltamos para o tema da delinquência e da tendên-
Para Winnicott (1964), a agressão tem dois significa- cia antissocial, entendendo que também a agressividade
dos: constitui uma reação direta ou indireta à frustração presente nesse tipo de comportamento tem para a criança,
e, por outro lado, é uma das muitas fontes de energia de ou para o adulto, um significado em seu mundo psíquico,
um indivíduo. Para o entendimento dessas duas manifes- ou seja, o de provocar no ambiente um controle que inter-
tações, devemos resgatar o surgimento da agressividade namente ele não conseguiu estabelecer. Nas palavras de
na vida do bebê. No início, ela se expressa no movimento, Winnicott:
no prazer oriundo da capacidade de se movimentar e se Sem tentar aprofundar-nos muito na origem das for-
encontrar com algo. Nesse momento, por não haver, por ças que lutam pelo predomínio dentro da personalidade,
parte da criança, uma razão clara para a ação, essa não posso assinalar que quando as forças cruéis ou destrutivas
pode ser interpretada como uma pancada ou pontapé. ameaçam dominar as forças de amor, o indivíduo tem de
Acompanhando esses movimentos, compreendemos que fazer alguma coisa para salvar-se, e uma das coisas que ele
essas “pancadas infantis” propiciam à criança o início da faz é pôr para fora do seu íntimo, dramatizar exteriormente
descoberta do mundo externo e de sua relação com obje- o mundo interior, representar ele próprio o papel destru-
tos externos. Desse modo: tivo e provocar seu controle por uma autoridade externa.
O que logo será comportamento agressivo não passa, O controle pode ser estabelecido desse modo, na fantasia
portanto, no início, de um simples impulso que leva a um dramatizada, sem sufocação séria dos instintos, ao passo
movimento e aos primeiros passos de uma exploração. A que o controle interno necessitaria ser geralmente aplicado
agressão está sempre ligada, dessa maneira, ao estabele- e resultaria num estado de coisas conhecido clinicamente
cimento de uma distinção entre o que é e o que não é o como depressão (Winnicott, 1939, p. 93-94).
eu (p. 98). O problema enfrentado frequentemente, segundo
O desenvolvimento sadio é marcado então por mo- Winnicott (1939), quase sempre essa dramatização da re-
vimentos naturais e uma tendência para bater contra as alidade interior, que não pode ser tolerada por ser muito
coisas, que são gradualmente usados pelo bebê, com ou- ruim.

21
CONHECIMENTOS ESPECÍFICOS - PSICÓLOGO

Finalizando nossa reflexão sobre agressividade, gosta-


ríamos de expor brevemente as alternativas que Winnicott NATUREZA E ORIGENS DA TENDÊNCIA
apresenta para o que considera um dos principais proble- ANTISSOCIAL.
mas de adultos e crianças: encontrar formas seguras de eli-
minar a maldade.
Uma primeira alternativa está ligada à dramatização
e resolução (falsa) obtida nos rituais de cuidados decor- A TENDÊNCIA ANTISSOCIAL E O ESTÁGIO DE
rentes da eliminação de elementos físicos provenientes do AMADURECIMENTO
corpo. Uma outra possibilidade é a utilização de jogos ou
trabalhos que envolvam uma ação distinta, que possa ser As características centrais do amadurecimento são o
desfrutada com prazer e que possibilite a eliminação do ambiente facilitador e a tendência inata do bebê à integra-
sentimento de frustração e ofensa: “um menino que luta ção. O ambiente facilitador é constituído da mãe ambiente
boxe ou chuta bola sente-se melhor com o que está fa- ou de qualquer outra pessoa que possa fazer às vezes da
zendo, em parte porque gosta de agredir e dar pontapés mãe.
e em parte porque sente inconscientemente (falsamente) A mãe ambiente, com os seus cuidados na medida cer-
que está expulsando a maldade através dos punhos e dos ta, ajuda o bebê a integrar as experiências do si mesmo,
pés” (Winnicott, 1939, p. 94-95). tornando-se um Eu Sou. (WINNICOTT, cf.,1958c)
Outro modo de eliminar a agressão madura é o encon- Na base da tendência antissocial, há uma falha am-
trado na competição em jogos e trabalhos, presente entre biental, ou melhor, uma falha da “mãe ambiente”. Winni-
os adolescentes. Outra forma, e essa nos interessa mais cott caracteriza a tendência antissocial como um distúrbio,
de perto, é a orientada pelo medo. Esse tipo de agressão resultado de uma falha ambiental que acometeu a relação
tem como objetivo encontrar o controle e forçá-lo a fun- mãe-bebê no estágio da dependência relativa.
cionar. Quando isso ocorre, o adulto deve ter sensibilidade Na linha do amadurecimento, a tendência antissocial se
suficiente para impedir que essa agressão fuja ao controle; localiza no estágio de dependência relativa, quando já há
para tanto, deve proporcionar uma autoridade confiante, uma integração, um Eu Sou constituído, aproximadamente
a qual permita que certo grau de maldade possa ser dra- entre um e dois anos de idade. (WINNICOTT, 1979)
matizada e usufruída sem perigo. A retirada gradual des- A falha da mãe, para se constituir em um trauma, preci-
sa autoridade deve ocorrer ao longo do desenvolvimento sa acontecer de forma sutil ao longo do tempo e se estabe-
da criança. O reconhecimento desse importante papel do lecer como um padrão de comunicação na relação mãe-be-
adulto diante da agressividade das crianças fez com que bê. A falha se estabelece como trauma, quando interrompe
Winnicott (1939) enfatizasse que: a continuidade de ser do bebê. (WINNICOTT, cf., 1958c)
A mãe deve falhar em satisfazer as necessidades instin-
É tarefa de pais e professores cuidar para que as crian- tivas, mas pode ser perfeitamente bem-sucedida em jamais
ças nunca se vejam diante de uma autoridade tão fraca a deixar que o bebê se sinta desamparado, provendo as suas
ponto de ficarem livres de qualquer controle ou, por medo, necessidades egoicas, até o momento em que ele já pos-
assumirem elas próprias a autoridade. A assunção de auto- sua introjetada uma mãe que apoia o ego, e tenha idade
ridade provocada por ansiedade significa ditadura, e aque- suficiente para manter essa introjeção, apesar das falhas do
les que tiveram a experiência de deixar as crianças contro- ambiente a este respeito. (WINNICOTT, 1958c)
larem seus próprios destinos sabem que o adulto tranquilo Todo bebê sofre de privação instintual. A mãe boa falha
é menos cruel, enquanto autoridade, do que uma criança necessariamente na sua adaptação em satisfazer as exigên-
poderá se tornar se for sobrecarregada com responsabili- cias instintuais do bebê. No entanto, a mãe que está em sin-
dades (p. 95). tonia com o seu bebê, não o desampara; e, ao realizar que
errou, oferece alguma reparação, e tudo retorna ao estado
Fonte: de tranquilidade anterior.
LUZ, I. R.da.. A agressividade na concepção de Winni- Quando ocorre a tendência antissocial, aconteceu uma
cott e suas implicações para a Educação Infantil privação propriamente dita (não uma simples privação), ou
WINNICOTT, D. W. Privação e Delinquência São Paulo, seja, deu-se a perda de algo bom, de caráter positivo na
WMF, Martins Fontes, 2012. experiência da criança até um certo momento, no qual este
elemento positivo foi retirado. (WINNICOTT, 1958c)
A mãe falha com o seu bebê quando retira o elemento
positivo da relação por um período longo demais para que
fosse capaz de manter viva a memória da experiência.
A tendência antissocial tem, na sua origem, uma priva-
ção. Diferentemente da privação que ocorre num momento
anterior e mais primitivo do amadurecimento emocional, a
criança que sofre uma de privação já tem maturidade de
ego suficiente para perceber que a falha é do ambiente. A
criança entende que algo lhe foi tirado e que a falha é ex-
terna a ela – do ambiente.

22
CONHECIMENTOS ESPECÍFICOS - PSICÓLOGO

Esse é o ponto de origem da tendência antissocial, e O desafio do tratamento consiste em estabelecer con-
aí se inicia o que toma conta da criança, sempre que ela se tato com a parte saudável da criança, que ainda tem espe-
sente esperançosa, e compele a uma atividade que é antis- rança na reparação da falha ambiental. No entanto, na pre-
social, até que alguém reconheça e tente corrigir a falha do sença de um ambiente suficientemente bom, a esperança
ambiente. WINNICOTT, 1979) sempre retorna.
A tendência antissocial é, portanto, uma reação da
criança à falha ambiental. O comportamento antissocial O sintoma e suas manifestações
ocorre quando a criança se enche de esperança, na expec-
tativa por uma reparação proveniente do ambiente pelo A manifestação do sintoma na tendência antissocial in-
dano que lhe foi causado. A privação inclui tanto o trauma clui o roubo, a mentira, a enurese, a encorpese, a destruição
de propriedade, e a desordem. A remoção do sintoma é
(quebra na continuidade de ser), como a situação traumá-
importante para o ambiente, pois ocasiona uma diminui-
tica que se prolonga no tempo.
ção do estresse, resultando em um alívio que possibilita o
holding.
A tendência antissocial e a delinquência O caráter perturbador e desagregador do sintoma é
explorado pela criança de forma persistente, provocando
Segundo Winnicott, a tendência antissocial não é uma reações do ambiente, o que é uma característica essencial
categoria, mas sim um distúrbio psíquico, relacionado a e valiosa, uma vez que aponta em direção à saúde e à cura.
dificuldades inerentes ao desenvolvimento emocional, po- O trauma na tendência antissocial corresponde ao
dendo vir acompanhado às psicoses, às depressões ou às momento da ocorrência da falha ambiental, superando a
neuroses. capacidade egoica do bebê de absorvê-la, gerando a de-
A tendência antissocial não é um diagnóstico. Não se sintegração.
pode compará-la, diretamente, com outros tipos de diag- Na desintegração, ocorre a cisão entre os impulsos libi-
nósticos, tais como neurose ou psicose. Pode ser encontra- dinais e motores (agressividade), provocando distorções na
da tanto em indivíduos normais quanto em neuróticos ou personalidade. A fusão das raízes agressivas com os instin-
psicóticos. (1958c p. 408) tos libidinais representaria a cura.
É denominada de tendência porque pode não ocorrer A criança, que pratica o ato antissocial, procura resgatar
se for compreendida logo no início, isto é, se a mãe enten- uma provisão ambiental perdida, busca os cuidados da mãe
der como uma falha sua e corrigir. Neste caso, a tendência que sente lhe é devido porque ela é sua criação.
No início, a mãe está em total sintonia com o bebê,
não se concretiza.
adaptando-se ativamente às necessidades dele. A criança,
Quando o ambiente não reconhece a falha e não ofere-
neste estágio, não sabe da existência do mundo externo,
ce uma reparação, o comportamento antissocial se crista- tudo é parte dela; ela tem a sensação de ser capaz de criar
liza, e a tendência se transforma em delinquência. A psico- o mundo.
patia seria um nível mais avançado da doença. A mãe responde à criatividade primária do bebê, tor-
O ato antissocial é um pedido de ajuda da criança, nando-se o objeto que estava ali para ser encontrado e a
quando ela volta a ter esperança na capacidade de repa- criança pronta para encontrar. O bebê cria o seio. É somente
ração do ambiente. a partir dessa ilusão inicial de onipotência, que o bebê pode
dar sentido à mãe objetivamente percebida.
O ato antissocial, portanto, vem carregado de esperan- A criança, com comportamento antissocial, busca recu-
ça nas provisões do ambiente. A criança faz um movimento perar a experiência de onipotência de um bebê bem aten-
em direção à cura. dido e que se viu, prematuramente, privado ao ser afastado
Ao cometer o ato antissocial a criança tem esperança da mãe. A ilusão de onipotência, mantida pela mãe am-
no retorno dos cuidados do ambiente. Ela sente que o am- biente, preservou-o de viver a sua extrema vulnerabilidade
biente tem uma dívida para com ela e busca uma reparação e dependência.
dele. Este sentimento de onipotência, que não pode ser vivi-
O momento de esperança é para a criança, sem este do e integrado por uma realidade psíquica favorecida pelo
sentimento, a luz no final do túnel, é a possibilidade de bom ambiente, irá se expressar em atitudes e atos de vio-
lência.
retomar o processo de se sentir inteira; é uma tentativa de
O contato do bebê com a realidade externa acontece
voltar a um tempo anterior à situação traumática.
aos poucos, auxiliado pelas suas funções intelectuais e pe-
Na delinquência, o ambiente fracassa em não entender las falhas no ambiente (quando a mãe erra ao atender às
o pedido de socorro da criança. Uma vez não compreen- necessidades do bebê). O bebê começa, então, a se relacio-
dida, ela se afasta do trauma inicial, e passa a desenvolver nar com um outro ser humano – período da desilusão da
habilidade e ganhar experiência na ação resultante do ato onipotência.
antissocial. A busca do objeto e a destruição nos remetem ao pe-
Na delinquência, a criança já fez uma “carreira” e de- ríodo da dependência relativa e às tarefas da mãe, que são
senvolveu uma “expertise”, ou seja, os chamados ganhos sobreviver à destrutividade do bebê e sustentar esta situa-
secundários, os quais afastam a criança do trauma, dificul- ção no tempo, permitindo-o integrar a instintualidade e a
tando o tratamento. agressividade (motilidade).

23
CONHECIMENTOS ESPECÍFICOS - PSICÓLOGO

No ciclo benigno, a mãe objeto (o seio) é algo que o Se essa falha é meticulosa, isto é, persiste ao longo
bebê cria, usa e destrói. Quando a ela sobrevive à vivaci- do tempo, ela passa a se constituir em um padrão de co-
dade e à voracidade do filho, surge a capacidade de repa- municação entre a mãe e seu bebê. A falha como trauma
rar do bebê, possibilitando que seus impulsos instintuais e pode ser percebida no poema de Mário Quintana – “Veio
agressivos se integrem em uma pessoa inteira – o Eu Sou. o Todo que apagou o vestígio de tudo” (tudo representado
A percepção da criança da incapacidade da mãe de pelo ambiente bom de outrora). Sendo assim, a criança que
sustentar e sobreviver à agressividade, contida na instin- apresenta o comportamento antissocial está compulsiva-
tualidade, consiste na base de sua dificuldade de reunir, mente exigindo, ao mesmo tempo, um reconhecimento e
dentro de si, os sentimentos de amor e ódio ali presentes. uma reparação do ambiente, o qual precisa retroceder à
Segundo Winnicott, o grau maior ou menor de pertur- situação de carência e reconhecer sua responsabilidade.
bação no ambiente causada pela criança antissocial é um O ato antissocial ocorre quando o ambiente apresenta
aspecto favorável, pois aponta para um potencial de recu- sinais de esperança de que existe a possibilidade de repa-
peração da fusão entre os impulsos libidinais (instintuais) ração. O ato antissocial é para a criança sem esperança uma
e motores.
perspectiva de retorno aos cuidados ambientais, ao amor
da mãe. Os versos abaixo descrevem, a meu ver, muito bem
Aspectos do tratamento e cura
essa brecha de esperança, que é percebida pela criança,
A compreensão de que o ato antissocial é uma expres- que coloca a sua vivacidade em ação no ato antissocial
são de esperança é vital para o tratamento de crianças que O reconhecimento do elemento positivo, na compul-
apresentam essa tendência. (1958c p.409) são pelo suporte da tensão gerada pelo comportamento
A criança antissocial é uma criança que sofreu uma antissocial, possibilita à criança uma certa dose de confia-
privação e, como tal, tem esperança numa reparação do bilidade, em uma atitude humana que permite buscar o
ambiente. O tratamento e a cura consistem no ambiente suprimento ambiental que se perdeu.
reconhecer esta dívida que tem para com a criança. Restaurada a confiabilidade no ambiente, a criança vol-
A criança que apresenta o comportamento antissocial ta a ter liberdade de agir, se mover e se agitar, na medida
busca um ambiente que suporte o embate gerado pelo ato em que passa a integrar os impulsos do id com a energia
antissocial. “Vezes sem-conta assistimos a momentos de motora, agora não mais uma ameaça ao ego, mas dentro
esperança serem desperdiçados ou minimizados por um de um contexto de integração e crescimento.
manejo equivocado ou por intolerância”(1958c p.407).
Portanto, o ambiente precisa ser testado, várias vezes,
na sua capacidade de suportar a agressão, reparar a des- PREVENÇÃO E EFEITOS DA PRIVAÇÃO
truição, e em tolerar o incômodo que tudo isto acarreta. MATERNA.
“O tratamento da tendência antissocial não é a psicanálise,
mas o manejo, ir ao encontro do momento de esperança
e corresponder-lhe” (1958c p409). Ou seja, o ambiente tem
que ir no sentido oposto à punição. A IMPORTÂNCIA DO CUIDADO E COMUNICAÇÃO
Quando este processo é percebido no seu estágio ini- ENTRE A MÃE E O BEBÊ
cial, a mãe pode reverter o complexo de privação, miman-
do o seu bebê. Toda a família pode participar e contribuir ANDRADE, T. O.
para a cura. É o ambiente que fornece o novo contexto
relacional, onde a criança possa experimentar, junto aos Winnicott foi pediatra antes de se tornar psiquiatra in-
adultos, dando sentido aos impulsos do id e recebendo a fantil e psicanalista, exercendo a clínica pediátrica durante
compreensão que tinha esperança de encontrar um dia. a maior parte da sua vida. Foi o psicanalista que mais traba-
O tratamento da tendência antissocial exige a não re-
lhou e estudou sobre o papel do ambiente na constituição
taliação do ambiente; isto é, o estabelecimento de limites
total do indivíduo ou suas doenças psíquicas.
para contenção dos atos antissociais não pode humilhar
Sendo a psicanálise, um método para tratar pessoas
ou denegrir a pessoa humana. Por exemplo, não se deve
exigir confissão de espécie alguma da criança que rouba. com doenças psíquicas através de meios psicológicos, por-
A criança que comete o ato de roubar ignora os motivos tanto, é um termo e um corpo teórico que diz respeito ao
da compulsão, porque não houve uma integração dos seus desenvolvimento emocional do ser humano. A psicanálise
impulsos libidinais e motores, ambos contidos tanto na ins- avança onde a fisiologia se detém, tentando resolver e en-
tintualidade quanto na agressividade. tender os fenômenos da personalidade, do sentimento e
A organização defensiva zela pela permanência da ci- do conflito humano. Então, o que a psicanálise nos diz de
são destes impulsos, pois se ela for desfeita, a criança passa mais importante sobre as pessoas? Aponta Winnicott que:
a reviver a situação traumática. “Ela nos fala a respeito do inconsciente, da vida profunda
e oculta de cada indivíduo humano que tem raízes na vida
A tendência antissocial está relacionada a uma falha real e imaginária da infância mais precoce” (1896 - 1971).
do ambiente. No ato antissocial, a criança sem esperan- Nesse contexto, Winnicott seguiu princípios diferentes
ça deposita o seu momento de esperança no retorno do daqueles que regem a clínica psicanalítica tradicional, di-
ambiente, que foi um dia suficientemente bom e capaz de vergindo em alguns pontos de Freud (o pai da psicanálise)
prover-lhe o que necessitava. e seus seguidores como: Klein, Lacan, Bion e outros, que

24
CONHECIMENTOS ESPECÍFICOS - PSICÓLOGO

baseiam a psicanálise no complexo de Édipo, e outras teo- Nesse ínterim, o desenvolvimento do indivíduo come-
rias e dimensões psicanalíticas tradicionais que seguem os ça logo ao nascer, a história do ser humano não começa
pensamentos Freudianos. Winnicott, porém, prática a psi- aos 4 anos nem aos 2, nem aos 3 meses, mas ao nascer.
canálise não edipiana, ou seja, baseia seu trabalho em uma Cada bebê é desde o começo uma pessoa, necessitando de
psicanálise não centrada no Édipo. cuidados, compreensão e a com necessidade de ser conhe-
A teoria psicanalítica tradicional explica o papel central cida por alguém e, ninguém pode conhecer mais o bebê do
do complexo de Édipo, considerando-o complexo nuclear que sua mãe. Winnicott foi um dos primeiros autores a des-
das neuroses e, de um modo geral, das doenças psíqui- tacar a importância e o papel da mãe no funcionamento
cas, baseando-se essa teoria na sexualidade, é o fenômeno mental da criança, considerando a mãe como interventora
principal da vida sexual. O problema de Édipo é o proble- ativa, construtora do espaço sadio e mental da criança.
ma central das neuroses e doenças psíquicas e a solução Surge dessa teoria Winnicottiana a importância da mãe
exemplar desses problemas. e do ambiente facilitador no desenvolvimento do bebê,
Nos anos 30, o complexo de Édipo era geralmente bem como, o caminho para uma vida normal e saudável
aceito como ponto essencial e ainda era tido como nuclear. em busca da independência. Logo, surgem boas razões so-
Então, surge a psicanálise Winnicottiana com uma crescen- ciais para instigar pesquisas sobre as primeiras fases das
te convicção de que existem muitos problemas iniciais da relações entre o bebê e a mãe. A teoria psicanalítica de
vida do indivíduo que podem ser descritas, identificadas e Winnicott tem o ser humano não como um objeto da natu-
que não são explicáveis por meio da teoria edipiana. Cha- reza, mas sim como uma pessoa que para existir precisa de
mando esses problemas de agonias e angústias. cuidados e atenção de outro ser humano. O que interessa
As agonias e angustias se dão diante do encontro do a Winnicott “é a relação entre a mãe e o seu bebê, antes
indivíduo com o mundo, inesperado e ainda incompreensí- deste nascer e nas primeiras semanas e meses após o nas-
vel pelo bebê, em seu estágio de desenvolvimento e ama- cimento”. (WINNICOTT, 1964 – 1957, p.10).
durecimento, portanto, nomeando esse processo como A teoria do amadurecimento é um conjunto de prin-
teoria do amadurecimento humano. Apontando que a úni- cípios meramente racional fundamentado no desenvolvi-
ca herança do homem é o processo de amadurecimento e mento emocional e físico da criança no ambiente em que
ela vive, sendo que a saúde mental do bebê está sendo
isso acontece no início da vida.
construída desde início pela mãe, que oferece um ambien-
Tirzard (1986) mostra que: “A validade da psicanálise,
te aconchegante e facilitador. A mãe é a mediadora que
não só enquanto recurso de aprendizagem, mas como te-
oferece um ambiente em que os processos evolutivos e in-
rapia, continua sendo objeto de controvérsia, mas o fato
terações naturais do bebê acontecem, ajudando sem per-
de que o extraordinário conhecimento de Winnicott sobre
ceber, a construir as bases da saúde mental do indivíduo.
o desenvolvimento humano inicial tenha origem não só na
observação direta de bebês e de suas mães [...], mas tam-
Os efeitos e importância do cuidado materno
bém na psicanálise de adultos.
A criança começa a se desenvolver assim que chega
A teoria Winnicottiana do amadurecimento humano ao mundo trazendo consigo um potencial para amadure-
altera de certa forma todos os elementos teóricos voltados cimento, para integração, porém, nasce indefesa, e ainda é
para o complexo de Édipo e pela psicanálise tradicional: um ser desintegrado, sujeito a diferentes estímulos do ex-
no lugar da sexualidade infantil, a dependência; no lugar terior. O seu desenvolvimento evolutivo dependerá de um
da mãe libidinal a mãe cuidadosa e preocupada com o es- ambiente facilitador que forneça cuidados suficientemente
tágio inicial do seu bebê; no lugar da mãe-objeto pulsio- bons, sendo que no início esse ambiente é representado
nal a mãe-ambiente; no lugar do sujeito com constituição pela mãe, que tem um papel de extrema importância no
biológico-dinâmico-mental, o bebê que tem como herança desenvolvimento psíquico do bebê.
o processo de amadurecimento. No essencial, a teoria da A tarefa da mãe é construir suporte adequado para
progressão das zonas erógenas é trocada pelo processo de que as condições de amadurecimento alcancem um bom
desenvolvimento incerto e instável de interpretação do in- desenvolvimento, suprindo as necessidades do bebê, sa-
divíduo. tisfazendo e reconhecendo suas particularidades, pois, da
Winnicott revoluciona os conceitos científicos e substi- relação saudável que ocorre entre a mãe o bebê que emer-
tui os paradigmas da psicanálise tradicional pela teoria do ge os fundamentos da constituição da pessoa e do desen-
amadurecimento pessoal, não utilizando mais a teoria da volvimento emocional-afetivo da criança. “Nada estabelece
função sexual, sendo a sexualidade apenas um importante tão clara e satisfatoriamente a concepção infantil da mãe
processo de amadurecimento humano e o estudo da natu- como um ser humano total do que as boas experiências
reza humana. durante a excitação, com gratificação e satisfação”. (WIN-
NICOTT, 1964 – 1957, p. 57).
O desenvolvimento do ser humano é um processo O bebê é um ser humano que está em processo de de-
contínuo. Tal como no desenvolvimento do corpo, assim senvolvimento, é imaturo e extremamente dependente, em
também no da personalidade e no da capacidade de rela- seu processo de amadurecimento está constantemente ar-
ções. Nenhuma fase pode ser suprimida ou impedida sem mazenando experiências. E essas progressões de desenvol-
efeitos perniciosos. (WINNICOTT). vimento têm uma enorme importância prática para todos

25
CONHECIMENTOS ESPECÍFICOS - PSICÓLOGO

que se ocupam dos estágios iniciais. A mãe que está ligada A sustentação do holding protege o bebê contra as
diretamente ao bebê tem a capacidade de se identificar causas fisiológicas, sempre levando em consideração a
com seu filho permitindo-lhe satisfazer uma função nome- sensibilidade da criança – tato, sensibilidade auditiva, tem-
ada por Winnicott como holding. Todas as particularidades peratura, sensibilidade visual, sensibilidade às quedas –
do cuidado materno, que antecedem ou advêm depois do pois, como a criança não conhece a existência de tudo o
nascimento dirigem-se para a formação do ambiente de que não seja ela própria, por isso a importância da rotina
holding. O holding é essencial ao bebê ao longo de seu de cuidados em torno do bebê ao longo do dia e da noite.
desenvolvimento, o ambiente de holding jamais perde sua A mãe funciona sempre como um ego auxiliar, essa relação
importância. e vínculo entre mãe e bebê que sustentará e assentará as
Segundo a teoria de Winnicott holding é: bases para o desenvolvimento saudável das capacidades
Os cuidados com as crianças giram em torno do ter- do indivíduo.
mo “segurar”, principalmente se permitimos que seu sig- O holding é necessário desde a dependência absoluta
nificado se amplie à medida que o bebê cresce e que seu até a independência da criança, até quando as estruturas
mundo vai se tornando complexo. O termo pode incluir, psíquicas entre a mãe e o filho estejam completamente e
com muita propriedade, a função do grupo familiar, e, de perfeitamente distintas. É importante reconhecer o fato da
uma forma mais sofisticada, pode também ser empregado dependência, já que os bebês não conseguem fazer nada
para caracterizar o trabalho com casos, tal como ele se dá sozinhos. Entretanto, o fato dos bebês serem dependen-
nas profissões cuja característica básica é a prestação de tes no início de suas vidas, eles são afetados por tudo que
cuidados. (WINNICOTT). acontece em seu meio, sendo à história do desenvolvimen-
to infantil uma história de dependência absoluta, mas que
Logo, holding é o ato físico de segurar e manipular a avança fortemente em direção a independência.
estrutura física do bebê que resulta em situações satisfató- O bebê, tendo alcançado a integração por períodos
rias. Segurar bem uma criança facilita os processos de ma- mais longos e sentindo-se habitado em seu próprio corpo,
turação, e segura-la mal significa uma ruptura do processo depara-se com a realidade externa. Num desenvolvimento
de maturação, devido às reações do bebê às transgressões emocional dito normal, a mãe, que está identificada com
de adaptação. seu bebê, o protege das invasões do mundo externo, ou
seja, ela não permite que o meio-ambiente invada o self
O holding é uma forma de amar, e a mãe quando ma-
do bebê, até que através de um gesto espontâneo o bebê
nipula, toca, aconchega, fala com seu bebê, ela promove
venha descobrir o meio ambiente. (Valler, 1990).
um arranjo somático entre o organismo físico do bebê e
a psique. O toque é suficientemente bom e inaugura uma
A maneira pela qual a mãe e o ambiente apresentam o
psique que habita soma, pois, afirma Winnicott que “a qua-
mundo ao bebê que estabelece a relação de objeto, logo,
lidade dos cuidados maternos no início da vida é responsá-
esse conhecimento acontece para o bebê em pequenas do-
vel pela saúde mental do indivíduo, pois livra-o da psicose”.
sagens. A capacidade em que a mãe tem de ir de encontro
(WINNICOTT)
às necessidades do bebê que se encontra em processo de
Winnicott refere-se o toque e a psique da soma como mutação e desenvolvimento, permite ao bebê uma trajetó-
“personificação”, o que significa que o bebê passa a sen- ria de vida relativamente saudável e contínua. Permitindo
tir, como uma consequência do toque amoroso, e que seu ao bebê vivenciar situações fragmentadas e harmoniosas,
sentimento de self (unitário de toda criança em desenvolvi- a partir da confiança que deposita na mãe.
mento) centra-se no interior de seu próprio corpo. Empre- O bebê passa, com muita desenvoltura, da dependên-
ga “personificação” a condição no qual o indivíduo passa cia à independência e começa a acumular conhecimentos e
pela separação mente-corpo em que não se sente perten- experiências, formando uma base para confiar nos proces-
cente ao seu próprio corpo: sos externos, que até então não era unitário, essas experi-
“No início ser amado significa ser aceito... A criança ências que levará o bebê a integração total como unidade.
possui uma cópia daquilo que é normal, o que é certamen- Contextualizando, a psicologia do desenvolvimento
te uma questão de forma e funcionamento de seu próprio emocional, precisa de um ambiente facilitador para que
corpo... possa se concretizar, só um ser humano pode conhecer um
A maioria das crianças foram aceitas no último está- bebê de forma a levá-lo ao amadurecimento e a adapta-
gio anterior ao nascimento, mas amor é demonstrado em ções cada vez melhores e, esse ser humano sem dúvida
termos de cuidados físicos, o que é geralmente adequado nenhuma é a mãe.
quando se trata do feto no ventre. É com toda certeza uma A mãe é essencial para a teoria do desenvolvimento
questão fundamental, uma vez que a criança precisa ser se- emocional de Winnicott. A mãe é o primeiro ambiente, tan-
gurada por uma pessoa cuja necessidade de envolvimento to em termos biológicos quanto psicológico. A maturação
emocional esteja em jogo. O início dessa parte do desen- e desenvolvimento nos estágios iniciais de vida e, na ver-
volvimento do bebê a que chamo de personalização, e que dade ao longo da vida, é com certeza uma questão de inte-
pode ser descrita como um habitar da psique no somar, gração com o mundo e as pessoas. A maneira como a mãe
encontra-se na capacidade da mãe de envolver emocio- se comporta e se sente em relação ao seu bebê exercerá
nalmente, o que originalmente se dá em termos físicos e uma grande influência sobre a saúde do bebê, começando
psicológicos”. ao nascer e prosseguindo pelo resta da vida.

26
CONHECIMENTOS ESPECÍFICOS - PSICÓLOGO

Fontes: A separação mobiliza uma avalanche de sentimentos.


WINNICOTT, Donald W., 1896-1971. Os bebês e suas Em meio a tantas coisas desfeitas, surge a necessidade de
mães/ D. W. Winnicott; tradução Jefferson Luiz Camargo; refazer-se. Como Maria Tereza Maldonado aponta, para
revista técnica Maria Helena Souza Patto. - 3º Ed. – São além do sofrimento e do pesar, a separação representa a
Paulo: Martins Fontes, 2006. – (Psicologia e pedagogia). busca de um novo equilíbrio.
WINNICOTT, Donald W., 1964, 1957. A criança e seu
mundo. Copyright 1982 by – LTC – Livros Técnicos e Cien- Separações, transformações
tíficos Editora S.A. O rompimento de um vínculo afetivo causa transfor-
WINNICOTT, Donald W., 1986 – 1971. Tudo começa em mações profundas na vida de um casal. A sensação de
casa / D. W. Winnicott; tradução Paulo Sandler. – 4º Ed. – desequilíbrio, a perda do lugar seguro e os pesares com
São Paulo: Martins Fontes, 2011. – (Psicologia e pedago- o anúncio do fim. Como mostra a autora, “a convivência
gia). envolve a formação de hábitos, rotinas, modos de vida e
WINNICOTT, Donald W., Seminários Paulistas / São valores comuns. Logo, a separação acarreta um rompimen-
Paulo: Casa do Psicólogo, 2001. to de tudo isso, provocando mudanças em hábitos, estilos
WINNICOTT, Donald W., Espaço potencial Winnicott: de vida, valores pessoais”.
diversidade e interlocução/ [Organização de Afrânio de
Matos Ferreira]. – São Paulo: Landy Editora, 2007. As pessoas ficam casadas por tantos motivos...
WINNICOTT, Donald W., A linguagem de Winnicott: di- O que mantém tantos casais infelizes juntos, amargu-
cionário das palavras e expressões utilizadas por Donald rando-se, machucando-se, remoendo mágoas? Como ar-
Winnicott. – Rio de Janeiro: Revinter Editora, 1996. gumenta Maria Tereza, o casamento pode ser o início do
fim, já que “é comum buscar no outro um messias que vai
nos resgatar de dificuldades”. Quando esta expectativa se
desfaz, o frágil laço afetivo desmobiliza-se. Há casais que se
AS INTER-RELAÇÕES FAMILIARES: mantêm juntos por conveniência econômica ou social. Há
CASAMENTO, CONFLITO CONJUGAL, ainda aqueles que suportam o casamento em uma ideali-
SEPARAÇÃO, GUARDA DOS FILHOS, zação que não correspondeu à realidade. “Na visão mágica
VIOLÊNCIA DOMÉSTICA. dos contos de fada (‘e aí se casaram e viveram felizes para
sempre’), a idéia do amor eterno e da indissolubilidade do
matrimônio reforça uma imagem estática do vínculo, como
se o casamento viesse com selo de garantia”. Em outros
CASAMENTO E SEPARAÇÃO casos, exemplifica Maria Tereza, o casamento é vivido como
um “teatro da perfeição”: o casal representa papéis de per-
feição ditados por regras que não correspondem aos reais
Casamento, término e reconstrução revela a travessia
anseios de cada participante. É o conhecido “casal perfeito”,
pelos conflitantes sentimentos despertados durante o pro-
vivendo um faz-de-conta que pode cair por terra. Nestes
cesso de separação. O término do casamento desperta um
jogos – em que o amor nem sempre é o protagonista – ou-
turbilhão de sentimentos no casal que está se separando,
tros elementos contracenam: poder, status, pressões exter-
mas também em seus pais, filhos e amigos. A psicóloga
nas, desejos de infelicidade, inveja e incômodo.
Maria Tereza Maldonado disseca os sentimentos experi-
mentados pela decisão de separar-se. Tristeza, raiva e má-
Já decidi: vou mesmo me separar
goa pelos sonhos desfeitos e pelos projetos interrompidos,
Diante do abismo inevitável, os casais reagem de ma-
caminhos encontrados para nos refazermos, possibilidades neiras distintas. Uns antecipam-se e tomam a decisão sem
de reconstrução da vida com novos projetos, novos sonhos um diálogo aprofundado, outros entram em consenso,
e a renovação do amor. A obra analisa ainda as transforma- e outros, ainda, submetem os parceiros a dominações e
ções trazidas nas duas últimas décadas. ameaças na esperança de que o vínculo se mantenha. Há
Como mostra a autora, a separação é o resultado de os que, por impossibilidade de admitir as dificuldades, “ru-
um longo processo de desgaste. O término, assim como a minam” secretamente a decisão sem dar ao outro a chance
formação do vínculo, é construído por marido e mulher. Em de saber o que se passa. Em muitos casos, a separação vai
alguns casos, sendo construída ao longo de anos, acumulando mágoas e
“A separação vai sendo construída passo a passo, de alimentando o ódio. Em outros, a separação adquire clima
modo progressivo e inexorável. Mágoas engolidas e acumu- de terror. No entanto, como lembra Maria Tereza, imagina-
ladas, que não se dissolveram e se transformam em rancor, -se que a exclusão do parceiro, que representa o mal, levará
discussões não resolvidas, brigas intermináveis – tudo isso diretamente ao fim dos problemas, sem assumir responsa-
vai, pouco a pouco, minando o vínculo, trazendo uma trans- bilidades pelos conflitos e pelas queixas.
formação dos sentimentos, bem como intolerância e distân-
cia. (...) Como um armário infestado de cupins que aparen- Vítima e algoz
temente está intacto, mas desprende uma poeira discreta, Como ocorre comumente, apenas um dos dois assume
denunciando a corrosão interior, até que apodrece e se des- a dianteira e comunica a decisão. Dá-se, então, um jogo
mantela”. de vítima e algoz no qual quem anuncia a separação passa

27
CONHECIMENTOS ESPECÍFICOS - PSICÓLOGO

a ser tratado como vilão. Neste jogo de representações, o agregam ao universo dos filhos novas crianças, trazidas
casal perde de vista as responsabilidades mútuas diante do das uniões anteriores do novo parceiro. Estas novas famí-
vínculo em deterioração e podem vir à tona comportamen- lias agregadas podem ou não acarretar novos conflitos e
tos antes impensados: violência, depreciação do outro, bar- modificações de hábitos entre os filhos. Como reconstruir
ganha, negociações, ameaças, desrespeito, desvalorização. laços parentais com esta nova família enquanto ainda há
Instala-se um jogo destrutivo. conflitos gerados pelo fim da união anterior?

“Logo depois que a gente se separou...” Refazendo


Alívio, horror, depressão, perplexidade, euforia, ator- Toda a reorganização social, econômica e psicológica
doamento, embotamento, confusão, desorganização da gerada pela separação necessita de uma revisão das metas
conduta, culpa, tristeza, a coexistência e a alternância de e dos valores, já que a separação pode implicar em suspen-
sentimentos diferentes e intensos são a tônica do perío- são das metas e dos valores antes compartilhados. A nova
do pós-separação. É um momento em que “tudo oscila, fase iniciada pelo rompimento deflagra também um perío-
principalmente, entre a confiança e a falta de confiança”, do de reconstrução. É quando nos perguntamos: “Valeu a
um verdadeiro caleidoscópio de sentimentos, lembranças
pena? O que quero da vida?” A nova fase pode ser marcada
e amarguras.
por profundas inseguranças, levando a medos, descontro-
les, ânsias de viver novas emoções. Como mostra a autora,
Familiares e amigos
O impacto da separação é vivido também por aque- é um momento delicado, quando precisamos “construir um
les que cercam o casal, em especial, os filhos. A recepção código pessoal, passar da etapa de fazer a própria vida para
da notícia pode interferir na dinâmica familiar, no compro- a etapa de fazer-se”.
misso amoroso com os filhos, e as conseqüências podem
marcar profundamente a criança. Com a separação, é co- Bibliografia
mum que haja uma reestruturação do círculo de amizades MALDONADO, Maria Tereza. Casamento, Término e Re-
e novas companhias passam a fazer parte da vida dos ex- construção. Editora Intregare, 2009.
-maridos. Quem não conhece situações em que, numa se-
paração, os amigos do casal tomam o lado de um dos en- RELAÇÕES CONJUGAIS
volvidos e cortam relações com o outro? Familiares, filhos e
amigos podem, nestes casos, assumir papel de mediadores No que se refere ao padrão de conjugalidade que cada
ou catalisadores da separação entre o homem e a mulher. um se dispõe a viver, houve, a partir da década de 1960, um
questionamento sobre os papéis pré-estabelecidos defini-
E os filhos? dos por gênero, demonstrando transformação nas relações
No auge dos conflitos, pai e mãe podem deixar de per- homem-mulher. Entre muitos casais podemos perceber
ceber ou atender as necessidades amorosas dos filhos, que uma relação mais igualitária, caracterizada por uma maior
passam a se questionar sobre o amor que os pais têm por confiança e reciprocidade.
eles, uma vez que o amor vivido entre pai e mãe cai por Podemos verificar, porém, que na sociedade, a reso-
terra. Para a autora, é importante que as crianças saibam lução da equação conjugal ainda não foi superada, uma
que suas necessidades básicas continuarão a ser atendidas. vez que mulheres e homens buscam a liberdade de movi-
Afinal, a separação dos pais implica também uma reorgani- mentos e pensamentos. Porém, é possível também, nesse
zação de sentimentos e hábitos da criança. É comum per- contexto da vida matrimonial, uma reflexão de sua própria
cebermos efeitos de excesso e ausência (de amor, sustento, maneira de ser, podendo contribuir, dessa forma, para a
carinho e presentes). De que forma anunciar o rompimento construção da própria identidade.
aos filhos? Como passar a eles que pai e mãe viverão se-
Segundo Gueiros (2002, p. 109):
parados, mas o amor e a atenção pelos filhos continuarão
O casamento e a família sofreram influências das mu-
intactos?
danças sociais mais gerais e, principalmente, do movimen-
No meio da linha de fogo to feminista, e nas três últimas décadas do século passado
Da mesma forma como são influenciados pela decisão observa-se, no que se refere ao casamento, uma tendên-
dos pais, as crianças podem, por vezes, gerar transtornos cia para o debate/embate de questões como: relações de
no contexto da separação. Como a autora explicita, o rom- gênero; redefinição dos papéis públicos e privados; com-
pimento do casamento modifica a distribuição de poder portamento sexual definido segundo o sexo; constituição
entre os membros da família e muitas alterações de com- da mulher como indivíduo e construção da individualidade
portamento podem ocorrer. Em algumas famílias, o rompi- e da identidade pessoal. Neste contexto, entende-se que
mento do matrimônio gera perda de contato entre irmãos, as questões cruciais do casamento contemporâneo dizem
ódios, chantagens, cobranças, disputas pelo amor dos pais respeito à dimensão da intimidade e às próprias questões
e dos filhos. advindas da perspectiva da valorização da individualidade
e da necessidade de, ao mesmo tempo, criar-se vínculos de
Os meus, os seus e os nossos reciprocidade entre o casal [...]
Hoje muito mais presentes na sociedade, as famílias Devido às transformações existentes na sociedade,
extensas mostram as possibilidades de crescimento quan- nem sempre a família tem o seu início através de um ca-
do homens e mulheres divorciados voltam a se casar e samento. Muitos iniciam a união pelo fato de estarem dis-

28
CONHECIMENTOS ESPECÍFICOS - PSICÓLOGO

postos a constituírem uma vida em comum, seja através do Aceitar, contudo, não significa resignar-se diante de
amor existente entre ambos, ou de um sentimento forte atitudes egoístas e desrespeitosas dos parceiros, não sig-
que os une, como também em decorrência da própria his- nifica submissão, e também não significa sentimentos de
tória que foi sendo construída. posse sob o parceiro.
Pode ser que a união que ocorre atualmente na socie- Antes de esposo e esposa, é necessário compreender
dade não seja aquela na qual a sociedade considera como que os casais precisam ser companheiros, buscando in-
ideal e seja diferente do modelo considerado “correto”. Po- teração tanto através do diálogo, quanto pela relação de
rém, não há como voltar atrás, pois as mudanças vão ocor- reciprocidade e de respeito mútuo. A relação a dois não
rendo com o desenvolvimento da sociedade, assim como significa a perda de identidade, mas sim a busca do cul-
no interior de cada família. tivo da individualidade e da unidade. Anton (2000, p.196)
Anton (2000) afirma que existe uma dissociação en- afirma que alguns casais, com o passar do tempo, vão se
tre o casamento de fato e o casamento sonhado, e que tornando cada vez mais amigos e encontrando prazer nas
a sociedade alimenta esse tipo de dissociação, que con- atividades em comum, ou em estar juntos, sendo que a
tinua idealizando amor e casamento, na medida em que volta para casa é uma ‘alegria’. Nesses casos específicos,
defende ideias como gratuidade e doação total. Ocorre ela conclui que “o lar se constitui num ambiente aconche-
que em uma relação, segundo Anton, os fatos nem sempre gante, em que cada um pode se reabastecer, efetuando as
correspondem aos desejos, e desejar nem sempre significa trocas íntimas mais significativas. ”
concretização dos objetivos, podendo gerar dificuldades e Ocorre que, na atualidade, com o mundo globalizado,
fracassos na escolha do companheiro e na evolução do re- pode ser que o afeto, a convivência familiar e o cuidado
lacionamento a dois. mútuo percam espaço no cotidiano, e que as pessoas não
consigam ver no outro um ser social, um sujeito dentro da
Em um relacionamento podem existir afetos positivos e sociedade, mas acabam usufruindo os outros como se fos-
negativos, como experiências de convívio familiar, que são sem objetos.
singulares, podendo existir sentimentos de amor e cumpli- Tanto o casamento quanto as uniões estáveis consti-
cidade. As relações afetivas que envolvem os laços conju- tuem realidade complexa, com caracterização própria, con-
gais tornam essa relação complexa, pois tal relacionamento ceitos e legislações que permeiam o cotidiano do casal.
envolve uma série de experiências comuns que só podem Eles podem se constituir instrumentos de redefinição de
ser divididas entre ambos. identidade, pois irão dar início a nova identidade, sendo
A convivência com o diferente pode ser experiência que ela deve ser estabelecida no diálogo e constantemente
desafiadora no cotidiano familiar. Compreender o outro repensada.
não significa concordar plenamente com tudo o que o Callil (1987) afirma que a escolha do parceiro é atra-
parceiro ou a parceira pensa e faz. Significa que o outro vessada pelo aspecto psicológico e que as motivações que
é pessoa diferente e que tem pensamento diferente, mas impulsionam o indivíduo a escolher alguém para o casa-
que nem sempre está equivocado. Diante dessas questões, mento estão relacionadas a aspectos inconscientes, assim
podemos verificar que o contato cotidiano permite a vivên- como o que provoca a atração entre os cônjuges, e não aos
cia com uma realidade próxima a si mesmo, mas que pode atributos individuais. Ele ainda coloca que nessas escolhas
ser, concomitantemente, distante daquilo que se tem como há encaixe das personalidades de cada um, sendo que essa
ideal de vida conjugal. escolha é realizada, na maioria das vezes, buscando a com-
Pode ser que haja uma falsa compreensão do real sig- plementaridade. Acreditamos que essa escolha do parcei-
nificado do sentimento que é gerado através da convivên- ro passa também pela via psicológica, mas não somente
cia cotidiana com a outra pessoa. Alguns pensam que é por esse ângulo, pois senão iremos desconsiderar toda a
sinônimo de submissão, como podem também estar rela- historicidade presente na construção do sujeito enquanto
cionando esse sentimento com desejos de posse, ou com ser social e dessa forma, não compreendê-lo enquanto su-
alguma espécie de egoísmo, tentando afirmar-se através jeito histórico social.
da pessoa do parceiro. É certo que com o companheirismo, convívio e da
Podemos recorrer a Anton (2000, p.190, destaque do socialização pode ocorrer o crescimento individual de
autor), quando afirma que cada um nesta relação, mas essa complementaridade se dá
[...] o amor pressupõe, sim o conhecimento dos va- através da vivência dos desafios existentes na história do
lores do amado, a admiração e o respeito por ele. Mas há casal.
uma certa medida neste encantamento, pois ele deve ter Através da convivência diária, um pode se mostrar ao
bases reais e situar-se dentro dos limites do real. Amor e outro, deixando transparecer a sua história, cultura e pro-
adoração não são sinônimos. Nem amor e paixão. Mesmo jetos. A sua individualidade deve existir naturalmente, as-
que tais sentimentos (paixão, adoração) se façam presentes sim como é preciso que haja respeito à individualidade do
num primeiro estágio, eles devem desaparecer em algum outro.
tempo. O amor subsiste às intempéries, pois implica na O que é vivenciado no cotidiano das relações conjugais
aceitação do outro, também em suas limitações, em seus começa, segundo Callil (1987, p. 120) a ser descoberto des-
senões. de a concepção:

29
CONHECIMENTOS ESPECÍFICOS - PSICÓLOGO

Através da ampla gama de relacionamentos com pai, Diante das relações diretas no relacionamento conju-
mãe, pai-mãe e irmãos, etc., a criança desenvolve um reser- gal, é possível refletir sobre a complexidade existente den-
vado acumulado de potencial relacional com estas figuras, tro da família, podendo concluir que há uma diversidade
que se tornam, então, modelos internalizados de relacio- de fatores que influenciam nesta relação, nas suas diversas
namento. Esses modelos estão a serviço de relacionamen- etapas. É possível verificar também que existem transfor-
tos futuros, especialmente no casamento, paternidade ou mações individuais dentro da família, e que essas mudan-
maternidade. ças podem ocasionar insatisfações ou satisfações para cada
A relação entre os membros da família, dessa forma, membro do grupo familiar.
sofre influências da maneira pela qual cada membro viven- Não é preciso fazer uma análise profunda para veri-
ciou as suas relações e emoções durante o percurso social. ficar que nestes modelos específicos, há espaço para o
Existem experiências comuns que o casal vivencia que só desenvolvimento dos interesses individuais de cada uma
poderão ser partilhadas entre ambos. O impacto que cada das partes, assim como para o desenvolvimento da relação
um pode causar sobre o outro é imenso, durante a vida conjugal, criando espaço favorável para o desenvolvimento
conjugal pode existir a mútua socialização, a ajuda e as ati- humano de cada um.
vidades cooperativas, assim como o companheirismo. Pode No entanto, para discorrer sobre as uniões é preciso
também ocorrer comportamentos conflituosos, agressivos passar também pelas questões referentes às relações de
e que trazem uma experiência distante da harmonia. gênero, intrínsecas ao ambiente familiar. É preciso entender
Essa relação, no entanto, pode trazer também aprendi- o significado dessas relações no âmbito cultural, histórico,
zado de habilidades e sentimentos que, por meio das tro- do que é ser mulher e homem na sociedade e como essas
cas cotidianas, pode passar a fazer parte das características relações se reproduzem no cotidiano da vida familiar.
de cada cônjuge. Conforme Scott (1992, p. 86, destaque do autor):
Não se pode deixar, contudo, de compreender que
essas experiências servem para as demais relações sociais [...] “Gênero” foi o termo usado para teorizar a ques-
que serão vivenciadas fora do núcleo familiar. tão da diferença sexual. Nos Estados Unidos, o termo é
É importante lembrar que as relações possuem extraído tanto da gramática, com suas implicações sobre
variações e especificidades conforme cada família, que irão as convenções ou regras (feitas pelo homem) do uso da
desdobrar-se em outras e possivelmente influenciar tanto linguística, quanto dos estudos de sociologia dos papéis
as relações intrafamiliares, quanto as relações sociais de sociais designados às mulheres e aos homens.
uma maneira geral. Krom (2000) traz uma reflexão sobre
os mitos familiares e afirma que a maneira como o casal Assim, a diferença sexual nas relações familiares é visí-
construirá o seu casamento está diretamente ligada aos vel, sendo estas as responsáveis tanto pela complementa-
mitos que advém de suas famílias de origem. Assim, cada ridade existente no âmbito dessas diferenças, quanto pelos
cônjuge traz sua história de vida e o diálogo entre ambos conflitos decorrentes de tais particularidades.
é experiência difícil. A união conjugal propicia a junção O importante na relação familiar é entender que a
desses mitos familiares e pode ser ‘benéfica’ ou ‘nociva’ questão de gênero perpassa por todos os lares e que tal
ao relacionamento conjugal e familiar, dependendo da questão pode ser um fator de crescimento e de diversidade
maneira pela qual as relações interpessoais vão acontecer. na relação conjugal e que os conflitos decorrentes dessas
É preciso refletir sobre a maneira pela qual esses mitos diferenças nem sempre são negativos, podendo exercer
irão influenciar direta ou indiretamente as expectativas em papel importante no cotidiano familiar, que deve ser ‘cui-
relação ao casamento, uma vez que estes podem dificultar dado’ para que não se torne rotineiro.
as relações conjugais. Pode ser que os cônjuges entrem no Continuando sua reflexão o autor Scott afirma:
casamento procurando buscar o que o outro não tem, mas Embora os usos sociológicos de “gênero” possam in-
esperando que tenha. Isso pode ocasionar sentimento de corporar tônicas funcionalistas ou essencialistas, as femi-
frustração diante de uma situação contrária à esperada, nistas escolheram enfatizar as conotações físicas de sexo.
podendo desencadear desilusão nas relações conjugais. Também enfatizaram o aspecto relacionado do gênero: não
A união entre duas pessoas de gêneros diferentes se pode conceber mulheres, exceto se elas forem defini-
numa convivência íntima e intensa pode ocasionar em ho- das em relação aos homens, nem homens, exceto quando
mens e mulheres muitas mudanças de concepções que já eles forem diferenciados das mulheres. Além disso, uma
tinham sido construídas desde sua concepção, e que, mui- vez que o gênero foi definido como relativo aos contextos
tas vezes, estava enrijecida pela delimitação do que é papel social e cultural, foi possível pensar em termos de diferen-
masculino e o que é papel feminino, podendo dificultar a tes sistemas de gênero e nas relações daqueles com outras
expansão de novos papéis no relacionamento a dois. categorias como raça, classe ou etnia.
Pode-se observar que a relação de gênero é utilizada
O convívio a dois se estabelece nas diferenças, que para definir as diferenças existentes entre homens e mu-
podem ser tanto positivas – quando são entendidas como lheres, e também pode demonstrar que as desigualdades
espaços favoráveis para as possibilidades de crescimento decorrentes dessas diferenças não podem ser determina-
do casal, quanto negativas – se o desafio existente no coti- das somente pelas diferenças biológicas existentes entre
diano da vida a dois se tornar impossível de ser superado, ambos, mas são construídas socialmente e tais construções
podendo gerar frustrações diante das expectativas de cada são expressas nas atribuições que são definidas pela socie-
cônjuge. dade para homens e mulheres.

30
CONHECIMENTOS ESPECÍFICOS - PSICÓLOGO

Dessa maneira, podemos dizer que o padrão conven- É importante pensar sobre a relação de gênero no
cional de relações de gênero se define de acordo com o conjunto das relações sociais. Não é possível dissociar a
contexto social. Podemos recorrer ao Brasil, país diversifi- questão de classe de relações interpessoais, pois estas
cado social, cultural e economicamente, no qual as rela- estão dentro da estrutura e obedecem às normas que
ções de gênero não podem ser generalizadas. Vejamos a permeiam a sociedade por inteiro. A concepção de relações
mulher: mesmo que a sociedade continue reproduzindo o interpessoais dissociada da estrutura de classes representa
modelo da mulher que vive no lar e para o lar e o homem visão que não contribui para esclarecer o porquê da
viva para o trabalho, historicamente, houve uma mudança, sociedade comportar violência intrafamiliar, doméstica,
pois a mulher passou a acumular atividades e responsabi- contra mulheres e de gênero. É preciso perceber as relações
lidades não somente no âmbito público, mas também no
internas da sociedade, pois corremos o risco de perda da
privado.
visão da sociedade como totalidade. É preciso entender a
Trabalhar dentro e fora do lar tem significado ímpar nas
reflexões acerca das relações de gênero na vida cotidiana. sociedade em sua inteireza, com tudo o que ela contém:
A mulher adentrando no mundo do trabalho vive as maze- contradições, desigualdades, iniquidades (SAFFIOTI, 2002,
las desse próprio mundo de maneira diferente do homem online).
inserido no universo masculino. Afirmar que a vida conjugal perpassa o contexto social,
Já o homem, mesmo ampliando suas atividades no assim como pela questão de gênero, é importante para a
cotidiano do lar, no cuidado com os filhos e em demais compreensão do significado que as uniões vêm tomando
atividades que anteriormente eram atribuições exclusivas no cenário brasileiro. Em meio às mudanças ocorridas nes-
do universo feminino, este ainda continua sendo priorida- sas últimas décadas, em especial, através das constituições
de no mundo público, não sendo responsabilizado por não diversificadas de família, e dos avanços na sociedade, po-
prover o lar e tampouco por não reconhecer o filho – nos demos afirmar que há diversidade de uniões conjugais, seja
aspectos morais e legais. através do casamento propriamente dito, ou das uniões
As discussões acerca da questão de gênero perpassam consensuais existentes na contemporaneidade.
pela questão do feminino e do masculino e sobre as in- Atualmente, percebemos que as legislações que defi-
fluências que estes possuem, em especial, quando detêm nem direitos e deveres dos cônjuges, trazem, em seu bojo,
o poder. as possibilidades de atingir a ‘harmonia familiar’. Ocorre
Saffioti (2002, online, destaque do autor) levanta os se- que nem sempre tais objetivos são alcançados, e dessa
guintes questionamentos:
forma, fica difícil atingir os objetivos dessas leis, que estas
Se o “gênero é uma maneira primordial de significar
serviriam de fonte de apoio para as possíveis adversidades
relações de poder” (Scott, 1988, p. 42), nem homens nem
mulheres podem situar-se fora dele. Obviamente, esta que ocorrerem no período da convivência conjugal e fa-
mobilidade pelas distintas matrizes de gênero permite a miliar. Podemos compreender também as especificidades
ressignificação das relações de poder, o que constitui o do cotidiano da vida familiar, para que estas possam ser
objetivo prioritário das diferentes vertentes do feminismo. repensadas a cada momento, neste cotidiano pode ocorrer
Praticamente toda a bibliografia aqui utilizada defende a o inesperado, o novo e situações que sejam de difícil reso-
idéia desta precedência do gênero na constituição da iden- lução no contexto intrafamiliar.
tidade, ou, se se preferir uma fórmula mais maleável, das
subjetividades dos seres humanos (Safiotti, 1997). Conflitos e Separações Conjugais

Admitir o gênero como precedente na constituição da Costumo dizer que todo fascínio e toda dificuldade
identidade do indivíduo, pode significar que este possui de ser casal, reside no fato de o casal encerrar, ao mes-
parcela de importância ímpar nessa constituição, já que o mo tempo, na sua dinâmica, duas individualidades e uma
ser humano, que também possui subjetividade, está a cada conjugalidade, ou seja, de o casal conter dois sujeitos, dois
dia se construindo e se constituindo. desejos, duas inserções no mundo, duas percepções do
Apesar dos avanços na interpretação entre os gêneros, mundo, duas histórias de vida, dois projetos de vida, duas
as transformações socioculturais existentes e toda a gama identidades individuais que, na relação amorosa, convivem
de informações que norteia a sociedade moderna, não é
com uma conjugalidade, um desejo conjunto, uma história
possível afirmar que existe uma maneira igualitária de in-
de vida conjugal, um projeto de vida de casal, uma identi-
corporação dos direitos e deveres da mulher e do homem.
Eles vivenciam as diferenças entre os próprios gêneros. dade conjugal.
Ainda que avanços tenham ocorrido, a questão social Baseando-nos nesta reflexão que pretendemos con-
que permeia esta relação, especialmente entre os cônjuges, tinuar a nossa discussão sobre a relação conjugal, e, em
é caracterizada pelas relações de dominação e de poder especial, sobre o convívio conjugal dentro da dinâmica da
que a própria cultura incorporou na execução dos papéis família, inserida nesta sociedade contemporânea.
familiares. A família, em sua relação interior, é influencia- As tensões existentes entre as individualidades, tão
da pela prerrogativa do poder e da dominação que foram presentes na relação conjugal não poderiam deixar de se
explicitados, principalmente, no seio da família patriarcal, e manifestarem, vivemos em uma sociedade marcada pelo
atualmente, apesar de tantas superações, ainda estão pre- individualismo, característica primordial do liberalismo e
sentes no cotidiano da vida familiar. que se renova no neoliberalismo.

31
CONHECIMENTOS ESPECÍFICOS - PSICÓLOGO

A relação a dois pode ser influenciada por uma diver- A relação, com o passar do tempo, pode vir a se des-
sidade de fatores que estão presentes na realidade, estas gastar, a cair na rotina e a decepcionar o casal. Nesse caso,
questões não podem ser desconsideradas no cotidiano do podem começar a aparecer os conflitos no relacionamento,
casal. que através de influências socioculturais podem vir a inter-
Existe, por um lado, a predominância do individualis- ferir no cotidiano do casamento.
mo, que, de certa forma, pode influenciar na autonomia É através das relações formadas na família, com-
dos cônjuges, e por outro lado, há a necessidade de vi- preendidas num contexto amplo, que as transformações
venciar a realidade do casal, como os desejos e possíveis ocorrem ao longo da vida de cada indivíduo. Surge, dessa
projetos conjugais. forma, um questionamento: será que pode existir uma de-
terminação de fatores que podem estar intervindo em um
É certo que cada casal irá conceber o casamento e
conflito familiar num determinado momento? Talvez seria
o relacionamento intrafamiliar de uma maneira, e esta
isto que os profissionais, parentes, ou até mesmo os cônju-
experiência irá determinar os limites e as possibilidades ges envolvidos nesse conflito desejariam para amenizar os
de tal relação. Dessa maneira, será construída a identidade desentendimentos, na busca de soluções para a problemá-
conjugal. tica apresentada.
A partir do momento que as expectativas dos cônjuges A vida pode levar a separações, transformações, per-
não são satisfeitas, eles podem assumir uma crise confli- das e ganhos a cada passagem do desconhecido para o
tuosa na união, que, dependendo da maneira pela qual os novo. Casamentos e separações também podem acarretar
mesmos vão enfrentar esse desafio, podem não suportar um período de rupturas, adaptação às mudanças de estilo
tais questões. de vida, valores e hábitos do cotidiano. As trajetórias pes-
Consideramos que na atualidade é preciso que o ca- soais da vida do homem e da mulher podem impossibilitar
sal venha a conciliar o novo modo de ser família, com a a continuidade do relacionamento.
vida familiar e a realização pessoal. Isso pode não ser tarefa Quando existe o desgaste da relação, pode ser que
simples, pois é constituída por contradições e regida pela alguns casais vivenciem um conjunto de sentimentos: des-
prezo, ataques, maus tratos. Se esses dissabores continua-
característica do neoliberalismo: o individualismo.
rem, há a possibilidade de existirem insatisfações, críticas
Essas características da vida em comum na atualidade e exigências do parceiro, que, muitas vezes, pode ser ins-
podem trazer para a família e, em especial, ao casal, uma trumento de tortura, gerando sentimentos variados, como
diversidade de maneiras de convivência. Tais convivências infelicidade e frustração.
podem ser fatores impulsionadores de crescimento pessoal Nesse sentido, muitos cônjuges permanecem em si-
e familiar, assim como podem ser motivos de enfraqueci- tuações de vida, na esperança de ficarem protegidos e res-
mento dos vínculos familiares. guardados de maiores mudanças, arrastando pela vida um
O cotidiano do casal é influenciado pelas mudanças relacionamento infeliz. Essa infelicidade leva a construção
societárias, que afetam as estruturas e o contexto no qual da conjugalidade carregada de insatisfações.
os cônjuges estão inseridos. Cada casal, entretanto, pos- As dificuldades de entendimentos entre os cônjuges,
sui uma maneira de vivenciar essas transformações histó- a incapacidade vida harmônica com o outro, prejudicam
ricas. O cotidiano, carregado de historicidade, é revestido o relacionamento do casal, podendo gerar transtorno nas
pelo tempo presente. Segundo Heller (2004), o tempo é relações conjugais e familiares.
É difícil amar incondicionalmente. As condições que
a irreversibilidade dos acontecimentos. O tempo históri-
são colocadas constituem forma de critério, de escolha, ou
co é a irreversibilidade dos acontecimentos sociais. Todo até mesmo na opção de exclusão no próprio relacionamen-
acontecimento é irreversível do mesmo modo; por isso, é to. Quando impomos uma condição, supomos que por trás
absurdo dizer que, nas várias épocas históricas, o tempo desta vem o desejo pessoal, a vontade própria. Numa re-
decorre em alguns casos ‘lentamente’ e em outros, “com lação, quando um coloca em primeiro lugar a sua escolha,
maior rapidez”. O que se altera não é o tempo, mas o rit- a sua vontade, não pensando no outro, fica complicada a
mo da alteração das estruturas sociais. Mas esse ritmo é questão da harmonia.
diferente nas esferas heterogêneas. É esse o fundamento Há pessoas que lamentam mais o que perdem que se
da desigualdade do desenvolvimento, que constitui uma alegram com o que ganham, e isso pode gerar conflitos e
categoria central da concepção marxista da história. confusões, uma vez que não conseguem ver o lado sadio das
coisas e somente ficam com suas lamentações, ressaltando
Dessa maneira, há também a possibilidade de essas mais as tristezas do que as alegrias e realizações.
mudanças interferirem no cotidiano do casal, este é refle- Segundo Jablonski (1998, p.86):
Quando o amor ‘acaba’, ou melhor, se transforma, os
xo de todo o contexto existente. Os cônjuges podem estar
casais se sentem traídos, tendendo a culpar seus pares ou a
sendo impedidos, pelo reflexo do individualismo que paira
si mesmos pelo ‘fracasso’, e não à cultura que lhe empurrou
na sociedade contemporânea, de viverem a conjugalidade, um modelo não muito compatível com a própria realidade.
e, consequentemente, deixando de viverem a realidade co-
mum do casal. Não se pode negar o fato de que o amor vem sendo
Essa nova maneira de ser casal, pode levar a um au- banalizado pela própria sociedade, seja através dos meios
mento de expectativas, idealização do outro e uma exigên- de comunicação de massa, como a televisão, que pene-
cia consigo mesmo, podendo levar aos conflitos conjugais. tra na maioria dos lares, como também por outros meios,

32
CONHECIMENTOS ESPECÍFICOS - PSICÓLOGO

como rádios, revistas, jornais. As influências vão desde can- Porchat (1992) coloca que essas perdas criam um es-
ções que mostram uma forma diferente de amar, como se paço vazio, o qual suscita nas pessoas a dor de estar só,
o amor pudesse ser negociado no mundo globalizado, até impotente e, em muitos casos, vivendo o sentimento de
produtos que o mercado oferece como formas de fazer do fracasso difícil de suportar.
amor uma forma de circulação nessa sociedade monopo- A ruptura conjugal pode trazer à tona as outras sepa-
lizada pelo capitalismo. Nesse sentido, o amor, na união rações vivenciadas desde a primeira infância até a perda
contemporânea, pode ou não existir. de um ente querido. A dor da separação pode ser intensa
Del Priore (2006, p. 320) traz um comentário sobre essa ou não, conforme a assimilação das perdas que sentiram
mudança da maneira de se vivenciar o amor: durante a vida. Essa ruptura pode englobar também várias
Vimos que há séculos o chamado amor romântico, nas- perdas, como a de amigos, de filhos, estilo de vida, perfil
cido com os trovadores medievais, fundou a idéia de uma socioeconômico, além da perda da autoestima e do senti-
união mística entre os amantes. A idealização temporária, do da vida.
típica do amor-paixão, juntou-se ao apego mais duradouro Muitas vezes, por falta de coragem, de recursos pró-
do objeto de amor. O amor romântico, que começa a exercer prios, ou até mesmo por comodismo diante de determi-
sua influência a partir de meados do século XIX, inspirou- nada situação, os cônjuges não conseguem romper com
-se em ideais desse tipo e incorporou elementos do amor/
a união e tendem a levar essa vida separada e dissolvida
paixão. Não foi à toa, lembram os especialistas, que o nas-
dentro do próprio lar, onde os mesmos passam a viver
cimento do amor romântico coincide com a aparição do
como amigos, irmãos, e as características de casal passam
romance: ambos têm em comum nova forma de narrativa.
à distância.
Aquela em que duas pessoas são a alma da história, sem
referência necessária a processos sociais que existiam em A decisão de assumir a separação pode causar impac-
torno delas. [...] A reorganização das atividades cotidianas tos na vida pessoal e abalar, por vezes, a estrutura emocio-
ocasionou uma reorganização profunda na vida emocional nal, fato que pode empacar a separação. Por um lado, há o
que ainda está por ser estudada. Ambas, contudo, ajudaram desejo de separar-se; por outro há o medo de concretizar
a sepultar, devagarzinho, antigas tradições referentes à esco- a decisão.
lha dos pares e às formas de dizer o amor. Para a consumação da separação existem muitos ti-
pos de tomada de decisão - desde o nível do sonhado,
É possível concordar com a autora, na medida em que do planejado, do conversado, do concretizado, até que se
na sociedade contemporânea há uma disparidade entre o passe a viver em casas separadas, ou efetivem a separação
discurso e o real vivenciado pelas pessoas. Apesar de muitas judicialmente. Os sonhos e as expectativas do casal podem
uniões não se iniciarem através do amor, ele continua se romper juntamente com o fim da união.
sendo um sentimento importante, um instrumento de
sonho para determinados homens e mulheres. A mudança O início da insatisfação conjugal, perder as esperanças
está na maneira de se vivenciar e nomear o amor. Dessa de melhoria do casamento, decidir separar-se e, finalmente
maneira, podemos verificar que o amor é um sentimento concretizar a separação em si, pode durar muito tempo, até
cuja manifestação está vinculada ao seu tempo, ao seu mesmo vários anos.
contexto. Vejamos as mudanças na maneira de manifestar Apesar de existir a lei que oficializa a separação, pode
esse amor ao ser amado – beijos, abraços, carícias - essas ser que as pessoas não estejam preparadas para enfrentar
foram sofrendo transformações ao longo da história. grandes mudanças, sendo que a separação poderia ser vis-
Pode ser que a união por aparência exista, e que as ta como um fracasso, juntamente com a pressão familiar
pessoas se sujeitem a determinadas condições para con- contra o término do casamento e a pressão da própria so-
viver com o outro, mesmo que verdadeiramente este não ciedade sobre essa possível situação transformada.
exista a conjugalidade. Ao decidir pela separação e oficializá-la, nos termos da
A união é entre duas pessoas e não adianta somente lei, o casal passa a assumir a situação de não estar juntos,
uma parte querer mudar a situação, enquanto a outra parte
ou seja, de estar separados. A legalização pode envolver
continuar com os mesmos costumes, os mesmos defeitos,
aspectos importantes na vida pessoal, como a de mudança
não permitindo que o casal se supere enquanto cônjuges.
de identidade, refletida na mudança de estado civil.
Em cada crise, ou passagem para o novo, pode ha-
Para a pessoa que decidiu se separar, do ponto de vista
ver uma mudança de aspectos importantes da pessoa e
de seu modo de ser. Quando esses sentimentos se aliviam, jurídico, existem as leis que regulamentam a separação ju-
as pessoas podem enxergar novas perspectivas de vida e dicial, o divórcio, a dissolução da sociedade de fato.
caminhar decididamente. Os vários tipos de dissolução da sociedade conjugal
Segundo Vicente (2002), o vínculo é aspecto funda- são denominados conforme a forma de união estabelecida.
mental na condição humana, essencial ao desenvolvimen- Quando existe o casamento no civil, o processo chama-se
to. Talvez seja por esse motivo que fica tão difícil o rompi- separação judicial, que pode ser consensual ou litigiosa.
mento temporário e definitivo do mesmo. A pessoa não Para que ocorra a separação por mútuo consentimento
consegue agir, pois existe o medo do sofrimento e da dor, (separação consensual) é preciso que a união tenha mais
ocasionados pelas rupturas, que bloqueiam a decisão de se de um ano, conforme estabelece o artigo 1574 do Código
separar do cônjuge. Civil:

33
CONHECIMENTOS ESPECÍFICOS - PSICÓLOGO

Dar-se-á a separação conjugal por mútuo consenti- Quanto à dissolução da sociedade de fato, o artigo
mento dos cônjuges se forem casados por mais de um ano, 7° da Lei n° 9278, de 19 de maio de 1996, estabelece que:
e manifestarem perante o juiz sendo por ele devidamente “Dissolvida a união estável por rescisão, a assistência mate-
homologada a convenção. Parágrafo Único. O juiz pode re- rial prevista nesta Lei será prestada por um dos conviventes
cusar a homologação e não decretar a separação judicial se ao que dela necessitar, a título de alimentos”. Ressaltamos
apurar que a convenção não preserva suficientemente os que a família, na atualidade, nem sempre é constituída pelo
interesses dos filhos ou de um dos cônjuges. casamento oficial, mas, em grande parcela, pelas uniões
A separação litigiosa ocorre quando uma das partes consensuais.
não concorda com a ruptura da união, aliado ao descum- Geralmente a decisão de separar-se é parcial, e existe
primento dos deveres do casamento, tornando insuportá- uma pessoa que solicita a separação, e esta futuramente
vel a vida em comum. Ela pode ser requerida a qualquer
poderá ser considerada a culpada, enquanto a outra, fica
tempo do casamento. O artigo 1572 do Código Civil dispõe
como vítima, ou seja, enquanto existe um que deixa e o
sobre essa forma de dissolução:
Qualquer dos cônjuges poderá propor a ação de se- outro é deixado; é a vítima.
paração judicial, imputando ao outro qualquer ato que Ao tomar consciência desse processo de separação,
importe grave violação dos deveres do casamento e torne a pessoa pode passar por etapas semelhantes a qualquer
insuportável a vida em comum. outra passagem do desconhecido para o novo, só que
§ 1° A separação judicial pode também ser pedida se com um sentimento diferente, uma sensação de que algo
um dos cônjuges provar ruptura da vida em comum há de ruim está acontecendo, por mais que essa vontade de
mais de um ano e a impossibilidade de sua reconstituição. separar-se já estivesse instalada há muito tempo. Essa é
§ 2° O cônjuge pode ainda pedir a separação judicial uma questão cultural, que vai além do controle individual.
quando o outro estiver acometido de doença mental grave, A concepção que a sociedade possui a respeito de pessoas
manifestada após o casamento, que torne impossível ‘separadas’ é carregada de conceitos pré-estabelecidos e
a continuação da vida em comum, desde que, após moralistas, sendo, na maioria das vezes, acusatórios e dis-
uma duração de dois anos, a enfermidade tenha sido criminatórios.
reconhecida de cura improvável. Quando a separação é concluída, a pessoa se defron-
§ 3° No caso do § 2°, reverterão ao cônjuge enfermo, que ta com a etapa de adaptação à nova situação com todas
não houver pedido a separação judicial, os remanescentes as mudanças que esse processo envolve. Por um lado, há
dos bens que levou para o casamento, e se o regime dos possibilidade de se arruinar com o sentimento de culpa de
bens adotado o permitir, a meação dos adquiridos na
ter deixado a outra pessoa, por outro, existe a possibilidade
constância da sociedade conjugal.
de uma vida realizada, possivelmente, assemelhando-se à

Nery Junior e Nery (2003) comentam que para a sepa- felicidade.
ração de fato basta a comprovação de que os cônjuges já No permear do processo de dissolução do vínculo
não suportam mais a vida em comum, sendo esta condição conjugal, no momento em que é chegada a hora de sair
impossível de se reconstituir. Esta separação normalmente de casa, ou de ser deixado, as expectativas, as apreensões
precede a separação judicial e o divórcio. parecem tomar conta da pessoa, ao mesmo tempo em que
Quando existe violência, ameaças e agressões, po- a ansiedade vem à tona. Podem ocorrer reações diferen-
demos realizar também, primeiramente, como medida de ciadas, sentimentos diversificados a partir do momento em
prevenção, denominada medida cautelar, a separação de que o ex-cônjuge deixa o lar.
corpos, que normalmente é mais rápida que a separação Mesmo quando a relação existente entre os cônjuges
judicial, e de certa forma torna-se proteção para o cônjuge era conflituosa e desgastante, e as partes envolvidas no
agredido e ameaçado. processo sentem até um alívio ao concretizar a separação
Já o artigo 1573 do Código Civil vem tratar sobre os conjugal, para a maioria das pessoas, os primeiros dias de-
motivos que acarretam a impossibilidade da vida a dois: pois da separação podem ser um choque. É como se hou-
Podem caracterizar a impossibilidade de comunhão de vesse quebra na rotina familiar.
vida a ocorrência dos seguintes motivos:
I – Adultério;
Com a separação, existem mudanças de aspectos im-
II – Tentativa de morte;
portantes na vida da pessoa, que podem ser intensificadas
III – Servícia ou injúria grave;
IV – Abandono voluntário do lar conjugal; quando, além da ruptura da convivência, as mudanças pas-
V – Condenação por crime infamante; sam a ser radicais. Um exemplo dessa situação é a ques-
VI – Conduta desonrosa. tão da queda do nível financeiro, que pode ocorrer, onde
Parágrafo único. O juiz poderá considerar outros fatos as pessoas, passam de determinado padrão de vida, cujas
que tornem evidente a impossibilidade da vida em comum. características possibilitavam algumas regalias, tais como
empregada, carros, eletrodomésticos sofisticados, para
Após prévia separação judicial por mais de um ano, ou outro padrão, cujas necessidades passam a serem revistas.
se for comprovada a separação de fato por mais de dois Nas famílias consideradas economicamente sem condições
anos, e o casal optar pela ruptura total do vínculo conjugal, de arcar com seu sustento, é possível que a dificuldade fi-
conforme o artigo 226, § 6°, do Capítulo VII da Constituição nanceira aumente, excluindo ainda mais a pessoa do aces-
Federal, o casamento pode ser dissolvido pelo divórcio. so aos bens necessários para a sobrevivência.

34
CONHECIMENTOS ESPECÍFICOS - PSICÓLOGO

No processo de separação conjugal, há tanto a separa- Quanto à guarda dos filhos, anteriormente ficava a
ção das partes envolvidas, quanto a separação e a partilha cargo da mãe. Atualmente, com o Novo Código, os filhos
dos bens do casal. Algumas pessoas deixam tudo para o ficam com o que possuir melhores condições de cuidar dos
outro como uma forma de livrar-se de um incômodo, mas mesmos e de educá-los. Nesses casos, se a decisão não for
comumente é rara a pessoa que tenha uma real considera- de comum acordo do casal, há a necessidade da realização
ção pelo outro no momento da partilha. de estudo social e psicológico, buscando intervenções no
Quanto à partilha de bens, sua regulamentação legal, sentido de amenizar os problemas vivenciados pela crian-
no Código Civil Brasileiro, prevê: ça. A realidade brasileira, permeada por mudanças con-
temporâneas, traz à tona uma nova modalidade de guarda
Art. 1575. A sentença de separação judicial importa a de filhos. Trata-se da guarda compartilhada. Segundo a
separação de corpos e a partilha de bens. Constituição Federal de 1988, no artigo 5º, parte I a igual-
Parágrafo único. A partilha de bens poderá ser feita dade entre o homem e a mulher. O artigo 226, parágrafo
mediante proposta dos cônjuges e homologada pelo juiz 5º traz a questão dos direitos e deveres referentes à socie-
ou por este decidida. dade conjugal, que devem ser igualmente exercidos pelo
A divisão de bens será feita conforme o regime de bens homem e pela mulher. Diante dessa realidade, podemos
escolhido para o casamento, se é comunhão parcial de refletir sobre a guarda compartilhada. Se atualmente hou-
bens, separação de bens, comunhão universal de bens e do ve uma evolução nos conceitos de família, podemos veri-
regime de participação final nos aquestos. Lembramos que ficar que tanto o referencial paterno quanto o referencial
no regime de comunhão parcial, os bens e dívidas adquiri- materno exercem igual influência para o desenvolvimento
dos após a celebração do casamento passam a ser comum
da criança, com exceção das situações excepcionais, como,
ao casal. No regime de comunhão universal os bens pre-
por exemplo, na fase da amamentação.
sentes e futuros dos cônjuges e suas dívidas passivas, pas-
O conceito de guarda compartilhada, segundo Barreto
sam a ser comuns entre o casal. No regime de separação
(2003, online) pode ser entendido como:
de bens a administração é exclusiva de cada um dos côn-
[...] um sistema onde os filhos de pais separados per-
juges, sendo que cada um se responsabiliza por seus bens
manecem sob a autoridade equivalente de ambos os genito-
e dívidas, mesmo após a união. No regime de participação
res, que vêm a tomar em conjunto as decisões importantes
final nos aquestos cada cônjuge possui patrimônio próprio,
quanto ao seu bem-estar, educação e criação. É tal espécie
e lhes cabe o direito de metade dos bens adquiridos pelo
casal na constância do casamento. de guarda um dos meios de exercício da autoridade paren-
Quanto à dissolução da sociedade conjugal, na parti- tal, quando fragmentada a família, buscando-se assemelhar
lha de bens encontramos a seguinte regulamentação, Art. as relações pai/filho e mãe/filho – que naturalmente tendem
1725. do Código Civil: “Na união estável, salvo contrato a modificar-se nesta situação – às relações mantidas antes
escrito entre os companheiros, aplica-se às relações da dissolução da convivência, o tanto quanto possível.
patrimoniais, no que couber, o regime da comunhão parcial Não podemos negar o fato de que a guarda comparti-
de bens.” lhada, enquanto nova maneira de vivenciar o poder paren-
Várias jurisprudências discorrem a respeito da união tal, é alternativa que possui uma intenção de rompimento
estável, como é o fato da Revista dos Tribunais 778/238 com o tradicional, cujas características eram unilaterais, e,
que afirma: em especial, a mãe, na maioria das vezes, é quem se res-
De acordo com o art. 5º da Lei 9.278/96, caracterizada ponsabilizava pelos filhos.
a sociedade de fato entre o casal, não há que ser exigida a Porém, consideramos a existência de diversos fatores
prova do esforço comum para a formação do patrimônio, que podem impedir que a guarda compartilhada se efetive
uma vez que o mesmo é presumido, ainda que, à época da concretamente. Esses fatores são referentes aos resquícios
aquisição do bem, a união estável fosse nascente. de atritos entre os ex-cônjuges, às condições em que os
mesmos se encontram, à liberdade de escolha dos próprios
Durante a realização da partilha dos bens, é comum filhos, dentre outros fatores.
ver as decepções retratadas pelos rostos dos cônjuges que O autor Barreto (2008) se refere à guarda como se
acabam lutando por seus direitos de forma radical, não fosse uma espécie de continuidade das relações pai/fi-
pensando no outro, buscando seus próprios interesses, lho e mãe/filho que existiam durante a união dos pais. Ao
passando por cima de todos, se preciso for, para conseguir considerarmos que a família vivencia o contexto que in-
alcançar seus objetivos. As decisões sobre pensões e divi- fluencia diretamente nas relações entre seus membros, não
são dos bens sofrem várias oscilações durante esse clima podemos pensar que pelo fato da guarda escolhida ser a
de tensões. Se a mulher tenta tirar o máximo que puder do compartilhada, as relações serão semelhantes às mantidas
homem, este tenta dar o menos possível, enganando sobre antes da separação.
seus reais rendimentos, pagando pensões irrisórias e insu- A Lei 6.515/77, do Divórcio, traz em seu artigo 9º: “no
ficientes. As dificuldades de oficializar a separação conjugal caso da dissolução da sociedade conjugal, pela separação
com todas as divisões que a envolvem - pensão, partilha, consensual (art. 4º) observar-se-á o que os cônjuges acor-
visitas aos filhos, refletem as dificuldades que ambos pos- darem sobre a guarda de filhos”. No artigo 27 a Lei dispõe
suem de se separarem, a necessidade de manter o vínculo, sobre o fato de que “o divórcio não modificará os direitos e
ainda que seja através da briga. deveres em relação aos filhos”.

35
CONHECIMENTOS ESPECÍFICOS - PSICÓLOGO

Também o Estatuto da Criança e do Adolescente (ECA) que ficam no meio da linha de fogo, que são usadas nesse
- Lei 8.069/90, traz dispositivos que convergem com con- momento de tensão. Essa é a expressão do ódio, junto com
cessão da guarda compartilhada: a competição pelo afeto dos filhos. Há a necessidade de
mostrar ao outro que é maior. Como consequência desses
Art. 19 – Toda criança ou adolescente tem direito de ser fatores, os filhos podem carregar consigo problemas que
criado e educado no seio de sua família [...]. vão desde o comprometimento da autoestima, até a visão
Art. 27 – Aos pais incumbe o dever de sustento, guarda ruim das imagens do pai e da mãe.
e educação dos filhos menores, cabendo-lhes ainda, no in-
teresse destes, a obrigação de cumprir e fazer cumprir das É pior ainda quando uma das partes encontra novo
determinações judiciais. relacionamento. A parte que está só tem a tendência de
denegrir mais ainda o ex-cônjuge e também o seu parcei-
A guarda compartilhada, apesar de todas as dificulda- ro, transmitindo diretamente aos filhos toda essa revolta,
des para ser de fato efetivada, vem oferecer um meio de colocando-os como aliados fiéis para não deixarem o outro
efetivação do poder familiar, ainda que a separação conju- permanecer com tal relacionamento. Notamos que muitas
gal tenha se consumado. vezes o desejo de que os pais tornem a viverem juntos é
Com o tempo, pode ser que o homem constitua uma o desejo embutido de os ver mais frequentemente. É in-
nova família, e a tendência é a tentativa de diminuição do teressante ressaltar que não é somente a separação em si
valor da pensão ou o distanciamento dos filhos, devido ao que é traumática, e nem sempre ela é tida como tal; é ne-
fato de seus gastos aumentarem. cessário que os filhos sintam que pai e mãe, mesmo sem
Muitas vezes, principalmente quando se trata de fa- viverem juntos, continuam assumindo a responsabilidade
mílias economicamente necessitadas, o processo de se- de cuidarem deles com amor e atenção. A questão das
paração, tão lento e gradual quanto todos os outros, não visitas é tema bastante complicado: por um lado, alguns
encontra bens para partilhar e dividir. Nesse caso, os filhos pais não visitam frequentemente os filhos para vingar, de
passam a ser alvos de brigas, seja pelo motivo da recusa ao certa forma, a ex-esposa. Em outros casos, é a mulher que,
pagamento de pensão alimentícia, seja pela solicitação da por ressentimento, cria inúmeras maneiras de impedir o
guarda, pelas visitas. contato entre o pai e os filhos, proibindo a aproximação
Pode ser que algumas pessoas pensem que separar dos mesmos, ainda que esteja resguardado esse direito nas
do cônjuge significa separar também da família – incluin- cláusulas da separação. Diante dessa guerra de fogo, a dor
do filhos, sogros, cunhados, tios, dentre outros. Essa visão dos filhos é grande: por um lado, sentem-se confusos com
pequena de separação pode levar a muitos desencontros, relação ao desejo de estar com os pais e ao mesmo tempo
desavenças e também pode gerar nos filhos um sentimen- desejam ser leais e amorosos com a mãe, que, muitas ve-
to de vazio, de desprezo dos pais. Separar-se do cônjuge zes, sofre com a traição. Esse sentimento é muito ruim para
não significa separar-se de tudo o que ligava os dois, dos as crianças.
A questão é a seguinte: do cônjuge é possível se sepa-
amigos, parentes. Pode ser que isso aconteça justamente
rar quando a convivência se torna insuportável ou quando
pelo fato de que desligando cada vez mais das pessoas
se chega à conclusão de que a pessoa com quem pensou
que faziam parte do cotidiano do casal, a separação possa
em viver o resto de sua vida não é a ideal. Entre pais e fi-
ser amenizada pelo distanciamento. Mas, é certo que esse
lhos, porém, a questão é diferente, mesmo quando não se
não é o melhor caminho para superar e enfrentar a realida-
tem os pais ou os filhos que gostariam de ter. O processo
de. Se essa for a forma escolhida para a desvinculação do
de desvinculação é mais difícil, ou até impossível.
outro, certamente eles vão acabar chegando frente a frente
Maldonado (2000, p. 253), coloca que:
com a realidade e assim poderá ser mais difícil conseguir
[...] cada grande transição da nossa vida traz uma revi-
superar esse desafio. Se as incompatibilidades conjugais
são de valores e de metas existenciais que, às vezes, entram
são irreversíveis, os problemas são constantes na vida a
em choque com antigas crenças e posturas e, sobretudo,
dois e o casal vive num clima de tensão constante, com com valores transmitidos nas duas primeiras décadas de
opressão, mal-estar, esse ambiente é extremamente pesa- nossa vida pela família, pela escola e pela Igreja.
do tanto para o casal quanto para os filhos. Assim, em mui- É muito importante refletir sobre esse momento da
tos casos, a separação representa alívio ao invés de trauma. vida, sobre a forma de se viver, ou mudar o rumo da vida,
Na medida em que a separação ganha espaço social descobrindo lições únicas que precisam ser aprendidas.
de validação, como alternativa de vida viável, sendo me- Sempre existiram e sempre existirão problemas e di-
lhor do que arrastar um casamento destrutivo, as situações ficuldades a serem enfrentados pela vida, e é possível en-
traumáticas, impostas pelo próprio contexto social, tendem frentar esses desafios do mundo como sinal de própria in-
a diminuir. serção na sociedade em transformação.
Existem pessoas que mesmo estando separadas con- Podemos afirmar que os casamentos, separações, de-
tinuam casadas pelo desejo de vingança, colocando os suniões que envolvem a história, o próprio contexto social,
filhos como torpedo, envolvendo-os diretamente nas ba- se desencadeiam pela vida das pessoas e têm repercussões
talhas. Pais se denigrem mutuamente na frente dos filhos,

36
CONHECIMENTOS ESPECÍFICOS - PSICÓLOGO

no cotidiano de cada um. A construção desse processo his- D) CASAMENTO NUNCUPATIVO é o celebrado pe-
tórico depende da forma de posicionamento de cada um, los próprios cônjuges, na presença de 06 testemunhas que
do seu passado, do seu presente e também das escolhas ao não sejam parentes em linha reta ou colateral, até segundo
longo da vida, como sonhos para o futuro. Essa é a respon- grau. A autorização legal de dispensa das formalidades de-
sabilidade que se leva na dinâmica das passagens da vida. corre da possibilidade de morte iminente de um dos con-
traentes.
E) CASAMENTO POR PROCURAÇÃO é o realizado
Bibliografia
entre um dos cônjuges e um procurador do outro, devida-
OLIVEIRA, N. H. D. Recomeçar: família, filhos e desafios. mente constituído por instrumento público.
Franca: UNESP, 2009
- São NULOS os casamentos realizados:
DIREITO DE FAMÍLIA a) com o enfermo mental sem discernimento para
prática de atos civis;
DO CASAMENTO (art. 1.511 a 1.590) b) entre ascendente e descendente, seja o parentes-
co civil ou natural;
O casamento é uma união legal entre homem e mulher c) entre irmãos, unilaterais ou bilaterais e entre cola-
terais, até 3º grau inclusive;
com a finalidade de constituição de uma família e basea-
d) entre afins em linha reta (ex: sogro e nora);
da na igualdade de direitos e deveres dos cônjuges. Para e) entre adotante com o ex-cônjuge do adotado e
sua realização é necessário o cumprimento de formalidades entre adotado com ex-cônjuge do adotante;
legais, através de um processo de habilitação, no qual se f) o adotado com o filho do adotante;
verifica a existência de impedimentos. O processo de habili- g) com pessoa casada;
tação corre perante o Oficial do Registro Civil e compreende h) entre um cônjuge e o homicida de seu consorte
quatro etapas: documentação, proclamas (fixação do edi- (ainda que o crime tenha sido apenas tentado).
tal), certificado (de habilitação) e registro (dos proclamas).
- São anuláveis os casamentos:
Com o certificado de habilitação, os noivos poderão reque-
a) realizados por quem não completou a idade míni-
rer a designação de data para a realização da cerimônia. O ma para casar (idade mínima: homens e mulheres 16 anos
casamento se realiza quando o casal manifesta sua vontade com autorização dos pais ou representantes legais);
perante um juiz de paz e produz efeitos a partir da data b) do menor em idade núbil, quando não autorizado
de sua celebração. Nossa legislação consagra o casamento por seu representante legal;
monogâmico. c) do incapaz de consentir ou manifestar o consenti-
Existe também o casamento religioso com efeitos civis mento;
ou registro civil do casamento religioso. O celebrante (mi- d) realizados com vício de vontade (quando há erro
nistro religioso) comunica a ocorrência do casamento reli- essencial quanto à pessoa do outro cônjuge ou quando
realizado em virtude de coação);
gioso ao ofício competente para efetivação do registro civil.
e) realizado pelo mandatário, sem que ele ou o outro
contraente soubessem da revogação do mandato;
A) CASAMENTO NULO é o realizado mesmo com a f) por incompetência da autoridade celebrante.
existência de impedimento causador de nulidade e peran-
te autoridade incompetente (nulidade absoluta). Efeitos Os prazos para interposição da ação de anulação de
ex tunc: retroage até a data do casamento. CASAMENTO casamento contam-se a partir da data de sua celebração e
ANULÁVEL é o realizado mesmo com a existência de impe- são todos decadenciais.
dimentos e com erro essencial quanto à pessoa do cônju- As autorizações concedidas pelos representantes legais
ge (nulidade relativa - vícios de consentimento). Efeitos ex autorizando o casamento de menores podem ser revoga-
das até a data do casamento e, caso a revogação baseie- se
nunc: os efeitos a partir da sentença que declara a nulidade
em motivo injusto, o Juiz poderá supri-la.
do ato. O casamento contraído de boa-fé por ambos os côn-
B) CASAMENTO INEXISTENTE é aquele em que o ato juges, ainda que seja nulo ou anulável, produzirá todos os
foi praticado com defeito tão notório e grave que dispen- efeitos em relação aos consortes e aos filhos, até o dia da
sa a declaração judicial de nulidade. (ex: casamento entre sentença anulatória. Se apenas um dos cônjuges estava de
pessoas do mesmo sexo, decretação do casamento sem o boa-fé, apenas a este e aos filhos os efeitos aproveitarão.
consentimento de um dos cônjuges, casamento sem habi- Ainda que celebrado de má-fé por ambos, o casamento
litação prévia). produzirá todos os efeitos civis com relação aos filhos.
Existem as causas suspensivas do casamento que de-
C) CASAMENTO PUTATIVO é o casamento nulo ou
vem ser supridas para a realização do matrimônio (ex: reali-
anulável que, contraído de boa-fé por pelo menos um dos zação da partilha de bens do casal divorciado para que eles
cônjuges, produzirá efeitos civis (mesmos efeitos de casa- possam contrair novo matrimônio; realização de inventário
mento válido) em relação aos cônjuges e aos filhos. O ca- e partilha de bens entre a viúva e os herdeiros). Os nuben-
samento foi realizado na suposição de estarem cumpridas tes podem solicitar judicialmente que não lhes sejam apli-
todas as exigências legais. (ex: casamento entre irmãos por cadas as causas suspensivas, devendo provar a inexistência
ignorância do parentesco). de prejuízo.

37
CONHECIMENTOS ESPECÍFICOS - PSICÓLOGO

São EFEITOS do casamento na esfera pessoal: o dever DIVÓRCIO


de fidelidade recíproca; o direito de o cônjuge acrescer ao
seu nome o do outro; convívio no domicílio conjugal; pla- O Divórcio põe termo ao casamento e aos efeitos civis
nejamento familiar para constituição de uma família; as- do casamento religioso. Depois de divorciados os cônjuges
sistência mútua e dever de sustento, guarda e educação só poderão se unir novamente com a celebração de novo
da prole. Na esfera patrimonial podem ser citados os se- casamento.
guintes efeitos: assistência financeira ao cônjuge e à prole; Existem dois tipos de divórcio:
direitos sucessórios e direito real de habitação do cônjuge • Divórcio indireto ou por conversão: é o requerido por
viúvo.
um ou ambos os cônjuges após 01 (um) ano da decretação
O matrimônio se extingue pela nulidade, morte ou di-
vórcio. A sociedade conjugal termina também pela sepa- da separação judicial. Em princípio são mantidas as cláusu-
ração. las e condições da separação.
• Divórcio direto: é o requerido por um ou ambos os
DA SEPARAÇÃO E DO DIVÓRCIO cônjuges e baseia-se na separação de fato do casal há mais
de 02 (dois) anos. Este divórcio não é precedido de ação
SEPARAÇÃO de separação.
Em qualquer dos tipos, no caso de o divórcio ser liti-
A SEPARAÇÃO não dissolve o vínculo matrimonial, mas gioso, a outra parte poderá alegar na contestação a falta de
põe fim aos deveres conjugais e ao regime de bens do ca- decurso de prazo e, no por conversão, o descumprimento
samento. dos termos da separação.
Pode ser consensual ou litigiosa. Existe a possibilidade de realização de divórcios e se-
A separação consensual só pode ser requerida após o parações consensuais diretamente nos cartórios de notas,
lapso temporal de 02 anos da data do casamento e da pe-
dispensando-se a necessidade de homologação judicial.
tição constará todo o pacto acordado pelos cônjuges. O
Para tanto, devem ser cumpridos os seguintes requisitos:
Juiz sempre tentará a reconciliação dos cônjuges. Caso não
seja possível um consenso quanto à partilha de bens, esta a) requisito temporal idêntico ao processo judicial (prazo
poderá ser feita posteriormente. tanto para separação quanto para divórcio direto ou por
A separação litigiosa pode ser requerida por um dos conversão); b) o casal não pode ter filhos menores ou in-
cônjuges, a qualquer tempo, e se baseará em uma das três capazes. Da escritura pública constarão as disposições re-
hipóteses: ferentes a descrição e partilha de bens, pensão alimentícia
• Separação-sansão - Imputação de conduta desonro- e à manutenção do nome de casado, constituindo-se em
sa ou qualquer outro ato que importe grave violação dos título hábil para o registro civil e de imóveis.
deveres conjugais, tornando insuportável a vida em co-
mum. (ex: abandono do lar, infidelidade). REGIME DE BENS (art. 1.639 a 1.688)
• Separação-falência - Separação de fato do casal há
mais de 01 (um) ano. Neste caso não há alegação de culpa O Regime de Bens do casamento inicia sua vigência na
do outro cônjuge e a prova a ser produzida restringe-se ao
data deste, sendo possível sua alteração, mediante autori-
lapso temporal. Nesta hipótese os filhos do casal permane-
zação judicial em pedido motivado, desde que não traga
cerão sob a guarda do que já estavam.
• Separação-remédio - Pedido baseado no fato de es- prejuízo a terceiros.
tar o outro cônjuge acometido de grave doença mental, O Regime de bens dito oficial ou legal e vigente é o da
manifestada após o casamento, de cura improvável e de comunhão parcial, podendo ser alterado através de pacto
duração superior a 5 anos. A separação não será decreta- antenupcial realizado através de escritura pública.
da se ficar provado que a separação poderá causar grande
inconveniente ao cônjuge doente. No caso de decretação A) Regime da Comunhão total ou universal de bens
da separação, o cônjuge requerente fica obrigado a prestar Há a comunicação de todos os bens presentes e futu-
alimentos ao doente. ros dos cônjuges, ou seja, dos bens adquiridos antes ou de-
pois do casamento pelos cônjuges e das dívidas passivas.
OBS: Mesmo após a decretação da Separação Judicial, São excluídos da comunhão: os bens doados ou herdados
a sociedade conjugal poderá ser restabelecida, a qualquer gravados com cláusula de incomunicabilidade e os sub-
tempo, nos termos em que fora constituída, mediante sim- -rogados em seu lugar; os bens gravados de fideicomisso
ples requerimento dirigido ao juízo da separação.
e o direito do herdeiro fideicomissário, antes de realizada
SEPARAÇÃO DE CORPOS: medida cautelar que pode a condição suspensiva; as dívidas anteriores ao casamen-
ser requerida ao Juiz para que um dos cônjuges deixe o to, salvo se provierem de despesas com seus aprestos, ou
lar conjugal, sendo sempre baseada em fatos graves que reverterem em proveito comum; as doações antenupciais
impliquem na impossibilidade da vida em comum (ex: feitas por um dos cônjuges ao outro com cláusula de in-
agressões físicas). No caso de ser decretada a separação comunicabilidade; os bens de uso pessoal, os livros e ins-
de corpos do casal, o prazo de 01 ano para interposição do trumentos de profissão; os proventos do trabalho pessoal
pedido de conversão da separação em divórcio será conta- de cada cônjuge; as pensões, meios-soldos, montepios e
do da decretação. outras rendas semelhantes.

38
CONHECIMENTOS ESPECÍFICOS - PSICÓLOGO

B) Regime da Comunhão Parcial de bens Já o concubinato impuro caracteriza-se também pela


união de duas pessoas sem a celebração do casamento,
Como regra geral há a comunicação apenas dos bens no entanto nestes casos estão as mesmas impedidas de se
adquiridos na constância do casamento por ato oneroso unirem por impedimentos públicos (incesto, bigamia e ho-
ou por fato eventual (ex: prêmios de loteria), ficando ex- micídio). Neste caso os conviventes não possuem proteção
cluídos os bens já pertencentes a cada um dos cônjuges, legal, a não ser quanto à pessoa dos filhos, se houverem.
os recebidos por doação ou herança ou em sub-rogação Quanto ao regime de bens, pode-se dizer que se equi-
de bens particulares. Há presunção de que todos os bens vale ao da comunhão parcial, comunicando-se apenas os
móveis foram adquiridos na constância do matrimônio, bens adquiridos, a título oneroso, na constância da união,
admitindo-se prova em contrário. salvo estipulação contratual contrária.
Os conviventes podem ser habilitados como depen-
C) Regime da Separação de bens dentes junto à Previdência Social, poderão utilizar o sobre-
nome do outro e terão direito de requerer pensão alimen-
Por este regime os bens dos cônjuges, adquiridos antes tícia. No caso de falecimento, o cônjuge sobrevivente terá
ou depois do casamento, são incomunicáveis. Para que o direito à herança, direito real de habitação e de usufruto
regime de separação de bens seja pleno deve constar do semelhante ao cônjuge.
pacto antenupcial que os bens, mesmo adquiridos depois Os conviventes poderão requerer ao juiz, a qualquer
do casamento, não se comunicarão, pertencendo a apenas tempo, desde que não haja impedimento, a conversão da
um dos cônjuges. Cada cônjuge permanecerá na adminis- união em casamento, com o devido assento no Registro
tração e fruição de seus bens, podendo aliená-los livre- Civil.
mente, ainda que se trate de bens imóveis. Caso não conste
do pacto tal disposição, os bens adquiridos na constância RELAÇÕES DE PARENTESCO (art. 1.591 a 1.638)
do matrimônio pertencerão a ambos. Este tipo de regime
de bens ocorre também nos casos previstos em lei, sen- Vínculo de Parentesco é a relação entre pessoas per-
do denominado separação legal, e obrigatório em casos tencentes a um mesmo grupo familiar.
como o casamento do maior de 60 e da maior de 50 anos Parentesco legítimo é aquele que decorre de um casa-
(salvo se já viverem juntos há mais de 10 anos), dos que mento e o ilegítimo é o que não decorre, sendo provenien-
dependerem de autorização judicial para casar e do viúvo te do adultério, concubinato puro e do incesto.
ou viúva que tiver filho do cônjuge falecido enquanto não O parentesco consanguíneo ou natural é o que une in-
der partilha aos herdeiros. divíduos descendentes de um mesmo tronco familiar e o
parentesco civil é o que resulta da adoção.
D) Regime de participação final nos aquestos Parentesco por afinidade é o que une uma pessoa e os
parentes de seu cônjuge. O parentesco por afinidade em
Este regime é novo e pode ser considerado misto, já linha reta não se extingue com a dissolução do casamento,
que, durante o casamento aplicam- se as regras do regi- ou seja, continuam as proibições matrimoniais.
me de separação total de bens, e com a sua dissolução Parentes em linha reta são as pessoas interligadas pela
aplicam-se as regras da comunhão parcial. Ou seja, cada relação de ascendentes e descendentes (ex: pai, avô, filho,
cônjuge tem seu próprio patrimônio e só será partilhado neto) e os em linha colateral são os descendentes do mes-
o patrimônio adquirido, a título oneroso, durante o casa- mo tronco, até o 6º grau, sem relação de ascendente e des-
mento, quando da dissolução da sociedade conjugal. No cendente (ex: irmão, tio, sobrinho).
entanto, no caso de venda de imóvel, será necessária a au- O grau de parentesco entre as pessoas é contado, em
torização do outro cônjuge, salvo disposição expressa em linha reta e na colateral, pelo número de gerações, sendo
contrário no pacto antenupcial. Os bens adquiridos antes que neste último caso deve-se contar da pessoa até o as-
do casamento e os que os substituíram, bem como os rece- cendente comum e descer novamente até o parente.
bidos por doação ou herança não se comunicarão. Aques-
tos: patrimônio adquirido, a título oneroso, na constância FILIAÇÃO
do casamento.
Com a promulgação da Constituição Federal/88, fo-
UNIÃO ESTÁVEL (art. 1.723 a 1.727) ram proibidas as classificações discriminatórias dos filhos,
tendo sido igualados e conferidos os mesmos direitos aos
A União Estável entre duas pessoas (homem e mulher) filhos havidos ou não do casamento, ou por adoção.
caracteriza-se pela convivência mútua e pública visando Os irmãos podem ser germanos, quando filhos do
a constituição de uma família, sem a ocorrência de casa- mesmo pai e da mesma mãe, ou unilaterais, quando tive-
mento. A convivência deve ser duradoura e contínua para rem apenas um em comum.
que haja o reconhecimento como entidade familiar, não A paternidade é provada pela certidão de nascimento,
havendo necessidade de coabitação e nem prazo mínimo mas há casos em que a paternidade é presumida, como
de convivência para reconhecimento da união. O Código no caso dos filhos nascidos na constância do matrimônio,
Civil permite a união estável entre pessoas solteiras, viúvas, admitindo-se prova em contrário. Há a possibilidade de o
divorciadas, separadas judicialmente ou separadas de fato. marido contestar a paternidade em ação própria, sendo

39
CONHECIMENTOS ESPECÍFICOS - PSICÓLOGO

imprescritível tal direito. No que pertine à paternidade é dade familiar destinado para moradia permanente. Apenas
possível ainda a interposição de uma ação que tem por fim um imóvel é considerado bem de família e seu reconheci-
negar a paternidade, podendo ser esta intentada pelo filho mento como tal independe de registro junto ao Registro
ou por qualquer outra pessoa interessada que possa ser de Imóveis. Os bens que guarnecem a residência também
beneficiada. são impenhoráveis, salvo os adornos suntuosos e obras de
A prova da maternidade é bem mais fácil e pode se arte. Essa impenhorabilidade pode ser oposta em qualquer
dar de diversas maneiras, como a gravidez ou o registro do processo de execução civil, fiscal, previdenciária, trabalhista
parto. A ação de contestação de maternidade poderá ser ou de outra natureza, saldo as seguintes exceções de
intentada também a qualquer tempo e por interessado que processo: para cobrança de créditos trabalhistas ou pre-
possa se beneficiar com o resultado. videnciários de trabalhadores da própria residência; para
A ação de investigação de maternidade ou paternida- cobrança de dívida decorrente do financiamento destinado
de é privativa do filho ou de seu representante legal, no à aquisição ou construção do imóvel; para cobrança de
caso de incapaz. No caso de filho maior, só este poderá pensão alimentícia; cobrança de impostos prediais ou terri-
ingressar com a ação. Na sentença que julgar a paternida- toriais, bem como taxas e contribuições devidas em decor-
de serão fixados alimentos, quando devidos. Esta fixação rência da propriedade do bem; para execução de hipoteca;
poderá ocorrer também por concessão de liminar, com a por ter sido o imóvel adquirido com produto de crime; para
fixação dos alimentos provisórios, no caso de haver prova cobrança de obrigação decorrente de fiança locatícia.
documental do parentesco. No caso de o casal ou entidade familiar possuir mais
No caso de a criança ser registrada só pela mãe, deve de um imóvel destinado à moradia, a impenhorabilidade
o Oficial do Registro enviar ao Juiz os dados do suposto recairá sobre o de menor valor, salvo se outro estiver re-
pai, para que ocorra uma verificação extraoficial sobre o gistrado como bem de família nos termos do Código Civil.
assunto. A mãe e o suposto pai serão ouvidos e no caso
de restar confirmada a paternidade, será lavrado termo de Observação: Os artigos podem ser conferidos e es-
reconhecimento. No caso de o suposto pai não comparecer tudados no código civil, Lei no 10.406, de 10 de janeiro
ou, comparecendo, negar a paternidade, o Juiz remeterá os de 2002, no site do Planalto. (http://www.planalto.gov.
autos ao Ministério Público para que ingresse com a neces- br/ccivil_03/leis/2002/L10406.htm)
sária Ação de Investigação de Paternidade.
GUARDA DOS FILHOS
O reconhecimento dos filhos poderá ser feito a qual-
quer momento, independente de serem havidos dentro
Poder familiar
ou fora do casamento e do estado civil de quem os reco-
nhece. O reconhecimento poderá ser realizado através de
Poder familiar refere-se à somatória de direitos e obri-
registro de nascimento, escritura pública, escrito particular,
gações assumidos pelos pais e responsáveis legais, instituí-
testamento ou verbalmente perante o Juiz, sendo que o
dos para a proteção dos interesses de subsistência, saúde,
reconhecimento de filho maior dependerá de seu consen-
educação e criação dos filhos. É um poder-dever indisponí-
timento. vel, irrenunciável, não é passível de transação e é imprescri-
tível (MESSEDER, 2010).
BEM DE FAMÍLIA (art. 1.711 a 1.722) Essa expressão passou a ser utilizada a partir do Códi-
go Civil (CC) promulgado em 2002 em substituição a pá-
O bem de família pode ser instituído pelos cônjuges ou trio poder, como constava no Código Civil de 1916. Em sua
entidade familiar, através de escritura pública ou testamen- origem, o pátrio poder chegava ao extremo de o pai ter o
to e recairá sobre prédio urbano ou rural. O terceiro poderá poder absoluto sobre o filho, inclusive de morte (FONSECA,
instituir bem de família, por testamento ou doação. Através 2004). A expressão poder familiar visou abranger o conjun-
deste ato, destina- se parte do patrimônio, que não pode to de direitos e deveres de mãe e pai sobre filhos, em con-
ultrapassar 1/3 (um terço) do total líquido, para constituir dições de igualdade. Entretanto, segundo o autor citado,
o bem de família. ainda é passível de críticas, porque continua enfatizando
Neste caso serão mantidas as regras de impenhora- o poder e sugerindo algo diferente do que exprime, pois,
bilidade do bem de família, lembrando que são também a terminologia “poder familiar” não é um poder da família,
impenhoráveis os bens que guarnecem a residência (per- mas uma obrigação dos pais (ibid.). Vale ressaltar que al-
tenças e acessórios destinados ao domicílio familiar). guns estudos têm sugerido o uso de “poder parental”, por
O bem de família do Código Civil tem que ser regis- considerá-lo mais abrangente.
trado no Registro de Imóveis e é isento de execução por Os filhos estão sujeitos ao poder familiar em qualquer
dívidas posteriores à sua instituição, salvo as decorrentes natureza da filiação. Se nascidos fora do casamento, ou da
de tributos decorrentes da propriedade do prédio ou de união estável, precisam ser legalmente reconhecidos, ou
despesas de condomínio. adotados para se estabelecer o parentesco. Se houver ape-
A dissolução da sociedade conjugal não extingue o nas a maternidade estabelecida, o poder familiar será exer-
bem de família. cido somente pela mãe. Também na falta de um dos pais
Existe ainda em nossa legislação o Bem de Família ins- – por morte, desaparecimento, ausência ou impedimento
tituído pela Lei 8009/90 que dispõe acerca da impenhora- por doença grave –, o outro exercerá o poder familiar com
bilidade do imóvel residencial próprio do casal ou da enti- exclusividade (FONSECA, 2004).

40
CONHECIMENTOS ESPECÍFICOS - PSICÓLOGO

O poder familiar, portanto, não decorre do casamento social das pessoas envolvidas: a situação dos pais e mães
ou da união estável, mas da paternidade, da maternidade e que abandonam ou entregam seus filhos a outras pessoas
da filiação, seja biológica ou civil. Assim, os direitos e deve- para serem criados e o que esta desistência representa para
res de pai e de mãe continuam em condições de igualdade eles; o momento de vida que atravessam para que isso
nos casos de separação judicial ou não, levando em conta o ocorra – que pode envolver grande pressão e sofrimento
que é acordado ou determinado judicialmente em relação emocional, fragilidades de ordem material, falta ou difi-
à guarda dos filhos. culdades de acesso a serviços de proteção social, ou ain-
da (ou cumulativamente) a decisão pela entrega a outros,
Destituição do poder familiar num ato de afeto, por acreditarem que a criança será mais
bem cuidada (FÁVERO, 2007). Como a presente pesquisa
A destituição do poder familiar é uma sanção grave im- constata (conforme dados analisados no tópico III), muitas
posta aos pais quando violam as obrigações inerentes ao vezes as condições de degradação humana, no interior de
poder familiar, observando-se sempre o que for de melhor um processo de apartação social e de violação de direitos
interesse da criança. É uma medida extrema e permanente, de adultos, podem condicionar a violação de direitos dos
e deve ser decretada apenas após avaliação da intensida- filhos pelos pais, exigindo a intervenção do Estado para a
de, da frequência, das circunstâncias e das consequências proteção imediata da criança – o que nem sempre é acom-
dos atos de violação de direitos – no seu sentido amplo –, panhado de ações de proteção social à família.
impostos pelos pais aos filhos menores de 18 anos. Sua de-
terminação se dá por sentença judicial, devendo ser asse- Colocação em família substituta
gurado aos pais o direito ao contraditório e à ampla defesa.
É providência necessária e prévia para a medida de adoção. A colocação em família substituta deve ser uma me-
(FONSECA, 2004). dida extrema, quando esgotadas todas as tentativas de
As causas de destituição do poder familiar são enume- manutenção da convivência da criança e do adolescente
radas no art. 1.638 do atual Código Civil, e assim descritas: em sua família biológica ou extensa. De forma ainda mais
I – castigar imoderadamente o filho; excepcional deve ocorrer a colocação em família substituta
II – deixar o filho em abandono; estrangeira, em razão da ruptura definitiva que acontece
III – praticar atos contrários à moral e aos bons costu- tanto dos vínculos familiares, de afinidade e afetividade,
mes; como com o meio social, cultural e linguístico, impondo a
IV – incidir, reiteradamente, nas faltas previstas no arti- privação, muitas vezes definitiva, de qualquer contato da
go antecedente.
criança ou do adolescente com sua família biológica (FA-
O Estatuto da Criança e do Adolescente declara a crian-
CHINETTO, 2009).
ça e o adolescente como sujeitos de direitos, devendo ser
A colocação em família substituta pode ocorrer através
garantido a eles que se desenvolvam livremente e em meio
da tutela, guarda e adoção (ECA, art. 28). Pode ser com-
aberto, no convívio com a família natural ou, como último
preendida como uma medida de proteção e garantia do
recurso, com a família substituta – em detrimento da insti-
direito à convivência familiar.
tucionalização.
Em relação à destituição (medida definitiva) ou suspen-
Guarda
são (medida temporária) do poder familiar, o ECA dispõe
em seu artigo 24: “A perda e a suspensão do poder familiar
serão decretadas judicialmente, em procedimento contra- Trata-se da obrigação de prestar assistência material,
ditório, nos casos previstos na legislação civil, bem como moral e educacional à criança ou adolescente, conferindo
na hipótese de descumprimento injustificado dos deveres o direito, a quem a detém, de opor-se a terceiros, inclu-
e obrigação a que alude o art. 22”. sive aos pais (ECA, art. 33). A guarda pode ser provisória
Embora o art. 22 do ECA disponha que é dever dos ou definitiva, de acordo com o pedido e com a situação
pais sustentar e educar os filhos menores de 18 anos, o específica (MESSEDER, 2010). Ressalte-se que não é pré-
art. 23 deixa claro que a falta ou a carência de recursos -requisito para a guarda a perda do poder familiar, sendo
materiais não constituem motivo para a perda ou suspen- que os pais o mantêm enquanto um terceiro tenha a guar-
são do poder familiar17. De acordo com o ECA, o cuidado da da criança ou adolescente, ocorrendo geralmente, neste
e a responsabilidade com a criança e com o adolescente caso, a suspensão do poder familiar. Nesse sentido, vale
devem ser compartilhados pela família, pela sociedade e observar que a determinação da perda do poder familiar
pelo Estado, o que implica que, se os pais têm dificuldades de pais de crianças que estão em acolhimento institucional
econômicas/materiais para cuidar de seus filhos, é dever ou sob guarda de outra família sem que esteja inserida em
do poder público prestar assistência à família, garantindo a processo de adoção é uma medida que causa maiores da-
não destituição do poder familiar em razão da situação de nos à criança, pois implica, entre outros, a perda da filiação
pobreza em que vive. civil, sem que ela tenha a oportunidade e o direito de nova
Como a destituição do poder familiar é uma decisão filiação.
judicial definitiva na vida das pessoas envolvidas, sua apli- Por considerar que a guarda é uma forma de assegurar
cação exige muitos cuidados. Nesse sentido, podem ser ci- o direito à convivência familiar e comunitária, evitando a
tados alguns aspectos, em especial referentes à realidade institucionalização da criança ou do adolescente, o

41
CONHECIMENTOS ESPECÍFICOS - PSICÓLOGO

ECA (art. 34) prevê que “O poder público estimulará, é aquela em que os pais biológicos podem acompanhar
por meio de assistência jurídica, incentivos fiscais e subsí- o desenvolvimento da criança adotada por meio de fotos,
dios, o acolhimento, sob a forma de guarda, de criança ou relatos ou mesmo contato direto, ou outra forma acordada
adolescente afastado do convívio familiar”. entre eles e os pais adotivos.
- Adoção “à brasileira”: ocorre quando “filhos bioló-
Tutela gicos de determinados pais são registrados como filhos
legítimos de outros pais, como se estes fossem seus pais
A tutela refere-se ao poder instituído judicialmente a biológicos” (GUEIROS, 2007, p. 27). Essa prática, que é ca-
um adulto para ser o representante legal da criança ou do racterizada como crime pelo Código Penal, impede qual-
adolescente menor de 18 anos de idade e gerir sua vida e quer processo de avaliação quanto à adequação daqueles
administrar seus bens na falta dos pais devido à destituição que registram a criança no que se refere à capacidade e
do poder familiar ou a falecimento (AMB, s/d). disponibilidade para atender suas necessidades e direitos,
Segundo Messeder (2010), no caso de morte dos pais, bem como pode encobrir “comércio” e “tráfico” de crianças.
o tutor poderá ter sido nomeado por eles em testamento.
Caso não o tenha sido, será nomeado tutor um parente Bibliografia
consanguíneo e, na ausência de tutor testamentário ou le- FÁVERO, Eunice Teresinha relatório final – pesquisa II
gítimo (parentes), a autoridade judicial nomeará um tutor realidade social, direitos e perda do poder familiar: despro-
residente no mesmo domicílio da criança. teção social x direito à convivência familiar e comunitária.
FÁVERO, Eunice Teresinha. Questão Social e perda do
Adoção Poder Familiar. São Paulo: Veras Editora

A adoção é uma medida excepcional e irrevogável. Se- ESTATUTO DA CRIANÇA E DO ADOLESCENTE


gundo o art. 41 do ECA, “ela atribui a condição de filho (LEI nº 8.069)
ao adotado, com os mesmos direitos e deveres, inclusive
sucessórios, desligando-o de qualquer vínculo com pais e Subseção II
parentes, salvo impedimentos matrimoniais”. Da Guarda
No Brasil, as primeiras legislações sobre a adoção sur-
Art. 33. A guarda obriga a prestação de assistência
giram apenas no século XX. Nas últimas décadas, mudan-
material, moral e educacional à criança ou adolescente,
ças legais permitiram o reconhecimento de crianças e ado-
conferindo a seu detentor o direito de opor-se a terceiros,
lescentes como sujeitos de direitos e a adoção passou a
inclusive aos pais.
figurar como um instrumento capaz de garantir, em casos
§ 1º A guarda destina-se a regularizar a posse de fato,
excepcionais, o direito à convivência familiar e comunitária.
podendo ser deferida, liminar ou incidentalmente, nos
De acordo com o ECA, cabe ao juiz, ouvido o Ministério
procedimentos de tutela e adoção, exceto no de adoção
Público, a colocação da criança ou adolescente em adoção, por estrangeiros.
obedecendo ao Cadastro Nacional de Adoção, que reúne § 2º Excepcionalmente, deferir-se-á a guarda, fora dos
os interessados em adotar, devidamente inscritos e avalia- casos de tutela e adoção, para atender a situações peculiares
dos como aptos para tal. Entretanto, existem situações em ou suprir a falta eventual dos pais ou responsável, podendo
que a adoção ocorre sem a indicação de adotantes inscri- ser deferido o direito de representação para a prática de
tos no Cadastro, como é o caso da “adoção pronta”, e mes- atos determinados.
mo sem a intermediação da Justiça, a chamada “adoção à § 3º A guarda confere à criança ou adolescente a
brasileira”. condição de dependente, para todos os fins e efeitos de
- Adoção pronta: ocorre quando, geralmente, a mãe ou direito, inclusive previdenciários.
os pais biológicos escolhem para quem desejam entregar § 4º Salvo expressa e fundamentada determinação
seu filho. O ato de definir a quem entregar o filho é chama- em contrário, da autoridade judiciária competente, ou
do de “intuito personae”. Em muitos casos, o pretendente à quando a medida for aplicada em preparação para adoção,
adoção, com anuência da família biológica da criança, pro- o deferimento da guarda de criança ou adolescente a
cura a Vara da Infância e da Juventude para legalizar uma terceiros não impede o exercício do direito de visitas
convivência que já está acontecendo de fato (AMB, s/d). pelos pais, assim como o dever de prestar alimentos, que
A prática da escolha da família adotiva pela família serão objeto de regulamentação específica, a pedido do
biológica não é considerada legal no Brasil, e buscou-se interessado ou do Ministério Público.
coibi-la com a introdução do parágrafo único ao art. 13 do
ECA (pela Lei n. 12.010, de 2009): “Parágrafo único. As ges- Art. 34. O poder público estimulará, por meio de assis-
tantes ou mães que manifestem interesse em entregar seus tência jurídica, incentivos fiscais e subsídios, o acolhimento,
filhos para adoção serão obrigatoriamente encaminhadas à sob a forma de guarda, de criança ou adolescente afastado
Justiça da Infância e da Juventude”. do convívio familiar.
Embora a lei tenha buscado disciplinar essa prática, ela § 1º A inclusão da criança ou adolescente em programas
continua alvo de estudos, debates e polêmicas, existindo de acolhimento familiar terá preferência a seu acolhimento
defensores da modalidade de “adoção aberta” ou “flexi- institucional, observado, em qualquer caso, o caráter
bilizada” (na qual a “adoção pronta” pode se inserir), que temporário e excepcional da medida, nos termos desta Lei.

42
CONHECIMENTOS ESPECÍFICOS - PSICÓLOGO

§ 2º Na hipótese do § 1o deste artigo a pessoa ou Tomamos este conceito como parâmetro para a análise
casal cadastrado no programa de acolhimento familiar da temática em questão, no intuito de pensar as múltiplas
poderá receber a criança ou adolescente mediante guarda, faces da violência que podem envolver uma criança e/ou
observado o disposto nos arts. 28 a 33 desta Lei. adolescente, em especial quando vítima de abuso sexual.
§ 3º A União apoiará a implementação de serviços Para tratar especificamente da violência doméstica ou
de acolhimento em família acolhedora como política violência intrafamiliar, reportamo-nos a Azevedo Guerra,
pública, os quais deverão dispor de equipe que organize Deslandes, Faleiros, Souza, reconhecidos estudiosos e par-
o acolhimento temporário de crianças e de adolescentes ticipantes de movimentos que visam enfrentar a questão,
em residências de famílias selecionadas, capacitadas e bem como à abordagem dada pelo Plano Nacional de Pro-
acompanhadas que não estejam no cadastro de adoção. moção, Proteção e Defesa do Direito de Crianças e Adoles-
(Incluído pela Lei nº 13.257, de 2016) centes à Convivência Familiar e Comunitária (PNCFC).
§ 4º Poderão ser utilizados recursos federais, estaduais, Azevedo Guerra, com base no paradigma da prote-
distritais e municipais para a manutenção dos serviços ção integral, define a violência doméstica contra crianças
de acolhimento em família acolhedora, facultando-se
e adolescentes como “Todo ato ou omissão praticado por
o repasse de recursos para a própria família acolhedora.
pais, parentes ou responsáveis contra crianças e adoles-
(Incluído pela Lei nº 13.257, de 2016)
centes que – sendo capaz de causar dano físico, sexual e/
ou psicológico à vítima – implica, de um lado, numa trans-
Art. 35. A guarda poderá ser revogada a qualquer tem-
po, mediante ato judicial fundamentado, ouvido o Minis- gressão de poder/dever de proteção do adulto e, de outro,
tério Público. numa coisificação da infância, isto é, numa negação do di-
reito que crianças e adolescentes têm de ser tratados como
VIOLÊNCIA DOMÉSTICA/ABUSO SEXUAL CONTRA sujeitos e pessoas em condições peculiares de desenvolvi-
CRIANÇAS E ADOLESCENTES mento” (1998, p. 32).
A autora aponta ainda que a violência doméstica tem
Eunice Teresinha Fávero relação com a violência estrutural presente em uma socie-
dade desigual. Todavia, considera que “tem outros deter-
O objetivo não é aprofundar o tema da violência/abuso minantes que não apenas os estruturais. É um tipo de vio-
sexual, mas apresentar elementos importantes que possibi- lência que permeia todas as classes sociais como violência
litem o entendimento da dinâmica do abuso e da dinâmica de natureza interpessoal ”.
da denúncia, bem como os trâmites da fase processual pe- Deslandes, em trabalho que aborda a prevenção da
nal, de maneira a possibilitar a compreensão da proposta, violência, relaciona tal fenômeno especialmente com a cul-
e refletir sobre a atuação/participação do assistente social tura, apontando que “os maus-tratos contra a criança e o
nessa “metodologia”. adolescente podem ser praticados pela omissão, supressão
Conforme Chauí (1998), em análise sobre Ética e Vio- e transgressão dos seus direitos, então definidos por con-
lência, etimologicamente, violência vem do latim vis, força, venções legais ou normas culturais. A definição do que seja
e significa : “Tudo o que age usando a força para ir contra uma prática abusiva passa sempre por uma negociação en-
a natureza de algum ser (...); todo ato de força contra a es- tre a cultura, a ciência e os movimentos sociais” (1994).
pontaneidade, a vontade e a liberdade de alguém (é coagir, Os maus-tratos e/ou a violência contra crianças e ado-
constranger, torturar, brutalizar); todo ato de violação da lescentes acontecem privilegiadamente no âmbito do am-
natureza de alguém ou de alguma coisa valorizada positi- biente familiar, todavia, também em ambiente extrafamiliar,
vamente por uma sociedade; todo ato de transgressão con- em especial a violência sexual. A violência contra crianças
tra aquelas coisas e ações que alguém ou uma sociedade
e adolescentes se materializa de diversas maneiras, sen-
define como justas e como um direito. Consequentemente,
do geralmente identificada por meio da violência física,
violência é um ato de brutalidade, sevícia e abuso físico e/
psicológica, sexual, negligência e, muitas vezes, reunindo
ou psíquico contra alguém e caracteriza relações intersub-
todas essas expressões. Vários estudos demonstram que
jetivas e sociais definidas pela opressão, intimidação, pelo
medo e pelo terror. A violência se opõe à ética porque trata os maus-tratos contra crianças e adolescentes acontecem
seres racionais e sensíveis, dotados de linguagem e de li- desde os primórdios e ao longo da história da humanidade,
berdade como se fossem coisas, isto é, irracionais, insensí- e em todos os segmentos sociais, tendo, em sua gênese, o
veis, mudos, inertes ou passivos. Na medida em que a ética poder absoluto do adulto sobre a criança, dos pais sobre
é inseparável da figura do sujeito racional, voluntário, livre os filhos (especialmente do pai, por personificar na cultura
e responsável, tratá-lo como se fosse desprovido de razão, ocidental patriarcal a chefia da família, à qual filhos e mu-
vontade, liberdade e responsabilidade é tratá-lo não como lheres deviam obediência), ou do mais forte sobre o mais
humano e sim como coisa, fazendo-lhe violência nos cinco fraco.
sentidos em que demos a esta palavra.“ Na contemporaneidade, mesmo com a limitação posta
A definição de Chauí possibilita uma visão ampla e di- pelo Estado aos direitos e poderes dos pais sobre os filhos
nâmica do fenômeno; a violência “(...) se manifesta como em nossa sociedade, a violência contra a criança e o ado-
o exercício da dominação de um ser sobre o outro, e tem lescente tem se manifestado com frequência em índices
como conseqüência maior a violação da humanidade des- acentuados, sendo suas formas mais comuns conforme já
te – e, indo além, do próprio agressor” (SILVA, 2005, p. 16). citamos neste material.

43
CONHECIMENTOS ESPECÍFICOS - PSICÓLOGO

Faleiros, em estudo que coordenou, refere que “As prá- Marli Palma de Souza (2004) , em estudo que busca
ticas de violência sexual interpessoal e comercial contra mapear “os contornos dos serviços de proteção a crianças e
crianças e adolescentes são uma violação de seus direitos adolescentes que sofrem violência no âmbito familiar”, as-
humanos e sexuais, e dos direitos particulares de pessoa sinala que o crescente número de denúncias que envolvem
em desenvolvimento. Além de violação à integridade física esse tipo de violência tem feito com que o estabelecimento
e psicológica, ao respeito, à dignidade, à sexualidade res- de diagnóstico ocupe “significativa parcela de tempo dos
ponsável e protegida, é violado o direito ao processo de profissionais envolvidos, em detrimento das ações proteto-
desenvolvimento físico, psicológico, moral e sexual sadios. ras”, o que tem gerado “contextos que não permitem que
A violência sexual na família é uma violação ao direito a os usuários percebam os serviços como apoio e ajuda para
uma convivência familiar protetora. No mercado do sexo, rever e questionar alguns atos das famílias, impedir que se
além dos anteriormente mencionados, são violados os di- repitam através de uma proteção que não seja desquali-
reitos de não ser explorado e o de não trabalhar em con- ficadora das funções familiares, mas capaz de preservar e
dições indignas, perigosas e estigmatizantes. Ética, cultural resgatar potencialidades das práticas familiares cotidianas”.
e socialmente, a violência sexual contra crianças e adoles-
centes é uma violação de direitos humanos universais, de Continuando a análise, considera recomendável que a
regras sociais e familiares das sociedades em que ocorre. violência contra a criança e o adolescente seja vista “num
É, portanto, uma ultrapassagem dos limites humanos, le- contexto de necessidades mais amplo”. Explicando que, ge-
gais, culturais, sociais, físicos, psicológicos. Trata-se de uma ralmente, esse tipo de violação de direitos “constitui o pro-
transgressão e neste sentido é um crime, ou seja, é o uso blema que mais chama atenção em meio a tantos outros
delituoso, delinquente, criminoso e inumano da sexualida- que a família enfrenta”, pondera que é importante que se
de da criança e do adolescente“ (1998, p. 9). avalie “a necessidade das famílias em relação às políticas
Na atualidade, vêm sendo denunciados também, com públicas, quer seja de habitação, saúde, educação, quer de
relativa frequência, os crimes sexuais contra crianças e ado- mecanismos legais e jurídicos capazes de garantir direitos.
lescentes por meio da Internet, como pornografia e pedo- Se a família, do ponto de vista legal e social, deve oferecer
cuidado e proteção aos seus filhos, é preciso criar condi-
filia, e ainda a prostituição infantil, o turismo sexual e o
ções que lhe permitam desempenhar tais papéis”, ou seja,
tráfico de crianças e adolescentes com fins de exploração
é preciso que seus direitos sociais sejam assegurados, de
sexual, os quais geralmente envolvem interesses econômi-
maneira a que tenha maiores possibilidades de fazerem va-
cos, financeiros e comerciais.
ler seus direitos fundamentais (Souza, 2004).
O PNCFC (2006) aborda a questão da violência domés-
Nesse sentido, os Serviços de Proteção Social às Crian-
tica contra crianças e adolescentes considerando sua com-
ças e aos Adolescentes Vítimas de Violência, Abuso e Ex-
plexidade e multideterminação, justificando a necessidade
ploração Sexual e a Suas Famílias (Combate ao abuso e à
de políticas sociais eficazes para atendimento. Recorrendo
exploração sexual de crianças e adolescentes – Programa
a conceito utilizado por Veronese e Costa (2006), explica Sentinela), inseridos no Plano Nacional de Enfrentamento
que a “violência doméstica ou intrafamiliar é um fenôme- da Violência Sexual Infanto-Juvenil, têm como objetivos
no complexo e multideterminado em que podem interagir “Contribuir para a promoção, defesa e garantia de direi-
e potencializar-se mutuamente características pessoais do tos de crianças e adolescentes vítimas de violência sexual,
agressor, conflitos relacionais e, por vezes, transgeracionais, buscando: i. identificar o fenômeno e riscos decorrentes;
fatores relacionados ao contexto socioeconômico da famí- ii. prevenir o agravamento da situação; iii. promover a in-
lia e elementos da cultura. Isso explica o fato da violência terrupção do ciclo de violência; iii. contribuir para a devida
doméstica não ser exclusiva de uma classe desfavorecida, responsabilização dos autores da agressão ou exploração;
perpassando indistintamente todos os estratos sociais. Ela e iv. favorecer a superação da situação de violação de di-
acontece no espaço privado, na assimetria das microrrela- reitos, a reparação da violência vivida, o fortalecimento dos
ções de poder estabelecidas entre os membros da família, e vínculos familiares e comunitários, a potencialização da au-
abrange a violência física, a violência psicológica e a violên- tonomia e o resgate da dignidade”.
cia sexual, podendo acarretar sequelas gravíssimas e até a Com a implantação do Sistema Único de Assistência
morte da criança ou do adolescente”. Nesta conceituação, Social (Suas), esse Programa deve ser operacionalizado pe-
assinala a gravidade do fenômeno, situando-o no mesmo los Centros de Referência Especializados de Assistência So-
plano da violência urbana e da violência estrutural, o que cial (Creas), de abrangência local ou regional, obedecendo
exige a urgente “necessidade de preveni-lo e enfrentá-lo, às Normas Operacionais Básicas da Política Pública de As-
em todas as suas facetas e gradações”. Por isso, lembra sistência Social, devendo manter estreita articulação com
“que condições de vida tais como pobreza, desemprego, os demais serviços da Proteção Social Básica e Especial,
exposição à violência urbana, situações não assistidas de com as demais Políticas Públicas e instituições que com-
dependência química ou de transtorno mental, violência de põem o Sistema de Garantia de Direitos.
gênero e outras, embora não possam ser tomadas como
causas de violência contra a criança e o adolescente, po-
dem contribuir para a sua emergência no seio das relações
familiares” (PNCFC, 2006, p. 36). Daí, conforme aponta, a
necessidade de eficazes políticas sociais para atendimento.

44
CONHECIMENTOS ESPECÍFICOS - PSICÓLOGO

Como Crianças e Adolescentes Vítimas de Violência situação, destacando-se que tal trabalho pode incluir res-
Sexual São Ouvidos no Judiciário postas a quesitos, se levantados pelas partes envolvidas. A
critério de cada magistrado, pode ser definida a participa-
Toda a fase processual que envolve crimes deve ser re- ção do(s) profissional(is) na audiência para esclarecimentos
gida por preceitos postos pela Constituição Federal, que técnicos, com base no estudo realizado. Ainda que sejam
dizem respeito ao “devido processo legal, ampla defesa e conhecidas as dificuldades para se operacionalizar a arti-
o contraditório”. culação da rede de atendimento, esse trabalho deve incluir
O Código de Processo Penal (CPP), datado de 1940, em seu norte a articulação com a rede familiar e social para
disciplina a aplicação da legislação penal (processo legal) o necessário atendimento e acompanhamento do conjunto
em relação à prática de crimes, e, neste caso, aqueles que de pessoas envolvidas no abuso.
envolvem a violência sexual contra crianças e adolescentes,
da seguinte maneira: Bibliografia
a. denúncia oferecida pelo Promotor de Justiça ao Juiz CHAUI, M. Ética e violência. Revista Teoria e Debate,
de Direito, com base em inquérito policial, ou outras infor- São Paulo: Fundação Perseu Abramo, n. 39, 1998. Dispo-
mações, por exemplo, a denúncia feita diretamente pela nível em: <http://www.fpabramo.org.br> . Acesso em: 22
própria vítima; dez. 2007.
b. interrogação do acusado, acompanhado de defen- DESLANDES, S. F. Prevenir a violência: um desafio para
sor (os quais podem não comparecer); profissionais de saúde. 2. ed. Rio de Janeiro: Fiocruz/ENSP/
c. apresentação de defesa escrita; Claves-Jorge Careli,
d. audiência para oitiva da vítima e das testemunhas, FALEIROS, V. P.; FALEIROS, E. S. Indicadores de violência
quando o juiz faz sua própria inquirição às testemunhas e intra-familiar e exploração sexual comercial de crianças e
à vítima (no caso em estudo, a criança e/ou o adolescente) adolescentes. Distrito Federal: Cecria, 1998. Disponível em:
sobre o crime e, em seguida, faz a elas as perguntas levan- <http://www.cecria.org.br>. Acesso em: 27 dez. 2007.
tadas pelo Promotor de Justiça e defensor, com objetivos FÁVERO, E. T. Questão social e perda do poder familiar.
de “conhecer a verdade dos fatos ocorridos”. São Paulo: Veras, 2007.
Trata-se do chamado sistema “presidencial’, isto é, cabe ______; TOLOSA JORGE, M. R; MELÃO, M. J. Serviço so-
exclusivamente ao juiz (presidente da audiência) fazer to-
cial e psicologia no Judiciário – construindo saberes, con-
das as perguntas – inclusive as proferidas por acusação e
quistando direitos. São Paulo: Cortez, 2005.
defesa –, tendo ele o “poder-dever” de censurá-las se con-
GUERRA, I. A instrumentalidade do serviço social. São
siderá-las inadequadas ou impertinentes.
Paulo: Cortez, 1998.
Nesse processo, as declarações das vítimas são essen-
HELLER, Agnes. O cotidiano e a história. Tradução de
ciais, servindo como uma das provas para a formação da
Carlos N. Coutinho e Leandro Konder. Rio de Janeiro: Paz
convicção do juiz para que estabeleça o julgamento – isto
e Terra, 1985.
é, para que obtenha a “verdade dos fatos” para aplicação
IAMAMOTO, M. V. O serviço social na contemporanei-
da pena. Por decisão do magistrado, o acusado pode não
estar presente a esta audiência, via de regra como uma for- dade: trabalho e formação profissional. São Paulo: Cortez,
ma de proteção à vítima. 1998.
As normas para ouvir os adultos são as mesmas utili- LEI nº 8.069/90. Estatuto da Criança e do Adolescente.
zadas para ouvir crianças e adolescentes, o que tem sido LEI nº 8.662/93. Dispõe sobre a profissão de Assistente
questionado por profissionais de diferentes áreas, tendo Social.
em vista sua condição peculiar de desenvolvimento (de- MIOTO, R. C. T. Novas propostas e velhos princípios: a
senvolvimento cognitivo, intelectual, psicossocial e psicos- assistência às famílias no contexto de programas de orien-
sexual ). tação e apoio sociofamiliar. IN: SALES, M. A.; LEAL, M. C.;
Quando é possível estabelecer acordo entre defesa e MATOS, M. C. (Org.). Política social, família e juventude:
acusação, a audiência com a vítima (criança e/ou adoles- uma questão de direitos. São Paulo: Cortez, 2004.
cente) pode deixar de ser realizada, sendo substituída por PLANO Nacional de Enfrentamento da Violência Sexual
avaliação técnica, geralmente realizada por assistentes so- Infanto-Juvenil. Relatório do Monitoramento 2003-2004.
ciais, psicólogos e/ou psiquiatras, a qual vai instruir a ação Brasília, 2006.
processual – que pode dar-se tanto na Justiça da Infância PNCFC – Plano Nacional de Promoção, Proteção e De-
e Juventude – para as medidas de proteção à criança e ao fesa do Direito de Crianças e Adolescentes à Convivência
adolescente –, como na Justiça Criminal, onde se desen- Familiar e Comunitária. Brasília, dezembro de 2006.
volverá o processo penal em relação ao suposto abusador
. Neste tipo de avaliação, os profissionais realizam estudo VIOLÊNCIA INTRAFAMILAR: POSSÍVEIS ARTICULA-
social, psicológico e/ou psiquiátrico, individualmente ou ÇÕES
interdisciplinarmente (dependendo da composição e/ou
articulação da equipe técnica interna ao Judiciário, ou per- Dentre os diversos ramos que a psicologia jurídica
tencente à rede de atendimento, neste caso, geralmente pode abordar, o presente artigo trata do papel do psicó-
vinculada à área da saúde). Em seguida, apresentam o lau- logo forense no que se refere à violência intrafamiliar, bem
do que registra o trabalho e as conclusões a respeito da como sua articulação com demais instâncias envolvidas.

45
CONHECIMENTOS ESPECÍFICOS - PSICÓLOGO

O campo da violência doméstica é um “terreno mo- os direitos e o bem-estar da criança e/ou adolescente, a fim
vediço”, como afirma Miranda (1998), em que se mesclam de auxiliar o juiz na tomada de uma decisão que melhor
fantasia e realidade, cena que causa horror e curiosidade. atenda às necessidades dessas pessoas. (SILVA, 2003, p.39)
Diante do número imenso de variáveis culturais e psíquicas,
torna-se muito complexa a tarefa de bem lidar com este Como afirma Miranda (1998), constituiu-se a partir de
problema. então uma nova área de prática dos psicólogos: a psicolo-
O estudo da violência em suas causas, consequências, gia jurídica. O lugar ocupado por esta ainda é pouco defini-
objetivos, justificativas, ganhou atualidade, parecendo de- do. A relação entre a psicologia e as práticas jurídicas ainda
pender desta compreensão a possibilidade de sobrevivên- se dá de forma estremecida e o lugar do psicólogo nesta
cia da humanidade e a construção de alternativas para um área ainda está por se configurar.
futuro melhor. No que se refere à violência doméstica, intervir na fa-
Pode-se pensar na violência intrafamiliar como toda mília para proteger a criança representa um dilema: qual é
ação ou omissão que prejudique o bem-estar, a integrida- o limite entre a proteção aos direitos da criança e o respeito
de física, psicológica ou a liberdade e o direito ao pleno à convivência familiar? Que nível de violência intrafamiliar
desenvolvimento de outro membro da família. Pode ser justifica a intervenção? Em que circunstâncias afastar uma
cometida dentro ou fora de casa por algum membro da criança de seus pais biológicos pode representar um be-
família, incluindo pessoas que passam a assumir função pa- nefício?
rental, ainda que sem laços de consanguinidade, e em rela- Pensando nesta família como doente – sustento a ideia
ção de poder à outra. Portanto, quando se fala de violência de que famílias que maltratam têm como característica
intrafamiliar deve-se considerar qualquer tipo de relação básica o sofrimento psíquico, ou ainda são portadoras de
de abuso praticado no contexto privado da família contra transtornos mentais – evidencia-se a necessidade de auxí-
qualquer um de seus membros. Deve-se ainda ressaltar lio, independente da decisão que vai ser tomada a poste-
que o conceito de violência intrafamiliar não se refere ape- riori. Talvez a única alternativa em algumas situações seja o
nas ao espaço físico onde a violência ocorre, mas também afastamento, mas nunca sem antes usar de todos os recur-
às relações em que se constrói e efetua. sos possíveis para a reestruturação familiar.
Quando se fala em recursos tem que se pensar que
No que se refere à Psicologia Jurídica seu surgimento eles ainda são muito escassos. A maior parte das famílias
é bastante recente. A participação do psicólogo nas ques- não recebe apoio adequado para enfrentar a situação e ter
tões judiciais começou em 1980, no Tribunal de Justiça do possibilidades de revertê-la. Para verificar esta realidade
Estado de São Paulo, quando um grupo de psicólogos vo- basta observar os casos acompanhados pelos Conselhos
luntários orientava pessoas que lhes eram encaminhadas Tutelares, onde o que se observa é o pouco empenho de-
pelo Serviço Social, basicamente apoio a questões fami- dicado aos mesmos, uma vez que de nada adianta enca-
liares, tendo como objetivo principal sua reestruturação e minhar para tratamento e programas se não é dada uma
manutenção da criança no lar. Mais tarde, a Lei nº 500 do atenção especial a estas famílias, um “empurrãozinho” para
CPC instituiu a contratação do Psicólogo, a título precário, que as coisas aconteçam. Essa falta de amparo se dá tanto
por um ano, podendo ser recontratado após esse período. em relação ao núcleo familiar quanto à criança afastada.
Em 1985, o presidente do Tribunal de Justiça apresentou à Como consequência, percebe-se, muitas vezes, que crian-
Assembleia Legislativa um projeto criando o cargo de psi- ças afastadas da família por maus-tratos não retornam aos
cólogo judiciário, o que significou a consolidação do posto lares de origem.
de psicólogo no sistema judiciário. Para enfrentar a violência doméstica são necessárias,
A relação entre os saberes construídos pela Psicologia, além de medidas punitivas, ações que estejam voltadas
o Direito e as práticas judiciárias é muito antiga, mas ainda para a prevenção, e, ainda, medidas de apoio que permi-
pouco conhecida no Brasil. A partir da complexidade com tam, por um lado, à vítima e à sua família ter assistência so-
que foram se constituindo as regras de convivência huma- cial, psicológica e jurídica necessárias à recomposição após
na, as bases da lei foram se complexificando e absorvendo a violência sofrida e, por outro lado, que proporcionem a
cada vez mais contribuições dos diversos campos do saber. possibilidade de reabilitação dos agressores.
Brito (1999) nos fala que a ideia de que todo o Direito, Apesar da necessidade que as famílias nestas situações
ou grande parte dele, está impregnado de componentes têm de auxílio psicológico, há entraves para a consolidação
psicológicos justifica a colaboração da Psicologia com o da prática psi na instituição judiciária. Porém, hoje se sabe
propósito de obtenção de eficácia jurídica. também que é pouco provável que haja benefícios na ação
Em se tratando de violência perpetrada no lar estamos que se contenta em localizar agressores e vítimas, punir os
adentrando na Psicologia Jurídica aplicada à área Civil. primeiros e proteger os segundos. A violência, produto da
Dessa forma podemos pensar que: cultura que explode em relações interpessoais, deve ser
A função do profissional psi consiste em interpretar a vista de modo mais abrangente.
comunicação inconsciente que ocorre na dinâmica familiar
e pessoal [...] Seu objetivo é destacar e analisar os aspectos Se a ótica é de proteção à família, a transferência da
psicológicos das pessoas envolvidas, que digam respeito pena exclui o argumento, na medida em que a vítima e
a questões afetivo-comportamentais da dinâmica familiar, demais membros do grupo familiar do agente criminaliza-
ocultas por trás das relações processuais, e que garantam do serão, por extensão, também penalizados tendo, muitas

46
CONHECIMENTOS ESPECÍFICOS - PSICÓLOGO

vezes, que sair de seus lares para que sejam protegidos. Es- alternativos de compensação e de consolidação para a
tas são ainda as medidas mais comuns: ou se tira o agres- solução dos conflitos. Estes incluiriam a possibilidade de
sor, ou se afasta as crianças, colocando-as em instituições tratamento tanto para as vítimas quanto para o abusador.
por período indeterminado. O tratamento fica para segun- O objetivo de tais propostas é restabelecer (ou estabelecer)
do plano. um equilíbrio justo entre vítima e autor, prescindindo do
As instituições que prestam serviços - jurídicos, poli- processo penal (quando isto é possível). Assim, evitar-se-ia
ciais, de saúde e educação - ainda não contam, em sua a estigmatização de ambos (vítima e autor), possibilitando-
maioria, com sistemas de diagnósticos e registros apro- -se uma resposta mais eficiente aos anseios das partes en-
priados. A ciência avançou tremendamente, os modelos volvidas, principalmente das vítimas.
diagnósticos evoluíram significativamente e os juristas bra- Diante desse fato, constata-se no exercício da “Lei” a
sileiros se debatem numa questão primitiva - a questão da forte presença da punição e a pouca importância que se dá
materialidade nos casos de maus-tratos contra crianças e à reabilitação ou, ainda mais longe, à atenção primária, à
adolescentes sem lesões orgânicas compatíveis. Os diag- qualidade de vida das pessoas.
nósticos psicológicos não falam da “materialidade” espera- Compete às equipes de Saúde da Família conhecer,
da, tanto que os laudos de avaliação psicológica costumam discutir e buscar a identificação dos fatores de risco na
ser desprezados nos tribunais. Estes falam de marcas que população adscrita, para facilitar a definição de ações a
não são visíveis de forma concreta, mas que nem por isso serem desenvolvidas, com a finalidade de intervir preven-
representam menor dano ao desenvolvimento do sujeito, tivamente ou confirmar um diagnóstico, visando a adoção
muito pelo contrário, são danos que, diferente de um ma- das medidas adequadas às diversas situações de violência
chucado provocado por uma agressão física, podem ficar intrafamiliar.
impressos para o resto da vida.
É necessário considerar a complexidade da sociedade
A lei no Brasil ainda exige exame de corpo de delito. No brasileira organizada em distintos setores que devem
Rio Grande do Sul, os Institutos Médico-legais (IMLs) não ser envolvidos, de forma articulada, na luta contra a
possuem sequer uma sala reservada ou cama ginecológica violência: ONGs, mídia, partidos políticos, associações
para os exames. Conforme afirma Caminha (2000, p.18): de classe, associações de base, Igreja, empresariado,
“Não podemos esperar, também, que um sujeito que movimentos sociais, escolas e universidades, dentre outros.
está com as câmaras frigoríficas cheias de cadáveres a se- A instrumentalização de instâncias alternativas com tal
rem analisados, além dos costumeiros acidentes de trân- envergadura de envolvimento sóciocomunitário é o ca-
sito, brigas e etc., goze de grandes talentos e delicadezas minho mais propício para a concretização desse objetivo.
para atender crianças e adolescentes maltratados”. Tanto mais que o sistema judiciário representa uma opção
O perito, neste caso, é um pouco vítima, já que o Esta- cruel e incongruente não apenas para o agressor, mas tam-
do não lhe fornece nem condições materiais nem formação bém para a própria vítima. Este tem se mostrado incapaci-
técnica compatível para o atendimento destes casos. Como tado para cumprir qualquer função preventiva e/ou repara-
produto de tais exames temos um segundo abuso igual- tória nos casos de violência doméstica.
mente traumático. Este fato vem corroborar com a ideia de Cervini (1990)
Todos estes fatores, somados ao desconhecimento e quando argumenta que, na gradação de opções de contro-
temor da sociedade frente à dinâmica das relações intrafa- le, o procedimento jurídico formal é tão-somente uma das
miliares violentas, levam as pessoas (tanto vítimas quanto soluções possíveis, sem dúvida a mais impessoal e onerosa,
agentes sociais) a evitar olhar para ela. Entretanto, é preciso mas não necessariamente a mais eficaz, para a solução de
assinalar que, a cada dia que passa, esses aspectos vêm lodosos conflitos.
sendo superados, haja visto o número cada vez maior de Quanto ao papel do psicólogo, constata-se a necessi-
denúncias realizadas. dade de um olhar mais amplo, que contemple, além das
Com isso pode-se questionar: que tipo de auxílio e demandas particulares de cada sujeito (tratamento do abu-
proteção é oferecido pelo Estado nos casos de violência sador e do abusado), um envolvimento maior com o social,
doméstica? A resposta encontrada é que o sistema penal, pois não se pode descolar a violência do contexto social
confirmando uma tendência que não é nova, acaba por jo- em que ela está inserida.
gar na vala comum todos os conflitos domésticos, sem que Benevides (2002) nos fala sobre a articulação entre
se possa diferenciar os casos e com isso constatar que, em saúde mental, direitos humanos e profissionais psi, mos-
muitos deles, a ocorrência de transtornos mentais e cultu- trando que as situações sociais, aquelas em que se com-
ras familiares que se propagam estão presentes. partilham deveres e direitos, são geralmente percebidas
como pertinentes ao campo das ciências jurídicas, das
Na maior parte dos casos, o trabalho do Estado en- ciências sociais. Ressaltando que esta clara dicotomia - de
cerra-se na constatação da violência sofrida e na busca um lado o indivíduo, de outro a sociedade - não se instala
da preservação da criança de outros abusos. O acompa- sem consequências.
nhamento tanto do abusado como do abusador não são
contemplados pelo sistema. Resta desatendida a recomen- O caminho que pode conduzir a uma resposta coeren-
dação mais importante, ou seja, a que sugere que os Es- te deve ser trilhado de dentro para fora do judiciário. Abri-
tados introduzam em suas legislações nacionais processos -lo passa pela valorização da vontade das vítimas, que pre-

47
CONHECIMENTOS ESPECÍFICOS - PSICÓLOGO

tendem, na verdade, ao aportar neste sistema, encontrar É importante, ainda, que os mediadores possam
aí uma instância mediadora capaz de deter a escalada da contar com supervisão periódica por parte da equipe
violência e de assessorá-las na empreitada de repactuação multidisciplinar. Para tanto, deve ser-lhes oportunizado
de sua convivência doméstica. trazer, ao menos a cada trinta dias, os casos sob seu
Analisando formas de intervir nesta problemática, Her- atendimento à discussão grupal com outros mediadores,
mam (2000) nos apresenta o exemplo do Canadá. Este país sob a coordenação de profissional habilitado.
propôs soluções extraprocessuais para delitos ocorridos
no seio familiar ou no meio social que o circunda, visan- A política de prevenção deve atingir, sensibilizando e
do resolver a interação vítima-autor através de instâncias capacitando, todos os atores que tenham contato com pes-
de compensação. Em tais instâncias informais buscam-se soas vítimas de violência nas diferentes etapas do processo.
soluções através da atuação de equipe multidisciplinar em Isto inclui os profissionais de saúde, os agentes policiais,
serviços comunitários visando o aconselhamento, cuidado membros do Poder Judiciário, psicólogos e assistentes so-
e tratamento familiar. Fazendo com que muitos casos pos- ciais.
sam ser resolvidos sem que precisem chegar ao sistema A abordagem deve ser multidisciplinar, sendo que a
judiciário. assistência ambulatorial ou hospitalar precisa ser criterio-
Num segundo momento, em 1985, o Canadá formou a samente decidida pela equipe, particularizando cada caso.
Rede Pró-Justiça Comunitária e Solução de Conflitos com O trabalho junto à família é imprescindível e não deve ser
o propósito de viabilizar a troca de informações, a capaci- apenas pontual. Essa família dever ser acompanhada du-
tação de intermediários sociais e a instrumentalização de rante um período que permita avaliar suas demandas, pro-
novas instâncias de justiça informal. pondo-se a partir de então intervenções adequadas.
Através da rede interagem mediadores, psicólogos, Pensando ainda no contexto brasileiro, algumas pro-
advogados, docentes, investigadores sociais, adminis- postas neste sentido já estão sendo executadas. Estados
tradores, pessoal da justiça penal e os que são partes no como São Paulo, Ceará, Pernambuco, Minas Gerais, pos-
conflito, sendo dessa forma atacados vários núcleos, tais suem algumas localidades onde estão sendo produzidos
como rixas de vizinhos, reconciliação delinquente/vítima, trabalhos inovadores no que diz respeito ao tratamento
prestado à saúde mental e saúde da família como um todo,
conflitos familiares e ambientais, violência contra a crian-
fazendo com que muitas questões possam ser resolvidas
ça e o adolescente, enfim, alternativas de tratamento que
de forma mais humana, priorizando a saúde ao invés das
evitem o processo penal e priorizem a proteção das vítimas
punições.
em geral.
Estes exemplos nos mostram tentativas de resolução
de problemas que ficam mais centradas nas comunidades
A experiência canadense, como um todo, embora ain-
e muitas vezes não necessitam passar pelo sistema judiciá-
da minoritária, apresenta soluções que representam uma
rio. É um caminho promissor, que trabalha com sistemas
proposta concreta de retomada, por parte da sociedade,
mais compartimentalizados, prestando uma atenção foca-
de conflitos que lhe dizem respeito diretamente, abrindo lizada nas necessidades de cada população.
uma trilha que reinsere autor e vítima. É muito menos one-
rosa que o processo formal e o encarceramento: enquanto Dessa forma, conclui-se que a articulação entre os pro-
opção alternativa, representa um dispêndio anual de dois fissionais que trabalham com a violência intrafamiliar ainda
dólares por habitante, os encargos inerentes à operaciona- precisa ser bastante trabalhada, embora já existam alguns
lização do sistema penal oficial chegam a atingir mais de movimentos neste sentido. Segundo Silva (2003), desde
cem dólares por habitante/ano. Entretanto, é preciso frisar 1980 os juízes vêm sendo sensibilizados, através de um tra-
que a atuação da rede só atinge oito a dez por cento dos balho de esclarecimento, sobre a importância do aspecto
conflitos conhecidos. dinâmico e emocional e sobre a compreensão do que é
Uma das opções que poderia ser adaptada à realidade subjetividade, uma vez que esta visão começa a fazer parte
brasileira e especificamente aos casos de conflito domésti- da formação dos juízes na Escola Superior de Magistratura.
co é o “Serviço de mediação comunitária”, que administra Um dos pontos de entrave para este processo é o fato da
a interveniência de mediadores sociais, os quais sugerem rotatividade de juízes na Vara ser muito grande, dificultan-
caminhos para a composição extrajudicial. do o trabalho. Há diversidade de opiniões que passam pela
A estrutura da instância alternativa deve contar com instituição, de pessoas com diferentes orientações, des-
o trabalho de mediadores leigos, comunitariamente próxi- de aquele juiz que acredita na importância das questões
mos dos protagonistas do conflito, mas previamente pre- emocionais, da doença mental e que faz cursos de aper-
parados para a função por técnicos especializados, de pre- feiçoamento na área psi para melhor servir a população,
ferência por equipe multidisciplinar (psicólogo, assistente até aquele que vê a psicologia como uma área avessa ao
social, advogado). Essa preparação é relevante, na medida Direito e que este tem que se valer única e exclusivamente
em que os mediadores comunitários devem estar cons- das normas.
cientes de estar lidando com um conflito delicado, bem
como atentos a uma eventual escalada desse conflito, que O Estatuto da Criança e do Adolescente (ECA, 2002)
possa porventura desaguar em consequências mais sérias fala sobre a importância de o juiz ser dotado de sensibili-
(um homicídio, por exemplo). dade, a fim de poder julgar na área da infância e da juven-

48
CONHECIMENTOS ESPECÍFICOS - PSICÓLOGO

tude. E, mais do que possuir esta sensibilidade, tem o dever


de agir em fina sintonia com ela em prol do bem-estar do ASPECTOS PSICOSSOCIAIS DO
assistido. ENVELHECIMENTO.
Quanto aos Psicólogos que optam por trabalhar com
questões jurídicas acabam, muitas vezes, formando uma
pele de proteção que os torna rígidos, deslocando mais
para questões que dizem respeito a leis fixas, deixando de CONTRIBUIÇÕES DA ANTROPOLOGIA PARA UMA
lado o olhar singular às questões trazidas por cada sujeito. ABORDAGEM DAS QUESTÕES RELATIVAS À SAÚDE DO
Talvez pelo fato de que trabalhar com violência doméstica IDOSO
é trabalhar com frustrações constantes, com situações que
remetem a sentimentos diversos e confusos em alguns mo- Elizabeth Uchôa
mentos, onde o profissional que não está preparado aca- http://www.scielo.br/scielo.php?script=sci_
ba deixando-se levar por atravessamentos pessoais que o arttext&pid=S0102-311X2003000300017
impedem de ter uma visão clara da situação apresentada.
Um novo olhar se faz necessário no entendimento des- Apesar da entrada tardia no campo de estudos sobre o
ta prática, onde somente os testes psicológicos e as leis envelhecimento, a antropologia já deu algumas contribui-
jurídicas não podem dar conta da imensidão existente na ções fundamentais para inovar a abordagem das questões
configuração familiar, uma vez que esta traz situações e relativas à saúde do idoso. A primeira delas refere-se à re-
sentimentos que não podem ser mensurados unicamente lativização da visão universalista, usualmente adotada em
pelo objetivo, isto é, pela mensuração e aplicação de nor- estudos sobre o envelhecimento. Delimitado inicialmente
mas. Para isto, parcerias devem ser efetivadas e fortaleci- a partir de sua dimensão biológica, o envelhecimento foi
das. Os profissionais devem estar mais flexíveis, dispostos a associado à deterioração do corpo e, em conseqüência,
traçar novos percursos, criar novas alternativas que possam tratado como uma etapa da vida caracterizada pelo declí-
contemplar as demandas trazidas de forma mais saudável nio (Corin, 1985; Debert, 1999). Cristalizou-se, assim, uma
possível. A violência intrafamiliar deve ser tratada e não pu- visão orgânica do envelhecimento. Entretanto, estudos rea-
nida. Deve-se investigar as causas, usar as pesquisas para, a lizados em sociedades não ocidentais tornaram conhecidas
partir de um trabalho em equipe, tornar viável a reestrutu- imagens bem mais positivas da velhice e do envelhecimen-
ração familiar. O que se percebe é que as instâncias envol- to, questionando a universalidade da visão ocidental e en-
vidas nestes casos pouco fazem porque pouco acreditam sinando que uma representação de velhice enraizada nas
em resultados positivos, tendo em vista a complexidade ideias de deterioração e perda não é universal.
desta problemática. É preciso uma maior qualificação como Um exemplo clássico desta outra visão da velhice nos
profissional e como pessoa para que possamos trabalhar foi dado, há mais de cinquenta anos, por Evans-Pritchard
nesta área (tanto o conselheiro tutelar, como o psi, o assis- (1989) em seu estudo sobre os Nuer, grupo étnico do Su-
tente social, o juiz....). dão. Segundo este autor, a entrada na adolescência para
o Nuer do sexo masculino é marcada por um ritual de ini-
Pensando no psicólogo como facilitador da promoção ciação que define sua inserção em uma classe de idade e
da saúde, ele deve procurar garantir os direitos fundamen- determina seu estatuto de superioridade, igualdade ou in-
tais dos indivíduos, visando sua saúde mental e a busca ferioridade frente aos outros Nuer. Os membros de uma
da cidadania. Do contrário, será mais um agente repressor. classe de idade devem respeito aos da classe anterior, que
é composta por pessoas mais velhas e, portanto, superiores
Fonte: na hierarquia social. Estes respeito e deferência transpare-
CESCA, T. B. O papel do psicólogo jurídico na violência cem em todas as dimensões da vida social.
intrafamilar: possíveis articulações. Psicol. Soc. vol.16 no.3 Um exemplo atual nos é dado pelos Bambara do Mali,
Porto Alegre Sept./Dec. 2004 grupo com a qual tive oportunidade de trabalhar (Uchoa,
1988). Os Bambara consideram a velhice uma conquista.
Para eles, o envelhecimento é concebido como um pro-
cesso de crescimento que ensina, enriquece e enobrece o
ser humano. Ser velho significa ter vivido, ter criado filhos
e netos, ter acumulado conhecimento e ter conquistado,
através destas experiências, um lugar socialmente valoriza-
do. Os Bambara constituem um exemplo atual da situação
privilegiada dos idosos em algumas sociedades africanas.
Para os Bambara, a idade é um elemento determinante
da posição de cada indivíduo na sociedade. Toda a vida
social é organizada segundo o princípio da senioridade.
Considera-se que os mais velhos estão mais próximos dos
ancestrais e, por esta razão, detêm a autoridade. Respeito
e submissão marcam o conjunto de atitudes e comporta-
mentos dos mais jovens para com os mais velhos.

49
CONHECIMENTOS ESPECÍFICOS - PSICÓLOGO

Os comportamentos e atitudes dos jovens inuit (esqui- que podem ser vividos e resolvidos diferentemente atra-
mós) para com os idosos constituem um paradoxo que es- vés das culturas. Os autores focalizam aspectos estruturais,
pantou europeus e norte-americanos quando estes entra- culturais e vivenciais do envelhecimento em cinco socie-
ram em contato com a cultura deste povo (Guemple, 1989). dades distintas e nos convidam a examinar os problemas
Por um lado, eles demonstram carinho e afeição para com específicos do envelhecimento e as estratégias adaptativas
os parentes mais velhos, mas por outro podem abandoná- utilizadas pelos idosos em termos de uma articulação entre
-los no meio de uma estrada ou ajudá-los a cometer sui- as capacidades do indivíduo e os recursos do meio.
cídio por afogamento ou estrangulamento. Esta aparente Um outro exemplo vem de um estudo realizado por
crueldade enraíza-se em uma concepção particular de vida, Arcand (1989) junto aos Cuiva, população indígena da Co-
de morte e da própria essência do ser humano, podendo lômbia. Neste estudo, o autor nos leva a interrogar sobre a
coexistir, sem contradição, com atitudes de interesse e su- existência de modelos culturais que seriam determinantes
porte aos mais velhos da comunidade. no aparecimento de problemas específicos do envelhe-
Entretanto de uma maneira geral, estudos realizados cimento em diferentes sociedades. Segundo o autor, os
em sociedades não ocidentais enfatizam o poder, o ele- Cuiva negariam todas as formas de envelhecimento, pois,
vado status e o papel social central atribuído aos idosos preocupados com os ideais de igualdade e homogeneida-
nestas sociedades. Alguns estudos ressaltam o impacto ne- de que estruturam a sociedade, evitariam tudo o que po-
gativo do processo de modernização sobre o status dos deria provocar uma ruptura social. Segundo Arcand (1989),
idosos nessas sociedades (Cowgill & Holmes, 1972). os Cuiva não constroem uma idade da velhice: depois de
É evidente que não se pode ter uma visão idealizada e sair da infância, o indivíduo integra o grupo de adultos e
homogênea das sociedades não ocidentais, mas é preciso continua confundido neste grupo até sua morte. Não existe
reconhecer a contribuição trazida pelos estudos realizados espaço social particular para os velhos, nem atividades das
nessas sociedades. À medida que se documentou o quais eles são excluídos. Entre os Cuiva, ninguém é consi-
processo do envelhecimento em diferentes culturas e que se derado velho demais para produzir, tomar decisões, casar
constatou a diversidade de formas de envelhecer, a velhice ou qualquer outra coisa. Segundo Arcand (1989), a socie-
e o envelhecimento deixaram de ser encarados como dade Cuiva trata os velhos como se a velhice não existisse,
fatos naturais, para serem encarados como fenômenos evidenciando um modelo radicalmente oposto ao modelo
profundamente influenciados pela cultura. ocidental, em que todos os meios são utilizados para acen-
Uma segunda contribuição da antropologia, bastan- tuar a distinção entre as diferentes fases da vida.
te ligada à primeira, refere-se à construção de um quadro As reflexões de Ellen Corin (1982, 1985) e de Gogna-
conceitual e metodológico inovador. Com o aparecimento lons-Caillard (1979) nos trazem de volta ao Ocidente. Se-
dos estudos holísticos, o envelhecimento passa a ser es- gundo Corin (1985), a velhice é, geralmente, descrita no
tudado através de uma perspectiva êmica. O pesquisador Ocidente em termos negativos (perda, falta) do que é valo-
tenta penetrar em uma cultura específica para descobrir rizado socialmente. Gognalons-Caillard (1979) sugere que
como aspectos relativos à velhice e ao envelhecimento são esta exacerbação da visão deficitária do envelhecimento
organizados e adquirem significado. O envelhecimento estaria ligada à maneira pela qual a velhice no Ocidente
deixa, então, de ser encarado como um estado ao qual os situa-se na contracorrente de uma sociedade centrada na
indivíduos se submetem passivamente para ser encarado produção, no rendimento, na juventude e no dinamismo.
como um fenômeno biológico, ao qual os indivíduos rea- Segundo Corin (1982, 1985), é a partir da ideia de perda
gem a partir de suas referências pessoais e culturais (Corin, (perdas de papéis sociais, perda de capacidades intelec-
1982; Marshall, 1986, 1987). Esta nova tendência coincide tuais) que se estruturaram os programas destinados aos
com o desenvolvimento da antropologia interpretativa e idosos. A autora (1982, 1985) alerta também para a margi-
encontra em Geertz (1973) subsídios para a construção nalização que uma definição tão negativa da velhice impõe
de um novo paradigma para o estudo da relação entre aos idosos. Por esta razão, ela enfatiza que os estudos so-
envelhecimento e cultura. Geertz (1973), que se situa na bre o envelhecimento devem focalizar a articulação entre
origem desta corrente, define a cultura como sendo um os parâmetros externos (ligados ao contexto sociocultural)
universo de significados que permite aos indivíduos de um e os parâmetros internos (ligados à vivência pessoal), bem
grupo interpretar sua experiência e guiar suas ações. Para como sua contribuição para construir uma relação indivi-
ele, a cultura é o contexto que torna inteligíveis os diver- dual com o envelhecimento.
sos acontecimentos e situações da vida, como por exem- Finalmente, devemos nos referir à contribuição da an-
plo limitações e perdas tão frequentes nesta fase da vida. tropologia para o estudo dos fatores culturais que intervêm
Uma abordagem interpretativa do envelhecimento focaliza no campo da saúde. Embora sua contribuição situe-se em
a interação entre parâmetros culturais, traços individuais e um nível bem mais geral, a antropologia médica fornece al-
marcadores biológicos na construção de maneiras típicas guns elementos-chave para inovar a abordagem da saúde
de envelhecer e viver o envelhecimento (Corin, 1982; Lé- do idoso (Uchoa, 1997; Uchoa & Vidal, 1994).
tourneau, 1989). Nas últimas décadas, pesquisadores americanos e ca-
Meyerhoff & Simic (1978) publicaram uma coletânea nadenses desenvolveram um importante quadro teórico e
de estudos antropológicos sobre o envelhecimento. Em metodológico para estudar de maneira sistemática percep-
todos estes estudos, o envelhecimento é abordado como ções e comportamentos predominantes neste campo. A
um fenômeno universal que gera problemas comuns, mas introdução da perspectiva interpretativa no estudo destas

50
CONHECIMENTOS ESPECÍFICOS - PSICÓLOGO

questões constitui a base do corpo de conhecimentos da po, determina o desenvolvimento de estratégias resoluti-
antropologia médica norte americana (Corin et al., 1992; vas que podem ter repercussões negativas sobre a saúde
Good, 1977; Kleinman, 1981). Um elemento central nesta do idoso, como a aquisição parcial do medicamento ou a
abordagem é a distinção analítica entre “doença processo” redução da dose prescrita. Cabe aqui uma interrogação so-
(disease) e “doença experiência” (illness) (Kleinman, 1981). bre o papel de uma representação deste tipo no contexto
Esta distinção nos remete aos dois aspectos indissociáveis atual de promoção da saúde. Uma representação da saúde
dos fenômenos saúde e doença, ou seja, o processo pato- com sendo “dinheiro dependente” dificilmente incentivaria
lógico e a experiência psicossocial deste processo, à impor- a população idosa a desenvolver hábitos mais saudáveis.
tância de considerar que um aspecto não é redutível ao ou- Estas são certamente informações da maior importância
tro e às múltiplas e constantes interações que existem entre para o planejamento de intervenções adequadas às carac-
eles. Nesta abordagem, considera-se que a experiência da terísticas dessa população.
doença é uma construção cultural que conjuga normas de O estudo mencionado vem reafirmar a importância
conduta, valores e expectativas tanto individuais quanto de compreender como o idoso vivendo em determinado
coletivas e se expressa em formas específicas de pensar contexto percebe sua saúde, o que ele define como pro-
e agir. Trata-se de investigar lógicas culturais subjacentes blema e que estratégias usa para resolvê-lo. Nele, reafirma-
à identificação de problemas e às tentativas de resolução -se também a importância de contextualizar suas reações
destes problemas, assim como dos diversos elementos do frente às perdas físicas (auditivas, visuais ou motoras), afeti-
contexto (pessoal, social e econômico) que podem influen- vas ou mudanças sociais e econômicas que caracterizam o
ciar a tradução destas lógicas em comportamentos concre- avançar da idade. Finalmente, expressa-se, naquele traba-
tos (Good & Good, 1982; Kleinamn, 1981). lho, a necessidade de identificar os recursos (individuais e
coletivos) disponíveis para que os idosos possam enfrentar
Um estudo desenvolvido recentemente entre idosos estas dificuldades.
residentes na cidade de Bambuí, em Minas Gerais, ilustra Uma abordagem antropológica das questões relativas
a pertinência de integrar as contribuições da antropologia à saúde de populações idosas nos obriga a ampliar nosso
na investigação das questões relativas à da saúde do idoso campo de estudo para incluir uma apreensão da experiên-
(Uchôa et al., 2002). Neste estudo, utilizou-se uma abor- cia subjetiva e de sua interação com diversos elementos do
dagem interpretativa do envelhecimento e as informações contexto social e cultural. Trata-se, portanto, de uma ferra-
foram obtidas através de entrevistas com informantes-cha- menta fundamental para a Saúde Pública contemporânea.
ve e reconstruções de histórias de vida. Verificou-se que a
avaliação da gravidade e da relevância de um problema de Bibliografia
saúde pareciam ser claramente determinados pela possibi- Minayo, M.C e Coimbra Jr, C.E.A. Antropologia, Saúde
lidade de enfrentá-lo, muito mais que pelo problema em si. e Envelhecimento. Fiocruz Ed, 2011 – Introdução; Cap I En-
Esta possibilidade estava associada ao apoio familiar e ao velhecimento e Saúde: experiência e construção cultural –
acesso a cuidados médicos. Para um grupo de senhoras de Elizabeth Uchôa, Josélia O. A. Firmo e Maria Fernanda F. de
alto poder aquisitivo, a saúde não constituía um problema, Lima-Costa; Cap II Envelhecimento e Sentimento do Corpo
apesar dos vários problemas de saúde por elas relatados – Alda Britto da Motta; Cap III O Idoso em Processo de De-
(diabetes, doença de Chagas, fraturas, problema cardíaco). mência: o Impacto na Família – Célia parreira Caldas
Com exceção destas senhoras, que podiam consultar mé-
dicos particulares, todas as outras relataram dificuldades VELHICE, NATUREZA E CULTURA
para conseguir consultas no sistema público de saúde. A
primeira dificuldade referia-se à existência de grandes filas Da mesma forma como sempre as mulheres foram li-
para a retirada das fichas que garantem o acesso às con- gadas à ‘natureza’, como forma de dominação e contro-
sultas. Diante da impossibilidade de permanecer horas em le – e toda a fase inicial do feminismo dos anos 60/70 foi
pé nestas filas, várias senhoras relataram a necessidade de um enorme movimento de esclarecimento e recusa a esse
pagar a alguém para retirar as fichas para elas. O paga- determinismo bioideológico –, assim ainda é feito com ‘os
mento de consulta particular ­de um quarto, de um terço velhos’. Mas de maneira diferente – e pior. É como se eles
ou metade do preço regular ­era outra estratégia desenvol- estivessem numa dimensão não produtiva e terminal da
vida, muitas vezes com grande esforço, por estas senhoras natureza – resíduos da natureza, objetos de necessário des-
para evitar as grandes filas. Quando a questão da consulta carte. Não se reproduzem mais, não produzem trabalho e
era resolvida, surgia a do medicamento; poder comprar os bens materiais (ou não se permite que produzam, segundo
remédios prescritos pelo médico era também visto como os cânones do capitalismo). Em suma, não reproduziriam
um fator essencial para o alcance ou manutenção da saú- a sociedade. Portanto, ‘não pertencem’ a ela. Até o ponto
de. Assim, a situação econômica do próprio idoso e de sua da análise de Birman (1995), “este lugar impossível que a
família surgia como fator fundamental para a manutenção modernidade ocidental construiu para a velhice, (...) [onde]
da saúde. a individualidade deixa de existir”.
Esta representação da saúde como “dinheiro-depen- A modernidade capitalista construiu uma visão seg-
dente” reflete as desigualdades que caracterizam o acesso mentar das idades: periodiza as gerações, constrói e ‘des-
do brasileiro idoso aos serviços de saúde e aos medica- constrói’ ‘idades’, quase a cada século inventa mais uma.
mentos (Lima-Costa et al., 2003). Evoca e, ao mesmo tem- Primeiramente a infância e a juventude, no pré-capitalismo

51
CONHECIMENTOS ESPECÍFICOS - PSICÓLOGO

socialmente indiferenciadas da idade adulta (Ariès, 1978); sua longa vida de médico e mestre foi sempre contrário a
bem mais recentemente (década de 60), inventa uma ‘ter- receitar paliativos para a velhice: achava mais fácil suportar
ceira idade’, inserção de um novo período entre a maturi- as dores alheias que as próprias. (García Márquez, 1987)
dade e a velhice, ao mesmo tempo negação desta (Lenoir, A referência à imagem do corpo, no entanto, pode ser
1979). a mais dura:
Em seguida, logo reconhecida a incapacidade de dar Atreveu-se a explorar com a ponta dos dedos seu pes-
conta da atual longevidade dos mais velhos, talvez afastada coço flácido, o peito encouraçado de varetas, as cadeiras
da imagem, atualmente idealizada, dos idosos dinâmicos e de ossos carcomidos, as coxas de corça velha... Tinha os
alegres dos grupos de ‘terceira idade’, inventa-se uma ines- ombros enrugados, os seios caídos, e as costelas forradas
capável ‘quarta idade’, prenunciadora ainda de uma quin- de um pelame pálido e frio como o de uma rã. (García Már-
ta... E estas, quase ninguém quer estudar ou conhecer...1 quez, 1987:417)
Entretanto, a ainda majoritária terceira idade começa a É evidente que essa aproximação desmesurada que se
ser muito lucrativa para uma série de organizadores/gesto- faz do velho com a natureza não corresponde a uma in-
res de atividades, produtos e serviços para esta faixa etária tegração social/natural ou a uma superação da dicotomia
– que, entre programas de lazer que incluem cursos, festas corpo/espírito da cultura ocidental, mas, ao contrário, a um
e viagens para uma ‘velhice saudável’, “põem em circulação alargamento de fosso que torna a ‘natureza’ ‘unipresente’
o dinheiro dos velhos” – como expressou, de modo seve- e ‘final’.
ro, Ariès (1993:53) – mas, contraditoriamente (e felizmen- A cultura, no entanto, também está inscrita no corpo,
te) põem também em circulação social os próprios velhos, ao mesmo tempo condicionando e transformando a natu-
grande parte deles ‘sem lugar’ na sociedade contemporâ- reza. Não atua, sabemos, de modo homogêneo no interior
nea. Repõem-nos, porém, nos interstícios dela (Britto da de uma sociedade e em determinado período histórico. É
Motta, 1998, 1999b). E ao mesmo tempo deixam neles a conformada por determinados sistemas de relações sociais
sensação de estar mais vivos e, às vezes, alegres. em seus modos de realização, que se constituem, ao mes-
No cotidiano, entretanto, as ‘idades’ ainda são perce- mo tempo, em dimensões básicas da vida social e da sua
bidas principalmente como parte do passar do tempo, mi- análise, como as relações de classe, de gênero e entre as
metizando como duração e ritmo os ciclos da natureza e
gerações (Britto da Motta, 1999a).
as estações, o que é expresso no corpo das pessoas. Diz-se
Por isso, os corpos, além de sua forma e ‘natureza’ hu-
completar ‘quinze primaveras’, estar ‘na flor da idade’ ou
mana, diferenciam-se em cada período histórico no seu
‘ainda viçosa aos 50 anos’, ‘bem conservado(a)’, ‘no inverno
existir biossocial – como corpos de homem ou de mulher,
da vida’ etc. Expressa-se um tempo ‘da natureza’ em traje-
de jovem ou de velho – e de classe social, com diferentes
tórias pelo mundo da cultura no capitalismo.
práticas.
Se a atuação do indivíduo deixa de corresponder à
classificação bioetária socialmente esperada, dá-se um
Boltanski analisa, como expressão geral, o corpo em
‘escândalo lógico’ e, adiante, o ‘infrator’ será punido. Com
remissões de cruel mau gosto, como “está conservado(a) sua percepção e uso conforme a situação de classe:
em formol”, com o ridículo (“velhas peruas”) ou a censura. À medida que se sobe na hierarquia social, que cres-
A própria literatura veicula imagens cruéis da velhice, espe- ce o nível de instrução e que decresce correlativamente e
cialmente a das mulheres. García Márquez (1987:317) é um progressivamente o volume de trabalho manual em favor
exemplo, num livro que fala de amor: do trabalho intelectual, o sistema de regras que regem a
Os homens floresciam numa espécie de juventude ou- relação dos indivíduos com o corpo também se modifica:
tonal, pareciam mais dignos com as primeiras cãs, se torna- quando sua atividade profissional é essencialmente uma
vam engenhosos e sedutores, (...) enquanto que suas mur- atividade intelectual, não exigindo nem força nem compe-
chas esposas tinham que se aferrar ao braço deles, para tência física particulares, os agentes sociais tendem primei-
não tropeçarem até na própria sombra. ramente a estabelecer uma relação consciente com o corpo
Repare-se nas metáforas sazonais e no ‘castigo’, que e a treinar sistematicamente a percepção de suas sensa-
é sempre diferente segundo o sexo/gênero: “Poucos anos ções físicas e a expressão de suas sensações... (Boltanski,
depois, no entanto, os maridos despencavam (...) no preci- 1979:168)
pício de uma velhice infame do corpo e da alma...” (García Esse autor analisa também um aspecto interessan-
Márquez, 1987:317). te – mas ainda motivo de muita discussão – da situação
O livro inteiro é, na verdade, um libelo contra a velhi- de gênero: as mulheres, que teriam “um consumo médico
ce, como destruidora do corpo e da ‘alma’. O personagem maior do que os homens”, consomem também mais pro-
médico, de forma escondida e inútil, leva adiante sua luta dutos farmacêuticos; queixam-se mais do que os homens
pessoal: de perturbações digestivas, “dores indeterminadas”, enxa-
Levantava-se com os primeiros galos, e a essa hora co- quecas e outras “doenças vagas”; “escutam-se mais do que
meçava a tomar seus remédios secretos: brometo de po- eles, da mesma maneira que os membros das classes su-
tássio para levantar o ânimo, salicilatos para as dores dos periores se escutam mais facilmente que os membros das
ossos em tempo de chuva, gotas de cravagem de centeio classes populares e mantêm, mais frequentemente do que
para as vertigens, beladona para o bom dormir. Tomava al- os homens, uma relação sensitiva com o corpo” (Boltanski,
guma coisa a cada hora, sempre às escondidas, porque em 1979:174).

52
CONHECIMENTOS ESPECÍFICOS - PSICÓLOGO

Há, entretanto, outras dimensões ou nuances das rela- Há, naturalmente, da parte dos próprios idosos, a clara
ções sociais e ligadas não apenas ao gênero, que não po- percepção desse processo – tanto o do corpo como o da
dem ser analisadas mais amplamente senão como relações reação social a ele. Existem queixas, moderadamente, ou a
e socializações ligadas ao poder social – são, sobretudo, referência à ‘normalidade’ do que acontece, principalmente
as idades/gerações. No exercício desses mecanismos de em relação a dores – na coluna, nas pernas, ‘nos quartos’,
poder social (Bourdieu, 1983), constroem-se preconceitos nos braços... Há, também, quase uma ‘naturalidade’ sobre
que têm mais diretamente a ver com a disputa, entre as isso, e não apenas nas classes populares, entre os menos
idades, pelos postos de trabalho no mercado, mas tam- assistidos. O ator Paulo Autran, em recente entrevista a um
bém, e em outro extremo, com a aversão possível aos que, programa de televisão e em meio a animados comentários
já mais velhos, não possam manter as competências sociais sobre seu mais novo trabalho, definiu: “Ser velho é sentir
de controle corporal: “Degrees of loss impair the capacity uma dor a cada dia”.
to be counted as a competent adult...” (Featherstone, 1991) Essas referências à dor ou a problemas de saúde, nas
e, não por último, o medo ou recusa à idéia de morte... E de entrevistas que fiz, são sempre matizadas por um certo fair-
quem, em princípio, está mais perto dela (Britto da Motta, -play, uma ‘filosofia do cotidiano’ – ‘isso é da velhice’ – que
1998). alcança até o limite da ambiguidade. Vejam-se D. Marta, 72
Todos esses diferenciais referem-se, evidentemente, anos, e o Sr. Manoel, 73.
aos corpos em suas múltiplas manifestações ou expres- São ativos e participam de grupos de idosos. Eles falam
sões: linguagem, apresentação física (roupas, penteados, sobre o sentimento da ‘idade’:
pintura, adereços), gestos etc. Eu quase morri, e vivi. Depois tomei muito remédio.
O ‘gestual humano’, por exemplo, como ação biocultu- Agora me acho feliz da vida. Não fico mais doente, só gri-
ral – postura do corpo e meio de comunicação instantânea pe. O que me ataca mais é a coluna. (D. Marta)
–, é particularmente diferente segundo idades e gerações. Eu senti e sinto, cada dia mais. A gente sente aquele
No caso dos idosos, isso é enfatizado; o comportamento desânimo no corpo. Eu me sinto tão alegre que não pensei
corporal é demandado de fora, para que se coadune com que ia viver tanto. (Sr. Manoel)
o modelo cristalizado do preconceito social. Deles não se
espera vigor, leveza nem dinamismo. Até os próprios ve- Surpreendentemente, os de idade mais avançada tam-
lhos entram nesse ageism, embora, atualmente, cada vez bém não têm grandes queixas. Mesmo o Sr. Pedro, com
mais resistam a ele. Maria Pureza, de 61 anos, participante 100 anos e uma perna quebrada, conta que viveu bem e é
de uma associação de bairro, define ‘velho’ segundo esse feliz. D. Brígida revela que só se deu conta da ‘idade’ aos 95
modelo tradicional e por isso não se reconhece nele: anos, quando começou a ter tonturas. Até então, trabalha-
Velho é uma pessoa que anda assim... [Curva o corpo] va com a enxada. Curiosamente, a que relata mais sintomas
Eu sou uma mulher de idade. [Endireita o corpo, olhando é a vigorosa e incansável D. Lalu:
para a frente.] A gente sente dores nos braços, na coluna, a pressão
Osvaldo, 62 anos, de uma faculdade da terceira idade, vai ficando alta, dores nas pernas, a gente sente que é uma
reage: droga!
Encontramos barreiras impostas pela própria socieda- Deu-me a impressão de que falava não por si, mas por
de, que diz que você já está velho para determinadas coisas. um coletivo. Suas palavras pareciam bem distanciadas das
Você não pode brincar ou ter expressões corporais, que logo minhas anotações no diário de campo, porque, ao conhe-
dizem: ‘Que velho gaiato!’ ‘Isso não é coisa de velho!’ cê-la, eu havia anotado: “É uma senhora de ótimo porte,
aparentando bem menos que os seus 86 anos”. E ao fim do
Às vezes as pessoas fazem concessões e expressam dia que passei em sua casa:
um esteticismo abstrato, comentando a beleza de um ros- Saímos, logo depois do almoço, para a gruta, onde ela
to “marcado pelo tempo”, “um pergaminho”. Mas ninguém vai cerca de duas vezes por dia. É um percurso razoavel-
quer ter essa ‘beleza’, essa aparência associada ao desgaste mente longo e enladeirado, muitos sobe-e-desce, que ela
e à proximidade da morte. percorreu bem, andando na frente, sem demonstrar qual-
No imaginário social, o envelhecimento é um proces- quer cansaço, enquanto os mais moços ficavam de fôlego
so que concerne à marcação da ‘idade’ como algo que se curto e pernas, em alguns momentos, reclamando...
refere à ‘natureza’, e que se desenrola como desgaste, limi- Que resistência têm os idosos diante dos limites e per-
tações crescentes e perdas, físicas e de papéis sociais, em das? Por um lado, essa naturalidade assinalada e, por outro,
trajetória que finda com a morte. Não se costuma pensar uma dissociação até certo ponto confortadora:
em nenhum bem; quando muito, alguma experiência. Ne- As pessoas, principalmente as das classes populares,
nhum ganho, nessa ‘viagem ladeira abaixo’. estão sempre diferenciando corpo e mente, corpo e senti-
As perdas são tratadas principalmente como proble- mentos, atordoadas pelo fato de que as perdas e limitações
mas de saúde, expressas em grande parte na aparência se dão primeiro na aparência e pequenas diminuições de
do corpo, pelo sentimento em relação a ele e ao que lhe possibilidades físico/funcionais do corpo, enquanto ‘por
acontece: enrugamento, encolhimento, descoramento dos dentro eu ainda sou a mesma’.
cabelos, ‘enfeiamento’, reflexos mais lentos, menos agilida- São muitos os depoimentos desse teor nas minhas
de... Mas são expressas muito mais pelos outros do que pesquisas. Vários outros são relatados por Beauvoir (1980),
pelos próprios velhos. como também reunidos por Featherstone (1991), que pro-

53
CONHECIMENTOS ESPECÍFICOS - PSICÓLOGO

põe a expressão ‘máscara do envelhecimento’ (mask of Também é objeto de atenção e especulação o fato de
ageing) para ilustrar uma situação e um sentimento, muito que não se envelhece de modo homogêneo, nem de vez
generalizados, de se ter uma espécie de máscara imposta (Britto da Motta, 1998). Dalva, uma senhora que se aproxi-
ao corpo que esconderia a identidade mais profunda da mava dos 60 anos, colecionou, sempre com sentimento de
pessoa, a qual continuaria sendo essencialmente a mesma estranheza, esse rol modelar de segmentadas considera-
da juventude. ções médicas sobre seu corpo, num período de três anos:
Um depoimento especialmente expressivo de tal estra- Que beleza! Seu joelho tem 20 anos! (Ortopedista
nhamento foi recolhido, ainda por Featherstone, do escri- olhando, encantado, uma radiografia)
tor inglês J. B. Priestley, aos 79 anos: Sua mama! A senhora já fez plástica? Não? Mas está
É como se, descendo a Avenida Shaftesbury como um muito bem! (Primeiro ginecologista)
homem jovem, eu fosse subitamente raptado, arrastado Sua mama é de uma mulher de 40 anos! (Segundo gi-
para um teatro e obrigado a receber o cabelo grisalho, as necologista)
Sua tensão é de broto! A vagina... tem sua idade. Seria
rugas e outras características da velhice, e empurrado para
bom fazer reposição hormonal... (Terceiro ginecologista)
o palco. Atrás da aparência da idade eu sou a mesma pes-
Claramente saudável, ainda assim não escapou da
soa, com os mesmos pensamentos de quando eu era mais
projeção do modelo da juventude, do traçado da trajetó-
jovem. (Featherstone, 1991:379)2
ria cronológica, nem da sugestão de intervenção também
Esse sentimento, nada incomum, demonstra bem a ‘adequada’ à idade.
dificuldade de construção da identidade de velho. Se a Contudo, essa percepção segmentada não é simples-
formação das identidades de idade ou de geração já é mente ‘pós-moderna’, social e sem motivação existencial
difícil, porque são condições mutáveis rapidamente no direta. A vida é curta e as mudanças corporais se proces-
tempo – a cada ano se tem uma nova idade; a cada dez sam rapidamente, por isso, há sempre um sentimento de
ou vinte anos se é classificado numa outra geração (Britto brusquidão na (auto)percepção do envelhecimento. Ao
da Motta,1999a) –, pior a fixação da identidade de velho, mesmo tempo, o envelhecimento, como anteriormente
porque indesejada e dúplice, ou ambígua, principalmen- mencionado, não se processa de modo homogêneo – nem
te quando referida ao corpo: a aparência ‘desgastada’, seu cronológica, nem física, nem emocionalmente (Debert,
funcionamento não totalmente sincronizado e a mente – 1988; Britto da Motta, 1998). Há sempre partes, órgãos ou
ou a essência dos sentimentos – ‘jovem’. funções do corpo que se mantêm muito mais ‘jovens’, ‘con-
Um equilíbrio difícil, que me leva a propor ampliar a servados’ ou sadios do que outros. Assim como no terreno
enunciação de Ferreira, segundo quem “É no campo re- dos sentimentos e das representações, “a velhice nunca é
lacional que se estabelecem os limites entre juventude e um fato total. Ninguém se sente velho em todas as situa-
velhice (...) Pensar-se a si próprio é, na velhice, um duplo ções” (Debert, 1988:62). Nem diante de todos os projetos.
exercício, pois à medida que o sujeito se define, o faz por A literatura tem a clara percepção disso, que Marina Cola-
contraste com o outro” (Ferreira 1995:429). Diria eu: tam- santi expressa na dimensão do poema:
bém com aquele ‘outro’ que é o seu eu jovem. Pensei entrar na velhice por inteiro como um barco ou
Curiosamente, é também o recurso à máscara, desta um cavalo, mas me surpreendo jovem, velha e madura ao
vez ‘da juventude’, que encontro expresso na experiência mesmo tempo. (Colasanti, 1994:106)
de campo, quando uma mulher relata, indignada, o repeti- D. Georgina, de uma associação de moradores, tam-
do abuso, por parte dos motoristas de ônibus, em Salvador, bém afirma, poeticamente:
de não pararem no ponto quando quem nele está é um Nem me lembro que tô velha. Chego no espelho, vejo
idoso. minha cabeça [branca], minhas pelanca, mas por dentro eu
converso com as meninas da minha rua.
Queria ter uma máscara, uma máscara de jovem, para
O maior aliado do preconceito contra os velhos pode
que ele parasse. Aí [acompanha a fala com os gestos] eu
vir, mesmo, das assertivas e prescrições ‘científicas’. Se por
retirava a máscara e lhe ‘dava’ língua...
um lado há o discurso teórico de médicos mais lúcidos de
É ainda Featherstone (1994:50) que, em outro traba-
que velhice não é doença; se Veras (1994), por exemplo,
lho, lembra que “a construção das identidades depende da afirma ter encontrado, em sua pesquisa no Rio de Janeiro,
construção das imagens do corpo”. Fecha-se o ciclo... uma população idosa em que mais de 80% são saudáveis;
No campo científico, as expressões podem não ser por outro lado, encontra-se, a cada passo, o discurso explí-
muito diferentes daquelas do cotidiano. Os corpos são, cito da doença ou da(s) perda(s) como próprias da velhice.
cada vez mais, loteados pelas especialidades médicas e Sugerido, ou posto de fora, é incorporado frequentemente
afins, segundo aparelhos e sistemas fisiológicos... E idades. pelos idosos como expectativa de processo ou até aconte-
O corpo dos velhos é o corpo ‘diferente’, comparado – em cimento inarredável. Gomes (1997) fez ilustrativo estudo de
desvantagem – com o modelo de corpo e beleza jovens uma situação dessas, com idosos atendidos em um ambu-
vigente na sociedade, manipulável para se aproximar deste. latório de geriatria, em Salvador.
Uma série de profissionais cuida desse aspecto: ‘alimenta- Todos referiam-se à ‘perda da memória’, mas nenhum
ção saudável’, exercícios físicos, ainda mais eficazes se reali- apresentava distúrbios orgânicos que justificassem a quei-
zados ‘sob orientação especializada’ em academias ou com xa. A pesquisa revelou a expectativa medrosa do esqueci-
um personal trainer, dança de salão, moda mais jovem etc. mento como ‘normal’ na velhice, até o ponto de nenhuma

54
CONHECIMENTOS ESPECÍFICOS - PSICÓLOGO

falha, nenhum esquecimento, comum em qualquer idade, porque do ponto de vista do gênero as mulheres sempre
poder passar com tranquilidade. Trabalhos em grupo, jo- foram, tradicionalmente, avaliadas pela aparência física e
gos e exercícios mnemônicos devolveram muito da auto- pela capacidade reprodutiva. Em suma, pelo estado do seu
confiança perdida. corpo: pela beleza que possa exercer atração, pela saúde
Não se pretende ignorar as diferenças fisiológicas re- que permita reproduzir, pela docilidade de um corpo que
ais do corpo do idoso, em comparação com o dos jovens, se deixe moldar para tudo isso e também pela domestici-
mas não raro se exageram as diferenças, que nem sempre dade, objeto permanente de gestão social. Na velhice, mui-
pesam muito para o negativo. Em participação observante tos desses circuitos se perdem e elas se sentem, declarada-
em oficina ‘para a terceira idade’ na Escola de Dança da mente, mais livres (Britto da Motta, 1997, 1998) (embora,
UFBa, acompanhei, durante um mês de férias, outro expe- na sociedade de consumo, eles retornem, tentem retornar
rimento bem-sucedido – no qual, como costuma acontecer à consideração e à ação dos ‘especialistas’).
nos grupos, os homens não estavam. Vi os agradecimen- Se é verdade que as novas gerações já começam a ter
tos e o suave chorar de saudade de uma convivência e um diferentes vivências sociais – do corpo mais livre à expe-
trabalho estimulantes, em que fora possível a descoberta riência amarga e prematura da ‘inatividade’, do desem-
de que o corpo respondia a movimentos de que já não se prego –, os corpos de quem tem mais idade estão ainda
julgavam capazes. E não por enfermidade, mas porque era mais diretamente manipulados pelas estranhas estruturas
a expectativa ‘normal’. da reprodução capitalista – descartados como força de tra-
O velho – e seu corpo – é também visto como desva- balho e pretensamente dessexuados, mas teimosamente
lido, eterna clientela para as técnicas da assistência social. sexuados; trabalhando (às vezes não reconhecidamente) e
Embora comece a mudar esse tipo de atitude, dela ainda movimentando-se, hoje resistem.
se encontram expressões flagrantes e fortes, até no campo Movimentam-se, homens e mulheres, em sentidos di-
educacional, frequentemente informado por preceitos da ferenciados, conforme suas trajetórias de vida: os homens,
gerontologia, a exemplo da Proposta para a Universidade para o lazer e o ‘descanso’ (Britto da Motta, 1997) ou, em
da Terceira Idade da PUCCamp. Se aí o idoso já não aparece bem menor número, para atividades públicas políticas,
como desvalido, mas como alguém capaz de participar da
principalmente o movimento dos aposentados (Pereira et
sociedade, ao mesmo tempo o perfil traçado é, mais que
al., 1992; Simões, 1994); as mulheres, para atividades de
tradicional, desanimador: “discriminado, inativo, vivendo
mais clara liberação existencial, de lazer e cultura (Debert,
em condições precárias e situação de perda” (PUCCamp,
1994; Britto da Motta, 1998). Mas todos tendo, em comum,
1991). O que tem sido também, por muito tempo, o dis-
a intensificação ou retomada de uma universalmente dese-
curso do Estado e dos criadores e gestores das políticas
jada sociabilidade (Britto da Motta, 1999b; Peixoto, 1997).
públicas, muitos deles também gerontólogos.
Nesse movimentar-se, seu uso do corpo é retomado
em bases mais plenas. Numa aparente contradição, saem
Pesquisa realizada em Campinas por Guerrero (1994)
sobre a referida ‘universidade’ chega, entretanto, a conclu- do aprisionamento da ‘natureza’ socialmente (im)posta
sões bem mais otimistas: para – aí, sim! – usarem, ou pelo menos representarem, o
A Universidade para a 3a Idade mobiliza um público corpo de maneira mais natural ou – até o ponto em que
que se apresenta como muito ativo. (...) Estamos muito dis- a cultura ocidental permita – menos fragmentada: andam,
tantes do idoso em crise, solitário, inativo, vivendo em con- dançam, alegram-se (Britto da Motta, 1997).
dições precárias (...) tal como (...) é tratado na Proposta de Mas em novo processo contraditório, são também ins-
Ação [da PUCCamp]... (Guerrero, 1994:50) tados a submeter o corpo às já referidas ‘técnicas corporais’
Nas próprias ciências sociais, em pioneiros trabalhos (Mauss, 1974) de ginástica e ‘alimentação adequada’ para
sobre a velhice, ainda se pontuou essa desvalia, mesmo no alcançar uma ‘velhice sadia’... e uma participação privilegia-
belo e profundo estudo de Ecléa Bosi: “A mulher, o negro, da no mercado de consumo.
combatem pelos seus direitos, mas o velho não tem armas.
Nós é que temos de lutar por ele” (Bosi, 1987:39). Curiosamente, as especialidades médicas e afins que
Diante de perspectivas desse tipo, os velhos não teriam haviam realizado o já referido ‘loteamento’ do corpo das
qualquer dinamismo ou poder. Sequer gestão do próprio pessoas vêm retomando um movimento de ação total so-
corpo. bre esses corpos – não pelo simples retorno do clínico, cuja
Mas o momento, agora, começa a ser outro. Há todo falta sempre se lamentou, mas, de maneira abarcante, pelo
um processo de mudança em curso, como já estamos ven- ramo mais recente da geriatria – a qual, aliás, se duplica
do. como espécie de clínica geral, não apenas com frequência
De forma nada surpreendente, as referências ao enve- crescente, mas sobretudo porque se espraia em espesso
lhecimento e ao corpo são, ainda quando não explícitas, ‘receituário’ para se viver a velhice ‘com qualidade’ e com
feitas sobretudo às mulheres. Não apenas porque, do pon- tanto zelo que preconiza as primeiras visitas a partir dos 35
to de vista da idade, no curso da vida, elas vão-se tornando anos... Receituário que vem gerando uma expectativa social
bem mais numerosas que os homens (60% a 80% da popu- compressora sobre os que ‘não obedecem’, que deveriam
lação idosa, dependendo do estágio de envelhecimento e sentir-se culpados por estar perdendo a oportunidade de
do país em questão e apesar de o estresse contemporâneo se ‘prevenir’ do envelhecimento, pois hoje, mais do que
contribuir para aproximar essas cifras), mas principalmente nunca, ‘só é velho quem quer’...

55
CONHECIMENTOS ESPECÍFICOS - PSICÓLOGO

Há, entretanto, experiências pedagógicas valiosas com Quando admitem o impulso sexual vivo, frequente-
idosos, visando a contrabalançar as técnicas de domina- mente o remetem aos laços (indissolúveis até na morte) do
ção social, como assinalaram Lobato e Mendes (1993), na casamento:
Escola de Dança da UFBa, ao encontrarem os corpos dos Se meu velho estivesse vivo, eu estaria funcionando.
velhos, principalmente de velhas, ‘fragmentados’ pela ima- (Hercília, 85 anos) É raro este tipo de comentário:
gem social de recusa a eles. Depondo sobre esse exercício Os homens velhos, em relação ao sexo, estão ‘pimba’.
reconstrutor, Lobato (1993) detalhou em entrevista: Já a mulher, mesmo de bengala, sente prazer e não nega
Com os jovens, se trabalha do corpo inteiro para as fogo nunca. (Nadir, 75 anos)
partes. Com as idosas, que perdem a imagem do corpo
como um todo, por causa da reação da sociedade, se fez Percebe-se, nas próprias citações, o lugar afetivo-social
o caminho inverso: partimos da parte para o todo; parti- de onde falam os representantes de cada gênero, corres-
mos das extremidades. Primeiro os pés, porque pisam e pondendo às expectativas sociais que sempre houve em re-
sustentam o corpo, e em seguida as mãos, muito usadas lação a eles ou nunca houve em relação a elas. Os homens
no trabalho. falam da preocupação com o desempenho e a liberação do
Os homens não se interessam em participar desses prazer. As mulheres, da cobrança de ‘seriedade’; do alívio
trabalhos, informou Lobato. Preferem reunir-se em outros em relação ao marido autoritário, a ponto de não dese-
lugares e atividades, como evidenciou a pesquisa. E em um jar outro e até abdicar da vida sexual; ou simplesmente do
desses lugares vamos encontrá-los. desejo de uma liberdade antes nunca experimentada; ou,
Em uma praça de um bairro periférico de Salvador, enfim, da situação, extrema e sempre lamentada, de não
como em muitas outras praças da cidade, reúne-se diaria- ser desejada sequer pelos homens da sua própria geração.
mente um grupo de homens idosos, quase todos aposen- No ‘inverno’ da vida, os homens ainda tentam (re)pro-
tados, para conversar e ver o movimento dos passantes. duzir ‘tempestades’ afetivosexuais; as mulheres silenciam
Ouvi-los é uma oportunidade privilegiada de checar ou ou ‘congelam’ ciosamente (ou cansadas, pelo muito que
superar noções preconcebidas a respeito dos velhos. Num foram manipuladas em sua trajetória de vida) a sexualida-
dos bancos dessa praça, jovens pintaram a frase: “Cuidado: de; a ‘plateia’ acha graça (afinal, ‘quem gosta de velho é
banco do pau mole”. Os idosos fazem piadas com quem se reumatismo’).
Agora estamos, finalmente, no limiar de um reconhe-
senta lá (até com o jovem entrevistador da nossa equipe),
cimento social dos idosos, seja por uma imagem social de
sem aparentemente estarem preocupados com a pecha,
dinamismo que eles vêm construindo, seja pela sua partici-
mas nas suas falas, dentro e fora das entrevistas, demons-
pação em grupos (Britto da Motta, 1999a), seja pela cons-
tram estar muito voltados para o sexual (Ribeiro Júnior,
tante visibilidade a eles dada pela imprensa. Entretanto,
1995). Com alguma tristeza e risos compensatórios, falam
ainda que estejam progredindo muito, continuam pouco
sobre as mulheres que já não conseguem conquistar (“Me
inseridos nos grandes circuitos das relações sociais. São
respeita, velho”) como perda. Medem, ainda, suas possibili-
ainda, como expressou Birman (1995:43), “sujeito em sus-
dades, talvez ampliando-as um pouco:
pensão”. Na expectativa de um lugar que possa ser novo e
Sexualmente, dá para três vezes por semana; quando mais satisfatório e que dependerá precipuamente da con-
tá jovem, ele quer todo dia. (Antônio, 68 anos) solidação de uma identidade coletiva de idosos que apenas
Referem-se, também, à vida dos outros: começaram a construir, principalmente nos seus grupos, e
A mulher daquele ali [aponta] disse: ‘Tem mais de cinco de uma ação política que se imponha à sociedade.
anos que não ‘dorme’ comigo’.
Criticam os companheiros, mas também as mulheres: Fonte:
O homem, na velhice... se encolhe muito. A mulher MOTTA, A.B. Envelhecimento e Sentimento do Corpo
quer se ‘espalhar’, mas não pode. A mulher não se acha in: Antropologia, Saúde e Envelhecimento Rio de Janeiro:
velha, fica falando para os outros que os maridos não dão Fiocruz Ed., 2011.
pra nada... MOTTA, AB. Envelhecimento e sentimento do corpo. In:
João, 58 anos, vendo a moça bonita que passa, ri: MINAYO, MCS., and COIMBRA
Ser velho é isso. Só fica olhando. JUNIOR, CEA., orgs. Antropologia, saúde e envelhe-
As idosas, fiéis à trajetória feminina tradicional, costu- cimento [online]. Rio de Janeiro: Editora FIOCRUZ, 2002.
mam silenciar sobre sua sexualidade. Claro, se antes era Antropologia & Saúde collection, pp. 37-50. ISBN: 978-85-
tabu, se não tinham o hábito dessa confidência, por que se 7541-304-3.
sentiriam à vontade agora?
Instadas a falar, suas declarações são de renúncia ou
desesperança:
Eu sou uma senhora de idade, fico no meu lugar. (Maria
Pureza, 61 anos)
Já sei o gosto, já me agradei. Mas meu marido era mui-
to ciumento. (...) Eu quero é poder me divertir... (Augusta,
92 anos)
Hoje eu já tô velha, ninguém olha. (Matilde, 72 anos)

56
CONHECIMENTOS ESPECÍFICOS - PSICÓLOGO

Definição de violência contra criança e adolescente


ASPECTOS PSICOSSOCIAIS DO
FENÔMENO DA VIOLÊNCIA. A violência contra criança e adolescente é definida
como:
(...) todo ato ou omissão praticado por pais, parentes
ou responsáveis contra crianças e ou adolescentes que –
VITIMIZAÇÃO sendo capaz de causar dano físico, sexual, psicológico à
vítima - implica em um lado numa transgressão de poder/
dever de proteção do adulto e, de outro, numa coisificação
Considerações iniciais: da infância, isto é, numa negação do direito que a criança
ou adolescente têm de ser tratado como sujeitos e pes-
Um Processo Verificatório pode trazer subjacente a soas em condição peculiar de desenvolvimento. (Azevedo
violência contra crianças e adolescentes, violência essa que e Guerra, 1984).
pode ou não estar associada a questões estruturais e à au- A violência intrafamiliar ocorre quando existe algum
sência ou insuficiência de políticas públicas. A intervenção laço familiar (direto ou não) ou ainda quando existe res-
dos assistentes sociais e psicólogos exige conhecimentos ponsabilidade do agressor sobre a vítima. O agressor é
específicos para se apreender a realidade exposta nas rela- uma pessoa que com ela convive e em quem ela confia e,
ções intra e extrafamiliar. frequentemente, ama.
Em primeiro lugar, é necessário ter presente que a vio- A violência extrafamiliar ocorre fora âmbito familiar.
lência contra a criança e adolescente é uma das formas O abusador é, na maioria das vezes, alguém que a criança
mais graves de violência, posto que os adultos encarrega- conhece: vizinho ou amigos da família, educadores, res-
dos da sua proteção e cuidado são precisamente aqueles ponsáveis por atividades de lazer, médicos, religiosos, etc.
que os agridem. A violência institucional é aquela que acontece den-
A situação se agrava ainda mais, pela dependência físi- tro das instituições governamentais ou não governamentais
ca, emocional e econômica quase que total que estas crian- encarregadas de prover cuidados substitutivos da família:
ças e adolescentes têm com seus responsáveis. abrigos, escolas. Pode ocorrer entre as próprias crianças e
As famílias abusivas geralmente funcionam como um adolescentes ou entre estes e profissionais da instituição
Destacam-se aqui os conceitos sobre os diferentes
sistema fechado, refratário ao convívio social, impermeá-
tipos de violência para melhor embasamento da questão
vel às trocas. Dificilmente se integram com as pessoas da
tratada:
comunidade. O motivo deste isolamento está intrinseca- - Violência física - é o uso da força física de forma
mente associado à forma de manter o segredo da violência intencional, não acidental por parte dos pais ou responsá-
vivida. veis de crianças ou adolescentes, com o objetivo de ferir,
Na prática, percebe-se que as figuras parentais se danificar e até mesmo destruir, deixando ou não marcas
distanciam de suas funções corriqueiras e acabam fican- evidentes. (GUERRA, 1985)
do igualmente afastadas dos sistemas extrafamiliares, tais - Violência psicológica - é toda ação ou omissão que
como escola, festividades e demais eventos sociais. causa ou visa causar dano à autoestima, à identidade ou
Também inibem o estreitamento das relações entre a ao desenvolvimento da pessoa. Incluem ameaças, humilha-
criança/ adolescente e seus colegas, e deles com outros ções, chantagem, descriminação, isolamento, rejeição etc. É
adultos que possam perceber a problemática existente. mais difícil de ser identificada, apesar de ocorrer com fre-
Devem-se levar em consideração delicadas situações quência. (MINISTÉRIO DA JUSTIÇA - 2001)
como: - Negligência - se refere à falta de proteção e cuidado
- O receio da criança/adolescente de ser punida pela mínimo por parte de quem tem o dever de fazê-lo. Existe
figuras parentais; negligência quando os responsáveis pelo cuidado ou edu-
- A angústia de vir a ser abrigada; cação das crianças e adolescentes não atendem ou satisfa-
- O medo das consequências advindas da regulação zem as necessidades básicas, sejam elas, físicas, psicológi-
social, por parte do Judiciário; cas, sociais e intelectuais. (UNICEF, 2007)
- Temor de o acusado ser afastado de casa pelo juiz da - Abandono - é o grau extremo de negligência por
infância, parte dos adultos. (UNICEF, 2007)
- Receio de o agressor vir a ser preso. - Violência sexual - é todo tipo de ato ou jogo sexual,
relação hetero ou homossexual, cujo agressor esteja em
Outros aspectos a serem ressaltados dizem respeito: estágio de desenvolvimento psicossocial mais adiantado
que a criança ou adolescente. Tem por intenção estimulá-
A re-vitimização que ocorre quando a criança/adoles-
-la sexualmente ou obter satisfação sexual. (FIOCRUZ/ENS/
cente é atendida por diversos serviços e profissionais sen-
CLAVES, 1994).
do obrigada a repetir sua história violenta inúmeras vezes; - Exploração sexual - é a utilização de crianças ou
O descompasso temporal existente entre os procedi- adolescentes com intenção do lucro (pode entender tam-
mentos inerentes a proteção e a responsabilização. A famí- bém comercialização) seja financeiro ou de qualquer outra
lia atravessa inúmeras crises, tentando restabelecer o equi- espécie. Quase sempre existe a participação de um (a) ali-
líbrio, e quando consegue certa acomodação, novamente ciador (a). Pode ser compreendida através de quatro tipos:
recebe intimação da Vara Criminal, obrigando-a a reviver prostituição, pornografia, turismo sexual e tráfico. (ABRA-
o conflito. PIA, 2002).

57
CONHECIMENTOS ESPECÍFICOS - PSICÓLOGO

FORMAS DA VIOLÊNCIA SEXUAL Trabalho sexual infanto-juvenil autônomo é a venda


de sexo realizada por crianças e adolescentes. Muitas crian-
O abuso sexual intra e/ou extrafamiliar pode se expres- ças e adolescentes, mais adolescentes do que crianças, de
sar de diversas formas: Abuso sexual sem contato físico - ambos os sexos, se engajam em trabalho sexual e fazem
são práticas sexuais que não envolvem contato físico: dele sua principal estratégia de sobrevivência.
Assédio sexual caracteriza-se por propostas de rela-
ções sexuais. Baseia-se, na maioria das vezes, na posição Trabalho sexual infanto-juvenil agenciado é a venda
de poder do agente sobre a vítima, que é chantageada e de sexo intermediada por uma ou mais pessoas ou servi-
ameaçada pelo autor da agressão (ABRAPIA, 2002). ços. No primeiro caso, essas pessoas são chamadas rufiões,
Abuso sexual verbal pode ser definido por conversas cafetões e cafetinas e, no segundo, os serviços são normal-
abertas sobre atividades sexuais destinadas a despertar mente conhecidos como bordéis, serviços de acompanha-
o interesse da criança ou do adolescente ou a chocá-los mento, clubes noturnos. Os trabalhadores sexuais pagam a
(ABRAPIA, 2002). essas pessoas ou serviços um percentual do que ganham
Telefonemas obscenos são também uma modalidade em troca de residência, pensão alimentar, roupas, transpor-
de abuso sexual verbal. A maioria deles é feita por adultos,
te, maquiagem e proteção durante a realização do traba-
especialmente do sexo masculino. Podem gerar muita an-
lho. Normalmente os profissionais do sexo se transformam
siedade na criança, no adolescente e na família (ABRAPIA,
em reféns de seus agenciadores, caracterizando uma rela-
2002).
ção de exploração ou de semi-escravidão.
Exibicionismo é o ato de mostrar os órgãos genitais
ou se masturbar diante da criança ou do adolescente ou
no campo de visão deles. A experiência, contudo, pode ser Turismo orientado para a exploração sexual é comu-
assustadora para algumas crianças e adolescentes (ABRA- mente conhecido como “turismo sexual” ou “sexo-turismo”.
PIA, 2002). Caracteriza-se, por um lado, pela organização de “excur-
Voyeurismo é o ato de observar fixamente atos ou ór- sões” turísticas, com fins não declarados de proporcionar
gãos sexuais de outras pessoas quando elas não desejam prazer sexual a turistas estrangeiros ou de outras regiões
serem vistas e obter satisfação com essa prática. A expe- do país e, por outro lado, pelo agenciamento de crianças e
riência pode perturbar e assustar a criança e o adolescente adolescentes para oferta de serviços sexuais.
(ABRAPIA, 2002).
Abuso sexual com contato físico são atos físico-genitais Tráfico para fins de exploração sexual de crianças
que incluem carícias nos órgãos genitais, tentativas de re- e adolescentes é uma das modalidades mais perversas de
lações sexuais, masturbação, sexo oral, penetração vaginal exploração sexual. A prática envolve atividades de coop-
e anal. Pornografia é a exposição de imagens eróticas de tação e/ou aliciamento, rapto, intercâmbio, transferência
pessoas ou focando partes de corpos, de práticas sexuais e hospedagem da pessoa recrutada para essa finalidade.
entre adultos, adultos e crianças, entre crianças ou entre Todavia, o mais recorrente é que o tráfico para fins de ex-
adultos com animais, sendo estes expostos em revistas, li- ploração sexual de crianças e adolescentes ocorra de forma
vros, filmes e, principalmente, na internet. “maquiada” por agências de modelo, turismo, trabalho in-
Essa tecnologia tem servido como elemento facilitador ternacional, namoro-matrimônio. Muitas jovens, seduzidas
para criação de clubes de pedofilia ou vendas de pornogra- por uma rápida mudança de vida ou sucesso fácil, embar-
fia infantil. A pornografia envolvendo crianças e adolescen- cam para outros estados do país ou para outros países e
tes é considerada crime, tanto de quem fotografa crianças lá se vêem forçadas a entrar no mercado da exploração
nuas ou expõe suas imagens em posições sedutoras com sexual.
objetivos sexuais, como de quem mostra às crianças fotos,
vídeos ou cenas pornográficas.
A AVALIAÇÃO SOCIAL E PSICOLÓGICA NOS CASOS
Pornografia é um produto com fins comerciais desti-
DE VITIMIZAÇÃO
nado a provocar estímulo sexual dos indivíduos.
A pornografia infantil atende a uma demanda de mer-
Diante do referencial teórico exposto, fica ainda mais
cado em que certas pessoas, em geral pedófilos, sentem
prazer em seu consumo. evidente a necessidade de que toda intervenção em situa-
ções de violência tenha sempre presente o interesse supe-
Trocas sexuais é a oferta de sexo para obtenção de rior da criança e adolescente.
outros favores. Muitas crianças e adolescentes que fogem Segundo Vieira (2006), os tempos atuais viram cessar
de casa, que vivem nas ruas, mantêm relações sexuais com o ocultamento do abuso sexual. Segundo o estudioso no
adultos em troca de comida, de uma noite de sono num assunto, a revisão da literatura específica autoriza-o men-
hotel ou para adquirir sua quota de drogas. Crianças e ado- cionar que na atualidade não se fala mais em fantasia, mas
lescentes de classe média podem também trocar sexo por sim se é verdade ou mentira, ou ainda delírio dentro de um
drogas ou produtos “de marca” (roupa, tênis, etc). Essas quadro psicótico (o que é raro).
práticas são eventuais e realizadas juntamente com outras Sua experiência como psicólogo judiciário conclui, no
estratégias de sobrevivência, em que as trocas sexuais não entanto, que as pessoas devem apurar sua formação para
predominam ou predominam apenas temporariamente, ou aceitar a realidade dos fatos e conseguir oferecer à criança
não há ação continuada de trabalho sexual. uma ajuda mais adequada.

58
CONHECIMENTOS ESPECÍFICOS - PSICÓLOGO

Para a avaliação de uma situação de violência e, por- ou adolescente tenha respeitado os seus direitos enquanto
tanto, do risco, o assistente social e o psicólogo judiciário um ser em desenvolvimento e que necessita da convivência
devem sempre observar e levantar dados acerca da relação familiar e comunitária.
existente entre as crianças e/ ou adolescentes com os seus Recomenda-se que os profissionais da Vara da Infância
pais ou responsáveis, bem como aspectos destes respon- e Juventude solicitem relatórios trimestrais dos serviços de
sáveis, do entorno familiar, da violência propriamente dita, atendimento até que se verifiquem mudanças significativas
da vulnerabilidade e da resposta da família à intervenção no sistema familiar.
técnica realizada.
O parecer técnico deverá levar em consideração os AVALIAÇÃO PSICOLÓGICA
princípios Universais da Criança e do Adolescente que es-
tabelece que: A vitimização sexual envolve todas as pessoas da famí-
A criança gozará proteção contra quaisquer formas de lia como num sistema. Dessa forma, deve-se atentar não
negligência, crueldade e exploração. Não será jamais obje- só para subsistema vítima abusador, mas incluir uma com-
to de tráfico, sob qualquer forma. preensão abrangente, circular, não- linear, de todo o macro
[Princípio 9º da Declaração Universal dos Direitos da sistema.
Criança de 1959] Assim, a avaliação pode ser realizada não só por meio
Assim como o que está previsto no artigo 130 do ECA, de entrevistas individuais com os membros da família nu-
em que determina que se constatada a hipótese de maus- clear e extensa, como também atender em subgrupos, des-
-tratos, opressão ou abuso sexual que foi imposto pelos de que se tenha o cuidado de não se confrontar a vítima
pais ou responsável, o juiz poderá determinar como cautela com o agressor.
o afastamento do agressor da moradia comum. No atendimento às famílias abusivas, percebe-se uma
maneira peculiar de funcionamento, no qual todos os inte-
A denúncia que rompe “o complô do silêncio” nas
grantes estão envolvidos de alguma forma, seja de maneira
famílias abusivas pode ser comunicada a diversas insti-
passiva ou ativa no ato da violência.
tuições: Conselho Tutelar, Delegacia, Unidades Básicas de
No cenário da vitimização apresentada no cotidiano de
Saúde - UBS, Hospitais, Escolas e ainda os profissionais dos
nossa prática observa-se a participação de no mínimo três
Abrigos.
personagens nesta trama complexa e dramática: o agres-
Necessário se faz criar um fluxograma do sistema de
sor, a mãe – passiva e geralmente conivente ao abuso - e
notificação e atendimento, visando à possibilidade de ime-
a vítima.
diato acolhimento da criança, adolescente e de sua família
Existem certos mitos sobre a instituição familiar que
para a rápida elaboração do diagnóstico e possíveis des- podem indicar alguns motivos de se passar despercebida
dobramentos jurídicos (guarda, abrigamento, afastamento por tanto tempo a dinâmica abusiva. O mito de que a fa-
do autor da violência) e encaminhamentos para tratamento mília é intocável, sagrada e que, portanto, ninguém deve
nos serviços do Poder Executivo. interferir.
A forma de atuar do assistente social e do psicólogo Muitas vezes ouvimos em entrevista, a vítima mencio-
nos processos que envolvem violência intra ou extrafami- nar que havia compartilhado o segredo com algum adulto
liar exige contínuo aprofundamento no tema, no sentido de seu relacionamento, mas este não conseguiu protegê-la.
de aferir com acuidade as nuances de cada caso. Outro aspecto da assimetria de poder é noção difundi-
Nos últimos anos, observamos que vem ocorrendo o da no senso comum de que os pais devem educar usando
reconhecimento da importância do trabalho interdiscipli- a força física e que também podem fazer o que bem enten-
nar, tanto na rede interna (entre assistente social, psicólo- derem com os filhos. Desta forma, os genitores não conse-
go, promotor e juiz), quanto na externa (rede de proteção). guem reconhecê-los como sujeito de direitos e ao menos
O trabalho em rede é indispensável quando se tratar de perceber seus sofrimentos psíquicos.
atendimento à violência, pois sendo esta multifacetada há Embora a denúncia seja um procedimento determi-
necessidade de diferentes serviços e áreas de atendimento, nado pelo Código de Ética da Psicologia, a partir da Reso-
como saúde, jurídico, psicológico, social, educacional, etc. lução CFP 010/2005, item II dos Princípios Fundamentais e
Nos casos de vitimização, tanto aqueles que tiveram em seu artigo 10; e pelo ECA, art. 245, a subnotificação é
necessidade do afastamento vítima-agressor, como aque- uma realidade em nosso país; esta ocorre devido a vários
les em que a criança ou adolescente permanece convi- fatores, como sentimento de culpa, vergonha por parte da
vendo com os familiares e, por vezes, com o agressor, há vítima e de seus familiares; receio dos parentes e/ou vi-
necessidade de ser o grupo encaminhado para atendimen- zinhos das possíveis repercussões (polícia e judiciário), a
to especializado. Isto porque, como já foi explicado ante- resistência de alguns médicos, enfermeiros, professores,
riormente, a violência perpassa por todo grupo e para que psicólogos, assistentes sociais que lidam com a problemá-
haja uma mudança no padrão relacional há necessidade de tica em reconhecê-la e relatá-la.
atenção a todos. Outro dilema é a insistência na constatação de provas
Ocorrendo este atendimento, sugere-se que haja materiais exigidas por alguns juristas que buscam evidên-
aproximação e troca de informações entre o judiciário e cias físicas (não encontradas nos casos de assédio, caricias
o serviço especializado. É importante que haja a discus- dos genitais, e exposição à pornografia e outros anterior-
são e reavaliação do caso, visando garantir que a criança mente citados).

59
CONHECIMENTOS ESPECÍFICOS - PSICÓLOGO

Desta forma, a avaliação psicológica toma vulto neste Se a criança ou o adolescente tiver que ser abrigado,
tipo de violência tão silenciosa e sem testemunhas. Deve o encaminhamento para a terapia deve ser providenciado
ser empreendida uma investigação técnica criteriosa da com urgência, assim como é imprescindível o acompanha-
vítima e de seus familiares, e neste sentido, além da inclu- mento criterioso pelos técnicos do Judiciário, lembrando
são evidentemente de um trabalho em rede que privilegie que esta é uma medida excepcional e transitória até que
ações preventivas e terapêuticas, a capacitação profissional encontrem soluções mais saudáveis para o seu desenvol-
torna-se um instrumento valioso na luta pela preservação vimento.
dos direitos fundamentais da criança e do adolescente. Nos atendimentos nas varas, percebe-se que as figu-
Diante das suspeitas de vitimização física, psicológica ras parentais em sua grande maioria já sofreram algum
ou sexual observam-se alguns sinais importantes: diver- tipo de vitimização (física, psicológica, sexual, negligência,
gência entre os diversos relatos, hesitação para prestar abandono), em suas famílias de origem e as repetem na
as informações; a demora em buscar atendimento para a atualidade.
criança/adolescente agredida, pois quando o dano é pro- Durante o processo avaliativo é possível identificar,
duzido pela violência, os responsáveis relutam em buscar analisar e relacionar aspectos da estrutura e dinâmica das
auxílio. famílias quanto às regras familiares, papéis familiares, mi-
Observam-se histórias repetidas de acidentes ou evi-
tos e segredos, padrões de repetição, relações hierárquicas,
dências de traumas freqüentes. E ainda: atraso no desen-
padrões de afetividade (alianças e coalizões).
volvimento psicomotor, evasão escolar (visando à manu-
As escolhas afetivas inconscientes dos adultos foram
tenção do complô do silêncio) e atitude de distanciamento
dos pais ou responsáveis quando da observação da intera- determinadas pela história da família de origem e refletem
ção pai-filho, mãe-filho. repetições, como explicam a psicanálise e a terapia familiar.
A partir de uma análise deste grupo social, constata- A primeira escola considera que a carga pulsional é ge-
mos padrões de comportamentos que incluem certa au- nética, as pulsões de vida e de morte são transmitidas de
sência na delimitação entre as fronteiras parentais e filiais, modo inconsciente, associada às características peculiares
típicas da família emaranhada. da relação mãe-filho e, por vezes, pouco consciente, que
Geralmente, ocorre uma fusão, e dificuldade de dife- não ofereceu subsídios para esta pessoa, enquanto bebê,
renciação entre os indivíduos desta família e de suas famí- se diferenciar do outro e tornar-se capaz de resistir às frus-
lias de origem. Frequentemente o acusado é descrito como trações durante seu desenvolvimento pela vida.
“muito apegado à filha/o” (vítima), em consequência do Assim, na maturidade este indivíduo escolhe alguém
afeto se dar de forma bastante erotizada. complementar ao seu quadro, que também pode ter tido
E é muito comum a vítima ser descrita como “criadora um frágil acolhimento de seus pais, que não tinham cons-
de caso”, como àquela que traz problemas para a família, ciência e intenção de serem maléficos na transmissão de
que tem mente fantasiosa, e que portanto, não deve ser modelos identificatórios como figuras parentais.
levada em consideração. Neste sentido, Minuchin enfatizou que, “A terapia fa-
A relação afetiva e sexual do casal parental geralmente miliar também desafiou a crença na autodeterminação do
é distante, desvitalizada. Quando abordado o tema da se- self, ao iluminar o poder da família. Ela reconheceu homens
xualidade entre os adultos, as partes chegam a verbalizar e mulheres como partes de um todo mais amplo - como
que as relações sexuais eram esporádicas e sem vigor. subsistemas, mas subsistemas significativos, de sistemas
mais amplos”. (Minuchin, 1990, p.38)
Percebe-se ainda um primitivismo, infantilismo, traços Na obra freudiana a criança é trabalhada como sujeito
característicos da pedofilia. A pulsão, o desejo do agres- de desejo, onde o desejo está presente desde a infância.
sor torna-se preponderante à dor a ser infringida à crian- (Duarte, 2007, p.112). Nas pessoas normais e neuróticas, o
ça. Nesta linha é constatado o mito falocêntrico de que a desejo infantil se relaciona com uma fantasia e ele jamais
mulher deve servir o homem na cama e na mesa, então a
é satisfeito e será mais facilmente reconhecido quando na
vítima é escolhida para preencher esta falta.
maturidade a pessoa tiver oportunidade de se submeter à
As mensagens de comunicação entre a família abusiva
análise.
e a sociedade seguem o modelo de um sistema fechado,
em que as trocas de informações são filtradas, reprimidas, Nestes casos de vitimização sexual, a criança é colo-
sem espontaneidade, como uma forma de manter e con- cada em uma situação patológica, em que seu desejo ao
trolar o segredo da violência. invés de ser interditado, ao contrário, é atuado, trazendo
Assim é mantida a homeostase do sistema familiar, evidentemente muita angústia, culpa, baixa autoestima,
que é definida como um processo de autorregulação e que autoaniquilamento, sintomas decorrentes do mau funcio-
mantém a estabilidade do sistema, protegendo-o das mu- namento das relações intrafamiliares.
danças que pudessem destruir sua organização, de modo a Percebe-se uma extrema permissividade, dificuldade
preservar o seu funcionamento. das figuras parentais em colocar limites, as fronteiras e os
Quando o caso ingressa no judiciário e muito prova- papéis são confusos. A genitora por ser desvitalizada, frágil,
velmente já passou pelo conselho tutelar, delegacia ou passiva, porém as relações são sempre interativas e dinâ-
outras instituições, a dinâmica familiar sofreu um impacto, micas.
está totalmente alterada, todos estão assustados e com os Para a psicanálise, o psicodrama, a gestalt terapia, a
mecanismos de defesa acirrados. A prática comprova que terapia familiar sistêmica dentre outras teorias, a instituição
o acusado tende a negar na maior porcentagem dos casos. familiar possibilita um amplo campo de investigações e re-

60
CONHECIMENTOS ESPECÍFICOS - PSICÓLOGO

flexões, oferecendo a possibilidade de buscar esclarecer as tuados a escutar. Criação ou destruição: do que se tratam
complexas relações afetivas que ocorrem em seu interior, as transgressões? Do que se tratam as violentas manifesta-
como, por exemplo, as que dizem respeito ao exercício da ções juvenis?
maternidade e a paternidade. Enriquez, a partir de Canguilhem, tece reflexões impor-
Cabe ressaltar que dentre os diversos instrumentos à tantes para fundamentar essas questões.
disposição da Psicologia, os testes são ferramentas ainda “A vida é constituída de todas as forças que se opõem
muito utilizadas pelos profissionais do judiciário. à morte” (Canguilhem, 1943). Em outras palavras o ser hu-
No entanto, em face das recentes discussões que vem mano luta contra a entropia que espreita constantemente
sendo travadas sobre a fidedignidade destes. Conforme já o mundo físico e o mundo vivo, e é criador de neguentro-
sugerido é importante que os psicólogos consultem re- pia. Ela torna presente o “querer viver” schopenhaueriano e
gularmente os sites do Conselho Regional de Psicologia e nietzschiano, e a vitória da pulsão de vida sobre a pulsão de
Conselho Federal, para que se atualizem sobre resoluções morte. Mesmo quando está misturada com algum pathos
que regem o exercício da Psicologia, bem como a lista de (de sofrimento), e se às vezes se desvia no patético, ela faz
testes atualizados (www.pol.org.br e www.crpsp.org.br). surgir novos valores, pois o homem, como diz Nietzsche
“uma coisa que pode ser superada” e, acrescentemos, a
Bibliografia única que tem essa característica (...) Assim, como escreveu
BRASIL. Atuação dos profissionais de Serviço Social e Bataille (1975), “a transgressão não é a negação da proi-
Psicologia. Manual de procedimentos técnicos Tribunal de bição, ela a ultrapassa e a completa”. (Enriquez, 2002, p.
Justiça do Estado de São Paulo. v.i. Infância e Juventude 122-3)
Isto posto, retomemos da tradição psicanalítica o con-
VIOLÊNCIA, TRANSGRESSÃO, PULSÃO: MANIFES- ceito fundamental de pulsão – Trieb. Como aponta Luís
TAÇÕES DO PATHOS. Hanns
... Trieb, tal qual é usado em alemão, entrelaça quatro
Condições para subjetivação momentos que conduzem do geral ao singular. Abarca um
princípio maior que rege os seres viventes e que se manifesta
Este questionamento se faz a partir da posição da Psi- como força que coloca em ação os seres de cada espécie;
copatologia Fundamental que está interessada num sujei- que aparece fisiologicamente “no” corpo somático do sujei-
to trágico que é constituído e coincide com o pathos... O to como se brotasse dele e o aguilhoasse; e por fim, que se
que se figura na tragédia é pathos que, no sentido clássico, manifesta “para” o sujeito fazendo-se representar ao nível
quer dizer tudo o que se fez ou que acontece de novo, do interno e íntimo como se fosse sua vontade ou um impera-
ponto de vista daquele ao qual acontece. Nesse sentido, tivo pessoal. No texto freudiano também a palavra mantém
quando pathos acontece, algo da ordem do excesso, da essas características de uso. Trieb evoca a idéia de força po-
desmesura, se põe em marcha, sem que o eu possa se as- derosa e irresistível que impele. (Hanns, 1996, p. 338)
senhorear desse acontecimento, a não ser como paciente, Radmila Zygouris (1999) comenta: “A pulsão em sua
como ator. (Berlinck, 1998) origem não é, portanto, nem boa nem má, ela só procura
Ora, encontramos no sentido primeiro da palavra vio- satisfação: é apenas devido à inadequação do objeto e im-
lência a idéia de “irrupção de uma força intensa que deixa o potência do sujeito em encontrar o objeto adequado que
sujeito submisso aos efeitos dessa força sem que ele possa ela se torna destruidora do objeto tanto quanto do sujeito,
se livrar dela” (Toubiana, 1997, p. 167). Essa concepção re- tentando manter um estado de menor tensão” (p. 15). Diz
toma, no fundo, a etimologia do termo violência, que vem ainda a autora: “Desejar, falar, pensar, sonhar, fazer. Tudo
do latim, em que vis quer dizer a força. Aliás, Jean Bergeret isto implica nas pulsões de vida, implica no ato de se estar
pesquisa cuidadosamente o termo – demonstrando que na frente a frente com um outro, de ir em direção ao outro e
origem apresenta a idéia de vida sem a conotação destru- abre um caminho para estabelecer uma relação com o ou-
tiva ou erótica e com a qual estamos familiarizados (Ber- tro, uma relação de objeto, como é chamada” (p. 25).
geret, 1995). Violência apresenta também na língua por- No entanto, observa-se o quanto tem sido difícil, nos
tuguesa um caráter polissêmico, que compreende desde dias de hoje, a disponibilidade dos sujeitos em se fazerem
a noção de ação violenta até a idéia de impetuosidade ou “objetos” e enfrentarem essa violência do movimento pul-
veemência, passando pelas noções de constrangimento, ou sional, chegando a abandonar seus parceiros significativos
contrariar as normas, violar, etc. (filhos, cônjuges, alunos, etc.) à solidão e ao desamparo.
O mesmo se dá com o termo transgressão. Nos di- “O ato de crueldade é seu último avatar (do Jogo Pulsio-
cionários Aurélio e Larousse Cultural encontramos: “1. Ato nal – Eros e Tânatos), última tentativa de um fazer erótico,
ou efeito de transgredir, infração, violação – sendo que no de um ir em direção ao outro, nem que seja para destruí-
verbete transgredir, aparece a definição de passar além, in- -lo; quando na falta desse outro inacessível, o ‘eu’ tornado
fringir, atravessar, violar uma norma 2. (Geol.) Invasão do impotente, se abraça a si próprio num último gesto de po-
mar que acarreta a formação de depósitos marinhos onde tência” (Ibid., p. 25).
dantes era continente, provocando transformações na cos- A proposta da Psicopatologia Fundamental, na tradição
ta litorânea”. Portanto, analisando esses termos, depara-se do método clínico, “busca uma classificação compreensiva
com sua polissemia e a extensão de seus significados traz tão exclusiva que se confunde com a própria subjetividade
uma situação paradoxal a qual psicanalistas estamos habi- de cada um” (Berlinck, s/d, p. 11). Essa compreensão, como

61
CONHECIMENTOS ESPECÍFICOS - PSICÓLOGO

apontamos, na sua dimensão trágica, aproxima-se justa- Enriquez (1999) comenta que a obrigação do sofrimen-
mente do caráter violento que a emergência do sujeito, ou to para inscrever a sociedade no corpo não significa “Não
seja, que a expressão subjetiva apresenta. terás o desejo de poder” e sim “Não te revoltarás, aceitarás
a lei do pai (...) deves tornar-te um ser para sempre submis-
A adolescência e o drama da subjetivação so, que não deve mais ser inovador (como na época em
que eras jovem e louco), que não pode ser portador da dis-
Na perspectiva da análise proposta pode-se dizer que córdia. E, um dia, tu também, nesta condição, poderás fazer
o processo de subjetivação sempre retoma a questão de- outros sofrerem. Sofrer é então, aceitar não mais rivalizar
sesperada do sujeito de se fazer único e de ser reconhecido com o pai, é aceitar no corpo a marca da castração simbóli-
pelo mundo. ca, sofrer é entrar no mundo de submissão e de dominação
Nada melhor que a adolescência para nos lembrar dis- (...). Tornar-se homem é entrar nas relações assimétricas”,
so. Propõe-se assim considerar o adolescente o paradigma conclui Enriquez (p. 210-1).
do sujeito humano, ou seja, do drama de sua subjetivação A condição para tornar-se homem, analisada por Enri-
– como colocar-se na rede social mantendo singularidade quez, parece absurda na contemporaneidade, onde impera
e autonomia; ou, dito de outra forma, nas palavras de P. o narcisismo e o princípio do prazer é praticamente um
Jeammet, “o paradoxo chave do desenvolvimento humano regulador social.1 Sendo assim a idéia de sofrer é anacrô-
– aquele da necessidade de conciliar a necessidade fun- nica e a sensação de submissão intolerável. Como o jovem
damental de identificação e de se alimentar dos objetos irá então se situar em nossa sociedade? Como será reco-
investidos, com a necessidade de salvaguardar a autono- nhecido e acolhido? Numa sociedade que parece valorizar
mia narcísica” (1995, p. 89). Penso que é por isso que o tanto a infância e a juventude, o que significa a adesão à
adolescente incomoda: ele retoma dia-a-dia nosso conflito proposta de encarcerá-los precocemente?
essencial. Ele luta, não desiste; busca sempre transgredir e Poder-se-ia analisar que se trata de uma projeção de
por isso expõe nossa violência recalcada. nossa violência sobre uma classe social menos favorecida,
Renúncia narcísica, vicissitudes edípicas, parecem ser para quem a única possibilidade de reconhecimento numa
um tema tão banal em psicanálise, mas por que ele per- sociedade que a exclui cotidianamente é retornar com vio-
siste? O tema violência e a questão do adolescente que lência. Nessa perspectiva é possível entender seguindo a
hoje se articulam tão fortemente na sociedade – e que a tradição winnicotiana, a delinquência como mecanismo
psicanálise não pode se furtar a entender –, retomam sem de denúncia social: afirmação do Eu, a busca do mundo
dúvida esse drama fundante do humano: “A gestão de sua que foi perdido, rompido abrupta e arbitrariamente. Se
violência interna, seus votos incestuosos e parricidas, de pensarmos na sociedade contemporânea que acena com
sua agressividade legítima e estruturante, assim como às promessas de consumo ilimitadas e de prazer imediato, o
expressões problemáticas de suas pulsões de morte”. ato delinquencial não deixa de ser a denúncia de uma im-
É na adolescência, sabemos, o momento de romper, postura. Nesse ponto as classes sociais menos privilegiadas
decididamente, com o ideal narcísico dos pais, constituin- encontram uma desvantagem.
do, para além de suas determinações, a possibilidade de
um ideal de Eu. Ou seja, é momento de subjetivar-se, acei- O adolescente espelhando o ideal social: afirmação
tando as limitações sociais, posicionando-se criativamente. narcísica
É tempo de buscar, em novas relações, recuperar “amores
perdidos”, fundar-se num corpo que foi amado e que po- Considerando então esse cenário, entende-se que na
derá amar, que foi gerado e poderá gerar, enfim, de en- sociedade contemporânea, com o triunfo do individualis-
trar na realidade social, simbólica, com nome próprio. Esse mo e o império do narcisismo, resta ao jovem expressar
processo, porém, não é feito sem luta, sem transgressões. de forma radical a tentativa de apagar a ferida narcísica,
Pensemos na explosão pulsional que um corpo em pu- cumprindo a promessa do imaginário social narcísico, ou
berdade significa. Pensemos na multiplicidade de opções seja, apagar os vestígios do outro, portador de um desejo
de identificação, de ilusões imaginárias que a sociedade próprio.
contemporânea coloca para que um jovem se posicione, Freud (1915), ao afirmar que o Eu só advém na alterida-
estabelecendo-se assim um excesso de exigências, muito de, e que o outro é o primeiro inimigo do eu, nos permite
embora sob a forma de promessas de prazer e realização. pensar que o ato violento, paradoxalmente, convoca o ou-
A antropologia nos mostra que a adolescência é asso- tro. Uma forma de manter o objeto vivo seria atacá-lo. Por
ciada à violência entre inúmeros povos em todos os con- isso mesmo é fundamental o posicionamento do adulto
tinentes. Rituais iniciáticos de sociedades ditas primitivas, frente ao jovem. Senão se está lançado no desamparo. Isso
pela passagem da infância para a idade adulta, testemu- porque podemos dizer que
nham a necessidade do controle violento sobre a impetuo- ... a violência surge aí onde se experimenta o insupor-
sidade da puberdade; talvez por esses jovens, ao represen- tável indizível: aí onde o pensamento e as palavras faltam
tarem “mudança”, serem vistos como portadores do risco para dizer o sofrimento, ou simbolizá-lo. Aí onde a ameaça
de desorganização social. A cultura impõe sua ordem e os fantasmática do sucumbir narcísico força a que se faça mal
sujeitos devem a ela se submeter – todo ritual é uma forma ao mundo exterior. Não há violência que não se origine de
de violência e pode deixar marcas corporais dolorosas que um terrível sofrimento, quando saem de cena as forças de
testemunham a passagem do mundo da infância para o ligação em benefício da Pulsão de Morte. (Birraux, 1997,
mundo adulto. p. 138)

62
CONHECIMENTOS ESPECÍFICOS - PSICÓLOGO

Quando faltam o suporte e as palavras adequadas às em como a renúncia pulsional para o jovem está muito
manifestações pulsionais, desde as origens do sujeito, ou mais dificultada, hoje em dia: ele pode e deve escolher o
seja, a violência primária, conforme Piera Aulagnier discute, que quiser, seja em termos profissionais, seja nos valores
a violência em ato pode ser entendida como resposta de- pessoais, já que a família “deve” deixá-lo livre para escolher,
fensiva à violência pulsional, ou seja, a sua projeção. Não não impondo sua própria referência. O adolescente deve,
deixa de ser um grito que convoca o outro que falhou, o no mínimo, ser feliz, bonito, com o corpo perfeito e, para
objeto amado/odiado que não pôde suportá-lo e interpre- tal, deve consumir os objetos, as dietas, os exercícios que
tá-lo. são incessantemente oferecidos no mercado. A permissivi-
Simone Couraud, no seu trabalho “L’acte criminel à dade contemporânea frente à sexualidade, ao mesmo tem-
l’adolescence” (1997) faz uma importante contribuição no po que lhe autoriza todas as possibilidades de vivência e
sentido da importância do processo de julgamento dos jo- satisfação sexual, acaba exigindo um padrão de realização
vens criminosos. sexual que aumenta em muito as exigências pulsionais. O
Constata-se nesses jovens uma falta de referências, adolescente fica imerso numa excitação intensa que lhe re-
tanto externas quanto internas, que tornavam difícil sua ativa o desamparo.
inscrição social através de uma história pessoal, familiar, Claude Balier (1997), ao discutir o parricídio, vai rela-
social, de um passado, de um futuro. O estudo dos testes cioná-lo
projetivos e das entrevistas, mostra um fracasso na repre- ... aos transtornos narcísicos criados pela maturação
sentação das imagens parentais. sexual, a inquietação frente à emergência das pulsões, a
confrontação para uma realização daquilo que até então
A autora segue analisando como nesses casos fica pa-
estava apenas no registro do fantasma, a dificuldade de
tente a ausência da função paterna e a característica de
reviver as perdas para encontrar novos objetos, a incerteza
mães que remetem ora a um vazio (a “Mãe Morta” evoca-
dos limites dentro/fora, os problemas de identidade frente
da por Green [1988, p. 247]) ora à imagem arcaica da mãe às transformações corporais, etc. os conflitos dificilmente
todo poderosa, fusional e ameaçadora. Mas o fundamental gerenciados, podem então implodir e o recurso às defesas
desse trabalho é apontar como no processo de julgamento primárias conduzir à passagem ao ato. Não é por acaso
do jovem criminoso lhe é dada a oportunidade de colocar que os atos de parricídio são essencialmente cometidos na
em palavras a sua história, a sua relação com a vítima, refle- adolescência. (p. 91)
tir sobre a passagem ao ato. Instaura-se, enfim um rito ini-
ciático onde o sujeito pode interrogar-se sobre si próprio, Este autor segue discutindo como o adolescente tem
viver a culpa, responsabilizar-se e talvez assim deixar de ser capacidade para ficar imerso na excitação, revelando a fra-
vítima, submetido a um Outro insuportável. A importância gilidade de sua identidade. Ele tem a tentação de fazer de-
da função das palavras aqui se coloca ao se reconhecer a saparecer o Eu, renunciando a seu estatuto de sujeito. O
expressão de um sujeito que sofre e pede reconhecimento, fenômeno de drogadição, típico dessa fase, explicita essa
mesmo que seja para pagar a sua culpa. Aliás, como en- tendência. Isto revela uma tentação a manter a relação fu-
sina Freud (1916), os crimes muitas vezes são cometidos sional primitiva com a mãe, o estado de passivação, como
para aliviar o sentimento de culpa intenso provocado justa- apresenta Green (1990), o qual oferece toda a possibilida-
mente pelas moções pulsionais relativas essencialmente ao de para o indivíduo poupar-se do movimento doloroso da
complexo edípico. Importante notar que a expressão Ado- separação que conduz à afirmação de identidade. O fenô-
lescentes em conflito com a lei, já propõe um engajamento meno das gangues, ou mesmo das “tribos” não necessaria-
do social frente a um Eu em revolução. “É na medida onde mente violentas, tão característico da adolescência, expres-
não houve castração no seio da família que o jovem vai sa claramente essa tendência que nada mais é do que uma
provocar de modo concreto o bastão da polícia”, aponta busca massiva de indiferenciação – roupas, vocabulário,
Melman ao falar de delinquência (1992, p. 46). gestuário uniformes. O jovem aí encontra um mecanismo
de, na ilusão identificatória, escapar à angústia do ser ou
Desamparo e violência: desafios para o jovem não ser, ou seja, da sua emergência enquanto sujeito em
oposição ao outro.
Em “Psicologia das massas e análise do eu” (1921),
Com certeza não se trata de defender o bastão de po-
Freud fala do grande gozo de se abandonar sem reservas,
lícia quando se fala da falta da lei, analisando-se o declínio
misturando-se à massa, como forma de perder o sentimen-
da Função Paterna como responsável pelo desamparo do
to de seu próprio limite individual, e fundir-se ao imaginá-
sujeito contemporâneo. Mas como entender as vicissitudes rio de uma potência ilimitada na submissão a uma influên-
edípicas e o jogo da Castração no atual cenário? Sabe-se cia fascinante. O sujeito fica, assim, estruturalmente hipno-
que o regulador social contemporâneo passa muito mais tizado pela figura do líder “despótico, brutal e terrível”. É o
pela transmissão de uma promessa de gozo do que pela fascínio pelo poder do pai primitivo, todo-poderoso, que
consciência da dívida, do dever para com as gerações pas- reúne os indivíduos alienados na massa, concentrando os
sadas, como era na sociedade tradicional. Aí se colocam movimentos pulsionais violentos.
sérias dificuldades para a elaboração da castração. Como Ou seja, o fenômeno de gangue garante que o jovem
organizar o narcisismo se a castração na organização social fique protegido de seus votos de morte, e organize o seu
contemporânea vai apelar para a promessa do prazer, não desamparo que tão fortemente é suscitado na revivência
mais evocando o dever, nem a ameaça explícita de um pai edípica que esta fase retoma, principalmente se o pai faltou
violento, como o mito edípico costuma evocar? Pensemos na sua função de simbolizar a lei.

63
CONHECIMENTOS ESPECÍFICOS - PSICÓLOGO

É possível entender, então, como o jovem pode encon- do pai? Embora os adultos tenham que estar presentes,
trar na gangue, na tribo dos “manos” um apoio para seu mantendo-se vivos e sustentando a tentativa de destrui-
drama de subjetivação, para sua luta pessoal na busca do ção que os filhos lhes imputam, eles não podem negar a
pai ideal que possa “ser morto” e viver um luto que permite morte e, sim, autorizar que os filhos mantenham a conti-
identificação. Maria Rita Kehl apresenta um trabalho – “A nuidade da vida, vivam sua sexualidade e potência. Win-
fratria órfã” (2000) – em que analisa os grupos dos “ma- nicott (1975) aborda essa questão, lembrando que se trata
nos” jovens da periferia de São Paulo, admiradores de rap. sempre de uma questão de morte na adolescência, e que
Kehl propõe que a aliança fraterna pode possibilitar que os a melhor forma que os adultos têm de ajudá-los (aos jo-
sujeitos explorem e ampliem suas margens relativizando o vens) é sobrevivendo. Neste aspecto, é importante analisar
discurso da autoridade encarnado pela figura do pai real. É como a questão do envelhecimento e, consequentemente,
interessante considerar esta análise como alternativa à tão da morte, nos dias atuais, vem sendo negada: as limitações
insistente constatação do declínio da função paterna. da potência sexual são contornadas ou adiadas (Viagra, por
O apelo ao reconhecimento é geralmente endereça- exemplo). Métodos de rejuvenescimento permitem que
do ao pai. O irmão, o semelhante, será destinatário deste os adultos se ocupem incessantemente de sua afirmação
apelo apenas quando o pai dá as costas? Penso que não; identitária. Estabelece-se assim uma situação paradoxal:
o reconhecimento paterno, fundamental para que o sujeito de um lado, os adultos poderiam, com essas conquistas,
constitua uma certeza imaginária sobre “quem ele é” (de- sentir-se menos ameaçados, deixando mais espaço para o
sejo do pai) pode gerar também um aprisionamento nar- jovem buscar suas referências e diminuindo o jogo de hos-
císico. tilidade entre eles; de outro, se deixam os jovens sozinhos
para fazerem “o que quiserem” para cumprirem o ideal so-
Contudo, é fundamental considerar a posição dos cial de beleza e felicidade, esses adultos lançam os jovens
adultos nessa situação, senão a violência em ato será a úni- ao desamparo frente à violência das pulsões da puberdade
ca saída. porque estão eles próprios adultos comprometidos com
A violência seria, então, aquela da fusão dos indivíduos seu narcisismo.
na massa, mas também, de forma oposta, aquela do es- Soma-se a isso a falta de alternativas que a sociedade
touro dessa fusão na angústia do pânico que acontece, diz contemporânea apresenta no sentido de organizar rituais
Freud, quando o líder da massa desaparece ou não mais é de iniciação e de exercício de poder, culturalmente acei-
reconhecido, isto é, quando naufraga a autoridade pater- tos. Existem poucas oportunidades de organização social
na. É então o “sujeito do eu” que faz estruturalmente falta, para o enfrentamento da tempestade da puberdade. Ca-
sendo que anteriormente era investido como objeto. Sobra recem os anteparos que façam frente a essa energia, uma
aí o ódio, o ódio pelo objeto. Nos fenômenos de violên- oposição que valorize e enfrente o desafio colocado pelo
cia muitos adolescentes põe em cena uma destrutividade adolescente que luta para ser reconhecido. Os pais, na rea-
como que para constituir um objeto no ódio, argumentan- lidade, fogem da dialética “ou eu/ou o outro” ao evocarem
do a partir das carências objetais que pontuaram sua histó- o mote do “você é quem sabe de sua vida” sob pretexto de
ria. (Richard, 1997, p. 63) não querer frustrar ou violentar a individualidade do filho.
O manejo da violência na adolescência, portanto, não Quem sabe não se esteja reeditando de forma aparente-
é privilégio da sociedade contemporânea. Esse conflito já mente menos cruel (é “politicamente correto” não apelar
aparece nas concepções míticas sobre a fundação cultural para a violência) o mito de Cronos que devorava seus pró-
da civilização humana, na constituição da identidade de prios filhos assim que nasciam, como forma de manter a
um indivíduo na cultura – o tema do herói, os rituais de eternidade e a potência absoluta. “Pais e filhos, um para o
iniciação, etc. Não há como negar que todas essas configu- outro, representam a morte. Quero matar meus pais por-
rações apontam para o dilema entre, de um lado, afastar- que eles querem me impedir de viver, e também porque
-se do fascínio da indiferenciação, do prazer nirvânico e da os vejo velhos e mortais e não quero me tornar como eles.
ausência de conflito – o Princípio Narcísico por excelência Quero matar meus filhos porque sou mais velho que eles,
– e, de outro, poder enfrentar o desamparo. Isto vem à tona porque eles me tornam velho e pretendem viver depois de
quando se é defrontado com as faltas, falhas, incertezas mim, no meu lugar” (Reyes, p. 57).
na perda de um pai protetor, e/ou com a violência inter- O que se tem visto são adultos que não conseguem se
na que quer afastar e destruir esse “pai”, obstáculo para o fazer presentes e fazer frente à tentativa de destruição, sen-
gozo eterno. Mas para isso também é preciso um pai que do assim, a rigor, um suporte da violência juvenil, por negar
assuma o ideal de força, que facilite a identificação com ele sua própria violência, já que sustentam sua potência ima-
que, no fundo, traduz o ideal social, como aponta a tradi- ginária numa figura amável e condescendente. No entanto,
ção psicanalítica. conhecemos bem o que sustenta o poder – a violência.
Esses adultos estão, no fundo, exercendo o que chamei
Impasses contemporâneos de Violência Branca4 ao considerar o jovem um “aborres-
cente”. Se aborrece, então é melhor ignorá-lo, afastar-se
Neste ponto, temos que pensar o que se passa na con- dele, poupar-se do embate que ele propõe, desviar o olhar
temporaneidade. Será que esse “pai” tem suportado os desse corpo que pulsa, que insiste em transbordar de se-
ataques de seus filhos, mantendo-se vivo e forte, apesar de xualidade quer seja nas belas formas da mulher modelo ou
seu envelhecimento? Envelhecimento este que se constitui do jovem atleta, quer seja nas formas desajeitadas e feias
num fato com o qual o jovem se depara e que, de alguma (espinhas, gordura, etc.). Os infindáveis questionamentos
forma, permite-lhe pensar em algum dia assumir o lugar carregados de revolta e raiva, aliados à imagem de um cor-

64
CONHECIMENTOS ESPECÍFICOS - PSICÓLOGO

po jovem pleno de erotismo e potência criativa se tornam paço para garantir o sustento dos filhos, a sua permanência
insuportáveis para um adulto que também vive sob a égide na escola (que não dispõe de vagas suficientes), o seu aces-
da busca do prazer incessante e da obsessão pela imagem so a lazer e cultura, sem falar na assistência à saúde. Tantas
perfeita. Lembro que o princípio da submissão a uma or- vezes, nas classes baixas, são as crianças as responsáveis
dem dada, por exemplo, o da sucessão entre as gerações e pelo sustento das famílias mendigando nas ruas e sendo
o peso da dívida entre elas, não é o norteador da sociedade exploradas pelo trabalho infantil, ou sendo usadas no trá-
contemporânea. Ao contrário, os filhos é que deveriam ser fico de drogas, aproveitando-se de sua inimputabilidade.
felizes e livres para garantir o narcisismo dos pais. Não é di- Os equipamentos sociais que deveriam assistir a infância
fícil entender, então, como o adulto poupa-se de enfrentar e se complementar à família falham, e acabam reprodu-
sua própria violência, que é reativada nesse encontro com zindo uma violência, a que chamei de branca, ao evocar
os adolescentes ao considerá-los apenas “aborrescentes” e que numa sociedade livre e democrática, os indivíduos que
por isso não merecerem controle e cuidados. Ao contrário, tiveram vontade poderiam ter encontrado formas de se re-
o espelho se inverte e o adulto inveja o espetáculo que o alizar.
adolescente produz, abandonando-o ao seu próprio desti-
no heroico. Mas de herói a vilão os passos podem ser cur- Lembro que a palavra infantaria – os que estão na
tos. E se há excessos, não se hesita em condenar os jovens. frente dos fronts – vem de infans.
Violência crescente são os excessos da juventude malcria-
da, audaciosa, exigente e intolerante. A transgressão não O adolescente, como se procurou evidenciar, está em
pode ser reconhecida, já que os limites não existiam e as busca de filiação e de reconhecimento, tentando encontrar
proibições eram vagas.
desesperadamente no social, o lugar que lhe foi prometido,
acenado, vendido, e que, em tantos casos, está tão ina-
CONCEITO DESENVOLVIDO NO LIVRO VIOLÊNCIAS.
cessível. Volto a lembrar Winnicott (1987), que nos ensina
como nas condutas antissociais estão os sinais de esperan-
Adolescente: o estrangeiro contemporâneo
ça, os gritos de socorro – e tantas vezes, diria eu, a denún-
cia de uma impostura.
Para concluir, gostaria de fazer uma aproximação do lu-
Assim, depois de terem gozado junto aos adolescen-
gar do adolescente com a do estrangeiro. O social sempre
encontrou formas de exterminar o estrangeiro ou de co- tes, é melhor para os adultos pensarem que, na verdade,
locá-lo no lugar de exótico como aquele que representa e esses adolescentes são violentos. Se eles não foram capa-
atua tudo aquilo que a cultura de dada sociedade reprime. zes de aproveitar todas as oportunidades, e já que vivem
A noção do “estranho” (unheimlich) que Freud desenvolve como adultos, que respondam pelos seus atos longe de
em relação ao psiquismo também trabalha essa questão: nós – que sejam encarcerados.
“Na realidade, não é nada de novo nem de estrangeiro, é Enfim, a adolescência nos põe face a face ao mal-estar.
algo que muito tempo atrás foi familiar à vida psíquica e se Apesar da ciência, da psicologia, das pedagogias, estamos,
tornou estrangeiro a ela pela repressão”. como Freud bem já denunciava, no eterno impasse da pro-
Penso que, hoje, o adolescente pode estar ocupando blemática relação do indivíduo com o grupo, no que diz
o lugar do estrangeiro, tanto nos seus aspectos idealiza- respeito à capacidade de regulamentar a hostilidade e o
dos quanto execrados. Se ele não está nos fronts de guerra ódio na relação do indivíduo com o outro. A busca da lei –
contra os “maus estrangeiros”5, então deve ocupar o lugar os rigorosos rituais iniciáticos das sociedades primitivas, a
das rebeliões da Febem, transfigurando-se no incendiário, justiça dos direitos previstos nos códigos contemporâneos,
no perigoso, justificando seu extermínio ou controle coer- como o ECA –, que certamente organiza os restos de vio-
citivo que abate seu corpo e identidade. Rememore-se as lência dos encontros humanos, nada mais faz do que tentar
muitas cenas televisionadas das interferências das tropas sempre reencontrar um Pai protetor, ideal, que garantiria a
de choque policiais na Febem, expondo, em pelotões, seus condição do sujeito ser amado, reconhecido – ser, quem
jovens de corpo nu. Evocam-se cenários de guerra, cená- sabe, único.
rios de pré-fuzilamento. Não esqueçamos que Freud é veemente em afirmar o
Nesse contexto, quem é violento? Qual é o apazigua- mal-estar, e que a violência não cessa de retornar apesar
mento procurado? Em nome do que e de quem? Em nome dos esforços da civilização em contê-la e organizá-la. Ele
de poupar os adultos, que até então invejavam a capacida- não concebe o psiquismo sem o dualismo pulsional, ou
de desses jovens de gozar, de lutar, de expressar seu des- seja, o imbricamento de Eros e Tânatos, as forças de vida
contentamento em protestos violentos, de buscar o prazer e morte que impulsionam os sujeitos a se encontrar, lutar,
de forma incessante através do consumo de grifes, dro- amar, transgredir, destruir e construir.
gas, que buscam realizar a promessa de um mundo sem Adolescência: violenta e apaixonante condição huma-
conflitos e sofrimento. Em nome, também, de poupar uma na. Vamos enfrentá-la.
sociedade que exclui grande parcela de sua população dos
direitos mínimos de cidadania e que acena com promessas Fonte:
ilusórias de realização, desde que no fundo se seja capaz MARIN, Isabel da Silva Kahn. Violência e transgressão:
de consumir. Os meios para isso são discutíveis. A família interrogando a adolescência. 2003
que deveria ter servido de referência está desestruturada, MARIN, Isabel da Silva Khan. Violências São Paulo: Es-
não “foi capaz”, ou melhor, não encontrou na sociedade es- cuta/Fapesp, 2002

65
CONHECIMENTOS ESPECÍFICOS - PSICÓLOGO

Durante o divórcio a criança vive várias dissociações


A CRIANÇA E A SEPARAÇÃO DOS PAIS. com maior ou menor grau de desestruturação. Uma das
mais importantes e que deve ser mencionada é a dissocia-
ção no nível espacial, que repercute ao corpo e no nível da
afetividade, através de sentimentos dissociados, de acordo
QUANDO OS PAIS SE SEPARAM4 com Françoise Dolto (2003). Se a criança puder permanecer
na casa onde seus pais estavam unidos, há uma mediação
Talvez um dos mais fortes rompimentos dos laços afe- e o trabalho do divórcio é feito de maneira melhor para
tivos para uma criança, com exceção à morte dos pais, seja ela. Caso sua casa tenha de ser abandonada, um dos pais a
a separação de seus progenitores. deixa ou mesmo a criança tem que se mudar com um dos
Principalmente quando acontece numa idade em que cônjuges, a criança vivencia os dois níveis de dissociação
a criança já possui entendimento para saber, a seu modo, mencionados.
quais serão as implicações que ela supostamente acredita Se caso a criança seja obrigada a deixar a escola com a
que sofrerá, acarretando, assim, e de fato, sérios conflitos e qual já estava habituada, surge aí mais um nível de disso-
danos na sua formação e saúde mental futura. ciação, a do grupo em que vive. Ela ficará por certo bastan-
É para a criança toda sua segurança e estabilidade te dividida e terá atraso escolar.
que estão se rompendo, seu mundo como ela conhece Igualmente prejudicial é o fato de a criança deixar a es-
despedaçando; parecidamente ou ainda de maneira pior cola no decorrer do ano letivo porque foi morar em outro
que como mencionado no capítulo anterior, acontece num lugar. É abalado o ser individual que se encontra dividido
holding ineficiente, a sensação de despedaçamento do seu e desestruturado pela separação dos pais e seu ser social,
mundo, uma destruição do seu universo que ela terá que separando-se do grupo no qual está inserida.
aprender a reconstruir de novo. Tanto melhor se tiver a aju-
da dos pais que se divorciarem de maneira a entender que Em confluência com isso, costuma-se seguir uma ten-
não podem se separarem dos filhos, mas só do parceiro, tativa, em vão, de amenizar o processo de separação di-
esses sim poderão contribuir um pouco com a reconstru- zendo à criança que um dos pais viajou. Ora, a criança é
ção do mundo da criança. inteligente o suficiente para perceber que aquele que “via-
Entretanto, sabemos ser essa uma tarefa pouco exis- jou” não voltou como volta de viagem qualquer pessoa,
tente entre os casais que se separam. Eles se separam e temos, aí, então, outra dissociação do contexto também
esquecem que tem um filho para cuidar, não só física e ma- social, pois se ela continuar na mesma escola, não saberá
terialmente que é o assunto que os pais mais se preocu-
o que dizer aos seus amigos. É aquela criança apática que
pam, mas principalmente esquece-se de sua saúde mental,
mal fala e brinca em sala de aula, absorta em seus pensa-
seu estado emocional, suas emoções e aflições próprias do
mentos, alheia ao que se passa na escola. Num processo
processo em que foram inseridos.
linguístico não pode haver fala se a criança está fragmenta-
É importante ressaltar que essa criança ao permanece-
da, muito menos concentração quando a criança tem algo
rem num lar de brigas e discussões constantes não cons-
muito mais importante e aflitivo para pensar do que prestar
titui também um lar saudável. Para a criança torna-se um
atenção no que a professora ensina. A criança se encontra
constante estado de insegurança que parece não ter fim.
fragmentada em todos os aspectos. É um comportamento
Os casais acreditam que se separando darão a eles pró-
característico da criança abalada por completo, o mesmo
prios e a criança uma tranquilidade maior. Em casos extre-
mos, acredito ser essa a solução desde que o divórcio seja ocorre quando um dos pais adoece seriamente ou mesmo
realizado com consciência e sem grandes alardes, onde a na morte de um deles.
criança seja realmente levada em consideração e tudo seja Os pais que estão se separando evocam as desestrutu-
pensado de maneira a beneficiá-la ou, ao menos, exista rações bruscas da afetividade da criança e não sabem se é
o cuidado de não prejudicá-la ainda mais dado os outros como devem contar o fato a elas. O fato é que sempre as
conflitos que tal situação, em si, já irá ocasionar. crianças devem ser comunicadas sobre este fato desde o
Vale ressaltar que só e somente só em casos realmente início que ele venha ocorrendo.
extremos de intolerável convivência o processo de separa- Elas precisam saber como ficará sua condição de filho,
ção seja aceitável para a criança. Qualquer outro desenten- com quem ficará e todas as outras coisas que o juiz deter-
dimento que o casal venha a ter é preciso pensar se pelo minar. Quando o divórcio é escondido, passa ser algo er-
menos a convivência amistosa ainda é possível até que a rado, acompanhado de um sofrimento muito maior e des-
criança atinja um estado de maturidade para entender que necessário do que ele já impõe. Os pais devem humanizar
essa é a melhor saída para os pais e aceitar a situação, caso esse fato, se responsabilizarem por ele, para que a criança
contrário, as perdas e desvios de caminhada rumo a um não sinta somente o estado de angústia e pressintam o
ambiente saudável de desenvolvimento que tanto Winni- divórcio pela variação de humor dos pais. Quando nada é
cott (2008) aconselha e pessoalmente concordo, estaria explicado à criança ela acredita que como os pais voltaram
ruído. atrás na palavra dada de ficarem casados, também um dia
4 GOOS, A. F. G. Formação e rompimento dos laços afetivos. Arara- renunciarão a criança, o que não é verdade, mas que se não
quara, 2010. for bem conversado, distorce o equilíbrio da criança.

66
CONHECIMENTOS ESPECÍFICOS - PSICÓLOGO

Segundo Françoise Dolto (2003), os atos nos seres hu- Outro fato que perturba a criança e abala suas estru-
manos são sempre precedidos de projetos antes de serem turas futuras são os pais que não podem ou não querem
executados. Portanto, se os pais contam aos filhos sobre contribuir financeiramente, mas ficam presos à necessida-
o divórcio e logo passam à ação, isto se torna traumati- de de ver esses filhos sem serem capazes de ganhar di-
zante para a criança que já vive um processo de separação nheiro para eles. Esses pais desenvolvem na criança a idéia
que, por melhor que seja conduzido, já é suficientemente de menosprezo pela noção de responsabilidade. Alegando
traumatizante e estressante. É importante salientar que a prejuízo financeiro, muitos se refugiam na casa dos pais,
notícia do divórcio seja dada pelo pai e pela mãe juntos, a causando uma regressão para a criança que se vê ao lado
criança precisa saber que o divórcio é um mal menor, ele de pais transformados em irmãos mais velhos, machucados
vai tirar a “doença”, o mal que aflige o casal, para trazer a pela vida e que já não são modelos de adultos, de acordo
paz, a saúde da relação, pelo menos deveria ser essa a ati- com Françoise Dolto (2003). Morar com os avós não traz
tude em relação a esse fato. nenhuma solução para a criança. O preço dessa facilidade
Muitos pais dizem não contar para as crianças porque terá de ser pago mais tarde.
não veem sentido, uma vez que no dia seguinte fazem as
mesmas perguntas como se não soubessem o que se passa E dentre todas as palavras que é possível dizer a uma
ou como se não tivessem ouvido nada a respeito. É que criança, são sobretudo a dos avós ao neto que podem per-
quando algo é difícil demais de assumir, as crianças têm mitir a este discutir o assunto com eles e, desse modo, rela-
a necessidade de inventar. Aliás, muitos adultos que não tivizar aquilo que nele, enquanto é pequeno, no momento
aceitam o divórcio fazem o mesmo, fato esse, portanto, em que está sofrendo com a separação dos pais, o leva a
que não justifica recriminar, então, uma criança. Quando considerar responsável ou culpado um ou outro de seus
a realidade se faz intolerável, é necessária a fuga para a genitores. É muito bom que a criança receba palavras que
fantasia ou tão somente uma fuga. A criança “inventa” que lhe deem acesso a essa compreensão das relações do ca-
nada está errado ou diferente com seus pais; outras vezes sal, uma compreensão que lhe chegará com a experiência.
não admite que outra pessoa fale a respeito disto ou ainda (DOLTO, 2003)
criam a fantasia de que o pai ou a mãe viajou ou outra coisa
parecida. Quanto à guarda da criança, muito se é discutido sobre
O que não é “verdadeiramente falado”, não é huma- o que é melhor ou pior para a criança. Temos uma opção, a
nizado, segundo Françoise Dolto (2003), e o ser humano mais comum até os dias de hoje, que é a guarda dada a um
aprecia o valor da humanização. único genitor, onde o outro tem direito a visitas estipula-
Outra comum confusão acerca do divórcio é o fato dos das, normalmente, pelo juiz ou em raras ocasiões acertadas
pais acharem que privar a criança da convivência de um entre os pais da criança. Temos, ainda, a guarda comparti-
dos cônjuges seria melhor para ambos. lhada, também chamada de alternada, onde a criança tem
dois lares a sua disposição, devendo ficar um determinado
Não se protege a segurança da relação privando o fi- período em cada lar, com cada um dos genitores. Na pri-
lho do conhecimento do outro genitor. Ao contrário, isso meira opção, podemos observar que mesmo estando com
constitui a enorme promessa de uma enorme insegurança um único genitor, mesmo contra tudo que foi escrito, é cru-
futura, e que já estaria presente desde a instauração de tal cial ressaltar que se a criança foi desde pequena criada pelo
medida, visto que isso é uma anulação de uma parte da pai e pela mãe, mesmo que tendo especial carinho por um
criança através da qual lhe é indicado, implicitamente, que deles, ao ter sido criada, de fato, pelos dois, vai sentir um
esse outro é alguém desvalorizado e falho. (DOLTO, 2003, profundo abalo em suas emoções, ficando absorta em seus
p.52) pensamentos e desatenta a tudo o que não for pertinente
Muitas coisas se passam com as crianças durante o em relação à separação de seus pais, podendo, ainda, de-
processo de separação dos pais. São frequentes as dores senvolver na escola um grave déficit de atenção. O déficit
de cabeça, de barriga, os surtos de febre, entre outras coi- de atenção é o problema mais comum a ser desenvolvido
sas. Trata-se de uma linguagem que a criança não pode pelas crianças, filhas de um divórcio pacífico, bem realizado
expressar verbalmente, então o inconsciente se estrutura ou não. Todas as crianças em maior ou menor grau vão
como uma linguagem. Esses sintomas não são um mau reagir de alguma forma. Umas ficarão extremamente agi-
sinal. Somente faz-se necessário adaptar essa linguagem tadas no intuito de chamarem a atenção para si, outras, na
tornando-a compreensível de maneira que a criança pos- mesma tentativa, tornar-se-ão rebeldes, malcomportadas
sa se expressar através de palavras e não de sintomas, de e, quase sem exceção, poderemos encontrar essas crian-
maneira psicossomática. De acordo com Françoise Dolto ças perdidas no tempo e espaço, sem atenção ao professor,
(2003), isso não se deve ao comportamento pessoa de como muitos diriam, “no mundo da lua”, com seu déficit de
cada um, mas a uma situação. As situações que acontecem atenção em maior ou menor intensidade. Outros proble-
com o divórcio em si, são normalmente desestruturantes mas de comportamento mais graves surgirão, como vere-
para a criança pequena, porque as etapas de seu desenvol- mos mais adiante, também oriundos da separação dos pais
vimento afetivo coincidem com as aflições pelas quais seus ou do divórcio.
pais estão passando, numa época em que os pais deveriam A guarda compartilhada ou alternada, não é a guarda
estar principalmente garantindo sua segurança emocional compartilhada pelo pai e pela mãe onde ambos concor-
para um futuro mentalmente sadio. dam e estabelecem segundo acreditam ser o melhor para

67
CONHECIMENTOS ESPECÍFICOS - PSICÓLOGO

a criança, os dias em que poderão visitar o pai ou a mãe. É Outro grave problema oriundo do divórcio é quando
de acordo com a lei a criança ficar um tempo estabelecido normalmente a mãe “sacrifica” tudo pela família não se ca-
pelo juiz com o pai e outro com a mãe. A criança tem dois sando novamente. Esta atitude da mãe que sacrificou tudo
lares e não consegue estabelecer-se em nenhum dos dois. pelos filhos, que deu sua vida pelos filhos, repercutirá na
Não cria vínculos sociais nem tão pouco emocionais. Emo- vida destes filhos no futuro. Desejosos de compensar esta
cionalmente é uma criança desconecta do mundo, carente mãe, não se casarão eles mesmos para que possam, por
afetivamente e perdida, não se enquadrando nos grupos exemplo, dar seu dinheiro a mãe que tudo fez por ele ou,
sociais ao qual pertence. Têm dois lares e ao mesmo tempo ainda, não continuará seu projeto de estudo por ser caro
nenhum. demais e continuar dando despesas a essa mãe. Esses fi-
lhos irão viver com a mãe e com outras mulheres se tor-
A reação mais comum é o desenvolvimento da passivi- narão bloqueadas sexualmente falando e os meninos, com
dade no caráter da criança. Ela perde o gosto pela iniciativa, tendência homossexual, viverão relacionamentos supérflu-
tanto do ponto de vista escolar quanto do ponto de vista os com pessoas que não deixaram suas mães também. Tra-
das brincadeiras, e entra em estados de devaneio que não ta-se de neuroses bastante difíceis de suportar e superar.
levam à criatividade – porque existem devaneios fecundos,
mas aqui não se trata de um deles. (DOLTO, 2003) Consequências para a vida da criança na escola e/
ou no meio social em que está inserida
Muitos pais e principalmente mães privam-se de um
novo relacionamento dizendo fazê-lo pelos filhos que não A família, na atualidade, não é mais como as grandes
querem ver o pai e, principalmente, a mãe casada nova- famílias do passado. Tinham-se muitos irmãos, primos e to-
mente. As pessoas que agirem assim com seus filhos, os dos interagiam e contribuíam para a formação da criança
farão bebês por muito mais tempo. Serão imaturos e no em crescimento.
futuro ficarão com as vidas congeladas, segundo Françoi- Antes mesmo que a criança iniciasse a escola propria-
se Dolto (2003), impedidas libidinalmente e afetivamente. mente dita, esta já tinha seu círculo extrafamiliar de rela-
Tornam-se encarregadas deste cônjuge que tudo abdicou ções formadas. Eram vários coleguinhas na vizinhança com
os quais podiam brincar. A criança ia por si só à casa dos
pelos filhos, pelo resto de suas vidas mesmo que porven-
amigos e, às vezes, passava o dia todo em contato com
tura consigam se casar.
uma ou várias outras crianças e suas famílias, convivendo,
A relação com os novos parceiros dos pais é extre-
assim, com diferenças culturais com as quais ela apreen-
mamente saudável e preferível para a criança, filha de pais
dia e aperfeiçoava em suas relações e em seu convívio em
separados. Se a separação ocorreu quando essas crianças
grupo.
eram muito pequenas, elas poderão viver o complexo de
Hoje, quase só podemos contar com as escolas de
Édipo ou reviver uma nova variação dele, situação não só
educação infantil. Não é o fato de delegar à escola a tarefa
saudável como necessária a todas as crianças. Esses adultos
de desenvolver os filhos socialmente, mas é dela, em gran-
serão para elas modelos e rivais. de parte hoje, essa responsabilidade. A pequena família
Muitas vezes a criança que vive a situação mencionada moderna depende da escola para que seus filhos possam
acima, dirá que tem dois pais ou duas mães. Não é neces- interagir com outras crianças. Não há mais irmãos e primos
sário impedir-lhes disto. Trata-se de uma autodefesa em para brincar. Tão pouco se tornou seguro brincar com um
relação à curiosidade das outras pessoas. amiguinho que não se conhece os pais. A economia onde
De fato, quando há problemas com o padrasto ou com os desejos individuais são exacerbados e não sobra tem-
a madrasta, provém do genitor com quem a criança mora. po nem dinheiro para os filhos e onde a segurança é um
Este genitor não aceita muito bem o direito do outro de ter grande problema a se enfrentar, vemos que a família tende
um novo relacionamento; a criança, então, sente que recu- a ser sempre menor até se tornar extinta e a desconfiança
sando o padrasto ou madrasta estará mostrando-se a favor que não nos permite interagir, acabará por matar as rela-
do pai ou da mãe com quem mora. Se não houver este tipo ções sociais como conhecemos. Será nessas escolas o lugar
de interferência, as coisas se arranjam de maneira muito onde as crianças desfrutarão de espaço e de pessoas dis-
mais fácil. É claro que muitos filhos desejarão que seus pais, poníveis para lhes dar a atenção necessária e poderão, en-
livres um dia dos atuais parceiros, possam se reencontrar. tão, aprimorar suas relações sociais. De que outra maneira
É uma projeção da primeira infância na idade avança- esta criança pode brincar com outra criança sem preocu-
da. Várias desorientações surgirão no decorrer do divórcio pação para os pais que precisam ou que querem trabalhar
e algumas serão para toda a vida. Os filhos ficarão deso- o período todo? Não é possível, é óbvio, delegar à outra
rientados em relação a concepção do matrimônio, questio- pessoa a tarefa que deveria ser desempenhada na própria
nando-se sobre a possibilidade de casarem no futuro ou se família. Entretanto, ainda nestes termos, a escola se faz a
simplesmente o farão não para a vida toda, mas, simples- instituição mais capaz de promover o grupo de relaciona-
mente para divorciarem como seus pais. Também quando mento social da atualidade.
seus pais não se casam novamente, ficam na dúvida sobre a É também na escola que a criança pode ter uma pausa
possibilidade de se tornarem celibatários como estes. Para de tudo que vive em casa. Na escola, a atmosfera emocio-
essas crianças, os referenciais de orientação é que são os- nal é menos densa que no lar segundo Winnicott (2008).
cilantes. Isso propicia à criança uma pausa para o desenvolvimento

68
CONHECIMENTOS ESPECÍFICOS - PSICÓLOGO

pessoal. Há ainda a possibilidade de a criança viver novas cer as relações pessoais da criança com a própria família,
relações triangulares menos intensas que na própria casa, apresentando simultaneamente um mundo mais vasto de
contribuindo para seu desenvolvimento. pessoas e oportunidades” (WINNICOTT, 2008, p. 220). A es-
cola, antes de qualquer coisa, representará a liberdade de
A escola, que é um apoio, mas não alternativa para o tempo para que a mãe se descubra e encontre suas poten-
lar da criança, pode fornecer oportunidades para uma pro- cialidades maternas e como indivíduo e, ao mesmo tempo,
funda relação pessoal com outras pessoas que não os pais. a criança está sendo cuidada para que se desenvolva e su-
Essas oportunidades apresentam-se na pessoa das profes- pere os inevitáveis problemas psicológicos com que o ser
soras e das outras crianças e no estabelecimento de uma humano em desenvolvimento se defronta. O ambiente da
tolerante, mas sólida, estrutura em que as experiências po- escola maternal, assim como a professora, desempenha um
dem ser realizadas. (WINNICOTT, 2008) papel importante para o desenvolvimento psicológico da
criança. É também na escola maternal que a criança entrará
Entre os dois e sete anos de idade, a criança experi- em possíveis conflitos com outros de sua idade, aprenden-
mentou todos os resultados dos seus conflitos resultantes do a resolver suas frustrações decorrentes destes possíveis
das poderosas reações instintivas que viveu e, a partir dos confrontos, aprendendo a desenvolver a capacidade de re-
cinco anos, aproximadamente, começam a abandonar, aos lações harmoniosas em seu grupo e principalmente a com-
poucos, a fantasia consciente e inconsciente para dar lugar partilhar. As brincadeiras, principalmente entre as crianças,
a identificações maiores com os pais e mães, envolvendo, é uma atividade criadora essencial para o desenvolvimento
assim, excitações que se expandirão mais quando chegar a humano e a escola facilitará, e muito, que a criança tenha
puberdade. êxito no terceiro tipo de desenvolvimento que é a capaci-
Ao mesmo tempo as relações só agora foram estabele- dade de relações em que diversas pessoas estejam envol-
cidas entre seres humanos integrais. vidas. É a professora que ajudará nessa fase que a criança
A consequência resultante da solução destes conflitos conduza sua agressividade para canais construtivos e para
é o sofrimento que aí se inicia, resultando na formação de adquirir habilidades eficazes.
sintomas, como já foi mencionado, inibições e até “recal- Em todo este período existe um processo duplo entre
ques”. À medida que o desenvolvimento da criança prosse- o lar e a escola: quando acontece um problema em um dos
gue, ela consegue cada vez mais expressar seus sentimen- ambientes, automaticamente se transfere como perturba-
tos de formas mais diretas. Desse modo, o alívio, então, é ções no comportamento para esse outro ambiente.
obtido mediante a autoexpressão, ou seja, quer através de
brincadeiras ou quer através da fala. Colapsos no asseio, dificuldades na alimentação e no
Quando a criança entra no maternal, suas capacidades sono, atraso na fala, atividade motora defeituosa, estes e
são mais subjetivas do que objetivas, pois seu processo outros sintomas podem-se apresentar como problemas
de maturação ainda está se formando e a capacidade de normais do crescimento ou, numa forma exagerada, como
percepção exata ainda não está totalmente desenvolvida. desvios do normal. (WINNICOTT, 2008, p. 223)
Quando a angústia ameaça, a criança volta facilmente à po-
sição infantil de dependência. Sem a cuidadosa apresen- Até o último período de frequência na escola maternal,
tação da realidade externa que é papel dos pais, a criança haverá certa confusão entre o que é certo e errado, entre
não possui meios de estabelecer uma relação satisfatória a fantasia e o fato, entre o que é propriedade pessoal e o
com o mundo. que é dos outros.
Na escola maternal, criam-se condições para que a Há que se mencionar os dois tipos básicos de crianças
criança possa desenvolver seu estágio intermediário entre que se inserem na escola. O primeiro deles anseia pelo en-
o sonho e o real através de músicas, brincadeiras, desenhos sino propriamente dito. São aquelas crianças que tiveram
e histórias. É nesse contexto que a criança toma consciência todo o processo de desenvolvimento emocional e afetivo
de quando é livre e de quando lhe é requerido um deter- necessários para se tornarem uma criança sadia emocional-
minado comportamento em grupo. É também na escola mente. Criadas por pais que assumiram sua responsabilida-
maternal em que ela se reconhece como indivíduo, sendo de, essas crianças estão dispostas ao trabalho árduo para
chamada pelo seu nome, sendo vestida e tratada pelo que que possam ter êxito nos exames e um dia terminarem os
é de fato, tendo, assim, sua individualidade se afirmando estudos e trabalharem como seus pais na profissão escolhi-
de tal forma que no futuro é ela que vai querer aderir às da. Estão ávidos por lições que lhe ensinarão cada vez mais
atividades em grupo. Assim como com a mãe, as atividades o que precisarem.
na escola não poderão ser mecânicas, elas representarão a Já o outro grupo é composto por crianças com proble-
mãe para criança através da alimentação, troca de roupa e mas familiares que caracterizarão a escola como abrigo, ou
até mesmo os banhos. Dessa forma, análogo ao vivenciado seja, uma extensão do lar para que se possa desenvolver e
com a mãe, tais atividades poderão ser amadas (aceitas), resolver o que não foi conseguido no seio familiar. Elas vão
ou rejeitadas (não merecerem confiança). procurar um grupo social do qual elas possam fazer parte e
O papel da professora na escola é de vital importân- se sentirem estáveis emocionalmente.
cia. Servirá para criança e pais estabelecerem uma relação Ao professor caberia, portanto, a tarefa de se inteirar
de segurança e confiança com o novo lugar que a criança da vida de seus alunos antes de rotulá-los com possíveis
ficará. “O seu dever é, antes, manter, fortalecer e enrique- distúrbios e mais comumente com dificuldades de aprendi-

69
CONHECIMENTOS ESPECÍFICOS - PSICÓLOGO

zagem, muitas das quais se caracterizam por reflexos e sin- Art. 8º É assegurado a todas as mulheres o acesso aos
tomas já discutidos anteriormente, que pertencem a uma programas e às políticas de saúde da mulher e de plane-
fase da vida que a criança está vivendo e que com sorte jamento reprodutivo e, às gestantes, nutrição adequada,
será esquecida e superada. atenção humanizada à gravidez, ao parto e ao puerpério
Um dos distúrbios de aprendizagem mais diagnosti- e atendimento pré-natal, perinatal e pós-natal integral no
cado entre crianças que apresentam problemas no lar é a âmbito do Sistema Único de Saúde. (Redação dada pela Lei
síndrome do déficit de atenção. nº 13.257, de 2016)
A atenção é o processo pelo qual usamos as estraté- § 1º O atendimento pré-natal será realizado por
gias necessárias para captar as informações do meio em profissionais da atenção primária. (Redação dada pela Lei
que estamos inseridos. A atenção está relacionada intima- nº 13.257, de 2016)
mente com a percepção e nos permite selecionar e hierar- § 2º Os profissionais de saúde de referência da gestante
quizar os estímulos recebidos. Com déficit de atenção, não garantirão sua vinculação, no último trimestre da gestação,
é possível se concentrar para realizar o que a professora ao estabelecimento em que será realizado o parto,
pede em aula, uma vez que é necessária a concentração garantido o direito de opção da mulher. (Redação
no que foi explicado antes por ela. Com a atenção pre- dada pela Lei nº 13.257, de 2016)
sa aos acontecimentos desastrosos vivenciados em casa, § 3º Os serviços de saúde onde o parto for realizado
a criança que não tem um lar satisfatório, fica presa aos assegurarão às mulheres e aos seus filhos recém-nascidos
problemas que afetam sua vida diretamente e não conse- alta hospitalar responsável e contrarreferência na atenção
gue, portanto, focar a atenção no que aparentemente não primária, bem como o acesso a outros serviços e a grupos
lhe é urgente como seu dilema no lar. Isso acarretará uma de apoio à amamentação. (Redação dada pela Lei nº
dificuldade grande em reter conhecimentos que só são ad- 13.257, de 2016)
quiridos mediante a importância e a necessidade que tem. § 4º Incumbe ao poder público proporcionar assistência
A desatenção pura e simplesmente também acontece nas psicológica à gestante e à mãe, no período pré e pós-
mesmas circunstancias descritas. Entretanto, a duração e natal, inclusive como forma de prevenir ou minorar as
a intensidade com que essa desatenção ocorre é que vai consequências do estado puerperal.
caracterizá-la como a síndrome do déficit de atenção ou § 5º A assistência referida no § 4o deste artigo deverá
não. Contudo, é necessário salientar que essa desatenção ser prestada também a gestantes e mães que manifestem
leve mesmo não sendo diagnosticada como a síndrome, interesse em entregar seus filhos para adoção, bem como
vai de qualquer forma contribuir para uma dificuldade de a gestantes e mães que se encontrem em situação de
aprendizagem, mais ou menos intensa, dependendo das privação de liberdade. (Redação dada pela Lei nº 13.257,
circunstâncias, atrapalhando, certamente, o desenvolvi-
de 2016)
mento escolar da criança.
§ 6º A gestante e a parturiente têm direito a 1 (um)
acompanhante de sua preferência durante o período do
Fonte:
pré-natal, do trabalho de parto e do pós-parto imediato.
BOWLBY, J. Formação e Rompimento dos laços afeti-
(Incluído pela Lei nº 13.257, de 2016)
vos. São Paulo: Martins Editora, 2015
§ 7º A gestante deverá receber orientação sobre
WINNICOTT, D.W Tudo começa em casa. São Paulo:
aleitamento materno, alimentação complementar saudável
Martins Fontes, 2011
e crescimento e desenvolvimento infantil, bem como sobre
formas de favorecer a criação de vínculos afetivos e de
OS DIREITOS FUNDAMENTAIS DA estimular o desenvolvimento integral da criança. (Incluído
CRIANÇA E DO ADOLESCENTE. pela Lei nº 13.257, de 2016)
§ 8º A gestante tem direito a acompanhamento
saudável durante toda a gestação e a parto natural
cuidadoso, estabelecendo-se a aplicação de cesariana
LEI Nº 8.069, DE 13 DE JULHO DE 1990. e outras intervenções cirúrgicas por motivos médicos.
(Incluído pela Lei nº 13.257, de 2016)
Dispõe sobre o Estatuto da Criança e do Adoles- § 9º A atenção primária à saúde fará a busca ativa da
cente e dá outras providências. gestante que não iniciar ou que abandonar as consultas de
pré-natal, bem como da puérpera que não comparecer às
Título II consultas pós-parto. (Incluído pela Lei nº 13.257, de 2016)
Dos Direitos Fundamentais § 10. Incumbe ao poder público garantir, à gestante e
Capítulo I à mulher com filho na primeira infância que se encontrem
Do Direito à Vida e à Saúde sob custódia em unidade de privação de liberdade,
ambiência que atenda às normas sanitárias e assistenciais
Art. 7º A criança e o adolescente têm direito a proteção do Sistema Único de Saúde para o acolhimento do filho, em
à vida e à saúde, mediante a efetivação de políticas sociais articulação com o sistema de ensino competente, visando
públicas que permitam o nascimento e o desenvolvimento ao desenvolvimento integral da criança. (Incluído pela Lei
sadio e harmonioso, em condições dignas de existência. nº 13.257, de 2016)

70
CONHECIMENTOS ESPECÍFICOS - PSICÓLOGO

Art. 9º O poder público, as instituições e os empre- para a permanência em tempo integral de um dos pais ou
gadores propiciarão condições adequadas ao aleitamento responsável, nos casos de internação de criança ou ado-
materno, inclusive aos filhos de mães submetidas a medida lescente. (Redação dada pela Lei nº 13.257, de 2016)
privativa de liberdade.
§ 1º Os profissionais das unidades primárias de saúde Art. 13. Os casos de suspeita ou confirmação de castigo
desenvolverão ações sistemáticas, individuais ou coletivas, físico, de tratamento cruel ou degradante e de maus-tratos
visando ao planejamento, à implementação e à avaliação contra criança ou adolescente serão obrigatoriamente co-
de ações de promoção, proteção e apoio ao aleitamento municados ao Conselho Tutelar da respectiva localidade,
materno e à alimentação complementar saudável, de forma sem prejuízo de outras providências legais. (Redação dada
contínua. (Incluído pela Lei nº 13.257, de 2016) pela Lei nº 13.010, de 2014)
§ 2º Os serviços de unidades de terapia intensiva § 1º As gestantes ou mães que manifestem interesse em
neonatal deverão dispor de banco de leite humano ou entregar seus filhos para adoção serão obrigatoriamente
unidade de coleta de leite humano. (Incluído pela Lei nº encaminhadas, sem constrangimento, à Justiça da Infância
13.257, de 2016) e da Juventude. (Incluído pela Lei nº 13.257, de 2016)
§ 2º Os serviços de saúde em suas diferentes portas
Art. 10. Os hospitais e demais estabelecimentos de de entrada, os serviços de assistência social em seu
atenção à saúde de gestantes, públicos e particulares, são componente especializado, o Centro de Referência
obrigados a: Especializado de Assistência Social (Creas) e os demais
I - manter registro das atividades desenvolvidas, atra- órgãos do Sistema de Garantia de Direitos da Criança
vés de prontuários individuais, pelo prazo de dezoito anos; e do Adolescente deverão conferir máxima prioridade
II - identificar o recém-nascido mediante o registro de ao atendimento das crianças na faixa etária da primeira
sua impressão plantar e digital e da impressão digital da infância com suspeita ou confirmação de violência de
mãe, sem prejuízo de outras formas normatizadas pela au- qualquer natureza, formulando projeto terapêutico
toridade administrativa competente; singular que inclua intervenção em rede e, se necessário,
III - proceder a exames visando ao diagnóstico e tera- acompanhamento domiciliar. (Incluído pela Lei nº 13.257,
pêutica de anormalidades no metabolismo do recém-nas- de 2016)
cido, bem como prestar orientação aos pais;
IV - fornecer declaração de nascimento onde constem Art. 14. O Sistema Único de Saúde promoverá progra-
necessariamente as intercorrências do parto e do desen- mas de assistência médica e odontológica para a preven-
volvimento do neonato; ção das enfermidades que ordinariamente afetam a popu-
V - manter alojamento conjunto, possibilitando ao lação infantil, e campanhas de educação sanitária para pais,
neonato a permanência junto à mãe. educadores e alunos.
Art. 11. É assegurado acesso integral às linhas de cui- § 1º É obrigatória a vacinação das crianças nos
dado voltadas à saúde da criança e do adolescente, por casos recomendados pelas autoridades sanitárias.
intermédio do Sistema Único de Saúde, observado o prin- (Renumerado do parágrafo único pela Lei nº 13.257, de
cípio da equidade no acesso a ações e serviços para pro- 2016)
moção, proteção e recuperação da saúde. (Redação dada § 2º O Sistema Único de Saúde promoverá a atenção
pela Lei nº 13.257, de 2016) à saúde bucal das crianças e das gestantes, de forma
§ 1º A criança e o adolescente com deficiência serão transversal, integral e intersetorial com as demais linhas de
atendidos, sem discriminação ou segregação, em suas cuidado direcionadas à mulher e à criança. (Incluído pela
necessidades gerais de saúde e específicas de habilitação Lei nº 13.257, de 2016)
e reabilitação. (Redação dada pela Lei nº 13.257, de 2016) § 3º A atenção odontológica à criança terá função
§ 2º Incumbe ao poder público fornecer gratuitamente, educativa protetiva e será prestada, inicialmente, antes de
àqueles que necessitarem, medicamentos, órteses, próteses o bebê nascer, por meio de aconselhamento pré-natal, e,
e outras tecnologias assistivas relativas ao tratamento, posteriormente, no sexto e no décimo segundo anos de
habilitação ou reabilitação para crianças e adolescentes, vida, com orientações sobre saúde bucal. (Incluído pela Lei
de acordo com as linhas de cuidado voltadas às suas nº 13.257, de 2016)
necessidades específicas. (Redação dada pela Lei nº § 4º A criança com necessidade de cuidados
13.257, de 2016) odontológicos especiais será atendida pelo Sistema Único
§ 3º Os profissionais que atuam no cuidado diário de Saúde. (Incluído pela Lei nº 13.257, de 2016)
ou frequente de crianças na primeira infância receberão
formação específica e permanente para a detecção de Capítulo II
sinais de risco para o desenvolvimento psíquico, bem como Do Direito à Liberdade, ao Respeito e à Dignidade
para o acompanhamento que se fizer necessário. (Incluído
pela Lei nº 13.257, de 2016) Art. 15. A criança e o adolescente têm direito à liberda-
de, ao respeito e à dignidade como pessoas humanas em
Art. 12. Os estabelecimentos de atendimento à saúde, processo de desenvolvimento e como sujeitos de direitos
inclusive as unidades neonatais, de terapia intensiva e de civis, humanos e sociais garantidos na Constituição e nas
cuidados intermediários, deverão proporcionar condições leis.

71
CONHECIMENTOS ESPECÍFICOS - PSICÓLOGO

Art. 16. O direito à liberdade compreende os seguintes I - encaminhamento a programa oficial ou comunitário
aspectos: de proteção à família; Incluído pela Lei nº 13.010, de 2014)
I - ir, vir e estar nos logradouros públicos e espaços II - encaminhamento a tratamento psicológico ou psi-
comunitários, ressalvadas as restrições legais; quiátrico; (Incluído pela Lei nº 13.010, de 2014)
II - opinião e expressão; III - encaminhamento a cursos ou programas de orien-
III - crença e culto religioso; tação; (Incluído pela Lei nº 13.010, de 2014)
IV - brincar, praticar esportes e divertir-se; IV - obrigação de encaminhar a criança a tratamento
V - participar da vida familiar e comunitária, sem dis- especializado; (Incluído pela Lei nº 13.010, de 2014)
criminação; V - advertência. (Incluído pela Lei nº 13.010, de 2014)
VI - participar da vida política, na forma da lei;
VII - buscar refúgio, auxílio e orientação. Parágrafo único. As medidas previstas neste artigo se-
rão aplicadas pelo Conselho Tutelar, sem prejuízo de outras
Art. 17. O direito ao respeito consiste na inviolabilida- providências legais. (Incluído pela Lei nº 13.010, de 2014)
de da integridade física, psíquica e moral da criança e do
adolescente, abrangendo a preservação da imagem, da
Capítulo III
identidade, da autonomia, dos valores, ideias e crenças, dos
Do Direito à Convivência Familiar e Comunitária
espaços e objetos pessoais.
Seção I
Disposições Gerais
Art. 18. É dever de todos velar pela dignidade da crian-
ça e do adolescente, pondo-os a salvo de qualquer tra-
tamento desumano, violento, aterrorizante, vexatório ou Art. 19. É direito da criança e do adolescente ser criado
constrangedor. e educado no seio de sua família e, excepcionalmente, em
família substituta, assegurada a convivência familiar e co-
Art. 18-A. A criança e o adolescente têm o direito de munitária, em ambiente que garanta seu desenvolvimento
ser educados e cuidados sem o uso de castigo físico ou de integral. (Redação dada pela Lei nº 13.257, de 2016)
tratamento cruel ou degradante, como formas de correção, § 1º Toda criança ou adolescente que estiver inserido
disciplina, educação ou qualquer outro pretexto, pelos pais, em programa de acolhimento familiar ou institucional
pelos integrantes da família ampliada, pelos responsáveis, terá sua situação reavaliada, no máximo, a cada 6 (seis)
pelos agentes públicos executores de medidas socioeduca- meses, devendo a autoridade judiciária competente, com
tivas ou por qualquer pessoa encarregada de cuidar deles, base em relatório elaborado por equipe interprofissional
tratá-los, educá-los ou protegê-los. (Incluído pela Lei nº ou multidisciplinar, decidir de forma fundamentada pela
13.010, de 2014) possibilidade de reintegração familiar ou colocação em
Parágrafo único. Para os fins desta Lei, considera-se: família substituta, em quaisquer das modalidades previstas
(Incluído pela Lei nº 13.010, de 2014) no art. 28 desta Lei.
I - castigo físico: ação de natureza disciplinar ou puni- § 2º A permanência da criança e do adolescente em
tiva aplicada com o uso da força física sobre a criança ou programa de acolhimento institucional não se prolongará
o adolescente que resulte em: (Incluído pela Lei nº 13.010, por mais de 2 (dois) anos, salvo comprovada necessidade
de 2014) que atenda ao seu superior interesse, devidamente
a) sofrimento físico; ou (Incluído pela Lei nº 13.010, de fundamentada pela autoridade judiciária.
2014) § 3º A manutenção ou a reintegração de criança ou
b) lesão; (Incluído pela Lei nº 13.010, de 2014) adolescente à sua família terá preferência em relação a
II - tratamento cruel ou degradante: conduta ou forma qualquer outra providência, caso em que será esta incluída
cruel de tratamento em relação à criança ou ao adolescen- em serviços e programas de proteção, apoio e promoção,
te que: (Incluído pela Lei nº 13.010, de 2014)
nos termos do § 1o do art. 23, dos incisos I e IV do caput do
a) humilhe; ou (Incluído pela Lei nº 13.010, de 2014)
art. 101 e dos incisos I a IV do caput do art. 129 desta Lei.
b) ameace gravemente; ou (Incluído pela Lei nº 13.010,
(Redação dada pela Lei nº 13.257, de 2016).
de 2014)
§ 4º Será garantida a convivência da criança e do
c) ridicularize. (Incluído pela Lei nº 13.010, de 2014)
adolescente com a mãe ou o pai privado de liberdade, por
Art. 18-B. Os pais, os integrantes da família ampliada, meio de visitas periódicas promovidas pelo responsável ou,
os responsáveis, os agentes públicos executores de medi- nas hipóteses de acolhimento institucional, pela entidade
das socioeducativas ou qualquer pessoa encarregada de responsável, independentemente de autorização judicial.
cuidar de crianças e de adolescentes, tratá-los, educá-los (Incluído pela Lei nº 12.962, de 2014)
ou protegê-los que utilizarem castigo físico ou tratamento
cruel ou degradante como formas de correção, disciplina, Art. 20. Os filhos, havidos ou não da relação do casa-
educação ou qualquer outro pretexto estarão sujeitos, sem mento, ou por adoção, terão os mesmos direitos e quali-
prejuízo de outras sanções cabíveis, às seguintes medidas, ficações, proibidas quaisquer designações discriminatórias
que serão aplicadas de acordo com a gravidade do caso: relativas à filiação.
(Incluído pela Lei nº 13.010, de 2014)

72
CONHECIMENTOS ESPECÍFICOS - PSICÓLOGO

Art. 21. O poder familiar será exercido, em igualdade Art. 27. O reconhecimento do estado de filiação é di-
de condições, pelo pai e pela mãe, na forma do que dispu- reito personalíssimo, indisponível e imprescritível, podendo
ser a legislação civil, assegurado a qualquer deles o direito ser exercitado contra os pais ou seus herdeiros, sem qual-
de, em caso de discordância, recorrer à autoridade judiciá- quer restrição, observado o segredo de Justiça.
ria competente para a solução da divergência.
Seção III
Art. 22. Aos pais incumbe o dever de sustento, guarda Da Família Substituta
e educação dos filhos menores, cabendo-lhes ainda, no in- Subseção I
teresse destes, a obrigação de cumprir e fazer cumprir as Disposições Gerais
determinações judiciais.
Parágrafo único. A mãe e o pai, ou os responsáveis, Art. 28. A colocação em família substituta far-se-á me-
têm direitos iguais e deveres e responsabilidades compar- diante guarda, tutela ou adoção, independentemente da
tilhados no cuidado e na educação da criança, devendo ser situação jurídica da criança ou adolescente, nos termos
resguardado o direito de transmissão familiar de suas cren- desta Lei.
ças e culturas, assegurados os direitos da criança estabele- § 1º Sempre que possível, a criança ou o adolescente
cidos nesta Lei. (Incluído pela Lei nº 13.257, de 2016) será previamente ouvido por equipe interprofissional,
respeitado seu estágio de desenvolvimento e grau de
Art. 23. A falta ou a carência de recursos materiais não compreensão sobre as implicações da medida, e terá sua
constitui motivo suficiente para a perda ou a suspensão do opinião devidamente considerada.
poder familiar. § 2º Tratando-se de maior de 12 (doze) anos de idade,
§ 1º Não existindo outro motivo que por si só autorize será necessário seu consentimento, colhido em audiência.
a decretação da medida, a criança ou o adolescente § 3º Na apreciação do pedido levar-se-á em conta
será mantido em sua família de origem, a qual deverá o grau de parentesco e a relação de afinidade ou de
obrigatoriamente ser incluída em serviços e programas afetividade, a fim de evitar ou minorar as consequências
oficiais de proteção, apoio e promoção. (Redação dada decorrentes da medida.
pela Lei nº 13.257, de 2016) § 4º Os grupos de irmãos serão colocados sob adoção,
§ 2º A condenação criminal do pai ou da mãe não tutela ou guarda da mesma família substituta, ressalvada
implicará a destituição do poder familiar, exceto na a comprovada existência de risco de abuso ou outra
hipótese de condenação por crime doloso, sujeito à pena situação que justifique plenamente a excepcionalidade de
de reclusão, contra o próprio filho ou filha. (Incluído pela solução diversa, procurando-se, em qualquer caso, evitar o
Lei nº 12.962, de 2014) rompimento definitivo dos vínculos fraternais.
§ 5º A colocação da criança ou adolescente em família
Art. 24. A perda e a suspensão do poder familiar serão substituta será precedida de sua preparação gradativa
decretadas judicialmente, em procedimento contraditório, e acompanhamento posterior, realizados pela equipe
nos casos previstos na legislação civil, bem como na hipó- interprofissional a serviço da Justiça da Infância e da
tese de descumprimento injustificado dos deveres e obri- Juventude, preferencialmente com o apoio dos técnicos
gações a que alude o art. 22. responsáveis pela execução da política municipal de
garantia do direito à convivência familiar.
Seção II § 6º Em se tratando de criança ou adolescente indígena
Da Família Natural ou proveniente de comunidade remanescente de quilombo,
é ainda obrigatório:
Art. 25. Entende-se por família natural a comunidade I - que sejam consideradas e respeitadas sua identi-
formada pelos pais ou qualquer deles e seus descendentes. dade social e cultural, os seus costumes e tradições, bem
Parágrafo único. Entende-se por família extensa ou como suas instituições, desde que não sejam incompatíveis
ampliada aquela que se estende para além da unidade com os direitos fundamentais reconhecidos por esta Lei e
pais e filhos ou da unidade do casal, formada por parentes pela Constituição Federal;
próximos com os quais a criança ou adolescente convive e II - que a colocação familiar ocorra prioritariamente no
mantém vínculos de afinidade e afetividade. seio de sua comunidade ou junto a membros da mesma
etnia;
Art. 26. Os filhos havidos fora do casamento poderão III - a intervenção e oitiva de representantes do órgão
ser reconhecidos pelos pais, conjunta ou separadamente, federal responsável pela política indigenista, no caso de
no próprio termo de nascimento, por testamento, median- crianças e adolescentes indígenas, e de antropólogos, pe-
te escritura ou outro documento público, qualquer que seja rante a equipe interprofissional ou multidisciplinar que irá
a origem da filiação. acompanhar o caso.
Parágrafo único. O reconhecimento pode preceder o
nascimento do filho ou suceder-lhe ao falecimento, se dei- Art. 29. Não se deferirá colocação em família substituta
xar descendentes. a pessoa que revele, por qualquer modo, incompatibilidade
com a natureza da medida ou não ofereça ambiente fami-
liar adequado.

73
CONHECIMENTOS ESPECÍFICOS - PSICÓLOGO

Art. 30. A colocação em família substituta não admi- § 4º Poderão ser utilizados recursos federais, estaduais,
tirá transferência da criança ou adolescente a terceiros ou distritais e municipais para a manutenção dos serviços
a entidades governamentais ou não-governamentais, sem de acolhimento em família acolhedora, facultando-se
autorização judicial. o repasse de recursos para a própria família acolhedora.
(Incluído pela Lei nº 13.257, de 2016)
Art. 31. A colocação em família substituta estrangeira
constitui medida excepcional, somente admissível na mo- Art. 35. A guarda poderá ser revogada a qualquer tem-
dalidade de adoção. po, mediante ato judicial fundamentado, ouvido o Minis-
tério Público.
Art. 32. Ao assumir a guarda ou a tutela, o responsável
prestará compromisso de bem e fielmente desempenhar o Subseção III
encargo, mediante termo nos autos. Da Tutela

Subseção II Art. 36. A tutela será deferida, nos termos da lei civil, a
Da Guarda pessoa de até 18 (dezoito) anos incompletos.
Parágrafo único. O deferimento da tutela pressupõe a
Art. 33. A guarda obriga a prestação de assistência prévia decretação da perda ou suspensão do poder familiar
material, moral e educacional à criança ou adolescente, e implica necessariamente o dever de guarda.
conferindo a seu detentor o direito de opor-se a terceiros,
inclusive aos pais. Art. 37. O tutor nomeado por testamento ou qualquer
§ 1º A guarda destina-se a regularizar a posse de fato, documento autêntico, conforme previsto no parágrafo úni-
podendo ser deferida, liminar ou incidentalmente, nos co do art. 1.729 da Lei no 10.406, de 10 de janeiro de 2002
procedimentos de tutela e adoção, exceto no de adoção - Código Civil, deverá, no prazo de 30 (trinta) dias após a
por estrangeiros. abertura da sucessão, ingressar com pedido destinado ao
§ 2º Excepcionalmente, deferir-se-á a guarda, fora dos controle judicial do ato, observando o procedimento pre-
casos de tutela e adoção, para atender a situações peculiares visto nos arts. 165 a 170 desta Lei.
ou suprir a falta eventual dos pais ou responsável, podendo Parágrafo único. Na apreciação do pedido, serão ob-
ser deferido o direito de representação para a prática de servados os requisitos previstos nos arts. 28 e 29 desta
atos determinados. Lei, somente sendo deferida a tutela à pessoa indicada na
§ 3º A guarda confere à criança ou adolescente a disposição de última vontade, se restar comprovado que
condição de dependente, para todos os fins e efeitos de a medida é vantajosa ao tutelando e que não existe outra
direito, inclusive previdenciários. pessoa em melhores condições de assumi-la.
§ 4º Salvo expressa e fundamentada determinação
em contrário, da autoridade judiciária competente, ou Art. 38. Aplica-se à destituição da tutela o disposto no
quando a medida for aplicada em preparação para adoção, art. 24.
o deferimento da guarda de criança ou adolescente a
terceiros não impede o exercício do direito de visitas Subseção IV
pelos pais, assim como o dever de prestar alimentos, que Da Adoção
serão objeto de regulamentação específica, a pedido do
interessado ou do Ministério Público. Art. 39. A adoção de criança e de adolescente reger-se-
-á segundo o disposto nesta Lei.
Art. 34. O poder público estimulará, por meio de assis- § 1º A adoção é medida excepcional e irrevogável,
tência jurídica, incentivos fiscais e subsídios, o acolhimento, à qual se deve recorrer apenas quando esgotados os
sob a forma de guarda, de criança ou adolescente afastado recursos de manutenção da criança ou adolescente na
do convívio familiar. família natural ou extensa, na forma do parágrafo único do
§ 1º A inclusão da criança ou adolescente em programas art. 25 desta Lei.
de acolhimento familiar terá preferência a seu acolhimento § 2º É vedada a adoção por procuração.
institucional, observado, em qualquer caso, o caráter
temporário e excepcional da medida, nos termos desta Lei. Art. 40. O adotando deve contar com, no máximo, de-
§ 2º Na hipótese do § 1o deste artigo a pessoa ou zoito anos à data do pedido, salvo se já estiver sob a guar-
casal cadastrado no programa de acolhimento familiar da ou tutela dos adotantes.
poderá receber a criança ou adolescente mediante guarda,
observado o disposto nos arts. 28 a 33 desta Lei. Art. 41. A adoção atribui a condição de filho ao adota-
§ 3º A União apoiará a implementação de serviços do, com os mesmos direitos e deveres, inclusive sucessó-
de acolhimento em família acolhedora como política rios, desligando-o de qualquer vínculo com pais e parentes,
pública, os quais deverão dispor de equipe que organize salvo os impedimentos matrimoniais.
o acolhimento temporário de crianças e de adolescentes § 1º Se um dos cônjuges ou concubinos adota o filho do
em residências de famílias selecionadas, capacitadas e outro, mantêm-se os vínculos de filiação entre o adotado
acompanhadas que não estejam no cadastro de adoção. e o cônjuge ou concubino do adotante e os respectivos
(Incluído pela Lei nº 13.257, de 2016) parentes.

74
CONHECIMENTOS ESPECÍFICOS - PSICÓLOGO

§ 2º É recíproco o direito sucessório entre o adotado, § 3º Em caso de adoção por pessoa ou casal residente
seus descendentes, o adotante, seus ascendentes, ou domiciliado fora do País, o estágio de convivência,
descendentes e colaterais até o 4º grau, observada a ordem cumprido no território nacional, será de, no mínimo, 30
de vocação hereditária. (trinta) dias.
§ 4º O estágio de convivência será acompanhado pela
Art. 42. Podem adotar os maiores de 18 (dezoito) anos, equipe interprofissional a serviço da Justiça da Infância e
independentemente do estado civil. da Juventude, preferencialmente com apoio dos técnicos
§ 1º Não podem adotar os ascendentes e os irmãos do responsáveis pela execução da política de garantia do
adotando. direito à convivência familiar, que apresentarão relatório
§ 2º Para adoção conjunta, é indispensável que os minucioso acerca da conveniência do deferimento da
adotantes sejam casados civilmente ou mantenham união medida.
estável, comprovada a estabilidade da família.
Art. 47. O vínculo da adoção constitui-se por sentença
§ 3º O adotante há de ser, pelo menos, dezesseis anos
judicial, que será inscrita no registro civil mediante manda-
mais velho do que o adotando. do do qual não se fornecerá certidão.
§ 4º Os divorciados, os judicialmente separados e os ex- § 1º A inscrição consignará o nome dos adotantes
companheiros podem adotar conjuntamente, contanto que como pais, bem como o nome de seus ascendentes.
acordem sobre a guarda e o regime de visitas e desde que o § 2º O mandado judicial, que será arquivado, cancelará
estágio de convivência tenha sido iniciado na constância do o registro original do adotado.
período de convivência e que seja comprovada a existência § 3º A pedido do adotante, o novo registro poderá ser
de vínculos de afinidade e afetividade com aquele não lavrado no Cartório do Registro Civil do Município de sua
detentor da guarda, que justifiquem a excepcionalidade da residência.
concessão. § 4º Nenhuma observação sobre a origem do ato
§ 5º Nos casos do § 4o deste artigo, desde que poderá constar nas certidões do registro.
demonstrado efetivo benefício ao adotando, será § 5º A sentença conferirá ao adotado o nome do
assegurada a guarda compartilhada, conforme previsto adotante e, a pedido de qualquer deles, poderá determinar
no art. 1.584 da Lei no 10.406, de 10 de janeiro de 2002 - a modificação do prenome.
Código Civil. § 6º Caso a modificação de prenome seja requerida pelo
§ 6º A adoção poderá ser deferida ao adotante que, adotante, é obrigatória a oitiva do adotando, observado o
após inequívoca manifestação de vontade, vier a falecer no disposto nos §§ 1o e 2o do art. 28 desta Lei.
§ 7º A adoção produz seus efeitos a partir do trânsito
curso do procedimento, antes de prolatada a sentença.
em julgado da sentença constitutiva, exceto na hipótese
prevista no § 6o do art. 42 desta Lei, caso em que terá força
Art. 43. A adoção será deferida quando apresentar retroativa à data do óbito.
reais vantagens para o adotando e fundar-se em motivos § 8º O processo relativo à adoção assim como outros a
legítimos. ele relacionados serão mantidos em arquivo, admitindo-se
seu armazenamento em microfilme ou por outros meios,
Art. 44. Enquanto não der conta de sua administração e garantida a sua conservação para consulta a qualquer
saldar o seu alcance, não pode o tutor ou o curador adotar tempo.
o pupilo ou o curatelado. § 9º Terão prioridade de tramitação os processos de
adoção em que o adotando for criança ou adolescente
Art. 45. A adoção depende do consentimento dos pais com deficiência ou com doença crônica. (Incluído pela Lei
ou do representante legal do adotando. nº 12.955, de 2014)
§ 1º. O consentimento será dispensado em relação à
criança ou adolescente cujos pais sejam desconhecidos ou Art. 48. O adotado tem direito de conhecer sua origem
tenham sido destituídos do poder familiar. biológica, bem como de obter acesso irrestrito ao processo
§ 2º. Em se tratando de adotando maior de doze anos no qual a medida foi aplicada e seus eventuais incidentes,
de idade, será também necessário o seu consentimento. após completar 18 (dezoito) anos.
Parágrafo único. O acesso ao processo de adoção po-
derá ser também deferido ao adotado menor de 18 (dezoi-
Art. 46. A adoção será precedida de estágio de con-
to) anos, a seu pedido, assegurada orientação e assistência
vivência com a criança ou adolescente, pelo prazo que a
jurídica e psicológica.
autoridade judiciária fixar, observadas as peculiaridades do
caso. Art. 49. A morte dos adotantes não restabelece o poder
§ 1º O estágio de convivência poderá ser dispensado familiar dos pais naturais.
se o adotando já estiver sob a tutela ou guarda legal do
adotante durante tempo suficiente para que seja possível Art. 50. A autoridade judiciária manterá, em cada co-
avaliar a conveniência da constituição do vínculo. marca ou foro regional, um registro de crianças e adoles-
§ 2º A simples guarda de fato não autoriza, por si só, a centes em condições de serem adotados e outro de pes-
dispensa da realização do estágio de convivência. soas interessadas na adoção.

75
CONHECIMENTOS ESPECÍFICOS - PSICÓLOGO

§ 1º O deferimento da inscrição dar-se-á após § 13. Somente poderá ser deferida adoção em
prévia consulta aos órgãos técnicos do juizado, ouvido o favor de candidato domiciliado no Brasil não cadastrado
Ministério Público. previamente nos termos desta Lei quando:
§ 2º Não será deferida a inscrição se o interessado não I - se tratar de pedido de adoção unilateral;
satisfazer os requisitos legais, ou verificada qualquer das II - for formulada por parente com o qual a criança ou
hipóteses previstas no art. 29. adolescente mantenha vínculos de afinidade e afetividade;
§ 3º A inscrição de postulantes à adoção será precedida III - oriundo o pedido de quem detém a tutela ou guar-
de um período de preparação psicossocial e jurídica, da legal de criança maior de 3 (três) anos ou adolescente,
orientado pela equipe técnica da Justiça da Infância e da desde que o lapso de tempo de convivência comprove a
Juventude, preferencialmente com apoio dos técnicos fixação de laços de afinidade e afetividade, e não seja cons-
responsáveis pela execução da política municipal de tatada a ocorrência de má-fé ou qualquer das situações
garantia do direito à convivência familiar. previstas nos arts. 237 ou 238 desta Lei.
§ 4º Sempre que possível e recomendável, a preparação § 14. Nas hipóteses previstas no § 13 deste artigo, o
referida no § 3o deste artigo incluirá o contato com crianças candidato deverá comprovar, no curso do procedimento,
e adolescentes em acolhimento familiar ou institucional que preenche os requisitos necessários à adoção, conforme
em condições de serem adotados, a ser realizado sob previsto nesta Lei.
a orientação, supervisão e avaliação da equipe técnica
da Justiça da Infância e da Juventude, com apoio dos Art. 51. Considera-se adoção internacional aquela na
técnicos responsáveis pelo programa de acolhimento e qual a pessoa ou casal postulante é residente ou domici-
pela execução da política municipal de garantia do direito liado fora do Brasil, conforme previsto no Artigo 2 da Con-
à convivência familiar. venção de Haia, de 29 de maio de 1993, Relativa à Proteção
§ 5º Serão criados e implementados cadastros estaduais das Crianças e à Cooperação em Matéria de Adoção Inter-
e nacional de crianças e adolescentes em condições de nacional, aprovada pelo Decreto Legislativo no 1, de 14 de
serem adotados e de pessoas ou casais habilitados à janeiro de 1999, e promulgada pelo Decreto no 3.087, de
adoção. 21 de junho de 1999.
§ 1º A adoção internacional de criança ou adolescente
§ 6º Haverá cadastros distintos para pessoas ou casais
brasileiro ou domiciliado no Brasil somente terá lugar
residentes fora do País, que somente serão consultados
quando restar comprovado:
na inexistência de postulantes nacionais habilitados nos
I - que a colocação em família substituta é a solução
cadastros mencionados no § 5o deste artigo.
adequada ao caso concreto;
§ 7º As autoridades estaduais e federais em matéria de
II - que foram esgotadas todas as possibilidades de
adoção terão acesso integral aos cadastros, incumbindo-
colocação da criança ou adolescente em família substituta
lhes a troca de informações e a cooperação mútua, para
brasileira, após consulta aos cadastros mencionados no art.
melhoria do sistema.
50 desta Lei;
§ 8º A autoridade judiciária providenciará, no prazo III - que, em se tratando de adoção de adolescente,
de 48 (quarenta e oito) horas, a inscrição das crianças e este foi consultado, por meios adequados ao seu estágio
adolescentes em condições de serem adotados que não de desenvolvimento, e que se encontra preparado para a
tiveram colocação familiar na comarca de origem, e das medida, mediante parecer elaborado por equipe interpro-
pessoas ou casais que tiveram deferida sua habilitação à fissional, observado o disposto nos §§ 1o e 2o do art. 28
adoção nos cadastros estadual e nacional referidos no § 5o desta Lei.
deste artigo, sob pena de responsabilidade. § 2º Os brasileiros residentes no exterior terão
§ 9º Compete à Autoridade Central Estadual zelar preferência aos estrangeiros, nos casos de adoção
pela manutenção e correta alimentação dos cadastros, internacional de criança ou adolescente brasileiro.
com posterior comunicação à Autoridade Central Federal § 3º A adoção internacional pressupõe a intervenção
Brasileira. das Autoridades Centrais Estaduais e Federal em matéria
§ 10. A adoção internacional somente será deferida de adoção internacional.
se, após consulta ao cadastro de pessoas ou casais
habilitados à adoção, mantido pela Justiça da Infância e da Art. 52. A adoção internacional observará o procedi-
Juventude na comarca, bem como aos cadastros estadual e mento previsto nos arts. 165 a 170 desta Lei, com as se-
nacional referidos no § 5o deste artigo, não for encontrado guintes adaptações:
interessado com residência permanente no Brasil. I - a pessoa ou casal estrangeiro, interessado em adotar
§ 11. Enquanto não localizada pessoa ou casal criança ou adolescente brasileiro, deverá formular pedido
interessado em sua adoção, a criança ou o adolescente, de habilitação à adoção perante a Autoridade Central em
sempre que possível e recomendável, será colocado sob matéria de adoção internacional no país de acolhida, assim
guarda de família cadastrada em programa de acolhimento entendido aquele onde está situada sua residência habi-
familiar. tual;
§ 12. A alimentação do cadastro e a convocação II - se a Autoridade Central do país de acolhida con-
criteriosa dos postulantes à adoção serão fiscalizadas pelo siderar que os solicitantes estão habilitados e aptos para
Ministério Público. adotar, emitirá um relatório que contenha informações so-

76
CONHECIMENTOS ESPECÍFICOS - PSICÓLOGO

bre a identidade, a capacidade jurídica e adequação dos § 4º Os organismos credenciados deverão ainda:
solicitantes para adotar, sua situação pessoal, familiar e I - perseguir unicamente fins não lucrativos, nas condi-
médica, seu meio social, os motivos que os animam e sua ções e dentro dos limites fixados pelas autoridades compe-
aptidão para assumir uma adoção internacional; tentes do país onde estiverem sediados, do país de acolhi-
III - a Autoridade Central do país de acolhida enviará o da e pela Autoridade Central Federal Brasileira;
relatório à Autoridade Central Estadual, com cópia para a II - ser dirigidos e administrados por pessoas qualifi-
Autoridade Central Federal Brasileira; cadas e de reconhecida idoneidade moral, com comprova-
IV - o relatório será instruído com toda a documen- da formação ou experiência para atuar na área de adoção
tação necessária, incluindo estudo psicossocial elaborado internacional, cadastradas pelo Departamento de Polícia
por equipe interprofissional habilitada e cópia autenticada Federal e aprovadas pela Autoridade Central Federal Bra-
da legislação pertinente, acompanhada da respectiva prova
sileira, mediante publicação de portaria do órgão federal
de vigência;
competente;
V - os documentos em língua estrangeira serão devida-
mente autenticados pela autoridade consular, observados III - estar submetidos à supervisão das autoridades
os tratados e convenções internacionais, e acompanhados competentes do país onde estiverem sediados e no país
da respectiva tradução, por tradutor público juramentado; de acolhida, inclusive quanto à sua composição, funciona-
VI - a Autoridade Central Estadual poderá fazer exi- mento e situação financeira;
gências e solicitar complementação sobre o estudo psicos- IV - apresentar à Autoridade Central Federal Brasileira,
social do postulante estrangeiro à adoção, já realizado no a cada ano, relatório geral das atividades desenvolvidas,
país de acolhida; bem como relatório de acompanhamento das adoções in-
VII - verificada, após estudo realizado pela Autoridade ternacionais efetuadas no período, cuja cópia será encami-
Central Estadual, a compatibilidade da legislação estran- nhada ao Departamento de Polícia Federal;
geira com a nacional, além do preenchimento por parte V - enviar relatório pós-adotivo semestral para a Au-
dos postulantes à medida dos requisitos objetivos e sub- toridade Central Estadual, com cópia para a Autoridade
jetivos necessários ao seu deferimento, tanto à luz do que Central Federal Brasileira, pelo período mínimo de 2 (dois)
dispõe esta Lei como da legislação do país de acolhida, anos. O envio do relatório será mantido até a juntada de
será expedido laudo de habilitação à adoção internacional, cópia autenticada do registro civil, estabelecendo a cidada-
que terá validade por, no máximo, 1 (um) ano; nia do país de acolhida para o adotado;
VIII - de posse do laudo de habilitação, o interessado VI - tomar as medidas necessárias para garantir que os
será autorizado a formalizar pedido de adoção perante o
adotantes encaminhem à Autoridade Central Federal Bra-
Juízo da Infância e da Juventude do local em que se encon-
sileira cópia da certidão de registro de nascimento estran-
tra a criança ou adolescente, conforme indicação efetuada
pela Autoridade Central Estadual. geira e do certificado de nacionalidade tão logo lhes sejam
§ 1º Se a legislação do país de acolhida assim o concedidos.
autorizar, admite-se que os pedidos de habilitação à § 5º A não apresentação dos relatórios referidos no § 4o
adoção internacional sejam intermediados por organismos deste artigo pelo organismo credenciado poderá acarretar
credenciados. a suspensão de seu credenciamento.
§ 2º Incumbe à Autoridade Central Federal Brasileira § 6º O credenciamento de organismo nacional ou
o credenciamento de organismos nacionais e estrangeiros estrangeiro encarregado de intermediar pedidos de adoção
encarregados de intermediar pedidos de habilitação à internacional terá validade de 2 (dois) anos.
adoção internacional, com posterior comunicação às § 7º A renovação do credenciamento poderá ser
Autoridades Centrais Estaduais e publicação nos órgãos concedida mediante requerimento protocolado na
oficiais de imprensa e em sítio próprio da internet. Autoridade Central Federal Brasileira nos 60 (sessenta) dias
§ 3º Somente será admissível o credenciamento de anteriores ao término do respectivo prazo de validade.
organismos que: § 8º Antes de transitada em julgado a decisão que
I - sejam oriundos de países que ratificaram a Con- concedeu a adoção internacional, não será permitida a
venção de Haia e estejam devidamente credenciados pela saída do adotando do território nacional.
Autoridade Central do país onde estiverem sediados e no § 9º Transitada em julgado a decisão, a autoridade
país de acolhida do adotando para atuar em adoção inter-
judiciária determinará a expedição de alvará com
nacional no Brasil;
autorização de viagem, bem como para obtenção de
II - satisfizerem as condições de integridade moral,
competência profissional, experiência e responsabilidade passaporte, constando, obrigatoriamente, as características
exigidas pelos países respectivos e pela Autoridade Central da criança ou adolescente adotado, como idade, cor, sexo,
Federal Brasileira; eventuais sinais ou traços peculiares, assim como foto
III - forem qualificados por seus padrões éticos e sua recente e a aposição da impressão digital do seu polegar
formação e experiência para atuar na área de adoção in- direito, instruindo o documento com cópia autenticada da
ternacional; decisão e certidão de trânsito em julgado.
IV - cumprirem os requisitos exigidos pelo ordenamen- § 10. A Autoridade Central Federal Brasileira poderá, a
to jurídico brasileiro e pelas normas estabelecidas pela Au- qualquer momento, solicitar informações sobre a situação
toridade Central Federal Brasileira. das crianças e adolescentes adotados.

77
CONHECIMENTOS ESPECÍFICOS - PSICÓLOGO

§ 11. A cobrança de valores por parte dos organismos § 2º Na hipótese de não reconhecimento da adoção,
credenciados, que sejam considerados abusivos prevista no § 1o deste artigo, o Ministério Público deverá
pela Autoridade Central Federal Brasileira e que não imediatamente requerer o que for de direito para resguardar
estejam devidamente comprovados, é causa de seu os interesses da criança ou do adolescente, comunicando-
descredenciamento. se as providências à Autoridade Central Estadual, que fará
§ 12. Uma mesma pessoa ou seu cônjuge não podem a comunicação à Autoridade Central Federal Brasileira e à
ser representados por mais de uma entidade credenciada Autoridade Central do país de origem.
para atuar na cooperação em adoção internacional.
§ 13. A habilitação de postulante estrangeiro ou Art. 52-D. Nas adoções internacionais, quando o Brasil
domiciliado fora do Brasil terá validade máxima de 1 (um) for o país de acolhida e a adoção não tenha sido deferida
ano, podendo ser renovada. no país de origem porque a sua legislação a delega ao país
§ 14. É vedado o contato direto de representantes de de acolhida, ou, ainda, na hipótese de, mesmo com deci-
organismos de adoção, nacionais ou estrangeiros, com são, a criança ou o adolescente ser oriundo de país que não
dirigentes de programas de acolhimento institucional tenha aderido à Convenção referida, o processo de adoção
ou familiar, assim como com crianças e adolescentes em seguirá as regras da adoção nacional.
condições de serem adotados, sem a devida autorização
judicial. Capítulo IV
§ 15. A Autoridade Central Federal Brasileira poderá Do Direito à Educação, à Cultura, ao Esporte e ao
limitar ou suspender a concessão de novos credenciamentos Lazer
sempre que julgar necessário, mediante ato administrativo
fundamentado. Art. 53. A criança e o adolescente têm direito à edu-
cação, visando ao pleno desenvolvimento de sua pessoa,
Art. 52-A. É vedado, sob pena de responsabilidade e preparo para o exercício da cidadania e qualificação para o
descredenciamento, o repasse de recursos provenientes de trabalho, assegurando-se-lhes:
organismos estrangeiros encarregados de intermediar pe- I - igualdade de condições para o acesso e permanên-
didos de adoção internacional a organismos nacionais ou cia na escola;
II - direito de ser respeitado por seus educadores;
a pessoas físicas.
III - direito de contestar critérios avaliativos, podendo
Parágrafo único. Eventuais repasses somente poderão
recorrer às instâncias escolares superiores;
ser efetuados via Fundo dos Direitos da Criança e do Ado-
IV - direito de organização e participação em entidades
lescente e estarão sujeitos às deliberações do respectivo
estudantis;
Conselho de Direitos da Criança e do Adolescente.
V - acesso à escola pública e gratuita próxima de sua
Art. 52-B. A adoção por brasileiro residente no exterior
residência.
em país ratificante da Convenção de Haia, cujo processo de
Parágrafo único. É direito dos pais ou responsáveis ter
adoção tenha sido processado em conformidade com a le-
ciência do processo pedagógico, bem como participar da
gislação vigente no país de residência e atendido o dispos- definição das propostas educacionais.
to na Alínea “c” do Artigo 17 da referida Convenção, será
automaticamente recepcionada com o reingresso no Brasil. Art. 54. É dever do Estado assegurar à criança e ao ado-
§ 1º Caso não tenha sido atendido o disposto na Alínea lescente:
“c” do Artigo 17 da Convenção de Haia, deverá a sentença I - ensino fundamental, obrigatório e gratuito, inclusive
ser homologada pelo Superior Tribunal de Justiça. para os que a ele não tiveram acesso na idade própria;
§ 2º O pretendente brasileiro residente no exterior II - progressiva extensão da obrigatoriedade e gratui-
em país não ratificante da Convenção de Haia, uma vez dade ao ensino médio;
reingressado no Brasil, deverá requerer a homologação da III - atendimento educacional especializado aos por-
sentença estrangeira pelo Superior Tribunal de Justiça. tadores de deficiência, preferencialmente na rede regular
de ensino;
Art. 52-C. Nas adoções internacionais, quando o Brasil IV - IV – atendimento em creche e pré-escola às crian-
for o país de acolhida, a decisão da autoridade competente ças de zero a cinco anos de idade; (Redação dada pela Lei
do país de origem da criança ou do adolescente será co- nº 13.306, de 2016);
nhecida pela Autoridade Central Estadual que tiver proces- V - acesso aos níveis mais elevados do ensino, da pes-
sado o pedido de habilitação dos pais adotivos, que comu- quisa e da criação artística, segundo a capacidade de cada
nicará o fato à Autoridade Central Federal e determinará um;
as providências necessárias à expedição do Certificado de VI - oferta de ensino noturno regular, adequado às
Naturalização Provisório. condições do adolescente trabalhador;
§ 1º A Autoridade Central Estadual, ouvido o Ministério VII - atendimento no ensino fundamental, através de
Público, somente deixará de reconhecer os efeitos programas suplementares de material didático-escolar,
daquela decisão se restar demonstrado que a adoção é transporte, alimentação e assistência à saúde.
manifestamente contrária à ordem pública ou não atende § 1º O acesso ao ensino obrigatório e gratuito é direito
ao interesse superior da criança ou do adolescente. público subjetivo.

78
CONHECIMENTOS ESPECÍFICOS - PSICÓLOGO

§ 2º O não oferecimento do ensino obrigatório Art. 65. Ao adolescente aprendiz, maior de quatorze
pelo poder público ou sua oferta irregular importa anos, são assegurados os direitos trabalhistas e previden-
responsabilidade da autoridade competente. ciários.
§ 3º Compete ao poder público recensear os educandos
no ensino fundamental, fazer-lhes a chamada e zelar, junto Art. 66. Ao adolescente portador de deficiência é asse-
aos pais ou responsável, pela frequência à escola. gurado trabalho protegido.
Art. 55. Os pais ou responsável têm a obrigação de ma-
tricular seus filhos ou pupilos na rede regular de ensino. Art. 67. Ao adolescente empregado, aprendiz, em regi-
me familiar de trabalho, aluno de escola técnica, assistido
Art. 56. Os dirigentes de estabelecimentos de ensino em entidade governamental ou não-governamental, é ve-
fundamental comunicarão ao Conselho Tutelar os casos de: dado trabalho:
I - maus-tratos envolvendo seus alunos; I - noturno, realizado entre as vinte e duas horas de um
II - reiteração de faltas injustificadas e de evasão esco- dia e as cinco horas do dia seguinte;
lar, esgotados os recursos escolares; II - perigoso, insalubre ou penoso;
III - elevados níveis de repetência. III - realizado em locais prejudiciais à sua formação e ao
seu desenvolvimento físico, psíquico, moral e social;
Art. 57. O poder público estimulará pesquisas, expe- IV - realizado em horários e locais que não permitam a
riências e novas propostas relativas a calendário, seriação, frequência à escola.
currículo, metodologia, didática e avaliação, com vistas à
inserção de crianças e adolescentes excluídos do ensino Art. 68. O programa social que tenha por base o traba-
fundamental obrigatório. lho educativo, sob responsabilidade de entidade governa-
mental ou não-governamental sem fins lucrativos, deverá
Art. 58. No processo educacional respeitar-se-ão os va- assegurar ao adolescente que dele participe condições de
lores culturais, artísticos e históricos próprios do contexto capacitação para o exercício de atividade regular remune-
social da criança e do adolescente, garantindo-se a estes a rada.
liberdade da criação e o acesso às fontes de cultura. § 1º Entende-se por trabalho educativo a atividade
laboral em que as exigências pedagógicas relativas ao
desenvolvimento pessoal e social do educando prevalecem
Art. 59. Os municípios, com apoio dos estados e da
sobre o aspecto produtivo.
União, estimularão e facilitarão a destinação de recursos e
§ 2º A remuneração que o adolescente recebe pelo
espaços para programações culturais, esportivas e de lazer
trabalho efetuado ou a participação na venda dos produtos
voltadas para a infância e a juventude.
de seu trabalho não desfigura o caráter educativo.
Capítulo V
Art. 69. O adolescente tem direito à profissionalização
Do Direito à Profissionalização e à Proteção no
e à proteção no trabalho, observados os seguintes aspec-
Trabalho tos, entre outros:
I - respeito à condição peculiar de pessoa em desen-
Art. 60. É proibido qualquer trabalho a menores de volvimento;
quatorze anos de idade, salvo na condição de aprendiz. II - capacitação profissional adequada ao mercado de
(Vide Constituição Federal) trabalho.
Art. 61. A proteção ao trabalho dos adolescentes é re- A Doutrina da Proteção Integral no cenário da in-
gulada por legislação especial, sem prejuízo do disposto fância e adolescência brasileira
nesta Lei.
A Constituição da República Federativa do Brasil de
Art. 62. Considera-se aprendizagem a formação técni- 1988 foi inovadora ao adotar a Doutrina da Proteção In-
co-profissional ministrada segundo as diretrizes e bases da tegral na questão da infância e adolescência no Brasil. A
legislação de educação em vigor. referida doutrina teve seu crescimento primeiramente em
âmbito internacional, em convenções e documentos na
Art. 63. A formação técnico-profissional obedecerá aos área da criança, dentre os quais se destaca a Convenção In-
seguintes princípios: ternacional sobre os Direitos da Criança de 1989, aprovada
I - garantia de acesso e frequência obrigatória ao en- por unanimidade pela Assembleia Geral das Nações Uni-
sino regular; das. Conforme Liberati (2003, p. 20), a Convenção “repre-
II - atividade compatível com o desenvolvimento do sentou até agora, dentro do panorama legal internacional,
adolescente; o resumo e a conclusão de toda a legislação garantista de
III - horário especial para o exercício das atividades. proteção à infância”.
A Convenção definiu a base da Doutrina da Proteção
Art. 64. Ao adolescente até quatorze anos de idade é Integral ao proclamar um conjunto de direitos de nature-
assegurada bolsa de aprendizagem. za individual, difusa, coletiva, econômica, social e cultural,

79
CONHECIMENTOS ESPECÍFICOS - PSICÓLOGO

reconhecendo que criança e adolescente são sujeitos de [...]”. Assim, a doutrina da situação irregular não atingia a
direitos e, considerando sua vulnerabilidade, necessitam de totalidade de crianças e adolescentes, mas somente des-
cuidados e proteção especiais. Exige a Convenção, com for- tinava-se àqueles que representavam um obstáculo à or-
ça de lei internacional, que os países signatários adaptem dem, considerados como tais, os abandonados, expostos,
as legislações às suas disposições e os compromete a não transviados, delinquentes, infratores, vadios, pobres, que
violarem seus preceitos, instituindo, para isto, mecanismos recebiam todos do Estado a mesma resposta assistencialis-
de controle e fiscalização. (VERONESE; OLIVEIRA, 2008). ta, repressiva e institucionalizante. (CUSTÓDIO; VERONESE,
O Brasil, com base nas discussões sobre a Convenção, 2009).
adota no texto constitucional de 1988 a Doutrina da Prote-
ção Integral, consagrando-a em seu art. 227. Pela nova ordem estabelecida, criança e adolescente
são sujeitos de direitos e não simplesmente objetos de
“É dever da família, da sociedade e do Estado assegurar intervenção no mundo adulto, portadores não só de uma
à criança, ao adolescente e ao jovem[5], com absoluta prio- proteção jurídica comum que é reconhecida para todas as
ridade, o direito à vida, à saúde, à alimentação, à educação, pessoas, mas detém ainda uma “supraproteção ou prote-
ao lazer, à profissionalização, à cultura, à dignidade, ao res- ção complementar de seus direitos”. (BRUNÕL, 2001, p.92).
peito, à liberdade e à convivência familiar e comunitária, A proteção é dirigida ao conjunto de todas as crianças e
além de colocá-los a salvo de toda forma de negligência, adolescentes, não cabendo exceção.
discriminação, exploração, violência, crueldade e opressão.” O artigo 3º do Estatuto da Criança e do Adolescente
Segundo Saraiva (2002), pela primeira vez na história esclarece a proteção complementar instaurada pela nova
brasileira, a questão da criança e do adolescente é aborda- doutrina, ao afirmar que `a criança e ao adolescente são
da como prioridade absoluta e a sua proteção passa a ser garantidos todos os direitos fundamentais inerentes a pes-
dever da família, da sociedade e do Estado. soa humana, bem como são sujeitos a proteção integral.
Contudo, a interferência prática desta opção consti- “Art.3°A criança e o adolescente gozam de todos os
tucional coube à legislação especial, aprovada em 13 de direitos fundamentais inerentes à pessoa humana, sem
julho de 1990, através da promulgação da Lei Federal Nº prejuízo da proteção integral de que trata esta Lei, asse-
8.069/90 – o Estatuto da Criança e do Adolescente. gurando-se-lhes, por lei ou por outros meios, todas as
“A gama de direitos elencados basicamente no art. 227 oportunidades e facilidades, a fim de lhes facultar o de-
da Constituição Federal, os quais constituem direitos fun- senvolvimento físico, mental, moral, espiritual e social, em
damentais, de extrema relevância, não só pelo seu conteú- condições de liberdade e de dignidade.”
do como pela sua titularidade, devem, obrigatoriamente,
ser garantidos pelo Estatuto, e uma forma de tornar con- Fica evidenciado o princípio da igualdade de todas as
creta essa garantia deu-se, justamente, por meio do Estatu- crianças e adolescentes, estes compreendidos como todos
to da Criança e do Adolescente, o qual tem a nobre e difícil os seres humanos que contam entre zero e 18 anos, ou
tarefa de materializar o preceito constitucional.” (VERONE- seja, não há categorias distintas de crianças e adolescentes,
SE, 1996, p. 94). apesar de estarem em situações sociais, econômicas e cul-
turais diferenciadas.
Deste modo, para Veronese (1996) o surgimento de Lembra Machado (2003) que sistema especial de pro-
uma legislação que tratasse crianças e adolescentes como teção tem por base a vulnerabilidade peculiar de crianças e
sujeitos de direitos era imprescindível, evitando que os pre- adolescentes, que por sua vez influencia na aparente que-
ceitos constitucionais fossem reduzidos a meras intenções. bra do princípio da igualdade, isto por que:
Sendo crianças e adolescentes titulares de direitos próprios “a) distingue crianças e adolescentes de outros gru-
e especiais, em razão de sua condição específica de pes- pos de seres humanos simplesmente diversos da noção
soas em desenvolvimento, tornou-se necessária a existên- do homo médio; b) autoriza e opera a aparente quebra do
cia de uma proteção especializada, diferenciada, integral. princípio da igualdade – porque são portadores de uma
Complementa Paula (2002) ser da própria essência do desigualdade inerente, intrínseca, o ordenamento confere-
Direito da Criança e do Adolescente a presença da prote- -lhes tratamento mais abrangente como forma de equili-
ção integral: brar a desigualdade de fato e atingir a igualdade jurídica
“[...] me parece que a locução proteção integral seja material e não meramente formal.” (MACHADO, 2003).
autoexplicativa [...] Proteção Integral exprime finalidades Assim, com base na supremacia que o valor da dignida-
básicas relacionadas às garantias do desenvolvimento sau- de da pessoa humana recebeu na Constituição da Repúbli-
dável e da integridade, materializadas em normas subor- ca Federativa do Brasil de 1988, foi inaugurado um sistema
dinantes que propiciam a apropriação e manutenção dos especial de proteção à infância, expressamente referido no
bens da vida necessários para atingir destes objetivos. ” parágrafo 3º do artigo 227, também no artigo 228, artigo
(PAULA, 2002). 226, caput §§ 3º, 4º, 5º e 8º e 229, primeira parte da CF/88.
A Doutrina da Proteção Integral veio contrapor a Dou- Ainda, XXX e XXXIII do artigo 7º, e § 3º do artigo 208.
trina da Situação Irregular então vigente instituída pelo Extrai-se do art. 227 da Constituição Federal e art. 4º
Código de Menores de 1979, “[...] onde a criança era vista do Estatuto da Criança e do Adolescente que o dever de
como problema social, um risco à estabilidade, às vezes até assegurar este sistema especial de proteção cabe à família,
uma ameaça à ordem social [...] a infância era um mero comunidade, sociedade em geral, poder público, que o fa-
objeto de intervenção do Estado regulador da propriedade rão com absoluta prioridade.

80
CONHECIMENTOS ESPECÍFICOS - PSICÓLOGO

Liberati (2003) entende prioridade absoluta como es- Ocorre que a efetivação dos direitos fundamentais de
tar a criança e o adolescente em primeiro lugar na escala cidadania pressupõe a criação de um Sistema de Garan-
de preocupações dos governantes, que em primeiro lugar tia de Direitos, que atue na perspectiva da promoção, da
devem ser atendidas as necessidades das crianças e ado- defesa e do controle. Este direito deve ser produzido na
lescentes. Exemplifica: sociedade, onde se experimenta um intenso processo de
“Por absoluta prioridade, entende-se que, na área correlações de forças, considerando a histórica postura de
administrativa, enquanto não existirem creches, escolas, negligência e arbitrariedade com crianças e adolescentes
postos de saúde, atendimento preventivo e emergencial no Brasil.
às gestantes, dignas moradias e trabalho, não se deverão
asfaltar ruas, construir praças, sambódromos, monumentos Crianças e adolescentes são sujeitos de direitos fun-
artísticos etc, porque a vida, a saúde, o lar, a prevenção de damentais especiais
doenças são mais importantes que as obras de concreto,
que ficam para demonstrar o poder do governante.” (LIBE- A Doutrina da Proteção Integral instaurou um sistema
RATI, 2003). especial de proteção, delineando direitos nos artigos 227
e 228 da Constituição brasileira, tornando crianças e ado-
A lei ordinária nº 8.069/90, no parágrafo único do ar- lescentes sujeitos dos direitos fundamentais atribuídos a
tigo 4º, detalhou a garantia da prioridade absoluta como todos os cidadãos e ainda titulares de direitos especiais,
sendo: a) primazia de receber proteção e socorro em quais- com base na sua peculiar condição de pessoa em desen-
quer circunstâncias; b) precedência de atendimento nos volvimento.
serviços públicos ou de relevância pública; c) preferência Machado (2003) afirma serem os direitos elencados
na formulação e na execução das políticas sociais públicas; no caput do artigo 227 e 228 da CF/88 também direitos
d) destinação privilegiada de recursos públicos nas áreas fundamentais da pessoa humana, pois o direito à vida, à
relacionadas com a proteção à infância e à juventude. liberdade, à igualdade mencionados no caput do artigo 5º
Outra base que sustenta a nova doutrina é a com- da CF referem-se a mesma vida, liberdade, igualdade des-
preensão de que crianças e adolescentes estão em pecu- critas no artigo 227 e § 3º do artigo 228, ou seja, tratam-
liar condição de pessoas humanas em desenvolvimento, -se de direitos da mesma natureza, sendo todos direitos
encontram-se em situação especial e de maior vulnerabili- fundamentais.
dade, ainda não desenvolveram completamente sua perso- Porém, os direitos fundamentais de que trata o artigo
nalidade, o que enseja um regime especial de salvaguarda,
227 são direitos fundamentais de uma pessoa humana de
o que lhes permite construir suas potencialidades humanas
condições especiais, qual seja pessoa humana em fase de
em plenitude.
desenvolvimento. Neste sentido, Bobbio (2002, p.35) apon-
Neste sentido, afirma Machado (2003) que o direito
ta como sendo singular a proteção destinada às crianças e
peculiar de crianças e adolescentes desenvolver sua perso-
adolescentes:
nalidade humana adulta integra os direitos da personalida-
“Se se diz que “criança, por causa de sua imaturidade
de e é relevante tal noção por estar ligada estruturalmente
física e intelectual, necessita de uma proteção particular e
a distinção que os direitos da crianças e adolescentes rece-
bem do texto constitucional. de cuidados especiais”, deixa-se assim claro que os direitos
“[...] sustento, pode-se afirmar, ao menos sob uma da criança são considerados como um ius singulare com
ótica principiológica ou conceitual, que a possibilidade de relação a um ius commne; o destaque que se dá a essa
formar a personalidade humana adulta – que é exatamente especificidade do genérico, no qual se realiza o respeito à
o que estão “fazendo” crianças e adolescentes pelo sim- máxima suum cuique tribuere. ”.
ples fato de crescerem até a condição adulta – há de ser Os direitos fundamentais de crianças e adolescentes
reconhecida como direito fundamental do ser humano, são especiais e, de acordo com Machado (2003), eles po-
porque sem ela nem poderiam ser os demais direitos da dem ser diferenciados do direito dos adultos por dois as-
personalidade adulta, ou a própria personalidade adulta.” pectos, sendo um quantitativo, pois crianças e adolescen-
(MACHADO, 2003). tes são beneficiários de mais direitos do que os adultos, e
Entretanto, frisa a autora, que a personalidade infan- ainda podem ser classificados pelo seu aspecto qualitativo
to-juvenil não é valorizada somente como meio de o ser ou estrutural, por estarem os titulares de tais direitos em
humano atingir a personalidade adulta, isto seria um equí- peculiar condição de desenvolvimento.
voco, uma vez que a vida humana tem dignidade em si Na sequência serão analisados os direitos fundamen-
mesma, em todos os momentos da vida, seja no mais frágil, tais de crianças e adolescentes, apresentando certo deta-
como no momento em que o recém-nascido respira, seja lhamento sobre cada um deles. Tendo em vista a extensa
no momento de ápice do potencial de criação intelectual gama de direitos fundamentais, optou-se por realizada a
de um ser humano. Assim, o que gera e justifica a positiva- abordagem dos direitos elencados no art. 227 da CF, quais
ção da proteção especial às crianças e adolescentes não é sejam: “direito à vida, à saúde, à alimentação, à educação,
meramente a sua condição de seres diversos dos adultos, ao lazer, à profissionalização, à cultura, à dignidade, ao res-
mas soma-se a isto a maior vulnerabilidade destes em rela- peito, à liberdade e à convivência familiar e comunitária”.
ção aos seres humanos adultos, bem como a força poten-
cial que a infância e juventude representam à sociedade.
(MACHADO, 2003).

81
CONHECIMENTOS ESPECÍFICOS - PSICÓLOGO

Direito à Vida e à Saúde sem as duas pernas, seria indigno que se arrastasse no in-
tuito de se locomover, neste caso caberia providenciar uma
O Estatuto da Criança e do Adolescente e a Constitui- cadeira de rodas, eventual cirurgia para colocação de pró-
ção da República Federativa do Brasil de 1988 iniciam a tese, enfim todos os meios para assegurar dignidade na
exposição dos direitos fundamentais pelo direito à vida e forma de viver.
à saúde. No artigo 7º do ECA, lê-se: “A criança e o adoles-
cente têm direito a proteção à vida e à saúde, mediante a Direito à Alimentação
efetivação de políticas que permitam o nascimento e o de-
senvolvimento sadio e harmonioso, em condições dignas O art. 227 da Constituição Federal inclui, logo após o
de existência”. direito à vida e à saúde, o direito à alimentação no rol dos
O próprio ECA preceitua várias medidas de caráter pre- direitos fundamentais de crianças e adolescentes.
ventivo, além de políticas públicas que permitam o nas- É um direito especial de crianças e adolescentes
cimento sadio, configurando-se, segundo Elias (2005) o positivado, levando em consideração a maior
direito de nascer. vulnerabilidade por estarem em peculiar condição de
Assegura-se à gestante o atendimento pré e perinatal, pessoa em desenvolvimento. Este direito tem estreita
pelo Sistema Único de Saúde (art. 8). Às mães é assegurado ligação com o direito à vida e direito ao não- trabalho.
o aleitamento materno, mesmo se estiverem submetidas a Assim, a positivação deste direito criou para o Estado
medida privativa de liberdade (art.9). Aos hospitais e de- o dever de assegurar alimentação a todas as crianças e
mais estabelecimentos são impostas obrigações, tais como adolescentes que não tenham acesso a ela por meio dos
a manutenção de registros (prontuários) pelo período de pais ou responsáveis e, ainda, faz nascer o direito individual
18 anos, identificação do recém-nascido, proceder a exa- de exigir esta prestação. (MACHADO, 2003).
mes acerca de anormalidades no metabolismo do recém- Conforme determina o art. 1.696 do Código Civil de
-nascido, prestar orientação aos pais, fornecer declaração 2002, “o direito à prestação de alimentos é recíproco entre
de nascimento onde constem as intercorrências do parto e pais e filhos, e extensivo a todos os ascendentes, recain-
do desenvolvimento do neonato (art. 10). do a obrigação nos mais próximos em grau, uns na falta
Ainda, o Estatuto da Criança e do Adolescente garante de outros”, assim na falta dos genitores poderá a criança
o tratamento igualitário de todos os sujeitos, independen- e o adolescente pleitear os alimentos dos outros parentes,
temente da condição social (art. 11). Os portadores de de- respeitando a ordem de sucessão. Define o art. 2° da Lei
ficientes receberão tratamento especializado (§ 1º), incum- de Alimentos, n. 5.478/68, que o credor, ao postular pela
bindo ao poder público o fornecimento gratuito de medi- concessão dos alimentos, exporá suas necessidades e pro-
camentos, próteses e outros recursos quando necessários vará apenas o parentesco ou a obrigação de alimentar do
(§ 2º). No caso de internação da criança e do adolescente, devedor.
os hospitais deverão propiciar condições para que um dos
pais permaneça com o paciente (art.12). O Sistema Único Direito à Educação
de Saúde promoverá ainda programas de assistência mé-
dica, odontológica e campanhas de vacinação das crianças A educação figura na Constituição Federal de 1988
(art. 14). como direito fundamental do ser humano, buscando con-
ferir suporte ao desenvolvimento de crianças e adolescen-
Observa-se, desta forma, que o direito à vida, incuti- tes. Este direito está expresso nos art. 205 a 214 da Cons-
do no direito à saúde, é considerado o mais elementar e tituição Federal de 1988, na Lei 9.394/90 (Lei de Diretrizes
absoluto dos direitos fundamentais, pois é indispensável da Educação) e na Lei 8.069/90 (Estatuto da Criança e do
ao exercício de todos os outros direitos. Não pode ser con- Adolescente).
fundido com sobrevivência, pois o direito à vida implica o A Lei de Diretrizes da Educação Nacional, conhecida
reconhecimento do direito de viver com dignidade, direito como Lei Darcy Ribeiro, reafirma a obrigação solidária do
de viver bem, desde o momento da formação do ser hu- Poder Público, da família e da comunidade na busca de
mano. (AMIN, 2007). garantir a educação.
Neste sentido, Lenza (2007) afirma que o direito à vida “Art. 2º. A educação é direito de todos e dever da famí-
abrange tanto o direito de não ser morto, privado da vida, lia e do Estado, terá como bases os princípios de liberdade
portanto o direito de continuar vivo, como também o di- e os ideais de solidariedade humana e, como fim, a for-
reito de ter uma vida digna, garantindo-se as necessidades mação integral da pessoa do educando, a sua preparação
vitais básicas do ser humano, e proibindo qualquer trata- para o exercício da cidadania e a sua qualificação para o
mento indigno, como a tortura, penas de caráter perpétuo, trabalho.”
trabalhos forçados, cruéis, entre outros. Conforme descrito no artigo 54 do Estatuto da Criança
Amim (2007) ilustra a efetivação do direito à vida e à e do Adolescente, o Estado buscará a efetivação do Direito
saúde, apontando para a hipótese de adolescente que es- à educação, assegurando o ensino fundamental gratuito e
tando à beira da morte, deve ser assegurado a ele, minima- universal a todos (inciso I), com acesso a “programas suple-
mente, os recursos para tentar mantê-lo vivo, ou se for ine- mentares de material didático-escolar, transporte, alimen-
vitável a sua morte precoce, que ao menos haja tratamento tação e assistência à saúde” (inciso VII). Ainda, será ofereci-
digno. Ainda, na hipótese de uma criança ou adolescente do atendimento especializado aos portadores de deficiên-

82
CONHECIMENTOS ESPECÍFICOS - PSICÓLOGO

cias (inciso III), e educação infantil, em creche e pré-escola, construção de complexos ou simples ginásios poliespor-
às crianças de zero a seis anos de idade (inciso IV). A não tivos. A família deve buscar proporcionar o acesso a estes
oferta do ensino obrigatório importa em responsabilização direitos, e a escola tem papel importante na promoção des-
da autoridade competente (§ 2º). tes, quando realiza passeios ou forma grupos de teatro com
Fazendo alusão ao § 3º do artigo 54 do ECA, Machado os próprios alunos.
(2003) ressalta a prestação positiva imposta ao Estado em Aponta Machado (2003) que um direito que se despren-
assegurar o direito à educação, não bastando a oferta de deria do direito ao lazer, à convivência familiar e comunitária,
vagas, a Constituição exige do Estado o recenseamento de do direito ao não-trabalho, seria o direito de brincar. A ga-
crianças e adolescentes em idade escolar, que proceda a rantia deste direito auxiliaria no desenvolvimento cognitivo,
chamada deles e que zele, junto com os pais ou responsá- psicológico e social da criança e do adolescente.
veis, pela frequência à escola. Assegurar o direito de brincar encontra seu significado
Contudo, alerta Meneses (2008): quando inserido numa sociedade influenciada pela mídia
que passou a exigir um comportamento adulto daqueles
“[...] o aluno fora da sala de aula afronta a juridicidade. que ainda não o são. Assim, crianças e adolescentes assu-
Mas um aluno na sala de aula, sem espaço para o erro, mem uma agenda de horários similar a dos adultos, a outros
e por causa dele, desautorizado a reconstruir concepções, ainda é imposta a responsabilidade pelo cuidado de irmãos
afronta a proteção integral de pessoa em desenvolvimento. menores, correndo o risco de lhes faltar tempo para brincar,
Ainda o aluno na sala de aula, porque assim determina a conversar, se divertir. (AMIN, 2007).
lei, que não respeita a convivência com o educador e com
os outros alunos, liquida com a qualidade da relação [...]. ” Direito à Profissionalização e à Proteção no Trabalho
(MENESES, 2008).
Veronese e Oliveira (2008) esclarecem ser o direito de O direito ao trabalho “repousa basicamente na prote-
aprender, explícito no direito ao acesso à educação regular, ção do interesse individual de ter liberdade para exercer as
um dos direitos humanos fundamentais. Isto se deve a re- potencialidades que todo trabalho humano comporta e na
lação existente entre educação e cidadania. Cidadania en- proteção o interesse individual de prover as próprias neces-
tendida como “[...] um exercício contínuo de reivindicação sidades”. (MACHADO, 2003).
Observa, contudo, Machado (2003) que, quando a crian-
de direitos. Como reivindicar o que não se conhece? Daí
ça ou o adolescente exercitam o trabalho não mais como
decorre a necessidade de investimento em educação [...]”.
impulso de experimentação das suas potencialidades, mas,
Ainda, sendo crianças e adolescentes sujeitos de direitos
sim, como necessidade de prover seu próprio sustento, o
em processo de desenvolvimento, a educação se tornou
trabalho conflitua com outros interesses necessários ao seu
um direito indisponível, um requisito indispensável para
pleno desenvolvimento. O trabalho poderá retirar as forças
garantir o crescimento sadio, nos aspectos físico, cognitivo,
imprescindíveis para o acompanhamento das aulas regula-
afetivo e emocional.
res, limitando a capacidade de aprendizado e prejudicando
sua qualificação teórico-profissional. Ainda, o trabalho po-
Direito à Cultura, ao Esporte e ao Lazer derá representar um esforço superior ao seu estágio de cres-
cimento, comprometendo a saúde e o seu desenvolvimento
As crianças e adolescente necessitam de vários estí- cognitivo.
mulos na sua formação: emocionais, sociais, culturais, edu- Por estas razões, visando proteger crianças e adoles-
cativos, motores, entre outros. Assim, a cultura estimula centes e, ao mesmo tempo, assegurar-lhes o direito funda-
o pensamento de maneira diversa da educação formal. O mental à profissionalização, o ordenamento estabeleceu um
esporte desenvolve habilidades motoras, socializa o indi- regime especial de trabalho, com direitos e restrições.
víduo. O lazer envolve entretenimento, a diversão que são A Emenda Constitucional nº 20, de 15/12/98 alterou o
importantes para o desenvolvimento integral do indivíduo. inciso XXXIII do art. 7º restringindo o trabalho adolescente
(AMIN, 2007). a partir dos 16 anos, salvo na condição de aprendiz a par-
Cabe aos Municípios, com o apoio dos Estados e da tir dos 14 anos, conforme art. 403 da CLT e art. 60 da Lei
União, estimular e destinar recursos e espaços para pro- 8.069/90.
gramações culturais, esportivas e de lazer, voltadas para a
infância e a juventude, conforme art. 59 do ECA. Além da limitação etária, é proibido o trabalho notur-
Elias (2005) ressalta a importância da cultura, do espor- no, entre às 22 e 5 horas, o trabalho perigoso, insalubre ou
te e lazer no processo de formação dos indivíduos, sob o penoso, realizado em locais prejudiciais à formação e desen-
ponto de vista físico e mental. Desta forma, a municipaliza- volvimento físico, psíquico, moral e social do adolescente,
ção facilita o atendimento nestas áreas, contribuindo para bem como em horários que prejudiquem a sua frequência à
afastar crianças e adolescentes dos perigos das drogas e de escola (art. 67 do ECA e arts. 403, 404, 405 da CLT). Também
outros vícios que prejudicam o desenvolvimento de uma lhe são assegurados os direitos trabalhistas e previdenciá-
personalidade saudável, o que, no futuro, poderá levá-los a rios (art. 65 do ECA).
uma vida sem qualidade e à criminalidade. O direito ao trabalho protegido, exercido por adoles-
Para Amin (2007) estes direitos devem ser assegurados cente entre 14 a 18 anos, não pode ser confundido com o
pelo Estado através da construção de praças, instalação de direito à profissionalização, existindo na essência antago-
teatros populares, promoção de shows abertos ao público, nismos entre eles. De acordo com Machado (2003, p.188):

83
CONHECIMENTOS ESPECÍFICOS - PSICÓLOGO

“[...] o direito à profissionalização objetiva proteger o Reafirma o art. 18 do ECA, ser dever de todos zelar pela
interesse de crianças e adolescentes de se preparem ade- suprema dignidade de crianças e adolescentes, colocando-
quadamente para o exercício do trabalho adulto, do tra- -os a salvo de qualquer forma de tratamento desumano,
balho no momento próprio; não visa o próprio sustento aterrorizante, constrangedor, bem como qualquer espécie
durante a juventude, que é necessidade individual concreta de violência, seja a violência física, a psicológica ou a vio-
resultante das desigualdades sociais, que a Constituição lência moral.
visa reduzir.”
Direito à Convivência Familiar e Comunitária
Diante do mundo contemporâneo que exige qualifica-
ção elevada, da qual a educação é requisito necessário, a O art. 19 da Lei n. 8.069/90, assegura a toda criança
qualificação profissional dos adolescentes é garantidora de e adolescente o direito de ser criado e educado no seio
um mínimo de igualdade entre os cidadãos quando da in- da sua família e, excepcionalmente, em família substituta,
serção no mercado de trabalho. Entretanto, quando o ado-
assegurando a convivência familiar e comunitária, zelando
lescente passa a exercer o trabalho regular precocemente,
por um ambiente livre da presença de pessoas dependen-
mais se limitam suas chances de desenvolver adequada-
tes de substâncias entorpecentes.
mente sua profissionalização, para que possa, na idade
adulta, competir no mercado de trabalho, mantendo, desta Este direito tem por base a capacidade protetora da
forma, sua desigualdade na inserção social, pois a apren- criança e do adolescente na relação parental. Conforme
dizagem é limitada e precária, basicamente laboral e não Gueiros e Oliveira (2005, p.118), o direito à convivência fa-
educativa, que se norteia pelos princípios da produtividade miliar deve ser garantido tanto aos filhos, como também
do trabalho e lucro do empregador. (MACHADO, 2003). aos pais:

Direito à Liberdade, ao Respeito e à Dignidade “É fundamental defender o princípio de que o lugar


da criança é na família, mas é necessário pensar que essa
A criança e o adolescente têm direito à liberdade, ao é uma via de mão dupla – direito dos filhos, mas também
respeito e à dignidade, por serem pessoas em desenvolvi- de seus pais- e, assim, sendo, deve ser assegurado à crian-
mento e sujeitos de direitos civis, humanos e sociais. (art. ça o direito de convivência familiar, preferencialmente na
15 da Lei 8.069/90). família na qual nasceu, e aos pais o direito de poder criar e
O direito à liberdade é mais amplo do que o direito de educar os filhos que tiveram do casamento ou de vivências
ir e vir. O art. 16 do ECA compreende a liberdade também amorosas que não chegaram a se constituir como parcerias
como liberdade de opinião, expressão, crença e culto reli- conjugais.”
gioso, liberdade de brincar, praticar esportes e divertir-se,
Como fatores que dificultam a manutenção de crianças
participar da vida em família, na sociedade e vida política,
e adolescentes em suas famílias, são apontados as desi-
assim como buscar refúgio, auxílio e proteção.
gualdades sociais presentes na sociedade e a crescente ex-
Porém, conforme verificado no inciso I, do art. 16 são
impostas restrições legais ao direito à liberdade de crian- clusão social do mercado formal de trabalho que incidem
ças e adolescentes. Para Elias (2005), as limitações à liber- diretamente sobre a situação econômica das famílias, invia-
dade são impostas devido a própria condição de pessoas bilizando o provimento de condições mínimas necessárias
em desenvolvimento, para o seu bem-estar. Neste sentido, à sua sobrevivência, desta forma, vivem na negligência e
Machado (2003) justifica que as restrições à liberdade da abandono, tanto pais quanto filhos. No caso presente, faz-
pessoa física em fase de desenvolvimento têm suas espe- -se urgente que as famílias contem com políticas públicas
cificidades ligadas à questão da imaturidade de crianças e sociais que garantam o acesso a bens e serviços indispen-
adolescentes, o que auxilia que estas se protejam contra sáveis à cidadania. (GUEIROS; OLIVEIRA, 2008).
agressões aos seus direitos. É bem verdade que a pobreza dos genitores não
Por seu turno, o direito ao respeito é descrito no art. 17 constitui fator de perda ou suspensão do poder familiar,
do ECA como a “inviolabilidade da integridade física, psí- podendo somente serem decretadas judicialmente (art.
quica e moral da criança e do adolescente, abrangendo a 23 e 24 da Lei 8.069/90). O Poder Familiar é conceituado
preservação da imagem, da identidade, da autonomia, dos por Maciel (2007, p. 72) como um “complexo de direitos
valores, ideias e crenças, dos espaços e objetos pessoais. e deveres pessoais e patrimoniais com relação ao filho
“[...] Toda criança nasce com o direito de ser. É um erro menor, [...] que deve ser exercido no melhor interesse deste
muito grave, que ofende o direito de ser, conceber a crian- último [...]”.
ça como apenas um projeto de pessoa, como alguma coisa
A par disso, esclarece Ishida (2001), que nos procedi-
que no futuro poderá adquirir a dignidade de um ser hu-
mentos da infância e juventude, a preferência é sempre de
mano. É preciso reconhecer e não esquecer em momento
mantença da criança e do adolescente junto aos genitores
algum, que, pelo simples fato de existir, a criança já é uma
biológicos. Somente após acompanhamento técnico-jurí-
pessoa e por essa razão merecedora do respeito que é de-
dico que verifique a inexistência de condições dos geni-
vido exatamente na mesma medida a todas as pessoas.”
tores, havendo direitos fundamentais ameaçados ou viola-
(DALLARI; KORCZACK, 1986, p. 21).
dos, inicia-se a colocação em lar substituto.

84
CONHECIMENTOS ESPECÍFICOS - PSICÓLOGO

Conforme art. 100 da Lei n. 8.069/90, a manutenção e Interesse superior da criança e do adolescente: a in-
o fortalecimento dos vínculos devem ser observados tam- tervenção deve atender prioritariamente aos interesses e
bém na aplicação de medidas socioeducativas, preferindo direitos da criança e do adolescente, sem prejuízo da con-
aquelas medidas que favoreçam as relações afetivas que sideração que for devida a outros interesses legítimos no
o adolescente já tem construído em sua família e comu- âmbito da pluralidade dos interesses presentes no caso
nidade. concreto;
Privacidade: a promoção dos direitos e proteção da
Fonte: criança e do adolescente deve ser efetuada no respeito pela
http://www.ambitojuridico.com.br/site/index.php?n_ intimidade, direito à imagem e reserva da sua vida privada;
Intervenção precoce: a intervenção das autoridades
link=revista_artigos_leitura&artigo_id=9619
competentes deve ser efetuada logo que a situação de pe-
rigo seja conhecida;
Intervenção mínima: a intervenção deve ser exercida
AS MEDIDAS ESPECÍFICAS DE PROTEÇÃO exclusivamente pelas autoridades e instituições cuja ação
À CRIANÇA E AO ADOLESCENTE. seja indispensável à efetiva promoção dos direitos e à pro-
teção da criança e do adolescente;
Proporcionalidade e atualidade: a intervenção deve ser
a necessária e adequada à situação de perigo em que a
criança ou o adolescente se encontram no momento em
MEDIDAS DE PROTEÇÃO À CRIANÇA E que a decisão é tomada;
AO ADOLESCENTE Responsabilidade parental: a intervenção deve ser efe-
tuada de modo que os pais assumam os seus deveres para
As medidas de proteção à criança e ao adolescente são com a criança e o adolescente;
aplicáveis sempre que os direitos reconhecidos por Lei fo- Prevalência da família: na promoção de direitos e na
rem ameaçados ou violados: proteção da criança e do adolescente deve ser dada preva-
a) Por ação ou omissão da sociedade ou do Estado; lência às medidas que os mantenham ou reintegrem na sua
b) Por falta, omissão ou abuso dos pais ou responsável; família natural ou extensa ou, se isto não for possível, que
c) Em razão de sua conduta. promovam a sua integração em família substituta;
Obrigatoriedade da informação: a criança e o adoles-
Medidas Específicas de Proteção cente, respeitado seu estágio de desenvolvimento e capa-
cidade de compreensão, seus pais ou responsável devem
As medidas previstas poderão ser aplicadas isolada ou ser informados dos seus direitos, dos motivos que deter-
cumulativamente, bem como substituídas a qualquer tem- minaram a intervenção e da forma como esta se processa;
po. Oitiva obrigatória e participação: a criança e o adoles-
Na aplicação das medidas levar-se-ão em conta as ne- cente, em separado ou na companhia dos pais, de respon-
cessidades pedagógicas, preferindo-se aquelas que visem sável ou de pessoa por si indicada, bem como os seus pais
ao fortalecimento dos vínculos familiares e comunitários. ou responsável, têm direito a ser ouvidos e a participar nos
São também princípios que regem a aplicação das me- atos e na definição da medida de promoção dos direitos e
didas: de proteção, sendo sua opinião devidamente considerada
Condição da criança e do adolescente como sujeitos pela autoridade judiciária competente.
de direitos: crianças e adolescentes são os titulares dos di-
reitos previstos no Estatuto da Criança e Adolescente - Lei MEDIDAS
nº 8.069/1990 e em outras Leis, bem como na Constituição
Federal; Verificada qualquer das hipóteses previstas, a autori-
Proteção integral e prioritária: a interpretação e apli- dade competente poderá determinar, dentre outras, as se-
cação de toda e qualquer norma contida no Estatuto da guintes medidas:
Criança e Adolescente - Lei nº 8.069/1990 deve ser voltada
à proteção integral e prioritária dos direitos de que crianças Encaminhamento aos pais ou responsável, mediante
e adolescentes são titulares; termo de responsabilidade;
Responsabilidade primária e solidária do poder públi- Orientação, apoio e acompanhamento temporários;
co: a plena efetivação dos direitos assegurados a crianças e Matrícula e frequência obrigatórias em estabelecimento
a adolescentes pelo Estatuto da Criança e Adolescente - Lei oficial de ensino fundamental;
nº 8.069/1990 e pela Constituição Federal, salvo nos casos Inclusão em programa comunitário ou oficial de auxílio
por esta expressamente ressalvados, é de responsabilidade à família, à criança e ao adolescente;
primária e solidária das 3 (três) esferas de governo, sem
prejuízo da municipalização do atendimento e da possibi- Requisição de tratamento médico, psicológico ou psi-
lidade da execução de programas por entidades não go- quiátrico, em regime hospitalar ou ambulatorial;
vernamentais; Inclusão em programa oficial ou comunitário de auxílio,
orientação e tratamento a alcoólatras e toxicômanos;

85
CONHECIMENTOS ESPECÍFICOS - PSICÓLOGO

Acolhimento institucional; O acolhimento familiar ou institucional ocorrerá no lo-


Inclusão em programa de acolhimento familiar; cal mais próximo à residência dos pais ou do responsável
Colocação em família substituta. e, como parte do processo de reintegração familiar, sempre
Acolhimento institucional e acolhimento familiar que identificada a necessidade, a família de origem será
incluída em programas oficiais de orientação, de apoio e de
O acolhimento institucional e o acolhimento familiar promoção social, sendo facilitado e estimulado o contato
são medidas provisórias e excepcionais, utilizáveis como com a criança ou com o adolescente acolhido.
forma de transição para reintegração familiar ou, não sen- Verificada a possibilidade de reintegração familiar, o
do esta possível, para colocação em família substituta, não responsável pelo programa de acolhimento familiar ou ins-
implicando privação de liberdade. titucional fará imediata comunicação à autoridade judiciá-
Sem prejuízo da tomada de medidas emergenciais ria, que dará vista ao Ministério Público, pelo prazo de 5
para proteção de vítimas de violência ou abuso sexual; o (cinco) dias, decidindo em igual prazo.
afastamento da criança ou adolescente do convívio familiar
é de competência exclusiva da autoridade judiciária e im- Reintegração à família
portará na deflagração, a pedido do Ministério Público ou
de quem tenha legítimo interesse, de procedimento judicial Sendo constatada a impossibilidade de reintegração
contencioso, no qual se garanta aos pais ou ao responsável da criança ou do adolescente à família de origem, após seu
legal o exercício do contraditório e da ampla defesa. encaminhamento a programas oficiais ou comunitários de
Crianças e adolescentes somente poderão ser encami- orientação, apoio e promoção social, será enviado relató-
nhados às instituições que executam programas de acolhi- rio fundamentado ao Ministério Público, no qual conste a
mento institucional, governamentais ou não, por meio de descrição pormenorizada das providências tomadas e a ex-
uma Guia de Acolhimento, expedida pela autoridade ju- pressa recomendação, subscrita pelos técnicos da entida-
diciária, na qual obrigatoriamente constará, dentre outros: de ou responsáveis pela execução da política municipal de
garantia do direito à convivência familiar, para a destituição
do poder familiar, ou destituição de tutela ou guarda.
Sua identificação e a qualificação completa de seus pais
ou de seu responsável, se conhecidos;
Recebido o relatório, o Ministério Público terá o prazo
O endereço de residência dos pais ou do responsável,
de 30 (trinta) dias para o ingresso com a ação de destitui-
com pontos de referência;
ção do poder familiar, salvo se entender necessária a reali-
Os nomes de parentes ou de terceiros interessados em
zação de estudos complementares ou outras providências
tê-los sob sua guarda;
que entender indispensáveis ao ajuizamento da demanda.
Os motivos da retirada ou da não reintegração ao con-
A autoridade judiciária manterá, em cada comarca ou
vívio familiar.
foro regional, um cadastro contendo informações atuali-
zadas sobre as crianças e adolescentes em regime de aco-
Imediatamente após o acolhimento da criança ou do lhimento familiar e institucional sob sua responsabilidade,
adolescente, a entidade responsável pelo programa de com informações pormenorizadas sobre a situação jurídica
acolhimento institucional ou familiar elaborará um plano de cada um, bem como as providências tomadas para sua
individual de atendimento, visando à reintegração familiar, reintegração familiar ou colocação em família substituta.
ressalvada a existência de ordem escrita e fundamentada Terão acesso ao cadastro o Ministério Público, o Con-
em contrário de autoridade judiciária competente, caso em selho Tutelar, o órgão gestor da Assistência Social e os Con-
que também deverá contemplar sua colocação em família selhos Municipais dos Direitos da Criança e do Adolescente
substituta, observadas as regras e princípios estipuladas e da Assistência Social, aos quais incumbe deliberar sobre a
pelo Estatuto da Criança e Adolescente - Lei nº 8.069/1990. implementação de políticas públicas que permitam reduzir
O plano individual será elaborado sob a responsabili- o número de crianças e adolescentes afastados do convívio
dade da equipe técnica do respectivo programa de atendi- familiar e abreviar o período de permanência em programa
mento e levará em consideração a opinião da criança ou do de acolhimento.
adolescente e a oitiva dos pais ou do responsável.
Constarão do plano individual, dentre outros: Registros e certidões
Os resultados da avaliação interdisciplinar;
Os compromissos assumidos pelos pais ou responsá- Verificada a inexistência de registro anterior, o assento
vel; e de nascimento da criança ou adolescente será feito à vista
A previsão das atividades a serem desenvolvidas com dos elementos disponíveis, mediante requisição da autori-
a criança ou com o adolescente acolhido e seus pais ou dade judiciária.
responsável, com vista na reintegração familiar ou, caso Os registros e certidões necessários à regularização
seja esta vedada por expressa e fundamentada determi- são isentos de multas, custas e emolumentos, gozando de
nação judicial, as providências a serem tomadas para sua absoluta prioridade.
colocação em família substituta, sob direta supervisão da Caso ainda não definida a paternidade, será deflagrado
autoridade judiciária. procedimento específico destinado à sua averiguação.

86
CONHECIMENTOS ESPECÍFICOS - PSICÓLOGO

É dispensável o ajuizamento de ação de investigação § 4º Os grupos de irmãos serão colocados sob adoção,
de paternidade pelo Ministério Público se, após o não tutela ou guarda da mesma família substituta, ressalvada
comparecimento ou a recusa do suposto pai em assumir a comprovada existência de risco de abuso ou outra
a paternidade a ele atribuída, a criança for encaminhada situação que justifique plenamente a excepcionalidade de
para adoção. solução diversa, procurando-se, em qualquer caso, evitar o
rompimento definitivo dos vínculos fraternais.
ATUAÇÃO DO ASSISTENTE SOCIAL E PSICÓLOGO § 5º A colocação da criança ou adolescente em família
DE ACORDO COM AS MEDIDAS PROTETIVAS substituta será precedida de sua preparação gradativa
e acompanhamento posterior, realizados pela equipe
Conforme assinalado anteriormente, nem sempre o interprofissional a serviço da Justiça da Infância e da
processo tem início com a definição da medida de prote- Juventude, preferencialmente com o apoio dos técnicos
ção que deve ser aplicada. A avaliação do caso poderá indi- responsáveis pela execução da política municipal de
car a necessidade ou não de medida de proteção. Contudo, garantia do direito à convivência familiar.
outros processos podem ter início com uma solicitação es- § 6º Em se tratando de criança ou adolescente indígena
pecifica da medida que a parte deseja obter. ou proveniente de comunidade remanescente de quilombo,
As medidas de proteção estão estabelecidas no ECA no é ainda obrigatório:
artº 101 e podem ser aplicadas isolada ou cumulativamen- I - que sejam consideradas e respeitadas sua identi-
te, bem como substituídas a qualquer tempo (art 99) e na dade social e cultural, os seus costumes e tradições, bem
sua aplicação levarão em conta as necessidades pedagó- como suas instituições, desde que não sejam incompatíveis
gicas, preferindo-se aquelas que visem ao fortalecimento com os direitos fundamentais reconhecidos por esta Lei e
dos vínculos familiares e comunitários (art.100). pela Constituição Federal;
Ademais é de exclusividade do Judiciário a inserção II - que a colocação familiar ocorra prioritariamente no
em família substituta por meio das medidas legais de guar- seio de sua comunidade ou junto a membros da mesma
da, tutela ou adoção. etnia;
Em algumas comarcas existem programas alternativos III - a intervenção e oitiva de representantes do órgão
ao abrigamento como: Família Acolhedora ou Família Guar- federal responsável pela política indigenista, no caso de
diã. Esses programas têm vinculação com o artº 34 do ECA, crianças e adolescentes indígenas, e de antropólogos, pe-
que evidencia: rante a equipe interprofissional ou multidisciplinar que irá
O Poder Público estimulará, através de assistência ju- acompanhar o caso.
rídica, incentivos fiscais e subsídios, o acolhimento, sob a
forma de guarda de criança ou adolescente órfão ou aban- Art. 29. Não se deferirá colocação em família substituta
donado (art. 34 ECA). a pessoa que revele, por qualquer modo, incompatibilidade
De toda forma, quer o processo tenha início com a so- com a natureza da medida ou não ofereça ambiente fami-
licitação expressa da medida de proteção, quer no desen- liar adequado.
rolar da avaliação se defina a medida, caberá aos assisten-
tes sociais e psicólogos identificar as peculiaridades que Art. 30. A colocação em família substituta não admi-
devem ser observadas no estudo social e psicológico, que tirá transferência da criança ou adolescente a terceiros ou
serão tratados a seguir. a entidades governamentais ou não-governamentais, sem
autorização judicial.
Seção III
Da Família Substituta Art. 31. A colocação em família substituta estrangeira
Subseção I constitui medida excepcional, somente admissível na mo-
Disposições Gerais dalidade de adoção.

Art. 28. A colocação em família substituta far-se-á me- Art. 32. Ao assumir a guarda ou a tutela, o responsável
diante guarda, tutela ou adoção, independentemente da prestará compromisso de bem e fielmente desempenhar o
situação jurídica da criança ou adolescente, nos termos encargo, mediante termo nos autos.
desta Lei.
§ 1º Sempre que possível, a criança ou o adolescente A - GUARDA (art. 33 a 35 – ECA)
será previamente ouvido por equipe interprofissional,
respeitado seu estágio de desenvolvimento e grau de Na impossibilidade de manutenção da criança e ado-
compreensão sobre as implicações da medida, e terá sua lescente com seus pais, deve-se verificar na família extensa
opinião devidamente considerada. (avós, tios, padrinhos, irmãos, primos etc) ou na comunida-
§ 2º Tratando-se de maior de 12 (doze) anos de idade, de a existência de pessoas que mantenham com a criança
será necessário seu consentimento, colhido em audiência. ou adolescente vínculo e que se disponham a assumir a
§ 3º Na apreciação do pedido levar-se-á em conta responsabilidade de cuidá-los.
o grau de parentesco e a relação de afinidade ou de Especial atenção deve ser dada quando se tratar de
afetividade, a fim de evitar ou minorar as consequências grupos de irmãos que sejam colocados sob guarda de
decorrentes da medida. diferentes interessados (parentes ou indivíduos que man-

87
CONHECIMENTOS ESPECÍFICOS - PSICÓLOGO

tenham vinculação afetiva). Todos os esforços devem ser O psicólogo deve deixar claro no relatório para subsi-
realizados no sentido de preservar os vínculos entre a ir- diar as possíveis alterações a curto, médio e longo prazo
mandade, garantindo-lhes estreito convívio. na vida do infante. É necessário incluir orientações e acom-
Diante da existência de interessado em assumir os panhamento do caso, de forma a tornar tal situação menos
cuidados desta criança ou adolescente, deverá haver uma traumática.
avaliação. Sugere-se que o estudo social seja acrescido dos Havendo interessados e, sendo verificada a adequação
seguintes aspectos: desses e o desejo da criança ou adolescente em estar sob
- Grau de parentesco, convivência com a família nuclear a sua responsabilidade, deverá o profissional elaborar um
da criança/adolescente; relatório conclusivo, em que conste de forma clara se a me-
- Conhecimento ou participação na história de vida da dida pleiteada pelo interessado(a) atende as necessidades
criança/adolescente; da criança ou adolescente.
- Facilidades e dificuldades nas interações pessoais e no Poderá ocorrer o acompanhamento, desde que suge-
processo socioeducativo da criança ou adolescente; rido em relatório pelo profissional de serviço social e/ou
psicologia ao juiz, e isso só poderá se viabilizar se houver
- Motivos que levaram o (a) requerente a pleitear a
autorização/ determinação do juiz.
guarda;
Ressalta-se a importância da reavaliação do caso, após
- Configuração familiar dos requerentes, papéis e dinâ-
alguns meses de convivência da criança ou adolescente
mica familiar do interessado (a);
com o guardião, pois é nesta ocasião que poderá verifi-
- Natureza das relações familiares, outros grupos de car se a medida está de fato atendendo aos interesses da
convivência familiar e comunitária; criança ou adolescente, bem como irá se proceder às orien-
- Situação socioeconômica e cultural do interessado (a); tações necessárias.
- Recursos da rede de serviços disponíveis ou utilizados Paralelo ao acompanhamento da criança é também ne-
pela criança/adolescente e grupo familiar; cessário que a família de origem seja acompanhada com
- Expectativa do interessado em relação à criança ou a finalidade de se verificar, orientar e encaminhar em re-
adolescente; lação às condições em que a mesma se encontra dentro
- Compreensão dos fatos pela criança ou adolescente, dos diferentes aspectos de sua vida, principalmente aque-
afinidade em relação aos requerentes, expectativas. les motivadores do afastamento da criança/adolescente de
sua convivência. Isso significa ver de que maneira a família
O resultado do trabalho deverá ser documentado sob responde aos encaminhamentos a rede de serviços assis-
forma de relatório que contenha uma análise fundamen- tenciais, trabalho, saúde (como por exemplo: tratamento
tada da situação verificada, bem como a sugestão de en- psicológico; psiquiátrico; tratamento de desintoxicação)
caminhamentos que solucionem ou minimizem a situação entre outros.
de vulnerabilidade/risco vivenciada pelas partes – crianças, Também no caso de não haver proibição judicial e, se
adolescentes e familiares. Este estudo deverá ser oferecido for possível, como ocorrem os contatos da família de ori-
à autoridade judiciária dentro do prazo por ele estipulado. gem com a criança/adolescente e os guardiões. Isso po-
Caso a medida pleiteada pelo requerente atenda aos derá oferecer indícios de quais as reais possibilidades de
interesses e necessidades da criança e adolescente, o juiz reinserção da criança e adolescente em sua família natural.
poderá deferir a guarda. O guardião de fato poderá requerer a aplicação da me-
Na avaliação psicológica da solicitação da guarda, as- dida de guarda quando a criança ou adolescente por algu-
sim como em outras medidas, considera-se importante que ma razão está sob sua responsabilidade.
seja compreendido de forma abrangente e na perspectiva A busca do judiciário visa regulamentar a situação na
de se aprofundar as características de personalidade dos perspectiva de assegurar direitos, uma vez que confere a
criança/adolescente a condição de dependente, para todos
requerentes, a motivação inerente ao pedido.
os fins e efeitos de direito, inclusive previdenciários.
Concomitantemente deve ser realizada uma investi-
A guarda é uma medida intermediária a ações definiti-
gação psicológica consistente do funcionamento intrapsí-
vas como no caso da tutela e da adoção. Excepcionalmente,
quico dos genitores, os papéis que eles ocupam na família
essa medida poderá ser aplicada para atender a situações
de origem, aspectos do funcionamento do sistema familiar peculiares ou suprir eventual falta dos pais ou responsável.
como um todo e, principalmente, a percepção do impac- Sempre que possível, a criança ou adolescente deverá
to daquele afastamento das figuras parentais na vida da ser previamente ouvido e a sua opinião devidamente con-
criança/adolescente. siderada (Art 28 §1º - ECA).
Os procedimentos técnicos aplicados de maneira indi- Na apreciação do pedido levar-se-á em conta o grau
vidual com a criança/adolescente, mesmo se tratando de de parentesco e a relação de afinidade ou de afetividade, a
grupos de irmãos, nos mostram que cada um deles cons- fim de evitar ou minorar as consequências decorrentes da
trói seus vínculos afetivos de maneira peculiar e que, por- medida.
tanto, após a elaboração do psicodiagnóstico, é necessário
um trabalho de preparação concomitante ao de devolução
dos dados para ajudá-la(s) a enfrentar as novas situações
que se apresentam no cotidiano.

88
CONHECIMENTOS ESPECÍFICOS - PSICÓLOGO

LEI Nº 8.069, DE 13 DE JULHO DE 1990. VII - intervenção mínima: a intervenção deve ser exer-
cida exclusivamente pelas autoridades e instituições cuja
Título II ação seja indispensável à efetiva promoção dos direitos e à
Das Medidas de Proteção proteção da criança e do adolescente;
Capítulo I VIII - proporcionalidade e atualidade: a intervenção
Disposições Gerais deve ser a necessária e adequada à situação de perigo em
que a criança ou o adolescente se encontram no momento
Art. 98. As medidas de proteção à criança e ao adoles- em que a decisão é tomada;
cente são aplicáveis sempre que os direitos reconhecidos IX - responsabilidade parental: a intervenção deve ser
nesta Lei forem ameaçados ou violados: efetuada de modo que os pais assumam os seus deveres
I - por ação ou omissão da sociedade ou do Estado; para com a criança e o adolescente;
II - por falta, omissão ou abuso dos pais ou responsá- X - prevalência da família: na promoção de direitos e na
vel; proteção da criança e do adolescente deve ser dada preva-
III - em razão de sua conduta. lência às medidas que os mantenham ou reintegrem na sua
família natural ou extensa ou, se isto não for possível, que
promovam a sua integração em família substituta;
Capítulo II
XI - obrigatoriedade da informação: a criança e o ado-
Das Medidas Específicas de Proteção
lescente, respeitado seu estágio de desenvolvimento e ca-
pacidade de compreensão, seus pais ou responsável devem
Art. 99. As medidas previstas neste Capítulo poderão
ser informados dos seus direitos, dos motivos que determi-
ser aplicadas isolada ou cumulativamente, bem como subs- naram a intervenção e da forma como esta se processa;
tituídas a qualquer tempo. XII - oitiva obrigatória e participação: a criança e o ado-
lescente, em separado ou na companhia dos pais, de res-
Art. 100. Na aplicação das medidas levar-se-ão em ponsável ou de pessoa por si indicada, bem como os seus
conta as necessidades pedagógicas, preferindo-se aquelas pais ou responsável, têm direito a ser ouvidos e a participar
que visem ao fortalecimento dos vínculos familiares e co- nos atos e na definição da medida de promoção dos direi-
munitários. tos e de proteção, sendo sua opinião devidamente consi-
Parágrafo único. São também princípios que regem a derada pela autoridade judiciária competente, observado o
aplicação das medidas: disposto nos §§ 1o e 2o do art. 28 desta Lei.
I - condição da criança e do adolescente como sujeitos
de direitos: crianças e adolescentes são os titulares dos di- Art. 101. Verificada qualquer das hipóteses previstas no
reitos previstos nesta e em outras Leis, bem como na Cons- art. 98, a autoridade competente poderá determinar, den-
tituição Federal; tre outras, as seguintes medidas:
II - proteção integral e prioritária: a interpretação e I - encaminhamento aos pais ou responsável, mediante
aplicação de toda e qualquer norma contida nesta Lei deve termo de responsabilidade;
ser voltada à proteção integral e prioritária dos direitos de II - orientação, apoio e acompanhamento temporários;
que crianças e adolescentes são titulares; III - matrícula e frequência obrigatórias em estabeleci-
III - responsabilidade primária e solidária do poder pú- mento oficial de ensino fundamental;
blico: a plena efetivação dos direitos assegurados a crian- IV - inclusão em serviços e programas oficiais ou co-
ças e a adolescentes por esta Lei e pela Constituição Fede- munitários de proteção, apoio e promoção da família, da
ral, salvo nos casos por esta expressamente ressalvados, é criança e do adolescente; (Redação dada pela Lei nº 13.257,
de responsabilidade primária e solidária das 3 (três) esferas de 2016)
de governo, sem prejuízo da municipalização do atendi- V - requisição de tratamento médico, psicológico ou
mento e da possibilidade da execução de programas por psiquiátrico, em regime hospitalar ou ambulatorial;
VI - inclusão em programa oficial ou comunitário de
entidades não governamentais;
auxílio, orientação e tratamento a alcoólatras e toxicôma-
IV - interesse superior da criança e do adolescente: a
nos;
intervenção deve atender prioritariamente aos interesses e
VII - acolhimento institucional;
direitos da criança e do adolescente, sem prejuízo da con-
VIII - inclusão em programa de acolhimento familiar;
sideração que for devida a outros interesses legítimos no IX - colocação em família substituta.
âmbito da pluralidade dos interesses presentes no caso § 1º O acolhimento institucional e o acolhimento
concreto; familiar são medidas provisórias e excepcionais, utilizáveis
V - privacidade: a promoção dos direitos e proteção como forma de transição para reintegração familiar ou, não
da criança e do adolescente deve ser efetuada no respei- sendo esta possível, para colocação em família substituta,
to pela intimidade, direito à imagem e reserva da sua vida não implicando privação de liberdade.
privada; § 2º Sem prejuízo da tomada de medidas emergenciais
VI - intervenção precoce: a intervenção das autorida- para proteção de vítimas de violência ou abuso sexual
des competentes deve ser efetuada logo que a situação de e das providências a que alude o art. 130 desta Lei, o
perigo seja conhecida; afastamento da criança ou adolescente do convívio familiar

89
CONHECIMENTOS ESPECÍFICOS - PSICÓLOGO

é de competência exclusiva da autoridade judiciária e § 9º Em sendo constatada a impossibilidade de


importará na deflagração, a pedido do Ministério Público reintegração da criança ou do adolescente à família de
ou de quem tenha legítimo interesse, de procedimento origem, após seu encaminhamento a programas oficiais ou
judicial contencioso, no qual se garanta aos pais ou ao comunitários de orientação, apoio e promoção social, será
responsável legal o exercício do contraditório e da ampla enviado relatório fundamentado ao Ministério Público, no
defesa. qual conste a descrição pormenorizada das providências
§ 3º Crianças e adolescentes somente poderão ser tomadas e a expressa recomendação, subscrita pelos
encaminhados às instituições que executam programas técnicos da entidade ou responsáveis pela execução da
de acolhimento institucional, governamentais ou não, política municipal de garantia do direito à convivência
familiar, para a destituição do poder familiar, ou destituição
por meio de uma Guia de Acolhimento, expedida pela
de tutela ou guarda.
autoridade judiciária, na qual obrigatoriamente constará,
§ 10. Recebido o relatório, o Ministério Público terá
dentre outros: o prazo de 30 (trinta) dias para o ingresso com a ação de
I - sua identificação e a qualificação completa de seus destituição do poder familiar, salvo se entender necessária
pais ou de seu responsável, se conhecidos; a realização de estudos complementares ou outras
II - o endereço de residência dos pais ou do responsá- providências que entender indispensáveis ao ajuizamento
vel, com pontos de referência; da demanda.
III - os nomes de parentes ou de terceiros interessados § 11. A autoridade judiciária manterá, em cada
em tê-los sob sua guarda; comarca ou foro regional, um cadastro contendo
IV - os motivos da retirada ou da não reintegração ao informações atualizadas sobre as crianças e adolescentes
convívio familiar. em regime de acolhimento familiar e institucional sob sua
responsabilidade, com informações pormenorizadas sobre
§ 4º Imediatamente após o acolhimento da criança ou
a situação jurídica de cada um, bem como as providências
do adolescente, a entidade responsável pelo programa de tomadas para sua reintegração familiar ou colocação em
acolhimento institucional ou familiar elaborará um plano família substituta, em qualquer das modalidades previstas
individual de atendimento, visando à reintegração familiar, no art. 28 desta Lei.
ressalvada a existência de ordem escrita e fundamentada § 12. Terão acesso ao cadastro o Ministério Público,
em contrário de autoridade judiciária competente, caso em o Conselho Tutelar, o órgão gestor da Assistência Social
que também deverá contemplar sua colocação em família e os Conselhos Municipais dos Direitos da Criança e do
substituta, observadas as regras e princípios desta Lei. Adolescente e da Assistência Social, aos quais incumbe
§ 5º O plano individual será elaborado sob a deliberar sobre a implementação de políticas públicas que
responsabilidade da equipe técnica do respectivo permitam reduzir o número de crianças e adolescentes
programa de atendimento e levará em consideração a afastados do convívio familiar e abreviar o período de
permanência em programa de acolhimento.
opinião da criança ou do adolescente e a oitiva dos pais ou
do responsável. Art. 102. As medidas de proteção de que trata este Ca-
§ 6º Constarão do plano individual, dentre outros: pítulo serão acompanhadas da regularização do registro
I - os resultados da avaliação interdisciplinar; civil.
II - os compromissos assumidos pelos pais ou respon- § 1º Verificada a inexistência de registro anterior, o
sável; e assento de nascimento da criança ou adolescente será feito
III - a previsão das atividades a serem desenvolvidas à vista dos elementos disponíveis, mediante requisição da
com a criança ou com o adolescente acolhido e seus pais autoridade judiciária.
ou responsável, com vista na reintegração familiar ou, caso § 2º Os registros e certidões necessários à regularização
seja esta vedada por expressa e fundamentada determi- de que trata este artigo são isentos de multas, custas e
emolumentos, gozando de absoluta prioridade.
nação judicial, as providências a serem tomadas para sua
§ 3º Caso ainda não definida a paternidade, será
colocação em família substituta, sob direta supervisão da deflagrado procedimento específico destinado à sua
autoridade judiciária. averiguação, conforme previsto pela Lei no 8.560, de 29 de
§ 7º O acolhimento familiar ou institucional ocorrerá no dezembro de 1992.
local mais próximo à residência dos pais ou do responsável § 4º Nas hipóteses previstas no § 3o deste artigo,
e, como parte do processo de reintegração familiar, sempre é dispensável o ajuizamento de ação de investigação
que identificada a necessidade, a família de origem será de paternidade pelo Ministério Público se, após o não
incluída em programas oficiais de orientação, de apoio e de comparecimento ou a recusa do suposto pai em assumir
promoção social, sendo facilitado e estimulado o contato a paternidade a ele atribuída, a criança for encaminhada
com a criança ou com o adolescente acolhido. para adoção.
§ 8º Verificada a possibilidade de reintegração familiar, § 5º Os registros e certidões necessários à inclusão, a
qualquer tempo, do nome do pai no assento de nascimento
o responsável pelo programa de acolhimento familiar
são isentos de multas, custas e emolumentos, gozando de
ou institucional fará imediata comunicação à autoridade absoluta prioridade. (Incluído dada pela Lei nº 13.257, de
judiciária, que dará vista ao Ministério Público, pelo prazo 2016)
de 5 (cinco) dias, decidindo em igual prazo.

90
CONHECIMENTOS ESPECÍFICOS - PSICÓLOGO

§ 6º São gratuitas, a qualquer tempo, a averbação mental de Especialistas para Revisão do Esboço das Diretri-
requerida do reconhecimento de paternidade no assento zes Internacionais sobre Proteção e Cuidados Alternativos
de nascimento e a certidão correspondente. (Incluído dada de Crianças Privadas de Cuidados Parentais, apoiada pelo
pela Lei nº 13.257, de 2016) MDS, Fundo das Nações Unidas para a Infância (UNICEF) e
Comitê dos Direitos da Criança da ONU.
Fonte Além disso, o governo brasileiro, em consonância com
http://www.normaslegais.com.br/guia/clientes/medi- esse movimento de reordenamento e de construção de
das-protecao-crianca-adolescente.htm normativas, elaborou e aprovou em dezembro de 2006 um
plano que traça diretrizes nacionais para as medidas de
apoio sociofamiliar, de acolhimento familiar e institucional
e de adoção: o Plano Nacional de Promoção, Proteção e
A CRIANÇA E O ADOLESCENTE NO Defesa do Direito de Crianças e Adolescentes à Convivência
ACOLHIMENTO INSTITUCIONAL. Familiar e Comunitária (MDS & SEDH, 2006). Ao longo des-
te artigo, referimos a esse plano como Plano Nacional. Ele
busca incorporar na sua plenitude a “doutrina da proteção
integral”, tendo mobilizado no processo de sua construção
discussões sobre o acolhimento familiar entre os órgãos de
O ACOLHIMENTO FAMILIAR NO CENÁRIO NACIO-
classe, entre grupos de pesquisa e de intervenção ligados
NAL
à infância e juventude, entre magistrados, promotores da
infância e juventude, além de outros atores que compõem
O Levantamento Nacional de Abrigos para Crianças e o Sistema de Garantia de Direitos.
Adolescentes (Instituto de Pesquisa Econômica Aplicada
[IPEA], 2003; Silva, 2004) realizado pelo IPEA junto às ins- Fica evidente o empenho para o reordenamento da
tituições cadastradas na Rede de Serviços de Ação Con- política nacional de assistência com o foco na família, va-
tinuada (SAC) do Ministério do Desenvolvimento Social, lorizada como o espaço ideal de convivência para a criança
estimou que em torno de 80.000 crianças e adolescentes e o adolescente.
estão vivendo em instituições de abrigo no Brasil. Entretan-
to, a maioria das crianças e adolescentes dos abrigos inves- Este Plano constitui um marco nas políticas públicas
tigados tem famílias (86,7%), sendo que foram afastados no Brasil, ao romper com a cultura da institucionalização
de suas famílias por situações de negligência, abandono de crianças e adolescentes e ao fortalecer o paradigma da
ou violência e possuem um perfil considerado distante do proteção integral e da preservação dos vínculos familiares
desejado para adoção (58,5% meninos, 63% afrodescen- e comunitários preconizados pelo Estatuto da Criança e do
dentes e 61,3% com idade entre 7 e 15 anos). Muitos pas- Adolescente. A manutenção dos vínculos familiares e co-
sam longos períodos nos abrigos, existindo aqueles que munitários - fundamentais para a estruturação das crianças
passam toda a infância e adolescência. São provenientes e adolescentes como sujeitos e cidadãos - está diretamente
de famílias pauperizadas, que enfrentam várias dificulda- relacionada ao investimento nas políticas públicas de aten-
des quanto à sua manutenção e subsistência (Silva, 2004). ção à família (MDS & SEDH, 2006, p. 14).
Com o intuito de enfrentamento e transformação dessa Dessa forma, o acolhimento familiar, que já aparecia
realidade, o Ministério do Desenvolvimento Social (MDS), a na Política Nacional de Assistência Social (MDS, 2004), re-
Secretaria Especial dos Direitos Humanos (SEDH), o Conse- lacionado como um programa a ser implantado dentro dos
lho Nacional dos Direitos da Criança e do Adolescente (CO- serviços de Proteção Social Especial de Alta Complexidade,
NANDA), o Conselho Nacional de Assistência Social (CNAS), passa a ser reconhecido como parte integrante das políti-
entre outros, vêm protagonizando discussões para implan- cas sociais do país. Fica reconhecido como estratégia que
objetiva o acolhimento temporário de crianças e adoles-
tar diretrizes e ações de fortalecimento do paradigma da
centes, visando sua reintegração familiar e buscando evi-
proteção integral da criança e do adolescente, garantindo a
tar a institucionalização. Ou seja, o acolhimento familiar é
sua condição de sujeito de direitos e de pessoa em desen-
compreendido como uma medida protetiva, a qual possi-
volvimento. Essas ações buscam promover e garantir seus bilita à criança e ao adolescente em vulnerabilidade e afas-
vínculos familiares e comunitários, adequando-se ao que tado de sua família de origem ser colocado sob a guarda
é preconizado pelo Estatuto da Criança e do Adolescente de uma outra família. Essa família é previamente seleciona-
(ECA, 1990) e pela Convenção sobre os Direitos da Criança da, cadastrada e vinculada a um programa. Ela acolherá a
(Organização das Nações Unidas [ONU], 1989). criança ou adolescente por um período. Paralelamente ao
Tal processo acontece simultaneamente a discussões acolhimento, é necessário trabalhar as causas do afasta-
internacionais lideradas pelo Comitê dos Direitos da Crian- mento junto à família de origem de maneira a contribuir,
ça da ONU, sobre a necessidade de serem aprimorados os efetivamente, para uma reintegração familiar como preco-
mecanismos de proteção integral dos direitos da criança nizado pelo ECA (1990). “As estratégias, objetivos e dire-
privada dos cuidados parentais (em especial o direito à trizes deste Plano estão fundamentados primordialmente
convivência familiar e comunitária). Pretende-se com isso na prevenção ao rompimento dos vínculos familiares, na
a elaboração de nova normativa internacional sobre o as- qualificação do atendimento dos serviços de acolhimento
sunto. O Brasil empenha-se no fortalecimento dessas dis- e no investimento para o retorno ao convívio com a família
cussões, tendo sediado em 2006 a Reunião Intergoverna- de origem” (MDS & SEDH, 2006, p. 14).

91
CONHECIMENTOS ESPECÍFICOS - PSICÓLOGO

O acolhimento familiar como prática social não é algo condição. Nesse sentido, o acolhimento familiar é um pro-
novo no Brasil, tendo acontecido ao longo da história de cesso de criação e não uma redefinição jurídica do status
modo informal através das práticas de circulação de crian- familiar da criança. As famílias biológicas podem colaborar
ças, por famílias que acolheram os chamados “filhos de cria- com as famílias acolhedoras e ambas são acompanhadas e
ção ou afilhados” (Fonseca, 2004; Franco, 2000; Sarti, 1996). supervisionadas por profissionais da área psicossocial (Ca-
Contudo, somente agora ele é proposto como um programa bral, 2004; George & Van Oudenhoven, 2003).
oficial, que compõe uma política pública de atendimento à Outros países europeus, como França, Itália e Espanha,
criança e ao adolescente privado dos cuidados parentais, implementaram o acolhimento familiar a partir de meados
com suporte legal e acompanhamento técnico. Nesse sen- da década de 70, em resposta à necessidade de reorde-
tido, constitui uma experiência nova na história do país. Daí namento das políticas de atenção à infância e juventude
a necessidade de pesquisas que possam acompanhar tanto em situação de vulnerabilidade. Na Espanha, conforme Fer-
as experiências de programas já existentes a partir de inicia- nández, Álvarez e Bravo (2003) e Palacios e Amorós (2006)
tivas locais, como a implantação da medida a nível nacional relatam, o acolhimento familiar e a adoção mudaram como
através da criação dos programas municipais. Por exemplo, reflexo das modificações legislativas de 1987 e 1996, que
Cabral (2004), Fávero (1995) e Uriarte Bálsamo (2005) rela- passaram a priorizar “o melhor interesse da criança”. A nova
tam iniciativas históricas de acolhimento familiar no Brasil. legislação estabeleceu que esses interesses devem ser pro-
Essas investigações devem abordar diferentes aspectos do vidos por uma família e não por uma instituição, e que as
acolhimento familiar, focando os seus distintos protagonis- decisões sobre medidas de proteção à criança precisariam
tas (família acolhedora, equipe dos programas, criança e ser tomadas por uma equipe de profissionais habilitados
adolescente acolhidos, família de origem), como também as (psicólogos e assistentes sociais). Assim, as mudanças le-
significações que circulam em seu contexto quanto às ques- gislativas e sociais formaram um panorama que favore-
tões relacionadas a manutenção e rompimento de vínculos ceu novas práticas de acolhimento e, consequentemente,
afetivos, reintegração familiar, acolhimento em famílias, den- novas concepções de família acolhedora. Isso porque não
tre outras questões envolvidas nessa prática de acolhimento. existia na Espanha uma cultura de acolhimento, sendo o
acolhimento familiar, então, um recurso pouco conhecido
O acolhimento familiar como alternativa de prote- pela sociedade e pelos próprios administradores públicos.
ção Ele era restrito à modalidade de acolhimento pré-adotivo
(onde não é realizado um trabalho com a família de ori-
gem) e pouco atendia às necessidades das crianças que
O acolhimento familiar foi instituído como política as-
precisavam do Sistema de Proteção (Amorós & Palacios,
sistencial em diferentes países em resposta a condições
2004; Amorós & Peláez, 2004).
históricas específicas, tais como situações de guerra e pós-
Essa situação mudou a partir da década de 90, sendo
-guerra, mudanças de paradigmas em relação à infância e
que atualmente o acolhimento familiar naquele país subdi-
ao status da família, crises econômicas, avaliações sobre os
vide-se em diferentes modalidades e conta com um banco
efeitos prejudiciais da institucionalização de crianças (Luna,
de famílias cadastradas, selecionadas e formadas para aco-
2004). Nos EUA e Canadá, por exemplo, o acolhimento fami-
lher. Além disso, ele busca absorver toda a demanda de
liar foi implementado, ainda no século XIX. Posteriormente, crianças e adolescentes em vulnerabilidade que necessitam
trabalhos como os de Bowlby, Spitz e Winnicott fundamen- de acolhimento, embora ainda existam pequenas institui-
taram melhor a construção dessa medida de proteção, e já ções de abrigamento (Amorós, Palacios, Fuentes, & León,
há alguns anos as políticas públicas desses países preconi- 2003). Hoje, na Espanha, o acolhimento familiar constitui
zam que todas as crianças e adolescentes separados dos uma alternativa de proteção mais utilizada que o acolhi-
seus pais sejam acolhidos em uma outra família (Brasil, 2004; mento institucional (Del Valle & Rateava, 2003).
George & Van Oudenhoven, 2003). Vale ressaltar que o aco- No Brasil, de modo semelhante ao que acontecia an-
lhimento familiar na Inglaterra e em Israel data do período teriormente na Espanha, o abrigamento em instituições
pós Segunda Guerra e que nos Estados Unidos já havia in- constituiu-se historicamente na modalidade de proteção
dicação de que seria melhor acolher crianças e adolescentes mais utilizada para crianças retiradas de sua família por
em famílias substitutas do que em instituições, desde o 1° negligência, violência, abandono ou orfandade (Baptista,
Congresso Sobre a Infância, ocorrido em 1909 (Luna, 2004). 2006; Fonseca, 1995; Justo, 1997; Rizina & Rizina, 2004).
Atualmente, na Inglaterra, o foster care caracteriza-se Desde a criação das Rodas de Expostos para os enjeitados,
pela transferência temporária ou permanente dos deve- as políticas de assistência às crianças e adolescentes em
res e direitos parentais dos pais biológicos para um outro vulnerabilidade priorizaram o internamento em institui-
adulto com quem a criança ou adolescente, em geral, não ções, com pouco ou nenhum convívio comunitário e social
tem relações consanguíneas. Para isso, as autoridades locais e com favorecimento do rompimento dos vínculos familia-
devem traçar um plano de intervenção que justifique essa res (Rizina, 1993). É importante pontuar que esse tipo de
transferência como fundamental para o bem-estar da crian- assistência foi criado para os desvalidos, filhos de famílias
ça ou adolescente. A colocação em uma família acolhedora com poucas condições socioeconômicas, pauperizadas,
visa garantir os cuidados básicos (alimentação e suporte ao que eram vistos como “carentes”, possíveis “delinquentes”
desenvolvimento físico, emocional e educacional), mas não ou “vadios” e, portanto, precisavam ser educados e contro-
altera a identidade da criança ou de seus pais biológicos, lados, enquanto suas famílias eram consideradas incompe-
nem substitui os direitos e obrigações que envolvem tal tentes para cuidar e educar.

92
CONHECIMENTOS ESPECÍFICOS - PSICÓLOGO

Fonseca (2004) afirma que com a insuficiência de po- No que tange às normativas internacionais ratifica-
líticas públicas adequadas, o abrigo ainda funciona como das pelo Brasil que podem auxiliar a argumentação pelo
um “colégio interno” para crianças pobres, quando suas acolhimento familiar como uma modalidade de proteção,
mães vivenciam o esgotamento dos recursos financeiros podem ser citadas aquelas que foram desconstruindo pa-
e de sua rede social de apoio. França (2006) ratifica esse radigmas e concretizando discussões sobre a criança ou o
argumento ao descrever a experiência do projeto de Famí- adolescente como sujeito de direitos em condição peculiar
lias Acolhedoras de Diadema (SP), em que várias mães ou de desenvolvimento. São exemplos: a Declaração sobre os
parentes dos acolhidos diziam preferir que suas crianças Direitos da Criança (1959); o Pacto Internacional dos Direi-
tivessem sido encaminhadas para um abrigo a que fos- tos Civis e Políticos (1966); o Pacto de São José da Costa
sem encaminhadas para famílias acolhedoras. Acreditavam Rica (1969); o Protocolo Adicional à Convenção das Nações
que na instituição não corriam o risco de perder a crian- Unidas contra o Crime Organizado Transnacional Relativo
ça, pois significavam o abrigo como um local onde ela é à Prevenção, Repressão e Punição do Tráfico de Pessoas,
cuidada, tem acesso ao estudo e tem maior estabilidade, em especial Mulheres e Crianças (ratificado pelo Brasil em
enquanto as mães e/ou familiares tentam conseguir sua 2004); e o Protocolo Facultativo à Convenção sobre os Di-
sobrevivência. “As mães acreditavam que, após resolverem reitos da Criança Referente à Venda de Crianças, à Prostitui-
ção Infantil e à Pornografia Infantil, ratificado pelo Brasil em
seus problemas, teriam seus filhos de volta naturalmente,
2004 (MDS & SEDH, 2006). No entanto, deve-se destacar o
desconhecendo que estavam abrindo mão da guarda, seja
papel preponderante da Convenção sobre os Direitos da
em favor do abrigo, seja em favor da família acolhedora”
Criança (ONU, 1989).
(França, 2006, p. 76).
Essa convenção, em seu preâmbulo, define o direito
Conforme se pode observar, implantar uma nova pro- da criança à proteção especial e acrescenta a esses outros
posta de acolhimento para crianças e adolescentes cujos direitos (de liberdade de opinião, expressão, pensamento,
direitos são violados, que seja de qualidade e mais eficaz religião etc), que demarcam duas prerrogativas que a so-
que a antiga institucionalização, exige tempo e mudanças. ciedade e o Estado devem possibilitar à criança e ao ado-
Mudanças de ordem legal, de práticas institucionais, de pa- lescente, para operacionalizar a proteção dos seus Direitos
radigmas de família, de concepções de criança e adoles- Humanos: cuidados e responsabilidades. A discussão de
cência, de práticas e políticas sociais. medidas de acolhimento e proteção relaciona-se, então,
aos cuidados que o Estado e a sociedade devem ter com a
Do ponto visto legal, já ocorreram transformações que infância e juventude do país.
possibilitam ao acolhimento familiar ancorar-se em leis Dessa forma, é possível perceber que o acolhimento
e normativas. A Constituição da República Federativa do familiar vem sendo compreendido como uma medida de
Brasil (1988), o ECA (1990), a Lei Orgânica da Assistência proteção a crianças e adolescentes que vivem situações de
Social ([LOAS], 1993), somados à Política Nacional de As- violação de direitos. Entretanto, ele abre questões que ain-
sistência Social (MDS, 2004) e ao Plano Nacional (MDS & da demandam aprofundamento, discussão e adequação à
SEDH, 2006), deram base e constituíram o marco legal para nossa realidade cultural. Uma delas diz respeito a um dos
a argumentação, justificativa e inclusão do acolhimento aspectos desenvolvimentais da criança e do adolescen-
familiar como uma política pública nacional. Vale destacar te acolhido: a construção de vínculos e relações afetivas
que ocorreram iniciativas do Poder Judiciário e municipais na família acolhedora. Outra se refere à transformação de
antes da elaboração desses dois últimos documentos na- uma cultura de acolhimento na sociedade, que comporte
cionais que preconizam o acolhimento familiar (vide a ex- o acolhimento familiar, com novas significações de família,
periência dos projetos de Acolhimento Familiar do Estado maternidade, paternidade. Conforme Cabral (2004) argu-
de São Paulo), mas nesses casos trata-se de políticas locais menta, o acolhimento familiar como programa alternativo
de assistência social. à instituição merece mais estudo e aprofundamento teóri-
Ao afirmar que “a família é a base da sociedade” (art. co-prático, pois existe pouco conhecimento técnico sobre
o tema. Para a autora, a implantação do acolhimento fami-
226 e 227) e que a criança ou adolescente tem direito à
liar “não significa puramente a transposição de um mode-
“convivência familiar e comunitária”, a Constituição Federal
lo de atendimento para o outro, como se o Acolhimento
(1988) reforça o papel da família na vida da criança e do
Familiar fosse uma ‘solução’ para substituir o estado atual
adolescente como elemento fundamental dentro do pro-
das coisas” (p. 12).
cesso de proteção integral e como parceira do Estado nes- Além disso, alguns problemas se configuram como
sa proteção. Desse modo, quando esgotados os recursos fundamentais para serem analisados quando se pensa na
ou possibilidades da permanência da criança na família de constituição, fomento e sistematização de programas de
origem, dispõe a lei que os operadores sociais e do direito acolhimento familiar. Por exemplo: Como eles estão se
devem buscar a colocação da criança em família substituta estruturando? Quais crianças visam atender (vitimizadas,
na forma de guarda, tutela ou adoção ou, ainda, a coloca- grupo de irmãos, bebês, adolescentes, etc)? Que tipo de
ção em acolhimento institucional. Para tanto, são necessá- suporte especializado vão poder oferecer às famílias aco-
rias políticas e programas que viabilizem essas colocações, lhedoras, às famílias de origem e aos acolhidos? Pode-se
e o acolhimento familiar conta então com base legal para ainda perguntar se essa será uma alternativa de proteção
sua execução. adequada para crianças em todas as faixas etárias. Essas são

93
CONHECIMENTOS ESPECÍFICOS - PSICÓLOGO

questões que se levantam num momento de transição e de Bowlby é bastante citado na escassa literatura nacional
criação de uma nova cultura de acolhimento, que objetiva sobre acolhimento familiar. A sua visão de que a institucio-
contemplar outros modelos de proteção diferenciados da nalização, o rompimento de vínculos e a privação mater-
antiga institucionalização. Questões que estão intimamen- na deixariam marcas profundas e prejudiciais na criança é
te relacionadas às concepções de família, de acolhimento, frequentemente usada na argumentação para criação de
de desenvolvimento da criança e do adolescente, de víncu- programas de acolhimento familiar como alternativa ao
lo afetivo, que os projetos esposam. acolhimento em instituições:
O fracasso no desenvolvimento da personalidade nas
Perspectivas Teóricas de Vinculação Afetiva em Si- crianças que sofreram privação é, talvez, melhor compreen-
tuações de Acolhimento dido quando se considera que é a mãe que, nos primeiros
anos de vida da criança, funciona como sua personalida-
A fundamentação teórica, na qual o ECA (1990) e os de e consciência. A criança em instituição nunca teve es-
programas de acolhimento familiar já implantados no Brasil tas experiências, dessa forma, não pode nunca completar
baseiam-se para valorizar a família como contexto primor- a primeira fase do desenvolvimento - estabelecer uma re-
dial de desenvolvimento da criança, articula-se a algumas lação com uma figura materna claramente definida. Tudo
teorias dentro da Psicologia, especialmente a Psicologia do que teve foi uma sucessão de agentes paliativos, cada um
Desenvolvimento. A própria inclusão do direito à convivên- auxiliando-a de uma forma limitada, mas nenhum deles
cia familiar e comunitária nas normativas internacionais e proporcionando-lhe a continuidade no tempo, que faz par-
nacionais foi subsidiada por teorias que ressaltavam a im- te da essência da personalidade. É bem possível que essas
portância dos vínculos afetivos com as figuras parentais crianças, gravemente privadas por nunca terem sido objeto
para o desenvolvimento saudável da criança, como tam- de um cuidado por parte de um mesmo ser humano, nunca
bém os efeitos nocivos ao desenvolvimento causado pelo tenham tido a oportunidade de aprender os processos de
processo de institucionalização. abstração e de organização do comportamento no tempo
Dessa forma, segundo Vicente (1998), o vínculo afetivo e no espaço. Certamente, suas graves deformações psíqui-
passou a ser valorizado nesses documentos e adquiriu uma cas são exemplos claros do princípio de que os danos in-
dimensão política, visto que seu desenvolvimento e manu- fligidos muito cedo produzem perturbações generalizadas
tenção, quando não ocorre na família de origem, neces- no crescimento (Bowlby, 1952/1995, p. 61).
sita de proteção do Estado, no sentido de assegurar essa Nesse trecho é evidente a argumentação de que a
possibilidade às crianças e adolescentes. Assim, o direito à privação materna favoreceria a origem de uma psicopa-
convivência familiar passa a fazer parte de um conjunto de tologia, o que, para o autor, estaria associada à vivência
elementos das políticas públicas e fundamenta ações de
institucional. Bowlby (1979/1997) considerava que muitos
colocação em famílias substitutas (sob guarda, tutela ou
distúrbios psiconeuróticos e da personalidade eram refle-
adoção) ou acolhedoras. Tais questões nos levam a refletir
xos de distúrbios na capacidade de estabelecimento de
sobre os aportes teóricos, especialmente da Psicologia, em
vínculos afetivos, originados numa “falha” de desenvolvi-
que se baseiam as práticas e políticas de acolhimento e
mento na infância ou em transtornos posteriores. Em seus
assistência às crianças e adolescentes sob violação de di-
escritos cita várias pesquisas realizadas na década de 60
reitos.
que tinham esse enfoque e que traduziam uma concepção
Das teorias que discutem a importância do estabele- quase hegemônica na ciência psiquiátrica e psicológica de
cimento de vínculos afetivos entre a criança e figuras pa- associarem a psicopatia, sociopatia, depressão e a delin-
rentais, destaca-se a Teoria do Apego, desenvolvida por quência com a privação materna, paterna ou orfandade.
Bowlby (1969/1990, 1952/1995, 1979/1997) e Ainsworth, Rutter (1972) teceu críticas bem fundamentadas sobre es-
Blehar, Waters e Wall (1978). Segundo ela, o ser humano é ses estudos, que observavam comportamentos de pacien-
portador de uma história filogenética que lhe garante um tes psiquiátricos ou crianças institucionalizadas, apontando
aparato biológico, que o auxilia no estabelecimento e ma- que as patologias presentes derivavam mais das péssimas
nutenção de vínculos afetivos com o outro, principalmente condições institucionais a que eles foram submetidos do
com a mãe (biológica ou substituta), sendo a constância que propriamente à privação materna.
dessa primeira vinculação afetiva crucial ao seu desenvol-
vimento psicossocial saudável. Nessa teoria, verificam-se Outro aspecto a se considerar é que tal concepção so-
concepções sobre a importância da constância de uma bre o desenvolvimento humano coloca o contexto familiar
pessoa (geralmente associada à figura materna) no cuida- como o locus privilegiado para o estabelecimento das re-
do dispensado ao bebê e sobre a determinação de um pe- lações de apego, contexto em que a criança teria um cui-
ríodo sensível para o desenvolvimento do apego, que seria dador individualizado. Também apresenta uma tendência à
dos seis primeiros meses de vida aos três anos de idade. patologização de contextos coletivos de desenvolvimento
Assim, os autores tratam da primazia da relação de apego da criança (como os abrigos), especialmente para a criança
mãe-criança, apontando para consequências disfuncionais pequena. Nesse caso, pouco valor é dado para o estabe-
sobre o desenvolvimento social, psicológico e emocional lecimento de vínculos afetivos com outros cuidadores que
da criança, caso a relação não se estabeleça, seja interrom- não a mãe ou substituta, ou mesmo, com coetâneos. Essa
pida ou se mantenha de modo inadequado (Rossetti-Fer- visão de estabelecimento de apego centra-se na análise de
reira, 1986, 2006). relações diádicas, pouco considerando o dinamismo das

94
CONHECIMENTOS ESPECÍFICOS - PSICÓLOGO

relações familiares, os outros interatores, as significações e construção de vínculos afetivos, dentro de uma perspectiva
práticas presentes nesse contexto, que são marcados pela contextual e sistêmica que possa abordar a complexida-
cultura em que se inserem (Rossetti-Ferreira, 2006). de do assunto. É interessante considerar as ideias de Lewis
A supremacia dos laços consanguíneos para o cuida- (1999), quando ele argumenta que a múltipla determina-
do de crianças é, sem dúvida, uma ideologia dominante ção, acasos, os encontros acidentais e a interação desses,
em nossa sociedade. Em parte, é dessa idéia que emana são problemas inerentes ao desenvolvimento humano e
a concepção hegemônica encontrada em diversas tendên- que dificultam a previsão do curso da vida humana. O au-
cias teóricas da predestinação à psicopatologia das crian- tor critica ideias das teorias do desenvolvimento, que ele
ças separadas da família biológica. Seus históricos de vida, chama de “ideias fixas”, as quais são inquestionavelmente
às vezes marcados por aquilo que a literatura clássica de aceitas, como: a perspectiva de continuidade e evolução
desenvolvimento humano considera estressores sociais de no desenvolvimento, a causalidade no passado e a idéia
diversas ordens, aos quais se somam vivências institucio- de que o passado atua sobre o presente e o futuro. Tudo
nais, são vistos a partir de uma conotação negativa, idéia isso compõe o que ele chama de um modelo organicista
predominante que se espraia em diferentes âmbitos da de desenvolvimento, que sempre considera a criança no
vida social, permeando tanto os discursos do senso comum processo de vir-a-ser, no que ela se tornará no futuro, e,
como os do meio científico (Rossetti-Ferreira, 2006). sobre o qual, a maioria das políticas públicas de atenção
Dessa forma, a ameaça ou existência de rupturas afe- à infância e juventude se baseia. Para o autor, é necessário
tivas anteriores parece criar enredamentos ou tramas que intervir quando se presencia sofrimento e injustiça, mas fa-
as pessoas em interação reeditam nas suas práticas dialó- zer isso buscando alcançar um efeito no futuro “pode ser
gicas e discursivas, co-construindo no momento atual, os uma bobagem, porque nós e as forças sobre as quais não
problemas ou uma visão de desenvolvimento inadequado temos controle - alteramos nossos destinos todos os dias,
para essas crianças. Rossetti-Ferreira (2007) argumenta de forma imprevisível” (Lewis, 1999, p. 31).
que a Teoria do Apego constitui um exemplo da relevância
das ciências do desenvolvimento humano para a definição Lewis (1999) propõe um modelo contextual de desen-
das políticas e práticas sociais de educação e proteção de volvimento que considere a natureza do ambiente onde
crianças e jovens, sobretudo daqueles que vivem situações a criança cresce, o “comportamento-em-contexto”, a rede
de vulnerabilidade social. Isso porque as redes de significa- social em que se insere, inclusive para se falar em desenvol-
ções que permeiam as políticas e práticas de acolhimento vimento afetivo. Argumenta que as políticas sociais devem
de crianças e adolescentes historicamente esbarram em considerar o momento presente da família, o momento do
certas ideias predominantes sobre apego, desenvolvimen- acolhimento, e que a idéia do curar deve dar lugar à idéia
to normal/anormal, fatores de risco/fatores protetores e do cuidar, numa alusão às teorias organicistas que estão
sobre a concepção de que a instituição familiar, particu- sempre amarradas ao passado, preocupadas com o futuro
larmente a do tipo nuclear, é a única e a ideal, aquela que e com a cura.
melhor oferece um ambiente pleno ao desenvolvimento Em consonância com essas ideias, consideramos que
de uma criança. Assim, as visões teórico-metodológicas da para se compreender o desenvolvimento afetivo e o ape-
área contribuem para construir realidades sociais que po- go é necessário desfocar das relações diádicas e buscar
dem influir, modificar e restringir o desenvolvimento e a apreender a rede de relações (e significações) em que a
qualidade de vida das pessoas, muitas vezes inserindo-as criança está imersa, a partir de uma perspectiva processual,
em um movimento de exclusão. relacional, situada e discursiva. Entendemos que o apego é
Cabral (2004), ao defender a criação dos programas de construído nas e através das interações e relações recípro-
acolhimento familiar, retoma a questão da construção de cas, que ocorrem em contextos específicos. Tais contextos
vínculos afetivos como um dos seus argumentos centrais. envolvem discursos vivenciados e situados, os quais colo-
Afirma que o acolhimento em famílias garante o direito a cam os parceiros em certas posições. Esse processo favo-
uma convivência familiar com dignidade e a possibilida- rece a construção de certos sentidos e um repertório de
de de construção de laços afetivos. Entretanto, temos nos papéis possíveis, circunscrevendo (estabelecendo limites e
perguntado como seria essa “propagandeada” constru- possibilidades para) assim o fluxo de comportamentos e o
ção de vínculos dentro de um programa de acolhimento desenvolvimento dos sujeitos (Rossetti-Ferreira, 2007).
familiar, visto ser esperado que eles sejam estabelecidos,
porém com um caráter de provisoriedade, pois todos os Gailey (2000), ao abordar a realidade social norte-ame-
envolvidos devem saber que haverá o momento de nova ricana, discute que a adoção e o acolhimento familiar cons-
separação, quando o acolhido retornará à sua família de tituem um desafio à noção de que a maternidade, a pater-
origem ou será colocado em adoção. Como os programas nidade e a possibilidade de vinculação afetiva das crianças
estão concebendo a possibilidade da criança ou adoles- estão associadas a conexões genéticas ou de nascimento.
cente vincular-se a uma ou mais famílias de acolhimento Segundo a autora, eles fornecem uma lente através da qual
e depois retornar para a família de origem? E, ainda, como podem ser vistos os vínculos de afetividade e parentesco
manter o vínculo com a família de origem? sendo construídos.
Essas são questões complexas, que instigam a con- Tal discussão nos leva a considerar que a compreen-
dução de pesquisas, entretanto, consideramos necessário são de como as equipes dos programas de acolhimento
que seja assumido outro paradigma para compreensão da familiar estão significando o processo de vinculação afeti-

95
CONHECIMENTOS ESPECÍFICOS - PSICÓLOGO

va daqueles envolvidos no acolhimento (acolhidos, família O art. 101 do Estatuto da Criança e Adolescente em seu
acolhedora e família de origem) é de grande importância § 1º e 2º estabelece:
para o desenvolvimento dos projetos, conforme afirma o
Plano Nacional. § 1º O acolhimento institucional e o acolhimento
A decisão sobre a separação é de grande responsabili- familiar são medidas provisórias e excepcionais, utilizáveis
dade e deve estar baseada em uma recomendação técnica, como forma de transição para reintegração familiar ou, não
a partir de um estudo diagnóstico, caso a caso, realizado sendo esta possível, para colocação em família substituta,
por equipe interdisciplinar, com a devida fundamentação não implicando privação de liberdade.
teórica - desenvolvimento infantil, etapas do ciclo de vida § 2º Sem prejuízo da tomada de medidas emergenciais
individual e familiar, teoria dos vínculos e estratégias de para proteção de vítimas de violência ou abuso sexual
sobrevivência de famílias em situação de extrema vulne- e das providências a que alude o art. 130 desta Lei, o
rabilidade. A realização deste estudo diagnóstico deve ser afastamento da criança ou adolescente do convívio familiar
realizada em estreita articulação com a Justiça da Infância é de competência exclusiva da autoridade judiciária e
e da Juventude e o Ministério Público, de forma a subsidiar importará na deflagração, a pedido do Ministério Público
tal decisão (MDS & SEDH, 2006, p. 71, grifos nossos). ou de quem tenha legítimo interesse, de procedimento
judicial contencioso, no qual se garanta aos pais ou ao
responsável legal o exercício do contraditório e da ampla
Fonte:
defesa.
Nina Rosa do Amaral Costa; Maria Clotilde Rossetti-
É necessário ressaltar que existem medidas anteriores
-Ferreira. Acolhimento familiar: uma alternativa de prote- ao acolhimento que visam a proteção dos direitos de
ção para crianças e adolescentes crianças e adolescentes de forma preventiva, e objetivando
o fortalecimento das famílias, justamente para evita o rom-
O ACOLHIMENTO INSTITUCIONAL COMO MEDIDA pimento do vínculo familiar que poderá ocorrer com a reti-
DE PROTEÇÃO rada da criança de seu ambiente doméstico.

O art. 19 do Estatuto da Criança e do Adolescente, trata O acolhimento institucional além de determinação ju-
do direito à convivência familiar e comunitária e afirma: É dicial, também acontece por encaminhamentos dos Servi-
direito da criança e do adolescente ser criado e educado no ços de Alta Complexidade do Sistema Único de Assistência
seio de sua família e, excepcionalmente, em família subs- Social, através do CREAS [4], podendo levar a medida de
tituta, assegurada a convivência familiar e comunitária, em proteção a um patamar de insegurança jurídica se, a inter-
ambiente que garanta seu desenvolvimento integral. (Re- venção do Ministério público e Poder Judiciário não forem
dação dada pela Lei nº 13.257, de 2016). imediatas.
No entanto, quando se esgotam as possibilidades de O Conselho Nacional do Ministério Público, em uma
favorecer a família no tocante a proteção de suas crian- publicação baseada em resultados das inspeções anuais
ças e adolescentes a lei autoriza seu afastamento do am- realizadas pelos Promotores de Justiça da infância e juven-
biente familiar, considerando critérios estabelecidos pelo tude em todo o país[5], levantam dados sobre acolhimento
ECA quando regula o acolhimento institucional como uma institucional no ano de 2013, que apontam mais de 30 mil
medida provisória e excepcional, utilizada como forma de crianças e adolescentes afastados da convivência familiar,
transição para reintegração familiar. pois se encontram em acolhimento institucional.
As instituições de acolhimento são regidas pelo Estatu- O mesmo relatório aponta que em 27,9% das institui-
to da criança e do adolescente e cabe aos Conselhos Muni- ções de acolhimento, há registro de crianças ou adolescen-
cipais de Direitos da Criança e do Adolescente acompanhar tes encaminhadas ao serviço sem ordem judicial.
o período de acolhimento institucional, para que crianças
Nestes casos pela quantidade de crianças e adolescen-
e adolescentes tenham assegurados o retorno a família de
tes em acolhimento institucional, a medida de proteção
origem ou colocação em família substituta garantindo as-
que baseia sua aplicação no princípio da excepcionalidade,
sim o direito a convivência familiar.
erroneamente tem se revelado uma saída rápida para os
problemas sociais de famílias em vulnerabilidade, demons-
O Acolhimento institucional, como medida de prote- trando o descompasso entre a legislação e a realidade des-
ção, está disposto no Estatuto da criança e do adolescente ta medida de proteção.
dentre um rol exemplificativo de medidas que, em grande
maioria submete a família a um programa de atendimento, Outro legitimado para encaminhar crianças e adoles-
objetivando socorrer a criança ou adolescente de seus ris- centes para acolhimento institucional é o Conselho Tutelar,
cos ainda no ambiente familiar. quando menores de 18 anos têm seus direitos violados ou
Esgotado tais possibilidades resta então o afastamento ameaçados, seja por omissão ou ação da sociedade e do
da criança e adolescente do convívio familiar, medida de Estado ou ainda em decorrência de falta, omissão ou abuso
competência exclusiva da autoridade judiciária, onde res- dos pais ou responsáveis, necessariamente nas situações
tará garantido aos pais ou responsáveis o devido processo previstas no artigo 98 do Estatuto da Criança e do Adoles-
legal. cente, são destinatárias das medidas de proteção imediata
e urgente.

96
CONHECIMENTOS ESPECÍFICOS - PSICÓLOGO

O Conselho Tutelar, atendendo tais casos, tem atribui- Em 2003, o Instituto de Pesquisa Econômica Aplicada -
ção de aplicar as medidas de proteção do artigo 101, inci- IPEA realizou um estudo nacional sobre a situação de crian-
sos I a VII, excluindo-se, portanto, a colocação em família ças e adolescentes encontradas em abrigos que receberam
substituta que depende sempre de procedimento judicial. recursos do governo federal.
O Conselho decide o caso e tais decisões devem ter Foram avaliadas 589 entidades de acolhimento insti-
origem de um procedimento previamente disciplinado no tucional, e constatou-se que mais da metade (55,2%) das
seu Regimento Interno, de sorte que todos os atos pra- cerca de 20 mil crianças e adolescentes encontrados estava
ticados para instruir a decisão sejam formalizados. Isto é nos abrigos há um período que variava entre sete meses e
imprescindível para se aferir a legalidade das decisões do cinco anos.
Conselho, que são sujeitas à revisão judicial mediante pro- A parcela mais significativa (32,9%), porém, estava nos
vocação por quem tenha legítimo interesse. abrigos há um período entre dois e cinco anos, 13,3% entre
seis e dez anos e 6,4% por mais de dez anos, ultrapassando
o prazo estipulado pelo ECA e infringindo a temporarieda-
Embora o acolhimento institucional possa se dá através
de da medida de acolhimento institucional.
do Conselho Tutelar, de forma administrativa, o art. 137 do
Além disso, a grande maioria dos abrigados tinha fa-
ECA afirma que: As decisões do Conselho Tutelar somente
mília (86,7%) e dentre os motivos que os levaram aos abri-
poderão ser revistas pela autoridade judiciária a pedido de gos, a pobreza foi a mais citada (24,2%), depois o abando-
quem tenha legítimo interesse. no (18,9%), a violência doméstica (11,7%), a dependência
Portanto resta evidente que a partir do estabelecimen- química dos pais ou responsáveis, incluindo alcoolismo
to de medida de proteção, acolhimento institucional, faz-se (11,4%), a vivência de rua (7,0%) e a orfandade (5,2%)[8].
necessário a defesa por parte de advogados como propõe Compreender no que consiste as palavras proviso-
Artigo 206 da Lei nº 8.069 de 13 de julho de 1990. riedade e excepcionalidade que são fundamentais para a
A criança ou o adolescente, seus pais ou responsável, determinação de medida de proteção em acolhimento ins-
e qualquer pessoa que tenha legítimo interesse na solução titucional e de suma importância para a sua observação e
da lide poderão intervir nos procedimentos de que trata cumprimento.
esta Lei, através de advogado, o qual será intimado para
todos os atos, pessoalmente ou por publicação oficial, res- EXCEPCIONALIDADE
peitado o segredo de justiça.
A Excepcionalidade consiste em somente adotar a me-
Sobre esse tema Frasseto, na época Defensor Público dida quando não houver mais possibilidade alguma de ma-
observa: nutenção da criança ou do adolescente na família natural,
já tendo sido esgotadas medidas anteriores elencadas que
As decisões de afastamento de crianças de suas famí- são voltadas a favorecer a família e auxilia-las em suas ne-
lias são frequentemente tomadas sem que a família se faça cessidades físicas e sociais conforme estabelece Art. 101.
representar por advogado/defensor público. São poucas as Verificada qualquer das hipóteses previstas no art. 98, a
famílias que procuram espontaneamente um defensor para autoridade competente poderá determinar, dentre outras,
apoia-las em sua pretensão de recuperar a guarda de filhos as seguintes medidas:
em acolhimento institucional. I - encaminhamento aos pais ou responsável, mediante
Neste sentido a dificuldade em se manifestar no pro- termo de responsabilidade;
cesso de afastamento do ambiente familiar, poderá concor- II - orientação, apoio e acompanhamento temporários;
III - matrícula e frequência obrigatórias em estabeleci-
rer para longo período de acolhimento institucional, fragili-
mento oficial de ensino fundamental;
zando a premissa de excepcionalidade e provisoriedade da
IV - inclusão em programa comunitário ou oficial de
medida e ainda impedindo a realização do devido processo
auxílio à família, à criança e ao adolescente;
legal dos quais são titulares os pais de crianças em acolhi- V - requisição de tratamento médico, psicológico ou
mento institucional. psiquiátrico, em regime hospitalar ou ambulatorial;
VI - inclusão em programa oficial ou comunitário de
EXCEPCIONALIDADE E PROVISORIEDADE DA ME- auxílio, orientação e tratamento a alcoólatras e toxicôma-
DIDA nos;
VII - acolhimento institucional;
O acolhimento institucional e o acolhimento familiar VIII - inclusão em programa de acolhimento familiar;
são medidas provisórias e excepcionais, utilizáveis como IX - colocação em família substituta.
forma de transição para reintegração familiar ou, não sen-
do esta possível, para colocação em família substituta, não O manejo inicial de aplicação da medida deve sujeitar
implicando privação de liberdade. Lei 8069/90 Art. 101, § a decisão em adotar o acolhimento institucional a um cri-
1o tério rigoroso de triagem, através de um pré diagnostico,
Quando se trata de medidas que em seu fundamen- elaborado por equipes de referência, que detém o poder
to legal se baseia em provisoriedade e excepcionalidade, de encaminhar crianças e adolescentes ao acolhimento,
os dados de pesquisas levantam sérios questionamentos observando o que determina o Estatuto da Criança e do
sobre a os critérios aplicados nas decisões para o afasta- Adolescente quanto as possibilidades anteriores a aplica-
mento do lar. ção da medida.

97
CONHECIMENTOS ESPECÍFICOS - PSICÓLOGO

Portanto entende-se que, para que a criança ou ado- PROVISORIEDADE


lescente seja encaminhada ao acolhimento institucional,
deve-se observar que várias medidas administrativa/sociais Consiste em permanecer na instituição de acolhimento
podem ser aplicadas à família até que se decida pelo afas- por tempo estritamente necessário, não podendo ultrapas-
tamento de seu lar. sar o período de dois anos, salvo por comprovada necessi-
Os procedimentos administrativos que antecedem o dade, conforme o § 2º do art. 19. Do Estatuto da Criança e
acolhimento institucional, ou seja, o pré diagnostico, ela- do adolescente.
borado por equipe de referência do CREAS ou pelo conse-
lho Tutelar devem ser sobremodo minucioso, visando não A situação deve ser reavaliada a cada seis meses e a
corroborar para a retirada da criança ou adolescente de seu instituição deverá manter atualizado o Plano Individual de
ambiente familiar sem a precisão necessária na avaliação. Atendimento do (a) acolhido (a), visando sempre à reinte-
Estabelece a lei que, na aplicação das medidas deverão gração familiar, preferencialmente, ou à colocação em fa-
mília substituta.
ser levadas em conta as que atendam as necessidades pe-
Neste sentido as instituições que acolhem crianças e
dagógicas e que visem o fortalecimento dos vínculos fami-
adolescentes são convocadas a aderir o que determina a lei
liares e comunitários, e por este motivo várias ações de âm-
8.069/90 no que tange a preparação para o desligamento
bito administrativo são oferecidas pelas políticas públicas gradativo do acolhido que, se inicia no momento que a
para contribuir na permanência da criança ou adolescente criança ou adolescente é acolhido, sobre isso expõe o § 4º
no seio da família. do art. 101.
Outrossim podemos ressaltar o que aduz o ECA, art. [...] imediatamente após o acolhimento da criança ou
23, “A falta ou carência de recursos materiais não constitui do adolescente, a entidade responsável pelo acolhimento
motivo suficiente para a perda ou a suspensão do pátrio institucional elaborará um plano individual de atendimen-
poder”, não justificando, portanto, que o acolhimento seja to, visando a reintegração familiar.
consubstanciado de forma administrativa por motivo de [...], com exceção de ser determinado judicialmente
vulnerabilidade e risco social quando sinônimos de pobre- que a reintegração familiar não ocorra, e sobretudo levan-
za. do em consideração a opinião da criança e também a oitiva
A retirada de crianças e adolescentes de seu ambiente dos pais, levando ao entendimento que, interação poder
familiar se justifica apenas quando os seus direitos forem judiciário, família e instituições de acolhimento são a cha-
ameaçados ou violados, e o Estatuto da Criança e do Ado- ve para que o período de acolhimento institucional seja o
lescente elenca princípios que se devem observar para que mais breve possível.
se imponha a medida de afastamento, o art. 100 de forma É necessário o compromisso de instituições de
brilhante inclui a consideração e participação da própria acolhimento facilitando, estimulando e favorecendo
criança ou adolescente e seus pais, na adoção da medida, o processo de fortalecimento de vínculos da criança e
em seus incisos XI e XII: adolescente acolhidos com sua família de origem visando
XI - obrigatoriedade da informação: a criança e o ado- facilitar o seu retorno ao convívio familiar.
lescente, respeitado seu estágio de desenvolvimento e ca- Este procedimento se dá a partir de avaliações, en-
pacidade de compreensão, seus pais ou responsável devem caminhamentos e relatórios técnicos tanto do acolhido,
ser informados dos seus direitos, dos motivos que determi- quanto da sua família, elaborados pela Equipe técnica do
naram a intervenção e da forma como esta se processo acolhimento em consonância com o art. 19 do ECA:
XII - oitiva obrigatória e participação: a criança e o ado- § 1º Toda criança ou adolescente que estiver inserido
em programa de acolhimento familiar ou institucional
lescente, em separado ou na companhia dos pais, de res-
terá sua situação reavaliada, no máximo, a cada 6 (seis)
ponsável ou de pessoa por si indicada, bem como os seus
meses, devendo a autoridade judiciária competente, com
pais ou responsável, têm direito a ser ouvidos e a participar
base em relatório elaborado por equipe interprofissional
nos atos e na definição da medida de promoção dos direi- ou multidisciplinar, decidir de forma fundamentada pela
tos e de proteção, sendo sua opinião devidamente consi- possibilidade de reintegração familiar ou colocação em
derada pela autoridade judiciária competente, observado o família substituta, em quaisquer das modalidades previstas
disposto nos §§ 1o e 2o do art. 28 desta Lei. no art. 28 desta Lei.
É fundamental afirmar que a medida de afastamento Para tanto, as instituições acolhedoras necessitam re-
de crianças e adolescentes de seu lar, ressalvados casos que ceber investimento de recursos públicos de forma priori-
forem verificada de forma emergencial hipótese de abuso tária para adequação e capacitação continuada de equipe
sexual ou violência, [...] é de competência exclusiva da au- multidisciplinar, no mínimo composta por Assistentes So-
toridade judiciária e importará na deflagração, a pedido do ciais, Psicólogos, Pedagogos e Advogados, dos quais serão
Ministério Público ou de quem tenha legítimo interesse, de os principais agentes na recondução da criança ou adoles-
procedimento judicial contencioso, no qual se garanta aos cente acolhido ao convívio familiar.
pais ou ao responsável legal o exercício do contraditório e O Estatuto da Criança e do Adolescente, reafirma, em
da ampla defesa. Art. 101§ 2o. seus arts. 4º, 5º, o enunciado do art. 227, da Constituição
Federal, no parágrafo único de seu art. 4º, detalhando e ex-
plicando como deve se aplicar de forma prática a garantia
de prioridade absoluta contida no citado Texto Constitu-
cional:

98
CONHECIMENTOS ESPECÍFICOS - PSICÓLOGO

Art. 4º. É dever da família, da comunidade, da socie- Nos casos em que o motivo que ensejaria a aplicação
dade em geral e do Poder Público assegurar, com abso- da medida de abrigo referir-se à falta ou precariedade de
luta prioridade, a efetivação dos direitos referentes à vida, condições de habitação da família, deve-se recorrer a me-
à saúde, à alimentação, à educação, ao esporte, ao lazer, à didas que preservem o convívio familiar e mantenham a
profissionalização à cultura, à dignidade, ao respeito, à li- família, a criança e o adolescente em condições de segu-
berdade e à convivência familiar e comunitária. rança e proteção, como a inclusão imediata de todos seus
membros conjuntamente em serviços de acolhimento para
Parágrafo único. A garantia de prioridade compreende: adultos com crianças ou adolescentes e acesso à moradia
[...] c) preferência na formulação e a execução das polí- subsidiada, dentre outras. Paralelamente, deve ser provi-
ticas sociais públicas; denciado, junto às políticas de habitação e trabalho, e ou-
d) destinação privilegiada de recursos públicos nas tras que o caso indicar, os encaminhamentos necessários
áreas relacionadas com a proteção à infância e à juventude. para alcançar soluções mais definitivas para a situação[12].
Com este intuito, Estatuto da Criança e do Adolescente
concebeu um Sistema de Garantia de Direitos – SGD, cujo Nesta seara continua o art. 101 do Estatuto da Criança
modelo estabelece uma ampla parceria entre o Poder Públi- e Adolescente em seu § 7º
co e a sociedade civil para elaborar e monitorar a execução
de todas as políticas públicas voltadas para o universo da O acolhimento familiar ou institucional ocorrerá no lo-
infância e adolescência. cal mais próximo à residência dos pais ou do responsável
Atualmente, na medida em que os direitos instituídos e, como parte do processo de reintegração familiar, sempre
pelo Estatuto da Criança se consolidam, os investimentos que identificada a necessidade, a família de origem será
de recursos públicos nas áreas da proteção à infância e ju- incluída em programas oficiais de orientação, de apoio e de
ventude tem sido melhor distribuído, porém ainda é neces- promoção social, sendo facilitado e estimulado o contato
sário maior investimento, visto que as demandas técnicas com a criança ou com o adolescente acolhido.
para os serviços têm aumentando gradativamente, na me- Entende-se, portanto que o cuidado em permanecer
dida em que se organiza e profissionaliza. a criança ou adolescente em medida de proteção, acolhi-
Não obstante os avanços no processo de construção mento institucional, perto do local de sua residência além
das políticas públicas, o Brasil ainda precisa avançar mais, de dar continuidade a convivência familiar é principalmen-
especialmente no que diz respeito à municipalização das te para que, no período de acolhimento sejam reparados,
políticas públicas e no que diz respeito à prioridade absolu- através da equipe técnica da instituição junto com a família,
ta em relação às crianças e adolescentes as situações que expõe a criança ao risco e ou a violação
O entendimento que se consolida é de que, o acolhi- de seus direitos.
mento institucional que em outros tempos fora considerado Tal ato se dá através do planejamento individual de
uma solução eficaz para os problemas de crianças e adoles- atendimento, que deverá ser elaborado pela equipe multi-
centes e suas famílias, atualmente é reconhecido como um disciplinar em conjunto com o acolhido e sua família, pro-
dos desencadeadores da violação ao direito de convivência cedimento gradativo que deverá evoluir para o desacolhi-
familiar e ao princípio da prevalência na família. mento da criança e seu retorno a família de origem.
A este respeito dotado de interesse em subsidiar a
PRINCÍPIO DA PREVALÊNCIA NA FAMÍLIA regulamentação, pelo Conselho Nacional dos Direitos da
Criança e do Adolescente – CONANDA e pelo Conselho
O ECA tem o cuidado de estabelecer princípios que re- Nacional de Assistência Social - CNAS, dos serviços de aco-
gem as medidas de proteção, tais princípios são de funda- lhimento para crianças e adolescente orienta a Cartilha:
mental importância para a aplicação das medidas. Tão logo, a criança ou o adolescente seja, encaminhado
para um serviço de acolhimento deve ser iniciado um es-
O presente estudo será baseado no princípio que es- tudo psicossocial para a elaboração de um plano de aten-
tabelece o inciso X do art. 101 do Estatuto, e versa sobre dimento, com vistas à promoção da reintegração familiar.
prevalência da família: Esse planejamento deve envolver de modo participativo
Na promoção de direitos e na proteção da criança e do a família de origem e, sempre que possível, a criança e o
adolescente deve ser dada prevalência às medidas que os adolescente, prevendo encaminhamentos, intervenções e
mantenham ou reintegrem na sua família natural ou extensa procedimentos que possam contribuir para o atendimento
ou, se isto não for possível, que promovam a sua integração das demandas identificadas. Devem ser delineadas medi-
em família substituta; das que contribuam para o fortalecimento da capacidade
Destaca-se, portanto, a premissa de que, quando im- da família para o desempenho do papel de cuidado e pro-
prescindível a medida deve-se atentar que o período de teção, bem como para sua gradativa participação nas ativi-
acolhimento seja o mais breve possível, e que, seja a asse- dades que envolvam a criança e o adolescente.
gurada pela instituição de acolhimento o direito a convi-
vência familiar. A proteção legal de que crianças e adolescentes man-
Sobre este tema orienta a Cartilha de Orientações Téc- tidos sob o cuidado direto de suas famílias é destacado no
nicas de Serviços de Acolhimento para Crianças e Adoles- Capítulo III, do Estatuto da criança e do adolescente, Do
centes do CONANDA. Direito à Convivência Familiar e Comunitária, quando de

99
CONHECIMENTOS ESPECÍFICOS - PSICÓLOGO

forma objetiva expõe o cuidado que deve ser considerado trário, o fator tempo será desencadeador de rompimento
para a manutenção da convivência familiar, destacado nes- severo das relações familiares, o que causaria um dano de
se capitulo o caput do Art. 19: difícil reparação à criança ou adolescente em acolhimento
Toda criança ou adolescente tem direito a ser criado e institucional.
educado no seio da sua família e, excepcionalmente, em
família substituta, assegurada a convivência familiar e co- Síntese
munitária, em ambiente livre da presença de pessoas de-
pendentes de substâncias entorpecentes. Em virtude ao que foi analisado, através de pesqui-
A pesquisa do IPEA, conclui que o afastamento do sas bibliográficas, fica evidenciado a maestria com a que
convívio familiar pode ter repercussão negativa sobre o a Constituição Federal conduz a proteção da criança e do
processo de desenvolvimento da criança e do adolescente, adolescente quando, responsabiliza não só o poder públi-
considerando ainda que a retomada do convívio familiar e co, mas também a família e a sociedade na efetivação de
reintegração a família de origem são processos complexos. tal proteção.
De fato, o seio familiar apresenta-se como o local pró- Através do art. 226, caput, disposto de maneira expres-
prio para o desenvolvimento pessoal em todos os sentidos. sa “a família, base da sociedade, tem especial proteção do
De certo que a família é o centro essencial para o desen- Estado”, e o § 8º, determinado que “o Estado assegurará
volvimento de todo ser humano, fica claro que qualquer a assistência à família na pessoa de cada um dos que a
impedimento para a realização dessa convivência familiar integram, criando mecanismos para coibir a violência no
se caracteriza grave violação ao princípio da dignidade da âmbito de suas relações.
pessoa humana. Faz-se necessário sobretudo a priorização do fortale-
cimento das famílias e que a estruturação da garantia de
A família deixa de ser uma instituição que surge ape- direitos seja baseada na incorporação de outros eixos do
nas do matrimônio e sua função não se limita a econômi- sistema, tais como saúde, moradia e educação, que são di-
ca, mas sua representação passa a ser fundamental para reitos fundamentais do ser humano.
o desenvolvimento da personalidade e dignidade de seus Observada a atenção a família de forma preventiva,
a necessidade de que a medida de proteção, acolhimen-
integrantes através de um fator essencial que é o vínculo
to institucional venha a se concretizar, poderá diminuir de
familiar e dá ao mesmo a sua devida importância.
forma satisfatória, atingindo o objetivo de ser medida ex-
Expõe a doutrina:
cepcional.
Nesse contexto, o resgate e valorização do direito à
As medidas de proteção à criança e adolescentes es-
convivência familiar e comunitária, como direito funda-
tabelecidas no Estatuto da Criança e do Adolescente, são
mental, pressupõe que a família – não apenas na sua con-
abrangentes e conseguem por si só garantir a proteção in-
cepção estritamente jurídica – deve ser vista como local
tegral, favorecendo o vínculo familiar e reunindo a família,
ideal de criação dos filhos, importando, concomitantemen-
o Estado e a sociedade em uma só responsabilidade nesta
te, em uma cruzada pela desinstitucionalização de crianças proteção.
e adolescentes. Fica evidente que, o pré diagnostico a ser realizado por
A partir deste princípio muitas garantias de preserva- entes que promovem encaminhamentos de crianças e ado-
ção de permanência contínua para crianças e adolescentes lescente ao acolhimento institucional, a saber Conselho Tu-
junto aos seus pais foram defendidas pelo ECA, a exemplo telar e CREAS, devem ser procedimento exigido pelo poder
o que dispõe o Art. 12 “Os estabelecimentos de atendi- judiciário, de forma prioritária e imediata, visando coibir a
mento à saúde deverão proporcionar condições para per- banalização da aplicação da medida.
manência em período integral de um dos pais ou respon- Ademais é notório a necessidade de entendimento por
sável, nos casos de internação de crianças e adolescentes. parte dos serviços de acolhimento institucional de que, a
Todas as ações voltadas a valorização da família e de partir do momento em que uma criança está sob sua pro-
sua importância para o desenvolvimento de crianças e teção, é de sua responsabilidade que este tempo seja pro-
adolescentes sob o domínio da proteção integral, irão re- visório e que, ainda tem a criança o direito fundamental a
sultar em garantias à dignidade da pessoa humana. convivência familiar.
E mesmo quando acolhidas em instituições, Estatuto Denota-se que a construção da metodologia de aten-
da Criança e do Adolescente, corrobora com a garantia dimento dos serviços de acolhimento institucional reivin-
do Princípio da Prevalência na Família, quando em seu art. dica profissionais cada vez mais preparados, que dominem
101§ 8º, estabelece: os procedimentos processuais que regem a medida judicial
Verificada a possibilidade de reintegração familiar, o de acolhimento institucional, sem tal conhecimento, o re-
responsável pelo programa de acolhimento familiar ou ins- torno da criança acolhida à sua família é prejudicado.
titucional fará imediata comunicação à autoridade judiciá-
ria, que dará vista ao Ministério Público, pelo prazo de 5 Neste sentido encontram-se novamente as relações de
(cinco) dias, decidindo em igual prazo. dependência de instituições de acolhimento com o poder
Deste modo, a medida de proteção não deverá ser vista público no que se refere a investimento financeiro na área
por nenhum dos sujeitos desta relação jurídica como me- da criança e adolescente, para que possam além de contra-
dida permanente nem mesmo de longo prazo, caso con- tar profissionais, consigam capacita-los de forma contínua.

100
CONHECIMENTOS ESPECÍFICOS - PSICÓLOGO

Conclui-se, portanto, que permanência de crianças e - avaliação do processo onde se conclua pelo total de-
adolescentes em acolhimento institucional por longo pe- sinteresse dos pais em relação aos filhos abrigados, respei-
ríodo fere o princípio da prevalência na família, podendo tados os padrões culturais, a sua capacidade de compreen-
prejudicar todo seu desenvolvimento físico e psicossocial, são e discernimento, bem como a história dos vínculos fa-
causando-lhe um evento danoso de grave ou difícil repa- miliares. (Dal Pizzol e Silva,2001, p 64)
ração. Poderão impetrar com ação de DPF, além do Ministério
Ainda assim, em casos excepcionais onde deverá ser Público, todos que tenham legítimo interesse e, neste caso,
aplicado a medida, deverá ser articulado com a instituição pressupõe-se a obrigatoriedade da representação por ad-
de acolhimento, com a família e com o poder judiciário, vogado.
uma metodologia de preparação para o desacolhimento É comum que pessoas que assumiram de fato a guar-
gradativo, para que o período seja de fato provisório, e que da de uma criança resolvam adotá-la (imediatamente ou
vise acima de tudo o melhor interesse da criança. após algum tempo de convivência) e ingressem com pedi-
do de adoção no Juízo de Infância e
Fonte: Juventude. Isso pressupõe a anuência dos genitores ou
SANTOS, L. D. B. A excepcionalidade e provisoriedade a sua destituição.
do acolhimento institucional nas medidas de proteção à Recebida a petição inicial, o Juiz poderá, ouvindo o
criança. Disponível em: http://lucianaderbe.jusbrasil.com. Ministério Público, se este não for o autor, liminarmente,
br/artigos/213902440/a-excepcionalidade-e-provisorieda- havendo motivo grave, decretar a suspensão do Poder Fa-
de-do-acolhimento-institucional-nas-medidas-de-prote- miliar até julgamento final da causa.
cao-a-crianca Entretanto, se não decretar liminarmente, a suspensão
do Poder Familiar poderá ser decretada no curso do pro-
cesso, pelo mesmo motivo. Os pais serão citados para, em
dez dias, oferecerem resposta escrita, por meio de advo-
A COLOCAÇÃO EM FAMÍLIA gado.
SUBSTITUTA. Caso não contestado o pedido, após a manifestação do
Ministério Público, a autoridade decidirá. Caso contestado,
o Juiz designará audiência de instrução e julgamento.
Em ambas as hipóteses poderão ser realizados estudo
sociais e psicológicos, bem como a oitiva de testemunha(s),
A SUSPENSÃO E A DESTITUIÇÃO
além da produção de outras provas que se fizerem neces-
DO PODER FAMILIAR
sárias (ex: realização de exame de corpo de delito).
Julgado procedente o pedido, o Juiz decretará a sus-
Muitos dos casos que culminam em ação de desti-
pensão ou perda do Poder Familiar, cuja decisão, após o
tuição do poder familiar podem ser iniciados como Pro-
trânsito em julgado - esgotado todos os recursos disponí-
cedimento Verificatório, inclusive, com a participação de
veis -, será averbada à margem do registro de nascimento
assistente social e/ou psicólogo. Nestes estudos poderão
da criança ou adolescente (art. 163 ECA).
já estar evidentes os aspectos que revelem as condições
socioeconômicas, culturais e afetivo-emocionais dos geni- A PERÍCIA SOCIAL E A PERÍCIA PSICOLÓGICA NO
tores e família de origem e as possibilidades de permane- PROCESSO CONTRADITÓRIO: DESTITUIÇÃO DO PO-
cer ou reassumir seu(s) filho(s). DER FAMILIAR
É nesse processo que o Ministério Público encontrará
subsídios para fundamentar o pedido de Destituição ou No processo Contraditório, poderá ou não ocorrer es-
Suspensão do Poder Familiar (DPF). tudos sociais e psicológicos, dependendo de determinação
Dessa forma, o descumprimento dos deveres inerentes judicial. Não obstante, a perícia técnica poderá ser deter-
ao Poder Familiar, como sustento, guarda, educação, prote- minada de ofício pelo Juiz ou requerida pelas partes ou
ção e assistência moral poderão implicar em suspensão ou Ministério Público (art. 167 ECA).
destituição do poder familiar. Na prática do Tribunal de Justiça de São Paulo, a perí-
Segundo Dal Pizzol e Silva (2001), muito embora seja cia é realizada pelo Serviço Social e Psicologia, concomi-
difícil identificar critérios para que a DPF seja feita, ele le- tantemente ou não. Este estudo será consubstanciado em
vantou algumas incidências: um laudo técnico, contendo subsídios para a decisão da
- pais com problemas psiquiátricos graves, sem supor- medida.
te familiar e comunitário (...); Na realização do estudo, os profissionais deverão levar
- pais com dependência ao álcool e outras drogas, com em conta os princípios do ECA, dispostos nos artigos 19,
reiteradas tentativas de tratamento, que não superam os 22 e 23, quanto ao direito da criança e do adolescente de
problemas por assumirem uma atitude de resistência total ser criado em sua família e, excepcionalmente, em família
a mudança; substituta.
- ausência de contato ou visita a criança cumulada com Cabe ressaltar que a elaboração de uma perícia social
uma das situações anteriormente mencionadas, por perío- em relação à DPF irá exigir do profissional uma redobrada
do superior a seis meses; atenção para que ele se atenha ao seu papel, sem deixar-

101
CONHECIMENTOS ESPECÍFICOS - PSICÓLOGO

-se levar por ideias pré-concebidas que o impeça de apro- A perícia psicológica terá o objetivo de avaliar sobre:
ximar-se da complexidade dos aspectos envoltos naquela • O grau de incapacidade para o exercício das funções
realidade. parentais;
Como Fávero (2000) aponta: • A motivação que determina a decisão de ser desti-
tuído (nos casos em que os pais estão decididos a serem
Existe um consenso de que a destituição do poder destituídos), se os motivos prendem-se a fatores externos,
familiar tem sua importância e função se aplicada como situacionais, não controláveis pelo indivíduo, ou motivação
medida excepcional e se tiver o propósito de um melhor interna (rejeição ao filho, certeza de não poder dar-lhe o
encaminhamento para a vida da criança – e ai é preciso amor, perspectivas de vida onde não entre o filho, convic-
pensar essa medida em relação à idéia de família substi- ção de que ele será mais feliz em outro lar);
tuta(...)Em se tratando de destituição do pátrio poder e • Firmeza da decisão tomada;
adoção, os profissionais, como agentes do Judiciário, não • Compreensão do significado da medida;
têm o papel de “salvadores” de uma criança da situação de • Condições psicoafetivas dos pais, identificando a exis-
pobreza familiar ( e suas decorrências).Essa família sempre tência de pontos favoráveis ou não à mudança, mediante
estará na história e na vida da criança, uma história que a algum nível de intervenção (psicoterapia, orientação/acon-
ela pertence, independentemente do Poder Judiciário ou selhamento, tratamento especializado - casos de alcoolis-
dos pais adotivos e da “nova vida” que passa a ter junto mo, drogadição e outros);
deles. (Fávero,2000, p.107-8) • Identificação de conflitos psíquicos, doenças mentais
Evidencia-se que as mães ou famílias que acabam por ou físicas, avaliando-se os riscos e as consequências psi-
ser destituídas do poder familiar apresentam uma situa- cológicas para criança/adolescente conviverem com pais
ção de desamparo e abandono, com conflitos diversos, portadores dessa problemática, considerando-se também
sem apoio familiar ou da rede social de serviços. A falta de a extensão de danos ocorridos à criança (casos de maus
condições socioeconômicas está quase sempre presente e, tratos, abuso sexual, etc.);
junta o desemprego, por muitas vezes o alcoolismo, a dro- • Condições para assumir ou não o papel materno/ pa-
gadição, a doença física e mental. terno;
Há uma incidência de casos de mulheres sozinhas, cuja • Existência e qualidade do vínculo com a criança/ado-
gestação pode ser fruto de um relacionamento passageiro lescente;
• Pesquisar dados de anamnese sobre a gravidez, par-
ou separadas devido a fatores que passam pela violência
to, histórico de vida da criança no sentido de se proceder
intrafamiliar, perdas e problemas com a manutenção e so-
às orientações e esclarecimentos necessários à possível fa-
brevivência da família de diversas ordens.
mília substituta, caso esta venha a ser colocada em adoção.
É necessário avaliar a situação como um todo, em seu
Na perícia social serão abordados os seguintes aspec-
contexto geral, não se atendo unicamente à análise do fato
tos:
que está possibilitando que a medida de Destituição do
• História da família, identificando suas crises e fatos
Poder Familiar seja determinada.
significativos; Para a realização de seu estudo, o psicólogo poderá
• Estrutura da família, papéis e padrões de funciona- fazer uso de entrevistas, aplicação de testes, observações
mento, quem é este pai e esta mãe; lúdicas de conduta, estudo por meio de visitas domicilia-
• Existência e qualidade das interrelações do núcleo res ou institucionais; aplicação de técnicas de orientação e
familiar; aconselhamento.
• Conduta dos pais em relação ao trabalho, convivência Por vezes, alguns casos necessitam de exames comple-
comunitária, participação em atos delitivos, etc.; mentares. Caso seja necessário, pode ser solicitado que o
• Presença de outros adultos significativos (avós, tios, Juiz oficie a algum órgão especializado, público ou privado,
compadres, vizinhos), que possam proporcionar apoio a fa- para estas avaliações (IMESC, IML, APAE, Ambulatório de
mília em crise; Saúde Mental do Estado).
• Recursos da rede de serviços (creches, escolas, centro
sociais), disponíveis e ou utilizados pelo grupo familiar; A CRIANÇA QUE SERÁ COLOCADA EM FAMÍLIA
• Receptividade da família para receber ajuda orienta- SUBSTITUTA: O TRABALHO NECESSÁRIO A SER DE-
ção e cumprimento de medidas determinadas pela autori- SENCADEADO
dade judiciária;
• Conhecer sobre a história da gestação do filho espe- Todas as crianças colocadas em adoção possuem um
rado ou não e se contou com apoio e de quem; histórico que é o nosso objeto estudo. Ela pode ou não ter
• Expectativas da família em relação à criança ou ado- passado por privações de diferentes naturezas, vivenciado
lescente, o motivo da entrega do filho e avaliar se tem pos- situações de violência, negligência, abandono e, caso seja
sibilidade ou não de permanecer com a criança. recém-nascida, muitas vezes, a uma história de rejeição,
• Esclarecer quanto às consequências da destituição do tentativa de aborto e sofrimento da mãe.
poder familiar; Pode se tratar de apenas uma criança ou de um grupo
• Encaminhar para a rede de serviços do município, de irmãos de diversas idades, o que passa a ser mais um
visando o bem-estar físico e mental de quem está sendo componente para análise e que poderá implicar maiores
destituído do poder familiar dificuldades na colocação em família substituta.

102
CONHECIMENTOS ESPECÍFICOS - PSICÓLOGO

Em muitos casos, ela poderá, antes que sua situação ração com profissionais do abrigo e professores e de outras
tenha sido definida judicialmente, ter vivido em diversas instituições, caso freqüente;
famílias ou, principalmente, ter sido institucionalizada. • No caso de se tratar de grupo de irmãos, como as
Neste caso de abrigamento, impõe-se a necessidade questões acima se dão individualmente;
de se buscar articular um trabalho entre os profissionais • Qual a relação entre irmãos;
da VIJ e os técnicos do abrigo, visando identificar estraté- • Quais as expectativas em relação à família substituta.
gias para abordar com as crianças e adolescentes as ques-
tões referentes ao não retorno para família de origem, bem Esta etapa requer uma delicadeza e preparo técnico,
como da possibilidade de ser adotado. pois a família que temos ou desejamos ter são objetos de
São os técnicos do abrigo, principalmente, que aco- idealizações e depositária de sentimentos positivos e ne-
lhem dúvidas e angústias da criança ou adolescente de gativos.
que não mais haverá retorno para a família de origem e é É clara a necessidade da intervenção técnica, por
comum, nessa situação, que a criança ou adolescente apre- meio de entrevistas interprofissionais, visitas no abrigo,
sentem diferentes manifestações como o contar e recontar observação lúdica e aplicação de testes para conhecermos
a sua própria história. a estrutura psíquica da criança e a capacidade de reatar
Estes profissionais devem respeitar o tempo do luto novos vínculos afetivos, identificando como ela vivenciou
por esse rompimento, oferecendo um espaço de escuta sua história de abandono e perdas, o tipo de vínculo – real
verdadeira e sensível diante das questões que surgem em ou idealizado - que estabeleceu com as figuras parentais
torno da espera por essa nova família. e o quanto está disponível afetivamente para uma nova
O diálogo fundado na verdade e no respeito ao papel relação afetiva com seus pais adotivos.
ativo da criança/adolescente em sua história pode ser rea- Feitas às considerações acima é importante dizer que
lizado pelos profissionais que convivem com ele no abrigo. os Setores Técnicos só poderão iniciar a fase efetivamente
Mas os assistentes sociais e psicólogos do Judiciário do preparo da criança ou adolescente quando o juiz enca-
não podem se omitir desse preparo, tendo em vista seu minhar o processo com a determinação para verificar pre-
papel fundamental em todo o processo da adoção. tendente à adoção.
Deste modo, a organização do tempo e do espaço para Concomitantemente, inicia-se o trabalho entre os pro-
os procedimentos da preparação para a adoção deve con- fissionais da VIJ e do abrigo, visando o preparo da criança
templar esses princípios, respeitando-se o tempo da crian- ou adolescente para sua colocação em família substituta,
ça/adolescente e também a brevidade necessária. ao mesmo tempo, é realizado consulta no Cadastro de Pre-
A colocação em uma nova família apontará para a ne- tendentes à Adoção (CPA) para reconhecer se há pessoas
cessidade de conhecer que criança é essa, em seus aspec- que atendem às necessidades das crianças em questão.
tos singulares e particulares. Portanto, é de fundamental Ademais, um trabalho conjunto pode sensibilizar o
importância a aproximação da realidade da criança para re- abrigo a não fazer aproximações de pretendentes sem pré-
conhecer sua(s) necessidade(s) a fim de cumprir a premissa via discussão com os técnicos do judiciário, bem como o
de se localizar a família adequada. Toda essa fase supõe a esclarecimento junto a eles quanto ao funcionamento do
interlocução permanente entre os profissionais da VIJ e do Cadastro de Pretendentes à Adoção.
Abrigo.
Não se pode esquecer que a criança ou adolescente,
independentemente do tempo que possa ter permanecido
no abrigo, pode ter estabelecido vínculos com os profissio-
nais e companheiros do abrigo, o que deve ser considera-
do durante a sua preparação para sua colocação em família
substituta.
Este parece ser outro aspecto importante que reforça a
necessidade de estabelecer a parceria do Judiciário com os
profissionais que estão convivendo com a criança/adoles-
cente no abrigo, visando uma intervenção fundamentada
na sua realidade, além de contribuir em muito o seu prepa-
ro para futura adoção.
Assim há pontos importantes a serem abordados em
uma avaliação com a criança ou adolescente e grupo de
irmãos:

O Serviço Social deverá verificar entre outros aspectos:


• Como está para a criança a compreensão de sua his-
tória de vida e de sua família de origem;
• Como vem lidando com a situação atual;
• Como estabelece seus relacionamentos interpessoais
com o grupo do abrigo, outros colegas da escola, sua inte-

103
CONHECIMENTOS ESPECÍFICOS - PSICÓLOGO

104
CONHECIMENTOS ESPECÍFICOS - PSICÓLOGO

OS PASSOS PARA LOCALIZAR FAMÍLIA SUBSTITUTA Bibliografia


POR MEIO DO CADASTRO DE PRETENDENTES À ADO- BRASIL. Atuação dos profissionais de Serviço Social e
ÇÃO (CPA) Psicologia. Manual de procedimentos técnicos Tribunal de
Justiça do Estado de São Paulo. v.i. Infância e Juventude
A adoção é prerrogativa do Judiciário e quando há de-
terminação judicial para que uma criança seja colocada em
família substituta por meio do instituto da adoção.
O processo deve vir para as Seções Técnicas que, após AS MEDIDAS SOCIOEDUCATIVAS.
terem detalhes sobre a criança, consultam o Livro de Regis-
tros dos Pretendentes à Adoção – que deve estar na guarda
das Seções Técnicas (conforme Cap. XI NCGJ).
Cada Juízo da Infância e Juventude é responsável em LEI Nº 8.069, DE 13 DE JULHO DE 1990
manter e atualizar Cadastro de Pretendentes à Adoção, da
área territorial de sua jurisdição, o qual trataremos de modo Capítulo IV
mais detalhado adiante. Das Medidas Socioeducativas
Caberá seguir a ordem de inscrição dos pretendentes. Seção I
Disposições Gerais
Entretanto, a atenção central efetiva-se em buscar uma famí-
lia que atenda às necessidades da criança/adolescente ou do
Art. 112. Verificada a prática de ato infracional, a au-
grupo de irmãos.
toridade competente poderá aplicar ao adolescente as se-
Na impossibilidade de localizar no cadastro da própria
guintes medidas:
localidade, a critério do juiz, poderá ser consultado cadas- I - advertência;
tro das comarcas vizinhas, e, no caso de se efetivar a guarda II - obrigação de reparar o dano;
com vistas à adoção, o juízo deverá seguir a rotina de praxe, III - prestação de serviços à comunidade;
comunicando a comarca oriunda, assim como oficiar a CEJAI IV - liberdade assistida;
Comissão Estadual Judiciária Adoção Internacional. V - inserção em regime de semiliberdade;
Caso não seja localizado pretendente à adoção na região, VI - internação em estabelecimento educacional;
os profissionais deverão informar no processo as tentativas VII - qualquer uma das previstas no art. 101, I a VI.
empregadas na busca e, inclusive, se chegaram a entrar em § 1º A medida aplicada ao adolescente levará em
contato com algum pretendente, informar o nº de inscrição, conta a sua capacidade de cumpri-la, as circunstâncias e a
seu processo de CPA, o motivo de sua recusa ou outra razão. gravidade da infração.
Neste caso, o assistente social e/ou psicólogo poderão suge- § 2º Em hipótese alguma e sob pretexto algum, será
rir que o juiz solicite pesquisa no Cadastro Estadual (CEJAI). admitida a prestação de trabalho forçado.
A CEJAI encaminhará uma listagem e poderá ser frutífero § 3º Os adolescentes portadores de doença ou
que os assistentes sociais e psicólogos da Vara entrem em deficiência mental receberão tratamento individual e
contato telefônico com as Varas responsáveis pela habilita- especializado, em local adequado às suas condições.
ção dos pretendentes no sentido de conhecer e discutir um
pouco o caso com aqueles que atuaram na sua habilitação, Art. 113. Aplica-se a este Capítulo o disposto nos arts.
obtendo aprofundamento dos dados sobre eles, inclusive se 99 e 100.
já houve aproximações com outra criança, dificuldades, supe-
rações, detalhes, etc. Art. 114. A imposição das medidas previstas nos incisos
Cabe ressaltar que, após está consulta, será importante II a VI do art. 112 pressupõe a existência de provas suficien-
que os elementos colhidos sejam objeto de discussão inter- tes da autoria e da materialidade da infração, ressalvada a
disciplinar com vistas a buscar identificar quem melhor po- hipótese de remissão, nos termos do art. 127.
derá atender às necessidades específicas de cada criança/ Parágrafo único. A advertência poderá ser aplicada
sempre que houver prova da materialidade e indícios su-
adolescente ou do grupo de irmãos que estiver em questão.
ficientes da autoria.
Essa discussão deverá resultar em um relatório, o qual
será incluído no processo, de forma a levar a conhecimento à
Seção II
autoridade judiciária.
Da Advertência
Não encontrando pretendente na listagem encaminhada
poderá ser solicitada outra listagem a CEJAI, que enviará a Art. 115. A advertência consistirá em admoestação ver-
sequência dos inscritos até esgotados todos aqueles do Es- bal, que será reduzida a termo e assinada.
tado que atendam as características da criança/adolescente.
No caso de não ser possível localizar pretendente inscrito
residente no estado de São Paulo na CEJAI, poderá o juiz do
feito requerer à CEJAI o envio de pretendentes internacio-
nais habilitados a adotar a criança ou o grupo de irmãos em
questão.

105
CONHECIMENTOS ESPECÍFICOS - PSICÓLOGO

Seção III Seção VI


Da Obrigação de Reparar o Dano Do Regime de Semiliberdade

Art. 116. Em se tratando de ato infracional com reflexos Art. 120. O regime de semiliberdade pode ser deter-
patrimoniais, a autoridade poderá determinar, se for o caso, minado desde o início, ou como forma de transição para o
que o adolescente restitua a coisa, promova o ressarcimen- meio aberto, possibilitada a realização de atividades exter-
to do dano, ou, por outra forma, compense o prejuízo da nas, independentemente de autorização judicial.
vítima. § 1º São obrigatórias a escolarização e a profissionalização,
Parágrafo único. Havendo manifesta impossibilidade, a devendo, sempre que possível, ser utilizados os recursos
medida poderá ser substituída por outra adequada. existentes na comunidade.
§ 2º A medida não comporta prazo determinado
Seção IV aplicando-se, no que couber, as disposições relativas à
Da Prestação de Serviços à Comunidade internação.

Art. 117. A prestação de serviços comunitários con- Seção VII


siste na realização de tarefas gratuitas de interesse geral, Da Internação
por período não excedente a seis meses, junto a entidades
assistenciais, hospitais, escolas e outros estabelecimentos Art. 121. A internação constitui medida privativa da li-
congêneres, bem como em programas comunitários ou berdade, sujeita aos princípios de brevidade, excepcionali-
governamentais. dade e respeito à condição peculiar de pessoa em desen-
Parágrafo único. As tarefas serão atribuídas conforme volvimento.
as aptidões do adolescente, devendo ser cumpridas du- § 1º Será permitida a realização de atividades externas,
rante jornada máxima de oito horas semanais, aos sába- a critério da equipe técnica da entidade, salvo expressa
dos, domingos e feriados ou em dias úteis, de modo a não determinação judicial em contrário.
prejudicar a frequência à escola ou à jornada normal de § 2º A medida não comporta prazo determinado,
trabalho. devendo sua manutenção ser reavaliada, mediante decisão
fundamentada, no máximo a cada seis meses.
§ 3º Em nenhuma hipótese o período máximo de
Seção V internação excederá a três anos.
Da Liberdade Assistida § 4º Atingido o limite estabelecido no parágrafo anterior,
o adolescente deverá ser liberado, colocado em regime de
Art. 118. A liberdade assistida será adotada sempre que semiliberdade ou de liberdade assistida.
se afigurar a medida mais adequada para o fim de acompa- § 5º A liberação será compulsória aos vinte e um anos
nhar, auxiliar e orientar o adolescente. de idade.
§ 1º A autoridade designará pessoa capacitada para § 6º Em qualquer hipótese a desinternação será precedi-
acompanhar o caso, a qual poderá ser recomendada por da de autorização judicial, ouvido o Ministério Público.
entidade ou programa de atendimento. § 7º A determinação judicial mencionada no § 1o poderá
§ 2º A liberdade assistida será fixada pelo prazo mínimo ser revista a qualquer tempo pela autoridade judiciária.
de seis meses, podendo a qualquer tempo ser prorrogada, (Incluído pela Lei nº 12.594, de 2012) (Vide)
revogada ou substituída por outra medida, ouvido o
orientador, o Ministério Público e o defensor. Art. 122. A medida de internação só poderá ser aplicada
quando:
Art. 119. Incumbe ao orientador, com o apoio e a su- I - tratar-se de ato infracional cometido mediante grave
pervisão da autoridade competente, a realização dos se- ameaça ou violência a pessoa;
guintes encargos, entre outros: II - por reiteração no cometimento de outras infrações
I - promover socialmente o adolescente e sua família, graves;
fornecendo-lhes orientação e inserindo-os, se necessário, III - por descumprimento reiterado e injustificável da
em programa oficial ou comunitário de auxílio e assistência medida anteriormente imposta.
social; § 1º O prazo de internação na hipótese do inciso III deste
II - supervisionar a frequência e o aproveitamento es- artigo não poderá ser superior a 3 (três) meses, devendo
colar do adolescente, promovendo, inclusive, sua matrícula; ser decretada judicialmente após o devido processo legal.
III - diligenciar no sentido da profissionalização do (Redação dada pela Lei nº 12.594, de 2012) (Vide)
adolescente e de sua inserção no mercado de trabalho; § 2º. Em nenhuma hipótese será aplicada a internação,
IV - apresentar relatório do caso. havendo outra medida adequada.

Art. 123. A internação deverá ser cumprida em entidade


exclusiva para adolescentes, em local distinto daquele desti-
nado ao abrigo, obedecida rigorosa separação por critérios
de idade, compleição física e gravidade da infração.

106
CONHECIMENTOS ESPECÍFICOS - PSICÓLOGO

Parágrafo único. Durante o período de internação, inclu- o Conselho Federal de Psicologia abre espaço em sua
sive provisória, serão obrigatórias atividades pedagógicas. respeitável revista Psicologia: Ciência e Profissão para
expor percursos profissionais variados em sua edição
Art. 124. São direitos do adolescente privado de liberda- histórica, que comemora meio século da regulamentação
de, entre outros, os seguintes: da profissão no Brasil.
I - entrevistar-se pessoalmente com o representante do Escrever sobre a nossa trajetória remete-nos a refle-
Ministério Público; xões fundamentais e à expectativa de poder incentivar e
II - peticionar diretamente a qualquer autoridade; disseminar o conhecimento a outros profissionais sobre a
III - avistar-se reservadamente com seu defensor; importância da Psicologia no âmbito das Varas de Família.
IV - ser informado de sua situação processual, sempre Na virada do século e já na segunda década do novo
que solicitada; milênio, olhamos com respeito para a nossa história, para
V - ser tratado com respeito e dignidade; uma profissão felizmente com muitas lutas passadas para
VI - permanecer internado na mesma localidade ou recordar-se. Na área da Psicologia jurídica, tivemos, em
naquela mais próxima ao domicílio de seus pais ou res- 1955, a publicação da obra de Emilio Mira y Lopes que já
ponsável; havia sido editada na Europa (1932) e que foi intitulada
VII - receber visitas, ao menos, semanalmente; Manual de Psicologia Jurídica. A obra defendia o teor cien-
VIII - corresponder-se com seus familiares e amigos; tífico da Psicologia e a aplicabilidade de seus conhecimen-
IX - ter acesso aos objetos necessários à higiene e as- tos e instrumentos, e, além disso, valorizava uma Psicologia
seio pessoal; aplicada às instituições jurídicas.
X - habitar alojamento em condições adequadas de hi- Discorrer sobre o tipo de atuação possível de ser de-
giene e salubridade; senvolvida junto às famílias em conflito seria um terreno
XI - receber escolarização e profissionalização; mais usual, debatido e extremamente organizado pelas
XII - realizar atividades culturais, esportivas e de lazer: várias correntes teóricas do conhecimento. Diferente dis-
XIII - ter acesso aos meios de comunicação social; so, nosso propósito é apresentar a atuação do psicólogo
XIV - receber assistência religiosa, segundo a sua cren- nas Varas de Família, trazendo pontos históricos e também
ça, e desde que assim o deseje; elementos atuais que já apontam demandas profissionais
XV - manter a posse de seus objetos pessoais e dispor – específicas e avançadas – em relação às famílias que ju-
de local seguro para guardá-los, recebendo comprovante dicializam seus conflitos, ou seja, que levam ao Poder Ju-
daqueles porventura depositados em poder da entidade; diciário ou a um operador de Direito propriamente dito o
XVI - receber, quando de sua desinternação, os docu- pedido de auxílio e de soluções.
mentos pessoais indispensáveis à vida em sociedade. É sabido que o sistema de Justiça não consegue
§ 1º Em nenhum caso haverá incomunicabilidade. fazer frente a tudo que lhe chega, seja na quantidade de
§ 2º A autoridade judiciária poderá suspender casos, seja na complexidade dos assuntos. Nesse sentido,
temporariamente a visita, inclusive de pais ou responsável, passa a realizar cada vez mais seus questionamentos aos
se existirem motivos sérios e fundados de sua prejudiciali- profissionais de diferentes áreas, tais como o psicólogo,
dade aos interesses do adolescente. para que lhe apresentem contribuições que possam ir
“além do que o cidadão comum pode intuitivamente
Art. 125. É dever do Estado zelar pela integridade físi- compreender ou explicar (Fonseca, Matos, & Simões, 2008,
ca e mental dos internos, cabendo-lhe adotar as medidas p.3).
adequadas de contenção e segurança. A participação da Psicologia na discussão dos inúme-
ros conflitos que chegam à Justiça foi impondo-se e mos-
trando suas possibilidades de contribuição para edificar um
campo de saber diferenciado, que hoje recebe a denomi-
O PSICÓLOGO NO ATENDIMENTO AOS CASOS nação de Psicologia jurídica, judiciária ou forense (não nos
NAS VARAS DA INFÂNCIA E DA JUVENTUDE, dedicaremos aqui a traçar as nuances que definem cada
NAS VARAS DA FAMÍLIA E DAS SUCESSÕES, tipo de nomenclatura, pois, para o nosso propósito, isso
VIOLÊNCIA DOMÉSTICA E AO IDOSO. não se faz importante).
A enormidade de problemas atrelados às áreas da
infância, juventude, família e idoso, da violência contra a
criança e a mulher, do doente mental que delinque, da rea-
PSICOLOGIA JURÍDICA: UM PERCURSO NAS VA- bilitação do detento, da bioética e tantos outros, buscam
RAS DE FAMÍLIA DO TRIBUNAL DE JUSTIÇA DO ESTA- o sistema de Justiça para a resolução ou o auxílio em suas
DO DE SÃO PAULO dificuldades, e necessitam de uma compreensão não pau-
tada apenas nas luzes da letra fria da lei. O prisma subjetivo
Evani Zambon Marques da Silva edifica-se e impõese por detrás desses inúmeros proces-
sos que recebem uma capa, um número e que encerram
É com satisfação que escrevemos sobre uma área o drama da vida de cada um. Indo além, podemos pensar
de atuação específica da Psicologia que nos absorve há também na riqueza que o Judiciário nos dá, nomeando as
mais de duas décadas, especialmente na ocasião em que ações que ali adentram como processos.

107
CONHECIMENTOS ESPECÍFICOS - PSICÓLOGO

(...) ninguém é dono de verdades absolutas a respeito se tornou referência para o restante do País. Foram criados
do Homem, se é que seja este suscetível de verdades ab- 65 cargos efetivos e mais 16 cargos de chefia, o que consti-
solutas. De modo que tentar compreendê-lo em estado de tuiu um marco para o serviço público da época. Logo após
sofrimento, como costuma apresentar-se aos profissionais a distribuição dos psicólogos aprovados para a atuação
de Direito, nos conflitos que lhe vem da inserção familiar, na área da infância e juventude, legitima-se o espaço para
é tarefa árdua e, para usar de paradoxo, quase desumana, a atuação nas Varas de Família, sendo que o Fórum João
porque supõe não apenas delicadeza de espírito e dispo- Mendes, de São Paulo (capital), sediou a primeira equipe
sição de ânimo, mas preparação intelectual e técnica tão de profissionais especializados na área.
vasta e apurada que já não entra no cabedal pretensioso Por força da Lei Federal n° 8069/90, que implantou o
dalgum jurista solitário. (Peluso, 1997, p. 7) Estatuto da Criança e do Adolescente, as chamadas equi-
A história da Psicologia dentro dos Tribunais brasileiros pes técnicas ou interprofissionais (artigos nos 150 e 151)
já foi apresentada por muitos estudiosos, dos quais desta- foram qualificando-se, integrando-se e não só sendo acei-
camos Rovinski (2009), Lago et al. (2009), Bernardi (2005) tas como tornando-se obrigatórias em todo o Brasil. Nesse
e Martins (1999), só para citarmos alguns. O nosso recorte sentido, citamos aqui a Recomendação nº 2, do Conselho
será a Psicologia desenvolvida dentro do Tribunal de Jus- Nacional de Justiça, que faz alusão a esse fato e que será
tiça paulista, que foi edificando-se juntamente ao próprio explicada mais adiante.
desenvolvimento da profissão no País. A prática no Tribunal paulista indica-nos que a proxi-
No início da década de 70 (século XX), já tivemos no- midade dos juízes e outros operadores do Direito com os
tícias de juízes e de desembargadores que se valiam de profissionais da Psicologia, ainda em sua maioria atuantes
avaliações psicológicas em algum momento do processo; nas Varas de Infância e Juventude, foi paulatinamente dis-
eram profissionais que consideramos verdadeiramente à seminando o valor da ciência psicológica, e suas conclu-
frente de seu tempo. Dentre eles, destacamos Antonio Luis sões acabaram valorizadas em diversos territórios jurídicos.
Chaves Camargo, falecido em 2007, que começou a propor No interior paulista, é comum ainda os profissionais esta-
em 1980 a participação de psicólogos voluntários nas au- rem lotados nas Varas de Infância e Juventude, mas presta-
diências, criando assim, à época, as chamadas audiências rem serviços às Varas de Família ou Cíveis. Na capital, nos
interprofissionais (Camargo, 1982), que ocorriam nas então Fóruns regionais, poucos são os setores técnicos que estão
denominadas Varas de Menores, hoje Varas de Infância e individualizados e que prestam serviços diferenciados en-
Juventude. tre os assuntos das Varas de Infância e Juventude e os das
Varas da Família. Normalmente, segundo estudos desen-
As audiências interprofissionais representavam uma volvidos pela Associação dos Assistentes Sociais e Psicó-
forma de atendimento dos casos, em juízo, através da pré- logos do Tribunal de Justiça de São Paulo (AASPTJ-SP), os
via apuração da equipe técnica, composta por assistente profissionais acumulam as duas áreas.
social e psicólogo. Tais profissionais tinham como dever A união e os esforços dos psicólogos que atuam nos
não só apresentar as medidas cabíveis dentro de suas res- Tribunais de todo o País foram reconhecidos. Os profissio-
pectivas áreas mas também confeccionar relatórios circuns- nais se organizaram como categoria e criaram associações
tanciados visando à decisão do processo (Camargo, 1982). regionais, das quais destacamos a Associação dos Assis-
A profissão de psicólogo foi regulamentada no Brasil tentes Sociais e Psicólogos do Tribunal de Justiça de São
pela Lei nº 4.112, de 27 de agosto de 1962, e, desde então, Paulo (AASPTJSP), que possui a finalidade de defender os
o Decreto nº 53.464, de 1964, em seu artigo 4º, prevê que interesses gerais e as reivindicações desses profissionais e
cabe ao psicólogo realizar perícias e emitir pareceres sobre de promover o aprimoramento técnico-profissional e cul-
a matéria de Psicologia”. O primeiro Código de Ética da tural de seus associados. A referida associação paulista foi
profissão, que surgiu em 1987, 18 anos após a criação dos criada em 1992, e conta atualmente com sede própria e
Conselhos de Psicologia (Pereira, 1991), mencionava um com aproximadamente 1.230 associados entre psicólogos
item específico para discorrer sobre as relações do psicó- e assistentes sociais atuantes e aposentados dos quadros
logo com a Justiça. Já o atual Código é datado de 2005, e técnicos do Tribunal de Justiça de São Paulo.
também traz artigos elencando questões de sigilo, perícia e Na área acadêmica, os psicólogos jurídicos também
prestação de esclarecimentos em juízo, dentre outros, que demarcam firmemente sua presença participando de en-
balizam a atividade cotidiana. contros, congressos e simpósios nacionais e internacionais.
Diversos outros documentos de respaldo profissional Destacamos eventos que desde 1987 são realizados em
foram surgindo junto aos Conselhos Regionais e Conselho São Paulo (I Encontro de Assistentes Sociais e Psicólogos
Federal de Psicologia, tais como resoluções, cadernos te- do Poder Judiciário) e em diversos Estados brasileiros, tais
máticos e manual de referência (os quais serão menciona- como Minas Gerais, Bahia, Rio Grande do Sul, Santa Catari-
dos mais adiante). Tais documentos mostram a efervescên- na e Rio de Janeiro, só para citarmos alguns. No âmbito in-
cia da área e a necessidade de normatizações específicas ternacional, foi realizado, em 2012, o VII Congresso Iberoa-
que visem, dentre outras coisas, à orientação ao psicólogo mericano de Psicologia Jurídica, no Equador, evidenciando
que atua com questões atinentes às Varas de Família. ampla participação do psicólogo nas questões da Justiça,
A partir da entrada dos psicólogos no Tribunal de Jus- seja no volume de trabalhos e conferências apresentadas,
tiça de São Paulo, na década de 80, e com o primeiro con- seja na projeção e na apresentação de discussões e pro-
curso público, em 1985, os profissionais foram ocupando postas cada vez mais atinentes a um contexto de crescente
espaços e galgando posições na instituição judiciária que complexidade.

108
CONHECIMENTOS ESPECÍFICOS - PSICÓLOGO

Apesar de a disciplina Psicologia Jurídica (ou forense Na tentativa de dar suprimento à formação profissional
ou judiciária) já fazer parte de algumas grades curricula- ou de levar conhecimentos atuais para aqueles que assim o
res de cursos superiores de Psicologia em todo o País, ela desejam, diversos cursos estão sendo oferecidos em todo
ainda não integra obrigatoriamente o currículo dos futuros o País. Em São Paulo, destacamos o proposto pelo Instituto
psicólogos. Muitas instituições oferecem disciplinas que Sedes Sapientiae, que, embora não seja credenciado pela
tangenciam o assunto da Psicologia jurídica, mas que ainda ABEP (Associação de Ensino em Psicologia) como espe-
deixam um grande espaço para que os alunos conheçam cialização, possui, em seu quadro de docentes, psicólogos
e ampliem sua formação, inclusive sobre cidadania, ética pioneiros nas diferentes áreas de atuação da Psicologia ju-
e assuntos específicos que atravessam a subjetividade dos rídica.
indivíduos no mundo atual. Na área de pós-graduação strictu sensu, também inú-
Na experiência dos profissionais pioneiros do Judi- meros trabalhos estão sendo apresentados ao longo das
ciário paulista, dentre os quais nos incluímos, a formação últimas décadas, em uma tentativa de sistematizar o co-
curricular nada trazia sobre a possibilidade de a Psicolo- nhecimento, que, ao menos em se tratando do Estado de
gia ser aplicada aos tribunais. Assim, apesar de matérias São Paulo, teve seu início de forma prática e pouco emba-
que falavam de instituições, psicodiagnóstico, técnicas de sada teoricamente, já que não havia um corpo de conhe-
avaliações psicométricas diversas, teorias de abordagem e cimentos sistematizado na área. Nossos atuais Mestres e
conhecimento das pessoas e de seus problemas, desenvol- Doutores são profissionais que, se não atuam diretamente
vimento infantil e adolescente, ética e estatística, só para na área, debruçam-se a pesquisar a Psicologia jurídica em
citarmos algumas, nada havia especificamente para nortear profundidade, redigindo dissertações e teses de alto nível e
o profissional que fosse atuar em tribunais. Perguntas eram com grande legitimação pela comunidade acadêmica.
formuladas: Como responder à demanda? Como organizar Paralelamente a esse avanço, em 2004, tivemos a cria-
uma avaliação com fins periciais? Como lidar com o sigilo? ção do Conselho Nacional de Justiça (CNJ). O órgão, insta-
Como discutir o caso com um operador do Direito? Quais lado em 2005, foi criado em obediência ao determinado na
os limites da atuação que deveriam ser fornecidos? Como Constituição Federal, nos termos do art. 103-B, e é voltado
redigir e comunicar o trabalho? Quem é o nosso cliente? E para a reformulação de quadros e meios no Judiciário, so-
assim por diante.
bretudo no que diz respeito ao controle e à transparência
Ainda hoje padecemos com essas dificuldades, ou seja,
administrativa e processual. Suas ações são voltadas para
profissionais que vão a campo sem o conhecimento das
o planejamento, a coordenação, o controle administrativo
ferramentas adequadas ou propícias para a atuação junto
e o aperfeiçoamento do serviço público na prestação da
aos operadores do Direito. Tal postura causa uma exposi-
Justiça.
ção negativa do psicólogo, já que, na atualidade, diga-se
O Conselho Nacional de Justiça vem editando resolu-
de passagem, ninguém está livre de, ainda que não tenha
ções que paulatinamente colaboram para o ajustamento
optado por trabalhar no sistema da Justiça, ter seu trabalho
solicitado para ilustrar um processo judicial qualquer, para dos psicólogos nas áreas da Justiça. Assim, podemos desta-
participar de uma audiência e outras entradas comuns. car a Resolução nº 75, que propõe, dentre outras matérias,
Para nós, a Psicologia jurídica está espalhada em qualquer a Psicologia Judiciária, como parte da formação importante
âmbito, sendo seu conhecimento imperioso para aqueles e necessária para o ingresso às carreiras da magistratura.
profissionais da clínica, da escola, dos hospitais, das em- A Resolução nº 75 menciona o que especificamente
presas. deve ser ensinado ao futuro magistrado (estadual, federal,
Em complementação ao nosso entendimento sobre do trabalho e militar) sobre a matéria Psicologia jurídica.
isso, Maciel e Cruz, ao escreverem sobre a avaliação psico- O texto se subdivide em quatro itens previamente indica-
lógica em processos judiciais, colocam que: dos pelo Conselho Nacional de Justiça, os quais tivemos
O trabalho do psicólogo no campo jurídico, ainda que a oportunidade de desenvolver em obra própria (Silva &
tenha se ampliado, não somente no campo pericial, ainda Castro, 2011).
carece de discussão e desenvolvimento curricular que lhe Antes disso, a Resolução nº 2, editada em 2006, já re-
faça referência. Essa deficiência reflete-se, também, nos ca- comendava aos Tribunais de Justiça dos Estados que, em
sos em que são necessárias intervenções dos Conselhos de observância à legislação de regência, adotassem as provi-
Psicologia, principalmente dos Comitês de Ética, quando dências necessárias à implantação de equipes interprofis-
tratam sobre laudos e conteúdos afins, onde é possível ob- sionais, próprias ou mediante convênios com instituições
servar a escassez de profissionais que trabalham com essa universitárias.
demanda e que apresentem competência técnica especí- O objetivo da citada resolução é que os tribunais, por
fica para compreender a natureza, limites e possibilidades meio de suas equipes, possam dar atendimento às Comar-
do trabalho pericial. (2009, p. 46) cas dos Estados nas causas relacionadas a família, crianças
Para estudiosos europeus, por exemplo, é sabido que e adolescentes, devendo, no prazo de seis meses, informar
“não basta uma simples transferência directa dos conheci- a esse Conselho Nacional de Justiça as providências ado-
mentos (ou resultados científicos) da Psicologia ou da psi- tadas.
quiatria para o domínio da Justiça (...) Exige-se, além disso, Uma rápida consulta a essas resoluções pode mostrar-
que a informação fornecida pelos peritos esteja ancorada -nos a disposição do órgão federal em inserir o profissional
numa sólida metodologia científica” (Fonseca, Matos, & Si- psicólogo de forma cada vez mais organizada tecnicamen-
mões, 2008, p. 6). te nos quadros dos Tribunais de Justiça brasileiros, valori-

109
CONHECIMENTOS ESPECÍFICOS - PSICÓLOGO

zando não só a contribuição histórica com laudos, parece- Os artigos que fazem referência ao trabalho do perito são
res e relatórios mas também com a formação mais ampla o nº 139 (que menciona o perito como um auxiliar da Jus-
do operador de Direito. tiça) e os nos 145 a 147 e 420 a 439 (que dispõem sobre as
Outro destaque importante, embora mais genérico, so- funções do perito e a prova pericial).
bre a atuação do psicólogo é a Resolução nº 125, editada A perícia é um meio probatório (Rosa, 1999; Ama-
em 2010, que dispõe sobre a Política Nacional de Trata- ral Santos, 1994), sendo que, nela, o profissional verifica
mento Adequado dos Conflitos de Interesse no âmbito do e analisa fatos interessantes à causa, transmitindo ao juiz
Poder Judiciário. Tal medida representa um avanço, uma seu respectivo parecer. Existem outros meios de provas, tais
vez que os tribunais brasileiros deverão organizar-se para como a testemunhal e a documental, porém a prova peri-
oferecer, por exemplo, a conhecida mediação, que vem a cial difere das demais por ser realizada por um especialista
ser, dentre outras, uma técnica alternativa (à adversarial) de na matéria.
resolução de conflitos. A perícia é uma atividade técnica e processual, que se
A legislação interna do Tribunal de Justiça paulista des- materializa no processo através do laudo ou de qualquer
de 1985 possui normatização própria para a organização outra forma legalmente prevista. Os elementos para a perí-
dos Setores Técnicos (Normas de Serviço da Corregedoria cia são, basicamente, a nomeação do perito, a indicação de
Geral – Cap. XI). O Provimento n.º 236/85, do Conselho Su- assistentes técnicos pelas partes, a apresentação de quesi-
perior da Magistratura do Egrégio Tribunal de Justiça de tos, e, posteriormente, as considerações críticas das partes,
São Paulo, já dispunha sobre a organização dos setores e outros esclarecimentos do perito e dos assistentes dentro
técnicos (Serviço Social e Psicologia), e, mais adiante, sur- ou fora da audiência.
ge o Provimento nº 6/91, da Corregedoria Geral da Justiça, Em tese, todos os profissionais psicólogos são conside-
propondo um grau maior de organização das equipes. rados peritos, já que qualquer profissional que se encontre
Em 2005, visando a organizar e a atender as demandas registrado junto ao seu órgão de classe e que possua quali-
ficação técnica para responder as questões formuladas em
internas, o Tribunal de Justiça de São Paulo cria o Núcleo
juízo está qualificado como expert, ou seja, é considerado
de Apoio Profissional ao Assistente Social e Psicólogo do
perito em matéria de Psicologia. Rovinski (2004) explicita
Tribunal de Justiça de São Paulo. Esse núcleo foi criado
que não há nenhuma necessidade de formação específica
tendo em vista a necessidade de assessoramento técnico
na área forense, ainda que esta possa trazer maior credibi-
aos profissionais e a padronização das rotinas existentes, a
lidade ao relatório do expert.
normatização e a centralização de diretrizes de trabalho de
A perícia é materializada por técnicas conhecidas de
ordem técnica e administrativa e a orientação e o acompa- avaliação psicológica, e requer conhecimentos específicos
nhamento de profissionais (de Serviço Social e Psicologia) em processos básicos de investigação: compreensão da
no exercício de suas funções interdisciplinares: demanda instalada e sua relação com o domínio jurídico
As atribuições dos assistentes sociais e psicólogos do a ela direcionada, definição das estratégias e instrumentos
Tribunal de Justiça de São Paulo foram construídas por de exame psicológico mais adequados à demanda instala-
meio de intensas discussões dos profissionais do Judiciá- da, verificação da pertinência dos quesitos apresentados
rio. Essas atribuições foram normatizadas pela Secretaria em juízo ao perito e elaboração de comunicação psicológi-
de Recursos Humanos (antigo Departamento Técnico de ca por meio de laudo (Maciel & Cruz, 2009, p.46).
Recursos Humanos), e, após aprovação da Presidência do O trabalho estritamente pericial sempre foi solitário. No
Tribunal de Justiça de São Paulo, publicadas. O Comunica- máximo, conseguíamos atuar de forma multidisciplinar, ou
do nº 308/2004 (D.O.J. de 12/03/2003) versa sobre as atri- seja, levando nosso ponto de vista ao assistente social ou
buições dos assistentes sociais e o de nº 345/2004 (D.O.J. ao juiz, sem maiores construções. Sobre isso, no trabalho
de 26/05/2004) sobre as atribuições dos psicólogos. (TJSP, de Shine e Strong (1998), fica demarcado que Psicologia
2008) e Direito são campos distintos, com atuações diferentes,
Assim, por nos sentirmos parte dessa história que com e que o Direito não é interdisciplinar, mas multidisciplinar,
tanto carinho, fôlego e profissionalismo tentamos construir não cabendo, ao menos na organização atual, falar-se em
é que chamamos a atenção dos psicólogos interessados no participação na sentença, na intervenção e na decisão judi-
assunto. É necessária a ocupação responsável desse cres- cial, por exemplo.
cente campo de atuação, mas que, como qualquer outro, é Hoje conseguimos enxergar e experimentar avanços no
exigente e, por vezes, disciplinador. âmbito da Psicologia jurídica, pois cada vez mais os ope-
radores do Direito se valem de elementos da subjetividade
A atuação nas Varas de Família – o caminho cons- para fundamentar suas sentenças e buscam o profissional
truído para discussões antes de redigir seu trabalho.
Como exemplo recente, podemos citar a decisão do
O psicólogo iniciou sua atuação nas Varas de Família Superior Tribunal de Justiça, em voto proferido pela Minis-
como perito. Foi sendo chamado, como já dito inicialmen- tra Fátima Nancy Andrighi, que reconheceu o abandono
te, na década de 70 (século XX), a emitir parecer sobre ma- afetivo de um pai em relação à filha, estipulando um va-
téria que envolvia disputas de guarda e regulamentações lor para o ressarcimento do aludido dano emocional. Além
de visitas de filhos. disso, na mesma esteira, ainda tratou da alienação parental,
O respaldo legal de um perito judicial, seja ele de qual- de dano familiar e de dever de cuidar (Recurso Especial nº
quer área, está definido pelo Código de Processo Civil (Lei 1.159.242, Terceira Turma do STJ, julgado em 24/04/2012 –
nº 5.869/73), alterado parcialmente em 1992 (Lei nº 8.455). www.stj.jus.br ).

110
CONHECIMENTOS ESPECÍFICOS - PSICÓLOGO

Apesar de não ser este o local de discussão do as- Interpretar e avaliar não significa julgar. Comprometer-
sunto com seus liames psicológicos, limitamonos apenas -se com um diagnóstico ou com um prognóstico – quando
a apontar que a construção do raciocínio do mencionado possível – não significa impingir ao outro sofrimento, mas
tribunal superior foi pautada em conhecimentos da área da sim, possibilidade de libertação por meios necessários da
Psicologia (em parte) para entender e proferir a decisão, aplicação de uma justiça que ele (usuário da Justiça) pró-
que certamente possuirá um eco emocional na vida dos prio procurou, de uma lei que ele próprio buscou e de um
envolvidos. atendimento e de uma escuta que ele clama, como cliente
Sobre isso, já observou Nalini quando destaca a im- de um Judiciário que é público, ou seja, que está para servir
portância de o magistrado não ser apenas um compilador o povo, e não para servir-se desse mesmo povo.
de jurisprudência, limitando-se a aplicar a orientação do- Sobre isso, gostaríamos de indicar que, segundo o do-
minante à hipótese submetida ao seu julgamento, já que cumento redigido pelo Setor de Psicologia das Varas de
esse tipo de serviço poderia ser prestado por um compu- Família do Fórum Central de São Paulo, por ocasião da con-
tador” (1977, p.6). Esclarece o autor, na obra citada, que o sulta pública do Centro de Referências Técnicas em Psicolo-
juiz não deve limitar-se a ser um burocrata repetidor de gia e Políticas Públicas (CREPOP), em 2010, e apresentado
decisões alheias, sugerindo que ele prolate suas sentenças em evento do CRPSP, no mesmo ano, entendemos que não
com sentimento, fazendo um paralelo entre os radicais dos há uma descontinuidade entre o ato de avaliar e o de julgar.
vocábulos sentença e sentimento. Julgar reduz-se, na realidade, a resolver de certa maneira,
Em outra oportunidade, já escrevemos (Silva, 2005) depois de se ter analisado uma situação. O perito, tenha ou
que a visão interdisciplinar que a Psicologia jurídica vem não consciência disso, não é neutro em sua avaliação, que
buscando junto aos tribunais é o verdadeiro resgate da tem por objetivo servir como uma das bases de julgamento
dignidade da pessoa humana, como ser que pensa, vive e é para o juiz. É óbvio que o juiz é quem julgará do ponto de
dotado de subjetividade. Transformar os sujeitos não ape- vista jurídico. O perito emitirá um parecer sobre qual geni-
nas em sujeitos de direitos mas em indivíduos dignos e em tor lhe parece mais indicado, se é que há algum, do ponto
plenas condições de exercer sua cidadania tem sido o lema de vista psicológico, para permanecer, usando o exemplo
dessa grande área, que, pela sua importância, foi alçada a acima proposto, com a guarda: compartilhada ou unilate-
uma especialidade pelo Conselho Federal de Psicologia em ral, ou com outro membro da família. O que se pede de um
2001 (Resolução CFP nº 002/2001). perito é que, dentro de sua área de conhecimento, den-
tro dos referenciais científicos atuais, diga sobre a matéria
A demanda inicialmente pericial para os psicólogos das que está sendo discutida no processo. Não adianta apenas
Varas de Família foi alternandose com pedidos de acompa- descrever a personalidade de cada um dos envolvidos. É
nhamento de casos e de intervenções que paulatinamente necessário emitir um parecer sobre essa questão, em certos
delineiam modalidade de atuação mais aberta e mais inter- casos, até demonstrar que não há uma alternativa melhor
ventiva. Pode-se dizer que houve boa aceitação por parte que outra (CRP SP, 2010, p.8).
dos juízes sobre isso, uma vez que grande parte deles, na Assim, a prática atual nas Varas de Família vem revelan-
verdade, desconhecia as diversas possibilidades de atua- do cada vez mais a necessidade de o psicólogo possuir um
ção do psicólogo. Para a Justiça, o trabalho é importante e raciocínio pluridimensional, sem deixar, porém, de avaliar e
válido quando consegue auxiliar os juízes a dirimirem suas de diagnosticar, revendo constantemente a inserção socio-
dúvidas e entenderem melhor os aspectos psicológicos do -histórica dessa avaliação e, dentre outras coisas, a visão de
caso. homem que está por detrás do escrutínio.
A própria complexidade da sociedade, o avanço das De uma vez por todas, a avaliação psicológica precisa
comunicações e a rapidez com que se buscam soluções ser compreendida como competência imprescindível ao
foram atingindo quase que em sua totalidade o fazer pro- psicólogo – como é disposto nas diretrizes curriculares que
fissional dos psicólogos voltados para as questões familia- regem a formação profissional – e como área que requer
res. Hoje as questões que buscam as soluções no Judiciário formação específica e atualizada (...) Avaliar é necessário
envolvem tamanha complexidade que os recursos de ava- em diversos contextos de atuação, pois, além de fornecer
liação até então conhecidos passam a não mais conseguir informações preciosas dos aspectos psicológicos dos pro-
auxiliar ou oferecer respostas. cessos avaliados, contribui para uma atuação mais refinada
Assim, definirmos possibilidades de risco que envol- e precisa (...) (Noronha & Reppold, 2010, p. 200)
vem filhos, atividades emocionais desestruturantes de ge-
nitores, tais como o que hoje se discute como alienação Seria interessante se a perícia conseguisse desenvolver
parental, sofrimento de bullying, assédios de várias nature- ou estimular o surgimento de uma autonomia interna na
zas, nexos causais gerados por abandonos afetivos e tantas clientela das Varas de Família, já que, em alguns casos, o
outras situações que chegam ao psicólogo necessitam de momento pericial, quando outro trabalho não teve ou não
uma implementação específica dos estudos e pesquisas pôde ter lugar (como, por exemplo, a psicoterapia, a me-
para que possamos situar-nos e entendermos que prote- diação), é o primeiro e único de que a família dispõe para
ção nos cabe fornecer ao ser humano acerca de sua saúde entrar minimamente em contato com seu mundo interno.
mental, ou ainda, como podemos colaborar com o sistema No entanto, sabemos que as estruturas de personalidade e
de Justiça e com o ser humano de forma integral, quando fatores cruciais concretos muitas vezes impossibilitam esse
nos solicitam institucionalmente nossas opiniões. esperado crescimento.

111
CONHECIMENTOS ESPECÍFICOS - PSICÓLOGO

Embora não seja o caso de fugirmos ao papel pericial, mos a utilização de ferramentas que ajudem aqueles que
aquele que nos enquadra na tarefa de Vara de Família, a estão em conflito a encontrarem, por si sós, o “leme do
nossa postura e, sem dúvida alguma, a documentação es- próprio barco interior” (Silva, 1999) sem precisarem recor-
crita realizada pelo psicólogo, aqui chamada de laudo, são rer a um terceiro, no caso o Judiciário, para a resolução de
ferramentas de grande valia para um estímulo e incentivo suas dificuldades ou amarguras.
aos indivíduos. Este último é um instrumento limitado, sa-
bemos, já que pode ser usado como alimento/ fermento Como já tivemos a oportunidade de escrever em traba-
do litígio, mas é o que temos como próprio para a tradução lho de doutorado (Silva, 2005) sobre a Psicologia jurídica,
do nosso trabalho, que nem sempre pode ser findado com sempre devemos partir do pensamento que as pessoas são
entrevistas devolutivas, tal como no modelo clínico. capazes de decidir sobre a própria vida; além disso, obser-
Nesse sentido, devido à nossa atuação profissional, vando-se os povos através da história da civilização, pode-
desenvolver-se especificamente junto às Varas de Família, a mos notar que eles buscam maneiras diversas de resolver
demanda de casos recebidos é, na maioria das vezes, para suas desavenças, atendo-se ao momento político e histó-
a realização de perícias. Por intermédio desse tipo de traba- ricosocial vigente. Dessa maneira, surgem como uma forte
lho, busca-se a definição, como já explicitado em outro tra- tendência do momento atual as discussões que priorizam
balho, “quem do ponto de vista emocional possui maiores formas alternativas de resolução dos conflitos, ou seja, for-
ou melhores condições, naquele momento, de assumir o mas alternativas à via judicial clássica, que é a adversarial.
filhos, em caso de disputas de guarda, por exemplo” (Silva, Além disso, o século XXI aponta-nos com clareza a
1999). ineficiência da Justiça no que se refere à administração do
Esse fato vem trazendo-nos inquietações e forçosa- tempo, um problema crônico do sistema judiciário brasilei-
mente remetendo-nos a um questionamento sobre as ro e que acarreta graves consequências sociais. O aumento
possibilidades efetivas que o psicólogo possui para ler a da população e da litigiosidade descortina uma realidade
situação emocional delineada, e, para além disso, também em que o Estado é incapaz de resolver todos os conflitos e
intervir. De algum modo, é importante darmos sentido para necessita adotar caminhos com novos parâmetros atrela-
que a coleta de dados, as entrevistas e as observações pos- dos a uma mentalidade de composição, de autorregulação.
sam fazer algum tipo de eco dentro de um contexto viven- Métodos conhecidos como mediação, conciliação e
cial mais amplo e profundo de sua história.
negociação vêm sendo adotados paulatinamente pelos tri-
bunais brasileiros nos mais diferentes tipos de processos,
A alternativa proposta por Martins é no sentido de
construindo uma verdadeira cultura que se contrapõe à
utilizarmos as avaliações psicológicas dentro do âmbito
cultura litigante, que é a denominada cultura da paz.
jurídico de forma a não apenas compreendermos as difi-
A profusão de ideias e pessoas ocorre numa velocidade
culdades internas das pessoas mas também seus recursos
tal que reclama por estruturas flexíveis, renovadas e adap-
individuais, familiares e relacionais, que podem ser utiliza-
táveis, através de procedimentos simples, céleres e eficazes
dos na solução das questões. Nossas sugestões inseridas
nos laudos são aceitas em 94,23% dos casos, segundo e que tenham condições de se contrapor ao peso do for-
uma pesquisa sobre a influência dos laudos psicológicos malismo da Justiça tradicional, materializada em um tribu-
nas decisões judiciais nas Varas da Família do Tribunal de nal e no complexo tecnicismo processual que caracteriza o
Justiça de São Paulo (Rodrigues, Couto, & Hungria, 2005, seu funcionamento. (Silva & Castro, 2011, p. 64)
p.31). Isso, é obvio, agrega não só maior responsabilidade
ao trabalho mas também a possibilidade de torná-lo um Na ponta profissional em que atuamos, os casos infe-
instrumento de transformação. lizmente já chegam bastante comprometidos do ponto de
Concordamos com Reis quando afirma que os psicó- vista emocional, muitas vezes com distorções significativas
logos que atuam nesse meio “muitas vezes caminham no na apreciação do mundo familiar e global a sua volta. As si-
sentido de reproduzir os conflitos familiares, em vez de in- tuações invariavelmente envolvem filhos com necessidades
ventar novos meios de desfazê-los ou transformálos (2009, básicas negadas, distorcidas, ou pior, destruídas de forma
p.19), mas também apontamos que, apesar de considerar- perigosa não só para a sua integridade psíquica como tam-
mos que as Varas de Família sejam o local onde se exer- bém para a física.
citam poderes, traduzidos muito bem pela citada autora Parte das situações que chegam para as perícias psico-
como uma verdadeira guerra dos sexos, há de se destrin- lógicas não possuem qualquer possibilidade de serem me-
char tecnicamente, e valendo-se de pressupostos científi- diadas. Há neles psicopatologias, queixas de abusos (físi-
cos, quais são os casos em que realmente isso está prevale- cos, sexuais e psicológicos) por vezes declaradas, por vezes
cendo e quais exibem clara ou subliminarmente situações escamoteadas, além de graves distorções nos dinamismos
de risco para o(s) filho(s) envolvido(s). psíquicos que, em última instância, necessitam de um ins-
Qualquer avaliação que envolva crianças pode ser vista trumento legal para salvaguardar o(s) filho(s) de inúmeras
hoje sob o princípio da dignidade humana, ou seja, de le- possibilidades de risco. Segundo Vezzulla,
var o direito de ser digno a um ser merecedor de respeito (...) É fundamental que o mediador tenha bem presen-
(Marques da Silva & Miranda, 2009). te, para poder transmitir aos mediados, que a mediação
Trabalhos realizados fora do âmbito judicial devem ser apenas os poderá ajudar se eles desejarem preservar o re-
implementados, e são, na maior parte das vezes, a melhor lacionamento, melhorá-lo ou pelo menos não o prejudicar.
alternativa ao trabalho pericial. Não há como desvalorizar- Se esse interesse não existir, a mediação perde a maior de

112
CONHECIMENTOS ESPECÍFICOS - PSICÓLOGO

suas forças e os acordos correm o risco de não serem cum- (...) Em psicodiagnóstico clínico, muitas vezes ouvi-
pridos, pela falta de desejo de reconstruir ou preservar o mos a máxima de que o que importa não é a realidade
relacionamento anterior à contenda, ou de recriar um novo objetiva. O que importa é como a criança vivenciou uma
relacionamento, já que o antigo fracassou. (2001, p. 34) determinada situação, tenha ela acontecido ou não. No Ju-
Além disso, o contingente de profissionais psicólogos diciário, importa não só como foram introjetadas as figuras
concursados, que atuam no tribunal paulista junto às Varas parentais mas também como os pais são na realidade. Se
de Família, possui o impedimento legal de realizar traba- há alegação de abuso sexual, é necessário que haja uma
lhos de conciliação, já que o psicólogo é remunerado nessa investigação minuciosa para concluir se, de fato, ocorreu.
instituição. A título de ilustração, mencionamos o provi- (Castro, 2003, p. 37)
mento nº 953/2005, do Conselho Superior de Magistratura: Salientamos, dessa forma, a necessidade constante
Artigo 3º - Poderão atuar como conciliadores, vo lun- de atualização do profissional que atua nos Tribunais, não
tários e não remunerados, magistrados, membros do Mi- apenas em relação às descobertas da ciência psicológica
nistério Público e Procuradores do Estado, todos aposenta- mas também no tocante aos códigos legais já estabeleci-
dos, advogados, estagiários, psicólogos, assistentes sociais, dos e aos avanços constantes empreendidos pelas socie-
outros profissionais selecionados, todos com experiência, dades, sempre inscritas em momentos históricos e culturais
reputação ilibada e vocação para a conciliação, previamen- que as remetem a um contínuo movimento.
te aferida pela Comissão de Juízes ou Juiz coordenador, Os processos que envolvem avaliações de responsabi-
quando não constituída a Comissão. lidades parentais são reconhecidos na literatura como os
§ 1º - Os conciliadores não terão vínculo empregatício mais complexos, com situações de litígio intenso e reitera-
e sua atuação não acarretará despesas para o Tribunal de do, e, não raro, com o desajustamento das crianças envolvi-
Justiça (grifo nosso). das. Muitos autores sugerem inclusive que se trata de uma
das áreas mais difíceis da avaliação psicológica forense,
Nos casos que envolvem disputa de guarda, por exem- chegando a considerá-la um contexto de risco para os pro-
plo, é evidente que a criança necessita de ambos os genito- fissionais, aquela onde é mais provável virem a confrontar-
res; pai e mãe têm importante parcela na formação do filho, -se com queixas e processos que questionam e impugnam
mas, por exemplo, diante do recrudescimento da criança,
as suas práticas (Pereira & Matos, 2011, p.315).
ou seja, de sua negativa veemente de não mais residir com
Já há algum tempo, diversos países estrangeiros, aten-
a genitora, temos necessariamente que avaliar dados de
tos ao surgimento de queixas nos Comitês de Ética sobre
sua personalidade e dinamismos psíquicos relativos aos
as perícias nas áreas que estudam as regulações das res-
seus padrões cognitivos e afetivoemocionais, que, no cru-
ponsabilidades parentais e também alertados pela incon-
zamento com as necessidades do filho, podem estar sendo
sistência e pela arbitrariedade metodológica desses traba-
mal assimilados na relação. Necessitamos avaliar as possi-
lhos, começam a publicar pequenos guias para o direcio-
bilidades (ou distúrbios emocionais) na família, nos indiví-
namento dessa atuação. Segundo Pereira e Matos (2011,
duos ou nos vínculos que possam estar sobrecarregando
e indicando que talvez essa criança sofra prejuízos em seu p.315), a Associação Americana de Psicologia (APA) publica
desenvolvimento emocional. as primeiras orientações para a prática pericial forense em
Para um psicólogo jurídico, atuante no tribunal, é im- 1994, seguindo-se depois muitas outras. São os chamados
portante apreender o leque de possibilidades e de interna- guidelines, que também trazem suas vicissitudes e devem
lizações que a criança e o adolescente fazem acerca de suas ser vistos com cautela e criticidade em função de, entre
figuras parentais; para tal, as técnicas de avaliação muito outras coisas, não haver uma padronização para as diversas
contribuem para o refinamento do olhar psicodiagnóstico. culturas que os utilizarão.
No entanto, saibamos olhá-las com humildade, uma vez
que, como nos disseram Simões e Lopes, “a avaliação é Além das sugestões da referida associação, outras
uma tarefa difícil e, ao mesmo tempo, redutora e imprecisa orientações internacionais foram surgindo, como é o caso
quando se trata de resumir toda a complexidade das con- das boas práticas publicadas online pela AFCC – Associa-
dutas humanas” (2004, p.195). tion of Family and Conciliation Courts, que é uma associa-
No entanto, o olhar investigativo, que vai além da ção internacional e interdisciplinar de profissionais dedica-
curiosidade científica e vai também examinar e ponderar dos à resolução de conflitos familiares (Pereira & Matos,
sobre a realidade factual em si, é muito importante tam- 2011, p.318).
bém, já que dará indicativos acerca de quem são aquelas No Brasil, foi oferecido à categoria pelo Conselho Fe-
figuras parentais na realidade e se podem apresentar ou deral de Psicologia o documento de referências para atua-
não algum tipo de ameaça ou risco ao crescimento físico- ção do psicólogo em Vara de Família (CFP, 2010), o qual se
-emocional da criança (Castro, 2003). pautou em metodologia do Centro de Referência Técnica
Identificamo-nos muito com o trabalho desenvolvido em Psicologia e Políticas Públicas (CREPOP). As referências
por Castro sobre disputa de guarda de filhos, pois a autora são, no entender no órgão federal, baseadas nos princípios
assume, com maestria científica, a necessidade de também éticos e políticos norteadores do trabalho dos psicólogos,
se avaliar como são os pais na realidade, demarcando uma e possibilitam a elaboração de parâmetros compartilhados
das diferenças entre o psicodiagnóstico clínico daquele e legitimados pela participação crítica e reflexiva da cate-
realizado por ocasião da perícia psicológica. goria.

113
CONHECIMENTOS ESPECÍFICOS - PSICÓLOGO

Sobre o conteúdo do documento, a equipe paulista, A PSICOLOGIA


pioneira na atuação com casos das Varas de Família e, diga-
-se de passagem, a mais antiga que milita na área, manifes- Um levantamento realizado por Bernardi (1999) indica
touse em documento próprio cuja relevância motivou um que, em 1981, os psicólogos já prestavam serviços voluntá-
evento no Conselho Regional de Psicologia – 6ª Região. As rios no Tribunal de Justiça, as agências de colocação fami-
considerações tecidas integram documento próprio, não liar (Lei Estadual nº 560 de 1949), com uma atuação de ca-
publicado, porém protocolado (no CRP – 6ª Região) em ráter terapêutico e de intervenção clínica junto às famílias.
função da consulta pública que foi realizada. O ingresso foi através de um estágio, na Fundação Es-
tadual do Bem-Estar do Menor, para dar início a um tra-
O Conselho Regional de Psicologia de São Paulo, con- balho que, mais tarde, seria sedimentado no Tribunal de
siderando o número crescente de representações relativas Justiça do Estado de São Paulo.
ao trabalho do Psicólogo no contexto do Poder Judiciá- A entrada do Psicólogo se deu oficialmente nas Varas
rio, especificamente nas questões de família, realizou dois de Menores (atuais Varas de Infância e Juventude), local
encontros (2005 e 2006) a fim de debater a relação dos
em que as questões atinentes à menoridade deveriam ser
peritos com os assistentes técnicos e de construir subsídios
estudadas. Posteriormente, surgiu a entrada nas Varas de
para um melhor exercício profissional. Disso decorreu a ne-
Família e Sucessões.
cessidade da criação de um grupo de trabalho (GT) com
Pode-se dizer que esta inserção foi fundamentada na
representantes do Conselho Regional de Psicologia de São
Paulo (CRP SP), profissionais e entidades representativas na necessidade de se oferecer aos Juízes uma assessoria es-
área. Das discussões do mencionado grupo, derivou uma pecializada, sempre que um direito da criança e do adoles-
recomendação ao Tribunal de Justiça de São Paulo, que o cente fosse ameaçado ou violado.
aprovou na forma do Comunicado nº 01/2.008, do Núcleo A finalidade era apresentar subsídios verbais e escritos
de Apoio Profissional de Serviço Social e Psicologia do Tri- sobre a natureza e as causas de uma dada situação envol-
bunal de Justiça de São Paulo, publicado no Diário Oficial vendo todo o grupo familiar, contribuindo para a reflexão e
da Justiça em 14 de outubro de 2008 (CRP SP, 2010). análise sobre a melhor medida legal a ser aplicada ao caso
Após a publicação, em novembro de 2008, o Conse- concreto, bem como as consequências desta aplicação. O
lho Regional de São Paulo realiza outro evento, chamado foco sempre foi à convivência familiar, possibilitando uma
agora A Ética Própria da Psicologia – Mudanças na Relação intervenção ativa na família, ao desenvolver um serviço de
Assistente Técnico e Perito, visando à promoção de um am- diagnóstico situacional, orientação e aconselhamento.
plo debate sobre o assunto. Todo o percurso paulista foi A contribuição desse campo da ciência, aos operadores
traduzido em uma publicação na série Cadernos Temáticos, de Direito, estava amparada legalmente, já que o Código
que objetiva registrar e divulgar os debates realizados no de Menores de 1979 estabelecia a diferenciação dos aten-
Conselho em diversos campos de atuação da Psicologia dimentos realizados pela Promoção Social e pelo Judiciário.
(CRP SP, 2010). Em 1980, os psicólogos passam a atuar nas chama-
Acompanhando a discussão nacional, em junho de das audiências interprofissionais, propostas por Camargo
2010, o Conselho Federal de Psicologia edita a Resolução (1982), que se constituíam numa forma de atendimento
nº 8, que, em suma, dispõe sobre a relação perito e assis- dos casos em juízo, por meio da prévia apuração da equipe
tente técnico no Poder Judiciário. A sobreposição das atua- técnica, composta por Assistente Social e Psicólogo. Tais
ções profissionais evidenciou necessitar de uma regulação, profissionais tinham como dever não só apresentar as me-
muito bem realizada por esse instrumento. didas cabíveis dentro de suas respectivas áreas, mas tam-
Em síntese, não basta uma simples transferência direta bém, confeccionarem relatórios circunstanciados visando à
dos resultados e dos conhecimentos científicos da Psico- decisão do processo.
logia para as questões que envolvem perícias nas Varas de
Esta proposta constituiu-se num verdadeiro marco
Família. Em conformidade com a nossa prática e em face
para a entrada definitiva do psicólogo nos quadros da ins-
da literatura consultada, é importante traçar metodologias
tituição jurídica, sendo que em 1981 os psicólogos foram
específicas e tratar o assunto com rigor e seriedade. Vimos
legalmente contratados, passando a integrar praticamente
que, pela utilização que tem sido feita dos laudos psico-
lógicos e das discussões da ciência psicológica, ainda há todas as Varas de Infância e Juventude da capital.
muito o que percorrer. Em 1985 ocorreu o primeiro concurso público para o
Não se trata de colocar meramente um ramo do saber ingresso de psicólogos nos quadros do Tribunal de Justiça
à disposição do outro, como outrora se dizia, também não de São Paulo com a criação de 65 cargos efetivos e mais 16
se trata de apresentar verdades não sabidas a quem precise cargos de chefia.
ouvi-las, uma vez que a Psicologia tem uma concepção de
verdade que difere de outros campos. Trata-se de formali- DO SERVIÇO SOCIAL E DA PSICOLOGIA
zar uma coconstrução, uma possibilidade de configurar um
olhar sobre elementos comuns às duas ciências – Direito e O Provimento CXVI, do Conselho Superior da Magis-
Psicologia – que, dentre outras coisas, passa pela previsão, tratura, de 17/04/1980, normatizou a atuação dos assisten-
explicação e construção de políticas voltadas para a saúde tes sociais nas Varas de Família e Sucessões e em 12 Varas
mental. Distritais da Comarca de São Paulo.

114
CONHECIMENTOS ESPECÍFICOS - PSICÓLOGO

Em 1985, o Provimento nº 236/85 do Conselho Supe- • a criação em 2006 do SINASE – Sistema Nacional
rior da Magistratura, retomou alguns pontos do anterior e de Atendimento Sócio Educativo.
incluiu a atuação dos
Psicólogos, dispondo também sobre a organização dos Cabe ainda mencionar que recentemente o Conselho
Setores Técnicos (Serviço Social e Psicologia). Um pouco Nacional de Justiça (Emenda Constitucional nº 45/2004)
mais adiante, o Provimento de nº 6/91 da Corregedoria Ge- mostrou-se sensível à importância inquestionável da atua-
ral da Justiça, atualiza a inserção das equipes técnicas. ção do psicólogo e assistente social junto às questões que
Hoje, a atuação dos assistentes sociais e psicólogos se apresentam no âmbito do Poder Judiciário, e editou a
está regulamentada pelos Provimentos do Conselho Su- Recomendação nº 2, em 25 de abril de 2.006.
perior da Magistratura nº 838/04 e Corregedoria Geral da Essa medida recomenda que os Tribunais de Justiça
Justiça, nº 07/2004 (todos inseridos nas Normas de Serviço dos Estados adotem as providências necessárias à implan-
da Corregedoria Geral – Cap. XI). tação de equipes interprofissionais, próprias ou mediante
Com a inserção legitimada na instituição, tanto os psi- convênios com instituições universitárias, que possam dar
cólogos como os assistentes sociais, iniciaram uma movi- atendimento às comarcas dos Estados nas causas relacio-
mentação, visando sua organização mais diferenciada den- nadas à família, crianças e adolescentes, a exemplo das que
tro do judiciário. Ocorreram inúmeros seminários, encon- versam sobre perda e suspensão do poder familiar, guarda,
tros, pesquisas, cursos e supervisões, com a proposta de adoção e tutela, além da aplicação de medidas socioedu-
ampliação, divulgação e discussão dos papéis profissionais. cativas, devendo, no prazo de seis meses, informar ao Con-
Do ingresso dos psicólogos em 1980 em diante, as selho sobre as providências adotadas.
chamadas equipes técnicas ou interprofissionais foram se No Estado de São Paulo, os profissionais estão distri-
qualificando e se integrando, sendo aceitas como obriga- buídos nos fóruns da capital – Fórum Central (Vara de In-
tórias em todo Brasil, também por força de Lei Federal nº fância e Juventude e Varas de Família e Sucessões), 10 Fó-
8.069/90, que implanta o Estatuto da Criança e do Adoles- runs Regionais (Vara de Infância e Varas de Família e Suces-
cente. sões) e 4 Varas Especiais, e em 56 Circunscrições Judiciárias,
O texto legal ressalta a autonomia dos profissionais, compostas por Comarcas e Fóruns Distritais, nas diversas
regiões do Estado de São Paulo, totalizando, na atualidade,
que deverão ter total liberdade para expressar suas con-
cerca de 1166 profissionais no corpo técnico.
clusões e sugestões técnicas, por meio de documentos que
Cabe ainda ressaltar que os Assistentes Sociais e Psi-
auxiliarão o juiz na resolução do caso.
cólogos também estão inseridos em diversas Unidades de
As atribuições dos assistentes sociais e psicólogos do
Departamentos para responder a uma demanda interna
Tribunal de Justiça de São Paulo foram construídas por
institucional, com o objetivo de melhorar as relações do
meio de intensas discussões dos profissionais do judiciá-
indivíduo no trabalho e trazendo maior qualidade e resul-
rio. Essas atribuições foram normatizadas pela Secretaria
tados organizacionais.
de Recursos Humanos (antigo Departamento Técnico de
Atualmente além de exercerem suas funções nos Servi-
Recursos ços para atendimento a usuários das Varas da Infância e da
Humanos) e após aprovação da Presidência do Tribu- Juventude, Varas Especiais e Varas de Família e Sucessões,
nal de Justiça de São Paulo, publicadas. O Comunicado nº o Serviço Social e a Psicologia também atuam em áreas
308/2004 (D.O.J. de 12/03/2003) versa sobre as atribui- administrativas como:
ções dos assistentes sociais e o de nº 345/2004 (D.O.J. de Seção de Concessão e Controle do Auxílio Creche-Es-
26/05/2004) sobre as atribuições dos psicólogos. cola, da DIRETORIA DE GESTÃO DE RECURSOS HUMANOS,
As alterações nos dispositivos legais e as mudanças na onde há assistentes sociais atuando desde 1982, no sentido
política de atendimento à criança e aos adolescentes têm de planejar e desenvolver ações, visando à administração
possibilitado a redefinição de funções entre o Judiciário e de benefícios;
Executivo. Na DIRETORIA DE DESENVOLVIMENTO DE RECURSOS
Ressalta-se que, diante da compreensão da incomple- HUMANOS, em uma Diretoria de gestão de capacitação,
tude institucional há necessidade de um novo posiciona- responsável pelo Treinamento e Desenvolvimento dos fun-
mento para o trato das questões do campo Sociojurídico. cionários em geral, trabalho iniciado em 1991 e reestrutu-
Pode-se citar alguns fatores que vem modificando a rado através da Portaria 7.254/2005, publicada em 1º de
atuação profissional dos Assistentes Sociais e Psicólogos agosto de 2005; GRUPO DE APOIO TÉCNICO E ADMINIS-
Judiciários no Tribunal de Justiça, contribuindo também TRATIVO AOS JUÍZES CORREGEDORES NA PRIMEIRA VICE-
para a ampliação do quadro funcional como: -PRESIDÊNCIA, trabalho iniciado em 1993, desenvolvendo
• a necessidade de democratização e acesso à Justi- assessoria nos processos administrativos de funcionários;
ça; SERVIÇO DE ATENDIMENTO PSICOSSOCIAL AOS MAGIS-
• a implementação dos Conselhos Municipais de Di- TRADOS E FUNCIONÁRIOS DO TRIBUNAL DE JUSTIÇA
reito da Criança e do Adolescente e Conselhos Tutelares; DO ESTADO DE SÃO PAULO, criado em 1995, Portaria nº
• a Criação do Conselho Nacional de Justiça instituí- 2.839/95, contando com seis unidades no interior, com
do em 2004, presidido pelo Supremo Tribunal Federal; objetivo de intervir e fornecer atenção apropriada aos as-
• o Plano Nacional de Convivência Familiar e Comu- pectos humanos do trabalhador, já afetados em sua orga-
nitária; nização mental e emocional; SERVIÇO PSICOSSOCIAL VO-

115
CONHECIMENTOS ESPECÍFICOS - PSICÓLOGO

CACIONAL AOS MAGISTRADOS E FUNCIONÁRIOS DO TRIBUNAL DE JUSTIÇA DO ESTADO DE SÃO PAULO existe desde
1998 e tem como função precípua a avaliação psicossocial dos candidatos à Magistratura; o acompanhamento e reavaliação
psicossocial dos juízes em estágio probatório. Além disso, desenvolve programas de orientação profissional aos filhos, ne-
tos e outros dependentes dos servidores do Tribunal de Justiça de São Paulo;
Na DIRETORIA DA ÁREA MÉDICA E ODONTOLÓGICA, foi criada a Seção Especial de Acompanhamento Psicossocial e
Readaptação, objetivando o acompanhamento dos servidores em licença saúde afastados há mais de 6 meses, com inter-
venções visando o restabelecimento da saúde e consequente retorno ao trabalho, atividade iniciada em 1998;
Na COMISSÃO JUDICIÁRIA DE ADOÇÃO INTERNACIONAL, desde 2002, elaborando instrumentos de registro e controle
das adoções realizadas, atendimento e orientação aos assistentes sociais e psicólogos judiciários em matérias relativas ao
CEJAI, bem como manifestações nos autos de habilitação de pretendentes à adoção internacional.
Em 09 de junho de 2005 surge o Núcleo de Apoio Profissional de Serviço Social e Psicologia do Tribunal de Justiça de
São Paulo criado pela Portaria n. º 7243/2005 e subordinado diretamente à Corregedoria Geral da Justiça.
O Núcleo foi criado tendo em vista a necessidade de assessoramento técnico aos profissionais e a padronização das
rotinas existentes; a normatização e centralização de diretrizes de trabalho de ordem técnica e administrativa, a orientação
e acompanhamento de profissionais (de Serviço Social e Psicologia) no exercício de suas funções interdisciplinares. Com
pouco mais de dois anos de existência, vem avaliando e qualificando também, projetos profissionais que trazem uma subs-
tancial melhoria na atuação de ambas as áreas dentro do judiciário paulista.

A SUBORDINAÇÃO DOS PROFISSIONAIS DE SERVIÇO SOCIAL E PSICOLOGIA

O Tribunal de Justiça definiu que nos fóruns onde há Vara Especializada da Infância e Juventude, os profissionais devem
ficar lotados nesse juízo e respondem disciplinarmente ao Juiz Corregedor Permanente da Vara. Comum é o assistente so-
cial e psicólogo que atua em processos que tramitam em outras Varas. Isso significa dizer que os profissionais respondem
diretamente pelos trabalhos que desenvolvem para cada um dos juízes dessas Varas, ou seja, para o Juiz do Feito (NSGC
Cap.XI, seção IV, artº 24.2).
Nos locais onde não há Vara Especializada da Infância e da Juventude a lotação dos assistentes sociais e psicólogos se
dá na Secretaria do Fórum, o que equivale dizer que o Juiz Diretor do Fórum é o superior hierárquico desses profissionais.
Abaixo apresentamos um organograma para dar maior clareza à subordinação do Assistente Social e do Psicólogo
atuantes nas áreas técnicas do Tribunal de Justiça de São Paulo.

116
CONHECIMENTOS ESPECÍFICOS - PSICÓLOGO

É importante a compreensão de que os assistentes sociais e psicólogos são subordinados hierarquicamente ao juiz. Ao
Diretor administrativo caberá o trato das questões relativas à esfera administrativa. Como providenciar os recursos neces-
sários para os profissionais desenvolverem suas ações, assegurar o uso da viatura, encaminhar frequência e férias.
Os profissionais devem manter informado o juiz a quem é subordinado sobre a necessidade de comparecer em ativi-
dades extra-fórum como reuniões, visitas a recursos da comunidade e outras, próprias da prática dos assistentes sociais e
psicólogos. Sugere-se que preferencialmente faça a informação por escrito, e as anexe. No caso de, por exemplo, ter um
recebido convite, recomenda-se que se possível, despache diretamente com o magistrado.
A assinatura do ponto dos profissionais deve ocorrer diariamente nas Varas, Diretorias ou Setores em que estiverem
lotados. Nas Varas da Capital, onde existe designação de chefia, o controle do ponto é de responsabilidade direta dessa
chefia (NSGC, Cap. XI), assim como a designação do processo (quando o processo não vem com prévia designação), escala
de Plantão Diário, do uso da viatura, escala de férias, dentre outras atividades relativas à gestão, supervisão, etc.
Não há regulamentação de chefia técnica para as equipes do interior, o que pode representar algumas dificuldades
na organização e gerenciamento do setor em equipes com vários profissionais. Como forma de proporcionar um melhor
funcionamento dos setores tem sido comum, que um profissional de cada área assuma a coordenação técnica. Notada-
mente os juízes têm reconhecido à importância da coordenação, pois sem dúvida isso contribui para uma maior eficiência
no trabalho.

O ESTUDO PSICOLÓGICO

O Estudo Psicológico é privativo do profissional da Psicologia. Este profissional deve escolher os procedimentos técni-
cos de acordo com a formação e linha teórica que vem desenvolvendo em sua prática.
Lembra-se o descrito no artigo 151 do ECA, de que os profissionais da equipe interprofissional têm a autonomia e
liberdade para atuarem.
Usualmente, utilizam-se técnicas de entrevistas apropriadas tanto para o atendimento à criança/adolescente quanto
aos adultos; sendo que estas podem ser aplicadas de maneira individual, com o casal, em subgrupo ou em grupo.
Quando se trata de criança em tenra idade, lança-se mão da observação lúdica, da observação da interação “mãe-filho”
e/ou deste com a figura substituta, entre outras técnicas, já que a linguagem verbal está de certa forma limitada. Sabe-se
da preponderância da linguagem não-verbal sobre a última.
Quanto ao uso de testes, orienta-se que o profissional consulte o site do CRP, www.pol.org.br e www.crp.org.br. Esta lista
vem sendo constantemente atualizada e nesta estão assinalados todos os testes que estão em condições de uso.
A Resolução 007/2003 do CFP instituiu o Manual de Elaboração de documentos escritos e produzidos pelo Psicólogo
decorrente da avaliação psicológica e revoga a resolução do CFP- 017/2002.

117
CONHECIMENTOS ESPECÍFICOS - PSICÓLOGO

Do ponto de vista psicológico podem ser levantados prioritariamente os seguintes aspectos:


Fases de desenvolvimento da personalidade da criança/adolescente e de seus irmãos (distúrbios de comportamentos,
disfunções cognitivas, psico- motoras e afetivas);
Percepção dos vínculos afetivos da criança/adolescente com as principais figuras de apego no passado, na atualidade,
e perspectivas de prognóstico para o futuro, levando sempre em consideração, o melhor interesse da criança;
Aspectos da psicodinâmica da estrutura de personalidade das figuras parentais e possíveis figuras substitutas; traços
patológicos e saudáveis de suas estruturas psíquicas; evidência de dependência química; comportamentos abusivos;
Avaliação das relações intrafamiliares e da família em sua inserção com outros sistemas; observação quanto ao cumpri-
mento dos papéis nestes espaços: creche, escola, saúde, rotina de vida em casa e na comunidade;
Características dos vínculos entre os adultos e de cada um deles com a criança ou adolescente em questão;
História de vida pessoal e familiar das figuras parentais. Pesquisar na família atual e na de origem dos genitores e /ou
requerentes, padrões de repetição de comportamentos que indicam disfuncionalidade, dados inconsistentes, segredos,
mitos que podem estar relacionados de alguma forma com a temática enfrentada;
Na entrevista de devolução, o psicólogo deve ter o cuidado em abordar com as partes os principais aspectos que
foram levantados no curso da avaliação e que serão descritos no relatório a serem anexados nos autos. Muitas vezes nos
questionamos sobre questões éticas, mas devemos ter claro que nosso maior comprometimento é com a criança e ou o
adolescente.

Quanto ao número de entrevistas, a escolha das técnicas deverá ser levada em conta a particularidade de cada caso; se
este é proveniente da Vara da Infância ou da Família, da complexidade, da gravidade e da urgência da situação, e principal-
mente, da percepção do profissional que está atuando.
Destacamos as ideias de Silva (2004) que ilustram bem o valor da atuação psicológica.
Dentro das instâncias jurídicas e em face do drama familiar de quem as procura, o psicólogo tem a real dimensão do valor
da lei, da sua importância para a organização da vida em sociedade e vale-se dela como uma aliada para auxiliar aqueles
casos atravessados, inclusive por acentuada periculosidade, quer seja física ou emocional. Geralmente apenas mediante a
realização de uma avaliação meticulosa é que se torna possível a detecção de tais perigos. (Silva, 2004, p.114)

118
CONHECIMENTOS ESPECÍFICOS - PSICÓLOGO

ESTUDO SOCIAL OU PSICOLÓGICO

Feitas às considerações acima, serão particularizadas alguns aspectos que os assistentes sociais e psicólogos neces-
sitam conhecer minimamente para realizar seus estudos bem como proceder às intervenções necessárias no sentido da
garantia de direitos.

Bibliografia
BRASIL. Atuação dos profissionais de Serviço Social e Psicologia. Manual de procedimentos técnicos Tribunal de Justiça
do Estado de São Paulo. v.i. Infância e Juventude

119
CONHECIMENTOS ESPECÍFICOS - PSICÓLOGO

O PAPEL DO PSICÓLOGO JURÍDICO NA VIOLÊNCIA A função do profissional psi consiste em interpretar a


INTRAFAMILAR: POSSÍVEIS ARTICULAÇÕES comunicação inconsciente que ocorre na dinâmica familiar
e pessoal [...] Seu objetivo é destacar e analisar os aspectos
Dentre os diversos ramos que a psicologia jurídica psicológicos das pessoas envolvidas, que digam respeito
pode abordar, o presente artigo trata do papel do psicó- a questões afetivo-comportamentais da dinâmica familiar,
logo forense no que se refere à violência intrafamiliar, bem ocultas por trás das relações processuais, e que garantam
como sua articulação com demais instâncias envolvidas. os direitos e o bem-estar da criança e/ou adolescente, a fim
O campo da violência doméstica é um “terreno mo- de auxiliar o juiz na tomada de uma decisão que melhor
vediço”, como afirma Miranda (1998), em que se mesclam atenda às necessidades dessas pessoas. (SILVA, 2003)
fantasia e realidade, cena que causa horror e curiosidade. Como afirma Miranda (1998), constituiu-se a partir de
Diante do número imenso de variáveis culturais e psíquicas, então uma nova área de prática dos psicólogos: a psicolo-
torna-se muito complexa a tarefa de bem lidar com este gia jurídica. O lugar ocupado por esta ainda é pouco defini-
problema. do. A relação entre a psicologia e as práticas jurídicas ainda
O estudo da violência em suas causas, consequências, se dá de forma estremecida e o lugar do psicólogo nesta
objetivos, justificativas, ganhou atualidade, parecendo de- área ainda está por se configurar.
pender desta compreensão a possibilidade de sobrevivên- No que se refere à violência doméstica, intervir na fa-
cia da humanidade e a construção de alternativas para um mília para proteger a criança representa um dilema: qual é
futuro melhor. o limite entre a proteção aos direitos da criança e o respeito
Pode-se pensar na violência intrafamiliar como toda à convivência familiar? Que nível de violência intrafamiliar
ação ou omissão que prejudique o bem-estar, a integrida- justifica a intervenção? Em que circunstâncias afastar uma
de física, psicológica ou a liberdade e o direito ao pleno criança de seus pais biológicos pode representar um be-
desenvolvimento de outro membro da família. Pode ser nefício?
cometida dentro ou fora de casa por algum membro da Pensando nesta família como doente – sustento a idéia
família, incluindo pessoas que passam a assumir função pa- de que famílias que maltratam têm como característica
rental, ainda que sem laços de consanguinidade, e em rela- básica o sofrimento psíquico, ou ainda são portadoras de
ção de poder à outra. Portanto, quando se fala de violência transtornos mentais – evidencia-se a necessidade de auxí-
intrafamiliar deve-se considerar qualquer tipo de relação lio, independente da decisão que vai ser tomada a poste-
de abuso praticado no contexto privado da família contra
riori. Talvez a única alternativa em algumas situações seja o
qualquer um de seus membros. Deve-se ainda ressaltar
afastamento, mas nunca sem antes usar de todos os recur-
que o conceito de violência intrafamiliar não se refere ape-
sos possíveis para a reestruturação familiar.
nas ao espaço físico onde a violência ocorre, mas também
Quando se fala em recursos tem que se pensar que
às relações em que se constrói e efetua.
eles ainda são muito escassos. A maior parte das famílias
No que se refere à Psicologia Jurídica seu surgimento
não recebe apoio adequado para enfrentar a situação e ter
é bastante recente. A participação do psicólogo nas ques-
possibilidades de revertê-la. Para verificar esta realidade
tões judiciais começou em 1980, no Tribunal de Justiça do
Estado de São Paulo, quando um grupo de psicólogos vo- basta observar os casos acompanhados pelos Conselhos
luntários orientava pessoas que lhes eram encaminhadas Tutelares, onde o que se observa é o pouco empenho de-
pelo Serviço Social, basicamente apoio a questões fami- dicado aos mesmos, uma vez que de nada adianta enca-
liares, tendo como objetivo principal sua reestruturação e minhar para tratamento e programas se não é dada uma
manutenção da criança no lar. Mais tarde, a Lei nº 500 do atenção especial a estas famílias, um “empurrãozinho” para
CPC instituiu a contratação do Psicólogo, a título precário, que as coisas aconteçam. Essa falta de amparo se dá tanto
por um ano, podendo ser recontratado após esse período. em relação ao núcleo familiar quanto à criança afastada.
Em 1985, o presidente do Tribunal de Justiça apresentou à Como consequência, percebe-se, muitas vezes, que crian-
Assembleia Legislativa um projeto criando o cargo de psi- ças afastadas da família por maus-tratos não retornam aos
cólogo judiciário, o que significou a consolidação do posto lares de origem.
de psicólogo no sistema judiciário. Para enfrentar a violência doméstica são necessárias,
além de medidas punitivas, ações que estejam voltadas
A relação entre os saberes construídos pela Psicologia, para a prevenção, e, ainda, medidas de apoio que permi-
o Direito e as práticas judiciárias é muito antiga, mas ainda tam, por um lado, à vítima e à sua família ter assistência so-
pouco conhecida no Brasil. A partir da complexidade com cial, psicológica e jurídica necessárias à recomposição após
que foram se constituindo as regras de convivência huma- a violência sofrida e, por outro lado, que proporcionem a
na, as bases da lei foram se complexificam e absorvendo possibilidade de reabilitação dos agressores.
cada vez mais contribuições dos diversos campos do saber. Apesar da necessidade que as famílias nestas situações
Brito (1999) nos fala que a idéia de que todo o Direito, têm de auxílio psicológico, há entraves para a consolidação
ou grande parte dele, está impregnado de componentes da prática psi na instituição judiciária. Porém, hoje se sabe
psicológicos justifica a colaboração da Psicologia com o também que é pouco provável que haja benefícios na ação
propósito de obtenção de eficácia jurídica. que se contenta em localizar agressores e vítimas, punir os
Em se tratando de violência perpetrada no lar estamos primeiros e proteger os segundos. A violência, produto da
adentrando na Psicologia Jurídica aplicada à área Civil. cultura que explode em relações interpessoais, deve ser
Dessa forma podemos pensar que: vista de modo mais abrangente.

120
CONHECIMENTOS ESPECÍFICOS - PSICÓLOGO

Se a ótica é de proteção à família, a transferência da nhamento tanto do abusado como do abusador não são
pena exclui o argumento, na medida em que a vítima e contemplados pelo sistema. Resta desatendida a recomen-
demais membros do grupo familiar do agente criminaliza- dação mais importante, ou seja, a que sugere que os Es-
do serão, por extensão, também penalizados tendo, muitas tados introduzam em suas legislações nacionais processos
vezes, que sair de seus lares para que sejam protegidos. Es- alternativos de compensação e de consolidação para a
tas são ainda as medidas mais comuns: ou se tira o agres- solução dos conflitos. Estes incluiriam a possibilidade de
sor, ou se afasta as crianças, colocando-as em instituições tratamento tanto para as vítimas quanto para o abusador.
por período indeterminado. O tratamento fica para segun- O objetivo de tais propostas é restabelecer (ou estabelecer)
do plano. um equilíbrio justo entre vítima e autor, prescindindo do
As instituições que prestam serviços - jurídicos, poli- processo penal (quando isto é possível). Assim, evitar-se-ia
ciais, de saúde e educação - ainda não contam, em sua a estigmatização de ambos (vítima e autor), possibilitando-
maioria, com sistemas de diagnósticos e registros apro- -se uma resposta mais eficiente aos anseios das partes en-
priados. A ciência avançou tremendamente, os modelos volvidas, principalmente das vítimas.
diagnósticos evoluíram significativamente e os juristas bra- Diante desse fato, constata-se no exercício da “Lei” a
sileiros se debatem numa questão primitiva - a questão da forte presença da punição e a pouca importância que se dá
materialidade nos casos de maus-tratos contra crianças e à reabilitação ou, ainda mais longe, à atenção primária, à
adolescentes sem lesões orgânicas compatíveis. Os diag- qualidade de vida das pessoas.
nósticos psicológicos não falam da “materialidade” espera-
da, tanto que os laudos de avaliação psicológica costumam Compete às equipes de Saúde da Família conhecer,
ser desprezados nos tribunais. Estes falam de marcas que discutir e buscar a identificação dos fatores de risco na
não são visíveis de forma concreta, mas que nem por isso população adscrita, para facilitar a definição de ações a
representam menor dano ao desenvolvimento do sujeito, serem desenvolvidas, com a finalidade de intervir preven-
muito pelo contrário, são danos que, diferente de um ma- tivamente ou confirmar um diagnóstico, visando a adoção
chucado provocado por uma agressão física, podem ficar das medidas adequadas às diversas situações de violência
impressos para o resto da vida. intrafamiliar.
A lei no Brasil ainda exige exame de corpo de delito. É necessário considerar a complexidade da sociedade
No Rio Grande do Sul, os Institutos Médico-legais (IMLs) brasileira organizada em distintos setores que devem
não possuem sequer uma sala reservada ou cama gineco- ser envolvidos, de forma articulada, na luta contra a
lógica para os exames. Conforme afirma Caminha (2000): violência: ONGs, mídia, partidos políticos, associações
de classe, associações de base, Igreja, empresariado,
“Não podemos esperar, também, que um sujeito que movimentos sociais, escolas e universidades, dentre
está com as câmaras frigoríficas cheias de cadáveres a se- outros. A instrumentalização de instâncias alternativas com
rem analisados, além dos costumeiros acidentes de trân- tal envergadura de envolvimento sociocomunitária é o
sito, brigas e etc., goze de grandes talentos e delicadezas caminho mais propício para a concretização desse objetivo.
para atender crianças e adolescentes maltratados”. Tanto mais que o sistema judiciário representa uma opção
O perito, neste caso, é um pouco vítima, já que o Esta- cruel e incongruente não apenas para o agressor, mas
do não lhe fornece nem condições materiais nem formação também para a própria vítima. Este tem se mostrado
técnica compatível para o atendimento destes casos. Como incapacitado para cumprir qualquer função preventiva e/
produto de tais exames temos um segundo abuso igual- ou reparatória nos casos de violência doméstica.
mente traumático. Este fato vem corroborar com a idéia de Cervini (1990)
Todos estes fatores, somados ao desconhecimento e quando argumenta que, na gradação de opções de contro-
temor da sociedade frente à dinâmica das relações intrafa- le, o procedimento jurídico formal é tão-somente uma das
miliares violentas, levam as pessoas (tanto vítimas quanto soluções possíveis, sem dúvida a mais impessoal e onerosa,
agentes sociais) a evitar olhar para ela. Entretanto, é preciso mas não necessariamente a mais eficaz, para a solução de
assinalar que, a cada dia que passa, esses aspectos vêm lodosos conflitos.
sendo superados, haja visto o número cada vez maior de Quanto ao papel do psicólogo, constata-se a necessi-
denúncias realizadas. dade de um olhar mais amplo, que contemple, além das
Com isso pode-se questionar: que tipo de auxílio e demandas particulares de cada sujeito (tratamento do abu-
proteção é oferecido pelo Estado nos casos de violência sador e do abusado), um envolvimento maior com o social,
doméstica? A resposta encontrada é que o sistema penal, pois não se pode descolar a violência do contexto social
confirmando uma tendência que não é nova, acaba por jo- em que ela está inserida.
gar na vala comum todos os conflitos domésticos, sem que Benevides (2002) nos fala sobre a articulação entre
se possa diferenciar os casos e com isso constatar que, em saúde mental, direitos humanos e profissionais psi, mos-
muitos deles, a ocorrência de transtornos mentais e cultu- trando que as situações sociais, aquelas em que se com-
ras familiares que se propagam estão presentes. partilham deveres e direitos, são geralmente percebidas
como pertinentes ao campo das ciências jurídicas, das
Na maior parte dos casos, o trabalho do Estado en- ciências sociais. Ressaltando que esta clara dicotomia - de
cerra-se na constatação da violência sofrida e na busca um lado o indivíduo, de outro a sociedade - não se instala
da preservação da criança de outros abusos. O acompa- sem consequências.

121
CONHECIMENTOS ESPECÍFICOS - PSICÓLOGO

O caminho que pode conduzir a uma resposta coeren- cientes de estar lidando com um conflito delicado, bem
te deve ser trilhado de dentro para fora do judiciário. Abri- como atentos a uma eventual escalada desse conflito, que
-lo passa pela valorização da vontade das vítimas, que pre- possa porventura desaguar em consequências mais sérias
tendem, na verdade, ao aportar neste sistema, encontrar (um homicídio, por exemplo).
aí uma instância mediadora capaz de deter a escalada da É importante, ainda, que os mediadores possam
violência e de assessorá-las na empreitada de repactuação contar com supervisão periódica por parte da equipe
de sua convivência doméstica. multidisciplinar. Para tanto, deve ser-lhes oportunizado
Analisando formas de intervir nesta problemática, Her- trazer, ao menos a cada trinta dias, os casos sob seu
mam (2000) nos apresenta o exemplo do Canadá. Este país atendimento à discussão grupal com outros mediadores,
propôs soluções extraprocessuais para delitos ocorridos sob a coordenação de profissional habilitado.
no seio familiar ou no meio social que o circunda, visan- A política de prevenção deve atingir, sensibilizando e
do resolver a interação vítima-autor através de instâncias capacitando, todos os atores que tenham contato com pes-
de compensação. Em tais instâncias informais buscam-se soas vítimas de violência nas diferentes etapas do processo.
soluções através da atuação de equipe multidisciplinar em Isto inclui os profissionais de saúde, os agentes policiais,
serviços comunitários visando o aconselhamento, cuidado membros do Poder Judiciário, psicólogos e assistentes so-
e tratamento familiar. Fazendo com que muitos casos pos- ciais.
sam ser resolvidos sem que precisem chegar ao sistema A abordagem deve ser multidisciplinar, sendo que a
judiciário. assistência ambulatorial ou hospitalar precisa ser criterio-
Num segundo momento, em 1985, o Canadá formou a samente decidida pela equipe, particularizando cada caso.
Rede Pró-Justiça Comunitária e Solução de Conflitos com O trabalho junto à família é imprescindível e não deve ser
o propósito de viabilizar a troca de informações, a capaci- apenas pontual. Essa família dever ser acompanhada du-
tação de intermediários sociais e a instrumentalização de rante um período que permita avaliar suas demandas, pro-
novas instâncias de justiça informal. pondo-se a partir de então intervenções adequadas.
Pensando ainda no contexto brasileiro, algumas pro-
Através da rede interagem mediadores, psicólogos, postas neste sentido já estão sendo executadas. Estados
advogados, docentes, investigadores sociais, adminis- como São Paulo, Ceará, Pernambuco, Minas Gerais, pos-
tradores, pessoal da justiça penal e os que são partes no suem algumas localidades onde estão sendo produzidos
conflito, sendo dessa forma atacados vários núcleos, tais trabalhos inovadores no que diz respeito ao tratamento
como rixas de vizinhos, reconciliação delinquente/vítima, prestado à saúde mental e saúde da família como um todo,
conflitos familiares e ambientais, violência contra a crian- fazendo com que muitas questões possam ser resolvidas
ça e o adolescente, enfim, alternativas de tratamento que de forma mais humana, priorizando a saúde ao invés das
evitem o processo penal e priorizem a proteção das vítimas punições.
em geral. Estes exemplos nos mostram tentativas de resolução
A experiência canadense, como um todo, embora ain- de problemas que ficam mais centradas nas comunidades
da minoritária, apresenta soluções que representam uma e muitas vezes não necessitam passar pelo sistema judiciá-
proposta concreta de retomada, por parte da sociedade, rio. É um caminho promissor, que trabalha com sistemas
de conflitos que lhe dizem respeito diretamente, abrindo mais compartimentalizados, prestando uma atenção foca-
uma trilha que reinsere autor e vítima. É muito menos one- lizada nas necessidades de cada população.
rosa que o processo formal e o encarceramento: enquanto Dessa forma, conclui-se que a articulação entre os pro-
opção alternativa, representa um dispêndio anual de dois fissionais que trabalham com a violência intrafamiliar ainda
dólares por habitante, os encargos inerentes à operaciona- precisa ser bastante trabalhada, embora já existam alguns
lização do sistema penal oficial chegam a atingir mais de movimentos neste sentido. Segundo Silva (2003), desde
cem dólares por habitante/ano. Entretanto, é preciso frisar 1980 os juízes vêm sendo sensibilizados, através de um tra-
que a atuação da rede só atinge oito a dez por cento dos balho de esclarecimento, sobre a importância do aspecto
conflitos conhecidos. dinâmico e emocional e sobre a compreensão do que é
Uma das opções que poderia ser adaptada à realidade subjetividade, uma vez que esta visão começa a fazer parte
brasileira e especificamente aos casos de conflito domésti- da formação dos juízes na Escola Superior de Magistratura.
co é o “Serviço de mediação comunitária”, que administra Um dos pontos de entrave para este processo é o fato da
a interveniência de mediadores sociais, os quais sugerem rotatividade de juízes na Vara ser muito grande, dificultan-
caminhos para a composição extrajudicial. do o trabalho. Há diversidade de opiniões que passam pela
A estrutura da instância alternativa deve contar com instituição, de pessoas com diferentes orientações, des-
o trabalho de mediadores leigos, comunitariamente próxi- de aquele juiz que acredita na importância das questões
mos dos protagonistas do conflito, mas previamente pre- emocionais, da doença mental e que faz cursos de aper-
parados para a função por técnicos especializados, de pre- feiçoamento na área psi para melhor servir a população,
ferência por equipe multidisciplinar (psicólogo, assistente até aquele que vê a psicologia como uma área avessa ao
social, advogado). Essa preparação é relevante, na medida Direito e que este tem que se valer única e exclusivamente
em que os mediadores comunitários devem estar cons- das normas.

122
CONHECIMENTOS ESPECÍFICOS - PSICÓLOGO

O Estatuto da Criança e do Adolescente (ECA, 2002) Profissional (CFP, 2005) estão em destaque a promoção
fala sobre a importância de o juiz ser dotado de sensibili- da liberdade, da dignidade, da igualdade e integridade do
dade, a fim de poder julgar na área da infância e da juven- ser humano; assim como a contribuição para eliminação
tude. E, mais do que possuir esta sensibilidade, tem o dever ou diminuição de negligências, violências, discriminação,
de agir em fina sintonia com ela em prol do bem-estar do crueldades, explorações e opressões. Dentre as subáreas
assistido. da Psicologia Forense, Leal (2008, p. 182) destaca a Psicolo-
Quanto aos Psicólogos que optam por trabalhar com gia Judiciária, sendo ela “toda prática psicológica realizada
questões jurídicas acabam, muitas vezes, formando uma a mando e a serviço da justiça”. Desse modo, o serviço do
pele de proteção que os torna rígidos, deslocando mais psicólogo está sob imediata subordinação das autoridades
para questões que dizem respeito a leis fixas, deixando de judiciárias.
lado o olhar singular às questões trazidas por cada sujeito. A atuação do psicólogo jurídico no Ministério Público,
Talvez pelo fato de que trabalhar com violência doméstica de acordo com Granjeiro e Costa (2008), tem início a partir
é trabalhar com frustrações constantes, com situações que do “momento em que o juízo solicita um estudo psicosso-
remetem a sentimentos diversos e confusos em alguns mo- cial para determinado caso”, sendo o momento em que o
mentos, onde o profissional que não está preparado aca- psicólogo é designado ao procedimento. Sobre a função
ba deixando-se levar por atravessamentos pessoais que o dos estudos psicossociais, Lima (2003, p. 16) afirma que
impedem de ter uma visão clara da situação apresentada. tem como principal dever auxiliar nas tomadas de decisões
Um novo olhar se faz necessário no entendimento des- dos magistrados, procurando promover intervenções fo-
ta prática, onde somente os testes psicológicos e as leis cais no sistema de justiça.
jurídicas não podem dar conta da imensidão existente na Segundo Granjeiro e Costa (2008), o maior objetivo do
configuração familiar, uma vez que esta traz situações e estudo psicossocial no Ministério Público é de coletar ele-
sentimentos que não podem ser mensurados unicamente mentos de prova que subsidiarão a acusação das partes em
pelo objetivo, isto é, pela mensuração e aplicação de nor- um posterior processo, se for necessário. Na Promotoria do
mas. Para isto, parcerias devem ser efetivadas e fortaleci- Idoso, o psicólogo deve buscar provas de que o idoso se
das. Os profissionais devem estar mais flexíveis, dispostos a encontra ou não em risco por meio de suas intervenções,
traçar novos percursos, criar novas alternativas que possam ao mesmo tempo em que visa a melhorar a situação em
que o mesmo se encontra, garantindo os direitos integrais
contemplar as demandas trazidas de forma mais saudável
da pessoa idosa.
possível. A violência intrafamiliar deve ser tratada e não pu-
Granjeiro e Costa (2008) entendem que a promotoria
nida. Deve-se investigar as causas, usar as pesquisas para, a
geralmente possui uma visão unilateral do caso, apenas fo-
partir de um trabalho em equipe, tornar viável a reestrutu-
cado na construção de um caso que acuse e culpabilize o
ração familiar. O que se percebe é que as instâncias envol-
agressor, sem se preocupar com a vítima ou a relação que
vidas nestes casos pouco fazem porque pouco acreditam
o agressor e a vítima possam ter. Dessa forma, consideram
em resultados positivos, tendo em vista a complexidade
fundamental cuidar da relação agressor-vítima, de forma
desta problemática. É preciso uma maior qualificação como
que haja uma mudança de mentalidade na sociedade e que
profissional e como pessoa para que possamos trabalhar possa ser promovida uma cultura de não agressão.
nesta área (tanto o conselheiro tutelar, como o psi, o assis- Com essa perspectiva, surge a técnica mediação de
tente social, o juiz....). conflitos e de práticas restaurativas. As práticas restaurati-
vas derivaram das justiças restaurativas, um movimento no
Pensando no psicólogo como facilitador da promoção campo da justiça criminal que, em vez de exclusivamente
da saúde, ele deve procurar garantir os direitos fundamen- punir os infratores, ela os sensibilizava para as consequên-
tais dos indivíduos, visando sua saúde mental e a busca cias de seus atos, colocando frente à frente vítima e agres-
da cidadania. Do contrário, será mais um agente repressor. sor. (COSTELLO; WACHTEL, J.; WACHTEL, T., 2012).

Fonte: Art. 3º É obrigação da família, da comunidade, da so-


CESCA, T. B. O papel do psicólogo jurídico na violência ciedade e do poder público assegurar ao idoso, com ab-
intrafamilar: possíveis articulações. Psicol. Soc. Vol.16 no.3 soluta prioridade, a efetivação do direito à vida, à saúde, à
Porto Alegre Sept./Dec. 2004. alimentação, à educação, à cultura, ao esporte, ao lazer, ao
trabalho, à cidadania, à liberdade, à dignidade, ao respeito
PSICOLOGIA E A DEFESA DOS DIREITOS DA PES- e à convivência familiar e comunitária. (BRASIL, 2013)
SOA IDOSA A partir do Artigo 3º do Estatuto do Idoso exposto
acima, observa-se uma série de direitos básicos da pessoa
De acordo com a citação acima, a função do psicólogo idosa que é de responsabilidade da família, da sociedade
jurídico seria contribuir com uma visão psicológica aos pro- e do Estado, assegurar e denunciar quaisquer atitudes que
cessos de justiça, seja nos aspectos de planejamento e na coloquem em risco esses direitos, considerando que sua
execução de políticas públicas, na prevenção de violências omissão também é considerada um crime (BRASIL, 2013).
ou nos aspectos de elaboração e interpretação das leis. Minayo (2005) trata a violência e os maus tratos aos
Além disso, entre os princípios fundamentais de atua- idosos como sendo sinônimos, e nessa perspectiva concei-
ção do psicólogo em seu Código de Ética tua a violência contra os idosos, como “um ato (único ou

123
CONHECIMENTOS ESPECÍFICOS - PSICÓLOGO

repetido) ou omissão que lhe cause dano ou aflição e que cool e drogas numa proporção três vezes mais elevada que
se produz em qualquer relação na qual exista expectativa os não abusadores. Isso foi também assinalado no estudo
de confiança.”. de Chaves e Costa (2003). (MINAYO apud SECRETARIA DA
A mesma autora também afirma que a violência contra SAÚDE DE SÃO PAULO, 2007, p. 33)
a pessoa idosa possui nove categorias internacionais, são De acordo com dados da Secretaria da Saúde de São
elas: Paulo (2007), entre algumas características do perfil do
a) A violência física, que seria o uso da força física agressor também são inclusas as pessoas que ingerem be-
para forçar o idoso a fazer algo que ele não deseja ou para bidas alcoólicas, drogas ilícitas, possuem algum transtorno
feri-lo e provocar dor; mental ou conflitos relacionais com o idoso.
b) A violência psicológica, que seria aquela caracteri- Além disso, a violência contra os idosos gera altos cus-
zada por agressões verbais e depreciativas com o objetivo tos na vida dos envolvidos, tanto pelo ponto de vista de
de humilhá-lo e isolá-lo do convívio social; investimento pessoal quanto pelo emocional:
c) A violência sexual, que seria o ato de obter excita- Os custos da violência contra pessoas idosas ainda que
ção, relação sexual ou práticas eróticas, por meio de alicia- não estejam suficientemente documentados têm implica-
mento e violência física; ções diretas e indiretas. Os custos diretos podem estar as-
d) O abandono, que é a ausência ou deserção dos sociados à prevenção e intervenção, assim como a presta-
responsáveis governamentais, institucionais ou familiares ção de serviços, processos jurídicos, assistência institucio-
de prestarem socorro; nal e programas de prevenção, educação e intervenção. Os
e) A negligência, que se caracteriza pela recusa ou custos indiretos referem-se a menor produtividade, baixa
omissão de cuidados de direito do idoso por familiares e qualidade de vida, dor e sofrimento emocional, a perda de
instituições; confiança e autoestima, as incapacidades e a morte prema-
f) Abuso financeiro ou econômico, que seria a utili- tura. (SECRETARIA DA SAÚDE, 2007)
zação inadequada ou ilegal de seus recursos financeiros e De um modo geral, a violência contra o idoso pode
patrimoniais; acarretar sérios problemas psicológicos, de autoestima, au-
g) A autonegligência, caracterizada por conduta do toconfiança e até antecipar a morte dessa população.
próprio idoso que coloque em risco sua saúde e seus cui- Martins e Macedo (2014) apontam a cultura patriarcal
dados. como possível fator que dificulta um maior índice de de-
núncia da violência doméstica, pois ela ainda se faz presen-
Além dessas categorias, alguns autores ainda incluem
te por meio de regras de que existem assuntos de família
a violência medicamentosa, que é descrita como “adminis-
e que devem ser resolvidos apenas nesse âmbito familiar.
tração por familiares, cuidadores e profissionais, dos me-
Também é destacado pelos autores supracitados o fato
dicamentos prescritos, de forma indevida, aumentando,
de que, com a inserção da mulher no mercado de trabalho,
diminuindo ou excluindo os medicamentos” (SECRETARIA
as crianças e os idosos acabam por ficar sobre cuidado de
MUNICIPAL DA SAÚDE, 2007, p. 30).
outros adultos e de instituições.
Segundo a Secretaria dos Direitos Humanos (2012), o
tipo de violência mais frequente no Disque Direitos Huma- Fonte:
nos (DDH) é as violações por negligência, com 68,7% dos LIMA, M. L. F. A atuação do psicólogo jurídico na pro-
casos; em segundo, violência psicológica, com 59,3%; e em motoria do idoso.
terceiro, abuso financeiro/econômico e violência patrimo-
nial, com uma porcentagem de 40,1%.
Outro dado estatístico interessante sobre a violência AVALIAÇÃO PSICOLÓGICA:
contra os idosos é que, segundo pesquisa do Tribunal de INSTRUMENTAIS E SUA PRÁTICA NA
Justiça do Distrito Federal e dos Territórios (2013), 80% dos INSTITUIÇÃO JUDICIÁRIA. A ENTREVISTA
casos foram cometidos por pessoas próximas do idoso,
PSICOLÓGICA.
como parentes, cuidadores, vizinhos. Enquanto isso, 12%
foram de caráter institucional, ou seja, violência feita por
órgãos públicos em forma de ação ou omissão, e os outros
8% não tiveram um caráter identificado. A avaliação psicológica é um procedimento clínico que
Essa mesma pesquisa, realizada em Brasília, ainda envolve um corpo organizado de princípios teóricos, mé-
apontou como principais agressores os filhos dos idosos, todos e técnicas de investigação tanto da personalidade
com uma porcentagem de 64,64% dos casos; outros pa- como de outras funções cognitivas, tais como: entrevista e
rentes, com porcentagem de 12,38% das vezes; e netos, os observações clínicas, testes psicológicos, técnicas projeti-
agressores em 8,39% das vezes. vas e outros procedimentos de investigação clínica, como
Dentre todos os fatores de vulnerabilidade dos idosos jogos, desenhos, o contar estórias, o brincar etc. A esco-
à violência familiar, a grande maioria dos estudiosos ressal- lha das estratégias e dos instrumentos empregados é fei-
ta a forte associação entre maus tratos e dependência quí- ta sempre de acordo com o referencial teórico, o objetivo
mica. Segundo Anetzberger et al (2004), 50% dos abusa- (clínico, profissional, educacional, forense etc.) e a finalida-
dores entrevistados por seu grupo tinham problemas com de (diagnóstico, indicação de tratamento e/ou prevenção),
bebidas alcoólicas. Esses autores e Chavez (2002) assinalam conforme Ocampo et al. (2005), Arzeno (2003) e Trinca
que os agressores físicos e emocionais dos idosos usam ál- (1984a).

124
CONHECIMENTOS ESPECÍFICOS - PSICÓLOGO

Nos últimos anos, o ensino e a prática da avaliação Neste cenário, cresceram entre os alunos de Psicologia
psicológica têm sido objetos de inúmeros estudos. Embo- o ceticismo em relação aos testes psicológicos e o desin-
ra desenvolvidos sob diferentes enfoques, todos eles têm teresse pela área de avaliação psicológica. As críticas mais
preocupações comuns como a qualidade da formação em frequentes dos alunos é que os testes “rotulam” e não são
avaliação psicológica, o conteúdo das disciplinas, o uso e confiáveis como instrumentos de diagnóstico e avaliação
a validação dos testes psicológicos, e a integração ensino- da personalidade, segundo apontam pesquisas (PEREIRA;
-aprendizagem e aplicação destes à prática profissional. CARELLOS, 1995; GOMES, 2000). Daí a importância de en-
Tais preocupações ganharam maior relevância com as volver docentes e pesquisadores nessa discussão não só
crescentes críticas dirigidas aos testes psicológicos, entre para resgatar o valor da área na formação profissional,
elas, a falta de respaldo científico e o mau uso e elaboração mas, especialmente, para incorporar as recentes mudanças
de laudos psicológicos, que em geral “rotulam” e repetem e oferecer aos alunos uma fundamentação teórica e
jargões psicológicos sem fundamentação teórica (PATTO, técnica mais ampla que lhes permita trabalhar com
1998). Tudo isso levou o Conselho Federal de Psicologia criatividade e flexibilidade, com as inúmeras possibilidades
(CFP) a criar, em 1997, a Câmara Interinstitucional de Ava- de diagnóstico e avaliação, tendo em vista os diferentes
liação Psicológica, com o objetivo de fazer um diagnóstico contextos e necessidades.
das condições de ensino na área, e, posteriormente, im- Sabe-se que, além das questões apontadas, a forma
plantar um Sistema de Avaliação dos Testes Psicológicos como essas técnicas são ensinadas interfere no interesse
usados no Brasil. Com a implantação desse sistema e entra- dos alunos, na apreensão e aplicação prática destas. In-
da em vigor da Resolução no 02/2003, o CFP passou a re- felizmente, há professores que continuam reproduzindo
comendar somente o uso dos testes avaliados com parecer mecanicamente o ensino de testes e técnicas sem nenhum
favorável da Comissão Consultiva. Os demais, com pare- questionamento ou articulação com as novas práticas e de-
cer desfavorável ou ainda não avaliados, continuam sendo mandas da Psicologia. Este trabalho de revisão teórica tem
usados apenas em pesquisa. como objetivos realizar uma sistematização do desenvol-
Embora essas medidas tenham sido cuidadas para dar vimento das práticas de diagnóstico e avaliação psicológi-
maior cientificidade aos instrumentos, na opinião de alguns ca, destacando suas principais influências e modelos. Em
autores ocorre um fenômeno contraditório que diz respei- seguida, a revisão focaliza duas estratégias diagnósticas
to à desvalorização dos testes psicológicos nas práticas de amplamente utilizadas pela Psicologia Clínica: o psicodiag-
avaliação. Por exemplo, Affonso (2005) comenta que, após nóstico, procedimento clínico estruturado que utiliza testes
a Resolução CRP no 02/2003 e divulgação da lista dos tes- psicológicos, e a entrevista clínica diagnóstica, que adota
tes com condições de uso, docentes e profissionais tiveram
procedimentos menos estruturados de investigação da
que rever suas estratégias de diagnóstico e avaliação psi-
personalidade, como o jogo, o brincar livre e espontâneo,
cológica. Muitos cursos de Psicologia reduziram a oferta
o desenhar e contar estórias.
de disciplinas de testes psicológicos e técnicas projetivas e,
O recorte prioriza práticas e técnicas fundamentadas
também, alteraram o seu conteúdo para dar maior ênfase
nas abordagens psicanalítica e fenomenológica. Tal escolha
às técnicas de entrevistas e a outras áreas como a Psicolo-
não significa, de forma alguma, que tais abordagens sejam
gia Hospitalar, a Psicologia Jurídica etc.
mais importantes que outras fundamentadas em outros
É possível que essa medida tenha acentuado um
processo que já vinha em curso, conforme aponta referenciais teóricos. A intenção é mostrar algumas pos-
pesquisa realizada por Alves, Alchieri e Marques (2001) sibilidades de intervenção dentro do vasto campo da ava-
sobre o panorama geral do ensino das técnicas de exame liação psicológica. Ao final, reafirma-se a importância de
psicológico no Brasil. Segundo essa pesquisa, a média contextualizar e atualizar a discussão dessas questões nos
geral dos cursos avaliados (64) é de 3,98 disciplinas, mas cursos de graduação em Psicologia, de forma a incorporar
alguns têm apenas uma disciplina de testes e técnicas de as mudanças que se processaram na área e na Psicologia,
avaliação psicológica. nos últimos anos. No momento atual isso é particularmen-
Sabe-se que essa desvalorização dos testes psicológi- te importante, tendo em vista a reforma curricular em curso
cos e, por extensão, da área de avaliação psicológica é con- nas instituições formadoras, para aplicação das Diretrizes
sequência também das mudanças ocorridas nas demandas Curriculares instituídas pela Câmara de Educação Superior
de intervenção e atuação da Psicologia, na atualidade, em do Conselho Nacional de Educação.
razão de novos processos de subjetivação e de questões
sociais e políticas que interferem diretamente na qualidade Principais influências e modelos
de vida e saúde da população e exigem de nossas teorias e
práticas constantes revisões e atualizações. Como apontam A Psicologia, assim como o desenvolvimento de suas
Féres-Carneiro e Lo Bianco (2005), no âmbito da Psicologia práticas de avaliação psicológica, foi, ao longo da histó-
Clínica isso resultou numa enorme expansão de aborda- ria, influenciada por duas principais tradições filosóficas: o
gens teóricas – psicanalíticas, fenomenológico-existenciais, positivismo e o humanismo. O positivismo, corrente filosó-
cognitivas, comportamentais, sistêmicas, corporais etc.; no fica que tem Augusto Comte (1973) como principal repre-
desenvolvimento de novas modalidades de intervenção – sentante, defende o conhecimento objetivo, por meio da
grupal, familiar, comunitária; e na atuação em outros set- neutralidade científica e da experimentação. Essa corrente
tings – instituições públicas e privadas, hospitais, unidades de pensamento fundamenta o método científico adotado
de saúde etc. pelas ciências naturais que foi, durante muito tempo, con-

125
CONHECIMENTOS ESPECÍFICOS - PSICÓLOGO

siderado “o modelo de ciência”. Na ótica positivista, o ho- Essa forma de pensar teve um papel marcante no de-
mem pode ser estudado como qualquer outro fenômeno senvolvimento de uma Psicologia humanista, influenciada
da natureza, ou seja, pode ser tomado como um objeto de por vertentes teóricas ligadas principalmente à Fenomeno-
estudo observável e mensurável. logia e à Psicanálise que enfatizam a subjetividade, a in-
Apoiam-se nessa tradição as práticas de avaliação psi- tencionalidade, o sentido e o significado das experiências
cológica, identificadas com os modelos médico e psico- (e dos sintomas), o inconsciente e a relação entre sujeito e
métrico, que caracterizam a primeira fase de atuação pro- objeto de estudo.
fissional do psicólogo – práticas que valorizam o uso dos Entre suas principais influências, estão Heidegger e
testes psicológicos, a eficiência e a objetividade do diag- Freud (FIGUEIREDO, 2004). Contra- pondo-se à visão redu-
nóstico como forma de garantir a cientificidade da psicolo- cionista da vertente positivista, a Psicologia humanista bus-
gia (TRINCA, 1984a; ANCONA-LOPEZ, 1984). cava uma compreensão global do homem, na apreensão
O modelo médico influenciou enormemente as práti- do mundo e do seu significado. Sob esse influxo, passou-se
cas de avaliação psicológica, principalmente no início da a questionar os modelos de avaliação classificatória, basea-
expansão da Psicologia, quando os psicólogos atuavam, dos apenas nos testes psicológicos (estruturados e padro-
basicamente, como auxiliares do médico no diagnóstico di- nizados). Outras práticas de diagnóstico, mais identificadas
ferencial de psicopatologias. Preocupados em avaliar com com a Psicanálise e a Fenomenologia, foram surgindo den-
objetividade, para indicar o tratamento mais eficaz, os psi- tro do chamado modelo psicológico, que deu origem ao
cólogos incorporaram às suas práticas de avaliação carac- psicodiagnóstico e a outros procedimentos de avaliação,
terísticas do modelo de diagnóstico médico, tais como: a como as entrevistas diagnósticas, com ou sem o uso de
ênfase nos sintomas, o uso da classificação nosológica e o testes ou técnicas (estruturadas ou não) de investigação da
emprego de testes (exames), para identificar determinadas personalidade.
características patológicas da personalidade do indivíduo. O psicodiagnóstico inaugurou uma nova visão da ava-
liação psicológica, diferente da realizada pelos “testólogos”
O modelo psicométrico manteve a preocupação de
da Psicometria. Ao adotar uma perspectiva clínica, mais
avaliar com objetividade e neutralidade e inaugurou uma
identificada com a teoria psicanalítica ou fenomenológi-
fase de maior prestígio da Psicologia, em que os testes
ca, distanciou-se da preocupação com a neutralidade e a
psicológicos passaram a ser usados na classificação e me-
objetividade, passando a enfatizar a importância da sub-
dida da capacidade intelectual e aptidões individuais. A
jetividade e dos aspectos transferenciais e contra transfe-
Psicometria ampliou a área de atuação da Psicologia – da
renciais presentes na relação. E o uso dos testes passou
clínica para as áreas escolar (diagnóstico de dificuldades
a ser complementado com outros procedimentos clínicos,
de aprendizagem das crianças) e profissional (seleção de
com o objetivo de integrar os dados levantados nos testes
indivíduos para funções específicas). Com essa expansão,
e na história clínica, para obter uma compreensão global
o psicólogo ganhou maior autonomia: os resultados dos da personalidade.
testes deixaram de ser obrigatoriamente entregues ao mé- No Brasil, o modelo de psicodiagnóstico, desenvolvi-
dico ou a outros profissionais; os próprios psicólogos co- do por Ocampo et al. (2005) e Arzeno (2003), tem nortea-
meçaram a prestar orientação aos pais e professores e até do o trabalho de grande parte dos profissionais da área.
mesmo aos médicos. No modelo psicométrico, tornou-se Além dele, os modelos compreensivos e fenomenológico
menos importante detectar e classificar os distúrbios psi- (ANCONA-LOPEZ, 1995; CUPERTINO, 1995; YEHIA, 1995)
copatológicos; a ênfase passou a ser dada à identificação também são bastante utilizados. O psicodiagnóstico pro-
das diferenças individuais e orientações específicas. Esse posto por Cunha (2000) é outra referência, não incluída no
modelo foi muito valorizado nos Estados Unidos, especial- recorte aqui feito, mas igualmente importante no contexto
mente durante a Segunda Guerra Mundial, quando se atri- brasileiro.
buiu à Psicologia a função de selecionar indivíduos aptos e
não-aptos para o exército, bem como avaliar os efeitos da O modelo compreensivo
guerra sobre os que retornavam (ANCONALOPEZ, 1984).
O humanismo apoia-se em correntes filosóficas que se O processo diagnóstico do tipo compreensivo, desen-
contrapõem à visão positivista e questionam a aplicação do volvido por Trinca (1984a), é outro modelo muito difundido
método das ciências naturais às ciências humanas. Defende entre os profissionais brasileiros, que trabalham com ava-
que não é possível uma total separação entre o sujeito e o liação psicológica na abordagem psicanalítica. Ele também
objeto de estudo, pois a subjetividade tem uma importân- busca uma visão totalizadora e integradora da personalida-
cia essencial: o sujeito está implicado com o seu objeto de de, por meio de uma compreensão abrangente das dinâ-
estudo, ele constitui o objeto e é constituído por ele. Se micas psíquicas, intrafamiliares e socioculturais. Para isso,
todo o conhecimento é estabelecido pelo homem, não se utiliza referenciais múltiplos – além da psicanálise, a análise
pode negar a participação da sua subjetividade, portanto é complementada com outros referenciais teóricos (teorias
não é possível estudar o homem como um mero objeto do desenvolvimento e maturação e da família). Tem ainda,
fazendo parte do mundo, pois o mundo não passa de um como características importantes, a valorização do pensa-
objeto intencional para o sujeito que o pensa (ANCONA- mento clínico e uma maior flexibilidade, na estruturação
-LOPEZ, 1984). do processo.

126
CONHECIMENTOS ESPECÍFICOS - PSICÓLOGO

O modelo compreensivo se estrutura de acordo com o Tal dimensionamento implica no estabelecimento


contexto. O uso ou não de testes psicológicos ou de outros de um programa de intervenção no caso, com avaliação,
procedimentos clínicos de investigação da personalidade acompanhamento, orientação e encaminhamento das pes-
fica na dependência do pensamento clínico empregado soas envolvidas. O compromisso do psicólogo não fica res-
(TRINCA, 1983). Na interpretação dos dados, o pensamen- trito ao fornecimento de informações ao magistrado para
to clínico funciona como um princípio organizador, define a decisão do processo judicial, mas, em trabalhar todas as
critérios, procedimentos e esquemas de raciocínio, para dimensões do caso, com vista à promoção e manutenção
integração dos dados e análise. Ele é influenciado não só de uma política de garantia de direitos da infância e ju-
pela teoria, mas, também, pela experiência clínica do pro- ventude. A intervenção psicológica nos casos, depende em
fissional, pelo contexto e pelas personalidades do cliente e grande parte, das relações estabelecidas com as políticas
do psicólogo. Para Trinca (1984b, p. 32): embora as teorias públicas e os programas sociais voltados para essa parcela
sejam fatores importantes no background do profissional, da população, cujas características precisam ser conheci-
é mister que sua atividade clínica seja empreendida com o das e contextualizadas pelo profissional. Seu trabalho não
cessa com a emissão de um parecer psicológico sobre o
mínimo de interferência de suas teorias sobre sua capaci-
caso, precisando, muitas vezes, reavaliar situações que se
dade de observar e captar os fatos relevantes.
transformam ao longo do processo judicial. Exemplifican-
do, podemos citar os casos de abrigamento provisório de
O modelo fenomenológico crianças, como medida excepcional de proteção em casos
de risco social, físico e/ou psicológico. Após a situação de
O psicodiagnóstico fenomenológico (ANCONA-LOPEZ, abrigamento, o psicólogo continua intervindo, na busca de
1995; CUPERTINO, 1995; YEHIA, 1995) introduz algumas implementação de medidas de proteção que preservem o
mudanças significativas no modelo proposto por Ocampo direito da criança à convivência familiar.
et al. (2005) e Arzeno (2003). Dentre suas inovações, desta- Essa dinâmica de atendimento, provoca várias manifes-
cam-se quatro características principais: tações do profissional nos autos, com informações e rela-
1. considera o processo psicodiagnóstico uma prática tórios circunstanciados, até que o caso seja encerrado com
interventiva: diagnóstico e intervenção são processos si- a emissão de um parecer interprofissional (social, psicoló-
multâneos e complementares; gico e do ministério público), uma sentença judicial final
2. propõe que a devolução seja feita durante o proces- com consequente determinação para o arquivamento do
so e não ao final; processo.
3. enfatiza o sentido da experiência dos envolvidos no Nesses casos, o relacionamento do psicólogo com as
processo; e pessoas, partes interessadas no processo, implica numa
4. redefine a relação paciente-psicólogo em termos de avaliação psicológica como um processo de compreensão
poder, papéis e realização de tarefas. e de intervenção, e também, no estabelecimento de reco-
No modelo fenomenológico, o cliente é um parceiro mendações terapêuticas e sociais pertinentes a realidade
ativo e envolvido no trabalho de compreensão e eventual dos implicados.
encaminhamento posterior. O psicólogo se afasta do lugar
de técnico ou especialista detentor do saber e estabelece Avaliação psicológica e perícia
com o paciente uma relação de cooperação, em que a ca-
pacidade de ambas as partes, de observarem, aprenderem Diferentemente desses casos verificatórios, há os con-
e compreenderem, constitui a base indispensável ao traba- tenciosos, cujas partes apresentam-se numa relação ju-
lho. Psicólogo e paciente se envolvem, a partir de pontos dicial litigiosa, em disputa por interesses contraditórios.
Nesses processos, as pessoas são representadas por ad-
de vista diferentes, mas igualmente importantes, na tare-
vogados, que provocam o Poder Judiciário visando a reso-
fa de construir os sentidos da existência de um deles – o
lução do conflito, com o restabelecimento dos direitos da
cliente (YEHIA, 1995).
pessoa prejudicada. Na área do direito de família, incluem-
-se os filhos e dependentes dos separados como partes
Fonte: ARAÚJO, M. F. Disponível em editorarevistas. interessadas no processo judicial. Também aqui, tal como
mackenzie.br na Justiça da Infância e Juventude. Os direitos da criança e
do adolescente prevalecem como prioridade absoluta, em
AVALIAÇÃO PSICOLÓGICA NO CONTEXTO JUDI- consonância com o Estatuto da Criança e do Adolescente.
CIÁRIO Porém, a natureza contenciosa desses casos, tem im-
plicado numa atuação pontual e específica do psicólogo,
A natureza dos processos judiciais e do rito judiciário prevista e regulamentada pelo Código de Processo Civil
utilizado para tratar a matéria em questão, determinam a como de perito.
forma de abordagem do caso pelo psicólogo. Assim, na A perícia, conforme o Código de Processo Civil, é o es-
matéria da infância e juventude, em que a maioria das tudo realizado por especialistas escolhidos pelos magistra-
questões implica em verificação da ameaça ou violação dos dos, de acordo com a matéria (Psicologia, Serviço Social,
direitos da criança e do adolescente (processos verificató- Antropologia, etc.) que funciona como “prova” no proces-
rios), cabe ao psicólogo utilizar os recursos de sua espe- so judicial, complementando as demais, tais como as do-
cialidade, para dimensionar a problemática psicológica dos cumentais, testemunhais e confessionais na resolução do
envolvidos na situação social e jurídica. processo judicial.

127
CONHECIMENTOS ESPECÍFICOS - PSICÓLOGO

Os peritos, como profissionais de confiança do juízo, Cabe também lembrar que a interface dos conheci-
assumem o compromisso de imparcialidade na avaliação mentos de psicologia, direito e serviço social pode auxiliar
dos casos, comprometendo-se a apresentar um parecer na manutenção ou na mudança de valores e crenças que
técnico psicológico sobre as questões formuladas pelo perpassam a atuação judiciária. As implicações ideológicas
magistrado e de responder aos quesitos formulados pelos do trabalho são inevitáveis, e por isso mesmo essenciais
advogados das partes e pelo ministério público. para a análise crítica das atividades desenvolvidas nesse
Ao psicólogo perito cabe fornecer um laudo psicoló- escopo de saber e poder judiciário.
gico com informações pertinentes ao processo judicial e à Representações sociais sobre família, infância e outros
problemática diagnosticada, visando auxiliar o magistrado conceitos, podem ser fortalecidos e/ou modificados pelo
na formação de seu convencimento sobre a decisão judicial conhecimento transmitido nos laudos psicológicos, cujo
a ser tomada, como forma de realização do direito objetivo conteúdo revelam, as concepções de sociedade, ciência e
das partes em oposição. Essa postura, própria do perito, de homem que dão suporte a ação profissional. Daí a ne-
determina também a dinâmica de atendimento do caso, na cessária precaução sobre os efeitos políticos e ideológicos
instituição judiciária. de nossa atuação, que vão muito além das decisões caso
Para tanto, o psicólogo estabelece um planejamen- a caso.
to da avaliação psicológicas das pessoas implicadas, com
base no estudo dos autos, isto é, de todos os documentos Fonte:
e provas que compõem o processo judicial. Os instrumen- BERNARDI, D. C. F. Avaliação Psicológica no âmbito das
tos diagnósticos são escolhidos de acordo com a formação instituições judiciárias.
teórica do profissional, das condições institucionais e da Disponível em:
situação emocional dos implicados no processo judicial. http://www.aasptjsp.org.br/ar tigo/
Considera-se e especificidade da situação judicial, em que a va l i a % C 3 % A 7 % C 3 % A 3 o - p s i c o l % C 3 % B 3 g i c a - n o -
as pessoas não escolheram a intervenção do psicólogo e -%C3%A2mbito-das-institui%C3%A7%C3%B5es-
estão numa posição defensiva, procurando fazer prevalecer -judici%C3%A1rias
seus interesses sobre terceiros, com quem, em geral, man-
DIMENSÕES DO PROCESSO DE AVALIAÇÃO PSICO-
tém vínculos afetivos conflituosos.
LÓGICA
Para as partes em oposição, está previsto o direito de
contratação de assistentes técnicos - psicólogos que esta-
A Avaliação Psicológica tem se desenvolvido na dire-
rão acompanhando os resultados da perícia realizada pelo
ção de uma extrema complexidade. Os desavisados talvez
profissional de confiança do juízo, abalizando ou não suas
não percebam, porém, as dimensões que a Avaliação Psico-
conclusões.
lógica atinge vão além da simplicidade de aplicar testes ou
Nessa situação, de acentuado sofrimento psíquico e de
fazer entrevistas com determinado objetivo.
animosidade entre as pessoas, a postura pericial acaba por
Qualquer psicólogo que pretenda trabalhar com ava-
privilegiar o magistrado como o referencial das decisões
liações deverá ter em mente as dimensões técnicas, rela-
judiciais. O compromisso ético do psicólogo perito com as cionais, éticas, legais, profissionais e sociais diretamente
pessoas atendidas, porém, dispõe a necessidade e perti- implicadas em seu trabalho. Se todas estas percepções ca-
nência de entrevistas devolutivas e encaminhamentos te- minham juntas e se inter-relacionam, é provável que pos-
rapêuticos. samos perceber a infinitude de cuidados e preocupações
O modelo pericial de atuação psicológica nas Varas da que devemos ter como profissionais que buscam, princi-
Família e Sucessões tem provocado inúmeros questiona- palmente e acima de tudo, o respeito e a “construção” da-
mentos sobre o alcance das avaliações em termos predi- quele que nos procura para se submeter a um processo de
tivos, da fidedignidade no uso dos instrumentos técnicos, Avaliação Psicológica.
uma vez que em um mesmo processo judicial, podem Avaliar nunca é simples, nem rápido, nem fácil. A res-
ocorrer laudos psicológicos conflitantes do perito e dos as- peito da dimensão técnica, o psicólogo necessita ter, antes
sistentes técnicos. de mais nada, um vasto conhecimento em relação às téc-
A postura coerente do psicólogo diante dessa realida- nicas que pretende utilizar, assim como uma possibilidade
de se faz fundamental para dirimir dúvidas sobre a vali- de crítica consciente em relação aos instrumentos de ava-
dade científica e operacional de seus conhecimentos. Ele liação que utiliza (testes, dinâmicas de grupo, observação,
precisa estar atento às limitações do caráter preditivo dos entrevista e outros). Obviamente, a formação dada pelas
instrumentos diagnósticos utilizados, bem como ao caráter Faculdades de Psicologia nesta área é insuficiente para o
situacional da avaliação realizada numa situação específica pleno domínio das técnicas e de si mesmo, principalmente,
de crise. porque ainda o nosso ensino é compartimentalizado e a
Ele deve refletir sobre as implicações éticas e políticas formação prioriza o aspecto técnico e não o científico em
de seu trabalho, considerando sempre que seus resultados geral. Aprendemos mecanicamente como aplicar diversas
podem ser determinantes da medida judicial aplicada ao técnicas, mas não experienciamos a integração dos dados
caso, embora o magistrado não esteja adstrito ao laudo obtidos, a análise acurada dos mesmos, o levantamento de
psicológico para sua decisão. hipóteses a partir dos dados coletados, a dinâmica, afinal,

128
CONHECIMENTOS ESPECÍFICOS - PSICÓLOGO

que sempre estará presente em um processo de avaliação. Principais técnicas de Avaliação Psicológica
Sempre, por melhor que tenha sido a formação do psicólo-
go, ele deve buscar cursos de pós-formação para aperfei- Entrevistas, observação, testes psicológicos, dinâmicas
çoar os seus conhecimentos. de grupo, observação lúdica, provas situacionais e outras.”
Também a dimensão relacional é importantíssima, pois Todas as técnicas acima citadas contribuem para a com-
nos informa a respeito de mecanismos transferenciais e preensão do outro (candidato, cliente, periciando, usuário,
contra transferenciais que sempre estão presentes no mo- etc.), sujeito do processo de Avaliação Psicológica. Todas as
mento da avaliação. Sem um conhecimento maior nessa técnicas são importantes para o processo de conhecimen-
área, os desavisados poderão entender manipulações to do outro, não esquecendo que a Avaliação Psicológica
evidentes, como simpatia, mecanismos de sedução como requer o planejamento adequado, a execução cuidadosa
educação e aí por diante.... O desenvolvimento da percep- do que foi planejado, a integração e análise dos dados ob-
ção mais acurada de si mesmo e do cliente somente pode tidos através de todas as técnicas utilizadas. A partir dessa
ser mais real e menos fantasioso através de um exercício integração das informações coletadas é que será possível
constante de auto percepção e autocrítica que, muitas ve- redigir ou entregar os resultados da mesma.
zes, é acompanhado de um processo psicoterapêutico.
A dimensão ética deve sempre direcionar qualquer Fonte
trabalho, especialmente este que tratamos. Falamos aqui Adriane Picchetto Machado e Célia Mazza de Souza
de respeito ao semelhante, à sua dor, às obrigações de cau- publicado na Revista Contato, do Conselho Regional de
sar o menor dano possível com a nossa intervenção e à Psicologia da 8ª Região – Ano 21 – nº 104 – Set/Out 2000
sustentação dos resultados, mesmo que havendo pressões
de todos os tipos (pais, chefias e outros). Também gostaria TESTES PSICOLÓGICOS
de ressaltar a obrigatoriedade de fazer entrevistas de de-
volução, premissa está muito esquecida pelos psicólogos, Os testes psicológicos são instrumentos de avaliação
e que pode servir como um momento muito especial de ou mensuração de características psicológicas, constituin-
crescimento para o nosso cliente, se bem realizada e levan- do-se um método ou uma técnica de uso privativo do psi-
do em conta todas as dimensões citadas. cólogo, em decorrência do que dispõe o § 1° do art. 13 da
A dimensão legal tem sido amplamente questionada: lei no 4.119/62. (Resolução CFP 002/2003)
o Estado tem o direito de investigar a vida de um cidadão Os testes psicológicos são procedimentos sistemáticos
que pretende ter um emprego ou uma Carteira Nacional de de observação e registro de amostras de comportamen-
Habilitação? Que consequências legais a interdição de um tos e respostas de indivíduos com o objetivo de descre-
membro de uma determinada família trará para o mesmo? ver e/ou mensurar características e processos psicológicos,
Qual o valor legal do uso de técnicas desatualizadas, não compreendidos tradicionalmente nas áreas emoção/afeto,
adaptadas à população brasileira? cognição/inteligência, motivação, personalidade, psicomo-
A dimensão profissional diz respeito a todas as im- tricidade, atenção, memória, percepção, dentre outras, nas
plicações e consequências de ordem profissional no mo- suas mais diversas formas de expressão, segundo padrões
mento de uma avaliação, da entrega de um laudo ou da definidos pela construção dos instrumentos. Segundo Al-
devolução de resultados. Qual é o nível de seriedade e chieri, Noronha e Primi (2003), os testes psicológicos são
isenção que esse profissional apresentou para tanto? Qual “instrumentos objetivos e padronizados de investigação do
é o seu posicionamento a respeito de uma avaliação rea- comportamento, que informam sobre a organização nor-
lizada pelo seu colega e que agora se encontra em suas mal dos comportamentos exigidos na execução de testes
mãos, em grau de recurso? Prefiro agradar o meu chefe e ou se suas perturbações em condições patológicas”. Assim,
manter o meu emprego a ser coerente com um trabalho e percebe-se que o teste dá ao profissional a possibilidade
uma imagem profissional? Quais as consequências para a de observar o comportamento de forma padronizada e jul-
classe de psicólogos de um trabalho malfeito, covarde, que gar se os comportamentos que observa durante a execu-
cede a pressões? ção deste encontram-se, segundo o próprio teste, dentro
A dimensão social se refere às reflexões mais amplas das condições observadas na população normal pesquisa-
da nossa sociedade. A instituição (Estado, empresa) tem o da durante a sua fabricação.
direito de investigar a vida de um cidadão que pretende ter A função do teste torna-se, dessa forma, medir as dife-
um emprego ou uma Carteira Nacional de Habilitação? Em renças entre indivíduos ou entre as reações do mesmo in-
que mecanismos sociais segregatórios a avaliação psicoló- divíduo em diferentes ocasiões. É importante ressaltar que
gica pode colaborar? Qual o uso deste tipo de trabalho e os testes psicológicos devem ser entendidos como instru-
que fins de manipulação ele pode ter? mentos auxiliares nesta coleta de dados que é a Avaliação
Desta forma, a avaliação psicológica nunca é simples, Psicológica e que, juntamente com as demais informações
nunca é isenta de consequências que podem ser muito organizadas pelo psicólogo, auxiliam na compreensão do
sérias para o cliente e para a imagem da nossa classe. In- problema estudado, de forma a facilitar a tomada de de-
dependentemente do local onde o psicólogo atue e dos cisões.
objetivos de sua avaliação, não é possível a prática de apli-
cações e correções de testes sem consciência e sem com-
preensão da complexidade deste trabalho.

129
CONHECIMENTOS ESPECÍFICOS - PSICÓLOGO

Classificação Partiu-se, para tanto, do princípio de que a responsa-


bilidade da escolha dos instrumentos, métodos e técnicas
Testes com respostas corretas: Funcionamento cogniti- no exercício profissional é do psicólogo, mas o Conselho
vo, conhecimento, habilidades ou capacidades; Federal de Psicologia (CFP) tem a competência para disci-
Testes nos quais não há respostas corretas: Inventários, plinar e fiscalizar o exercício profissional.
questionários, levantamentos, testes de personalidade, Assim, foi publicada, a Resolução CFP 02/2003, que de-
motivações, preferências, atitudes, interesses, opiniões, re- fine e regulamenta o uso, a elaboração e a comercialização
ações características. de testes psicológicos (ver Anexo 1). Nessa Resolução fo-
“Testes são ferramentas. São um meio para se alcançar ram definidos os requisitos mínimos que os testes precisam
um fim. Podem ser mal aplicados, o que limita ou anula sua apresentar para serem aprovados pelo Conselho:
utilidade. A responsabilidade última pelo uso e interpreta- - Fundamentação teórica;
ção apropriados dos testes é do psicólogo.” - Evidências empíricas de validade e precisão das inter-
Os testes psicológicos, ao longo da história da Psico- pretações propostas;
logia, foram protagonistas de uma série de problemáticas - Sistema de correção e interpretação dos escores;
- Descrição clara dos procedimentos de aplicação e
que contribuíram para o seu descrédito e pouca utilização.
correção; Manual contendo as informações.
De forma geral, a principal problemática se refere ao uso
inadequado das técnicas, sendo que, infelizmente, muitos
As técnicas devem ser apresentadas pelas editoras ou
psicólogos as usam de forma pouco criteriosa, com conhe-
laboratórios de Avaliação Psicológica, sendo que a primeira
cimento prévio pouco cuidadoso, não fazendo a análise lista de testes favoráveis foi publicada em 06 de novembro
necessária quanto a atualização das mesmas, utilizando de 2003. A lista publicada não é estática nem definitiva.
procedimentos que não apresentam condição de cientifi- Será considerada falta ética a utilização de testes psicológi-
cidade. cos que não constam da relação de testes aprovados pelo
Podemos considerar que a formação nesta área, nos CFP, salvo os casos de pesquisa.
cursos de graduação em Psicologia, tem contribuído para Tal medida foi alvo de muita discussão e polêmica, sen-
essas problemáticas, pois, de forma geral, os programas do que os psicólogos não acompanharam de forma mais
são estanques e primam pelo tecnicismo, ensinando as téc- próxima o processo como um todo, tendo sido recebida
nicas de avaliação de forma isolada e não como integrantes a primeira lista publicada como uma surpresa. Assim, os
de um processo dinâmico, como entende-se a Avaliação serviços de grande porte (Detran, Avaliação de Porte de
Psicológica. Arma, Avaliação de Vigilantes, etc.) tiveram que se adequar
Essas condições contribuíram para a pouca considera- rapidamente às novas normas, o que gerou, na época, mui-
ção pela própria classe em relação às técnicas de avaliação. tas dificuldades.
A partir das muitas reflexões realizadas nos Fóruns Regio- Posteriormente à publicação da primeira listagem, em
nais de Avaliação Psicológica e no Fórum Nacional de Ava- 2003, os psicólogos foram pouco a pouco investindo no
liação Psicológica, ocorridos em 1999 e promovidos pelo conhecimento de novas técnicas, apesar da resistência
Sistema Conselhos, ampliaram-se as discussões a respeito apresentada por grande parte dos usuários dos mesmos,
desse tema e foram localizadas algumas opções para mini- em deixar de usar esta ou aquela técnica com a qual já
mizar ou acabar com as problemáticas referidas, entre elas: estava acostumado.
- Oferecer uma formação integradora, que ofereça De forma geral, passados quase quatro anos da ini-
condições técnicas, éticas e críticas - reflexivas ao aluno; ciativa, pode-se perceber que os profissionais tiveram que
- Aprimorar os programas de formação - menos buro- investir na remodelagem dos seus processos de avaliação,
cracia, mais dinâmica; abrindo-se para novos testes, fazendo reciclagens e super-
- Desenvolver e validar novos instrumentos de avalia- visões, o que, por si só, já pode ser considerado um grande
ganho nesse trabalho.
ção;
Para que um teste possa ser utilizado pelos profissio-
- Possibilitar o acesso ao conhecimento em formação
nais da Psicologia, ele necessita responder a requisitos es-
continuada;
senciais. O primeiro grande ponto a ser discutido é a vali-
- Avaliar os instrumentos à disposição no mercado.
dade do teste em questão. Ao produzir um instrumento de
Segundo pesquisa desenvolvida por Noronha, Primi e avaliação, os autores devem estar certos de que ele men-
Alchieri, em 2004, apenas 28,8% dos testes apresentam as surará o que está se propondo. Em outras palavras, os au-
três condições mínimas (precisão, validade e padronização) tores precisam esclarecer se o teste de atenção em questão
para ser considerado um instrumento cientificamente fun- realmente mede atenção.
damentado. Outro ponto refere-se à fidedignidade do teste. Este
Com base neste diagnóstico, o plano de ação foi colo- assunto envolve o fato de o escore obtido na aplicação se
cado em prática, cabendo ao Conselho Federal de Psico- aproximar ao escore verdadeiro do sujeito. Ou seja, ao apli-
logia as medidas relativas à avaliação dos instrumentos de car um teste de atenção e obter o escore médio, significa
avaliação psicológica. Para tanto, foi organizado o Sistema que o indivíduo possui realmente uma atenção dentro da
de Avaliação dos Testes Psicológicos (SATEPSI) média? A fidedignidade é a “confiabilidade” do instrumen-
to de testagem ou a sua “precisão”.

130
CONHECIMENTOS ESPECÍFICOS - PSICÓLOGO

Ao formalizar um teste, os autores precisam estabele- Antes da aplicação do teste, é importante que o profis-
cer “regras” que sejam comuns a todos os usuários: é a sional observe o avaliando (ou o grupo de avaliados) nos
padronização, ou seja, a uniformidade dos procedimentos seguintes aspectos: condições físicas (medicações, estado
tanto de aplicação quanto de pontuação do teste. Isso ga- de cansaço, problemas visuais e auditivos, alimentação etc)
rantirá que todos os psicólogos que fizerem uso do mate- e condições psicológicas (problemas situacionais, altera-
rial em um mesmo sujeito, por exemplo, obterão o mes- ções comportamentais), procurando identificar possíveis
mo resultado. Por isso é muito importante ressaltar que, situações que possam influenciar na qualidade do desem-
quando o profissional muda uma consigna está ferindo os penho do sujeito.
princípios de padronização do teste e perdendo a confiabi- Além disso, estabelecer um bom rapport é imprescin-
dível: no início da avaliação é importante salientar os ob-
lidade de seu resultado.
jetivos da mesma, procurando esclarecer todas as dúvidas.
Por fim, o teste utilizado pelo psicólogo deve corres-
Durante a aplicação propriamente dita, é de respon-
ponder à realidade em que está sendo utilizado. Isso sig- sabilidade a observação dos princípios de padronização
nifica que ele deve estar adaptado às realidades social, do teste: reduções não previstas e instruções diferentes do
econômica e política do Brasil. Dessa forma, muitos tes- que estabelece o manual ferem os princípios básicos de
tes que são apenas traduzidos para a língua portuguesa sua utilização. Deve-se, no entanto, seguir rigorosamente
não são testes que podem ser utilizados pela Psicologia. as orientações do manual sem assumir postura estereoti-
Esses instrumentos necessitam passar por um processo de pada e rígida.
pesquisa em que será estudado e aplicado em populações Além disso, a utilização de materiais fotocopiados po-
específicas para poder ser considerado apto à população dem gerar problemas futuros.
daquele país. Para que se obtenha um desempenho adequado dos
avaliados, deve-se também observar a postura do psicólo-
A escolha dos instrumentos go na hora da execução: aplicar os testes de forma clara e
objetiva, inspirando tranquilidade e evitando, assim, acen-
Conhecer os princípios que regem a elaboração dos tuar a ansiedade situacional típica da avaliação.
testes é fundamental. Entretanto os profissionais, quando O trabalho pode ser facilitado se o psicólogo habituar-
em sua prática, ficam em dúvida quanto ao qual instrumen- -se a registrar os eventos de forma pormenorizada, ou seja,
to deverá utilizar para avaliar determinada característica. É os comportamentos durante a aplicação. Isso facilitará na
hora da sistematização dos dados e ajudará o profissional
importante ressaltar que não existem instrumentos melho-
na conclusão de seu trabalho.
res ou piores – visto que todos passaram por um processo
científico de elaboração – e sim instrumentos mais indica- A avaliação dos testes
dos ou menos indicados para determinada situação.
Os pontos abaixo podem ajudar o profissional a pensar Após a aplicação dos testes, a próxima etapa corres-
melhor e a escolher o teste mais adequada para a ocasião: ponde à avaliação dos mesmos. Deve-se, neste momento,
Que atributos ou características se quer avaliar: perso- observar todos os princípios contidos no manual e segui-
nalidade, atenção, inteligência etc.; -los com rigorosidade. A entrega dos resultados e a entre-
Quais as técnicas disponíveis e aprovadas: técnicas que vista de devolução baseiam-se na boa avaliação. Após isso,
constam na lista do CFP como aprovadas; é importante observar o resguardo e o sigilo das informa-
Idade, escolaridade, nível socioeconômico etc. do tes- ções, assegurando a guarda adequada dos materiais. Tra-
tando: perfil da pessoa a ser avaliada; taremos desse assunto de forma mais específica um pouco
Familiaridade com o instrumento: ter conhecimento adiante.
prévio do material antes de sua aplicação; A integração das informações obtidas quando há a uti-
Qualidade do instrumento: confiabilidade do material lização de mais de uma ferramenta de avaliação torna-se
mediante indicação ou não do CFP; Materiais originais: uti- um processo demorado, porém fundamental. Para que se
lização de materiais da editora e nunca fotocópias. possa ter uma real visão do objeto de estudo, é preciso
Dessa forma, há testes que não podem ser utilizados expor para si todas as informações obtidas e organizá-
-las. Identificar padrões de comportamento é o primeiro
com pessoas de baixa escolaridade, adultos, etc. e há ins-
passo, ou seja, quando o sujeito, em diferentes momen-
trumentos que exigem do profissional mais treino do que
tos, apresentar formas semelhantes de reação. Além disso,
outros. Isso deve ser pensado antes da formalização e da essas informações deverão ser correlacionadas com o que
escolha dos materiais que se pretende utilizar. se conhece sobre a história do indivíduo e de sua família,
bem como o registro contratransferencial do profissional,
Cuidados na aplicação dos testes levantando hipóteses explicativas. Deve-se, também, pro-
curar informações incoerentes e estudá-las, investigando e
Na prática dos profissionais, percebem-se muitos erros fornecendo respostas para elas.
e muitas dúvidas na hora da aplicação. Como é uma situa- A integração de todos os dados de uma avaliação psi-
ção que envolve pessoas e, dessa forma, são situações em cológica (entrevistas, testes, observações, dinâmicas) não
que fatos inesperados podem ocorrer, é importante que o é um trabalho fácil e exige do psicólogo muito treino e
psicólogo observe alguns pontos. competência.

131
CONHECIMENTOS ESPECÍFICOS - PSICÓLOGO

Observação ENTREVISTA

A avaliação psicológica é no seu fim último uma pes- A entrevista psicológica é uma conversação dirigida a
quisa. E fazer pesquisa implica em desenvolvimento de um um propósito definido de avaliação. Sua função básica é
repertório específico. Este repertório contempla a habili- prover ao avaliador subsídios técnicos acerca da conduta
dade de manejar diferentes ferramentas. Dentre as ferra- do candidato, completando os dados obtidos pelos demais
mentas disponíveis para a utilização do psicólogo em um instrumentos utilizados.
processo de avaliação, a observação é uma das principais. A entrevista é uma técnica de investigação científica em
“Observar é estudar, examinar, olhar com atenção, pes- psicologia, sendo um instrumento fundamental do método
quisar minuciosamente.” clínico. (...) Compreende o desenvolvimento de uma rela-
A observação é um método mais aberto de avaliação ção entre o entrevistado e o entrevistador, relacionada com
psicológica e sem dúvida o primeiro instrumento que o o significado da comunicação. Revela dados introspectivos
profissional de Psicologia aprende a utilizar. Assim, o seu (a informação do entrevistado sobre os seus sentimentos e
treino é fundamental para que haja clareza e exatidão nas
experiências), bem como o comportamento verbal e não-
informações coletadas.
-verbal do entrevistador e do entrevistado. (Cunha, 1986)
Observar significa tornar mensurável o comportamen-
Apesar de suas vantagens, a entrevista está sujeita a
to que se expõe por parte do sujeito que o manifesta. O
comportamento observado também produz reações (sen- interpretações subjetivas do examinador (valores, estere-
timentos, respostas) no observador, que o auxiliam a for- ótipos, preconceitos, etc.). Deve-se, portanto, planejar e
mular hipóteses sobre o mesmo. sistematizar indicadores objetivos de avaliação correspon-
dentes ao perfil examinado.
Exame do estado mental
Roteiro de entrevista
Atenção
Capacidade de concentração do psiquismo frente a de- De forma geral, os dados a serem obtidos deverão ser
terminado estímulo; observados de acordo com o contexto onde o atendimen-
Senso percepção to estará sendo realizado. Assim, em determinados contex-
Capacidade de receber um estímulo e transformá-lo tos, certas informações são imprescindíveis, enquanto que
em uma imagem; em outros, algumas informações se tornam irrelevantes.
Consciência A entrevista deve ser entendida como uma forma di-
Estado de clareza psíquica; nâmica, o que possibilitará o conhecimento necessário aos
Orientação objetivos da avaliação proposta. Consideramos como im-
Capacidade de situar-se em relação a si mesmo e ao portantes, nos mais diversos contextos, a investigação das
ambiente; seguintes informações:
Memória Dados de identificação
Capacidade de fixar, conservar, evocar e reconhecer um Dados socioculturais
estímulo; História familiar
Pensamento História escolar
Capacidade de elaborar, associar e criticar ideias. História e dados profissionais
Traduz a aptidão de elaborar conceitos, articulá-los em História e indicadores de saúde/doença
juízos e construir raciocínios de modo a solucionar proble- Aspectos da conduta social
mas;
Visão e valores associados a temática investigada
Linguagem
Características pessoais
Conjunto de sinais convencionados utilizados para se
Expectativas de futuro
expressar;
Afetividade
Capacidade de experimentar sentimentos e emoções; Ao mesmo tempo em que os dados objetivos são co-
Conduta letados, deve-se observar o processo de comunicação que
Tendência psicomotora da atividade psíquica. se estabelece entre o cliente e o profissional que o atende.
Quando se processa uma entrevista, o psicólogo tem que
Observação ter em mente que há outras formas de comunicação, além
da verbal, que seria a mais tradicional e óbvia. A comunica-
Aparência geral: ção não verbal muitas vezes é mais intensa e rica, comple-
Higiene, cuidados corporais e vestimenta mentando ou não a exposição oral.
Aspecto físico e de saúde A forma de organização espacial, a localização, os
Modo de comportar-se gestos, o olhar e a voz irão fornecer ao psicólogo treina-
Área sensorial e motora do adequadamente dados muito confiáveis a respeito dos
Visão sentimentos do cliente, assim como sobre as condições da
Audição comunicação que está acontecendo (vontade de favorecer
Movimento corporal a comunicação, bloqueios, inseguranças, etc.).

132
CONHECIMENTOS ESPECÍFICOS - PSICÓLOGO

Fonte: A entrevista psicológica pode ser também um processo


MACHADO, Adriane Picchetto. Manual de Avaliação grupal, isto é, com um ou mais entrevistadores e/ou en-
Psicológica / Adriane Pichetto Machado, Valéria Cristina trevistados. No entanto, esse instrumento é sempre em
Morona. – Curitiba: Unificado, 2007. função da sua dinâmica, um fenômeno de grupo, mesmo
que seja com a participação de um entrevistado e de um
A ENTREVISTA PSICOLÓGICA: UM BREVE HISTÓRI- entrevistador.
CO
A sequência temporal das entrevistas diagnósticas
A entrevista psicológica sofreu algumas modificações
no início do século XIX, quando predominava o modelo Essa sequência pode ser subdividida em: entrevista ini-
médico. Naquela época, Kraepelin usava a entrevista com cial; entrevistas subsequentes e entrevista de devolução,
o objetivo de detalhar o comportamento do paciente, e, caracterizadas de forma diferente, e mostrando objetivos
assim, poder identificar as síndromes e as doenças espe- distintos conforme o momento em que elas ocorram.
cíficas que as classificavam segundo a nosografia vigente.
Enquanto isso, Meyer, psiquiatra americano, se interessava a) Entrevista Inicial
pelo enfoque psicobiológico (aspectos biológicos, histó-
É a primeira entrevista de um processo de
ricos, psicológicos e sociais) do entrevistado. A partir de
psicodiagnóstico. Semi dirigida, durante a qual o sujeito
Hartman e Anna Freud o interesse da entrevista se deslo-
fica livre para expor seus problemas. Segundo Fiorini, o
cou para as defesas do paciente. Isto é, a psicanálise teve
empenho do terapeuta nessa primeira entrevista pode ter
sua influência na investigação dos processos psicológicos, uma influência decisiva na continuidade ou no abandono
sem enfatizar o aspecto diagnóstico, antes valorizado. do tratamento. Pinheiro salienta que a mesma ocorre num
Nos anos cinquenta, Deutsch e Murphy apresentaram certo contexto de relação constantemente negociada. O
sua técnica denominada Análise Associativa que considera- termo negociação se refere ao posicionamento definido
va importante registrar não somente o que o paciente dizia, como “um processo discursivo, através do qual [...] são si-
mas, também, em fornecer informações sobre o mesmo. tuados numa conversação como participantes observáveis,
Desse modo, desviou-se o foco sobre o comportamento subjetivamente coerentes em linhas de histórias conjunta-
psicopatológico para o comportamento dinâmico. Ainda mente produzidas”.
nesta década, Sullivan concebeu a entrevista como um fe- Essa entrevista, geralmente, inicia-se com a chamada
nômeno sociológico, uma díade de interferência mútua. telefônica de um outro técnico, encaminhando o entrevis-
Após este período, a entrevista e o Aconselhamento tado para a avaliação psicodiagnóstica, ou com a chamada
Psicológicos se deixaram influenciar, entre outros, por Carl do próprio entrevistado. Tem como objetivos discutir ex-
Rogers, cuja abordagem consiste em centrar no paciente. pectativas, clarear as metas do trabalho, e colher informa-
Ou seja, em procurar compreender, de acordo com o seu ções sobre o entrevistado, que não poderiam ser obtidas
referencial, significados e componentes emocionais, tendo de outras fontes. As primeiras impressões sobre o entrevis-
como base a sua aceitação incondicional por parte do en- tado, sua aparência, comportamento durante a espera, são
trevistador. dados que serão analisados pelo entrevistador, e que po-
dem facilitar o processo de análise do caso. Para Gilliéron,
Definição de entrevista psicológica a primeira entrevista deve permitir conhecer:
- O modo de chagada do paciente à consulta (por si
A entrevista psicológica é um processo bidirecional mesmo, enviado por alguém ou a conselho de alguém,
de interação, entre duas ou mais pessoas com o propósi- etc.);
to previamente fixado no qual uma delas, o entrevistador, - O tipo de relação que o paciente procura estabelecer
procura saber o que acontece com a outra, o entrevistado, com o seu terapeuta;
- As queixas iniciais verbalizadas pelo paciente, em par-
procurando agir conforme esse conhecimento. Enquan-
ticular a maneira pela qual ele formula seu pedido de ajuda
to técnica, a entrevista tem seus próprios procedimentos
(ou sua ausência de pedido).
empíricos através dos quais não somente se amplia e se
A partir dessas impressões e expectativas, entrevistador
verifica, mas, também, simultaneamente, absorve os co-
e entrevistado constroem mutuamente suas transferências,
nhecimentos científicos disponíveis. Nesse sentido, Bleger contratransferências, e resistências que foram ativadas bem
define a entrevista psicológica como sendo “um campo de antes de ocorrer o encontro propriamente dito. Um clima
trabalho no qual se investiga a conduta e a personalida- de confiança proporcionado pelo entrevistador facilita que
de de seres humanos”. Uma outra definição caracteriza a o entrevistando revele seus pensamentos e sentimentos
entrevista psicológica como sendo “uma forma especial sem tanta defesa, portanto, com menos distorções. No fi-
de conversão, um método sistemático para entrar na vida nal dessa entrevista devem ficar esclarecidos os seguintes
do outro, na sua intimidade”. Enfim, Gil compreende a en- pontos: horários, duração das sessões, honorários, formas
trevista como uma forma de diálogo assimétrico, em que de pagamento (quando particular), condições para admi-
uma das partes busca coletar dados e a outra se apresenta nistrar instrumentos de testagem e para as condições de
como fonte de informação. consulta a terceiros.

133
CONHECIMENTOS ESPECÍFICOS - PSICÓLOGO

b) Entrevistas Subsequentes nizado a história de sua vida, e um esquema de seu pre-


sente, e destes temos que deduzir o que ele não sabe. Ou
Após a entrevista inicial, em que é obtida uma primei- seja, “o que nos guia numa entrevista, do mesmo modo
ra impressão sobre a pessoa do paciente, esclarecimentos que em um tratamento, não é a fenomenologia reconhecí-
sobre os motivos da procura, e realização do contrato de vel, mas o ignorado, a surpresa”. Nessa perspectiva, Bleger
trabalho de psicodiagnóstico, via de regra são necessários compreende que, diferentemente da consulta e da anam-
mais alguns encontros. O objetivo das entrevistas subse- nese, a entrevista psicológica tenta o estudo e a utilização
quentes é a obtenção de mais dados com riqueza de de- do comportamento total do indivíduo em todo o curso da
talhes sobre a história do entrevistado, tais como: fases relação estabelecida com o técnico, durante o tempo que
do seu desenvolvimento, escolaridade, relações familiares, essa relação durar.
A entrevista psicológica funciona como uma situação
profissionais, sociais e outros.
onde se observa parte da vida do paciente. Mas, nesse
contexto não consegue emergir a totalidade do repertó-
c) Entrevista de Devolução ou Devolutiva
rio de sua personalidade, uma vez que não pode substi-
tuir, e nem excluir outros procedimentos de investigação
No término do psicodiagnóstico, o técnico tem algo a mais extensos e profundos, a exemplo de um tratamento
dizer ao entrevistado em relação ao que fundamenta a in- psicoterápico ou psicanalítico, o qual demanda tempo, e
dicação. Em 1991, Cunha, Freitas e Raymundo, elaboraram favorece para que possa emergir determinados núcleos da
algumas recomendações sobre a entrevista de devolução: personalidade. Este tipo de assistência, também não pode
- Após a interpretação dos dados, o entrevistador vai prescindir da entrevista. Esta que apresenta lacunas, dis-
comunicar-lhe em que consiste o psicodiagnóstico, e indi- sociações e contradições que levam alguns pesquisadores
car a terapêutica que julga mais adequada; a considerá-la um instrumento pouco confiável. Mas, com
- O entrevistador retoma os motivos da consulta, e a diz Bleger, essas dissociações e contradições, são inerentes
maneira como o processo de avaliação foi conduzido; à condição humana, e a entrevista oferece condições para
- A devolução inicia com os aspectos menos compro- que as mesmas sejam refletidas e trabalhadas.
metidos do paciente, ou seja, menos mobilizadores de an-
siedade; Tipos de entrevista
- Deve-se evitar o uso de jargão técnico (expressões
própria da ciência circulante entre os profissionais da área, Segundo Gil, as entrevistas podem ser classificadas em:
em outras palavras “gíria profissional”), e iniciar por sinto- informal, focalizada, por pautas e estruturada.
a) Entrevista Informal (livre ou não-estruturada) – É o
ma ligado diretamente à queixa principal;
tipo menos estruturado, e só se distingue da simples con-
- A entrevista de devolução deve encerrar com a indi-
versação porque tem como objetivo básico a coleta de da-
cação terapêutica. dos. O que se pretende é a obtenção de uma visão geral do
problema pesquisado, bem como a identificação de alguns
Diferença entre entrevista, consulta e anamnese aspectos da personalidade do entrevistado;
b) Entrevista Focalizada (semiestruturada ou semidi-
A técnica da entrevista procede do campo da medici- rigida) – É tão livre quanto a informal, todavia, enfoca um
na, e inclui procedimentos semelhantes que não devem ser tema bem específico. Permite ao entrevistado falar livre-
confundidos e nem superpostos à entrevista psicológica. mente sobre o assunto, mas quando este se desvia do tema
Consulta não é sinônimo de entrevista. A consulta consis- original o entrevistador deve se esforçar para sua retoma-
te numa assistência técnica ou profissional que pode ser da;
realizada ou satisfeita, entre as mais diversas modalidades, c) Entrevista por Pautas (semiestruturada ou semidi-
através da entrevista. A entrevista não é uma anamnese. rigida) – Apresenta certo grau de estruturação, já que se
Esta implica numa compilação de dados preestabelecidos, guia por uma relação de pontos de interesses que o entre-
que permitem fazer uma síntese, seja da situação presente, vistador vai explorando ao longo do seu curso. As pautas
ou da história de doença e de saúde do indivíduo. Embo- devem ser ordenadas e guardar certa relação entre si. O
ra, se faça a anamnese com base na utilização correta dos entrevistador faz poucas perguntas diretas e deixa o entre-
princípios que regem a entrevista, porém, são bem diferen- vistado falar livremente à medida que se refere às pautas
assimiladas. Quando este, por ventura, se afasta, o entrevis-
ciadas nas suas funções.
tador intervém de maneira sutil, para preservar a esponta-
Na anamnese, o paciente é o mediador entre sua vida,
neidade da entrevista;
sua enfermidade, e o médico. Quando por razões estatís-
d) Entrevista Estruturada (fechada) – Desenvolve-se
ticas ou para cumprir obrigações regulamentares de uma a partir de uma relação fixa de perguntas, cuja ordem e re-
instituição, muitas vezes, ela é feita pelo pessoal de apoio dação permanecem invariável para todos os entrevistados,
ou auxiliar. A anamnese trabalha com a suposição de que que geralmente são em grande número. Por possibilitar o
o paciente conhece sua vida e está, portanto, capacitado tratamento quantitativo dos dados, este tipo de entrevista
para fornecer dados sobre a mesma. Enquanto que, a hi- torna-se o mais adequado para o desenvolvimento de le-
pótese da entrevista é de que cada ser humano tem orga- vantamentos sociais.

134
CONHECIMENTOS ESPECÍFICOS - PSICÓLOGO

A entrevista quanto ao seu referencial teórico disposição: observação, hipótese e verificação. Uma boa
observação consiste, de algum modo, em formular hipó-
O processo de entrevista é orientado por seu refe- teses que vão sendo reformuladas durante a entrevista
rencial teórico. Aqui serão vistas, em síntese, algumas das em função das observações subsequentes. No entender
perspectivas: de Bleger, o trabalho do psicólogo somente adquire real
a) Perspectiva Psicanalítica – Tem como base os pres- envergadura e transcendência quando coincidem a investi-
supostos dos conteúdos inconscientes. O entrevistador gação e a tarefa profissional, porque estas são as unidades
busca avaliar a motivação inconsciente, o funcionamento de uma práxis que resguarda a tarefa mais humana: com-
psíquico e a organização da personalidade do entrevistado. preender e ajudar os outros. Assim, indagação e atuação,
A entrevista é orientada para a psicodinâmica da estrutu- teoria e prática, devem ser manejadas como momentos e
ra intrapsíquica ou das relações objetais e funcionamento aspectos inseparáveis do mesmo processo.
interpessoal;
b) Perspectiva Existencial-humanista – Não procura Segundo Bleger, a entrevista se diferencia de acordo
formular um diagnóstico, e sim, verificar se o interesse do com o beneficiário do resultado:
indivíduo está auto realizado ou não. Aqui não existe uma - A entrevista que se realiza em benefício do entrevista-
técnica específica de entrevista, estas são consideradas do, a exemplo da consulta psicológica ou psiquiátrica;
pelos existencialistas como manipulação. O entrevistador - A entrevista cujo objetivo é a pesquisa, valorizando,
reflete o que ouve, pergunta com cuidado, e tenta apenas, o resultado científico da mesma;
reconhecer os sentimentos do entrevistado; - A entrevista que se realiza para terceiro, neste caso, a
c) Perspectiva Fenomenológica – Estuda a influência serviço de uma instituição.
dos pressupostos e dos preconceitos sobre a mente, e que Com exceção do primeiro tipo de entrevista, os demais
os acionam ao estruturar a experiência e atribuir-lhe um
exigem do entrevistador que desperte interesse ou motive
significado. Além de uma atitude aberta e receptiva, é ne-
a participação do entrevistado.
cessário que o entrevistador atue como observador par-
ticipante, e que, assim, seja capaz de avaliar criticamente,
Segundo Gil, as entrevistas podem se dá em duas mo-
através de sua experiência clínica e conhecimento teórico,
dalidades: Face a face e por Telefone. A entrevista tradicio-
o que está ocorrendo na entrevista.
nal tem sido realizada face a face. No entanto, nas últimas
A entrevista quanto ao seu método décadas vem sendo desenvolvida a entrevista por telefone.
- Principais vantagens da entrevista por telefone, em
Segundo Ribeiro, a realização da entrevista psicológica relação à entrevista pessoal: custos mais baixos; facilidade
segue diferentes enfoques: na seleção da amostra; rapidez; maior aceitação dos mo-
a) Psicométrico – O entrevistador faz uso constante de radores das grandes cidades, que temem abrir suas por-
uma série de instrumentos: testes, pesquisas, controle es- tas para estranhos; facilidade de agendar o momento mais
tatístico, etc., predeterminados, enquanto dispositivos para apropriado para a realização da entrevista;
a aquisição de conhecimentos sobre o entrevistado. Nessa - Limitações da entrevista por telefone: interrupção da
situação, dificilmente o entrevistador conseguirá aprofun- entrevista pelo entrevistado; menor quantidade de infor-
dar a relação, o encontro permanece mais em nível formal mações; impossibilidade de descrever as características do
e informativo do que espontâneo, criativo e transformador. entrevistado ou as circunstâncias em que se realizou a en-
Isto não quer dizer que seja menos válida ou mais super- trevista; parcela significativa da população que não dispõe
ficial; de telefone ou não tem seu nome na lista.
b) Psicodinâmico – A relação poderá ser mais apro-
fundada devido ao fato do entrevistador contar com maior Segundo Erickson, algumas recomendações devem ser
disponibilidade de tempo para questionar o entrevistado, aplicáveis ao processo de entrevista psicológica:
e conduzir a situação de maneira “menos estruturada”. Sua - O entrevistador deve ter o cuidado para não transfor-
atenção não está no aqui e no agora, ela atende a uma mar a entrevista numa conversa social. “Como posso ajudá-
dinâmica de causa-efeito na qual sub mensagens poderão -lo?”, é uma boa maneira de se iniciar uma entrevista;
dificultar a comunicação; - O entrevistador não deve completar as frases do en-
c) Antropológico – Abrange a relação ambiente-orga- trevistado. Devem-se evitar perguntas que induzam res-
nismo na compreensão da comunicação. Qualquer dado postas do tipo “sim” ou “não”. Não interromper o fluxo do
será considerado, mas, nem sempre, é possível dizer em pensamento do entrevistado, a não ser que ele se perca em
que momento ele está e onde será utilizado. Esse tipo de ideias que fogem dos tópicos da entrevista;
entrevista parece mais complexo, assim sendo, exige mais - A atitude do entrevistador deve ser de aceitação
prática do entrevistador para analisar as informações. completa das vivências do entrevistado. Não deve haver
discussão de pontos de vista;
Técnicas de entrevista - As pausas e silêncios são, quase sempre, embaraço-
sos para o entrevistador. Nesses momentos, possivelmente,
Um dos aspectos essenciais da entrevista está na in- o entrevistado está revivendo experiências que não con-
vestigação que se realiza durante o seu transcurso. As ob- segue expressar verbalmente. Quando as pausas forem
servações são registradas em função das hipóteses que longas, o entrevistador poderá retomar um tópico anterior
o entrevistado emite. O entrevistador ordena na seguinte que estava sendo discutido;

135
CONHECIMENTOS ESPECÍFICOS - PSICÓLOGO

- O tempo de entrevista deve ser marcado, e o entre- análise da entrevista, não se deve atuar diante da rejeição,
vistado será comunicado de quanto tempo dispõe. Se ne- inveja ou qualquer outro sentimento do entrevistado. As
cessário, marca-se outra (s) entrevista (s). Deve-se limitar o atitudes deste não devem ser “domadas” ou subjugadas,
número de assuntos em cada sessão para não confundir o não se trata de querer triunfar e nem se impor perante o
entrevistado; mesmo. Compete ao entrevistador averiguar como essas
- É necessário trocar o pronome pessoal “eu”, pelo uso atitudes funcionam e como o afetam. O grau de repressão
de expressões mais vagas, tais como: “parece que ...”; “pa- do entrevistado, de um certo modo, tem uma relação dire-
rece melhor ...”; etc.; ta com o nível de repressão do entrevistador.
- Recomenda-se fazer o resumo do que fora discutido Necessariamente, o entrevistado que fala muito não
em cada final de entrevista. E que o entrevistador faça uma traz à tona aspectos relevantes das suas dificuldades. A lin-
síntese para o entrevistado do que foi abordado na sessão; guagem que é um meio de transmitir informação, mas po-
- O término da entrevista não deve transformar-se derá ser também uma maneira poderosa de se evitar uma
numa conversa social, sem nenhuma relação com os pro- verdadeira comunicação. Nem sempre, uma carga emocio-
blemas discutidos. Isto pode prejudicar o resultado da en- nal intensa significa uma evolução no processo. O silêncio
trevista. é uma expressão não-verbal que muitas vezes comunica
bem mais que as palavras. O silêncio é, geralmente, o fan-
Segundo Foddy, é aconselhável o investigador ou en- tasma do entrevistador iniciante. Ele pode ser também uma
trevistador: tentativa de encobrir a faceta de um momento o qual o
- Adotar uma atitude comum e casual. Ex. “Por acaso sujeito não consegue enfrentar. Castilho cita uma série de
você ...”; tipos de silêncio que são comuns nas dinâmicas de grupo,
- Empregar a técnica “Kinsey” de olhar os inquiridos mas que também ocorrem, com bastante frequência, no
bem nos olhos, e colocar a pergunta sem rodeios de modo processo de entrevista, etc. Para ilustrar foram destacados
a que eles tenham dificuldade em mentir; alguns tipos de silêncio:
- Adotar uma aproximação indireta de modo a que - Silêncio de Tensão – É a expressão da ansiedade. Fa-
os inquiridos forneçam a informação desejada sem terem cilmente observado através da postura corporal tensa ou
consciência disso, a exemplo das técnicas projetivas; inquieta do entrevistado, da sua respiração ofegante, do
- Colocar as perguntas perturbadoras na parte final do tamborilar dos dedos, etc.;
questionário ou da entrevista de modo a que as respostas - Silêncio de Medo – Deixa o entrevistado petrificado,
não sofram qualquer consequência desse efeito. na sua tentativa de fugir de uma situação psicologicamen-
te ameaçadora. Esse silêncio suscita muita tensão e, como
Segundo Gilliéron, pode-se estudar os comportamen- consequência, forte descarga psicossomática;
tos do paciente praticamente em relação a dois eixos: - Silêncio de Reflexão – Surge normalmente após a in-
- A anamnese do sujeito que permite a observação dos tervenção do entrevistador, ou logo após um feedback, ou
comportamentos repetitivos que dão uma ideia exata da mesmo depois do entrevistador ter passado por algum tipo
sua personalidade: trata-se do ponto de vista histórico; de vivência. Nele, observa-se a ausência de tensão, há um
- A observação do comportamento do paciente quan- recolhimento introspectivo de elaboração mental;
do da primeira entrevista também fornece indicações mui- - Silêncio de Desinteresse – O indivíduo perde o foco
to precisas sobre a organização da sua personalidade. da atenção, camufla resistência, se desinteressa pela situa-
ção externa porque interiormente ela o atinge.
Dinâmica da entrevista
A Ansiedade na Entrevista
O entrevistador, no seu papel de técnico, não deve
expor suas reações e nem sua história de vida. Não deve A ansiedade é parte da existência humana, todas as
permitir em ser considerado como um amigo pelo entre- pessoas a sentem em grau variado, por vezes consiste em
vistado e, nem entrar em relação comercial, de amizade ou uma resposta adaptativa do organismo. Para Bion, se duas
de qualquer outro benefício que não seja o pagamento dos pessoas estão numa sala de análise sem angústia, não está
seus honorários. Para Gilliéron, a investigação repousará: havendo análise. Calligaris, percebe que em todo encon-
- Na análise do comportamento do paciente com rela- tro, o outro está imediatamente implicado enquanto “se-
ção ao enquadre; melhante imaginário”, o que se busca primeiro é uma tela,
- Num modelo preciso suscetível de evidenciar a di- uma espécie de cumplicidade que supõe um sentido co-
nâmica relacional que se estabelece entre o paciente e o mum ao que estamos dizendo. Desse modo, a ansiedade
terapeuta; modelo de apoio objetal. é um indicativo do desenvolvimento de uma entrevista, e
deve ser controlada pelo entrevistador, a sua própria, e a
O entrevistado deve ser recebido com cordialidade, e que aparece no entrevistado.
não de forma efusiva. Diante de informações prévias forne- Durante a situação de entrevista, tanto à ansiedade
cidas por outra pessoa, se deixa claro que essas não serão quando os mecanismos de defesa do entrevistado podem
mantidas em reserva. Em função de não abalar a confian- aumentar, não somente devido a esse novo contexto ex-
ça do entrevistado, estas lhe serão comunicadas. A reação terno que ele enfrenta, mas também devido ao perigo, em
contra transferencial deve ser encarada com um dado de potencial, daquilo que desconhece em sua personalidade.

136
CONHECIMENTOS ESPECÍFICOS - PSICÓLOGO

O contato direto com seres humanos, coloca o técnico cida como o terreno em que se dá a problemática de um
diante da sua própria vida, saúde ou doença, conflitos e tratamento psicanalítico, pois são a sua instalação, as suas
frustrações. Considerando que o entrevistador é um agente modalidades, a sua interpretação e a sua resolução que as
ativo na investigação, sua ansiedade torna-se um dos fato- caracteriza.
res mais difíceis de lidar. Em sua tarefa, o psicólogo pode A transferência e a contratransferência são fenômenos
oscilar facilmente entre a ansiedade e o bloqueio, sem que que estão presentes em toda relação interpessoal, inclusive
isto o perturbe, desde que possa resolver na medida em na entrevista. Na transferência o entrevistado atribui papéis
que surja. ao entrevistador, e se comporta em função dos mesmos,
Toda investigação implica a presença de ansiedade transfere situações e modelos para a realidade presente e
frente ao desconhecido, e o investigador deve ter a capa- desconhecida, e tende à configurar esta última como situa-
cidade para tolerá-la, assim, poderá manter o controle da ção já conhecida, repetitiva. No entender de Gori, repetin-
situação. Há casos em que o investigador, devido aos seus do transferencialmente, evoca-se a lembrança e é somente
bloqueios e limitações, se vê oprimido pela ansiedade, e por meio da lembrança que temos acesso a história [...] Por
recorre a mecanismos de defesa para se sentir seguro, e
meio da transferência é forjado num lugar intermediário
assim, elimina a possibilidade de uma investigação eficaz,
entre a vida real e um ensaio de vida, para que o drama
uma vez que conduz a entrevista de maneira estereotipa-
humano possa ter um desfecho.
da. Um outro problema frequente diz respeito a uma certa
A articulação do conceito de “momento sensível” pas-
compulsão do entrevistador focalizar seu interesse ou en-
contrar perturbações exatamente na esfera que ele nega os sa pelo posicionamento do terapeuta. Esse instante preci-
seus próprios conflitos. so determina os mecanismos que instalam a transferência.
A manipulação técnica, de toda ansiedade, deve ser Com efeito, é o momento em que uma relação de trabalho
realizada com referência a personalidade do entrevistado, se torna possível. A abertura ao outro, a espera de ajuda
e o nível de timing (sincronização e ajustamento) que se te- vinda do exterior é forte e expõe o paciente tanto ao me-
nha estabelecido na relação. Toda interpretação fora desse lhor quanto ao pior dessa interação.
contexto implica em agressão ao paciente ou entrevista- Nessa perspectiva, Gilliéron diz que todo paciente pro-
do. Cabe ao psicólogo saber calar, na proporção inversa da cura obter alguma coisa do terapeuta. Ele não busca ape-
sua vontade compulsiva de interferir. Nessa ótica, Almeida nas a cura de um sintoma, mas também certa qualidade
& Wetzel dizem que a interpretação algumas vezes vem de relação. O entrevistado revela aspectos irracionais ou
de um desejo de intervenção com a finalidade de eliminar imaturos de sua personalidade, seu grau de dependência,
angústias (perda de continência), instados pela situação e sua onipotência e seu pensamento mágico. As transferên-
autorizados pelo setting. cias negativas e positivas podem coexistir num mesmo
Segundo Piaget, o bom entrevistador deve reunir duas processo, embora, quase sempre com predomínio relativo,
qualidades: saber observar (não desviar nada, não esgotar estável ou alterado, de uma delas. Segundo Sang, é a situa-
nada); saber buscar (algo de preciso, ter a cada instante ção analítica e não a sua pessoa o que levou a paciente a
uma hipótese de trabalho, uma teoria, verdadeira ou falsa, se apaixonar por ele, isto é, que o amor de transferência é
para controlar). Douglas corrobora com essa ideia quan- essencialmente impessoal. [...] o analista não deve nem re-
do afirma que entrevistar criativamente é ter determinação primir nem satisfazer as pretensões amorosas da paciente.
atendendo ao contexto, em vez de negar, ou não conseguir Deve sim, tratá-las como algo irreal. No que é confirmado
compreender. O que se passa numa situação de entrevista por Yalom, quando diz que os sentimentos que surgem na
é determinado pelo processo de perguntas e respostas, a situação terapêutica geralmente pertencem mais ao papel
entrevista criativa agarra o imediato, a situação concreta, que à pessoa, é um equívoco tomar a adoração transferen-
tenta perceber de que modo esta afetação vai sendo co- cial como um sinal de sua atratividade ou charme pessoal
municada e, ao compreender esses efeitos, modifica a re-
irresistível.
cepção do entrevistador, aumentando, assim, a descoberta
das verdades.
b) Contratransferência
Na contratransferência emerge do entrevistador rea-
Transferência e Contratransferência
ções que se originam do campo psicológico em que se
a) Transferência estrutura a entrevista. Porém, se constitui, quando bem
Freud (1914-1969) entende que a transferência é (...) conduzida, num indício de grande significação e valor para
apenas um fragmento da repetição e que a repetição é orientar o entrevistador no estudo que realiza. Seu manejo
uma transferência do passado esquecido (...) para todos requer preparação, experiência e um alto grau de equilíbrio
os aspectos da situação atual. A transferência é designa- mental, para que possa ser utilizada com validade e eficiên-
da pela psicanálise como um processo através do qual os cia. Na contratransferência, salienta Gilliéron, as emoções
desejos inconscientes se atualizam sobre determinados vividas pelo analista são consideradas reativas às do pa-
objetos, num certo tipo de relação estabelecida, eminen- ciente, vinculando-se, portanto, ao passado deste último,
temente, no quadro da relação analítica. A repetição de e não dizendo respeito diretamente à pessoa do analista.
protótipos infantis vividos com um sentimento de atuali- Manfredi, distingue cinco tendências de abordagens
dade acentuada. Classicamente a transferência é reconhe- desta questão:

137
CONHECIMENTOS ESPECÍFICOS - PSICÓLOGO

1 - A contratransferência não é mais considerada como I - PRINCÍPIOS NORTEADORES NA ELABORAÇÃO


uma criação unicamente do paciente, por ignorar a trans- DE DOCUMENTOS
ferência do analista;
2 - É problemático diferenciar a contratransferência O psicólogo, na elaboração de seus documentos, de-
normal da patológica (os dados à disposição do analista verá adotar como princípios norteadores as técnicas da lin-
não permitem, quase sempre, uma diferenciação); guagem escrita e os princípios éticos, técnicos e científicos
3 - A tolerância à contratransferência já seria suficiente, da profissão.
dada, aqui, a dificuldade da diferenciação dos sentimentos
envolvidos na dupla; 1– Princípios Técnicos da Linguagem Escrita
4 - Devia-se, mais sábia e humildemente, fazer também
a rota inversa: procurar no paciente, e não só procurar no O documento deve, na expressão escrita, apresentar
analista; uma redação bem estruturada e definida, ou seja, expres-
5 - A questão do confessar ou não, ou confessar/reve- sar o pensamento, o que se quer comunicar. Deve ter uma
lar até quando/quanto, os sentimentos contra transferen- ordenação que possibilite a compreensão por quem o lê, o
ciais despertados. que é fornecido pela estrutura, composição de parágrafos
ou frases, além da correção gramatical.
Fonte: O emprego de expressões ou termos deve ser compa-
SILVA, V. G. da. tível com as expressões próprias da linguagem profissional,
garantindo a precisão da comunicação e evitando a diver-
sidade de significações da linguagem popular.
A comunicação deve ainda apresentar como qualida-
ELABORAÇÃO DE DOCUMENTOS des a clareza, a concisão e a harmonia. A clareza se tra-
ESCRITOS: LAUDOS, RELATÓRIOS E duz, na estrutura frasal, pela sequência ou ordenamento
PARECERES PSICOLÓGICOS. adequado dos conteúdos, pela explicitação da natureza e
função de cada parte na construção do todo. A concisão se
verifica no emprego da linguagem adequada, da palavra
exata e necessária. Essa “economia verbal” requer do psi-
MANUAL DE ELABORAÇÃO DE DOCUMENTOS cólogo a atenção para o equilíbrio que evite uma redação
DECORRENTES DE AVALIAÇÕES PSICOLÓGICAS. lacônica ou o exagero de uma redação prolixa. Finalmente,
a harmonia se traduz na correlação adequada das frases,
Considerações Iniciais no aspecto sonoro e na ausência de cacofonias.

“A avaliação psicológica é entendida como o processo 2– Princípios Éticos e Técnicos


científico de coleta de dados, estudos e interpretação de
informações a respeito dos fenômenos psicológicos, que 2.1. Princípios Éticos
são resultantes da relação do indivíduo com a sociedade,
utilizando-se, para tanto, de estratégias psicológicas – mé- Na elaboração de DOCUMENTO, o psicólogo baseará
todos, técnicas e instrumentos. Os resultados das avalia- suas informações na observância dos princípios e dispo-
ções devem identificar os condicionantes sociais e seus sitivos do Código de Ética Profissional do Psicólogo. En-
efeitos no psiquismo, com a finalidade de serem instru- fatizamos aqui os cuidados em relação: aos deveres do
mentos para atuar não somente sobre o indivíduo, mas na psicólogo nas suas relações com a pessoa atendida, ao
modificação desses condicionantes sociais.” sigilo profissional, às relações com a justiça e ao alcance
O presente Manual tem como objetivos orientar o pro- das informações - identificando riscos e compromissos em
fissional psicólogo na confecção de documentos decor- relação à utilização das informações presentes nos docu-
rentes das avaliações psicológicas e fornecer os subsídios mentos em sua dimensão de relações de poder.
éticos e técnicos necessários para a elaboração qualificada
da comunicação escrita. 2.2. Princípios Técnicos
As modalidades de documentos aqui apresentadas
foram sugeridas durante o I Fórum Nacional de avaliação O processo de avaliação psicológica deve considerar
psicológica, ocorrido em dezembro de 2000. que os objetos deste procedimento (as questões de ordem
psicológica) têm determinações históricas, sociais, econô-
Este Manual compreende os seguintes itens: micas e políticas, sendo as mesmas elementos constitutivos
no processo de subjetivação. O DOCUMENTO, portanto,
I. Princípios Norteadores da elaboração documen- deve considerar a natureza dinâmica, não definitiva e não
tal; cristalizada do seu objeto de estudo.
II. Modalidades de documentos; Os psicólogos, ao produzirem documentos escritos,
III. Conceito e Finalidade / Estrutura; devem se basear exclusivamente nos instrumentais técni-
IV. Validade dos Documentos; cos (entrevistas, testes, observações, dinâmicas de grupo,
V. Guarda dos Documentos. escuta, intervenções verbais etc.) que se configuram como

138
CONHECIMENTOS ESPECÍFICOS - PSICÓLOGO

métodos e técnicas psicológicas para a coleta de dados, a) Afirmar como testemunha, por escrito, a informação
estudos e interpretações de informações a respeito da pes- ou estado psicológico de quem, por requerimento, o solici-
soa ou grupo atendidos, bem como sobre outros materiais ta, aos fins expressos por este;
e documentos produzidos anteriormente e pertinentes à b) Justificar faltas e/ou impedimentos do solicitante,
matéria em questão. atestando-os como decorrentes do estado psicológico in-
A linguagem nos documentos deve ser rigorosa, preci- formado;
sa, clara e inteligível. c) Solicitar afastamento e/ou dispensa do solicitante,
subsidiado na afirmação atestada do fato, em acordo com
II - MODALIDADES DE DOCUMENTOS o disposto na Resolução CFP nº 015/96.

1. Declaração 2.2. Estrutura do Atestado


2. Atestado Psicológico
3. Relatório Psicológico A formulação do Atestado deve restringir-se à infor-
4. Laudo Psicológico mação solicitada pelo requerente, contendo expressamen-
5. Parecer Psicológico
te o fato constatado. Embora seja um documento simples,
deve cumprir algumas formalidades:
III - CONCEITO / FINALIDADE / ESTRUTURA
a) Ser emitido em papel timbrado ou apresentar na
1 – Declaração
subscrição do documento o carimbo, em que conste seu
1.1. Conceito e finalidade da Declaração nome e sobrenome acrescido de sua inscrição profissional
(“Nome do Psicólogo / N.º da inscrição”).
É um documento que visa a informar a ocorrência de
fatos ou situações objetivas relacionados ao atendimento b) O Atestado deve expor:
psicológico, com a finalidade de: - Registro do nome e sobrenome do cliente;
a) Declarar comparecimentos do atendido; - Finalidade do documento;
b) Declarar o acompanhamento psicológico do aten- - Registro da informação pelo sintoma, situação ou es-
dido; tado psicológico que justifica o atendimento, afastamento
c) Informações diversas sobre o enquadre do atendi- ou falta – podendo registrar sob o indicativo do código da
mento (tempo de acompanhamento, dias ou horários); Classificação Internacional de Doenças (CID);
Não deve ser feito o registro de sintomas, situações ou - Registro do local e data da expedição do Atestado;
estados psicológicos. - Registro do nome completo do psicólogo, sua ins-
crição no CRP, e/ou carimbo com as mesmas informações;
1.2. Estrutura da Declaração - Assinatura do psicólogo acima da identificação do
psicólogo ou do carimbo.
a) Ser emitido em papel timbrado ou apresentar na
subscrição do documento o carimbo, em que conste nome Se a finalidade do Atestado for solicitar afastamento
e sobrenome do psicólogo acrescido de sua inscrição pro- ou dispensa, o registro da informação/pedido deverá estar
fissional (“Nome do Psicólogo / N.º da inscrição”). justificado pelo sintoma, situação ou estado psicológico.
b) A Declaração deve expor: Os registros deverão estar transcritos de forma corrida,
- Registro do nome e sobrenome do solicitante; ou seja, separados apenas pela pontuação, sem parágrafos,
- Finalidade do documento (por exemplo, para fins de evitando, com isso, riscos de adulterações. No caso em que
comprovação); seja necessária a utilização de parágrafos, o psicólogo de-
- Registro de informações solicitadas em relação ao
verá preencher esses espaços com traços.
atendimento (por exemplo: se faz acompanhamento psico-
lógico, em quais dias, qual horário);
3 – Relatório Psicológico
- Registro do local e data da expedição da Declaração;
- Registro do nome completo do psicólogo, sua ins-
crição no CRP, e/ou carimbo com as mesmas informações. 3.1. Conceito e finalidade do Relatório Psicológico
Assinatura do psicólogo acima da identificação do psi-
cólogo ou do carimbo. O Relatório Psicológico é uma apresentação descritiva
e/ou interpretativa acerca de situações ou estados psico-
2– Atestado Psicológico lógicos e suas determinações históricas, sociais, políticas e
culturais, pesquisadas no processo de Avaliação Psicológi-
2.1. Conceito e Finalidade do Atestado ca. Como todo DOCUMENTO, deve ser subsidiado em da-
dos colhidos e analisados à luz de um instrumental técnico
É um documento expedido pelo psicólogo que certifi- (entrevistas, dinâmicas, testes psicológicos, observação, es-
ca uma determinada situação ou estado psicológico, tendo cuta, intervenção verbal etc.), consubstanciado em referen-
como finalidade: cial técnico-filosófico e científico, adotado pelo psicólogo.

139
CONHECIMENTOS ESPECÍFICOS - PSICÓLOGO

A finalidade do Relatório Psicológico será sempre a de zados para coletar as informações, contextualizando fatos
apresentar resultados e conclusões da avaliação psicológi- e pessoas neles envolvidos e a metodologia empregada,
ca. Entretanto, em função da petição ou da solicitação do possibilitando assim, para quem lê, a compreensão do
interessado, o Relatório Psicológico poderá destinar-se a ocorrido, o que se está analisando, solicitando e/ou ques-
finalidades diversas, como: encaminhamento, intervenção, tionando.
diagnóstico, prognóstico, parecer, orientação, solicitação
de acompanhamento psicológico, prorrogação de prazo Portanto, a introdução tratará da narração:
para acompanhamento psicológico etc. Enfim, a solicitação a) dos fatos motivadores do pedido;
do requerente é que irá apontar o objetivo último do Rela- b) dos procedimentos e instrumentos utilizados na co-
tório Psicológico. leta de dados (número de encontros, pessoas ouvidas, ins-
trumentos utilizados), à luz do referencial teórico-filosófico
3.2. Estrutura que os embasa.

Independentemente das finalidades a que se destina, o 3.2.3. Descrição ou Desenvolvimento


Relatório Psicológico é uma peça de natureza e valor cien-
tíficos, devendo conter narrativa detalhada e didática, com É a parte do Relatório na qual o psicólogo faz uma
clareza, precisão e harmonia, tornando-se acessível e com- exposição descritiva de forma metódica, objetiva e fiel dos
preensível ao destinatário. dados colhidos e das situações vividas. Nessa exposição,
Os termos técnicos devem, portanto, estar acompa- deve respeitar a fundamentação teórica que sustenta o
nhados das explicações e/ou conceituação retiradas dos instrumental técnico utilizado, bem como princípios éticos,
fundamentos teórico-filosóficos que os sustentam. como as questões relativas ao sigilo das informações.
Independentemente também, da finalidade a que se Somente deve ser relatado o que for necessário para o
destina, o Relatório Psicológico deve conter, no mínimo, 3 esclarecimento do encaminhamento, como disposto no
(três) etapas: introdução, descrição e conclusão, além do parágrafo 2o. do Artigo 23 do Código de Ética Profissional.
cabeçalho. O psicólogo, ainda nessa parte, poderá se valer de ci-
tações ou transcrições, visando a reforçar as conclusões de
1. – Cabeçalho sua análise. Não deve fazer afirmações sem sustentação em
2. – Introdução ou Histórico fatos e/ou teorias, devendo ter linguagem precisa, espe-
3. – Descrição ou Desenvolvimento cialmente quando se referir a dados de natureza subjetiva,
4. – Conclusão
expressando-se de maneira clara e exata.
3.2.1. Cabeçalho
3.2.4. Conclusão
É a parte superior da primeira parte do Relatório
É a parte final, conclusiva, do Relatório Psicológico.
Psicológico com a finalidade de identificar:
Nela, o psicólogo vai espelhar e dar ênfase às evidências
O autor/relator – quem elabora o Relatório Psicológico;
encontradas na análise dos dados a partir das referências
O interessado – quem solicita o Relatório Psicológico;
adotadas, que subsidiaram o resultado a que o psicólogo
O assunto/finalidade – qual a razão/finalidade do Re-
latório Psicológico. chegou, sustentando assim a finalidade a que se propôs.
Escrita logo após a descrição, o psicólogo deve expor o re-
No identificador AUTOR/RELATOR, deverá ser colo- sultado e/ou considerações. Após a narração conclusiva, o
cado o(s) nome(s) do(s) psicólogo(s) que realizará(ão) a Relatório Psicológico é encerrado, com indicação do local,
avaliação, com a(s) respectiva(s) inscrição(ões) no Conselho data de emissão e assinatura do psicólogo.
Regional.
No identificador INTERESSADO, o psicólogo indicará o 4 – Laudo Psicológico ou Pericial
nome do autor do pedido (se a solicitação foi da Justiça, se 4.1. Conceito e finalidade do Laudo Psicológico ou
foi de empresas, entidades ou do cliente). Pericial
No identificador ASSUNTO, o psicólogo indicará a ra-
zão, o motivo do pedido (se para acompanhamento psi- A palavra laudo é originária do idioma latino, do geni-
cológico, prorrogação de prazo para acompanhamento ou tivo laud-is e significa originalmente mérito, valor, glória. É
outras razões pertinentes a uma avaliação psicológica). um documento conciso, minucioso e abrangente, que bus-
ca relatar, analisar e integrar os dados colhidos no processo
3.2.2. Introdução ou Histórico de avaliação psicológica tendo como objetivo apresentar
diagnóstico e/ou prognóstico, para subsidiar ações, deci-
Alguns psicólogos, em seus Relatórios, intitulam essa sões ou encaminhamentos. Portanto, diferencia-se do Re-
primeira parte como HISTÓRICO. Ela é destinada à narração latório Psicológico por ter como objetivo subsidiar uma to-
histórica e sucinta dos fatos que produziram o pedido do mada de decisão, por realizar uma extensa pesquisa cujas
Relatório Psicológico. Inicia-se com as razões do pedido, observações e dados colhidos deverão ser relacionados às
seguida da descrição do processo ou procedimentos utili- questões e situações levantadas pela decisão a ser tomada.

140
CONHECIMENTOS ESPECÍFICOS - PSICÓLOGO

4.2. Estrutura Alchieri3 caracterizam a utilização dos parâmetros técnicos


científicos recomendados pelo Conselho Federal de Psico-
Na sua estrutura básica, o laudo psicológico contém os logia como sendo indicador de qualidade na estruturação
seguintes itens: de laudo psicológico.
1. Identificação As primeiras versões do “Manual de Elaboração de Do-
2. Descrição da demanda cumentos Escritos produzidos pelo psicólogo, decorren-
3. Métodos e técnicas utilizadas tes de avaliação psicológica”, Resolução CFP N.º 30/2001
4. Conclusão e Resolução CFP N.º 17/2002 apresentavam o relatório
psicológico e o laudo psicológico como documentos dis-
4.2.1. Identificação tintos, isto é, ambas as resoluções apresentam uma pro-
posta para elaboração de laudos e outra para elaboração
Refere-se à descrição dos dados básicos do avaliado, de relatórios. Analisando-se as duas primeiras versões do
como nome, data de nascimento, idade, escolaridade, filia- “Manual de Elaboração de Documentos Escritos produzi-
ção, profissão etc. dos pelo psicólogo, decorrentes de avaliação psicológica”
observa-se que as modificações da primeira resolução para
sua sucessora, quanto ao conceito e finalidade do relatório
psicológico e laudo psicológico são tênues, sendo que o
4.2.2. Descrição da demanda
documento laudo psicológico para estas resoluções é com-
patível com laudo pericial, isto é, o documento produzido
Nesse item, o psicólogo apresenta as informações refe-
a partir de “avaliação psicológica que visa subsidiar uma
rentes a motivos, queixas ou problemáticas apresentadas, decisão”16,17; muito provavelmente as resoluções se refe-
esclarecendo quais ações, decisões ou encaminhamentos o rem a decisões judiciais, apesar de não estarem explicitas.
Laudo deverá subsidiar. No entanto, para a Resolução CFP N.º 007/2003, “lau-
do psicológico” e “relatório psicológico” é um mesmo tipo
4.2.3. Métodos e técnicas utilizadas de documento, não havendo distinção, com uma única
apresentação de objetivo, finalidade e estruturação do
Refere-se à descrição dos recursos utilizados e dos re- documento; a definição conceitual é referida como “uma
sultados obtidos. apresentação descritiva acerca de situações e/ou condi-
ções psicológicas e suas determinações históricas, sociais,
4.2.4. Conclusão políticas e culturais, pesquisadas no processo de avaliação
psicológica”14, apresenta ainda a observação de que:
Destina-se a apresentar uma síntese do diagnóstico [...] todo documento, deve ser subsidiado em dados
e/ou prognóstico da avaliação realizada e/ou encaminha- colhidos e analisados, à luz de um instrumental técnico
mentos, necessariamente relacionados à demanda. (entrevistas, dinâmicas, testes psicológicos, observação,
exame psíquico, intervenção verbal), consubstanciado em
LAUDOS PSICOLÓGICOS referencial técnico-filosófico e científico adotado pelo psi-
cólogo14.
Os laudos e relatórios psicológicos estão inseridos no
campo de conhecimento relacionado à avaliação psicoló- Quanto à finalidade do relatório psicológico, segundo
gica e assim sendo, parte dos artigos e livros encontrados a Resolução CFP N.º 007/2003, é de
apresenta a avaliação psicológica como principal eixo te-
mático, porém, visto sua inserção profissional em inúmeras [...] apresentar os procedimentos e conclusões gerados
áreas, também foram encontrados artigos que tratam so- pelo processo da avaliação psicológica, relatando sobre o
bre laudo em textos relacionados à clínica, encaminhamento, as intervenções, o diagnóstico, o prog-
nóstico e evolução do caso, orientação e sugestão de pro-
Psicologia escolar, jurídica, do trânsito e no ensino da
jeto terapêutico, bem como, caso necessário, solicitação de
psicologia, com destaque à área jurídica e de ensino da psi-
acompanhamento psicológico, limitando-se a fornecer so-
cologia, que foram as que apresentaram maior quantidade
mente as informações necessárias relacionadas à demanda,
de material.
solicitação ou petição14.
A Resolução do CFP N° 007/2003, a qual apresenta o
“Manual de Elaboração de Documentos Escritos produzi- Em relação à estruturação do laudo, a Resolução do
dos pelo psicólogo decorrentes de avaliação psicológica” CFP N° 007/2003 determina que a estrutura do laudo deve
apresenta uma proposição de parâmetros para a estrutu- conter no mínimo 5 (cinco) itens: identificação, descri-
ração do laudo psicológico, além de princípios técnicos da ção da demanda, procedimento, análise e conclusão. No
linguagem escrita e princípios éticos e técnicos relaciona- campo inicial do laudo, deve constar a identificação, que
dos à avaliação psicológica, no Artigo 3º fica determinado segundo a Resolução do CFP N° 007/2003 consta de três
que o psicólogo em toda comunicação por escrito decor- partes: a caracterização do autor do laudo (o nome do psi-
rente de avaliação psicológica deve seguir as diretrizes des- cólogo que realizou a avaliação psicológica, associado ao
critas no manual contido na referida resolução12. Silva e seu número de inscrição no Conselho Regional de Psicolo-

141
CONHECIMENTOS ESPECÍFICOS - PSICÓLOGO

gia - CRP), quem solicita o laudo (o nome do autor do pedido, associado a órgãos ou entidades, como a Justiça, empresas,
ou do cliente) e o assunto e finalidade do laudo (a razão, o motivo do pedido). A descrição da demanda é onde se explicita a
problemática apresentada e dos motivos, razões e expectativas que produziram o pedido do laudo, e ainda, a análise que o
profissional faz da demanda de forma a justificar o procedimento adotado. O procedimento é o campo do laudo para apre-
sentar os recursos e instrumentos técnicos utilizados, sendo que a referida resolução observa que o procedimento adotado
deve ser pertinente para avaliar a complexidade do que está sendo demandado. No campo analise é onde se apresenta, de
forma metódica, objetiva e fiel, a exposição descritiva dos dados colhidos relacionados à demanda em sua complexidade,
com respeito à fundamentação teórica que sustenta o instrumental técnico utilizado, além de sigilo e respeito a princípios
éticos na apresentação dos dados colhidos no processo de avaliação psicológica.
Na conclusão do laudo, constam as considerações do profissional a respeito de sua investigação gerada pelo processo
de avaliação psicológica. O campo final do laudo consta de indicação do local, data de emissão, assinatura do psicólogo e
o seu número de inscrição no CRP; as demais folhas devem ser rubricadas pelo profissional.
Visto que há a obrigatoriedade do psicólogo em seguir os parâmetros apresentados na Resolução do CFP N° 007/2003,
entende-se esta como um marco para a estruturação de laudos e relatórios psicológicos no Brasil. Assim, a presente pes-
quisa apresentará os textos que abordam os laudos/relatórios psicológicos, focando aspectos de estruturação e conteúdo,
separados em dois grandes períodos: textos anteriores a 2003 e textos posteriores a 2003 (respectivamente, anteriores e
posteriores à promulgação da Resolução do CFP N° 007/2003), os quais estão expostos no Quadro 1, com discussão apre-
sentada na sequência.

142
CONHECIMENTOS ESPECÍFICOS - PSICÓLOGO

143
CONHECIMENTOS ESPECÍFICOS - PSICÓLOGO

TEXTOS ANTERIORES À RESOLUÇÃO CFP N.º sendo “um documento ou peça processual habilitada le-
007/2003 galmente para servir de prova ou consulta esclarecedora
acerca de um fato”. O autor recomenda seis elementos es-
É importante destacar pelo pioneirismo, os livros de senciais quanto à estrutura do documento, a saber:
Minicuccie Skaf como sendo umas das poucas referências
que apresentam como objetivo da obra enfocar de forma a) Dados de identificação do cliente e período de re-
ampla os laudos e relatórios psicológicos, apresentando alização do exame;
propostas de estruturação dos documentos, conteúdos, b) Motivo do encaminhamento ou objetivo da avalia-
dentre outros aspectos relevantes. ção;
Minicucci refere que a palavra laudo vem do latim, c) Técnicas utilizadas, com seus respectivos focos de
laud-us, que significa mérito, valor e em outra obra Mini- exame;
cucci refere que como termo técnico, laudo significa o re- d) Sumário dos resultados (quantitativos e/o qualitati-
lato descritivo, interpretativo de um exame, que descreve vos); e) Conclusão (diagnóstico);
e/ou interpreta dados; refere ainda que também pode ser f) Identificação do profissional (registro CRP).
chamado de relato ou relatório. Quanto à estruturação do
laudo, Minicucci20-22 apresenta várias opções de modelos Cruz apresenta ainda cinco fatores a serem considera-
de acordo com variadas necessidades profissionais, como dos pelo psicólogo quanto ao conteúdo da redação técnica
por exemplo, os laudos de seleção profissional, psicope- do laudo pericial, sugerindo os seguintes elementos:
dagógico, clínico e para exame psicotécnico de motoris-
ta dentre outros modelos e exemplos de laudos que são a) Capacidade de discernimento sobre o assunto e
apresentados pelo autor. O termo “exame psicotécnico de suas implicações;
motorista” está em desuso, porém optou-se por manter-se b) Tensões, alterações e/ou complicações de ordem
a terminologia utilizada pelo autor. cognitiva e emocional;
As propostas de estruturação de laudos apresentadas c) Ação, dimensão e/ou repercussões sociais do
por Minicucci quanto à estruturação e conteúdo são varia- comportamento estudado;
das de acordo com a situação profissional a que o laudo d) Nexos entre a conduta e o desempenho na ativi-
teria utilidade, não há padronização quanto à estrutura- dade;
ção, assim há grande divergência se comparado com os e) Estressores ou facilitadores associados aos relacio-
parâmetros técnicos apresentados na Resolução do CFP N° namentos interpessoais.
007/2003.
Skaf apresenta uma definição de laudo voltada à perí- Maciel em pesquisa sobre a perícia psicológica relacio-
cia e o define como relatórios escritos e pormenorizados, nada a conflitos familiares, alerta para o fato de a escrita do
de tudo quanto os peritos julgarem útil informar dentro laudo contribuir na tomada de decisão pelo juiz. Por isso,
das limitações e circunstancias de cada caso, limitando- destaca a utilização da fundamentação teórica e referencia
-o ao campo jurídico, indicando que são peças periciais bibliográfica como parte do laudo conferindo caráter cien-
formais de grande importância nos processos jurídicos. tifico ao estudo pericial e a importância da apresentação
Quanto à estrutura do laudo, o autor propõe os seguintes da conclusão.
tópicos: Preâmbulo, Quesitos, Histórico ou Comemorativo, Ortiz enfoca o laudo pericial no âmbito da justiça e
Descrição, Discussão, Conclusões, Resposta aos Quesitos. alerta para que a redação do laudo, além de explícita e
A proposição apresentada por Skaf, quanto ao conte- clara, deve ser assertiva, para que não ocorram distorções
údo dos tópicos elencados pelo autor, tem proximidade à interpretativas pelos não versados no assunto, alertando
proposição da Resolução 007/2003, porem apresenta as- que este documento é destinado à leitura leiga ( juízes,
pectos específicos do contexto jurídico, como por exemplo curadores e advogados). A autora trabalha na perspecti-
Quesitos e Resposta aos Quesitos. va de se estabelecer um Diagnóstico Situacional, no qual
Rovinski ao abordar o informe pericial no campo jurí- utiliza diversos elementos para o exame dos conflitos da
dico o define como sendo o relatório redigido posterior- disputa familiar (dados processuais, estudo pormenoriza-
mente a uma perícia e a referida autora utiliza referencial do de entrevistas, anamneses), com objetivo de sintetizar
estrutural de laudo apresentado por Skaf, considerando a os elementos disponíveis para análise e estabelecer uma
qualidade da proposta do autor associada ao fato de que perspectiva abrangente da situação. A autora não apresen-
teve o apoio e foi editada pelo Conselho Regional de Psico- ta preocupação excessiva relativa à estruturação do laudo,
logia do Paraná (CRP/PR), recomendando-a como padrão a porem recomenda alguns cuidados: quanto a expressões
ser seguido à época. técnicas, apesar de ser um documento destinado a públi-
Cruz ao trabalhar a perícia psicológica e o laudo, des- co leigo, não deve prescindir de expressões técnicas e da
taca que elaborar um laudo, de forma ampla, é realizar um análise teórica que norteiam as conclusões do Diagnóstico
diagnóstico, em função de que o laudo deve produzir um Situacional, indicando que estas expressões devem ser
conhecimento especializado e o define como documen- traduzidas; indica também a consultas bibliográficas que
to técnico que representa materialmente o conhecimento podem ser citadas em notas de rodapé, equiparando à tra-
pelo qual se utiliza para as finalidade de consulta, estudo balhos científicos, indicando que os textos podem consti-
ou prova. Do ponto de vista judicial, o autor o define como tuir anexos do laudo. Em relação à síntese dos dados, reco-

144
CONHECIMENTOS ESPECÍFICOS - PSICÓLOGO

menda que apenas neste campo são apresentadas ilações pesquisas que envolvem processos éticos, os demais abor-
interpretativas, sendo característico do Diagnóstico Situa- dam o laudo/relatório sob alguns aspectos e ainda, alguns
cional. apenas fazem referência à Resolução CFP N.º 007/03 como
Shine em obra da área jurídica, especificadamente rela- parâmetro a ser seguido.
cionada à disputa de guarda em Varas de Família, apresen- No âmbito do trânsito, Silva e Alchieri avaliaram a qua-
ta capítulo para a apresentação de normas para redação de lidade dos laudos psicológicos produzidos por psicólogos
documentos e recomenda a utilização da Resolução CFP do trânsito que realizam avaliação psicológica para a con-
N.º 30/ 2001, recomendando-a como parâmetro ao psicó- cessão de Carteira Nacional de Habilitação (CNH), os auto-
logo. O autor esclarece que, sob o ponto de vista proces- res definem critérios de qualidade dos laudos baseados na
sual, o laudo é a peça mais importante da perícia, o qual Resolução CFP N.º 007/2003, de acordo com três aspectos:
consubstanciará as conclusões do perito, devendo ser inte- guarda de material, estrutura dos laudos de acordo com
ligível, autoexplicativo e convincente. O autor faz menção os 5 itens básicos e obrigatórios, e avaliação do preenchi-
aos quesitos, os quais são “perguntas escritas e articuladas mento dos laudos. Em relação à estrutura dos laudos ava-
relativas aos fatos a serem periciados” defendendo que o liados foram detectados vários tipos de problemas, como:
psicólogo perito deve transformá-los em quesito psico- inexistência de identificação do psicólogo; em vez da con-
legal na hora de respondê-los, no sentido de se atentar clusão, foi colocado parecer final; não foi expresso o local
para a interface da preocupação legal em jogo e abordar as de realização dos exames; quanto ao preenchimento dos
questões psicológicas pertinentes. laudos, foi houve vários indicativos de falta de qualidade
técnica, como o uso excessivo de abreviações na escrita;
Cunha ao trabalhar com a comunicação dos resulta- incorreções na denominação dos instrumentos; ausência
dos de psicodiagnóstico clínico argumenta que o laudo se dos resultados e não integração dos dados obtidos no pro-
constitui como uma unidade essencial do psicodiagnós- cesso avaliativo. A pesquisa indica que a maior parte dos
tico; quanto à estrutura, propõe que o laudo responda a laudos analisados não possuiu a “qualidade técnicocien-
questões como “o que”, “quanto”, “como”, “por que”, ‘para tífica recomendada pelo Conselho Federal de Psicologia”,
que” e quanto”, alertando que o conteúdo da comunica- porém mostram cautela com qualquer possibilidade de
ção é definido tanto pelas questões específicas, formuladas generalização quanto à qualidade dos laudos produzidos
no início do processo, como pela identidade do receptor, por psicólogos.
no sentido que, dependendo do receptor do documento Dois autores estudaram laudos psicológicos associa-
alguns conteúdos devem ser incluídos e outros não, aler-
dos a processos éticos junto a Conselhos Regionais de Psi-
tando quanto às questões éticas do sigilo profissional, que
cologia.
comprometem o psicólogo a não fornecer certas informa-
Assim, Shine estudou denúncias éticas contra psicó-
ções, ou presta-las somente a quem de direito e sempre
logos e que foram julgados pelo Conselho Regional de
contemplando o benefício do paciente; alerta ainda para
Psicologia de São Paulo em que estavam envolvidos lau-
que seja respeitado o nível sociocultural, intelectual e emo-
dos psicológicos associados ao campo da psicologia jurí-
cional do receptor, usando terminologia e linguagem ade-
dica e Vara de família; a pesquisa usou como parâmetro
quadas à cada necessidade.
Santos e Silva Neto propõe cuidados éticos na elabo- de análise da qualidade dos laudos a Resolução CFP N.º
ração de documentos psicológicos produzidos por psicó- 007/2003. A pesquisa concluiu que as falhas mais graves na
logos que trabalham com avaliação de candidatos à obten- estruturação dos laudos estudados não são relacionados à
ção de Carteira Nacional de Habilitação (CNH), tecem críti- linguagem nem à dificuldade de comunicação da matéria
cas aos jargões utilizados como termos técnicos, os quais, psicologia ao leitor leigo, por outro lado, estão associadas
na opinião dos autores têm inviabilizado a comunicação a estratégias de avaliação psicológica equivocadas refleti-
efetiva e tem se apresentado como dispositivo de poder das nos laudos psicológicos produzidos.
para “travestir a ignorância”. Também Frizzo em pesquisa realizada no Conselho Re-
Visando proposição de solução para o quadro de defi- gional de Psicologia de Santa Catarina, relativo a faltas éti-
ciência na atuação profissional, Noronha et al. propõe que cas no exercício profissional, desde a denúncia, passando
a formação em avaliação psicológica deve cobrir tópicos pela instância de apuração, até o momento da finalização
como teoria da medida e psicometria, avaliação da inteli- de processo disciplinar ético, sob forma de julgamento,
gência, avaliação da personalidade e práticas integrativas teve como resultado que 20,8% da amostra estudada ti-
de planejamento, execução e elaboração de laudos nos veram laudos psicológicos como provas documentais para
mais variados contextos. instruir os processos disciplinares éticos e 46% da amostra
tiveram caracterização da infração ética associada a falhas
TEXTOS POSTERIORES À RESOLUÇÃO CFP N.º na realização de perícia/avaliação psicológica e conse-
007/2003 quente estruturação dos laudos. A autora alerta para que
a produção científica possa ser incorporada à prática pro-
De todos os textos encontrados na pesquisa biblio- fissional, buscando ser tecnicamente competente e etica-
gráfica com data posterior à da Resolução CFP N.º 007/03, mente responsável. Neste sentido, os avanços no âmbito
apenas 3 deles têm como foco principal a elaboração de da avaliação psicológica necessitam estar refletidos na prá-
laudos/relatórios psicológicos, sendo um deles relacionado tica profissional do psicólogo, tanto em aspectos técnicos
à área do trânsito e dois relacionados à ética, visto serem quanto éticos.

145
CONHECIMENTOS ESPECÍFICOS - PSICÓLOGO

Primi alerta para a dificuldade de avaliar o quanto o do que deve refletir o caso particular exposto; quanto à
psicólogo está consumindo criticamente esse conhecimen- estruturação do laudo, apresenta a proposta da Resolução
to fomentado principalmente nos últimos anos e transfor- CFP N.º 007/2003, porem argumenta que é um modelo ge-
mando a prática profissional do psicólogo. nérico e não específico para a área forense e assim, propõe
Alguns autores, cujo foco do trabalho é avaliação psi- a estruturação dos laudos perícias segundo Skaf (1997).
cológica, trabalham com questões voltadas ao laudo ou Rovinski argumenta que laudo, segundo o Código de
relatório psicológico, porém não fazem menção à Resolu- Processo Civil (CPC) é o documento apresentado pelo pe-
ção CFP N.º 007/2003 e aos parâmetros de estruturação rito designado pelo juiz, diferenciando-o do documento
propostos: produzido pelo assistente técnico (parecer), recomendando
Guzzo e Pasquali propõem que a estruturação do laudo como parâmetro de estruturação o uso da Resolução CFP
pode variar de acordo com o objetivo da avaliação, porem N.º 007/2003, porem observa que há erro na resolução,
delimitam três grandes partes: “a história do indivíduo com apontando que é no laudo que o perito deve responder
o motivo da avaliação, as provas realizadas e seus resulta- aos quesitos apresentados, e não no parecer como cons-
dos gerais e a conclusão com indicadores de intervenção”. ta na referida resolução, acrescenta ainda para inclusão no
Os autores alertam para que o laudo equilibre os dados laudo de referências bibliográficas que sustentem as con-
com a teoria psicológica, com ilustração de comportamen- clusões do perito.
tos observados; propõe que o conteúdo do laudo apre- Ortiz, quando se propõe à análise institucional de dis-
sente identificação, motivo da consulta, descrição física, curso no âmbito jurídico com foco no trabalho do psicó-
impressão geral obtida durante o rapport, comportamento logo em Varas de Família, ressalta que o laudo é a voz do
do examinando, variáveis ambientais, instrumentos utiliza- perito junto ao processo, não sendo apenas a tradução e
dos (o objetivo e o nome de cada um deles), planejamen- resultado de um processo de avaliação psicológica, confi-
to, resultados dos testes, conclusão e limitações. Apontam gurando-se como peça processual ativa no conflito judicial,
ainda erros a serem evitados: argumentando que o laudo, mais que prova, tem o poten-
cial de levar as partes a se questionarem e a se reposiciona-
• excesso de termos técnicos rem em relação ao conflito. Neste sentido, a autora ressalta
• demonstração de cientificismo que o perito, principalmente através do laudo, introduz um
• apresentação de resultados sem uma visão integrada saber particular na cena jurídica e “afirma-se em uma certa
dos dados • uso de “chavões” • interferências de resultados vontade de verdade característica da instituição jurídica e,
em outro campo profissional. ao mesmo tempo, na intenção do perito de, como coadju-
vante da cena processual, nela produzir efeitos singulares”.
Tavares sugere que seja apresentada no relatório uma Fávero et al. ao discutirem sobre a atuação do serviço
descrição sumária do caso, com a situação que gerou o en- social e da psicologia no judiciário, apresentam pesquisa
caminhamento e das questões que se pretende responder, realizada com profissionais de ambas as profissões que tra-
as quais estão relacionadas aos procedimentos e instru- balhavam no Tribunal de Justiça do Estado de São Paulo,
mentos utilizados; faz recomendação de que o psicólogo com foco nas técnicas utilizadas, metodologia de traba-
ao fazer o contrato de avaliação psicológica, deva obter a lho, abordagem conceitual utilizada, dentre vários outros
permissão por escrito para comunicar-se oralmente e para aspectos; em relação aos documentos produzidos pelos
apresentar o relatório, especificando os profissionais e /ou profissionais do judiciário paulista, as autoras apresentam
instituições que deverão recebê-lo. discussão das peculiaridades de cada uma das profissões e
Noronha et al. recomendam que na fase do psicodiag- consequentes documentos produzidos. Quanto ao laudo/
nóstico relativo à comunicação dos resultados deve ser relatório psicológico as autoras apresentam o resultado da
identificado quem tem direito de recebê-las e serem elabo- pesquisa de que quase totalidade das psicólogas pesqui-
rados informes orais e escritos, restringindo as informações sadas elaboram relatórios psicológicos (73 sujeitos da pes-
a serem disponibilizadas; em relação ao método de ensino quisa de um total de 78 sujeitos pesquisados) e um número
das técnicas de avaliação psicológicas. Os autores propõem um pouco menor (63 sujeitos da pesquisa) indicam que ela-
estudos de caso e elaboração de documentos psicológicos boram laudos; na discussão dos resultados as autoras ava-
dentre outras metodologias. liam que alguns psicólogos apresentam indiferenciação de
Vários autores apenas fazem referência à recomenda- entendimento do que sejam relatórios, pareceres e laudos,
ção de uso da Resolução CFP N.º 007/200314 para estrutu- ressaltando que as diferenças entre eles podem ser muito
ração de laudos e relatórios psicológicos, não aprofundan- tênues, porém as autoras não apresentam o entendimento
do em aspectos de conteúdo. da pesquisa em relação a uma conceituação diferencial en-
Dentre os autores que tratam de aspectos relativos ao tre relatório psicológico e laudo psicológico; utilizam como
laudo psicológico, foram encontrados cinco textos que en- referência a Resolução CFP N° 007/2003; no entanto, para
focam o campo judiciário/forense: esta resolução não há diferenciação entre laudo e relatório
Rovinski, ao caracterizar o laudo pericial, apresenta-o psicológico, caracterizando-os como um único documento,
como sendo “uma comunicação técnicocientífica, de natu- diferentemente das discussões apresentadas na pesquisa.
reza oficial, que tem como destinatário o juízo que solicitou Silva baseia-se na Resolução CFP N.º 007/ 2003 para
a perícia”; ao discutir sobre a estrutura do laudo pericial recomendar a estruturação do laudo pericial, observan-
alerta para não torna-lo inflexível e mecanicista, ressaltan- do, do ponto de vista intrínseco, para que seja assinado e

146
CONHECIMENTOS ESPECÍFICOS - PSICÓLOGO

rubricado em todas as folhas, para evitar substituição de ciplinas ministradas e, enquanto metodologia de ensino,
folhas ou adulteração, sob aspecto extrínseco recomenda apenas 31,8% utilizava a elaboração de laudos, relatórios e
clareza, circunscrito ao objeto da perícia e fundamentado. documentos como atividade didática; desta forma, a cate-
Silveira recomenda a estruturação do laudo com os pa- goria de análise e princípios de elaboração de documento
râmetros apresentados pela Resolução CFP N° 017/2002, psicológico foi, juntamente com o histórico da avaliação
porém na data da publicação do artigo, novembro de 2003, psicológica, as categorias menos contempladas pelos pro-
a referida resolução não estava mais em vigor, pois foi re- fessores pesquisados. As autoras discutem a tendência à
vogada em junho de 2003. formação tecnicista, voltadas à instrumentalização da ava-
Na área da saúde Loli e Preto45 fazem recomendações liação psicológica em contraposição à contextualização da
relativas ao laudo para avaliação psicológica para a cirurgia avaliação psicológica enquanto área de estudo da psicolo-
bariátrica (gastroplastia); recomendando o uso da Resolu- gia e suas relações com a sociedade, como questões éticas
ção CFP N° 007/2003; ressaltam aspectos de estruturação e elaboração de documentos; sendo a elaboração adequa-
que estariam omissos na referida resolução: especificar no da de laudo entendida como uma competência profissional
documento a identificação da pessoa avaliada; acrescentar que reflete uma compreensão ampla de avaliação psicoló-
item referente a considerações éticas da psicologia em fun- gica é desejável em um bom profissional.
ção de que o destinatário do documento não é psicólogo
(médico) e recomendação de que o documento seja enca- Fonte:
minhado em envelope lacrado, constando a palavra “con- PRETO, Cássia Regina de Souza; FAJARDO, Renato Sal-
fidencial”; há ainda a sugestão de que o psicólogo fique viato. Laudo psicológico no Brasil: revisão da literatura com
com uma cópia idêntica ao encaminhado ao destinatário, foco em estruturação e conteúdo. Arch Health Invest (2015)
com autorização por escrito do paciente para que autorize PRETO, Cássia Regina de Souza. Laudo Psicológico –
a remessa de informações. Curitiba: Juruá.
Noronha et al. ressaltam que o ensino de avaliação psi-
cológica é um aspecto central na formação dos psicólogos 5 – Parecer
e que parte da responsabilidade por atuações profissionais
impróprias se deve a formação inconsistente nesta área. As 5.1. Conceito e finalidade do Parecer
diretrizes curriculares propõem competências que o curso
de graduação deve atender como requisito mínimo, sendo O Parecer é uma manifestação técnica fundamentada e
que na área de avaliação psicológica, por exemplo, é ne- resumida sobre uma questão focal do campo psicológico
cessário saber articular conhecimentos, habilidades e com- cujo resultado pode ser indicativo ou conclusivo.
petências em diversas áreas de atuação profissional, além O Parecer tem como finalidade apresentar resposta
de saber escolher e utilizar instrumentos e procedimentos esclarecedora, no campo do conhecimento psicológico,
de coleta de dados em Psicologia, realizar diagnóstico e através de uma avaliação técnica especializada, de uma
avaliar processos psicológicos e emitir laudos e documen- “questão-problema”, visando a dirimir dúvidas que estão
tos psicológicos, dentre outros. interferindo na decisão, sendo, portanto, uma resposta a
Nesse sentido, Nunes et al.48 elencam 27 competên- uma consulta, que exige de quem responde competência
cias básicas em avaliação psicológica, destacando-se a ela- no assunto.
boração de laudos e documentos psicológicos.
Com essa perspectiva, pesquisa que envolveu alunos 5.2. Estrutura
de graduação em psicologia investigou a importância con-
ferida e a percepção de domínio quanto a competências de O psicólogo nomeado perito deve fazer a análise do
avaliação psicológica. Para tanto, Noronha et al. utilizaram problema apresentado, destacar os aspectos relevantes e
20 itens baseados nas definições de competências essen- opinar a respeito, considerando os quesitos apontados e
ciais apontadas pela American Psychological Associacion e com fundamento em referencial teórico científico.
o resultado apontou para percepção dos alunos da impor- Deve-se rubricar todas as folhas dos documentos. Ha-
tância relacionada à comunicação de resultados e aspectos vendo quesitos, o psicólogo deve respondê-los de forma
éticos da avaliação psicológica. sintética e convincente, não deixando nenhum quesito sem
Pesquisas que envolvem o ensino de avaliação psicoló- resposta. Quando não houver dados para a resposta ou
gica na graduação focaram aspectos relacionados à emis- quando o psicólogo não puder ser categórico, deve-se uti-
são de laudos. Noronha49 avaliou ementas de disciplinas lizar a expressão “sem elementos de convicção”. Se o que-
de avaliação psicológica e apenas 26% elencaram o ensino sito estiver mal formulado, pode-se afirmar “prejudicado”,
de elaboração de laudos; em pesquisa mais recente Noro- “sem elementos” ou “aguarda evolução”.
nha et al. encontraram resultado semelhante, ao pesquisar
conteúdos ministrados e metodologias de ensino utilizadas O Parecer é composto de 4 (quatro) partes:
por professores de graduação em psicologia em disciplinas
relacionadas à avaliação psicológica. O resultado da pes- 1. Cabeçalho
quisa demonstrou, enquanto conteúdo ministrado, que a 2. Exposição de motivos
elaboração de documentos foi apontada como conteúdo 3. Discussão
contemplado por professores em apenas 36,4% das dis- 4. Conclusão

147
CONHECIMENTOS ESPECÍFICOS - PSICÓLOGO

5.2.1. Cabeçalho Em caso de extinção de serviço psicológico, o material


privativo e os documentos escritos devem permanecer em
É a parte que consiste em identificar o nome do perito posse do psicólogo responsável, que os manterá sob sua
e sua titulação, o nome do autor da solicitação e sua guarda pelo prazo previsto neste manual.
titulação. Atingido esse prazo, o psicólogo ou instituição respon-
sável pela guarda deverá destruir o material de forma a não
5.2.2. Exposição de Motivos permitir a quebra do sigilo das informações nele contidas.
O psicólogo responsável pelo documento escrito de-
Essa parte destina-se à transcrição do objetivo da con- corrente da avaliação psicológica deverá estar atento ao
sulta e os quesitos ou à apresentação das dúvidas levanta- artigo 24 do Código de Ética Profissional do Psicólogo, ga-
das pelo solicitante. Deve-se apresentar a “questão-proble- rantido, assim, o sigilo profissional.
ma”, não sendo necessária, portanto, a descrição detalhada
dos procedimentos, como os dados colhidos ou o nome
REGISTRO DE DOCUMENTOS:
dos envolvidos.
PRONTUÁRIOS E PARECERES
5.2.3. Discussão
Registro documental / prontuário
A discussão do PARECER constitui-se na análise minu-
ciosa da “questão-problema”, explanada e argumentada O que é o Registro Documental?
com base nos fundamentos necessários existentes, seja na
ética, na técnica ou no corpo conceitual da ciência psico- O registro documental é um documento de caráter si-
lógica. giloso e constitui-se em um conjunto de informações que
tem por objetivo contemplar de forma sucinta o trabalho
5.2.4. Conclusão prestado, a descrição e a evolução da atividade e os proce-
dimentos técnico-científicos adotados.
É a parte final do Parecer, em que o psicólogo irá
apresentar seu posicionamento, respondendo à questão O que é o Prontuário?
levantada. Ao final do posicionamento ou do Parecer
propriamente dito, informa o local e data em que foi Prontuários são definidos como arquivos, em papel ou
elaborado e assina o documento. informatizados, cuja finalidade é facilitar a manutenção e
o acesso às informações que os(as) usuários(as) fornecem
V - VALIDADE DOS DOCUMENTOS
durante o atendimento, incluindo os resultados de avalia-
O prazo de validade dos documentos escritos decor- ções e procedimentos realizados com finalidade diagnós-
rentes das avaliações psicológicas deverá considerar a le- tica ou de tratamento, lembrando que o(a) usuário(a) deve
gislação vigente nos casos já definidos. ser informado(a) da existência do prontuário.
Não havendo definição legal, o psicólogo, onde for
possível, indicará o prazo de validade em função das carac- Todo(a) psicólogo(a) está obrigado(a) a manter registro
terísticas avaliadas, das informações obtidas e dos objeti- documental dos serviços de psicologia prestados?
vos da avaliação.
Quando não for possível a indicação do prazo, infor- Sim. Conforme a Resolução CFP n.° 001/2009, todo(a)
mará o caráter situacional e temporal dos dados de uma psicólogo(a) deve manter registro documental de suas ati-
avaliação psicológica. vidades, e em formato de prontuário quando na saúde.
Ao definir o prazo, o psicólogo deve dispor dos funda-
mentos para a indicação, devendo apresentá-los sempre O(A) usuário(a) do serviço pode acessar o prontuário?
que solicitado.
O prontuário é de propriedade do(a) usuário(a) do
VI - GUARDA DOS DOCUMENTOS E CONDIÇÕES DE serviço ou responsável, o artigo 5º da resolução do CFP
GUARDA 001/2009 destaca em seu inciso II que fica garantido ao(à)
usuário(a) ou representante legal o acesso integral às in-
Os documentos escritos decorrentes de avaliação psi-
formações registradas, pelo(a) psicólogo(a), em seu pron-
cológica, bem como todo o material que os fundamen-
tou, deverão ser guardados pelo prazo mínimo de 5 anos, tuário, ou seja, o(a) usuário(a) poderá dispor do prontuário
observando-se a responsabilidade por eles tanto do psi- para verificação (conhecimento) em qualquer tempo.
cólogo quanto da instituição em que ocorreu a avaliação
psicológica. O(A) usuário(a) do serviço pode obter cópia do prontuá-
Esse prazo poderá ser ampliado nos casos previstos em rio?
lei, por solicitação judicial, ou ainda em casos específicos
em que seja necessária a manutenção da guarda por maior Sim. A concessão de cópia deverá ser garantida caso
tempo. haja solicitação do(a) usuário(a) ou representante legal.

148
CONHECIMENTOS ESPECÍFICOS - PSICÓLOGO

E se o serviço for multiprofissional? lógico, por razões que envolvam a restrição do comparti-
lhamento de informações com o usuário e/ou beneficiário
Neste caso, é recomendável que o registro seja reali- do serviço prestado.
zado em prontuário único, multiprofissional, devendo ser § 1°. O registro documental em papel ou informatizado
registradas apenas as informações necessárias ao cumpri- tem caráter sigiloso e constitui-se de um conjunto de
mento dos objetivos do trabalho, conforme a Resolução informações que tem por objetivo contemplar de forma
CFP nº 01/2009. sucinta o trabalho prestado, a descrição e a evolução da
atividade e os procedimentos técnicocientíficos adotados.
Como deve ser mantida a guarda dos registros docu- § 2º. Deve ser mantido permanentemente atualizado e
mentais/prontuários? organizado pelo psicólogo que acompanha o procedimento.
Deve existir um local reservado para a guarda destes Art. 2°. Os documentos agrupados nos registros do tra-
documentos, seja em arquivo, em armário ou qualquer ou- balho realizado devem contemplar:
tro móvel. O fundamental é garantir a restrição de acesso I – identificação do usuário/instituição;
de pessoas que não tenham relação com o atendimento,
II – avaliação de demanda e definição de objetivos do
principalmente, nos casos em que transitem, pelo local,
trabalho;
profissionais ou pessoas que não estão submetidos(as) ao
III – registro da evolução do trabalho, de modo a per-
sigilo profissional.
mitir o conhecimento do mesmo e seu acompanhamento,
E por quanto tempo devem ser guardados os registros bem como os procedimentos técnicocientíficos adotados;
documentais/prontuários? IV – registro de Encaminhamento ou Encerramento;
V – documentos resultantes da aplicação de instru-
O período de guarda deve ser de no mínimo 05 (cinco) mentos de avaliação psicológica deverão ser arquivados
anos, podendo ser ampliado nos casos previstos em lei. em pasta de acesso exclusivo do psicólogo.
VI – cópias de outros documentos produzidos pelo psi-
RESOLUÇÃO CFP Nº 001/2009 cólogo para o usuário/instituição do serviço de psicologia
prestado, deverão ser arquivadas, além do registro da data
Dispõe sobre a obrigatoriedade do registro documental de emissão, finalidade e destinatário”.
decorrente da prestação de serviços psicológicos.
Art. 3°. Em caso de serviço psicológico prestado em
O CONSELHO FEDERAL DE PSICOLOGIA, no uso de serviços-escola e campos de estágio, o registro deve con-
suas atribuições legais e regimentais, que lhe são conferi- templar a identificação e a assinatura do responsável técni-
das pela Lei nº 5.766, de 20 de dezembro de 1971 e; co/supervisor que responderá pelo serviço prestado, bem
CONSIDERANDO a necessidade de haver um registro como do estagiário. Parágrafo único. O supervisor técnico
das informações decorrentes da prestação de serviços psi- deve solicitar do estagiário registro de todas as atividades e
cológicos que possibilite a orientação e a fiscalização sobre acontecimentos que ocorrerem com os usuários do serviço
o serviço prestado e a responsabilidade técnica adotada; psicológico prestado.
CONSIDERANDO a necessidade de contemplar de for-
ma sucinta a assistência prestada, a descrição e a evolução Art. 4°. A guarda do registro documental é de respon-
do processo e os procedimentos técnico-científicos adota- sabilidade do psicólogo e/ou da instituição em que ocor-
dos no exercício profissional; reu o serviço.
CONSIDERANDO que o registro documental, além de § 1. ° O período de guarda deve ser de no mínimo 05
valioso para o psicólogo e para quem recebe atendimen-
anos, podendo ser ampliado nos casos previstos em lei,
to e, ainda, para as instituições envolvidas, é também ins-
por determinação judicial, ou ainda em casos específicos
trumento útil à produção e ao acúmulo de conhecimento
em que seja necessária a manutenção da guarda por maior
científico, à pesquisa, ao ensino, como meio de prova idô-
nea para instruir processos disciplinares e à defesa legal; tempo.
CONSIDERANDO o que está disposto no Código de § 2º. O registro documental deve ser mantido em local
Ética Profissional do que garanta sigilo e privacidade e mantenha-se à disposi-
Psicólogo; ção dos Conselhos de Psicologia para orientação e fiscali-
CONSIDERANDO a decisão do Plenário do Conselho zação, de modo que sirva como meio de prova idônea para
Federal de Psicologia, no dia 31 de janeiro de 2009, instruir processos disciplinares e à defesa legal.

RESOLVE: CAPÍTULO II
DOS PRONTUÁRIOS
CAPÍTULO I
DOS REGISTROS DOCUMENTAIS Art. 5º. Na hipótese de o registro documental de que
trata o art. 1º desta Resolução ser realizado na forma de
Art. 1º. Tornar obrigatório o registro documental so- prontuário, o seguinte deve ser observado:
bre a prestação de serviços psicológicos que não puder ser I – as informações a ser registradas pelo psicólogo são
mantido prioritariamente sob a forma de prontuário psico- as previstas nos incisos I a V do art. 2º desta Resolução;

149
CONHECIMENTOS ESPECÍFICOS - PSICÓLOGO

II – fica garantido ao usuário ou representante legal o O Perito auxilia o Juiz em questões técnicas. Há ques-
acesso integral às informações registradas, pelo psicólogo, tões problema a serem respondidas, e o profissional deve
em seu prontuário; formular resposta aos quesitos. Ele tem a função de exami-
III – para atendimento em grupo não eventual, o psicó- nar as pessoas envolvidas no litígio e formar um juízo sobre
logo deve manter, além dos registros dos atendimentos, a o que lhe foi questionado.
documentação individual referente a cada usuário; “Os fatos litigiosos nem sempre são simples de forma a
IV – a guarda dos registros de atendimento individual permitir sua integral revelação ao juiz, ou sua inteira com-
ou de grupo é de responsabilidade do profissional psicólo- preensão por ele, através apenas dos meios usuais de pro-
go ou responsável técnico e obedece ao disposto no Códi- va que são as testemunhas e documentos.
go de Ética Profissional e à Resolução CFP nº 07/2003, que Nem é admissível exigir que o juiz disponha de conhe-
institui o Manual de Documentos Escritos, produzidos pelo cimentos universais a ponto de examinar cientificamente
psicólogo, decorrente de avaliação psicológica. tudo sobre a veracidade e as consequências de todos os
fenômenos possíveis de figurar nos pleitos judiciais. Não
raras vezes, portanto, terá o juiz de se socorrer de auxílio
Art. 6º. Quando em serviço multiprofissional, o registro
de pessoas especializadas como engenheiros, agrimenso-
deve ser realizado em prontuário único.
res, médicos, contadores, químicos etc., para examinar as
Parágrafo único. Devem ser registradas apenas as in- pessoas, coisas ou documentos envolvidos no litígio e for-
formações necessárias ao cumprimento dos objetivos do mar sua convicção para julgar a causa, com a indispensável
trabalho. segurança.” (THEODORO JR., 2002, p. 428)
Art. 7º Esta resolução entra em vigor na data de sua Diferenças entre a avaliação psicológica e perícia (ava-
publicação. liação psicológica em contexto forense):

Art. 8º Revogam-se as disposições em contrário. I) Em relação ao seu objeto: é a questão pertinente que
a avaliação trata de investigar, ou posto de outra forma,
trata-se de um problema a resolver (Maloney and Ward
Fonte: (apud Grisso, 1986, p. 105; Cunha, J. A., 2000, p. 19), uma
http://www.crpsp.org.br/ questão a responder. Lembremos que a Psicologia funciona
por meio da busca de uma resposta a uma pergunta espe-
cífica (Qual é a inteligência do fulano? Por exemplo).
II) Em relação ao objetivo: será dado pela demanda que
O LUGAR DO SABER PSICOLÓGICO é feita ao psicólogo em sua avaliação. Por exemplo, em ca-
NA INSTITUIÇÃO JUDICIÁRIA. sos de disputa de guarda em Vara de Família, recorre-se ao
perito psicólogo no intuito de buscar respostas a questões-
-problemas de origem e natureza psicológicas, mas cujo
objetivo final é definir o guardião legal da criança: Quem
tem as melhores condições psicológicas para o exercício
A ATUAÇÃO DO PSICÓLOGO NO PODER JUDICIÁ- da guarda?
RIO: INTERFACES ENTRE A PSICOLOGIA E O DIREITO
A resolução do problema que a avaliação psicológica
visa sempre recairá sobre um sujeito (Shine, 2003).
Dr. Sidney Shine
A abordagem da Psicologia se caracteriza, então, pela
Psicólogo; Doutor e Mestre em Psicologia pela USP;
dimensão intersubjetiva; em última instância o objeto da
Perito em avaliação das famílias no Tribunal de Justiça de
Psicologia é sempre pertinente ao sujeito. Portanto, toda
São Paulo; Professor do curso ?Saúde Mental e Justiça? Do a questão técnica implica, necessariamente, em uma po-
Instituto Oscar Freire de Medicina Legal da USP sição ética em relação ao sujeito-objeto da avaliação e ao
demandante dela.
As atividades na Vara de Infância e Juventude são mais
relacionadas às Políticas Públicas/ Estado, enquanto nas - sujeito-objeto: quem vai ser avaliado.
Varas de Família e Sucessões atua-se mais na esfera do pri- - demandante: quem solicita a avaliação.
vado.
Mediante as denúncias éticas, o Conselho Regional de A partir das distinções acima, apresenta exemplos em
Psicologia passa a atuar como um segundo Tribunal de Jus- que se configuram as diferenças entre a atuação do Psicó-
tiça, passando o Psicólogo a ser o réu. Pelo fácil acesso a logo no enquadre clínico e no enquadre jurídico e os tipos
Advogados, não é de se estranhar que ocorram mais quei- de problemas que tendem a surgir neste campo.
xas sobre a atuação dos Psicólogos nesta área do Judiciário.
Os processos em Vara da Infância e Juventude correm, em Exemplo 1:
sua maioria, sem a representação por meio de Advogados O Psicólogo realizou perícia em Vara de Família em
constituídos pelas partes. Em casos de Vara de Família, to- uma Ação de Disputa de Guarda. Após entrevistar os adul-
dos são representados legalmente, mesmo que pelos Pro- tos em litígio, chamou as crianças de 10 e 13 anos para
curadores do Estado para aqueles que não podem pagar. uma entrevista psicológica.

150
CONHECIMENTOS ESPECÍFICOS - PSICÓLOGO

Na entrevista, soube que o avô materno manipulava Exemplo 3:


as reações das crianças, incentivando-as a escreverem “bi- O Psicólogo S. recebeu em seu consultório mais um
lhetes de amor” à mãe. No enquadre com as crianças, o menino, com cerca de quatro anos de idade, encaminha-
Psicólogo garantiu total sigilo para o que falassem como do pelo colega que se mudaria de cidade. Depois de um
meio de assegurar uma confiança no vínculo profissional- rápido diagnóstico, começou a atendê-lo, fazendo eventu-
-crianças. almente orientações com a mãe, que era separada do pai
da criança e levava, sozinha, o filho para a psicoterapia. O
Ao redigir o laudo, se deparou com quesitos comple- ex-marido estava em constante briga com a mãe, de modo
mentares do Advogado da parte contrária da mãe, em que que o garoto via o pai somente nos finais de semana, con-
se perguntava ao profissional se os “bilhetes escritos pelas forme estipulado pelo juiz, no processo de separação. No
crianças eram autênticos”. O profissional se viu confronta- entanto, ocorria uma disputa judicial, na qual o casal não
do com o dilema de informar o que sabia no desempenho brigava pela guarda do filho, mas pelo número de visitas
de seu papel e expor as crianças ou protegê-las à custa de feitas pelo pai.
uma informação que detinha de fato.
A mãe dizia sempre nas sessões de orientação que o
Aqui houve o manejo equivocado do enquadre no pai era agressivo, violento, que não era possível o diálogo
atendimento às crianças, garantindo-se um sigilo que não com ele e que era esta a causa de todos os sintomas apre-
pode ser respeitado quando o profissional age como au- sentados pelo menino e da impossibilidade de melhora
xiliar da Justiça e está compromissado a levar ao conheci- dos mesmos.
mento da autoridade judiciária o que for relevante ao pro-
cesso judicial. O Psicólogo, que cada vez mais sabia das agressões
e ameaças do pai via relato da mãe, pensou ser prudente
Toda a questão técnica implica, não se envolver com ele, trabalhando apenas com a mãe e
necessariamente, em uma posição o menino, de modo que nunca chamou o pai para qualquer
ética em relação ao sujeito-objeto da tipo de participação neste trabalho. Ao tomar esta decisão,
avaliação e ao demandante dela. preocupava-se principalmente com o bem-estar da criança
e zelava por seu espaço de terapia, na qual sempre eram
trazidas situações, referentes ao relacionamento com o pai.
Exemplo 2:
O presente Parecer trata de solicitação do Mm. Sr. Juiz
Cerca de três meses depois, a mãe da criança solicitou
Dr. ______________________, da _____ Vara de Família, da Co-
ao psicólogo um relatório sobre o estado de seu filho para
marca ____________, sobre a validade de Avaliação Psicoló-
que, na disputa com o marido, tivesse dados perante o juiz
gica. A Avaliação Psicológica, que se encontra nos Autos
que sustentassem e justificassem o pedido de redução do
do Processo N.º ___ de Separação Judicial, é peça utilizada
número de visitas do pai. O Psicólogo primeiramente hesi-
por uma das partes como prova alegada de incapacida- tou, mas depois, na tentativa de proteger a criança atendi-
de emocional da parte que ficou com a guarda dos filhos da, escreveu o documento, intitulado como ‘Laudo psico-
quando da separação, motivo pelo qual requer do juiz a? lógico’ e não apresentava endereçamento. Iniciava-se com
Revisão de guarda? A parte, agora contestando, solicita a alguns dados da criança e em seguida passava a expor uma
invalidação da Avaliação Psicológica alegando que o docu- análise psicológica da mesma, seguida de informações a
mento não tem respaldo ético legal, vez que o psicólogo respeito de sua relação com a figura paterna, a qual é des-
era muito amigo da parte que está pleiteando a guarda. Diz crita como descontrolada e agressiva.
ainda que aquela avaliação não está isenta da neutralidade
necessária, pois o Psicólogo deu informações baseadas na O profissional aponta os prejuízos causados ao menino
versão do “amigo” e que consigo só falou uma vez, apre- pelo contato com a figura paterna e pelas disputas desta
sentando interpretações pessoais e deturpadas. com a figura materna, cuja relação com a criança é avalia-
Requer, portanto, o Mm. Juiz, Parecer sobre a validade da positivamente. Diante da descrição da figura paterna,
da contestada Avaliação Psicológica. incluindo hipótese de transtorno psiquiátrico, o Psicólogo
faz sugestões quanto à periodicidade das visitas do pai. No
(Modelo de PARECER retirado da Resolução CFP N.º final, sua assinatura, sua inscrição no CRP e a data.
30/2001- revogada. A Resolução atualmente vigente é a de (Extraído do Psi Jornal de Psicologia do CRP SP, jan./
N.º 007/2003) fev. 2004, p. 09)

Este é um exemplo em que o Juiz da causa está soli- Aqui o erro do profissional psicoterapeuta é de extra-
citando um parecer sobre um objeto específico do campo polar a sua competência e seu campo de atuação. Assu-
psicológico - o laudo psicológico em questão. Configura- mindo o trabalho de psicoterapeuta de um menor a pedi-
-se um parecer, uma vez que o psicólogo demandado pelo do de um dos responsáveis, não entrou em contato com o
Juiz não irá reproduzir todo o procedimento de avaliação outro responsável.
psicológica com a família avaliada (isto seria uma nova pe- Além disto, teceu considerações sobre esta pessoa que
rícia), mas responder, pontualmente, sobre a validade téc- não foi avaliada por si, terminando por se intrometer na
nica do documento que ora é posto sobre dúvida. regulamentação de visita deste genitor ao filho.

151
CONHECIMENTOS ESPECÍFICOS - PSICÓLOGO

Há diferentes papéis que o psicólogo pode assumir en- ficar com quem?), o perito adversarial é, assim o denomi-
quanto Perito ou Assistente Técnico, mas há sempre uma namos (SHINE, 2003), aquele que escolhe alguém seja por
dimensão ética no trabalho técnico. um motivo ou outro.
Em outros termos, é o perito que toma a posição de
A TESTEMUNHA (FACTUAL): A testemunha é, por de- dar um laudo conclusivo, entendendo-se? Conclusivo? No
finição, “aquele que sabe porque viu ou ouviu” (Ferreira, sentido de ir ao mérito mesmo da ação que está sendo
1999). julgada.
“O que geralmente acontece é uma confusão entre Woody3 (1978) e Gardner4 (1982) são representantes
dois tipos de ação que o profissional pode ter junto à jus- desta forma de pensamento. A proposta destes autores,
tiça: a situação em que vai agir como testemunha e a outra segundo Berry (1989), é proceder a uma avaliação tão “im-
em que é solicitado aprestar esclarecimentos técnicos so- parcialmente” quanto possível, mas uma vez concluído, o
bre o paciente. perito deveria se colocar ativa e abertamente do lado do
A diferença é sutil, mas fundamental. Como testemu- genitor escolhido como o mais adequado.
nha o Psicólogo deverá prestar informações sobre fatos
PERITO IMPARCIAL: é neutro, não oferece recomenda-
concretos que tenha presenciado e que podem auxiliar na
ções conclusivas, não propõe desfecho.
resolução do caso em questão. Essas informações, portan-
Berry (1989) defende a posição de que o profissional
to, não podem ser baseadas nos depoimentos de seus pa- deve? Simplesmente apresentar as descobertas, opiniões e
cientes ou em inferências que o profissional possa fazer a previsões de forma imparcial e neutra? (Berry, 1989, p.140).
partir dos atendimentos que está realizando” Segundo essa visão, opiniões podem ser emitidas a respei-
(José Alberto Simões Correa, Conselheiro do CRP 06, to dos possíveis resultados de diferentes arranjos de guar-
na edição do Jornal do Conselho de março/abril de 1996, da, mas nunca oferecer recomendações conclusivas.
p. 16) Rovinski (1998) alerta para o perigo de se incorrer em
um julgamento, competência do juiz. Tal posição é, neces-
ASSISTENTE TÉCNICO: é um Perito parcial, porque é sariamente, moral, e, requer uma autoridade legal. Grisso5
um perito da parte, mas deve sempre ser isento. Está con- argumenta, segundo Rovinski (1998), que uma avaliação
dicionado ao que pode saber pela sua experiência (parte psicológica não pode definir operacionalmente um arranjo
do problema). de guarda específico. Isto porque, enquanto uma constru-
ção hipotética e legal, ela teria um componente que escapa
Há diferentes papéis que o psicólogo à competência do profissional de saúde mental. O autor
pode assumir enquanto Perito ou defende que a função do psicólogo seria discriminar os fa-
Assistente Técnico, mas há sempre tores psicológicos em jogo e expor o nível de congruência
uma dimensão ética no trabalho técnico. entre o que se faz (do lado dos pais) e do que se necessita
(do lado da criança), sem julgar se tal nível de congruência
Todo Psicólogo, em um sentido amplo, é um expert é suficiente ou não para o deferimento de pleito em favor
na sua matéria. Ou seja, no que diz respeito à sua área de de um ou de outro.
competência ele é um especialista1. Quando ele é contra-
tado pelo Advogado ou pela parte, ele se tornará um perito “Assim, voltando ao papel do perito na avaliação psico-
parcial dentro da arena jurídica. O termo corrente, mais co- lógica, podemos dizer que sua tarefa é descrever, da forma
mum, é Assistente Técnico2 mais clara e precisa possível, aquilo que o periciado sabe,
entende, acredita ou pode fazer. Não cabe a ele estabelecer,
de forma abreviada, um escore que represente a aceitabili-
A avaliação do Perito não deve
dade ou inaceitabilidade legal do desempenho do sujeito.
responder à questão final do
Quando o perito estiver avaliando incongruência entre as
julgamento, já que o Perito é auxiliar
habilidades de um examinando e as demandas de um con-
da justiça, e não substituto do juiz. texto particular, não deve tentar estabelecer critérios para
definir uma quantidade particular de incongruência que
PERITO ?PISTOLEIRO?: é aquele que faz um laudo a seja sugestiva de incompetência legal. Em outras palavras,
favor da parte, ressaltando o interessa da pessoa contra- sua avaliação não pode responder a questão final sobre o
tante. Não existe compromisso com a isenção, apenas em julgamento. O examinador deve descrever habilidades pes-
servir ao cliente. Porém, defender a parte omitindo dados soais, demandas situacionais e o seu grau de congruência,
é incompatível com a obrigação de dizer a verdade. Se o de maneira a evitar estabelecer o último julgamento ou a
Psicólogo é um pesquisador e um cientista no exercício de conclusão final sobre a competência legal”
sua profissão tal ação seria incompatível com o que se es- (Rovinski, 1998, p. 60).
peraria dele. Isto é diferente para o advogado.
Pela OAB, o Advogado não pode fornecer evidências A avaliação do Perito não deve responder à questão
contrárias ao cliente. final do julgamento, já que o Perito é auxiliar da justiça, e
não substituto do juiz. O Parecer não é sentença, mas fonte
PERITO ADVERSARIAL: escolhe um dos lados do lití- de informação ao juiz. Tal entendimento também é baliza-
gio, dá laudo conclusivo e vai ao mérito da ação. Quando do do ponto de vista do operador do Direito por meio de
a questão final a ser concluída é colocada (a guarda deve citação do mesmo THEODORO JR. (2002):

152
CONHECIMENTOS ESPECÍFICOS - PSICÓLOGO

Valor probante da perícia de educação; a educação deve estar a serviço do aperfei-


çoamento do homem; a educação deve agir em função de
O laudo pericial é o relato das impressões captadas um ideal e não limitar‑se à realidade vigente; a educação
pelo técnico, em torno do fato litigioso, por meio dos co- deve ser uma ciência e não uma mera prática mecânica de
nhecimentos especiais de quem o examinou. reprodução dos valores e crenças vigentes; a formação do
Vale pelas informações que contenha, não pela autori- futuro cidadão passa necessariamente por um embate no
dade de quem o subscreveu, razão pela qual deve o perito espaço público, já que é publicamente que ela se dá e é
indicar as razões em que se fundou para chegar às conclu- para o público que ela prepara. É verdade que o filósofo de
sões enunciadas no laudo (art. 433 do CPC). Konigsberg, na obra mencionada acima, está se referindo à
O perito é apenas um auxiliar da Justiça e não um subs- educação de crianças, mas os princípios educacionais po-
tituto do juiz na apreciação do evento probando. ? Deve dem ser estendidos à educação universitária.
apenas apurar a existência de fatos cuja certificação de- A transformação dos conceitos de moral e ética (do
penda de conhecimento técnico.? Seu parecer não é uma senso comum até um nível mais elaborado), e, mais do que
sentença, mas apenas fonte de informação para o juiz, que isso, a integração da dimensão ética na prática profissional
não fica adstrito ao laudo e pode formar sua convicção de está, em hipótese, relacionada a um ambiente acadêmico
modo contrário à base de outros elementos ou fatos pro- que ofereça oportunidades de assunção de responsabili-
vados no processo (art. 436). dades e de reflexão dirigida. A preocupação central deste
E, realmente, deve ser assim, pois do contrário, o laudo trabalho foi saber como alunos de psicologia avaliam seu
pericial deixaria de ser simples meio de prova para assumir ambiente acadêmico e como os conceitos de moral e éti-
o feitio de decisão arbitral6 e o perito se colocaria numa ca se apresentam no início e no final do curso. O objetivo
posição superior à do próprio juiz, tornando dispensável geral do presente estudo foi, então, comparar a percepção
até mesmo o pronunciamento jurisdicional. dos alunos de psicologia do primeiro e do quinto ano, no
(Theodoro Jr., 2002, p. 434). que diz respeito ao ambiente acadêmico e aos conceitos
de moral e ética. A seguir, serão retomados os conceitos
O psicólogo não pode se descuidar das diversas varia- de ética e moral, os objetivos da formação universitária,
ções do enquadre de trabalho para o seu posicionamento de modo geral, e a história da formação do psicólogo no
técnico e ético. Brasil. As demais seções são: metodologia, resultados e dis-
cussão e considerações finais.
Fonte: Os termos moral e ética são frequentemente utilizados
http://www.crpsp.org.br/portal/comunicacao/cader- hoje em dia e, pela proximidade etimológica, são confun-
nos_tematicos/10/frames/fr_aatuacao.aspx didos e, outras vezes, se fundem. Os conceitos de moral e
ética são empregados como sinônimos, referindo‑se a um
conjunto de regras de conduta consideradas como obriga-
ÉTICA PROFISSIONAL. tórias. Segundo La Taille, tal sinonímia é aceitável, já que
os vocábulos derivam de duas culturas (romana e grega),
que assim nomeavam o campo da reflexão sobre costumes.
Piaget define moral como “sistema de regras” e a morali-
A MORAL E ÉTICA EM PSICOLOGIA dade como o “respeito que o indivíduo adquire por essas
regras”. Em suas pesquisas, o autor entende que há dois
“Que lugar ocupam moral e ética na formação do tipos de moralidade: a heterônoma, ligada ao realismo mo-
psicólogo?”. Durante os anos de formação, o estudante ral e às restrições impostas pelos adultos ou outra figura de
acumula inúmeros conhecimentos, lê diversos autores, é autoridade, e a autônoma, ligada à cooperação e à respon-
apresentado a vários procedimentos e técnicas de traba- sabilidade subjetiva, que considera, além das aparências, as
lho. Entretanto, que relevância é atribuída à questão da intenções e motivos subjacentes. Embora Piaget não tenha
atuação moral e ética do profissional? Pressupõe‑se que estabelecido as diferenças conceituais entre moral e ética,
o ambiente acadêmico deva proporcionar ao graduando é possível considerarmos que, havendo diferentes níveis de
oportunidades para que o aluno possa, gradativamente, desenvolvimento moral, a ética se relaciona a níveis mais
tornar‑se capaz de perceber que sua atuação profissional evoluídos, uma vez que envolve a reflexão a respeito das
implica não apenas o conhecimento de regras, mas cons- regras e não necessariamente o seu cumprimento. Assim
ciência e reflexão a respeito de seus valores, visando a um como Piaget, Kohlberg também não estabeleceu essa di-
exercício profissional que tenha o ser humano como um ferença claramente, mas, na medida em que relaciona o
fim em si mesmo. imperativo categórico aos estágios pós‑convencionais,
Em sua obra Sobre a pedagogia, Kant discute como também permite que pensemos na ética como uma supe-
a formação e a educação dos homens devem ocorrer de ração do nível das regras para sua relativização em nome
modo que os tornem “capazes de desejar e buscar dig- de princípios. Vazquez conceitua moral como um conjunto
nidade e respeito igual para todos”. La Taille analisa essa de princípios, valores, prescrições e atos considerados vá-
mesma obra e destaca os princípios da educação sob o lidos pelo agrupamento social, enquanto a ética se ocupa
enfoque kantiano: O homem é a única criatura suscetível das condições objetivas e subjetivas do ato moral.

153
CONHECIMENTOS ESPECÍFICOS - PSICÓLOGO

Na perspectiva deste trabalho, é necessário que se faça nos leva a refletir sobre a necessidade de desenvolver o
a distinção entre os campos moral e ético. La Taille propõe potencial de agente de cada aluno, sua capacidade de ad-
que façamos a seguinte diferenciação: a moral refere‑se às ministrar seu desenvolvimento educativo ou sua capacida-
leis que normatizam as condutas humanas, e a ética corres- de de atuar com autonomia no processo de aprendizagem.
ponde aos ideais que dão sentido à vida. La Taille propõe Segundo Dias, estudos desenvolvidos por investiga-
que cada palavra seja associada a uma indagação, de modo dores brasileiros, sobre a influência da escola no desen-
que moral corresponde à pergunta: “como devo agir?”, en- volvimento da consciência moral dos alunos, indicam que
quanto ética corresponde a “que vida eu quero viver?”. As- o modelo educacional vigente na maioria das escolas é o
sim, chamaremos de moral, por exemplo, mandamentos heterônomo, com caráter coercitivo e uniformizante, o que
como “não matarás”, uma vez que consistem em leis que não contribui para a consecução dos objetivos educacio-
impõem deveres; de ética, chamaremos ideais como a dig- nais, que, em última análise, significam desenvolvimento
nidade do ser humano. de sujeitos racionais, críticos, livres e autônomos. Nessa di-
reção, as oportunidades de assunção de responsabilidades
Segundo Libâneo “educar (em latim, educare) é condu- são aquelas em que os alunos adquirem o conhecimento
zir de um estado a outro, é modificar numa certa direção em situações que eles próprios tenham tido possibilidade
o que é suscetível de educação”. O ato de educar implica de definir ou, ao menos, ajudar a delinear, ou seja, que pres-
atividade de interação entre os seres sociais, tanto no nível supõem seu envolvimento. Reiman explica que, na assun-
intrapessoal como no nível da influência do meio. Aranha ção de responsabilidade, a ação precede e dá substância à
acrescenta que se pode deduzir que exista a interação de conscientização intelectual (reflexão), que, por sua vez, se
três componentes na educação: um agente, uma mensa- desenvolve a partir da assunção de responsabilidade, como
gem transmitida e um educando. A mensagem transmitida em um círculo virtuoso. Na ausência de uma interação so-
só o é em um ambiente acadêmico que inclua atividades cial envolvendo um novo e mais complexo papel, parece
curriculares, semicurriculares e extracurriculares, além de improvável que o aluno inicie as reflexões necessárias para
qualquer outra atividade educacional que coloque o edu- mudanças estruturais.
cando como agente de seu processo de aprendizagem e O desafio proporcionado pelas atividades, porém, não
com possibilidade de refletir a respeito desse processo. é suficiente para estimular o desenvolvimento moral. Os
Nesse sentido, espera‑se que o ambiente acadêmico pro- estudantes necessitam também de reflexão dirigida, defi-
porcione ao graduando oportunidade de assumir respon- nida aqui como oportunidades de receber o adequado su-
sabilidades e de se tornar um profissional crítico e reflexivo, porte oferecido por professores e/ou alunos mais experien-
capaz de compreender o contexto em que vive, além de es- tes, para que se possam discutir os novos papéis, as novas
tar comprometido com a ética e a política. Para Rué, a au- experiências de aprendizagem. Desse modo, seria possível
torregulação, chave no desenvolvimento de aprender com dizer que o desafio deve ser seguido ou acompanhado por
autonomia, implica um sujeito atento à intencionalidade de um apoio, em forma de oportunidades de reflexão, acon-
sua ação e o questionamento a respeito de seu saber e agir selhamento competente ou feedback sobre problemas
ou saber/fazer. relacionados com o processo de aprendizagem. A expres-
Ao longo da história, a educação superior no Brasil so- são “ambiente de aprendizagem favorável” será utilizada
freu um intenso processo de mudanças, transformações, aqui como referência à combinação de oportunidades de
acontecimentos e crises, tanto na concepção ou propó- assunção de responsabilidade e reflexão dirigida, ou seja,
sito da universidade como instituição, como em relação de ação e reflexão. Tal ambiente implica uma abordagem
aos métodos de ensino. Há inúmeros questionamentos a não tradicional de educação, diferente daquela em que o
respeito de quais devam ser os objetivos da educação su- professor é o transmissor do conhecimento e o aluno o
perior. Oliveira chama a atenção para a tensão que existe depositário, em que as aulas são expositivas, sem possibili-
quanto à finalidade da educação, que pode estar voltada dades de discussão, e o valor primeiro é a memorização de
para a ética e cidadania ou para o mercado. Nesse con- conteúdo. Essa abordagem tem se mostrado ineficaz para
texto, Rué recorda um desejo e uma queixa constante dos a promoção do desenvolvimento cognitivo, em geral, e da
professores: que seus alunos sejam mais autônomos. Tal autonomia moral, em particular.
queixa está relacionada com o fato de eles não serem sufi- Cabe, agora, discutir de que modo a formação do
cientemente ou absolutamente autônomos. Por que esses psicólogo tem cumprido seu papel de formador do pro-
desejos não se realizam? O autor conclui que alcançar a au- fissional autônomo. Pereira e Pereira Neto propõem uma
tonomia depende, sobretudo, do desenvolvimento do po- compreensão da historicidade da profissionalização da psi-
tencial de autorregulação do sujeito. As referências sobre cologia para atingir essa avaliação a respeito de como é,
o que se espera do ambiente acadêmico para que o edu- atualmente, a formação desse profissional. A institucionali-
cando descubra seus pontos fortes, seus interesses, suas zação da formação do psicólogo ocorreu pela primeira vez
necessidades e competências na aprendizagem e para a em 1946. O psicólogo habilitado deveria frequentar os três
aprendizagem, apontam para novos pilares de desenvolvi- primeiros anos de filosofia, biologia, fisiologia, antropolo-
mento das relações de ensino e aprendizagem. Dentre es- gia ou estatística e fazer, então, cursos especializados de
ses novos pilares, incluem‑se o papel central do indivíduo psicologia. Em 1957, foi iniciada a formação do psicólogo
que aprende no processo educativo e as relações que ele é em estabelecimento de nível superior e com currículo ma-
capaz de estabelecer com aquilo que lhe é oferecido. Isso joritariamente dedicado à psicologia, na Pontifícia Univer-

154
CONHECIMENTOS ESPECÍFICOS - PSICÓLOGO

sidade Católica do Rio de Janeiro. Nas décadas de 1940 e estudo sobre o desenvolvimento moral dos profissionais
1950, o psicólogo passou a atuar, cada vez mais, nas áreas de saúde tem demonstrado que o ensino universitário por
da educação e do trabalho. Em 27 de agosto de 1962, foi si só não promove o desenvolvimento moral, mas que são
aprovada a Lei n. 4.119, que regulamentou a profissão de necessários outros fatores, dentre eles um ambiente de
psicólogo. Nesse mesmo ano, o Conselho Federal de Edu- ensino que ofereça oportunidades de desenvolvimento do
cação estabeleceu o currículo mínimo e a duração do curso raciocínio autônomo na interação com o meio.
de psicologia. Entre os anos de 1970 e 1980, existiam três
grandes áreas de atuação na psicologia: educação, traba- Fonte:
lho e clínica. Em 1988, o Conselho Federal de Psicologia BATAGLIA; P. U. R.; BORTOLANZA,M. R. Disponível em
fez o primeiro levantamento sobre a profissão, constatando http://pepsic.bvsalud.org/
que, nessa época, havia 58.277 profissionais inscritos (hoje
esse número está quadriplicado). Segundo Dimenstein, a A PRÁXIS DO PSICÓLOGO FACE AO CÓDIGO DE
década de 1980 foi o momento para a inserção do psicólo- ÉTICA PROFISSIONAL
go nos sistemas de saúde, já que se procuravam desenvol-
ver serviços substitutivos ao hospital, mais eficazes e com Ética profissional: fatos e possibilidades:
menor custo.
Com relação à metodologia de ensino‑aprendizagem, Para TEIXEIRA (1999), a questão ética profissional em
Ribeiro e Luzio e Ferreira Neto e Penna analisam como o Psicologia é discutida a partir de uma tentativa de análise
fenômeno psicológico é contemplado nas diretrizes curri- comportamental de alguns enunciados verbais sobre mo-
culares, portarias e documentos oficiais, ou seja, como pro- ral e ética. A dissociação entre “dizer” e o “fazer” éticos a
dução social e multideterminada. Os autores identificam complexidade envolvida na assimilação de contingências
que a formação do psicólogo visa ao modelo clínico tradi- sociais, regras ou leis são destacadas. Algumas situações
cional, em vez de uma perspectiva preventivo‑comunitária. circunscritas ao meio acadêmico e aos diversos campos de
Eles reconhecem que as diretrizes aspiram a uma mudança atuação do psicólogo são mencionadas como suscetíveis
paradigmática na formação do psicólogo, mas há proble- de questionamentos éticos. O reconhecimento e a atribui-
mas na operacionalização da mudança. ção de direitos são apontados como bases para o desen-
Bataglia realizou uma pesquisa com estudantes de volvimento de reconhecimentos éticos e para a superação
psicologia e psicólogos e os resultados mostraram que da dissociação entre o “dizer” e o “fazer” éticos. Essa dis-
a grande maioria apresentou um nível de juízo moral ca- sociação entre o “dizer” e o “fazer” éticos e a amplitude
das questões envolvendo ética, exigem um detalhamento
racterizado pelo relativismo instrumental hedonista e pela
da própria noção de ética que permita, posteriormente,
satisfação de valores grupais. Além disso, mostraram co-
sua especificação enquanto ética profissional, no campo
nhecimento de um único item do código de ética dos psi-
da Psicologia. Segundo Ferreira (1986), o estudo dos juí-
cólogos e, mesmo, de forma incompleta: o sigilo.
zos de apreciação referentes à conduta humana suscetível
Concluiu ser urgente uma intervenção na formação
de qualificação do ponto de vista do bem e do mal, seja
dos profissionais, começando pelo curso de Ética Profis-
relativamente a determinada sociedade, seja de modo ab-
sional, disciplina que tem por objetivo básico esclarecer o
soluto. A ética é a expressão da medida. É a garantia da
psicólogo acerca de seus deveres com relação a pacientes, harmonia que resulta da boa conduta da alma e que de-
colegas, sociedade etc. Em outra pesquisa, Bataglia levanta termina o lugar certo de qualquer coisa (e de qualquer ato)
dados que mostram que o curso de Ética Profissional, tal no mundo. Os limites do exercício da profissão do psicólo-
como está estruturado atualmente, não leva a uma mudan- go estão estabelecidos no Código de Ética Profissional dos
ça significativa na competência moral dos alunos, entendi- Psicólogos, aprovado pelo Conselho Federal de Psicologia
da aqui como a capacidade de emitir juízos morais e agir em 03/08/79. Desde então as questões éticas relacionadas
de acordo com tais juízos. Conclui, ainda, que não só a dis- a este campo de atuação profissional vêm se ampliando
ciplina Ética Profissional, mas todo o ambiente acadêmico e se delineando em novas formulações. Skinner (1982)
deve colaborar para a construção de um profissional ético. afirma que comportamento “moral ou justo é um produ-
Por outro lado, Schillinger constata uma clara influência to de tipos especiais de contingências sociais” e sugere
do ambiente acadêmico em termos de oportunidades de que “precisamos analisar tais contingências se pretende-
assunção de responsabilidade e reflexão dirigida na cons- mos construir um mundo em que as pessoas ajam moral
trução da competência moral. Em seu estudo, compara o e equitativamente.” O campo de atuação do psicólogo é
ambiente acadêmico e a competência moral em três cur- muito amplo, o que torna muito difícil a tarefa de analisá-
sos (Psicologia, Administração de Empresas e Medicina) -lo em suas implicações éticas. E por fim, afirmaria que o
em universidades de três países (Suíça, Alemanha e Brasil). desenvolvimento do comportamento ético é uma questão
Nas universidades que oferecem oportunidades para seus de aprendizagem, e esta pode ocorrer em qualquer lugar. A
alunos assumirem responsabilidade sobre seu processo de escola e o ensino formais não constituem as únicas condi-
ensino‑aprendizagem, bem como oportunidade de refle- ções para a aquisição desse repertório comportamental. Se
xão dirigida, percebe‑se um aumento significativo no esco- todos exigirem respeito de seus direitos, não haverá quem
re de competência moral. Por outro lado, em universidades os desrespeitem. O comportamento ético concebido, de-
em que tais oportunidades não ocorrem, há uma estabili- senvolvido e controlado dessa forma produziriam a supe-
dade ou até regressão nessa competência. Para Oliveira o ração da dissociação entre retórica e fato.

155
CONHECIMENTOS ESPECÍFICOS - PSICÓLOGO

Por uma ética na saúde: algumas reflexões sobre a Percepção de alunos de graduação em psicologia
ética e o ser ético na atuação do psicólogo: sobre a conduta ética dos psicólogos:
De acordo com WACHELKE e outros, nos ensinam que,
Segundo MEDEIROS (2002), a ética vem sendo tema com base no entendimento de que as experiências nos cur-
frequente em discussão, abarcando várias instâncias da sos de graduação em psicologia constituem uma influência
sociedade. Nas instituições de Saúde são prementes as relevante na formação das representações dos alunos, que
questões referentes a parâmetros e limites na intervenção por sua vez guiarão seus comportamentos futuros. Os psi-
sobre os seres humanos. O psicólogo, profissional presen- cólogos apresentavam alguns comportamentos pautados
te nas instituições de saúde, tem a sua prática atravessada
pelos princípios do Código de Ética Profissional do Psicólo-
por vivências de grande significado na vida das pessoas. A
go. A ética profissional do psicólogo é um assunto de suma
presença de situações como ansiedades, angústias, medos,
importância para os psicólogos, visto que devem basear
desencadeiam crises pessoais e familiares e impõem aos
profissionais vinculados aos cuidados com a saúde inúme- suas práticas profissionais no respectivo código de conduta
ras dúvidas com relação a práticas que sejam coerentes elaborado e fiscalizado pelo CFP. De acordo com o próprio
com o propósito ético da ciência que representam. Além código, o sistema ético proposto” ...permitirá uma atuação
das questões éticas que emergem em decorrência das profissional engajada social e politicamente no mundo, e
biotecnologias, são inúmeras as situações que constituem não um profissional a serviço exclusivo do individuo” (CRP,
dilemas éticos na relação do psicólogo com a pessoa aten- 2002). Embora fosse desejável que todos os profissionais
dida e/ou familiares desta, ou na relação com a equipe de respeitassem as orientações do Código, sabe-se que isso
trabalho. Até onde manter o sigilo profissional? É possível nem sempre ocorre. Os princípios fundamentais que re-
quebrar sigilo? Em quais situações? Como agir a atitudes gem o Código são pilares que norteiam a atuação profis-
antiéticas de colegas de trabalho? Além destas questões sional, mas nem sempre são cumpridos em sua totalidade.
é possível enumerar muitas outras. Autores como Herreo Alguns profissionais, como ocorre em todos os campos
(1999), Chiattone e Sebastiani (1997), Berlinguer (1996) e de atuação, utilizam-se dos conhecimentos e do estatuto
Lepargneur (1996), fazem referências a falência dos prin- proporcionados pela psicologia para benefício próprio, em
cípios morais em nossa época. Frente á essa problemática, detrimento do bem-estar da sociedade. As decorrências
são muitas as questões envolvendo a ética, constituindo de uma percepção dos psicólogos enquanto profissionais
desafios constantes para a Psicologia e para as demais pouco éticos são graves e numerosas. Em primeiro lugar,
Ciências. Chauí (1995) compreende ética como “Filosofia
são estimuladas relações de desrespeito e desconfiança
moral, isto é, uma reflexão que discuta, problematize e in-
rumo aos pares, o que pode empobrecer a atuação pro-
terprete o significado dos valores morais.” Considerando a
ética, convém questionar: será que a conduta ética pode fissional de psicólogos e enfraquecer a classe profissional
sustentar-se unicamente no cumprimento do Código de de modo geral. Ainda mais preocupante, seria a instalação
Ética Profissional do Psicólogo? E mais: O Código está no de uma atmosfera de permissividade entre os psicólogos
campo da ética ou no campo da moral? Faz-se necessária frente a comportamentos antiéticos; pensar que, aos depa-
a compreensão de que o Código não traz, em seus cin- rar-se com situações delicadas do ponto de vista do dever-
qüenta artigos, respostas parecidas às questões éticas. Daí -ser psicológico os profissionais tenham crenças como “Os
a necessidade de não limitar-se aos conteúdos inscritos no psicólogos agem assim mesmo”, ou então que “isso é nor-
Código. Fica demarcado, portanto, o caráter referencial do mal”, que acabem por pautar uma conduta inadequada. É
mesmo, que serve como um relevante norteador para as preciso que os profissionais de psicologia possuam sólida
atividades profissionais da categoria, pois trata de direitos, formação ética, incluindo a moral a conduta ética, apoiados
deveres e responsabilidades. As particularidades de cada pelo Código.
situação exigem uma ampla reflexão que inclui o Código
de Ética Profissional do Psicólogo, mas não se limita á ele. Fonte:
E também refere-se à ação do psicólogo guiada por seus REIS, D. K.. dos; RODRIGUES, A. S. e MELO, C. M. da S.
valores e princípios, construídos ao longo de sua formação
pessoal e profissional. Certamente devemos considerar CÓDIGO DE ÉTICA PROFISSIONAL DO PSICÓLOGO
que os princípios do Psicólogo são relevantes e devem ser
considerados e respeitados. Pautar o agir em princípios
RESOLUÇÃO CFP Nº 010/05
sustentados pela ética talvez fosse o recurso para uma ação
profissional livre de padrões fundamentados em regras,
normas ou valores pessoais, naquilo que o psicólogo Aprova o Código de Ética Profissional do Psicólogo.
valoriza ou considera verdadeiro, sem considerar o que
o outro acredita e valoriza, contribuem unicamente para O CONSELHO FEDERAL DE PSICOLOGIA, no uso de
a moralização, opressão e marginalização daqueles que suas atribuições legais e regimentais, que lhe são conferi-
esperam ser tratados com respeito e dignidade. das pela Lei no 5.766, de 20 de dezembro de 1971;
CONSIDERANDO o disposto no Art. 6º, letra “e”, da Lei
no 5.766 de 20/12/1971, e o Art. 6º, inciso VII, do Decreto
nº 79.822 de 17/6/1977;

156
CONHECIMENTOS ESPECÍFICOS - PSICÓLOGO

CONSIDERANDO o disposto na Constituição Federal Consoante com a conjuntura democrática vigente, o


de 1988, conhecida como Constituição Cidadã, que con- presente Código foi construído a partir de múltiplos espa-
solida o Estado Democrático de Direito e legislações dela ços de discussão sobre a ética da profissão, suas responsa-
decorrentes; bilidades e compromissos com a promoção da cidadania.
CONSIDERANDO decisão deste Plenário em reunião O processo ocorreu ao longo de três anos, em todo o país,
realizada no dia 21 de julho de 2005; com a participação direta dos psicólogos e aberto à socie-
dade.
RESOLVE: Este Código de Ética pautou-se pelo princípio geral de
aproximar-se mais de um instrumento de reflexão do que
Art. 1º - Aprovar o Código de Ética Profissional do Psi- de um conjunto de normas a serem seguidas pelo psicólo-
cólogo. go. Para tanto, na sua construção buscou-se:
a. Valorizar os princípios fundamentais como grandes
Art. 2º - A presente Resolução entrará em vigor no dia eixos que devem orientar a relação do psicólogo com a
27 de agosto de 2005. sociedade, a profissão, as entidades profissionais e a ciên-
cia, pois esses eixos atravessam todas as práticas e estas
Art. 3º - Revogam-se as disposições em contrário, em demandam uma contínua reflexão sobre o contexto social
especial a Resolução CFP n º 002/87. e institucional.
b. Abrir espaço para a discussão, pelo psicólogo, dos
Brasília, 21 de julho de 2005.
limites e interseções relativos aos direitos individuais e
coletivos, questão crucial para as relações que estabelece
Apresentação
com a sociedade, os colegas de profissão e os usuários ou
Toda profissão define-se a partir de um corpo de prá- beneficiários dos seus serviços.
ticas que busca atender demandas sociais, norteado por c. Contemplar a diversidade que configura o exercício
elevados padrões técnicos e pela existência de normas éti- da profissão e a crescente inserção do psicólogo em con-
cas que garantam a adequada relação de cada profissional textos institucionais e em equipes multiprofissionais.
com seus pares e com a sociedade como um todo. d. Estimular reflexões que considerem a profissão como
Um Código de Ética profissional, ao estabelecer padrões um todo e não em suas práticas particulares, uma vez que
esperados quanto às práticas referendadas pela respectiva os principais dilemas éticos não se restringem a práticas
categoria profissional e pela sociedade, procura fomentar a específicas e surgem em quaisquer contextos de atuação.
autorreflexão exigida de cada indivíduo acerca da sua prá- Ao aprovar e divulgar o Código de Ética Profissional do
xis, de modo a responsabilizá-lo, pessoal e coletivamente, Psicólogo, a expectativa é de que ele seja um instrumento
por ações e suas consequências no exercício profissional. A capaz de delinear para a sociedade as responsabilidades e
missão primordial de um código de ética profissional não deveres do psicólogo, oferecer diretrizes para a sua forma-
é de normatizar a natureza técnica do trabalho, e, sim, a de ção e balizar os julgamentos das suas ações, contribuindo
assegurar, dentro de valores relevantes para a sociedade para o fortalecimento e ampliação do significado social da
e para as práticas desenvolvidas, um padrão de conduta profissão.
que fortaleça o reconhecimento social daquela categoria.
Códigos de Ética expressam sempre uma concepção de ho- Princípios Fundamentais
mem e de sociedade que determina a direção das relações
entre os indivíduos. Traduzem-se em princípios e normas I. O psicólogo baseará o seu trabalho no respeito e na
que devem se pautar pelo respeito ao sujeito humano e promoção da liberdade, da dignidade, da igualdade e da
seus direitos fundamentais. Por constituir a expressão de integridade do ser humano, apoiado nos valores que em-
valores universais, tais como os constantes na Declaração basam a Declaração Universal dos Direitos Humanos.
Universal dos Direitos Humanos; socioculturais, que refle-
II. O psicólogo trabalhará visando promover a saúde
tem a realidade do país; e de valores que estruturam uma
e a qualidade de vida das pessoas e das coletividades e
profissão, um código de ética não pode ser visto como um
contribuirá para a eliminação de quaisquer formas de ne-
conjunto fixo de normas e imutável no tempo. As socie-
dades mudam, as profissões transformam-se e isso exige, gligência, discriminação, exploração, violência, crueldade e
também, uma reflexão contínua sobre o próprio código de opressão.
ética que nos orienta. III. O psicólogo atuará com responsabilidade social,
A formulação deste Código de Ética, o terceiro da pro- analisando crítica e historicamente a realidade política,
fissão de psicólogo no Brasil, responde ao contexto orga- econômica, social e cultural.
nizativo dos psicólogos, ao momento do país e ao estágio IV. O psicólogo atuará com responsabilidade, por meio
de desenvolvimento da Psicologia enquanto campo cien- do contínuo aprimoramento profissional, contribuindo
tífico e profissional. Este Código de Ética dos Psicólogos é para o desenvolvimento da Psicologia como campo cientí-
reflexo da necessidade, sentida pela categoria e suas enti- fico de conhecimento e de prática.
dades representativas, de atender à evolução do contexto V. O psicólogo contribuirá para promover a universali-
institucional-legal do país, marcadamente a partir da pro- zação do acesso da população às informações, ao conhe-
mulgação da denominada Constituição Cidadã, em 1988, e cimento da ciência psicológica, aos serviços e aos padrões
das legislações dela decorrentes. éticos da profissão.

157
CONHECIMENTOS ESPECÍFICOS - PSICÓLOGO

VI. O psicólogo zelará para que o exercício profissional Art. 2º – Ao psicólogo é vedado:
seja efetuado com dignidade, rejeitando situações em que
a Psicologia esteja sendo aviltada. a) Praticar ou ser conivente com quaisquer atos que
VII. O psicólogo considerará as relações de poder nos caracterizem negligência, discriminação, exploração, vio-
contextos em que atua e os impactos dessas relações sobre lência, crueldade ou opressão;
as suas atividades profissionais, posicionando-se de forma b) Induzir a convicções políticas, filosóficas, morais,
crítica e em consonância com os demais princípios deste ideológicas, religiosas, de orientação sexual ou a qualquer
Código. tipo de preconceito, quando do exercício de suas funções
profissionais;
Das Responsabilidades do Psicólogo c) Utilizar ou favorecer o uso de conhecimento e a uti-
lização de práticas psicológicas como instrumentos de cas-
Art. 1º – São deveres fundamentais dos psicólogos: tigo, tortura ou qualquer forma de violência;
d) Acumpliciar-se com pessoas ou organizações que
a) Conhecer, divulgar, cumprir e fazer cumprir este Có- exerçam ou favoreçam o exercício ilegal da profissão de
digo; psicólogo ou de qualquer outra atividade profissional;
b) Assumir responsabilidades profissionais somente e) Ser conivente com erros, faltas éticas, violação de
por atividades para as quais esteja capacitado pessoal, teó- direitos, crimes ou contravenções penais praticados por
rica e tecnicamente; psicólogos na prestação de serviços profissionais;
c) Prestar serviços psicológicos de qualidade, em con- f) Prestar serviços ou vincular o título de psicólogo a
dições de trabalho dignas e apropriadas à natureza desses serviços de atendimento psicológico cujos procedimentos,
serviços, utilizando princípios, conhecimentos e técnicas técnicas e meios não estejam regulamentados ou reconhe-
reconhecidamente fundamentados na ciência psicológica, cidos pela profissão;
na ética e na legislação profissional; g) Emitir documentos sem fundamentação e qualidade
d) Prestar serviços profissionais em situações de ca- técnico-científica;
lamidade pública ou de emergência, sem visar benefício h) Interferir na validade e fidedignidade de instrumen-
pessoal; tos e técnicas psicológicas, adulterar seus resultados ou fa-
e) Estabelecer acordos de prestação de serviços que
zer declarações falsas;
respeitem os direitos do usuário ou beneficiário de serviços
i) Induzir qualquer pessoa ou organização a recorrer a
de Psicologia;
seus serviços;
f) Fornecer, a quem de direito, na prestação de serviços
j) Estabelecer com a pessoa atendida, familiar ou ter-
psicológicos, informações concernentes ao trabalho a ser
ceiro, que tenha vínculo com o atendido, relação que possa
realizado e ao seu objetivo profissional;
interferir negativamente nos objetivos do serviço prestado;
g) Informar, a quem de direito, os resultados decorren-
k) Ser perito, avaliador ou parecerista em situações nas
tes da prestação de serviços psicológicos, transmitindo so-
quais seus vínculos pessoais ou profissionais, atuais ou an-
mente o que for necessário para a tomada de decisões que
teriores, possam afetar a qualidade do trabalho a ser reali-
afetem o usuário ou beneficiário;
h) Orientar a quem de direito sobre os encaminhamen- zado ou a fidelidade aos resultados da avaliação;
tos apropriados, a partir da prestação de serviços psico- l) Desviar para serviço particular ou de outra institui-
lógicos, e fornecer, sempre que solicitado, os documentos ção, visando benefício próprio, pessoas ou organizações
pertinentes ao bom termo do trabalho; atendidas por instituição com a qual mantenha qualquer
i) Zelar para que a comercialização, aquisição, doação, tipo de vínculo profissional;
empréstimo, guarda e forma de divulgação do material m) Prestar serviços profissionais a organizações con-
privativo do psicólogo sejam feitas conforme os princípios correntes de modo que possam resultar em prejuízo para
deste Código; as partes envolvidas, decorrentes de informações privile-
j) Ter, para com o trabalho dos psicólogos e de outros giadas;
profissionais, respeito, consideração e solidariedade, e, n) Prolongar, desnecessariamente, a prestação de ser-
quando solicitado, colaborar com estes, salvo impedimen- viços profissionais;
to por motivo relevante; o) Pleitear ou receber comissões, empréstimos, doa-
k) Sugerir serviços de outros psicólogos, sempre que, ções ou vantagens outras de qualquer espécie, além dos
por motivos justificáveis, não puderem ser continuados honorários contratados, assim como intermediar transa-
pelo profissional que os assumiu inicialmente, fornecendo ções financeiras;
ao seu substituto as informações necessárias à continuida- p) Receber, pagar remuneração ou porcentagem por
de do trabalho; encaminhamento de serviços;
l) Levar ao conhecimento das instâncias competentes q) Realizar diagnósticos, divulgar procedimentos ou
o exercício ilegal ou irregular da profissão, transgressões a apresentar resultados de serviços psicológicos em meios
princípios e diretrizes deste Código ou da legislação pro- de comunicação, de forma a expor pessoas, grupos ou or-
fissional. ganizações.

158
CONHECIMENTOS ESPECÍFICOS - PSICÓLOGO

Art. 3º – O psicólogo, para ingressar, associar-se ou §1° – No caso de não se apresentar um responsável
permanecer em uma organização, considerará a missão, a legal, o atendimento deverá ser efetuado e comunicado às
filosofia, as políticas, as normas e as práticas nela vigentes autoridades competentes;
e sua compatibilidade com os princípios e regras deste Có- §2° – O psicólogo responsabilizar-se-á pelos
digo. encaminhamentos que se fizerem necessários para garantir
Parágrafo único: Existindo incompatibilidade, cabe ao a proteção integral do atendido.
psicólogo recusar-se a prestar serviços e, se pertinente,
apresentar denúncia ao órgão competente. Art. 9º – É dever do psicólogo respeitar o sigilo pro-
fissional a fim de proteger, por meio da confidencialidade,
Art. 4º – Ao fixar a remuneração pelo seu trabalho, o a intimidade das pessoas, grupos ou organizações, a que
psicólogo: tenha acesso no exercício profissional.

a) Levará em conta a justa retribuição aos serviços Art. 10° – Nas situações em que se configure conflito
prestados e as condições do usuário ou beneficiário; entre as exigências decorrentes do disposto no Art. 9º e
b) Estipulará o valor de acordo com as características as afirmações dos princípios fundamentais deste Código,
da atividade e o comunicará ao usuário ou beneficiário an- excetuando-se os casos previstos em lei, o psicólogo po-
tes do início do trabalho a ser realizado; derá decidir pela quebra de sigilo, baseando sua decisão na
c) Assegurará a qualidade dos serviços oferecidos in- busca do menor prejuízo.
dependentemente do valor acordado. Parágrafo único – Em caso de quebra do sigilo previsto
no caput deste artigo, o psicólogo deverá restringir-se a
Art. 5º – O psicólogo, quando participar de greves ou prestar as informações estritamente necessárias.
paralisações, garantirá que:
Art. 11° – Quando requisitado a depor em juízo, o psi-
a) As atividades de emergência não sejam interrompi- cólogo poderá prestar informações, considerando o previs-
das; to neste Código.
b) Haja prévia comunicação da paralisação aos usuá-
rios ou beneficiários dos serviços atingidos pela mesma. Art. 12° – Nos documentos que embasam as atividades
em equipe multiprofissional, o psicólogo registrará apenas
Art. 6º – O psicólogo, no relacionamento com profis- as informações necessárias para o cumprimento dos obje-
sionais não psicólogos: tivos do trabalho.

a) Encaminhará a profissionais ou entidades habilita- Art. 13° – No atendimento à criança, ao adolescente


dos e qualificados demandas que extrapolem seu campo ou ao interdito, deve ser comunicado aos responsáveis o
de atuação; estritamente essencial para se promoverem medidas em
b) Compartilhará somente informações relevantes para seu benefício.
qualificar o serviço prestado, resguardando o caráter confi-
dencial das comunicações, assinalando a responsabilidade, Art. 14° – A utilização de quaisquer meios de registro
de quem as receber, de preservar o sigilo. e observação da prática psicológica obedecerá às normas
deste Código e a legislação profissional vigente, devendo o
Art. 7º – O psicólogo poderá intervir na prestação de usuário ou beneficiário, desde o início, ser informado.
serviços psicológicos que estejam sendo efetuados por ou-
tro profissional, nas seguintes situações: Art. 15° – Em caso de interrupção do trabalho do psicó-
logo, por quaisquer motivos, ele deverá zelar pelo destino
a) A pedido do profissional responsável pelo serviço; dos seus arquivos confidenciais.
b) Em caso de emergência ou risco ao beneficiário ou § 1° – Em caso de demissão ou exoneração, o psicólogo
usuário do serviço, quando dará imediata ciência ao pro- deverá repassar todo o material ao psicólogo que vier a
fissional; substituí-lo, ou lacrá-lo para posterior utilização pelo
c) Quando informado expressamente, por qualquer psicólogo substituto.
uma das partes, da interrupção voluntária e definitiva do § 2° – Em caso de extinção do serviço de Psicologia,
serviço; o psicólogo responsável informará ao Conselho Regional
d) Quando se tratar de trabalho multiprofissional e a de Psicologia, que providenciará a destinação dos arquivos
intervenção fizer parte da metodologia adotada. confidenciais.

Art. 8º – Para realizar atendimento não eventual de Art. 16° – O psicólogo, na realização de estudos, pes-
criança, adolescente ou interdito, o psicólogo deverá obter quisas e atividades voltadas para a produção de conheci-
autorização de ao menos um de seus responsáveis, obser- mento e desenvolvimento de tecnologias:
vadas as determinações da legislação vigente:

159
CONHECIMENTOS ESPECÍFICOS - PSICÓLOGO

a) Avaliará os riscos envolvidos, tanto pelos procedi- d) Suspensão do exercício profissional, por até 30 (trin-
mentos, como pela divulgação dos resultados, com o obje- ta) dias, ad referendum do Conselho Federal de Psicologia;
tivo de proteger as pessoas, grupos, organizações e comu- e) Cassação do exercício profissional, ad referendum
nidades envolvidas; do Conselho Federal de Psicologia.
b) Garantirá o caráter voluntário da participação dos
envolvidos, mediante consentimento livre e esclarecido, Art. 22° – As dúvidas na observância deste Código e os
salvo nas situações previstas em legislação específica e res- casos omissos serão resolvidos pelos Conselhos Regionais
peitando os princípios deste Código; de Psicologia, ad referendum do Conselho Federal de Psi-
c) Garantirá o anonimato das pessoas, grupos ou orga- cologia.
nizações, salvo interesse manifesto destes;
d) Garantirá o acesso das pessoas, grupos ou organi- Art. 23° – Competirá ao Conselho Federal de Psicologia
zações aos resultados das pesquisas ou estudos, após seu firmar jurisprudência quanto aos casos omissos e fazê-la
encerramento, sempre que assim o desejarem. incorporar a este Código.

Art. 17° – Caberá aos psicólogos docentes ou supervi- Art. 24° – O presente Código poderá ser alterado pelo
sores esclarecer, informar, orientar e exigir dos estudantes Conselho Federal de Psicologia, por iniciativa própria ou da
a observância dos princípios e normas contidas neste Có- categoria, ouvidos os Conselhos Regionais de Psicologia.
digo.
Art. 25°– Este Código entra em vigor em 27 de agosto
Art. 18° – O psicólogo não divulgará, ensinará, cede- de 2005.
rá, emprestará ou venderá a leigos instrumentos e técnicas
psicológicas que permitam ou facilitem o exercício ilegal QUESTÕES
da profissão.
01. (TJ/SP) Marin (1999), ao abordar o funcionamento
Art. 19° – O psicólogo, ao participar de atividade em institucional e seus efeitos sobre as pessoas, retoma a no-
ção de dupla determinação. A autora afirma que a fonte de
veículos de comunicação, zelará para que as informações
infelicidade e distorção psicológica se baseia nas estruturas
prestadas disseminem o conhecimento a respeito das atri-
(A) psicológicas dos pais e das crianças que precisa-
buições, da base científica e do papel social da profissão.
ram ser internadas em instituições.
(B) econômicas das famílias de baixa renda que têm
Art. 20° – O psicólogo, ao promover publicamente seus
pouca escolaridade e cultura.
serviços, por quaisquer meios, individual ou coletivamente:
(C) culturais das famílias de baixa renda que não pu-
deram acessar direitos básicos.
a) Informará o seu nome completo, o CRP e seu núme- (D) escolares da rede pública que se encontram de-
ro de registro; gradadas e defasadas.
b) Fará referência apenas a títulos ou qualificações pro- (E) alienadas das instituições e do sistema de produ-
fissionais que possua; ção e distribuição da riqueza.
c) Divulgará somente qualificações, atividades e recur-
sos relativos a técnicas e práticas que estejam reconhecidas 02. (TJ/SP) Marin (1999) analisou casos de crianças
ou regulamentadas pela profissão; institucionalizadas para as quais o processo institucional se
d) Não utilizará o preço do serviço como forma de pro- revelou como condição para um desenvolvimento satisfa-
paganda; tório. A autora afirma que, para tanto, é necessário que
e) Não fará previsão taxativa de resultados; (A) a instituição seja continente absoluto para as
f) Não fará autopromoção em detrimento de outros crianças.
profissionais; (B) a instituição não se camufle como uma família.
g) Não proporá atividades que sejam atribuições priva- (C) as famílias sejam negadas como modelo identifi-
tivas de outras categorias profissionais; catório. (D) as atendentes assumam o papel de mães para
h) Não fará divulgação sensacionalista das atividades as crianças.
profissionais. (E) as atendentes possuam uma personalidade ade-
quada para a continência.
Das Disposições Gerais
03. (TJ/SP) Marin (1999) ao estudar o lugar do outro
Art. 21° – As transgressões dos preceitos deste Códi- nas relações institucionais no âmbito da FEBEM discute o
go constituem infração disciplinar com a aplicação das se- desenvolvimento psicossocial da criança. Em seu texto, a
guintes penalidades, na forma dos dispositivos legais ou autora adota, principalmente, o conceito de
regimentais: (A) condicionamento.
(B) indivíduo.
a) Advertência; (C) privação.
b) Multa; (D) delinquência.
c) Censura pública; (E) identidade.

160
CONHECIMENTOS ESPECÍFICOS - PSICÓLOGO

04. (TJ/SP) De acordo com a Lei n.º 12.594, de 18 de (C) o aumento do número de atendimentos em servi-
janeiro de 2012, que institui o Sistema Nacional de Aten- ços públicos e privados.
dimento Socioeducativo (SINASE), as medidas socioeduca- (D) a universalização do acesso da população ao co-
tivas têm como um de seus objetivos a integração social nhecimento da ciência psicológica.
do adolescente e a garantia de seus direitos individuais e (E) a neutralidade de outros profissionais quanto às
sociais, por meio relações de poder.
(A) do isolamento do jovem em conflito com a lei até
os 18 anos. 08. (TJ/SP) De acordo com o § 2.º, do Art. n.º 8 do
(B) do cumprimento de seu plano individual de aten- Código de Ética Profissional do Psicólogo (Resolução CFP
n.º 010/05), o psicólogo responsabilizar-se-á pelos enca-
dimento.
minhamentos que se fizerem necessários para garantir a
(C) da privação da liberdade por tempo indetermina- proteção integral do atendido. Esse artigo diz respeito, es-
do para casos graves. pecificamente, ao atendimento não eventual de
(D) da privação da liberdade associada a estratégias (A) acidentados, trabalhadores e grupos em risco.
de punição efetivas. (B) pacientes hospitalares, idosos e usuários de dro-
(E) do tratamento psicológico do jovem com trans- gas.
tornos de personalidade. (C) crianças, adolescentes ou interdito.
(D) estrangeiros, indígenas e mulheres.
05. (TJ/SP) A Lei n.º 12.594, de 18 de janeiro de 2012, (E) aposentados, pensionistas e pessoas em situação
institui o Sistema Nacional de Atendimento Socioeducati- de rua.
vo (SINASE). Este documento legal regulamenta, na Seção
II do Capítulo IV, os programas de atendimento em meio 09. (TJ/SP) Spitz (2004) identificou uma série de pa-
aberto. De acordo com essa Lei, compete à direção do pro- drões prejudiciais de comportamento materno que se
mostraram ligados a distúrbios psicotóxicos da criança.
grama, entre outras atribuições,
Dentre eles, pode-se citar:
(A) contribuir para a qualificação e ação em rede dos (A) estabilidade de humor.
Sistemas de Atendimento Socioeducativo. (B) rejeição primária manifesta.
(B) criar e manter programas de atendimento para a (C) amor incondicional.
execução das medidas socioeducativas em meio aberto. (D) ausência de valores.
(C) prestar assessoria técnica e suplementação finan- (E) inteligência reduzida.
ceira aos Municípios para a oferta regular de programas de
meio aberto. 10. (TJ/SP) De acordo com o Conselho Federal de Psi-
(D) o detalhamento das atribuições e responsabilida- cologia, os documentos produzidos por psicólogos que
des do dirigente, de seus prepostos, dos membros da equi- atuam na área da justiça devem manter o rigor técnico e
pe técnica e dos demais educadores. ético exigido na Resolução CFP n.º 007/2003, que institui
(E) selecionar e credenciar orientadores, designando- (A) as normas éticas para atuação do psicólogo no
-os, caso a caso, para acompanhar e avaliar o cumprimento atendimento de adolescentes em conflito com a lei.
(B) os parâmetros psicométricos para a validação de
da medida.
instrumentos psicométricos para a realidade brasileira.
(C) o guia técnico para a orientação jurídica de psicó-
06. (TJ/SP) De acordo com o § 2.º, do Capítulo I da Lei logos que atuam como técnicos junto ao poder judiciário.
n. º 12.594, de 18 de janeiro de 2012, um dos objetivos da (D) a tabela de classificação psicométrica de acordo
medida socioeducativa é a desaprovação da conduta infra- com índices estatísticos internacionais.
cional. Segundo essa premissa, quando um jovem comete (E) o Manual de Elaboração de Documentos Decor-
um ato infracional, ele deve ser rentes de Avaliações Psicológicas.
(A) convencido sobre a necessidade de ser obediente.
(B) estimulado a se arrepender do que fez. 11. (TJ/SP) Bleger (2011), ao se referir à entrevista psi-
(C) punido severamente em função de seu delito. cológica, afirma a importância dos cuidados com a inter-
(D) orientado a reparar o dano que cometeu. pretação. Segundo ele, quanto maior for a compulsão para
(E) condenado a pagar pelo seu crime. interpretar, tanto mais será necessário (A) ser diretivo.
(B) calar-se.
(C) interpretar.
07. (TJ/SP) O Código de Ética Profissional do Psicólo-
(D) pontuar.
go (Resolução CFP n. º 010/05) consiste em um documento (E) assinalar.
que expressa uma concepção de homem e de sociedade.
Dentre os princípios fundamentais enunciados nesse docu- 12. (TJ/SP) De acordo com o Art. 145 do Código do
mento, pode-se destacar que o psicólogo deverá contribuir Processo Civil, quando a prova do fato depender de co-
para nhecimento técnico ou científico, o juiz será assistido por
(A) o desenvolvimento de serviços de saúde devida- perito, segundo o disposto no Art. 421. Nas localidades
mente remunerados. onde não houver profissionais qualificados que preencham
(B) a conservação dos limites técnicos da Psicologia os requisitos descritos nos parágrafos 1 e 2, do Art. 145, a
na Saúde. indicação dos peritos será

161
CONHECIMENTOS ESPECÍFICOS - PSICÓLOGO

(A) dispensada definitivamente. (A) garantir a primazia da responsabilidade científica


(B) dirigida a profissionais de outras áreas. na condução da política.
(C) de livre escolha do juiz. (B) a supremacia das exigências de rentabilidade eco-
(D) realizada livremente pelas partes. nômica sobre as necessidades sociais.
(E) de livre escolha do Órgão de Classe. (C) tornar o destinatário da ação assistencial alcançá-
vel pelas demais políticas públicas.
13. (TJ/SP) Bowlby (2006) aborda a rigidez de candi- (D) incluir prestadores de serviço dos setores público
datos a pais adotivos em relação às características da crian- e privado para garantir assistência.
ça. Estes fazem exigências quanto a detalhes relacionados
(E) ampliar os auxílios àqueles em precariedade e
à aparência, ao sexo e à inteligência.
O autor entende que essas atitudes estão relacionadas com maior necessidade econômica.
(A) a experiências anteriores de adoção.
(B) à ausência de tempo para tomar a decisão. 18. (TJ/SP) O Sistema Único de Assistência Social é
(C) à influência de terceiros no processo de escolha. considerado um importante avanço para a garantia de di-
(D) à falta de informação sobre as formas de adoção. reitos da população. Dentre as diretrizes para a organiza-
(E) a conflitos emocionais dos próprios candidatos. ção da assistência social, está a participação da população
por meio de
14. (TJ/SP) O conhecimento científico tem se demons- (A) organizações não governamentais (ONGs), na
trado de grande importância para auxiliar nas decisões ju- oferta de serviços de assistência social em todos os níveis.
diciais. O Art. 420 do Código de Processo Civil determina (B) associações filantrópicas, na formulação e oferta
que as provas periciais consistem em de serviços gratuitos a todos aqueles que necessitarem.
(A) exame, vistoria ou avaliação. (C) representantes nomeados pelo município, na ofer-
(B) análises, interpretações ou indicações.
ta de serviços de assistência social em todos os níveis.
(C) prescrições, documentos ou declarações.
(D) representantes indicados pelo poder público e de
(D) opinião técnica, depoimentos ou documentos.
(E) opiniões profissionais, interpretações ou docu- acordo com critérios técnicos para fiscalizar as políticas.
mentos. (E) organizações representativas, na formulação das
políticas e no controle das ações em todos os níveis.
15. (TJ/SP) No que diz respeito ao pagamento de
prestação alimentícia, de acordo com o § 1.º do Art. 733 19. (TJ/SP) proteção social básica consiste em um
do Código de Processo Civil, se o devedor não pagar, nem conjunto de serviços, programas, projetos e benefícios da
se escusar, o juiz decretar-lhe-á a prisão pelo prazo de (A) assistência social que visa prevenir situações de vulnerabi-
1(um) a 2 (dois) anos. lidade e risco social por meio do
(B) 2 (dois) a 3 (três) anos. (A) desenvolvimento de potencialidades e aquisições
(C) 1 (um) ano e 3 (três) meses. e do fortalecimento de vínculos familiares e comunitários.
(D) 1 (um) a 3 (três) meses. (B) desenvolvimento social e econômico, incentivan-
(E) 3 (três) a 5 (cinco) meses. do iniciativas geradoras de postos de trabalho.
(C) incentivo de uma conduta cidadã que garanta a
16. (TJ/SP) Em relação a outras medidas provisionais
descritas na Seção XV, o Art. 888 do Código de Processo responsabilidade de todos para o desenvolvimento social.
Civil determina que o juiz poderá ordenar ou autorizar, na (D) desenvolvimento econômico e pela distribuição
pendência da ação principal ou antes de sua propositura, o de recursos públicos para famílias de baixa renda.
depósito de menores ou incapazes (E) desenvolvimento sociocultural de grupos menos
(A) se o devedor não pagar a prestação alimentícia, escolarizados afim de capacitá-los ao trabalho qualificado.
nem se escusar ao juiz o débito de prestações vencidas.
(B) quando os cônjuges apresentarem comportamen- 20. (TJ/SP) O capítulo XI do Código Civil dispõe sobre
to conjugal pouco convencional e predominado por confli- a proteção da pessoa dos filhos e, os parágrafos 2.º e 3.º,
tos. do Art. 1583, indicam os fatores e circunstâncias que envol-
(C) cuja guarda e a educação dos filhos não foram vem a guarda unilateral. Esses fatores são:
concedidas aos avós e/ou familiares próximos. (A) afeto nas relações com o genitor e com o grupo
(D) em casos de interdição ou demolição de casas familiar, saúde, segurança e educação.
para preservar a saúde, a segurança ou o bem-estar geral.
(B) segurança afetiva na relação com o genitor, segu-
(E) castigados imoderadamente por seus pais, tutores
rança financeira, alimentação e educação.
ou curadores, ou por eles induzidos à prática de atos con-
trários à lei ou à moral. (C) orientação adequada, afeto familiar e comunitário,
segurança financeira, habitação e educação.
17. (TJ/SP) No ano de 2011, foi promulgada a lei que (D) autoridade na relação com o genitor e comunida-
regulamenta o Sistema Único de Assistência Social (SUAS). de, saúde, habitação e alimentação e educação.
Dentre seus princípios, está a universalização dos direitos (E) afeto para o bom desenvolvimento da personali-
sociais, que tem por objetivo dade, segurança financeira, educação e saúde.

162
CONHECIMENTOS ESPECÍFICOS - PSICÓLOGO

21. (TJ/SP) Bowlby (2006) comenta que o paradoxo 25. (TJ/SP) Segundo Costa (2012), o relatório psicos-
central do trabalho com crianças que sofrem privação da social acerca do jovem que cometeu ato infracional deve
mãe é que existem milhar es de lares sem filhos. Segundo servir a dois senhores, sendo peça de subsídio ao juiz e en-
ele, é provado que, para a saúde mental do bebê, é impor- riquecendo o modus operandi do judiciário. Isso ocorrerá
tante que ele seja adotado à medida que
(A) após o desenvolvimento da personalidade. (A) se programe uma nova estratégia de diálogo e ne-
(B) logo após o nascimento. gociação.
(C) por pais jovens e sem filhos. (B) as noções de ética e cidadania se encaminharem
(D) por pessoas da mesma classe social. para a garantia de direitos.
(E) posteriormente à estadia em lar substituto. (C) seja desconsiderado o histórico de infrator e valo-
rizada a educação.
22. (TJ/SP) De acordo com o § 1.º do Art.28 da Lei (D) aconteça o esclarecimento dos procedimentos de
12.010 de 2009, do ECA (Estatuto da Criança e do Adoles- avaliação psicológica.
cente) atualizado, sempre que possível, a criança ou o ado- (E) seja incluída no âmbito jurídico a compreensão do
lescente será previamente ouvido por equipe interprofis- sujeito inserido no meio social.
sional, respeitado seu estágio de desenvolvimento e grau
de compreensão sobre as implicações da medida, e terá
26. (TJ/SP) Ocampo e colaboradores (2009) apresen-
sua opinião devidamente considerada. Segundo disposto
tam as etapas do processo psicodiagnóstico. Nesse proces-
no § 2.º, tratando-se de maior de 12 anos de idade, será
necessário seu consentimento so, é muito importante que seja realizado o enquadramen-
(A) colhido por familiar. to, que tem por finalidade
(B) mediante presença de familiar. (A) orientar os pais da criança sobre a importância do
(C) colhido em audiência. sigilo no processo.
(D) mediante presença do psicólogo. (B) planejar as atividades a serem desenvolvidas du-
(E) colhido na instituição de abrigamento. rante o psicodiagnóstico.
(C) manter constantes certas variáveis que intervêm
23. (TJ/SP) O aperfeiçoamento do Estatuto da Criança no processo.
e do Adolescente (ECA), atualizado pela Lei n.º 12.010 de (D) reconhecer as demandas do sujeito em processo
2009, trata dos casos de criança ou adolescente indígena de avaliação.
ou proveniente de comunidade remanescente de quilom- (E) garantir condições mínimas de trabalho para o
bo. De acordo com o § 6.º do Art. 28, nessas situações, é psicólogo.
obrigatória a intervenção e oitiva perante a equipe inter-
profissional ou multidisciplinar que irá acompanhar o caso. 27. (TJ/SP) Ocampo e colaboradores (2009) destacam
Procedimento este executado por a importância da entrevista inicial com os pais e da avalia-
(A) antropólogos e representantes do órgão federal ção de sua capacidade de elaborar a situação diagnóstica.
responsável pela política indigenista. Para os autores, a presença de ambos os pais na entrevista
(B) pedagogos e representantes do conselho tutelar diagnóstica é imprescindível, pois
local. (A) os pais precisam ser orientados sobre os possíveis
(C) representantes do órgão federal responsável pela resultados.
política indigenista e a comunidade. (B) eles precisam ser entrevistados primeiro para que
(D) psicólogos e representantes das comunidades in- possam acolher a criança.
dígenas envolvidas. (C) a criança não tem autonomia para participar do
(E) psicólogos e representantes do órgão federal res- enquadre.
ponsável pela política indigenista.
(D) consideram a criança como emergente de um gru-
po familiar.
24. (TJ/SP) Costa (2012) afirma a importância do re-
(E) existe determinação do Conselho Federal de Psi-
latório psicossocial, não apenas para auxiliar o juiz, mas
também porque deve devolver a humanidade aos sujeitos cologia.
dependentes das decisões judiciais. Dessa forma, a Justiça
se constituirá na criação de condições para 28. (TJ/SP) Segundo Ocampo e colaboradores (2009),
(A) o cumprimento de medida socioeducativa de in- a hora de jogo diagnóstica consiste em uma experiência
ternação. nova para todos os envolvidos, entrevistador e entrevista-
(B) o arrependimento do jovem e a consolidação de dos. Esse momento implica, além do interjogo das séries
uma postura cidadã. complementares, o estabelecimento
(C) a reestruturação da personalidade desviante do (A) de uma postura interpretativa.
adolescente. (B) de atividades já concatenadas simbolicamente.
(D) que o adolescente recupere sua perspectiva de um (C) da tarefa dentro do referencial operativo.
ser em formação. (D) dos papéis socialmente aceitos.
(E) que o jovem seja punido de forma educativa e não (E) de um vínculo transferencial.
incorra em ato infracional.

163
CONHECIMENTOS ESPECÍFICOS - PSICÓLOGO

29. (TJ/SP) Dolto (2011) discute a problemática do di- (A) na entrevista psicológica com os pais.
vórcio sobre a ótica psicanalítica e enfoca as dissociações (B) na observação das audiências com o juiz.
vividas pela criança em decorrência dessa ruptura da estru- (C) por meio de testes psicológicos.
tura familiar. Para ela, quando os pais não estão mais pre- (D) pela observação clínica.
sentes e o espaço também não é o mesmo, a criança (E) nos grupos operativos com a família.
(A) faz a mediação da separação com maior facilidade.
(B) não confiará no analista e nos pais. 34. (TJ/SP) Shine (2003) discute a posição do psicólo-
(C) não mais se reconhece, nem mesmo em seu corpo. go no jogo de forças envolvido em processos de avaliação
(D) entende que suas vontades são desconhecidas por para determinação da guarda. O autor retoma as indica-
todos. ções de Bleger e aponta que também é preciso
(E) elabora a separação, pois não compreende o que (A) afirmar nossa identidade profissional, atendendo
ocorreu. às urgências do trabalho.
(B) agir como um expert, apoiado nos parâmetros das
30. (TJ/SP) Uma das temáticas que permeiam os escri- ciências da natureza.
tos de Dolto (2011), acerca dos processos de separação dos (C) atuar, como analista institucional, a despeito da
pais e suas possíveis consequências para o psiquismo das natureza jurídica do trabalho.
crianças, é a comunicação da separação para elas. Segundo (D) resistir à pressão da urgência e à onipotência e
ela, não falar sobre o assunto com o filho significa, de certa não tomar para si tarefas alheias.
forma, (E) entender que não se pode ocupar o lugar da resis-
(A) instruí-lo pelo não dito. tência às demandas.
(B) uma traição.
(C) não instruí-lo, limitando sua elaboração. 35. (TJ/SP) Dentre os procedimentos técnicos envol-
(D) negá-lo como sujeito. vidos no trabalho do psicólogo judiciário, Shine (2003)
(E) ignorá-lo em seu sofrimento e demandas. destaca a importância da leitura dos autos. Para ele, esse
procedimento consiste em
31. (TJ/SP) Winnicott (2005) enfatiza a importância (A) incorporar elementos jurídicos na entrevista psico-
dos métodos psicológicos na investigação de casos judi- lógica.
ciais, mas alerta que um dos maiores obstáculos para tanto (B) procurar subsídios para traçar uma estratégia de
é a atitude sentimentalista em relação ao crime. Para ele,
avaliação.
no sentimentalismo, (A) ocorre a identificação com o jovem
(C) diminuir o tempo para a realização da avaliação.
delinquente.
(D) garantir um enfoque multidisciplinar ao trabalho
(B) ocorrem sentimentos inconscientes de vingança
do juiz.
pública.
(E) evitar discrepâncias entre o trabalho de avaliação
(C) encobre-se o que há de comum na justiça.
e a perspectiva legal.
(D) a culpa opera obscurecendo a gravidade do crime.
36. (TJ/SP) Bleger (2011), de forma crítica, avalia que
(E) existe uma tendência de proteger o criminoso.
psicólogos inexperientes podem se valer dos testes em si-
32. (TJ/SP) Winnicott (2005) descreve que a tendência tuações de insegurança pessoal. Segundo o autor, o isola-
antissocial inclui roubo, mentira, incontinência e, de modo mento profissional contribui muito para a estereotipia, pois
geral, uma conduta desordenada. Quanto à terapêutica (A) favorece a manutenção da clivagem.
dessa tendência, ele afirma que (B) encobre as dificuldades com onipotência.
(A) ela é restrita ao campo psiquiátrico e medicamen- (C) permite a análise da própria personalidade.
toso. (D) evita a alienação no trabalho.
(B) o ambiente deve dar nova oportunidade à ligação (E) não permite a troca de informações.
egoica.
(C) o peso da transferência não deve se desenvolver 37. (TJ/SP) Segundo Bleger (2011), a entrevista se
fora da análise. constitui como um campo que depende principalmente
(D) a indicação é a psicanálise, pois ela controla as dis- das variáveis da personalidade de quem é entrevistado.
torções do ambiente. Portanto, para permitir maior engajamento da personali-
(E) é a psicanálise que poderá suprir a privação egoica. dade do entrevistado, aquilo que o entrevistador oferece
deve ser
33. (TJ/SP) Para Shine (2003), nos processos judiciais (A) um conjunto de variáveis objetivas.
pela guarda de crianças, é fundamental a obtenção de da- (B) livre de seus aspectos de personalidade.
dos psicológicos relativos a capacidades específicas, limita- (C) suficientemente ambíguo.
ções e dinâmica dos envolvidos. Nesse sentido, o material (D) livre de ambiguidades.
concreto, ao qual vários psicólogos podem se reportar, é (E) direto e objetivo.
produzido

164
CONHECIMENTOS ESPECÍFICOS - PSICÓLOGO

38. (TJ/SP) Dentre os métodos de avaliação da perso-


nalidade, destacam- -se as técnicas projetivas. Ocampo e
colaboradores (2009) afirma que a distinção entre percep-
ção cognitiva e apercepção de uma situação é, dinamica-
mente, uma
(A) falsa dicotomia.
(B) condição do trabalho psíquico.
(C) indicação de equilíbrio.
(D) situação psicopatológica.
(E) variável recorrente.

39. (TJ/SP) A violência sexual consiste em um proble-


ma que, em muitos casos, fica em segredo. É importante
que os profissionais saibam reconhecer indicadores desse
tipo de violência na conduta da criança e do adolescente.
Segundo Ferrari (2002), esses indicadores podem ser:
(A) regressão a comportamentos infantis e comporta-
mento agressivo.
(B) abuso de álcool e outras drogas.
(C) defesa da disciplina corporal severa e timidez.
(D) exigência de perfeição e desempenho superior às
possibilidades da criança/adolescente.
(E) postura insinuante, extremamente zelosa e prote-
tora.

40. (TJ/SP) Quanto ao atendimento psicológico a ca-


sos de violência familiar, Ferrari (2002) aborda estratégias
de intervenção psicoterápica e psicoprofiláticas. Faz parte
do atendimento o diagnóstico inicial ou avaliação sumária,
e nessa fase estabelece-se o (A) abrigamento da criança.
(B) responsável pela criança.
(C) procedimento disciplinar.
(D) grau de risco.
(E) atendimento psicossocial.

RESPOSTAS

01 – E/ 02 – B/ 03 – E/ 04 – B/ 05 – E/ 06 – D/ 07 – D/
08 – C/ 09 – B/ 10 – E/11 – B/ 12 – C/ 13 – E/ 14 - A / 15
– D/ 16 – E/ 17 – C/ 18 – E/ 19 – A/ 20 – A/21 – B/ 22 – C/
23 – A/ 24 – D/ 25 – E/ 26 – C/ 27 – D/ 28 – E/ 29 – C/ 30
- A /31 – B/ 32 - B 33- C/ 34 – D/ 35 – B/ 36 – B/ 37 – C/
38 - A /39 – A/ 40 – D/

165
EXERCICÍOS COMPLEMENTARES

EXERCICÍOS COMPLEMENTARES o Executivo e o Judiciário”. Somente resta a alternativa “D”,


que apesar de realmente trazer um objetivo da República
001 - (PC/SC - Agente de Polícia - ACAFE/2014) A Federativa brasileira – previsto no artigo 3º, IV, não tem a
República Federativa do Brasil, formada pela união indisso- ver com os princípios fundamentais, mas sim com os ob-
lúvel dos Estados e Municípios e do Distrito Federal, cons- jetivos.
titui-se em Estado Democrático de Direito (art. 1º da CF).
Com base no enunciado acima é correto afirmar, ex- 003 - (DPE/DF - Analista - Assistência Judiciária -
ceto: FGV/2014) Sobre os Princípios Fundamentais da República
A) são objetivos fundamentais da república federativa Federativa do Brasil, à luz do texto constitucional de 1988,
do Brasil erradicar a pobreza e a marginalização e reduzir é INCORRETO afirmar que:
as desigualdades sociais e regionais. A) a República Federativa do Brasil tem como funda-
B) a soberania, a cidadania e o pluralismo político não mentos: a soberania, a cidadania, a dignidade da pessoa
são fundamentos da república federativa do brasil. humana, os valores sociais do trabalho e da livre iniciativa e
C) ninguém será obrigado a fazer ou deixar de fazer o pluralismo político.
alguma coisa senão em virtude de lei. B) a República Federativa do Brasil tem como objetivos
D) é livre a manifestação de pensamento, sendo veda- fundamentais: construir uma sociedade livre, justa e solidá-
do o anonimato. ria; garantir o desenvolvimento nacional, erradicar a pobre-
E) construir uma sociedade livre, justa e solidária é um za e a marginalização e reduzir as desigualdades sociais e
dos objetivos fundamentais da república federativa do Bra- regionais; promover o bem de todos, sem preconceitos de
sil. origem, raça, sexo, cor, idade e quaisquer outras formas de
R: B. Todas as alternativas descrevem características, discriminação.
atributos do Estado Democrático de Direito que é a Re- C) todo o poder emana do povo, que o exerce unica-
pública Federativa brasileira, notadamente: erradicação da mente por meio de representantes eleitos.
pobreza e diminuição de desigualdades (artigo 3º, III, CF); D) entre outros, são princípios adotados pela Repú-
soberania, cidadania e pluralismo político (artigo 1º, I, II e V, blica Federativa do Brasil nas suas relações internacionais,
CF); princípio da legalidade (artigo 5º, II, CF); liberdade de os seguintes: a independência nacional, a prevalência dos
expressão (artigo 5º, IV, CF); construção de sociedade justa, direitos humanos e o repúdio ao terrorismo e ao racismo.
livre e solidária (artigo 3º, I, CF). Sendo assim, incorreta a E) a autodeterminação dos povos, a não intervenção
afirmação de que soberania, cidadania e pluralismo políti- e a defesa da paz são princípios regedores das relações
co não são fundamentos da República Federativa do Brasil, internacionais da República Federativa do Brasil.
pois estão como tais enumerados no artigo 1º, CF, além de R: C. A democracia brasileira adota a modalidade semi-
decorrerem da própria estrutura de um Estado Democráti- direta, porque possibilita a participação popular direta no
co de Direito. poder por intermédio de processos como o plebiscito, o re-
ferendo e a iniciativa popular. Como são hipóteses restritas,
002 - (DPE/GO - Defensor Público - UFG/2014) A pode-se afirmar que a democracia indireta é predominan-
propósito dos princípios fundamentais da República Fede- temente adotada no Brasil, por meio do sufrágio universal
rativa do Brasil, reconhece-se que: e do voto direto e secreto com igual valor para todos. Con-
A) o pluralismo político está inserido entre seus obje- tudo, não é a única maneira de se exercer o poder (artigo
tivos. 14, CF e artigo 1º, parágrafo único, CF).
B) a livre iniciativa é um de seus fundamentos e se con-
trapõe ao valor social do trabalho. 004 - (TJ/MG - Juiz - FUNDEP/2014) Assinale a alter-
C) a dignidade é também do nascituro, o que desauto- nativa que descreve COMPLETAMENTE os objetivos funda-
riza, portanto, a prática da interrupção da gravidez quando mentais da República Federativa do Brasil.
decorrente de estupro. A) Erradicar a pobreza e o analfabetismo.
D) a promoção do bem de todos, sem preconceito de B) Garantir o desenvolvimento pessoal dos cidadãos e
origem, raça, sexo, cor, idade e qualquer outra forma de construir a riqueza de sua gente.
discriminação, é um de seus objetivos. C) Construir uma sociedade livre, justa e solidária, ga-
E) o legislativo, o executivo e o judiciário, dependentes rantindo o desenvolvimento nacional com erradicação da
e harmônicos entre si, são poderes da união. pobreza e da marginalização, reduzindo as desigualdades
R: D. O artigo 1º, CF traz os princípios fundamentais sociais e regionais, promovendo o bem de todos, sem pre-
(fundamentos) da República Federativa do Brasil: “I - a so- conceitos de origem, raça, cor, idade e quaisquer outras for-
berania; II - a cidadania; III - a dignidade da pessoa huma- mas de discriminação.
na; IV - os valores sociais do trabalho e da livre iniciativa; D) Promover a defesa nacional contra atos de Estados
V - o pluralismo político”. O princípio de “A” se encontra no estrangeiros que intervierem nos assuntos internos da na-
inciso V; o de “B” no inciso IV; o de “C” no inciso III, pois ção.
viola a dignidade humana da mãe forçá-la a dar luz à um R: C. O constituinte trabalha no artigo 3º da Consti-
filho que resulte de estupro; o de “E” decorre dos incisos I e tuição Federal com os objetivos da República Federativa
II e é previsão do artigo 2º, que dispõe que “são Poderes da do Brasil, nos seguintes termos: “Art. 3º Constituem obje-
União, independentes e harmônicos entre si, o Legislativo, tivos fundamentais da República Federativa do Brasil: I -

166
EXERCICÍOS COMPLEMENTARES

construir uma sociedade livre, justa e solidária; II - garantir rindo nos órgãos de soberania e nem sequer nos demais
o desenvolvimento nacional; III - erradicar a pobreza e a órgãos do Estado, abrangendo de igual forma os estados
marginalização e reduzir as desigualdades sociais e regio- federados e os municípios;
nais; IV - promover o bem de todos, sem preconceitos de E) Constitui-se em exemplo de dispositivo de natureza
origem, raça, sexo, cor, idade e quaisquer outras formas de constitucional que trata diretamente da dignidade da pes-
discriminação”. Somente a alternativa “C” resume todos os soa humana o previsto no art. 79 do ADCT, que instituiu o
objetivos elencados neste dispositivo. Fundo de Combate e Erradicação da Pobreza, com o obje-
tivo de viabilizar a todos os brasileiros acesso a níveis dig-
005 - (TCE/PI - Assessor Jurídico - FCC/2014) Pelo nos de subsistência e com recursos para serem aplicados
princípio da justeza ou da conformidade funcional da em ações suplementares de nutrição, habitação, educação,
Constituição Federal, saúde, reforço de renda familiar e outros programas de re-
A) as normas constitucionais devem ser interpretadas levante interesse social voltados para melhoria da qualida-
no sentido de terem a mais ampla efetividade social, re- de de vida.
conhecendo a maior eficácia possível aos direitos funda- R: B. Em se tratando de norma constitucional, possui
mentais. força normativa e serve de parâmetro ao intérprete na apli-
B) partindo da ideia de unidade da constituição, os cação da lei e mesmo de diretriz para o controle de cons-
bens jurídicos constitucionalizados deverão coexistir de titucionalidade, pouco importando se a densidade norma-
forma harmônica na hipótese de eventual conflito ou con- tiva é baixa, ou seja, se são princípios genéricos e de difícil
corrência entre esses bens e princípios, por inexistir hierar- compreensão teórica na prática. Logo, a aplicação não é
quia entre eles. indireta, mas sim direta.
C) o intérprete máximo da constituição, ao concretizar
007 - (MPE/MA - MPE/2014) Atualmente a aplicação
a norma constitucional, será responsável por estabelecer
do preceito da dignidade da pessoa humana, conforme
sua força normativa, não podendo alterar a repartição de
previsão do inciso III, do art. 1º, da CF de 1988, tem se re-
funções constitucionalmente estabelecidas pelo constituinte velado relativamente constante nas decisões provenientes
originário. do STF e STJ, assim como dos demais órgãos judiciários.
D) as normas constitucionais devem ser interpretadas Sobre esse fundamento do Estado Democrático de Direito,
em sua globalidade, afastando-se as aparentes antinomias é correto afirmar que:
legais. A) Embora a dignidade da pessoa humana conste
E) na resolução dos problemas jurídico-constitucionais do rol dos direitos e garantias fundamentais expressos
deve-se dar primazia aos critérios que favoreçam a inte- na Magna Carta, foi também consagrada como princípio
gração política e social, e o reforço da unidade política do e valor fundamental e, como tal, deve servir de norte ao
estado. intérprete, ao qual incumbe a missão de assegurar-lhe a
R: C. O princípio da conformidade funcional ou da jus- necessária força normativa;
teza é um dos princípios interpretativos das normas cons- B) A qualificação normativa da dignidade da pessoa
titucionais e prescreve que ao intérprete da Constituição, humana como princípio fundamental traduz a certeza de
o Supremo Tribunal Federal, é defeso, proibido, modificar que o art. 1º, III, da Constituição contém apenas uma de-
a repartição de funções fixadas pela própria Constituição claração de conteúdo ético, na medida em que representa
Federal. uma norma jurídico-positiva não dotada, em sua plenitude,
de status constitucional formal e material;
006 - (MPE/MA - MPE/2014) Sobre os fundamentos C) Pelo menos no que concerne à sua dimensão prin-
da República Federativa do Brasil é incorreto afirmar que: cipiológica, a dignidade da pessoa humana atua como uma
A) Podemos falar na existência de soberania popu- espécie de mandado de otimização, ordenando a prote-
lar quando a soberania reside no povo (fonte do poder) e ção e promoção da dignidade da pessoa, a ser realizada
quando o poder pertence ao povo (titularidade do poder); na maior medida possível, ainda que desconsiderando as
B) Todos os preceitos que identificam o regime ado- possibilidades fáticas e jurídicas existentes;
tado como democrático são bens reveladores da ideia de D) A dignidade humana serve como elemento limita-
dor dos direitos fundamentais, pois age como justificativa
Direito ou da ordem de valores acolhida na constituição,
para a imposição de restrição a estes, podendo também
refletindo-se, contudo, apenas indiretamente nas normas
atuar como limite aos limites desses mesmos direitos, ao
atributivas de direitos e, portanto, não se impondo direta-
exercer restrições à atividade limitadora no âmbito dos
mente ao intérprete e aplicador das normas constitucionais direitos fundamentais, com o objetivo de coibir eventual
e legais; abuso que possa levar ao seu esvaziamento ou supressão;
C) A menção à democracia realizada no caput do art. E) Na sua atuação como limite à atuação estatal e da
1º da CF/88 incorpora uma regra prescritiva e não uma re- comunidade em geral, a dignidade implica apenas que a
gra negativa ou proibitiva, na medida em que obriga a que pessoa não pode ser reduzida à condição de mero objeto
na expressão e na organização políticas se observem as da ação própria e de terceiros, não provocando diretamen-
regras inerentes a uma ordem constitucional democrática; te a adoção de medidas contra a atuação estatal ou de ter-
D) A separação e a interdependência dos poderes, ceiros que a violem ou ameacem o nascimento de direitos
conforme previsto no art. 2º da CF/88, constitui-se em fundamentais negativos.
princípio coessencial ao Estado de Direito, não se exau-

167
EXERCICÍOS COMPLEMENTARES

R: D. Dignidade da pessoa humana é o principal valor A) defesa da paz, soberania nacional, não-intervenção
do ordenamento ético e, por consequência, jurídico que e repúdio a todas as formas de tratamento desumano ou
pretende colocar a pessoa humana como um sujeito ple- degradante.
no de direitos e obrigações na ordem internacional e na- B) autodeterminação dos povos, cooperação entre os
cional, cujo desrespeito acarreta a própria exclusão de sua povos para o progresso da humanidade e promoção do
personalidade. Logo, estabelece-se na noção de dignidade bem-estar e da justiça social.
da pessoa humana um aspecto intrínseco de correlação en- C) defesa da paz, solução pacífica dos conflitos, não
tre direitos e deveres. Os próprios direitos afirmados como -intervenção e repúdio ao terrorismo e ao racismo.
inerentes à dignidade da pessoa humana são obstáculo D) soberania nacional, proteção do meio ambiente
para o exercício de cada qual deles, pois nenhum direito é ecologicamente equilibrado, não intervenção e solução
absoluto, afinal, se todo ser humano é digno, todo ser hu-
pacífica dos conflitos.
mano deve poder exercê-los, sendo que o direito do outro
E) cooperação entre os povos para o progresso da
é o limite do exercício de cada direito pessoal.
humanidade, proteção do meio ambiente ecologicamente
equilibrado, promoção do bem-estar e da justiça social.
008 - (TJ/MT - Juiz - FMP-RS/2014) Assinale a alter-
R: C. O último artigo do título I trabalha com os princí-
nativa correta a respeito dos partidos políticos.
A) É vedado a eles o recebimento de recursos financei- pios que regem as relações internacionais da República bra-
ros por parte de empresas transnacionais. sileira: “Art. 4º A República Federativa do Brasil rege-se nas
B) É assegurado a eles o acesso gratuito à propaganda suas relações internacionais pelos seguintes princípios: I - in-
no rádio e na televisão, exceto aqueles que não possuam dependência nacional; II - prevalência dos direitos humanos;
representação no Congresso Nacional. III - autodeterminação dos povos; IV - não-intervenção; V -
C) Os partidos devem, obrigatoriamente, ter caráter igualdade entre os Estados; VI - defesa da paz; VII - solução
nacional. pacífica dos conflitos; VIII - repúdio ao terrorismo e ao
D) Os partidos devem, após cada campanha, apresen- racismo; IX - cooperação entre os povos para o progresso
tar ao Congresso Nacional a sua prestação de contas para da humanidade; X - concessão de asilo político. Parágrafo
aprovação. único. A República Federativa do Brasil buscará a integração
E) Em razão de sua importante função institucional, os econômica, política, social e cultural dos povos da América
partidos políticos possuem natureza jurídica de direito pú- Latina, visando à formação de uma comunidade latino-ame-
blico. ricana de nações”. Os grifos apontam que a alternativa “C”
R: C. O artigo 17 da Constituição Federal regulamen- descreveu corretamente 4 dos princípios que regem as rela-
ta os partidos políticos e coloca o caráter nacional como ções internacionais da República brasileira.
preceito que deva necessariamente se observado: “É livre
a criação, fusão, incorporação e extinção de partidos po- 010 - (PC/SP - Oficial Administrativo - VU-
líticos, resguardados a soberania nacional, o regime de- NESP/2014) A República Federativa do Brasil, formada
mocrático, o pluripartidarismo, os direitos fundamentais pela união indissolúvel dos Estados e Municípios e do Dis-
da pessoa humana e observados os seguintes preceitos: trito Federal, constitui-se em um Estado
I - caráter nacional; II - proibição de recebimento de re- A) democrático de Direito.
cursos financeiros de entidade ou governo estrangeiros ou B) burocrático.
de subordinação a estes; III - prestação de contas à Justiça C) o Congresso Nacional, o Senado e a Câmara dos
Eleitoral; IV - funcionamento parlamentar de acordo com a
Deputados.
lei. § 1º É assegurada aos partidos políticos autonomia para
D) socialista progressista.
definir sua estrutura interna, organização e funcionamento
E) humanitário social.
e para adotar os critérios de escolha e o regime de suas
R: A. Consta no caput do artigo 1º, CF: “a República
coligações eleitorais, sem obrigatoriedade de vinculação
Federativa do Brasil, formada pela união indissolúvel dos
entre as candidaturas em âmbito nacional, estadual, distri-
tal ou municipal, devendo seus estatutos estabelecer nor- Estados e Municípios e do Distrito Federal, constitui-se em
mas de disciplina e fidelidade partidária. § 2º Os partidos Estado Democrático de Direito [...]”.
políticos, após adquirirem personalidade jurídica, na forma
da lei civil, registrarão seus estatutos no Tribunal Superior (ANAC – TÉCNICO EM REGULAMENTAÇÃO DE
Eleitoral. § 3º Os partidos políticos têm direito a recursos AVIAÇÃO CIVIL - ESAF/2015 - adaptada) Leia o depoi-
do fundo partidário e acesso gratuito ao rádio e à televisão, mento a seguir para responder às questões
na forma da lei. § 4º É vedada a utilização pelos partidos
políticos de organização paramilitar”. Há quase dois anos fui empossado técnico administrati-
vo na ANAC de São Paulo e estou muito satisfeito de traba-
009 - (Prefeitura de Recife/PE - Procurador- lhar lá. Nesse tempo já fui nomeado para outros dois cargos
FCC/2014) Entre os princípios que regem, segundo a na administração pública, porém preferi ficar onde estou por
Constituição Federal, a República Federativa do Brasil nas diversos motivos, profissionais e pessoais. Sinceramente, sou
suas relações internacionais, encontram-se os seguintes: partidário do “não se mexe em time que está ganhando”.

168
EXERCICÍOS COMPLEMENTARES

Trabalho na área administrativa junto com outros téc- (ANAC – TÉCNICO EM REGULAMENTAÇÃO DE
nicos e analistas, além de ser gestor substituto do setor de AVIAÇÃO CIVIL - ESAF/2015 - adaptada) Leia o texto a
transportes da ANAC/SP. Tenho de analisar documentação, seguir para responder às questões
preparar processos solicitando pagamentos mensais para
empresas por serviços prestados, verificar se os termos do Se você é um passageiro frequente, certamente já pas-
contrato estão sendo cumpridos, resolver alguns “pepinos” sou por uma turbulência. A pior da minha vida foi no meio
que sempre aparecem ao longo do mês, além, é claro, de efe- do nada, sobrevoando o Atlântico, e durou uma boa hora. Já
tuar trabalhos eventuais que surgem conforme a demanda.
que estou aqui escrevendo esse artigo, sobrevivi.
<http://wordpress.concurseirosolitario.com.br/o-coti-
A turbulência significa que o avião vai cair? Ok, sabe-
dianode-
mos que não. Apesar de também sabermos que o avião é a
um-servidor-publico/> Acesso em: 17/12/2015> (com
forma mais segura de viagem, não é tão fácil lembrar disso
adaptações).
em meio a uma turbulência. Então, não custa lembrar que,
11-) Assinale a substituição proposta que causa erro mesmo quando o ar está “violento”, é impossível que ele
de morfossintaxe no texto. «arremesse» o avião para o chão.
<http://revistagalileu.globo.com/Tecnologia/noticia/2015/07/
substituir: por: turbulencia-dos-avioes-e-perigosa.html> Acesso em:15/12/2015
A) Há A (com adaptações).
B) Nesse tempo Durante esse tempo
C) junto juntamente 13-) Assinale a opção em que o primeiro período do
D) Tenho de Tenho que texto foi reescrito com correção gramatical.
E) ao longo do mês no decorrer do mês A) Na hipótese de você for um passageiro frequente, já
tinha passado por uma turbulência, com certeza.
A única substituição que causaria erro é a de “há” por B) Certamente, já deverá ter passado por uma turbu-
“a”, já que, quando empregado com o sentido de tempo lência, se você fosse um passageiro frequente.
passado, deve ser escrito com “h” (há). C) Na certa, acaso você seja um passageiro frequente,
RESPOSTA: A já aconteceu de passar por uma turbulência.
D) Com certeza, se você foi um passageiro frequente, já
12-) (ANAC – TÉCNICO EM REGULAMENTAÇÃO DE
tivesse passado por uma turbulência.
AVIAÇÃO CIVIL - ESAF/2015 - adaptada) Assinale a op-
E) Caso você seja um passageiro frequente, já deve,
ção em que a pontuação permanece correta, apesar de ter
com certeza, ter passado por uma turbulência.
sido modificada.
A) Há quase dois anos, fui empossado técnico admi-
nistrativo (...) Correções:
B) (...) na ANAC, de São Paulo e estou muito satisfeito A) Na hipótese de você for (SER) um passageiro fre-
de trabalhar lá. quente, já tinha passado (PASSOU) por uma turbulência,
C) (...) na administração pública, porém; preferi, ficar com certeza.
onde estou (…) B) Certamente, já deverá (DEVE) ter passado por uma
D) Sinceramente sou partidário, do “não se mexe, em turbulência, se você fosse (FOR) um passageiro frequente.
time que está ganhando”. C) Na certa, acaso você seja um passageiro frequente,
E) Trabalho na área administrativa, junto com outros já aconteceu de passar (PASSOU) por uma turbulência.
técnicos e analistas, além de ser, gestor substituto (…) D) Com certeza, se você foi (É) um passageiro frequen-
Fiz as correções: te, já tivesse passado (PASSOU) por uma turbulência.
B) na ANAC de São Paulo e estou muito satisfeito de E) Caso você seja um passageiro frequente, já deve,
trabalhar lá. com certeza, ter passado por uma turbulência.
C) na administração pública, porém preferi ficar onde RESPOSTA: E
estou (…)
D) Sinceramente, sou partidário do “não se mexe em
14-) (ANAC – TÉCNICO EM REGULAMENTAÇÃO DE
time que está ganhando”.
AVIAÇÃO CIVIL - ESAF/2015) A expressão sublinhada em
E) Trabalho na área administrativa junto com outros
“Já que estou escrevendo esse artigo, sobrevivi” tem sentido
técnicos e analistas, além de ser gestor substituto (…)
RESPOSTA: A de
A) conformidade.
B) conclusão.
C) causa.
D) dedução.
E) condição.

169
EXERCICÍOS COMPLEMENTARES

Subordinadas Adverbiais - Indicam que a oração su- 17-) (ANAC – TÉCNICO EM REGULAMENTAÇÃO DE
bordinada exerce a função de adjunto adverbial da princi- AVIAÇÃO CIVIL - ESAF/2015 - adaptada) Em relação às
pal. De acordo com a circunstância que expressam, classi- regras de acentuação, assinale a opção correta.
ficam-se em:
- Causais: introduzem uma oração que é causa da Por que é preciso passar pelo equipamento de raios X?
ocorrência da oração principal. As conjunções são: porque, São normas internacionais de segurança. É proibido por-
que, como (= porque, no início da frase), pois que, visto que, tar objetos cortantes ou perfurantes. Se você se esqueceu de
uma vez que, porquanto, já que, desde que, etc. despachá-los, esses itens terão de ser descartados no mo-
RESPOSTA: C mento da inspeção.
Como devo proceder na hora de passar pelo equipa-
15-) (ANAC – TÉCNICO EM REGULAMENTAÇÃO DE mento detector de metais?
AVIAÇÃO CIVIL - ESAF/2015 - adaptada) Sobre as vírgu- A inspeção dos passageiros por detector de metais é
las e as aspas empregadas no texto é correto afirmar que obrigatória. O passageiro que, por motivo justificado, não
A) a primeira vírgula separa duas orações coordenadas. puder ser inspecionado por meio de equipamento detector
B) a vírgula antes do “e” ocorre porque o verbo da ora- de metal deverá submeter-se à busca pessoal. As mulheres
ção “e durou uma boa hora” é diferente do verbo da oração grávidas podem solicitar a inspeção por meio de detector
anterior. manual de metais ou por meio de busca pessoal.
C) a vírgula antes de “sobrevivi” marca a diferença entre <http://www.infraero.gov.br/images/stories/guia/2014/
os tempos verbais de “estou escrevendo” e “sobrevivi”. guiapassageiro2014_portugues.pdf> Acesso em: 4/1/2016 (com
D) a vírgula que ocorre depois do “que” e a que ocorre adaptações).
depois de “violento” estão isolando oração intercalada. A) Acentua-se o verbo “é”, quando átono, para diferen-
E) as aspas nas palavras “violento” e “arremesse” se ciá-lo da conjunção “e”.
justificam porque tais palavras pertencem ao vocabulário B) “Você” é palavra acentuada por ser paroxítona termi-
técnico da aviação. nada na vogal “e” fechada.
C) “Despachá-los” se acentua pelo mesmo motivo de
A = Se você é um passageiro frequente, certamente já
“deverá”.
passou por uma turbulência – incorreta (subordinada ad-
D) Ocorre acento grave em “à busca pessoal” em razão
verbial condicional)
B = incorreta (vem depois de uma oração explicativa) do emprego de locução com substantivo no feminino.
C = incorreta (separando oração principal da causal) E) O acento agudo em “grávidas” se deve por se tratar
E = incorreta (empregadas em sentido figurado, facili- de palavra paroxítona terminada em ditongo.
tando a compreensão da descrição)
RESPOSTA: D Comentários:
A) Acentua-se o verbo “é”, quando átono, para diferen-
16-) (ANAC – TÉCNICO EM REGULAMENTAÇÃO ciá-lo da conjunção “e” = não é acento diferencial
DE AVIAÇÃO CIVIL - ESAF/2015) A frase sublinhada em B) “Você” é palavra acentuada por ser paroxítona ter-
“Apesar de também sabermos que o avião é a forma mais minada na vogal “e” fechada = acentua-se por ser oxítona
segura de viagem, não é tão fácil lembrar disso em meio a terminada em “e”
uma turbulência” mantém tanto seu sentido original quan- C) “Despachá-los” se acentua pelo mesmo motivo de
to sua correção gramatical na opção: “deverá” = correta (oxítona terminada em “a”). Lembre-se
A) Embora também sabemos … de que, em verbos com pronome oblíquo, este é desconsi-
B) Dado também saibamos … derado ao analisar a acentuação
C) Pelo motivo o qual também sabemos … D) Ocorre acento grave em “à busca pessoal” em razão
D) Em virtude de também sabermos … do emprego de locução com substantivo no feminino = o
E) Conquanto saibamos …
acento grave se deve à regência do verbo “submeter” que
pede preposição (submeter-se a)
Correções:
A) Embora também sabemos = saibamos E) O acento agudo em “grávidas” se deve por se tratar
B) Dado também saibamos = sabermos de palavra paroxítona terminada em ditongo = acentua-se
C) Pelo motivo o qual também sabemos = essa deixa o por ser proparoxítona
período confuso... RESPOSTA: C
D) Em virtude de também sabermos = sentido diferen-
te do original… 18-) (SABESP/SP – AGENTE DE SANEAMENTO AM-
E) Conquanto saibamos = conjunção que mantém o BIENTAL 01 – FCC/2014 - adaptada)
sentido original (concessivas: introduzem uma oração que ... a navegação rio abaixo entre os séculos XVIII e XIX,
expressa ideia contrária à da principal, sem, no entanto, im- começava em Araritaguaba...
pedir sua realização. São elas: embora, ainda que, apesar O verbo conjugado nos mesmos tempo e modo em
de que, se bem que, mesmo que, por mais que, posto que, que se encontra o grifado acima está em:
conquanto, etc.) (A) ... o Tietê é um regato.
RESPOSTA: E (B) ... ou perto delas moram 30 milhões de pessoas...

170
EXERCICÍOS COMPLEMENTARES

(C) O desenvolvimento econômico e demográfico cus- 20-) (PREFEITURA DE NOVA FRIBURGO-RJ – SECRE-
tou caro ao rio. TÁRIO ESCOLAR - EXATUS/2015 ) Assinale a alternativa
(D) O rio Tietê nasce acima dos mil metros de altitude... em que a palavra é acentuada pela mesma razão que “ce-
(E) ... e traziam ouro. rimônia”:
A) tendência – crônica.
“Começava” = pretérito imperfeito do Indicativo B) descartáveis – uísque.
(A) ... o Tietê é um regato. = presente do Indicativo C) búzios – vestuário.
(B) ... ou perto delas moram 30 milhões de pessoas... = D) ótimo – cipó.
presente do Indicativo Cerimônia = paroxítona terminada em ditongo
(C) O desenvolvimento econômico e demográfico cus- A) tendência = paroxítona terminada em ditongo / crô-
tou caro ao rio.= pretérito perfeito do Indicativo nica = proparoxítona
(D) O rio Tietê nasce acima dos mil metros de altitude... B) descartáveis = paroxítona terminada em ditongo /
= presente do Indicativo uísque = regra do hiato
(E) ... e traziam ouro. = pretérito imperfeito do Indica- C) búzios = paroxítona terminada em ditongo / vestuá-
tivo rio = paroxítona terminada em ditongo
RESPOSTA: “E” D) ótimo = proparoxítona / cipó = oxítona terminada
em “o”
19-) (TÉCNICO EM REGULAMENTAÇÃO DE AVIA- RESPOSTA: C
ÇÃO CIVIL - ESAF/2015) Assinale o trecho sem problemas
de ortografia. 21-) (PREFEITURA DE NOVA FRIBURGO-RJ – SECRE-
A) No caso de sentir-se prejudicado ou de ter seus TÁRIO ESCOLAR - EXATUS/2015 ) Os termos destacados
direitos desrespeitados, o passageiro de avião deve dirijir- abaixo estão corretamente analisados quanto à função sin-
se primeiro à empresa aérea contratada, para reinvindicar tática em:
seus direitos como consumidor. I - “O cidadão é livre” – predicativo do sujeito.
B) É possível, também, registrar reclamação contra a II - “A gente tem um ressaca” – objeto direto.
empresa aérea na ANAC, que analizará o fato. III - O Boldo resolve – predicado verbal.
C) Se a ANAC constatar descomprimento de normas da A) Apenas I e II.
aviação civil, poderá aplicar sanção administrativa à empresa. B) Apenas I e III.
D) No entanto, a ANAC não é parte na relação de con- C) Apenas II e III.
sumo firmada entre o passageiro e a empresa aérea, razão D) I, II e III.
pela qual não é possível buscar indenização na Agência.
E) Para exijir indenização por danos morais e/ou mate- I - “O cidadão é livre” – predicativo do sujeito = correta
riais, consulte os órgãos de defesa do consumidor, e averi- II - “A gente tem um ressaca” – objeto direto = correta
gúe antecipadamente se está de posse dos comprovantes III - O Boldo resolve – predicado verbal = correta
necessários. RESPOSTA: D
Trechos adaptados de <http://www.infraero.gov.br/
images/stories/ 22-) (JUCEPAR-PR – ADMINISTRADOR - FAU/2016)
guia/2014/guiapassageiro2014_portugues.pdf> Aces-
so em: A essência da infância
17/12/2015. Como a convivência íntima com os filhos é capaz de
transformar
Por itens: a relação das crianças consigo mesmas e com o mundo
A) No caso de sentir-se prejudicado ou de ter seus di-
reitos desrespeitados, o passageiro de avião deve dirijir-se Crianças permanentemente distraídas com o celular ou
(DIRIGIR-SE) primeiro à empresa aérea contratada, para re- o tablet. Agenda cheia de tarefas e aulas depois da escola.
invindicar (REIVINDICAR) seus direitos como consumidor. Pais que não conseguem impor limites e falar “não”. Os mo-
B) É possível, também, registrar reclamação contra a mentos de lazer que ficaram restritos ao shopping Center,
empresa aérea na ANAC, que analizará (ANALISARÁ) o fato. em vez de descobertas ao ar livre. Quais as implicações desse
C) Se a ANAC constatar descomprimento (DESCUM- conjunto de hábitos e comportamentos para nossos filhos?
PRIMENTO) de normas da aviação civil, poderá aplicar san- Para o pediatra Daniel Becker, esses têm sido verdadeiros
ção administrativa à empresa. pecados cometidos à infância, que prejudicarão as crianças
D) No entanto, a ANAC não é parte na relação de con- até a vida adulta. Pioneiro da Pediatria Integral, prática que
sumo firmada entre o passageiro e a empresa aérea, razão amplia o olhar e o cuidado para promover o desenvolvi-
pela qual não é possível buscar indenização na Agência. mento pleno e o bem-estar da criança e da família, Daniel
E) Para exijir (EXIGIR) indenização por danos morais e/ defende que devemos estar próximos dos pequenos – esse,
ou materiais, consulte os órgãos de defesa do consumidor, sim, é o melhor presente a ser oferecido. E que desenvolver
e averigúe (AVERIGUE) antecipadamente se está de posse intimidade com as crianças, além de um tempo reservado
dos comprovantes necessários. ao lazer com elas, faz a diferença. Para o bem-estar delas e
RESPOSTA: D para toda a família.
(Revista Vida Simples. Dezembro de 2015).

171
EXERCICÍOS COMPLEMENTARES

O tema central do texto a essência da infância refere- (C) Tempo.


se: (D) Concessão.
(A) Às tecnologias disponíveis. (E) Finalidade.
(B) À importância do convívio familiar.
(C) Às preocupações do pediatra Daniel Becker. O trecho apresenta uma condição para que façamos
(D) À importância de impor limites. um belíssimo passeio: não chover.
(E) Ao exagerado consumo. RESPOSTA: B
Fica clara a intenção do autor: mostrar a importância
do convívio familiar (E que desenvolver intimidade com as 27-) (SABESP/SP – AGENTE DE SANEAMENTO AM-
crianças, além de um tempo reservado ao lazer com elas, faz BIENTAL 01 – FCC/2014) Até o século passado, as margens
a diferença. Para o bem-estar delas e para toda a família). e várzeas do Tietê ...... pela população, ...... das enchentes e do
RESPOSTA: B risco de doenças que ...... depois delas.
Os espaços da frase acima estarão corretamente preen-
23-) (JUCEPAR-PR – ADMINISTRADOR - FAU/2016) chidos, na ordem dada, por:
No excerto: “... esses têm sido verdadeiros pecados cometi- (A) eram evitadas − temerosa − apareciam
dos à infância...”. O pronome em destaque refere-se a: (B) era evitadas − temerosa − aparecia
(A) Celular e tablet. (C) era evitado − temerosas − apareciam
(B) Agenda. (D) era evitada − temeroso − aparecia
(C) Aulas depois da escola. (E) eram evitadas − temeroso – aparecia
(D) Visitas ao shopping Center.
(E) Conjunto de hábitos. Destaquei os termos que se relacionam:
Até o século passado, as margens e várzeas do Tietê
Voltemos ao texto: “Quais as implicações desse conjun- eram evitadas pela população, temerosa das enchentes e
to de hábitos e comportamentos para nossos filhos? Para o do risco de DOENÇAS que APARECIAM depois delas.
pediatra Daniel Becker, esses têm sido (..)” Eram evitadas / temerosa / apareciam.
RESPOSTA: E RESPOSTA: A
24-) (JUCEPAR-PR – ADMINISTRADOR - FAU/2016) 28-) (JUCEPAR-PR – ADMINISTRADOR - FAU/2016)
No fragmento: “... além de um tempo reservado ao lazer Na frase: “O livro que estou lendo é muito interessante”. A
com elas...”. A palavra destacada expressa ideia de: palavra destacada é um:
(A) Ressalva. (A) Artigo.
(B) Conclusão. (B) Substantivo.
(C) Adição. (C) Adjetivo.
(D) Advertência.
(D) Verbo.
(E) Explicação.
(E) Pronome.
Dá-nos a ideia de adição.
Quando conseguimos substituir o “que” por “o qual”
RESPOSTA: C
temos um caso de pronome relativo – como na questão.
RESPOSTA: E
25-) (JUCEPAR-PR – ADMINISTRADOR - FAU/2016)
No período: “Para o pediatra Daniel Becker, esses têm sido
29-) (JUCEPAR-PR – ADMINISTRADOR - FAU/2016
verdadeiros pecados cometidos à infância, que prejudicarão
as crianças até a vida adulta”. O verbo destacado está res- - adaptada) No período: “ANS reforça campanha contra o
pectivamente no modo e tempo do: mosquito transmissor da dengue e zika”. O verbo em desta-
(A) Indicativo – presente. que apresenta-se:
(B) Subjuntivo – pretérito. (A) Na voz passiva.
(C) Subjuntivo – futuro. (B) Na voz ativa.
(D) Indicativo – futuro. (C) Na voz reflexiva.
(E) Indicativo – pretérito. (D) Na voz passiva analítica.
(E) Na voz passiva sintética.
Quando o verbo termina em “ão”: indica uma ação que
acontecerá – futuro do presente do Indicativo. Temos sujeito (ANS) praticando a ação (reforça), por-
RESPOSTA: D tanto voz ativa.
RESPOSTA: B
26-) (JUCEPAR-PR – ADMINISTRADOR - FAU/2016)
Na frase: Se não chover hoje à tarde faremos um belíssimo 30-) (JUCEPAR-PR – ADMINISTRADOR - FAU/2016)
passeio. Há indicação de: Na frase: Ao terminar a prova, todos os candidatos deverão
(A) Comparação. aguardar a verificação dos aplicadores. A oração destacada
(B) Condição. faz referência a

172
EXERCICÍOS COMPLEMENTARES

(A) Condição. (C) As ideias revolucionárias do século 18, apesar do


(B) Finalidade. isolamento do país, viajava na bagagem da pequena elite
(C) Tempo. brasileira que tivera oportunidade de estudar em Portugal.
(D) Comparação. (D) No final do século 18, haviam mudanças profundas
(E) Conformidade. na tecnologia, com a invenção das máquinas a vapor pro-
A frase nos dá a ideia do momento (tempo) em que de- tagonizadas pelos ingleses.
veremos aguardar a verificação por parte dos aplicadores. (E) Em 1776, ano da Independência dos Estados Uni-
RESPOSTA: C dos, havia nove universidades no país, incluindo a presti-
giada Harvard, e chegava a três milhões de exemplares por
31-) (DPE-RR – ASSISTENTE ADMINISTRATIVO - ano a circulação de jornais.
FCC/2015) Mas não vou pegá-lo − o poema já foi reescrito
várias vezes em outros poemas; e o meu boi no asfalto ainda Correções:
me enche de luz, transformado em minha própria estrela. (A) Deflagrada em 1789 com a queda da Bastilha – pri-
Atribuindo-se caráter hipotético ao trecho acima, os são parisiense onde se confinava (CONFINAVAM) crimino-
verbos sublinhados devem assumir a seguinte forma: sos e dissidentes políticos − a Revolução Francesa levou
milhares de condenados à guilhotina.
(A) iria − iria ser − teria enchido
(B) A maré das inovações democráticas na Europa e nos
(B) ia − tinha sido − encheria
Estados Unidos chegariam (CHEGARIA) com algum atraso
(C) viria − iria ser − encheria
ao Brasil, mas com efeito igualmente devastador.
(D) iria − teria sido − encheria (C) As ideias revolucionárias do século 18, apesar do
(E) viria − teria sido − teria enchido isolamento do país, viajava (VIAJAVMA) na bagagem da
pequena elite brasileira que tivera oportunidade de estu-
O modo verbal que trabalha com hipótese é o Sub- dar em Portugal.
juntivo. Façamos as transformações: Mas não iria pegá- (D) No final do século 18, haviam (HAVIA) mudanças
-lo − o poema já teria sido reescrito várias vezes em outros profundas na tecnologia, com a invenção das máquinas a
poemas; e o meu boi no asfalto ainda me encheria de luz, vapor protagonizadas pelos ingleses.
transformado em minha própria estrela. (E) Em 1776, ano da Independência dos Estados Uni-
RESPOSTA: D dos, havia nove universidades no país, incluindo a presti-
giada Harvard, e chegava a três milhões de exemplares por
32-) (METRÔ/SP – TÉCNICO SISTEMAS METROVIÁ- ano a circulação de jornais.
RIOS CIVIL – FCC/2014 - adaptada) RESPOSTA: E
...’sertanejo’ indicava indistintamente as músicas produ-
zidas no interior do país... 34-) (DPE-RR – ASSISTENTE ADMINISTRATIVO -
Transpondo-se a frase acima para a voz passiva, a for- FCC/2015 - adaptada) Considere o texto abaixo para res-
ponder à questão.
ma verbal resultante será:
O pesquisador e médico sanitarista Luiz Hildebrando Pe-
(A) vinham indicadas.
reira da Silva tornou-se professor titular de parasitologia em
(B) era indicado.
1997, assumindo a direção dos programas de pesquisa em
(C) eram indicadas. Rondônia − numa das frentes avançadas da USP na Ama-
(D) tinha indicado. zônia −, que reduziram o percentual de registros de malária
(E) foi indicada. em Rondônia de 40% para 7% do total de casos da doença
na região amazônica em uma década.
‘sertanejo’ indicava indistintamente as músicas produ- (Adaptado de: revistapesquisa.fapesp.
zidas no interior do país. br/2014/10/09/o-cientista-
As músicas produzidas no país eram indicadas pelo das-doencas-tropicais)
sertanejo, indistintamente.
RESPOSTA: C ... que reduziram o percentual de registros de malária
em Rondônia...
33-) (DPE-RR – ASSISTENTE ADMINISTRATIVO - O elemento que justifica a flexão do verbo acima é:
FCC/2015) As normas de concordância estão respeitadas (A) casos da doença.
em: (B) frentes avançadas da USP na Amazônia.
(A) Deflagrada em 1789 com a queda da Bastilha – pri- (C) registros de malária.
são parisiense onde se confinava criminosos e dissidentes (D) programas de pesquisa em Rondônia.
políticos − a Revolução Francesa levou milhares de conde- (E) investigações sobre a malária em Rondônia.
nados à guilhotina.
Recorramos ao texto: “assumindo a direção dos progra-
(B) A maré das inovações democráticas na Europa e nos
mas de pesquisa em Rondônia − numa das frentes avança-
Estados Unidos chegariam com algum atraso ao das da USP na Amazônia −, que reduziram o percentual”. O
Brasil, mas com efeito igualmente devastador.

173
EXERCICÍOS COMPLEMENTARES

termo entre “traços” é um aposto, uma informação a mais. (D) ...outra parte prioriza a participação popular através
O verbo se relaciona com o termo anteriormente citado da interatividade, bem como a cooperação técnica para o
(programas). reuso de dados, abertos por entidades e empresas. = in-
RESPOSTA: D correta
(E) Contudo, existem estudos, que apontam que bas-
35-) (DPE-RR – ASSISTENTE ADMINISTRATIVO - tariam meros quatro pontos de dados para identificar os
FCC/2015 - adaptada) Considere o texto abaixo para res- movimentos de uma pessoa na cidade. = incorreta
ponder à questão. RESPOSTA: B

Sobre a vinda ao Brasil, Luiz Hildebrando Pereira da Sil-


va afirmou: “Quando me aposentei na França, consideran-
do-me ainda válido, hesitei antes de tomar a decisão de me
reintegrar às atividades de pesquisa na Amazônia. Acabei
decidindo. (...) Eu me ...... um velho ranzinza se ....... ficado na
França plantando rosas”.
(Adaptado de: cremesp.org.br)

Considerado o contexto, preenchem corretamente as


lacunas da frase acima, na ordem dada:
(A) tornarei − tinha
(B) tornara − tivesse
(C) tornarei − tiver
(D) tornaria − tivesse
(E) tornasse – tivera

Pelo contexto, é possível identificar que se trata de


uma hipótese (se tivesse ficado na França, ele se tornaria
um velho ranzinza).
RESPOSTA: D

36-) (TRF 3ªREGIÃO – TÉCNICO JUDICIÁRIO - EDI-


FICAÇÕES - FCC/2016 - adaptada) O acréscimo de uma
vírgula após o termo sublinhado não altera o sentido nem
a correção do trecho:
(A) A ideia de cidade inteligente sempre aparece rela-
cionada à abertura de bases de dados por parte dos órgãos
públicos.
(B) Há experiências importantes em cidades brasileiras
também.
(C) ... uma parte prioriza a transparência como meio
de prestação de contas e responsabilidade política frente à
sociedade civil, como a ideia de governo aberto...
(D) ...outra parte prioriza a participação popular através
da interatividade, bem como a cooperação técnica para o
reuso de dados abertos por entidades e empresas.
(E) Contudo, existem estudos que apontam que bas-
tariam meros quatro pontos de dados para identificar os
movimentos de uma pessoa na cidade.

Vejamos:
(A) A ideia de cidade inteligente sempre aparece, rela-
cionada à abertura de bases de dados por parte dos órgãos
públicos. = incorreta
(B) Há experiências importantes em cidades brasileiras,
também. = correta
(C) ... uma parte, prioriza a transparência como meio
de prestação de contas e responsabilidade política frente
à sociedade civil, como a ideia de governo aberto... = in-
correta

174

Das könnte Ihnen auch gefallen